All Items

  1. Algebra, NAPX-I3-CA27 SA

    <div class="sm_mode"> {{{question}}} </div>

    [{"vars":[{"varval":"Claude is going on a fishing trip.\n\n\nHe wants to purchase fishing lures before the trip, which are sold in boxes of a dozen lures.\n\n\nThe more boxes of lures that Claude buys, the less it costs per box, as shown in the table below.\n\n<div class=\"sm-table row1-color5 heading-color5\">\n\n>>Fishing lure pricing\n\n>>| Number of boxes | Total Cost |\n|:-:|:-:|\n| 1 | $18.00 |\n| 2 | $35.50 |\n| 3 | $52.50 |\n|4|$69.00|\n\n</div>\n\n<br>If this price pattern continues, how much will it cost for 6 boxes of lures?\n"},{"varval":"Additional boxes reduce in cost by 50 cents.\n\n⇒ 4th box costs an extra $16.50\n\n⇒ 5th box costs an extra $16.00\n\n⇒ 6th box costs an extra $15.50\n\n<div class=\"aligned\">\n\n| | |\n| ------------- | ---------- |\n| $\\therefore$ Total cost of 6 boxes | \\= $69.00+16.00+15.50$ |\n| | \\= {{{prefix0}}}{{{correctAnswer0}}} |\n\n</div>\n"}]},{"vars":[{"varval":"Tiffany has a market stall where she sells crystals.\n\n\nShe wants to purchase crystals from a wholesaler, which are sold in boxes of five crystals.\n\n\nThe more boxes of crystals that Tiffany buys, the less it costs per box, as shown in the table below.\n\n<div class=\"sm-table row1-color3 heading-color3\">\n\n>>Crystal pricing\n\n>>| Number of boxes | Total Cost |\n|:-:|:-:|\n| 1 | $7.50 |\n| 2 | $14.50 |\n| 3 | $21.00 |\n|4|$27.00|\n\n</div>\n\n<br>If this price pattern continues, how much will it cost for 7 boxes of crystals?\n"},{"varval":"Additional boxes reduce in cost by 50 cents.\n\n⇒ 4th box costs an extra $6.00\n\n⇒ 5th box costs an extra $5.50\n\n⇒ 6th box costs an extra $5.00\n\n⇒ 7th box costs an extra $4.50\n\n<div class=\"aligned\">\n\n| | |\n| ------------- | ---------- |\n| $\\therefore$ Total cost of 7 boxes | \\= $27.00+5.50+5.00+4.50$ |\n| | \\= {{{prefix0}}}{{{correctAnswer0}}} |\n\n</div>\n"}]},{"vars":[{"varval":"Boyd is running an art class for beginners.\n\n\nHe wants to purchase pencils for his students, which are sold in boxes of 24 pencils.\n\n\nThe more boxes of pencils that Boyd buys, the less it costs per box, as shown in the table below.\n\n<div class=\"sm-table row1-color2 heading-color2\">\n\n>>Pencil pricing\n\n>>| Number of boxes | Total Cost |\n|:-:|:-:|\n| 1 | $15.20 |\n| 2 | $29.60 |\n| 3 | $43.20 |\n|4|$56.00|\n\n</div>\n\n<br>If this price pattern continues, how much will it cost for 6 boxes of pencils?\n"},{"varval":"Additional boxes reduce in cost by 80 cents.\n\n⇒ 4th box costs an extra $12.80\n\n⇒ 5th box costs an extra $12.00\n\n⇒ 6th box costs an extra $11.20\n\n<div class=\"aligned\">\n\n| | |\n| ------------- | ---------- |\n| $\\therefore$ Total cost of 6 boxes | \\= $56.00+12.00+11.20$ |\n| | \\= {{{prefix0}}}{{{correctAnswer0}}} |\n\n</div>\n"}]},{"vars":[{"varval":"Indira is purchasing Economics textbooks.\n\n\nThe more textbooks that Indira buys, the less it costs per textbook, as shown in the table below.\n\n<div class=\"sm-table row1-color4 heading-color4\">\n\n>>Textbook pricing\n\n>>| Number of books | Total Cost |\n|:-:|:-:|\n| 1 | $75.50 |\n| 2 | $147.50 |\n| 3 | $217.50 |\n|4|$285.00|\n\n</div>\n\n<br>If this price pattern continues, how much will it cost for 7 Economics textbooks?\n"},{"varval":"Additional textbooks reduce in cost by $2.50.\n\n⇒ 4th book costs an extra $67.50\n\n⇒ 5th book costs an extra $65.00\n\n⇒ 6th book costs an extra $62.50\n\n⇒ 7th book costs an extra $60.00\n\n<div class=\"aligned\">\n\n| | |\n| ------------- | ---------- |\n| $\\therefore$ Total cost of 7 textbooks | \\= $285.00+65.00+62.50+60.00$ |\n| | \\= {{{prefix0}}}{{{correctAnswer0}}} |\n\n</div>\n"}]},{"vars":[{"varval":"Jett is a builder.\n\n\nHe wants to purchase flat head nails for his latest construction job, which are sold in boxes of a fifty nails.\n\n\nThe more boxes of nails that Jett buys, the less it costs per box, as shown in the table below.\n\n<div class=\"sm-table row1-color1 heading-color1\">\n\n>>Nail pricing\n\n>>| Number of boxes | Total Cost |\n|:-:|:-:|\n| 1 | $27.35 |\n| 2 | $54.50 |\n| 3 | $81.45 |\n|4|$108.20|\n\n</div>\n\n<br>If this price pattern continues, how much will it cost for 6 boxes of flat head nails?\n"},{"varval":"Additional boxes reduce in cost by 20 cents.\n\n⇒ 4th box costs an extra $26.75\n\n⇒ 5th box costs an extra $26.55\n\n⇒ 6th box costs an extra $26.35\n\n<div class=\"aligned\">\n\n| | |\n| ------------- | ---------- |\n| $\\therefore$ Total cost of 7 boxes | \\= $108.20+26.55+26.35$ |\n| | \\= {{{prefix0}}}{{{correctAnswer0}}} |\n\n</div>\n"}]}]

  2. Algebra, NAPX-G4-NC25 SA - Same but with decimals in the table and fractions in the question

    <div class="sm_mode"> {{{question}}} </div>

    [{"vars":[{"varval":"The table below shows the relationship between the width of a tree's trunk in centimetres and how old it is in months.\n\n<br>\n\n<div class=\"sm-table col1-color1\">\n\n>>| Age in months |12|15|18|21|24|\n|:-:|:-:|:-:|:-:|:-:|:-:|\n| Width of trunk (cm) | 6.375|6.5|6.625|6.75|6.875|\n\n</div>\n\n<br>If this pattern continues, how old would you expect a tree with a 7$\\frac{1}{4}$ cm wide trunk to be?"},{"varval":"Continuing the pattern adding 0.125:\n\n<div class=\"sm-table col1-color1\">\n\n>>| Age in months | 24|27|30|33|\n|:-:|:-:|:-:|:-:|:-:|\n| Width of trunk (cm) | 6.875|7|7.125|7.25|\n\n</div>\n\n<br>$\\therefore$ Expected age = {{{correctAnswer0}}} {{{suffix0}}}"}]},{"vars":[{"varval":"The table below shows the relationship between the length of a skink in centimetres and how old it is in months.\n\n<br>\n\n<div class=\"sm-table col1-color2\">\n\n>>| Age in months | 4|6|8|10|12|\n|:-:|:-:|:-:|:-:|:-:|:-:|\n| Length (cm) | 8.375|88.5|8.625|8.75|8.875|\n\n</div>\n\n<br>If this pattern continues, how old would you expect a 9$\\frac{3}{8}$ cm skink to be?"},{"varval":"Continuing the pattern adding 0.125:\n\n<div class=\"sm-table col1-color2\">\n\n>>| Age in months | 12|14|16|18|20|\n|:-:|:-:|:-:|:-:|:-:|:-:|\n| Length (cm) | 8.875|9|9.125|9.250|9.375|\n\n</div>\n\n<br>$\\therefore$ Expected age = {{{correctAnswer0}}} {{{suffix0}}}"}]},{"vars":[{"varval":"The table below shows the relationship between the length of an axolotl in centimetres and how old it is in months.\n\n<br>\n\n<div class=\"sm-table col1-color3\">\n\n>>| Age in months | 7|10|13|16|19|\n|:-:|:-:|:-:|:-:|:-:|:-:|\n| Length (cm) | 2.7|2.9|3.1|3.3|3.5|\n\n</div>\n\n<br>If this pattern continues, how old would you expect a 4$\\frac{3}{10}$ cm axolotl to be?"},{"varval":"Continuing the pattern adding 0.2:\n\n<div class=\"sm-table col1-color3\">\n\n>>| Age in months | 19|22|25|28|31|\n|:-:|:-:|:-:|:-:|:-:|:-:|\n| Length (cm) | 3.5|3.7|3.9|4.1|4.3|\n\n</div>\n\n<br>$\\therefore$ Expected age = {{{correctAnswer0}}} {{{suffix0}}}"}]},{"vars":[{"varval":"The table below shows the relationship between the weight of a squirrel in grams and how old it is in days.\n\n<br>\n\n<div class=\"sm-table col1-color4\">\n\n>>| Age in days| 2|4|6|8|10|\n|:-:|:-:|:-:|:-:|:-:|:-:|\n| Weight in grams (g) | 15.8|16.1|16.4|16.7|17|\n\n</div>\n\n<br>If this pattern continues, how old would you expect a squirrel with a weight of 18$\\frac{1}{5}$ g to be?"},{"varval":"Continuing the pattern adding 0.3:\n\n<div class=\"sm-table col1-color4\">\n\n>>| Age in days| 10|12|14|16|18|\n|:-:|:-:|:-:|:-:|:-:|:-:|\n| Weight in grams (g) | 17|17.3|17.6|17.9|18.2|\n\n</div>\n\n<br>$\\therefore$ Expected age = {{{correctAnswer0}}} {{{suffix0}}}"}]},{"vars":[{"varval":"The table below shows the relationship between the height of a mushroom in millimetres and how old it is in days.\n\n<br>\n\n<div class=\"sm-table col1-color5\">\n\n>>| Age in days| 20|24|28|32|36|\n|:-:|:-:|:-:|:-:|:-:|:-:|\n| Height of mushroom (mm) | 30.125|30.5|30.875|31.25|31.625|\n\n</div>\n\n<br>If this pattern continues, how old would you expect a mushroom with a height of 32$\\frac{3}{4}$ mm to be?"},{"varval":"Continuing the pattern adding 0.375:\n\n<div class=\"sm-table col1-color5\">\n\n>>| Age in days| 36|40|44|48|\n|:-:|:-:|:-:|:-:|:-:|\n| Height of mushroom (mm)| 31.625| 32|32.375|32.75|\n\n</div>\n\n<br>$\\therefore$ Expected age = {{{correctAnswer0}}} {{{suffix0}}}"}]}]

  3. Algebra, NAPX-p106742v01

    <div class="sm_mode"> {{{question}}} </div>

    [{"vars":[{"varval":"A Cartesian plane is shown below..\n\n\nsm_img https://teacher.smartermaths.com.au/wp-content/uploads/2023/08/Algebra-NAP_20275-min.svg 300 indent vpad\n\nWhich statement is true?"},{"varval":"Points on the $\\large y$-axis : &nbsp;$\\large x$ = 0\n\nPoints below the $\\large x$-axis : &nbsp;$\\large y$ < 0\n\n$\\therefore$ {{{correctAnswer}}} is correct."}]},{"vars":[{"varval":"A Cartesian plane is shown below..\n\n\nsm_img https://teacher.smartermaths.com.au/wp-content/uploads/2023/08/Algebra-NAP_20275-min.svg 300 indent vpad\n\nWhich statement is true?"},{"varval":"Points on the left of the $\\large y$-axis : &nbsp;$\\large x$ < 0\n\nPoints below the $\\large x$-axis : &nbsp;$\\large y$ < 0\n\n$\\therefore$ {{{correctAnswer}}} is correct."}]},{"vars":[{"varval":"A Cartesian plane is shown below..\n\n\nsm_img https://teacher.smartermaths.com.au/wp-content/uploads/2023/08/Algebra-NAP_20275-min.svg 300 indent vpad\n\nWhich statement is true?"},{"varval":"Points to the right of the $\\large y$-axis : &nbsp;$\\large x$ > 0\n\nPoints below the $\\large x$-axis : &nbsp;$\\large y$ < 0\n\n$\\therefore$ {{{correctAnswer}}} is correct."}]},{"vars":[{"varval":"A Cartesian plane is shown below..\n\n\nsm_img https://teacher.smartermaths.com.au/wp-content/uploads/2023/08/Algebra-NAP_20275-min.svg 300 indent vpad\n\nWhich statement is true?"},{"varval":"Points to the right of the $\\large y$-axis : &nbsp;$\\large x$ > 0\n\nPoints above the $\\large x$-axis : &nbsp;$\\large y$ > 0\n\n$\\therefore$ {{{correctAnswer}}} is correct."}]},{"vars":[{"varval":"A Cartesian plane is shown below..\n\n\nsm_img https://teacher.smartermaths.com.au/wp-content/uploads/2023/08/Algebra-NAP_20275-min.svg 300 indent vpad\n\nWhich statement is true?"},{"varval":"Points to the left of the $\\large y$-axis : &nbsp;$\\large x$ < 0\n\nPoints above the $\\large x$-axis : &nbsp;$\\large y$ > 0\n\n$\\therefore$ {{{correctAnswer}}} is correct."}]},{"vars":[{"varval":"A Cartesian plane is shown below..\n\n\nsm_img https://teacher.smartermaths.com.au/wp-content/uploads/2023/08/Algebra-NAP_20275-min.svg 300 indent vpad\n\nWhich statement is true?"},{"varval":"Points to the right of the $\\large y$-axis : &nbsp;$\\large x$ > 0\n\nPoints on the $\\large x$-axis : &nbsp;$\\large y$ = 0\n\n$\\therefore$ {{{correctAnswer}}} is correct."}]}]

  4. Algebra, NAPX-E4-CA23 SA

    <div class="sm_mode"> {{{question}}} </div>

    [{"vars":[{"varval":"Vlad uses the formula below to estimate the population of wombats in Kosciuszko National Park over a three year period.\n\n>Year 1 = 900\n\n<br>\n\n>Year 2 = Year 1 + $\\dfrac{ \\text{Year 1}}{30}$\n\n<br>\n\n>Year 3 = Year 2 + $\\dfrac{ \\text{Year 2}}{30}$\n\n<br>\n\nEstimate the population of wombats in Kosciuszko National Park in Year 3?"},{"varval":"Year 1 = 900\n\nYear 2 = 900 + $\\dfrac{900}{30}$ = 930\n\nYear 3 = 930 + $\\dfrac{930}{30}$ = {{{correctAnswer0}}}"}]},{"vars":[{"varval":"Morgan uses the formula below to estimate the population of Tasmanian devils in a breeding program over a three year period.\n\n>Year 1 = 300\n\n<br>\n\n>Year 2 = Year 1 + $\\dfrac{ \\text{Year 1}}{10}$\n\n<br>\n\n>Year 3 = Year 2 + $\\dfrac{ \\text{Year 2}}{10}$\n\n<br>\n\nEstimate the population of Tasmanian devils in Year 3?"},{"varval":"Year 1 = 300\n\nYear 2 = 300 + $\\dfrac{300}{10}$ = 330\n\nYear 3 = 330 + $\\dfrac{330}{10}$ = {{{correctAnswer0}}}"}]},{"vars":[{"varval":"Kurt uses the formula below to estimate the population of humpback whales in the Eastern migration over a three year period.\n\n>Year 1 = 40 000\n\n<br>\n\n>Year 2 = Year 1 + $\\dfrac{ \\text{Year 1}}{40}$\n\n<br>\n\n>Year 3 = Year 2 + $\\dfrac{ \\text{Year 2}}{40}$\n\n<br>\n\nEstimate the population of humpback whales in Year 3?"},{"varval":"Year 1 = 40000\n\nYear 2 = 40000 + $\\dfrac{40000}{40}$ = 41000\n\nYear 3 = 41000 + $\\dfrac{41000}{40}$ = {{{correctAnswer0}}}"}]},{"vars":[{"varval":"Bianca uses the formula below to estimate the population of female sea turtles on the Caribbean coast over a three year period.\n\n>Year 1 = 30000\n\n<br>\n\n>Year 2 = Year 1 + $\\dfrac{ \\text{Year 1}}{30}$\n\n<br>\n\n>Year 3 = Year 2 + $\\dfrac{ \\text{Year 2}}{30}$\n\n<br>\n\nEstimate the population of female sea turtles in Year 3?"},{"varval":"Year 1 = 30000\n\nYear 2 = 30000 + $\\dfrac{30000}{20}$ = 31500\n\nYear 3 = 31500 + $\\dfrac{31500}{20}$ = {{{correctAnswer0}}}"}]},{"vars":[{"varval":"Gina uses the formula below to estimate the population of fairy penguins on Phillip Island over a three year period.\n\n>Year 1 = 32000\n\n<br>\n\n>Year 2 = Year 1 + $\\dfrac{ \\text{Year 1}}{16}$\n\n<br>\n\n>Year 3 = Year 2 + $\\dfrac{ \\text{Year 2}}{16}$\n\n<br>\n\nEstimate the population of fairy penguins in Year 3?"},{"varval":"Year 1 = 32000\n\nYear 2 = 32000 + $\\dfrac{32000}{16}$ = 34000\n\nYear 3 = 34000 + $\\dfrac{34000}{16}$ = {{{correctAnswer0}}}"}]},{"vars":[{"varval":"Skippy uses the formula below to estimate the population of kangaroos in New South Wales over a three year period.\n\n>Year 1 = 11.8 million\n\n<br>\n\n>Year 2 = Year 1 + $\\dfrac{ \\text{Year 1}}{10}$\n\n<br>\n\n>Year 3 = Year 2 + $\\dfrac{ \\text{Year 2}}{10}$\n\n<br>\n\nEstimate the population of kangaroos in New South Wales in Year 3?"},{"varval":"Year 1 = 11 800 000\n\nYear 2 = 11 800 000 + $\\dfrac{11 800 000}{10}$ = 12980000\n\nYear 3 = 12 980 000 + $\\dfrac{12 980 000}{10}$ = {{{correctAnswer0}}}"}]}]

  5. <div class="sm_mode"> {{{question}}} </div>

    [{"vars":[{"varval":"Iggy used this rule to make a number pattern.\n\n* Start with 2.\n\n* To get to the next number, multiply by 2 then add 3.\n\nWhat is the sum of the first three numbers in Iggy's pattern?"},{"varval":"First write down the pattern:\n\n<div class=\"aligned\">\n\n>>| | | |\n| ----------------: | ---------------------- |--------- |\n| First number | \\= 2 ||\n| Second number | \\= 2 $\\times$ 2 + 3| \\= 7 |\n| Third number|\\= 2 $\\times$ 7 + 3| \\= 17 |\n\n</div>\n\n<br>Then find the sum of the numbers\n\n<div class=\"aligned\">\n\n>>| | | \n| ----------------: | ---------------------- |\n| Sum of first 3 terms | \\= 2 + 7 + 17 = {{{correctAnswer0}}}|\n\n</div>"}]},{"vars":[{"varval":"Ellie used this rule to make a number pattern.\n\n* Start with 4.\n\n* To get to the next number, multiply by 3 then takeaway 1.\n\nWhat is the sum of the first three numbers in Ellie's pattern?"},{"varval":"First write down the pattern:\n\n<div class=\"aligned\">\n\n>>| | | |\n| ----------------: | ---------------------- |--------- |\n| First number | \\= 4 ||\n| Second number | \\= 3 $\\times$ 4 $-$ 1| \\= 11 |\n| Third number|\\= 3 $\\times$ 11 $-$ 1| \\= 32 |\n\n</div>\n\n<br>Then find the sum of the numbers\n\n<div class=\"aligned\">\n\n>>| | | \n| ----------------: | ---------------------- |\n| Sum of first 3 terms | \\= 4 + 11 + 32 = {{{correctAnswer0}}}|\n\n</div>"}]},{"vars":[{"varval":"Boss used this rule to make a number pattern.\n\n* Start with 10.\n\n* To get to the next number, multiply by 4 then add 5.\n\nWhat is the sum of the first three numbers in Boss's pattern?"},{"varval":"First write down the pattern:\n\n<div class=\"aligned\">\n\n>>| | | |\n| ----------------: | ---------------------- |--------- |\n| First number | \\= 10 ||\n| Second number | \\= 4 $\\times$ 10 + 5| \\= 45 |\n| Third number|\\= 4 $\\times$ 45 + 5| \\= 185 |\n\n</div>\n\n<br>Then find the sum of the numbers\n\n<div class=\"aligned\">\n\n>>| | | \n| ----------------: | ---------------------- |\n| Sum of first 3 terms | \\= 10 + 45 + 185 = {{{correctAnswer0}}}|\n\n</div>"}]},{"vars":[{"varval":"Jaydah used this rule to make a number pattern.\n\n* Start with 7.\n\n* To get to the next number, multiply by 5 then takeaway 3.\n\nWhat is the sum of the first three numbers in Jaydah's pattern?"},{"varval":"First write down the pattern:\n\n<div class=\"aligned\">\n\n>>| | | |\n| ----------------: | ---------------------- |--------- |\n| First number | \\= 7 ||\n| Second number | \\= 5 $\\times$ 7 $-$ 3| \\= 32 |\n| Third number|\\= 5 $\\times$ 32 $-$ 3| \\= 157 |\n\n</div>\n\n<br>Then find the sum of the numbers\n\n<div class=\"aligned\">\n\n>>| | | \n| ----------------: | ---------------------- |\n| Sum of first 3 terms | \\= 7 + 32 + 157 = {{{correctAnswer0}}}|\n\n</div>"}]},{"vars":[{"varval":"Joyce used this rule to make a number pattern.\n\n* Start with 12.\n\n* To get to the next number, multiply by 4 then takeaway 8.\n\nWhat is the sum of the first three numbers in Joyce's pattern?"},{"varval":"First write down the pattern:\n\n<div class=\"aligned\">\n\n>>| | | |\n| ----------------: | ---------------------- |--------- |\n| First number | \\= 12 ||\n| Second number | \\= 4 $\\times$ 12 $-$ 8| \\= 40 |\n| Third number|\\= 4 $\\times$ 40 $-$ 8| \\= 152 |\n\n</div>\n\n<br>Then find the sum of the numbers\n\n<div class=\"aligned\">\n\n>>| | | \n| ----------------: | ---------------------- |\n| Sum of first 3 terms | \\= 12 + 40 + 152 = {{{correctAnswer0}}}|\n\n</div>"}]}]

  6. Variation: Algebra, NAPX-F4-CA13, NAPX-F3-CA20

    <div class="sm_mode"> {{{question}}} </div>

    [{"vars":[{"varval":"Flame picked a total of 350 kilograms of pumpkins in 5 days.\n\nAfter the first day, she picked 20 more kilograms than the previous day.\n\nHow may kilograms of pumpkins did she pick on the first day?"},{"varval":"Average kilograms per day\n\n>= 350 $\\div$ 5\n\n>= 70\n\n<br>\n\n$\\Rightarrow$ 70 will be midpoint of the 5 kilogram values\n\n\n\n<br>\n\nKilograms each day were:\n\n>30, 50, 70, 90, 110\n\n<br>\n\n$\\therefore$ Kilograms of pumpkins picked on 1st day = {{{correctAnswer}}}"}]},{"vars":[{"varval":"Lenny walked a total of 400 kilometres 5 months.\n\nFor each month after the first month, she walked 30 more kilometres than the previous month.\n\nHow may kilometres did she walk in the first month?"},{"varval":"Average kilometres per month\n\n>= 400 $\\div$ 5\n\n>= 80\n\n<br>\n\n$\\Rightarrow$ 80 will be midpoint of the 5 kilometre values\n\n\n\n<br>\n\nkilometres walked each month were:\n\n>20, 50, 80, 110, 140\n\n<br>\n\n$\\therefore$ Kilometres walked during 1st month = {{{correctAnswer}}}"}]},{"vars":[{"varval":"Ursula did 600 burpees during a 5 day challenge at her gym.\n\nFor each day after the first day, she did 50 more burpees than the previous day.\n\nHow may burpees did she do on the first day of the challenge?"},{"varval":"Average burpees per day\n\n>= 600 $\\div$ 5\n\n>= 120\n\n<br>\n\n$\\Rightarrow$ 120 will be midpoint of the 5 burpee values\n\n\n\n<br>\n\nBurpees each day were:\n\n>20, 70, 120, 170, 220\n\n<br>\n\n$\\therefore$ Number of burpees on 1st day = {{{correctAnswer}}}"}]},{"vars":[{"varval":"Morris ran 400 kilometres during a 5 week challenge at his gym.\n\nFor each week after the first week, he ran 15 more kilometres than he had run the previous week.\n\nHow may kilometres did he run during the second week?"},{"varval":"Average kilometres per week\n\n>= 400 $\\div$ 5\n\n>= 80\n\n<br>\n\n$\\Rightarrow$ 80 will be midpoint of the 5 kilometre values\n\n\n\n<br>\n\nKilometres each week were:\n\n>50, 65, 80, 95, 110\n\n<br>\n\n$\\therefore$ Kilometres run during 2nd week = {{{correctAnswer}}}"}]},{"vars":[{"varval":"Keigan rode 800 kilometres during a 5 week cycling challenge.\n\nFor each week after the first week, he rode 30 more kilometres than he had run the previous week.\n\nHow may kilometres did he ride during the fifth week?"},{"varval":"Average kilometres per week\n\n>= 800 $\\div$ 5\n\n>= 160\n\n<br>\n\n$\\Rightarrow$ 160 will be midpoint of the 5 kilometre values\n\n\n\n<br>\n\nKilometres each week were:\n\n>100, 130, 160, 190, 220\n\n<br>\n\n$\\therefore$ Kilometres run during 5th week = {{{correctAnswer}}}"}]},{"vars":[{"varval":"Kirrily baked a total of 500 cookies in 5 days.\n\nEach day after the first day, she baked 20 more cookies than the previous day.\n\nHow may cookies did she bake on the fourth day?"},{"varval":"Average cookies per day\n\n>= 500 $\\div$ 5\n\n>= 100\n\n<br>\n\n$\\Rightarrow$ 100 will be midpoint of the 5 cookie values\n\n\n\n<br>\n\nCookies each day were:\n\n>60, 80, 100, 120, 140\n\n<br>\n\n$\\therefore$ Cookies baked on 4th day = {{{correctAnswer}}}"}]}]

  7. Algebra, NAP_10707 with number plane grid

    <div class="sm_mode"> {{{question}}} </div>

    [{"vars":[{"varval":"Autumn is drawing parallelograms on a number plane without crossing the sides of any other parallelograms.\n\nShe has 3 more points to join to form another parallelogram.\n\nsm_img https://teacher.smartermaths.com.au/wp-content/uploads/2023/08/Algebra-NAP_10095v0q-min.svg 450 indent2 vpad\n\nTwo of the points are ( 3 , 2 ) and ( 8 , 4 ).\n\nWhich of the following is the third point Autumn must connect?\n"},{"varval":"By connecting the points ( 3 , 2 ), **{{correctAnswer}}** and ( 8 , 4 ) we obtain another parallelogram. \n\nsm_img https://teacher.smartermaths.com.au/wp-content/uploads/2023/08/Algebra-NAP_10095v0-min.svg 450 indent vpad"}]},{"vars":[{"varval":"Summer is drawing parallelograms on a number plane without crossing the sides of any other parallelograms.\n\nShe has 3 more points to join to form another parallelogram.\n\nsm_img https://teacher.smartermaths.com.au/wp-content/uploads/2023/08/Algebra-NAP_10095v0q-min.svg 450 indent2 vpad\n\nTwo of the points are ( 4 , 0 ) and ( 9 , 2 ).\n\nWhich of the following is the third point Summer must connect?"},{"varval":"By connecting the points ( 4 , 0 ), **{{correctAnswer}}** and ( 9 , 2 ) we obtain another parallelogram. \n\nsm_img https://teacher.smartermaths.com.au/wp-content/uploads/2023/08/Algebra-NAP_10095v1ws-min.svg 450 indent vpad"}]},{"vars":[{"varval":"Winter is drawing parallelograms on a number plane without crossing the sides of any other parallelograms.\n\nShe has 3 more points to join to form another parallelogram.\n\nsm_img https://teacher.smartermaths.com.au/wp-content/uploads/2023/08/Algebra-NAP_10095v2q.svg 450 indent2 vpad\n\nTwo of the points are ( 3 , 1 ) and ( 7 , 3 ).\n\nWhich of the following is the third point Winter must connect?"},{"varval":"By connecting the points ( 3 , 1 ), **{{correctAnswer}}** and ( 7 , 3 ) we obtain another parallelogram. \n\nsm_img https://teacher.smartermaths.com.au/wp-content/uploads/2023/08/Algebra-NAP_10095v2ws.svg 450 indent vpad"}]},{"vars":[{"varval":"April is drawing parallelograms on a number plane without crossing the sides of any other parallelograms.\n\nShe has 3 more points to join to form another parallelogram.\n\nsm_img https://teacher.smartermaths.com.au/wp-content/uploads/2023/08/Algebra-NAP_10095v2q.svg 450 indent2 vpad\n\n\nTwo of the points are ( 2 , 3 ) and ( 6 , 5 ).\n\nWhich of the following is the third point April must connect?"},{"varval":"By connecting the points ( 2 , 3 ), **{{correctAnswer}}** and ( 6 , 5 ) we obtain another parallelogram. \n\nsm_img https://teacher.smartermaths.com.au/wp-content/uploads/2023/08/Algebra-NAP_10095v3ws-min.svg 450 indent vpad"}]},{"vars":[{"varval":"Didier is drawing rectangles on a number plane without crossing the sides of any other rectangles .\n\nHe has 3 more points to join to form another rectangle.\n\nsm_img https://teacher.smartermaths.com.au/wp-content/uploads/2023/08/Algebra-NAP_10095v4-5q-min.svg 420 indent2 vpad\n\nTwo of the points are ( $-$ 2 , 1 ) and ( 1 , 2 ).\n\nWhich of the following is the third point Didier must connect?"},{"varval":"By connecting the points ( $-$ 2 , 1 ), **{{correctAnswer}}** and ( 1 , 2 ) we obtain another rectangle. \n\nsm_img https://teacher.smartermaths.com.au/wp-content/uploads/2023/08/Algebra-NAP_10095v4ws-min.svg 420 indent vpad"}]},{"vars":[{"varval":"Marley is drawing rectangles on a number plane without crossing the sides of any other rectangles .\n\nShe has 3 more points to join to form another rectangle.\n\nsm_img https://teacher.smartermaths.com.au/wp-content/uploads/2023/08/Algebra-NAP_10095v4-5q-min.svg 420 indent2 vpad\n\nTwo of the points are ( $-$ 3 , $-$ 2 ) and ( 2 , $-$ 3 ).\n\nWhich of the following is the third point Marley must connect?"},{"varval":"By connecting the points ( $-$ 3 , $-$ 2 ), **{{correctAnswer}}** and ( 2 , $-$ 3 ) we obtain another rectangle. \n\nsm_img https://teacher.smartermaths.com.au/wp-content/uploads/2023/08/Algebra-NAP_10095v5ws-min.svg 420 indent vpad"}]},{"vars":[{"varval":"Bellamy is drawing parallelograms on a number plane without crossing the sides of any other parallelograms.\n\nHe has 3 more points to join to form another parallelogram.\n\nsm_img https://teacher.smartermaths.com.au/wp-content/uploads/2023/08/Algebra-NAP_10095v6q-min.svg 420 indent2 vpad\n\nTwo of the points are ( $-$ 4 , $-$ 1 ) and ( 2 , 0 ).\n\nWhich of the following is the third point Bellamy must connect?"},{"varval":"By connecting the points ( $-$ 4 , $-$ 1 ), **{{correctAnswer}}** and ( 2 , 0 ) we obtain another parallelogram. \n\n\nsm_img https://teacher.smartermaths.com.au/wp-content/uploads/2023/08/Algebra-NAP_10095v6ws-min.svg 420 indent vpad"}]},{"vars":[{"varval":"Bridgette is drawing parallelograms on a number plane without crossing the sides of any other parallelograms.\n\nShe has 3 more points to join to form another parallelogram.\n\nsm_img https://teacher.smartermaths.com.au/wp-content/uploads/2023/08/Algebra-NAP_10095v7ws-min.svg 420 indent2 vpad\n\n\nTwo of the points are ( $-$ 4 , $-$ 1 ) and ( $-$ 1 , 4 ).\n\nWhich of the following is the third point Bridgette must connect?"},{"varval":"By connecting the points ( $-$ 4 , $-$ 1 ), **{{correctAnswer}}** and ( $-$ 1 , 4 ) we obtain another parallelogram. \n\nsm_img https://teacher.smartermaths.com.au/wp-content/uploads/2023/08/Algebra-NAP_10095v6ws-min.svg 420 indent vpad"}]}]

  8. Algebra, NAPX-F3-CA18

    <div class="sm_mode"> {{{question}}} </div>

    [{"vars":[{"varval":"Ignatius used this rule to make a number pattern.\n\n\n* Start with 1.\n\n\n* To get the next number, multiply by 2 then add 3.\n\n<br>\n\nThe first four numbers in his pattern were 1 , 5 , 13 , 29.\n\n\nWhat was the sixth number in Ignatius' pattern?"},{"varval":"Fifth number\n\n>29 × 2 + 3 = 58 + 3 = 61\n\n<br>\n\n\nSixth number\n\n>61 × 2 + 3 = 122 + 3 = {{{correctAnswer}}}"}]},{"vars":[{"varval":"Mo used this rule to make a number pattern.\n\n\n* Start with $-$ 2.\n\n\n* To get the next number, add 3 then multiply the answer by 2.\n\n<br>\n\nThe first four numbers in her pattern were $-$ 2 , 2 , 10 , 26 .\n\n\nWhat was the sixth number in Mo's pattern?"},{"varval":"Fifth number\n\n>(26 + 3) × 2 = 29 × 2 = 58\n\n<br>\n\n\nSixth number\n\n>(58 + 3) × 2 = 61 × 2 = {{{correctAnswer}}}"}]},{"vars":[{"varval":"Brock used this rule to make a number pattern.\n\n\n* Start with 6.\n\n\n* To get the next number, multiply by 2 then subtract 5.\n\n<br>\n\nThe first four numbers in his pattern were 6 , 7 , 9 , 13 .\n\n\nWhat was the sixth number in Brock's pattern?"},{"varval":"Fifth number\n\n>13 × 2 $-$ 5 = 26 $-$ 5 = 21\n\n<br>\n\n\nSixth number\n\n>21 × 2 $-$ 5 = 42 $-$ 5 = {{{correctAnswer}}}"}]},{"vars":[{"varval":"Jeddah used this rule to make a number pattern.\n\n\n* Start with $-$ 2.\n\n\n* To get the next number, multiply by 2 then add 4.\n\n<br>\n\nThe first four numbers in her pattern were $-$ 2 , 0 , 4 , 12 .\n\n\nWhat was the sixth number in Jeddah's pattern?"},{"varval":"Fifth number\n\n>12 × 2 + 4 = 24 + 4 = 28\n\n<br>\n\n\nSixth number\n\n>28 × 2 + 4 = 56 + 4 = {{{correctAnswer}}}"}]},{"vars":[{"varval":"Ted used this rule to make a number pattern.\n\n\n* Start with $-$ 5.\n\n\n* To get the next number, multiply by 2 then add 6.\n\n<br>\n\nThe first four numbers in his pattern were $-$ 5 , $-$ 4 , $-$ 2 , 2 .\n\n\nWhat was the sixth number in Ted's pattern?"},{"varval":"Fifth number\n\n>2 × 2 + 6 = 2 + 6 = 10\n\n<br>\n\n\nSixth number\n\n>10 × 2 + 6 = 20 + 6 = {{{correctAnswer}}}"}]}]

  9. Algebra, NAPX-J4-CA07, NAPX-J3-CA15

    <div class="sm_mode"> {{{question}}} </div>

    [{"vars":[{"varval":"Mia writes the sequence:\n\n\n>>$40,0.4,0.004,0.00004, …$\n\n<br>\nDescribe the rule she uses to write the next number in this sequence.\n"},{"varval":"<div class=\"aligned\">\n\n| | |\n| ------------: | ---------- |\n| 40 ÷ 100 | \\= 0.4 |\n| 0.4 ÷ 100 | \\= 0.004 |\n| 0.004 ÷ 100 | \\= 0.00004 |\n\n</div>\n\n<br>\n\n$\\therefore$ Rule: {{{correctAnswer}}}"}]},{"vars":[{"varval":"Thomas writes the sequence:\n\n\n>>$18,1.8,0.18,0.018, …$\n\n<br>\nDescribe the rule he uses to write the next number in this sequence."},{"varval":"<div class=\"aligned\">\n\n| | |\n| ------------: | ---------- |\n| 18 ÷ 10 | \\= 1.8 |\n| 1.8 ÷ 10 | \\= 0.18 |\n| 0.18 ÷ 10 | \\= 0.018 |\n\n</div>\n\n<br>\n\n$\\therefore$ Rule: {{{correctAnswer}}}"}]},{"vars":[{"varval":"Happy writes the sequence:\n\n\n>>$0.0024,0.024,0.24,2.4, …$\n\n<br>\nDescribe the rule he uses to write the next number in this sequence."},{"varval":"<div class=\" aligned\">\n\n| | |\n| ------------: | ---------- |\n| 0.0024 $\\times$ 10 | \\= 0.024 |\n| 0.024 $\\times$ 10 | \\= 0.24 |\n| 0.24 $\\times$ 10 | \\= 2.4 |\n\n</div>\n\n<br>\n\n$\\therefore$ Rule: {{{correctAnswer}}}"}]},{"vars":[{"varval":"Jacques writes the sequence:\n\n\n>>$890,8.9,0.089,0.00089, …$\n\n<br>\nDescribe the rule he uses to write the next number in this sequence."},{"varval":"<div class=\"aligned\">\n\n| | |\n| ------------: | ---------- |\n| 890 ÷ 100 | \\= 8.9 |\n| 8.9 ÷ 100 | \\= 0.089 |\n| 0.089 ÷ 100 | \\= 0.00089 |\n\n</div>\n\n<br>\n\n$\\therefore$ Rule: {{{correctAnswer}}}"}]},{"vars":[{"varval":"Bernard writes the sequence:\n\n\n>>$0.00202,0.0202,0.202,2.02, …$\n\n<br>\nDescribe the rule he uses to write the next number in this sequence."},{"varval":"<div class=\"aligned\">\n\n| | |\n| ------------: | ---------- |\n| 0.00202 $\\times$ 10 | \\= 0.0202 |\n| 0.0202 $\\times$ 10 | \\= 0.202 |\n| 0.202 $\\times$ 10 | \\= 2.02 |\n\n</div>\n\n<br>\n\n$\\therefore$ Rule: {{{correctAnswer}}}"}]},{"vars":[{"varval":"Heathcliff writes the sequence:\n\n\n>>$1.01,0.101,0.0101,0.00101, …$\n\n<br>\nDescribe the rule he uses to write the next number in this sequence."},{"varval":"<div class=\"aligned\">\n\n| | |\n| ------------: | ---------- |\n| 1.01 ÷ 10 | \\= 0.101 |\n| 0.101÷ 10 | \\= 0.0101 |\n| 0.0101 ÷ 10 | \\= 0.00101 |\n\n</div>\n\n<br>\n\n$\\therefore$ Rule: Divide by 10 = Multiply by $\\frac{1}{10}$ = {{{correctAnswer}}}"}]}]

  10. <div class="sm_mode"> {{{question}}} </div>

    [{"vars":[{"varval":"Reba is taking part in a push-up challenge that lasts for 7 days.\n\nOn the first day he completes 22 push-ups. Every day after this he intends to increase the number of push-ups he does by 8.\n\nHow many push-ups will Reba do on the last day of the challenge?"},{"varval":"Strategy 1:\n\nConsider the push-ups Reba completes each day\n\n>Number of push-ups Day 1 = 22\n\n>Number of push-ups Day 2 = 22 + 8 = 30\n\n>Number of push-ups Day 3 = 30 + 8 = 38\n\n>Number of push-ups Day 4 = 38 + 8 = 46\n\n>Number of push-ups Day 5 = 46 + 8 = 54\n\n>Number of push-ups Day 6 = 54 + 8 = 62\n\n>Number of push-ups Day 7 = 62 + 8 = {{{correctAnswer}}}\n\n<br>Therefore, Reba completed {{{correctAnswer}}} push-ups on the seventh day.\n\nStrategy 2: (Advanced)\n\n<div class=\"aligned\">\n\n| | |\n| ------------- | ---------- |\n| Push-ups | = 22 + 8 $\\times$ (day number - 1) |\n| | = 22 + 8 $\\times$ (7 $-$ 1) |\n| | = 22 + 8 $\\times$ 6 |\n| | \\= {{{correctAnswer}}} |\n\n</div>\n<br>Therefore, Reba completed {{{correctAnswer}}} push-ups on the seventh day.\n\n\n\n\n\n"}]},{"vars":[{"varval":"Huxley is taking part in a running challenge that lasts for 8 weeks.\n\nDuring the first week he runs a total of 30 kilometres. Every week after that he intends to increase the number of kilometres he runs by 4.\n\nHow many kilometres will Huxley run on the last day of the challenge?"},{"varval":"Strategy 1:\n\nConsider the kilometres Huxley completes each week\n\n>Number of kilometres week 1 = 30\n\n>Number of kilometres week 2 = 30 + 4 = 34\n\n>Number of kilometres week 3 = 34 + 4 = 38\n\n>Number of kilometres week 4 = 38 + 4 = 42\n\n>Number of kilometres week 5 = 42 + 4 = 46\n\n>Number of kilometres week 6 = 46 + 4 = 50\n\n>Number of kilometres week 7 = 50 + 4 = 54\n\n>Number of kilometres week 8 = 54 + 4 = {{{correctAnswer}}}\n\n<br>Therefore, Huxley completed {{{correctAnswer}}} kilometres during the 8th week.\n\nStrategy 2: (Advanced)\n\n<div class=\"aligned\">\n\n| | |\n| ------------- | ---------- |\n| Kilometres | = 30 + 4 $\\times$ (week number - 1) |\n| | = 30 + 4 $\\times$ (8 $-$ 1) |\n| | = 30 + 4 $\\times$ 7 |\n| | \\= {{{correctAnswer}}} |\n\n</div>\n<br>Therefore, Huxley completed {{{correctAnswer}}} kilometres during the 8th week."}]},{"vars":[{"varval":"Burt is saving to go on a holiday in 6 months time.\n\nHe already has $470 saved and intends to add a futher $140 each month for the next 6 months.\n\nHow much money will Burt have saved, in total, by the time he goes on holidays?"},{"varval":"Strategy 1:\n\nConsider the amount Burt saves each month\n\n>Amount already saved = $470\n\n>Amount saved after 1 month = 470 + 140 = 610\n\n>Amount saved after 2 months = 610 + 140 = 750\n\n>Amount saved after 3 months = 750 + 140 = 890\n\n>Amount saved after 4 months = 890 + 140 = 1030\n\n>Amount saved after 5 months = 1030 + 140 = 1170\n\n>Amount saved after 6 months = 1170 + 140 = {{{correctAnswer}}}\n\n<br>Therefore, Burt has saved {{{correctAnswer}}} for his holiday.\n\nStrategy 2: (Advanced)\n\n<div class=\"aligned\">\n\n| | |\n| ------------- | ---------- |\n| Savings | = 470 + 140 $\\times$ month number |\n| | = 470 + 140 $\\times$ 6 |\n| | = 470 + 840 |\n| | \\= {{{correctAnswer}}} |\n\n</div>\n<br>Therefore, Burt has saved {{{correctAnswer}}} for his holiday."}]},{"vars":[{"varval":"Betty is taking part in a walking challenge that lasts for 5 weeks.\n\nDuring the first week she walks 37 kilometres. Every week after this she intends to increase the number of kilometres she walks by 3.\n\nHow many kilometres will Betty walk during the last week of the challenge?"},{"varval":"Strategy 1:\n\nConsider the kilometres Betty walks each week\n\n>Number of kilometres week 1 = 37\n\n>Number of kilometres week 2 = 37 + 3 = 40\n\n>Number of kilometres week 3 = 40 + 3 = 43\n\n>Number of kilometres week 4 = 43 + 3 = 46\n\n>Number of kilometres week 5 = 46 + 3 = {{{correctAnswer}}}\n\n<br>Therefore, Betty walked {{{correctAnswer}}} kilometres during the 5th week.\n\nStrategy 2: (Advanced)\n\n<div class=\"aligned\">\n\n| | |\n| ------------- | ---------- |\n| Kilometres | = 37 + 3 $\\times$ (week number - 1) |\n| | = 37 + 3 $\\times$ (5 $-$ 1) |\n| | = 37 + 3 $\\times$ 4 |\n| | \\= {{{correctAnswer}}} |\n\n</div>\n<br>Therefore, Betty walked {{{correctAnswer}}} kilometres during the 5th week."}]},{"vars":[{"varval":"George is taking part in a 100 metre sprinting challenge and he records his best 100 metre time each week for 6 weeks.\n\nDuring the first week he records a best time of 31 seconds. Every week after this he intends to reduce his weekly best time by 2 seconds.\n\nWhat should George's best time be during the last week of the challenge if he has achieved his goal?"},{"varval":"Strategy 1:\n\nConsider the times George records each week\n\n>Best time week 1 = 31\n\n>Best time week 2 = 31 $-$ 2 = 29\n\n>Best time week 3 = 29 $-$ 2 = 27\n\n>Best time week 4 = 27 $-$ 2 = 25\n\n>Best time week 5 = 25 $-$ 2 = 23\n\n>Best time week 6 = 23 $-$ 2 = {{{correctAnswer}}}\n\n<br>Therefore, George records a best time of {{{correctAnswer}}} seconds during the 6th week of the Challenge.\n\nStrategy 2: (Advanced)\n\n<div class=\"aligned\">\n\n| | |\n| ------------- | ---------- |\n| Push-ups | = 31 $-$ 2 $\\times$ (week number - 1) |\n| | = 31 $-$ 2 $\\times$ (6 $-$ 1) |\n| | = 31 $-$ 2 $\\times$ 5 |\n| | \\= {{{correctAnswer}}} |\n\n</div>\n<br>Therefore, George records a best time of {{{correctAnswer}}} seconds during the 6th week of the Challenge."}]}]

  11. <div class="sm_mode"> {{{question}}} </div>

    [{"vars":[{"varval":"Patience is setting out exam desks in the school hall using the pattern shown below.\n\nsm_img https://teacher.smartermaths.com.au/wp-content/uploads/2023/08/Algebra-NAP_20025v0-min.svg 550 indent vpad\n\nIf this pattern continues until there are six rows, what is the total number of exam desks in the hall when Patience has finished?"},{"varval":"The number of desks in each row is decreasing by 1\n\n>Desks in Row 1 = 8\n\n>Desks in Row 2 = 7\n\n>Desks in Row 3 = 6\n\n>Desks in Row 4 = 5\n\n>Desks in Row 5 = 4\n\n>Desks in Row 6 = 3\n\n<br>\n \nTherefore, the total number of desks = 8 + 7 + 6 + 5 + 4 + 3 = {{{correctAnswer}}}\n"}]},{"vars":[{"varval":"Arima is planting flowers in his garden using the pattern shown below.\n\nsm_img https://teacher.smartermaths.com.au/wp-content/uploads/2023/08/Algebra-NAP_20025v1-min.svg 550 indent vpad\n\nIf this pattern continues until there are seven rows, what is the total number of flowers in the garden when Arima is finished planting?"},{"varval":"The number of plants in each row is decreasing by 2\n\n>Plants in Row 1 = 14\n\n>Plants in Row 2 = 12\n\n>Plants in Row 3 = 10\n\n>Plants in Row 4 = 8\n\n>Plants in Row 5 = 6\n\n>Plants in Row 6 = 4\n\n>Plants in Row 7 = 2\n\n\n<br>\n \nTherefore, the total number of plants = 14 + 12 + 10 + 8 + 6 + 4 + 2 = {{{correctAnswer}}}"}]},{"vars":[{"varval":"Jansui is arranging a display of cakes using the pattern shown below.\n\nsm_img https://teacher.smartermaths.com.au/wp-content/uploads/2023/08/Algebra-NAP_20025v2-min.svg 550 indent vpad\n\nIf this pattern continues until there are 5 rows, what is the total number of cakes in the display when Jansui has finished?"},{"varval":"The number of cakes in each row is decreasing by 3\n\n>Cakes in Row 1 = 13\n\n>Cakes in Row 2 = 10\n\n>Cakes in Row 3 = 7\n\n>Cakes in Row 4 = 4\n\n>Cakes in Row 5 = 1\n\n<br>\n \nTherefore, the total number of Cakes = 13 + 10 + 7 + 4 + 1 = {{{correctAnswer}}}"}]},{"vars":[{"varval":"Nat is arranging band members for the parade using a pattern shown below.\n\nsm_img https://teacher.smartermaths.com.au/wp-content/uploads/2023/08/Algebra-NAP_20025v3-min.svg 550 indent vpad\n\nIf this pattern continues until there are six rows, what is the total number of band members in the parade when Nat has finished?"},{"varval":"The number of band members in each row is decreasing by 2\n\n>Band members in Row 1 = 13\n\n>Band members in Row 2 = 11\n\n>Band members in Row 3 = 9\n\n>Band members in Row 4 = 7\n\n>Band members in Row 5 = 5\n\n>Band members in Row 6 = 3\n\n<br>\n \nTherefore, the total number of desks = 13 + 11 + 9 + 7 + 5 + 3 = {{{correctAnswer}}}"}]},{"vars":[{"varval":"Faith is planting Christmas trees in rows using the pattern shown below.\n\nsm_img https://teacher.smartermaths.com.au/wp-content/uploads/2023/08/Algebra-NAP_20025v4a-min.svg 650 indent vpad\n\nIf this pattern continues until there are five rows, what is the total number of Christmas trees Faith has planted?"},{"varval":"The number of Christmas trees in each row is decreasing by 5\n\n>Christmas trees in Row 1 = 21\n\n>Christmas trees in Row 2 = 16\n\n>Christmas trees in Row 3 = 11\n\n>Christmas trees in Row 4 = 6\n\n>Christmas trees in Row 5 = 1\n\n<br>\n \nTherefore, the total number of Singers = 21 + 16 + 11 + 6 + 1 = {{{correctAnswer}}}"}]}]

  12. Algebra, NAPX-G3-NC08

    <div class="sm_mode"> {{{question}}} </div>

    [{"vars":[{"varval":"Cata uses sticks to make a pattern.\n\nShe starts with 2 sticks for Design 1.\n\nsm_img https://teacher.smartermaths.com.au/wp-content/uploads/2023/08/Algebra-NAP_20021v1q-min.svg 725 indent vpad\n\nHow many sticks does Cata need for Design 5?"},{"varval":"Strategy One\n\nDrawing design 5:\n\n\nsm_img https://teacher.smartermaths.com.au/wp-content/uploads/2023/08/Algebra-NAP_20021v1ws-min.svg 135 indent vpad\n\n∴ {{{correctAnswer}}} sticks are required for design 5 \n\n<br>\n\n\n\nStrategy Two\n\n\n>Design 2 has 4 more sticks than design 1.\n\n>Design 3 has 6 more sticks than design 2.\n\n>Design 4 has 8 more sticks than design 3.\n\n<br>Continuing this pattern,\n\n>Design 5 will have 10 more sticks than design 4 = 20 + 10 = {{{correctAnswer}}} sticks.\n"}]},{"vars":[{"varval":"Alexia uses sticks to make a pattern.\n\nShe starts with 3 sticks for Design 1.\n\nsm_img https://teacher.smartermaths.com.au/wp-content/uploads/2023/08/Algebra-NAP_20021v0q-min.svg 725 indent vpad\n\nHow many sticks does Alexia need for Design 5?"},{"varval":"Strategy One\n\nDrawing design 5:\n\n\nsm_img https://teacher.smartermaths.com.au/wp-content/uploads/2023/08/Algebra-NAP_20021v0ws-min.svg 135 indent vpad\n\n∴ {{{correctAnswer}}} sticks are required for design 5 \n\n<br>\n\n\n\nStrategy Two\n\n\n>Design 2 has 6 more sticks than design 1.\n\n>Design 3 has 9 more sticks than design 2.\n\n>Design 4 has 12 more sticks than design 3.\n\n<br>Continuing this pattern,\n\n>Design 5 will have 15 more sticks than design 4 = 30 + 15 = {{{correctAnswer}}} sticks.\n"}]},{"vars":[{"varval":"Felix uses sticks to make a pattern.\n\nHe starts with 3 sticks for Design 1.\n\nsm_img https://teacher.smartermaths.com.au/wp-content/uploads/2023/08/Algebra-NAP_20021v2q-min.svg 725 indent vpad\n\nHow many sticks does Felix need for Design 5?"},{"varval":"Strategy One\n\nDrawing design 5:\n\n\nsm_img https://teacher.smartermaths.com.au/wp-content/uploads/2023/08/Algebra-NAP_20021v2ws-min.svg 180 indent vpad\n\n∴ {{{correctAnswer}}} sticks are required for design 5 \n\n<br>\n\n\n\nStrategy Two\n\n\n>Design 2 has 6 more sticks than design 1.\n\n>Design 3 has 9 more sticks than design 2.\n\n>Design 4 has 12 more sticks than design 3.\n\n<br>Continuing this pattern,\n\n>Design 5 will have 15 more sticks than design 4 = 30 + 15 = {{{correctAnswer}}} sticks.\n"}]},{"vars":[{"varval":"Hugh uses sticks to make a pattern.\n\nHe starts with 4 sticks for Design 1.\n\nsm_img https://teacher.smartermaths.com.au/wp-content/uploads/2023/08/Algebra-NAP_20021v3q-min.svg 725 indent vpad\n\nHow many sticks does Hugh need for Design 5?"},{"varval":"Strategy One\n\nDrawing design 5:\n\n\nsm_img https://teacher.smartermaths.com.au/wp-content/uploads/2023/08/Algebra-NAP_20021v3ws-min.svg 180 indent vpad\n\n∴ {{{correctAnswer}}} sticks are required for design 5 \n\n<br>\n\n\n\nStrategy Two\n\n\n>Design 2 has 8 more sticks than design 1.\n\n>Design 3 has 12 more sticks than design 2.\n\n>Design 4 has 16 more sticks than design 3.\n\n<br>Continuing this pattern,\n\n>Design 5 will have 20 more sticks than design 4 = 40 + 20 = {{{correctAnswer}}} sticks."}]},{"vars":[{"varval":"Aaron uses sticks to make a pattern.\n\nHe starts with 5 sticks for Design 1.\n\nsm_img https://teacher.smartermaths.com.au/wp-content/uploads/2023/08/Algebra-NAP_20021v4q-min.svg 725 indent vpad\n\nHow many sticks does Aaron need for Design 5?"},{"varval":"Strategy One\n\nDrawing design 5:\n\n\nsm_img https://teacher.smartermaths.com.au/wp-content/uploads/2023/08/Algebra-NAP_20021v4ws-min.svg 180 indent vpad\n\n∴ {{{correctAnswer}}} sticks are required for design 5 \n\n<br>\n\n\n\nStrategy Two\n\n\n>Design 2 has 10 more sticks than design 1.\n\n>Design 3 has 15 more sticks than design 2.\n\n>Design 4 has 20 more sticks than design 3.\n\n<br>Continuing this pattern,\n\n>Design 5 will have 25 more sticks than design 4 = 50 + 25 = {{{correctAnswer}}} sticks."}]}]

  13. <div class="sm_mode"> {{{question}}} </div>

    [{"vars":[{"varval":"If the pattern below continues, what is the next image in the pattern?\n\n<br>\n\nsm_img https://teacher.smartermaths.com.au/wp-content/uploads/2023/08/Algebra-NAP_19001v01-min.svg 600 indent vpad\n"},{"varval":"{{{correctAnswer}}}"}]},{"vars":[{"varval":"If the pattern below continues, what is the next image in the pattern?\n\n<br>\n\nsm_img https://teacher.smartermaths.com.au/wp-content/uploads/2023/08/Algebra-NAP_19001v1-min.svg 600 indent vpad"},{"varval":"{{{correctAnswer}}}"}]},{"vars":[{"varval":"If the pattern below continues, what is the next image in the pattern?\n\n<br>\n\nsm_img https://teacher.smartermaths.com.au/wp-content/uploads/2023/08/Algebra-NAP_19001v2-min.svg 600 indent vpad"},{"varval":"{{{correctAnswer}}}"}]},{"vars":[{"varval":"If the pattern below continues, what is the next image in the pattern?\n\n<br>\n\nsm_img https://teacher.smartermaths.com.au/wp-content/uploads/2023/08/Algebra-NAP_19001v3-min.svg 600 indent vpad"},{"varval":"{{{correctAnswer}}}"}]},{"vars":[{"varval":"If the pattern below continues, what is the next image in the pattern?\n\n<br>\n\nsm_img https://teacher.smartermaths.com.au/wp-content/uploads/2023/08/Algebra-NAP_19001v4-min.svg 600 indent vpad"},{"varval":"{{{correctAnswer}}}"}]},{"vars":[{"varval":"If the pattern below continues, what is the next image in the pattern?\n\n<br>\n\nsm_img https://teacher.smartermaths.com.au/wp-content/uploads/2023/08/Algebra-NAP_19001v5-min.svg 600 indent vpad"},{"varval":"{{{correctAnswer}}}"}]}]

  14. <div class="sm_mode"> {{{question}}} </div>

    [{"vars":[{"varval":"Ferris is writing down a number pattern.\n\n>23, 22, 20, 17, <span class=\"sm-text color1\">?</span>\n\n<br>\n\nWhat number should Ferris write down next?"},{"varval":"The amount being subtracted each time is increasing by 1.\n<div class=\"aligned\">\n\n||||\n|-:|-|-|\n|1st number|= 23||\n|2nd number|= 23 $-$ **1**| = 22|\n|3rd number|= 22 $-$ **2**| = 20|\n|4th number|= 20 $-$ **3**| = 17 |\n|<span class=\"sm-text color1\">?</span>| = 17 $-$ **4**| = {{{correctAnswer}}}|\n\n</div>"}]},{"vars":[{"varval":"Prince is writing down a number pattern.\n\n>35, 30, 24, 17, <span class=\"sm-text color2\">?</span>\n\n<br>\n\nWhat number should Prince write down next?"},{"varval":"The amount being subtracted each time is increasing by 1.\n<div class=\"aligned\">\n\n||||\n|-:|-|-|\n|1st number|= 35||\n|2nd number|= 35 $-$ **5**| = 30|\n|3rd number|= 30 $-$ **6**| = 24|\n|4th number|= 24 $-$ **7**| = 17 |\n|<span class=\"sm-text color2\">?</span>| = 17 $-$ **8**| = {{{correctAnswer}}}|\n\n</div>"}]},{"vars":[{"varval":"Hugo is writing down a number pattern.\n\n>40, 35, 31, 28, <span class=\"sm-text color3\">?</span>\n\n<br>\n\nWhat number should Hugo write down next?"},{"varval":"The amount being subtracted each time is decreasing by 1.\n<div class=\"aligned\">\n\n||||\n|-:|-|-|\n|1st number|= 40||\n|2nd number|= 40 $-$ **5**| = 35|\n|3rd number|= 35 $-$ **4**| = 31|\n|4th number|= 31 $-$ **3**| = 28 |\n|<span class=\"sm-text color3\">?</span>| = 28 $-$ **2**| = {{{correctAnswer}}}|\n\n</div>"}]},{"vars":[{"varval":"Pat is writing down a number pattern.\n\n>60, 50, 41, 33, <span class=\"sm-text color4\">?</span>\n\n<br>\n\nWhat number should Pat write down next?"},{"varval":"The amount being subtracted each time is decreasing by 1.\n<div class=\"aligned\">\n\n||||\n|-:|-|-|\n|1st number|= 60||\n|2nd number|= 60 $-$ **10**| = 50|\n|3rd number|= 50 $-$ **9**| = 41|\n|4th number|= 41 $-$ **8**| = 33 |\n|<span class=\"sm-text color4\">?</span>| = 33 $-$ **7**| = {{{correctAnswer}}}|\n\n</div>"}]},{"vars":[{"varval":"Bjorn is writing down a number pattern.\n\n>102, 112, 132, 162, <span class=\"sm-text color2\">?</span>\n\n<br>\n\nWhat number should Bjorn write down next?"},{"varval":"The amount being added each time is increasing by 10.\n<div class=\"aligned\">\n\n>||||\n|-:|-|-|\n|1st number|= 102||\n|2nd number|= 102 + **10**| = 112|\n|3rd number|= 112 + **20**| = 132|\n|4th number|= 132 + **30**| = 162 |\n|<span class=\"sm-text color2\">?</span>| = 162 + **40**| = {{{correctAnswer}}}|\n\n</div>"}]},{"vars":[{"varval":"Isobel is writing down a number pattern.\n\n>43, 48, 58, 73, <span class=\"sm-text color3\">?</span>\n\n<br>\n\nWhat number should Isobel write down next?"},{"varval":"The amount being added each time is increasing by 5.\n<div class=\"aligned\">\n\n>||||\n|-:|-|-|\n|1st number|= 43||\n|2nd number|= 43 + **5**| = 48|\n|3rd number|= 48 + **10**| = 58|\n|4th number|= 58 + **15**| = 73 |\n|<span class=\"sm-text color3\">?</span>| = 73 + **20**| = {{{correctAnswer}}}|\n\n</div>"}]}]

  15. <div class="sm_mode"> {{{question}}} </div>

    [{"vars":[{"varval":"Horatio wrote down the following number pattern?\n\n>7, 8, 10, 13, <span class=\"sm-text color1\">?</span>\n\n<br>\n\nWhat is the number Horatio should write down next? "},{"varval":"The amount being added each time is increasing by one.\n<div class=\"aligned\">\n\n||||\n|-:|-|-|\n|1st number|= 7||\n|2nd number|= 7 + **1**| = 8|\n|3rd number|= 8 + **2**| = 10|\n|4th number|= 10 + **3**| = 13 |\n|<span class=\"sm-text color1\">?</span>| = 13 + **4**| = {{{correctAnswer}}}|\n\n</div>"}]},{"vars":[{"varval":"Georgio wrote down the following number pattern?\n\n>26, 27, 29, 32, <span class=\"sm-text color2\">?</span>\n\n<br>\n\nWhat is the number Georgio should write down next? "},{"varval":"The amount being added each time is increasing by one.\n<div class=\"aligned\">\n\n||||\n|-:|-|-|\n|1st number|= 26||\n|2nd number|= 26 + **1**| = 27|\n|3rd number|= 27 + **2**| = 29|\n|4th number|= 29 + **3**| = 32 |\n|<span class=\"sm-text color2\">?</span>| = 32 + **4**| = {{{correctAnswer}}}|\n\n</div>"}]},{"vars":[{"varval":"Claudio wrote down the following number pattern?\n\n>18, 19, 21, 24, <span class=\"sm-text color3\">?</span>\n\n<br>\n\nWhat is the number Claudio should write down next? "},{"varval":"The amount being added each time is increasing by one.\n<div class=\"aligned\">\n\n||||\n|-:|-|-|\n|1st number|= 18||\n|2nd number|= 18 + **1**| = 19|\n|3rd number|= 19 + **2**| = 21|\n|4th number|= 21 + **3**| = 24 |\n|<span class=\"sm-text color3\">?</span>| = 24 + **4**| = {{{correctAnswer}}}|\n\n</div>"}]},{"vars":[{"varval":"Luigi wrote down the following number pattern?\n\n>9, 11, 15, 21, <span class=\"sm-text color4\">?</span>\n\n<br>\n\nWhat is the number Luigi should write down next? "},{"varval":"The amount being added each time is increasing by 2.\n<div class=\"aligned\">\n\n||||\n|-:|-|-|\n|1st number|= 9||\n|2nd number|= 9 + **2**| = 11|\n|3rd number|= 11 + **4**| = 15|\n|4th number|= 15 + **6**| = 21 |\n|<span class=\"sm-text color4\">?</span>| = 21 + **8**| = {{{correctAnswer}}}|\n\n</div>"}]},{"vars":[{"varval":"Juliet wrote down the following number pattern?\n\n>21, 23, 27, 33, <span class=\"sm-text color5\">?</span>\n\n<br>\n\nWhat is the number Juliet should write down next? "},{"varval":"The amount being added each time is increasing by 2.\n<div class=\"aligned\">\n\n||||\n|-:|-|-|\n|1st number|= 21||\n|2nd number|= 21 + **2**| = 23|\n|3rd number|= 23 + **4**| = 27|\n|4th number|= 27 + **6**| = 33 |\n|<span class=\"sm-text color5\">?</span>| = 33 + **8**| = {{{correctAnswer}}}|\n\n</div>"}]},{"vars":[{"varval":"Angeline wrote down the following number pattern?\n\n>75, 77, 81, 87, <span class=\"sm-text color6\">?</span>\n\n<br>\n\nWhat is the number Angeline should write down next? "},{"varval":"The amount being added each time is increasing by 2.\n<div class=\"aligned\">\n\n||||\n|-:|-|-|\n|1st number|= 75||\n|2nd number|= 75 + **2**| = 77|\n|3rd number|= 77 + **4**| = 81|\n|4th number|= 81 + **6**| = 87 |\n|<span class=\"sm-text color6\">?</span>| = 87 + **8**| = {{{correctAnswer}}}|\n\n</div>"}]}]

  16. <div class="sm_mode"> {{{question}}} </div>

    [{"vars":[{"varval":"Johnny created a number pattern by adding four to a number to get the next number.\n\nWhich of these could be the first four numbers in Johnny's pattern?"},{"varval":"The next number is found by adding 4 to the previous number\n\n<div class=\"aligned\">\n\n||||\n|-:|-|-|\n|Starting number|= 18||\n|2nd number|= 18 + 4|= 22|\n|3rd number|= 22 + 4|= 26|\n|4th number|= 26 + 4|= 30|\n\n<br>$\\therefore$ Johnny's pattern is: &nbsp;{{{correctAnswer}}}\n\n</div>"}]},{"vars":[{"varval":"Pearl created a number pattern by adding seven to a number to get the next number.\n\nWhich of these could be the first four numbers in Pearl's pattern?"},{"varval":"The next number is found by adding 7 to the previous number\n\n<div class=\"aligned\">\n\n||||\n|-:|-|-|\n|Starting number|= 9||\n|2nd number|= 9 + 7|= 16|\n|3rd number|= 16 + 7|= 23|\n|4th number|= 23 + 7|= 30|\n\n<br>$\\therefore$ Pearl's pattern is: &nbsp;{{{correctAnswer}}}\n\n</div>"}]},{"vars":[{"varval":"Kym created a number pattern by adding nine to a number to get the next number.\n\nWhich of these could be the first four numbers in Kym's pattern?"},{"varval":"The next number is found by adding 9 to the previous number\n\n<div class=\"aligned\">\n\n||||\n|-:|-|-|\n|Starting number|= 23||\n|2nd number|= 23 + 9|= 32|\n|3rd number|= 32 + 9|= 41|\n|4th number|= 41 + 9|= 50|\n\n<br>$\\therefore$ Kim's pattern is: &nbsp;{{{correctAnswer}}}\n\n</div>"}]},{"vars":[{"varval":"Poh created a number pattern by adding five to a number to get the next number.\n\nWhich of these could be the first four numbers in Poh's pattern?"},{"varval":"The next number is found by adding 5 to the previous number\n\n<div class=\"aligned\">\n\n||||\n|-:|-|-|\n|Starting number|= 41||\n|2nd number|= 41 + 5|= 46|\n|3rd number|= 46 + 5|= 51|\n|4th number|= 51 + 5|= 56|\n\n<br>$\\therefore$ Poh's pattern is: &nbsp;{{{correctAnswer}}}\n\n</div>"}]},{"vars":[{"varval":"Letty created a number pattern by adding eight to a number to get the next number.\n\nWhich of these could be the first four numbers in Letty's pattern?"},{"varval":"The next number is found by adding 8 to the previous number\n\n<div class=\"aligned\">\n\n||||\n|-:|-|-|\n|Starting number|= 31||\n|2nd number|= 31 + 8|= 39|\n|3rd number|= 39 + 8|= 47|\n|4th number|= 47 + 8|= 55|\n\n<br>$\\therefore$ Letty's pattern is: &nbsp;{{{correctAnswer}}}\n\n</div>"}]},{"vars":[{"varval":"Neil created a number pattern by adding eleven to a number to get the next number.\n\nWhich of these could be the first four numbers in Neil's pattern?"},{"varval":"The next number is found by adding 11 to the previous number\n\n<div class=\"aligned\">\n\n||||\n|-:|-|-|\n|Starting number|= 31||\n|2nd number|= 18 + 4|= 42|\n|3rd number|= 22 + 4|= 53|\n|4th number|= 26 + 4|= 64|\n\n<br>$\\therefore$ Neil's pattern is: &nbsp;{{{correctAnswer}}}\n\n</div>"}]}]

  17. Algebra, NAPX-C1-14

    <div class="sm_mode"> {{{question}}} </div>

    [{"vars":[{"varval":"The flags below make a pattern.\n\nsm_img https://teacher.smartermaths.com.au/wp-content/uploads/2023/08/Algebra-NAP_40044v0a-min.svg 550 indent vpad\n\nWhich of these shows the order of the next four flags?"},{"varval":"{{{correctAnswer}}}\n"}]},{"vars":[{"varval":"The flags below make a pattern.\n\nsm_img https://teacher.smartermaths.com.au/wp-content/uploads/2023/08/Algebra-NAP_40044v1-min.svg 550 indent vpad\n\nWhich of these shows the order of the next four flags?"},{"varval":"{{{correctAnswer}}}"}]},{"vars":[{"varval":"The flags below make a pattern.\n\nsm_img https://teacher.smartermaths.com.au/wp-content/uploads/2023/08/Algebra-NAP_40044v2-min.svg 550 indent vpad\n\nWhich of these shows the order of the next four flags?"},{"varval":"{{{correctAnswer}}}"}]},{"vars":[{"varval":"The flags below make a pattern.\n\nsm_img https://teacher.smartermaths.com.au/wp-content/uploads/2023/08/Algebra-NAP_40044v3-min.svg 550 indent vpad\n\nWhich of these shows the order of the next four flags?"},{"varval":"{{{correctAnswer}}}"}]},{"vars":[{"varval":"The flags below make a pattern.\n\nsm_img https://teacher.smartermaths.com.au/wp-content/uploads/2023/08/Algebra-NAP_40044v4-min.svg 550 indent vpad\n\nWhich of these shows the order of the next four flags?"},{"varval":"{{{correctAnswer}}}"}]},{"vars":[{"varval":"The flags below make a pattern.\n\nsm_img https://teacher.smartermaths.com.au/wp-content/uploads/2023/08/Algebra-NAP_40044v5-min.svg 550 indent vpad\n\nWhich of these shows the order of the next four flags?"},{"varval":"{{{correctAnswer}}}"}]}]

  18. <div class="sm_mode"> {{{question}}} </div>

    [{"vars":[{"varval":"These numbers form a pattern.\n\n>> 17 , 21 , 25 , 29 , <span class=\"sm-text color1\">?</span> , <span class=\"sm-text color1\">?</span>\n\n\n<br>The next 2 numbers in the pattern are:"},{"varval":"The next term is found by adding 4 to the previous term\n\n<div class=\"aligned\">\n\n>>||||\n|-:|-|-|\n|Starting number|= 17||\n|2nd number|= 17 + 4|= 21|\n|3rd number|= 21 + 4|= 25|\n|4th number|= 25 + 4|= 29|\n|5th number|= 29 + 4|= 33|\n|6th number|= 33 + 4|= 37|\n\n<br>Therefore, the next 2 numbers are {{{correctAnswer}}}.\n\n</div>"}]},{"vars":[{"varval":"These numbers form a pattern.\n\n>> 23 , 20 , 17 , 14 , <span class=\"sm-text color2\">?</span> , <span class=\"sm-text color2\">?</span>\n\n\n<br>The next 2 numbers in the pattern are:"},{"varval":"The next term is found by subtracting 3 from the previous term\n\n<div class=\"aligned\">\n\n>>||||\n|-:|-|-|\n|Starting number|= 23||\n|2nd number|= 23 $-$ 3|= 20|\n|3rd number|= 20 $-$ 3|= 17|\n|4th number|= 17 $-$ 3|= 14|\n|5th number|= 14 $-$ 3|= 11|\n|6th number|= 11 $-$ 3|= 8|\n\n<br>Therefore, the next 2 numbers are {{{correctAnswer}}}.\n\n</div>"}]},{"vars":[{"varval":"These numbers form a pattern.\n\n>> 9 , 15 , 21 , 27 , <span class=\"sm-text color4\">?</span> , <span class=\"sm-text color4\">?</span>\n\n\n<br>The next 2 numbers in the pattern are:"},{"varval":"The next term is found by adding 6 to the previous term\n\n<div class=\"aligned\">\n\n>>||||\n|-:|-|-|\n|Starting number|= 9||\n|2nd number|= 9 + 6|= 15|\n|3rd number|= 15 + 6|= 21|\n|4th number|= 21 + 6|= 27|\n|5th number|= 27 + 6|= 33|\n|6th number|= 33 + 6|= 39|\n\n<br>Therefore, the next 2 numbers are {{{correctAnswer}}}.\n\n</div>"}]},{"vars":[{"varval":"These numbers form a pattern.\n\n>> 82 , 73 , 64 , 55 , <span class=\"sm-text color5\">?</span> , <span class=\"sm-text color5\">?</span>\n\n\n<br>The next 2 numbers in the pattern are:"},{"varval":"The next term is found by subtracting 9 from the previous term\n\n<div class=\"aligned\">\n\n>>||||\n|-:|-|-|\n|Starting number|= 82||\n|2nd number|= 82 $-$ 9|= 73|\n|3rd number|= 73 $-$ 9|= 64|\n|4th number|= 64 $-$ 9|= 55|\n|5th number|= 55 $-$ 9|= 46|\n|6th number|= 46 $-$ 9|= 37|\n\n<br>Therefore, the next 2 numbers are {{{correctAnswer}}}.\n\n</div>"}]},{"vars":[{"varval":"These numbers form a pattern.\n\n>> 800 , 400 , 200 , 100 , <span class=\"sm-text color6\">?</span> , <span class=\"sm-text color6\">?</span>\n\n\n<br>The next 2 numbers in the pattern are:"},{"varval":"The next number in the pattern is found by halving the previous number.\n\n<div class=\"aligned\">\n\n>>||||\n|-:|-|-|\n|Starting number|= 800||\n|2nd number|= 800 $\\div$ 2|= 400|\n|3rd number|= 400 $\\div$ 2|= 200|\n|4th number|= 200 $\\div$ 2|= 100|\n|5th number|= 100 $\\div$ 2|= 50|\n|6th number|= 50 $\\div$ 2|= 25|\n\n<br>Therefore, the next 2 numbers are {{{correctAnswer}}}.\n\n</div>"}]},{"vars":[{"varval":"These numbers form a pattern.\n\n>> 1 , 2 , 4 , 8 , <span class=\"sm-text color3\">?</span> , <span class=\"sm-text color3\">?</span>\n\n\n<br>The next 2 numbers in the pattern are:"},{"varval":"The next number in the pattern is found by doubling the previous number.\n\n<div class=\"aligned\">\n\n>>||||\n|-:|-|-|\n|Starting number|= 1||\n|2nd number|= 1 $\\times$ 2|= 2|\n|3rd number|= 2 $\\times$ 2|= 4|\n|4th number|= 4 $\\times$ 2|= 8|\n|5th number|= 8 $\\times$ 2|= 16|\n|6th number|= 16 $\\times$ 2|= 32|\n\n<br>Therefore, the next 2 numbers are {{{correctAnswer}}}.\n\n</div>\n"}]}]

  19. <div class="sm_mode"> {{{question}}} </div>

    [{"vars":[{"varval":"Greg is building patterns using hexagons.\n\nsm_img https://teacher.smartermaths.com.au/wp-content/uploads/2023/08/Algebra-NAP_10099v0-min.svg 400 indent vpad\n\nHow many hexagons will Greg need to make Pattern 4?"},{"varval":"Each pattern has the bottom row removed.\n\n>>Pattern 1 = 1 + 2 + 3 + 4 + 5 + 6 = 21 hexagons\n\n>>Pattern 2 = 1 + 2 + 3 + 4 + 5 = 15 hexagons\n\n>>Pattern 3 = 1 + 2 + 3 + 4 = 10 hexagons\n\n>>Pattern 4 = 1 + 2 + 3 = {{{correctAnswer}}} hexagons\n"}]},{"vars":[{"varval":"Maui is building patterns using hexagons.\n\nsm_img https://teacher.smartermaths.com.au/wp-content/uploads/2023/08/Algebra-NAP_10099v1-min.svg 400 indent vpad\n\nHow many hexagons will Maui need to make Pattern 4?"},{"varval":"Each pattern has 2 hexagons added.\n\n>>Pattern 1 = 6 hexagons\n\n>>Pattern 2 = 8 hexagons\n\n>>Pattern 3 = 10 hexagons\n\n>>Pattern 4 = {{{correctAnswer}}} hexagons\n"}]},{"vars":[{"varval":"Heather is building patterns using hexagons.\n\nsm_img https://teacher.smartermaths.com.au/wp-content/uploads/2023/08/Algebra-NAP_10099v2-min.svg 350 indent vpad\n\nHow many hexagons will Heather need to make Pattern 4?"},{"varval":"Each pattern has 4 hexagons added.\n\n>>Pattern 1 = 5 hexagons\n\n>>Pattern 2 = 9 hexagons\n\n>>Pattern 3 = 13 hexagons\n\n>>Pattern 4 = {{{correctAnswer}}} hexagons\n"}]},{"vars":[{"varval":"Kendra is building patterns using hexagons.\n\nsm_img https://teacher.smartermaths.com.au/wp-content/uploads/2023/08/Algebra-NAP_10099v3-min.svg 350 indent vpad\n\nHow many hexagons will Kendra need to make Pattern 4?"},{"varval":"Each pattern has 3 hexagons added.\n\n>>Pattern 1 = 5 hexagons\n\n>>Pattern 2 = 8 hexagons\n\n>>Pattern 3 = 11 hexagons\n\n>>Pattern 4 = {{{correctAnswer}}} hexagons"}]},{"vars":[{"varval":"Polly is building patterns using hexagons.\n\nsm_img https://teacher.smartermaths.com.au/wp-content/uploads/2023/08/Algebra-NAP_10099v4.svg 350 indent vpad\n\nHow many hexagons will Polly need to make Pattern 4?"},{"varval":"Each pattern has 3 hexagons added.\n\n>>Pattern 1 = 1 hexagon\n\n>>Pattern 2 = 4 hexagons\n\n>>Pattern 3 = 7 hexagons\n\n>>Pattern 4 = {{{correctAnswer}}} hexagons"}]}]

  20. <div class="sm_mode"> {{{question}}} </div>

    [{"vars":[{"varval":"What is the first number in this number pattern?\n\n>> <span class=\"sm-text color3\">?</span>, 88, 92, 96, 100 "},{"varval":"The difference between numbers is:\n<div class=\"aligned\">\n\n>>|||\n|-|-|\n|92 $-$ 88|= 4|\n|96 $-$ 92|= 4|\n|100 $-$ 96|= 4|\n\n<br>Therefore, to get to the first number you do the reverse and take away 4.\n\n</div>\n\n<div class=\"aligned\">\n\n>>|||\n|-|-|\n|<span class=\"sm-text color3\">?</span>|= 2nd number $-$ 4|\n||= 88 $-$ 4|\n||= {{{correctAnswer}}}|\n\n</div>\n"}]},{"vars":[{"varval":"What is the first number in this number pattern?\n\n>> <span class=\"sm-text color2\">?</span>, 25, 31, 37, 43 "},{"varval":"The difference between numbers is:\n<div class=\"aligned\">\n\n>>|||\n|-|-|\n|31 $-$ 25|= 6|\n|37 $-$ 31|= 6|\n|43 $-$ 37|= 6|\n\n<br>Therefore, to get to the first number you do the reverse and take away 6.\n\n</div>\n\n<div class=\"aligned\">\n\n>>|||\n|-|-|\n|<span class=\"sm-text color2\">?</span>|= 2nd number $-$ 6|\n||= 25 $-$ 6|\n||= {{{correctAnswer}}}|\n\n</div>\n"}]},{"vars":[{"varval":"What is the first number in this number pattern?\n\n>> <span class=\"sm-text color1\">?</span>, 60, 52, 44, 36 "},{"varval":"The difference between numbers is:\n<div class=\"aligned\">\n\n>>|||\n|-|-|\n|52 $-$ 60|= $-$ 8|\n|44 $-$ 52|= $-$ 8|\n|36 $-$ 44|= $-$ 8|\n\n<br>Therefore, to get to the first number you do the reverse and add 8.\n\n</div>\n\n<div class=\"aligned\">\n\n>>|||\n|-|-|\n|<span class=\"sm-text color1\">?</span>|= 2nd number $+$ 8|\n||= 60 $+$ 8|\n||= {{{correctAnswer}}}|\n\n</div>\n"}]},{"vars":[{"varval":"What is the first number in this number pattern?\n\n>> <span class=\"sm-text color4\">?</span>, 32, 27, 22, 17"},{"varval":"The difference between numbers is:\n<div class=\"aligned\">\n\n>>|||\n|-|-|\n|27 $-$ 32|= $-$ 5|\n|22 $-$ 27|= $-$ 5|\n|17 $-$ 22|= $-$ 5|\n\n<br>Therefore, to get to the first number you do the reverse and add 5.\n\n</div>\n\n<div class=\"aligned\">\n\n>>|||\n|-|-|\n|<span class=\"sm-text color4\">?</span>|= 2nd number $+$ 5|\n||= 32 $+$ 5|\n||= {{{correctAnswer}}}|\n\n</div>\n"}]},{"vars":[{"varval":"What is the first number in this number pattern?\n\n>> <span class=\"sm-text color5\">?</span>, 110, 113, 116, 119 "},{"varval":"The difference between numbers is:\n<div class=\"aligned\">\n\n>>|||\n|-|-|\n|113 $-$ 110|= 3|\n|116 $-$ 113|= 3|\n|119 $-$ 116|= 3|\n\n<br>Therefore, to get to the first number you do the reverse and take away 3.\n\n</div>\n\n<div class=\"aligned\">\n\n>>|||\n|-|-|\n|<span class=\"sm-text color5\">?</span>|= 2nd number $-$ 3|\n||= 110 $-$ 3|\n||= {{{correctAnswer}}}|\n\n</div>\n"}]},{"vars":[{"varval":"What is the first number in this number pattern?\n\n>> <span class=\"sm-text color6\">?</span>, 250, 220, 190, 160 "},{"varval":"The difference between numbers is:\n<div class=\"aligned\">\n\n>>|||\n|-|-|\n|220 $-$ 250|= $-$ 30|\n|190 $-$ 220|= $-$ 30|\n|160 $-$ 190|= $-$ 30|\n\n<br>Therefore, to get to the first number you do the reverse and add 30.\n\n</div>\n\n<div class=\"aligned\">\n\n>>|||\n|-|-|\n|<span class=\"sm-text color6\">?</span>|= 2nd number $+$ 30|\n||= 250 $+$ 30|\n||= {{{correctAnswer}}}|\n\n</div>\n"}]}]

  21. <div class="sm_mode"> {{{question}}} </div>

    [{"vars":[{"varval":"Didier has placed numbered cards on the table in a pattern as shown below.\n\n\nsm_img https://teacher.smartermaths.com.au/wp-content/uploads/2023/08/Algebra-NAP_30045v0-min.svg 300 indent2 vpad\n\n\nWhen Didier turns the middle card, what number should be on the card?"},{"varval":"The difference between numbers on cards is:\n<div class=\"aligned\">\n\n>>|||\n|-|-|\n|27 $-$ 21|= 6|\n|45 $-$ 39|= 6|\n\n<br>Therefore the middle card will be:\n\n</div>\n\n<div class=\"aligned\">\n\n>>|||\n|-|-|\n||= 2nd card $+$ 6|\n||= 27 $+$ 6|\n||= {{{correctAnswer}}}|\n\n</div>\n\n"}]},{"vars":[{"varval":"Gamora has placed numbered cards on the table in a pattern as shown below.\n\n\nsm_img https://teacher.smartermaths.com.au/wp-content/uploads/2023/08/Algebra-NAP_30045v1-min.svg 300 indent2 vpad\n\n\nWhen Gamora turns the middle card, what number should be on the card?"},{"varval":"The difference between numbers on cards is:\n<div class=\"aligned\">\n\n>>|||\n|-|-|\n|78 $-$ 81|= $-$ 3|\n|69 $-$ 72|= $-$ 3|\n\n<br>Therefore the middle card will be:\n\n</div>\n\n<div class=\"aligned\">\n\n>>|||\n|-|-|\n||= 2nd card $-$ 3|\n||= 78 $-$ 3|\n||= {{{correctAnswer}}}|\n\n</div>\n"}]},{"vars":[{"varval":"Thor has placed numbered cards on the table in a pattern as shown below.\n\n\nsm_img https://teacher.smartermaths.com.au/wp-content/uploads/2023/08/Algebra-NAP_30045v2-min.svg 300 indent2 vpad\n\n\nWhen Thor turns the middle card, what number should be on the card?"},{"varval":"The difference between numbers is:\n<div class=\"aligned\">\n\n>>|||\n|-|-|\n|32 $-$ 21|= 11|\n|65 $-$ 54|= 11|\n\n<br>Therefore the middle card will be:\n\n</div>\n\n<div class=\"aligned\">\n\n>>|||\n|-|-|\n||= 2nd card $+$ 11|\n||= 32 $+$ 11|\n||= {{{correctAnswer}}}|\n\n</div>"}]},{"vars":[{"varval":"Jansen has placed numbered cards on the table in a pattern as shown below.\n\n\nsm_img https://teacher.smartermaths.com.au/wp-content/uploads/2023/08/Algebra-NAP_30045v3b-min.svg 300 indent2 vpad\n\n\nWhen Jansen turns the middle card, what number should be on the card?"},{"varval":"The difference between numbers is:\n<div class=\"aligned\">\n\n>>|||\n|-|-|\n|52 $-$ 56|= $-$ 4|\n|40 $-$ 44|= $-$ 4|\n\n<br>Therefore the middle card will be:\n\n</div>\n\n<div class=\"aligned\">\n\n>>|||\n|-|-|\n||= 2nd card $-$ 4|\n||= 52 $-$ 4|\n||= {{{correctAnswer}}}|\n\n</div>\n"}]},{"vars":[{"varval":"Loz has placed numbered cards on the table in a pattern as shown below.\n\n\nsm_img https://teacher.smartermaths.com.au/wp-content/uploads/2023/08/Algebra-NAP_30045v4-min.svg 300 indent2 vpad\n\n\nWhen Loz turns the middle card, what number should be on the card?"},{"varval":"The difference between numbers is:\n<div class=\"aligned\">\n\n>>|||\n|-|-|\n|25 $-$ 19|= 6|\n|43 $-$ 37|= 6|\n\n<br>Therefore the middle card will be:\n\n</div>\n\n<div class=\"aligned\">\n\n>>|||\n|-|-|\n||= 2nd card $+$ 6|\n||= 25 $+$ 6|\n||= {{{correctAnswer}}}|\n\n</div>\n"}]},{"vars":[{"varval":"Patrick has placed numbered cards on the table in a pattern as shown below.\n\n\nsm_img https://teacher.smartermaths.com.au/wp-content/uploads/2023/08/Algebra-NAP_30045v5-min.svg 300 indent2 vpad\n\n\nWhen Patrick turns the middle card, what number should be on the card?"},{"varval":"The difference between numbers is:\n<div class=\"aligned\">\n\n>>|||\n|-|-|\n|72 $-$ 65|= 7|\n|93 $-$ 86|= 7|\n\n<br>Therefore the middle card will be:\n\n</div>\n\n<div class=\"aligned\">\n\n>>|||\n|-|-|\n||= 2nd card $+$ 7|\n||= 72 $+$ 7|\n||= {{{correctAnswer}}}|\n\n</div>"}]}]

  22. <div class="sm_mode"> {{{question}}} </div>

    [{"vars":[{"varval":"What is the rule you would use to find the next number in this number pattern?\n\n>> 87, 91, 95, 99, <span class=\"sm-text color3\">?</span>"},{"varval":"The difference between numbers is:\n<div class=\"aligned\">\n\n>>|||\n|-|-|\n|91 $-$ 87|= 4|\n|95 $-$ 91|= 4|\n|90 $-$ 95|= 4|\n\n<br>Therefore to get to the next number add 4.\n\n</div>\n\n<div class=\"aligned\">\n\n>>|||\n|-|-|\n|<span class=\"sm-text color3\">?</span>|= last number $+$ 4 = 103|\n|Rule|= {{{correctAnswer}}}|\n\n</div>\n"}]},{"vars":[{"varval":"What is the rule you would use to find the next number in this number pattern?\n\n>> 102, 96, 90, 84, <span class=\"sm-text color4\">?</span>"},{"varval":"The difference between numbers is:\n<div class=\"aligned\">\n\n>>|||\n|-|-|\n|96 $-$ 102|= $-$ 6|\n|90 $-$ 96|= $-$ 6|\n|84 $-$ 90|= $-$ 6|\n\n<br>Therefore to get to the next number takeaway 6.\n\n</div>\n\n<div class=\"aligned\">\n\n>>|||\n|-|-|\n|<span class=\"sm-text color4\">?</span>|= last number $-$ 6 = 78|\n|Rule|= {{{correctAnswer}}}|\n\n</div>\n\n"}]},{"vars":[{"varval":"What is the rule you would use to find the next number in this number pattern?\n\n>> 34, 46, 58, 70, <span class=\"sm-text color5\">?</span>"},{"varval":"The difference between numbers is:\n<div class=\"aligned\">\n\n>>|||\n|-|-|\n|46 $-$ 34|= 12|\n|58 $-$ 46|= 12|\n|70 $-$ 58|= 12|\n\n<br>Therefore to get to the next number add 12.\n\n</div>\n\n<div class=\"aligned\">\n\n>>|||\n|-|-|\n|<span class=\"sm-text color5\">?</span>|= last number $+$ 12 = 82|\n|Rule|= {{{correctAnswer}}}|\n\n</div>\n\n"}]},{"vars":[{"varval":"What is the rule you would use to find the next number in this number pattern?\n\n>> 71, 68, 65, 62, <span class=\"sm-text color6\">?</span>"},{"varval":"The difference between numbers is:\n<div class=\"aligned\">\n\n>>|||\n|-|-|\n|68 $-$ 71|= $-$ 3|\n|65 $-$ 68|= $-$ 3|\n|62 $-$ 65|= $-$ 3|\n\n<br>Therefore to get to the next number take away 3.\n\n</div>\n\n<div class=\"aligned\">\n\n>>|||\n|-|-|\n|<span class=\"sm-text color6\">?</span>|= last number $-$ 3 = 59|\n|Rule|= {{{correctAnswer}}}|\n\n</div>\n\n"}]},{"vars":[{"varval":"What is the rule you would use to find the next number in this number pattern?\n\n>> 135, 142, 149, 156, <span class=\"sm-text color8\">?</span>"},{"varval":"The difference between numbers is:\n<div class=\"aligned\">\n\n>>|||\n|-|-|\n|142 $-$ 135|= 7|\n|149 $-$ 142|= 7|\n|156 $-$ 149|= 7|\n\n<br>Therefore to get to the next number add 7.\n\n</div>\n\n<div class=\"aligned\">\n\n>>|||\n|-|-|\n|<span class=\"sm-text color8\">?</span>|= last number $+$ 7 = 163|\n|Rule|= {{{correctAnswer}}}|\n\n</div>\n"}]},{"vars":[{"varval":"What is the rule you would use to find the next number in this number pattern?\n\n>> 118, 109, 100, 91, <span class=\"sm-text color1\">?</span>"},{"varval":"The difference between numbers is:\n<div class=\"aligned\">\n\n>>|||\n|-:|-|\n|109 $-$ 118|= $-$ 9|\n|100 $-$ 109|= $-$ 9|\n|91 $-$ 100|= $-$ 9|\n\n<br>Therefore to get to the next number take away 9.\n\n</div>\n\n<div class=\"aligned\">\n\n>>|||\n|-|-|\n|<span class=\"sm-text color1\">?</span>|= last number $-$ 9 = 82|\n|Rule|= {{{correctAnswer}}}|\n\n</div>\n\n"}]}]

  23. Algebra, NAPX-J3-CA04 SA, NAPX-J2-11

    <div class="sm_mode"> {{{question}}} </div>

    [{"vars":[{"varval":"Using the table, a teacher allocates coloured pencils to class groups depending on the number of students in each group.\n\n<br>\n\n<div class=\"sm-table row1-color1\">\n\n>>| Number of <br> students in group | Coloured <br> pencils| \n|:-:|:-:|\n| 2 | 10|\n| 4 | 20|\n| 6 | **?**|\n|8|40|\n\n</div>\n\n<br>Using the pattern in the table, how many pencils should a group of 6 students receive?"},{"varval":"The pattern shows that each student receives\n 5 coloured pencils.\n\n>>Pieces = 5 $\\times$ 6 = {{{correctAnswer0}}}\n\n<br>∴ A group of 6 will be given {{{correctAnswer0}}} pieces."}]},{"vars":[{"varval":"Using the table, a soccer trainer allocates balls to teams depending on the number of players in each training group.\n\n<br>\n\n<div class=\"sm-table row1-color2\">\n\n>>| Number of <br> players in group| Number of <br> balls | \n|:-:|:-:|\n| 6 | 3|\n| 8 | 4|\n| 10 | **?**|\n|12|6|\n\n</div>\n\n<br>Using the pattern in the table, how many balls should a training group of 10 players receive?"},{"varval":"The pattern shows that there is 1 ball for every 2 players.\n\nsm_nogap Number of balls \n\n>>= Players $\\div$ 2\n\n>>= 10 $\\div$ 2\n\n>> = {{{correctAnswer0}}}\n\n<br>∴ A team of 10 will be given {{{correctAnswer0}}} balls."}]},{"vars":[{"varval":"Using the table, a chef allocates cups of flour to a cupcake recipe depending on the number of cupcakes being baked.\n\n<br>\n\n<div class=\"sm-table row1-color3\">\n\n>>| Number of cupcakes <br> being baked | Cups of <br> flour| \n|:-:|:-:|\n| 12 | 2|\n| 24 | 4|\n| 36 | **?**|\n|48|8|\n\n</div>\n\n<br>Using the pattern in the table, how many cups of flour should be allocated for a recipe making 36 cupcakes?"},{"varval":"The pattern shows that there is 1 cup of flour used for every 6 cupcakes baked\n\nsm_nogap Cups of flour \n\n>>= Cupcakes $\\div$ 6\n\n>>= 36 $\\div$ 6\n\n>> = {{{correctAnswer0}}}\n\n<br>∴ A batch of 36 cupcakes will be allocated {{{correctAnswer0}}} cups of flour."}]},{"vars":[{"varval":"Using the table, a caterer allocates mini pies to groups depending on the number of people in each group.\n\n<br>\n\n<div class=\"sm-table row1-color4\">\n\n>>| Number of <br> people in group | Number of <br> mini pies| \n|:-:|:-:|\n| 4 | 16|\n| 6 | 24|\n| 8 | **?**|\n|10|40|\n\n</div>\n\n<br>Using the pattern in the table, how many mini pies should a group of 8 people receive?"},{"varval":"The pattern shows that each person receives 4 mini pies.\n\n>>Mini pies for 8 people = 4 $\\times$ 8 = {{{correctAnswer0}}}\n\n<br>∴ A group of 8 will be given {{{correctAnswer0}}} mini pies."}]},{"vars":[{"varval":"Using the table, a Little Athletics coordinator allocates points depending on the number of events a competitor completes.\n\n<br>\n\n<div class=\"sm-table row1-color5\">\n\n>>| Number of <br> events | Points <br> allocated| \n|:-:|:-:|\n| 2 | 6|\n| 4 | 12|\n| 6 | **?**|\n|8|24|\n\n</div>\n\n<br>Using the pattern in the table, how many points should a competitor completing 6 events receive?"},{"varval":"The pattern shows that each event receives\n 3 points.\n\n>>Points = 3 $\\times$ 6 = {{{correctAnswer0}}}\n\n<br>∴ A competitor completing 6 events will receive {{{correctAnswer0}}} points."}]}]

  24. Patterns Number, NAPX-p110532v01

    <div class="sm_mode"> {{{question}}} </div>

    [{"vars":[{"varval":"Dean earns $20 in the first week of his casual job.\n\nHe then earns twice as much as he did the previous week for the next 4 weeks.\n\nDean's total earnings, in dollars, in any given week will be which of the following:"},{"varval":"1st week: $20 (even)\n\n2nd week: 20 + 40 = $60 (even)\n\n3rd week: 60 + 120 = $180 (even)\n\nAny week: even + even = even\n\n∴ Total amount is always an even number."}]},{"vars":[{"varval":"Bindera earns $9 in her first day selling homemade lemondade.\n\nShe then earns twice as much as she did the previous day for the next 4 days.\n\nBindera's total earnings, in dollars, on any given day will be which of the following:"},{"varval":"1st week: $9 (odd)\n\n2nd week: $9 + $18 = $27 (odd)\n\n3rd week: $27 + $54 = $81 (odd)\n\nAny week: odd + even = odd\n\n∴ Total amount is always an odd number."}]},{"vars":[{"varval":"TJ sells 3 watermelons on Monday.\n\nHe then sells twice as many as he did the previous day for the next 4 days.\n\nTJ's total sales on any given day will be which of the following:"},{"varval":"Monday: 3 (odd)\n\nTuesday: 3 + 6 = 9 (odd)\n\nWednesday: 9 + 18 = 27 (odd)\n\nAny week: odd + even = odd\n\n∴ Total amount is always an odd number."}]},{"vars":[{"varval":"Jimmy walks 4 kilometres on the first day of his fitness program.\n\nHe then walks twice as far as he did the previous day for the next 4 days.\n\nJimmy's total distance, in kilometres, on any given day will be which of the following:"},{"varval":"Day 1: 4 kms (even)\n\nDay 2: 4 + 8 = 12 kms (even)\n\nDay 3: 12 + 24 = 36 kms (even)\n\nAny day : even + even = even\n\n∴ Total distance is always an even number."}]},{"vars":[{"varval":"Justine deposits $7 in her money box.\n\nShe then deposits four times as much as she did the previous week for the next 4 weeks.\n\nJustine's total savings, in dollars, in any given week will be which of the following:"},{"varval":"1st week: $3 (odd)\n\n2nd week: 3 + 12 = $15 (odd)\n\n3rd week: 15 + 60 = $75 (odd)\n\n4th week: 75 + 300 = $75 (odd)\n\nAny week: odd + odd = odd\n\n∴ Total amount is always an odd number."}]},{"vars":[{"varval":"Pimm earns $10 in the first week of her casual job.\n\nShe then earns 4 times as much as she did the previous week for the next 4 weeks.\n\nPimm's total earnings, in dollars, in any given week will be which of the following:"},{"varval":"1st week: $10 (even)\n\n2nd week: 10 + 40 = $50 (even)\n\n3rd week: 50 + 200 = $250 (even)\n\nAny week: even + even = even\n\n∴ Total amount is always an even number."}]}]

  25. Number, NAPX-p110532v01

    <div class="sm_mode"> {{{question}}} </div>

    [{"vars":[{"varval":"Arlo puts $22 into his piggy bank to start saving money.\n\nHe then puts $7 into the piggy bank at the end of each month.\n\nThe total amount of money in his piggy bank at the end of each month will be which of the following:"},{"varval":"Starting amount = $22 (even)\n\nEnd of 1st month = 22 + 7 = $29 (odd)\n\nEnd of 2nd month = 29 + 7 = $36 (even)\n\n$\\therefore$ Total amount will be either an odd or even number."}]},{"vars":[{"varval":"On Monday Newton harvests 8 pumpkins to sell at the markets.\n\nOn each of the following three days he harvests another 5 pumpkins.\n\nThe total of pumpkins he has ready for market at the end of each day will be which of the following:"},{"varval":"Starting number = 8 (even)\n\nEnd of Tuesday = 8 + 5 = 13 (odd)\n\nEnd of Wednesday = 13 + 5 = 18 (even)\n\nEnd of Thursday = 18 + 5 = 23 (odd)\n\n$\\therefore$ Total number of pumpkins will be either an odd or an even number."}]},{"vars":[{"varval":"Gretchen puts $80 into a savings account to start saving money.\n\nShe then puts $24 into the savings account at the end of each month.\n\nThe total amount of money in hers savings account at the end of each month will be which of the following:"},{"varval":"Starting amount = $80 (even)\n\nEnd of 1st month = 80 + 24 = $104 (even)\n\nEnd of 2nd month = 104 + 24 = $128 (even)\n\nEnd of 3rd month = 128 + 24 = $152 (even)\n\n$\\therefore$ Total amount will be always be an even number."}]},{"vars":[{"varval":"Brandon puts $100 into his savings account to start saving for a holiday.\n\nHe then puts $25 into the savings account at the end of each fortnight.\n\nThe total amount of money in his savings account at the end of each fortnight will be which of the following:"},{"varval":"Starting amount = $100 (even)\n\nEnd of 1st fortnight = 100 + 25 = $125 (odd)\n\nEnd of 2nd fortnight = 125 + 25 = $150 (even)\n\nEnd of 3rd fortnight = 150 + 25 = $175 (odd)\n\n$\\therefore$ Total amount will be either an odd or even number."}]},{"vars":[{"varval":"Midge puts $140 into her savings account to start saving money.\n\nShe then puts $50 into the savings account at the end of each month.\n\nThe total amount of money in Midge's savings account at the end of each month will be which of the following:"},{"varval":"Starting amount = $140 (even)\n\nEnd of 1st month = 140 + 50 = $190 (even)\n\nEnd of 2nd month = 190 + 50 = $240 (even)\n\nEnd of 3rd month = 240 + 50 = $290 (even)\n\n$\\therefore$ Total amount will always be an even number."}]},{"vars":[{"varval":"Jackie puts $25 into her piggy bank to start saving money.\n\nShe then puts $15 into the piggy bank at the end of each month.\n\nThe total amount of money in Jackie's piggy bank at the end of each month will be which of the following:"},{"varval":"Starting amount = $25 (odd)\n\nEnd of 1st month = 25 + 15 = $40 (even)\n\nEnd of 2nd month = 40 + 15 = $55 (odd)\n\n$\\therefore$ Total amount will be either an odd or even number."}]}]

  26. Algebra, NAPX-F1-10 (363 difficulty)

    <div class="sm_mode"> {{{question}}} </div>

    [{"vars":[{"varval":"This repeating pattern is made by turning a square tile.\n\nsm_img https://teacher.smartermaths.com.au/wp-content/uploads/2023/08/Algebra-NAP_10020v0q-min.svg 500 indent vpad\n\nWhich of these tiles comes next in this pattern?"},{"varval":"The square is turned clockwise 90 degrees each time.\n\n$\\therefore$ The next tile in the pattern is:\n\n{{{correctAnswer}}}\n"}]},{"vars":[{"varval":"This repeating pattern is made by turning a square tile.\n\nsm_img https://teacher.smartermaths.com.au/wp-content/uploads/2023/08/Algebra-NAP_10020v1.svg 500 indent vpad\n\nWhich of these tiles comes next in this pattern?"},{"varval":"The square is turned clockwise 90 degrees each time.\n\n$\\therefore$ The next tile in the pattern is:\n\n{{{correctAnswer}}}\n"}]},{"vars":[{"varval":"This repeating pattern is made by turning a square tile.\n\nsm_img https://teacher.smartermaths.com.au/wp-content/uploads/2023/08/Algebra-NAP_10020v2q.svg 500 indent vpad\n\nWhich of these tiles comes next in this pattern?"},{"varval":"The square is turned clockwise 90 degrees each time.\n\n$\\therefore$ The next tile in the pattern is:\n\n{{{correctAnswer}}}\n"}]},{"vars":[{"varval":"This repeating pattern is made by turning a square tile.\n\nsm_img https://teacher.smartermaths.com.au/wp-content/uploads/2023/08/Algebra-NAP_10020v3q.svg 500 indent vpad\n\nWhich of these tiles comes next in this pattern?"},{"varval":"The square is turned anti-clockwise 90 degrees each time.\n\n$\\therefore$ The next tile in the pattern is:\n\n{{{correctAnswer}}}\n"}]},{"vars":[{"varval":"This repeating pattern is made by turning a square tile.\n\nsm_img https://teacher.smartermaths.com.au/wp-content/uploads/2023/08/Algebra-NAP_10020v4q.svg 500 indent vpad\n\nWhich of these tiles comes next in this pattern?"},{"varval":"The square is turned anti-clockwise 90 degrees each time.\n\n$\\therefore$ The next tile in the pattern is:\n\n{{{correctAnswer}}}\n"}]},{"vars":[{"varval":"This repeating pattern is made by turning a square tile.\n\nsm_img https://teacher.smartermaths.com.au/wp-content/uploads/2023/08/Algebra-NAP_10020v5q.svg 500 indent vpad\n\nWhich of these tiles comes next in this pattern?"},{"varval":"The square is turned anti-clockwise 90 degrees each time.\n\n$\\therefore$ The next tile in the pattern is:\n\n{{{correctAnswer}}}\n"}]}]

  27. <div class="sm_mode"> {{{question}}} </div>

    [{"vars":[{"varval":"The table below shows the distances thrown by javelin competitors at the athletics carnival.\n\n<div class=\"sm-table row1-color1 heading-color1\">\n\n>>**Javelin Competition**\n\n>>| **Name** | **Distance in Metres** |\n|:-:|:-:|\n| James | 15 |\n| Allen | 12 |\n| Will | 8 |\n| Kenji | 13 |\n| Owen | 9 |\n\n</div>\n\n<br>What is the difference between the longest throw and the shortest throw?"},{"varval":"sm_nogap Difference between longest and shortest throw\n\n<div class=\"aligned\">\n\n>> | |\n| ---------- |\n| \\= 15 $-$ 8|\n| \\= {{{correctAnswer}}} metres |\n\n</div>"}]},{"vars":[{"varval":"The table below shows the masses of dogs who weighed in at the vet's on Saturday.\n\n<div class=\"sm-table row1-color2 heading-color2\">\n\n>>**Dog Weigh In**\n\n>>| **Dog** | **Mass in Kilograms** |\n|:-:|:-:|\n| Spot | 24 |\n| Brutus | 59 |\n| Jack | 16 |\n| Bella | 48 |\n| Callie | 37 |\n\n</div>\n\n<br>What is the difference, in kilograms, between the heaviest dog and the lightest dog?"},{"varval":"sm_nogap Difference between heaviest and lightest dog\n\n<div class=\"aligned\">\n\n>> | |\n| ---------- |\n| \\= 59 $-$ 16|\n| \\= {{{correctAnswer}}} kilograms |\n\n</div>"}]},{"vars":[{"varval":"The table below shows the number of donuts sold at a cafe on different days.\n\n<div class=\"sm-table row1-color3 heading-color3\">\n\n>>**Donut Sales**\n\n>>| **Day** | **Number of Donuts** |\n|:-:|:-:|\n| Monday | 56 |\n| Tuesday | 47 |\n| Wednesday| 29 |\n| Thursday | 74 |\n| Friday | 61 |\n\n</div>\n\n<br>What is the difference in the number of donuts sold on Wednesday compared to Thursday?"},{"varval":"sm_nogap Difference in number of donuts sold\n\n<div class=\"aligned\">\n\n>> | |\n| ---------- |\n| \\= 74 $-$ 29|\n| \\= 74 $-$ 20 $-$ 9 |\n| \\= 54 $-$ 9 |\n| \\= {{{correctAnswer}}} donuts |\n\n</div>"}]},{"vars":[{"varval":"The table below shows the number of cupcakes sold at a cafe on different days.\n\n<div class=\"sm-table row1-color4 heading-color4\">\n\n>>**Cupcake Sales**\n\n>>| **Day** | **Number of Cupcakes** |\n|:-:|:-:|\n| Wednesday| 34 |\n| Thursday | 27 |\n| Friday | 42 |\n| Saturday | 51 |\n| Sunday | 35 |\n</div>\n\n<br>How many cupcakes were sold in total on Friday and Saturday?"},{"varval":"sm_nogap Total sold on Friday and Saturday\n\n<div class=\"aligned\">\n\n>> | |\n| ---------- |\n| \\= 42 + 51|\n| \\= {{{correctAnswer}}} cupcakes |\n\n</div>"}]},{"vars":[{"varval":"The table below shows the number of pies sold at a bakery last week.\n\n<div class=\"sm-table row1-color5 heading-color5\">\n\n>>**Pie Sales**\n\n>>| **Day** | **Number of Pies** |\n|:-:|:-:|\n| Monday| 71 |\n| Tuesday| 83 |\n| Wednesday| 67 |\n| Thursday | 59 |\n| Friday | 89 |\n| Saturday | 54 |\n| Sunday | 36 |\n</div>\n\n<br>How many pies were sold in total on Wednesday and Saturday?"},{"varval":"sm_nogap Total sold on Wednesday and Saturday\n\n<div class=\"aligned\">\n\n>> | |\n| ---------- |\n| \\= 67 + 54|\n| \\= 60 + 7 + 50 + 4|\n| \\= 110 + 11|\n| \\= {{{correctAnswer}}} pies|\n\n</div>"}]},{"vars":[{"varval":"The table below shows the number of soccer boots sold at an online sports store last week.\n\n<div class=\"sm-table row1-color6 heading-color6\">\n\n>>**Soccer Boot Sales**\n\n>>| **Day** | **Number of Pairs** |\n|:-:|:-:|\n| Monday| 15 |\n| Tuesday| 22 |\n| Wednesday| 17 |\n| Thursday | 30 |\n| Friday | 28 |\n| Saturday | 42 |\n| Sunday | 18 |\n</div>\n\n<br>How many pairs of soccer boots were sold in total on Monday, Tuesday and Wednesday?"},{"varval":"sm_nogap Total sold on Monday, Tuesday and Wednesday\n\n<div class=\"aligned\">\n\n>> | |\n| ---------- |\n| \\= 15 + 22 + 17|\n| \\= 10 + 5 + 20 + 2 + 10 + 7|\n| \\= 40 + 14|\n| \\= {{{correctAnswer}}} soccer boots |\n\n</div>"}]}]

  28. <div class="sm_mode"> {{{question}}} </div>

    [{"vars":[{"varval":"Sandra walks her dog around the local park every afternoon.\n\nThe table below shows the number of laps she and her dog walked over the last 5 days.\n\n<div class=\"sm-table col1-color3\">\n\n>>| **Day** |Monday|Tuesday|Wednesday|Thursday|Friday|\n|:-:|:-:|:-:|:-:|:-:|:-:|\n|**Laps** | 2|3|3|0|1|\n\n</div>\n\n<br>\n<div class=\"sm-table col1-color3\">\n\n>>|**1 Lap = 3 kilometres** |\n|:-:|\n\n</div>\n\n<br>What is the total distance Sandra and her dog walked on Wednesday afternoon?"},{"varval":"sm_nogap Kilometres walked on Wednesday\n\n<div class=\"aligned\">\n\n>> | |\n| ---------- |\n| \\= 3 $\\times$ 3 |\n| \\= {{{correctAnswer}}} |\n\n</div>\n"}]},{"vars":[{"varval":"The table below shows the number of laps of the bay Kinglsey paddled his kayak last week.\n\n<div class=\"sm-table row1-color2\">\n\n>>| **Day** |**Laps**|\n|:-:|:-:|\n|Wednesday | 1|\n|Thursday | 3|\n|Friday | 2|\n|Saturday | 4|\n|Sunday | 2|\n\n</div>\n<br>\n<div class=\"sm-table col1-color2\">\n\n>>|**1 Lap = 2 kilometres** |\n|:-:|\n\n</div>\n\n<br>What is the total distance, in kilometres, Kingsley paddled his kayak on Wednesday and Thursday last week?"},{"varval":"sm_nogap Kilometres kayaked on Wednesday and Thursday\n\n<div class=\"aligned\">\n\n>> | |\n| ---------- |\n| \\= 2 $\\times$ (total laps) |\n| \\= 2 $\\times$ (1 + 3) |\n| \\= 2 $\\times$ 4 |\n| \\= {{{correctAnswer}}} kilometres|\n\n</div>\n"}]},{"vars":[{"varval":"The table below shows the number of Park Runs Bligh ran over a six month period.\n\n<div class=\"sm-table row1-color4\">\n\n>>| **Day** |**Laps**|\n|:-:|:-:|\n|September | 1|\n|October | 2|\n|November | 3|\n|December | 2|\n|January | 4|\n|February | 3|\n\n</div>\n<br>\n<div class=\"sm-table col1-color4\">\n\n>>|**1 Park Run = 5 kilometres** |\n|:-:|\n\n</div>\n\n<br>What is the total distance, in kilometres, Bligh ran during November and January?"},{"varval":"sm_nogap Kilometres run on November and January\n\n<div class=\"aligned\">\n\n>> | |\n| ---------- |\n| \\= 5 $\\times$ (total park runs) |\n| \\= 5 $\\times$ (3 + 4) |\n| \\= 5 $\\times$ 7 |\n| \\= {{{correctAnswer}}} kilometres |\n\n</div>\n"}]},{"vars":[{"varval":"The table below shows the number of bunches of flowers that a florist made last week.\n\n<div class=\"sm-table row1-color5\">\n\n>>| **Day** |**Bunches of Flowers**|\n|:-:|:-:|\n|Monday | 9|\nTuesday | 5|\n|Wednesday | 12|\n|Thursday | 15|\n|Friday | 21|\n|Saturday | 18|\n\n\n</div>\n<br>\n<div class=\"sm-table col1-color5\">\n\n>>|**1 Bunch = 10 flowers** |\n|:-:|\n\n</div>\n\n<br>What is the total number of flowers used in the bunches made on Monday and Tuesday?"},{"varval":"sm_nogap Flowers used on Monday and Tuesday\n\n<div class=\"aligned\">\n\n>> | |\n| ---------- |\n| \\= 10 $\\times$ (total bunches) |\n| \\= 10 $\\times$ (9 + 5) |\n| \\= 10 $\\times$ 14 |\n| \\= {{{correctAnswer}}} flowers |\n\n</div>\n"}]},{"vars":[{"varval":"The table below shows the number of exercise classes Tia attended last week.\n\n<div class=\"sm-table row1-color6\">\n\n>>| **Day** |**Exercise Classes**|\n|:-:|:-:|\n|Monday | 2|\nTuesday | 3|\n|Wednesday | 1|\n|Thursday | 2|\n|Friday | 1|\n|Saturday | 3|\n\n\n</div>\n<br>\n<div class=\"sm-table col1-color6\">\n\n>>|**1 Exercise Class = 30 minutes** |\n|:-:|\n\n</div>\n\n<br>What is the total number of minutes that Tia exercised for on Monday, Wednesday and Friday?"},{"varval":"sm_nogap Minutes exercise on Monday, Wednesday and Friday\n\n<div class=\"aligned\">\n\n>> | |\n| ---------- |\n| \\= 30 $\\times$ (total classes) |\n| \\= 30 $\\times$ (2 + 1 +1) |\n| \\= 30 $\\times$ 4 |\n| \\= {{{correctAnswer}}} minutes |\n\n</div>"}]},{"vars":[{"varval":"The table below shows the number of boxes of apples that Jonathon packed last week.\n\n<div class=\"sm-table row1-color8\">\n\n>>| **Day** |**Boxs of Apples**|\n|:-:|:-:|\n|Monday | 21|\nTuesday | 35|\n|Wednesday | 18|\n|Thursday | 30|\n|Friday | 25|\n\n\n\n</div>\n<br>\n<div class=\"sm-table col1-color8\">\n\n>>|**1 Box = 100 apples** |\n|:-:|\n\n</div>\n\n<br>What is the total number of apples Jonathon packed on Tuesday and Friday?"},{"varval":"sm_nogap Apples packed on Tuesday and Friday\n\n<div class=\"aligned\">\n\n>> | |\n| ---------- |\n| \\= 100 $\\times$ (total boxes) |\n| \\= 100 $\\times$ (35 + 25) |\n| \\= 100 $\\times$ 60 |\n| \\= {{{correctAnswer}}} apples |\n\n</div>"}]}]

  29. <div class="sm_mode"> {{{question}}} </div>

    [{"vars":[{"varval":"The graph below shows the times Kerry spent, in minutes, performing different exercises at the gym over the last fortnight.\n\nsm_img https://teacher.smartermaths.com.au/wp-content/uploads/2023/08/Statistics-NAP_50011_v0-min.svg 380 indent2 vpad\n\nHow many minutes in total did Kerry spend cycling and walking?"},{"varval":"sm_nogap Total time spent Cycling and Walking\n\n<div class=\"aligned\">\n\n>> | |\n| ---------- |\n| \\= 90 + 60 |\n| \\= {{{correctAnswer}}} |\n\n</div>\n"}]},{"vars":[{"varval":"The graph below shows the times Kerry spent, in minutes, performing different exercises at the gym over the last fortnight.\n\nsm_img https://teacher.smartermaths.com.au/wp-content/uploads/2023/08/Statistics-NAP_50011_v0-min.svg 380 indent2 vpad\n\nHow many more minutes did Kerry spend doing HIIT than rowing?"},{"varval":"sm_nogap Difference between HIIT and rowing\n\n<div class=\"aligned\">\n\n>> | |\n| ---------- |\n| \\= 75 $-$ 40 |\n| \\= {{{correctAnswer}}} |\n\n</div>\n"}]},{"vars":[{"varval":"The graph below shows the outdoor activities chosen by students at the Year 7 camp.\n\nsm_img https://teacher.smartermaths.com.au/wp-content/uploads/2023/08/Statistics-NAP_50011_v2.svg 380 indent2 vpad\n\nHow many students in total chose either high ropes or abseiling?"},{"varval":"sm_nogap Total High Ropes and Abseiling\n\n<div class=\"aligned\">\n\n>> | |\n| ---------- |\n| \\= 9 + 12 |\n| \\= {{{correctAnswer}}} |\n\n</div>\n"}]},{"vars":[{"varval":"The graph below shows the outdoor activities chosen by students at the Year 7 camp.\n\nsm_img https://teacher.smartermaths.com.au/wp-content/uploads/2023/08/Statistics-NAP_50011_v2.svg 380 indent2 vpad\n\nHow many more students chose the Giant Swing than Hiking?"},{"varval":"sm_nogap Difference between Giant Swing and Hiking\n\n<div class=\"aligned\">\n\n>> | |\n| ---------- |\n| \\= 24 $-$ 15 |\n| \\= {{{correctAnswer}}} |\n\n</div>"}]},{"vars":[{"varval":"The graph below shows the favourite subjects of a group of 100 students.\n\nsm_img https://teacher.smartermaths.com.au/wp-content/uploads/2023/08/Statistics-NAP_50011_v4-min.svg 480 indent2 vpad\n\nHow many students in total chose either Technology or English as their favourite subject?"},{"varval":"sm_nogap Total Technology and English\n\n<div class=\"aligned\">\n\n>> | |\n| ---------- |\n| \\= 10 + 13 |\n| \\= {{{correctAnswer}}} |\n\n</div>\n"}]},{"vars":[{"varval":"The graph below shows the favourite subjects of a group of 100 students.\n\nsm_img https://teacher.smartermaths.com.au/wp-content/uploads/2023/08/Statistics-NAP_50011_v4-min.svg 480 indent2 vpad\n\nHow many more students chose Maths as their favourite subject than History?"},{"varval":"sm_nogap Difference between Maths and History\n\n<div class=\"aligned\">\n\n>> | |\n| ---------- |\n| \\= 19 $-$ 9 |\n| \\= {{{correctAnswer}}} |\n\n</div>"}]}]

  30. <div class="sm_mode"> {{{question}}} </div>

    [{"vars":[{"varval":"Beenie recorded her classmates favourite colour in a table.\n\n<div class=\"sm-table col1-color1\">\n\n>>| **Colour** | Red| Blue|Green |Other|\n|:-:|:-:|:-:|:-:|:-:|\n|**Number** | 14|5|8|2|\n\n</div>\n\n<br>She then drew a graph to represent the information.\n\n<br>\n\nsm_img https://teacher.smartermaths.com.au/wp-content/uploads/2023/08/Statistics-NAP_20051_v0a-min.svg 420 indent vpad\n\nBeenie has made a mistake in her graph. Which colour is recorded incorrectly?\n\n\n\n"},{"varval":"{{{correctAnswer}}} is drawn incorrectly with a height of 4."}]},{"vars":[{"varval":"Robin recorded the training distances he ran last month in a table.\n\n<div class=\"sm-table col1-color8\">\n\n>>| **Distance** | 5 km| 10 km|15 km|20 km|\n|:-:|:-:|:-:|:-:|:-:|\n|**Number of Runs** | 7|11|2|4|\n\n</div>\n\n<br>He then drew a graph to represent the information.\n\n<br>\n\nsm_img https://teacher.smartermaths.com.au/wp-content/uploads/2023/08/Statistics-NAP_20051_v1-min.svg 420 indent vpad\n\nRobin has made a mistake in his graph. Which distance is recorded incorrectly?\n\n\n\n"},{"varval":"{{{correctAnswer}}} is drawn incorrectly with a height of 8."}]},{"vars":[{"varval":"Melissa recorded the services she performed at her hair dressing studio on Friday and Saturday.\n\n<div class=\"sm-table col1-color3\">\n\n>>| **Service** | Wash|Cut|Blow Dry|Colour|\n|:-:|:-:|:-:|:-:|:-:|\n|**Number** | 12|8|15|7|\n\n</div>\n\n<br>She then drew a graph to represent the information.\n\n<br>\n\nsm_img https://teacher.smartermaths.com.au/wp-content/uploads/2023/08/Statistics-NAP_20051_v2-min-1.svg 420 indent vpad\n\nMelissa has made a mistake in her graph. Which service is recorded incorrectly?"},{"varval":"{{{correctAnswer}}} is drawn incorrectly with a height of 17."}]},{"vars":[{"varval":"Ping recorded the fish he caught over the last month.\n\n<div class=\"sm-table col1-color5\">\n\n>>| **Fish Type** | Bream|Whiting|Flathead|Tailor|\n|:-:|:-:|:-:|:-:|:-:|\n|**Number** | 11|9|10|13|\n\n</div>\n\n<br>He then drew a graph to represent the information.\n\n<br>\n\nsm_img https://teacher.smartermaths.com.au/wp-content/uploads/2023/08/Statistics-NAP_20051_v3a.svg 420 indent vpad\n\nPing has made a mistake in his graph. Which fish type is recorded incorrectly?"},{"varval":"{{{correctAnswer}}} is drawn incorrectly with a height of 14."}]}]

  31. <div class="sm_mode"> {{{question}}} </div>

    [{"vars":[{"varval":"This graph shows the number of cars cleaned at Shining Car Wash last week.\n\nsm_img https://teacher.smartermaths.com.au/wp-content/uploads/2023/08/Statistics-NAP_30081v0-min.svg 500 indent vpad\n\nHow many more cars were cleaned on Wednesday than on Tuesday?\n"},{"varval":"<div class=\"aligned\">\n\n>| | \n|--------|\n| Cars cleaned on Wednesday = 6 $\\times$ 6 = 36|\n| Cars cleaned on Tuesday = 6 $\\times$ 3 + 6 $\\times$ $\\frac{1}{2}$ = 21|\n| Difference = 36 $-$ 21 = {{{correctAnswer}}}|\n\n</div>\n<br>Therefore, there were {{{correctAnswer}}} more cars cleaned on Wednesday.\n"}]},{"vars":[{"varval":"This graph shows the number of cars cleaned at Shining Car Wash last week.\n\nsm_img https://teacher.smartermaths.com.au/wp-content/uploads/2023/08/Statistics-NAP_30081v0-min.svg 500 indent vpad\n\nHow many cars were cleaned altogether last week?"},{"varval":"<div class=\"aligned\">\n\n>| | | \n|-:|:|\n| Total cars cleaned| = 6 $\\times$ (4.5 + 3.5 + 6 + 4 + 5.5)|\n| |= 6 $\\times$ 23.5 |\n| |= {{{correctAnswer}}}|\n\n</div>\n"}]},{"vars":[{"varval":"This graph shows the number of babies who attended Funtimes Daycare last week.\n\nsm_img https://teacher.smartermaths.com.au/wp-content/uploads/2023/08/Statistics-NAP_30081v1b-min.svg 400 indent vpad\n\nHow many more babies attended on Friday than on Wednesday?"},{"varval":"<div class=\"aligned\">\n\n>| | \n|--------|\n| Babies who attended on Friday = 4 $\\times$ 6 = 24|\n| Babies who attended on Wednesday = 4 $\\times$ 3 = 12|\n| Difference = 24 $-$ 12 = {{{correctAnswer}}}|\n\n</div>\n<br>Therefore, {{{correctAnswer}}} more babies attended on Friday."}]},{"vars":[{"varval":"This graph shows the number of babies who attended Funtimes Daycare last week.\n\nsm_img https://teacher.smartermaths.com.au/wp-content/uploads/2023/08/Statistics-NAP_30081v1b-min.svg 400 indent vpad\n\nHow many babies attended in total last week?"},{"varval":"<div class=\"aligned\">\n\n>| | | \n|-:|:|\n| Total babies| = 4 $\\times$ (4 + 5 + 3 + 5 + 6)|\n| |= 4 $\\times$ 23 |\n| |= {{{correctAnswer}}}|\n\n</div>"}]},{"vars":[{"varval":"This graph shows the number of dogs groomed at K-Nine's grooming last week.\n\nsm_img https://teacher.smartermaths.com.au/wp-content/uploads/2023/08/Statistics-NAP_30081v5-min.svg 500 indent vpad\n\nHow many more dogs were groomed on Thursday than on Monday?"},{"varval":"<div class=\"aligned\">\n\n| | \n|--------|\n| Dogs groomed on Thursday = 8 $\\times$ 6 = 48|\n| Dogs groomed on Monday = 8 $\\times$ 2 = 16|\n| Difference = 48 $-$ 16 = {{{correctAnswer}}}|\n\n</div>\n<br>Therefore, there were {{{correctAnswer}}} more dogs groomed on Thursday."}]},{"vars":[{"varval":"This graph shows the number of dogs groomed at K-Nine's grooming last week.\n\nsm_img https://teacher.smartermaths.com.au/wp-content/uploads/2023/08/Statistics-NAP_30081v5-min.svg 500 indent vpad\n\nHow many dogs were groomed in total last week?"},{"varval":"<div class=\"aligned\">\n\n>| | | \n|-:|:|\n| Total dogs groomed| = 8 $\\times$ (2 + 5 + 3 + 6 + 4)|\n| |= 8 $\\times$ 20 |\n| |= {{{correctAnswer}}}|\n\n</div>"}]}]

  32. <div class="sm_mode"> {{{question}}} </div>

    [{"vars":[{"varval":"Nataya had a market stall selling cookies.\n\nThe graph below shows how many of each type of cookie she sold.\n\nChocolate chip cookies were the most popular.\n\nCoconut were less popular than blueberry but more popular than choc mint.\n\n<br>\n\nsm_img https://teacher.smartermaths.com.au/wp-content/uploads/2023/08/Statistics-NAP_20071_v0-min.svg 350 indent2 vpad\n\nWhich column on the graph represents blueberry cookies?"},{"varval":"From the information given, the order of popularity is:\n\n1. >Chocolate chip (favourite)\n2. >Blueberry\n3. >Coconut (not as popular as Blueberry but more than choc mint)\n4. >Choc mint\n\nTherefore, **{{{correctAnswer}}}** represents blueberry cookies.\n\n"}]},{"vars":[{"varval":"Sanjay sold comics at the markets on the weekend.\n\nThe graph below shows how many of each type of comic he sold.\n\nStar Wars comics were the least popular.\n\nSpiderman comics were less popular than Marvel Universe but more popular than The Hulk.\n\n<br>\n\nsm_img https://teacher.smartermaths.com.au/wp-content/uploads/2023/08/Statistics-NAP_20071_v1-min.svg 350 indent2 vpad\n\nWhich column on the graph represents Spiderman comics?"},{"varval":"From the information given, the order of popularity is:\n\n1. >Marvel Universe\n2. >Spiderman (not as popular as Marvel Universe but more than The Hulk)\n3. >The Hulk\n4. >Star Wars (least popular)\n\nTherefore, **{{{correctAnswer}}}** represents Spiderman comics.\n"}]},{"vars":[{"varval":"A group of teenagers were surveyed to find out their favourite sports.\n\nThe graph below shows the results of the survey listing the four most popular sports.\n\nSoccer was the most popular.\n\nNetball was less popular than basketball but more popular than baseball.\n\n<br>\n\nsm_img https://teacher.smartermaths.com.au/wp-content/uploads/2023/08/Statistics-NAP_20071_v2-min.svg 350 indent2 vpad\n\nWhich column on the graph represents netball?"},{"varval":"From the information given, the order of popularity is:\n\n1. >Soccer (most popular)\n2. >Basketball\n3. >Netball (not as popular as basketball but more than baseball )\n4. >Baseball\n\nTherefore, **{{{correctAnswer}}}** represents netball.\n"}]},{"vars":[{"varval":"Jocelyn sold fruit at the markets on the weekend.\n\nThe graph below shows how many of each type of fruit she sold.\n\nApples were the least popular.\n\nBananas were the less popular than pears but more popular than oranges.\n\n<br>\n\nsm_img https://teacher.smartermaths.com.au/wp-content/uploads/2023/08/Statistics-NAP_20071_v3-min.svg 350 indent2 vpad\n\nWhich column on the graph represents pears?"},{"varval":"From the information given, the order of popularity is:\n\n1. >Pears \n2. >Bananas(not as popular as Pears but more popular than oranges)\n3. >Oranges \n4. >Apples (least favourite)\n\nTherefore, **{{{correctAnswer}}}** represents pears.\n"}]},{"vars":[{"varval":"Jake was sorting coins from his moneybox.\n\nThe graph below shows how many of each type of coin he had.\n\nHe had more $2 coins than any other coin.\n\nThe number of 50 cent coins was more than $1 coins but less than 20 cent coins.\n\n<br>\n\nsm_img https://teacher.smartermaths.com.au/wp-content/uploads/2023/08/Statistics-NAP_20071_v4-min.svg 350 indent2 vpad\n\nWhich column on the graph represents $1 coins?"},{"varval":"From the information given the order of popularity is:\n\n1. >$2 coins (Most)\n2. >20 cent\n3. >50 cent (more than $1 coins but less than 20 cent coins)\n4. >$1 coins\n\nTherefore, **{{{correctAnswer}}}** represents $1 coins.\n"}]},{"vars":[{"varval":"Marlon went fishing on the weekend.\n\nHe recorded the number of each type of fish he caught in the graph below.\n\nHe caught more bream than any other type of fish.\n\nTailor numbered more than flathead but not as many as snapper.\n\n<br>\n\nsm_img https://teacher.smartermaths.com.au/wp-content/uploads/2023/08/Statistics-NAP_20071_v5-min.svg 350 indent2 vpad\n\nWhich column on the graph represents tailor?"},{"varval":"From the information given the order of popularity is:\n\n1. >Bream (most)\n2. >Snapper\n3. >Tailor (more than flathead but not as many as snapper)\n4. >Flathead\n\nTherefore, **{{{correctAnswer}}}** represents tailor.\n"}]}]

  33. SJ Tables

    <div class="sm_mode"> {{{question}}} </div>

    [{"vars":[{"varval":"Three families flew from Melbourne to Queensland for a holiday.\n\nEach person was allowed to carry 21 kilograms of luggage.\n\n<br>\n\nsm_img https://teacher.smartermaths.com.au/wp-content/uploads/2023/08/Statistics-NAP_10031v0a-min.svg 330 indent2 vpad\n\n<br>Which family took more than 21 kilograms of luggage per person?\n"},{"varval":"Checking each Option\n\n\nsm_nogap > Option 1: Wilmott allowed baggage = 21 $\\times$ 3 = 63 kg $\\checkmark$\n\n>Option 2: Bergen allowed baggage = 21 $\\times$ 2 = 42 kg $\\checkmark$\n\n>Option 3: Tran allowed baggage = 21 $\\times$ 5 = 105 kg (5 kg overweight)\n\n>Option 2: O'Brien allowed baggage = 21 $\\times$ 4 = 84 kg $\\checkmark$\n\n\n<br>Therefore, the {{{correctAnswer}}} family took more than 21 kg of luggage per person.\n\n"}]},{"vars":[{"varval":"Four groups of friends caught the Indian Pacific train from Sydney to Perth for a holiday.\n\nEach person was allowed to carry 25 kilograms of luggage.\n\n<br>\n\nsm_img https://teacher.smartermaths.com.au/wp-content/uploads/2023/08/Statistics-NAP_10031v1a-min.svg 330 indent2 vpad\n\n<br>Which group took more than 25 kilograms of luggage per person?\n"},{"varval":"Checking each Option\n\n\nsm_nogap > Option 1: Group 1 allowed baggage = 25 $\\times$ 5 = 125 kg $\\checkmark$\n\n>Option 2: Group 2 allowed baggage = 25$\\times$ 2 = 50 kg $\\checkmark$\n\n>Option 3: Group 3 allowed baggage = 25 $\\times$ 3 = 75 kg $\\checkmark$\n\n>Option 2: Group 4 allowed baggage = 25 $\\times$ 4 = 100 kg (5 kg overweight)\n\n\n\n<br>Therefore, {{{correctAnswer}}} took more than 25 kg of luggage per person.\n"}]},{"vars":[{"varval":"Four families were travelling by plane to England.\n\nEach person was able to carry a maximum of 7 kilograms of hand luggage.\n\n<br>\n\nsm_img https://teacher.smartermaths.com.au/wp-content/uploads/2023/08/Statistics-NAP_10031v2-min.svg 430 indent2 vpad\n\n<br>Which family took more than 7 kilograms of hand luggage per person?\n"},{"varval":"Checking each Option\n\n\nsm_nogap > Option 1: Miscevski family allowed hand luggage = 7 $\\times$ 5 = 35kg $\\checkmark$\n\n>Option 2: Smith family allowed hand luggage = 7 $\\times$ 8 = 56 kg $\\checkmark$\n\n>Option 3: Barnes family allowed hand luggage = 7 $\\times$ 3 = 21 kg (1 kg overweight)\n\n>Option 2: McWilliams family allowed hand luggage = 7 $\\times$ 6 = 42 kg $\\checkmark$\n\n\n\n<br>Therefore, the {{{correctAnswer}}} family took more than 7 kg of hand luggage per person.\n"}]},{"vars":[{"varval":"A company uses 4 barges to transport goods in containers across a river.\n\nEach container can carry a maximum of 600 kilograms.\n\n<br>\n\nsm_img https://teacher.smartermaths.com.au/wp-content/uploads/2023/08/Statistics-NAP_10031v3a-min.svg 430 indent2 vpad\n\n<br>Which barge had containers holding more than 600 kilograms per container?\n"},{"varval":"Checking each Option\n\n\nsm_nogap > Option 1: Barge 1 allowed container mass = 600 $\\times$ 3 = 1800 kg (100 kg overweight)\n\n>Option 2: Barge 2 allowed container mass = 600 $\\times$ 2 = 1200 kg $\\checkmark$\n\n>Option 3: Barge 3 allowed container mass = 600 $\\times$ 4 = 2400 kg $\\checkmark$\n\n>Option 2: Barge 4 allowed container mass = 600 $\\times$ 1 = 600 kg $\\checkmark$\n\n\n\n<br>Therefore, {{{correctAnswer}}} had containers holding more than 600 kg per container."}]}]

  34. <div class="sm_mode"> {{{question}}} </div>

    [{"vars":[{"varval":"Which of the following pairs of spinners have the same chance of landing on the coloured section?"},{"varval":"Counting the coloured sections:\n\n<div class=\"sm_img_inline\">\n\n>Option 1\n\n<br>\n\n>sm_img https://teacher.smartermaths.com.au/wp-content/uploads/2023/08/spinner-8-2-x-5-min.svg 200 indent vpad\n\n>Spinner 1 has 5 sections coloured\n\n>Spinner 2 has 5 sections coloured\n\n<br>\n\n>Option 2\n\n<br>\n\n>sm_img https://teacher.smartermaths.com.au/wp-content/uploads/2023/08/spinner-8-5b-x-4b.svg 200 indent2 vpad \n\n>Spinner 1 has 5 sections coloured\n\n>Spinner 2 has 4 sections coloured\n\n<br>\n\n>Option 3\n\n<br>\n\n>sm_img https://teacher.smartermaths.com.au/wp-content/uploads/2023/08/spinner-8-3-x-5a.svg 200 indent2 vpad \n\n>Spinner 1 has 3 sections coloured\n\n>Spinner 2 has 5 sections coloured\n\n\n<br>\n\n>Option 4\n\n<br>\n\n>sm_img https://teacher.smartermaths.com.au/wp-content/uploads/2023/08/spinner-8-2a-x-6.svg 200 indent2 vpad \n\n>Spinner 1 has 2 sections coloured\n\n>Spinner 2 has 6 sections coloured\n\n<br>\n\n>Therefore, Option 1 spinners have the same chance of landing on\n\n>the coloured sections as they both have 5 sections coloured.\n\n</div>\n\n\n"}]},{"vars":[{"varval":"Which of the following pairs of spinners have the same chance of landing on the coloured section?"},{"varval":"Counting the coloured sections:\n\n<div class=\"sm_img_inline\">\n\n>Option 1\n\n<br>\n\n>sm_img https://teacher.smartermaths.com.au/wp-content/uploads/2023/08/spinner-8-2a-x-6.svg 250 indent2 vpad \n\n>Spinner 1 has 2 sections coloured\n\n>Spinner 2 has 6 sections coloured\n\n<br>\n\n>Option 2\n\n<br>\n\n>sm_img https://teacher.smartermaths.com.au/wp-content/uploads/2023/08/spinner-8-4-x-4b.svg 250 indent2 vpad \n\n>Spinner 1 has 4 sections coloured\n\n>Spinner 2 has 4 sections coloured\n\n<br>\n\n>Option 3\n\n<br>\n\n>sm_img https://teacher.smartermaths.com.au/wp-content/uploads/2023/08/spinner-8-3-x-5a.svg 250 indent2 vpad \n\n>Spinner 1 has 3 sections coloured\n\n>Spinner 2 has 5 sections coloured\n\n\n<br>\n\n>Option 4\n\n<br>\n\n>sm_img https://teacher.smartermaths.com.au/wp-content/uploads/2023/08/spinner-8-5b-x-4b.svg 250 indent2 vpad \n\n>Spinner 1 has 5 sections coloured\n\n>Spinner 2 has 4 sections coloured\n\n<br>\n\n>Therefore, Option 2 spinners have the same chance of landing on\n\n>the coloured sections as they both have 4 sections coloured.\n\n</div>\n"}]},{"vars":[{"varval":"Which of the following pairs of spinners have the same chance of landing on the coloured section?"},{"varval":"Counting the coloured sections:\n\n<div class=\"sm_img_inline\">\n\n>Option 1\n\n<br>\n\n>sm_img https://teacher.smartermaths.com.au/wp-content/uploads/2023/08/spinner-8-5b-x-4b.svg 200 indent2 vpad \n\n>Spinner 1 has 5 sections coloured\n\n>Spinner 2 has 4 sections coloured\n\n<br>\n\n>Option 2\n\n<br>\n\n>sm_img https://teacher.smartermaths.com.au/wp-content/uploads/2023/08/spinner-8-2a-x-6.svg 200 indent2 vpad \n\n>Spinner 1 has 2 sections coloured\n\n>Spinner 2 has 6 sections coloured\n\n<br>\n\n>Option 3\n\n<br>\n\n>sm_img https://teacher.smartermaths.com.au/wp-content/uploads/2023/08/spinner-8-3-x-2.svg 200 indent2 vpad \n\n>Spinner 1 has 3 sections coloured\n\n>Spinner 2 has 2 sections coloured\n\n\n<br>\n\n>Option 4\n\n<br>\n\n>sm_img https://teacher.smartermaths.com.au/wp-content/uploads/2023/08/spinner-8-3-x-3a.svg 200 indent2 vpad \n\n>Spinner 1 has 3 sections coloured\n\n>Spinner 2 has 3 sections coloured\n\n<br>\n\n>Therefore, Option 4 spinners have the same chance of landing on\n\n>the coloured sections as they both have 3 sections coloured.\n\n</div>\n"}]}]

  35. <div class="sm_mode"> {{{question}}} </div>

    [{"vars":[{"varval":"These identical coloured discs were in a bag.\n\nBrandon selected one disc.\n\nsm_img https://teacher.smartermaths.com.au/wp-content/uploads/2023/08/discs-12-min.svg 220 indent2 vpad \n\nWhich is the colour that is most likely to be selected by Brandon?\n"},{"varval":"Counting the discs of each type:\n\n<div class=\"sm_img_inline\">\n\n>5 $\\times$ ![](https://teacher.smartermaths.com.au/wp-content/uploads/2023/08/discs-blue.svg)\n\n<br>\n\n>4 $\\times$ ![](https://teacher.smartermaths.com.au/wp-content/uploads/2023/08/discs-green.svg)\n\n<br>\n\n>3 $\\times$ ![](https://teacher.smartermaths.com.au/wp-content/uploads/2023/08/discs-yellow.svg)\n\n<br>\n\n>Option 4 is incorrect as there are different numbers of each colour.\n\n<br>Therefore, ![](https://teacher.smartermaths.com.au/wp-content/uploads/2023/08/discs-blue.svg) is most likely.\n\n</div>"}]},{"vars":[{"varval":"These identical coloured discs were in a bag.\n\nEli selected one disc.\n\nsm_img https://teacher.smartermaths.com.au/wp-content/uploads/2023/08/discs-12-min.svg 220 indent2 vpad \n\nWhich is the colour that is least likely to be selected by Eli?\n"},{"varval":"Counting the discs of each type:\n\n<div class=\"sm_img_inline\">\n\n>5 $\\times$ ![](https://teacher.smartermaths.com.au/wp-content/uploads/2023/08/discs-blue.svg)\n\n<br>\n\n>4 $\\times$ ![](https://teacher.smartermaths.com.au/wp-content/uploads/2023/08/discs-green.svg)\n\n<br>\n\n>3 $\\times$ ![](https://teacher.smartermaths.com.au/wp-content/uploads/2023/08/discs-yellow.svg)\n\n<br>\n\n>Option 4 is incorrect as there are different numbers of each colour.\n\n<br>\n\nTherefore, ![](https://teacher.smartermaths.com.au/wp-content/uploads/2023/08/discs-yellow.svg) is least likely.\n\n</div>"}]},{"vars":[{"varval":"These identical numbered discs were in a bag.\n\nLivvy selected one disc.\n\nsm_img https://teacher.smartermaths.com.au/wp-content/uploads/2023/08/discs-numbered-20.svg 220 indent2 vpad \n\nWhat is the chance that the disc Livvy selected is an even number?\n"},{"varval":"Counting the discs of each type:\n\n<div class=\"sm_img_inline\">\n\n>Even Numbers:\n\n<br>\n\n>3 $\\times$ ![](https://teacher.smartermaths.com.au/wp-content/uploads/2023/08/discs-number-2-min.svg) \n\n<br>\n\n>4 $\\times$ ![](https://teacher.smartermaths.com.au/wp-content/uploads/2023/08/discs-number-4-min.svg)\n\n<br>\n\n>1 $\\times$ ![](https://teacher.smartermaths.com.au/wp-content/uploads/2023/08/discs-number-6-min.svg) \n\n<br>\n\n>2 $\\times$ ![](https://teacher.smartermaths.com.au/wp-content/uploads/2023/08/discs-number-8-min.svg)\n\n<br>\n\n>>Total number of even numbers is 10.\n\n<br>\n\n>Odd Numbers:\n\n<br>\n\n>3 $\\times$ ![](https://teacher.smartermaths.com.au/wp-content/uploads/2023/08/discs-number-1-min.svg) \n\n<br>\n\n>2 $\\times$ ![](https://teacher.smartermaths.com.au/wp-content/uploads/2023/08/discs-number-3-min.svg)\n\n<br>\n\n>3 $\\times$ ![](https://teacher.smartermaths.com.au/wp-content/uploads/2023/08/discs-number-5-min.svg) \n\n<br>\n\n>2 $\\times$ ![](https://teacher.smartermaths.com.au/wp-content/uploads/2023/08/discs-number-9-min.svg)\n\n<br>\n\n>>Total number of odd numbers is 10.\n\n</div>\n\n<br>\n\n>Therefore, the chance that an odd number is selected is {{correctAnswer}}."}]},{"vars":[{"varval":"These identical numbered discs were in a bag.\n\nManni selected one disc.\n\nsm_img https://teacher.smartermaths.com.au/wp-content/uploads/2023/08/discs-numbered-20.svg 220 indent2 vpad \n\nWhat is the chance that the disc Manni selected is an odd number?"},{"varval":"Counting the discs of each type:\n\n<div class=\"sm_img_inline\">\n\n>Even Numbers:\n\n<br>\n\n>3 $\\times$ ![](https://teacher.smartermaths.com.au/wp-content/uploads/2023/08/discs-number-2-min.svg) \n\n<br>\n\n>4 $\\times$ ![](https://teacher.smartermaths.com.au/wp-content/uploads/2023/08/discs-number-4-min.svg)\n\n<br>\n\n>1 $\\times$ ![](https://teacher.smartermaths.com.au/wp-content/uploads/2023/08/discs-number-6-min.svg) \n\n<br>\n\n>2 $\\times$ ![](https://teacher.smartermaths.com.au/wp-content/uploads/2023/08/discs-number-8-min.svg)\n\n<br>\n\n>>Total number of even numbers is 10.\n\n<br>\n\n>Odd Numbers:\n\n<br>\n\n>3 $\\times$ ![](https://teacher.smartermaths.com.au/wp-content/uploads/2023/08/discs-number-1-min.svg) \n\n<br>\n\n>2 $\\times$ ![](https://teacher.smartermaths.com.au/wp-content/uploads/2023/08/discs-number-3-min.svg)\n\n<br>\n\n>3 $\\times$ ![](https://teacher.smartermaths.com.au/wp-content/uploads/2023/08/discs-number-5-min.svg) \n\n<br>\n\n>2 $\\times$ ![](https://teacher.smartermaths.com.au/wp-content/uploads/2023/08/discs-number-9-min.svg)\n\n<br>\n\n>>Total number of odd numbers is 10.\n\n</div>\n\n<br>\n\n>Therefore, the chance that an odd number is selected is {{correctAnswer}}."}]},{"vars":[{"varval":"These identical numbered discs were in a bag.\n\nGay selected one disc.\n\nsm_img https://teacher.smartermaths.com.au/wp-content/uploads/2023/08/discs-numbered-20.svg 220 indent2 vpad \n\nWhat is the chance that the disc Gay selected had a 7 on it?"},{"varval":"The discs have the following numbers:\n\n<div class=\"sm_img_inline\">\n\n<br>\n\n>>![](https://teacher.smartermaths.com.au/wp-content/uploads/2023/08/discs-number-1-min.svg) ![](https://teacher.smartermaths.com.au/wp-content/uploads/2023/08/discs-number-2-min.svg)\n![](https://teacher.smartermaths.com.au/wp-content/uploads/2023/08/discs-number-3-min.svg) \n![](https://teacher.smartermaths.com.au/wp-content/uploads/2023/08/discs-number-4-min.svg)\n![](https://teacher.smartermaths.com.au/wp-content/uploads/2023/08/discs-number-5-min.svg) \n![](https://teacher.smartermaths.com.au/wp-content/uploads/2023/08/discs-number-6-min.svg)\n ![](https://teacher.smartermaths.com.au/wp-content/uploads/2023/08/discs-number-8-min.svg) \n![](https://teacher.smartermaths.com.au/wp-content/uploads/2023/08/discs-number-9-min.svg)\n\n<br>\n\n>There are no discs with a 7 on them\n\n</div>\n\n<br>\n\nTherefore, the chance that a disc with a 7 on it is selected is {{correctAnswer}}."}]},{"vars":[{"varval":"These identical numbered discs were in a bag.\n\nJulio selected one disc.\n\nsm_img https://teacher.smartermaths.com.au/wp-content/uploads/2023/08/discs-numbered-20.svg 220 indent2 vpad \n\nWhat is the chance that the disc Julio selected is a number less than 10?"},{"varval":"All the discs have numbers less than ten.\n\n<div class=\"sm_img_inline\">\n\n<br>\n\n>>![](https://teacher.smartermaths.com.au/wp-content/uploads/2023/08/discs-number-1-min.svg) ![](https://teacher.smartermaths.com.au/wp-content/uploads/2023/08/discs-number-2-min.svg)\n![](https://teacher.smartermaths.com.au/wp-content/uploads/2023/08/discs-number-3-min.svg) \n![](https://teacher.smartermaths.com.au/wp-content/uploads/2023/08/discs-number-4-min.svg)\n![](https://teacher.smartermaths.com.au/wp-content/uploads/2023/08/discs-number-5-min.svg) \n![](https://teacher.smartermaths.com.au/wp-content/uploads/2023/08/discs-number-6-min.svg)\n ![](https://teacher.smartermaths.com.au/wp-content/uploads/2023/08/discs-number-8-min.svg) \n![](https://teacher.smartermaths.com.au/wp-content/uploads/2023/08/discs-number-9-min.svg)\n\n<br>\n\n</div>\n\nTherefore, the chance of a number being selected that is less than 10 is {{{correctAnswer}}}."}]}]

  36. <div class="sm_mode"> {{{question}}} </div>

    [{"vars":[{"varval":"A group of students tallied the different types of pets they owned.\n\nA table of their responses is shown below.\n\n<br>\n\nsm_img https://teacher.smartermaths.com.au/wp-content/uploads/2023/08/Statistics-NAP_10011_pets-min.svg 280 indent2 vpad\n\n<br>How many students own a cat?"},{"varval":"sm_img https://teacher.smartermaths.com.au/wp-content/uploads/2023/08/Statistics-NAP_10011_pets-ws-min.svg 350 indent2 vpad\n\nTherefore, {{{correctAnswer}}} children own a cat.\n"}]},{"vars":[{"varval":"A group of students tallied the different types of pets they owned.\n\nA table of their responses is shown below.\n\n<br>\n\nsm_img https://teacher.smartermaths.com.au/wp-content/uploads/2023/08/Statistics-NAP_10011_pets-min.svg 280 indent2 vpad\n\n<br>How many more cats were owned that fish?"},{"varval":"sm_img https://teacher.smartermaths.com.au/wp-content/uploads/2023/08/Statistics-NAP_10011_pets-ws-min.svg 350 indent2 vpad\n\nsm_nogap Difference between cats and fish\n\n<div class=\"aligned\">\n\n>> | |\n| ---------- |\n| \\= 12 $-$ 6 |\n| \\= {{{correctAnswer}}} |\n\n</div>\n\n\n"}]},{"vars":[{"varval":"A group of students tallied the different types of pets they owned.\n\nA table of their responses is shown below.\n\n<br>\n\nsm_img https://teacher.smartermaths.com.au/wp-content/uploads/2023/08/Statistics-NAP_10011_pets-min.svg 280 indent2 vpad\n\n<br>How many pets were owned in total?"},{"varval":"sm_img https://teacher.smartermaths.com.au/wp-content/uploads/2023/08/Statistics-NAP_10011_pets-ws-min.svg 350 indent2 vpad\n\nsm_nogap Total pets owned\n\n<div class=\"aligned\">\n\n>> | |\n| ---------- |\n| \\= 8 + 12 + 6 + 2 + 1 |\n| \\= {{{correctAnswer}}} |\n\n</div>\n"}]},{"vars":[{"varval":"A group of students tallied the different types of cars that passed the school during a 10 minute period.\n\nA table of their responses is shown below.\n\n<br>\n\nsm_img https://teacher.smartermaths.com.au/wp-content/uploads/2023/08/Statistics-NAP_10011_car-min.svg 280 indent2 vpad\n\n<br>How many Kia cars passed the school in the 10 minute period?"},{"varval":"sm_img https://teacher.smartermaths.com.au/wp-content/uploads/2023/08/Statistics-NAP_10011_car_ws-min.svg 350 indent2 vpad\n\nTherefore, {{{correctAnswer}}} Kia cars passed the school in the 10 minute period.\n"}]},{"vars":[{"varval":"A group of students tallied the different types of cars that passed the school during a 10 minute period.\n\nA table of their responses is shown below.\n\n<br>\n\nsm_img https://teacher.smartermaths.com.au/wp-content/uploads/2023/08/Statistics-NAP_10011_car-min.svg 280 indent2 vpad\n\n<br>How many more Ford cars passed the school than Hyundai cars?"},{"varval":"sm_img https://teacher.smartermaths.com.au/wp-content/uploads/2023/08/Statistics-NAP_10011_car_ws-min.svg 350 indent2 vpad\n\nsm_nogap Difference between Ford and Hyundai\n\n<div class=\"aligned\">\n\n>> | |\n| ---------- |\n| \\= 15 $-$ 9 |\n| \\= {{{correctAnswer}}} |\n\n</div>"}]},{"vars":[{"varval":"A group of students tallied the different types of cars that passed the school during a 10 minute period.\n\nA table of their responses is shown below.\n\n<br>\n\nsm_img https://teacher.smartermaths.com.au/wp-content/uploads/2023/08/Statistics-NAP_10011_car-min.svg 280 indent2 vpad\n\n<br>How many cars in total passed the school in the 10 minute period?"},{"varval":"sm_img https://teacher.smartermaths.com.au/wp-content/uploads/2023/08/Statistics-NAP_10011_car_ws-min.svg 350 indent2 vpad\n\nsm_nogap Total pets owned\n\n<div class=\"aligned\">\n\n>> | |\n| ---------- |\n| \\= 15 + 12 + 9 + 14 |\n| \\= {{{correctAnswer}}} |\n\n</div>\n"}]}]

  37. <div class="sm_mode"> {{{question}}} </div>

    [{"vars":[{"varval":"Complete the number sentence\n"},{"varval":"<div class=\"aligned\">\n\n| | |\n| ------------- | ---------- |\n| <span class=\"sm_box\"> ` ` ` ` </span> | \\= 65 $-$ 24 |\n| | \\= 65 $-$ 20 $-$ 4 |\n| | \\= 45 $-$ 4 |\n| | \\= {{{correctAnswer0}}} |\n\n\n</div>\n\n"}]},{"vars":[{"varval":"Complete the number sentence"},{"varval":"<div class=\"aligned\">\n\n| | |\n| ------------- | ---------- |\n| <span class=\"sm_box\"> ` ` ` ` </span> | \\= 39 $-$ 13 |\n| | \\= 39 $-$ 10 $-$ 3 |\n| | \\= 29 $-$ 3 |\n| | \\= {{{correctAnswer0}}} |\n\n\n</div>\n"}]},{"vars":[{"varval":"Complete the number sentence"},{"varval":"<div class=\"aligned\">\n\n| | |\n| ------------- | ---------- |\n| <span class=\"sm_box\"> ` ` ` ` </span> | \\= 78 $-$ 43 |\n| | \\= 78 $-$ 40 $-$ 3 |\n| | \\= 38 $-$ 3 |\n| | \\= {{{correctAnswer0}}} |\n\n\n</div>\n"}]},{"vars":[{"varval":"Complete the number sentence"},{"varval":"<div class=\"aligned\">\n\n| | |\n| ------------- | ---------- |\n| <span class=\"sm_box\"> ` ` ` ` </span> | \\= 84 $-$ 55 |\n| | \\= 84 $-$ 50 $-$ 5 |\n| | \\= 34 $-$ 5 |\n| | \\= {{{correctAnswer0}}} |\n\n\n</div>\n"}]},{"vars":[{"varval":"Complete the number sentence"},{"varval":"<div class=\"aligned\">\n\n| | |\n| ------------- | ---------- |\n| <span class=\"sm_box\"> ` ` ` ` </span> | \\= 92 $-$ 38 |\n| | \\= 92 $-$ 30 $-$ 8 |\n| | \\= 62 $-$ 8 |\n| | \\= {{{correctAnswer0}}} |\n\n\n</div>\n"}]},{"vars":[{"varval":"Complete the number sentence"},{"varval":"<div class=\"aligned\">\n\n| | |\n| ------------- | ---------- |\n| <span class=\"sm_box\"> ` ` ` ` </span> | \\= 61 $-$ 37 |\n| | \\= 61 $-$ 30 $-$ 7 |\n| | \\= 31 $-$ 7 |\n| | \\= {{{correctAnswer0}}} |\n\n\n</div>\n"}]}]

  38. SJ Units

    <div class="sm_mode"> {{{question}}} </div>

    [{"vars":[{"varval":"Which of the following is greater than 1 metre?"},{"varval":"Compare all options to the conversions\n\n>>1 metre = 100 centimetres = 1000 millimetres\n\n<br>\n\n>>Option 1 = 95 centimetres is less than 1 metres\n\n>>Option 2 = 500 millimetres is less than 1 metre\n\n>>Option 3 = 900 millimetres is less than 1 metre\n\n>>Option 4 = {{{correctAnswer}}} is greater than 1 metre $\\checkmark$\n\n\n<br>$\\therefore$ {{{correctAnswer}}} is greater than 1 metre\n"}]},{"vars":[{"varval":"Which of the following is greater than 2 metres?"},{"varval":"Compare all options to the conversions\n\n>>2 metres = 200 centimetres = 2000 millimetres\n\n<br>\n\n>>Option 1 = 1500 millimetres = 1.5 metres ( $\\times$ )\n\n>>Option 2 = {{{correctAnswer}}} = 2.5 metres ( $\\checkmark$ )\n\n\n>>Option 3 = 50 centimetres = 0.5 metres ( $\\times$ )\n\n>>Option 4 = 150 centimetres = 1.5 metres( $\\times$ )\n\n\n<br>$\\therefore$ {{{correctAnswer}}} is greater than 1 metre"}]},{"vars":[{"varval":"Which of the following is greater than 900 centimetres?"},{"varval":"Compare all options to the conversions\n\n>>900 centimetres = 9 metres = 9000 millimetres\n\n<br>\n\n>>Option 1 = 8000 mm = 800 cm ( $\\times$ )\n\n>>Option 2 = 6 m = 600 cm ( $\\times$ )\n\n>>Option 3 = {{{correctAnswer}}} = 950 cm ( $\\checkmark$ )\n\n>>Option 4 = 8 m = 800 cm ( $\\times$ )\n\n\n<br>$\\therefore$ {{{correctAnswer}}} is greater than 1 metre"}]},{"vars":[{"varval":"Which of the following is greater than 500 centimetres?"},{"varval":"Compare all options to the conversions\n\n>>100 centimetres = 1 metres = 1000 millimetres\n\n>>500 centimetres = 5 metres = 5000 millimetres\n\n<br>\n\n>>Option 1 = 6000 mm = 600 cm ( $\\checkmark$ )\n\n>>Option 2 = 3 m = 300 cm ( $\\times$ )\n\n>>Option 3 = 600 mm = 60 cm ( $\\times$ )\n\n>>Option 4 = 0.5 m = 50 cm ( $\\times$ )\n\n\n<br>$\\therefore$ {{{correctAnswer}}} is greater than 1 metre"}]}]

  39. SJ Units

    <div class="sm_mode"> {{{question}}} </div>

    [{"vars":[{"varval":"Which of the following is the same as 3 kilometres?"},{"varval":"1 kilometre = 1000 metres\n\n$\\therefore$ 3 kilometres = {{{correctAnswer}}}"}]},{"vars":[{"varval":"Which of the following is the same as 2 kilometres?"},{"varval":"1 kilometre = 1000 metres\n\n$\\therefore$ 2 kilometres = {{{correctAnswer}}}"}]},{"vars":[{"varval":"Which of the following is the same as 10 kilometres?"},{"varval":"1 kilometre = 1000 metres\n\n$\\therefore$ 10 kilometres = {{{correctAnswer}}}"}]},{"vars":[{"varval":"Which of the following is the same as 5 metres?"},{"varval":"1 metre = 100 centimetres\n\n$\\therefore$ 5 metres = {{{correctAnswer}}}"}]},{"vars":[{"varval":"Which of the following is the same as 6 metres?"},{"varval":"1 metre = 1000 millimetres\n\n$\\therefore$ 6 metres = {{{correctAnswer}}}"}]},{"vars":[{"varval":"Which of the following is the same as 8 metres?"},{"varval":"1 metre = 100 centimetres\n\n$\\therefore$ 8 metres = {{{correctAnswer}}}"}]}]

  40. SJ Units

    <div class="sm_mode"> {{{question}}} </div>

    [{"vars":[{"varval":"Which of the following is the same as 2 kilograms?"},{"varval":"1 kilogram = 1000 grams\n\n$\\therefore$ 2 kilograms = {{{correctAnswer}}}"}]},{"vars":[{"varval":"Which of the following is the same as 5 kilograms?"},{"varval":"1 kilogram = 1000 grams\n\n$\\therefore$ 5 kilograms = {{{correctAnswer}}}"}]},{"vars":[{"varval":"Which of the following is the same as 3 kilograms?"},{"varval":"1 kilogram = 1000 grams\n\n$\\therefore$ 3 kilograms = {{{correctAnswer}}}"}]},{"vars":[{"varval":"Which of the following is the same as 4 kilograms?"},{"varval":"1 kilogram = 1000 grams\n\n$\\therefore$ 4 kilograms = {{{correctAnswer}}}"}]},{"vars":[{"varval":"Which of the following is the same as 9 kilograms?"},{"varval":"1 kilogram = 1000 grams\n\n$\\therefore$ 9 kilograms = {{{correctAnswer}}}"}]},{"vars":[{"varval":"Which of the following is the same as 11 kilograms?"},{"varval":"1 kilogram = 1000 grams\n\n$\\therefore$ 11 kilograms = {{{correctAnswer}}}"}]}]

  41. SJ Units

    <div class="sm_mode"> {{{question}}} </div>

    [{"vars":[{"varval":"Which is greater, 2 litres or 2000 millilitres?"},{"varval":"1 litre = 1000 millilitres\n\n2 litres = 2000 millilitres\n\n$\\therefore$ {{{correctAnswer}}}"}]},{"vars":[{"varval":"Which is smaller, 5 kilometres or 4500 metres?"},{"varval":"1000 metres = 1 kilometre\n\n5000 metres = 5 kilometres\n\n4500 metres < 5000 metres\n\nTherefore, {{{correctAnswer}}} is smaller."}]},{"vars":[{"varval":"Which is greater, 1 centimetre or 15 millimetres?"},{"varval":"1 centimetre = 10 millimetres\n\n10 millimetres < 15 millimetres\n\nTherefore, {{{correctAnswer}}} is greater."}]},{"vars":[{"varval":"Which is greater, 5000 grams or 5 kilograms?"},{"varval":"1000 grams = 1 kilogram\n\n5000 grams = 5 kilograms\n\n$\\therefore$ {{{correctAnswer}}}"}]},{"vars":[{"varval":"Which is smaller, 50 centimetres or 1 metre?"},{"varval":"1 metre = 100 centimetres\n\n50 centimetres < 100 centimetres\n\nTherefore, {{{correctAnswer}}} is smaller."}]},{"vars":[{"varval":"Which is greater, 40 centimetres or 400 millimetres?"},{"varval":"1 centimetre = 10 millimetres\n\n4 centimetres = 40 millimetres\n\n40 centimetres = 400 millimetres\n\n$\\therefore$ {{{correctAnswer}}}"}]}]

  42. SJ Units

    <div class="sm_mode"> {{{question}}} </div>

    [{"vars":[{"varval":"Which of the following would be the best estimate for the weight of a cat?\n\nsm_img https://teacher.smartermaths.com.au/wp-content/uploads/2023/07/cat-1-min.svg 150 indent vpad"},{"varval":"A cat would weigh approximately {{{correctAnswer}}}"}]},{"vars":[{"varval":"Which of the following would be the best estimate for the capacity of a carton of milk?\n\nsm_img https://teacher.smartermaths.com.au/wp-content/uploads/2023/07/milk-1-min.svg 150 indent2 vpad"},{"varval":"A carton of milk would have a capacity of approximately {{{correctAnswer}}}"}]},{"vars":[{"varval":"Which of the following would be the best estimate for the weight of a fire truck?\n\nsm_img https://teacher.smartermaths.com.au/wp-content/uploads/2023/07/fire-truck-1-min.svg 250 indent vpad"},{"varval":"A fire truck would weigh approximately {{{correctAnswer}}}"}]},{"vars":[{"varval":"Which of the following would be the best estimate for the height of a fridge?\n\n<br>\n\nsm_img https://teacher.smartermaths.com.au/wp-content/uploads/2023/07/fridge-1.svg 100 indent2 vpad"},{"varval":"The height of a fridge would be approximately {{{correctAnswer}}}"}]},{"vars":[{"varval":"Which of the following would be the best estimate for the capacity of a teaspoon?\n\n<br>\n\nsm_img https://teacher.smartermaths.com.au/wp-content/uploads/2023/07/spoon-1a-min.svg 150 indent2 vpad"},{"varval":"A teaspoon would have a capacity of approximately {{{correctAnswer}}}"}]},{"vars":[{"varval":"Which of the following would be the best estimate for the capacity of a bucket?\n\n<br>\n\nsm_img https://teacher.smartermaths.com.au/wp-content/uploads/2023/07/bucket-1-min.svg 180 indent vpad"},{"varval":"A bucket would have a capacity of approximately {{{correctAnswer}}}"}]}]

  43. <div class="sm_mode"> {{{question}}} </div>

    [{"vars":[{"varval":"Which 2 animals are about the same height?\n\nsm_img https://teacher.smartermaths.com.au/wp-content/uploads/2023/07/animals-4b_same-height_1-min.svg 350 indent vpad"},{"varval":"Animals {{{correctAnswer}}} are about the same height."}]},{"vars":[{"varval":"Which 2 children are about the same height?\n\nsm_img https://teacher.smartermaths.com.au/wp-content/uploads/2023/07/kids-4-height_same-min.svg 350 indent vpad"},{"varval":"Children {{{correctAnswer}}} are about the same height."}]},{"vars":[{"varval":"Which 2 cars are about the same length?\n\nsm_img https://teacher.smartermaths.com.au/wp-content/uploads/2023/07/cars-4x4-same_length-min.svg 350 indent vpad"},{"varval":"Cars {{{correctAnswer}}} are about the same length."}]},{"vars":[{"varval":"Which 2 fish are about the same length?\n\nsm_img https://teacher.smartermaths.com.au/wp-content/uploads/2023/07/fish-same_length-min.svg 350 indent vpad"},{"varval":"Fish {{{correctAnswer}}} are about the same length."}]},{"vars":[{"varval":"Which 2 buildings are about the same height?\n\nsm_img https://teacher.smartermaths.com.au/wp-content/uploads/2023/07/buildings-4-same-height-min.svg 350 indent vpad"},{"varval":"Buildings {{{correctAnswer}}} are about the same height."}]},{"vars":[{"varval":"Which 2 pencils are about the same length?\n\n<br>\n\nsm_img https://teacher.smartermaths.com.au/wp-content/uploads/2023/07/pencils-x4-same-length-min.svg 250 indent vpad"},{"varval":"Pencils {{{correctAnswer}}} are about the same length."}]}]

  44. SJ Area

    <div class="sm_mode"> {{{question}}} </div>

    [{"vars":[{"varval":"Which figure has an area of 6 square units?"},{"varval":"Count the squares in each option\n\n<div class=\"aligned\">\n\n>>|||\n|-|-|\n|Option 1|= 7 square units| \n|Option 2|= 7 square units|\n|Option 3|= 6 square units|\n|Option 4|= 7 square units|\n\n</div>\n\n<br>\n\nTherefore, Option 3 has an area of 6 square units.\n\n{{{correctAnswer}}}"}]},{"vars":[{"varval":"Which figure has an area of 7 square units?"},{"varval":"Count the squares in each option\n\n<div class=\"aligned\">\n\n>>|||\n|-|-|\n|Option 1|= 6 square units| \n|Option 2|= 7 square units|\n|Option 3|= 8 square units|\n|Option 4|= 9 square units|\n\n</div>\n\n<br>\n\nTherefore, Option 2 has an area of 7 square units.\n\n{{{correctAnswer}}}"}]},{"vars":[{"varval":"Which figure has an area of 8 square units?"},{"varval":"Count the squares in each option\n\n<div class=\"aligned\">\n\n>>|||\n|-|-|\n|Option 1|= 8 square units| \n|Option 2|= 7 square units|\n|Option 3|= 9 square units|\n|Option 4|= 7 square units|\n\n</div>\n\n<br>\n\nTherefore, Option 1 has an area of 8 square units.\n\n{{{correctAnswer}}}"}]},{"vars":[{"varval":"Which figure has an area of 9 square units?"},{"varval":"Count the squares in each option\n\n<div class=\"aligned\">\n\n>>|||\n|-|-|\n|Option 1|= 8 square units| \n|Option 2|= 7 square units|\n|Option 3|= 10 square units|\n|Option 4|= 9 square units|\n\n</div>\n\n<br>\n\nTherefore, Option 4 has an area of 9 square units.\n\n{{{correctAnswer}}}"}]},{"vars":[{"varval":"Which figure has an area of 5 square units?"},{"varval":"Count the squares in each option\n\n<div class=\"aligned\">\n\n>>|||\n|-|-|\n|Option 1|= 6 square units| \n|Option 2|= 7 square units|\n|Option 3|= 6 square units|\n|Option 4|= 5 square units|\n\n</div>\n\n<br>\n\nTherefore, Option 4 has an area of 5 square units.\n\n{{{correctAnswer}}}"}]},{"vars":[{"varval":"Which figure has an area of 6 square units?"},{"varval":"Count the squares in each option\n\n<div class=\"aligned\">\n\n>>|||\n|-|-|\n|Option 1|= 5 square units| \n|Option 2|= 6 square units|\n|Option 3|= 7 square units|\n|Option 4|= 7 square units|\n\n</div>\n\n<br>\n\nTherefore, Option 2 has an area of 6 square units.\n\n{{{correctAnswer}}}"}]}]

  45. SJ Length

    <div class="sm_mode"> {{{question}}} </div>

    [{"vars":[{"varval":"What is the correct order from tallest to shortest for these trees?\n\nsm_img https://teacher.smartermaths.com.au/wp-content/uploads/2023/07/trees-1-Order-min.svg 250 indent vpad "},{"varval":"{{{correctAnswer}}}"}]},{"vars":[{"varval":"What is the correct order from tallest to shortest for these children?\n\nsm_img https://teacher.smartermaths.com.au/wp-content/uploads/2023/07/kids-4-ORDERED-min.svg 300 indent vpad "},{"varval":"{{{correctAnswer}}}"}]},{"vars":[{"varval":"What is the correct order from shortest to tallest for these yachts?\n\nsm_img https://teacher.smartermaths.com.au/wp-content/uploads/2023/07/boat-4x4-ordered-min.svg 350 indent vpad "},{"varval":"{{{correctAnswer}}}"}]},{"vars":[{"varval":"What is the correct order from tallest to shortest for these animals?\n\nsm_img https://teacher.smartermaths.com.au/wp-content/uploads/2023/07/animals-4x4-min.svg 350 indent vpad "},{"varval":"{{{correctAnswer}}}"}]},{"vars":[{"varval":"What is the correct order from shortest to tallest for these farm animals?\n\nsm_img https://teacher.smartermaths.com.au/wp-content/uploads/2023/07/animals-4b-order-min.svg 350 indent vpad "},{"varval":"{{{correctAnswer}}}"}]},{"vars":[{"varval":"What is the correct order from tallest to shortest for these horses?\n\nsm_img https://teacher.smartermaths.com.au/wp-content/uploads/2023/07/horse-4x4-min.svg 380 indent vpad"},{"varval":"{{{correctAnswer}}}"}]}]

  46. SJ Length

    <div class="sm_mode"> {{{question}}} </div>

    [{"vars":[{"varval":"Johnny ruled four lines in his science book.\n\nHow many lines are longer than line 2?\n\n<br>\n\nsm_img https://teacher.smartermaths.com.au/wp-content/uploads/2023/07/lines-4-min.svg 300 indent vpad\n"},{"varval":"{{{correctAnswer}}} lines are longer than line 2"}]},{"vars":[{"varval":"Messie ruled four lines in his geography book.\n\nHow many lines are longer than line 4?\n\n<br>\n\nsm_img https://teacher.smartermaths.com.au/wp-content/uploads/2023/07/lines-4-min.svg 300 indent vpad\n"},{"varval":"{{{correctAnswer}}} lines are longer than line 4"}]},{"vars":[{"varval":"Rebecca cut 4 pieces of ribbon for her textiles class.\n\nHow many pieces of ribbon are longer than ribbon 4?\n\n<br>\n\nsm_img https://teacher.smartermaths.com.au/wp-content/uploads/2023/07/ribbon-1x4-min.svg 300 indent vpad\n"},{"varval":"{{{correctAnswer}}} pieces of ribbon are longer than ribbon 4"}]},{"vars":[{"varval":"Jayce cut four pieces of ribbon.\n\nHow many pieces of ribbon are longer than ribbon 1?\n\n<br>\n\nsm_img https://teacher.smartermaths.com.au/wp-content/uploads/2023/07/ribbon-1x4-min.svg 300 indent vpad\n"},{"varval":"{{{correctAnswer}}} piece of ribbon is longer than ribbon 1"}]},{"vars":[{"varval":"George cut 4 pieces of timber in his technology class.\n\nHow many pieces of timber are longer than piece 4?\n\n<br>\n\nsm_img https://teacher.smartermaths.com.au/wp-content/uploads/2023/07/plank-1x4a-min.svg 500 indent vpad\n"},{"varval":"{{{correctAnswer}}} pieces of timber are longer than piece 4"}]},{"vars":[{"varval":"Mark cut 4 pieces of timber in his technology class.\n\nHow many pieces of timber are longer than piece 1?\n\n<br>\n\nsm_img https://teacher.smartermaths.com.au/wp-content/uploads/2023/07/plank-1x4a-min.svg 500 indent vpad"},{"varval":"{{{correctAnswer}}} piece of timber is longer than piece 1"}]}]

  47. SJ Length

    <div class="sm_mode"> {{{question}}} </div>

    [{"vars":[{"varval":"Percy ruled four lines in his book.\n\nWhich line is the longest?\n\nsm_img https://teacher.smartermaths.com.au/wp-content/uploads/2023/07/lines-4-min.svg 300 indent vpad\n"},{"varval":"{{{correctAnswer}}} is the longest."}]},{"vars":[{"varval":"Svetlana ruled four lines in her book.\n\nWhich line is the shortest?\n\nsm_img https://teacher.smartermaths.com.au/wp-content/uploads/2023/07/lines-4-min.svg 300 indent vpad\n"},{"varval":"{{{correctAnswer}}} is the shortest."}]},{"vars":[{"varval":"Dane ruled four lines in his book.\n\nWhich line is the longest?\n\nsm_img https://teacher.smartermaths.com.au/wp-content/uploads/2023/07/lines-4-2-min.svg 300 indent vpad\n\n\n"},{"varval":"{{{correctAnswer}}} is the longest."}]},{"vars":[{"varval":"Hercules ruled four lines in his book.\n\nWhich line is the shortest?\n\nsm_img https://teacher.smartermaths.com.au/wp-content/uploads/2023/07/lines-4-2-min.svg 300 indent vpad\n\n"},{"varval":"{{{correctAnswer}}} is the shortest."}]},{"vars":[{"varval":"Pia cut four lengths of ribbon.\n\nWhich ribbon is the longest?\n\n<br>\n\nsm_img https://teacher.smartermaths.com.au/wp-content/uploads/2023/07/ribbon-1x4-min.svg 300 indent vpad\n\n\n\n"},{"varval":"{{{correctAnswer}}} is the longest."}]},{"vars":[{"varval":"Madge cut four lengths of ribbon.\n\nWhich ribbon is the shortest?\n\n<br>\n\nsm_img https://teacher.smartermaths.com.au/wp-content/uploads/2023/07/ribbon-1x4-min.svg 300 indent vpad\n\n"},{"varval":"{{{correctAnswer}}} is the shortest."}]},{"vars":[{"varval":"Blanco cut four pieces of timber.\n\nWhich piece of timber is the longest?\n\n<br>\n\nsm_img https://teacher.smartermaths.com.au/wp-content/uploads/2023/07/plank-1x4a-min.svg 500 indent vpad\n"},{"varval":"{{{correctAnswer}}} is the longest."}]},{"vars":[{"varval":"Olive cut four pieces of timber.\n\nWhich piece of timber is the shortest?\n\n<br>\n\nsm_img https://teacher.smartermaths.com.au/wp-content/uploads/2023/07/plank-1x4a-min.svg 500 indent vpad\n"},{"varval":"{{{correctAnswer}}} is the shortest."}]}]

  48. SJ Time

    <div class="sm_mode"> {{{question}}} </div>

    [{"vars":[{"varval":"How long is a quarter of an hour?"},{"varval":"1 hour = 60 minutes\n\n<div class=\"aligned\">\n\nsm_nogap $\\therefore$ One quarter of an hour \n\n>>| |\n| --------------------- |\n| = 60 $\\div$ 4 |\n| = {{correctAnswer}} |\n\n</div>\n\n"}]},{"vars":[{"varval":"How long is half an hour?"},{"varval":"1 hour = 60 minutes\n\n<div class=\"aligned\">\n\nsm_nogap $\\therefore$ Half an hour \n\n>>| |\n| --------------------- |\n| = 60 $\\div$ 2 |\n| = {{correctAnswer}} |\n\n</div>"}]},{"vars":[{"varval":"How long is three quarters of an hour?"},{"varval":"1 hour = 60 minutes\n\n<div class=\"aligned\">\n\nsm_nogap $\\therefore$ One quarter of an hour \n\n>>| |\n| --------------------- |\n| = 60 $\\div$ 4 |\n| = 15 minutes |\n\n</div>\n<div class=\"aligned\">\n\nsm_nogap $\\therefore$ Three quarters of an hour \n\n>>| |\n| --------------------- |\n| = 15 $\\times$ 3 |\n| = {{correctAnswer}} |\n\n</div>"}]},{"vars":[{"varval":"How long is an hour and a half?"},{"varval":"1 hour = 60 minutes\n\n<div class=\"aligned\">\n\nsm_nogap $\\therefore$ Half an hour \n\n>>| |\n| --------------------- |\n| = 60 $\\div$ 2 |\n| = 3 minutes |\n\n</div>\n<div class=\"aligned\">\n\nsm_nogap $\\therefore$ An hour and a half \n\n>>| |\n| --------------------- |\n| = 60 + 30 |\n| = {{correctAnswer}} |\n\n</div>"}]},{"vars":[{"varval":"How long is an hour and a quarter?"},{"varval":"1 hour = 60 minutes\n\n<div class=\"aligned\">\n\nsm_nogap $\\therefore$ One quarter of an hour \n\n>>| |\n| --------------------- |\n| = 60 $\\div$ 4 |\n| = 15 minutes |\n\n</div>\n<div class=\"aligned\">\n\nsm_nogap $\\therefore$ An hour and a quarter\n\n>>| |\n| --------------------- |\n| = 60 + 15 |\n| = {{correctAnswer}} |\n\n</div>"}]},{"vars":[{"varval":"How long is an hour and three quarters?"},{"varval":"1 hour = 60 minutes\n\n<div class=\"aligned\">\n\nsm_nogap $\\therefore$ One quarter of an hour \n\n>>| |\n| --------------------- |\n| = 60 $\\div$ 4 |\n| = 15 minutes |\n\n</div>\n<div class=\"aligned\">\n\nsm_nogap $\\therefore$ Three quarters of an hour \n\n>>| |\n| --------------------- |\n| = 15 $\\times$ 3 |\n| = 45 |\n\n</div>\n<div class=\"aligned\">\n\nsm_nogap $\\therefore$ An hour and three quarters \n\n>>| |\n| --------------------- |\n| = 60 + 45 |\n| = {{{correctAnswer}}} |\n\n</div>"}]}]

  49. SJ Time

    <div class="sm_mode"> {{{question}}} </div>

    [{"vars":[{"varval":"What time is shown on this smartwatch?\n\nsm_img https://teacher.smartermaths.com.au/wp-content/uploads/2023/07/watch-145-min.svg 150 indent vpad"},{"varval":"60 minutes = 1 hour\n\n15 minutes = quarter of 1 hour\n\n45 minutes past = a quarter to the next hour\n\nTime is {{{correctAnswer}}}"}]},{"vars":[{"varval":"What time is shown on this clock?\n\nsm_img https://teacher.smartermaths.com.au/wp-content/uploads/2023/07/analog-1130-min.svg 150 indent vpad"},{"varval":"60 minutes = 1 hour\n\n30 minutes = half of 1 hour\n\nTime is {{{correctAnswer}}}"}]},{"vars":[{"varval":"What time is shown on this clock?\n\nsm_img https://teacher.smartermaths.com.au/wp-content/uploads/2023/07/analog-615-min.svg 150 indent vpad"},{"varval":"60 minutes = 1 hour\n\n15 minutes = quarter of 1 hour\n\nTime is {{{correctAnswer}}}"}]},{"vars":[{"varval":"What time is shown on this smartphone?\n\nsm_img https://teacher.smartermaths.com.au/wp-content/uploads/2023/07/android-1045a-min.svg 120 indent2 vpad"},{"varval":"60 minutes = 1 hour\n\n15 minutes = quarter of 1 hour\n\n45 minutes past = a quarter to the next hour\n\n{{{correctAnswer}}}"}]},{"vars":[{"varval":"What time is shown on this smartphone?\n\nsm_img https://teacher.smartermaths.com.au/wp-content/uploads/2023/07/smartphone-115-min.svg 120 indent2 vpad"},{"varval":"60 minutes = 1 hour\n\n15 minutes = quarter of 1 hour\n\n15 minutes past the hour\n\n$\\therefore$ {{{correctAnswer}}}"}]},{"vars":[{"varval":"What time is shown on this smartphone?\n\nsm_img https://teacher.smartermaths.com.au/wp-content/uploads/2023/07/iphone-1145-min.svg 120 indent2 vpad"},{"varval":"60 minutes = 1 hour\n\n15 minutes = quarter of 1 hour\n\n45 minutes past = a quarter to the next hour\n\n{{{correctAnswer}}}"}]}]

  50. SJ Time

    <div class="sm_mode"> {{{question}}} </div>

    [{"vars":[{"varval":"Cayden is catching the train at a quarter to 4.\n\nWhat time is this in digital time?"},{"varval":"60 minutes = 1 hour\n\n15 minutes = one quarter of an hour\n\nA quarter to 4 = 15 minutes before 4 o'clock = {{{correctAnswer}}}\n\n\n"}]},{"vars":[{"varval":"Binky's movie is starting at a quarter past 1.\n\nWhat time is this in digital time?"},{"varval":"60 minutes = 1 hour\n\n15 minutes = one quarter of an hour\n\nA quarter past 1 = 15 minutes after 1 o'clock = {{{correctAnswer}}}\n"}]},{"vars":[{"varval":"Blake starts his gym class at half past 5.\n\nWhat time is this in digital time?"},{"varval":"60 minutes = 1 hour\n\n30 minutes = one half of an hour\n\nHalf past 5 = 30 minutes after 5 o'clock = {{{correctAnswer}}}\n"}]},{"vars":[{"varval":"Mandy is catching a train at a quarter to 9.\n\nWhat time is this in digital time?"},{"varval":"60 minutes = 1 hour\n\n15 minutes = one quarter of an hour\n\nA quarter to 9 = 15 minutes before 9 o'clock = {{{correctAnswer}}}\n"}]},{"vars":[{"varval":"Stephen has a doctor's appointment at a quarter past 10.\n\nWhat time is this in digital time?"},{"varval":"60 minutes = 1 hour\n\n15 minutes = one quarter of an hour\n\nA quarter past 10 = 15 minutes after 10 o'clock = {{{correctAnswer}}}\n"}]},{"vars":[{"varval":"Aldo's pizza will be ready at half past 8.\n\nWhat time is this in digital time?"},{"varval":"60 minutes = 1 hour\n\n30 minutes = one half of an hour\n\nHalf past 8 = 30 minutes after 8 o'clock = {{{correctAnswer}}}\n"}]}]

  51. SJ from Measurement, RAPH14 Q19-21

    <div class="sm_mode"> {{{question}}} </div>

    [{"vars":[{"varval":"Which smartwatch shows a quarter past 4?"},{"varval":"60 minutes = 1 hour\n\n15 minutes = quarter of 1 hour\n\n{{{correctAnswer}}}"}]},{"vars":[{"varval":"Which smartwatch shows a quarter to 6?"},{"varval":"60 minutes = 1 hour\n\n15 minutes = quarter of 1 hour\n\n{{{correctAnswer}}}"}]},{"vars":[{"varval":"Which smartwatch shows a quarter past 8?"},{"varval":"60 minutes = 1 hour\n\n15 minutes = quarter of 1 hour\n\n{{{correctAnswer}}}"}]},{"vars":[{"varval":"Which smartwatch shows half past 9?"},{"varval":"60 minutes = 1 hour\n\n30 minutes = half of 1 hour\n\n{{{correctAnswer}}}"}]},{"vars":[{"varval":"Which smartwatch shows half past one?"},{"varval":"60 minutes = 1 hour\n\n30 minutes = half of 1 hour\n\n{{{correctAnswer}}}"}]}]

  52. SJ - Time -calendar

    <div class="sm_mode"> {{{question}}} </div>

    [{"vars":[{"varval":"The calendar below shows the month of June 2017.\n\nsm_img https://teacher.smartermaths.com.au/wp-content/uploads/2023/07/calendar-June-min.svg 250 indent2 vpad\n\nWhat is the date of the first Friday?"},{"varval":"{{{correctAnswer}}}"}]},{"vars":[{"varval":"The calendar below shows the month of September 2017.\n\nsm_img https://teacher.smartermaths.com.au/wp-content/uploads/2023/07/calendar-September-min.svg 250 indent2 vpad\n\nWhat is the date of the third Wednesday?"},{"varval":"{{{correctAnswer}}}"}]},{"vars":[{"varval":"The calendar below shows the month of May 2017.\n\nsm_img https://teacher.smartermaths.com.au/wp-content/uploads/2023/07/calendar-May-min.svg 250 indent2 vpad\n\nWhat is the date of the fourth Friday?"},{"varval":"{{{correctAnswer}}}"}]},{"vars":[{"varval":"The calendar below shows the month of April 2017.\n\n\nsm_img https://teacher.smartermaths.com.au/wp-content/uploads/2023/07/calendar-April-min.svg 250 indent2 vpad\n\nWhat is the date of the third Saturday?"},{"varval":"{{{correctAnswer}}}"}]},{"vars":[{"varval":"The calendar below shows the month of December 2017.\n\nsm_img https://teacher.smartermaths.com.au/wp-content/uploads/2023/07/calendar-December-min.svg 250 indent2 vpad\n\nWhat is the date of the fourth Monday?"},{"varval":"{{{correctAnswer}}}"}]},{"vars":[{"varval":"The calendar below shows the month of January 2017.\n\nsm_img https://teacher.smartermaths.com.au/wp-content/uploads/2023/07/calendar-January-min.svg 250 indent2 vpad\n\nWhat is the date of the second Thursday?\n\n"},{"varval":"{{{correctAnswer}}}"}]}]

  53. Units

    <div class="sm_mode"> {{{question}}} </div>

    [{"vars":[{"varval":"Which of the items below could be used to measure a 4 centimetre caterpillar?"},{"varval":"{{{correctAnswer}}}\n\nA ruler marked in centimetres."}]},{"vars":[{"varval":"Which of the following items would you use to measure 2 litres of water?"},{"varval":"{{{correctAnswer}}}\n\nA jug marked in litres."}]},{"vars":[{"varval":"Which of the following items could be used to calculate the time 4 hours from now?"},{"varval":"{{{correctAnswer}}}\n\nAn analog clock."}]},{"vars":[{"varval":"Which of the following could be used to calculate the outside temperature at lunch time?"},{"varval":"{{{correctAnswer}}}\n\nA thermometer."}]},{"vars":[{"varval":"Which of the following could be used to measure the speed you are travelling in your car?"},{"varval":"{{{correctAnswer}}}\n\nAn odometer."}]},{"vars":[{"varval":"Which of the following could be used to measure a kilogram of flour?"},{"varval":"{{{correctAnswer}}}\n\nA kitchen scale."}]}]

  54. Sj Measurement 248-333

    <div class="sm_mode"> {{{question}}} </div>

    [{"vars":[{"varval":"Which bottle holds the most milk?"},{"varval":"{{{correctAnswer}}}"}]},{"vars":[{"varval":"Which box has the greatest mass?"},{"varval":"{{{correctAnswer}}}"}]},{"vars":[{"varval":"Which beaker contains the least liquid?"},{"varval":"{{{correctAnswer}}}"}]},{"vars":[{"varval":"Which pumpkin has the least mass?"},{"varval":"{{{correctAnswer}}}"}]},{"vars":[{"varval":"Which apple has the greatest mass?"},{"varval":"{{{correctAnswer}}}"}]},{"vars":[{"varval":"Which kettlebell has the least mass?"},{"varval":"{{{correctAnswer}}}"}]},{"vars":[{"varval":"Which kettlebell has the greatest mass?"},{"varval":"{{{correctAnswer}}}"}]},{"vars":[{"varval":"Which apple has the smallest mass?"},{"varval":"{{{correctAnswer}}}"}]},{"vars":[{"varval":"Which pumpkin has the greatest mass?"},{"varval":"{{{correctAnswer}}}"}]},{"vars":[{"varval":"Which beaker has the most liquid?"},{"varval":"{{{correctAnswer}}}"}]},{"vars":[{"varval":"Which box has the smallest mass?"},{"varval":"{{{correctAnswer}}}"}]},{"vars":[{"varval":"Which bottle holds the least milk?"},{"varval":"{{{correctAnswer}}}"}]}]

  55. SJ Measurement

    <div class="sm_mode"> {{{question}}} </div>

    [{"vars":[{"varval":"Which tree is the tallest?\n\nsm_img https://teacher.smartermaths.com.au/wp-content/uploads/2023/07/trees-1-min.svg 280 indent vpad"},{"varval":"Tree {{{correctAnswer}}} is the tallest."}]},{"vars":[{"varval":"Which car is the shortest?\n\nsm_img https://teacher.smartermaths.com.au/wp-content/uploads/2023/07/cars-4x4-min.svg 180 indent vpad"},{"varval":"Car {{{correctAnswer}}} is the shortest."}]},{"vars":[{"varval":"Which seahorse is the longest?\n\nsm_img https://teacher.smartermaths.com.au/wp-content/uploads/2023/07/seahorse-1x4-min.svg 380 indent vpad"},{"varval":"Seahorse {{{correctAnswer}}} is the longest."}]},{"vars":[{"varval":"Which boat is the tallest?\n\nsm_img https://teacher.smartermaths.com.au/wp-content/uploads/2023/07/boat-4x4-1.svg 350 indent vpad"},{"varval":"Boat {{{correctAnswer}}} is the tallest."}]},{"vars":[{"varval":"Which animal is the largest?\n\nsm_img https://teacher.smartermaths.com.au/wp-content/uploads/2023/07/animals-4x4-min.svg 300 indent vpad"},{"varval":"Animal {{{correctAnswer}}} is the largest."}]},{"vars":[{"varval":"Which horse is the tallest?\n\nsm_img https://teacher.smartermaths.com.au/wp-content/uploads/2023/07/horse-4x4-min.svg 380 indent vpad"},{"varval":"Horse {{{correctAnswer}}} is the tallest."}]}]

  56. SJ Measurement

    <div class="sm_mode"> {{{question}}} </div>

    [{"vars":[{"varval":"Reba wants to measure the length of her loungeroom.\n\nWhich object below should she use?"},{"varval":"{{{correctAnswer}}}\n\nA tape measure\n"}]},{"vars":[{"varval":"Kyra wants to calculate the time 90 minutes from now.\n\nWhich of the objects below should she use?"},{"varval":"{{{correctAnswer}}}\n\nA clock"}]},{"vars":[{"varval":"Floyd has just finished running a marathon and wants to measure how much weight he has lost during his run.\n\nWhich of the items below should he use?"},{"varval":"{{{correctAnswer}}}\n\nScales"}]},{"vars":[{"varval":"Cyrus wants to estimate the outside temperature.\n\nWhich of the objects below should he use?"},{"varval":"{{{correctAnswer}}}\n\nA thermometer"}]}]

  57. SJ Multiply/Divide From Algebra, NAPX-G2-34 SA

    <div class="sm_mode"> {{{question}}} </div>

    [{"vars":[{"varval":"\n\nIn a card game, John gets 5 cards and earns points.\n\n<br>\n\nsm_img https://teacher.smartermaths.com.au/wp-content/uploads/2023/07/stars-35.svg 350 indent vpad\n\n<br>Each white star is worth double the points of each black star.\n\nHow many points is a white star worth?\n\n"},{"varval":"\nJohn's hand is worth the same as having\n 7 black stars.\n\n<div class=\"aligned\">\n\n<div class=\"sm_img_inline\" style=\"height: 20px\">\n\n>| | |\n| ------------: | ---------- |\n| 7 $\\times$ <img src=\"https://teacher.smartermaths.com.au/wp-content/uploads/2023/07/stars-black.svg\" style=\"height: 40px;\"> | \\= 35 |\n| <img src=\"https://teacher.smartermaths.com.au/wp-content/uploads/2023/07/stars-black.svg\" style=\"height: 40px;\"> | \\= 35 $\\div$ 7 = 5 |\n\n</div>\n\n</div>\n\n<br>A white star is worth 2 $\\times$ black\n\n<div class=\"aligned\">\n\n\n>| | |\n| ------------: | ---------- |\n| <img src=\"https://teacher.smartermaths.com.au/wp-content/uploads/2023/07/stars-white.svg\" style=\"height: 40px;\"> | \\= 2 $\\times$ 5 |\n| | \\= {{{correctAnswer0}}} |\n\n</div>\n\n"}]},{"vars":[{"varval":"\nIn a card game, Francine gets 5 cards and earns points.\n\n<br>\n\nsm_img https://teacher.smartermaths.com.au/wp-content/uploads/2023/07/stars-90.svg 350 indent vpad\n\n<br>Each white star is worth double the points of each black star.\n\nHow many points is a white star worth?"},{"varval":"Francine's hand is worth the same as having 6 black stars.\n\n<div class=\"aligned\">\n\n<div class=\"sm_img_inline\" style=\"height: 20px\">\n\n>| | |\n| ------------: | ---------- |\n| 6 $\\times$ <img src=\"https://teacher.smartermaths.com.au/wp-content/uploads/2023/07/stars-black.svg\" style=\"height: 40px;\"> | \\= 90 |\n| <img src=\"https://teacher.smartermaths.com.au/wp-content/uploads/2023/07/stars-black.svg\" style=\"height: 40px;\"> | \\= 90 $\\div$ 6 = 15 |\n\n</div>\n\n</div>\n\n<br>A white star is worth 2 $\\times$ black\n\n<div class=\"aligned\">\n\n\n>| | |\n| ------------: | ---------- |\n| <img src=\"https://teacher.smartermaths.com.au/wp-content/uploads/2023/07/stars-white.svg\" style=\"height: 40px;\"> | \\= 2 $\\times$ 15 |\n| | \\= {{{correctAnswer0}}} |\n\n</div>"}]},{"vars":[{"varval":"In a card game, Lucy gets 6 cards and earns points.\n\n<br>\n\nsm_img https://teacher.smartermaths.com.au/wp-content/uploads/2023/07/stars-96a.svg 350 indent vpad\n\n<br>Each white star is worth double the points of each black star.\n\nHow many points is a white star worth?"},{"varval":"Lucy's hand is worth the same as having 8 black stars.\n\n<div class=\"aligned\">\n\n<div class=\"sm_img_inline\" style=\"height: 20px\">\n\n>| | |\n| ------------: | ---------- |\n| 8 $\\times$ <img src=\"https://teacher.smartermaths.com.au/wp-content/uploads/2023/07/stars-black.svg\" style=\"height: 40px;\"> | \\= 96 |\n| <img src=\"https://teacher.smartermaths.com.au/wp-content/uploads/2023/07/stars-black.svg\" style=\"height: 40px;\"> | \\= 96 $\\div$ 8 = 12 |\n\n</div>\n\n</div>\n\n<br>A white star is worth 2 $\\times$ black\n\n<div class=\"aligned\">\n\n\n>| | |\n| ------------: | ---------- |\n| <img src=\"https://teacher.smartermaths.com.au/wp-content/uploads/2023/07/stars-white.svg\" style=\"height: 40px;\"> | \\= 2 $\\times$ 12 |\n| | \\= {{{correctAnswer0}}} |\n\n</div>"}]},{"vars":[{"varval":"In a card game, Grant gets 6 cards and earns points.\n\n<br>\n\nsm_img https://teacher.smartermaths.com.au/wp-content/uploads/2023/07/stars-63.svg 350 indent vpad\n\n<br>Each white star is worth double the points of each black star.\n\nHow many points is a white star worth?"},{"varval":"Grant's hand is worth the same as having 9 black stars.\n\n<div class=\"aligned\">\n\n<div class=\"sm_img_inline\" style=\"height: 20px\">\n\n>| | |\n| ------------: | ---------- |\n| 9 $\\times$ <img src=\"https://teacher.smartermaths.com.au/wp-content/uploads/2023/07/stars-black.svg\" style=\"height: 40px;\"> | \\= 63 |\n| <img src=\"https://teacher.smartermaths.com.au/wp-content/uploads/2023/07/stars-black.svg\" style=\"height: 40px;\"> | \\= 63 $\\div$ 9 = 7 |\n\n</div>\n\n</div>\n\n<br>A white star is worth 2 $\\times$ black\n\n<div class=\"aligned\">\n\n\n>| | |\n| ------------: | ---------- |\n| <img src=\"https://teacher.smartermaths.com.au/wp-content/uploads/2023/07/stars-white.svg\" style=\"height: 40px;\"> | \\= 2 $\\times$ 7 |\n| | \\= {{{correctAnswer0}}} |\n\n</div>"}]},{"vars":[{"varval":"In a card game, Gavin gets 7 cards and earns points.\n\n<br>\n\nsm_img https://teacher.smartermaths.com.au/wp-content/uploads/2023/07/stars-22.svg 380 indent vpad\n\n<br>Each white star is worth double the points of each black star.\n\nHow many points is a white star worth?"},{"varval":"Gavin's hand is worth the same as having 11 black stars.\n\n<div class=\"aligned\">\n\n<div class=\"sm_img_inline\" style=\"height: 20px\">\n\n>| | |\n| ------------: | ---------- |\n| 11 $\\times$ <img src=\"https://teacher.smartermaths.com.au/wp-content/uploads/2023/07/stars-black.svg\" style=\"height: 40px;\"> | \\= 22 |\n| <img src=\"https://teacher.smartermaths.com.au/wp-content/uploads/2023/07/stars-black.svg\" style=\"height: 40px;\"> | \\= 22 $\\div$ 11 = 2 |\n\n</div>\n\n</div>\n\n<br>A white star is worth 2 $\\times$ black\n\n<div class=\"aligned\">\n\n\n>| | |\n| ------------: | ---------- |\n| <img src=\"https://teacher.smartermaths.com.au/wp-content/uploads/2023/07/stars-white.svg\" style=\"height: 40px;\"> | \\= 2 $\\times$ 2 |\n| | \\= {{{correctAnswer0}}} |\n\n</div>"}]},{"vars":[{"varval":"In a card game, Trinity gets 7 cards and earns points.\n\n<br>\n\nsm_img https://teacher.smartermaths.com.au/wp-content/uploads/2023/07/stars-20.svg 380 indent vpad\n\n<br>Each white star is worth double the points of each black star.\n\nHow many points is a white star worth?"},{"varval":"Trinity's hand is worth the same as having 10 black stars.\n\n<div class=\"aligned\">\n\n<div class=\"sm_img_inline\" style=\"height: 20px\">\n\n>| | |\n| ------------: | ---------- |\n| 10 $\\times$ <img src=\"https://teacher.smartermaths.com.au/wp-content/uploads/2023/07/stars-black.svg\" style=\"height: 40px;\"> | \\= 20 |\n| <img src=\"https://teacher.smartermaths.com.au/wp-content/uploads/2023/07/stars-black.svg\" style=\"height: 40px;\"> | \\= 20 $\\div$ 10 = 2 |\n\n</div>\n\n</div>\n\n<br>A white star is worth 2 $\\times$ black\n\n<div class=\"aligned\">\n\n\n>| | |\n| ------------: | ---------- |\n| <img src=\"https://teacher.smartermaths.com.au/wp-content/uploads/2023/07/stars-white.svg\" style=\"height: 40px;\"> | \\= 2 $\\times$ 2 |\n| | \\= {{{correctAnswer0}}} |\n\n</div>"}]}]

  58. SJ Multiply/divide Algebra, NAPX-J2-28

    <div class="sm_mode"> {{{question}}} </div>

    [{"vars":[{"varval":"Hughes has 71 match sticks.\n\nHe uses 6 match sticks to make two small triangles.\n\n<br>\n\nsm_img https://teacher.smartermaths.com.au/wp-content/uploads/2018/12/NAP-J2-28-v1.svg 300 indent vpad\n\n<br>What is the largest number of small triangles, like this, that Hughes can make with his 71 match sticks?\n"},{"varval":"\n3 match sticks are used for 1 triangle.\n\n<div class=\"aligned\">\n\n| | |\n| ------------- | ---------- |\n| Number of triangles\t | \\= 71 $\\div$ 3 |\n| | \\= {{{correctAnswer}}} remainder 2 |\n\n</div>\n\n<br>\n\n\n$\\therefore$ {{{correctAnswer}}} triangles (round down).\n4 match sticks are used for 1 square.\n"}]},{"vars":[{"varval":"Anita has 57 match sticks.\n\nShe uses 8 match sticks to make two small squares.\n\n<br>\n\nsm_img https://teacher.smartermaths.com.au/wp-content/uploads/2023/07/match-2x2-min.svg 300 indent vpad\n\n<br>What is the largest number of small squares, like this, that Anita can make with her 57 match sticks?\n"},{"varval":"<div class=\"aligned\">\n\n| | |\n| ------------- | ---------- |\n| Number of squares\t | \\= 57 $\\div$ 4 |\n| | \\= {{{correctAnswer}}} remainder 1 |\n\n</div>\n\n<br>\n\n\n$\\therefore$ {{{correctAnswer}}} squares (round down)."}]},{"vars":[{"varval":"Brutus has 81 match sticks.\n\nHe uses 12 match sticks to make two small fish shapes.\n\n<br>\n\nsm_img https://teacher.smartermaths.com.au/wp-content/uploads/2023/07/fish-2x2-min.svg 180 indent2 vpad\n\n<br>What is the largest number of small fish shapes, like this, that Brutus can make with his 81 match sticks?\n\n\n"},{"varval":"<div class=\"aligned\">\n\n| | |\n| ------------- | ---------- |\n| Number of small fish\t | \\= 81 $\\div$ 6 |\n| | \\= {{{correctAnswer}}} remainder 3 |\n\n</div>\n\n<br>\n\n\n$\\therefore$ {{{correctAnswer}}} small fish (round down)."}]},{"vars":[{"varval":"Burgess has 94 match sticks.\n\nHe uses 16 match sticks to make these two daisy shapes.\n\n<br>\n\nsm_img https://teacher.smartermaths.com.au/wp-content/uploads/2023/07/match-2x8-min.svg 300 indent vpad\n\n<br>What is the largest number of daisy shapes, like this, that Burgess can make with his 94 match sticks?\n"},{"varval":"<div class=\"aligned\">\n\n| | |\n| ------------- | ---------- |\n| Number of daisies | \\= 94 $\\div$ 8 |\n| | \\= {{{correctAnswer}}} remainder 6 |\n\n</div>\n\n<br>\n\n\n$\\therefore$ {{{correctAnswer}}} daisies (round down)."}]},{"vars":[{"varval":"Bianca has 75 match sticks.\n\nShe uses 8 match sticks to make these two crosses.\n\n<br>\n\nsm_img https://teacher.smartermaths.com.au/wp-content/uploads/2023/07/match-2x4-cross-min.svg 300 indent vpad\n\n<br>What is the largest number of crosses, like this, that Bianca can make with her 75 match sticks?\n"},{"varval":"<div class=\"aligned\">\n\n| | |\n| ------------- | ---------- |\n| Number of crosses| \\= 75 $\\div$ 4 |\n| | \\= {{{correctAnswer}}} remainder 3 |\n\n</div>\n\n<br>\n\n\n$\\therefore$ {{{correctAnswer}}} crosses (round down)."}]},{"vars":[{"varval":"Beau has 85 match sticks.\n\nHe uses 14 match sticks to make these two shapes.\n\n<br>\n\nsm_img https://teacher.smartermaths.com.au/wp-content/uploads/2023/07/match-2x7-min.svg 300 indent vpad\n\n<br>What is the largest number of shapes, like this, that Beau can make with his 85 match sticks?\n"},{"varval":"<div class=\"aligned\">\n\n| | |\n| ------------- | ---------- |\n| Number of shapes\t | \\= 85 $\\div$ 7 |\n| | \\= {{{correctAnswer}}} remainder 1 |\n\n</div>\n\n<br>\n\n\n$\\therefore$ {{{correctAnswer}}} shapes (round down)."}]}]

  59. SJ Add/Subtract

    <div class="sm_mode"> {{{question}}} </div>

    [{"vars":[{"varval":"Ginger measured the height of her drink bottle by measuring it against her ruler.\n\nsm_img https://teacher.smartermaths.com.au/wp-content/uploads/2023/07/water-ruler-2-min.svg 200 indent3 vpad\n\nWhat is the height of her drink bottle in centimetres?"},{"varval":"<div class=\"aligned\">\n\n| | |\n| ------------- | ---------- |\n| Height | \\= 28 − 4 |\n| | \\= {{{correctAnswer}}} centimetres |\n\n</div>\n"}]},{"vars":[{"varval":"George found a butterfly and measured the width of its wingspan using part of his ruler.\n\nsm_img https://teacher.smartermaths.com.au/wp-content/uploads/2023/07/butterfly-ruler-1-min.svg 350 indent3 vpad\n\nWhat is the width of the butterfly's wingspan, in centimetres?"},{"varval":"<div class=\"aligned\">\n\n| | |\n| ------------- | ---------- |\n| Width | \\= 19 − 13 |\n| | \\= {{{correctAnswer}}} centimetres |\n\n</div>\n"}]},{"vars":[{"varval":"Min measured the length of her diary by measuring it against part of her ruler.\n\nsm_img https://teacher.smartermaths.com.au/wp-content/uploads/2023/07/ruler-diary-min.svg 500 indent vpad\n\nWhat is the length of her diary, in centimetres?"},{"varval":"<div class=\"aligned\">\n\n| | |\n| ------------- | ---------- |\n| Length | \\= 43 − 24 |\n| | \\= {{{correctAnswer}}} centimetres |\n\n</div>\n"}]},{"vars":[{"varval":"Virgil measured the length of his pencil using part of his ruler.\n\nsm_img https://teacher.smartermaths.com.au/wp-content/uploads/2023/07/ruler-pencil-min.svg 550 vpad\n\nWhat is the length of his pencil, in centimetres?"},{"varval":"<div class=\"aligned\">\n\n| | |\n| ------------- | ---------- |\n| Height | \\= 45 − 27 |\n| | \\= {{{correctAnswer}}} centimetres |\n\n</div>\n"}]},{"vars":[{"varval":"Milo measured the length of the fish he caught, measuring against part of his ruler.\n\nsm_img https://teacher.smartermaths.com.au/wp-content/uploads/2023/07/ruler-fish-min.svg 500 indent vpad\n\nWhat is the length of the fish, in centimetres?"},{"varval":"<div class=\"aligned\">\n\n| | |\n| ------------- | ---------- |\n| Height | \\= 49 − 22 |\n| | \\= {{{correctAnswer}}} centimetres |\n\n</div>\n"}]},{"vars":[{"varval":"Iris measured the length of a dart using part of her ruler.\n\nsm_img https://teacher.smartermaths.com.au/wp-content/uploads/2023/07/ruler-dart-min.svg 450 indent2 vpad\n\nWhat is the length of the dart, to the nearest centimetre?"},{"varval":"<div class=\"aligned\">\n\n| | |\n| ------------- | ---------- |\n| Height | \\= 41 − 26 |\n| | \\= {{{correctAnswer}}} centimetres |\n\n</div>\n"}]}]

  60. SJ Add/subtract

    <div class="sm_mode"> {{{question}}} </div>

    [{"vars":[{"varval":"Bev has $66 saved and Mickey has $24 saved.\n\nHow much money should Bev give to Mickey for them each to have the same amount of money?"},{"varval":"<div class=\"aligned\">\n\n|||\n|-|-|\n|Difference|= \\$66 $−$ \\$24|\n||= \\$46 $-$ \\$4|\n||= $42|\n\n</div>\n\n<br>Bev needs to give half of the $42 to Mickey.\n\nHalf of $42 is ${{{correctAnswer0}}}.\n\nBev needs to give Mickey ${{{correctAnswer0}}} of her money.\n\n(Check: \\$66 $-$ \\${{{correctAnswer0}}} = $45 and $24 + ${{{correctAnswer0}}} = $45)\n"}]},{"vars":[{"varval":"Bing has saved $75 and Dean has saved $41 .\n\nHow much money should Bing give to Dean for them to each have the same amount of money?"},{"varval":"<div class=\"aligned\">\n\n|||\n|-|-|\n|Difference|= \\$75 $−$ \\$41|\n||= \\$35 $-$ \\$1|\n||= $34|\n\n</div>\n\n<br>Bing needs to give half of the $34 to Dean.\n\nHalf of $34 is ${{{correctAnswer0}}}.\n\nBing needs to give Dean ${{{correctAnswer0}}} of his money.\n\n(Check: \\$75 $-$ \\${{{correctAnswer0}}} = $58 and $41 + ${{{correctAnswer0}}} = $58)"}]},{"vars":[{"varval":"Edgar has saved $97 and Allen has saved $45.\n\nHow much money should Edgar give to Allen for them to each have the same amount of money?"},{"varval":"<div class=\"aligned\">\n\n|||\n|-|-|\n|Difference|= \\$97 $−$ \\$45|\n||= \\$57 $-$ \\$5|\n||= $52|\n\n</div>\n\n<br>Edgar needs to give half of the $52 to Allen.\n\nHalf of $52 is ${{{correctAnswer0}}}.\n\nEdgar needs to give Allen ${{{correctAnswer0}}} of his money.\n\n(Check: \\$97 $-$ \\${{{correctAnswer0}}} = $71 and $45 + ${{{correctAnswer0}}} = $71)"}]},{"vars":[{"varval":"Arjun has saved $168 and Rishi has saved $92.\n\nHow much money should Arjun give to Rishi for them to each have the same amount of money?"},{"varval":"<div class=\"aligned\">\n\n|||\n|-|-|\n|Difference|= \\$168 $−$ \\$92|\n||= \\$78 $-$ \\$2|\n||= $76|\n\n</div>\n\n<br>Arjun needs to give half of the $76 to Rishi.\n\nHalf of $76 is ${{{correctAnswer0}}}.\n\nArjun needs to give Rishi ${{{correctAnswer0}}} of his money.\n\n(Check: \\$168 $-$ \\${{{correctAnswer0}}} = $130 and $92 + ${{{correctAnswer0}}} = $130)"}]},{"vars":[{"varval":"Mei has saved $225 and Xiang has saved $119.\n\nHow much money should Mei give to Xiang for them each to have the same amount of money?"},{"varval":"<div class=\"aligned\">\n\n|||\n|-|-|\n|Difference|= \\$225 $−$ \\$119|\n||= \\$115 $-$ \\$9|\n||= $106|\n\n</div>\n\n<br>Mei needs to give half of the $106 to Xiang.\n\nHalf of $106 is ${{{correctAnswer0}}}.\n\nMei needs to give Xiang ${{{correctAnswer0}}} of her money.\n\n(Check: \\$225 $-$ \\${{{correctAnswer0}}} = $172 and $119 + ${{{correctAnswer0}}} = $172)"}]},{"vars":[{"varval":"Niamh has saved $454 and Enya has saved $342.\n\nHow much money should Niamh give to Enya for them each to have the same amount of money?"},{"varval":"<div class=\"aligned\">\n\n|||\n|-|-|\n|Difference|= \\$454 $−$ \\$342|\n||= \\$154 $-$ \\$42|\n||= \\$114 $-$ \\$2|\n||= $112|\n\n</div>\n\n<br>Niamh needs to give half of the $112 to Enya.\n\nHalf of $112 is ${{{correctAnswer0}}}.\n\nNiamh needs to give Enya ${{{correctAnswer0}}} of her money.\n\n(Check: \\$454 $-$ \\${{{correctAnswer0}}} = $398 and $342 + ${{{correctAnswer0}}} = $398)"}]}]

  61. SJ Add/subtract

    <div class="sm_mode"> {{{question}}} </div>

    [{"vars":[{"varval":"A tourist travelled 2248 kilometres on the first week of their holiday.\n\nThey travelled another 1872 kilometres on the second week of their holiday.\n\nWhich of these could be used to calculate the total number of kilometres travelled by the tourist on their two week holiday?\n"},{"varval":"<div class=\"aligned\">\n\n| | |\n| ------------- | ---------- |\n| 2248 + 872 | \\= 2000 + 200 + 40 + 8 + 800 + 70 + 2 |\n| | \\= 2000 + 200 + 800 + 40 + 70 + 8 + 2 |\n| | \\= {{{correctAnswer}}}|\n\n</div>"}]},{"vars":[{"varval":"Tae travelled 1339 kilometres Adelaide to Sydney.\n\nHe then travelled another 981 kilometres from Sydney to Brisbane.\n\nWhich of these could be used to calculate the total number of kilometres Tae travelled from Adelaide to Brisbane?\n"},{"varval":"<div class=\"aligned\">\n\n| | |\n| ------------- | ---------- |\n| 1339 + 921 | \\= 1000 + 300 + 30 + 9 + 900 + 80 + 1 |\n| | \\= 1000 + 300 + 900 + 30 + 80 + 9 + 1 |\n| | \\= {{{correctAnswer}}}|\n\n</div>"}]},{"vars":[{"varval":"A truck weighs 7709 kilograms with no load.\n\nA load of 971 kilograms is added to the truck.\n\nWhich of these could be used to calculate the total weight of the loaded truck?\n"},{"varval":"<div class=\"aligned\">\n\n| | |\n| ------------- | ---------- |\n| 7709 + 971 | \\= 7000 + 700 + 9 + 900 + 70 + 1 |\n| | \\= 7000 + 700 + 900 + 70 + 9 + 1 |\n| | \\= {{{correctAnswer}}}|\n\n</div>"}]},{"vars":[{"varval":"Aziz travelled 1093 kilometres on the first week of his holiday.\n\nHe then travelled another 487 kilometres on the second week of his holiday.\n\nWhich of these could be used to calculate the total number of kilometres travelled by the Aziz on his holiday?\n"},{"varval":"<div class=\"aligned\">\n\n| | |\n| ------------- | ---------- |\n| 1093 + 487 | \\= 1000 + 90 + 3 + 400 + 80 + 7 |\n| | \\= 1000 + 400 + 90 + 80 + 3 + 7 |\n| | \\= {{{correctAnswer}}}|\n\n</div>"}]},{"vars":[{"varval":"Ali cycled 1754 kilometres in total from January to June.\n\nHe then cycled another 496 kilometres from July to December.\n\nWhich of these could be used to calculate the number of kilometres cycled by Ali from January to December?\n"},{"varval":"<div class=\"aligned\">\n\n| | |\n| ------------- | ---------- |\n| 1754 + 496 | \\= 1000 + 700 + 50 + 4 + 400 + 90 + 6 |\n| | \\= 1000 + 700 + 400 + 50 + 90 + 4 + 6 |\n| | \\= {{{correctAnswer}}}|\n\n</div>"}]},{"vars":[{"varval":"Blanch ran a total of 1396 kilometres in 2021.\n\nShe then ran another 784 kilometres in 2022.\n\nWhich of these could be used to calculate the total number of kilometres run by Blanch during 2021 and 2022?\n"},{"varval":"<div class=\"aligned\">\n\n| | |\n| ------------- | ---------- |\n| 1396 + 784 | \\= 1000 + 300 + 90 + 6 + 700 + 80 + 4 |\n| | \\= 1000 + 300 + 700 + 90 + 80 + 6 + 4 |\n| | \\= {{{correctAnswer}}}|\n\n</div>"}]}]

  62. SJ Add subtract

    <div class="sm_mode"> {{{question}}} </div>

    [{"vars":[{"varval":"<div class=\"sm_mode\">\n\nA company has 345 green highlighters in a warehouse.\n\nAnother 485 highlighters are delivered to the warehouse.\n\nWhich of these could be used to calculate the correct number of highlighters in the warehouse?\n\n</div>\n"},{"varval":"<div class=\"aligned\">\n\n| | |\n| ------------- | ---------- |\n| 345 + 485 | \\= 300 + 40 + 5 + 400 + 80 + 5 |\n| | \\= 300 + 400 + 40 + 80 + 5 + 5 |\n| | \\= {{{correctAnswer}}}|\n\n</div>\n"}]},{"vars":[{"varval":"<div class=\"sm_mode\">\n\nA farm has 233 dairy cows in a paddock.\n\nAnother 187 dairy cows are in the milking sheds.\n\nWhich of these could be used to calculate the number of dairy cows, in total, on the farm?\n\n</div>\n"},{"varval":"<div class=\"aligned\">\n\n| | |\n| ------------- | ---------- |\n| 233 + 187 | \\= 200 + 30 + 3 + 100 + 80 + 7 |\n| | \\= 200 + 100 + 80 + 30 + 3 + 7 |\n| | \\= {{{correctAnswer}}}|\n\n</div>\n"}]},{"vars":[{"varval":"<div class=\"sm_mode\">\n\nA school has 395 students who walk to school.\n\nAnother 685 students travel to school by bus.\n\nWhich of these could be used to calculate the total number of students who either walk to school or travel by bus?\n\n</div>\n"},{"varval":"\n<div class=\"aligned\">\n\n| | |\n| ------------- | ---------- |\n| 395 + 685 | \\= 300 + 90 + 5 + 600 + 80 + 5 |\n| | \\= 300 + 600 + 90 + 80 + 5 + 5 |\n| | \\= {{{correctAnswer}}}|\n\n</div>\n"}]},{"vars":[{"varval":"<div class=\"sm_mode\">\n\nOn Monday a gym had 195 members who trained in the morning.\n\nAnother 275 members trained in the afternoon and evening.\n\nWhich of these could be used to calculate the total number of members training at the gym on Monday?\n\n</div>\n"},{"varval":"<div class=\"aligned\">\n\n| | |\n| ------------- | ---------- |\n| 195 + 275 | \\= 100 + 90 + 5 + 200 + 70 + 5 |\n| | \\= 100 + 200 + 90 + 70 + 5 + 5 |\n| | \\= {{{correctAnswer}}}|\n\n</div>\n"}]},{"vars":[{"varval":"<div class=\"sm_mode\">\n\nA farmer collects 625 eggs on Monday.\n\nAnother 785 eggs were collected on Tuesday.\n\nWhich of these could be used to calculate the total number of eggs collected on Monday and Tuesday?\n\n</div>\n"},{"varval":"<div class=\"aligned\">\n\n| | |\n| ------------- | ---------- |\n| 625 + 785 | \\= 600 + 20 + 5 + 700 + 80 + 5 |\n| | \\= 600 + 700 + 20 + 80 + 5 + 5 |\n| | \\= {{{correctAnswer}}}|\n\n</div>\n"}]},{"vars":[{"varval":"<div class=\"sm_mode\">\n\nA plant nursery has 545 indoor plants.\n\nThe nursery has another 895 plants that are outdoors.\n\nWhich of these could be used to calculate the total number of indoor and outdoor plants in the nursery?\n\n</div>\n"},{"varval":"<div class=\"aligned\">\n\n| | |\n| ------------- | ---------- |\n| 545 + 895 | \\= 500 + 40 + 5 + 800 + 90 + 5 |\n| | \\= 500 + 800 + 40 + 90 + 5 + 5 |\n| | \\= {{{correctAnswer}}}|\n\n</div>\n"}]}]

  63. SJ Add/subtract

    <div class="sm_mode"> {{{question}}} </div>

    [{"vars":[{"varval":"The table shows the weights of 5 friends.\n\n<div class=\"sm-table col1-color1 row1-color1 top-left-cell-hidden\">\n\n>| | Annie| Brian| Hamza| Vivian|Cade|\n|:-:|:-:|:-:|:-:|:-:|:-:|\n|Weight (kilograms)| 50| 57|54|55|61|\n\n</div>\n\n<br>\nWho is the second lightest friend?\n"},{"varval":"\nOrdered from lightest to heaviest:\n\n>Annie (50)\n\n>Hamza (54)\n\n>Vivian (55)\n\n>Brian (57)\n\n>Cade (61)\n\n<br>\n\n$\\therefore$ {{{correctAnswer}}} is the second lightest.\n\n"}]},{"vars":[{"varval":"The table shows the lengths of 4 samples of wood.\n\n<div class=\"sm-table col1-color2 row1-color2 top-left-cell-hidden\">\n\n>| | Sample 1| Sample 2| Sample 3| Sample 4|\n|:-:|:-:|:-:|:-:|:-:|\n| Length (centimetres)| 113| 104|118|109|\n\n</div>\n\n<br>\nWhich wood sample is the second longest?\n"},{"varval":"Ordered from shortest to longest:\n\n>Sample 2 (104)\n\n>Sample 4 (109)\n\n>Sample 1 (113)\n\n>Sample 3 (118)\n\n<br>\n\n$\\therefore$ {{{correctAnswer}}} is the second longest.\n"}]},{"vars":[{"varval":"The table shows the times of 4 runners in a 200 metre race.\n\n<div class=\"sm-table col1-color3 row1-color3 top-left-cell-hidden\">\n\n>| | Runner 1| Runner 2| Runner 3| Runner 4|\n|:-:|:-:|:-:|:-:|:-:|\n| Time (seconds)| 23| 27|24|26|\n\n</div>\n\n<br>\nWhich runner is the second slowest?\n"},{"varval":"Ordered from fastest to slowest:\n\n>Runner 1 (23)\n\n>Runner 3 (24)\n\n>Runner 4 (26)\n\n>Runner 2 (27)\n\n<br>\n\n$\\therefore$ {{{correctAnswer}}} is the second slowest."}]},{"vars":[{"varval":"The table shows the distances of 4 mountain hikes.\n\n<div class=\"sm-table col1-color4 row1-color4 top-left-cell-hidden\">\n\n>| | Hike 1| Hike 2| Hike 3| Hike 4|\n|:-:|:-:|:-:|:-:|:-:|\n| Distance (kilometres)| 43| 28|37|35|\n\n</div>\n\n<br>\nWhich hike is the second longest?\n\n"},{"varval":"Ordered from shortest to longest:\n\n>Hike 2 (28)\n\n>Hike 4 (35)\n\n>Hike 3 (37)\n\n>Hike 1 (43)\n\n<br>\n\n$\\therefore$ {{{correctAnswer}}} is the second longest."}]},{"vars":[{"varval":"The table shows the running times of 4 movies.\n\n<div class=\"sm-table col1-color5 row1-color5 top-left-cell-hidden\">\n\n>| | Movie 1| Movie 2| Movie 3| Movie 4|\n|:-:|:-:|:-:|:-:|:-:|\n| Time (minutes)| 127|125|119|126|\n\n</div>\n\n<br>\nWhich movie has the second shortest running time?\n"},{"varval":"Ordered from shortest to longest:\n\n>Movie 3 (119)\n\n>Movie 2 (125)\n\n>Movie 4 (126)\n\n>Movie 1 (127)\n\n<br>\n\n$\\therefore$ {{{correctAnswer}}} is the second shortest."}]},{"vars":[{"varval":"The table shows the size of 4 blocks of land in square metres.\n\n<div class=\"sm-table col1-color6 row1-color6 top-left-cell-hidden\">\n\n>| | Block 1| Block 2| Block 3| Block 4|\n|:-:|:-:|:-:|:-:|:-:|\n| Size (sq metres)| 867| 1004|790|640|\n\n</div>\n\n<br>\nWhich block size is the second largest?\n"},{"varval":"Ordered from smallest to largest:\n\n>Block 4 (640)\n\n>Block 3 (790)\n\n>Block 1 (867)\n\n>Block 2 (1004)\n\n<br>\n\n$\\therefore$ {{{correctAnswer}}} is the second largest block."}]}]

  64. SJ Add/subtract From Algebra, NAPX-p124265v01

    <div class="sm_mode"> {{{question}}} </div>

    [{"vars":[{"varval":"Gina has saved $92.\n\n<br>\n\nsm_img https://teacher.smartermaths.com.au/wp-content/uploads/2019/01/NAPX-G2-13v2.svg 170 indent3 vpad\n\n<br>How much more money does she need to buy the tennis racquet?"},{"varval":"sm_nogap Extra money needed for racquet\n\n<div class=\"aligned\">\n\n>>| | |\n| ------------- | ---------- |\n| | \\= $174 − $92 |\n| | \\= ${{{correctAnswer}}} |\n\n</div>\n"}]},{"vars":[{"varval":"Andrew has saved $140.\n\n<br>\n\nsm_img https://teacher.smartermaths.com.au/wp-content/uploads/2023/06/fishing-rod-1_1-min.svg 250 indent vpad\n\n<br>How much more money does he need to buy the fishing rod?"},{"varval":"sm_nogap Extra money needed for fishing rod\n\n<div class=\"aligned\">\n\n>>| | |\n| ------------- | ---------- |\n| | \\= $213 − $140 |\n| | \\= ${{{correctAnswer}}} |\n\n</div>\n"}]},{"vars":[{"varval":"Rohan has saved $394.\n\n<br>\n\nsm_img https://teacher.smartermaths.com.au/wp-content/uploads/2023/06/bike-2_2-min.svg 270 indent vpad\n\n<br>How much more money does he need to buy the mountain bike?"},{"varval":"sm_nogap Extra money needed for mountain bike\n\n<div class=\"aligned\">\n\n>>| | |\n| ------------- | ---------- |\n| | \\= $500 − $394 |\n| | \\= ${{{correctAnswer}}} |\n\n</div>\n"}]},{"vars":[{"varval":"Mark has saved $385.\n\n<br>\n\nsm_img https://teacher.smartermaths.com.au/wp-content/uploads/2023/06/surfboard-1_1-min.svg 100 indent vpad\n\n<br>How much more money does he need to buy the surfboard?"},{"varval":"sm_nogap Extra money needed for surfboard\n\n<div class=\"aligned\">\n\n>>| | |\n| ------------- | ---------- |\n| | \\= $560 − $385 |\n| | \\= ${{{correctAnswer}}} |\n\n</div>\n"}]},{"vars":[{"varval":"Thea has saved $67.\n\n<br>\n\nsm_img https://teacher.smartermaths.com.au/wp-content/uploads/2023/06/kite-1_1-min.svg 170 indent vpad\n\n<br>How much more money does she need to buy the kite?"},{"varval":"sm_nogap Extra money needed for kite\n\n<div class=\"aligned\">\n\n>>| | |\n| ------------- | ---------- |\n| | \\= $123 − $67 |\n| | \\= ${{{correctAnswer}}} |\n\n</div>\n"}]},{"vars":[{"varval":"Romi has saved $88.\n\n<br>\n\nsm_img https://teacher.smartermaths.com.au/wp-content/uploads/2023/06/shoes-1_1-min.svg 170 indent vpad\n\n<br>How much more money does she need to buy the training shoes?"},{"varval":"sm_nogap Extra money needed for training shoes\n\n<div class=\"aligned\">\n\n>>| | |\n| ------------- | ---------- |\n| | \\= $157 − $88 |\n| | \\= ${{{correctAnswer}}} |\n\n</div>\n"}]}]

  65. SJ Add/subtract From: Algebra, NAPX-p124052v01

    <div class="sm_mode"> {{{question}}} </div>

    [{"vars":[{"varval":"The runs scored by Billy and Ray in 4 games of cricket are recorded in the table below.\n\n<br>\n\n\n<div class=\"sm-table col1-color2 row1-color2 top-left-cell-hidden\">\n\n>>| | Billy | Ray |\n|:-:|:-:|:-:|\n|Game 1| 35| 22|\n|Game 2| 78| 64|\n|Game 3| 29| 41|\n|Game 4| 18| 26|\n\n</div>\n\n<br>In which game did Ray score 12 more runs than Billy?"},{"varval":"Consider each game,\n \nGame 1: &nbsp;35 $-$ 22 = 13 runs (more by Billy)\n\nGame 2: &nbsp;78 $-$ 64 = 14 runs (more by Billy)\n\nGame 3: &nbsp;41 $-$ 29 = 12 runs (more by Ray)\n\nGame 4: &nbsp;26 $-$ 18 = 8 runs (more by Ray)\n\nTherefore, in {{{correctAnswer}}} Ray scored 12 more runs than Billy."}]},{"vars":[{"varval":"The goals scored by Jett and Jax in 4 games of basketball are recorded in the table below.\n\n<br>\n\n\n<div class=\"sm-table col1-color1 row1-color1 top-left-cell-hidden\">\n\n>>| | Jett | Jax |\n|:-:|:-:|:-:|\n|Game 1| 18| 24|\n|Game 2| 44| 29|\n|Game 3| 26| 12|\n|Game 4| 6| 34|\n\n</div>\n\n<br>In which game did Jett score 15 more goals than Jax?"},{"varval":"Consider each game,\n \nGame 1: &nbsp;24 $-$ 18 = 6 goals (more by Jax)\n\nGame 2: &nbsp;44 $-$ 29 = 15 goals (more by Jett)\n\nGame 3: &nbsp;26 $-$ 12 = 14 goals (more by Jett)\n\nGame 4: &nbsp;34 $-$ 6 = 28 goals (more by Jax)\n\nTherefore, in {{{correctAnswer}}} Jett scored 15 more runs than Jax."}]},{"vars":[{"varval":"The goals scored by Erin and Emily in 4 seasons of netball are recorded in the table below.\n\n<br>\n\n\n<div class=\"sm-table col1-color3 row1-color3 top-left-cell-hidden\">\n\n>>| | Erin | Emily |\n|:-:|:-:|:-:|\n|Season 1| 67| 64|\n|Season 2| 76| 92|\n|Season 3| 74| 58|\n|Season 4| 46| 20|\n\n</div>\n\n<br>In which Season did Erin score 16 more goals than Emily?"},{"varval":"Consider each season,\n \nSeason 1: &nbsp;67 $-$ 64 = 3 goals (more by Erin)\n\nSeason 2: &nbsp;92 $-$ 76 = 16 goals (more by Emily)\n\nSeason 3: &nbsp;74 $-$ 58 = 16 goals (more by Erin)\n\nSeason 4: &nbsp;46 $-$ 20 = 26 goals (more by Erin)\n\nTherefore, in {{{correctAnswer}}} Erin scored 16 more goals than Emily."}]},{"vars":[{"varval":"The points scored by Nat and Alex in 4 rounds of trivia are recorded in the table below.\n\n<br>\n\n\n<div class=\"sm-table col1-color4 row1-color4 top-left-cell-hidden\">\n\n>>| | Nat | Alex |\n|:-:|:-:|:-:|\n|Round 1| 27| 32|\n|Round 2| 36| 47|\n|Round 3| 48| 29|\n|Round 4| 42| 31|\n\n</div>\n\n<br>In which round did Nat score 11 more points than Alex?"},{"varval":"Consider each game,\n \nRound 1: &nbsp;32 $-$ 27 = 5 points (more by Alex)\n\nRound 2: &nbsp;47 $-$ 36 = 11 points (more by Alex)\n\nRound 3: &nbsp;48 $-$ 29 = 19 points (more by Nat)\n\nRound 4: &nbsp;42 $-$ 31 = 11 points (more by Nat)\n\nTherefore, in {{{correctAnswer}}} Nat scored 11 more points than Alex."}]},{"vars":[{"varval":"The goals scored by Zane and Taren in 4 rounds of soccer are recorded in the table below.\n\n<br>\n\n\n<div class=\"sm-table col1-color5 row1-color5 top-left-cell-hidden\">\n\n>>| | Zane | Taren |\n|:-:|:-:|:-:|\n|Round 1| 18| 23|\n|Round 2| 17| 9|\n|Round 3| 16| 22|\n|Round 4| 9| 4|\n\n</div>\n\n<br>In which round did Taren score 5 more goals than Zane?"},{"varval":"Consider each game,\n \nRound 1: &nbsp;23 $-$ 18 = 5 goals (more by Taren)\n\nRound 2: &nbsp;17 $-$ 9 = 8 goals (more by Zane)\n\nRound 3: &nbsp;22 $-$ 16 = 6 goals (more by Taren)\n\nRound 4: &nbsp;9 $-$ 4 = 5 goals (more by Zane)\n\nTherefore, in {{{correctAnswer}}} Taren scored 5 more goals than Zane."}]},{"vars":[{"varval":"The runs scored by Rich and Viv in 4 games of cricket are recorded in the table below.\n\n<br>\n\n\n<div class=\"sm-table col1-color6 row1-color6 top-left-cell-hidden\">\n\n>>| | Rich | Viv |\n|:-:|:-:|:-:|\n|Game 1| 98| 87|\n|Game 2| 43| 52|\n|Game 3| 105| 86|\n|Game 4| 62| 81|\n\n</div>\n\n<br>In which game did Viv score 19 more runs than Rich?"},{"varval":"Consider each game,\n \nGame 1: &nbsp;98 $-$ 87 = 11 runs (more by Rich)\n\nGame 2: &nbsp;52 $-$ 43 = 9 runs (more by Viv)\n\nGame 3: &nbsp;105 $-$ 86 = 19 runs (more by Rich)\n\nGame 4: &nbsp;81 $-$ 62 = 19 runs (more by Viv)\n\nTherefore, in {{{correctAnswer}}} Viv scored 19 more runs than Rich."}]}]

  66. SJ Add/subtract

    <div class="sm_mode"> {{{question}}} </div>

    [{"vars":[{"varval":"Julius did two long bush walks during the school holidays.\n\nHe went 68 kilometres on the first bush walk and 82 kilometres on the second bush walk.\n\nWhich number sentence could be used to find the total number of kilometres that Julius walked?"},{"varval":"sm_nogap Total amount walked\n\n<div class=\"aligned\">\n\n>| | |\n| ------------- | ---------- |\n| | \\= 68 + 82 |\n| | \\= 60 + 8 + 80 + 2 |\n| | \\= 60 + 80 + 10 |\n| | \\= 150 |\n\n</div>\n"}]},{"vars":[{"varval":"Charles had 2 different casual jobs.\n\nLast week he earned $74 from the first and $66 from the second.\n\nWhich number sentence could be used to find the total amount Charles earned last week?"},{"varval":"sm_nogap Total amount earned\n\n<div class=\"aligned\">\n\n>| | |\n| ------------- | ---------- |\n| | \\= 74 + 66 |\n| | \\= 70 + 4 + 60 + 6 |\n| | \\= 70 + 60 + 10 |\n| | \\= 140 |\n\n</div>"}]},{"vars":[{"varval":"Ang bought groceries from 2 different supermarkets.\n\nAt the first he spent $47 and at the second he spent $83.\n\nWhich number sentence could be used to find the total amount of money Ang spent on groceries?"},{"varval":"sm_nogap Total amount spent\n\n<div class=\"aligned\">\n\n>| | |\n| ------------- | ---------- |\n| | \\= 47 + 83 |\n| | \\= 40 + 7 + 80 + 3 |\n| | \\= 40 + 80 + 10 |\n| | \\= 130 |\n\n</div>"}]}]

  67. SJ Add/subtract

    <div class="sm_mode"> {{{question}}} </div>

    [{"vars":[{"varval":"What number is 10 less than 1037?"},{"varval":"1037 $-$ 10 = {{{correctAnswer}}}"}]},{"vars":[{"varval":"What number is 9 less than 708?"},{"varval":"<div class=\"aligned\">\n\n|||\n|-|-|\n| 708 $-$ 9 | \\= (708 $-$ 10) + 1 |\n| | \\= 698 + 1 |\n| | \\= {{{correctAnswer}}} |\n\n</div>\n"}]},{"vars":[{"varval":"What number is 9 less than 404?"},{"varval":"<div class=\"aligned\">\n\n|||\n|-|-|\n| 404 $-$ 9 | \\= (404 $-$ 10) + 1 |\n| | \\= 394 + 1 |\n| | \\= {{{correctAnswer}}} |\n\n</div>"}]},{"vars":[{"varval":"What number is 19 less than 882?"},{"varval":"<div class=\"aligned\">\n\n|||\n|-|-|\n| 882 $-$ 19 | \\= (882 $-$ 20) + 1 |\n| | \\= 862 + 1 |\n| | \\= {{{correctAnswer}}} |\n\n</div>"}]},{"vars":[{"varval":"What number is 19 less than 748?"},{"varval":"<div class=\"aligned\">\n\n|||\n|-|-|\n| 748 $-$ 19 | \\= (748 $-$ 20) + 1 |\n| | \\= 728 + 1 |\n| | \\= {{{correctAnswer}}} |\n\n</div>"}]},{"vars":[{"varval":"What number is 29 less than 977?"},{"varval":"<div class=\"aligned\">\n\n|||\n|-|-|\n| 977 $-$ 29 | \\= (977 $-$ 30) + 1 |\n| | \\= 947 + 1 |\n| | \\= {{{correctAnswer}}} |\n\n</div>"}]}]

  68. SJ Add/Subtract

    <div class="sm_mode"> {{{question}}} </div>

    [{"vars":[{"varval":"Jonas was packing bananas in boxes that he would sell at the market.\n\nHe packed 8 boxes and had an extra 4 kilograms of bananas left over that he would sell individually.\n\nIf each box contained 9 kilograms of bananas, which of the following number sentences shows how many kilograms of bananas there were in total?\n"},{"varval":"<div class=\"aligned\">\n\n| | |\n| ----------------------- | ---------------------- |\n| Total kilograms of bananas | \\= (8 $\\times$ 9) + 4 |\n| | \\= {{{correctAnswer}}} |\n\n</div>"}]},{"vars":[{"varval":"Mort was packing tiger prawns in boxes that he would sell at the fish market.\n\nHe packed 11 boxes and had an extra 5 kilograms of tiger prawns left over that he would sell individually.\n\nIf each box contained 8 kilograms of tiger prawns, which of the following number sentences shows how many kilograms of tiger prawns there were in total?\n"},{"varval":"<div class=\"aligned\">\n\n| | |\n| ----------------------- | ---------------------- |\n| Total kilograms of tiger prawns | \\= (11 $\\times$ 8) + 5 |\n| | \\= {{{correctAnswer}}} |\n\n</div>"}]},{"vars":[{"varval":"Opal was packing cupcakes in boxes that she would sell at the market.\n\nShe packed 7 boxes and had an extra 5 cupcakes left over that she would sell individually.\n\nIf each box contained 6 cupcakes, which of the following number sentences shows how many cupcakes there were in total?\n"},{"varval":"<div class=\"aligned\">\n\n| | |\n| ----------------------- | ---------------------- |\n| Total cupcakes | \\= (7 $\\times$ 6) + 5 |\n| | \\= {{{correctAnswer}}} |\n\n</div>"}]},{"vars":[{"varval":"Winter was packing cookies in boxes that she would sell at the local coffee shop.\n\nShe packed 5 boxes and had an extra 8 cookies left over that she would sell individually.\n\nIf each box contained 10 cookies, which of the following number sentences shows how many cookies there were in total?\n"},{"varval":"<div class=\"aligned\">\n\n| | |\n| ----------------------- | ---------------------- |\n| Total cookies | \\= (5 $\\times$ 10) + 8 |\n| | \\= {{{correctAnswer}}} |\n\n</div>"}]},{"vars":[{"varval":"Lennox was packing eggs in cartons that she would sell at the local market.\n\nShe packed 7 cartons and had an extra 11 eggs left over that she would sell individually.\n\nIf each carton contained 12 eggs, which of the following number sentences shows how many eggs there were in total?\n"},{"varval":"<div class=\"aligned\">\n\n| | |\n| ----------------------- | ---------------------- |\n| Total kilograms of bananas | \\= (7 $\\times$ 12) + 11 |\n| | \\= {{{correctAnswer}}} |\n\n</div>"}]},{"vars":[{"varval":"Brax was packing grapes in boxes that he would sell at the market.\n\nHe packed 4 boxes and had an extra 70 kilograms of grapes left over that he would sell individually.\n\nIf each box contained 100 kilograms of grapes, which of the following number sentences shows how many kilograms of grapes there were in total?\n"},{"varval":"<div class=\"aligned\">\n\n| | |\n| ----------------------- | ---------------------- |\n| Total kilograms of bananas | \\= (4 $\\times$ 100) + 70 |\n| | \\= {{{correctAnswer}}} |\n\n</div>"}]}]

  69. SJ Multiply/Divide

    <div class="sm_mode"> {{{question}}} </div>

    [{"vars":[{"varval":"Ezra swam 1800 metres during training on Wednesday.\n\nIsaiah swam half as far as Ezra.\n\nHow far did Isaiah swim?"},{"varval":"<div class=\"aligned\">\n\n| | |\n| ------------: | ---------- |\n| Isaiah's distance | \\= 1800 $\\div$ 2 |\n| | \\= {{{correctAnswer}}} metres|\n\n</div>\n"}]},{"vars":[{"varval":"Jordy has 20 lollipops.\n\nJock has half as many lollipops as Jordy.\n\nHow many lollipops does Jock have?"},{"varval":"<div class=\"aligned\">\n\n| | |\n| ------------: | ---------- |\n| Jock's lollipops| \\= 20 $\\div$ 2 |\n| | \\= {{{correctAnswer}}} |\n\n</div>\n"}]},{"vars":[{"varval":"Sadie caught 18 fish.\n\nCora caught one third as many fish as Sadie.\n\nHow many fish did Cora catch?"},{"varval":"<div class=\"aligned\">\n\n| | |\n| ------------: | ---------- |\n| Cora's fish | \\= 18 $\\div$ 3 |\n| | \\= {{{correctAnswer}}} |\n\n</div>\n"}]},{"vars":[{"varval":"Serenity has 28 cupcakes.\n\nBliss has one half the number of cupcakes that Serenity has.\n\nHow many cupcakes does Bliss have?"},{"varval":"<div class=\"aligned\">\n\n| | |\n| ------------: | ---------- |\n| Bliss's cupcakes | \\= 28 $\\div$ 2 |\n| | \\= {{{correctAnswer}}} |\n\n</div>\n"}]},{"vars":[{"varval":"Mateo picked 64 kilograms of peaches.\n\nConstance picked one half the number of kilograms of peaches that Mateo picked.\n\nHow many kilograms of peaches did Constance pick?"},{"varval":"<div class=\"aligned\">\n\n| | |\n| ------------: | ---------- |\n| Constance's peaches | \\= 64 $\\div$ 2 |\n| | \\= {{{correctAnswer}}} kilograms|\n\n</div>\n"}]},{"vars":[{"varval":"Wyatt rode 160 kilometres on his bike over the weekend.\n\nCarter rode half as far as Wyatt.\n\nHow far did Carter ride over the weekend?"},{"varval":"<div class=\"aligned\">\n\n| | |\n| ------------: | ---------- |\n| Carter's distance | \\= 160 $\\div$ 2 |\n| | \\= {{{correctAnswer}}} |\n\n</div>\n"}]}]

  70. SJ Multiply/Divide

    <div class="sm_mode"> {{{question}}} What number makes the above number sentence correct? </div>

    [{"vars":[{"varval":"80 $\\div$ 8 = 5 $\\times$ <span class=\"sm-text\">?</span>"},{"varval":"<div class=\"aligned\">\n\n|||\n|-:|-|\n|80 $\\div$ 8|= 10|\n|5 $\\times$ <span class=\"sm-text\">?</span>|= 10|\n\n</div>\n<br>\nTherefore, <span class=\"sm-text\">?</span> = {{{correctAnswer}}}"}]},{"vars":[{"varval":"30 $\\div$ 5 = 2 $\\times$ <span class=\"sm-text\">?</span>"},{"varval":"<div class=\"aligned\">\n\n|||\n|-:|-|\n|30 $\\div$ 5|= 6|\n|2 $\\times$ <span class=\"sm-text\">?</span>|= 6|\n\n</div>\n<br>\nTherefore, <span class=\"sm-text\">?</span> = {{{correctAnswer}}}"}]},{"vars":[{"varval":"32 $\\div$ 4 = 2 + <span class=\"sm-text\">?</span>"},{"varval":"<div class=\"aligned\">\n\n|||\n|-:|-|\n|32 $\\div$ 4|= 8|\n|2 + <span class=\"sm-text\">?</span>|= 8|\n\n</div>\n<br>\nTherefore, <span class=\"sm-text\">?</span> = {{{correctAnswer}}}"}]},{"vars":[{"varval":"3 $\\times$ 7 = 17 + <span class=\"sm-text\">?</span>"},{"varval":"<div class=\"aligned\">\n\n|||\n|-:|-|\n|3 $\\times$ 7|= 21|\n|17 + <span class=\"sm-text\">?</span>|= 21|\n\n</div>\n<br>\nTherefore, <span class=\"sm-text\">?</span> = {{{correctAnswer}}}"}]},{"vars":[{"varval":"13 + 9 = 2 $\\times$ <span class=\"sm-text\">?</span>"},{"varval":"<div class=\"aligned\">\n\n|||\n|-:|-|\n|13 + 9|= 22|\n|2 $\\times$ <span class=\"sm-text\">?</span>|= 22|\n\n</div>\n<br>\nTherefore, <span class=\"sm-text\">?</span> = {{{correctAnswer}}}"}]},{"vars":[{"varval":"5 $\\times$ 4 = 15 + <span class=\"sm-text\">?</span>"},{"varval":"<div class=\"aligned\">\n\n|||\n|-:|-|\n|5 $\\times$ 4|= 20|\n|15 + <span class=\"sm-text\">?</span>|= 20|\n\n</div>\n<br>\nTherefore, <span class=\"sm-text\">?</span> = {{{correctAnswer}}}"}]}]

  71. SJ Multiply/Divide

    <div class="sm_mode"> {{{question}}} What number makes the above number sentence correct? </div>

    [{"vars":[{"varval":"\n20 $\\div$ <span class=\"sm-text\">?</span> = 10 $\\div$ 2\n\n"},{"varval":"<div class=\"aligned\">\n\n|||\n|-:|-|\n|10 $\\div$ 2|= 5|\n|20 $\\div$ <span class=\"sm-text\">?</span>|= 5|\n\n</div>\n<br>\nTherefore, <span class=\"sm-text\">?</span> = {{{correctAnswer}}}"}]},{"vars":[{"varval":"30 $\\div$ <span class=\"sm-text\">?</span> = 15 $\\div$ 3"},{"varval":"<div class=\"aligned\">\n\n|||\n|-:|-|\n|15 $\\div$ 3|= 5|\n|30 $\\div$ <span class=\"sm-text\">?</span>|= 5|\n\n</div>\n<br>\nTherefore, <span class=\"sm-text\">?</span> = {{{correctAnswer}}}"}]},{"vars":[{"varval":"40 $\\div$ <span class=\"sm-text\">?</span> = 20 $\\div$ 5"},{"varval":"<div class=\"aligned\">\n\n|||\n|-:|-|\n|20 $\\div$ 5|= 4|\n|40 $\\div$ <span class=\"sm-text\">?</span>|= 4|\n\n</div>\n<br>\nTherefore, <span class=\"sm-text\">?</span> = {{{correctAnswer}}}"}]},{"vars":[{"varval":"60 $\\div$ <span class=\"sm-text\">?</span> = 30 $\\div$ 3"},{"varval":"<div class=\"aligned\">\n\n|||\n|-:|-|\n|30 $\\div$ 3|= 10|\n|60 $\\div$ <span class=\"sm-text\">?</span>|= 10|\n\n</div>\n<br>\nTherefore, <span class=\"sm-text\">?</span> = {{{correctAnswer}}}"}]},{"vars":[{"varval":"42 $\\div$ <span class=\"sm-text\">?</span> = 21 $\\div$ 3."},{"varval":"<div class=\"aligned\">\n\n|||\n|-:|-|\n|21 $\\div$ 3|= 7|\n|42 $\\div$ <span class=\"sm-text\">?</span>|= 7|\n\n</div>\n<br>\nTherefore, <span class=\"sm-text\">?</span> = {{{correctAnswer}}}"}]},{"vars":[{"varval":"36 $\\div$ <span class=\"sm-text\">?</span> = 18 $\\div$ 6"},{"varval":"<div class=\"aligned\">\n\n|||\n|-:|-|\n|18 $\\div$ 6|= 3|\n|36 $\\div$ <span class=\"sm-text\">?</span>|= 3|\n\n</div>\n<br>\nTherefore, <span class=\"sm-text\">?</span> = {{{correctAnswer}}}"}]}]

  72. <div class="sm_mode"> {{{question}}} </div>

    [{"vars":[{"varval":"Cindy had a box of apples.\n\nShe gave 4 of her friends 6 apples each and had 2 apples left over for herself.\n\nHow many apples, in total, were in the box?"},{"varval":"<div class=\"aligned\">\n\n|||\n|-:|-|\n|Total apples|= 4 $\\times$ 6 + 2|\n| |= {{{correctAnswer}}}|\n\n</div>"}]},{"vars":[{"varval":"Chablis had a bag of musksticks.\n\nShe gave 4 of her friends 5 musksticks each and had 3 left over for herself. \n\nHow many musksticks, in total, were in the bag?"},{"varval":"<div class=\"aligned\">\n\n|||\n|-:|-|\n|Total musksticks|= 4 $\\times$ 5 + 3|\n| |= {{{correctAnswer}}}|\n\n</div>"}]},{"vars":[{"varval":"Paisley had a bag of coins.\n\nShe gave 6 of her friends 3 coins each and had 2 left over for herself. \n\nHow many coins, in total, were in the bag?"},{"varval":"<div class=\"aligned\">\n\n|||\n|-:|-|\n|Total coins|= 6 $\\times$ 3 + 2|\n| |= {{{correctAnswer}}}|\n\n</div>"}]},{"vars":[{"varval":"Autumn had a bag of pine cones.\n\nShe gave 3 of her friends 11 pine cones each and had 7 left over for herself. \n\nHow many pine cones, in total, were in the bag?"},{"varval":"<div class=\"aligned\">\n\n|||\n|-:|-|\n|Total pine cones|= 3 $\\times$ 11 + 7|\n| |= {{{correctAnswer}}}|\n\n</div>"}]},{"vars":[{"varval":"Quinn had a bunch of flowers.\n\nShe gave 4 of her friends 8 flowers each and had 7 left over for herself. \n\nHow many flowers, in total, were in the bunch?"},{"varval":"<div class=\"aligned\">\n\n|||\n|-:|-|\n|Total flowers|= 4 $\\times$ 8 + 7|\n| |= {{{correctAnswer}}}|\n\n</div>"}]},{"vars":[{"varval":"Vickery had a box of apples.\n\nHe gave 5 of his friends 6 apples each and had 5 left over for himself. \n\nHow many apples, in total, were in the box?"},{"varval":"<div class=\"aligned\">\n\n|||\n|-:|-|\n|Total musksticks|= 5 $\\times$ 6 + 5|\n| |= {{{correctAnswer}}}|\n\n</div>"}]}]

  73. SJ Multiply/divide

    <div class="sm_mode"> {{{question}}} </div>

    [{"vars":[{"varval":"Cassie has 14 cookies.\n\nShe needs to divide the cookies into packets of 3.\n\nHow many cookies will she have left over?\n"},{"varval":"sm_nogap Number of packets\n\n<div class=\"aligned\">\n\n>|||\n|-:|-|\n|14 $\\div$ 3|= 4 remainder 2|\n\n</div>\n<br>\nTherefore, there will be {{{correctAnswer}}} cookies leftover."}]},{"vars":[{"varval":"Madge has 25 oranges.\n\nShe needs to divide the oranges into packets of 4.\n\nHow many oranges will she have left over?"},{"varval":"sm_nogap Number of packets\n\n<div class=\"aligned\">\n\n>|||\n|-:|-|\n|25 $\\div$ 4|= 6 remainder 1|\n\n</div>\n<br>\nTherefore, there will be {{{correctAnswer}}} orange leftover."}]},{"vars":[{"varval":"Bryn has 50 coloured discs.\n\nHe needs to divide the coloured discs into packets of 8.\n\nHow many coloured discs will he have left over?"},{"varval":"sm_nogap Number of packets\n\n<div class=\"aligned\">\n\n>|||\n|-:|-|\n|50 $\\div$ 8|= 6 remainder 2|\n\n</div>\n<br>\nTherefore, there will be {{{correctAnswer}}} coloured discs leftover."}]},{"vars":[{"varval":"Bonnie has 33 pairs of socks.\n\nShe needs to divide the pairs of socks into packets of 5.\n\nHow many pairs of socks will she have left over?"},{"varval":"sm_nogap Number of packets\n\n<div class=\"aligned\">\n\n>|||\n|-:|-|\n|33 $\\div$ 5|= 6 remainder 3|\n\n</div>\n<br>\nTherefore, there will be {{{correctAnswer}}} pairs of socks leftover."}]},{"vars":[{"varval":"Violet has 36 roses.\n\nShe needs to divide the roses into bunches of 8.\n\nHow many roses will she have left over?"},{"varval":"sm_nogap Number of packets\n\n<div class=\"aligned\">\n\n>|||\n|-:|-|\n|36 $\\div$ 8|= 4 remainder 4|\n\n</div>\n<br>\nTherefore, there will be {{{correctAnswer}}} roses leftover."}]},{"vars":[{"varval":"Addison has 51 cupcakes.\n\nShe needs to divide the cupcakes into packets of 4.\n\nHow many cupcakes will she have left over?"},{"varval":"sm_nogap Number of packets\n\n<div class=\"aligned\">\n\n>|||\n|-:|-|\n|51 $\\div$ 4|= 12 remainder 3|\n\n</div>\n<br>\nTherefore, there will be {{{correctAnswer}}} cupcakes leftover."}]},{"vars":[{"varval":"Bentley has 23 vintage cars.\n\nHe needs to fit the vintage cars into 5 garages.\n\nHow many vintage cars will be left without a garage?"},{"varval":"sm_nogap Number of cars remaining\n\n<div class=\"aligned\">\n\n>|||\n|-:|-|\n|23 $\\div$ 5|= 4 remainder 3|\n\n</div>\n<br>\nTherefore, there will be {{{correctAnswer}}} vintage cars without a garage."}]}]

  74. SJ Multiply/Divide

    <div class="sm_mode"> {{{question}}} </div>

    [{"vars":[{"varval":"Mark has these ice creams.\n\nsm_img https://teacher.smartermaths.com.au/wp-content/uploads/2023/06/12x-ice-cream-2-min.svg 400 indent vpad\n\nHe shares the ice creams equally between his 3 friends.\n\nHow many ice creams does each friend receive?"},{"varval":"<div class=\"aligned\">\n\n| | |\n| ------------- | ---------- |\n| Number of ice creams each | \\= 12 $\\div$ 3|\n| | \\= {{{correctAnswer}}} |\n\n</div>\n\n\n"}]},{"vars":[{"varval":"Jannie has these ice creams.\n\nsm_img https://teacher.smartermaths.com.au/wp-content/uploads/2023/06/12x-ice-cream-2-min.svg 400 indent vpad\n\nHe shares the ice creams equally between his 4 friends.\n\nHow many ice creams does each friend receive?"},{"varval":"<div class=\"aligned\">\n\n| | |\n| ------------- | ---------- |\n| Number of ice creams each | \\= 12 $\\div$ 4|\n| | \\= {{{correctAnswer}}} |\n\n</div>"}]},{"vars":[{"varval":"Bernice has these apples.\n\nsm_img https://teacher.smartermaths.com.au/wp-content/uploads/2023/06/15-x-apple-1-min.svg 350 indent vpad\n\nShe shares the apples equally between her 5 friends.\n\nHow many apples does each friend receive?"},{"varval":"<div class=\"aligned\">\n\n| | |\n| ------------- | ---------- |\n| Number of apples each | \\= 15 $\\div$ 5|\n| | \\= {{{correctAnswer}}} |\n\n</div>"}]},{"vars":[{"varval":"Candy has these apples.\n\nsm_img https://teacher.smartermaths.com.au/wp-content/uploads/2023/06/15-x-apple-1-min.svg 350 indent vpad\n\nShe shares the apples equally between her 3 friends.\n\nHow many apples does each friend receive?"},{"varval":"<div class=\"aligned\">\n\n| | |\n| ------------- | ---------- |\n| Number of apples each | \\= 15 $\\div$ 3|\n| | \\= {{{correctAnswer}}} |\n\n</div>"}]},{"vars":[{"varval":"Jenna has these marbles.\n\nsm_img https://teacher.smartermaths.com.au/wp-content/uploads/2023/06/2-x-marbles-9-min.svg 350 indent vpad\n\nShe shares the marbles equally between her 3 friends.\n\nHow many marbles does each friend receive?"},{"varval":"<div class=\"aligned\">\n\n| | |\n| ------------- | ---------- |\n| Number of marbles each | \\= 18 $\\div$ 3|\n| | \\= {{{correctAnswer}}} |\n\n</div>"}]},{"vars":[{"varval":"Nora has these marbles.\n\nsm_img https://teacher.smartermaths.com.au/wp-content/uploads/2023/06/2-x-marbles-9-min.svg 350 indent vpad\n\nShe shares the marbles equally between her 6 friends.\n\nHow many marbles does each friend receive?"},{"varval":"<div class=\"aligned\">\n\n| | |\n| ------------- | ---------- |\n| Number of marbles each | \\= 18 $\\div$ 6|\n| | \\= {{{correctAnswer}}} |\n\n</div>"}]}]

  75. SJ multiply/divide

    <div class="sm_mode"> {{{question}}} </div>

    [{"vars":[{"varval":"Here are some sheep in a field.\n\nsm_img https://teacher.smartermaths.com.au/wp-content/uploads/2023/06/sheep-4-min.svg 450 indent vpad\n\nWhich of the following is **NOT** a way to work out the total number of legs on the sheep?"},{"varval":"<div class=\"aligned\">\n\n| | |\n| ------------- | ---------- |\n| Number of legs | \\= number of sheep $\\times$ 4|\n| |\\= 4 $\\times$ 4|\n| | \\= 16 |\n\n</div>\n\nOption 1 = 8\n\nOption 2 = 16\n\nOption 3 = 16\n\nOption 4 = 16\n\nTherefore , {{{correctAnswer}}} is not a way to work out the number of legs.\n"}]},{"vars":[{"varval":"Here are some birthday cakes with candles.\n\nsm_img https://teacher.smartermaths.com.au/wp-content/uploads/2023/06/3xcake-5.svg 400 indent vpad\n\nWhich of the following is **NOT** a way to work out the total number of candles on the cakes?"},{"varval":"<div class=\"aligned\">\n\n| | |\n| ------------- | ---------- |\n| Number of candles | \\= number of cakes $\\times$ 5|\n| |\\= 3 $\\times$ 5|\n| | \\= 15 |\n\n</div>\n\nOption 1 = 15\n\nOption 2 = 9\n\nOption 3 = 15\n\nOption 4 = 15\n\nTherefore , {{{correctAnswer}}} is not a way to work out the number of candles."}]},{"vars":[{"varval":"Here are some birthday cakes with candles.\n\nsm_img https://teacher.smartermaths.com.au/wp-content/uploads/2023/06/7xcake-3-min.svg 300 indent vpad\n\nWhich of the following is **NOT** a way to work out the total number of candles on the cakes?"},{"varval":"<div class=\"aligned\">\n\n| | |\n| ------------- | ---------- |\n| Number of candles | \\= number of cakes $\\times$ 3|\n| |\\= 7 $\\times$ 3|\n| | \\= 21 |\n\n</div>\n\nOption 1 = 21\n\nOption 2 = 21\n\nOption 3 = 21\n\nOption 4 = 49\n\nTherefore , {{{correctAnswer}}} is not a way to work out the number of candles."}]},{"vars":[{"varval":"Here are some birthday cakes with candles.\n\nsm_img https://teacher.smartermaths.com.au/wp-content/uploads/2023/06/6xcake-5-min.svg 350 indent vpad\n\nWhich of the following is **NOT** a way to work out the total number of candles on the cakes?"},{"varval":"<div class=\"aligned\">\n\n| | |\n| ------------- | ---------- |\n| Number of candles | \\= number of cakes $\\times$ 5|\n| |\\= 6 $\\times$ 5|\n| | \\= 30 |\n\n</div>\n\nOption 1 = 30\n\nOption 2 = 30\n\nOption 3 = 36\n\nOption 4 = 30\n\nTherefore , {{{correctAnswer}}} is not a way to work out the number of candles."}]},{"vars":[{"varval":"Here are some birthday cakes with candles.\n\nsm_img https://teacher.smartermaths.com.au/wp-content/uploads/2023/06/4xcake-7-min.svg 550 indent vpad\n\nWhich of the following is **NOT** a way to work out the total number of candles on the cakes?"},{"varval":"<div class=\"aligned\">\n\n| | |\n| ------------- | ---------- |\n| Number of candles | \\= number of cakes $\\times$ 7|\n| |\\= 4 $\\times$ 7|\n| | \\= 28 |\n\n</div>\n\nOption 1 = 16\n\nOption 2 = 28\n\nOption 3 = 28\n\nOption 4 = 28\n\nTherefore , {{{correctAnswer}}} is not a way to work out the number of candles."}]},{"vars":[{"varval":"Here are some cows in a field.\n\nsm_img https://teacher.smartermaths.com.au/wp-content/uploads/2023/06/6xcow-1-min.svg 400 indent vpad\n\nWhich of the following is **NOT** a way to work out the total number of legs on the cows?"},{"varval":"<div class=\"aligned\">\n\n| | |\n| ------------- | ---------- |\n| Number of legs | \\= number of cows $\\times$ 4|\n| |\\= 6 $\\times$ 4|\n| | \\= 24 |\n\n</div>\n\nOption 1 = 24\n\nOption 2 = 24\n\nOption 3 = 36\n\nOption 4 = 24\n\nTherefore , {{{correctAnswer}}} is not a way to work out the number of legs.\n"}]}]

  76. SJ Number and Place

    <div class="sm_mode"> {{{question}}} </div>

    [{"vars":[{"varval":"Which group of numbers is ordered from smallest to largest?"},{"varval":"Considering each option in order from smallest to largest:\n\nOption 4 is in the correct order. \n\n>> 405, 450, 504, 540\n"}]},{"vars":[{"varval":"\nWhich group of numbers is ordered from smallest to largest?"},{"varval":"Considering each option in order from smallest to largest:\n\nOption 3 is in the correct order. \n\n>> 238, 283, 328, 382"}]},{"vars":[{"varval":"Which group of numbers is ordered from smallest to largest?"},{"varval":"Considering each option in order from smallest to largest:\n\nOption 1 is in the correct order. \n\n>> 576, 657, 675, 765"}]},{"vars":[{"varval":"Which group of numbers is ordered from smallest to largest?"},{"varval":"Considering each option in order from smallest to largest:\n\nOption 2 is in the correct order. \n\n>> 898, 899, 989, 998 "}]},{"vars":[{"varval":"Which group of numbers is ordered from smallest to largest?"},{"varval":"Considering each option in order from smallest to largest:\n\nOption 3 is in the correct order. \n\n>> 767, 770, 780, 787 "}]},{"vars":[{"varval":"Which group of numbers is ordered from smallest to largest?"},{"varval":"Considering each option in order from smallest to largest:\n\nOption 2 is in the correct order. \n\n>> 345, 354, 435, 453"}]}]

  77. SJ Number and Place

    <div class="sm_mode"> {{{question}}} </div>

    [{"vars":[{"varval":"Which group of numbers is written from largest to smallest?"},{"varval":"Considering each option in order from largest to smallest:\n\nOption 2 is in the correct order. \n\n>> 787, 780, 770, 767\n\n"}]},{"vars":[{"varval":"Which group of numbers is written from largest to smallest?"},{"varval":"Considering each option in order from largest to smallest:\n\nOption 3 is in the correct order. \n\n>> 453, 435, 354, 345\n\n"}]},{"vars":[{"varval":"Which group of numbers is written from largest to smallest?"},{"varval":"Considering each option in order from largest to smallest:\n\nOption 2 is in the correct order. \n\n>> 998, 989, 899, 898\n\n"}]},{"vars":[{"varval":"Which group of numbers is written from largest to smallest?"},{"varval":"Considering each option in order from largest to smallest:\n\nOption 4 is in the correct order. \n\n>> 765, 675, 657, 576\n\n"}]},{"vars":[{"varval":"Which group of numbers is written from largest to smallest?"},{"varval":"Considering each option in order from largest to smallest:\n\nOption 1 is in the correct order. \n\n>> 382, 328, 283, 238\n\n"}]},{"vars":[{"varval":"Which group of numbers is written from largest to smallest?"},{"varval":"Considering each option in order from largest to smallest:\n\nOption 4 is in the correct order. \n\n>> 540, 504, 450, 405\n\n"}]}]

  78. SJ Number and Place

    <div class="sm_mode"> {{{question}}} </div>

    [{"vars":[{"varval":"Which of these numbers has the smallest value?"},{"varval":"Arrange the numbers from smallest to the largest. \n\n>>2035 < 2305 < 2350 < 2530\n\n<br>\n\n$\\therefore$ {{{correctAnswer}}} is the smallest number.\n\n"}]},{"vars":[{"varval":"Which of these numbers has the smallest value?"},{"varval":"Arrange the numbers from smallest to the largest. \n\n>>3047 < 3074 < 3407 < 3470\n\n<br>\n\n$\\therefore$ {{{correctAnswer}}} is the smallest number."}]},{"vars":[{"varval":"Which of these numbers has the smallest value?"},{"varval":"Arrange the numbers from smallest to the largest. \n\n>>1021 < 1102 < 1120 < 1201\n\n<br>\n\n$\\therefore$ {{{correctAnswer}}} is the smallest number."}]},{"vars":[{"varval":"Which of these numbers has the smallest value?"},{"varval":"Arrange the numbers from smallest to the largest. \n\n>>709 < 719 < 790 < 791\n\n<br>\n\n$\\therefore$ {{{correctAnswer}}} is the smallest number."}]},{"vars":[{"varval":"Which of these numbers has the smallest value?"},{"varval":"Arrange the numbers from smallest to the largest. \n\n>>123 < 132 < 231 < 312\n\n<br>\n\n$\\therefore$ {{{correctAnswer}}} is the smallest number."}]},{"vars":[{"varval":"Which of these numbers has the smallest value?"},{"varval":"Arrange the numbers from smallest to the largest. \n\n>>4055 < 4505 < 4550 < 5045\n\n<br>\n\n$\\therefore$ {{{correctAnswer}}} is the smallest number."}]}]

  79. SJ Add/Subtract

    <div class="sm_mode"> {{{question}}} </div>

    [{"vars":[{"varval":"On Sunday 56 millimetres of snow fell at Perisher. On Monday another 25 millimetres fell.\n\nHow much snow fell, in total, at Perisher on Sunday and Monday?"},{"varval":"sm_nogap One strategy:\n\n<div class=\"aligned\">\n\n| | |\n| --------------------- | -------------- |\n| 56 + 25| = 50 + 6 + 20 + 5|\n|| = 70 + 11|\n| | \\= {{{correctAnswer}}} millimetres|\n\n</div>"}]},{"vars":[{"varval":"During September Jane's tomato plants grew 18 millimetres and in October they grew another 37 millimetres.\n\nHow much did Jane's tomato plants grow during September and October?"},{"varval":"sm_nogap One strategy:\n\n<div class=\"aligned\">\n\n| | |\n| --------------------- | -------------- |\n| 18 + 37| = 10 + 8 + 30 + 7|\n|| = 40 + 15|\n| | \\= {{{correctAnswer}}} millimetres |\n\n</div>"}]},{"vars":[{"varval":"Cyrus bought a 58 millimetre carp for his garden pond. \n\nWhen he measured it the following year it had grown another 23 millimetres.\n\nHow long was Cyrus's carp when he measured it the following year?"},{"varval":"sm_nogap One strategy:\n\n<div class=\"aligned\">\n\n| | |\n| --------------------- | -------------- |\n| 58 + 23| = 50 + 8 + 20 + 3|\n|| = 70 + 11|\n| | \\= {{{correctAnswer}}} millimetres |\n\n</div>"}]},{"vars":[{"varval":"Last week bailey worked for 27 hours and this week he worked 29 hours.\n\nHow many hours, in total, has Bailey worked over the last two weeks."},{"varval":"sm_nogap One strategy:\n\n<div class=\"aligned\">\n\n| | |\n| --------------------- | -------------- |\n| 27 + 29| = 20 + 7 + 20 + 9|\n|| = 40 + 16|\n| | \\= {{{correctAnswer}}} hours |\n\n</div>"}]},{"vars":[{"varval":"Last week Maude harvested 87 kilograms of strawberries from her hobby farm. This week she harvested another 72 kilograms of strawberries.\n\nHow many kilograms of strawberries has Maude harvested in the last two weeks?"},{"varval":"sm_nogap One strategy:\n\n<div class=\"aligned\">\n\n| | |\n| --------------------- | -------------- |\n| 87 + 72| = 80 + 7 + 70 + 2|\n|| = 150 + 9|\n| | \\= {{{correctAnswer}}} kilograms |\n\n</div>"}]},{"vars":[{"varval":"Bernice has a weekend market stall. On Saturday she sold 48 kilograms of beans and on Sunday she sold 67 kilograms of beans.\n\nHow many kilograms of beans did Bernice sell on the weekend?"},{"varval":"sm_nogap One strategy:\n\n<div class=\"aligned\">\n\n| | |\n| --------------------- | -------------- |\n| 48 + 67| = 40 + 8 + 60 + 7|\n|| = 100 + 15|\n| | \\= {{{correctAnswer}}} kilograms|\n\n</div>"}]}]

  80. SJ Add/subtract

    <div class="sm_mode"> {{{question}}} </div>

    [{"vars":[{"varval":"Millie is 18 and her mother is 31 years older than Millie.\n\nHow old is Millie's mother?"},{"varval":"sm_nogap One strategy:\n\n<div class=\"aligned\">\n\n| | |\n| --------------------- | -------------- |\n| 18 + 31| = 10 + 8 + 30 + 1|\n|| = 40 + 9|\n| | \\= {{{correctAnswer}}} |\n\n</div>"}]},{"vars":[{"varval":"Jordy is 27 and his grandmother is 63 years older than Jordy.\n\nHow old is Jordy's grandmother?"},{"varval":"sm_nogap One strategy:\n\n<div class=\"aligned\">\n\n| | |\n| --------------------- | -------------- |\n| 27 + 63| = 20 + 7 + 60 + 3|\n|| = 80 + 10|\n| | \\= {{{correctAnswer}}} |\n\n</div>"}]},{"vars":[{"varval":"Moulan is 16 and her grandfather is 57 years older than Moulan.\n\nHow old is Moulan's grandfather?"},{"varval":"sm_nogap One strategy:\n\n<div class=\"aligned\">\n\n| | |\n| --------------------- | -------------- |\n| 16 + 57| = 10 + 6 + 50 + 7|\n|| = 60 + 13|\n| | \\= {{{correctAnswer}}} |\n\n</div>"}]},{"vars":[{"varval":"Jiah is 26 and her father is 48 years older than Jiah.\n\nHow old is Jiah's father?"},{"varval":"sm_nogap One strategy:\n\n<div class=\"aligned\">\n\n| | |\n| --------------------- | -------------- |\n| 26 + 48| = 20 + 6 + 40 + 8|\n|| = 60 + 14|\n| | \\= {{{correctAnswer}}} |\n\n</div>"}]},{"vars":[{"varval":"Kwan is 38 and his brother is 17 years older than Kwan.\n\nHow old is Kwan's brother?"},{"varval":"sm_nogap One strategy:\n\n<div class=\"aligned\">\n\n| | |\n| --------------------- | -------------- |\n| 38 + 17| = 30 + 8 + 10 + 7|\n|| = 40 + 15|\n| | \\= {{{correctAnswer}}} |\n\n</div>"}]},{"vars":[{"varval":"Aliyeh is 22 and her sister is 29 years older than Aliyeh.\n\nHow old is Aliyeh's sister?"},{"varval":"sm_nogap One strategy:\n\n<div class=\"aligned\">\n\n| | |\n| --------------------- | -------------- |\n| 22 + 29| = 20 + 2 + 20 + 9|\n|| = 40 + 11|\n| | \\= {{{correctAnswer}}} |\n\n</div>"}]}]

  81. SJ Financial/Money 3-6

    <div class="sm_mode"> {{{question}}} </div>

    [{"vars":[{"varval":"Which of the following groups of coins has the highest value?"},{"varval":"Consider options: \n\nOption 1: \n\n0.50 + 0.20 + 0.50 + 0.20 + 0.20 + 0.10 + 0.10 = $1.80 \n\nOption 2: \n\n0.50 + 0.10 + 0.10 + 0.50 + 0.50 + 0.05 = $1.75\n\nOption 3:\n\n0.50 + 0.50 + 0.10 + 0.10 + 0.50 + 0.05 + 0.10 = $1.85\n\nOption 4:\n\n$1.00 + 0.20 + 0.20 + 0.05 + 0.10 + 0.10 = $1.65\n\nTherefore, Option 3 is the largest.\n\n{{{correctAnswer}}}\n"}]},{"vars":[{"varval":"Which of the following groups of coins has the smallest value?"},{"varval":"Consider options: \n\nOption 1: \n\n0.50 + 0.20 + 0.50 + 0.20 + 0.20 + 0.10 + 0.10 = $1.80 \n\nOption 2: \n\n$1.00 + 0.20 + 0.20 + 0.05 + 0.10 + 0.10 = $1.65\n\nOption 3:\n\n0.50 + 0.50 + 0.10 + 0.10 + 0.50 + 0.05 + 0.10 = $1.85\n\nOption 4:\n\n0.50 + 0.10 + 0.10 + 0.50 + 0.50 + 0.05 = $1.75\n\nTherefore, Option 2 is the smallest.\n\n{{{correctAnswer}}}\n"}]}]

  82. SJ Non-calc 7/8 Add/subtract 4/5/6

    <div class="sm_mode"> {{{question}}} </div>

    [{"vars":[{"varval":"On a fish farm 162 salmon were released in the first month and 128 in the second month.\n\nIn total, how many fish were released in these two months?"},{"varval":"sm_nogap One strategy:\n\n<div class=\"aligned\">\n\n| | |\n| --------------------- | -------------- |\n| 162 + 128| = 100 + 60 + 2 + 100 + 20 + 8|\n|| = 200 + 80 + 10|\n|| = 200 + 90|\n| | \\= {{{correctAnswer}}} |\n\n</div>"}]},{"vars":[{"varval":"On a farm there are 145 cows and 63 sheep.\n\nIn total, how many animals are there on the farm?"},{"varval":"sm_nogap One strategy:\n\n<div class=\"aligned\">\n\n| | |\n| --------------------- | -------------- |\n| 145 + 63| = 100 + 40 + 5 + 60 + 3|\n|| = 100 + 100 + 8|\n|| = 200 + 8|\n| | \\= {{{correctAnswer}}} |\n\n</div>"}]},{"vars":[{"varval":"On a science excursion there are 165 year 7 students and 142 year 9 students.\n\nIn total, how many students are there on the science excursion?"},{"varval":"sm_nogap One strategy:\n\n<div class=\"aligned\">\n\n| | |\n| --------------------- | -------------- |\n| 165 + 142| = 100 + 60 + 5 + 100 + 40 + 2|\n|| = 200 + 100 + 7|\n|| = 300 + 7|\n| | \\= {{{correctAnswer}}} |\n\n</div>"}]},{"vars":[{"varval":"On an Air New Zealand flight there are 243 economy passengers and 87 premium economy passengers.\n\nHow many passengers are on the flight altogether?"},{"varval":"sm_nogap One strategy:\n\n<div class=\"aligned\">\n\n| | |\n| --------------------- | -------------- |\n| 243 + 87| = 200 + 40 + 3 + 80 + 7|\n|| = 200 + 120 + 10|\n|| = 320 + 10|\n| | \\= {{{correctAnswer}}} |\n\n</div>"}]},{"vars":[{"varval":"In an orchard, there are 125 orange trees and 135 lemon trees.\n\nHow many citrus trees are there in the orchard altogether?"},{"varval":"sm_nogap One strategy:\n\n<div class=\"aligned\">\n\n| | |\n| --------------------- | -------------- |\n| 125 + 135| = 100 + 20 + 5 + 100 + 30 + 5|\n|| = 200 + 50 + 10|\n| | \\= {{{correctAnswer}}} |\n\n</div>"}]},{"vars":[{"varval":"Maryanne watches 2 movies on Saturday afternoon.\n\nThe first movie runs for 127 minutes and the second for 92 minutes.\n\nFor how many minutes, in total, did Maryanne watch movies on Saturday afternoon?"},{"varval":"sm_nogap One strategy:\n\n<div class=\"aligned\">\n\n| | |\n| --------------------- | -------------- |\n| 127 + 92| = 100 + 20 + 7 + 90 + 2|\n|| = 100 + 20 + 90 + 9|\n|| = 100 +110 +9\n| | \\= {{{correctAnswer}}} |\n\n</div>"}]}]

  83. SJ_add/subtract and non-calc

    <div class="sm_mode"> {{{question}}} </div>

    [{"vars":[{"varval":"Which of these gives the largest total?"},{"varval":"sm_nogap Checking options:\n\n<div class=\"aligned\">\n\n>>| | |\n| --------------------- | -------------- |\n| 4 + 200| = 204|\n|20 + 400| = 420|\n|2 + 400| = 402|\n|40 + 200| = 240|\n\n</div>\n<br>\nTherefore, option 2 gives the largest total\n"}]},{"vars":[{"varval":"Which of these gives the smallest total?"},{"varval":"sm_nogap Checking options:\n\n<div class=\"aligned\">\n\n>>| | |\n| --------------------- | -------------- |\n| 4 + 200| = 204|\n|20 + 400| = 420|\n|2 + 400| = 402|\n|40 + 200| = 240|\n\n</div>\n<br>\nTherefore, option 1 gives the smallest total\n"}]},{"vars":[{"varval":"Which of these gives the largest total?"},{"varval":"sm_nogap Checking options:\n\n<div class=\"aligned\">\n\n>>| | |\n| --------------------- | -------------- |\n|3 + 500| = 503|\n|50 + 300| = 350|\n|30 + 500| = 530|\n|5 + 300| = 305|\n\n\n</div>\n<br>\nTherefore, option 3 gives the largest total"}]},{"vars":[{"varval":"Which of these gives the smallest total?"},{"varval":"sm_nogap Checking options:\n\n<div class=\"aligned\">\n\n>>| | |\n| --------------------- | -------------- |\n|3 + 500| = 503|\n|50 + 300| = 350|\n|30 + 500| = 530|\n|5 + 300| = 305|\n\n\n</div>\n<br>\nTherefore, option 4 gives the smallest total"}]},{"vars":[{"varval":"Which one of these gives the largest total?"},{"varval":"sm_nogap Checking options:\n\n<div class=\"aligned\">\n\n>>| | |\n| --------------------- | -------------- |\n|9 + 800| = 809|\n|8 + 900| = 908|\n|80 + 900| = 980|\n|90 + 800| = 890|\n\n\n</div>\n<br>\nTherefore, option 3 gives the largest total"}]},{"vars":[{"varval":"Which of these gives the smallest total?"},{"varval":"sm_nogap Checking options:\n\n<div class=\"aligned\">\n\n>>| | |\n| --------------------- | -------------- |\n|9 + 800| = 809|\n|80 + 900| = 980|\n|8 + 900| = 908|\n|90 + 800| = 890|\n\n\n</div>\n<br>\nTherefore, option 1 gives the smallest total."}]}]

  84. SJ add/subtract

    <div class="sm_mode"> {{{question}}} </div>

    [{"vars":[{"varval":"Bridget has a part time job. \n\nIn February she was paid $65 and $74.\n\nHow much, in dollars, did Bridget earn in February?\n"},{"varval":"sm_nogap One strategy:\n\n<div class=\"aligned\">\n\n| | |\n| --------------------- | -------------- |\n| $65 + $74| = $60 + $5 + $70 + $4|\n|| = $130 + $5 + $4|\n| | \\= {{{correctAnswer}}} |\n\n</div>\n"}]},{"vars":[{"varval":"Nyal is saving for a holiday. \n\nSo far he has saved $84 from mowing lawns and $66 from his birthday money.\n\nHow much, in dollars, has Nyal saved altogether?"},{"varval":"sm_nogap One strategy:\n\n<div class=\"aligned\">\n\n| | |\n| --------------------- | -------------- |\n| $84 + $66| = $80 + $4 + $60 + $6|\n|| = $140 + $4 + $6|\n| | \\= {{{correctAnswer}}} |\n\n</div>"}]},{"vars":[{"varval":"Kyra and Kelly both have part time jobs.\n\nLast week Kyra was paid $39 and Kelly was paid $48.\n\nHow much have they been paid, in dollars, in the last week?"},{"varval":"sm_nogap One strategy:\n\n<div class=\"aligned\">\n\n| | |\n| --------------------- | -------------- |\n| $39 + $48| = $30 + $9 + $40 + $8|\n|| = $70 + $9 + $8|\n| | \\= {{{correctAnswer}}} |\n\n</div>"}]},{"vars":[{"varval":"James and Vinnie both collect marbles.\n\nJames has 37 and Vinnie has 55.\n\nHow many marbles do they have altogether?"},{"varval":"sm_nogap One strategy:\n\n<div class=\"aligned\">\n\n| | |\n| --------------------- | -------------- |\n| 37 + 55| = 30 + 7 + 50 + 5|\n|| = 80 + 7 + 5|\n| | \\= {{{correctAnswer}}} |\n\n</div>"}]},{"vars":[{"varval":"Jimmy and May are collecting eggs at their uncle's chicken farm.\n\nJimmy has collected 18 eggs and May has collected 24.\n\nHow many eggs have they collected altogether?"},{"varval":"sm_nogap One strategy:\n\n<div class=\"aligned\">\n\n| | |\n| --------------------- | -------------- |\n| 18 + 24| = 10 + 8 + 20 + 4|\n|| = 30 + 8 + 4|\n| | \\= {{{correctAnswer}}} |\n\n</div>"}]},{"vars":[{"varval":"On Monday the school canteen had 39 bananas and 25 oranges.\n\nHow many pieces of fruit did the canteen have altogether? "},{"varval":"sm_nogap One strategy:\n\n<div class=\"aligned\">\n\n| | |\n| --------------------- | -------------- |\n| 39 + 25| = 30 + 9 + 20 + 5|\n|| = 50 + 9 + 5|\n| | \\= {{{correctAnswer}}} |\n\n</div>"}]}]

  85. SJ_Ratio/Rates

    <div class="sm_mode"> {{{question}}} </div>

    [{"vars":[{"varval":"Tina lives 2 kilometres from her school.\n\nShe walks to school at a constant speed of 4 kilometres per hour.\n\nHow many minutes does it take for Tina to walk to school?"},{"varval":"<div class=\"aligned\">\n\n| | |\n| -------------- |-------|\n| 4 kilometres/hour|\\= 4 kilometres/60 minutes |\n| |\\= 2 kilometres/{{{correctAnswer}}} minutes |\n\n</div>\n<br>\nTherefore it takes Tina {{{correctAnswer}}} minutes to walk to school."}]},{"vars":[{"varval":"Jake lives 2 kilometres from his work.\n\nHe walks to work at a constant speed of 6 kilometres per hour.\n\nHow many minutes will it take for Jake to walk to work?"},{"varval":"<div class=\"aligned\">\n\n| | |\n| -------------- |-------|\n| 6 kilometres/hour|\\= 6 kilometres/60 minutes |\n| |\\= 2 kilometres/{{{correctAnswer}}} minutes |\n\n</div>\n<br>\nTherefore it takes Jake {{{correctAnswer}}} minutes to walk to work."}]},{"vars":[{"varval":"Dan lives 5 kilometres from the football stadium.\n\nHe runs to the stadium at a constant speed of 10 kilometres per hour.\n\nHow many minutes will it take for Dan to run to the football stadium?"},{"varval":"<div class=\"aligned\">\n\n| | |\n| -------------- |-------|\n| 10 kilometres/hour|\\= 10 kilometres/60 minutes |\n| |\\= 5 kilometres/{{{correctAnswer}}} minutes |\n\n</div>\n<br>\nTherefore it takes Dan {{{correctAnswer}}} minutes to run to the football stadium."}]},{"vars":[{"varval":"Pixie lives 12 kilometres from the tram station.\n\nShe walks to the station at a constant speed of 8 kilometres per hour.\n\nHow many minutes will it take for Pixie to walk to the station?"},{"varval":"<div class=\"aligned\">\n\n| | |\n| -------------- |-------|\n| 8 kilometres/hour|\\= 8 kilometres/60 minutes |\n||\\= 4 kilometres/30 minutes |\n| |\\= 12 kilometres/{{{correctAnswer}}} minutes |\n\n</div>\n<br>\nTherefore it takes Pixie {{{correctAnswer}}} minutes to walk to the station."}]},{"vars":[{"varval":"Barak lives 50 kilometres from his work.\n\nHe drives to work at a constant speed of 75 kilometres per hour.\n\nHow many minutes will it take for Barak to drive to work?"},{"varval":"<div class=\"aligned\">\n\n| | |\n| -------------- |-------|\n| 75 kilometres/hour|\\= 75 kilometres/60 minutes |\n||\\= 25 kilometres/20 minutes |\n| |\\= 50 kilometres/{{{correctAnswer}}} minutes |\n\n</div>\n<br>\nTherefore it takes Barak {{{correctAnswer}}} minutes to drive to work."}]},{"vars":[{"varval":"Michelle lives 60 kilometres from her work.\n\nShe drives to work at a constant speed of 80 kilometres per hour.\n\nHow many minutes will it take for Michelle to drive to work?"},{"varval":"<div class=\"aligned\">\n\n| | |\n| -------------- |-------|\n| 80 kilometres/hour|\\= 80 kilometres/60 minutes |\n||\\= 40 kilometres/30 minutes |\n||\\= 20 kilometres/15 minutes |\n| |\\= 60 kilometres/{{{correctAnswer}}} minutes |\n\n</div>\n<br>\nTherefore it takes Michelle {{{correctAnswer}}} minutes to drive to work."}]}]

  86. SJ_Ratio/Rates

    <div class="sm_mode"> {{{question}}} </div>

    [{"vars":[{"varval":"Esther can run 5 kilometres in 20 minutes.\n\nRunning at the same speed, how long will it take Esther to run 3 kilometres?"},{"varval":"<div class=\"aligned\">\n\n| | |\n| --------------------: | -------------- |\n| Minutes per kilometre | = $\\dfrac{20}{5}$ |\n| | = 4 |\n\n</div>\n\n<br>\n\n<div class=\"aligned\">\n\n| | |\n| --------------------: | -------------- |\n| Minutes for 3 kilometres | = 3 $\\times$ 4 |\n| | = {{{correctAnswer}}} minutes|\n\n</div>"}]},{"vars":[{"varval":"Emilio can run 8 kilometres in 40 minutes.\n\nRunning at the same speed, how long will it take Emilio to run 5 kilometres?"},{"varval":"<div class=\"aligned\">\n\n| | |\n| --------------------: | -------------- |\n| Minutes per kilometre | = $\\dfrac{40}{8}$ |\n| | = 5 |\n\n</div>\n\n<br>\n\n<div class=\"aligned\">\n\n| | |\n| --------------------: | -------------- |\n| Minutes for 5 kilometres | = 5 $\\times$ 5 |\n| | = {{{correctAnswer}}} minutes|\n\n</div>"}]},{"vars":[{"varval":"Izzy can run 3 kilometres in 18 minutes.\n\nRunning at the same speed, how long will it take Izzy to run 7 kilometres?"},{"varval":"<div class=\"aligned\">\n\n| | |\n| --------------------: | -------------- |\n| Minutes per kilometre | = $\\dfrac{18}{3}$ |\n| | = 6 |\n\n</div>\n\n<br>\n\n<div class=\"aligned\">\n\n| | |\n| --------------------: | -------------- |\n| Minutes for 7 kilometres | = 7 $\\times$ 6 |\n| | = {{{correctAnswer}}} minutes|\n\n</div>"}]},{"vars":[{"varval":"Michael can ride 25 kilometres in 50 minutes.\n\nRiding at the same speed, how long will it take Michael to ride 13 kilometres?"},{"varval":"<div class=\"aligned\">\n\n| | |\n| --------------------: | -------------- |\n| Minutes per kilometre | = $\\dfrac{50}{25}$ |\n| | = 2 |\n\n</div>\n\n<br>\n\n<div class=\"aligned\">\n\n| | |\n| --------------------: | -------------- |\n| Minutes for 13 kilometres | = 13 $\\times$ 2 |\n| | = {{{correctAnswer}}} minutes|\n\n</div>"}]},{"vars":[{"varval":"Courtney can walk 5 kilometres in 60 minutes.\n\nWalking at the same speed, how long will it take Courtney to walk 6 kilometres?"},{"varval":"<div class=\"aligned\">\n\n| | |\n| --------------------: | -------------- |\n| Minutes per kilometre | = $\\dfrac{60}{5}$ |\n| | = 12 |\n\n</div>\n\n<br>\n\n<div class=\"aligned\">\n\n| | |\n| --------------------: | -------------- |\n| Minutes for 6 kilometres | = 6 $\\times$ 12 |\n| | = {{{correctAnswer}}} minutes|\n\n</div>"}]},{"vars":[{"varval":"Bron can walk 8 kilometres in 72 minutes.\n\nWalking at the same speed, how long will it take Bron to walk 5 kilometres?"},{"varval":"<div class=\"aligned\">\n\n| | |\n| --------------------: | -------------- |\n| Minutes per kilometre | = $\\dfrac{72}{8}$ |\n| | = 9 |\n\n</div>\n\n<br>\n\n<div class=\"aligned\">\n\n| | |\n| --------------------: | -------------- |\n| Minutes for 5 kilometres | = 5 $\\times$ 9 |\n| | = {{{correctAnswer}}} minutes|\n\n</div>"}]}]

  87. SJ_Ratio/Rates

    <div class="sm_mode"> {{{question}}} </div>

    [{"vars":[{"varval":"Freya makes $96 selling 12 cupcakes at the market.\n\nAll her cupcakes are the same price.\n\nHow much money will she make selling 20 cupcakes?"},{"varval":"<div class=\"aligned\">\n\n| | |\n| -------|------- |\n|Cost per cupcake| = 96 $\\div$ 12 |\n| |= $8 |\n\n</div>\n\n<br>\n\n<div class=\"aligned\">\n\n| | |\n| -------|------- |\n|Cost for 20 cupcakes| = 20 $\\times$ 8 |\n| |= {{{correctAnswer}}} |\n\n</div>\n"}]},{"vars":[{"varval":"Bjork makes $1000 selling 10 scarves at the market.\n\nAll her scarves are the same price.\n\nHow much money will she make selling 13 scarves?"},{"varval":"<div class=\"aligned\">\n\n| | |\n| -------|------- |\n|Cost per scarf| = 1000 $\\div$ 10 |\n| |= $100 |\n\n</div>\n\n<br>\n\n<div class=\"aligned\">\n\n| | |\n| -------|------- |\n|Cost for 13 scarves| = 13 $\\times$ 100 |\n| |= {{{correctAnswer}}} |\n\n</div>\n\n\n"}]},{"vars":[{"varval":"Byron makes $180 selling 20 poetry books on eBay.\n\nAll his poetry books are the same price.\n\nHow much money will he make selling 11 poetry books?"},{"varval":"<div class=\"aligned\">\n\n| | |\n| -------|------- |\n|Cost per poetry book| = 180 $\\div$ 20 |\n| |= $9 |\n\n</div>\n\n<br>\n\n<div class=\"aligned\">\n\n| | |\n| -------|------- |\n|Cost for 11 poetry books| = 11 $\\times$ 9 |\n| |= {{{correctAnswer}}} |\n\n</div>\n\n\n"}]},{"vars":[{"varval":"Bliss makes $2000 on 10 hair treatments.\n\nAll her hair treatments are the same price.\n\nHow much money will she make for 8 hair treatments?"},{"varval":"<div class=\"aligned\">\n\n| | |\n| -------|------- |\n|Cost per hair treatment| = 2000 $\\div$ 10 |\n| |= $200 |\n\n</div>\n\n<br>\n\n<div class=\"aligned\">\n\n| | |\n| -------|------- |\n|Cost for 8 hair treatments| = 8 $\\times$ 200 |\n| |= {{{correctAnswer}}} |\n\n</div>\n\n\n"}]},{"vars":[{"varval":"Jack makes $250 000 building 2 granny flats.\n\nAll his granny flats are the same price.\n\nHow much money will he make selling 3 granny flats?"},{"varval":"<div class=\"aligned\">\n\n| | |\n| -------|------- |\n|Cost per granny flat| = 250 000 $\\div$ 2 |\n| |= $125 000 |\n\n</div>\n\n<br>\n\n<div class=\"aligned\">\n\n| | |\n| -------|------- |\n|Cost for 3 granny flats| = 3 $\\times$ 125 000 |\n| |= {{{correctAnswer}}} |\n\n</div>\n\n\n"}]},{"vars":[{"varval":"Bindy makes $8000 selling 200 tickets to her nature park.\n\nAll the tickets are the same price.\n\nHow much money will she make selling 8 tickets?"},{"varval":"<div class=\"aligned\">\n\n| | |\n| -------|------- |\n|Cost per ticket| = 8000 $\\div$ 200 |\n| |= $40 |\n\n</div>\n\n<br>\n\n<div class=\"aligned\">\n\n| | |\n| -------|------- |\n|Cost for 8 tickets| = 8 $\\times$ 40 |\n| |= {{{correctAnswer}}} |\n\n</div>\n\n\n"}]}]

  88. SJ_Ratio/Rates

    <div class="sm_mode"> {{{question}}} </div>

    [{"vars":[{"varval":"Geoff saves $40 each week.\n\nHow long will it take Geoff to save $1600?"},{"varval":"<div class=\"aligned\"> \n\n| | |\n| -----: | -------------------- |\n| Weeks | \\= $\\dfrac{1600}{40}$ |\n| | \\= {{{correctAnswer}}} |\n\n</div>"}]},{"vars":[{"varval":"Anthony saves $300 each month.\n\nHow long will it take him to save $1500?"},{"varval":"<div class=\"aligned\"> \n\n| | |\n| -----: | -------------------- |\n| Months | \\= $\\dfrac{1500}{300}$ |\n| | \\= {{{correctAnswer}}} |\n\n</div>"}]},{"vars":[{"varval":"Joyce saves $25 each week.\n\nHow long will it take for her to save $500?"},{"varval":"<div class=\"aligned\"> \n\n| | |\n| -----: | -------------------- |\n| Weeks | \\= $\\dfrac{500}{25}$ |\n| | \\= {{{correctAnswer}}} |\n\n</div>"}]},{"vars":[{"varval":"Vinh saves $80 per month.\n\nHow many months will it take him to save $560?"},{"varval":"<div class=\"aligned\"> \n\n| | |\n| -----: | -------------------- |\n| Months | \\= $\\dfrac{560}{80}$ |\n| | \\= {{{correctAnswer}}} |\n\n</div>"}]},{"vars":[{"varval":"Vito saves $2.50 per week.\n\nHow long will it take him to save $50?"},{"varval":"<div class=\"aligned\"> \n\n| | |\n| -----: | -------------------- |\n| Weeks | \\= $\\dfrac{50}{2.50}$ |\n| | \\= {{{correctAnswer}}} |\n\n</div>"}]},{"vars":[{"varval":"Chelsea works in a coffee shop and receives tips from customers.\n\nEach day she receives $6 in tips.\n\nHow many days will it take her to have $54 in tips?"},{"varval":"<div class=\"aligned\"> \n\n| | |\n| -----: | -------------------- |\n| Days | \\= $\\dfrac{54}{6}$ |\n| | \\= {{{correctAnswer}}} |\n\n</div>"}]}]

  89. Created new. Q 30 Non Calc 2010

    <div class="sm_mode"> {{{question}}} </div>

    [{"vars":[{"varval":"Barney was picking fruit from his citrus trees.\n\nExactly 40% of the fruit he picked were lemons.\n\nWhat is the smallest number of lemons that he could have picked?"},{"varval":"<div class=\"aligned\">\n\n| | |\n| ------------------------------ | ------------------------------------- |\n| Fraction of lemons | \\= $\\dfrac{40}{100}$ |\n| | \\= $\\dfrac{2}{5}$ |\n\n</div>\n<br>\n<div class=\"aligned\">\n\n$\\Rightarrow$ Smallest number of citrus picked \\= 5\n\n$\\therefore$ Smallest number of lemons picked \\= {{{correctAnswer0}}}\n\n</div>"}]},{"vars":[{"varval":"Po was slicing tomatoes in her restaurant.\n\nExactly 55% of the tomatoes have been sliced.\n\nWhat is the smallest number of tomatoes that she could have already sliced?"},{"varval":"<div class=\"aligned\">\n\n| | |\n| ------------------------------ | ------------------------------------- |\n|Fraction of tomatoes sliced | \\= $\\dfrac{55}{100}$ |\n| | \\= $\\dfrac{11}{20}$ |\n\n</div>\n<br>\n<div class=\"aligned\">\n\n$\\Rightarrow$ Smallest number of tomatoes to be sliced \\= 20\n\n$\\therefore$ Smallest number of tomatoes already sliced \\= {{{correctAnswer0}}}\n\n</div>"}]},{"vars":[{"varval":"Jessie was laying floor tiles in his bathroom.\n\nExactly 46% of the tiles have been laid.\n\nWhat is the smallest number of tiles that he could have already laid?"},{"varval":"<div class=\"aligned\">\n\n| | |\n| ------------------------------ | ------------------------------------- |\n| Fraction of tiles | \\= $\\dfrac{46}{100}$ |\n| | \\= $\\dfrac{23}{50}$ |\n\n</div>\n<br>\n<div class=\"aligned\">\n\n$\\Rightarrow$ Smallest number of tiles to be laid \\= 50\n\n$\\therefore$ Smallest number of tiles already laid \\= {{{correctAnswer0}}}\n\n</div>"}]},{"vars":[{"varval":"Joyce was collecting shells on the beach.\n\nExactly 18% of the shells were pippies.\n\nWhat is the smallest number of pippies that she could have collected?"},{"varval":"<div class=\"aligned\">\n\n| | |\n| ------------------------------ | ------------------------------------- |\n| Fraction of shells| \\= $\\dfrac{18}{100}$ |\n| | \\= $\\dfrac{9}{50}$ |\n\n</div>\n<br>\n<div class=\"aligned\">\n\n$\\Rightarrow$ Smallest number of shells collected \\= 50\n\n$\\therefore$ Smallest number of pippies collected \\= {{{correctAnswer0}}}\n\n</div>"}]},{"vars":[{"varval":"Ali was scuba diving on the Barrier Reef.\n\nExactly 45% of the sharks he sighted were reef sharks.\n\nWhat is the smallest number of reef sharks that he could have sighted?"},{"varval":"<div class=\"aligned\">\n\n| | |\n| ------------------------------ | ------------------------------------- |\n| Fraction of sharks| \\= $\\dfrac{45}{100}$ |\n| | \\= $\\dfrac{9}{20}$ |\n\n</div>\n<br>\n<div class=\"aligned\">\n\n$\\Rightarrow$ Smallest number of sharks sighted \\= 20\n\n$\\therefore$ Smallest number of reef sharks sighted \\= {{{correctAnswer0}}}\n\n</div>"}]},{"vars":[{"varval":"Aubrey recorded the whales he sighted in Hervey Bay during the month of June.\n\nExactly 72% of the whales he sighted were humpbacks.\n\nWhat is the smallest number of humpbacks that he could have sighted?"},{"varval":"<div class=\"aligned\">\n\n| | |\n| ------------------------------ | ------------------------------------- |\n| Fraction of whales| \\= $\\dfrac{72}{100}$ |\n| | \\= $\\dfrac{18}{25}$ |\n\n</div>\n<br>\n<div class=\"aligned\">\n\n$\\Rightarrow$ Smallest number of whales sighted \\= 25\n\n$\\therefore$ Smallest number of humpbacks sighted \\= {{{correctAnswer0}}}\n\n</div>"}]}]

  90. Needs correct Title

    <div class="sm_mode"> {{{question}}} </div>

    [{"vars":[{"varval":"A farmer sells a box of oranges to a supermarket with a 15% markup.\n\n\r\n\r\nThe supermarket then adds a further 20% on the already increased price.\n\n\r\n\r\nWhat is the overall percentage markup on the box of oranges?\n"},{"varval":"Strategy 1\n\n\r\nAssume the box of oranges costs $1.00 (say)\n\n\r\nCost after 15% markup = $1.15\n\n\r\nsm_nogap Cost after further 20% increase\n\n<div class=\"aligned\">\n\n>>||\n|-|\n|= 1.15 + (20% × 1.15)|\n|= 1.15 + 0.23|\n|= $1.38|\n\r\n\r\n</div>\n\n<br>\n\n<div class=\"aligned\">\n\n| | |\n| --------------------- | -------------- |\n| $\\therefore$ Overall Markup | \\= (1.38 − 1.00) × 100 |\n| | \\= {{{correctAnswer}}} |\n\n</div>\n\n<br>\n\r\n\r\r\n\r\n\nsm_nogap Strategy 2 (advanced)\n\n<div class=\"aligned\">\n\n| | |\n| --------------------- | -------------- |\n| Overall markup | \\= (1 × 1.15 × 1.20) − 1 |\n| | \\= 1.38 − 1.00 |\n|| \\= 0.38|\n|| \\= {{{correctAnswer}}}|\n\n</div>"}]},{"vars":[{"varval":"An unrestored Kombi van sells at auction with a 30% markup.\n\n\r\n\r\nThe Kombi is then restored and sold for 60% above the already increased price.\n\n\r\n\r\nWhat is the overall percentage markup on the Kombi van?\n"},{"varval":"Strategy 1\n\n\r\nAssume the Kombi van costs $1.00 (say)\n\n\r\nCost after 30% markup = $1.30\n\n\r\nsm_nogap Cost after further 60% increase\n\n<div class=\"aligned\">\n\n>>||\n|-|\n|= 1.30 + (60% × 1.30)|\n|= 1.30 + 0.78|\n|= $2.08|\n\r\n\r\n</div>\n\n<br>\n\n<div class=\"aligned\">\n\n| | |\n| --------------------- | -------------- |\n| $\\therefore$ Overall Markup | \\= (2.08 − 1.00) × 100 |\n| | \\= {{{correctAnswer}}} |\n\n</div>\n\n<br>\n\r\n\r\r\n\r\n\nsm_nogap Strategy 2 (advanced)\n\n<div class=\"aligned\">\n\n| | |\n| --------------------- | -------------- |\n| Overall markup | \\= (1 × 1.30 × 1.60) − 1 |\n| | \\= 2.08 − 1.00 |\n|| \\= 1.08|\n|| \\= {{{correctAnswer}}}|\n\n</div>"}]},{"vars":[{"varval":"A famous painting increased in value in ten years by 25%.\n\n\r\n\r\nThe painting is estimated to increase a further 40% in the next 10 years.\n\n\r\n\r\nWhat is the overall percentage increase in value of the painting after 20 years?\n"},{"varval":"Strategy 1\n\n\r\nAssume the painting is valued at $1.00 (say)\n\n\r\nCost after 25% increase = $1.25\n\n\r\nsm_nogap Value after further 40% increase\n\n<div class=\"aligned\">\n\n>>||\n|-|\n|= 1.25 + (40% × 1.25)|\n|= 1.25 + 0.50|\n|= $1.75|\n\r\n\r\n</div>\n\n<br>\n\n<div class=\"aligned\">\n\n| | |\n| --------------------- | -------------- |\n| $\\therefore$ Overall increase | \\= (1.75 − 1.00) × 100 |\n| | \\= {{{correctAnswer}}} |\n\n</div>\n\n<br>\n\r\n\r\r\n\r\n\nsm_nogap Strategy 2 (advanced)\n\n<div class=\"aligned\">\n\n| | |\n| --------------------- | -------------- |\n| Overall increase | \\= (1 × 1.25 × 1.40) − 1 |\n| | \\= 1.75 − 1.00 |\n|| \\= 0.75|\n|| \\= {{{correctAnswer}}}|\n\n</div>"}]},{"vars":[{"varval":"A laptop is sold by the wholesaler with a 15% markup.\n\n\r\n\r\nThe laptop is then sold by the retailer for 40% above the already increased price.\n\n\r\n\r\nWhat is the overall percentage markup on the laptop?\n"},{"varval":"Strategy 1\n\n\r\nAssume the laptop costs $1.00 (say)\n\n\r\nCost after 15% markup = $1.15\n\n\r\nsm_nogap Cost after further 40% increase\n\n<div class=\"aligned\">\n\n>>||\n|-|\n|= 1.15 + (40% × 1.15)|\n|= 1.15 + 0.46|\n|= $1.61|\n\r\n\r\n</div>\n\n<br>\n\n<div class=\"aligned\">\n\n| | |\n| --------------------- | -------------- |\n| $\\therefore$ Overall Markup | \\= (1.61 − 1.00) × 100 |\n| | \\= {{{correctAnswer}}} |\n\n</div>\n\n<br>\n\r\n\r\r\n\r\n\nsm_nogap Strategy 2 (advanced)\n\n<div class=\"aligned\">\n\n| | |\n| --------------------- | -------------- |\n| Overall markup | \\= (1 × 1.15 × 1.40) − 1 |\n| | \\= 1.61 − 1.00 |\n|| \\= 0.61|\n|| \\= {{{correctAnswer}}}|\n\n</div>"}]},{"vars":[{"varval":"Courtney's salary increased by 5% last year.\n\n\r\n\r\nHer salary is set to increase by a further 6% this year.\n\n\r\n\r\nWhat is the overall percentage increase in Courtney's salary over the two years?\n"},{"varval":"Strategy 1\n\n\r\nAssume Courtney's salary is $1.00 (say)\n\n\r\nSalary after 5% increase = $1.05\n\n\r\nsm_nogap Cost after further 5% increase\n\n<div class=\"aligned\">\n\n>>||\n|-|\n|= 1.05 + (5% × 1.05)|\n|= 1.05 + 0.0525|\n|= $1.1025|\n\r\n\r\n</div>\n\n<br>\n\n<div class=\"aligned\">\n\n| | |\n| --------------------- | -------------- |\n| $\\therefore$ Overall Markup | \\= (1.1025− 1.00) × 100 |\n| | \\= {{{correctAnswer}}} |\n\n</div>\n\n<br>\n\r\n\r\r\n\r\n\nsm_nogap Strategy 2 (advanced)\n\n<div class=\"aligned\">\n\n| | |\n| --------------------- | -------------- |\n| Overall markup | \\= (1 × 1.05 × 1.05) − 1 |\n| | \\= 1.1025 − 1.00 |\n|| \\= 0.1025|\n|| \\= {{{correctAnswer}}}|\n\n</div>"}]},{"vars":[{"varval":"The price of petrol increased by 22% last month.\n\n\r\n\r\nThe price increase by a further 8% this month.\n\n\r\n\r\nWhat is the overall percentage increase in the price of petrol over the two months?\n"},{"varval":"Strategy 1\n\n\r\nAssume The original price of petrol is $1.00 (say)\n\n\r\nSalary after 22% increase = $1.22\n\n\r\nsm_nogap Cost after further 8% increase\n\n<div class=\"aligned\">\n\n>>||\n|-|\n|= 1.22 + (8% × 1.22)|\n|= 1.22 + 0.0976|\n|= $1.3176|\n\r\n\r\n</div>\n\n<br>\n\n<div class=\"aligned\">\n\n| | |\n| --------------------- | -------------- |\n| $\\therefore$ Overall Markup | \\= (1.3176− 1.00) × 100 |\n| | \\= {{{correctAnswer}}} |\n\n</div>\n\n<br>\n\r\n\r\r\n\r\n\nsm_nogap Strategy 2 (advanced)\n\n<div class=\"aligned\">\n\n| | |\n| --------------------- | -------------- |\n| Overall markup | \\= (1 × 1.22 × 1.08) − 1 |\n| | \\= 1.3176 − 1.00 |\n|| \\= 0.3176|\n|| \\= {{{correctAnswer}}}|\n\n</div>"}]}]

  91. Needs a correct Title

    <div class="sm_mode"> {{{question}}} </div>

    [{"vars":[{"varval":"A car dealer decreases the price of a car from $50 000 to $45 000.\n\n\r\n\r\nWhat is the percentage decrease in the price?\n"},{"varval":"sm_nogap Price decrease = 50 000 $−$ 45 000 = $5 000\n\n<div class=\"aligned\">\n\n| | |\n| --------------------- | -------------- |\n| $\\therefore$ Percentage decrease | \\= $\\dfrac{5000}{50000}$ × 100 |\n| | \\= 0.10 × 100 |\n|| \\= {{{correctAnswer0}}}{{{suffix0}}}|\n\n</div>\n"}]},{"vars":[{"varval":"A department store advertises a sale. John purchases a suit for $300 that was originally priced at $500.\n\n\r\n\r\nWhat is the percentage decrease in the price?\n"},{"varval":"sm_nogap Price decrease = 500 $−$ 300 = $200\n\n<div class=\"aligned\">\n\n| | |\n| --------------------- | -------------- |\n| $\\therefore$ Percentage decrease | \\= $\\dfrac{200}{500}$ × 100 |\n| | \\= 0.40 × 100 |\n|| \\= {{{correctAnswer0}}}{{{suffix0}}}|\n\n</div>\n"}]},{"vars":[{"varval":"A lounge suite decreases in price from $3300 to $2640.\n\n\r\n\r\nWhat is the percentage decrease in the price?\n"},{"varval":"sm_nogap Price decrease = 3300 $−$ 2640 = $660\n\n<div class=\"aligned\">\n\n| | |\n| --------------------- | -------------- |\n| $\\therefore$ Percentage decrease | \\= $\\dfrac{660}{3300}$ × 100 |\n| | \\= 0.20 × 100 |\n|| \\= {{{correctAnswer0}}}{{{suffix0}}}|\n\n</div>\n"}]},{"vars":[{"varval":"A supermarket decreases the price of a kilogram of gala apples from $4.50 to $3.60.\n\n\r\n\r\nWhat is the percentage decrease in the price?\n"},{"varval":"sm_nogap Price decrease = 4.50 $−$ 3.60 = $0.90\n\n<div class=\"aligned\">\n\n| | |\n| --------------------- | -------------- |\n| $\\therefore$ Percentage decrease | \\= $\\dfrac{0.9}{4.50}$ × 100 |\n| | \\= 0.20 × 100 |\n|| \\= {{{correctAnswer0}}}{{{suffix0}}}|\n\n</div>\n"}]},{"vars":[{"varval":"A family holiday to Disneyworld in Florida is reduced in price from $4100 to $3895.\n\n\r\n\r\nWhat is the percentage decrease in the price?"},{"varval":"sm_nogap Price decrease = 4100 $−$ 3895 = $205\n\n<div class=\"aligned\">\n\n| | |\n| --------------------- | -------------- |\n| $\\therefore$ Percentage decrease | \\= $\\dfrac{205}{4100}$ × 100 |\n| | \\= 0.05 × 100 |\n|| \\= {{{correctAnswer0}}}{{{suffix0}}}|\n\n</div>\n"}]},{"vars":[{"varval":"A family ticket to Taronga Zoo in Sydney reduced in price from $90 to $73.80.\n\nWhat is the percentage decrease in the price?"},{"varval":"sm_nogap Price decrease = 90 $−$ 73.80 = $16.20\n\n<div class=\"aligned\">\n\n| | |\n| --------------------- | -------------- |\n| $\\therefore$ Percentage decrease | \\= $\\dfrac{16.20}{90}$ × 100 |\n| | \\= 0.18 × 100 |\n|| \\= {{{correctAnswer0}}}{{{suffix0}}}|\n\n</div>"}]}]

  92. A version of easier questions on 30144. Without the variable focus.

    <div class="sm_mode"> {{{question}}} </div>

    [{"vars":[{"varval":"Brandon purchased one Lacoste Big Croc Tee and one Fila PLR Polo in a sale.\n\nsm_img https://teacher.smartermaths.com.au/wp-content/uploads/2020/07/Q35_var1-r.png 900 indent vpad\n\nHow much does Brandon save, in total, off the normal price of these two items?"},{"varval":"<div class=\"sm_mode\">\n\n<div class=\"aligned\">\n\n| | |\n| --------------------: | -------------- |\n| Savings on Big Croc Tee | = 70 $-$ 49 |\n| | = $21 |\n\n</div>\n\n<br>\n\n<div class=\"aligned\">\n\n| | |\n| --------------------: | -------------- |\n| Savings on Polo | = 58 $-$ 43.50 |\n| | = $14.50 |\n\n</div>\n\n<br>\n\n<div class=\"aligned\">\n\n| | |\n| --------------------: | -------------- |\n| $\\therefore$ Total savings | = 21 + 14.50 |\n| | = {{{correctAnswer}}} |\n\n</div>\n\n</div>"}]},{"vars":[{"varval":"Gagadeep purchased one Lacoste Big Croc Tee and one pair of Lacoste Novak Shorts in a sale.\n\nsm_img https://teacher.smartermaths.com.au/wp-content/uploads/2020/07/Q35_var1-r.png 900 indent vpad\n\nHow much does Gagadeep save, in total, off the normal price of these two items?"},{"varval":"<div class=\"sm_mode\">\n\n<div class=\"aligned\">\n\n| | |\n| --------------------: | -------------- |\n| Savings on Big Croc Tee | = 70 $-$ 49 |\n| | = $21 |\n\n</div>\n\n<br>\n\n<div class=\"aligned\">\n\n| | |\n| --------------------: | -------------- |\n| Savings on Shorts | = 80 $-$ 56 |\n| | = $24 |\n\n</div>\n\n<br>\n\n<div class=\"aligned\">\n\n| | |\n| --------------------: | -------------- |\n| $\\therefore$ Total savings | = 21 + 24 |\n| | = {{{correctAnswer}}} |\n\n</div>\n\n</div>"}]},{"vars":[{"varval":"Chablis buys her daughter an Ella Coat and Fur Hooded Parka at a sale.\n\nsm_img https://teacher.smartermaths.com.au/wp-content/uploads/2020/07/Q35_var2-r.png 900 indent vpad\n\nHow much does Chablis save, in total, off the normal price of these two items? "},{"varval":"<div class=\"sm_mode\">\n\n<div class=\"aligned\">\n\n| | |\n| --------------------: | -------------- |\n| Savings on Ella Coat | = 87 $-$ 44 |\n| | = $43 |\n\n</div>\n\n<br>\n\n<div class=\"aligned\">\n\n| | |\n| --------------------: | -------------- |\n| Savings on Parka | = 105 $-$ 55 |\n| | = $50 |\n\n</div>\n\n<br>\n\n<div class=\"aligned\">\n\n| | |\n| --------------------: | -------------- |\n| $\\therefore$ Total savings | = 43 + 50 |\n| | = {{{correctAnswer}}} |\n\n</div>\n\n</div>"}]},{"vars":[{"varval":"Connie buys her daughter a Hooded Gillet and a Crew Neck Sweat at a sale.\n\nsm_img https://teacher.smartermaths.com.au/wp-content/uploads/2020/07/Q35_var2-r.png 900 indent vpad\n\nHow much does Connie save, in total, off the normal price of these two items? "},{"varval":"<div class=\"sm_mode\">\n\n<div class=\"aligned\">\n\n| | |\n| --------------------: | -------------- |\n| Savings on Gillet | = 75 $-$ 39.20 |\n| | = $35.80 |\n\n</div>\n\n<br>\n\n<div class=\"aligned\">\n\n| | |\n| --------------------: | -------------- |\n| Savings on Crew Neck Sweat | = 45 $-$ 23 |\n| | = $22 |\n\n</div>\n\n<br>\n\n<div class=\"aligned\">\n\n| | |\n| --------------------: | -------------- |\n| $\\therefore$ Total savings | = 35.80 + 22 |\n| | = {{{correctAnswer}}} |\n\n</div>\n\n</div>"}]},{"vars":[{"varval":"Rebecca purchased one pair of Otik workboots and one pair of Carina Square Toe Ballet shoes at a sale.\n\nsm_img https://teacher.smartermaths.com.au/wp-content/uploads/2020/07/Q35_var3-r.png 900 indent vpad\n\nApproximately how much does Rebecca save, in total, off the normal price of these two items?"},{"varval":"<div class=\"sm_mode\">\n\n<div class=\"aligned\">\n\n| | |\n| --------------------: | -------------- |\n| Savings on Otik Woorkboots | $\\approx$ 80 $-$ 40 |\n| | $\\approx$ $40 |\n\n</div>\n\n<br>\n\n<div class=\"aligned\">\n\n| | |\n| --------------------: | -------------- |\n| Savings on Ballet shoes | $\\approx$ 20 $-$ 10 |\n| | $\\approx$ $10 |\n\n</div>\n\n<br>\n\n<div class=\"aligned\">\n\n| | |\n| --------------------: | -------------- |\n| $\\therefore$ Total savings | $\\approx$ 40 + 10 |\n| | $\\approx$ {{{correctAnswer}}} |\n\n</div>\n\n</div>"}]},{"vars":[{"varval":"Dale purchased one Asics Travel Polo and one pair of Asics Shorts at a sale.\n\nsm_img https://teacher.smartermaths.com.au/wp-content/uploads/2020/07/Q35_var4-r.png 900 indent vpad\n\nApproximately how much does Dale save, in total, off the normal price of these two items?"},{"varval":"<div class=\"sm_mode\">\n\n<div class=\"aligned\">\n\n| | |\n| --------------------: | -------------- |\n| Savings on Polo | $\\approx$ 85 $-$ 60 |\n| | $\\approx$ $25 |\n\n</div>\n\n<br>\n\n<div class=\"aligned\">\n\n| | |\n| --------------------: | -------------- |\n| Savings on Shorts | = 35 $-$ 22 |\n| | = $13 |\n\n</div>\n\n<br>\n\n<div class=\"aligned\">\n\n| | |\n| --------------------: | -------------- |\n| $\\therefore$ Total savings | $\\approx$ 25 + 13 |\n| | $\\approx$ {{{correctAnswer}}} |\n\n</div>\n\n</div>"}]},{"vars":[{"varval":"Pele bought a pair of Shin Guards and one pair of Adidas Gloves at a sale.\n\nsm_img https://teacher.smartermaths.com.au/wp-content/uploads/2020/07/Q35_var5-r.png 900 indent vpad\n\nHow much did Pele save, in total, off the normal price of these two items?"},{"varval":"<div class=\"sm_mode\">\n\n<div class=\"aligned\">\n\n| | |\n| --------------------: | -------------- |\n| Savings on Shin Guards | = 20 $-$ 14 |\n| | = $6 |\n\n</div>\n\n<br>\n\n<div class=\"aligned\">\n\n| | |\n| --------------------: | -------------- |\n| Savings on Gloves | = 25 $-$ 17.50 |\n| | = $7.50 |\n\n</div>\n\n<br>\n\n<div class=\"aligned\">\n\n| | |\n| --------------------: | -------------- |\n| $\\therefore$ Total savings | = 6 + 7.50 |\n| | = {{{correctAnswer}}} |\n\n</div>\n\n</div>"}]}]

  93. <div class="sm_mode"> {{{question}}} </div>

    [{"vars":[{"varval":"What number is 10 less than 202?"},{"varval":"202 $-$ 10 = {{{correctAnswer}}}"}]},{"vars":[{"varval":"What number is 10 less than 307?"},{"varval":"307 $-$ 10 = {{{correctAnswer}}}"}]},{"vars":[{"varval":"What number is 9 less than 204?"},{"varval":"204 $-$ 9 = {{{correctAnswer}}}"}]},{"vars":[{"varval":"What number is 12 less than 404?"},{"varval":"404 $-$ 12 = {{{correctAnswer}}}"}]}]

  94. Number and Algebra, NAP-54123

    <div class="sm_mode"> {{{question}}} </div>

    [{"vars":[{"varval":"Sully bought KFC for his soccer team to celebrate their grand final victory.\n\nHe bought 12 wings, 16 drumsticks and 7 breast pieces.\n\nIn total, how many pieces of fried chicken did Sully buy?"},{"varval":"sm_nogap One strategy:\n\n<div class=\"aligned\">\n\n| | |\n| --------------------- | -------------- |\n| 12 + 16 + 7| = 10 + 2 + 10 + 6 + 7|\n|| = 20 + 15|\n| | \\= {{{correctAnswer}}} |\n\n</div>"}]},{"vars":[{"varval":"Clarissa was donating cans of food in her pantry to flood victims\n\nShe donated 23 cans of tuna, 9 cans of soup and 16 cans of beans.\n\nIn total, how many cans of food did Clarissa donate?"},{"varval":"sm_nogap One strategy:\n\n<div class=\"aligned\">\n\n| | |\n| --------------------- | -------------- |\n| 23 + 9 + 16| = 20 + 3 + 9 + 10 + 6|\n|| = 30 + 18|\n| | \\= {{{correctAnswer}}} |\n\n</div>"}]},{"vars":[{"varval":"Strop worked at a animal refuge.\n\nDuring the bushfires, Strop rescued 19 koalas, 13 kangaroos and 9 wombats.\n\nIn total, how many animals did Strop rescue during the bushfires?"},{"varval":"sm_nogap One strategy:\n\n<div class=\"aligned\">\n\n| | |\n| --------------------- | -------------- |\n| 19 + 13 + 9| = 10 + 9 + 10 + 3 + 9|\n|| = 20 + 21|\n| | \\= {{{correctAnswer}}} |\n\n</div>"}]},{"vars":[{"varval":"Charlie wanted to start a reptile park in his backyard.\n\nHe caught 13 blue tongue lizards, 25 geckos and 7 goannas to show visitors when the park opened.\n\nIn total, how many reptiles did Charlie have?"},{"varval":"sm_nogap One strategy:\n\n<div class=\"aligned\">\n\n| | |\n| --------------------- | -------------- |\n| 13 + 25 + 7| = 10 + 3 + 20 + 5 + 7|\n|| = 30 + 15|\n| | \\= {{{correctAnswer}}} |\n\n</div>"}]},{"vars":[{"varval":"Jules supplied coloured shirts for his 3 oztag teams.\n\nHe had 13 blue, 11 yellow and 5 green shirts.\n\nIn total, how many coloured shirts did Jules supply?"},{"varval":"sm_nogap One strategy:\n\n<div class=\"aligned\">\n\n| | |\n| --------------------- | -------------- |\n| 13 + 11 + 5| = 10 + 3 + 10 + 1 + 5|\n|| = 20 + 9|\n| | \\= {{{correctAnswer}}} |\n\n</div>"}]},{"vars":[{"varval":"Vicki is cutting up fruit to make fruit salad for the school canteen.\n\nShe has 9 bananas, 31 strawberries and 12 apples.\n\nIn total, how many pieces of fruit is Vicki using in the fruit salad?"},{"varval":"sm_nogap One strategy:\n\n<div class=\"aligned\">\n\n| | |\n| --------------------- | -------------- |\n| 9 + 31 + 12| = 9 + 30 + 1 + 10 + 2|\n|| = 40 + 9 + 1 + 2|\n| | \\= {{{correctAnswer}}} |\n\n</div>"}]}]

  95. <div class="sm_mode"> The default is a light blue background which is color3 for sm-text in a span such as: here <span class="sm-text">?</span> is the inline item. And you can add the class colorN where N=1..8 for more color control such as here <span class="sm-text color1">?</span>, <span class="sm-text color2">?</span>, <span class="sm-text color3">?</span>, <span class="sm-text color4">?</span>, <span class="sm-text color5">?</span>, <span class="sm-text color6">?</span>, <span class="sm-text color7">?</span>, <span class="sm-text color8">?</span> And fyi, any text can be in the span, such as: <span class="sm-text color6">This text!</span> </div>

    [{"vars":null}]

  96. <div class="sm_mode"> # New and improved tables! - Tables are built in markdown, can use 2 asterisks for bold, 2 greater than signs for indenting, and colons in the tables for right/left/center alignment, and should be surrounded by a div with the options below. - Options that can be mixed and matched in tables are: - sm-table (required) - top-left-cell-hidden - row1-color1 (colorN where N is 1..8) - col1-color1 (colorN where N is 1..8) - heading-color1 (only used if need a spanning heading row at top, and colorN where N is 1..8) ## Examples: ### 1. Merged Title Row A <div class="sm-table row1-color1 heading-color1"> >>Time Spent <br> Training >>| Day | Time | |:-:|:-:| | Monday | 3h 15min | | Tuesday | 2h 10min | | Thursday | 1h 5min | </div> <br> ### 1. Merged Title Row B <div class="sm-table col1-color1 heading-color1"> >>Type II Diabetes in USA >>| Year | 2007 | 2008 | 2009 | |:-:|:-:|:-:|:-:| | Millions | 31.16| 32.4 |32.9| </div> <br> ### 2. Color in title row/column <div class="sm-table col1-color1 row1-color1 top-left-cell-hidden"> >>| | Skateboard | No Skate | |:-:|:-:|:-:| | Bike | 55| 2| | No Bike | 7| 20| </div> <br> ### 3. Color in title column <div class="sm-table col1-color1"> >>| Peter | Number of things: 5 | |:-:|:-:| | Andrew | Number of things: 2| | Rex | Number of things: 7| </div> <br> ### 4. Color in title row <div class="sm-table row1-color8"> >>| A title | B title | C title | |:-:|:-:|:-:| | 1 | 2| 3| | 3 | 7| 20| </div> <br> </div>

    [{"vars":null}]

  97. <div class="sm_mode"> {{{question}}} </div>

    [{"vars":[{"varval":"Five cars are parked side by side.\n\nsm_img https://teacher.smartermaths.com.au/wp-content/uploads/2021/10/RAPH16-67.svg 440 indent vpad\n\nWhich car is parked directly on the left of the red car?"},{"varval":"The cars are facing you so their left side is your right side.\n\nTherefore, the {{{correctAnswer}}} is parked directly on the left of the red car. "}]},{"vars":[{"varval":"Three friends are holding some items in their hands.\n\nsm_img https://teacher.smartermaths.com.au/wp-content/uploads/2021/10/RAPH16-68.svg 440 indent vpad\n\nWhich item is being held by Richard in his right hand?"},{"varval":"The children are facing you so their right side is your left side.\n\nTherefore, the {{{correctAnswer}}} is held by Richard in his right hand.\n"}]},{"vars":[{"varval":"Five stores are located along a street.\n\nsm_img https://teacher.smartermaths.com.au/wp-content/uploads/2021/10/RAPH16-69.svg 550 indent vpad\n\nWhich store is located on the left of the coffee shop?"},{"varval":"The stores are facing you so their left side is your right side.\n\nTherefore, the {{{correctAnswer}}} is located on the left of the coffee shop."}]}]

  98. <div class="sm_mode"> {{{question}}} </div>

    [{"vars":[{"varval":"Annie has only these coins.\n\nsm_img https://teacher.smartermaths.com.au/wp-content/uploads/2021/10/RAPH16-64.svg 440 indent vpad\r\n\nShe buys a slice of cake for $1.25 and a cup of tea for 85 cents.\n\n\r\nHow much money does Annie have left?\r\n"},{"varval":"Calculate total money Annie starts with:\n\n>>0.20 + 1.00 + 0.20 + 0.50 + 0.10 + 0.05 + 0.10 + 0.50 = 2.65\r\n\n<br>\n\nCalculate the total cost of cake and tea:\r\n\n>>1.25 + 0.85 = 2.1\r0\n\n<br>\n\nCalculate the money left:\r\n\n>>2.65 – 2.10 = {{{correctAnswer}}}\n"}]},{"vars":[{"varval":"John has only these coins.\n\nsm_img https://teacher.smartermaths.com.au/wp-content/uploads/2021/10/RAPH16-65.svg 440 indent vpad\r\n\nHe buys an ice cream for $2.10 and a wafer for 50 cents.\r\n\nHow much money does John have left?\r"},{"varval":"Calculate the total money of John:\r\n\n>>1.00 + 0.20 + 0.50 + 0.10 + 0.05 + 0.05 + 0.10 + 0.50 + 0.05 + 1.00 + 0.50 = 4.05\n\n<br>\n\nCalculate the total cost of ice cream and wafer:\r\n\n>>2.10 + 0.50 = 2.6\r0\n\n<br>\n\nCalculate the money left:\r\n\n>>4.05 – 2.60 = {{{correctAnswer}}}\n"}]},{"vars":[{"varval":"Trisha has only these coins.\n\nsm_img https://teacher.smartermaths.com.au/wp-content/uploads/2021/10/RAPH16-66.svg 440 indent vpad\n\nShe buys a lollipop for 40 cents and a chocolate bar for $1.10.\r\n\nHow much money does Trisha have left?\r"},{"varval":"Calculate total money Trisha starts with:\n\n>>0.10 + 0.10 + 0.20 + 1.00 + 0.20 + 0.50 + 0.20 + 0.10 + 0.05 + 0.10 = 2.55\r\n\n<br>\n\nCalculate the total cost of lollipop and chocolate bar:\r\n\n>>0.40 + 1.10 = 1.50\n\n<br>\n\nCalculate the money left:\r\n\n>>2.55 – 1.50 = {{{correctAnswer}}}\n"}]},{"vars":[{"varval":"Corey has only these coins.\n\nsm_img https://teacher.smartermaths.com.au/wp-content/uploads/2021/10/RAPH16-64.svg 440 indent vpad\n\nHe buys an orange juice for $1.45 and a cupcake for 95 cents.\n\n\nHow much money does Corey have left?\n"},{"varval":"Calculate total money Corey starts with:\n\n>>0.20 + 1.00 + 0.20 + 0.50 + 0.10 + 0.05 + 0.10 + 0.50 = $2.65\n\n<br>\n\nCalculate the total cost of orange juice and cupcake:\n\n>>1.45 + 0.95 = $2.40\n\n<br>\n\nCalculate the money left:\n\n>>2.65 – 2.40 = {{{correctAnswer}}}\n"}]},{"vars":[{"varval":"Jacinta has only these coins.\n\nsm_img https://teacher.smartermaths.com.au/wp-content/uploads/2021/10/RAPH16-65.svg 440 indent vpad\n\nShe buys an sandwich for $2.80 and a bottle of water for 95 cents.\n\nHow much money does Jacinta have left?\n"},{"varval":"Calculate total money Jacinta starts with:\n\n>>1.00 + 0.20 + 0.50 + 0.10 + 0.05 + 0.05 + 0.10 + 0.50 + 0.05 + 1.00 + 0.50 = 4.05\n\n<br>\n\nCalculate the total cost of the sandwich and bottle of water\n\n>>2.80 + 0.95 = 3.75\n\n<br>\n\nCalculate the money left:\n\n>>4.05 – 3.75 = {{{correctAnswer}}}"}]},{"vars":[{"varval":"Rip has only these coins.\n\nsm_img https://teacher.smartermaths.com.au/wp-content/uploads/2021/10/RAPH16-66.svg 440 indent vpad\n\nHe buys a soft drink for $1.45 and a yoghurt bar for 70 cents.\n\nHow much money does Rip have left?\n"},{"varval":"Calculate total money Rip starts with:\n\n>>0.10 + 0.10 + 0.20 + 1.00 + 0.20 + 0.50 + 0.20 + 0.10 + 0.05 + 0.10 = 2.55\n\n<br>\n\nCalculate the total cost of soft drink and yoghurt bar:\n\n>>1.45 + 0.70 = 2.15\n\n<br>\n\nCalculate the money left:\n\n>>2.55 – 2.15 = {{{correctAnswer}}}\n"}]}]

  99. <div class="sm_mode"> {{{question}}} </div>

    [{"vars":[{"varval":"The table below shows the weights of four cars.\n\n<div class=\"outline\">\n\n> > | Car| A | B | C | D |\n> > | :-----: | :-: | :-: | :-: | :-: |\n> > | Weight| 2345 kg | 2354 kg | 2355 kg | 2344 kg |\n\n</div>\n\n<br>\nWhich is the second lightest car?"},{"varval":"Arrange the weight of the cars from lightest to the heaviest.\r\n\n>>2344 < 2345 < 2354 < 2355\r\n\n<br>\n\n$\\therefore$ Car A is the 2nd lightest car.\r\n"}]},{"vars":[{"varval":"The table below shows the distance of four places from Evelyn’s house.\n\n<div class=\"outline\">\n\n> > | Place | Park | Market | Mall | Hospital |\n> > | :-----: | :-: | :-: | :-: | :-: |\n> > | Distance | 1670 m | 1770 m | 1660 m | 1760 m |\n\n</div>\n\n<br>\nWhich is the second farthest place from Evelyn’s home?"},{"varval":"Arrange the distance of the places from nearest to the farthest.\r\n\n>>1660 < 1670 < 1760 < 1770\r\n\n<br>$\\therefore$ the hospital is the second farthest place from Evelyn’s house.\r\n"}]},{"vars":[{"varval":"The table below shows the money saved by four friends.\n\n<div class=\"outline\">\n\n> > | Name | Henry | Lei | Janice | Dorian |\n> > | :-----: | :-: | :-: | :-: | :-: |\n> > | Savings | $5632 | $5623 | $5633 | $5622 |\n\n</div>\n\n<br>\nWho has the second largest savings?"},{"varval":"Arrange the savings from smallest to largest.\r\n\n>>5622 < 5623 < 5632 < 5633\r\n\n<br>\n\n$\\therefore$ Henry has the second largest savings.\r\n"}]},{"vars":[{"varval":"The table below shows the distances of four children from the finish line in a running race.\n\n<div class=\"outline\">\n\n> > | Child| Anna | Ben | Carrie | Dennis |\n> > | :-----: | :-: | :-: | :-: | :-: |\n> > | Distance| 185 m | 158 m | 174 m | 147 m |\n\n</div>\n\n<br>\nWhich child has the second shortest distance to travel to the finish line?"},{"varval":"Arrange the distances from the finish line from shortest to the longest. \n\nNote the shortest distance is closest to the finish line.\n\n>>147 < 158 < 174 < 185\n\n<br>\n\n$\\therefore$ {{{correctAnswer}}} has the second shortest distance to travel.\n"}]},{"vars":[{"varval":"The table below shows the number of push-ups four gym members have completed in a push-up challenge.\n\n<div class=\"outline\">\n\n> > | Member| Art | Bic | Clint | Dray |\n> > | :-----: | :-: | :-: | :-: | :-: |\n> > | Push-ups| 123 | 321 | 231 | 312 |\n\n</div>\n\n<br>\nWho has done the second largest number of push-ups?"},{"varval":"Arrange the number of push-ups from smallest to largest. \n\n>>123 < 231 < 312 < 321\n\n<br>\n\n$\\therefore$ {{{correctAnswer}}} has done the second largest number of push-ups."}]},{"vars":[{"varval":"The table below shows the total distance 4 friends walked on a four day hike.\n\n<div class=\"outline\">\n\n> > | Name| Ariel | Beryl | Christa | Doula |\n> > | :-----: | :-: | :-: | :-: | :-: |\n> > | Distance| 93 km | 89 km | 98 km | 95 km |\n\n</div>\n\n<br>\nWhich friend walked the second smallest distance?"},{"varval":"Arrange the distances from smallest to largest. \n\n>>89 < 93 < 95 < 98\n\n<br>\n\n$\\therefore$ {{{correctAnswer}}} walked the second smallest distance."}]}]

  100. <div class="sm_mode"> {{{question}}} </div>

    [{"vars":[{"varval":"Arthur creates a square on the Cartesian plane and labels each vertex.\r\n\nWhat are the coordinates of point X?\n\n\r\nsm_img https://teacher.smartermaths.com.au/wp-content/uploads/2021/09/raph13-27.png 300 indent vpad\n\n"},{"varval":"Looking at the graph the point X is located at the intersection of x = –2 and y = –2"}]},{"vars":[{"varval":"Neil drew a circle on the Cartesian plane.\r\n\nWhat are the coordinates of center?\n\n\r\nsm_img https://teacher.smartermaths.com.au/wp-content/uploads/2021/09/raph13-28.png 300 indent vpad"},{"varval":"Looking at the graph, the center is located at the intersection of x = –2 and y = 1."}]}]

  101. <div class="sm_mode"> {{{question}}} </div>

    [{"vars":[{"varval":"Students were asked to fill the questionnaires on what subject was the most difficult.\r\n\nThe graph shows the results.\n\n\r\nsm_img https://teacher.smartermaths.com.au/wp-content/uploads/2021/09/raph13-25.svg 360 indent vpad\n\n<br> What percentage is Physics?"},{"varval":"Analyzing the graph, Math is 50% and English is 25%.\r\n\nThis makes Physics to be less than 25%.\r\n"}]},{"vars":[{"varval":"People were asked what their favorite colour is.\r\n\nThe graph shows the results.\r\n\nsm_img https://teacher.smartermaths.com.au/wp-content/uploads/2021/09/raph13-26.svg 360 indent vpad\n\nWhat percentage is Red?"},{"varval":"Analyzing the graph, yellow is 25%.\r\n\nRed is larger than yellow but not twice as much.\r\n"}]}]

  102. Note: MUST BE REFORMATTED WITH MULTI "correctAnswer:" since current formatting as at 2022-06-02 will fail.

    <div class="sm_mode"> {{{question}}} </div>

    [{"vars":[{"varval":"A survey was conducted which movie genre was the best.\r\n\nThe table shows the result of the survey but two values are missing.\r\nAltogether:\r\n\n* 52 comedy\r\n\n* 36 action\r\n\n* 41 romance\r\n\r\n\nComplete the table by filling in the missing values.\n\n\r\n<div class=\"outline\">\r\n\r\n\n| Movie Genre | Male | Female |\r\n| :----------:- | :--------: | :--: |\r\n| Comedy | 25 | 27 |\r\n| Action | | 14 |\n| Romance | 28 | |\r\n\r\n\n</div>\r\n"},{"varval":"For Action: 36 – 14 = 22\r\n\nFor Romance: 41 – 28 = 13\r\n"}]},{"vars":[{"varval":" A bakery asked for customers’ feedback on which is the best cake flavor.\r\n\nThe table shows the result of the feedback but three values are missing.\r\n\nAltogether:\r\n\r\n* 11 chocolate\r\n* 24 vanilla\r\n* 27 red velvet\r\n* 19 sponge cake\r\n\r\nComplete the table by filling in the missing values.\n\r\n\n<div class=\"outline\">\r\n\r\n\n| Cake Flavor | Male | Female |\r\n| :----------:- | :--------: | :--: |\r\n| Chocolate | 9 | 2 |\r\n| Vanilla | 7 | |\n| Red Velvet | 12 | |\n| Sponge cake | | 11 |\r\n\r\n\n</div>"},{"varval":"For Vanilla: 24 – 7 = 17\r\n\nFor Red Velvet: 27 – 12 = 15\r\n\nFor Sponge Cake: 19 – 11 = 8\r\n"}]}]

  103. <div class="sm_mode"> {{{question}}} </div>

    [{"vars":[{"varval":"There are 36,422 people in Dubbo. \r\n\nEach day a person uses an average of 154 litres of water.\r\n\r\n\n<br>What is the estimate for the total number of litres used by people in Dubbo per day?\r\n"},{"varval":">> 36,422 ≈ 36, 000 \r\n\n>> 154 ≈ 150\r\n\nTherefore, the estimated total number of litres of water used per day is 36000 x 150.\r\n"}]},{"vars":null}]

  104. <div class="sm_mode"> {{{question}}} </div>

    [{"vars":[{"varval":"Fiona has a rectangular area of grass which is 5 metres long and 4 metres wide.\r\n\nShe plants a fence post in one corner\r and then puts posts every 1 metre around the rectangular area.\r\n\nHow many posts does she use altogether?\r\n"},{"varval":"sm_nogap Starting along a 5-metre side and working around the rectangle:\n\n<div class=\"aligned\">\r\n\r\n| | |\r\n| ------------- | ---------- |\r\n| Posts needed | \\= 6 + 4 + 5 + 3 |\r\n| | \\= {{{correctAnswer0}}} {{{suffix0}}} |\r\n\r\n</div>\r\n"}]},{"vars":[{"varval":"Adam has a rectangular backyard which is 7 metres long and 5 metres wide.\r\n\nHe puts a fence post in one corner\r and then puts posts every 1 metre around the entire backyard.\r\n\nHow many posts does he use altogether?\r\n"},{"varval":"sm_nogap Starting along a 7-metre side and working around the yard:\n\n<div class=\"aligned\">\r\n\r\n| | |\r\n| ------------- | ---------- |\r\n| Posts needed | \\= 8 + 5 + 7 + 4 |\r\n| | \\= {{{correctAnswer0}}} {{{suffix0}}} |\r\n\r\n</div>\r\n"}]},{"vars":[{"varval":"Lucas has a rectangular vegetable garden which is 9 metres long and 4 metres wide.\r\n\nHe plants a fence post in one corner\r and then plants posts every 1 metre around the whole vegetable garden.\r\n\nHow many posts does he use altogether?\r\n"},{"varval":"sm_nogap Starting along a 9-metre side and working around the rectangle:\n\n<div class=\"aligned\">\r\n\r\n| | |\r\n| ------------- | ---------- |\r\n| Posts needed | \\= 10 + 4 + 9 + 3 |\r\n| | \\= {{{correctAnswer0}}} {{{suffix0}}} |\r\n\r\n</div>\r\n"}]}]

  105. <div class="sm_mode"> {{{question}}} </div>

    [{"vars":[{"varval":"John and Steve picked up 35 pizzas for a party.\r\n\nSteve took 7 more pizzas back to the party than John.\r\n\nHow many pizzas did John take back to the party?\r\n"},{"varval":"sm_nogap Strategy one:\n\n<div class=\"aligned\">\r\n\r\n| | |\r\n| ------------- | ---------- |\r\n| John's pizzas| \\= (35 – 7) $\\div$ 2 |\n| | \\= 28 $\\div$ 2 |\r\n| | \\= {{{correctAnswer0}}} |\r\n\r\n</div>\r\n\n<br>\n\nStrategy two (by trial and error):\n\nJohn takes less than half &nbsp$\\rightarrow$ &nbsp;try 16\n\nIf John 16, Steve 23 $\\rightarrow$ &nbsp;total = 39 (too high)\n\nIf John 15, Steve 22 $\\rightarrow$ &nbsp;total = 37 (too high)\n\nIf John 14, Steve 21 $\\rightarrow$ &nbsp;total = 35 (correct)\n\n\n"}]},{"vars":[{"varval":"Mark and Ivan went fishing together and caught 26 fish together.\r\n\nMark caught 8 more fish than Ivan.\r\n\nHow many fish did Ivan catch?\r\n"},{"varval":"sm_nogap Strategy one:\n\n<div class=\"aligned\">\r\n\r\n| | |\r\n| ------------- | ---------- |\r\n| Ivan's fish| \\= (26 – 8) $\\div$ 2 |\n| | \\= 18 $\\div$ 2 |\r\n| | \\= {{{correctAnswer0}}} |\r\n\r\n</div>\r\n\n<br>\n\nStrategy two (by trial and error):\n\nIvan catches less than half &nbsp$\\rightarrow$ &nbsp;try 12\n\nIf Ivan 12, Mark 20 $\\rightarrow$ &nbsp;total = 32 (too high)\n\nIf Ivan 10, Mark 18 $\\rightarrow$ &nbsp;total = 28 (too high)\n\nIf Ivan 9, Mark 17 $\\rightarrow$ &nbsp;total = 26 (correct)\n"}]},{"vars":[{"varval":"Altogether, Natasha and Shirley scored 34 points in a basketball game.\r\n\nNatasha scored 10 more points than Shirley.\r\n\nHow many points did Shirley score?\r\n"},{"varval":"sm_nogap Strategy one:\n\n<div class=\"aligned\">\r\n\r\n| | |\r\n| ------------- | ---------- |\r\n| Shirley’s points| \\= (34 – 10) $\\div$ 2 |\n| | \\= 24 $\\div$ 2 |\r\n| | \\= {{{correctAnswer0}}} |\r\n\r\n</div>\r\n\n<br>\n\nStrategy two (by trial and error):\n\nShirley scores less than half the total points &nbsp$\\rightarrow$ &nbsp;try 9\n\nIf Shirley 9, Natasha 19 $\\rightarrow$ &nbsp;total = 28 (too low)\n\nIf Shirley 11, Natasha 21 $\\rightarrow$ &nbsp;total = 32 (too low)\n\nIf Shirley 12, Natasha 22 $\\rightarrow$ &nbsp;total = 34 (correct)\n"}]},{"vars":[{"varval":"Altogether, Biggsy and Smacka had saved $67 for a skateboard.\n\nBiggsy has saved $9 more than Smacka.\n\nHow much has Smacka saved?"},{"varval":"sm_nogap Strategy one:\n\n<div class=\"aligned\">\r\n\r\n| | |\r\n| ------------- | ---------- |\r\n| Smacka savings| \\= (67 – 9) $\\div$ 2 |\n| | \\= 58 $\\div$ 2 |\r\n| | \\= {{{prefix0}}}{{{correctAnswer0}}} |\r\n\r\n</div>\r\n\n<br>\n\nStrategy two (by trial and error):\n\nSmacka has saved less than half the total &nbsp$\\rightarrow$ &nbsp;try 32\n\nIf Smacka 32, Biggsy 41 $\\rightarrow$ &nbsp;total = 73 (too high)\n\nIf Smacka 30, Biggsy 39 $\\rightarrow$ &nbsp;total = 69 (too high)\n\nIf Smacka 29, Biggsy 38 $\\rightarrow$ &nbsp;total = 67 (correct)\n"}]},{"vars":[{"varval":"Crystal and Shandy bought a total of 71 Merino sheep at the auction.\n\nCrystal bought 13 more sheep than Shandy.\n\nHow many sheep did Shandy buy?"},{"varval":"sm_nogap Strategy one:\n\n<div class=\"aligned\">\r\n\r\n| | |\r\n| ------------- | ---------- |\r\n| Shandy's sheep| \\= (71 – 13) $\\div$ 2 |\n| | \\= 58 $\\div$ 2 |\r\n| | \\= {{{correctAnswer0}}} |\r\n\r\n</div>\r\n\n<br>\n\nStrategy two (by trial and error):\n\nShandy purchased less than half the total sheep &nbsp$\\rightarrow$ &nbsp;try 34\n\nIf Shandy 34, Crystal 47 $\\rightarrow$ &nbsp;total = 81 (too high)\n\nIf Shandy 30, Crystal 43 $\\rightarrow$ &nbsp;total = 73 (too high)\n\nIf Shandy 29, Crystal 42 $\\rightarrow$ &nbsp;total = 71 (correct)"}]}]

  106. <div class="sm_mode"> {{{question}}} </div>

    [{"vars":[{"varval":"Fredrick made this number sentence with cards.\n\nsm_img https://teacher.smartermaths.com.au/wp-content/uploads/2022/01/RAPH14-Q97-99-var-0.svg 500 indent vpad\n\nWhich of these cards will make the number sentence correct?"},{"varval":"sm_nogap Simplify each side of the expression:\n\n<div class=\"aligned\">\r\n\r\n| | |\r\n| ------------: | ---------- |\r\n| 36 + 10 | \\= 36 + 6 ?? |\r\n| 46 | \\= 42 ?? |\r\n\r\n</div>\n\n\r\n<br>\n\nsm_nogap Checking each option:\r\n\n> 42 + 4 = 46 $\\checkmark$\n\n> 42 – 4 = 38 (Incorrect)\r\n\n> 42 + 6 = 48 (Incorrect)\r\n\n> 42 – 6 = 36 (Incorrect)\r\n"}]},{"vars":[{"varval":"Arianne made this number sentence with cards.\n\nsm_img https://teacher.smartermaths.com.au/wp-content/uploads/2022/01/RAPH14-Q97-99-var-1.svg 500 indent vpad\n\nWhich of these cards will make the number sentence correct?"},{"varval":"sm_nogap Simplify each side of the expression:\n\n<div class=\"aligned\">\r\n\r\n| | |\r\n| ------------: | ---------- |\r\n| 41 + 21 | \\= 41 + 19 ?? |\r\n|62 | \\= 60 ?? |\r\n\r\n</div>\n\n\r\n<br>\n\nThen check each option:\r\n\n>>60 – 2 = 58 (Incorrect)\r\n\n>>60 + 9 = 69 (Incorrect)\r\n\n>>60 + 2 = 62 $\\checkmark$\n\n>>60 – 9 = 51 (Incorrect)\r\n"}]},{"vars":[{"varval":"Bogdan made this number sentence with cards.\n\nsm_img https://teacher.smartermaths.com.au/wp-content/uploads/2022/01/RAPH14-Q97-99-var-2.svg 500 indent vpad\n\nWhich of these cards will make the number sentence correct?"},{"varval":"sm_nogap Simplify each side of the expression:\n\n<div class=\"aligned\">\r\n\r\n| | |\r\n| ------------: | ---------- |\r\n| 52 + 36 | \\= 52 + 39 ?? |\r\n| 88 | \\= 91 ?? |\r\n\r\n</div>\r\n\n<br>\n\nsm_nogap Check each option:\r\n\n>91 + 3 = 94 (Incorrect)\r\n\n>91 – 3 = 88 $\\checkmark$\n\n>91 + 6 = 97 (Incorrect)\r\n\n>91 – 6 = 85 (Incorrect)\r\n"}]}]

  107. <div class="sm_mode"> {{{question}}} </div>

    [{"vars":[{"varval":"Anne is using a broken ruler to measure the length of a pencil.\n\n<br>\n\nsm_img https://teacher.smartermaths.com.au/wp-content/uploads/2021/09/94.png 500 indent vpad\n\nWhat is the length of the pencil?"},{"varval":"<div class=\"aligned\">\r\n\r\n| | |\r\n| ------------- | ---------- |\r\n| Pencil’s length | \\= 19 – 12 |\r\n| | \\= {{{correctAnswer}}} |\r\n\r\n</div>\r\n"}]},{"vars":[{"varval":"Selene is using a broken ruler to measure the length of a pin.\n\n<br>\n\nsm_img https://teacher.smartermaths.com.au/wp-content/uploads/2021/09/95.png 550 indent vpad\n\nWhat is the length of the pin?"},{"varval":"<div class=\"aligned\">\r\n\r\n| | |\r\n| ------------- | ---------- |\r\n| Pin’s length | \\= 16 – 11 |\r\n| | \\= {{{correctAnswer}}} |\r\n\r\n</div>\r\n"}]},{"vars":[{"varval":"Eugene measured a wooden pin using a broken ruler.\n\n<br>\n\nsm_img https://teacher.smartermaths.com.au/wp-content/uploads/2021/09/96.png 500 indent vpad\n\nWhat is the length of the wooden pin?"},{"varval":"<div class=\"aligned\">\r\n\r\n| | |\r\n| ------------- | ---------- |\r\n| Length of wooden pin | \\= 19 – 14 |\r\n| | \\= {{{correctAnswer}}} |\r\n\r\n</div>\r\n"}]}]

  108. <div class="sm_mode"> {{{question}}} </div>

    [{"vars":[{"varval":"Matt, Trish, and Kent went to a pizza restaurant.\r\n\nMatt and Trish each ate $\\dfrac{1}{8}$ of the pizza served.\r\n\nWhat fraction of the pizza was left for Kent?\r\n"},{"varval":"<div class=\"aligned\">\r\n\r\n| | |\r\n| ------------- | ---------- |\r\n| Pizza left | \\= 1 – ( $\\dfrac{1}{8}$ + $\\dfrac{1}{8}$ ) |\n| | \\= 1 – $\\dfrac{1}{4}$ |\n| | \\= $\\dfrac{4}{4}$ – $\\dfrac{1}{4}$ |\r\n| | \\= {{{correctAnswer}}} |\r\n\r\n</div>\r\n"}]},{"vars":[{"varval":"Ruth, April, and Yoni shared a box of chocolates.\r\n\nRuth and April each ate $\\dfrac{1}{6}$ of the chocolates in the box.\r\n\nWhat fraction of the chocolates were left for Yoni?\r\n"},{"varval":"<div class=\"aligned\">\r\n\r\n| | |\r\n| ------------- | ---------- |\r\n| Chocolates left | \\= 1 – ( $\\dfrac{1}{6}$ + $\\dfrac{1}{6}$ ) |\n| | \\= 1 – $\\dfrac{2}{6}$ |\n| | \\= $\\dfrac{6}{6}$ – $\\dfrac{2}{6}$ |\r\n| | \\= {{{correctAnswer}}} |\r\n\r\n</div>\r\n"}]},{"vars":[{"varval":"Manuel, Jericho, and Tom shared a bucket of fried chicken.\r\n\nManuel and Jericho each ate $\\dfrac{3}{7}$ of the fried chicken.\r\n\nWhat fraction of the fried chicken was left for Tom?\r\n"},{"varval":"<div class=\"aligned\">\r\n\r\n| | |\r\n| ------------- | ---------- |\r\n| Fried chicken left | \\= 1 – ( $\\dfrac{3}{7}$ + $\\dfrac{3}{7}$ ) |\n| | \\= 1 – $\\dfrac{6}{7}$ |\n| | \\= $\\dfrac{7}{7}$ – $\\dfrac{6}{7}$ |\r\n| | \\= {{{correctAnswer}}} |\r\n\r\n</div>\r\n"}]}]

  109. <div class="sm_mode"> {{{question}}} </div>

    [{"vars":[{"varval":"<div class=\"sm_img_inline\">\n\nAron wrote &nbsp;<span class=\"sm-text color8\">?</span> $–$ 3 = 7\r\n\nIn which of these does the &nbsp;<span class=\"sm-text color8\">?</span> &nbsp;have the same value as above?\r\n\n</div>"},{"varval":"sm_nogap By rearranging the equation: \n\n<div class=\"aligned sm_img_inline\">\n\n|||\n|-:|-|\n|&nbsp;<span class=\"sm-text color8\">?</span> $–$ 3|= 7|\n|&nbsp;<span class=\"sm-text color8\">?</span> |= 7 + 3|\n|3 + 7|= &nbsp;<span class=\"sm-text color8\">?</span> |\n\n</div>\n\n\n"}]},{"vars":[{"varval":"<div class=\"sm_img_inline\">\n\nKrishna wrote &nbsp;<span class=\"sm-text color3\">?</span> $–$ 2 = 9 \r\n\nIn which of these does the &nbsp;<span class=\"sm-text color3\">?</span> &nbsp;have the same value as above?\r\n\n</div>\n"},{"varval":"sm_nogap By rearranging the equation:\n\n<div class=\"aligned sm_img_inline\">\n\n|||\n|-:|-|\n|&nbsp;<span class=\"sm-text color3\">?</span> $–$ 2|= 9|\n|&nbsp;<span class=\"sm-text color3\">?</span> |= 9 + 2|\n|9 + 2 |= &nbsp;<span class=\"sm-text color3\">?</span> |\n\n</div>\n\n"}]},{"vars":[{"varval":"<div class=\"sm_img_inline\">\n\nMay wrote &nbsp;<span class=\"sm-text color2\">?</span> + 2 = 10\r\n\nIn which of these does the &nbsp;<span class=\"sm-text color2\">?</span> &nbsp;have the same value as above?\r\n\n</div>\n"},{"varval":"sm_nogap By rearranging the equation:\n\n<div class=\"aligned sm_img_inline\">\n\n|||\n|-:|-|\n|&nbsp;<span class=\"sm-text color2\">?</span> + 2|= 10|\n|&nbsp;<span class=\"sm-text color2\">?</span> |= 10 $-$ 2|\n|10 $-$ 2 |= &nbsp;<span class=\"sm-text color2\">?</span> |\n\n</div>\n\n\n"}]}]

  110. {{{question}}}

    [{"vars":[{"varval":"Gary thinks of a number.\r\n\nHe writes the clues so his friends can guess it.\r\n\n<div style=\"background-color: lightyellow; padding: 5px 25px 20px 15px; border: 2px solid; width: max-content; margin-left: 30px\" >\n\n* The number is less than 600\n* The number has 20 tens\n* The digits in the ones place is the same as the digit in the hundreds place\n\n</div>\n\n<br> What is Gary’s number?"},{"varval":"sm_nogap Check each option:\r\n\n> 22 is less than 600, it doesn’t have 20 tens (Incorrect)\r\n\n> 202 is less than 600, it has 20 tens, and the ones digit is the same as the hundreds digit (Correct)\r\n\n>266 is less than 600, it has 20 tens, and the ones digit is NOT the same as the hundreds digit (Incorrect)\r\n\n>20 is less than 600, it doesn’t have 20 tens (Incorrect)\r\n"}]},{"vars":[{"varval":"Josh thinks of a number.\r\n\nHe writes the clues so his friends can guess it.\n\r\n\n<div style=\"background-color: lightyellow; padding: 5px 25px 20px 15px; border: 2px solid; width: max-content; margin-left: 30px\" >\n\n* The number is less than 500\n* The number has 40 tens\n* The digit in the ones place is twice the the digit in the hundreds place\n\n</div>\n\n<br> What is Josh’s number?"},{"varval":"sm_nogap Check each option:\r\n\n>402 is less than 500, it has 40 tens, and the ones digit is half the digit in the hundreds place (Incorrect)\r\n\n>480 is less than 500, it has 48 tens (Incorrect)\r\n\n>40 is less than 500, it only has 4 tens (Incorrect)\r\n\n>408 is less than 500, it has 40 tens, and the ones digit is twice the hundreds digit (Correct)\n"}]},{"vars":[{"varval":"Lily thinks of a number.\r\n\nShe writes the clues so her friends can guess it.\n\n\r\n<div style=\"background-color: lightyellow; padding: 5px 25px 20px 15px; border: 2px solid; width: max-content; margin-left: 30px\" >\n\n* The number is less than 900\r\n* The digits in the ones place is the same as the digit in the hundreds place\r\n* The number has 70 tens\r\n\n</div>\n\n<br> What is Lily’s number?"},{"varval":"sm_nogap Check each option:\r\n\n>77 is less than 900, it only has 7 tens (Incorrect)\r\n\n>770 is less than 900, it has 70 tens, and its ones digit is NOT the same as the hundreds digit. (Incorrect)\r\n\n>707 is less than 900, it has 70 tens, and its ones digit is the same as the hundreds digit. (Correct)\r\n\n>70 is less than 900, it only has 7 tens (Incorrect)\r\n"}]}]

  111. <div class="sm_mode"> {{{question}}} </div>

    [{"vars":[{"varval":"Kristian is using a number line to show a pattern.\n\n<br>\n\nsm_img https://teacher.smartermaths.com.au/wp-content/uploads//2021/09/82.svg 550 indent vpad\n\n<br>In Kristian’s pattern, what number comes before 22?"},{"varval":"The pattern has a common difference of 4.\r\n\r\n\n<div class=\"aligned\">\r\n\r\n| | |\r\n| ------------- | ---------- |\r\n| Number before 22 | \\= 22 – 4 |\r\n| | \\= {{{correctAnswer}}} |\r\n\r\n</div>\r\n"}]},{"vars":[{"varval":"Kristel is using a number line to show a pattern.\n\n<br>\n\nsm_img https://teacher.smartermaths.com.au/wp-content/uploads//2021/09/83.svg 600 indent vpad\n\n<br>In Kristel’s pattern, what number comes before 38?"},{"varval":"The pattern has a common difference of 4.\r\n\n<div class=\"aligned\">\r\n\r\n| | |\r\n| ------------- | ---------- |\r\n| Number before 38 | \\= 38 – 4 |\r\n| | \\= {{{correctAnswer}}} |\r\n\r\n</div>\r\n"}]},{"vars":[{"varval":"Matthew is using a number line to show a pattern.\n\n<br>\n\nsm_img https://teacher.smartermaths.com.au/wp-content/uploads//2021/09/84.svg 550 indent vpad\n\n<br>In Matthew’s pattern, what number comes before 14?"},{"varval":"The pattern has a common difference of 5.\r\n\n\r\n<div class=\"aligned\">\r\n\r\n| | |\r\n| ------------- | ---------- |\r\n| Number before 14 | \\= 14 – 5 |\r\n| | \\= {{{correctAnswer}}} |\r\n\r\n</div>\r\n"}]}]

  112. <div class="sm_mode"> {{{question}}} </div>

    [{"vars":[{"varval":"Henry and Paolo are playing a game where Henry tosses a coin and Paolo guesses if it is heads or tails.\n\nHenry calls heads on the first and second toss and wins both times.\n\nWhich of the following is true for the third toss?\r\n"},{"varval":"Every time a fair coin is tossed the chance of landing on a head or tails is the same.\n\n$\\rightarrow$ {{{correctAnswer}}}"}]},{"vars":[{"varval":"Wally and Katherine are best friends and eat lunch together every weekday.\n\nTo decide where they eat, Wally flips a coin. If it lands on heads, Wally chooses and if it lands on tails, Katherine chooses.\n\nOne week, Katherine chose where they eat for the first four days in a row.\n\nWhich of the following is true for the fifth day?\n"},{"varval":"Every time a fair coin is tossed the chance of landing a head or tails is the same.\n\n$\\rightarrow$ {{{correctAnswer}}}"}]},{"vars":[{"varval":"John and Richard flip a coin to see who will be able to use the family computer for the next hour.\r\n\nRichard calls tails every time and got to use the computer for the first two hours in a row.\n\nWhich of the following is true for the third hour?\r\n"},{"varval":"Every time a fair coin is tossed the chance of landing on a head or tails is the same.\n\n$\\rightarrow$ {{{correctAnswer}}}"}]}]

  113. <div class="sm_mode"> {{{question}}} </div>

    [{"vars":[{"varval":"Terry and Ray have 32 toys altogether.\r\n\nRay has 19 toys.\r\n\nWhich number sentence can be used to find the number of toys Terry has?\r\n"},{"varval":"Total toys $-$ Ray's toys = Terry's toys\n\nTherefore, correct number sentence is:\n\n{{{correctAnswer}}}\n\n"}]},{"vars":[{"varval":"Michael and Andre have saved $92 altogether.\r\n\nMichael has saved $51.\r\n\nWhich number sentence can be used to find the amount of money Andre has saved?\r\n"},{"varval":"Total saved $-$ Michael's savings = Andre's savings\n\nTherefore, correct number sentence is:\n\n{{{correctAnswer}}}"}]},{"vars":[{"varval":"Gerry and David own 37 vintage cars altogether.\r\n\nGerry has 21 vintage cars.\r\n\nWhich number sentence can be used to find the number of vintage cars owned by David?\r\n"},{"varval":"Total vintage cars $-$ Gerry's cars = David's cars\n\nTherefore, the correct number sentence is:\n\n{{{correctAnswer}}}"}]},{"vars":[{"varval":"Altogether, a farmer has 78 cows that live in his paddock.\n\nOne morning, 42 cows are in the shed being milked.\n\nHow many cows are left in the paddock?"},{"varval":"Total cows $-$ cows in shed = cows in paddock\n\nTherefore, the correct number sentence is:\n\n{{{correctAnswer}}}"}]},{"vars":[{"varval":"A rancher has 46 brumbies living on her property.\n\nShe captures 19 of the brumbies and sells them.\n\nWhich number sentence can be used to find the number of brumbies that remain living on her property?"},{"varval":"Total Brumbies $-$ Brumbies sold = Brumbies left\n\nTherefore, the correct number sentence is:\n\n{{{correctAnswer}}}"}]}]

  114. <div class="sm_mode"> {{{question}}} </div>

    [{"vars":[{"varval":"Which of the following figures when folded will create a square pyramid?"},{"varval":"This figure when folded will create a square pyramid.\n\nsm_img https://teacher.smartermaths.com.au/wp-content/uploads/2021/09/83a.svg 150 indent vpad"}]},{"vars":[{"varval":"Which of the following figures when folded will create a trapezoidal prism?"},{"varval":"This figure when folded will create a trapezoidal prism.\n\nsm_img https://teacher.smartermaths.com.au/wp-content/uploads/2023/07/trap-prism-2.svg 150 indent vpad"}]}]

  115. <div class="sm_mode"> {{{question}}} </div>

    [{"vars":[{"varval":"Which of the following spinners will most likely land on the coloured area?"},{"varval":"sm_img https://teacher.smartermaths.com.au/wp-content/uploads/2021/09/81c.svg 150 indent vpad\n\nThis spinner has the most area painted therefore has the highest chance of landing on a coloured area.\n\n"}]},{"vars":[{"varval":"Which of the following spinners will most likely land on the coloured area?"},{"varval":"sm_img https://teacher.smartermaths.com.au/wp-content/uploads/2021/09/82b.svg 150 indent vpad\n\nThis spinner has the most area painted therefore has the highest chance of landing on a coloured portion of the spinner."}]},{"vars":[{"varval":"Which of the following spinners will most likely land on the coloured area?"},{"varval":"{{{correctAnswer}}}\n\nThis spinner has the most area painted therefore has the highest chance of landing on a coloured portion of the spinner."}]},{"vars":[{"varval":"Which of the following spinners will most likely land on the coloured area?"},{"varval":"{{{correctAnswer}}}\n\nThis spinner has the most area painted therefore has the highest chance of landing on a coloured portion of the spinner."}]},{"vars":[{"varval":"Which of the following spinners is the least likely to land on the coloured area?"},{"varval":"{{{correctAnswer}}}\n\nThis spinner has the least area painted therefore has the smallest chance of landing on a coloured portion of the spinner."}]},{"vars":[{"varval":"Which of the following spinners is the least likely to land on the coloured area?"},{"varval":"{{{correctAnswer}}}\n\nThis spinner has the least area painted therefore has the smallest chance of landing on a coloured portion of the spinner."}]}]

  116. <div class="sm_mode"> {{{question}}} </div>

    [{"vars":[{"varval":"A group of 7 students scored 57 points in a game.\r\n5 students scored 7 points each.\r\n\nThe other two, Wilson and Harold, scored the remaining points equally.\r\n\nHow many points did Wilson score?\r\n"},{"varval":"Remaining points = 57 $-$ (5 x 7) = 57 $-$ 35 = 22\r\n\nWilson’s points = 22 ÷ 2 = {{{correctAnswer}}}\n"}]},{"vars":[{"varval":"12 farmers harvested 156 banana trees in a week.\r\n\n10 farmers harvested 12 banana trees each.\n\r\n\nThe other two, Greg and Ryan, harvested the remaining trees equally.\r\n\nHow many banana trees did Greg harvest?\r\n"},{"varval":"Remaining trees = 156 $-$ (10 x 12) = 156 $-$ 120 = 36\r\n\nGreg’s trees = 36 ÷ 2 = {{{correctAnswer}}}"}]},{"vars":[{"varval":"A team of 6 runners completed a 52 kilometre relay event. \n\nThe first 4 runners ran 9 kilometres each. The other two, Carrie and Michael, ran the remaining distance equally.\n\nHow many kilometres did Carrie run?"},{"varval":"Remaining kilometres = 52 $-$ (4 x 9) = 52 $-$ 36 = 16\n\nCarrie’s kilometres = 16 ÷ 2 = {{{correctAnswer}}}\n"}]}]

  117. <div class="sm_mode"> {{{question}}} </div>

    [{"vars":[{"varval":"Jay made a figure consisting of squares and 2 of them are painted grey.\n\nsm_img https://teacher.smartermaths.com.au/wp-content/uploads/2021/09/77.svg 200 indent vpad\n\nJay flipped and rotated the figure.\n\nsm_img https://teacher.smartermaths.com.au/wp-content/uploads/2021/09/77s.svg 200 indent vpad\n\nWhich two numbers show the positions of the grey squares?"},{"varval":"The numbers {{{correctAnswer0}}} and {{{correctAnswer1}}} are in the positions of the grey squares. "}]},{"vars":[{"varval":"Omar created a figure which is composed of squares and 2 of them are painted grey.\n\nsm_img https://teacher.smartermaths.com.au/wp-content/uploads/2021/09/78.svg 250 indent vpad\n\nOmar rotated the figure.\n\nsm_img https://teacher.smartermaths.com.au/wp-content/uploads/2021/09/78s.svg 130 indent vpad\n\nWhich two numbers show the positions of the grey squares?"},{"varval":"The numbers {{{correctAnswer0}}} and {{{correctAnswer1}}} are in the positions of the grey squares. "}]}]

  118. SJ add/subtract

    <div class="sm_mode"> {{{question}}} </div>

    [{"vars":[{"varval":"Complete the number sentence\r\n\n\r\n"},{"varval":"<div class=\"aligned\">\n\n| | |\n| ------------- | ---------- |\n| <span class=\"sm_box\"> ` ` ` ` </span> | \\= 26 $-$ 15 |\n| | \\= {{{correctAnswer0}}} |\n\n</div>\n\n"}]},{"vars":[{"varval":"Complete the number sentence\r\n"},{"varval":"<div class=\"aligned\">\n\n| | |\n| ------------- | ---------- |\n| <span class=\"sm_box\"> ` ` ` ` </span> | \\= 84 $-$ 32 |\n| | \\= 84 $-$ 30 $-$ 2 |\n| | \\= 54 $-$ 2 |\n| | \\= {{{correctAnswer0}}} |\n\n\n</div>\n\n"}]},{"vars":[{"varval":"Complete the number sentence\n"},{"varval":"<div class=\"aligned\">\n\n| | |\n| ------------- | ---------- |\n| <span class=\"sm_box\"> ` ` ` ` </span> | \\= 92 $-$ 51 |\n| | \\= 92 $-$ 50 $-$ 1 |\n| | \\= 42 $-$ 1 |\n| | \\= {{{correctAnswer0}}} |\n\n</div>\n"}]},{"vars":[{"varval":"Complete the number sentence\n"},{"varval":"<div class=\"aligned\">\n\n| | |\n| ------------- | ---------- |\n| <span class=\"sm_box\"> ` ` ` ` </span> | \\= 97 $-$ 52 |\n| | \\= 97 $-$ 50 $-$ 2 |\n| | \\= 47 $-$ 2 |\n| | \\= {{{correctAnswer0}}} |\n\n</div>\n"}]},{"vars":[{"varval":"Complete the number sentence"},{"varval":"<div class=\"aligned\">\n\n| | |\n| ------------- | ---------- |\n| <span class=\"sm_box\"> ` ` ` ` </span> | \\= 185 $-$ 51 |\n| | \\= 185 $-$ 50 $-$ 1 |\n| | \\= 135 $-$ 1 |\n| | \\= {{{correctAnswer0}}} |\n\n</div>\n"}]},{"vars":[{"varval":"Complete the number sentence\n"},{"varval":"<div class=\"aligned\">\n\n| | |\n| ------------- | ---------- |\n| <span class=\"sm_box\"> ` ` ` ` </span> | \\= 128 $-$ 47 |\n| | \\= 128 $-$ 40 $-$ 7 |\n| | \\= 88 $-$ 7 |\n| | \\= {{{correctAnswer0}}} |\n\n</div>\n"}]}]

  119. <div class="sm_mode"> {{{question}}} </div>

    [{"vars":[{"varval":"A statistic shows that on a certain highway the chance of an accident happening on any day is 5%.\r\n\nWhich of the following describes the chance of having an accident today?\r"},{"varval":"Anything that has 100% chance of occurring is certain to happen.\r\n\nThe chance of an accident happening today is 5%.\r\n\nTherefore, it is **unlikely** to happen since the chance is very low.\t\r\n"}]},{"vars":[{"varval":"According to previous events, the chance of passing the entrance exam for a certain university is around 75%.\r\n\nWhich of the following describes the chance of passing in the university’s entrance exam?\r\n"},{"varval":"Anything that has 100% chance of occurring is certain to happen.\r\n\nThe chance of passing the exam is 75%.\r\n\nTherefore, you are **likely** to pass the exam since the chance is high.\t\r\n"}]}]

  120. <div class="sm_mode"> {{{question}}} </div>

    [{"vars":[{"varval":"sm_nogap Arthur bought the items shown.\n\nsm_img https://teacher.smartermaths.com.au/wp-content/uploads/2021/09/67.png 650 indent vpad\n\nHe gives the cashier $400\r.\n\nHow much change should Arthur get?\r\n"},{"varval":"sm_nogap Strategy 1 (add up rounded numbers): \n\n<div class=\"aligned\">\r\n\r\n| | |\r\n| ------------- | ---------- |\r\n| Total cost | \\= 311 + 52 + 19 |\r\n| | \\= $382 |\r\n\r\n</div>\r\n\n<br>\n\n<div class=\"aligned\">\r\n\r\n| | |\r\n| ------------- | ---------- |\r\n| Change | $\\approx$ $400 – $381 |\r\n| | $\\approx$ $19 |\r\n\r\n</div>\r\n\n<br>\n\n$\\rightarrow$ $18.10 is the closest option\n\n<br>\n\nsm_nogap Strategy 2: \n\n<div class=\"aligned\">\r\n\r\n| | |\r\n| ------------- | ---------- |\r\n| Total cost | \\= 310.50 + 52.30 + 19.10 |\r\n| | \\= $381.90 |\r\n\r\n</div>\r\n\n<br>\n\n<div class=\"aligned\">\r\n\r\n| | |\r\n| ------------- | ---------- |\r\n| Change | \\= $400 – $381.90 |\r\n| | \\= {{{correctAnswer}}} |\r\n\r\n</div>\r\n"}]},{"vars":[{"varval":"Mary bought each of the following items at her local cafe.\n\nsm_img https://teacher.smartermaths.com.au/wp-content/uploads/2021/09/68.png 350 indent vpad\n\nShe gave the cashier $5.00\r.\n\nHow much change should she receive?\r\n"},{"varval":"<div class=\"aligned\">\r\n\r\n| | |\r\n| ------------- | ---------- |\r\n| Total cost of items | \\= 1.20 + 0.80 + 2.20 |\r\n| | \\= $4.20 |\r\n\r\n</div>\r\n\n<br>\n\n<div class=\"aligned\">\r\n\r\n| | |\r\n| ------------- | ---------- |\r\n| Change | \\= $5 – $4.20 |\r\n| | \\= {{{correctAnswer}}} |\r\n\r\n</div>\r\n"}]},{"vars":[{"varval":"Raymond bought the items shown below.\n\nsm_img https://teacher.smartermaths.com.au/wp-content/uploads/2021/09/69.png 450 indent vpad\n\nHe gave the store owner $50\r.\n\nHow much change should he get?\r\n"},{"varval":"<div class=\"aligned\">\r\n\r\n| | |\r\n| ------------- | ---------- |\r\n| Total cost of items | \\= 40.80 + 3.20 + 1.50 |\r\n| | \\= $45.50 |\r\n\r\n</div>\r\n\n<br>\n\n<div class=\"aligned\">\r\n\r\n| | |\r\n| ------------- | ---------- |\r\n| Change | \\= $50 – $45.50 |\r\n| | \\= {{{correctAnswer}}} |\r\n\r\n</div>\r\n"}]}]

  121. <div class="sm_mode"> {{{question}}} </div>

    [{"vars":[{"varval":"<div class=\"sm_img_inline\">\n\nShawn drew a treasure map.\r\n\nHe marked the treasure with an &nbsp<img src=\"https://teacher.smartermaths.com.au/wp-content/uploads/2021/09/64_x.png\" width=\"30\" height=\"30\">.\n\n<br>\n\n<img src=\"https://teacher.smartermaths.com.au/wp-content/uploads/2021/09/64.png\" width=\"550\"> \n\n\n<br>In which cell is the &nbsp<img src=\"https://teacher.smartermaths.com.au/wp-content/uploads/2021/09/64_x.png\" width=\"30\" height=\"30\"> &nbsp;located?\n\n</div>"},{"varval":"<div class=\"sm_img_inline\">\n\n\nBased on the clues, the rows and columns should be labeled as shown below:\n\nsm_img https://teacher.smartermaths.com.au/wp-content/uploads/2021/09/64_sol.png 250 indent vpad\n\n<img src=\"https://teacher.smartermaths.com.au/wp-content/uploads/2021/09/64_x.png\" width=\"30\" height=\"30\"> &nbsp;is located at cell {{{correctAnswer}}}\n\n</div>"}]},{"vars":[{"varval":"<div class=\"sm_img_inline\">\n\nTrisha drew a map which contains a treasure.\r\n\nShe marked the treasure with an &nbsp;<img src=\"https://teacher.smartermaths.com.au/wp-content/uploads/2021/09/64_x.png\" width=\"30\" height=\"30\"> .\n\n</div>\n\n<br>\n\n<img src=\"https://teacher.smartermaths.com.au/wp-content/uploads/2021/09/65.png\" width=\"550\">\n\n<div class=\"sm_img_inline\">\n\n<br>In which cell is the &nbsp;<img src=\"https://teacher.smartermaths.com.au/wp-content/uploads/2021/09/64_x.png\" width=\"30\" height=\"30\"> &nbsp;located?\n\n</div>"},{"varval":"<div class=\"sm_img_inline\">\n\nBased on the clues, the rows and columns should be labeled as shown below:\n\nsm_img https://teacher.smartermaths.com.au/wp-content/uploads/2021/09/65_sol.png 250 indent vpad\n\n<img src=\"https://teacher.smartermaths.com.au/wp-content/uploads/2021/09/64_x.png\" width=\"30\" height=\"30\"> &nbsp;is located at cell {{{correctAnswer}}}\n\n</div>\n"}]},{"vars":[{"varval":"<div class=\"sm_img_inline\">\n\nJohnny drew a treasure map.\r\n\nHe marked the treasure with an &nbsp;<img src=\"https://teacher.smartermaths.com.au/wp-content/uploads/2021/09/64_x.png\" width=\"30\" height=\"30\"> .\n\r\n</div>\n\n<br>\n\n<img src=\"https://teacher.smartermaths.com.au/wp-content/uploads/2021/09/66.png\" width=\"550\">\n\n<div class=\"sm_img_inline\">\n\n<br>In which cell is the &nbsp;<img src=\"https://teacher.smartermaths.com.au/wp-content/uploads/2021/09/64_x.png\" width=\"30\" height=\"30\"> &nbsp;located?\n\n</div>"},{"varval":"<div class=\"sm_img_inline\">\n\nBased on the clues, the rows and columns should be labeled as shown below:\n\nsm_img https://teacher.smartermaths.com.au/wp-content/uploads/2021/09/66_sol.png 250 indent vpad\n\n<img src=\"https://teacher.smartermaths.com.au/wp-content/uploads/2021/09/64_x.png\" width=\"30\" height=\"30\"> &nbsp;is located at cell {{{correctAnswer}}}\n\n</div>"}]}]

  122. <div class="sm_mode"> {{{question}}} </div>

    [{"vars":[{"varval":"A group of 8 volunteers collected a total of 64 cans to be recycled.\r\n\nEach volunteer collected the same number of cans.\r\n\nHow many cans did each volunteer collect?\r\n"},{"varval":"<div class=\"aligned\">\r\n\r\n| | |\r\n| ------------- | ---------- |\r\n| Number of cans each | \\= $64 \\div 8$ |\r\n| | \\= {{{correctAnswer}}} cans |\r\n\r\n</div>\r\n"}]},{"vars":[{"varval":"6 farmers picked a total of 42 pumpkins.\r\n\nEach farmer picked the same number of pumpkins.\r\n\nHow many did each farmer pick?\r\n"},{"varval":"<div class=\"aligned\">\r\n\r\n| | |\r\n| ------------- | ---------- |\r\n| Number of vegetables | \\= $42 \\div 6$ |\r\n| | \\= {{{correctAnswer}}} pumpkins |\r\n\r\n</div>\r\n"}]},{"vars":[{"varval":"At a birthday party a total of 48 lollipops were given to 8 children.\r\n\nEach child received the same number of lollipops.\r\n\nHow many did each child receive?\r\n"},{"varval":"<div class=\"aligned\">\r\n\r\n| | |\r\n| ------------- | ---------- |\r\n| Number of lollipops | \\= $48 \\div 8$ |\r\n| | \\= {{{correctAnswer}}} |\r\n\r\n</div>\r\n"}]},{"vars":[{"varval":"A charity gave 108 cans of food to 9 families at Christmas.\n\nEach family recieved the same number of cans.\n\nHow many did each family recieve?"},{"varval":"<div class=\"aligned\">\r\n\r\n| | |\r\n| ------------- | ---------- |\r\n| Number of cans | \\= $108 \\div 9$ |\r\n| | \\= {{{correctAnswer}}} |\r\n\r\n</div>\r"}]},{"vars":[{"varval":"A teacher has 84 marking pens and gives them to 7 groups in her class for an art project.\n\nEach group recieves the same number of marking pens.\n\nHow many does each group recieve."},{"varval":"<div class=\"aligned\">\r\n\r\n| | |\r\n| ------------- | ---------- |\r\n| Number of pens | \\= $84 \\div 7$ |\r\n| | \\= {{{correctAnswer}}} |\r\n\r\n</div>\r"}]},{"vars":[{"varval":"A horse sale sold a total of 72 horses to 6 breeders.\n\nIf the breeders bought the same number of horses, how many did each one get?"},{"varval":"<div class=\"aligned\">\r\n\r\n| | |\r\n| ------------- | ---------- |\r\n| Number of horses | \\= $72 \\div 6$ |\r\n| | \\= {{{correctAnswer}}} |\r\n\r\n</div>\r"}]},{"vars":[{"varval":"A wedding planner hired 112 chairs for a wedding.\n\nThe chairs were split between 7 tables.\n\nIf each table had the same number of chairs, how many did each one have?"},{"varval":"<div class=\"aligned\">\r\n\r\n| | |\r\n| ------------- | ---------- |\r\n| Chairs on each table | \\= $112 \\div 7$ |\r\n| | \\= {{{correctAnswer}}} |\r\n\r\n</div>\r"}]},{"vars":[{"varval":"A sheep dog rounded up 114 sheep into 6 pens.\n\nEach pen had the same number of sheep in it.\n\nHow many sheep did each pen have?"},{"varval":"<div class=\"aligned\">\r\n\r\n| | |\r\n| ------------- | ---------- |\r\n| Sheep in each pen | \\= $114 \\div 6$ |\r\n| | \\= {{{correctAnswer}}} |\r\n\r\n</div>\r"}]}]

  123. <div class="sm_mode"> {{{question}}} </div>

    [{"vars":[{"varval":"Molly poured one glass of water into four different containers.\r\n\nWhen full, which container will hold the most water?\r\n"},{"varval":"The container with the largest capacity is:\r\n\n<br>\n\r\n{{{correctAnswer}}}"}]},{"vars":[{"varval":"Jonas poured one cup of oil into four different containers.\r\n\nWhen full, which container will hold the most oil?\r\n"},{"varval":"The container with the most capacity is:\n\n<br>\n\n{{{correctAnswer}}}"}]},{"vars":[{"varval":"Kevin poured one cup of liquid into four different containers.\r\n\nWhich container will hold the most liquid when full?\r\n"},{"varval":"The container that can fit the most cups of liquid when full is:\n\n<br>\n\n{{{correctAnswer}}}"}]}]

  124. <div class="sm_mode"> {{{question}}} </div>

    [{"vars":[{"varval":"Richard uses the number sentence &nbsp12 $\\times$ 4 = 48 &nbspto solve a problem.\r\n\nWhich problem could he solve with this number sentence?\r\n"},{"varval":"{{{correctAnswer}}}\n"}]}]

  125. Needs difficulty

    <div class="sm_mode"> {{{question}}} </div>

    [{"vars":[{"varval":"Oriel bought drinks for each of his 15 friends.\r\n\nHe put the drinks on the tables of his friends who were sitting in groups of 3.\r\n\nWhich of the following shows the 15 drinks in groups of 3?\r\n"},{"varval":"<div class=\"aligned\">\r\n\r\n| | |\r\n| ------------- | ---------- |\r\n| Number of groups | \\= 15 $\\div$ 3|\r\n| | \\= 5 |\n\r\n\r\n</div>\r\n\n<br>\n\n\n{{{correctAnswer}}} "}]},{"vars":[{"varval":"Twenty people have participated in an event.\r\n\nThey were asked to form themselves into groups of 4.\r\n\nWhich of the following shows the 20 people in groups of 4?\r\n"},{"varval":"<div class=\"aligned\">\r\n\r\n| | |\r\n| ------------- | ---------- |\r\n| Number of groups | \\= 20 $\\div$ 4|\r\n| | \\= 5 |\n\r\n\r\n</div>\r\n\n<br>\n\n\n{{{correctAnswer}}} "}]},{"vars":[{"varval":"Mark owns 16 books and wants to organise them in groups of 4.\r\n\nWhich of the following shows the 16 books in groups of 4?\r\n"},{"varval":"<div class=\"aligned\">\r\n\r\n| | |\r\n| ------------- | ---------- |\r\n| Number of groups | \\= 16 $\\div$ 4|\r\n| | \\= 4 |\n\r\n\r\n</div>\r\n\n<br>\n\n\n{{{correctAnswer}}} "}]}]

  126. <div class="sm_mode"> {{{question}}} </div>

    [{"vars":[{"varval":"Daniella is comparing the weights of four different objects using a balance scale.\n\n<br>\n\nsm_img https://teacher.smartermaths.com.au/wp-content/uploads//2021/09/73.svg 650 indent vpad\n\n<br> Which object is the heaviest?"},{"varval":"<div class=\"sm_img_inline\">\n\nBy comparing the weight of the objects:\r\n\n>> <img src=\"https://teacher.smartermaths.com.au/wp-content/uploads//2021/09/73b.svg\" width=\"50px\" height=\"50px\"> &nbsp;is heavier than &nbsp;<img src=\"https://teacher.smartermaths.com.au/wp-content/uploads//2021/09/73c.svg\" width=\"50px\" height=\"50px\"> \r\n\n<br>\n\n>> <img src=\"https://teacher.smartermaths.com.au/wp-content/uploads//2021/09/73b.svg\" width=\"50px\" height=\"50px\"> &nbsp;is the same weight as<img src=\"https://teacher.smartermaths.com.au/wp-content/uploads//2021/09/73a.svg\" width=\"55px\" height=\"50px\">\r\n\n<br>\n\n>> <img src=\"https://teacher.smartermaths.com.au/wp-content/uploads//2021/09/73d.svg\" width=\"50px\" height=\"50px\"> &nbsp;is heavier than <img src=\"https://teacher.smartermaths.com.au/wp-content/uploads//2021/09/73a.svg\" width=\"55px\" height=\"50px\"> \r\n\n<br>Therefore, &nbsp;<img src=\"https://teacher.smartermaths.com.au/wp-content/uploads//2021/09/73d.svg\" width=\"50px\" height=\"50px\"> &nbsp;is the heaviest object.\r\n\n</div>\n"}]},{"vars":[{"varval":"Anthony is comparing the weights of four different objects using a balance scale.\n\n<br>\n\nsm_img https://teacher.smartermaths.com.au/wp-content/uploads//2021/09/74.svg indent vpad\n\n<br>Which object is the heaviest?"},{"varval":"<div class=\"sm_img_inline\">\n\nBy comparing the weight of the objects:\r\n\n>> <img src=\"https://teacher.smartermaths.com.au/wp-content/uploads//2021/09/74d.svg\" width=\"55px\" height=\"55px\"> &nbsp;is heavier than &nbsp;<img src=\"https://teacher.smartermaths.com.au/wp-content/uploads//2021/09/74a.svg\" width=\"55px\" height=\"55px\">\r\n\n<br>\n\n>> <img src=\"https://teacher.smartermaths.com.au/wp-content/uploads//2021/09/74d.svg\" width=\"55px\" height=\"55px\"> &nbsp;is the same weight as &nbsp;<img src=\"https://teacher.smartermaths.com.au/wp-content/uploads//2021/09/74c.svg\" width=\"55px\" height=\"55px\"> \r\n\n<br>\n\n>> <img src=\"https://teacher.smartermaths.com.au/wp-content/uploads//2021/09/74b.svg\" width=\"55px\" height=\"55px\"> &nbsp;is heavier than &nbsp;<img src=\"https://teacher.smartermaths.com.au/wp-content/uploads//2021/09/74c.svg\" width=\"55px\" height=\"55px\">\r\n\n<br>Therefore, <img src=\"https://teacher.smartermaths.com.au/wp-content/uploads//2021/09/74b.svg\" width=\"55px\" height=\"55px\"> &nbsp;is the heaviest object.\r\n\n</div>\n"}]},{"vars":[{"varval":"Kate is comparing the weights of four different objects using a balance scale.\n\n<br>\n\nsm_img https://teacher.smartermaths.com.au/wp-content/uploads//2021/09/75.svg indent vpad\n\n<br>Which object is the heaviest?"},{"varval":"<div class=\"sm_img_inline\">\n\nBy comparing the weights of the objects:\r\n\n>> <img src=\"https://teacher.smartermaths.com.au/wp-content/uploads//2021/09/75c.svg\" width=\"55px\" height=\"55px\"> &nbsp;is heavier than &nbsp;<img src=\"https://teacher.smartermaths.com.au/wp-content/uploads//2021/09/75b.svg\" width=\"55px\" height=\"55px\">\r\n\n<br>\n\n>> <img src=\"https://teacher.smartermaths.com.au/wp-content/uploads//2021/09/75c.svg\" width=\"55px\" height=\"55px\"> &nbsp;is the same weight as &nbsp;<img src=\"https://teacher.smartermaths.com.au/wp-content/uploads//2021/09/75d.svg\" width=\"100px\" height=\"55px\">\r\n\n<br>\n\n>> <img src=\"https://teacher.smartermaths.com.au/wp-content/uploads//2021/09/75a.svg\" width=\"55px\" height=\"55px\"> &nbsp;is heavier than &nbsp;<img src=\"https://teacher.smartermaths.com.au/wp-content/uploads//2021/09/75d.svg\" width=\"100px\" height=\"55px\">\r\n\n<br>Therefore, <img src=\"https://teacher.smartermaths.com.au/wp-content/uploads//2021/09/75a.svg\" width=\"55px\" height=\"55px\"> is the heaviest object.\r\n\n</div>"}]}]

  127. <div class="sm_mode"> {{{question}}} </div>

    [{"vars":[{"varval":"<div class=\"sm_img_inline\">\n\nPeter spins the arrow on this spinner once.\n\n<br>\n\n>> <img src=\"https://teacher.smartermaths.com.au/wp-content/uploads/2021/09/51.png\" width=\"250\" height=\"250\">\n\n<br>What is the chance of the arrow landing on a &nbsp; <img src=\"https://teacher.smartermaths.com.au/wp-content/uploads/2021/09/51_m.png\" width=\"45\" height=\"45\"> ?\n\n</div>"},{"varval":"<div class=\"sm_img_inline center\">\n\nSince there are more &nbsp; <img src=\"https://teacher.smartermaths.com.au/wp-content/uploads/2021/09/51_m.png\" width=\"45\" height=\"45\"> &nbsp; than&nbsp; <img src=\"https://teacher.smartermaths.com.au/wp-content/uploads/2021/09/51_f.png\" width=\"50\" height=\"50\"> , it is \"likely\" that the arrow will land on &nbsp; <img src=\"https://teacher.smartermaths.com.au/wp-content/uploads/2021/09/51_m.png\" width=\"45\" height=\"45\"> .\n\n</div>\n"}]}]

  128. <div class="sm_mode"> {{{question}}} </div>

    [{"vars":[{"varval":"Allie made these patterns by painting some squares.\r\n\nAll small squares are the same size.\r\n\nWhich of these patterns has the largest area painted?\r\n"},{"varval":"Count the number of painted squares in each option:\n\n>1st pattern – 20 squares are painted\r\n\t\n>2nd pattern – 22 squares are painted\r\n\n>3rd pattern – 16 squares are painted\r\n\n>4th pattern – 18 squares are painted\r\n\n<br>Therefore, the pattern with the largest area painted is:\n\n{{{correctAnswer}}}\n"}]},{"vars":[{"varval":"Patrick made these patterns by painting some triangles.\r\n\nAll small triangles are the same size.\r\n\nWhich of these patterns has the largest area painted?\r\n"},{"varval":"Count the number of painted triangles in each option:\n\n>1st pattern – 8 triangles are painted\r\n\n>2nd pattern – 10 triangles are painted\r\n\n>3rd pattern – 6 triangles are painted\r\n\n>4th pattern – 9 triangles are painted\n\r\n\n<br>Therefore, the pattern with the largest area painted is:\n\n{{{correctAnswer}}}\n"}]},{"vars":[{"varval":"Bella made these patterns by painting some hexagons.\r\n\nAll of the hexagons are the same size.\r\n\nWhich of these patterns has the largest area painted?\r\n"},{"varval":"Count the number of painted hexagons in each option:\n\n>1st pattern – 10 hexagons are painted\r\n\n>2nd pattern – 7 hexagons are painted\r\n\n>3rd pattern – 9 hexagons are painted\r\n\n>4th pattern – 9 hexagons are painted\r\n\n<br>Therefore, the pattern with the largest area painted is:\n\n{{{correctAnswer}}}\n"}]}]

  129. This question had no tables. I worked backwards from the question. Not sure if they were picture graphs but they work. Needs difficulty

    <div class="sm_mode"> {{{question}}} </div>

    [{"vars":[{"varval":"Two groups were sent to the nearby sea to catch some fish.\r\n\r\n\nThe total number of fish caught is shown in the table below.\n\n<br>\r\n\nsm_img https://teacher.smartermaths.com.au/wp-content/uploads/2022/11/RAPH-Job-12-71_v0.svg 420 indent vpad\n\nHow many more fishes did Group 1 catch compared to Group 2."},{"varval":"Fishes caught by Group 1 = 6 x 5 = 30\r\n\nFishes caught by Group 2 = 4 x 5 = 20\r\n\nDifference = 30 – 20 = {{{correctAnswer}}} fishes\r\n"}]},{"vars":[{"varval":"Two farmers harvested apples from two different farms.\r\n\r\n\nThe total number of apples harvested is shown in the table below.\n\n<br>\n\nsm_img https://teacher.smartermaths.com.au/wp-content/uploads/2022/11/RAPH-Job-12-71_v1.svg 450 indent vpad\n\r\n\nHow many more apples were harvested at Farm B compared to Farm A."},{"varval":"Apples harvested at Farm A = 4 x 20 = 80\r\n\nApples harvested at Farm B = 9 x 20 = 180\r\n\nDifference = 180 – 80 = {{{correctAnswer}}} apples\r\n"}]},{"vars":[{"varval":"Two florists harvested flowers from two different gardens.\n\n\nThe total number of flowers harvested is shown in the table below.\n\n<br>\n\nsm_img https://teacher.smartermaths.com.au/wp-content/uploads/2023/08/garden-pictograph-min.svg 450 indent vpad\n\n\nHow many more flowers were harvested at Garden 2 compared to Garden 1."},{"varval":"Flowers harvested at Garden 1 = 8 x 6 = 48\n\nFlowers harvested at Garden 2 = 11 x 6 = 66\n\nDifference = 66 – 48 = {{{correctAnswer}}} flowers\n"}]},{"vars":[{"varval":"People attending a function were divided into two groups.\n\nThe total number of people attending is shown in the table below.\n\n<br>\n\nsm_img https://teacher.smartermaths.com.au/wp-content/uploads/2023/08/people-pictograph-min.svg 350 indent vpad\n\nHow many more people were in Group 2 compared to Group 1."},{"varval":"People in Group 1 = 7 x 30 = 210\n\nPeople in Group 2 = 10 x 30 = 300\n\nDifference = 300 – 210 = {{{correctAnswer}}} people \n"}]},{"vars":[{"varval":"Year 7 and Year 8 have been collecting cans for recycling.\n\nThe total number of cans collected is shown in the table below.\n\n<br>\n\nsm_img https://teacher.smartermaths.com.au/wp-content/uploads/2023/08/recycle-pictograph-2-min.svg 450 indent vpad\n\nHow many more cans were collected by Year 7 compared to Year 8."},{"varval":"Cans collected by Year 7 = 10 x 15 = 150\n\nCans collected by Year 8 = 4 x 15 = 60\n\nDifference = 150 – 60 = {{{correctAnswer}}} cans\n"}]},{"vars":[{"varval":"Two trucks were transporting pineapples to the farmers market.\n\n\nThe total number of pineapples being transported is shown in the table below.\n\n<br>\n\nsm_img https://teacher.smartermaths.com.au/wp-content/uploads/2023/08/Pineapple-pictograph-min.svg 450 indent vpad\n\n\nHow many more pineapples were transported by Truck 1 compared to Truck 2."},{"varval":"Pineapples transported by Truck 1 = 9 x 50 = 450\n\nPineapples transported by Truck 2 = 4 x 50 = 200\n\nDifference = 450 – 200 = {{{correctAnswer}}} pineapples"}]}]

  130. Needs difficulty

    <div class="sm_mode"> {{{question}}} </div>

    [{"vars":[{"varval":"What number is being represented by the arrow F?\n\nsm_img https://teacher.smartermaths.com.au/wp-content/uploads/2021/09/67.svg 350 indent vpad"},{"varval":"From the number line each interval is equal to 2.\r\n\nTherefore, the arrow F is pointing to the number 86.\r\n"}]},{"vars":[{"varval":"What number is being represented by the arrow E?\n\nsm_img https://teacher.smartermaths.com.au/wp-content/uploads/2021/09/68.svg 350 indent vpad"},{"varval":"From the number line each interval is equal to 2.\r\n\nTherefore, the arrow E is pointing to the number {{correctAnswer}}.\n"}]}]

  131. Raph 12 69-70

    <div class="sm_mode"> {{{question}}} </div>

    [{"vars":[{"varval":"Chester is drawing parallelograms on a square grid without crossing the sides of any other parallelograms.\r\n\nHe has 3 more points to join to form another parallelogram.\r\n\nsm_img https://teacher.smartermaths.com.au/wp-content/uploads/2021/09/69.svg 350 indent vpad\n\nTwo of the points are H2 and C0.\n\nWhich of the following is the third point to connect?\n"},{"varval":"By connecting the points H2, {{correctAnswer}} and C0 we obtain another parallelogram. \n\nsm_img https://teacher.smartermaths.com.au/wp-content/uploads/2021/09/69s.svg 350 indent vpad"}]},{"vars":[{"varval":"Rick is drawing 3 triangles on a square grid without crossing the sides of any other triangles.\r\n\nHe has 2 more points to join to form another triangle.\r\n\nsm_img https://teacher.smartermaths.com.au/wp-content/uploads/2021/09/70.svg 350 indent vpad\n\nOne of the points is E0\r\n\nWhich of the following is the second point to connect?\r\n"},{"varval":"By connecting the points E0 and B2 we obtain another triangle.\n\nsm_img https://teacher.smartermaths.com.au/wp-content/uploads/2021/09/70s.svg 350 indent vpad"}]},{"vars":[{"varval":"Salma is drawing parallelograms on a square grid without crossing the sides of any other parallelograms.\n\nShe has 3 more points to join to form another parallelogram.\n\nsm_img https://teacher.smartermaths.com.au/wp-content/uploads/2023/08/Algebra-NAP_10707_v2.svg 450 indent vpad\n\nTwo of the points are I2 and D0.\n\nWhich of the following is the third point to connect?"},{"varval":"By connecting the points I2, {{correctAnswer}} and D0 we obtain another parallelogram. \n\nsm_img https://teacher.smartermaths.com.au/wp-content/uploads/2023/08/Algebra-NAP_10707_v2ws.svg 450 indent vpad"}]},{"vars":[{"varval":"Olga is drawing parallelograms on a square grid without crossing the sides of any other parallelograms.\n\nShe has 3 more points to join to form another parallelogram.\n\nsm_img https://teacher.smartermaths.com.au/wp-content/uploads/2023/08/Algebra-NAP_10707_v2.svg 450 indent vpad\n\nTwo of the points are C2 and H4.\n\nWhich of the following is the third point to connect?"},{"varval":"By connecting the points C2, {{correctAnswer}} and H4 we obtain another parallelogram. \n\nsm_img https://teacher.smartermaths.com.au/wp-content/uploads/2023/08/Algebra-NAP_10707_v3ws.svg 450 indent vpad"}]},{"vars":[{"varval":"Aitana is drawing 3 triangles on a square grid without crossing the sides of any other triangles.\n\nShe has 2 more points to join to form another triangle.\n\nsm_img https://teacher.smartermaths.com.au/wp-content/uploads/2023/08/Algebra-NAP_10707_v4.svg 300 indent vpad\n\n\nOne of the points is E1.\n\nWhich of the following is the second point to connect?\n"},{"varval":"By connecting the points E1 and {{correctAnswer}} we obtain another triangle.\n\nsm_img https://teacher.smartermaths.com.au/wp-content/uploads/2023/08/Algebra-NAP_10707_v4ws.svg 300 indent vpad"}]}]

  132. Needs difficulty

    <div class="sm_mode"> {{{question}}} </div>

    [{"vars":[{"varval":"Which of these triangles have the least fraction of area shaded?"},{"varval":"All of the triangles have their area shaded by at least $\\dfrac{1}{2}$\n\nExcept for this figure:\n\r\n\n<br>\n\n{{{correctAnswer}}}\n\n<br>Which has only $\\dfrac{1}{4}$ of its area shaded."}]},{"vars":[{"varval":"Which of these parallelograms has the greatest fraction of area shaded?"},{"varval":"All of the parallelograms have their area shaded by less than $\\dfrac{1}{2}$ \r\n\nExcept for this figure:\n\n<br>\n\n\r\n\n{{{correctAnswer}}} \n\n<br>\n\n<br>Which has exactly $\\dfrac{1}{2}$ of its area shaded."}]}]

  133. <div class="sm_mode"> {{{question}}} </div>

    [{"vars":[{"varval":"Gina gave 5 of her friends 9 toys each.\r\n\nWhich of the following can be used to find how many toys, in total, Gina gave to her friends?\n"},{"varval":"sm_nogap Since Gina gave 5 of her friends 9 toys each:\n\n<div class=\"aligned\">\n\n|||\n|-:|-|\n|Total toys|= 5 $\\times$ 9|\n| |= {{{correctAnswer}}}|\n\n</div>\n\n"}]},{"vars":[{"varval":"Oli gave 4 of his friends 3 marbles each.\r\n\nWhich of the following can be used to find how many marbles, in total, Oli gave to his friends?\r\n"},{"varval":"sm_nogap Since Oli gave 4 of his friends 3 marbles each:\n\n<div class=\"aligned\">\n\n|||\n|-:|-|\n|Total marbles|= 4 $\\times$ 3|\n| |= {{{correctAnswer}}}|\n\n</div>"}]},{"vars":[{"varval":"Lance gave 7 of his friends 3 AFL cards each.\n\nWhich of the following can be used to find how many AFL cards, in total, Lance gave to his friends?"},{"varval":"sm_nogap Since Lance gave 7 of his friends 3 AFL cards each:\n\n<div class=\"aligned\">\n\n|||\n|-:|-|\n|Total cards|= 7 $\\times$ 3|\n| |= {{{correctAnswer}}}|\n\n</div>"}]},{"vars":[{"varval":"Jodie gave 5 of her friends 3 donuts each.\n\nWhich of the following can be used to find how many donuts, in total, Jodie gave to her friends?"},{"varval":"sm_nogap Since Jodie gave 5 of her friends 3 donuts each:\n\n<div class=\"aligned\">\n\n|||\n|-:|-|\n|Total donuts|= 5 $\\times$ 3|\n| |= {{{correctAnswer}}}|\n\n</div>"}]},{"vars":[{"varval":"Maureen gave 4 of her friends 8 Easter eggs.\n\nWhich of the following can be used to find how many Easter eggs, in total, Maureen gave to her friends?"},{"varval":"sm_nogap Since Maureen gave 4 of her friends 8 Easter eggs each:\n\n<div class=\"aligned\">\n\n|||\n|-:|-|\n|Total Easter eggs|= 4 $\\times$ 8|\n| |= {{{correctAnswer}}}|\n\n</div>"}]},{"vars":[{"varval":"Django gave 5 of his friends 9 coloured pencils.\n\nWhich of the following can be used to find how many coloured pencils, in total, Django gave to his friends?"},{"varval":"sm_nogap Since Django gave 5 of his friends 9 coloured pencils each:\n\n<div class=\"aligned\">\n\n|||\n|-:|-|\n|Total coloured pencils|= 5 $\\times$ 9|\n| |= {{{correctAnswer}}}|\n\n</div>"}]}]

  134. <div class="sm_mode"> {{{question}}} </div>

    [{"vars":[{"varval":"The table shows the total number of dogs that Timothy and Gabbi own.\n\n<br>\n\n<div class=\"outline\">\n\n> > | | German Shepherd| Poodle| Labrador|\n> > | :---: | :------: | :-----: | :------: |\n> > | Gabbi | 1 | 2 | 2 |\n> > | Timothy | 1 | 1 | 3 |\n\n</div>\n\n<br>They draw a picture graph showing the correct number of dogs they own altogether.\r\n\nWhich picture graph did they draw?\r\n"},{"varval":"German Shepherds = 1 +1 = 2\r\n\nPoodles = 2 + 1 = 3\r\n\nLabradors =2 + 3 = 5\r\n\nTherefore, the correct picture graph is:\n\n\r\n{{{correctAnswer}}}"}]}]

  135. <div class="sm_mode"> {{{question}}} </div>

    [{"vars":[{"varval":"Sarah travelled one thousand and eighty-six kilometres from Melbourne to Sydney.\n\n<br>\n\r\nsm_img https://teacher.smartermaths.com.au/wp-content/uploads//2021/09/42.svg 300 indent vpad\n\n<br>The distance can be written as:"},{"varval":"{{{correctAnswer}}}"}]}]

  136. SJ Add/subtract

    <div class="sm_mode"> {{{question}}} </div>

    [{"vars":[{"varval":"The scale is balanced.\n\n<br>\n\nsm_img https://teacher.smartermaths.com.au/wp-content/uploads//2021/09/39.svg 350 indent vpad\n\n<br>What is the weight of the square?"},{"varval":"sm_nogap Since the scale is balanced, both sides must weigh 37 grams.\n\n<div class=\"aligned\">\r\n\r\n| | |\r\n| ------------- | ---------- |\r\n| Weight of square | \\= 37 $–$ 23 |\r\n| | \\= {{{correctAnswer0}}} |\r\n\r\n</div>\r\n"}]},{"vars":[{"varval":"The scale is balanced.\n\n<br>\n\nsm_img https://teacher.smartermaths.com.au/wp-content/uploads/2023/06/balance-1_1a-min.svg 300 indent vpad\n\n<br>What is the weight of the square?"},{"varval":"sm_nogap Since the scale is balanced, both sides must weigh 45 kilograms.\n\n<div class=\"aligned\">\n\n| | |\n| ------------- | ---------- |\n| Weight of square | \\= 45 $–$ 19 |\n| | \\= {{{correctAnswer0}}} |\n\n</div>\n"}]},{"vars":[{"varval":"The scale is balanced.\n\n<br>\n\nsm_img https://teacher.smartermaths.com.au/wp-content/uploads/2023/06/balance-1_2a-min.svg 300 indent vpad\n\n<br>What is the weight of the circle?"},{"varval":"sm_nogap Since the scale is balanced, both sides must weigh 105 grams.\n\n<div class=\"aligned\">\n\n| | |\n| ------------- | ---------- |\n| Weight of circle | \\= 105 $–$ 71 |\n| | \\= {{{correctAnswer0}}} |\n\n</div>\n"}]},{"vars":[{"varval":"The scale is balanced.\n\n<br>\n\nsm_img https://teacher.smartermaths.com.au/wp-content/uploads/2023/06/balance-1_3-min.svg 300 indent vpad\n\n<br>What is the weight of the rectangle on the left?"},{"varval":"sm_nogap Since the scale is balanced, both sides must weigh the same.\n\n<div class=\"aligned\">\n\n| | |\n| ------------- | ---------- |\n| Weight of rectangle | \\= 39 $+$ 45 |\n| | \\= {{{correctAnswer0}}} |\n\n</div>\n"}]},{"vars":[{"varval":"The scale is balanced.\n\n<br>\n\nsm_img https://teacher.smartermaths.com.au/wp-content/uploads/2023/06/balance-1_4-min.svg 300 indent vpad\n\n<br>What is the weight of the rectangle on the left?"},{"varval":"sm_nogap Since the scale is balanced, both sides must weigh the same.\n\n<div class=\"aligned\">\n\n| | |\n| ------------- | ---------- |\n| Weight of rectangle | \\= 117 + 144 |\n| | \\= {{{correctAnswer0}}} |\n\n</div>\n"}]},{"vars":[{"varval":"The scale is balanced.\n\n<br>\n\nsm_img https://teacher.smartermaths.com.au/wp-content/uploads/2023/06/balance-1_5-min.svg 350 indent vpad\n\n<br>What is the weight of the circle?"},{"varval":"sm_nogap Since the scale is balanced, both sides must weigh 195 grams.\n\n<div class=\"aligned\">\n\n| | |\n| ------------- | ---------- |\n| Weight of circle | \\= 195 $–$ 106 |\n| | \\= {{{correctAnswer0}}} |\n\n</div>\n"}]}]

  137. <div class="sm_mode"> {{{question}}} </div>

    [{"vars":[{"varval":"This pattern is made by turning a tile.\n\n<br>\n\nsm_img https://teacher.smartermaths.com.au/wp-content/uploads//2021/09/36.svg 580 indent vpad\n\n<br>Which tile is missing from the pattern?"},{"varval":"{{{correctAnswer}}}"}]}]

  138. <div class="sm_mode"> {{{question}}} </div>

    [{"vars":[{"varval":"Jason folded this piece of paper along the dotted lines to make a model.\n\n<br>\n\nsm_img https://teacher.smartermaths.com.au/wp-content/uploads//2021/09/31.svg 250 indent3 vpad\n\n<br>Which model did Jason make?"},{"varval":"sm_img https://teacher.smartermaths.com.au/wp-content/uploads//2021/09/31sol.svg 450 indent vpad"}]},{"vars":[{"varval":"Michelle has a piece of paper and folded it along the dotted lines to make a figure.\n\n<br>\n\nsm_img https://teacher.smartermaths.com.au/wp-content/uploads//2021/09/32.svg?a 200 indent3 vpad\n\n<br>Which model did Michelle make?"},{"varval":"sm_img https://teacher.smartermaths.com.au/wp-content/uploads//2021/09/32sol.svg 450 indent vpad"}]},{"vars":[{"varval":"Christopher folded this piece of paper along the dotted lines into a model.\n\n<br>\n\nsm_img https://teacher.smartermaths.com.au/wp-content/uploads//2021/09/33.svg 310 indent3 vpad\n\n<br>Which model did Christopher make?"},{"varval":"sm_img https://teacher.smartermaths.com.au/wp-content/uploads//2021/09/33sol.svg 550 indent vpad"}]}]

  139. <div class="sm_mode"> {{{question}}} </div>

    [{"vars":[{"varval":"Gabriel has a book which has more than 628 pages but less than 682 pages.\n\nWhich of these could be the number of pages in Gabriel’s book?\r\n"},{"varval":"sm_nogap Check each option:\r\n\n>>665 is more than 628 and less than 682 (Correct)\r\n\n>>620 is less than 628 (Incorrect)\r\n\n>>689 is more than 682 (Incorrect)\r\n\n>>606 is less than 628 (Incorrect\r)\n"}]}]

  140. <div class="sm_mode"> {{{question}}} </div>

    [{"vars":[{"varval":"Some students were asked to name their favourite ice-cream flavour.\r\n\nThe table shows their responses.\n\n\r<br>\n\nsm_img https://teacher.smartermaths.com.au/wp-content/uploads//2021/09/27.svg 310 indent vpad\n\n<br>How many students were asked, altogether?"},{"varval":"<div class=\"aligned\">\r\n\r\n| | |\r\n| ------------- | ---------- |\r\n| Total number | \\= 16 + 14 + 14 |\r\n| | \\= {{{correctAnswer0}}} |\r\n\r\n</div>\r\n"}]}]

  141. <div class="sm_mode"> {{{question}}} </div>

    [{"vars":[{"varval":"Jake sold his surfboard and received these notes.\n\n<br>\n\nsm_img https://teacher.smartermaths.com.au/wp-content/uploads//2021/09/23.svg 550 indent vpad\n\nHow much did Jake sell his surfboard for?"},{"varval":"<div class=\"aligned\">\r\n\r\n| | |\r\n| ------------- | ---------- |\r\n| Total money | \\= (3 $\\times$ 50) + (2 $\\times$ 10) + 5 |\n| | = 150 + 20 + 5 |\n| | \\= {{{prefix0}}}{{{correctAnswer0}}} |\r\n\r\n</div>\r\n"}]}]

  142. Raph14-20 Raph14-21 Raph15-25 Raph15-27 SJ: Added_v4,v5,v6 Analog

    <div class="sm_mode"> {{{question}}} </div>

    [{"vars":[{"varval":"Which clock shows half-past five?"},{"varval":"{{{correctAnswer}}}\n\n"}]},{"vars":[{"varval":"Which clock shows half-past nine?"},{"varval":"{{{correctAnswer}}}\n\n"}]},{"vars":[{"varval":"Which clock shows quarter to 6?"},{"varval":"60 minutes = 1 hour\n\n15 minutes = quarter of 1 hour\n\n{{{correctAnswer}}}"}]},{"vars":[{"varval":"Which clock shows quarter past 11?"},{"varval":"60 minutes = 1 hour\n\n15 minutes = quarter of 1 hour\n\n{{{correctAnswer}}}"}]},{"vars":[{"varval":"Which clock shows half-past three?"},{"varval":"{{{correctAnswer}}}"}]},{"vars":[{"varval":"Which clock shows half-past eleven?"},{"varval":"{{{correctAnswer}}}"}]},{"vars":[{"varval":"Which clock shows half-past eight?"},{"varval":"{{{correctAnswer}}}"}]}]

  143. <div class="sm_mode"> {{{question}}} </div>

    [{"vars":[{"varval":"An apartment building has 9 flats.\n\nEach flat has 3 beds inside.\r\n\n<br>\n\nsm_img https://teacher.smartermaths.com.au/wp-content/uploads/2021/09/Raph14-18.png 250 indent3 vpad\n\n<br>How many beds are there in the apartment building, altogether?"},{"varval":"<div class=\"aligned\">\n\n| | |\n| --------------------- | ----------------- |\n| Number of beds| = 3 $\\times$ 9 |\n| | = {{{correctAnswer}}} |\n\n</div>"}]},{"vars":[{"varval":"A large wooden crate contains 5 boxes.\n\nEach box has 12 apples\r in it.\n\nsm_img https://teacher.smartermaths.com.au/wp-content/uploads/2023/06/box_apple-min.svg 250 indent vpad\n\nHow many apples are there in total?"},{"varval":"<div class=\"aligned\">\r\n\r\n| | |\r\n| ------------- | ---------- |\r\n| Total apples | \\= 5 $\\times$ 12 |\r\n| | \\= {{{correctAnswer}}} apples |\r\n\r\n</div>\r\n"}]},{"vars":[{"varval":"A clothes chest has 3 drawers\r.\n\nEach drawer has 8 pairs of football socks in it.\n\nsm_img https://teacher.smartermaths.com.au/wp-content/uploads/2023/06/drawers_3_a-min.svg 250 indent vpad\n\n\r\nHow many pairs of football socks are in the clothes chest altogether?"},{"varval":"<div class=\"aligned\">\r\n\r\n| | |\r\n| ------------- | ---------- |\r\n| Total pairs of socks | \\= 3 $\\times$ 8 |\r\n| | \\= {{{correctAnswer}}} |\r\n\r\n</div>\r\n"}]},{"vars":[{"varval":"A tram has 4 carriages.\n\nEach carriage has seating for 21 passengers.\n\nsm_img https://teacher.smartermaths.com.au/wp-content/uploads/2023/06/tramway-1294365.svg 200 indent vpad\n\n\nHow many seats are on the tram altogether?"},{"varval":"<div class=\"aligned\">\n\n| | |\n| ------------- | ---------- |\n| Total number of seats | \\= 4 $\\times$ 21 |\n| | \\= {{{correctAnswer}}} |\n\n</div>\n"}]},{"vars":[{"varval":"Jess buys 5 boxes of donuts.\n\nEach box contains 6 donuts.\n\nsm_img https://teacher.smartermaths.com.au/wp-content/uploads/2023/06/donut-2013343.svg 150 indent vpad\n\n\nHow many donuts has Jess purchased altogether?"},{"varval":"<div class=\"aligned\">\n\n| | |\n| ------------- | ---------- |\n| Total number of seats | \\= 5 $\\times$ 6 |\n| | \\= {{{correctAnswer}}} |\n\n</div>\n"}]},{"vars":[{"varval":"James is collecting eggs at his grandparent's farm. \n\nThere are 14 laying boxes and each laying box contains 10 eggs.\n\nsm_img https://teacher.smartermaths.com.au/wp-content/uploads/2023/06/chicken-45929.svg 300 indent vpad\n\n\nHow many eggs will James have to collect altogether?"},{"varval":"<div class=\"aligned\">\n\n| | |\n| ------------- | ---------- |\n| Total number of seats | \\= 14 $\\times$ 10 |\n| | \\= {{{correctAnswer}}} |\n\n</div>"}]}]

  144. <div class="sm_mode"> {{{question}}} </div>

    [{"vars":[{"varval":"Norman has these two shapes\n\nsm_img https://teacher.smartermaths.com.au/wp-content/uploads//2021/09/13.svg 200 indent vpad\n\nWhich shape could he make by joining them together?"},{"varval":"sm_img https://teacher.smartermaths.com.au/wp-content/uploads//2021/09/13-sol.svg 450 indent vpad"}]},{"vars":[{"varval":"Raven has these two shapes.\n\nsm_img https://teacher.smartermaths.com.au/wp-content/uploads//2021/09/14.svg 190 indent vpad"},{"varval":"sm_img https://teacher.smartermaths.com.au/wp-content/uploads//2021/09/14-sol.svg 480 indent vpad\n"}]},{"vars":[{"varval":"William has these two shapes.\n\nsm_img https://teacher.smartermaths.com.au/wp-content/uploads//2021/09/15.svg 200 indent vpad\n\n<br>Which shape could he make by joining them together?"},{"varval":"sm_img https://teacher.smartermaths.com.au/wp-content/uploads//2021/09/15_sol.svg 450 indent vpad"}]}]

  145. <div class="sm_mode"> {{{question}}} </div>

    [{"vars":[{"varval":"This is a map of a city.\n\nsm_img https://teacher.smartermaths.com.au/wp-content/uploads/2021/09/Math-Job-15-103.svg 350 indent vpad\n\nRaymond travelled along the path from the start to end.\r\n\nFor how many metres did Raymond travelled north?"},{"varval":"Mark all the path where he is travelling north.\n\nsm_img https://teacher.smartermaths.com.au/wp-content/uploads/2021/09/Math-Job-15-103Ans.svg 350 indent vpad\n\nThere are 7 squares that Raymond covered while travelling north.\r\n\nDistance that Raymond travelled north:\r\n\n7 x 20 = {{{correctAnswer0}}}"}]}]

  146. <div class="sm_mode"> {{{question}}} </div>

    [{"vars":[{"varval":"Amira has blocks with six rectangular faces.\r\n\nsm_img https://teacher.smartermaths.com.au/wp-content/uploads/2021/09/Math-Job-15-100.svg 40 indent3 vpad\n\n\r\nShe makes this model.\r\n\nsm_img https://teacher.smartermaths.com.au/wp-content/uploads/2021/09/Math-Job-15-100_1.svg 200 indent2 vpad\n\nWhat is the height of the model in centimetres?"},{"varval":"Height of a single block:\r\n\n24 ÷ 2 = 12 cm\r\n\r\n\nHeight of the block = 4 $\\times$ short edge of block\n\r\n\nLength of the short edge of the block:\r\n\n12 ÷ 4 = 3 cm\r\n\r\n\nHeight of the model:\r\n\n3 + 12 + 3 = {{{correctAnswer0}}}\r {{{suffix0}}}"}]},{"vars":[{"varval":"Henry has blocks with two square faces.\n\nsm_img https://teacher.smartermaths.com.au/wp-content/uploads/2021/09/Math-Job-15-101.svg 50 indent3 vpad\n\nHe makes this model.\n\nsm_img https://teacher.smartermaths.com.au/wp-content/uploads/2021/09/Math-Job-15-101_1.svg 150 indent2 vpad\n\nWhat is the height of the model in centimetres?"},{"varval":"Height of a single block:\r\n\n16 ÷ 2 = 8 cm\r\n\r\n\nLength of the side of the square block:\r\n\n8 ÷ 2 = 4 cm\r\n\r\n\nHeight of the model:\r\n\n4 + 8 + 4 = {{{correctAnswer0}}}\r {{{suffix0}}}"}]},{"vars":[{"varval":"Charles has blocks with six rectangular faces.\n\nsm_img https://teacher.smartermaths.com.au/wp-content/uploads/2021/09/Math-Job-15-102.svg 50 indent3 vpad\n\nHe makes this model.\n\nsm_img https://teacher.smartermaths.com.au/wp-content/uploads/2021/09/Math-Job-15-102_1.svg 240 indent2 vpad\n\nWhat is the height of the model in centimetres?"},{"varval":"Height of a single block:\r\n\n40 ÷ 2 = 20 cm\r\n\r\n\nLength of the short edge of the rectangular block:\r\n\n20 ÷ 4 = 5 cm\r\n\r\n\nHeight of the model:\r\n\n5 + 20 + 5 = {{{correctAnswer0}}}\r {{{suffix0}}}"}]}]

  147. Raph15-97 Raph15-98 Raph15-99

    <div class="sm_mode"> {{{question}}} </div>

    [{"vars":[{"varval":"Roger has these cards.\n\nsm_img https://teacher.smartermaths.com.au/wp-content/uploads/2021/08/Math-Job-15-97.svg 250 indent vpad\n\nHere are two ways Roger can arrange the cards to make a total.\n\nsm_img https://teacher.smartermaths.com.au/wp-content/uploads/2021/08/Math-Job-15-97_1.svg 340 indent vpad\n\nWhat is the largest total Roger can make using all the cards?"},{"varval":"Largest 3-digit number = 954\r\n\r\n<div class=\"aligned\">\n\n| | |\n| -:|-|\n|Largest number|= 954 + 2 |\n| |= {{{correctAnswer0}}}|\n\n</div>"}]},{"vars":[{"varval":"Patrick has these cards.\n\nsm_img https://teacher.smartermaths.com.au/wp-content/uploads/2021/09/Math-Job-15-99.svg 250 indent vpad\n\nHere are two ways he can arrange the cards to make a total.\n\nsm_img https://teacher.smartermaths.com.au/wp-content/uploads/2021/09/Math-Job-15-99_1.svg 340 indent vpad\n\nWhat is the largest total Patrick can make using all the cards?"},{"varval":"Largest 3-digit number = 874\n\r\n<div class=\"aligned\">\n\n| | |\n| -:|-|\n|Largest number|= 874 + 3 |\n| |= {{{correctAnswer0}}}|\n\n</div>"}]},{"vars":[{"varval":"Zander has these cards.\n\nsm_img https://teacher.smartermaths.com.au/wp-content/uploads/2021/08/Math-Job-15-98.svg 250 indent vpad\n\nHere are two ways he can arrange the cards to make a total.\n\nsm_img https://teacher.smartermaths.com.au/wp-content/uploads/2021/08/Math-Job-15-98_1.svg 350 indent vpad\n\nWhat is the largest total Zander can make using all the cards?"},{"varval":"Largest 3-digit number = 763\n\r\n<div class=\"aligned\">\n\n| | |\n| -:|-|\n|Largest number|= 763 + 1 |\n| |= {{{correctAnswer0}}}|\n\n</div>"}]}]

  148. Raph15-94 Raph15-95 Raph15-96

    <div class="sm_mode"> {{{question}}} </div>

    [{"vars":[{"varval":"Princess buys a slice of cake and a cup of tea.\n\nsm_img https://teacher.smartermaths.com.au/wp-content/uploads/2021/10/Math-Job-15-94.png 350 indent vpad\n\nShe pays with a $5 note and gets back $1.30 in change.\n\nHow much does the cake cost?"},{"varval":"<div class=\"aligned\">\n\n| | |\n| -:|-|\n|Cost of cake|= 5 $-$ (2.20 + 1.30) |\n| |= 5 $-$ 3.50 |\n| |= {{{prefix0}}}{{{correctAnswer0}}}|\n\n</div>"}]},{"vars":[{"varval":"Candice buys a lollipop and a chocolate bar.\n\nsm_img https://teacher.smartermaths.com.au/wp-content/uploads/2021/10/Math-Job-15-95.png 300 indent3 vpad\n\nShe pays with a $5 note and gets back exactly $1.00 in change.\n\nHow much does the lollipop cost?"},{"varval":"<div class=\"aligned\">\n\n| | |\n| -:|-|\n|Cost of lollipop|= 5 $-$ (2.90 + 1.00) |\n| |= 5 $-$ 3.90 |\n| |= {{{prefix0}}}{{{correctAnswer0}}}|\n\n</div>"}]},{"vars":[{"varval":"James buys a robot and a toy car from the toy store.\n\nsm_img https://teacher.smartermaths.com.au/wp-content/uploads/2021/10/Math-Job-15-96.png 350 indent3 vpad\n\nHe pays with a $20 note and gets back $2.20 in change.\n\nWhat price did James pay for the robot?"},{"varval":"<div class=\"aligned\">\n\n| | |\n| -:|-|\n|Cost of robot|= 20 $-$ (5.50 + 2.20) |\n| |= 20 $-$ 7.70 |\n| |= {{{prefix0}}}{{{correctAnswer0}}}|\n\n</div>"}]}]

  149. Raph15-91 Raph15-92 Raph15-93

    <div class="sm_mode"> {{{question}}} </div>

    [{"vars":[{"varval":"Leo cut a triangular prism from a cube to make this object.\r\n\n<br>\n\nsm_img https://teacher.smartermaths.com.au/wp-content/uploads/2021/08/Math-Job-15-91.svg 200 indent3 vpad\n\n<br>How many edges does the object have?"},{"varval":"Edges on top face = 5\n\nEdges on bottom face = 5\n\nVertical edges = 5\n\nTotal edges = 5 + 5 + 5 = {{{correctAnswer}}}"}]},{"vars":[{"varval":"Sebastian cut a rectangular prism from a cube to make this object.\r\n\n<br>\n\nsm_img https://teacher.smartermaths.com.au/wp-content/uploads/2021/08/Math-Job-15-92.svg 200 indent3 vpad\n\n<br>How many edges does the object have?"},{"varval":"Edges on top face = 8\n\nEdges on bottom face = 8\n\nVertical edges = 8\n\nTotal edges = 8 + 8 + 8 = {{{correctAnswer}}}"}]},{"vars":[{"varval":"Milat had a wooden cube.\n\nHe cut away half of the cube to make this object.\r\n\n<br>\n\nsm_img https://teacher.smartermaths.com.au/wp-content/uploads/2021/08/Math-Job-15-93.svg 200 indent vpad\n\n<br>How many edges does the object have?"},{"varval":"Edges on top face = 3\n\nEdges on bottom face = 3\n\nVertical edges = 3\n\nTotal edges = 3 + 3 + 3 = {{{correctAnswer}}}"}]},{"vars":[{"varval":"Lolo cut a \rpentagonal prism from a cube to make this object.\n\n<br>\n\nsm_img https://teacher.smartermaths.com.au/wp-content/uploads/2022/09/Geom_Raph15-91_v3.svg 180 indent3 vpad\n\n<br>How many edges does the object have?"},{"varval":"Edges on top face = 9\n\nEdges on bottom face = 9\n\nVertical edges = 9\n\nTotal edges = 9 + 9 + 9 = {{{correctAnswer}}}"}]},{"vars":[{"varval":"Lars cut 4 identical rectangular prisms from a cube to make this object.\n\n<br>\n\nsm_img https://teacher.smartermaths.com.au/wp-content/uploads/2022/09/Geom_Raph15-91_v4a.svg 200 indent3 vpad\n\n<br>How many edges does the object have?"},{"varval":"Edges on front face = 20\n\nEdges on back face = 20\n\nHorizontal edges = 20\n\nTotal edges = 20 + 20 + 20 = {{{correctAnswer}}}"}]},{"vars":[{"varval":"Lindley cut 4 identical triangular prisms from a cube to make this object.\n\n<br>\n\nsm_img https://teacher.smartermaths.com.au/wp-content/uploads/2022/09/Geom_Raph15-91_v5.svg 200 indent3 vpad\n\n<br>How many edges does the object have?"},{"varval":"Edges on front face = 16\n\nEdges on back face = 16\n\nHorizontal edges = 16\n\nTotal edges = 16 + 16 + 16 = {{{correctAnswer}}}"}]}]

  150. Raph15-88 Raph15-89 Raph15-90

    <div class="sm_mode"> {{{question}}} </div>

    [{"vars":[{"varval":"This table shows the types of pizzas a customer of a pizza restaurant ordered each month over a 12 month period.\n\n<div class=\"sm-table col1-color1 row1-color1 top-left-cell-hidden\">\n\n>| | Jan | Feb | Mar| Apr | May | Jun | Jul | Aug | Sep | Oct | Nov | Dec |\n| - | :-: |:-:|:-:|:-:|:-:|:-:|:-:|:-:|:-:|:-:|:-:|:-:|\n| Hawaiian | $\\checkmark$ || $\\checkmark$ | $\\checkmark$ |$\\checkmark$ ||| $\\checkmark$ | $\\checkmark$ || $\\checkmark$ ||\n| Pepperoni | $\\checkmark$ | $\\checkmark$ || $\\checkmark$ |$\\checkmark$ || $\\checkmark$ | $\\checkmark$ | $\\checkmark$ | $\\checkmark$ || $\\checkmark$ |\n| Cheese || $\\checkmark$ | $\\checkmark$ || | $\\checkmark$ |$\\checkmark$|| | $\\checkmark$ |$\\checkmark$||\n| Vegetarian | $\\checkmark$ || $\\checkmark$ |$\\checkmark$|| $\\checkmark$ | $\\checkmark$ | $\\checkmark$ ||| $\\checkmark$ |\n\n</div>\n\n<br>\n\nIn how many months did the customer order pepperoni pizza but not vegetarian pizza."},{"varval":"There are {{{correctAnswer}}} months where Faye ordered pepperoni but not vegetarian pizza:\n\n$\\rightarrow$ Feb,\r May\r, Sep, Oct, Dec"}]},{"vars":[{"varval":"This table shows the different flavours of ice cream Irene ate over a 12-month period.\n\n<div class=\"sm-table col1-color8 row1-color8 top-left-cell-hidden\">\n\n>| | Jan | Feb | Mar| Apr | May | Jun | Jul | Aug | Sep | Oct | Nov | Dec |\n| - | :-: |:-:|:-:|:-:|:-:|:-:|:-:|:-:|:-:|:-:|:-:|:-|\n| Chocolate | $\\checkmark$ || $\\checkmark$ | $\\checkmark$ |$\\checkmark$ ||| $\\checkmark$ | $\\checkmark$ || $\\checkmark$ ||\n| Strawberry | $\\checkmark$ | $\\checkmark$ || $\\checkmark$ |$\\checkmark$ || $\\checkmark$ | | $\\checkmark$ | $\\checkmark$ |||\n| Cookies n cream | $\\checkmark$ | $\\checkmark$ | $\\checkmark$ | $\\checkmark$ | | $\\checkmark$ |$\\checkmark$| | | $\\checkmark$ |$\\checkmark$| $\\checkmark$ |\n| Vanilla || $\\checkmark$ | $\\checkmark$ |$\\checkmark$|| $\\checkmark$ | | $\\checkmark$ |$\\checkmark$|$\\checkmark$| $\\checkmark$ |$\\checkmark$ |\n\n</div>\n\n<br>\n\nIn how many months did Irene eat vanilla ice cream but not chocolate ice cream."},{"varval":"There are {{{correctAnswer}}} months where Irene ate vanilla but not chocolate ice cream.\r\n\n$\\rightarrow$ Feb, Jun, Oct, Dec"}]},{"vars":[{"varval":"This table shows the months when a fruit grower picked different types of fruit.\n\n<div class=\"sm-table col1-color3 row1-color3 top-left-cell-hidden\">\n\n>| | Jan | Feb | Mar| Apr | May | Jun | Jul | Aug | Sep | Oct | Nov | Dec |\n| - | :-: |:-:|:-:|:-:|:-:|:-:|:-:|:-:|:-:|:-:|:-:|:-|\n| Apple | $\\checkmark$ | $\\checkmark$ | $\\checkmark$ | $\\checkmark$ ||||||| $\\checkmark$ | $\\checkmark$ |\n| Cherry | $\\checkmark$ | $\\checkmark$ |||||||$\\checkmark$ | $\\checkmark$ | $\\checkmark$ | $\\checkmark$ |\n| Peach ||| $\\checkmark$ | $\\checkmark$ | $\\checkmark$ | $\\checkmark$ |$\\checkmark$||||$\\checkmark$ ||\n| Orange |$\\checkmark$|||||| $\\checkmark$ | $\\checkmark$ | $\\checkmark$ | $\\checkmark$ | $\\checkmark$ | $\\checkmark$ |\n\n</div>\n\n<br>\n\nDuring how many months did the fruit grower pick cherries but not peaches."},{"varval":"There are {{{correctAnswer}}} months where the farmer harvested cherries but not peach.\r\n\n$\\rightarrow$ Jan, Feb, Sep, Oct, Dec"}]}]

  151. SJ Added Multiply/divide (v3-v5)

    <div class="sm_mode"> {{{question}}} </div>

    [{"vars":[{"varval":"A shop sells apples in boxes.\r\n\nA small box has 4 apples.\r\n\nA medium box has 9 apples.\r\n\nsm_img https://teacher.smartermaths.com.au/wp-content/uploads/2021/08/Math-Job-15-87.svg 300 indent vpad\n \r\n\r\nKira buys a total of 50 apples.\r\n\nShe buys 2 medium boxes and a number of small boxes.\r\n\nHow many small boxes does Kira buy?\r"},{"varval":"sm_nogap Number of apples in 2 medium boxes:\r\n\n>>2 x 9 = 18\r\n\nsm_nogap Number of apples needed:\r\n\n>>50 $-$ 18 = 32\r\n\nsm_nogap Number of small boxes:\r\n\n>>32 ÷ 4 = {{{correctAnswer0}}}\r"}]},{"vars":[{"varval":"A shop sells donuts in trays and boxes.\r\n\nEach tray holds 5 donuts.\r\n\nEach box holds 10 donuts.\r\n\nsm_img https://teacher.smartermaths.com.au/wp-content/uploads/2021/08/Math-Job-15-85.svg 200 indent vpad\n\nJudy buys a total of 60 donuts.\r\n\nShe buys 3 boxes and some trays.\r\n\nHow many trays does Judy buy?\r"},{"varval":"<div class=\"sm_mode\">\n\nsm_nogap Number of donuts in 3 boxes:\r\n\n>>3 x 10 = 30\r\n\nsm_nogap Number of donuts needed:\r\n\n>>60 $-$ 30 = 30\r\n\nsm_nogap Number of trays:\r\n\n>>30 ÷ 5 = {{{correctAnswer0}}}\r\n\n</div>"}]},{"vars":[{"varval":"A shop sells cookies in trays and boxes.\r\n\nEach tray holds 6 cookies.\r\n\nEach box holds 12 cookies.\r\n\nsm_img https://teacher.smartermaths.com.au/wp-content/uploads/2021/08/Math-Job-15-86.svg 200 indent vpad\n \r\n\r\nJ-Dawg buys a total of 90 cookies.\r\n\nHe buys 4 boxes and some trays.\r\n\nHow many trays does J-Dawg buy?\r"},{"varval":"sm_nogap Number of cookies in 4 boxes:\r\n\n>>4 x 12 = 48\r\n\nsm_nogap Number of cookies needed:\r\n\n>>90 $-$ 48 = 42\r\n\nsm_nogap Number of trays:\r\n\n>>42 ÷ 6 = {{{correctAnswer0}}}"}]},{"vars":[{"varval":"A shop sells pears in boxes.\n\nA small box has 3 pears.\n\nA medium box has 6 pears.\n\nsm_img https://teacher.smartermaths.com.au/wp-content/uploads/2023/07/pear-boxes-min.svg 200 indent vpad\n \n\nAmie buys a total of 45 pears.\n\nShe buys 4 medium boxes and a number of small boxes.\n\nHow many small boxes does Amie buy?\n\n"},{"varval":"sm_nogap Number of pears in 4 medium boxes:\n\n>>4 x 6 = 24\n\nsm_nogap Number of pears needed:\n\n>>45 $-$ 24 = 21\n\nsm_nogap Number of small boxes:\n\n>>21 ÷ 3 = {{{correctAnswer0}}}\n"}]},{"vars":[{"varval":"A shop sells capsicums in boxes.\n\nA small box has 3 capsicums.\n\nA medium box has 6 capsicums.\n\nsm_img https://teacher.smartermaths.com.au/wp-content/uploads/2023/07/bell-peppers-box-min.svg 300 indent vpad\n \n\nBillie buys a total of 27 capsicums.\n\nShe buys 2 medium boxes and a number of small boxes.\n\nHow many small boxes does Billie buy?\n"},{"varval":"sm_nogap Number of capsicums in 2 medium boxes:\n\n>>2 x 6 = 12\n\nsm_nogap Number of capsicums needed:\n\n>>27 $-$ 12 = 15\n\nsm_nogap Number of small boxes:\n\n>>15 ÷ 3 = {{{correctAnswer0}}}"}]},{"vars":[{"varval":"A shop sells cupcakes in boxes.\n\nA small box has 10 cupcakes.\n\nA medium box has 15 cupcakes.\n\nsm_img https://teacher.smartermaths.com.au/wp-content/uploads/2023/07/cakes-box-min.svg 400 indent vpad\n \n\nBodhi buys a total of 75 cupcakes.\n\nHe buys 3 medium boxes and a number of small boxes.\n\nHow many small boxes does Bodhi buy?\n"},{"varval":"sm_nogap Number of cupcakes in 3 medium boxes:\n\n>>3 x 15 = 45\n\nsm_nogap Number of cupcakes needed:\n\n>>75 $-$ 45 = 30\n\nsm_nogap Number of small boxes:\n\n>>30 ÷ 10 = {{{correctAnswer0}}}"}]}]

  152. Raph15-82 Raph15-83 Raph15-84

    <div class="sm_mode"> {{{question}}} </div>

    [{"vars":[{"varval":"Ramon cuts a 100-centimetre rod into two pieces.\r\n\nThe longer piece is four times the length of the shorter piece.\r\n\nWhat is the length of the longer piece?\r"},{"varval":"sm_nogap Equal parts = 4 + 1 = 5\n\n<div class=\"aligned\">\n\n|||\n|-:|-|\n|Length of 1 part |= 100 $\\div$ 5|\n||= 20 centimetres|\n\n</div>\n\n<br>\n\n<div class=\"aligned\">\n\n|||\n|-:|-|\n|Length of longer piece |= 4 $\\times$ 20|\n||= {{{correctAnswer0}}} {{{suffix0}}}|\n\n</div>"}]},{"vars":[{"varval":"Lina cuts a 12-metre length of ribbon into two pieces.\r\n\nThe longer piece is five times the length of the shorter piece.\r\n\nWhat is the length of the longer piece?\r"},{"varval":"sm_nogap Equal parts = 5 + 1 = 6\n\n<div class=\"aligned\">\n\n|||\n|-:|-|\n|Length of 1 part |= 12 $\\div$ 6|\n||= 2 metres|\n\n</div>\n\n<br>\n\n<div class=\"aligned\">\n\n|||\n|-:|-|\n|Length of longer piece |= 5 $\\times$ 2|\n||= {{{correctAnswer0}}} {{{suffix0}}}|\n\n</div>"}]},{"vars":[{"varval":"Karen cuts a 9-centimetre stick into two pieces.\r\n\nThe longer piece is two times the length of the shorter piece.\r\n\nWhat is the length of the longer piece?\r"},{"varval":"sm_nogap Equal parts = 2 + 1 = 3\n\n<div class=\"aligned\">\n\n|||\n|-:|-|\n|Length of 1 part |= 9 $\\div$ 3|\n||= 3 centimetres|\n\n</div>\n\n<br>\n\n<div class=\"aligned\">\n\n|||\n|-:|-|\n|Length of longer piece |= 2 $\\times$ 3|\n||= {{{correctAnswer0}}} {{{suffix0}}}|\n\n</div>"}]}]

  153. Raph15-80 Raph15-81

    <div class="sm_mode"> {{{question}}} </div>

    [{"vars":[{"varval":"Monica hangs 3 flags on a horizontal pole to make a banner.\n\nAny flags next to each other share a pin.\n\nShe uses 4 pins.\r\n\n<br>\n\nsm_img https://teacher.smartermaths.com.au/wp-content/uploads/2021/08/Math-Job-15-80.svg 350 indent3 vpad\n\n<br>How many pins would Monica use to hang 8 flags next to each other?"},{"varval":"1 flag = 2 pins\n\n2 flags = 3 pins\n\n$\\rightarrow$ 8 flags = {{{correctAnswer0}}} pins"}]},{"vars":[{"varval":"Light posts are placed every 100 metres on a road.\r\n\nFor the 300 metres of road below, there are 4 light posts.\r\n\n<br>\n\nsm_img https://teacher.smartermaths.com.au/wp-content/uploads/2021/08/Math-Job-15-79.svg 550 indent vpad\n\n<br>How many lights posts are there for a 900 metre road?"},{"varval":"100 metre road = 2 light posts\n\n200 metre road = 3 light posts\n\n$\\rightarrow$ 900 metre road = {{{correctAnswer0}}} light posts"}]},{"vars":[{"varval":"Electric towers are installed for every 2 kilometres.\r\n\nFor the 6 km distance below, there are 4 electric towers.\r\n\n<br>\n\nsm_img https://teacher.smartermaths.com.au/wp-content/uploads/2021/08/Math-Job-15-81.svg 550 indent vpad\n\n<br>How many electric towers are there for a distance of 12 km?"},{"varval":"2 km distance = 2 towers\n\n4 km distance = 3 towers\n\n$\\rightarrow$ 12 km distance = (12 $\\div$ 2) + 1 = {{{correctAnswer0}}} towers"}]}]

  154. Raph15-77 Raph15-78

    <div class="sm_mode"> {{{question}}} </div>

    [{"vars":[{"varval":"Josie put one shape on top of another shape to make this figure.\n\n<br>\n\nsm_img https://teacher.smartermaths.com.au/wp-content/uploads/2021/08/Math-Job-15-76.svg 120 indent vpad\n\n<br>Which two shapes could Josie have used?"},{"varval":"sm_img https://teacher.smartermaths.com.au/wp-content/uploads/2021/08/Math-Job-15-76d.svg 150 indent2 vpad\n\n<br>\n\nsm_img https://teacher.smartermaths.com.au/wp-content/uploads/2021/08/Math-Job-15-76Ans.svg 100 indent3 vpad"}]},{"vars":[{"varval":"Tim put one shape on top of another shape to make this figure.\n\nsm_img https://teacher.smartermaths.com.au/wp-content/uploads/2021/08/Math-Job-15-77.svg 155 indent3 vpad\n\nWhich two shapes could Tim have used?"},{"varval":"{{{correctAnswer}}}\n\n<br>\n\nsm_img https://teacher.smartermaths.com.au/wp-content/uploads/2021/08/Math-Job-15-77Ans.svg 150 indent3 vpad"}]},{"vars":[{"varval":"Ana put one shape on top of another shape to make this figure.\n\n<br>\n\nsm_img https://teacher.smartermaths.com.au/wp-content/uploads/2021/08/Math-Job-15-78.svg 170 indent3 vpad\n\n<br>Which two shapes could Ana have used?"},{"varval":"{{{correctAnswer}}}\n\n<br>\n\nsm_img https://teacher.smartermaths.com.au/wp-content/uploads/2021/08/Math-Job-15-78Ans.svg 170 indent3 vpad"}]}]

  155. <div class="sm_mode"> {{{question}}} </div>

    [{"vars":[{"varval":"Jack has $2.20 in 20-cent coins.\r\n\nHow many 20-cent coins does Jack have?\r"},{"varval":"One strategy:\n\n$2.20 = 220 cents\r\n\n>5 $\\times$ 20 = 100 cents\r\n\n>10 $\\times$ 20 = 200 cents\r\n\t\n>11 $\\times$ 20 = 220 cents\r\n\n<br>Therefore, Jack has {{{correctAnswer0}}} 20-cent coins.\r"}]},{"vars":[{"varval":"Kim has $1.30 in 5-cent coins.\r\n\nHow many 5-cent coins does Kim have?\r"},{"varval":"One strategy:\n\n$1.30 = 130 cents\r\n\n>10 $\\times$ 5 = 50 cents\r\n\n>20 $\\times$ 5 = 100 cents\r\n\t\n>26 $\\times$ 5 = 130 cents\r\n\n<br>Therefore, Kim has {{{correctAnswer0}}} 5-cent coins.\r"}]},{"vars":[{"varval":"Hailey has $1.80 in 5-cent coins.\r\n\nHow many 5-cent coins does Hailey have?\r"},{"varval":"One strategy:\n\n$1.80 = 180 cents\r\n\n>10 $\\times$ 5 = 50 cents\r\n\n>20 $\\times$ 5 = 100 cents\r\n\n>30 $\\times$ 5 = 150 cents\r\n\t\n>36 $\\times$ 5 = 180 cents\r\n\n<br>Therefore, Hailey has {{{correctAnswer0}}} 5-cent coins.\r"}]},{"vars":[{"varval":"Danielle has $4.60 in 20-cent coins.\n\nHow many 20-cent coins does Danielle have?"},{"varval":"$4.60 = 460 cents\r\n\n>5 x 20 = 100 cents\r\n\n>10 x 20 = 200 cents\n\n>20 x 20 = 400 cents\r\n\n>23 x 20 = 460 cents\r\n\n<br>\nTherefore, Danielle has {{{correctAnswer0}}} 20-cent coins.\r"}]},{"vars":[{"varval":"Herb has $12.50 in 50-cent coins.\n\nHow many 50-cent coins does Herb have?"},{"varval":">2 x 50 = $1\n\n>20 x 50 = $10\n\n>25 x 50 = $12.50\n\n<br>\nTherefore, Herb has {{{correctAnswer0}}} 50-cent coins.\r"}]}]

  156. <div class="sm_mode"> {{{question}}} </div>

    [{"vars":[{"varval":"This graph shows how much money Benny saves each month.\n\n<br>\n\nsm_img https://teacher.smartermaths.com.au/wp-content/uploads/2021/08/Math-Job-15-70.svg 500 indent2 vpad\n\n<br>On how many months did Benny save at least $55?"},{"varval":"Months where Benny saved at least $55:\r\n\nFebruary, March, May, June, September and November.\n\n$\\rightarrow$ Saved at least $55 in {{{correctAnswer}}} months"}]},{"vars":[{"varval":"This graph shows how many pizzas Johan cooks each day.\n\n<br>\n\nsm_img https://teacher.smartermaths.com.au/wp-content/uploads/2021/08/Math-Job-15-71.svg 500 indent3 vpad\n\n<br>On how many days did Johan bake at least 5 pizzas?"},{"varval":"Days where 5 or more pizzas were cooked:\n\nMonday, Wednesday, Saturday and Sunday.\n\n$\\rightarrow$ At least 5 pizzas cooked on {{{correctAnswer}}} days."}]},{"vars":[{"varval":"This graph shows how many customers a barber shop has each day over a week.\n\n<br>\n\nsm_img https://teacher.smartermaths.com.au/wp-content/uploads/2021/08/Math-Job-15-72.svg 500 indent3 vpad\n\n<br>On how many days did the barber shop have at least 6 customers?"},{"varval":"Days where shop had 6 customers or more:\n\nMonday, Thursday and Sunday\n\n$\\rightarrow$ At least 6 customers on {{{correctAnswer}}} days"}]}]

  157. Raph15-68 Raph15-69

    <div class="sm_mode"> {{{question}}} </div>

    [{"vars":[{"varval":"Dorothy has this picture on her computer.\n\n<br>\n\nsm_img https://teacher.smartermaths.com.au/wp-content/uploads/2021/08/Math-Job-15-67.svg 150 indent3 vpad\n\n<br>She makes the picture half as high and twice as wide.\n\n\r\nHow will the picture look after he does that?\r"},{"varval":"{{{correctAnswer}}}"}]},{"vars":[{"varval":"Carmella is given this picture in her art class.\n\n<br>\n\nsm_img https://teacher.smartermaths.com.au/wp-content/uploads/2021/08/Math-Job-15-68.svg 150 indent3 vpad\n\n<br>She redraws the picture twice as wide with the same height.\n\nWhat will Carmella's drawing look like?\r"},{"varval":"{{{correctAnswer}}}"}]},{"vars":[{"varval":"Greg has this picture on his laptop.\n\n<br>\n\nsm_img https://teacher.smartermaths.com.au/wp-content/uploads/2021/08/Math-Job-15-69.svg 150 indent3 vpad\n\n<br>He makes a copy of the picture with the same width but half as high.\n\n\r\nHow will the picture look now?\r"},{"varval":"{{{correctAnswer}}}"}]}]

  158. Raph15-65 Raph15-66

    <div class="sm_mode"> {{{question}}} </div>

    [{"vars":[{"varval":"This table shows the favourite sport of 600 people.\n\n<div class=\"outline\">\n\n> > | Favourite Sport | Number of people|\n> > | ------ | :----------------: |\n> > | Basketball| 245 |\n> > | Baseball| 132 |\n> > | Soccer| 120 |\n> > | Tennis | 103 |\n\n</div>\n<br>\nWhich of these is true?"},{"varval":"Number who prefer baseball = 132\n\nNumber who prefer tennis = 103\n\nCorrect statement: {{{correctAnswer}}}"}]},{"vars":[{"varval":"This table shows the favourite fruit of 500 people.\n\n<div class=\"outline\">\n\n> > | Favourite Fruit | Number of people|\n> > | ------ | :----------------: |\n> > | Apple | 85 |\n> > | Orange | 92 |\n> > | Banana | 67 |\n> > | Strawberry | 256 |\n\n</div>\n\n<br>\nWhich of these is true?"},{"varval":"Number who like strawberries = 256\n\nTotal people = 500\n\nCorrect statement: {{{correctAnswer}}}"}]},{"vars":[{"varval":"This table shows the grade level of 800 students.\n\n<div class=\"outline\">\n\n> > | Grade level | Number of people|\n> > | ------ | :----------------: |\n> > | Grade 1 | 176 |\n> > | Grade 2 | 155 |\n> > | Grade 3 | 298 |\n> > | Grade 4 | 171 |\n\n</div>\n\n<br>\nWhich of these is true?"},{"varval":"Number of students in 3rd grade = 298\n\nTotal students = 800\n\nCorrect statement: {{{correctAnswer}}}"}]}]

  159. <div class="sm_mode"> {{{question}}} </div>

    [{"vars":[{"varval":"Lyle had a circular piece of paper.\n\n\r\nHe folded it in half twice and cut a piece out as shown.\r\n\nsm_img https://teacher.smartermaths.com.au/wp-content/uploads/2021/08/Math-Job-15-63.svg 450 indent vpad\n\nHow will the piece of paper look when he unfolds it?\n\nsm_img https://teacher.smartermaths.com.au/wp-content/uploads/2021/08/Math-Job-15-63opt.svg 450 indent vpad\n"},{"varval":"The top-left quadrant of paper {{{correctAnswer}}} is similar to Lyle’s paper.\n\nsm_img https://teacher.smartermaths.com.au/wp-content/uploads/2021/08/Math-Job-15-63ans.svg 150 indent vpad\n"}]}]

  160. Raph15-61 Raph15-62 Raph15-63

    <div class="sm_mode"> {{{question}}} </div>

    [{"vars":[{"varval":"Jane had a circular piece of paper.\n\r\nShe folded it in half twice and cut a piece out as shown.\r\n\n<br>\n\nsm_img https://teacher.smartermaths.com.au/wp-content/uploads/2021/08/Math-Job-15-61.svg 400 indent vpad\n\nWhich image shows the paper after it has been unfolded?\n\nsm_img https://teacher.smartermaths.com.au/wp-content/uploads/2021/08/Math-Job-15-61opt.svg 400 indent vpad"},{"varval":"The shape cut out in the top left quadrant is the same in each quadrant.\n\n<br>\n\nsm_img https://teacher.smartermaths.com.au/wp-content/uploads/2021/08/Math-Job-15-61ans.svg 200 indent3 vpad\n\n<br>The unfolded paper is {{{correctAnswer}}}."}]},{"vars":[{"varval":"Paula had a circular piece of paper.\n\r\nShe folded it in half twice and cut a piece out as shown.\r\n\n<br>\n\nsm_img https://teacher.smartermaths.com.au/wp-content/uploads/2021/08/Math-Job-15-62.svg 400 indent vpad\n\nWhich image shows the paper after it has been unfolded?\n\nsm_img https://teacher.smartermaths.com.au/wp-content/uploads/2021/08/Math-Job-15-62opt.svg 400 indent vpad"},{"varval":"The shape cut out in the top left quadrant is the same in each quadrant.\n\nsm_img https://teacher.smartermaths.com.au/wp-content/uploads/2021/08/Math-Job-15-62ans.svg 150 indent3 vpad\n\n<br>The unfolded paper looks like {{{correctAnswer}}}."}]},{"vars":[{"varval":"Lin had a circular piece of paper.\n\nShe folded it in half twice and cut a piece out as shown.\r\n\n<br>\n\nsm_img https://teacher.smartermaths.com.au/wp-content/uploads/2021/08/Math-Job-15-63.svg 450 indent vpad\n\nWhich image shows the paper after it has been unfolded?\n\nsm_img https://teacher.smartermaths.com.au/wp-content/uploads/2021/08/Math-Job-15-63opt.svg 450 indent vpad"},{"varval":"The shape cut out in the top left quadrant is the same in each quadrant.\n\nsm_img https://teacher.smartermaths.com.au/wp-content/uploads/2021/08/Math-Job-15-63ans.svg 150 indent3 vpad\n\n<br> <br>The unfolded paper looks like option {{{correctAnswer}}}."}]}]

  161. RAPH12-1

    <div class="sm_mode"> {{{question}}} </div>

    [{"vars":[{"varval":"Yohan went to the store to buy some chips.\r\n\nIt costs $15 to buy 5 bags of chips.\r\n\nIf each bag of chips costs the same, how many bags of chips could he buy for $54?\r\n"},{"varval":"<div class=\"aligned\">\n\n| | |\n| --------------------- | -------------- |\n| Cost of 1 bag | \\= 15 $\\div$ 5 |\n| | \\= $3 |\n\n</div>\n\n<br>\n\n<div class=\"aligned\">\n\n| | |\n| --------------------- | -------------- |\n| $\\therefore$ Number of bags | \\= 54 $\\div$ 3 |\n| | \\= {{{correctAnswer}}} bags |\n\n</div>\n\n"}]},{"vars":[{"varval":"Julia wants to buy a T-shirt in a mall.\r\n\nA certain shop sells 2 T-shirts for the cost of $14.\r\n\nIf each T-shirt costs the same, how many T-shirts can she buy if she has $91?\r\n"},{"varval":"<div class=\"aligned\">\n\n| | |\n| --------------------- | -------------- |\n| Cost of 1 shirt | \\= 14 $\\div$ 2 |\n| | \\= $7 |\n\n</div>\n\n<br>\n\n<div class=\"aligned\">\n\n| | |\n| --------------------- | -------------- |\n| $\\therefore$ Number of bags | \\= 91 $\\div$ 7 |\n| | \\= {{{correctAnswer}}} shirts |\n\n</div>"}]},{"vars":[{"varval":"Bryn went to the store to buy some apples.\n\nIt costs $16 to buy 4 kilograms of apples.\n\nIf each kilogram of apples costs the same, how many kilograms of apples could he buy for $68?\n"},{"varval":"<div class=\"aligned\">\n\n| | |\n| --------------------- | -------------- |\n| Cost of 1 kilogram | \\= 16 $\\div$ 4 |\n| | \\= $4 |\n\n</div>\n\n<br>\n\n<div class=\"aligned\">\n\n| | |\n| --------------------- | -------------- |\n| $\\therefore$ Number of kilograms | \\= 68 $\\div$ 4 |\n| |= (60 $\\div$ 4) + (8 $\\div$ 4) |\n| |= 15 + 2 |\n| | \\= {{{correctAnswer}}} kilograms |\n\n</div>\n\n"}]},{"vars":[{"varval":"Ingrid went to the market to buy some flowers.\n\nIt costs $42 to buy 6 bunches of flowers.\n\nIf each bunch of flowers costs the same, how many bunches of flowers could she buy for $84?\n"},{"varval":"<div class=\"aligned\">\n\n| | |\n| --------------------- | -------------- |\n| Cost of 1 bunch | \\= 42 $\\div$ 6 |\n| | \\= $7 |\n\n</div>\n\n<br>\n\n<div class=\"aligned\">\n\n| | |\n| --------------------- | -------------- |\n| $\\therefore$ Number of bunches | \\= 84 $\\div$ 7 |\n| | \\= {{{correctAnswer}}} bunches |\n\n</div>\n\n"}]},{"vars":[{"varval":"Ester went to the market to buy some potatoes.\n\nIt costs $27 to buy 9 bags of potatoes.\n\nIf each bag of potatoes costs the same, how many bags of potatoes could she buy for $90?\n"},{"varval":"<div class=\"aligned\">\n\n| | |\n| --------------------- | -------------- |\n| Cost of 1 bag | \\= 27 $\\div$ 9 |\n| | \\= $3 |\n\n</div>\n\n<br>\n\n<div class=\"aligned\">\n\n| | |\n| --------------------- | -------------- |\n| $\\therefore$ Number of bags | \\= 90 $\\div$ 3 |\n| | \\= {{{correctAnswer}}} bags |\n\n</div>\n\n"}]},{"vars":[{"varval":"Marcus went to the store to buy some socks.\n\nIt costs $28 to buy 4 pairs of socks.\n\nIf each pair of socks costs the same, how many pairs of socks could he buy for $63?\n"},{"varval":"<div class=\"aligned\">\n\n| | |\n| --------------------- | -------------- |\n| Cost of 1 pair of socks | \\= 28 $\\div$ 4 |\n| | \\= $7 |\n\n</div>\n\n<br>\n\n<div class=\"aligned\">\n\n| | |\n| --------------------- | -------------- |\n| $\\therefore$ Number of pairs of socks | \\= 63 $\\div$ 7 |\n| | \\= {{{correctAnswer}}} pairs|\n\n</div>\n\n"}]}]

  162. Raph15-59 Raph15-60

    <div class="sm_mode"> {{{question}}} </div>

    [{"vars":[{"varval":"sm_img https://teacher.smartermaths.com.au/wp-content/uploads/2021/08/Math-Job-15-58.svg 440 indent vpad\n\nCatherine’s birthday is October 27.\n\nKyle’s birthday is 8 days after Catherine’s birthday.\n\nOn which day of the week is Kyle’s birthday?"},{"varval":"Count 8 days after October 27.\n\n\rIt will be {{{correctAnswer}}}.\r\n\nsm_img https://teacher.smartermaths.com.au/wp-content/uploads/2021/08/Math-Job-15-58Ans.svg 350 indent vpad"}]},{"vars":[{"varval":"sm_img https://teacher.smartermaths.com.au/wp-content/uploads/2021/08/Math-Job-15-59.svg 450 indent vpad\n\nMark’s birthday is November 29.\n\nRon’s birthday is 5 days after Mark’s birthday.\n\nOn which day of the week is Ron’s birthday?\r"},{"varval":"Count 5 days after November 29.\r\n\nIt will be {{{correctAnswer}}}.\r\n\nsm_img https://teacher.smartermaths.com.au/wp-content/uploads/2021/08/Math-Job-15-59Ans.svg 350 indent vpad"}]},{"vars":[{"varval":"sm_img https://teacher.smartermaths.com.au/wp-content/uploads/2021/08/Math-Job-15-60.svg 450 indent vpad\n\nHarold’s birthday is December 8.\n\nTyrone’s birthday is 9 days before Harold’s birthday.\n\nOn which day of the week is Tyrone’s birthday?\r"},{"varval":"Count 9 days before December 8.\n\nIt will be {{{correctAnswer}}}.\n\r\nsm_img https://teacher.smartermaths.com.au/wp-content/uploads/2021/08/Math-Job-15-60Ans.svg 350 indent vpad"}]}]

  163. <div class="sm_mode"> {{{question}}} </div>

    [{"vars":[{"varval":"Nora shaded these 4 shapes on grid paper.\n\nWhich shape has the **least** shading?\r"},{"varval":"Count the number of shaded squares for each shape (2 half triangles are equal to 1 square).\n\r\n>A = 12\r\n\n>B = 9\r\n\n>C = 13\n\n>D = 12\r\n\r\n\n<br>The shape with the least shading is:\n\n{{{correctAnswer}}}"}]},{"vars":[{"varval":"Karen shaded these 4 shapes on grid paper.\n\n\rWhich shape has the **least** shading?\r"},{"varval":"Count the number of shaded squares for each shape (2 half triangles are equal to 1 square).\n\n>A = 11\n\n>B = 12\r\n\n>C = 15\r\n\n>D = 9\r\n\r\n\n<br>The shape with the least shading is\n\n{{{correctAnswer}}}\n"}]},{"vars":[{"varval":"Nobe shaded these 4 shapes on grid paper.\n\nWhich shape has the **least** shading?\r"},{"varval":"Count the number of shaded squares for each shape (2 half triangles are equal to 1 square).\r\n\n>A = 12\r\n\n>B = 16\r\n\n>C = 13\r\n\n>D = 15\r\n\r\n\n<br>The shape with the least shading. is\n\n{{{correctAnswer}}}"}]}]

  164. Raph15-53 Raph15-54

    <div class="sm_mode"> {{{question}}} </div>

    [{"vars":[{"varval":"The table shows the weights of 5 friends.\n\n<div class=\"outline\">\n\n> |Name |Annie|Brian|Hamza|Vivian|Cade|\n> |:-:|:-:|:-:|:-:|:-:|:-:|\n|Weight (pounds)|110|126|119|122|135|\n\n</div>\n\n<br>\nWho is 9 pounds lighter than the heaviest person?"},{"varval":"Heaviest person &nbsp;$\\rightarrow$ &nbsp;Cade at 135 pounds.\n\n\r9 pounds lighter = 135 $-$ 9 = 126 pounds\r\n\n$\\therefore$ {{{correctAnswer}}}"}]},{"vars":[{"varval":"The table shows the cost of 5 items.\n\n<div class=\"outline\">\n\n> > |Name |Bicycle|Phone|Bag|Shoes|Laptop|\n> > |:-:|:-:|:-:|:-:|:-:|:-:|\n|Price|$597|$522|$468|$471|$556|\n\n</div>\n\n<br>\nWhich item is $41 cheaper than the most expensive item?"},{"varval":"Most expensive &nbsp;$\\rightarrow$ &nbsp;Bicycle at $597.\n\n\\$41 cheaper = 597 $-$ 41 = $556 \n\n$\\therefore$ {{{correctAnswer}}}"}]},{"vars":[{"varval":"The table shows the ages of 5 grandparents.\n\n<div class=\"outline\">\n\n> > |Name |Linda|Barbara|William|James|Mary|\n> > |:-:|:-:|:-:|:-:|:-:|:-:|\n|Age|92|85|74|98|86|\n\n</div>\n\n<br>\nWho is 6 years younger than the oldest person?"},{"varval":"Oldest &nbsp;$\\rightarrow$ &nbsp;James at 98.\n\n6 years younger = 98 $-$ 6 = 92 \n\n$\\therefore$ {{{correctAnswer}}}"}]},{"vars":[{"varval":"The table shows the highest individual scores in cricket of 5 batsmen.\n\n<div class=\"outline\">\n\n> > |Name |Vivian|Ricky|Gary|Darryl|Alistair|\n> > |:-:|:-:|:-:|:-:|:-:|:-:|\n|Score|291|257|275|277|263|\n\n</div>\n\n<br>\nWho has a highest score of 14 runs less than the highest individual score of all 5 batsmen?"},{"varval":"Highest individual score &nbsp;$\\rightarrow$ &nbsp;Vivian at 291.\n\n14 runs less = 291 $-$ 14 = 277 \n\n$\\therefore$ {{{correctAnswer}}}"}]},{"vars":[{"varval":"The table shows the quokka population in a nature reserve over a 5-month period.\n\n<div class=\"outline\">\n\n> > |Month |May|June|July|August|September|\n> > |:-:|:-:|:-:|:-:|:-:|:-:|\n|Population|352|379|355|339|382|\n\n</div>\n\n<br>\nWhich month recorded a quokka population of 27 less than the highest population over the 5-month period?"},{"varval":"Highest population &nbsp;$\\rightarrow$ &nbsp;September at 382\n\n382 $-$ 27 = 355 \n\n$\\therefore$ {{{correctAnswer}}}"}]},{"vars":[{"varval":"The table shows the number of sea turtles tagged each month by a research group over a five month period.\n\n<div class=\"outline\">\n\n> > |Month |Aug|Sep|Oct|Nov|Dec|\n> > |:-:|:-:|:-:|:-:|:-:|:-:|\n|Turtles tagged|34|41|28|46|39|\n\n</div>\n\n<br>\nWhich month recorded the number of turtles tagged at 7 less than highest number tagged over the five months?"},{"varval":"Highest number tagged &nbsp;$\\rightarrow$ &nbsp;Nov at 46\n\n46 $-$ 7 = 39 \n\n$\\therefore$ {{{correctAnswer}}}"}]}]

  165. Raph15-50 Raph15-51

    <div class="sm_mode"> {{{question}}} </div>

    [{"vars":[{"varval":"sm_img https://teacher.smartermaths.com.au/wp-content/uploads/2021/08/Math-Job-15-49.svg 300 indent vpad\n\nWhat number is marked with X on this number line?"},{"varval":"Determine the number pattern\n\n60 – 30 = 30\r\n\n$\\rightarrow$ Numbers increase by 30\r\n\r\nMissing number = 90 + 30 = {{{correctAnswer}}}\r"}]},{"vars":[{"varval":"sm_img https://teacher.smartermaths.com.au/wp-content/uploads/2021/08/Math-Job-15-51.svg 280 indent vpad\n\nWhat number is marked with X on this number line?"},{"varval":"Determine the number pattern\n\n30 – 15 = 15\r\n\n$\\rightarrow$ Numbers increase by 15\r\n\r\n\nMissing number = 45 + 15 = {{{correctAnswer}}}\r"}]},{"vars":[{"varval":"sm_img https://teacher.smartermaths.com.au/wp-content/uploads/2021/08/Math-Job-15-50.svg 290 indent vpad\n\nWhat number is marked with X on this number line?"},{"varval":"Determine the number pattern\n\n40 – 20 = 20\n\n\r\n$\\rightarrow$ Numbers increase by 20\r\n\r\n\nMissing number = 60 + 20 = {{{correctAnswer}}}\r"}]}]

  166. Raph15-47 Raph15-48

    <div class="sm_mode"> {{{question}}} </div>

    [{"vars":[{"varval":"Amanda is looking at some books in the direction shown by the arrow.\n\n<br>\n\nsm_img https://teacher.smartermaths.com.au/wp-content/uploads/2021/08/Math-Job-15-46.svg 340 indent1 vpad\n\n<br>What does she see?"},{"varval":"Looking from the direction of the arrow, 3 books have the same height.\n\n\r\nThe shortest book is 2nd from the left.\n\n<br>\n\r\n{{{correctAnswer}}}"}]},{"vars":[{"varval":"Jalepeno is looking at some boxes in the direction shown by the arrow.\n\n<br>\n\nsm_img https://teacher.smartermaths.com.au/wp-content/uploads/2021/08/Math-Job-15-47.svg 340 indent vpad\n\nWhat does he see?"},{"varval":"Looking from the direction of the arrow, 2 boxes in the centre are the shortest and of the same height.\n\n\r\nThe 2 outer boxes are the tallest and also of the same height.\n\n<br>\n\n{{{correctAnswer}}}"}]},{"vars":[{"varval":"Hamed is looking at some boxes in the direction shown by the arrow.\n\n<br>\n\nsm_img https://teacher.smartermaths.com.au/wp-content/uploads/2021/08/Math-Job-15-48.svg 340 indent vpad\n\nWhat does he see?"},{"varval":"Looking from the direction of the arrow, the 2 two boxes on the left are smallest and of the same height.\n\n<br>\n\r\n{{{correctAnswer}}}"}]}]

  167. Raph15-44 Raph15-45

    <div class="sm_mode"> {{{question}}} </div>

    [{"vars":[{"varval":"Mara cut some whole oranges into quarters.\r\n\nAll the quarters are shown below.\r\n\n<br>\n\nsm_img https://teacher.smartermaths.com.au/wp-content/uploads/2021/10/Math-Job-15-43-rev.png 340 indent3 vpad\n\n<br>How many whole oranges did Mara cut?"},{"varval":"sm_nogap There are 16 quarters.\r\n\n\n<div class=\"aligned\">\n\n|||\n|-:|-|\n|Whole oranges|= 16 $\\div$ 4|\n||= {{{correctAnswer0}}}|\n\n</div>"}]},{"vars":[{"varval":"Tina cut some whole pizzas into thirds.\n\nAll the slices are shown below.\n\n<br>\n\r\n\nsm_img https://teacher.smartermaths.com.au/wp-content/uploads/2021/10/Math-Job-15-44.png 440 indent3 vpad\n\n<br>How many whole pizzas did Tina cut?"},{"varval":"sm_nogap There are 9 thirds.\r\n\n\n<div class=\"aligned\">\n\n|||\n|-:|-|\n|Whole pizzas|= 9 $\\div$ 3|\n||= {{{correctAnswer0}}}|\n\n</div>"}]},{"vars":[{"varval":"Malika cut some whole lemons in half.\n\n\r\nAll the halves are shown below.\r\n\n<br>\n\nsm_img https://teacher.smartermaths.com.au/wp-content/uploads/2021/10/Math-Job-15-45.png 440 indent vpad\n\n<br>How many whole lemons did Malika cut?"},{"varval":"sm_nogap There are 10 halves.\r\n\n\n<div class=\"aligned\">\n\n|||\n|-:|-|\n|Whole lemons|= 10 $\\div$ 2|\n||= {{{correctAnswer0}}}|\n\n</div>"}]}]

  168. <div class="sm_mode"> Emma and Tulip both competed in the shot put at their school's athletics carnival. Emma's best throw was 7.34 metres. Tulip's best throw was 6.56 metres. What was the difference, in centimetres, between their best throws? </div>

    [{"vars":null}]

  169. Raph15-41 Raph15-42 SJ v3 - v5

    <div class="sm_mode"> {{{question}}} </div>

    [{"vars":[{"varval":"Aaron is using crayons to measure this piece of cardboard.\n\n<br>\n\nsm_img https://teacher.smartermaths.com.au/wp-content/uploads/2021/08/Math-Job-15-40.svg 440 indent3 vpad\n\n<br>What is the total length of all four sides?"},{"varval":"Length = 6 crayons\n\nWidth = 4 crayons\n\n<div class=\"aligned\">\n\n\n| | |\n|-:|-|\n|Total length|= 6 + 4 + 6 + 4|\n| | = {{{correctAnswer}}}|\n\n\r\n</div>"}]},{"vars":[{"varval":"Helga is using spoons to measure a table.\n\n<br>\n\nsm_img https://teacher.smartermaths.com.au/wp-content/uploads/2021/08/Math-Job-15-41.svg 440 indent3 vpad\n\n<br>What is the total length of all four sides?"},{"varval":"Length = 8 spoons\n\nWidth = 5 spoons\n\n<div class=\"aligned\">\n\n| | |\n|-:|-|\n|Total length|= 8 + 5 + 8 + 5|\n| |= {{{correctAnswer}}} |\n\n\r\n</div>"}]},{"vars":[{"varval":"Tray is using erasers to measure this paper.\n\n<br>\n\nsm_img https://teacher.smartermaths.com.au/wp-content/uploads/2021/08/Math-Job-15-42.svg 440 indent3 vpad\n\n<br>What is the total length of all four sides?"},{"varval":"Length = 4 erasers\n\nWidth = 3 erasers\n\n<div class=\"aligned\">\n\n\n| | |\n|-:|-|\n|Total length|= 4 + 3 + 4 + 3|\n| |= {{{correctAnswer}}} |\r\n\n</div>"}]},{"vars":[{"varval":"Donovan is using crayons to measure this equilateral triangle.\n\n<br>\n\nsm_img https://teacher.smartermaths.com.au/wp-content/uploads/2023/07/triangle-equilateral-crayon-min.svg 250 indent3 vpad\n\n<br>What is the total length of all three sides?"},{"varval":"Side length = 2 crayons\n\n<div class=\"aligned\">\n\n\n| | |\n|-:|-|\n|Total length|= 2 + 2 + 2|\n| |= {{{correctAnswer}}} |\n\n</div>"}]},{"vars":[{"varval":"Christian is using crayons to measure this equilateral triangle.\n\n<br>\n\nsm_img https://teacher.smartermaths.com.au/wp-content/uploads/2023/07/triangle-equilateral-crayon-3.svg 250 indent3 vpad\n\n<br>What is the total length of all three sides?"},{"varval":"Side length = 3 crayons\n\n<div class=\"aligned\">\n\n\n| | |\n|-:|-|\n|Total length|= 3 + 3 + 3|\n| |= {{{correctAnswer}}} |\n\n</div>"}]},{"vars":[{"varval":"Betty is using crayons to measure this square.\n\n<br>\n\nsm_img https://teacher.smartermaths.com.au/wp-content/uploads/2023/07/square-crayon-4-min.svg 200 indent3 vpad\n\n<br>What is the total length of all four sides?"},{"varval":"Side length = 4 crayons\n\n<div class=\"aligned\">\n\n\n| | |\n|-:|-|\n|Total length|= 4 + 4 + 4 + 4|\n| |= {{{correctAnswer}}} |\n\n</div>"}]}]

  170. Raph15-38 Raph15-39

    <div class="sm_mode"> {{{question}}} </div>

    [{"vars":[{"varval":"Gary needs to buy 23 apples to make pies.\n\n\rThe apples are sold in packets of 8.\n\n\rHow many packets does he need to buy?\r"},{"varval":"sm_nogap Number of packets:\n\n<div class=\"aligned\">\n\n>|||\n|-:|-|\n|23 $\\div$ 8|= 2 remainder 7|\n| |= {{{correctAnswer}}} (round up)|\n\n</div>\n"}]},{"vars":[{"varval":"Martin needs to buy 27 hotdogs for a camping trip.\n\nThe hotdogs are sold in packets of 7.\n\nHow many packets does he need to buy?\r"},{"varval":"sm_nogap Number of packets\n\n<div class=\"aligned\">\n\n>|||\n|-:|-|\n|27 $\\div$ 7|= 3 remainder 6|\n| |= {{{correctAnswer}}} (round up)|\n\n</div>"}]},{"vars":[{"varval":"Sheila needs to buy 19 flowers.\n\nThe flowers are sold in bouquets of 6.\n\nHow many bouquets does she need to buy?\r"},{"varval":"sm_nogap Number of bouquets needed\n\n<div class=\"aligned\">\n\n>|||\n|-:|-|\n|19 $\\div$ 6|= 3 remainder 1|\n| |= {{{correctAnswer}}} (round up)|\n\n</div>"}]},{"vars":[{"varval":"Cassandra is buying 23 toffee apples for a party.\n\nThey are sold in bags of 5.\n\nHow many bags does she need to buy?\r"},{"varval":"sm_nogap Number of bags needed\n\n<div class=\"aligned\">\n\n>|||\n|-:|-|\n|23 $\\div$ 5|= 4 remainder 3|\n| |= {{{correctAnswer}}} (round up)|\n\n</div>"}]},{"vars":[{"varval":"Blart is buying 37 donuts for his office morning tea.\n\nThey are sold in bags of 5.\n\nHow many bags does he need to buy?\r"},{"varval":"sm_nogap Number of bags needed\n\n<div class=\"aligned\">\n\n>|||\n|-:|-|\n| 37 $\\div$ 5|= 7 remainder 2|\n| |= {{{correctAnswer}}} (round up)|\n\n</div>"}]},{"vars":[{"varval":"Delilah is buying 33 crystals for her shop.\n\nThey are sold in boxes of 4.\n\nHow many boxes does she need to buy?\r"},{"varval":"sm_nogap Number of boxes needed\n\n<div class=\"aligned\">\n\n>|||\n|-:|-|\n| 33 $\\div$ 4|= 8 remainder 1|\n| |= {{{correctAnswer}}} (round up)|\n\n</div>"}]},{"vars":[{"varval":"Alma is buying 44 cards from the newsagent.\n\nThey are sold in packets of 3.\n\nHow many packets does she need to buy?\r"},{"varval":"sm_nogap Number of packets needed\n\n<div class=\"aligned\">\n\n>|||\n|-:|-|\n| 44 $\\div$ 3|= 14 remainder 2|\n| |= {{{correctAnswer}}} (round up)|\n\n</div>"}]},{"vars":[{"varval":"Cassie needs to buy 14 pencils for her class.\n\nThe pencils are sold in packets of 3.\n\nHow many packets does she need to buy?\r"},{"varval":"sm_nogap Number of packets\n\n<div class=\"aligned\">\n\n>|||\n|-:|-|\n|14 $\\div$ 3|= 4 remainder 2|\n| |= {{{correctAnswer}}} (round up)|\n\n</div>"}]},{"vars":[{"varval":"Arthur needed to buy 14 eggs from the market.\n\nThe eggs are sold in cartons of 6 eggs.\n\nHow many cartons does Arthur need to buy?\r"},{"varval":"sm_nogap Number of cartons\n\n<div class=\"aligned\">\n\n>|||\n|-:|-|\n|14 $\\div$ 6|= 2 remainder 2|\n| |= {{{correctAnswer}}} (round up)|\n\n</div>"}]}]

  171. <div class="sm_mode"> {{{question}}} </div>

    [{"vars":[{"varval":"Fill in the box to make this number sentence correct."},{"varval":"<div class=\"aligned\">\n\n| | |\n|-:|-|\n| 22 + <span class=\"sm_box\"> ` ` ` ` </span> |= 48|\n| | |\n| | |\n|<span class=\"sm_box\"> ` ` ` ` </span> |= 48 $-$ 22|\n||= {{{correctAnswer0}}}|\n\n</div>"}]},{"vars":[{"varval":"Fill in the box to make this number sentence correct."},{"varval":"<div class=\"aligned\">\n\n| | |\n|-:|-|\n| 28 + <span class=\"sm_box\"> ` ` ` ` </span> |= 67|\n| | |\n| | |\n|<span class=\"sm_box\"> ` ` ` ` </span> |= 67 $-$ 28|\n||= {{{correctAnswer0}}}|\n\n</div> "}]},{"vars":[{"varval":"Fill in the box to make this number sentence correct."},{"varval":"<div class=\"aligned\">\n\n| | |\n|-:|-|\n| 34 + <span class=\"sm_box\"> ` ` ` ` </span> |= 53|\n| | |\n| | |\n|<span class=\"sm_box\"> ` ` ` ` </span> |= 53 $-$ 34 |\n||= {{{correctAnswer0}}}|\n\n</div> "}]},{"vars":[{"varval":"Fill in the box to make this number sentence correct."},{"varval":"<div class=\"aligned\">\n\n| | |\n|-:|-|\n| 25 + <span class=\"sm_box\"> ` ` ` ` </span> |= 49|\n| | |\n| | |\n|<span class=\"sm_box\"> ` ` ` ` </span> |= 49 $-$ 25 |\n||= {{{correctAnswer0}}}|\n\n</div> "}]},{"vars":[{"varval":"Fill in the box to make this number sentence correct."},{"varval":"<div class=\"aligned\">\n\n| | |\n|-:|-|\n| 13 + <span class=\"sm_box\"> ` ` ` ` </span> |= 36|\n| | |\n| | |\n|<span class=\"sm_box\"> ` ` ` ` </span> |= 36 $-$ 13 |\n||= {{{correctAnswer0}}}|\n\n</div> "}]},{"vars":[{"varval":"Fill in the box to make this number sentence correct."},{"varval":"<div class=\"aligned\">\n\n| | |\n|-:|-|\n| 22 + <span class=\"sm_box\"> ` ` ` ` </span> |= 79|\n| | |\n| | |\n|<span class=\"sm_box\"> ` ` ` ` </span> |= 79 $-$ 22 |\n||= {{{correctAnswer0}}}|\n\n</div> "}]},{"vars":[{"varval":"Fill in the box to make this number sentence correct."},{"varval":"<div class=\"aligned\">\n\n| | |\n|-:|-|\n| 17 + <span class=\"sm_box\"> ` ` ` ` </span> |= 95|\n| | |\n| | |\n|<span class=\"sm_box\"> ` ` ` ` </span> |= 95 $-$ 17 |\n||= {{{correctAnswer0}}}|\n\n</div> "}]},{"vars":[{"varval":"Fill in the box to make this number sentence correct."},{"varval":"<div class=\"aligned\">\n\n| | |\n|-:|-|\n| 28 + <span class=\"sm_box\"> ` ` ` ` </span> |= 93|\n| | |\n| | |\n|<span class=\"sm_box\"> ` ` ` ` </span> |= 93 $-$ 28 |\n||= {{{correctAnswer0}}}|\n\n</div> "}]}]

  172. Was same question as Geometry, Raph15-31 so have changed image in v0

    <div class="sm_mode"> {{{question}}} </div>

    [{"vars":[{"varval":"Matt has started to make a model of a triangular pyramid using toothpicks and clay.\n\nsm_img https://teacher.smartermaths.com.au/wp-content/uploads/2023/07/Tripyramid-min.svg 200 indent vpad\n\nHow many more toothpicks does Matt need to finish the model?\n"},{"varval":"A triangular pyramid has 6 edges.\n\r\nTherefore, Matt needs {{{correctAnswer}}} more toothpick.\n\nsm_img https://teacher.smartermaths.com.au/wp-content/uploads/2023/07/Tripyramid_ws-min.svg 150 indent vpad"}]},{"vars":[{"varval":"Navi has started to make a model of a triangular prism using toothpicks and putty.\n\nsm_img https://teacher.smartermaths.com.au/wp-content/uploads/2023/07/tri-prism-min.svg 260 indent vpad\n\nHow many more toothpicks does Navi need to finish the model?"},{"varval":"A triangular prism has 9 edges.\n\nTherefore, Navi needs {{{correctAnswer}}} more toothpicks.\n\nsm_img https://teacher.smartermaths.com.au/wp-content/uploads/2023/07/tri-prism_ws-min.svg 230 indent vpad"}]},{"vars":[{"varval":"Aiesha has started to make a model of a rectangular prism using toothpicks and clay.\n\nsm_img https://teacher.smartermaths.com.au/wp-content/uploads/2023/07/rect-prism-min.svg 200 indent vpad\n\nHow many more toothpicks does Aiesha need to finish the model?"},{"varval":"A rectangular prism has 12 edges.\n\nTherefore, Aiesha needs {{{correctAnswer}}} more toothpicks.\n\nsm_img https://teacher.smartermaths.com.au/wp-content/uploads/2023/07/rect-prism_ws-min.svg 160 indent vpad"}]},{"vars":[{"varval":"Antony has started to make a model of a rectangular pyramid using toothpicks and clay.\n\nsm_img https://teacher.smartermaths.com.au/wp-content/uploads/2023/07/rect-pyr-min.svg 220 indent vpad\n\nHow many more toothpicks does Antony need to finish the model?\n"},{"varval":"A rectangular pyramid has 8 edges.\n\nTherefore, Antony needs {{{correctAnswer}}} more toothpicks.\n\nsm_img https://teacher.smartermaths.com.au/wp-content/uploads/2023/07/rect-pyr-ws-min.svg 180 indent vpad"}]}]

  173. Raph15-32 Raph15-33

    <div class="sm_mode"> {{{question}}} </div>

    [{"vars":[{"varval":"John has started to make a model of a square pyramid using sticks and clay balls.\n\nsm_img https://teacher.smartermaths.com.au/wp-content/uploads/2021/08/Math-Job-15-31.svg 300 indent vpad\n\nHow many more sticks does John need to finish the model?"},{"varval":"A square pyramid has 8 edges.\r\n\nTherefore, John needs {{{correctAnswer}}} more sticks.\r"}]},{"vars":[{"varval":"<div class=\"sm_mode\">\n\nDonna has started to make a model of a rectangular prism using sticks and clay balls.\n\nsm_img https://teacher.smartermaths.com.au/wp-content/uploads/2021/08/Math-Job-15-32.svg 340 indent vpad\n\nHow many more sticks does Donna need to finish the model?\n\n</div>"},{"varval":"A rectangular prism has 12 edges.\n\nTherefore, Donna needs {{{correctAnswer}}} more sticks.\r"}]},{"vars":[{"varval":"Carl has started to make a model of a triangular pyramid using rods and clay balls.\n\nsm_img https://teacher.smartermaths.com.au/wp-content/uploads/2021/08/Math-Job-15-33.svg 340 indent vpad\n\nHow many more rods does Carl need to finish the model?"},{"varval":"A triangular pyramid has 6 edges.\n\n\r\nTherefore, Carl needs {{{correctAnswer}}} more rods.\r"}]}]

  174. RAPH15-28 RAPH15-29 RAPH15-30

    <div class="sm_mode"> {{{question}}} </div>

    [{"vars":[{"varval":"Which of these will look most like a cube when closed?"},{"varval":"A cube has 6 square faces.\n\n{{{correctAnswer}}}"}]},{"vars":[{"varval":"Which of these will look most like a sphere when closed?"},{"varval":"A sphere has the shape of a ball.\n\n{{{correctAnswer}}}"}]},{"vars":[{"varval":"Which of these will look most like a cone when closed?"},{"varval":"{{{correctAnswer}}}"}]}]

  175. Raph15-22 Raph15-23 Raph15-24

    <div class="sm_mode"> {{{question}}} </div>

    [{"vars":[{"varval":"Which phone is the most expensive?\n\n<br>\n\nsm_img https://teacher.smartermaths.com.au/wp-content/uploads/2021/10/Math-Job-15-22.png 380 indent3 vpad"},{"varval":"The highest price = $541\n\n$\\rightarrow$ Phone {{{correctAnswer}}}\n"}]},{"vars":[{"varval":"Which pack of cookies has the lowest calories?\n\n<br>\n\nsm_img https://teacher.smartermaths.com.au/wp-content/uploads/2021/10/Math-Job-15-23.png 350 indent3 vpad"},{"varval":"The lowest calories of any pack = 789\n\n$\\rightarrow$ Packet {{{correctAnswer}}} "}]},{"vars":[{"varval":"Which car travelled the longest distance?\n\nsm_img https://teacher.smartermaths.com.au/wp-content/uploads/2021/10/Math-Job-15-24.png 420 indent3 vpad"},{"varval":"The longest distance travelled was 321 km.\n\n$\\rightarrow$ Car {{{correctAnswer}}}"}]},{"vars":[{"varval":"Which desktop monitor is the most expensive?\n\n<br>\n\nsm_img https://teacher.smartermaths.com.au/wp-content/uploads/2023/06/Number-_Raph-15-20-min.svg 420 indent2 vpad"},{"varval":"The highest price = $342\n\n$\\rightarrow$ Desktop monitor {{{correctAnswer}}}\n"}]},{"vars":[{"varval":"Which mountain bike is the least expensive?\n\n<br>\n\nsm_img https://teacher.smartermaths.com.au/wp-content/uploads/2023/06/bikes_v4_2-min.svg 500 indent vpad"},{"varval":"The lowest price = $748\n\n$\\rightarrow$ Mountain bike {{{correctAnswer}}}\n"}]}]

  176. Raph15-19 Raph15-20 Raph15-21

    <div class="sm_mode"> {{{question}}} </div>

    [{"vars":[{"varval":"Lisa and Andy both worked in part time jobs. \n\nLisa earned $36 while Andy earned $37.\n\nHow much, in dollars, did they earn altogether?\r"},{"varval":"sm_nogap One strategy:\n\n<div class=\"aligned\">\n\n| | |\n| --------------------- | -------------- |\n| 36 + 37| = 30 + 6 + 30 + 7|\n|| = 60 + 6 + 7|\n| | \\= {{{correctAnswer}}} |\n\n</div>\n"}]},{"vars":[{"varval":"Ramon is making a cabinet. \n\nHe bought 35 nails and 46 screws.\n\nHow many nails and screws did he buy altogether?\r"},{"varval":"sm_nogap One strategy:\n\n<div class=\"aligned\">\n\n| | |\n| --------------------- | -------------- |\n| 35 + 46| = 30 + 5 + 40 + 6|\n|| = 70 + 5 + 6|\n| | \\= {{{correctAnswer}}} |\n\n</div>"}]},{"vars":[{"varval":"A restaurant owner purchased 29 lettuces and 45 eggplants from the market.\n\nHow many vegetables did he purchase in total?\r"},{"varval":"sm_nogap One strategy:\n\n<div class=\"aligned\">\n\n| | |\n| --------------------- | -------------- |\n| 29 + 45| = 20 + 9 + 40 + 5|\n|| = 60 + 9 + 5|\n| | \\= {{{correctAnswer}}} |\n\n</div>"}]},{"vars":[{"varval":"A bull catcher caught 127 bulls in the first week of a job and 86 in the second week.\n\nIn total, how many bulls did he catch?"},{"varval":"sm_nogap One strategy:\n\n<div class=\"aligned\">\n\n| | |\n| --------------------- | -------------- |\n| 127 + 86| = 100 + 20 + 7 + 80 + 6|\n|| = 100 + 100 + 7 + 6|\n|| = 200 + 13|\n| | \\= {{{correctAnswer}}} |\n\n</div>"}]},{"vars":[{"varval":"Leisa collected 128 football cards in April and 175 more cards in May.\n\nIn total, how many football cards did Leisa collect in April and May?"},{"varval":"sm_nogap One strategy:\n\n<div class=\"aligned\">\n\n| | |\n| --------------------- | -------------- |\n| 128 + 175| = 100 + 20 + 8 + 100 + 70 + 5|\n|| = 200 + 90 + 8 + 5|\n|| = 290 + 13|\n| | \\= {{{correctAnswer}}} |\n\n</div>"}]}]

  177. Raph15-17 Raph15-18 SJ Corrected setup so workedSolution variable added as more than one variant.

    <div class="sm_mode"> {{{question}}} </div>

    [{"vars":[{"varval":"Dennis has 18 pieces of fruit.\n\n10 are apples and 8 are oranges.\n\n\r\nWhich of these correctly shows the number of each fruit?\r"},{"varval":"{{{correctAnswer}}}"}]},{"vars":[{"varval":"Emile has 22 students.\n\n10 are boys and 12 are girls.\n\nWhich of these correctly shows this tally?"},{"varval":"{{{correctAnswer}}}"}]}]

  178. Raph15-13 Raph15-14

    <div class="sm_mode"> {{{question}}} </div>

    [{"vars":[{"varval":"Alfred has some boxes of crayons and 6 extra crayons.\n\n<br>\n\nsm_img https://teacher.smartermaths.com.au/wp-content/uploads/2021/08/Raph15-14.png 580 indent2 vpad\r\n\n\n<br>How many crayons does Alfred have in total?"},{"varval":"<div class=\"aligned\">\n\n| | |\n| -:|-|\n|Total number of crayons|= (12 $\\times$ 3) + 6 |\n| |= 36 + 6 |\n| |= {{{correctAnswer}}}|\n\n</div>"}]},{"vars":[{"varval":"James buys a number of boxes of cupcakes and 4 extra cupcakes.\n\n<br>\n\nsm_img https://teacher.smartermaths.com.au/wp-content/uploads/2021/08/Raph15-13.png 540 indent vpad\r\n\n<br>How many cupcakes does James have in total?"},{"varval":"<div class=\"aligned\">\n\n| | |\n| -:|-|\n|Total number of cupcakes|= (5 $\\times$ 8) + 4 |\n| |= 40 + 4 |\n| |= {{{correctAnswer}}}|\n\n</div>"}]},{"vars":[{"varval":"Pollo collects eggs from his hen house.\n\nOne morning, he collected enough eggs to fill 3 cartons and with 9 eggs left over.\n\nIf each carton holds 12 eggs, how many eggs has Pollo collected in total."},{"varval":"<div class=\"aligned\">\n\n| | |\n| -:|:|\n| Total number of eggs | \\= (3 $\\times$ 12) + 9 |\n| | \\= 36 + 9 |\n| | \\= {{{correctAnswer}}} |\n\n\n</div>"}]},{"vars":[{"varval":"Steve was packing apples in boxes that he would sell at the market.\n\nHe packed 12 boxes and had an extra 5 apples left over that he would sell individually.\n\nIf each box contained 9 apples, how many apples were there in total?"},{"varval":"<div class=\"aligned\">\n\n| | |\n| ----------------------- | ---------------------- |\n| Total number of apples | \\= (12 $\\times$ 9) + 5 |\n| | \\= 108 + 5 |\n| | \\= {{{correctAnswer}}} |\n\n</div>"}]},{"vars":[{"varval":"Johan has some boxes each containing some donuts and 5 extra donuts.\n\n<br>\n\nsm_img https://teacher.smartermaths.com.au/wp-content/uploads/2023/07/donuts-boxes-8-min.svg 400 indent2 vpad\n\n\n<br>How many donuts does Johan have in total?"},{"varval":"<div class=\"aligned\">\n\n| | |\n| -:|-|\n|Total number of donuts|= (8 $\\times$ 3) + 5 |\n| |= 24 + 5 |\n| |= {{{correctAnswer}}}|\n\n</div>"}]},{"vars":[{"varval":"Briony has some boxes of pencils and 12 extra pencils.\n\n<br>\n\nsm_img https://teacher.smartermaths.com.au/wp-content/uploads/2023/07/box-5-pencils12A-min.svg 450 indent2 vpad\n\n\n<br>How many pencils does Briony have in total?"},{"varval":"<div class=\"aligned\">\n\n| | |\n| -:|-|\n|Total number of pencils|= (5 $\\times$ 15) + 12 |\n| |= 75 + 12 |\n| |= {{{correctAnswer}}}|\n\n</div>"}]}]

  179. Raph15-10 Raph15-11 Raph15-12

    <div class="sm_mode"> {{{question}}} </div>

    [{"vars":[{"varval":"Angus is counting down by twos.\n\r\n28, 26, 24, 22, <span class=\"sm-text color2\">?</span>\n\r\nWhich number comes next?\r"},{"varval":"<div class=\"aligned\">\n\n|||\n|-|-|\n|<span class=\"sm-text color2\">?</span>|= last number $-$ 2|\n||= 22 $-$ 2|\n||= {{{correctAnswer}}}|\n\n</div>"}]},{"vars":[{"varval":"Wendy is counting down by twos.\n\n\r\n47, 45, 43, 41, <span class=\"sm-text color3\">?</span>\n\n\r\nWhich number comes next?\r"},{"varval":"<div class=\"aligned\">\n\n|||\n|-|-|\n|<span class=\"sm-text color3\">?</span>|= last number $-$ 2|\n||= 41 $-$ 2|\n||= {{{correctAnswer}}}|\n\n</div>"}]},{"vars":[{"varval":"Almira is counting down by ones.\n\n74, 73, 72, 71, 70, <span class=\"sm-text color3\">?</span>\n\nWhich number comes next?"},{"varval":"<div class=\"aligned\">\n\n|||\n|-|-|\n|<span class=\"sm-text color3\">?</span>|= last number $-$ 1|\n||= 70 $-$ 1|\n||= {{{correctAnswer}}}|\n\n</div>"}]},{"vars":[{"varval":"Callum is counting down by fours.\n\n43, 39, 35, 31, <span class=\"sm-text color3\">?</span>\n\nWhat is the next number in this pattern?"},{"varval":"<div class=\"aligned\">\n\n|||\n|-|-|\n|<span class=\"sm-text color3\">?</span>|= last number $-$ 4|\n||= 31 $-$ 4|\n||= {{{correctAnswer}}}|\n\n</div>"}]},{"vars":[{"varval":"Albert wrote down a number pattern.\n\n25, 29, 33, 37, <span class=\"sm-text color2\">?</span>\n\nWhat is the next number in the pattern?"},{"varval":"<div class=\"aligned\">\n\n|||\n|-|-|\n|<span class=\"sm-text color2\">?</span>|= last number + 4|\n||= 37 + 4|\n||= {{{correctAnswer}}}|\n\n</div>"}]},{"vars":[{"varval":"Jevin writes down a number pattern.\n\n15, 22, 29, 36, <span class=\"sm-text color2\">?</span>\n\nWhat is the next number in Jevin's pattern?"},{"varval":"<div class=\"aligned\">\n\n|||\n|-|-|\n|<span class=\"sm-text color2\">?</span>|= last number + 7|\n||= 36 + 7|\n||= {{{correctAnswer}}}|\n\n</div>"}]},{"vars":[{"varval":"Clinton writes down a number pattern.\n\n63, 55, 47, <span class=\"sm-text color4\">?</span>\n\nWhat number should Clinton write next?"},{"varval":"<div class=\"aligned\">\n\n|||\n|-|-|\n|<span class=\"sm-text color4\">?</span>|= last number $-$ 8|\n||= 47 $-$ 8|\n||= {{{correctAnswer}}}|\n\n</div>"}]},{"vars":[{"varval":"Jerome created a number pattern by adding four to a number to get the next number.\n\nWhich of these could be the first four numbers in Jerome's pattern?"},{"varval":"17 + 4 = 21\n\n21 + 4 = 25\n\n25 + 4 = 29\n\n$\\therefore$ Jerome's pattern is: &nbsp;17, 21, 25, 29"}]},{"vars":[{"varval":"Chapelle created a number pattern by adding six to a number to get the next number.\n\nWhich of these could be the first four numbers in Chapelle's pattern?"},{"varval":"15 + 6 = 21\n\n21 + 6 = 27\n\n27 + 6 = 33\n\n$\\therefore$ Chapelle's pattern is: &nbsp;15, 21, 27, 33"}]},{"vars":[{"varval":"Leonardo wrote down the following number pattern?\n\n4, 5, 7, 10, <span class=\"sm-text color2\">?</span>\n\nWhat number should Leonardo write down next? "},{"varval":"<div class=\"aligned\">\n\n|||\n|-:|-|\n|2nd number|= 4 + 1 = 5|\n|3rd number|= 5 + 2 = 7|\n|4th number|= 7 + 3 = 10 |\n|<span class=\"sm-text color2\">?</span>| = 10 + 4 = {{{correctAnswer}}}|\n\n</div>"}]},{"vars":[{"varval":"Bonacci is writing down a number pattern.\n\n>17, 15, 12, 8, <span class=\"sm-text color2\">?</span>\n\n<br>\n\nWhat number should Bonacci write down next?"},{"varval":"<div class=\"aligned\">\n\n|||\n|-:|-|\n|2nd number|= 17 $-$ 2 = 15|\n|3rd number|= 15 $-$ 3 = 12|\n|4th number|= 12 $-$ 4 = 8 |\n|<span class=\"sm-text color2\">?</span>| = 8 $-$ 5 = {{{correctAnswer}}}|\n\n</div>"}]},{"vars":[{"varval":"Isaac creates a number pattern.\n\n4, 7, 12, 19, <span class=\"sm-text color3\">?</span>\n\nWhat number should Isaac write down next?"},{"varval":"<div class=\"aligned\">\n\n|||\n|-:|-|\n|2nd number|= 4 + 3 = 7|\n|3rd number|= 7 + 5 = 12|\n|4th number|= 12 + 7 = 19 |\n| <span class=\"sm-text color3\">?</span>| = 19 + 9 = {{{correctAnswer}}}|\n\n</div>"}]}]

  180. <div class="sm_mode"> {{{question}}} </div>

    [{"vars":[{"varval":"This is a map of a zoo.\n\nsm_img https://teacher.smartermaths.com.au/wp-content/uploads/2021/08/Math-Job-15-9.svg 320 indent vpad\n\nWhich enclosure is furthest away from the entrance?\n"},{"varval":"By looking at the path inside the zoo, the zebra's enclosure {{{correctAnswer}}} is the furthest from the entrance."}]}]

  181. Needs difficulty

    <div class="sm_mode"> {{{question}}} </div>

    [{"vars":[{"varval":"This is a map of a playground.\n\nsm_img https://teacher.smartermaths.com.au/wp-content/uploads/2021/08/Math-Job-15-8.svg 280 indent vpad\n\nWhich equipment is furthest away from the ice cream store?"},{"varval":"The slide, {{{correctAnswer}}}, is on the other side of the playground opposite to the ice cream store and is therefore, furthest from the ice cream store."}]}]

  182. <div class="sm_mode"> {{{question}}} </div>

    [{"vars":[{"varval":"This is a map of a village.\n\nsm_img https://teacher.smartermaths.com.au/wp-content/uploads/2021/08/Math-Job-15-7.svg 280 indent vpad\n\nWhich of these houses is furthest away from the market?"},{"varval":"Houses {{{correctAnswer}}} and D are further away from the market than houses B and C.\n\n\r\nBetween house {{{correctAnswer}}} and house D, house {{{correctAnswer}}} is further because it has a bigger horizontal distance from the market.\r\n\nsm_img https://teacher.smartermaths.com.au/wp-content/uploads/2021/08/Math-Job-15-7-Ans.svg 280 indent vpad\n"}]}]

  183. RAPH 15-4 RAPH 15-5

    <div class="sm_mode"> {{{question}}} </div>

    [{"vars":[{"varval":"Four horses are walking up a hill.\n\r\n<br>\n\nsm_img https://teacher.smartermaths.com.au/wp-content/uploads/2023/07/horse-4-min.svg 300 indent2 vpad\n\n<br>Which horse is second from the bottom?\r"},{"varval":"Bottom = Horse D\n\n2nd from the bottom = {{{correctAnswer}}}"}]},{"vars":[{"varval":"Four cars are travelling down the hill.\n\n<br>\n\nsm_img https://teacher.smartermaths.com.au/wp-content/uploads/2021/08/Math-Job-15-5.svg 300 indent2 vpad\n\n<br>Which one is second from the top?\n\r"},{"varval":"Top = Vehicle A\n\n2nd from top = {{{correctAnswer}}}"}]},{"vars":[{"varval":"Four vehicles are travelling down the hill.\n\n<br>\n\nsm_img https://teacher.smartermaths.com.au/wp-content/uploads/2023/07/carx4-hill-min.svg 300 indent2 vpad\n\n<br>Which one is second from the bottom?\n\n"},{"varval":"Bottom = Vehicle D\n\n2nd from bottom = {{{correctAnswer}}}"}]},{"vars":[{"varval":"Four friends are standing in a line.\n\n<br>\n\nsm_img https://teacher.smartermaths.com.au/wp-content/uploads/2023/07/kids-4-min.svg 250 indent2 vpad\n\n<br>Which one is second from the left?\n\n"},{"varval":"Furthest left = Friend A\n\n2nd left = {{{correctAnswer}}}"}]},{"vars":[{"varval":"Four dogs are standing in a line.\n\n<br>\n\nsm_img https://teacher.smartermaths.com.au/wp-content/uploads/2023/07/dog-4a-min.svg 300 indent vpad\n\n<br>Which one is second from the right?\n\n"},{"varval":"Furthest right = Dog D\n\n2nd right = {{{correctAnswer}}}"}]},{"vars":[{"varval":"Four farm animals are standing in a line.\n\n<br>\n\nsm_img https://teacher.smartermaths.com.au/wp-content/uploads/2023/07/animals-4b-min.svg 350 indent2 vpad\n\n<br>Which one is second from the left?"},{"varval":"Furthest left = Animal A\n\n2nd left = {{{correctAnswer}}}"}]}]

  184. SJ: I changed these images for v1 and v2 as the others uploaded really slowly in the online tests.

    <div class="sm_mode"> {{{question}}} </div>

    [{"vars":[{"varval":"Which building is the shortest?\n\n<br>\n\nsm_img https://teacher.smartermaths.com.au/wp-content/uploads/2023/07/buildings-4v0-min.svg 300 indent vpad\n"},{"varval":"Shortest = Lowest\n\n\r\nTherefore, building {{{correctAnswer}}} is the shortest.\r"}]},{"vars":[{"varval":"Which of the children is the tallest?\n\n<br>\n\nsm_img https://teacher.smartermaths.com.au/wp-content/uploads/2023/07/children-4-min.svg 300 indent vpad"},{"varval":"Tallest = Highest\n\nTherefore, child {{{correctAnswer}}} is the tallest.\r"}]},{"vars":[{"varval":"Which building is the tallest?\n\n<br>\n\nsm_img https://teacher.smartermaths.com.au/wp-content/uploads/2023/07/buildings-4v0-min.svg 300 indent vpad\n"},{"varval":"Tallest = Highest\n\nTherefore, building {{{correctAnswer}}} is the tallest.\n"}]},{"vars":[{"varval":"Which of the children is the shortest?\n\n<br>\n\nsm_img https://teacher.smartermaths.com.au/wp-content/uploads/2023/07/children-4-min.svg 300 indent vpad"},{"varval":"Shortest = Lowest\n\nTherefore, child {{{correctAnswer}}} is the shortest.\n"}]},{"vars":[{"varval":"Which plane is flying the furthest from the ground?\n\n<br>\n\nsm_img https://teacher.smartermaths.com.au/wp-content/uploads/2023/07/aeroplane-2-abcd-min.svg 300 indent vpad"},{"varval":"Furthest from the ground = Highest\n\nTherefore, plane {{{correctAnswer}}} is the furthest from the ground.\n"}]},{"vars":[{"varval":"Which plane is flying the closest to the ground?\n\n<br>\n\nsm_img https://teacher.smartermaths.com.au/wp-content/uploads/2023/07/aeroplane-2-abcd-min.svg 300 indent vpad"},{"varval":"Closest to the ground = Lowest\n\nTherefore, plane {{{correctAnswer}}} is closest to the ground."}]}]

  185. Number, NAPX-J2-26

    <div class="sm_mode"> {{{question}}} </div>

    [{"vars":[{"varval":"A cupcake recipe contains $\\dfrac{3}{5}$ cup of flour.\n\n\r\n\r\nWhich of the following represents the same amount of flour?"},{"varval":"<div class=\"aligned\">\n\n|||\n|-|-|\n|$\\dfrac{3}{5}$|= $\\dfrac{3 \\times 2}{5 \\times 2}$|\n||= $\\dfrac{6}{10}$|\n\n</div>"}]},{"vars":[{"varval":"A cupcake recipe contains $\\dfrac{3}{4}$ cup of sugar.\n\n\r\n\r\nWhich of the following represents the same amount of sugar?"},{"varval":"<div class=\"aligned\">\n\n|||\n|-|-|\n|$\\dfrac{3}{4}$|= $\\dfrac{3 \\times 3}{4 \\times 3}$|\n||= $\\dfrac{9}{12}$|\n\n</div>"}]}]

  186. Number, NAPX-p73018v01 Number, NAPX-I2-27

    <div class="sm_mode"> {{{question}}} </div>

    [{"vars":[{"varval":"Jonah, Murray, and Colin shared a box full of kiwi fruit.\n\n\r\n\r\nJonah and Murray took $\\dfrac{4}{11}$ of the box of kiwi fruits each.\n\n\r\n\r\nWhat fraction was left for Colin?"},{"varval":"sm_nogap Fraction that Jonah and Murray took\n\n<div class=\"aligned\">\n\n>>||\n|-|\n|= $\\dfrac{4}{11} + \\dfrac{4}{11}$|\n|= $\\dfrac{8}{11}$|\n\n</div>\n\n<br>\n\nsm_nogap $\\therefore$ Fraction left for Colin\n\n<div class=\"aligned\">\n\n>>||\n|-|\n|= $1 - \\dfrac{8}{11}$|\n|= $\\dfrac{3}{11}$|\n\n</div>"}]},{"vars":[{"varval":"Karl, Mary and Lily shared a whole pizza.\n\nMary and Lily ate $\\dfrac{2}{5}$ of the whole pizza each.\n\nWhat fraction of the pizza was left for Karl?"},{"varval":"sm_nogap Fraction that Mary and Lily ate\n\n<div class=\"aligned\">\n\n>>||\n|-|\n|= $\\dfrac{2}{5} + \\dfrac{2}{5}$|\n|= $\\dfrac{4}{5}$|\n\n</div>\n\n<br>\n\nsm_nogap $\\therefore$ Fraction left for Karl\n\n<div class=\"aligned\">\n\n>>||\n|-|\n|= $1 - \\dfrac{4}{5}$|\n|= $\\dfrac{1}{5}$|\n\n</div>\n\n\r\n\n"}]},{"vars":[{"varval":"Curly, Larry and Moe share a bag of strawberries.\n\n\r\n\r\nCurly and Larry get $\\dfrac{1}{8}$ of the strawberries each.\n\n\r\n\r\nWhat fraction of the bag of strawberries does Moe get?"},{"varval":"sm_nogap Fraction that Curly and Larry receive\n\n<div class=\"aligned\">\n\n>>||\n|-|\n|= $\\dfrac{1}{8} + \\dfrac{1}{8}$|\n|= $\\dfrac{2}{8}$|\n\n</div>\n\n<br>\n\nsm_nogap $\\therefore$ Fraction left for Moe\n\n<div class=\"aligned\">\n\n>>||\n|-|\n|= $1 - \\dfrac{2}{8}$|\n|= $\\dfrac{6}{8}$|\n\n</div>\n\n\r\n\n"}]}]

  187. Number, NAPX-p73124v01

    <div class="sm_mode"> {{{question}}} </div>

    [{"vars":[{"varval":"Julian was driving into town and hit a kangaroo $\\dfrac{3}{4}$ of a kilometre into his trip.\n\n\r\n\r\nWhich of these represent where Julian hit the kangaroo?"},{"varval":"Each spacing is worth $\\dfrac{1}{4}$ km.\n\n<br>\n\nsm_img https://teacher.smartermaths.com.au/wp-content/uploads/2021/04/Number-NAPX-p73124v02-iv.png 240 indent vpad\r\n"}]},{"vars":[{"varval":"Axe went jogging and stopped after $\\dfrac{2}{6}$ of a kilometre to take a rest.\n\n\r\n\r\nWhich of these represents where Axe stopped jogging?"},{"varval":"Each spacing is worth $\\dfrac{1}{6}$ km.\n\n<br>\n\nsm_img https://teacher.smartermaths.com.au/wp-content/uploads/2021/03/NAPX5-29-1-1.png 240 indent vpad\r"}]}]

  188. Number, NAPX-J2-25

    <div class="sm_mode"> {{{question}}} </div>

    [{"vars":[{"varval":"Stu is weighing different types of balls used in his favourite sports.\n\n<br>\n\nsm_img https://teacher.smartermaths.com.au/wp-content/uploads/2019/01/NAPX-J2-25-v2_2-768x393.png 380 indent vpad\n\n\r\n<br>Stu knows that a cricket ball weighs more than a baseball but less than a soccer ball.\n\n\r\n\r\nWhich of these could be the weight of a cricket ball?"},{"varval":"0.36 kg is greater than 0.15 kg and less\r than 0.4 kg"}]},{"vars":[{"varval":"Oscar, Khan and Paul are weighing their grapes.\n\n<br>\n\nsm_img https://teacher.smartermaths.com.au/wp-content/uploads/2019/01/NAPX-J2-25-v1-768x357.png 460 indent vpad\r\n\n<br>Khan's grapes weigh more than Paul's, but less than Oscar's grapes.\n\n\r\n\r\nWhich of these could be the weight of Khan's grapes?"},{"varval":"0.4 kg \n\n\r\nSince 0.25 < 0.4 < 0.6"}]}]

  189. Raph14-10 Raph14-11 Raph14-12 Redid the graphs. Changed "table" to "graph" in var0 and var2 question. Used Police Officer instead of Policeman and changed Businessman to Carpenter in var2.

    <div class="sm_mode"> {{{question}}} </div>

    [{"vars":[{"varval":"Students are asked what is their favourite colour.\n\nThe results are recorded in the graph\nbelow.\n<br>\n\nsm_img https://teacher.smartermaths.com.au/wp-content/uploads/2022/11/Raph-14-Q10-12_v0.svg 500 indent vpad\n\n<br>Which statement is true?"},{"varval":"The bar representing the colour orange is the smallest.\n\nTherefore, orange is the favourite colour of the smallest number of students."}]},{"vars":[{"varval":"The graph below shows the amount of vegetables, by weight in kilograms, that were harvested at a farm.\n\n<br>\n\nsm_img https://teacher.smartermaths.com.au/wp-content/uploads/2022/11/Raph-14-Q10-12_v1.svg 500 indent vpad\n\n<br>Which statement is true?"},{"varval":"The bar representing potatoes is the tallest. \n\nTherefore, more potatoes were picked than any other vegetable."}]},{"vars":[{"varval":"Students were asked what they wanted to be when they grew up.\n\nTheir answers are recorded in the graph below.\n\n<br>\n\nsm_img https://teacher.smartermaths.com.au/wp-content/uploads/2022/11/Raph-14-Q10-12_v2.svg 500 indent vpad\n\n<br>Which statement is true?"},{"varval":"The bar representing nurse is the taller than police officer.\n\nTherefore, more students want to be a nurse than a police officer. "}]}]

  190. Number, NAPX-p116764v02 Number, NAPX-p116764v03 Number, NAPX-p116764v01 Number, NAPX-I2-19

    <div class="sm_mode"> {{{question}}} </div>

    [{"vars":[{"varval":"Pringle has 55 twenty-cent coins in his money pouch.\n\nsm_img https://teacher.smartermaths.com.au/wp-content/uploads/2021/04/q38-300x283.png 240 indent vpad\n\nHow much money does Pringle have?"},{"varval":"5 twenty-cent coins = $1.00\n\n<div class=\"aligned\">\n\n|||\n|-|-|\n|$\\therefore$ 55 twenty-cent coins\t |= 11 × $1.00|\n||= $11.00|\n\n</div>"}]},{"vars":[{"varval":"Peach had 8 fifty-cent coins and 15 twenty-cent coins in her purse.\n\n\r\n\r\nHow much money did she have in total?"},{"varval":"<div class=\"aligned\">\n\n|||\n|-|-|\n|Total|= 8 × 50¢ + 15 × 20¢|\n||= $4 + $3|\n||= $7|\n\n</div>\n"}]},{"vars":[{"varval":"Sheena has 26 twenty-cent coins in her piggy bank.\n\nsm_img https://teacher.smartermaths.com.au/wp-content/uploads/2021/04/q37.png 200 indent vpad\n\nHow much money does Sheena have?"},{"varval":"5 twenty-cent coins = $1.00\n\n<div class=\"aligned\">\n\n|||\n|-|-|\n|$\\therefore$ 26 twenty-cent coins\t |= 5 × $1.00 + 20 cents\t|\n||= $5.20|\n\n</div>\n"}]},{"vars":[{"varval":"Richie had 45 twenty cent coins in his pocket.\n\n\r\n\r\n\r\n\r\nHow much money did he have in total?"},{"varval":"Since 5 × 20¢ = $1\n\n\r\n<div class=\"aligned\">\n\n|||\n|-|-|\n|45 × 20¢|= 9 × $1|\n||= $9|\n\n</div>"}]},{"vars":[{"varval":"Cassius had 25 twenty-cent coins in his pocket.\n\n\r\n\r\nHow much money did he have in total?"},{"varval":"Since 5 × 20¢ = $1,\n\n<div class=\"aligned\">\n\n|||\n|-|-|\n|25 × 20¢|= 5 × $1|\n||= $5|\n\n</div>"}]}]

  191. Number, NAPX-p123173v01

    <div class="sm_mode"> {{{question}}} </div>

    [{"vars":[{"varval":"Gordon buys $\\dfrac{1}{5}$ of a kilogram of grapes.\n\n<br>\n\nsm_img https://teacher.smartermaths.com.au/wp-content/uploads/2019/01/NAPX-J2-20-v3-300x204.png 160 indent vpad\n\n<br>Which is another way to write $\\dfrac{1}{5}$ of a kilogram?"},{"varval":"$\\dfrac{1}{5}$ kilogram = 0.2 kilogram"}]},{"vars":[{"varval":"Carrie purchases $\\dfrac{7}{10}$ of a kilogram of gold.\n\nsm_img https://teacher.smartermaths.com.au/wp-content/uploads/2018/10/NAPX-I2-18-gold_rev.svg 145 indent vpad\n\nWhich is another way to write $\\dfrac{7}{10}$ of a kilogram?"},{"varval":"$\\dfrac{7}{10}$ kilogram = 0.7 kilogram"}]}]

  192. Number, NAPX-p116838v03 Number, NAPX-p116838v02 Number, NAPX-I2-16

    <div class="sm_mode"> {{{question}}} </div>

    [{"vars":[{"varval":"In a bakery Michelle bought one slice of cake, one donut and a croissant.\n\n<br>\n\nsm_img https://teacher.smartermaths.com.au/wp-content/uploads/2021/04/raph8-q32.svg 400 indent vpad\n\n<br>She gave the cashier $10.00.\n\n\r\n\r\nHow much change should Michelle receive?"},{"varval":"<div class=\"aligned\">\n\n|||\n|-|-|\n|Total cost|= 1.35 + 1.20 + 0.75|\n||= $3.30|\n\n</div>\n\n<div class=\"aligned\">\n\n|||\n|-|-|\n|Change|= 10 $-$ 3.3|\n||= $6.70|\n\n</div>\n"}]},{"vars":[{"varval":"Alexander went shopping and bought the following items.\n\n<br>\n\nsm_img https://teacher.smartermaths.com.au/wp-content/uploads/2021/04/q31-768x469.png 400 indent vpad\n\n<br>He gave the cashier $50.\n\n\r\n\r\nHow much change should Alexander receive?"},{"varval":"<div class=\"aligned\">\n\n|||\n|-|-|\n|Total cost|= 9.50 + 12.10 + 11.40|\n||= $33.00|\n\n</div>\n\n<div class=\"aligned\">\n\n|||\n|-|-|\n|Change|= 50 $-$ 33.00|\n||= $17.00|\n\n</div>\n"}]},{"vars":[{"varval":"Marvin buys the following items.\n\n<br>\n\nsm_img https://teacher.smartermaths.com.au/wp-content/uploads/2018/10/NAPX-I2-16-v3.svg 400 indent vpad\n\n<br>He gives the shop owner $40.\n\n\r\n\r\nHow much change should she get?"},{"varval":"<div class=\"aligned\">\n\n|||\n|-|-|\n|Total cost|= $6.20 + $7.30 + $21.50|\n||= $35|\n\n</div>\n\n<div class=\"aligned\">\n\n|||\n|-|-|\n|Change|= 40 $-$ 35|\n||= $5|\n\n</div>\n"}]},{"vars":[{"varval":"Cynthia buys the following items.\n\n<br>\n\nsm_img https://teacher.smartermaths.com.au/wp-content/uploads/2018/10/NAPX-I2-16-v1.svg 400 indent vpad\n\n<br>Cynthia gives the shop owner $50.\n\n\r\n\r\nHow much change should she get?"},{"varval":"<div class=\"aligned\">\n\n|||\n|-|-|\n|Total cost|= $4.40 + $5.20 + $31.40|\n||= $41|\n\n</div>\n\n<div class=\"aligned\">\n\n|||\n|-|-|\n|Change|= 50 $-$ 41|\n||= $9|\n\n</div>\n"}]}]

  193. <div class="sm_mode"> {{{question}}} </div>

    [{"vars":[{"varval":"Sylvester painted squares white and grey on the grids pictured below.\n\n\r\n\r\nAll the squares in the grids are the same size.\n\n\r\n\r\nWhich grid has the largest area painted grey?"},{"varval":"The largest area painted grey is 16 squares.\n\n{{{correctAnswer}}}\n"}]},{"vars":[{"varval":"Betty painted squares white and grey on the grids pictured below.\n\n\r\n\r\nAll the squares in the grids are the same size.\n\n\r\n\r\nWhich grid has the smallest area painted white?"},{"varval":"The smallest area painted white is 9 squares.\n\n{{{correctAnswer}}}\n"}]}]

  194. <div class="sm_mode"> Mischa has poured cordial into some measuring cups. She makes them all exactly one quarter full. Mischa counts by quarters to find out, in total, how many full cups of cordial she has. sm_img https://teacher.smartermaths.com.au/wp-content/uploads/2018/10/NAPX-J2-12-v1_1.svg 440 indent vpad Which number does Mischa count next? </div>

    [{"vars":null}]

  195. <div class="sm_mode"> {{{question}}} </div>

    [{"vars":[{"varval":"sm_img https://teacher.smartermaths.com.au/wp-content/uploads/2019/01/NAPX-J2-07-v2_1.png 420 indent vpad\n\nWhich set of coins shows the **same** amount of money as the coins above?​"},{"varval":"\n\nCoins need to add up to:\n \n$1 + 50¢ + 20¢ + 5¢ = $1.75\n\n \n\nConsider the 2nd option:\n\n\nsm_nogap 50¢ + 10¢ + 10¢ + 50¢ + 50¢ + 5¢\n\n<div class=\"aligned\">\n\n>>||\n|-|\n|= 175¢|\n|= $1.75|\n\n\n</div>\n\n<br>\n\n{{{correctAnswer}}}"}]},{"vars":[{"varval":"sm_img https://teacher.smartermaths.com.au/wp-content/uploads/2019/01/NAPX-J2-07-v2b-768x141.png 420 indent vpad\n\nWhich set of coins shows the **same** amount of money as the coins above?​"},{"varval":"\n\nCoins need to add up to:\n \n50¢ + 10¢ + 10¢ + 50¢ + 50¢ + 5¢ = $1.75\n \n\nConsider the 1st option:\n\n\nsm_nogap $1 + 50¢ + 20¢ + 5¢ \n\n<div class=\"aligned\">\n\n>>||\n|-|\n|= 175¢|\n|= $1.75|\n\n\n</div>\n\n<br>\n\n{{{correctAnswer}}}"}]}]

  196. Note: For these special HTML backbround bullet points, you cannot use them inside an sm_mode div. <div style="background-color: lightyellow; padding: 5px 25px 20px 15px; border: 2px solid; width: max-content; margin-left: 30px" > * It is **greater than 3000** but less than five hundred * First table with a color background * A third row </div> <br> <div style="background-color: lightcyan; padding: 5px 25px 20px 15px; border: 2px solid; width: max-content; margin-left: 30px" > * It is greater than 3000 but less than five hundred * First **table with a color** background * A third row </div>

    [{"vars":null}]

  197. Number, NAPX-K2-01

    <div class="sm_mode"> {{{question}}} </div>

    [{"vars":[{"varval":"Which of these coins has the greatest value?"},{"varval":"The $2 coin has the greatest value.\n\nsm_img https://teacher.smartermaths.com.au/wp-content/uploads/2020/03/cropped-2aud.png 70 indent vpad"}]},{"vars":[{"varval":"Which of these coins has the greatest value?"},{"varval":"The $1 coin has the greatest value.\n\nsm_img https://teacher.smartermaths.com.au/wp-content/uploads/2018/12/nap-K2-01f.svg 90 indent vpad"}]},{"vars":[{"varval":"Which of these coins has the greatest value?"},{"varval":"The $2 coin has the greatest value.\n\nsm_img https://teacher.smartermaths.com.au/wp-content/uploads/2020/03/2aud.png 100 indent vpad"}]},{"vars":[{"varval":"Which of these coins has the least value?"},{"varval":"The 10 cent coin has the least value.\n\nsm_img https://teacher.smartermaths.com.au/wp-content/uploads/2020/03/10cau.png 120 indent vpad\n"}]},{"vars":[{"varval":"Which of these coins has the least value?"},{"varval":"The 20 coin has the least solution.\n\n\nsm_img https://teacher.smartermaths.com.au/wp-content/uploads/2018/11/nap-K2-01c.svg 110 indent vpad\n"}]}]

  198. <div class="sm_mode"> ### Special Rules/Restrictions for variables/answers and workarounds... The markdown we specify in variables and answers is stored in a format called yaml that has the following restrictions: * Colons are used to delimit fields, so if you have just numbers and colons, the system gets confused and you must backslash the colons. This probably only will occur with clock times as per the following: * 12:12 will fail since it is numbers and colons only, and should instead be 12:12 * 1:2:3 should be 1:2:3 * 12:12 pm is ok since it has a space then pm so the system knows its text. * Answer is 1:2:3 is ok also for the same reason. * There are reserved words in the yaml language that if we want them in variables/answers we have to treat in a certain way. * The reserved words are: * yes no on off true false y n * Just like with clock times above, a reserved word alone is a problem, but with other text it is no problem! For example: * Problem: * yes * true * But the following are no problem: * yes is the answer * it is in fact true * To encode a reserved word as a variable for example, do the following: 1. Instead of Yes, make the variable be <span>Yes</span> 2. Refer to {{{variable}}} with the three curly brackets not just the normal two - - - ### Force a space to be rendered This is a sentence where all spaces are collapsed (normal). This is a sentence with a forced extra space. - - - ### Red box ** NOTE: THIS IS NOW OLD/DEPRECATED as at 2021-08-10 ** No spaces: <span class="sm_box"></span> One space inside: <span class="sm_box"> ` ` </span> Two spaces inside: <span class="sm_box"> ` ` ` ` </span> </div>

    [{"vars":null}]

  199. <div class="sm_mode"> ## Docs - Katex ### Dollar sign within katex/latex $$\text{\textdollar 9000 here is the dollar sign}$$ or even easier is to specify a backslash before the dollar sign either inside or outside the katex: $$\$9\alpha$$ ### Percentage Sign To use percentage sign, use a backslash. $$\%$$ ### When bold text is not working Make sure to put proper break tag, for example: Grant joined the two objects below. <br> sm_img https://teacher.smartermaths.com.au/wp-content/uploads/2017/12/nap-b4-nc09.svg 160 indent3 vpad <br> Which object below could **not** be made using Grant's 2 objects? - - - ### Force katex font (1em size) Force the use of katex fonts and over-ride the SmarterMaths default of using the system font. This is required in rare cases where a katex symbol/instruction is not rendering appropriately (due to issues in using the system font for rendering that particular instruction.) You can use the class="sm_katex-font" inside any div or span tag to force the katex-font. For example: * Using system font: $$\ne 123 2^2 \alpha$$. Notice the not equals symbol does not render correctly. * Forcing the katex font: <span class="sm_katex-font">$$\ne \angle$$</span>$$123 2^2 \alpha$$. Notice the not equals symbol now renders correctly. And the remaining katex is specified *outside* of the span so it continues to match/use the system font. </div>

    [{"vars":null}]

  200. <div class="sm_mode"> ### Table Please see [GitHub flavoured markdown](https://github.github.com/gfm/#tables-extension-) for further details such as using the colon to right/left align the cell contents. Table Notes: 1. SmarterMaths indenting format (greater than signs on the first line) works with tables, but will be put on all lines if you use Rich Text to edit...both are fine. 2. If you edit these tables in Rich Text mode although it will space out the table appropriately in markdown mode which can be convenient, it will also unfortunately remove any colon right or left alignment if you've specified them, so be careful or you may have to reapply your alignment. 3. Specify a table type with a div tag specifying a class as shown * Table type "no-outline" has the following format: <div class="no-outline"> > > | Name | Distance | > > | ------ | ----------------- | > > | Billy | 56 656 | > > | Sandra | {{correctAnswer}} | </div> * Table type "outline" has the following format: <div class="outline"> > > | Name | Distance | > > | ------ | ----------------- | > > | Billy | 56 656 | > > | Sandra | {{correctAnswer}} | </div> * Table type "aligned" has the following format: sm_nogap Example table that can be used for alignment. (Leave header row empty but defined, and colons define alignment. No indent is default except for 3rd example below.) <div class="aligned"> | | | | --------------------- | -------------- | | 10 km/hr x 30 min | \= 10 x 0.5 hr | | $$ \therefore $$ eggs | \= 70% yolk | </div> <br> sm_nogap And another example below (no indent): <div class="aligned"> | | | | --------------------- | ------------------------------------------- | | $$ \text{Distance} $$ | \= $$ \text{speed} \times \text{time} $$ | | | \= $$ \text{speed} \times \dfrac{30}{60} $$ | | | \= {{correctAnswer}} | </div> <br> sm_nogap 3rd example - when only one column (indent used here): <br> sm_nogap Total distance travelled <div class="aligned"> > > | | > > | ------------------------------------------- | > > | \= $$ \text{speed} \times \text{time} $$ | > > | \= $$ \text{speed} \times \dfrac{30}{60} $$ | > > | \= {{correctAnswer}} | </div> Note: End of sm_mode div is below. </div>

    [{"vars":null}]

  201. Docs 2

    <div class="sm_mode"> * The sm_mode in the div (above it is viewable in the editor) with a closing tag at the bottom of this post modifies some of the regular markdown functionality, as show in this post below. - - - #### Please note: This page functions as a reference for post encoding usage, and will continue to be updated. - - - ### Markdown Please see [this link](https://www.markdownguide.org/basic-syntax/) to understand how to use markdown. This is a sentence in it's own paragraph. Notice there is a blank line above and below it. And another regular line. But this one is one the same line as the last...you need a blank line in markdown to signal a new paragraph. Please read this in the editor, and then see how it actually prints out in the right hand pane, the preview/viewer. #### 1a. Spaces and new lines Spaces and blank lines are treated differently in markdown.\ They are all collapsed into a single space as indicated by this line, which is actually on a new line in the editor, but doesn't have a blank line above/below. In these cases, multiple spaces and even a new line is all translated into a single space. #### 1.b How to have text be immediately on the next line without a new paragraph If you want a new line but don't want to have an extra spacing line, or you already have one, but need another, then use the backward slash\ such as this line\ To skip a line, you can use a single space with a backslash\ \ \ \ such as on the lines above this one. #### 1.c When the backslash newline won't work, do this... Sometimes this will not work (specifically if the next line is a markdown block) and you can use the "sm_nogap" indication, as shown below. sm_nogap This is a special line (used here before an indent block, but can be used anywhere) that has no margin below. > This is an indented markdown block.\ > and another! <br> #### 1.d Another way of achieving blank lines * Every so often you may need to force a new line(s) with one or more <br> (without the backslashes) tag but that should be an exception! Use sparingly. - - - ### Indenting text > This is an indented markdown block. To indent, just put a greaterthan sign on the first line of a block of text.\ > and another line! > > and yet another on a new line. Notice that the greater than sign is a markdown code, so you can either have a blank line above/below OR use the continuation backslash line to ensure the text continues on the next line. > > > and now 2 levels of indent\ > > and another one at 2 levels\ > > \ > > and some more with formulas $$\therefore x^2 \alpha$$ on this line > > and now back to one level of indent > > $$\alpha$$ and some more\ > Images can be embedded here also, but the indent setting on the image over-rides any number of the greater than signs. e.g. sm_img https://teacher.smartermaths.com.au/wp-content/uploads/2020/06/Functions-2ADV-F1-EQ-Bank-6.svg 200 indent vpad - - - ### Image with indent (note indent can be indent2 or indent3 also) * You can remove the vpad or the indent at the end of the line to NOT indent and NOT vertical pad, but normally you would want those. sm_img https://teacher.smartermaths.com.au/wp-content/uploads/2020/06/Functions-2ADV-F1-EQ-Bank-6.svg 200 indent vpad - - - ### Side by side images (note indent can be indent2 or indent3 also) sm_img https://teacher.smartermaths.com.au/wp-content/uploads/2020/06/Functions-2ADV-F1-EQ-Bank-6.svg 200 indent vpad sm_img https://teacher.smartermaths.com.au/wp-content/uploads/2020/06/Functions-2ADV-F1-EQ-Bank-6.svg 200 indent vpad sm_img https://teacher.smartermaths.com.au/wp-content/uploads/2020/06/Functions-2ADV-F1-EQ-Bank-6.svg 200 indent vpad </div>

    [{"vars":null}]

  202. Markdown tables

    <div class="sm_mode"> In the tables, you can use color1 through color19 ... mostly you should use no color (the default is light blue when no color is specified) or color 1 through color6 which are from our color palette for images and are duplicated here for the tables. In addition, when no coloring at all is required, use color_white. For example: #### Tables with outline Default (no color explicitly specified) <div class="outline"> > | WHITE | YELLOW | BLUE | > | :-----------: | :----------: | :------------: | > | 34 | 2 | 1 | </div> <br> Color 1 (color1) <div class="outline color1"> > | WHITE | YELLOW | BLUE | > | :-----------: | :----------: | :------------: | > | 34 | 2 | 1 | </div> <br> Color 2 <div class="outline color2"> > | WHITE | YELLOW | BLUE | > | :-----------: | :----------: | :------------: | > | 34 | 2 | 1 | </div> <br> Color 3 <div class="outline color3"> > | WHITE | YELLOW | BLUE | > | :-----------: | :----------: | :------------: | > | 34 | 2 | 1 | </div> <br> Color 4 <div class="outline color4"> > | WHITE | YELLOW | BLUE | > | :-----------: | :----------: | :------------: | > | 34 | 2 | 1 | </div> <br> Color 5 <div class="outline color5"> > | WHITE | YELLOW | BLUE | > | :-----------: | :----------: | :------------: | > | 34 | 2 | 1 | </div> Color 6 <br> <div class="outline color6"> > | WHITE | YELLOW | BLUE | > | :-----------: | :----------: | :------------: | > | 34 | 2 | 1 | </div> <br> Color 7 <div class="outline color7"> > | WHITE | YELLOW | BLUE | > | :-----------: | :----------: | :------------: | > | 34 | 2 | 1 | </div> Color 8 <br> <div class="outline color8"> > | WHITE | YELLOW | BLUE | > | :-----------: | :----------: | :------------: | > | 34 | 2 | 1 | </div> Color 9 <br> <div class="outline color9"> > | WHITE | YELLOW | BLUE | > | :-----------: | :----------: | :------------: | > | 34 | 2 | 1 | </div> Color 10 <br> <div class="outline color10"> > | WHITE | YELLOW | BLUE | > | :-----------: | :----------: | :------------: | > | 34 | 2 | 1 | </div> <br> Color 11 <div class="outline color11"> > | WHITE | YELLOW | BLUE | > | :-----------: | :----------: | :------------: | > | 34 | 2 | 1 | </div> <br> Color 12 <div class="outline color12"> > | WHITE | YELLOW | BLUE | > | :-----------: | :----------: | :------------: | > | 34 | 2 | 1 | </div> <br> Color 13 <div class="outline color13"> > | WHITE | YELLOW | BLUE | > | :-----------: | :----------: | :------------: | > | 34 | 2 | 1 | </div> <br> Color 14 <div class="outline color14"> > | WHITE | YELLOW | BLUE | > | :-----------: | :----------: | :------------: | > | 34 | 2 | 1 | </div> <br> Color 15 <div class="outline color15"> > | WHITE | YELLOW | BLUE | > | :-----------: | :----------: | :------------: | > | 34 | 2 | 1 | </div> <br> Color 16 <div class="outline color16"> > | WHITE | YELLOW | BLUE | > | :-----------: | :----------: | :------------: | > | 34 | 2 | 1 | </div> <br> Color 17 <div class="outline color17"> > | WHITE | YELLOW | BLUE | > | :-----------: | :----------: | :------------: | > | 34 | 2 | 1 | </div> <br> Color 18 <div class="outline color18"> > | WHITE | YELLOW | BLUE | > | :-----------: | :----------: | :------------: | > | 34 | 2 | 1 | </div> <br> Color 19 <div class="outline color19"> > | WHITE | YELLOW | BLUE | > | :-----------: | :----------: | :------------: | > | 34 | 2 | 1 | </div> <br> Color White (color_white) <div class="outline color_white"> > | WHITE | YELLOW | BLUE | > | :-----------: | :----------: | :------------: | > | 34 | 2 | 1 | </div> <br> #### Tables with no-outline <div class="no-outline"> > | WHITE | YELLOW | BLUE | > | :-----------: | :----------: | :------------: | > | 34 | 2 | 1 | </div> <br> <div class="no-outline color1"> > | WHITE | YELLOW | BLUE | > | :-----------: | :----------: | :------------: | > | 34 | 2 | 1 | </div> <br> <div class="no-outline color2"> > | WHITE | YELLOW | BLUE | > | :-----------: | :----------: | :------------: | > | 34 | 2 | 1 | </div> <br> <div class="no-outline color3"> > | WHITE | YELLOW | BLUE | > | :-----------: | :----------: | :------------: | > | 34 | 2 | 1 | </div> <br> <div class="no-outline color4"> > | WHITE | YELLOW | BLUE | > | :-----------: | :----------: | :------------: | > | 34 | 2 | 1 | </div> <br> <div class="no-outline color5"> > | WHITE | YELLOW | BLUE | > | :-----------: | :----------: | :------------: | > | 34 | 2 | 1 | </div> <br> <div class="no-outline color6"> > | WHITE | YELLOW | BLUE | > | :-----------: | :----------: | :------------: | > | 34 | 2 | 1 | </div> <br> <div class="no-outline color7"> > | WHITE | YELLOW | BLUE | > | :-----------: | :----------: | :------------: | > | 34 | 2 | 1 | </div> <br> <div class="no-outline color8"> > | WHITE | YELLOW | BLUE | > | :-----------: | :----------: | :------------: | > | 34 | 2 | 1 | </div> <br> <div class="no-outline color9"> > | WHITE | YELLOW | BLUE | > | :-----------: | :----------: | :------------: | > | 34 | 2 | 1 | </div> <br> <div class="no-outline color10"> > | WHITE | YELLOW | BLUE | > | :-----------: | :----------: | :------------: | > | 34 | 2 | 1 | </div> <br> <div class="no-outline color11"> > | WHITE | YELLOW | BLUE | > | :-----------: | :----------: | :------------: | > | 34 | 2 | 1 | </div> <br> <div class="no-outline color12"> > | WHITE | YELLOW | BLUE | > | :-----------: | :----------: | :------------: | > | 34 | 2 | 1 | </div> <br> <div class="no-outline color13"> > | WHITE | YELLOW | BLUE | > | :-----------: | :----------: | :------------: | > | 34 | 2 | 1 | </div> <br> <div class="no-outline color14"> > | WHITE | YELLOW | BLUE | > | :-----------: | :----------: | :------------: | > | 34 | 2 | 1 | </div> <br> <div class="no-outline color15"> > | WHITE | YELLOW | BLUE | > | :-----------: | :----------: | :------------: | > | 34 | 2 | 1 | </div> <br> <div class="no-outline color16"> > | WHITE | YELLOW | BLUE | > | :-----------: | :----------: | :------------: | > | 34 | 2 | 1 | </div> <br> <div class="no-outline color17"> > | WHITE | YELLOW | BLUE | > | :-----------: | :----------: | :------------: | > | 34 | 2 | 1 | </div> <br> <div class="no-outline color18"> > | WHITE | YELLOW | BLUE | > | :-----------: | :----------: | :------------: | > | 34 | 2 | 1 | </div> <br> <div class="no-outline color19"> > | WHITE | YELLOW | BLUE | > | :-----------: | :----------: | :------------: | > | 34 | 2 | 1 | </div> <br> <div class="no-outline color_white"> > | WHITE | YELLOW | BLUE | > | :-----------: | :----------: | :------------: | > | 34 | 2 | 1 | </div> <br> The one below this inside a variable {{{question1}}} </div>

    [{"vars":[{"varval":"<div class=\"outline color2\">\n\n> | WHITE | YELLOW | BLUE |\n> | :-----------: | :----------: | :------------: |\n> | 123 | 234 | 345 |\n\n</div>"}]}]

  203. Number, NAPX-p73014v02 Number, NAPX-p73014v01 Number, NAPX-p117011v01 Number, NAPX-I2-26

    {{{question}}}

    [{"vars":[{"varval":"Idriss thinks of a number.\n\n\r\n\r\nHe writes these clues for his classmates.\n\n\n\n<div style=\"background-color: lightyellow; padding: 5px 25px 20px 15px; border: 2px solid; width: max-content; margin-left: 20px\" >\n\n* **The number is an odd number**\n* **It is greater than 300 but less than 500**\n* **The tens digit is the same as its hundreds digit**\n\n</div>\n\n\n\n<br>\n\nWhat is Idriss' number?"},{"varval":"By elimination:\n\n\r\nNumber is between 300 and 500 ⇒ Eliminate 665.\n\n\r\nTen's digit = hundred's digit\n\n\r\n⇒ Eliminate 363 and 477.\n\n\r\n$\\therefore$ Number must be 447."}]},{"vars":[{"varval":"Fatima thinks of a number.\n\n\r\n\r\nShe writes these clues so her friends can guess it.\r\n\n\n\n\n<div style=\"background-color: lightcyan; padding: 5px 25px 20px 15px; border: 2px solid; width: max-content; margin-left: 20px\" >\n\n* **The number is an even number**\n* **It is greater than 100 but less than 1000**\n* **The tens digit is the same as its hundreds digit**\n\n</div>\n\n<br>\n\nWhat is Fatima's's number?"},{"varval":"By elimination:\n\n\r\nNumber is even ⇒ Eliminate 355 and 883\n\n\r\nTen's digit = hundred's digit\n\n\r\n⇒ Eliminate 808.\n\n\r\n$\\therefore$ Number must be 992."}]},{"vars":[{"varval":"Yasmine thinks of a number.\n\n\r\n\r\nShe writes these clues so her friends can guess it.\n\n\n<div style=\"background-color: lightyellow; padding: 5px 25px 20px 15px; border: 2px solid; width: max-content; margin-left: 20px\" >\n\n* **The number is an odd number**\n* **It is greater than 500 but less than 800**\n* **The ones digit is the same as its hundreds digit**\n\n</div>\n\n<br>\n\nWhat is Yasmine's number?"},{"varval":"By elimination: \n\n\r\nNumber is odd ⇒ Eliminate 606\n\n\r\nOne's digits = hundred's digit\n\n\r\n⇒ Eliminate 655 and 779.\n\n\r\n$\\therefore$ Number must be 737."}]},{"vars":[{"varval":"\r\nJade writes a number on a piece of paper and covers it.\n\n\r\n\r\nShe gives her friend the following clues about her number.\n\n\n\n<div style=\"background-color: lightcyan; padding: 5px 25px 20px 15px; border: 2px solid; width: max-content; margin-left: 20px\" >\n\n* **The number is less than 1000**\n* **The number has 80 tens**\n* **The digit in the ones place is half the digit in the hundreds place.**\n\n</div>\n\n<br>\n\n\r\nWhat is Jade's number?"},{"varval":"Consider the following clues,\n\n\r\nNumber has 80 tens,\n\n\r\n⇒ Number is 3 digits, starting with 80_\n\n\r\n\r\n\r\nThe “ones” digit is half the “hundreds” digit,\n\n\r\n$\\therefore$ Number must be 804."}]},{"vars":[{"varval":"Wyatt writes a number on a piece of paper and hides it.\n\n\r\n\r\nHe gives his friend the following clues about his number.\n\n<br>\n\n<div style=\"background-color: lightyellow; padding: 5px 25px 20px 15px; border: 2px solid; width: max-content; margin-left: 20px\" >\n\n* **The number is less than 600**\n* **The number has 40 tens**\n* **The digit in the ones place is two less than the digit in the hundreds place.**\n\n</div>\n\n<br>\n\n\r\nWhat is Wyatt's number?"},{"varval":"Consider the following clues,\n\n\r\nNumber has 40 tens,\n\n\r\n⇒ Number is 3 digits, starting with 40_\n\n\r\n\r\n\r\nThe “ones” digit is two less than the “hundreds” digit,\n\n\r\n$\\therefore$ Number must be 402."}]}]

  204. Number, NAPX-H2-25

    <div class="sm_mode"> {{{question}}} </div>

    [{"vars":[{"varval":"Which of these pairs of numbers has a difference of 23?"},{"varval":"51 $-$ 28 = 23\r\n\n$\\therefore$ 28 and 51"}]},{"vars":[{"varval":"Which of these pairs of numbers has a difference of 19?"},{"varval":"77 $-$ 58 = 19\n\n\r\n$\\therefore$ 58 and 77"}]}]

  205. Number, NAPX-p124236v01

    <div class="sm_mode"> {{{question}}} </div>

    [{"vars":[{"varval":"sm_img https://teacher.smartermaths.com.au/wp-content/uploads/2019/01/NAPX-G2-11v1.svg 320 indent vpad\n\nWhat is the missing number on this number line?"},{"varval":"sm_nogap Each interval is worth an extra 25.\n\n<div class=\"aligned\">\n\n|||\n|-|-|\n|$\\therefore$ Missing number|= 75 + 25|\n||= 100|\n\n</div>"}]},{"vars":[{"varval":"sm_img https://teacher.smartermaths.com.au/wp-content/uploads/2019/01/NAPX-G2-11v2.svg 320 indent vpad\n\nWhat is the missing number on this number line?"},{"varval":"sm_nogap Each interval is worth an extra 20.\n\n<div class=\"aligned\">\n\n|||\n|-|-|\n|$\\therefore$ Missing number|= 60 + 20 + 20|\n||= 100|\n\n</div>"}]}]

  206. <div class="sm_mode"> {{{question}}} </div>

    [{"vars":[{"varval":"In which one of these numbers does the numeral 4 represent 4 hundreds?"},{"varval":"6427\n\n\r\nsm_nogap In this number:\n\n\r\n>> 6 → thousands\n\n\r\n>> 4 → hundreds\n\n\r\n>> 2 → tens\n\n\r\n>> 7 → ones"}]},{"vars":[{"varval":"In which one of these numbers does the numeral 3 represent 3 hundreds?"},{"varval":"1304\n\nsm_nogap In this number:\n\n\r\n>> 1 → thousands\n\n\r\n>> 3 → hundreds\n\n\r\n>> 0 → tens\n\n\r\n>> 4 → ones"}]}]

  207. Number, NAPX-p116817v02 Number, NAPX-p116817v01

    <div class="sm_mode"> {{{question}}} What number makes the above number sentence correct? </div>

    [{"vars":[{"varval":"3 $\\times$ <span class=\"sm-text\">?</span> = 87\n\n"},{"varval":"sm_nogap Test each option:\n\n<div class=\"aligned\">\n\n|||\n|-|-|\n|3 × 29|= 3 × 20 + 3 × 9|\n||= 60 + 27|\n||= 87|\n\n</div>"}]},{"vars":[{"varval":"3 $\\times$ <span class=\"sm-text\">?</span> = 81"},{"varval":"sm_nogap Test each option:\n\n<div class=\"aligned\">\n\n|||\n|-|-|\n|3 × 27|= 3 × 20 + 3 × 7|\n||= 60 + 21|\n||= 81|\n\n</div>"}]},{"vars":[{"varval":"5 $\\times$ <span class=\"sm-text\">?</span> = 65"},{"varval":"sm_nogap Test each ooption:\n\n<div class=\"aligned\">\n\n\r\n\r\n|||\r\n|-|-|\r\n|5 × 13|= 5 × 10 + 5 × 3|\r\n||= 50 + 15|\r\n||= 65|\n\n\r\n\r\n</div>"}]},{"vars":[{"varval":"4 $\\times$ <span class=\"sm-text\">?</span> = 68"},{"varval":"sm_nogap Test each option:\n\n<div class=\"aligned\">\n\n\r\n\r\n|||\r\n|-|-|\r\n|4 × 17|= 4 × 10 + 4 × 7|\r\n||= 40 + 28|\r\n||= 68|\n\n\r\n\r\n</div>"}]}]

  208. Number, NAPX-p116801v01 Number, NAPX-H2-05

    <div class="sm_mode"> {{{question}}} </div>

    [{"vars":[{"varval":"What number is 10 less than 1005?"},{"varval":"1005 $−$ 10 = 995"}]},{"vars":[{"varval":"What number is 10 less than 1108?"},{"varval":"1108 $−$ 10 = 1098"}]},{"vars":[{"varval":"What number is 10 less than 2002?"},{"varval":"2002 $−$ 10 = 1992"}]}]

  209. Number, NAPX-p116663v01 Number, NAPX-I2-06 Number, NAPX-I2-p116663v02

    <div class="sm_mode"> {{{question}}} </div>

    [{"vars":[{"varval":"Danica travelled a total of two thousand, nine hundred and nine kilometres from Brisbane to Adelaide.\n\n<br>\n\nsm_img https://teacher.smartermaths.com.au/wp-content/uploads/2021/04/RAPH8-Q12.svg 280 indent3 vpad\n\n<br>The distance travelled can be written as:"},{"varval":"Two thousand, nine hundred and nine = 2909"}]},{"vars":[{"varval":"Pamela skips rope. In one hour she completed five thousand two hundred and five skips.\n\n<br>\n\nsm_img https://teacher.smartermaths.com.au/wp-content/uploads/2018/10/NAPX-I2-06-v2.svg 250 indent3 vpad\n\n<br>The number of skips can be written as:"},{"varval":"sm_nogap Five thousand two hundred and five contains:\n\n<div class=\"aligned\">\n\n>>||\n|-|\n|⇒ 5 “thousands”|\n|⇒ 2 “hundreds”|\n|⇒ 0 “tens”|\n|⇒ 5 “ones”|\n\n\r\n</div>\n\n<br>\n\n\r\n$\\therefore$ Five thousand two hundred and five = 5205"}]},{"vars":[{"varval":"Olive skips rope.\n\r\nIn twenty-five minutes she completed two thousand and thirty nine skips.\n\n<br>\n\nsm_img https://teacher.smartermaths.com.au/wp-content/uploads/2018/10/NAPX-I2-06-v1.svg 200 indent3 vpad\n\n<br>The number of skips can be written as:"},{"varval":"sm_nogap Two thousand and thirty nine contains:\n\n<div class=\"aligned\">\n\n>>||\n|-|\n|⇒ 2 “thousands”|\n|⇒ 0 “hundreds”|\n|⇒ 3 “tens”|\n|⇒ 9 “ones”|\n\r\n\r\n\r</div>\n\n<br>\n\n\r\n$\\therefore$ Two thousand and thirty nine = 2039"}]},{"vars":[{"varval":"Arnie competed in the World Championship for the most push-ups in one hour.\n\r\nHe completed two thousand and eighty six push-ups in one hour.\n\n<br>\n\nsm_img https://teacher.smartermaths.com.au/wp-content/uploads/2019/01/NAPX-I2-06-v3.svg 400 indent vpad\n\n\r\n<br>The number of push-ups can be written as:"},{"varval":"\r\nsm_nogap Two thousand and eighty six contains:\n\n<div class=\"aligned\">\n\n>>||\n|-|\n|⇒ 2 “thousands”|\n|⇒ 0 “hundreds”|\n|⇒ 8 “tens”|\n|⇒ 6 “ones”|\n\n</div>\n\n<br>\n\r\n$\\therefore$ Two thousand and eighty six = 2086"}]}]

  210. Number, NAPX-p116787v01

    <div class="sm_mode"> {{{question}}} </div>

    [{"vars":[{"varval":"sm_img https://teacher.smartermaths.com.au/wp-content/uploads/2021/03/NAPX5-TLA-3-v2.svg 360 indent vpad\n\n<br>Which arrow points to the number that is halfway between 10 and 26?"},{"varval":"sm_nogap Halfway between 10 and 26\n\n<div class=\"aligned\">\n\n>>||\n|-|\n|= $\\dfrac{10 + 26}{2}$|\n|= $\\dfrac{36}{2}$|\n|= 18|\n\n</div>\n\n<br>\n\n$\\rArr C$ points to halfway."}]},{"vars":[{"varval":"sm_img https://teacher.smartermaths.com.au/wp-content/uploads/2021/03/NAPX5-TLA-3-v1.svg 330 indent vpad\n\n<br>Which arrow points to the number that is halfway between 18 and 30?"},{"varval":"sm_nogap Halfway between 18 and 30\n\n<div class=\"aligned\">\n\n>>||\n|-|\n|= $\\dfrac{18 + 30}{2}$|\n|= $\\dfrac{48}{2}$|\n|= 24|\n\n</div>\n\n<br>\n\n$\\rArr B$ points to halfway."}]},{"vars":[{"varval":"sm_img https://teacher.smartermaths.com.au/wp-content/uploads/2018/10/NAPX-H2-03-v2.svg 460 indent vpad\n\n<br>Which arrow points to the number that is halfway between 15 and 45?"},{"varval":"Solution 1\n\nsm_nogap Halfway between 15 and 45\n\n<div class=\"aligned\">\n\n>>||\n|-|\n|= $\\dfrac{15 + 45}{2}$|\n|= $\\dfrac{60}{2}$|\n|= 30|\n\n</div>\n\n<br>\n\nSolution 2\n\n\r\n(This point can be found by the three interval jumps needed from both 15 and 45 to get to 30)\n\nsm_img https://teacher.smartermaths.com.au/wp-content/uploads/2018/10/NAPX-H2-03-v2-Answer.svg 450 indent vpad\n\n<br>$\\rArr B$ points to halfway."}]}]

  211. Number, NAPX-p116616v02 Number, NAPX-p116616v01 Number, NAPX-I2-03

    <div class="sm_mode"> {{{question}}} </div>

    [{"vars":[{"varval":"Eric owns a book which has more than 258 pages but less than 285 pages.\n\n\r\n\r\nWhich of these could represent the number of pages in Eric’s book?"},{"varval":"Check each option:\n\n\r\n287 is more than 285 ⇒ incorrect\n\n\r\n294 is more than 285 ⇒ incorrect\n\n\r\n249 is less than 258 ⇒ incorrect\n\n\r\n262 is more than 258 and less than 285 ⇒ correct"}]},{"vars":[{"varval":"Harold is travelling between Alice Springs and the Davenport Ranges.\n\n\r\n\r\nHe knows it is less than 476 kilometres but more than 448 kilometres.\n\n\r\n\r\nWhich of these could be the number of kilometres that Harold has to travel?"},{"varval":"468 is less than 476 and more than 448."}]},{"vars":[{"varval":"Nick is travelling between Margaret River and Geraldton.\n\n\r\n\r\nHe knows it is more than 646 kilometres but less than 664 kilometres.\n\n\r\n\r\nWhich of these could be the number of kilometres that Nick has to travel?"},{"varval":"662 is more than 646 and less than 664."}]},{"vars":[{"varval":"Kate is travelling from Byron Bay to Port Macquarie.\n\n\r\n\r\nShe knows that it is more 357 kilometres but less than 375 kilometres.\n\n\r\n\r\nWhich of these could be the number of kilometres that Kate has to travel?"},{"varval":"373 is more than 357 and less than 375."}]}]

  212. <div class="sm_mode"> {{{question}}} </div>

    [{"vars":[{"varval":"James rolled a dice once.\n\nThe number he rolled was odd and greater than one.\n\nWhich of these numbers could James have rolled?\n"},{"varval":"{{{correctAnswer}}}"}]},{"vars":[{"varval":"Kin rolled a dice once.\n\nThe number he rolled was even and greater than two.\n\nWhich of these numbers could Kin have rolled?\n"},{"varval":"{{{correctAnswer}}}"}]},{"vars":[{"varval":"Leilani rolled a dice once.\n\nThe number she rolled was odd and greater than three.\n\nWhich of these numbers could Leilani have rolled?"},{"varval":"{{{correctAnswer}}}"}]},{"vars":[{"varval":"Armani rolled a dice once.\n\nThe number he rolled was even and greater than two.\n\nWhich of these numbers could Armani have rolled?"},{"varval":"{{{correctAnswer}}}"}]},{"vars":[{"varval":"Huxley rolled a dice once.\n\nThe number he rolled was odd and less than three.\n\nWhich of these numbers could Huxley have rolled?"},{"varval":"{{{correctAnswer}}}"}]},{"vars":[{"varval":"Scarlett rolled a dice once.\n\nThe number she rolled was even and greater than four.\n\nWhich of these numbers could Scarlett have rolled?"},{"varval":"{{{correctAnswer}}}"}]}]

  213. <div class="sm_mode"> {{{question}}} </div>

    [{"vars":[{"varval":"Which one of these gives an answer of 63?"},{"varval":"{{{correctAnswer}}}"}]},{"vars":[{"varval":"Which one of these gives an answer of 49?"},{"varval":"{{{correctAnswer}}}"}]},{"vars":[{"varval":"Which one of these gives an answer of 85?"},{"varval":"{{{correctAnswer}}}"}]},{"vars":[{"varval":"Which one of these gives an answer of 97?"},{"varval":"{{{correctAnswer}}}"}]},{"vars":[{"varval":"Which one of these gives an answer of 59?"},{"varval":"{{{correctAnswer}}}"}]},{"vars":[{"varval":"Which one of these gives an answer of 105?"},{"varval":"{{{correctAnswer}}}"}]}]

  214. <div class="sm_mode"> This is a map of a pathway through Valentine's school. <br> sm_img https://teacher.smartermaths.com.au/wp-content/uploads/2017/04/NAP-G1-351.png 600 indent vpad <br>Valentine walked along the path from the start to the end. For about how many metres did Valentine walk west? </div>

    [{"vars":null}]

  215. <div class="sm_mode"> {{{question}}} </div>

    [{"vars":[{"varval":"Nathan places a notepad along a line of symmetry of a hidden figure.\n\nsm_img https://teacher.smartermaths.com.au/wp-content/uploads/2021/03/NAPX7-23-1-300x260.png 200 indent vpad\n\nHow many points does the whole figure have?"},{"varval":"Number of points = {{{correctAnswer}}}\n\nsm_img https://teacher.smartermaths.com.au/wp-content/uploads/2021/03/NAPX7-23-1-1-300x286.png 160 indent vpad\n"}]},{"vars":[{"varval":"A piece of paper is placed along a line of symmetry of a flower.\n\n<br>\n\nsm_img https://teacher.smartermaths.com.au/wp-content/uploads/2020/09/NAPX-I3-NC10_1.svg 265 indent3 vpad\n\n<br>\n\nHow many petals does the flower have?"},{"varval":"{{{correctAnswer}}}"}]},{"vars":[{"varval":"A calculator is placed along a line of symmetry of a hidden figure.\n\n<br>\n\nsm_img https://teacher.smartermaths.com.au/wp-content/uploads/2022/10/Geom_NAPX-p72864v01_v2.svg 205 indent3 vpad\n\n<br>\n\nHow many points does the whole figure have?"},{"varval":"Number of points = {{{correctAnswer}}}\n\nsm_img https://teacher.smartermaths.com.au/wp-content/uploads/2022/10/Geom_NAPX-p72864v01_v2a.svg 160 indent vpad"}]},{"vars":[{"varval":"A truck partially hides a sign along a line of symmetry.\n\n<br>\n\nsm_img https://teacher.smartermaths.com.au/wp-content/uploads/2022/10/Geom_NAPX-p72864v01_v3.svg 255 indent3 vpad\n\n<br>\n\nHow many points does the whole sign have?"},{"varval":"Number of points = {{{correctAnswer}}}\n\nsm_img https://teacher.smartermaths.com.au/wp-content/uploads/2022/10/Geom_NAPX-p72864v01_v3a.svg 160 indent vpad"}]},{"vars":[{"varval":"Georgiou has completed half of his new logo design.\n\n<br>\n\nsm_img https://teacher.smartermaths.com.au/wp-content/uploads/2022/10/Geom_NAPX-p72864v01_v4qa.svg 125 indent3 vpad\n\n<br>\n\nHow many points will the completed logo have?"},{"varval":"Number of points = {{{correctAnswer}}}\n\nsm_img https://teacher.smartermaths.com.au/wp-content/uploads/2022/10/Geom_NAPX-p72864v01_v4wsa.svg 250 indent vpad"}]},{"vars":[{"varval":"A calculator is placed along a line of symmetry of a hidden figure.\n\n<br>\n\nsm_img https://teacher.smartermaths.com.au/wp-content/uploads/2022/10/Geom_NAPX-p72864v01_v5q.svg 205 indent3 vpad\n\n<br>\n\nHow many points does the whole figure have?"},{"varval":"Number of points = {{{correctAnswer}}}\n\nsm_img https://teacher.smartermaths.com.au/wp-content/uploads/2022/10/Geom_NAPX-p72864v01_v5ws.svg 200 indent vpad"}]}]

  216. Geometry, NAPX-p116721v02 Geometry, NAPX-I2-08

    <div class="sm_mode"> {{{question}}} </div>

    [{"vars":[{"varval":"Sarah drew a straight line on a shape.\n\n\r\n\r\nThe line divided the shape into two rectangles.\n\n\r\n\r\nWhich of these could have been Sarah’s shape?"},{"varval":"sm_img https://teacher.smartermaths.com.au/wp-content/uploads/2021/04/RAPH8-Q16-sol.svg 400 indent vpad\n\nDrawing any horizontal straight line on this shape divides it into two rectangles."}]},{"vars":[{"varval":"Troy drew a straight line through a shape.\n\n\r\n\r\nThe line divided the shape into two equal quadrilaterals.\n\n\r\n\r\nWhich of these could not have been Troy’s shape?"},{"varval":"sm_img https://teacher.smartermaths.com.au/wp-content/uploads/2021/04/RAPH8-Q15-b.svg 100 indent vpad\n\nDrawing a diagonal line will produce two quadrilaterals (4 sides)."}]},{"vars":[{"varval":"Colin drew a straight line on a shape.\n\n\r\n\r\nThe line divided the shape into two triangles.\n\n\r\n\r\nWhich shape below could not have been Colin's original shape?"},{"varval":"All other shapes with 4 sides could have been divided into 2 triangles.\n\nsm_img https://teacher.smartermaths.com.au/wp-content/uploads/2018/10/NAPX-I2-08d-v1.svg 100 indent vpad\n"}]}]

  217. <div class="sm_mode"> Brandon is making a pattern by turning this shape a quarter turn clockwise in each box. <br> sm_img https://teacher.smartermaths.com.au/wp-content/uploads/2017/04/naplan-Y3-2009-19mc-rev-768x140.png 600 indent vpad <br>What will the shape in the last box look like? </div>

    [{"vars":null}]

  218. <div class="sm_mode"> Henry drew a map. He plans on leaving his house, walking to the bike shop, and then riding to the waterfall. <br> sm_img https://teacher.smartermaths.com.au/wp-content/uploads/2017/04/NAP-I1-225-768x342.png 500 indent vpad <br>In which cell is the waterfall located? </div>

    [{"vars":null}]

  219. Geometry, NAPX-p116732v01 Geometry, NAPX-I2-13

    <div class="sm_mode"> {{{question}}} </div>

    [{"vars":[{"varval":"A bridge is drawn below.\n\n<br>\n\nsm_img https://teacher.smartermaths.com.au/wp-content/uploads/2021/04/RAPH8-Q26.svg 400 indent2 vpad\n\n<br>The angle $\\large y\\degree$ is marked from the foot of the bridge to the diagonal support.\n\n\r\n\r\nThe value of angle $\\large y\\degree$ is\n"},{"varval":"sm_img https://teacher.smartermaths.com.au/wp-content/uploads/2021/04/RAPH8-Q26-sol.svg 400 indent vpad\n\n<br>The angle from the foot to the diagonal support is {{{correctAnswer}}}"}]},{"vars":[{"varval":"sm_img https://teacher.smartermaths.com.au/wp-content/uploads/2018/10/NAPX-I2-13-v2.svg 120 indent vpad\n\nThe angle marked $\\ \\large x$$\\degree$ &nbsp;is"},{"varval":"A right angle is 90$\\degree$\n\n\nsm_img https://teacher.smartermaths.com.au/wp-content/uploads/2018/10/NAPX-I2-13ans-v2.svg 180 indent vpad\n\n$\\therefore$ The angle marked $\\ \\large x$$\\degree$ &nbsp;is {{{correctAnswer}}}"}]},{"vars":[{"varval":"A door wedge is pictured in the diagram below.\n\n<br>\n\nsm_img https://teacher.smartermaths.com.au/wp-content/uploads/2018/10/NAPX-I2-13-v1.svg 160 indent vpad\n\n<br>The angle marked $\\ \\large x$$\\degree$ &nbsp;is"},{"varval":"A right angle (bottom right in diagram) is 90$\\degree$\n\n<br>\n\nsm_img https://teacher.smartermaths.com.au/wp-content/uploads/2018/10/NAPX-I2-13-v1.svg 160 indent vpad\n\n<br>$\\therefore$ The angle marked $\\ \\large x$$\\degree$ &nbsp;is {{{correctAnswer}}}"}]}]

  220. <div class="sm_mode"> {{{question}}} </div>

    [{"vars":[{"varval":"Michael’s house is shown in the map.\n\n<br>\n\nsm_img https://teacher.smartermaths.com.au/wp-content/uploads/2021/04/RAPH8-Q20.svg 550 indent vpad\n\n<br>Michael travels to work by driving 5 kilometres west, and then walking 3 kilometres south and then 1 kilometre west.\n\nIn which cell on the map is Michael’s work?"},{"varval":"sm_img https://teacher.smartermaths.com.au/wp-content/uploads/2021/04/RAPH8-Q20-SOL.svg 340 indent vpad\n\nEach grid width (or height) on the map represents 1 kilometre.\n\n\r\n$\\therefore$ Michael’s work is at {{{correctAnswer}}} on the map."}]},{"vars":[{"varval":"\nsm_img https://teacher.smartermaths.com.au/wp-content/uploads/2017/04/NAP-C1-08-300x231.png 240 indent vpad\n\nPeter moves the marker left 3 squares and then up 2 squares.\n\n\r\n\r\nWhat cell does Peter move the marker to?"},{"varval":"3 squares left: &nbsp;E3 $\\ \\rightarrow$ &nbsp;B3\n\n2 squares up: &nbsp;B3 $\\ \\rightarrow$ &nbsp;{{{correctAnswer}}}"}]}]

  221. Geometry, NAPX-K2-10

    <div class="sm_mode"> {{{question}}} </div>

    [{"vars":[{"varval":"Krusty takes off in a helicopter and heads for Sun Island.\n\n<br>\n\nsm_img https://teacher.smartermaths.com.au/wp-content/uploads/2018/12/NAPX-K2-10v3.svg 400 indent3 vpad\n\n<br>In which direction is Krusty flying?"},{"varval":"Krusty is flying north-west"}]},{"vars":[{"varval":"Corey takes off in a helicopter and heads for Hat Island.\n\n<br>\n\nsm_img https://teacher.smartermaths.com.au/wp-content/uploads/2018/12/NAPX-K2-10v1.svg 400 indent3 vpad\n\n<br>In which direction is Corey flying?\n"},{"varval":"Corey is flying south-west."}]}]

  222. <div class="sm_mode"> {{{question}}} </div>

    [{"vars":[{"varval":"\nRashid uses balls and sticks to make a model of a triangular pyramid.\n\nEach stick is the same length.\n\n<br>\n\nsm_img https://teacher.smartermaths.com.au/wp-content/uploads/2018/12/NAP-J2-22_1.svg 200 indent2 vpad\n\n<br>Rashid is given 4 more balls to change his model into a cube.\n\n\r\n\r\nHow many more sticks will Rashid need to make the cube?\n\n"},{"varval":"Rashid will need {{{correctAnswer0}}} more sticks (a total of 12).\n\n<br>\n\nsm_img https://teacher.smartermaths.com.au/wp-content/uploads/2019/01/NAPX-J2-22-v1.svg 170 indent2 vpad"}]},{"vars":[{"varval":"Riley uses balls and sticks to make a model of a triangular pyramid.\n\nEach stick is the same length.\n\n<br>\n\nsm_img https://teacher.smartermaths.com.au/wp-content/uploads/2018/12/NAP-J2-22_1.svg 200 indent2 vpad\n\n<br>Riley is given 6 more balls to change her model into a pentagonal prism.\n\n\r\n\r\nHow many more sticks will Riley need to make the pentagonal prism?\n"},{"varval":"A pentagonal prism has 5 edges at each end.\n\nRiley will need {{{correctAnswer0}}} more sticks (a total of 15).\n\n<br>\n\nsm_img https://teacher.smartermaths.com.au/wp-content/uploads/2022/10/Geom_NAPX-J2-22_v1a.svg 200 indent2 vpad"}]},{"vars":[{"varval":"Ringo uses balls and sticks to make a model of a triangular pyramid.\n\nEach stick is the same length.\n\n<br>\n\nsm_img https://teacher.smartermaths.com.au/wp-content/uploads/2018/12/NAP-J2-22_1.svg 150 indent2 vpad\n\n<br>Ringo is given 8 more balls to change his model into a hexagonal prism.\n\n\r\n\r\nHow many more sticks will Ringo need to make the hexagonal prism?\n"},{"varval":"A hexagonal prism has 6 edges at each end.\n\nRingo will need {{{correctAnswer0}}} more sticks (a total of 18).\n\n<br>\n\nsm_img https://teacher.smartermaths.com.au/wp-content/uploads/2022/10/Geom_NAPX-J2-22_v2.svg 200 indent2 vpad"}]},{"vars":[{"varval":"Rishi uses balls and sticks to make a model of a square pyramid.\n\nEach stick is the same length.\n\n<br>\n\nsm_img https://teacher.smartermaths.com.au/wp-content/uploads/2022/10/Geom_NAPX-J2-22_v3.svg 200 indent2 vpad\n\n<br>Rishi is given 7 more balls to change his model into a hexagonal prism.\n\n\r\n\r\nHow many more sticks will Rishi need to make the hexagonal prism?"},{"varval":"A hexagonal prism has 6 edges at each end.\n\nRishi will need {{{correctAnswer0}}} more sticks (a total of 18).\n\n<br>\n\nsm_img https://teacher.smartermaths.com.au/wp-content/uploads/2022/10/Geom_NAPX-J2-22_v3aws.svg 220 indent2 vpad"}]},{"vars":[{"varval":"Valentina uses balls and sticks to make a model of a square pyramid.\n\nEach stick is the same length.\n\n<br>\n\nsm_img https://teacher.smartermaths.com.au/wp-content/uploads/2022/10/Geom_NAPX-J2-22_v3.svg 200 indent2 vpad\n\n<br>Valentina is given 3 more balls to change her model into a cube.\n\n\r\n\r\nHow many more sticks will Valentina need to make the cube?"},{"varval":"Valentina will need {{{correctAnswer0}}} more sticks (a total of 12).\n\n<br>\n\nsm_img https://teacher.smartermaths.com.au/wp-content/uploads/2022/10/Geom_NAPX-J2-22_v4ws.svg 200 indent2 vpad"}]}]

  223. Geometry, NAPX-H2-27

    <div class="sm_mode"> {{{question}}} </div>

    [{"vars":[{"varval":"Which net folds into a 3D object with 8 edges?"},{"varval":"Consider the 3D object after folding in option 1:\n\nsm_img https://teacher.smartermaths.com.au/wp-content/uploads/2017/03/naplan-Y5-2016-4mciii.png 120 indent vpad\n\nThis 3D object has 8 edges."}]},{"vars":[{"varval":"Which net folds into a 3D object with 6 edges?"},{"varval":"Consider the net in option 4:\n\nsm_img https://teacher.smartermaths.com.au/wp-content/uploads/2018/12/NAPX-H2-27-v1-Answer.svg 100 indent vpad\n\nThe 3D object after folding has 6 edges."}]},{"vars":[{"varval":"Which net folds into a 3D object with 9 edges?"},{"varval":"Consider the 3D object after folding in option 3:\n\nsm_img https://teacher.smartermaths.com.au/wp-content/uploads/2018/12/NAPX-H2-27-v2b.svg 170 indent vpad\n\nThis 3D object has 9 edges."}]},{"vars":[{"varval":"Which net folds into a 3D object with 12 edges?"},{"varval":"Consider the 3D object after folding in option 4:\n\nsm_img https://teacher.smartermaths.com.au/wp-content/uploads/2018/12/NAPX-H2-27-v1b.svg 170 indent vpad\n\nThis 3D object has 12 edges."}]},{"vars":[{"varval":"Which net folds into a 3D object with 15 edges?"},{"varval":"Consider the 3D object after folding in option 2:\n\nsm_img https://teacher.smartermaths.com.au/wp-content/uploads/2022/10/Geom_NAPX-p120861v01_v4a.svg 190 indent vpad\n\n\nThis 3D object has 15 edges."}]},{"vars":[{"varval":"Which net folds into a 3D object with 10 edges?"},{"varval":"Consider the 3D object after folding in option 4:\n\nsm_img https://teacher.smartermaths.com.au/wp-content/uploads/2022/10/Geom_NAPX-p120861v01_v4b.svg 170 indent vpad\r\n\r\n\r\n\nThis 3D object has 10 edges."}]}]

  224. <div class="sm_mode"> Lucy went camping. The tent she pitched is shown below. <br> sm_img https://teacher.smartermaths.com.au/wp-content/uploads/2017/04/NAP-C1-181-300x155.png 300 indent3 vpad <br>Lucy's tent is most like a </div>

    [{"vars":null}]

  225. <div class="sm_mode"> Sarah makes a sculpture using identical cubes. There were six cubes in the bottom three rows. <br> sm_img https://teacher.smartermaths.com.au/wp-content/uploads/2018/10/NAPX-J2-09-v1.svg 200 indent3 vpad <br>How many cubes did Sarah put in the top two rows altogether? </div>

    [{"vars":null}]

  226. <div class="sm_mode"> Rory made a tower using cylinders. <br> sm_img https://teacher.smartermaths.com.au/wp-content/uploads/2017/04/naplan-Y3-2011-20mc-300x266.png 180 indent3 vpad <br>Which of these shows Rory's tower from above? </div>

    [{"vars":[]}]

  227. Geometry, NAPX-J2-06

    <div class="sm_mode"> {{{question}}} </div>

    [{"vars":[{"varval":"Fiona is standing **inside** a tree house. She looks through a window with four shapes in it.\n\n<br>\n\nsm_img https://teacher.smartermaths.com.au/wp-content/uploads/2018/10/NAPX-J2-06-v2.svg 140 indent3 vpad\n\n<br>The shapes can be seen from both sides of the window.\n\n\r\n\r\nHow would the window look when viewed from **outside** the tree house?"},{"varval":"sm_img https://teacher.smartermaths.com.au/wp-content/uploads/2018/10/NAPX-J2-06-v2b.svg 140 indent vpad "}]},{"vars":[{"varval":"Vince is standing **inside** a pre-school classroom. He looks through a window with four shapes in it.\n\n<br>\n\nsm_img https://teacher.smartermaths.com.au/wp-content/uploads/2018/10/NAPX-J2-06-v1.svg 140 indent3 vpad\n\n<br>The shapes can be seen from both sides of the window.\n\n\r\n\r\nHow would the window look when viewed from **outside** the classroom?"},{"varval":"sm_img https://teacher.smartermaths.com.au/wp-content/uploads/2018/10/NAPX-J2-06-v1d.svg 140 indent vpad"}]}]

  228. <div class="sm_mode"> {{{question}}} </div>

    [{"vars":[{"varval":"Gary folded this piece of paper along the dashed lines to make a solid figure.\n\n<br>\n\nsm_img https://teacher.smartermaths.com.au/wp-content/uploads/2021/04/RAPH8-Q8.svg 320 indent2 vpad\n\n<br>Which of these models did Gary make?\n"},{"varval":"By folding the paper along the dashed lines we could obtain a figure of:\n\n<br>\n\nsm_img https://teacher.smartermaths.com.au/wp-content/uploads/2021/04/RAPH8-Q8-sol.svg 120 indent3 vpad"}]},{"vars":[{"varval":"Lance folded this piece of cardboard along the dotted lines to make a solid figure.\n\n<br>\n\nsm_img https://teacher.smartermaths.com.au/wp-content/uploads/2021/04/RAPH8-Q7.svg 260 indent2 vpad\n\n<br>Which of these models did Lance made?"},{"varval":"By folding the cardboard along the dotted lines we could obtain a figure of:\n\n<br>\n\nsm_img https://teacher.smartermaths.com.au/wp-content/uploads/2021/04/RAPH8-Q7-solution.svg 200 indent3 vpad\n"}]},{"vars":[{"varval":"Albert folded this piece of paper along the dotted lines to make a model.\n\n<br>\n\nsm_img https://teacher.smartermaths.com.au/wp-content/uploads/2018/10/NAPX-I2-04-v2.svg 200 indent2 vpad\n\n<br>Which of these models did Albert make?​"},{"varval":"The net folds into a triangular prism\n\n{{{correctAnswer}}}\n"}]},{"vars":[{"varval":"The net below is folded to form a prism.\n\nsm_img https://teacher.smartermaths.com.au/wp-content/uploads/2018/10/NAPX-H2-06-v1.svg 260 indent vpad\n\nWhich prism could it make?"},{"varval":"sm_img https://teacher.smartermaths.com.au/wp-content/uploads/2017/01/NAP-H3-NC01d.png 160 indent vpad"}]},{"vars":[{"varval":"The net below is folded to form a prism.\n\nsm_img https://teacher.smartermaths.com.au/wp-content/uploads/2018/07/NAPX-H3-NC01.svg 250 indent vpad\n\nWhich prism could it make?"},{"varval":"sm_img https://teacher.smartermaths.com.au/wp-content/uploads/2018/07/NAPX-H3-NC01d.svg 160 indent2 vpad"}]}]

  229. <div class="sm_mode"> Nancy is making a rectangular prism using plastic balls and sticks. <br> sm_img https://teacher.smartermaths.com.au/wp-content/uploads//2021/03/NAPX5-TLA-2-v2.svg 260 indent vpad <br>How many more sticks does Nancy need to finish the rectangular prism? </div>

    [{"vars":null}]

  230. Changed var0 as solution was incorrect. I still question whether the LHS of the reflection should also be included. Also whether the reflection line should be included in the worked solution image. This question is visually much more difficult than var1. Both need difficulty.

    <div class="sm_mode"> {{{question}}} </div>

    [{"vars":[{"varval":"A letter V was written on a piece of paper.\r\n\nThe paper was folded along the dotted line as shown in the figure below.\n\n\r\nsm_img https://teacher.smartermaths.com.au/wp-content/uploads/2022/11/RAPH12-Q61-62_v0_b.svg 200 indent vpad\n\nWhich of the following represents the reflected image if it was folded along the line?"},{"varval":"By folding the piece of paper along the dotted line we obtain a reflection of\n\nsm_img https://teacher.smartermaths.com.au/wp-content/uploads/2022/11/Raph-12-Q61-62_ws_v0.svg 480 indent vpad\n"}]},{"vars":[{"varval":"A curve was drawn on a piece of paper using wet paint.\r\n\nThe paper is then folded along the dotted line.\r\n"},{"varval":"By folding the piece of paper along the dotted line we obtain a reflection of\n\nsm_img https://teacher.smartermaths.com.au/wp-content/uploads//2021/09/62sol.svg 450 indent vpad\n\n"}]}]

  231. Measurement, NAPX-p116903v01 Measurement, NAPX-I2-18

    <div class="sm_mode"> {{{question}}} </div>

    [{"vars":[{"varval":"Banjo compared the weight of four different solid figures by using a balancing scale.\n\n<br>\n\nsm_img https://teacher.smartermaths.com.au/wp-content/uploads/2021/04/q35-1-300x239.png 180 indent vpad\n\nsm_img https://teacher.smartermaths.com.au/wp-content/uploads/2021/04/q35-2-300x252.png 180 indent vpad\n\nsm_img https://teacher.smartermaths.com.au/wp-content/uploads/2021/04/q35-3-300x261.png 180 indent vpad\n\n<br>Which of the four objects is the heaviest?"},{"varval":"By inspection:\n\n\r\nSphere → heavier than the cube.\n\n\r\nSphere → same weight as the cylinder.\n\n\r\nPentagonal prism → heavier than the cylinder.\n\r\n \r\n\r\n$\\therefore$ Pentagonal prism is the heaviest."}]},{"vars":[{"varval":"Bjorn balances four objects on a set of scales, as pictured below.\n\n<br>\n\nsm_img https://teacher.smartermaths.com.au/wp-content/uploads/2018/10/NAPX-I2-18-v2.svg 600 indent vpad\n\n<br>Which object is the heaviest?\n"},{"varval":"sm_img https://teacher.smartermaths.com.au/wp-content/uploads/2018/10/NAPX-I2-18-ball.svg 60 indent vpad\n\nBowling ball → heavier than barrel and money bag \n\n\r\nMoney bag → heavier than gold\n\n\r\n$\\therefore$ Bowling ball must be the heaviest."}]},{"vars":[{"varval":"Angelina balances four objects on a set of scales, as pictured below.\n\n<br>\n\nsm_img https://teacher.smartermaths.com.au/wp-content/uploads/2018/10/NAPX-I2-18-v1.svg 600 indent vpad\n\n<br>Which object is the heaviest?"},{"varval":"sm_img https://teacher.smartermaths.com.au/wp-content/uploads/2018/10/NAPX-I2-18-gold_rev.svg 75 indent vpad\n\nMoney bag → heavier than bowling ball and barrel\n\n\r\nGold → heavier than the money bag\n\n\r\n$\\therefore$ Gold must be the heaviest."}]}]

  232. <div class="sm_mode"> {{{question}}} </div>

    [{"vars":[{"varval":"The scales are balanced.\n\n<br>\n\nsm_img https://teacher.smartermaths.com.au/wp-content/uploads/2017/04/naplan-Y3-2015-27mc.png 420 indent vpad\n\n<br>Which one of these weighs the least?\n"},{"varval":"4 cubes weigh the same as 1 triangle.\n\n\n3 balls weigh the same as 1 triangle.\n\n\n⇒ A cube must be the lightest.\n\n{{{correctAnswer}}}\n"}]},{"vars":[{"varval":"These scales are balanced.\n\n<br>\n\nsm_img https://teacher.smartermaths.com.au/wp-content/uploads/2023/07/balance_1-min.svg 220 indent vpad\n\n<br>Which one of these weighs the least?"},{"varval":"2 cubes weigh the same as 1 pyramid.\n\n4 balls weigh the same as 1 pyramid.\n\n⇒ A ball must be the lightest.\n\n{{{correctAnswer}}}\n"}]},{"vars":[{"varval":"These scales are balanced.\n\n<br>\n\nsm_img https://teacher.smartermaths.com.au/wp-content/uploads/2023/07/balance_2-min.svg 220 indent vpad\n\n<br>Which one of these weighs the least?"},{"varval":"2 pyramids weigh the same as 1 cube.\n\n3 balls weigh the same as 1 cube.\n\n⇒ A ball must be the lightest.\n\n{{{correctAnswer}}}\n"}]},{"vars":[{"varval":"These scales are balanced.\n\nsm_img https://teacher.smartermaths.com.au/wp-content/uploads/2023/07/banana-fruit-min.svg 390 indent vpad\n\nWhich piece of fruit weighs the least?\n\n\n"},{"varval":"3 bananas weigh the same as 2 pears.\n\n4 apples weigh the same as 2 pears.\n\n⇒ An apple must weigh the least.\n\n{{{correctAnswer}}}"}]},{"vars":[{"varval":"These scales are balanced.\n\nsm_img https://teacher.smartermaths.com.au/wp-content/uploads/2023/07/vegetable-balance-min.svg 390 indent vpad\n\nWhich vegetable weighs the least?\n\n\n"},{"varval":"2 egg plant weigh the same as 1 pumpkin.\n\n4 carrots weigh the same as 1 pumpkin.\n\n⇒ A carrot must weigh the least.\n\n{{{correctAnswer}}}"}]},{"vars":[{"varval":"These scales are balanced.\n\nsm_img https://teacher.smartermaths.com.au/wp-content/uploads/2023/07/balance-kitchen-min.svg 350 indent vpad\n\nWhich item weighs the least?"},{"varval":"4 glasses weigh the same as 1 vase.\n\n5 mugs weigh the same as 1 vase.\n\n⇒ A mug must weigh the least.\n\n{{{correctAnswer}}}"}]}]

  233. <div class="sm_mode"> {{{question}}} </div>

    [{"vars":[{"varval":"Valerie looked at the calendar on 23 March.\n\nShe plans to go to the theatre on 15 April and circled this on the calendar below.\n\nsm_img https://teacher.smartermaths.com.au/wp-content/uploads/2019/01/NAPX-K2-40.svg 220 indent3 vpad\n\nWhat day of the week is 23 March?"},{"varval":"Working backwards:\n\n\r\n>>1 April → Tuesday (from calendar)\n\n\r\n>>31 March → Monday (31 days in March)\n\n\r\n>>24 March → Monday \r\n\n<br> \r\n\r\n$\\therefore$ 23 March is a {{{correctAnswer}}}"}]},{"vars":[{"varval":"Curtly looked at the calendar on 5 March.\n\nHe plans to go to the cricket on 15 April and circled this on the calendar below.\n\nsm_img https://teacher.smartermaths.com.au/wp-content/uploads/2019/01/NAPX-K2-40.svg 220 indent3 vpad\n\nWhat day of the week is 5 March?"},{"varval":"Working backwards:\n\n\r\n>>1 April → Tuesday (from calendar)\n\n\r\n>>31 March → Monday (31 days in March)\n\n>>24 March → Monday \n\n>>17 March → Monday \n\n>>10 March → Monday \n\r\n>>3 March → Monday \r\n\n<br> \r\n\r\n$\\therefore$ 5 March is a {{{correctAnswer}}}"}]},{"vars":[{"varval":"Jevin looked at the calendar on 14 May.\n\nHe plans to go to the AFL the following month on 9 June, shown in the calendar below.\n\n<br>\n\nsm_img https://teacher.smartermaths.com.au/wp-content/uploads/2019/01/NAPX-G2-17.svg 220 indent3 vpad\n\n<br>What day of the week is 14 May?"},{"varval":"Working backwards:\n\n\r\n>>1 June → Friday (from calendar)\n\n\r\n>>31 May → Thursday (31 days in May)\n\n>>24 May → Thursday \n\n>>17 May → Thursday \n\n<br> \r\n\r\n$\\therefore$ 14 May is a {{{correctAnswer}}}"}]}]

  234. <div class="sm_mode"> {{{question}}} </div>

    [{"vars":[{"varval":"Grant takes 15 minutes to walk to the beach for board training.\n\n\r\n\r\nLast week he arrived at training at 4:05 pm, which was 25 minutes late.\n\n\r\n\r\nAt what time would Grant leave home to arrive at board training on time?"},{"varval":"sm_nogap Start of training\n\n<div class=\"aligned\">\n\n>>||\n|-|\n|= 4:05 less 25 minutes|\n|= 3:40 pm|\n\n</div>\n\n\r<br>\n\r\n\r\nsm_nogap $\\therefore$ Time to leave home\n\n<div class=\"aligned\">\n\n>>||\n|-|\n|= 3:40 less 15 minutes|\n|= {{{correctAnswer}}}|\n\r\n\n</div>"}]},{"vars":[{"varval":"Yves took 25 minutes to walk to her job interview.\n\nWhen Yves arrived at 11:15 am, she was told she was 35 minutes late.\n\nAt what time should Yves have left home to arrive at her job interview on time?"},{"varval":"sm_nogap Time of job interview\n\n<div class=\"aligned\">\n\n>>||\n|-|\n|= 11:15 less 35 minutes|\n|= 10:40 pm|\n\n</div>\n\n\r<br>\n\r\n\r\nsm_nogap $\\therefore$ Time to leave home\n\n<div class=\"aligned\">\n\n>>||\n|-|\n|= 10:40 less 25 minutes|\n|= {{{correctAnswer}}}|\n\r\n\n</div>"}]}]

  235. Measurement, NAPX-p116865v03 Measurement, NAPX-p116865v02 Measurement, NAPX-p116865v01 Measurement, NAPX-p124595v01 Measurement, NAPX-G2-17

    <div class="sm_mode"> {{{question}}} </div>

    [{"vars":[{"varval":"\r\nScott expects his wine to be delivered on the 28th March.\n\nHe is told by the distributor that its delivery will be delayed by 5 days.\n\n<br>\n\nsm_img https://teacher.smartermaths.com.au/wp-content/uploads/2021/03/Measurement-NAPX5-TLA-6-v2-300x300.png 260 indent3 vpad\n\n<br>What day of the week will the wine now be delivered?"},{"varval":"28th March is a Thursday.\n\n\r\n$\\therefore$ 5 days later is a Tuesday."}]},{"vars":[{"varval":"Marty expects his gold mining drench to be delivered on the 28th of June.\n\nDelays at customs mean it will be delivered 6 days later.\n\n<br>\n\nsm_img https://teacher.smartermaths.com.au/wp-content/uploads/2017/03/naplan-Y5-2015-9mc-rev-300x300.png 260 indent3 vpad\n\n<br>Which day of the week should the drench now be delivered?"},{"varval":"28th June is a Tuesday.\n\n\r\n$\\therefore$ 6 days later is a Monday."}]},{"vars":[{"varval":"Abbey's birthday is on the 3th of August.\n\nHer sister Lily had her birthday 6 days earlier.\n\n<br>\n\nsm_img https://teacher.smartermaths.com.au/wp-content/uploads/2018/10/NAPX-H2-09-v2.png 330 indent3 vpad\n\n<br>Which day of the week was Lily's birthday in 2017?"},{"varval":"3rd August was on a Thursday.\r\n\n$\\therefore$ Lily's birthday was on a {{{correctAnswer}}}."}]},{"vars":[{"varval":"Basil's birthday is on the 29th of August.\n\nHis father's birthday is 6 days later.\n\n<br>\n\nsm_img https://teacher.smartermaths.com.au/wp-content/uploads/2018/10/NAPX-H2-09-v2.png 330 indent3 vpad\n\n<br>Which day of the week was his father's birthday in 2017?"},{"varval":"29th August was a Tuesday.\n\n\r\n$\\therefore$ Father's birthday was on {{{correctAnswer}}}."}]},{"vars":[{"varval":"sm_img https://teacher.smartermaths.com.au/wp-content/uploads/2019/01/NAPX-G2-17.svg 260 indent3 vpad\n\nDavid has a dentist appointment on 4 June.\n\n5 days before the appointment, David's tooth fell out.\n\nOn which day of the week did his tooth fall out?"},{"varval":"Working backwards:\n\n\r\n1 day before 4 June → Sunday\n\n\r\n2 days before 4 June → Monday\n\n$\r\\therefore$ 5 days before 4 June → {{{correctAnswer}}}"}]},{"vars":[{"varval":"sm_img https://teacher.smartermaths.com.au/wp-content/uploads/2019/01/NAPX-G2-17.svg 240 indent3 vpad\n\nThe French Open women's tennis final was held on 2 June.\n\n\r6 days before the final, Lleyton arrived in Paris.\n\nOn what day of the week did Lleyton arrive in Paris?"},{"varval":"Working backwards:\n\n\r\n1 day before 2 June → Friday\n\n\r\n2 days before 2 June → Thursday, etc...\n\n$\r\\therefore$ 6 days before 2 June → {{{correctAnswer}}}"}]},{"vars":[{"varval":"Stella's birthday is on the 30th of March.\n\r\nBlanche's birthday is four days after Stella's.\n\n<br>\n\nsm_img https://teacher.smartermaths.com.au/wp-content/uploads/2017/04/naplan-Y3-2015-17mc.png 290 indent2 vpad\n\n\n<br>Which day of the week does Blanche's birthday fall on in 2014?"},{"varval":"30th March ⇒ Thursday\n\n\r\nOne day after ⇒ Friday\n\n\r\nTwo days after ⇒ Saturday\n\n$\\therefore$ Four days after ⇒ {{{correctAnswer}}}"}]}]

  236. <div class="sm_mode"> Lin's alarm clock is upside down. When he looked at it, he saw this. <br> sm_img https://teacher.smartermaths.com.au/wp-content/uploads/2017/04/NAP-E1-141-292x300.png 200 indent2 vpad <br>At what time did he look at his alarm clock? </div>

    [{"vars":null}]

  237. <div class="sm_mode"> Stella's birthday is on the 30th of March. Blanche's birthday is four days after Stella's. <br> sm_img https://teacher.smartermaths.com.au/wp-content/uploads/2017/04/naplan-Y3-2015-17mc-300x300.png 240 indent2 vpad <br>Which day of the week does Blanche's birthday fall on in 2014? </div>

    [{"vars":null}]

  238. SJ v3 - v7 added

    <div class="sm_mode"> {{{question}}} </div>

    [{"vars":[{"varval":"How many months are there in 2 years?"},{"varval":"sm_nogap Months in 2 years\n\n<div class=\"aligned\">\n\n>>||\n|-|\n|= 2 × 12|\n|= {{{correctAnswer}}}|\n\n</div>"}]},{"vars":[{"varval":"How many months are there in 4 years?"},{"varval":"sm_nogap Months in 4 years\n\n<div class=\"aligned\">\n\n>>||\n|-|\n|= 4 × 12|\n|= {{{correctAnswer}}}|\n\n</div>"}]},{"vars":[{"varval":"How many months are there in one and a half years?"},{"varval":"sm_nogap Months in one and a half years\n\n<div class=\"aligned\">\n\n>>||\n|-|\n|= 12 + $\\dfrac{1}{2}\\ \\times$ 12|\n|= 12 + 6|\n|= {{{correctAnswer}}}|\n\n</div>"}]},{"vars":[{"varval":"How many months are there in 3 years?"},{"varval":"sm_nogap Months in 3 years\n\n<div class=\"aligned\">\n\n>>||\n|-|\n|= 3 × 12|\n|= {{{correctAnswer}}}|\n\n</div>"}]},{"vars":[{"varval":"How many months in 5 years?"},{"varval":"sm_nogap Months in 5 years\n\n<div class=\"aligned\">\n\n>>||\n|-|\n|= 5 × 12|\n|= {{{correctAnswer}}}|\n\n</div>"}]},{"vars":[{"varval":"How many months in two and a half years?"},{"varval":"sm_nogap Months in two and a half years\n\n<div class=\"aligned\">\n\n>>||\n|-|\n|= 2 $\\times$ 12 + $\\dfrac{1}{2}\\ \\times$ 12|\n|= 24 + 6|\n|= {{{correctAnswer}}}|\n\n</div>"}]},{"vars":[{"varval":"How many months in half a year?"},{"varval":"sm_nogap Months in half a year\n\n<div class=\"aligned\">\n\n>>||\n|-|\n|= $\\dfrac{1}{2}\\ \\times$ 12|\n|= {{{correctAnswer}}}|\n\n</div>"}]},{"vars":[{"varval":"How many months in a quarter of a year?"},{"varval":"sm_nogap Months in a quarter of a year\n\n<div class=\"aligned\">\n\n>>||\n|-|\n|= $\\dfrac{1}{4}\\ \\times$ 12|\n|= {{{correctAnswer}}}|\n\n</div>"}]}]

  239. Corrected English. Difficulty needed.

    <div class="sm_mode"> {{{question}}} </div>

    [{"vars":[{"varval":"A pattern was made by Nathalie as shown below.\n\nsm_img https://teacher.smartermaths.com.au/wp-content/uploads//2021/09/59.svg 550 indent vpad\n\nWhich of the following, when repeated, will result in the pattern shown above?"},{"varval":"{{{correctAnswer}}}\n\nRepeating this pattern three times will result in the pattern above."}]},{"vars":[{"varval":"A unique pattern is shown below.\n\nsm_img https://teacher.smartermaths.com.au/wp-content/uploads//2021/09/60.svg 650 indent vpad\n\nWhich of the following, when repeated, will result in the pattern shown above?"},{"varval":"{{{correctAnswer}}}\n\nRepeating this pattern two times will result in the pattern above."}]}]

  240. <div class="sm_mode"> {{{question}}} </div>

    [{"vars":[{"varval":"Timothy attended a meeting which started a\rt 8:30 a.m.\n\nThe meeting ended at 11:16 a.m.\n\n\rFor how long was the meeting held?\n"},{"varval":"From 8:30 a.m. to 9:00 a.m., 30 minutes have passed.\r\n\nFrom 9:00 a.m. to 11:00 a.m., 2 hours have passed.\r\n\nFrom 11:00 a.m. to 11:16 a.m., 16 minutes have passed.\r\n\nTotal time = 2 hours 46 minutes\r\n"}]},{"vars":[{"varval":"Morty attended a seminar about lifestyle which started at 2:05 p.m.\n\nThe seminar ended at 4:23 p.m.\n\nFor how long was the seminar held?\r\n"},{"varval":"Starting from 2:05 p.m. up to 2:23 p.m., 18 minutes have passed.\r\n\nFrom 2:23 p.m. to 4:23 p.m., 2 hours have passed.\r\n\nTotal time = 2 hours 18 minutes\r\n"}]}]

  241. <div class="sm_mode"> {{{question}}} </div>

    [{"vars":[{"varval":"A moon and star image is shown below\n\nsm_img https://teacher.smartermaths.com.au/wp-content/uploads//2021/08/55.svg 200 indent vpad\n\nThe image is rotated a quarter of a turn in a clockwise direction\r\n\nWhich of the following is the resulting position of this image?\r\n"},{"varval":"After rotating the image by 90° (a quarter turn) clockwise, the resulting image is:\n\nsm_img https://teacher.smartermaths.com.au/wp-content/uploads//2021/08/55sol.svg 400 indent vpad"}]},{"vars":[{"varval":"A cowboy boot is shown in the image below.\n\nsm_img https://teacher.smartermaths.com.au/wp-content/uploads//2021/08/56.svg 200 indent vpad\n\nIt is rotated two quarter turns in an anti-clockwise direction. \n\nWhich of the following is the resulting position of this image?\r\n"},{"varval":"After rotating the image by 2 quarter turns or 180°, the resulting image is:\t \n\nsm_img https://teacher.smartermaths.com.au/wp-content/uploads//2021/08/56sol.svg 350 indent vpad"}]}]

  242. Needs difficulty

    <div class="sm_mode"> {{{question}}} </div>

    [{"vars":[{"varval":"<div class=\"sm_img_inline\" style=\"height: 20px\">\n\n\n<img src=\"https://teacher.smartermaths.com.au/wp-content/uploads//2021/08/57.svg\" style=\"height: 50px;\"> represents $\\dfrac{4}{7}$\r\n\r\nWhich of the following represents a whole?\r\n\n\n</div>"},{"varval":"Each pointed figure represents $\\dfrac{1}{7}$\r\n\nTherefore, 7 figures represent a whole\n\n\r\n{{{correctAnswer}}}\n\n"}]},{"vars":[{"varval":"<div class=\"sm_img_inline\" style=\"height: 20px\">\n\n<img src=\"https://teacher.smartermaths.com.au/wp-content/uploads//2021/08/58.svg\" style=\"height: 50px;\"> represents $\\dfrac{1}{3}$\r\n\r\nWhich of the following represents a whole?\r\n\n\n</div>"},{"varval":"<div class=\"sm_img_inline\" style=\"height: 20px\">\n\n<img src=\"https://teacher.smartermaths.com.au/wp-content/uploads//2021/08/58.svg\" style=\"height: 50px;\"> represents $\\dfrac{1}{3}$ $=$ $\\dfrac{2}{6}$\n\nTherefore a single spiral represents $\\dfrac{1}{6}$ of a whole\r\n\nTherefore, 6 spirals represent a whole\n\n\r\n{{{correctAnswer}}}\n\n</div>"}]}]

  243. NAP-F2-18

    <div class="sm_mode"> {{{question}}} </div>

    [{"vars":[{"varval":"Flint has his birthday on the 21st of May.\n\n<br>\n\nsm_img https://teacher.smartermaths.com.au/wp-content/uploads//2021/08/51.svg 250 indent3 vpad\n\n<br>His father’s birthday is 12 days after his.\r\n\nWhat day is his father’s birthday?\r\n"},{"varval":"Counting 12 days from May 21:\n\nDay 10 $\\rightarrow$ Monday, 31-May\n\nDay 11 $\\rightarrow$ Tuesday, 1-Jun\n\nDay 12 $\\rightarrow$ Wednesday, 2-Jun\n\n$\\therefore$ Father’s birthday is on {{{correctAnswer}}}."}]},{"vars":[{"varval":"Trish’s graduation day is on April 6.\n\n<br>\n\nsm_img https://teacher.smartermaths.com.au/wp-content/uploads//2021/08/52.svg 250 indent3 vpad\n\n<br>She plans to go to a concert 8 days before her graduation.\r\n\nWhat day is the concert on?\r\n"},{"varval":"Counting 8 days backwards from Apr 6:\n\nDay 5 $\\rightarrow$ Thursday, 1-Apr\n\nDay 6 $\\rightarrow$ Wednesday, 31-Mar\n\nDay 7 $\\rightarrow$ Tuesday, 30-Mar\n\nDay 8 $\\rightarrow$ Monday, 29-Mar\n\n$\\therefore$ The concert is on a {{{correctAnswer}}}.\n"}]}]

  244. <div class="sm_mode"> {{{question}}} </div>

    [{"vars":[{"varval":"The graph below represents data gathered from 9 professional workers.\n\n<br>\n\nsm_img https://teacher.smartermaths.com.au/wp-content/uploads/2021/05/RAPH10_72.svg 340 indent vpad\n\n<br>What kind of data could be represented in the graph shown above?\n"},{"varval":"{{{correctAnswer}}}\n\n* Each data category needs to have many numerical possibilities\n\n* The chosen pair need a general relationship where one moves higher at the same time as the other. \n\nAge and salary is the most likely.\n"}]},{"vars":[{"varval":"The graph below represents data gathered from a winter clothing store.\n\n<br>\n\nsm_img https://teacher.smartermaths.com.au/wp-content/uploads/2022/08/Stats_NAPX-p108432v02_v1.svg 340 indent3 vpad\n\n<br>What kind of data could be represented in the graph shown above?\n"},{"varval":"{{{correctAnswer}}}\n\n* Each data category needs to have many numerical possibilities\n\n* The chosen pair need a general relationship where one moves lower at the same time as the other gets higher. \n\nTemperature and sales is the most likely.\n"}]},{"vars":[{"varval":"The graph below represents data gathered about a beach.\n\n<br>\n\nsm_img https://teacher.smartermaths.com.au/wp-content/uploads/2022/08/Stats_NAPX-p108432v02_v2.svg 340 indent3 vpad\n\n<br>What kind of data could be represented in the graph shown above?\n"},{"varval":"{{{correctAnswer}}}\n\n* Each data category needs to have many numerical possibilities\n\n* The chosen pair need a general relationship where one moves higher at the same time as the other. \n\nNumber of people at the beach and temperature are most likely.\n\n"}]},{"vars":[{"varval":"The graph below represents data gathered from a class of 20 students.\n\n<br>\n\nsm_img https://teacher.smartermaths.com.au/wp-content/uploads/2022/08/Stats_NAPX-p108432v02_v3.svg 340 indent3 vpad\n\n<br>What kind of data could be represented in the graph shown above?\n"},{"varval":"{{{correctAnswer}}}\n\n* Each data category needs to have many numerical possibilities\n\n* The chosen pair need a general relationship where one moves higher at the same time as the other. \n\nScore on a Quiz and Hours of Study is the most likely.\n"}]},{"vars":[{"varval":"The graph below represents data gathered from a person at the gym.\n\n<br>\n\nsm_img https://teacher.smartermaths.com.au/wp-content/uploads/2022/08/Stats_NAPX-p108432v02_v4.svg 340 indent3 vpad\n\n<br>What kind of data could be represented in the graph shown above?\n"},{"varval":"{{{correctAnswer}}}\n\n* Each data category needs to have many numerical possibilities\n\n* The chosen pair need a general relationship where one measurement moves lower as the other moves higher. \n\nInitially the heartrate is high and as times goes by the heartrate returns to normal. \n\nSo Minutes since the gym class completed and Heart rate are most likely.\n"}]},{"vars":[{"varval":"The graph below represents data gathered from 14 university students.\n\n<br>\n\nsm_img https://teacher.smartermaths.com.au/wp-content/uploads/2022/08/Stats_NAPX-p108432v02_v5.svg 340 indent3 vpad\n\n<br>What kind of data could be represented in the graph shown above?\n"},{"varval":"{{{correctAnswer}}}\n\n* Each data category needs to have many numerical possibilities\n\n* The chosen pair need a general relationship where one moves higher at the same time as the other. \n\nWeight of Backpack and Number of Books is the most likely.\n"}]}]

  245. NAP-F2-19

    <div class="sm_mode"> {{{question}}} </div>

    [{"vars":[{"varval":"A fruit stall sold half of its pineapples in the first hour at a market.\r\n\nIt then sold 9 more and had 3 pinepples left to sell.\r\n\nHow many pineapples did the stall have for sale at the start of the day?\r\n"},{"varval":"Working backwards:\n\nAdd the 9 pineapples sold to the 3 that were left\n\n$\\rightarrow$ 12 pineapples after 1 hour\n\n$\\rightarrow$ 12 pineapples were half of the starting number\n\n$\\therefore$ Pineapples for sale = 12 $\\times$ 2 = {{{correctAnswer}}}\r\n"}]},{"vars":[{"varval":"Kelly shared half of her sherbet cones with her friends at school.\r\n\nShe then gave 13 more sherbert cones to her AFL team mates.\r\n\nNow she had 7 sherbert cones left.\r\n\nHow many sherbert cones did Kelly have at the start?\r\n"},{"varval":"Working backwards:\n\nAdd the 13 to the 7 sherbert cones left.\n\n$\\rightarrow$ 20 sherbert cones\n\n$\\rightarrow$ 20 sherbert cones is half of the starting number\n\n$\\therefore$ Sherbert cones at start = 20 $\\times$ 2 = {{{correctAnswer}}}\r"}]}]

  246. NAP-F2-20

    <div class="sm_mode"> {{{question}}} </div>

    [{"vars":[{"varval":"Lester has a goal of cycling a total distance of 6200 metres.\r\n\nHis GPS tracker recorded that he had travelled 4286 metres.\r\n\nWhat distance does he still need to cycle to reach his goal?\r\n"},{"varval":"One strategy:\n\nSubtract thousands first, then hundreds, then the balance (as per the solution).\n\n<div class=\"aligned\">\r\n\r\n| | |\r\n| ------------- | ---------- |\r\n| Distance left | \\= 6200 – 4286 |\r\n| | \\= 2200 – 286 |\r\n| | \\= 2000 – 86 |\r\n| | \\= {{{correctAnswer}}} metres |\r\n\r\n</div>\r\n"}]},{"vars":[{"varval":"Derek has a goal of jogging a total distance of 4150 metres.\r\n\nHis GPS tracker shows he has already jogged a distance of 2317 metres.\r\n\nHow much further does he have to jog to reach his goal?\r\n"},{"varval":"One strategy:\n\nSubtract thousands first, then hundreds, then the balance (as per the solution).\n\n<div class=\"aligned\">\r\n\r\n| | |\r\n| ------------- | ---------- |\r\n| Distance left | \\= 4150 – 2317 |\r\n| | \\= 2150 – 317 |\r\n| | \\= 1850 – 17 |\r\n| | \\= {{{correctAnswer}}} metres |\r\n\r\n</div>\r\n"}]},{"vars":[{"varval":"Carol's research shows the number of privately owned tigers in Texas in 2019 is 6734.\n\nIn 2020, there are 4816 privately owned tigers in Texas.\n\nWhich number shows the reduction of tigers owned privately between 2019 and 2020?"},{"varval":"One strategy:\n\nSubtract thousands first, then hundreds, then the balance (as per the solution).\n\n<div class=\"aligned\">\r\n\r\n| | |\r\n| ------------- | ---------- |\r\n| Distance left | \\= 6734 – 4816 |\r\n| | \\= 2734 – 816 |\r\n| | \\= 1934 – 16 |\r\n| | \\= {{{correctAnswer}}} tigers |\r\n\r\n</div>\r"}]},{"vars":[{"varval":"Dick was flying his helicopter to a destination 3407 kilometres away.\n\nHis radar showed that he had flown 1721 kilometres.\n\nHow much further does Dick need to fly to reach his destination?"},{"varval":"One strategy:\n\nSubtract thousands first, then hundreds, then the balance (as per the solution).\n\n<div class=\"aligned\">\r\n\r\n| | |\r\n| ------------- | ---------- |\r\n| Distance left | \\= 3407 – 1721 |\r\n| | \\= 2407 – 721 |\r\n| | \\= 1707 – 21 |\r\n| | \\= {{{correctAnswer}}} kilometres |\r\n\r\n</div>\r"}]}]

  247. NAP-F2-21

    <div class="sm_mode"> {{{question}}} </div>

    [{"vars":[{"varval":"Keira’s shoe print is shown in the picture below:\n\nsm_img https://teacher.smartermaths.com.au/wp-content/uploads//2021/08/45.svg 120 indent3 vpad\n\nWhich shoe below could have made the print?"},{"varval":"The correct shoe has the mirror image of the shoe print.\n\nsm_img https://teacher.smartermaths.com.au/wp-content/uploads//2021/08/45sol.svg 300 indent vpad"}]},{"vars":[{"varval":"Tristan has the glove shown below and is trying to find its pair.\n\n<br>\n\nsm_img https://teacher.smartermaths.com.au/wp-content/uploads//2021/08/46.svg 120 indent3 vpad\n\n<br>Which of the following is the pair of Tristan’s gloves?"},{"varval":"The correct pair of the glove is the mirror image.\n\nsm_img https://teacher.smartermaths.com.au/wp-content/uploads//2021/08/46sol.svg 250 indent vpad"}]}]

  248. NAP-F2-22

    <div class="sm_mode"> {{{question}}} </div>

    [{"vars":[{"varval":"A survey of four villages asked people if they had gone to hospital in the last 3 years.\n\r\nThe results are shown in the graph below.\n\n\r\nsm_img https://teacher.smartermaths.com.au/wp-content/uploads//2021/08/43.svg 550 indent vpad\n\nWhich village had the highest total number of people that had been to hospital in the last 3 years?"},{"varval":"sm_nogap Calculate the total of each village:\r\n\n> Village A = 25 + 35 = 60\r\n\n>Village B = 40 + 20 = 60\r\n\n>Village C = 45 + 30 = 75\r\n\n>Village D = 30 + 35 = 65\r\n\n<br>\n\nTherefore, {{{correctAnswer}}} has the most number of people that owns a pet.\r\n"}]},{"vars":[{"varval":"Four companies were asked how many men and women work in the company.\r\n\nThe results are shown in the graph below.\r\n\nsm_img https://teacher.smartermaths.com.au/wp-content/uploads//2021/08/44.svg 550 indent vpad\n\nWhich company has the highest total number of employees?"},{"varval":"sm_nogap Calculate the total employees in each company:\n\r\n\n>Company A = 35 + 40 = 75\r\n\n>Company B = 25 + 45 = 70\r\n\n>Company C = 45 + 35 = 80\r\n\n>Company D = 40 + 45 = 85\r\n\n<br>\n\nTherefore, {{{correctAnswer}}} has the highest number of employees.\r\n"}]},{"vars":[{"varval":"Two coffee shop franchises, Cafe A and Cafe B, are the only two coffee shops that sell coffee at four different shopping centres.\n\nPeople were asked at each shopping centre whether they purchased their coffee from Cafe A or Cafe B.\r\n\nThe results are shown in the graph below.\r\n\nsm_img https://teacher.smartermaths.com.au/wp-content/uploads/2022/09/Stat_Raph12-43-44_v2a.svg 550 indent vpad\n\nWhich shopping centre had the highest number of people completing the survey?"},{"varval":"Calculate the total people surveyed at each shopping centre:\n\r\n\n>Cafeteria A = 20 + 45 = 65\r\n\n>Cafeteria B = 35 + 15 = 50\r\n\n>Cafeteria C = 45 + 10 = 55\r\n\n>Cafeteria D = 40 + 20 = 60\r\n\n<br>\n\nTherefore, {{{correctAnswer}}} had the highest number of people completing the survey.\r\n"}]},{"vars":[{"varval":"Four soccer clubs were asked how many men and women were registered to play in the 2022 season.\r\n\nThe results are shown in the graph below.\r\n\nsm_img https://teacher.smartermaths.com.au/wp-content/uploads/2022/09/Stat_Raph12-43-44_v3.svg 580 indent vpad\n\nWhich soccer club has the highest total number of players?"},{"varval":"Calculate the total players registered at each club:\n\r\n\n>Club A = 110 + 70 = 180\r\n\n>Club B = 120 + 90 = 210\r\n\n>Club C = 95 + 110 = 205\r\n\n>Club D = 90 + 115 = 205\r\n\n<br>\n\nTherefore, {{{correctAnswer}}} has the highest number of players.\r\n"}]},{"vars":[{"varval":"Four companies recorded the sales of Product A and Product B in 2019.\r\n\nThe results are shown in the graph below.\r\n\nsm_img https://teacher.smartermaths.com.au/wp-content/uploads/2022/09/Stat_Raph12-43-44_v4.svg 550 indent vpad\n\nWhich company had the highest total sales of the two products in 2019?"},{"varval":"Calculate the total sales for each company:\n\r\n\n>Company A = 1700 + 1800 = 3500\r\n\n>Company B = 1400 + 1900 = 3300\r\n\n>Company C = 1600 + 1700 = 3300\r\n\n>Company D = 1700 + 1600 = 3300\r\n\n<br>\n\nTherefore, {{{correctAnswer}}} had the highest total sales.\r\n"}]},{"vars":[{"varval":"Four hardware companies were asked how many men and women customers purchased goods between 8 am and 7 pm on Thursday.\r\n\nThe results are shown in the graph below.\r\n\nsm_img https://teacher.smartermaths.com.au/wp-content/uploads/2022/09/Stat_Raph12-43-44_v5_1.svg 550 indent vpad\n\nWhich company has the highest total number of customers?"},{"varval":"Calculate the total customers for each company:\n\r\n\n>Company A = 5200 + 3600 = 8800\r\n\n>Company B = 4500 + 5100 = 9600\r\n\n>Company C = 4300 + 4900 = 9200\r\n\n>Company D = 5000 + 4200 = 9200\r\n\n<br>\n\nTherefore, {{{correctAnswer}}} had the highest number of customers.\r\n"}]}]

  249. Raph14-8 Raph14-9 These questions all use cents in the questions and the images were all of coins in Euro. I have added an image to var0 but it is grainy. Is it possible for Myca to get new images for these questions?

    <div class="sm_mode"> {{{question}}} </div>

    [{"vars":[{"varval":"Beth has 7 five-cent coins.\n\nsm_img https://teacher.smartermaths.com.au/wp-content/uploads/2022/11/Raph-14-Q7-9.svg 250 indent vpad\n\nHow much money does Beth have altogether?"},{"varval":"<div class=\"aligned\">\r\n\r\n| | |\r\n| ------------- | ---------- |\r\n| Total money | \\= 5 $\\times$ 7 |\r\n| | \\= {{{correctAnswer}}} |\r\n\r\n</div>\r\n"}]},{"vars":[{"varval":"Jane received 6 twenty-cent coins from her friend.\n\nsm_img https://teacher.smartermaths.com.au/wp-content/uploads//2021/08/raph14_q8.svg 320 indent vpad\n\nHow much money does Jane have altogether?"},{"varval":"<div class=\"aligned\">\r\n\r\n| | |\r\n| ------------- | ---------- |\r\n| Total money | \\= 20 $\\times$ 6 |\r\n| | \\= {{{correctAnswer}}} |\r\n\r\n</div>\r\n"}]},{"vars":[{"varval":"Jonathan has 6 five-cent coins.\n\nsm_img https://teacher.smartermaths.com.au/wp-content/uploads//2021/08/raph14_q9.svg 250 indent vpad\n\nHow much money does Jonathan have altogether?"},{"varval":"<div class=\"aligned\">\r\n\r\n| | |\r\n| ------------- | ---------- |\r\n| Total money | \\= 5 $\\times$ 6 |\r\n| | \\= {{{correctAnswer}}} |\r\n\r\n</div>\r\n"}]}]

  250. RAPH14-4 RAPH14-6 Changed some grammar and deleted an extra full stop but can't see much else wrong with these questions.

    <div class="sm_mode"> {{{question}}} </div>

    [{"vars":[{"varval":"Carl and Joshua each had a pen.\n\nThey each measured their pen against a line of identical pins.\r\n\n<br>\n\nsm_img https://teacher.smartermaths.com.au/wp-content/uploads/2021/08/Raph14-5.png 550 indent vpad\n\n<br>Which of the following statements is true?\n"},{"varval":"Joshua’s pen measures almost the length of 3 pins whilst Carl’s pen measures around 2 pins.\r\n\nTherefore, {{{correctAnswer}}}\n"}]},{"vars":[{"varval":"Andrea and Kelly each had a spoon.\r\n\nThey measured their spoons using paddle pop sticks.\r\n\n<br>\n\nsm_img https://teacher.smartermaths.com.au/wp-content/uploads/2021/08/Raph14-6.png 550 indent vpad\n\n<br>Which of these statements is true?\n"},{"varval":"Andrea’s spoon measures around 2 and a half paddle pop sticks, whilst Kelly’s spoon measures around 3 paddle pop sticks.\r\n\nTherefore, {{{correctAnswer}}}\r\n"}]},{"vars":[{"varval":"Juliet and Roman each had a pencil.\n\nThey each measured their pencil against a line of identical paperclips.\n\n<br>\n\nsm_img https://teacher.smartermaths.com.au/wp-content/uploads/2023/07/paperclip-1-pencil-1-min.svg 550 indent vpad\n\n<br>Which of the following statements is true?\n"},{"varval":"Roman’s pencil measures the length of 5 paperclips whilst Juliet’s pencil measures 4 paperclips.\n\nTherefore, {{{correctAnswer}}}\n"}]},{"vars":[{"varval":"Siren and Atlas each had a feather.\n\nThey each measured their feather against a line of identical paperclips.\n\n<br>\n\nsm_img https://teacher.smartermaths.com.au/wp-content/uploads/2023/07/feather-paperclip-1-min.svg 550 indent vpad\n\n<br>Which of the following statements is true?\n"},{"varval":"Atlas’s feather measures about the length of 5 and a half feathers whilst Siren’s feather measures about 3 feathers.\n\nTherefore, {{{correctAnswer}}}\n"}]},{"vars":[{"varval":"Molly and Joy each had a highlighter.\n\nThey each measured their highlighter against a line of identical pushpins.\n\n<br>\n\nsm_img https://teacher.smartermaths.com.au/wp-content/uploads/2023/07/pushpins-1-highlighters-min.svg 550 indent vpad\n\n<br>Which of the following statements is true?\n"},{"varval":"Molly’s highlighter measures about the length of 4 and a half pushpins and Joy’s highlighter measures about 4 and a half pushpins.\n\nTherefore, {{{correctAnswer}}}\n"}]},{"vars":[{"varval":"Andrew and Sue each caught a fish.\n\nThey each measured their fish against a line of identical pens.\n\n<br>\n\nsm_img https://teacher.smartermaths.com.au/wp-content/uploads/2023/07/Fish-pens-min.svg 550 indent vpad\n\n<br>Which of the following statements is true?\n"},{"varval":"Andrew’s fish measures about the length of 3 and a half pens whilst Sue’s fish measures about 3 pens.\n\nTherefore, {{{correctAnswer}}}\n"}]}]

  251. Raph14-1 Raph14-2 Raph14-3 Changed some punctuation and re-worded one variant but can't see anything else wrong with these questions.

    <div class="sm_mode"> {{{question}}} </div>

    [{"vars":[{"varval":"Who has the most hearts?"},{"varval":"sm_nogap Check each option:\r\n\n>Mary has 7 hearts\r\n\n>Sharon has 11 hearts\r\n\n>Hina has 9 hearts\r\n\n>Julia has 8 hearts\r\n\n<br>\n\nTherefore, Sharon has the most hearts."}]},{"vars":[{"varval":"Which group has the most diamonds?"},{"varval":"sm_nogap Check each option:\r\n\n>Oxford has 9 diamonds\r\n\n>Lincoln has 11 diamonds\r\n\n>Rutherford has 6 diamonds\r\n\n>William has 13 diamonds\r\n\n<br>\n\nTherefore, the William's group has the most diamonds. \n"}]},{"vars":[{"varval":"Who has the most leaves?"},{"varval":"sm_nogap Check each option:\r\n\n>John has 8 leaves\r\n\n>Neil has 11 leaves\r\n\n>Adrian has 12 leaves\r\n\n>Ryan has 14 leaves\r\n\n<br>\n\nTherefore, Ryan has the most leaves.\r\n"}]}]

  252. Measurement, NAPX-p73132v02

    <div class="sm_mode"> {{{question}}} </div>

    [{"vars":[{"varval":"What is the perimeter of the shape below given each square has a side length of 1 cm.\r\n\n<br>\n\nsm_img https://teacher.smartermaths.com.au/wp-content/uploads/2018/10/NAPX-I2-21-v2.svg 280 indent vpad"},{"varval":"sm_nogap Starting at the bottom right and moving clockwise:\n\n<div class=\"aligned\">\n\n|||\n|-|-|\n|Perimeter|= 6 + 1 + 1 + 1 + 1 + 1 + 3 + 2 + 1 + 3|\n||= {{{correctAnswer}}}|\n\n</div>"}]},{"vars":[{"varval":"Each square in the figure has a side length which measures 1 centimetre.\n\n<br>\n\nsm_img https://teacher.smartermaths.com.au/wp-content/uploads/2021/03/NAPX7-21-2-3-300x228.png 200 indent vpad\n\n<br>What is the perimeter of the figure?"},{"varval":"Starting at the bottom right corner and\r\nmoving clockwise:\n\nsm_img https://teacher.smartermaths.com.au/wp-content/uploads/2021/03/NAPX7-21-2-2-283x300.png 200 indent vpad\n\n<div class=\"aligned\">\n\n|||\n|-|-|\n|Perimeter|= 4 + 1 + 1 + 1 + 1 + 1 + 1 + 1 + 1 + 2|\n||= {{{correctAnswer}}}|\n\n</div>"}]}]

  253. Probability, NAPX-p116940v02 Probability, NAPX-p116940v03 Probability, NAPX-p116940v04

    <div class="sm_mode"> {{{question}}} </div>

    [{"vars":[{"varval":"The arrow pictured below is spun once:\n\n<br>\n\nsm_img https://teacher.smartermaths.com.au/wp-content/uploads/2018/10/NAPX-I2-20-v2.svg 200 indent3 vpad\n\n<br> Which shape is the spinner least likely to land on?"},{"varval":"Least likely to land on the shape that takes up the\r\nthe smallest area on the target.\n\n{{{correctAnswer}}}"}]},{"vars":[{"varval":"A spinning wheel has 3 different colours.\n\n<br>\n\nsm_img https://teacher.smartermaths.com.au/wp-content/uploads/2021/04/q39.png 220 indent3 vpad\n\n<br> Which colour in the wheel is most likely to land on?"},{"varval":"By inspection, grey is the most likely as it it shades the\r\nlargest area of the wheel."}]},{"vars":[{"varval":"A small disk is thrown onto the table top pictured below.\n\n<br>\n\nsm_img https://teacher.smartermaths.com.au/wp-content/uploads/2021/04/NAPX5-TLA-12-v1.svg 200 indent3 vpad\n\n<br> It has an equal chance of landing in any square.\r\n\r\nWhich numbered square is the disk least likely to land in?"},{"varval":"Only 1 square is numbered 4 (all other numbers have 2 squares).\n\n$\\therefore$ Least likely to land in square 4"}]},{"vars":[{"varval":"A spinning wheel has sections labelled with different numbers.\n\n<br>\n\nsm_img https://teacher.smartermaths.com.au/wp-content/uploads/2021/04/p116940v03.png 200 indent3 vpad\n\n<br> If the spinner has an equal chance of landing in each section, which of the numbers is the spinner most likely to land on?"},{"varval":"Since the spinner is divided into 8 equal sections and each number has\r 2 sections, all numbers are equally likely."}]}]

  254. <div class="sm_mode"> {{{question}}} </div>

    [{"vars":[{"varval":"Eight buttons, numbered 1 to 8, are placed in a bag.\r\n\r\nRobin picks three buttons out of the bag without looking.\r\n\r\nOnce chosen, a button is not put back into the bag.\r\n\r\nThe first button is number 2.\r\n\r\nWhich of the following cannot happen?"},{"varval":"Once number 2 is picked, it cannot be\r picked again (no replacement).\n\n$\\therefore$ The third button cannot be 2."}]},{"vars":[{"varval":"Six buttons, numbered 1 to 6, are placed in a bag.\r\n\r\nBarney picks four buttons out of the bag without looking.\r\n\r\nOnce chosen, a button is not put back into the bag.\r\n\r\nThe first three buttons Barney chose, in order, are 5, 1 and 3.\r\n\r\nWhich of the following cannot happen?"},{"varval":"Once numbers 5, 3 and 1 are picked, they cannot be\r picked again (no replacement).\n\n$\\therefore$ The 4th button chosen cannot be odd $\\rightarrow$ only even numbers left."}]},{"vars":[{"varval":"Seven buttons, numbered 1 to 7, are placed in a bag.\r\n\r\nZac picks four buttons out of the bag without looking.\r\n\r\nOnce chosen, a button is not put back into the bag.\r\n\r\nThe first two buttons Zac chose are 6 and 2.\r\n\r\nWhich of the following cannot happen?"},{"varval":"Once numbers 6 and 2 are picked, they cannot be\r picked again (no replacement).\n\n$\\therefore$ The 3rd and 4th picks cannot both be even $\\rightarrow$ only one even number is left."}]}]

  255. SJ v1 - v5

    <div class="sm_mode"> {{{question}}} </div>

    [{"vars":[{"varval":"Ricky has three boxes of balls.\n\n<br>\n\nsm_img https://teacher.smartermaths.com.au/wp-content/uploads/2017/04/naplan-Y3-2011-18mc.png 450 indent vpad\n\n<br>He picks one ball out of each box.\n\nWhich of these could be the balls he chooses?\n"},{"varval":"{{{correctAnswer}}}"}]},{"vars":[{"varval":"Nicholas has three boxes of Christmas decorations.\n\n<br>\n\nsm_img https://teacher.smartermaths.com.au/wp-content/uploads/2023/08/christmas-dec-box-min.svg 600 indent vpad\n\n<br>He picks one decoration out of each box.\n\nWhich of these could be the decorations he chooses?"},{"varval":"{{{correctAnswer}}}"}]},{"vars":[{"varval":"The Grinch has three boxes of Christmas decorations.\n\n<br>\n\nsm_img https://teacher.smartermaths.com.au/wp-content/uploads/2023/08/christmas-dec-box-min.svg 600 indent vpad\n\n<br>He picks one decoration out of each box.\n\nWhich of these could be the decorations he chooses?"},{"varval":"{{{correctAnswer}}}"}]},{"vars":[{"varval":"Scarlett has three boxes of Christmas decorations.\n\n<br>\n\nsm_img https://teacher.smartermaths.com.au/wp-content/uploads/2023/08/christmas-dec-box-min.svg 600 indent vpad\n\n<br>She picks one decoration out of each box.\n\nWhich of these could be the decorations she chooses?"},{"varval":"{{{correctAnswer}}}"}]},{"vars":[{"varval":"Jett has organised his t-shirts into three colour groups as shown below.\n\n<br>\n\nsm_img https://teacher.smartermaths.com.au/wp-content/uploads/2023/08/t-shirt-x-9a-min.svg 200 indent vpad\n\n<br>He picks one t-shirt from each group.\n\nWhich of these could be the t-shirts he chooses?"},{"varval":"{{{correctAnswer}}}"}]},{"vars":[{"varval":"Norman has organised his t-shirts into three colour groups as shown below.\n\n<br>\n\nsm_img https://teacher.smartermaths.com.au/wp-content/uploads/2023/08/t-shirt-x-9a-min.svg 200 indent vpad\n\n<br>He picks one t-shirt from each group.\n\nWhich of these could be the t-shirts he chooses?"},{"varval":"{{{correctAnswer}}}"}]}]

  256. Probability, NAPX-J2-02

    <div class="sm_mode"> {{{question}}} </div>

    [{"vars":[{"varval":"Shapes are drawn on the balls below and placed in a bag.\r\n\nsm_img https://teacher.smartermaths.com.au/wp-content/uploads/2018/10/NAPX-J2-02-v1.svg 300 indent vpad\n\nBilly reaches into the bag and takes out a ball without looking.\r\n\r\nWhich type of ball is he least likely to take out?"},{"varval":"Counting the balls of each type:\n\n<div class=\"sm_img_inline\">\n\n>6 $\\times$ ![](https://teacher.smartermaths.com.au/wp-content/uploads/2018/10/NAPX-J2-02-v1c.svg)\n\n>5 $\\times$ ![](https://teacher.smartermaths.com.au/wp-content/uploads/2018/10/NAPX-J2-02-v1b.svg)\n\n>3 $\\times$ ![](https://teacher.smartermaths.com.au/wp-content/uploads/2018/10/NAPX-J2-02-v1d.svg)\n\n>2 $\\times$ ![](https://teacher.smartermaths.com.au/wp-content/uploads/2018/10/NAPX-J2-02-v1a.svg)\n\n$\\therefore$ Least likely is ![](https://teacher.smartermaths.com.au/wp-content/uploads/2018/10/NAPX-J2-02-v1a.svg)\n\n</div>"}]},{"vars":[{"varval":"Shapes are drawn on the balls below and placed in a bag.\r\n\nsm_img https://teacher.smartermaths.com.au/wp-content/uploads/2018/10/NAPX-J2-02-v1.svg 350 indent vpad\n\nBigalow reaches into the bag and takes out a ball without looking.\r\n\r\nWhich type of ball is he most likely to take out?\n"},{"varval":"Counting the balls of each type\n\n<div class=\"sm_img_inline\">\n\n>6 $\\times$ ![](https://teacher.smartermaths.com.au/wp-content/uploads/2018/10/NAPX-J2-02-v1c.svg)\n\n>5 $\\times$ ![](https://teacher.smartermaths.com.au/wp-content/uploads/2018/10/NAPX-J2-02-v1b.svg)\n\n>3 $\\times$ ![](https://teacher.smartermaths.com.au/wp-content/uploads/2018/10/NAPX-J2-02-v1d.svg)\n\n>2 $\\times$ ![](https://teacher.smartermaths.com.au/wp-content/uploads/2018/10/NAPX-J2-02-v1a.svg)\n\n$\\therefore$ Most likely is ![](https://teacher.smartermaths.com.au/wp-content/uploads/2018/10/NAPX-J2-02-v1c.svg)\n\n</div>"}]}]

  257. Number, NAPX-I2-17

    <div class="sm_mode"> {{{question}}} </div>

    [{"vars":[{"varval":"Jane made 4 patterns on a grid paper.\n\n\r\n\r\nWhich pattern has the most shading?"},{"varval":"Consider the shading on each option: \n\n\r\nOption 1: 10 squares + 2 triangles = 11 squares\n\n\r\nOption 2: 6 squares + 6 triangles = 9 squares\n\n\r\nOption 3: 6 squares + 8 triangles = 10 squares\n\n\r\nOption 4: 6 squares + 8 triangles = 10 squares\n\nsm_img https://teacher.smartermaths.com.au/wp-content/uploads/2019/01/NAPX-G2-12a-v3.svg 140 indent vpad"}]},{"vars":[{"varval":"Sylvester painted squares white and grey on the grids pictured below.\n\n\r\n\r\nAll the squares in the grids are the same size.\n\n\r\n\r\nWhich grid has the largest area painted grey?"},{"varval":"16 squares painted grey is the most.\n\nsm_img https://teacher.smartermaths.com.au/wp-content/uploads/2018/10/NAPX-I2-17-v1c.svg 140 indent vpad"}]}]

  258. Measurement, NAPX-p116876v04 Measurement, NAPX-p116872v01 Measurement, NAPX-H2-10

    <div class="sm_mode"> {{{question}}} </div>

    [{"vars":[{"varval":"Ella builds the shape below using identical small blocks.\n\n<br>\n\nsm_img https://teacher.smartermaths.com.au/wp-content/uploads/2021/03/NAPX5-TLA-5-v1.svg 180 indent3 vpad\n\n<br>What is the smallest number of blocks Ella needs to build a cube?\n"},{"varval":"The smallest cube will be &nbsp;3 × 3 × 3\n\nExtra cubes needed (by row):\n\n\r\nBottom row = 2\n\n\r\n2nd row = 3\n\n\r\nTop row = 5\n\n\r\n$\\therefore$ {{{correctAnswer}}} blocks needed"}]},{"vars":[{"varval":"Flanno builds the shape below using identical small blocks.\n\n<br>\n\nsm_img https://teacher.smartermaths.com.au/wp-content/uploads/2021/03/NAPX5-TLA-5-v2.svg 200 indent2 vpad\n\n<br>What is the smallest number of blocks Flanno needs to build a cube?"},{"varval":"The smallest cube will be &nbsp;3 × 3 × 3\n\nExtra cubes needed (by row):\n\n\r\nBottom row = 1\n\n\r\n2nd row = 5\n\n\r\nTop row = 7\n\n\r\n$\\therefore$ 13 blocks needed"}]},{"vars":[{"varval":"Xavier builds the shape below using identical small blocks.\n\n<br>\n\nsm_img https://teacher.smartermaths.com.au/wp-content/uploads/2018/10/NAPX-H2-10-v2.svg 200 indent2 vpad\n\n<br>What is the smallest number of blocks Xavier needs to build a cube?"},{"varval":"The smallest cube will be &nbsp;3 × 3 × 3\n\nExtra cubes needed (by row):\n\n\r\nBottom row = 1\n\n\r\n2nd row = 5\n\n\r\n3rd row = 8\n\n\r\n$\\therefore$ Smallest number of blocks = {{{correctAnswer}}}"}]},{"vars":[{"varval":"This is a view of a solid made by stacking 24 identical cubes.\n\n<br>\n\nsm_img https://teacher.smartermaths.com.au/wp-content/uploads/2018/10/NAPX-H2-10-v1.svg 200 indent3 vpad\n\n<br>How many cubes are completely hidden in this view?"},{"varval":"Hidden cubes:\n\n\r\nBottom row = 6\n\n\r\n2nd row = 3\n\n\r\n3rd row = 1\n\n\r\n$\\therefore$ {{{correctAnswer}}} cubes are completely hidden."}]}]

  259. <div class="sm_mode"> {{{question}}} </div>

    [{"vars":[{"varval":"Derek used pencils to measure his desk.\n\n<br>\n\nsm_img https://teacher.smartermaths.com.au/wp-content/uploads/2017/04/NAP-G1-14-300x155.png 300 indent vpad\n\n<br>What is the total length of all four sides?"},{"varval":"sm_nogap Length of all 4 sides\n\n<div class=\"aligned\">\n\n>>||\n|-|\n|= 6 + 3 + 6 + 3|\n|= {{{correctAnswer}}}|\n\n</div>"}]}]

  260. <div class="sm_mode"> {{{question}}} </div>

    [{"vars":[{"varval":"The shapes below are made up of identical squares.\n\nWhich shape covers the largest area?"},{"varval":"Option 1 has the greatest number of squares (10) and therefore the largest area.\n\n{{{correctAnswer}}}"}]},{"vars":[{"varval":"The shapes below are made up of identical squares.\n\nWhich shape covers the largest area?"},{"varval":"Option 3 has the greatest number of squares (16) and therefore the largest area.\n\n{{{correctAnswer}}}"}]},{"vars":[{"varval":"The shapes below are made up of identical squares.\n\nWhich shape covers the largest area?"},{"varval":"Option 2 has the greatest number of squares (12) and therefore the largest area.\n\n{{{correctAnswer}}}"}]},{"vars":[{"varval":"The shapes below are made up of identical squares.\n\nWhich shape covers the largest area?"},{"varval":"Option 4 has the greatest number of squares (13) and therefore the largest area.\n\n\n{{{correctAnswer}}}"}]},{"vars":[{"varval":"The shapes below are made up of identical squares.\n\nWhich shape covers the smallest area?"},{"varval":"Option 2 has the least number of squares (9) and therefore the smallest area.\n\n\n{{{correctAnswer}}}"}]},{"vars":[{"varval":"The shapes below are made up of identical squares.\n\nWhich shape covers the smallest area?"},{"varval":"Option 3 has the least number of squares (9) and therefore the smallest area.\n\n{{{correctAnswer}}}"}]}]

  261. <div class="sm_mode"> {{{question}}} </div>

    [{"vars":[{"varval":"Which of these holds about the same amount of water as a kettle?"},{"varval":"{{{correctAnswer}}}"}]},{"vars":[{"varval":"Which of these would hold about the same amount of water as a bucket?\n\nsm_img https://teacher.smartermaths.com.au/wp-content/uploads/2023/07/bucket-1-min.svg 150 indent vpad"},{"varval":"{{{correctAnswer}}}"}]},{"vars":[{"varval":"Which of these would hold about the same amount as a cup of coffee?\n\n\nsm_img https://teacher.smartermaths.com.au/wp-content/uploads/2023/07/coffee-1-min.svg 250 indent vpad"},{"varval":"{{{correctAnswer}}}"}]},{"vars":[{"varval":"Which of these would hold about the same amount as a teaspoon?\n\nsm_img https://teacher.smartermaths.com.au/wp-content/uploads/2023/07/spoon-1.svg 150 indent vpad"},{"varval":"{{{correctAnswer}}}"}]},{"vars":[{"varval":"Which of these would hold about the same amount as a can of drink?\n\nsm_img https://teacher.smartermaths.com.au/wp-content/uploads/2023/07/soda-1-min.svg 100 indent vpad"},{"varval":"{{{correctAnswer}}}"}]},{"vars":[{"varval":"Which of these would hold about the same amount as a sandpit?\n\nsm_img https://teacher.smartermaths.com.au/wp-content/uploads/2023/07/sandpit-1-min.svg 280 indent2 vpad"},{"varval":"{{{correctAnswer}}}"}]}]

  262. Algebra, NAPX-p169224v02 SJ v2-5

    <div class="sm_mode"> {{{question}}} </div>

    [{"vars":[{"varval":"A delivery company uses a formula to determine the cost of shipping different sizes of boxes.\r\n\r\n\nThe formula they use is as follows:\r\n\r\n\n>>Size of box = length + width + height\r\n\r\n\n<br>\n\nThe maximum size that can be shipped is 240 cm.\r\n\r\nWhich box is oversized?"},{"varval":"Check each option:\r\n\nOption 1 - 100 + 80 + 60 = 240\r\n\nOption 2 - 70 + 60 + 90 = 220\r\n\nOption 3 - 90 + 90 + 50 = 230\r\n\nOption 4 - 90 + 110 + 50 = 250 (Oversized)"}]},{"vars":[{"varval":"A company ships crates overseas and uses a formula for calculating the size and cost of shipping.\r\n\r\n\nThe formula is shown below:\r\n\r\n\n>>Size of crate = Length + Width + Height\r\n\r\n\n<br>\n\nThe maximum size of crates to be shipped overseas is 350 cm.\r\n\r\nWhich of the following crates is oversized?"},{"varval":"Check each option:\r\n\nOption 1 - 200 + 60 + 80 = 340\r\n\nOption 2 - 150 + 130 + 90 = 370 (Oversized)\r\n\nOption 3 - 160 + 100 + 70 = 330\r\n\nOption 4 - 130 + 120 + 100 = 350"}]},{"vars":[{"varval":"A furniture company uses a formula to determine the cost of shipping different sizes of furniture crates.\n\n\nThe formula they use is as follows:\n\n\n>>Size of box = length + width + height\n\n\n<br>\n\nThe maximum size that can be shipped is 640 cm.\n\nWhich crate is oversized?"},{"varval":"Check each option:\n\nOption 1 - 200 + 300 + 140 = 640\n\nOption 2 - 150 + 260 + 190 = 600\n\nOption 3 - 290 + 190 + 180 = 660 (Oversized)\n\nOption 4 - 130 + 270 + 210 = 610 "}]},{"vars":[{"varval":"Australia Post uses a formula to determine the cost of delivering boxes of different sizes.\n\n\nThe formula they use is as follows:\n\n\n>>Size of box = length + width + height\n\n\n<br>\n\nThe maximum size that can be shipped is 360 cm.\n\nWhich box is oversized?"},{"varval":"Check each option:\n\nOption 1 - 170 + 120 + 80 = 370 (Oversized)\n\nOption 2 - 250 + 30 + 30 = 310\n\nOption 3 - 140 + 140 + 60 = 340\n\nOption 4 - 190 + 110 + 60 = 360"}]},{"vars":[{"varval":"A delivery company uses a formula to determine the cost of shipping boxes of different sizes.\n\n\nThe formula they use is as follows:\n\n\n>>Size of box = length + width + height\n\n\n<br>\n\nThe maximum size that can be shipped is 580 cm.\n\nWhich box is oversized?"},{"varval":"Check each option:\n\nOption 1 - 220 + 240 + 90 = 550\n\nOption 2 - 370 + 80 + 130 = 580\n\nOption 3 - 20 + 250 + 300 = 570\n\nOption 4 - 160 + 180 + 250 = 590 (Oversized)"}]},{"vars":[{"varval":"A delivery company uses a formula to determine the cost of shipping crates of different sizes.\n\n\nThe formula they use is as follows:\n\n\n>>Size of box = length + width + height\n\n\n<br>\n\nThe maximum size that can be shipped is 425 cm.\n\nWhich box is oversized?"},{"varval":"Check each option:\n\nOption 1 - 165 + 85 + 160 = 410\n\nOption 2 - 205 + 90 + 130 = 425\n\nOption 3 - 195 + 105 + 130 = 430 (Oversized)\n\nOption 4 - 230 + 110 + 80 = 420"}]}]

  263. Statistics, NAPX-p72862v01

    <div class="sm_mode"> {{{question}}} </div>

    [{"vars":[{"varval":"A group of students were asked what job they would like when they grow up.\r\n\r\nThe results are represented in the graph below.\n\n<br>\n\nsm_img https://teacher.smartermaths.com.au/wp-content/uploads/2021/03/NAPX7-22-2.svg 350 indent2 vpad\n\n<br>How many students were asked?"},{"varval":"<div class=\"aligned\">\r\n\r\n| | |\r\n| ------------- | ---------- |\r\n| Students asked | \\= 5 + 6 + 4 + 7 |\r\n| | \\= {{{correctAnswer}}} |\r\n\r\n</div>\r\n"}]},{"vars":[{"varval":"There were different coloured new model cars in a car show.\r\n\r\nThe colours were represented in the graph below.\n\n<br>\n\nsm_img https://teacher.smartermaths.com.au/wp-content/uploads/2021/03/NAPX7-22-1.svg 350 indent2 vpad\n\n<br>How many new model cars were in the car show?"},{"varval":"<div class=\"aligned\">\r\n\r\n| | |\r\n| ------------- | ---------- |\r\n| New model cars | \\= 3 + 2 + 5 + 3 |\r\n| | \\= {{{correctAnswer}}} |\r\n\r\n</div>\r\n"}]}]

  264. Statistics, NAPX-p116802v02 Statistics, NAPX-p116802v01 Statistics, NAPX-I2-14

    <div class="sm_mode"> {{{question}}} </div>

    [{"vars":[{"varval":"In a neighborhood 10 villagers were asked if how many pets they own.\r\n\r\nThe results were: 3, 2, 1, 1, 1, 4, 2, 4, 5, 2\r\n\r\nSelect the dot plot that displays the data recorded."},{"varval":"A dot represents the number of persons that owns N pets\r\n\nWriting the data in order:\r\n\n1, 1, 1, 2, 2, 2, 3, 4, 4, 5\n\n\n$\\therefore$ The correct dot plot is:\n\n<br>\n\nsm_img https://teacher.smartermaths.com.au/wp-content/uploads/2021/04/RAPH8-Q28-a.svg 200 indent2 vpad"}]},{"vars":[{"varval":"Ten school children were asked how many times they went to the beach last month.\r\n\r\nTheir answers were: 5, 3, 0, 2, 0, 1, 5, 5, 4, 1\r\n\r\nWhich of the dot plots below correctly displays the data​?"},{"varval":"Writing the data in order:\r\n\n0, 0, 1, 1, 2, 3, 4, 5, 5, 5\n\n<br>\n\nsm_img https://teacher.smartermaths.com.au/wp-content/uploads/2018/10/NAPX-I2-14-v3c.svg 150 indent2 vpad"}]},{"vars":[{"varval":"Nine school children were asked how many brothers and sisters they have in their family.\r\n\r\nTheir answers were: 1, 1, 3, 0, 4, 2, 2, 1, 0\r\n\r\nWhich of the dot plots below correctly displays the data​?"},{"varval":"Writing the data in order:\r\n\n0, 0, 1, 1, 1, 2, 2, 3, 4\n\n<br>\n\nsm_img https://teacher.smartermaths.com.au/wp-content/uploads/2018/10/NAPX-I2-14-v2b.svg 120 indent2 vpad"}]},{"vars":[{"varval":"Writing the data in order:\r\n\n0, 0, 1, 2, 2, 2, 3, 3, 4, 5\n\n<br>\n\nsm_img https://teacher.smartermaths.com.au/wp-content/uploads/2018/10/NAPX-I2-14-v1d.svg 140 indent2 vpad"},{"varval":"Ten school children were asked how many sick days they had off school last month.\r\n\r\nTheir answers were: 0, 3, 2, 2, 1, 5, 3, 4, 2, 0\r\n\r\nWhich of the dot plots below correctly displays the data"}]}]

  265. Statistics, NAPX-I2-02 SA

    <div class="sm_mode"> {{{question}}} </div>

    [{"vars":[{"varval":"In a school, a group of students were given a list of 4 subjects and asked to pick their favourite.\r\n\r\nA table of their responses is shown below.\n\n<br>\n\nsm_img https://teacher.smartermaths.com.au/wp-content/uploads/2021/04/NAPX-p116609v03-SA.png 490 indent2 vpad\n\n<br>How many students were asked this question altogether?"},{"varval":"Mathematics = 5 + 5 + 5 + 2 = 17\r\n\nScience = 5 + 5 = 10\r\n\nEnglish = 5 + 1 = 6\r\n\nHistory = 5 + 5 + 1 = 11\r\n\n<br>\n\n$\\therefore$ Total = 17 + 10 + 6 + 11 = {{{correctAnswer0}}}"}]},{"vars":[{"varval":"Over one hour, Cher recorded the colour of all cars that drove past her school for a class project.\r\n\r\nThe results were recorded in the table below.\n\nsm_img https://teacher.smartermaths.com.au/wp-content/uploads/2018/10/NAPX-I2-02-SA-v1.svg 280 indent vpad\n\nHow many cars drove past Cher's school in the hour?"},{"varval":"sm_nogap Number of cars\n\n<div class=\"aligned\">\r\n\r\n>> | |\r\n| ---------- |\r\n| \\= 7 + 17 + 13 + 6 |\r\n| \\= {{{correctAnswer0}}} |\r\n\r\n</div>\r\n"}]},{"vars":[{"varval":"Demeter records the number of items she recycles in one week.\r\n\r\nThe tally table shows Demeter's results.\n\n<br>\n\nsm_img https://teacher.smartermaths.com.au/wp-content/uploads/2020/03/STATISTICS-NAPX-K2-04v1-SA-table2.svg 280 indent2 vpad\n\n<br>How many items does Demeter recycle in total?"},{"varval":"<div class=\"aligned\">\r\n\r\n| | |\r\n| ------------- | ---------- |\r\n| Total items | \\= 4 + 2 + 7 + 17 |\r\n| | \\= {{{correctAnswer0}}} |\r\n\r\n</div>\r"}]},{"vars":[{"varval":"Nellie records the number of items she recycles in one week.\n\nThe tally table shows Nellie's results.\n\r\n<br>\n\nsm_img https://teacher.smartermaths.com.au/wp-content/uploads/2020/03/STATISTICS-NAPX-K2-04v1-SA-table-1.svg 280 indent2 vpad\n\n<br>How many items does Nellie recycle in total?"},{"varval":"<div class=\"aligned\">\r\n\r\n| | |\r\n| ------------- | ---------- |\r\n| Total items | \\= 7 + 13 + 2 + 5 |\r\n| | \\= {{{correctAnswer0}}} |\r\n\r\n</div>\r"}]},{"vars":[{"varval":"Sharnie records the number of items she recycles in one week.\r\n\r\nThe tally table shows Sharnie's results.\r\n\n<br>\n\nsm_img https://teacher.smartermaths.com.au/wp-content/uploads/2018/12/NAPX-K2-04v1.svg 280 indent vpad\n\n<br>How many items does Sharnie recycle in total?"},{"varval":"<div class=\"aligned\">\r\n\r\n| | |\r\n| ------------- | ---------- |\r\n| Total items \t | \\= 9 + 12 + 3 + 1 |\r\n| | \\= {{{correctAnswer0}}} |\r\n\r\n</div>\r"}]}]

  266. Removed repeated sentence. Added categories.

    <div class="sm_mode"> {{{question}}} </div>

    [{"vars":[{"varval":"Bruno is in Year 5. He wants to know the most popular colour for boys in Year 5.\r\n\r\nWhich of these would best help Bruno to solve this problem?"},{"varval":"The best data will come from Year 5 boys.\n\n$\\therefore$ He should ask all the Year 5 boys.\n"}]}]

  267. <div class="sm_mode"> {{{question}}} </div>

    [{"vars":[{"varval":"Four friends were asked what pets they owned.\n\n<br>\n\n<div class=\"sm-table col1-color8 row1-color8 top-left-cell-hidden\">\n\n>>| | dog | cat | horse | goldfish | budgie|turtle|\n|:-:|:-:|:-:|:-:|:-:|:-:|:-:|\n| Chandler | $\\checkmark$| $\\checkmark$|$\\checkmark$|$\\checkmark$|||\n| Ross | $\\checkmark$| $\\checkmark$|$\\checkmark$||||\n| Phoebe | $\\checkmark$| ||$\\checkmark$|$\\checkmark$|$\\checkmark$|\n| Monica | | $\\checkmark$||$\\checkmark$|$\\checkmark$||\n\n</div>\n\n\n<br>Who had both a cat and a budgie as pets?\n"},{"varval":"{{{correctAnswer}}}"}]},{"vars":[{"varval":"Four friends were asked what pets they owned.\n\n<br>\n\n<div class=\"sm-table col1-color2 row1-color2 top-left-cell-hidden\">\n\n>>| | dog | cat | horse | goldfish | budgie|turtle|\n|:-:|:-:|:-:|:-:|:-:|:-:|:-:|\n| Marcus | $\\checkmark$| $\\checkmark$|$\\checkmark$|$\\checkmark$|||\n| Brett | $\\checkmark$| $\\checkmark$|$\\checkmark$||||\n| Raine | $\\checkmark$| ||$\\checkmark$|$\\checkmark$|$\\checkmark$|\n| Mila | | $\\checkmark$||$\\checkmark$|$\\checkmark$||\n\n</div>\n\n\n<br>Who had both a dog and a turtle as pets?\n"},{"varval":"{{{correctAnswer}}}"}]},{"vars":[{"varval":"Four friends were asked what rides they went on at Luna Park.\n\n<br>\n\n<div class=\"sm-table col1-color5 row1-color5 top-left-cell-hidden\">\n\n>>| | Dodgem cars | Tango Train | Hair Raiser | Wild Mouse | Ferris Wheel|turtle|\n|:-:|:-:|:-:|:-:|:-:|:-:|\n| Chris |$\\checkmark$| $\\checkmark$||$\\checkmark$|| \n| Bluey | $\\checkmark$| $\\checkmark$|$\\checkmark$|||\n| Penny | $\\checkmark$||||$\\checkmark$|\n| Jules | | $\\checkmark$|$\\checkmark$|$\\checkmark$||\n\n</div>\n\n\n<br>Who went on both the Dodgem Cars and the Wild Mouse?\n"},{"varval":"{{{correctAnswer}}}"}]},{"vars":[{"varval":"Four workers recorded the days they worked last week.\n\n<br>\n\n<div class=\"sm-table col1-color4 row1-color4 top-left-cell-hidden\">\n\n>>| | Monday | Tuesday | Wednesday | Thursday | Friday|\n|:-:|:-:|:-:|:-:|:-:|:-:|\n| Booth | | $\\checkmark$|$\\checkmark$|$\\checkmark$||\n| TJ | $\\checkmark$| $\\checkmark$||||\n| James | $\\checkmark$| |$\\checkmark$|$\\checkmark$|$\\checkmark$|\n| Harry | | |$\\checkmark$|$\\checkmark$||\n\n</div>\n\n\n<br>Who worked both Monday and Wednesday last week?\n"},{"varval":"{{{correctAnswer}}}"}]},{"vars":[{"varval":"Four friends were asked what sports they watch on TV.\n\n<br>\n\n<div class=\"sm-table col1-color1 row1-color1 top-left-cell-hidden\">\n\n>>| | AFL | Tennis | Golf | Netball | \n|:-:|:-:|:-:|:-:|:-:|\n| Greg | $\\checkmark$| |$\\checkmark$|$\\checkmark$|\n| Annie | $\\checkmark$||$\\checkmark$||\n| Maree | $\\checkmark$|$\\checkmark$||$\\checkmark$|\n| Carl | | $\\checkmark$||$\\checkmark$|\n\n</div>\n\n<br>Which friend watches Netball but **not** tennis?\n"},{"varval":"{{{correctAnswer}}}"}]},{"vars":[{"varval":"Four friends were asked what sports they played.\n\n<br>\n\n<div class=\"sm-table col1-color7 row1-color7 top-left-cell-hidden\">\n\n>>| | Football | Netball | Baseball | Basketball | \n|:-:|:-:|:-:|:-:|:-:|\n| Ricky || |$\\checkmark$|$\\checkmark$|\n| Ross | | $\\checkmark$|$\\checkmark$||\n| Maryanne | $\\checkmark$| ||$\\checkmark$|\n| Mikaila| | $\\checkmark$||$\\checkmark$|\n\n</div>\n\n\n<br>Which sport do both Ricky and Ross play?\n"},{"varval":"{{{correctAnswer}}}"}]}]

  268. <div class="sm_mode"> Ziggy threw darts at the board below. He hit the board with every throw. <br> sm_img https://teacher.smartermaths.com.au/wp-content/uploads/2017/03/naplan-Y5-2015-2mca.png 200 indent vpad <br>He recorded each time he hit a colour in the table below. <br> sm_img https://teacher.smartermaths.com.au/wp-content/uploads/2018/10/NAPX-H2-02-v1.svg 250 indent vpad <br>How many darts did Ziggy throw in total? </div>

    [{"vars":null}]

  269. <div class="sm_mode"> Seven buttons, numbered 1 to 7, are placed in a bag. Shelley picks four buttons out of the bag without looking and without replacement. The first button is number 6. Which of the following cannot happen? </div>

    [{"vars":null}]

  270. Statistics, NAPX-K2-06

    <div class="sm_mode"> {{{question}}} </div>

    [{"vars":[{"varval":"The table shows the time (in seconds) some children can run 100 meters.\n\n<br>\n\n<div class=\"outline color7\">\n\n>> | Name | Time (seconds) |\n> | :-----------: | :----------: |\n> | Rog | 18 |\n> | Ty | 19 |\n> | Ian | 20 |\n> | Sue | 13 |\n\n</div>\n\n<br>\n\nWhich column graph correctly shows the data in the table?"},{"varval":"{{{correctAnswer}}}"}]},{"vars":[{"varval":"The table shows the time (in seconds) some children can run across a football field.\r\n\n<br>\n\n<div class=\"outline color4\">\n\n>> | Name | Time (seconds) |\n> | :-----------: | :----------: |\n> | Di | 16 |\n> | Meg | 12 |\n> | Ian | 17 |\n> | Pip | 14 |\n\n</div>\n\n<br>Which column graph correctly shows the data in the table?"},{"varval":"{{{correctAnswer}}}"}]}]

  271. <div class="sm_mode"> Kris has these candy canes. <br> sm_img https://teacher.smartermaths.com.au/wp-content/uploads/2018/10/NAPX-J2-03-v2.svg 400 indent vpad <br>He gives half of them to his friend Jasper. How many candy canes does Jasper get? </div>

    [{"vars":[]}]

  272. Number, NAPX-p116602v03 Algebra, NAPX-p116602v01 Algebra, NAPX-p116602v02

    <div class="sm_mode"> {{{question}}} </div>

    [{"vars":[{"varval":"\r\nJordan lives in Perth and receives 10 cents for every glass bottle she recycles at the depot.\r\n\r\nJordan takes 33 glass bottles to the depot.\r\n\r\nHow much money will she receive?"},{"varval":"<div class=\"aligned\">\r\n\r\n| | |\r\n| ------------- | ---------- |\r\n| 33 $\\times$ 10¢ | \\= 330¢ |\r\n| | \\= {{{correctAnswer}}} |\r\n\r\n</div>\r\n"}]},{"vars":[{"varval":"George wants to make doughnuts that cost him 10 cents to make each.\n\nsm_img https://teacher.smartermaths.com.au/wp-content/uploads/2021/04/RAPH8-Q2.svg 300 indent vpad\n\nIf he makes 28 of these donuts, how much will it cost him?"},{"varval":"<div class=\"aligned\">\r\n\r\n| | |\r\n| ------------: | ---------- |\r\n| Total money earned | \\= 28 $\\times$ 10 cents |\r\n| | \\= {{{correctAnswer}}} |\r\n\r\n</div>\r\n"}]},{"vars":[{"varval":"Karl recycles the plastic bottles his family use and receives 10 cents for each bottle he recycles.\r\n\r\nLast month, he collected 43 plastic bottles.\r\n\r\nHow much will he receive?"},{"varval":"<div class=\"aligned\">\r\n\r\n| | |\r\n| ------------- | ---------- |\r\n| 43 $\\times$ 10¢ | \\= 430¢ |\r\n| | \\= {{{correctAnswer}}} |\r\n\r\n</div>\r\n"}]},{"vars":[{"varval":"Arnie recycles aluminum cans and receives 10 cents per can at the local recycling depot.\r\n\r\nArnie has collected 84 aluminum cans.\r\n\r\nHow much will he receive from the depot?"},{"varval":"<div class=\"aligned\">\r\n\r\n| | |\r\n| ------------- | ---------- |\r\n| 84 $\\times$ 10¢ | \\= 840¢ |\r\n| | \\= {{{correctAnswer}}} |\r\n\r\n</div>\r\n"}]},{"vars":[{"varval":"May sells buttons at the local market and receives 10 cents per button.\n\nLast week may sold 56 buttons.\n\nHow much will she receive from her button sales?"},{"varval":"<div class=\"aligned\">\n\n| | |\n| ------------- | ---------- |\n| 56 $\\times$ 10¢ | \\= 560¢ |\n| | \\= {{{correctAnswer}}} |\n\n</div>\n"}]},{"vars":[{"varval":"Jewel makes 10 cents profit for every jar of jam she sells at her market stall.\n\nLast week may sold 92 jars of jam.\n\nHow much will Jewel receive from her jam sales?"},{"varval":"<div class=\"aligned\">\n\n| | |\n| ------------- | ---------- |\n| 92 $\\times$ 10¢ | \\= 920¢ |\n| | \\= {{{correctAnswer}}} |\n\n</div>\n"}]}]

  273. Algebra, NAPX-p122115v02 Algebra, NAPX-p122115v01 Added categories.

    <div class="sm_mode"> {{{question}}} </div>

    [{"vars":[{"varval":"Calvin had 72 coloured pencils.\r\n\r\nHe lined them up so there are 8 pencils in each row.\r\n\r\nHow many rows are there?"},{"varval":"<div class=\"aligned\">\r\n\r\n| | |\r\n| ------------- | ---------- |\r\n| Number of rows | \\= 72 $\\div$ 8 |\r\n| | \\= 9 |\r\n\r\n</div>\r\n"}]},{"vars":[{"varval":"Vivian had 54 cricket balls.\r\n\r\nHis club had 6 cricket teams.\r\n\r\nHe gave the same number of balls to each team.\r\n\r\nHow many balls did each team receive?"},{"varval":"<div class=\"aligned\">\r\n\r\n| | |\r\n| ------------- | ---------- |\r\n| Number of rows | \\= 54 $\\div$ 6 |\r\n| | \\= 9 |\r\n\r\n</div>\r\n"}]},{"vars":[{"varval":"Pablo caught 56 blowfish.\r\n\r\nHe lined them up in rows so there were 7 blowfish in each row.\r\n\r\nHow many rows are there?"},{"varval":"<div class=\"aligned\">\r\n\r\n| | |\r\n| ------------- | ---------- |\r\n| Number of rows | \\= 56 $\\div$ 7 |\r\n| | \\= 8 |\r\n\r\n</div>\r\n"}]}]

  274. SJ v2-v5 Multiply/divide

    <div class="sm_mode"> {{{question}}} </div>

    [{"vars":[{"varval":"Aditi had 78 blocks.\r\n\r\nShe built identical shapes that were made up of 15 blocks each.\r\n\r\nShe had 3 blocks left over.\r\n\r\nHow many shapes did Aditi make?"},{"varval":"Strategy 1\n\n2 shapes = 30 blocks\n\n4 shapes = 60 blocks\n\n5 shapes = 60 + 15 = 75 (3 blocks left)\n\n<br>\n\nStrategy 2\n\n78 $\\div$ 15 = 5 remainder 3\r\n\n$\\therefore$ Aditi made 5 shapes."}]},{"vars":[{"varval":"Godiva made 83 chocolates.\r\n\r\nShe placed them in boxes of 7 chocolates.\r\n\r\nWhat is the maximum number of boxes she can fill up?"},{"varval":"sm_nogap Boxes that can be filled\r\n\n<div class=\"aligned\">\r\n\r\n| | |\r\n| ------------- | ---------- |\r\n| | \\= 83 $\\div$ 7 |\r\n| | \\= 11 remainder 6 |\r\n\r\n</div>\r\n\r\n\n<br>\n\n\r\n\r\n$\\therefore$ Godiva can fill up 11 boxes."}]},{"vars":[{"varval":"Lecretia had 92 sweets for Halloween.\n\nShe put them into bags containing 11 sweets each.\n\nShe had 4 sweets left over.\n\nHow many bags did Lecretia fill?"},{"varval":"<div class=\"aligned\">\n\n| | |\n| -------------: | ---------- |\n|Sweets that fit in the full bags | \\= 92 - 4 = 88 |\n|Number of bags|= 88 $\\div$ 11|\n| | \\= {{{correctAnswer}}} |\n\n</div>\n\n<br>\n\n$\\therefore$ Lecretia can fill up {{{correctAnswer}}} bags."}]},{"vars":[{"varval":"Danielle had 35 litres of milk.\n\nShe poured the milk into bottles, each with a capacity of 4 litres.\n\nShe had 3 litres left over.\n\nHow many bottles did Danielle fill?"},{"varval":"<div class=\"aligned\">\n\n| | |\n| -------------: | ---------- |\n|Milk that fits into the full bottles | \\= 35 - 3 = 32 |\n|Number of bottles|= 32 $\\div$ 4|\n| | \\= {{{correctAnswer}}} |\n\n</div>\n\n<br>\n\n$\\therefore$ Danielle can fill up {{{correctAnswer}}} bottles."}]},{"vars":[{"varval":"Bonn had 50 guitar strings.\n\nHe packaged them in bags, each containing 6 strings.\n\nHe had 2 strings left over.\n\nHow many bags did Bonn fill?"},{"varval":"<div class=\"aligned\">\n\n| | |\n| -------------: | ---------- |\n|Strings that fit into the full bags | \\= 50 - 2 = 48 |\n|Number of bags|= 48 $\\div$ 6|\n| | \\= {{{correctAnswer}}} |\n\n</div>\n\n<br>\n\n$\\therefore$, Bonn can fill up {{{correctAnswer}}} bags."}]},{"vars":[{"varval":"Merlin had 57 eggs.\n\nHe packed them into cartons, each of which held 6 eggs.\n\nHe had 3 eggs left over.\n\nHow many cartons did Merlin fill?"},{"varval":"<div class=\"aligned\">\n\n| | |\n| -------------: | ---------- |\n|Eggs that fit in the full cartons | \\= 57 - 3 = 54 |\n|Number of cartons|= 54 $\\div$ 6|\n| | \\= {{{correctAnswer}}} |\n\n</div>\n\n<br>\n\n$\\therefore$ Merlin can fill up {{{correctAnswer}}} cartons."}]}]

  275. Algebra, NAPX-p116672v02

    <div class="sm_mode"> {{{question}}} </div>

    [{"vars":[{"varval":"Rudy uses the number sentence &nbsp;24 – 6 = 18\r\n\r\nWhich of the following problems can he solve with this number sentence?"},{"varval":"Rudy has 24 slices of cake and gave 6 slices to his friends. \r\n\nExpressing this as an equation:\n\n<div class=\"aligned\">\r\n\r\n| | |\r\n| ------------: | ---------- |\r\n| Slices Rudy has left\t | \\= 24 − 6 |\r\n| 18 | \\= 24 − 6 |\r\n\r\n</div>\r\n"}]},{"vars":[{"varval":"Shawn uses the number sentence &nbsp;20 $\\times$ 6 = 120\r\n\r\nWhich of the following problems can he solve with this number sentence?"},{"varval":"Shawn has 20 nephews and gives 6 toys to each one.\r\n\nExpressing this as an equation:\n\n<div class=\"aligned\">\r\n\r\n| | |\r\n| ------------: | ---------- |\r\n| Total toys to nephews | \\= 20 $\\times$ 6 |\r\n| 120 | \\= 20 $\\times$ 6 |\r\n\r\n</div>\r\n"}]}]

  276. <div class="sm_mode"> There are 56 students in year 7 at a school. Every student is either right-handed or left-handed. There are 6 times more right-handers than left-handers. How many left handers are there in Year 7? </div>

    [{"vars":null}]

  277. Added detail to the question. Added categories.

    <div class="sm_mode"> {{{question}}} </div>

    [{"vars":[{"varval":"Tom bought these apples.\n\nsm_img https://teacher.smartermaths.com.au/wp-content/uploads/2017/04/naplan-Y3-2009-16mc.png 350 indent vpad\n\nHe put a quarter of the apples in his fridge.\r\n\r\nHow many apples did Tom put in his fridge?\n"},{"varval":"<div class=\"aligned\">\r\n\r\n| | |\r\n| ------------- | ---------- |\r\n| Total apples \t | \\= 3 $\\times$ 8 |\r\n| | \\= 24 |\r\n\r\n</div>\r\n\n<br>\n\n<div class=\"aligned\">\r\n\r\n| | |\r\n| ------------- | ---------- |\r\n| $\\Rarr$ Apples put in fridge | \\= $\\dfrac{1}{4} \\ \\times\\ 24$ |\r\n| | \\= {{{correctAnswer0}}} |\r\n\r\n</div>"}]}]

  278. <div class="sm_mode"> A group of 8 friends are planning a trip to Leyland Brothers World. Lunch will cost $7 per person. Entry will cost $13 per person. Hiring a bus for transport will cost $180 for the whole group. What is the expected total cost of the trip for the whole group? </div>

    [{"vars":null}]

  279. <div class="sm_mode"> Gillian has 84 brochures to give out at a conference. She puts an equal number of brochures onto each table at the conference and has none left over. Select the option that shows the number of tables that could not be at the conference. </div>

    [{"vars":null}]

  280. Algebra, NAPX-p72911v02 Number, NAPX-I3-NC14, NAPX-I2-30

    <div class="sm_mode"> {{{question}}} </div>

    [{"vars":[{"varval":"Patrick earns money by doing some part time jobs.\r\n\r\nFor each job he completes he earns $2.00\r\n\r\nHe records the number of jobs he does in a week using the table below.\r\n\n<br>\n\nsm_img https://teacher.smartermaths.com.au/wp-content/uploads/2021/03/NAPX5-30-1.png 380 indent vpad\n\n<br>If Patrick does the same jobs for two weeks, how much money will he earn altogether?"},{"varval":"sm_nogap Total jobs in 1 week\n\n<div class=\"aligned\">\r\n\r\n>>| |\r\n| ------------- | ---------- |\r\n| \\= 4 + 5 + 8 |\r\n| \\= 17 |\r\n\r\n</div>\n\n\nsm_nogap $\\therefore$ Money earned in 2 weeks\n\n<div class=\"aligned\">\r\n\r\n>>| |\r\n| ------------- | ---------- |\r\n| \\= 2 $\\times$ (17 $\\times$ 2) |\r\n| \\= 2 $\\times$ 34 |\n| \\= {{{correctAnswer}}} |\r\n\r\n</div>\r\n"}]},{"vars":[{"varval":"John donates money to 4 different charities.\r\n\r\nHe gives $5 each time he donates.\r\n\r\nHe records the number of donations he makes in a month using the table below.\n\n<br>\n\nsm_img https://teacher.smartermaths.com.au/wp-content/uploads/2021/03/NAPX5-30-2-768x277.png 390 indent2 vpad\n\n<br>If John makes the same number of donations for 2 months in a row, how much money will he donate altogether?"},{"varval":"sm_nogap Total number of donations in 1 month\n\n<div class=\"aligned\">\r\n\r\n>>| |\r\n| ------------- | ---------- |\r\n| \\= 5 + 6 + 2 + 3 |\r\n| \\= 16 |\r\n\r\n</div>\n\n\nsm_nogap $\\therefore$ Money donated in 2 months\n\n<div class=\"aligned\">\r\n\r\n>>| |\r\n| ------------- | ---------- |\r\n| \\= 2 $\\times$ (16 $\\times$ 5) |\r\n| \\= 2 $\\times$ 80 |\n| \\= {{{correctAnswer}}} |\r\n\r\n</div>\r\n"}]},{"vars":[{"varval":"Mia does odd jobs around the house to earn pocket money.\r\n\r\nShe gets $4 per job.\r\n\r\nThe table below shows all the jobs she did in one week.\n\n<br>\n\nsm_img https://teacher.smartermaths.com.au/wp-content/uploads/2018/06/NAPX-I3-NC14.svg 200 indent3 vpad\n\n<br>If Mia does the same jobs for two weeks, how much money will she earn altogether?"},{"varval":"sm_nogap Total jobs in 1 week\n\n<div class=\"aligned\">\r\n\r\n>>| |\r\n| ------------- | ---------- |\r\n| \\= 6 + 4 + 3 |\r\n| \\= 13 |\r\n\r\n</div>\n\n\n\nsm_nogap $\\therefore$ Money earned in 2 weeks\n\n<div class=\"aligned\">\r\n\r\n>>| |\r\n| ------------- | ---------- |\r\n| \\= 2 $\\times$ 13 $\\times$ 4 |\r\n| \\= {{{correctAnswer}}} |\r\n\r\n</div>\r\n"}]}]

  281. <div class="sm_mode"> Bonnie earned $4 each time she fed her neighbour's horses when they were on holidays. In Summer, she fed the horses 7 times each week for 4 weeks in a row. How much money did Bonnie earn during this time? </div>

    [{"vars":null}]

  282. <div class="sm_mode"> Olivia had a bag of 37 lollipops. She gave an equal number of lollipops to 7 of her friends and kept the rest for herself. If her friends were given the highest number of lollipops possible, how many did Olivia keep for herself? </div>

    [{"vars":[]}]

  283. Algebra, NAPX-p122115v02 Algebra, NAPX-p122115v03

    <div class="sm_mode"> {{{question}}} </div>

    [{"vars":[{"varval":"Pablo caught 56 blowfish.\r\n\r\nHe lined them up in rows so there were 7 blowfish in each row.\r\n\r\nHow many rows are there?"},{"varval":"<div class=\"aligned\">\r\n\r\n| | |\r\n| ------------: | ---------- |\r\n| Number of rows | \\= 56 $\\div$ 7 |\r\n| | \\= {{{correctAnswer0}}} |\r\n\r\n</div>\r\n"}]},{"vars":[{"varval":"Vivian had 54 cricket balls.\r\n\r\nHis club had 6 cricket teams.\r\n\r\nHe gave the same number of balls to each team.\r\n\r\nHow many balls did each team receive?"},{"varval":"<div class=\"aligned\">\r\n\r\n| | |\r\n| ------------: | ---------- |\r\n| Number of rows | \\= 54 $\\div$ 6 |\r\n| | \\= {{{correctAnswer0}}} |\r\n\r\n</div>\r\n"}]},{"vars":[{"varval":"Calvin had 72 coloured pencils.\r\n\r\nHe lined them up so there are 8 pencils in each row.\r\n\r\nHow many rows are there?"},{"varval":"<div class=\"aligned\">\r\n\r\n| | |\r\n| ------------: | ---------- |\r\n| Number of rows | \\= 72 $\\div$ 8 |\r\n| | \\= {{{correctAnswer0}}} |\r\n\r\n</div>\r\n"}]}]

  284. Algebra, NAPX-K2-34

    <div class="sm_mode"> {{{question}}} </div>

    [{"vars":[{"varval":"Torvil has 18 scarves.\r\n\r\nDean has 5 less scarves than Torvil.\r\n\r\nWhich of these shows how to work out the total number of scarves they have altogether?"},{"varval":"Torvil's scarves = 18\r\n\nDean's scarves = 18 − 5\n\n<br>\n\n<div class=\"aligned\">\r\n\r\n| | |\r\n| ------------: | ---------- |\r\n| $\\therefore$ Total scarves | \\= 18 + 18 − 5 |\r\n| | \\= {{{correctAnswer}}} |\r\n\r\n</div>\r\n"}]},{"vars":[{"varval":"Bilbo has 14 rings.\r\n\r\nGandalf has 7 more rings than Bilbo.\r\n\r\nWhich of these shows how to work out the total number of rings they have altogether?"},{"varval":"Bilbo's rings = 14\r\n\nGandalf's rings = 14 + 7\n\n<br>\n\n<div class=\"aligned\">\r\n\r\n| | |\r\n| ------------: | ---------- |\r\n| $\\therefore$ Total rings | \\= 14 + 14 + 7 |\r\n| | \\= {{{correctAnswer}}} |\r\n\r\n</div>\r\n"}]},{"vars":[{"varval":"Bailey has saved $54.\r\n\r\nLogan has saved $8 more than Bailey.\r\n\r\nWhich of these shows how to work out the total savings they have altogether?"},{"varval":"Bailey's savings = $54\r\n\nLogan's savings = $54 + $8\n\n<br>\n\n<div class=\"aligned\">\r\n\r\n| | |\r\n| ------------: | ---------- |\r\n| $\\therefore$ Total savings | \\= 54 + 54 + 8 |\r\n| | \\= {{{correctAnswer}}} |\r\n\r\n</div>\r\n"}]},{"vars":[{"varval":"Rachel has knitted 15 soft toys for charity.\r\n\r\nCharli has knitted 4 soft toys less than Rachel.\r\n\r\nWhich of these shows how to work out the total number of soft toys they have knitted altogether?"},{"varval":"Rachel's soft toys = 15\r\n\nCharli's soft toys = 15 − 4\n\n<br>\n\n<div class=\"aligned\">\r\n\r\n| | |\r\n| ------------: | ---------- |\r\n| $\\therefore$ Total soft toys | \\= 15 + 15 − 4 |\r\n| | \\= {{{correctAnswer}}} |\r\n\r\n</div>\r\n"}]},{"vars":[{"varval":"Gayle collected 34 eggs from her chickens yesterday.\r\n\r\nToday Gayle collected 8 more eggs than yesterday.\r\n\r\nWhich of these shows how to work out the total number of eggs Gayle collected altogether?"},{"varval":"Yesterday's eggs = 34\r\n\nToday's eggs = 34 + 8\n\n<br>\n\n<div class=\"aligned\">\r\n\r\n| | |\r\n| ------------: | ---------- |\r\n| $\\therefore$ Total eggs | \\= 34 + 34 + 8 |\r\n| | \\= {{{correctAnswer}}} |\r\n\r\n</div>\r\n"}]},{"vars":[{"varval":"Samantha baked 90 cupcakes today.\r\n\r\nYesterday she baked 24 more cupcakes than today.\r\n\r\nWhich of these shows how to work out the total number of cupcakes Samantha has baked altogether?"},{"varval":"Today's cupcakes = 90\r\n\nYesterday's cupcakes = 90 + 24\n\n<br>\n\n<div class=\"aligned\">\r\n\r\n| | |\r\n| ------------: | ---------- |\r\n| $\\therefore$ Total cupcakes | \\= 90 + 90 + 24 |\r\n| | \\= {{{correctAnswer}}} |\r\n\r\n</div>"}]}]

  285. Algebra, NAPX-G2-27 SA

    <div class="sm_mode"> {{{question}}} </div>

    [{"vars":[{"varval":"Complete the number sentence"},{"varval":"<div class=\"aligned\">\r\n\r\n| | |\r\n| ------------: | ---------- |\n| 400 | \\= 80 $\\times$ <span class=\"sm_box\"> ` ` ` ` </span> |\r\n| | |\n| <span class=\"sm_box\"> ` ` ` ` </span> | \\= $\\dfrac{400}{80}$ |\n| | |\n| | \\= $\\dfrac{4 \\times 10 \\times 10}{2 \\times 4 \\times 10}$ |\r\n| | \\= 5 |\r\n\r\n</div>\r\n"}]},{"vars":[{"varval":"Complete the number sentence"},{"varval":"<div class=\"aligned\">\r\n\r\n| | |\r\n| ------------: | ---------- |\r\n| <span class=\"sm_box\"> ` ` ` ` </span> | \\= $\\dfrac{300}{60}$ |\n| | \\= $\\dfrac{5 \\times 60}{60}$ |\r\n| | \\= 5 |\r\n\r\n</div>\r\n"}]}]

  286. Algebra, NAPX-p124011v01 Algebra, NAPX-J2-30

    <div class="sm_mode"> {{{question}}} </div>

    [{"vars":[{"varval":"Seamus uses the number sentence &nbsp;7 × 14 = 98 &nbsp;to solve a problem.\r\n\r\nWhich of the following could be the problem?"},{"varval":"14 nerf guns $\\times$ $7 per gun = $98\r\n \r\n\r\n$\\therefore$ {{{correctAnswer}}}"}]},{"vars":[{"varval":"Lelei uses the number sentence &nbsp;6 × 42 = 252 &nbsp;to solve a problem.\r\n\r\nWhich of the following could be the problem?"},{"varval":"42 shirts × $6 per shirt = $252\n\n$\\therefore$ {{{correctAnswer}}}"}]},{"vars":[{"varval":"Dimitry uses the number sentence &nbsp;32 × 8 = 256 &nbsp;to solve a problem.\r\n\r\nWhich of the following could be the problem?"},{"varval":"8 ties × $32 per tie = $256\n\n$\\therefore$ {{{correctAnswer}}}"}]}]

  287. Algebra, NAPX-K2-20

    <div class="sm_mode"> {{{question}}} </div>

    [{"vars":[{"varval":"On Thursday, Sandra went to the doctor and was given 21 tablets.\r\n\r\nSandra was to take 5 tablets each day starting from Thursday.\r\n\r\nOn which day did she take the last tablet?"},{"varval":"<div class=\"aligned\">\r\n\r\n| | |\r\n| ------------- | ---------- |\r\n| Number of days | \\= 21 $\\div$ 5|\r\n| | \\= 4 remainder 1 |\r\n\r\n</div>\r\n\n<br>\n\n$\\Rarr$ Last tablet taken on the 5th day.\n\n$\\therefore$ {{{correctAnswer}}}"}]},{"vars":[{"varval":"On Tuesday, Dinesh went to the doctor and was given 28 tablets.\r\n\r\nDinesh was to take 6 tablets each day starting from Tuesday.\r\n\r\nOn which day did Dinesh take the last tablet?"},{"varval":"<div class=\"aligned\">\r\n\r\n| | |\r\n| ------------- | ---------- |\r\n| Number of days | \\= 28 $\\div$ 6|\r\n| | \\= 4 remainder 4 |\r\n\r\n</div>\r\n\n<br>\n\n$\\Rarr$ Last tablet taken on the 5th day.\n\n$\\therefore$ {{{correctAnswer}}}"}]}]

  288. <div class="sm_mode"> {{{question}}} </div>

    [{"vars":[{"varval":"sm_img https://teacher.smartermaths.com.au/wp-content/uploads/2017/04/naplan-Y3-2009-16mc.png 400 indent vpad\n\nJoe put half of the apples in his fridge.\r\n\r\nHow many apples did Joe put in his fridge?"},{"varval":"<div class=\"aligned\">\r\n\r\n| | |\r\n| ------------- | ---------- |\r\n| Total apples | \\= 3 $\\times$ 8 |\r\n| | \\= 24 |\r\n\r\n</div>\n\n\r\n<br>\n\n<div class=\"aligned\">\r\n\r\n| | |\r\n| ------------- | ---------- |\r\n| $\\Rarr$ Apples put in fridge | \\= $\\dfrac{1}{2} \\ \\times 24$ |\r\n| | \\= {{{correctAnswer0}}} |\r\n\r\n</div>\r"}]},{"vars":[{"varval":"Jonah's soccer team got oranges at half time.\r\n\r\nThe quarters of oranges were put in a container.\n\n<br>\n\nsm_img https://teacher.smartermaths.com.au/wp-content/uploads/2017/04/NAP-G1-15.png 350 indent vpad\n\n<br>How many whole oranges were cut into quarters and put in the container?"},{"varval":"There are 16 orange quarters in the container.\r\n\n$\\Rarr$ Each orange is cut into 4 quarters.\r\n\n$\\therefore$ 4 full oranges are needed."}]}]

  289. <div class="sm_mode"> {{{question}}} </div>

    [{"vars":[{"varval":"Emily walks for 17 minutes to get from her house to Diamond Station.\n\n\r\n\r\nShe then travels by train to Ruby Station.\n\n\r\n\r\nEmily then walks 21 minutes to the supermarket where she works.\n\nsm_img https://teacher.smartermaths.com.au/wp-content/uploads/2021/05/p169867v02-768x240.png 400 indent vpad\n\nIf Emily's shift starts at 6:30, what is the latest time she can leave home to arrive on time?"},{"varval":"Working backwards from arriving at the supermarket at 6:30\n\n\r\n6:30 less 21 minutes walking = 6:09\n\n\r\n1st train to arrive before 6:09 leaves Diamond Station at 5:55\n\n\r\n5:55 less than 17 minutes walking = 5:38\n\n\r\n$\\therefore$ Emily must leave her house by 5:38."}]},{"vars":[{"varval":"It takes Terry 32 minutes to walk from his home to Train Station A.\n\nHe then travels by train to Station C.\n\n\r\nTerry then walks to an amusement park which takes him 18 minutes.\n\n\n\nsm_img https://teacher.smartermaths.com.au/wp-content/uploads/2021/05/p169867v01-300x109.png 300 indent vpad\n\nWhat is the latest time that Terry can leave his home to arrive at the amusement park by 5:00?"},{"varval":"Working backwards from arriving at the amusement park at 5:00\n\n\r\n5:00 less 18 minutes walking = 4:42\n\n\r\n1st train to arrive before 4:42 leaves Station A at 3:40\n\n\r\n3:40 less than 32 minutes walking = 3:08\n\n\r\n$\\therefore$ Terry must leave his house by 3:08."}]}]

  290. I can't find anything wrong with this question. I have done 4 more variants and added staging_suejones in the categories.

    <div class="sm_mode"> {{{question}}} </div>

    [{"vars":[{"varval":"Valerie looked at the calendar on the 23rd of March.\n\n\r\n\r\nShe plans to go to the theatre on the 15th of April and circled this on the calendar below.\r\n\nsm_img https://teacher.smartermaths.com.au/wp-content/uploads/2019/01/NAPX-K2-40.svg 200 indent vpad\n\nWhat day of the week is the 23rd of March?"},{"varval":"Working backwards:\n\n\r\n1 April → Tuesday (from calendar)\n\n\r\n31 March → Monday (31 days in March)\n\n\r\n24 March → Monday \r\n \r\n\r\n$\\therefore$ 23 March is a {{{correctAnswer}}}"}]},{"vars":[{"varval":"Ben looked at the calendar on the 18th of March.\n\n\r\n\r\nHe plans to go to a midweek football game on the 15th of April and circled this on the calendar below.\n\nsm_img https://teacher.smartermaths.com.au/wp-content/uploads/2019/01/NAPX-K2-40.svg 200 indent vpad\n\nWhat day of the week is the 18th of March?"},{"varval":"Working backwards:\n\n\r\n1 April → Tuesday (from calendar)\n\n\r\n31 March → Monday (31 days in March)\n\n\r\n24 March → Monday \n\r\n \r\n\r\n17 March → Monday \n\n$\\therefore$ 18 March is a {{{correctAnswer}}}"}]},{"vars":[{"varval":"Margaret looked at the calendar on the 10th of March.\n\n\r\n\r\nShe has an appointment with the doctor on the 15th of April and circled this on the calendar below.\n\nsm_img https://teacher.smartermaths.com.au/wp-content/uploads/2019/01/NAPX-K2-40.svg 200 indent vpad\n\nWhat day of the week is the 10th of March?"},{"varval":"Working backwards:\n\n\r\n1 April → Tuesday (from calendar)\n\n\r\n31 March → Monday (31 days in March)\n\n\r\n24 March → Monday \n\r\n \r\n\r\n17 March → Monday \n\n$\\therefore$ 10 March is a {{{correctAnswer}}}"}]},{"vars":[{"varval":"Michael looked at the calendar on the 1st of March.\n\n\r\n\r\nHe has booked a flight to Queensland on the 15th of April and circled this on the calendar below.\n\nsm_img https://teacher.smartermaths.com.au/wp-content/uploads/2019/01/NAPX-K2-40.svg 200 indent vpad\n\nWhat day of the week is the 1st of March?"},{"varval":"Working backwards:\n\n\r\n1 April → Tuesday (from calendar)\n\n\r\n31 March → Monday (31 days in March)\n\n\r\n24 March → Monday \n\r\n17 March → Monday\n\n10 March → Monday\n\n3 March → Monday\n\n$\\therefore$ 1 March is a {{{correctAnswer}}}"}]},{"vars":[{"varval":"Chase looked at the calendar on the 7th of March.\n\n\r\n\r\nHe has an assessment task on the 15th of April and circled this on the calendar below.\n\nsm_img https://teacher.smartermaths.com.au/wp-content/uploads/2019/01/NAPX-K2-40.svg 200 indent vpad\n\nWhat day of the week is the 7th of March?"},{"varval":"Working backwards:\n\n\r\n1 April → Tuesday (from calendar)\n\n\r\n31 March → Monday (31 days in March)\n\n\r\n24 March → Monday \n\r\n \r\n\r\n17 March → Monday\n\n10 March → Monday\n\n$\\therefore$ 7 March is a {{{correctAnswer}}}"}]}]

  291. Can't find anything wrong with this question. I changed the worked solution to include a minus sign instead of saying "less than". I've added 4 more variants and added staging_suejones to categories.

    <div class="sm_mode"> {{{question}}} </div>

    [{"vars":[{"varval":"Dawn takes 4 minutes 15 seconds to swim 200 metres in the pool.\n\n\r\n\r\nMurray swims the same distance in 2 minutes 25 seconds.\n\n\r\n\r\nIf they start at the same time, how long after Murray finishes will Dawn finish?"},{"varval":"One strategy:\n\n<div class=\"aligned\">\n\n|||\n|-|-|\n|Extra time|= 4 min 15 sec $-$ 2 min 25 sec|\n||= 2 min 15 sec $-$ 25 sec|\n||= {{{correctAnswer}}}|\n\n</div>"}]},{"vars":[{"varval":"Jerry takes 10 minutes and 35 seconds to walk to the supermarket via the park.\n\n\r\n\r\nAnnette takes a different route and arrives in 7 minutes and 15 seconds.\n\n\r\n\r\nIf they start at the same time, how long after Annette arrives will Jerry arrive at the supermarket?"},{"varval":"<div class=\"aligned\">\n\n|||\n|-|-|\n|Extra time|= 10 min 35 sec $-$ 7 min 15 sec|\n||= 3 min 35 sec $-$ 15 sec|\n||= {{{correctAnswer}}}|\n\n</div>"}]},{"vars":[{"varval":"Geoffrey ran the 1500 metre race at the school carnival in 7 minutes and 25 seconds.\n\n\r\n\r\nHe hopes to run the same distance in 6 minutes and 40 seconds at the zone carnival next month.\n\n\r\n\r\nBy how much must he improve his time to achieve this goal?"},{"varval":"One strategy:\n\n<div class=\"aligned\">\n\n|||\n|-|-|\n|Time difference|= 7 min 25 sec $-$ 6 min 40 sec|\n||= 1 min 25 sec $-$ 40 sec|\n||= {{{correctAnswer}}}|\n\n</div>"}]},{"vars":[{"varval":"Georgia caught the \"all stops\" train from Sydney to Newcastle. The trip took 3 hours and 4 minutes.\n\n\r\n\r\nJulia chose to drive and the trip took 2 hours and 17 minutes.\n\n\r\n\r\nIf they start at the same time, how long after Julia arrives will Georgia arrive?"},{"varval":"One strategy:\n\n<div class=\"aligned\">\n\n|||\n|-|-|\n|Extra time|= 3 hours 4 min $-$ 2 hours 17 min|\n||= 1 hour 4 min $-$ 17 min|\n||= 64 min $-$ 17 min|\n||= {{{correctAnswer}}}|\n\n</div>"}]},{"vars":[{"varval":"Billy takes 13 minutes 5 seconds to ride his bike to soccer training.\n\n\r\n\r\nHis brother James catches the bus and the trip takes 5 minutes and 25 seconds.\n\n\r\n\r\nIf they leave home at the same time, how long after James arrives will Billy arrive?"},{"varval":"One strategy:\n\n<div class=\"aligned\">\n\n|||\n|-|-|\n|Extra time|= 13 min 5 sec $-$ 5 min 25 sec|\n||= 8 min 5 sec $-$ 25 sec|\n||= {{{correctAnswer}}}|\n\n</div>"}]}]

  292. <div class="sm_mode"> Grant takes 15 minutes to walk to the beach for board training. Last week he arrived at training at 4:05 pm, which was 25 minutes late. At what time would Grant leave home to arrive at board training on time? </div>

    [{"vars":null}]

  293. Can't find any errors in this question. Added 4 more variants.

    <div class="sm_mode"> {{{question}}} </div>

    [{"vars":[{"varval":"A father and his son attended a video game convention, arriving at 9:15 am.\n\n\r\n\r\nThey left the convention at 2:05 pm.\n\n\r\n\r\nHow long did they spend at the convention?"},{"varval":"9:15 am to 10:00 am → 45 minutes\n\n\r\n10:00 am to 2:00 pm → 4 hours\n\n\r\n2:00 pm to 2:05 pm → 5 minutes\n\n\r\n$\\therefore$ Time at convention = {{{correctAnswer}}}."}]},{"vars":[{"varval":"Julius caught the train from Wyong station at 7:03am.\n\n\r\n\r\nHe arrived at Town Hall station at 8:54am.\n\n\r\n\r\nHow long was Julius' train trip?"},{"varval":"One strategy:\n\n7:03 am to 8:00 am → 57 minutes\n\n\r\n8:00 am to 8:54 pm → 54 minutes\n\n\r\nTotal time = 111 minutes\n\n\r\n$\\therefore$ Length of train trip = {{{correctAnswer}}}."}]},{"vars":[{"varval":"Jacquie watched a movie that started at 10:35 am and finished at 1:15 pm.\n\n\r\n\r\n\r\n\r\nHow long was the movie?"},{"varval":"10:35 am to 11:00 am → 25 minutes\n\n\r\n11:00 am to 1:00 pm → 2 hours\n\n\r\n1:00 pm to 1:15 pm → 15 minutes\n\n\r\n$\\therefore$ Time at convention = {{{correctAnswer}}}."}]},{"vars":[{"varval":"On Monday Bernie logged on to an online course at 10:40 am and logged off at 3:15 pm.\n\n\r\n\r\n\r\n\r\nFor how long was he logged on to the online course on Monday?"},{"varval":"One strategy: \n\n10:40 am to 11:00 am → 20 minutes\n\n\r\n11:00 am to 3:00 pm → 4 hours\n\n\r\n3:00 pm to 3:15 pm → 15 minutes\n\n\r\n$\\therefore$ Time logged on = {{{correctAnswer}}}."}]},{"vars":[{"varval":"James started playing Minecraft at 11:25 am and stopped for lunch at 1:45 pm.\n\nFor how long was James playing Minecraft before lunch?"},{"varval":"11:25 am to 12:00 am → 35 minutes\n\n\r\n12:00 am to 1:00 pm → 1 hours\n\n\r\n1:00 pm to 1:45 pm → 45 minutes\n\n\r\n$\\therefore$ Time playing Minecraft = 1 hour 80 minutes = {{{correctAnswer}}}."}]}]

  294. Measurement, NAPX-G2-17 Can't find any errors. Changed Lleyton to Serena in Var1 as they were attending the Women's tennis final.

    <div class="sm_mode"> {{{question}}} </div>

    [{"vars":[{"varval":"sm_img https://teacher.smartermaths.com.au/wp-content/uploads/2019/01/NAPX-G2-17.svg 200 indent vpad\n\nDavid has a dentist appointment on 4 June.\n\n\r\n\r\n5 days before the appointment, David's tooth fell out.\n\n\r\n\r\nOn which day of the week did his tooth fall out?"},{"varval":"Working backwards:\n\n\r\n1 day before 4 June → Sunday\n\n\r\n2 days before 4 June → Monday\n\n\r\n>>⋮\n\n\r\n5 days before 4 June → Wednesday"}]},{"vars":[{"varval":"sm_img https://teacher.smartermaths.com.au/wp-content/uploads/2019/01/NAPX-G2-17.svg 200 indent vpad\n\nThe French Open women's tennis final was held on 2 June.\n\n\r\n\r\n6 days before the final, Serena arrived in Paris.\n\n\r\n\r\nOn what day of the week did Serena arrive in Paris?"},{"varval":"Working backwards:\n\n\r\n1 day before 2 June → Friday\n\n\r\n2 days before 2 June → Thursday\n\n\r\n>>⋮\n\n\r\n6 days before 2 June → Sunday"}]}]

  295. <div class="sm_mode"> {{{question}}} </div>

    [{"vars":[{"varval":"What number makes this number sentence correct?"},{"varval":" <span class=\"sm_box\"> ` ` ` ` </span> &nbsp;− 47 + 29 = 51\n\n<div class=\"aligned\">\r\n\r\n| | |\r\n| ------------: | ---------- |\r\n| <span class=\"sm_box\"> ` ` ` ` </span> | \\= 51 + 47 − 29 |\n| | \\= 98 − 29 |\r\n| | \\= {{{correctAnswer0}}} |\r\n\r\n</div>\r"}]},{"vars":[{"varval":"What number makes this number sentence correct?"},{"varval":" <span class=\"sm_box\"> ` ` ` ` </span> &nbsp;− 34 + 68 = 57\n\n<div class=\"aligned\">\r\n\r\n| | |\r\n| ------------: | ---------- |\r\n| <span class=\"sm_box\"> ` ` ` ` </span> | \\= 57 + 34 − 68 |\n| | \\= 91 − 68 |\r\n| | \\= {{{correctAnswer0}}} |\r\n\r\n</div>\r"}]},{"vars":[{"varval":"What number makes this number sentence correct?"},{"varval":" <span class=\"sm_box\"> ` ` ` ` </span> &nbsp;− 29 + 72 = 44\n\n<div class=\"aligned\">\r\n\r\n| | |\r\n| ------------: | ---------- |\r\n| <span class=\"sm_box\"> ` ` ` ` </span> | \\= 44 + 29 − 72 |\n| | \\= 73 − 72 |\r\n| | \\= {{{correctAnswer0}}} |\r\n\r\n</div>\r"}]},{"vars":[{"varval":"What number makes this number sentence correct?"},{"varval":" <span class=\"sm_box\"> ` ` ` ` </span> &nbsp;− 25 + 47 = 83\n\n<div class=\"aligned\">\r\n\r\n| | |\r\n| ------------: | ---------- |\r\n| <span class=\"sm_box\"> ` ` ` ` </span> | \\= 83 + 25 − 47 |\n| | \\= 108 − 47 |\r\n| | \\= {{{correctAnswer0}}} |\r\n\r\n</div>\r"}]}]

  296. <div class="sm_mode"> {{{question}}} </div>

    [{"vars":[{"varval":"A new hammock costs $335.\n\nsm_img https://teacher.smartermaths.com.au/wp-content/uploads/2018/12/NAP-J2-37-v1.svg 350 indent vpad\n\nKeanu has saved a total of $128.\r\n\r\nWhich number sentence can be solved to find the amount of money Keanu still needs to buy the hammock?"},{"varval":"Savings + money needed = total cost\n\n{{{correctAnswer}}}\n"}]}]

  297. <div class="sm_mode"> Ambrosia wanted to measure the height of a soda can by measuring it against a section of her measuring tape. sm_img https://teacher.smartermaths.com.au/wp-content/uploads/2018/12/NAPX-I2-33.svg 200 indent3 vpad What is the height of the soda can? </div>

    [{"vars":null}]

  298. Algebra, NAPX-p123181v02 Algebra, NAPX-p123181v01

    <div class="sm_mode"> {{{question}}} </div>

    [{"vars":[{"varval":"Mike enters a motorcycle race.\r\n\r\nOn Day 1, he rides 620 kilometres.\r\n\r\nOn Day 2, he rides 483 kilometres.\r\n\r\nWhich number sentence shows how many more kilometres he rides on Day 1 than Day 2?"},{"varval":"{{{correctAnswer}}}"}]},{"vars":[{"varval":"Sean enters a motorcycle race.\r\n\r\nOn Day 1, he rides 530 kilometres.\r\n\r\nOn Day 2, he rides 286 kilometres.\r\n\r\nWhich number sentence shows how many more kilometres he rides on Day 1 than Day 2?"},{"varval":"{{{correctAnswer}}}"}]},{"vars":[{"varval":"At breakfast, a coffee shop sold 290 cups of coffee.\r\n\r\nAt lunchtime on the same day, the shop sold 177 cups of coffee.\r\n\r\nWhich number sentence shows how many more cups of coffee were sold at breakfast than at lunchtime?"},{"varval":"{{{correctAnswer}}}"}]},{"vars":[{"varval":"A restaurant has an all day breakfast menu and used 83 eggs at breakfast.\r\n\r\nAt lunchtime on the same day, the restaurant used 41 eggs.\r\n\r\nWhich number sentence shows how many more eggs were used at breakfast than at lunchtime?"},{"varval":"{{{correctAnswer}}}"}]},{"vars":[{"varval":"At lunchtime, a pizza shop sold 147 pizzas.\r\n\r\nAt dinnertime on the same day, the pizza shop sold 233 pizzas.\r\n\r\nWhich number sentence shows how many more pizzas were sold at dinnertime than at lunchtime?"},{"varval":"{{{correctAnswer}}}"}]},{"vars":[{"varval":"Ferenc enters a charity walkathon.\r\n\r\nDuring Week 1, he walks 294 kilometres.\r\n\r\nDuring Week 2, he walks 357 kilometres.\r\n\r\nWhich number sentence shows how many more kilometres he walks during Week 2 than Week 1?"},{"varval":"\t\r\n{{{correctAnswer}}}"}]}]

  299. Var: Algebra, NAPX-p72866v01

    <div class="sm_mode"> {{{question}}} </div>

    [{"vars":[{"varval":"The first number in a pattern is 5.85\r\n\r\nEach number in the pattern is formed by subtracting 2.25 from the previous number.\r\n\r\nWhat is the 3rd number in the pattern?"},{"varval":"2nd number: 5.85 − 2.25 = 3.60\r\n\n3rd number: 3.60 − 2.25 = 1.35"}]},{"vars":[{"varval":"The first number in the pattern is 4.1\r\n\r\nEach number in the pattern is formed by adding 1.25 to the previous number\r.\n\r\nWhat is the 4th number in the pattern?"},{"varval":"2nd number: &nbsp;4.1 + 1.25 = 5.35\r\n\n3rd number: &nbsp;5.35 + 1.25 = 6.6\r\n\n4th number: &nbsp;6.6 + 1.25 = 7.85"}]}]

  300. Algebra, NAPX-p116707v02 Algebra, NAPX-p116707v01

    <div class="sm_mode"> {{{question}}} </div>

    [{"vars":[{"varval":"Louise donated money to two charities\r.\n\r\nShe donated $125 to the first charity and donated $45 to the second charity.\r\n\r\nWhich number sentence could be used to find total amount of money Louise donated?"},{"varval":"sm_nogap Total amount donated to charity\n\n<div class=\"aligned\">\r\n\r\n>| | |\r\n| ------------- | ---------- |\r\n| | \\= 125 + 45 |\n| | \\= 130 − 5 + 50 − 5 |\n| | \\= 130 + 50 − 10 |\r\n| | \\= 170 |\r\n\r\n</div>\r\n"}]},{"vars":[{"varval":"Shane played two basketball games.\r\n\r\nShe scored 35 points in the first game.\r\n\r\nIn the second game she scored 25 points.\r\n\r\nWhich number sentence could be used to find the total number of points Shane scored in the first two games?"},{"varval":"sm_nogap Total points in first 2 games\n\n<div class=\"aligned\">\r\n\r\n>| | |\r\n| ------------- | ---------- |\r\n| | \\= 35 + 25 |\n| | \\= 30 + 5 + 20 + 5 |\n| | \\= 30 + 20 + 10 |\r\n| | \\= 60 |\r\n\r\n</div>\r\n"}]},{"vars":[{"varval":"Leonardo was his soccer team's top goal scorer over two seasons.\r\n\r\nIn season 1, he scored 55 goals.\r\n\r\nIn season 2, he scored 65 goals.\r\n\r\nWhich number sentence could be used to find the total number of goals Leonardo scored over both seasons?"},{"varval":"sm_nogap Total goals over 2 seasons\n\n<div class=\"aligned\">\r\n\r\n>| | |\r\n| ------------- | ---------- |\r\n| | \\= 55 + 65 |\n| | \\= 50 + 5 + 60 + 5 |\n| | \\= 50 + 60 + 10 |\r\n| | \\= 120 |\r\n\r\n</div>\r\n"}]}]

  301. <div class="sm_mode"> The runs scored by Josh and Patrick in 4 games of cricket are recorded in the table below. <br> sm_img https://teacher.smartermaths.com.au/wp-content/uploads/2019/01/NAPX-G2-05v2.svg 210 indent vpad <br>In which game did Patrick score 5 more runs than Josh? </div>

    [{"vars":null}]

  302. <div class="sm_mode"> George, Martha, Nick and Honey are in a band. The years they were born are recorded in the table below. sm_img https://teacher.smartermaths.com.au/wp-content/uploads/2018/10/NAPX-H2-13-v1.svg 550 indent vpad Who is the second youngest band member? </div>

    [{"vars":null}]

  303. Algebra, NAPX-p116624v04 Algebra, NAPX-p116624v03 Algebra, NAPX-I2-05 SA

    <div class="sm_mode"> {{{question}}} </div>

    [{"vars":[{"varval":"The scales below are evenly balanced.\n\n<br>\n\nsm_img https://teacher.smartermaths.com.au/wp-content/uploads//2021/03/NAPX5-TLA-4-v2_.svg 350 indent vpad\n\n<br>What is the mass of the small cube?"},{"varval":"<div class=\"aligned\">\r\n\r\n| | |\r\n| ------------- | ---------- |\r\n| Mass of small cube | \\= 44 − 18 |\r\n| | \\= {{{correctAnswer0}}} {{{suffix0}}}|\r\n\r\n</div>\r\n"}]},{"vars":[{"varval":"The scales below are evenly balanced.\n\n<br>\n\nsm_img https://teacher.smartermaths.com.au/wp-content/uploads//2021/03/NAPX5-TLA-4-v1_.svg 350 indent vpad\n\n<br>What is the mass of the small cube?\n\n"},{"varval":"<div class=\"aligned\">\r\n\r\n| | |\r\n| ------------- | ---------- |\r\n| Mass of small cube | \\= 32 − 19 |\r\n| | \\= {{{correctAnswer0}}} {{{suffix0}}} |\r\n\r\n</div>\r\n"}]},{"vars":[{"varval":"The scale pictured below is balanced.\n\n<br>\n\nsm_img https://teacher.smartermaths.com.au/wp-content/uploads/2021/04/p116624v03-300x201.png 350 indent vpad\n\n<br>What is the weight of the cube?"},{"varval":"Since the scale is balanced, weight on both sides = 44 grams\n\n<div class=\"aligned\">\r\n\r\n| | |\r\n| ------------- | ---------- |\r\n| Weight of cube | \\= 44 − 13 |\r\n| | \\= 31 grams |\r\n\r\n</div>\r\n"}]},{"vars":[{"varval":"The scales below are evenly balanced.\n\n<br>\n\nsm_img https://teacher.smartermaths.com.au/wp-content/uploads/2018/10/NAPX-I2-05-v1.svg 350 indent vpad\n\n<br>What is the weight of the small cube?"},{"varval":"<div class=\"aligned\">\r\n\r\n| | |\r\n| ------------- | ---------- |\r\n| Weight of small cube | \\= 26 − 15 |\r\n| | \\= {{{correctAnswer0}}} {{{suffix0}}} |\r\n\r\n</div>"}]}]

  304. var: Algebra, NAPX-p121288v02 Algebra, NAPX-p121288v03

    <div class="sm_mode"> {{{question}}} </div>

    [{"vars":[{"varval":"Write a number in the box to make this number sentence true."},{"varval":"<div class=\"aligned\">\r\n\r\n| | |\r\n| ------------: | ---------- |\r\n| 7 + 6 | \\= 10 + <span class=\"sm_box\"> ` ` </span> |\n| | |\n| 13 | \\= 10 + <span class=\"sm_box\"> ` ` </span> |\r\n| | |\n| <span class=\"sm_box\"> ` ` </span> | \\= {{{correctAnswer0}}} |\r\n\r\n</div>\r\n"}]},{"vars":[{"varval":"Write a number in the box to make this number sentence true."},{"varval":"<div class=\"aligned\">\r\n\r\n| | |\r\n| ------------: | ---------- |\r\n| 8 + 8 | \\= 10 + <span class=\"sm_box\"> ` ` </span> |\n| | |\n| 16 | \\= 10 + <span class=\"sm_box\"> ` ` </span> |\r\n| | |\n| <span class=\"sm_box\"> ` ` </span> | \\= {{{correctAnswer0}}} |\r\n\r\n</div>\r\n"}]},{"vars":[{"varval":"Write a number in the box to make this number sentence true."},{"varval":"<div class=\"aligned\">\r\n\r\n| | |\r\n| ------------: | ---------- |\r\n| 9 + 7 | \\= 11 + <span class=\"sm_box\"> ` ` </span> |\n| | |\n| 16 | \\= 11 + <span class=\"sm_box\"> ` ` </span> |\r\n| | |\n| <span class=\"sm_box\"> ` ` </span> | \\= {{{correctAnswer0}}} |\r\n\r\n</div>\r\n"}]},{"vars":[{"varval":"Write a number in the box to make this number sentence true."},{"varval":"<div class=\"aligned\">\r\n\r\n| | |\r\n| ------------: | ---------- |\r\n| 8 + 5 | \\= 10 + <span class=\"sm_box\"> ` ` </span> |\n| | |\n| 13 | \\= 10 + <span class=\"sm_box\"> ` ` </span> |\r\n| | |\n| <span class=\"sm_box\"> ` ` </span> | \\= {{{correctAnswer0}}} |\r\n\r\n</div>\r"}]},{"vars":[{"varval":"Write a number in the box to make this number sentence true."},{"varval":"<div class=\"aligned\">\r\n\r\n| | |\r\n| ------------: | ---------- |\r\n| 8 + 7 | \\= 10 + <span class=\"sm_box\"> ` ` </span> |\n| | |\n| 15 | \\= 10 + <span class=\"sm_box\"> ` ` </span> |\r\n| | |\n| <span class=\"sm_box\"> ` ` </span> | \\= {{{correctAnswer0}}} |\r\n\r\n</div>\r"}]}]

  305. var : Algebra, NAPX-p116683v02

    <div class="sm_mode"> {{{question}}} </div>

    [{"vars":[{"varval":"Kylie planted 175 lemon trees in her orchard.\r\n\r\n23 lemon trees died in the first month.\r\n\r\nHow many lemon trees survived past the first month?"},{"varval":"<div class=\"aligned\">\r\n\r\n| | |\r\n| ------------- | ---------- |\r\n| Lemon trees left\t | \\= 175 − 23 |\r\n| | \\= {{{correctAnswer0}}} |\r\n\r\n</div>\r\n"}]},{"vars":[{"varval":"Rodney planted 227 small tress on his property.\r\n\r\nSome of the small trees died so there was only 203 trees left.\r\n\r\nHow many of the small trees died?"},{"varval":"<div class=\"aligned\">\r\n\r\n| | |\r\n| ------------- | ---------- |\r\n| Trees that died | \\= 227 − 203 |\r\n| | \\= 24 |\r\n\r\n</div>\r\n"}]}]

  306. Algebra, NAPX-K2-05 SA

    <div class="sm_mode"> {{{question}}} </div>

    [{"vars":[{"varval":"In a bowls competition, a player won 12 games and lost the other 9 games.\r\n\r\nFinish the subtraction sentence below to show the number of games he played."},{"varval":"<div class=\"aligned\">\r\n\r\n| | |\r\n| ------------- | ---------- |\r\n| <span class=\"sm_box\"> ` ` ` ` </span> | \\= 9 + 12 |\r\n| | \\= {{{correctAnswer0}}} |\r\n\r\n</div>\r\n"}]},{"vars":[{"varval":"In a squash competition Michelle won 9 games and lost the other 7 games.\r\n\r\nFinish the subtraction sentence below to show the number of games Michelle played."},{"varval":"<div class=\"aligned\">\r\n\r\n| | |\r\n| ------------- | ---------- |\r\n| <span class=\"sm_box\"> ` ` ` ` </span> | \\= 7 + 9 |\r\n| | \\= {{{correctAnswer0}}} |\r\n\r\n</div>\r\n"}]},{"vars":[{"varval":"In a basketball competition, the Red Devils won 17 games and lost the other 14 games.\r\n\r\nFinish the subtraction sentence below to show the number of games the Red Devils played."},{"varval":"<div class=\"aligned\">\r\n\r\n| | |\r\n| ------------- | ---------- |\r\n| <span class=\"sm_box\"> ` ` ` ` </span> | \\= 17 + 14 |\r\n| | \\= {{{correctAnswer0}}} |\r\n\r\n</div>\r"}]},{"vars":[{"varval":"In a rugby league competition, the Bulldogs won 7 games and lost the other 19 games.\r\n\r\nFinish the subtraction sentence below to show the number of games the Bulldogs played."},{"varval":"<div class=\"aligned\">\r\n\r\n| | |\r\n| ------------- | ---------- |\r\n| <span class=\"sm_box\"> ` ` ` ` </span> | \\= 7 + 19 |\r\n| | \\= {{{correctAnswer0}}} |\r\n\r\n</div>\r"}]},{"vars":[{"varval":"In a netball league, the Swifts won 15 games and lost the other 8 games.\r\n\r\nFinish the subtraction sentence below to show the number of games the Swifts played."},{"varval":"<div class=\"aligned\">\r\n\r\n| | |\r\n| ------------- | ---------- |\r\n| <span class=\"sm_box\"> ` ` ` ` </span> | \\= 15 + 8 |\r\n| | \\= {{{correctAnswer0}}} |\r\n\r\n</div>\r"}]}]

  307. <div class="sm_mode"> Geoff has $88. <br> sm_img https://teacher.smartermaths.com.au/wp-content/uploads/2019/01/NAPX-G2-13v2.svg 170 indent3 vpad <br>How much more money does he need to buy the tennis racquet? </div>

    [{"vars":null}]

  308. Can't find any errors.

    <div class="sm_mode"> Ralph has a rectangular puppy playground in his backyard with an area of 10 m$^2$. He decides to make it bigger and increases each side to 3 times its current length. What is the new area of the puppy playground? </div>

    [{"vars":null}]

  309. <div class="sm_mode"> Mario sketches an irregular plot of land, pictured below. sm_img https://teacher.smartermaths.com.au/wp-content/uploads/2021/05/RAPH10_80.svg 240 indent vpad What is the perimeter of this polygon in metres? </div>

    [{"vars":null}]

  310. Error in var1 - Divide shape into 3 parts - should be 2 parts.

    <div class="sm_mode"> {{{question}}} </div>

    [{"vars":[{"varval":"Timothy sketched a plot of land with the following measurements in metres.\n\nsm_img https://teacher.smartermaths.com.au/wp-content/uploads/2021/05/46.svg 240 indent vpad\n\nWhat is the area of the land in square metres?"},{"varval":"Divide the shape into 3 parts:\n\nsm_img https://teacher.smartermaths.com.au/wp-content/uploads/2021/05/46s.svg 300 indent vpad\n\n<div class=\"aligned\">\r\n\n|||\n|-|-|\n|Total Area|= Area 1 + Area 2 + Area 3|\n||= (15 × 12) + (23 × 10) + $\\bigg( \\dfrac{1}{2} \\times 11 \\times 14 \\bigg)$|\n||= 180 + 230 + 77|\n||= 487 m$^2$|\n\n</div>"}]},{"vars":[{"varval":"Hana sketches a shape with the following measurements in centimetres.\n\nsm_img https://teacher.smartermaths.com.au/wp-content/uploads/2021/05/45.svg 260 indent vpad\n\nWhat is the area of the shape sketched by Hana?"},{"varval":"Divide the shape into 2 parts:\n\nsm_img https://teacher.smartermaths.com.au/wp-content/uploads/2021/05/45s.svg 260 indent vpad\n\n<div class=\"aligned\">\r\n\n|||\n|-|-|\n|Total Area|= Area of triangle + Area of rectangle|\n||= $\\bigg( \\dfrac{1}{2} \\times 4 \\times 9 \\bigg) + (8 \\times 9)$|\n||= 18 + 72|\n||= 90 m$^2$|\n\n</div>"}]}]

  311. <div class="sm_mode"> {{{question}}} </div>

    [{"vars":[{"varval":"Barney's blocks have two square faces.\n\n<br>\n\nsm_img https://teacher.smartermaths.com.au/wp-content/uploads/2019/01/NAPX-G2-35i.svg 80 indent3 vpad\n\n<br>He builds the model below using the blocks.\n\n<br>\n\nsm_img https://teacher.smartermaths.com.au/wp-content/uploads/2019/01/NAPX-G2-35ii.svg 230 indent3 vpad\n\n<br>What is the height of the model in centimetres?"},{"varval":"sm_nogap Side length of the square face\n\n<div class=\"aligned\">\n\n>>||\n|-|\n|= 30 ÷ 6|\n|= 5 cm|\n\n</div>\n\n<br>\n\n<div class=\"aligned\">\n\n|||\n|-|-|\n|$\\therefore$ Height|= 4 × 5|\n||= {{{correctAnswer0}}} {{{suffix0}}} |\n\n</div>\n"}]},{"vars":[{"varval":"Bill's blocks have two square faces.\n\n<br>\n\nsm_img https://teacher.smartermaths.com.au/wp-content/uploads/2022/08/Measurement_NAPX-G2-35-SA_v1q.svg 120 indent3 vpad\n\n<br>He builds the model below using the blocks.\n\n<br>\n\nsm_img https://teacher.smartermaths.com.au/wp-content/uploads/2022/08/Measurement_NAPX-G2-35-SA_v1qb.svg 300 indent3 vpad\n\n<br>What is the height of the model in centimetres?"},{"varval":"sm_nogap Side length of the square faces\n\n<div class=\"aligned\">\n\n>>||\n|-|\n|= 56 ÷ 8|\n|= 7 cm|\n\n</div>\n\n<br>\n\n<div class=\"aligned\">\n\n|||\n|-|-|\n|$\\therefore$ Height|= 7 × 5|\n||= {{{correctAnswer0}}} {{{suffix0}}} |\n\n</div>\n"}]},{"vars":[{"varval":"Betty's blocks have two square faces.\n\n<br>\n\nsm_img https://teacher.smartermaths.com.au/wp-content/uploads/2022/08/Measurement_NAPX-G2-35-SA_v2q.svg 100 indent3 vpad\n\n<br>She builds the model below using the blocks.\n\n<br>\n\nsm_img https://teacher.smartermaths.com.au/wp-content/uploads/2022/08/Measurement_NAPX-G2-35-SA_v2qb.svg 210 indent3 vpad\n\n<br>What is the height of the model in centimetres?"},{"varval":"sm_nogap Side length of the square faces\n\n<div class=\"aligned\">\n\n>>||\n|-|\n|= 72 ÷ 6|\n|= 12 cm|\n\n</div>\n\n<br>\n\n<div class=\"aligned\">\n\n|||\n|-|-|\n|$\\therefore$ Height|= 12 × 6|\n||= {{{correctAnswer0}}} {{{suffix0}}} |\n\n</div>\n"}]},{"vars":[{"varval":"Damo's blocks have two square faces.\n\n\nsm_img https://teacher.smartermaths.com.au/wp-content/uploads/2022/08/Measurement_NAPX-G2-35-SA_v3q.svg 100 indent3 vpad\n\nHe builds the model below using the blocks.\n\n\nsm_img https://teacher.smartermaths.com.au/wp-content/uploads/2022/08/Measurement_NAPX-G2-35-SA_v3qb.svg 230 indent3 vpad\n\nWhat is the height of the model in centimetres?"},{"varval":"sm_nogap Side length of the square faces\n\n<div class=\"aligned\">\n\n>>||\n|-|\n|= 78 ÷ 6|\n|= 13 cm|\n\n</div>\n\n<br>\n\n<div class=\"aligned\">\n\n|||\n|-|-|\n|$\\therefore$ Height|= 13 × 5|\n||= {{{correctAnswer0}}} {{{suffix0}}} |\n\n</div>\n"}]},{"vars":[{"varval":"Veronica's blocks have two square faces.\n\n\nsm_img https://teacher.smartermaths.com.au/wp-content/uploads/2022/08/Measurement_NAPX-G2-35-SA_v4q.svg 110 indent3 vpad\n\nShe builds the model below using the blocks.\n\n\nsm_img https://teacher.smartermaths.com.au/wp-content/uploads/2022/08/Measurement_NAPX-G2-35-SA_v4qa.svg 280 indent3 vpad\n\nWhat is the height of the model in millimetres?"},{"varval":"sm_nogap Side length of the square faces\n\n<div class=\"aligned\">\n\n>>||\n|-|\n|= 272 ÷ 8|\n|= 34 mm|\n\n</div>\n\n<br>\n\n<div class=\"aligned\">\n\n|||\n|-|-|\n|$\\therefore$ Height|= 34 × 7|\n||= {{{correctAnswer0}}} {{{suffix0}}} |\n\n</div>\n"}]},{"vars":[{"varval":"Callie's blocks have two square faces.\n\n\nsm_img https://teacher.smartermaths.com.au/wp-content/uploads/2022/08/Measurement_NAPX-G2-35-SA_v5q.svg 100 indent3 vpad\n\nShe builds the model below using the blocks.\n\n\nsm_img https://teacher.smartermaths.com.au/wp-content/uploads/2022/08/Measurement_NAPX-G2-35-SA_v5qb.svg 370 indent3 vpad\n\nWhat is the height of the model in millimetres?"},{"varval":"sm_nogap Side length of the square faces\n\n<div class=\"aligned\">\n\n>>||\n|-|\n|= 204 ÷ 12|\n|= 17 mm|\n\n</div>\n\n<br>\n\n<div class=\"aligned\">\n\n|||\n|-|-|\n|$\\therefore$ Height|= 17 × 5|\n||= {{{correctAnswer0}}} {{{suffix0}}} |\n\n</div>"}]}]

  312. Measurement, NAPX-p167302v01

    <div class="sm_mode"> {{{question}}} </div>

    [{"vars":[{"varval":"The image below is a plan of a commercial garden.\n\nsm_img https://teacher.smartermaths.com.au/wp-content/uploads/2021/04/Math-Job-Q44.svg 340 indent vpad\n\nThe total planted area is 48 square metres.\n\n\r\n\r\nWhat is the total unplanted area?"},{"varval":"6 planted areas = 48 m$^2$\n\n\r\n1 hexagon grid = 48 ÷ 6 = 8 m$^2$\n\n\r\nNumber of unplanted areas: 8\n\n<div class=\"aligned\">\n\n|||\n|-|-|\n|$\\therefore$ Total unplanted area|= 8 × 8|\n||= 64 m$^2$|\n\n</div>"}]},{"vars":[{"varval":"The image below shows a layout of a village.\n\nsm_img https://teacher.smartermaths.com.au/wp-content/uploads/2021/04/Math-Job-Q43.svg 400 indent vpad\n\nThe total area of the occupied lots is 60 m$^2$.\n\n\r\n\r\nWhat is the total area of the unoccupied lots in the village?"},{"varval":"12 occupied lots = 60 m$^2$\n\n\r\n1 lot = 60 ÷ 12 = 5 m$^2$\n\n\r\nNumber of unoccupied lots = 10\n\n<div class=\"aligned\">\n\n|||\n|-|-|\n|$\\therefore$ Area of unoccupied lots|= 10 × 5|\n||= 50 m$^2$|\n\n</div>"}]}]

  313. Words left out of first sentence in question.

    <div class="sm_mode"> Brock decided to cut out a small rectangle from a piece of rectangular paper. sm_img https://teacher.smartermaths.com.au/wp-content/uploads/2021/05/p169727v02-296x300.png 200 indent vpad The rectangle cut out has a length of 60 mm and a height of 40 mm. Which of the following expressions gives the area of the paper that was left after Brock cut out the smaller rectangle? </div>

    [{"vars":null}]

  314. Error in worked solution. 15 + 5 instead of 15 + 15.

    <div class="sm_mode"> The perimeter of a rectangular box is 130 m. sm_img https://teacher.smartermaths.com.au/wp-content/uploads/2021/04/raph10-q15.svg 300 indent vpad What is the value of the width $x$? </div>

    [{"vars":null}]

  315. Measurement, NAPX-p167451v01 Error in worked solution. 82m should be 84m. Changed to {{{correctAnswer}}} to avoid errors.

    <div class="sm_mode"> {{{question}}} </div>

    [{"vars":[{"varval":"A triangular lot is for sale.\n\nsm_img https://teacher.smartermaths.com.au/wp-content/uploads/2021/04/RAPH9-20.svg 240 indent vpad\n\nWhat is the perimeter of the lot?"},{"varval":"<div class=\"aligned\">\n\n|||\n|-|-|\n|Perimeter|= 12 + 35 + 37|\n||= {{{correctAnswer}}} |\n\n</div>"}]},{"vars":[{"varval":"Dexter owns a backyard in the shape of a trapezoid.\n\nsm_img https://teacher.smartermaths.com.au/wp-content/uploads/2021/04/RAPH9-19.svg 320 indent vpad\n\nWhat is the perimeter of the backyard owned by Dexter?"},{"varval":"<div class=\"aligned\">\n\n|||\n|-|-|\n|Perimeter|= (2 × 68) + 212 + 189|\n||= 136 + 212 + 189|\n||= {{{correctAnswer}}}|\n\n</div>"}]}]

  316. Added worked solution as there was only a bald answer of 98 m

    <div class="sm_mode"> {{{question}}} </div>

    [{"vars":[{"varval":"Derrick has a backyard in the shape of a rectangle.\n\nsm_img https://teacher.smartermaths.com.au/wp-content/uploads/2021/05/RAPH10-61.svg 300 indent vpad\n\nThe longer side is &nbsp;$1 \\dfrac{1}{3}$&nbsp; times longer than the shorter side.\r\n\r\nWhat is the perimeter of Derick’s backyard?"},{"varval":"<div class=\"aligned\">\n\n| | |\n| ---- | --------------------------------------- |\n| Perimeter | \\= 2 $\\times$ 21 + $\\dfrac{4}{3}$ $\\times$ 2 $\\times$ 21 |\n| | \\= 42 + 56 |\n| | \\= {{{correctAnswer}}} |\n\n</div>"}]}]

  317. Added area calculation in worked solution as only image of correct triangle included.

    <div class="sm_mode"> {{{question}}} </div>

    [{"vars":[{"varval":"Four triangular shaped playgrounds are shown below.\n\n\r\n\r\nWhich of these play grounds has the least surface area?\n"},{"varval":"<div class=\"aligned\">\n\n|||\n|-|-|\n| Area| = $\\dfrac{1}{2} \\times 34 \\times 15$ |\n||= 255 m$^2$|\n\n<br>\n\n</div>\n\n{{{correctAnswer}}}"}]}]

  318. Worked solution incorrect. Stated area of triangle instead of rectangle. Also error in final line of solution corrected.

    <div class="sm_mode"> Which shape has an area of exactly 12 square units? sm_img https://teacher.smartermaths.com.au/wp-content/uploads/2021/04/Math-Job-Q2.png 300 indent vpad </div>

    [{"vars":null}]

  319. Added "painted grey" to the question to eliminate those that may be painted white. Moved correct answer to sit below the calculation. Couldn't find any other errors.

    <div class="sm_mode"> Sam painted grids in the larger square areas pictured below. All of the grids are the same size. Which of the following squares has the largest area painted grey? </div>

    [{"vars":null}]

  320. Fixed incorrect units in solution - were 185mm and 300mm - now 185 cm and 300 cm.

    <div class="sm_mode"> Wooden beams for building house frames can be purchased in a number of lengths. A hardware store can supply lengths of 1.9 metres, 290 millimetres, 185 centimetres, 0.295 metres and 300 centimetres. Which wooden beam is the longest? </div>

    [{"vars":null}]

  321. Measurement, NAPX-p106471v01 SA Incorrect number used in worked solution 468 -> 470. Changed wording in var1 to correct grammar and ambiguity.

    <div class="sm_mode"> {{{question}}} </div>

    [{"vars":[{"varval":"Martin has a piece of wire that measures 1.32 metres.\n\n\r\n\r\nHe cuts off a 470 millimetre piece of wire.\n\n\r\n\r\nHow long is the remaining wire in millimetres?"},{"varval":"sm_nogap Conversion: 1 metre = 1000 millimetres\n\n<div class=\"aligned\">\n\n|||\n|-|-|\n|Length remaining|= (1.320 × 1000) $-$ 470|\n||= 1320 $-$ 470|\n||= {{{correctAnswer0}}} {{{suffix0}}}|\n\n</div>"}]},{"vars":[{"varval":"Henry is 175 centimetres tall.\n\nHe stands on top of a chair that is 425 millimetre tall.\n\nHow tall is Henry now, including the chair, in centimetres?\n"},{"varval":"sm_nogap Conversion: 10 millimetres = 1 centimetre\n\n<div class=\"aligned\">\n\n|||\n|-|-|\n|Total height|= (425 ÷ 10) + 175|\n||= 42.5 + 175|\n||= {{{correctAnswer0}}} {{{suffix0}}}|\n\n</div>"}]}]

  322. Measurement, NAPX-J2-27

    <div class="sm_mode"> {{{question}}} </div>

    [{"vars":[{"varval":"A bottle of milk can fill 5 glasses exactly.\n\n<br>\n\nsm_img https://teacher.smartermaths.com.au/wp-content/uploads/2019/01/NAPX-J2-27-v1.svg 260 indent3 vpad\n\n<br>Which of the following holds the most milk?"},{"varval":"Converting each option to the number of glasses.\n\n\r\nOption 1: 1 × 5 + 8 = 13 glasses\n\n\r\nOption 2: 2 × 5 + 2 = 12 glasses\n\n\r\nOption 3: 12 glasses\n\n\r\nOption 4: 1 × 5 + 3 = 8 glasses\n\nsm_img https://teacher.smartermaths.com.au/wp-content/uploads/2018/12/NAP-J2-27-v1_b1.svg 200 indent vpad\n"}]},{"vars":[{"varval":"A bottle of milk can fill 5 glasses exactly.\n\n<br>\n\nsm_img https://teacher.smartermaths.com.au/wp-content/uploads/2019/01/NAPX-J2-27-v1.svg 270 indent3 vpad\n\n<br>Which of the following holds the most milk?"},{"varval":"Converting each option to the number of glasses.\n\n\r\nOption 1: 3 × 5 = 15 glasses\n\n\r\nOption 2: 1 × 5 + 8 = 13 glasses\n\n\r\nOption 3: 2 × 5 + 1 = 11 glasses\n\n\r\nOption 4: 2 × 5 + 4 = 14 glasses\n\nsm_img https://teacher.smartermaths.com.au/wp-content/uploads/2018/12/NAP-J2-27-v1_a.svg 170 indent vpad"}]}]

  323. <div class="sm_mode"> {{{question}}} </div>

    [{"vars":[{"varval":"Miles makes a fruit drink by mixing juice concentrate with water.\n\n\n\r\nMiles adds 85 millilitres (mL) of juice concentrate to 1 litre (L) of water.\n\n\n\r\nHow much fruit drink did Miles make?"},{"varval":"Converting to mL:\n\n\r\nsm_nogap 1 L = 1000 mL\n\n<div class=\"aligned\">\n\n|||\n|-|-|\n|$\\therefore$ Total fruit drink|= 1000 + 85|\n||= 1085 mL|\n\n</div>"}]},{"vars":[{"varval":"Veronica is making green cordial.\n\nIt is made by mixing water with green concentrate.\n\nVeronica adds 2 litres (L) of water to 60 millilitres (mL) of green concentrate.\n\nHow much cordial does Veronica make?"},{"varval":"Converting to mL:\n\n\r\nsm_nogap 2 L = 2000 mL\n\n<div class=\"aligned\">\n\n|||\n|-|-|\n|$\\therefore$ Total cordial|= 2000 + 60|\n||= 2060 mL|\n\n</div>"}]}]

  324. <div class="sm_mode"> Mary used the scale below to measure the mass of a necklace. sm_img https://teacher.smartermaths.com.au/wp-content/uploads/2018/12/NAPX-H2-18-v2.svg 140 indent3 vpad What is the mass of the necklace on the scales? </div>

    [{"vars":null}]

  325. Measurement, NAPX-p169212v01 Worked Solutions were not showing as they were not set up correctly as variables. Also question was repeated in worked solution.

    <div class="sm_mode"> {{{question}}} </div>

    [{"vars":[{"varval":"At the start of an experiment, Stephanie has 215 millilitres of chemical solution in a flask.\n\n\r\n\r\nStephanie then used some of the solution in the experiment.\n\n\r\n\r\nThe image below shows the volume of the chemical solution left in the flask.\n\nsm_img https://teacher.smartermaths.com.au/wp-content/uploads/2021/05/Raph11-14-MC.svg 200 indent vpad\n\nHow much of the solution was used?"},{"varval":"<div class=\"aligned\">\n\n|||\n|-|-|\n|Volume used|= 215 $-$ 100|\n||= 115 millilitres|\n\n\n</div>"}]},{"vars":[{"varval":"Rosa started with 227 millitres of chemical solution in a flask.\n\n\r\n\r\nShe then poured some of the chemical solution into a test tube.\n\n\r\n\r\nThe level of chemical solution left in the flask is shown in the image below.\n\nsm_img https://teacher.smartermaths.com.au/wp-content/uploads/2021/05/Raph11-13-MC.svg 200 indent vpad\n\nHow much solution did Rosa pour into the test tube?"},{"varval":"<div class=\"aligned\">\n\n|||\n|-|-|\n|Volume used|= 227 $-$ 200|\n||= 27 millilitres|\n\n\n\n</div>"}]}]

  326. Measurement, NAPX-p169002v01 Worked solutions were not showing as not set up as variable correctly.

    <div class="sm_mode"> {{{question}}} </div>

    [{"vars":[{"varval":"Anne was driving from her house to the supermarket to buy some groceries.\n\n\r\n\r\nShe drove a total distance of 2 kilometres and 54 metres.\n\n\r\n\r\nWhich of these shows the distance, in metres, that Anne drove to get to the supermarket?"},{"varval":"Conversion: 1 kilometre = 1000 metres\n\n<div class=\"aligned\">\n\n|||\n|-|-|\n|Distance|= (2 × 1000) + 54|\n||= 2000 + 54|\n||= 2054 m|\n\n</div>"}]},{"vars":[{"varval":"John walked from his office to his home.\n\n\r\n\r\nHe walked a total of 1 kilometre and 62 metres.\n\n\r\n\r\nWhich of these shows the distance John walked in metres."},{"varval":"Conversion: 1 kilometre = 1000 metres\n\n<div class=\"aligned\">\n\n|||\n|-|-|\n|Distance walked|= 1000 + 62|\n||= 1062 m|\n\n</div>"}]}]

  327. Measurement, NAPX p124557v03 Measurement, NAPX p124557v02 RAPH11-13, RAPH11-14

    <div class="sm_mode"> {{{question}}} </div>

    [{"vars":[{"varval":"Gina started with 635 millilitres of liquid in a flask.\n\r\nAfter pouring some some liquid out, the flask is pictured below.\n\n<br>\n\nsm_img https://teacher.smartermaths.com.au/wp-content/uploads/2019/01/NAPX-I2-15v2.svg 100 indent3 vpad\n\nHow much liquid did Gina pour out of the flask?"},{"varval":"Since the flask now has 600 mL,\n\n\r\nsm_nogap Volume of liquid poured out\n\n<div class=\"aligned\">\n\n>>||\n|-|\n|= 635 $−$ 600|\n|= {{{correctAnswer}}}|\n\n</div>"}]},{"vars":[{"varval":"Carlos started with 460 millilitres of liquid in a flask.\n\r\nAfter he poured some of the liquid into a cup, the level of liquid in the flask is pictured below.\n\n<br>\n\nsm_img https://teacher.smartermaths.com.au/wp-content/uploads/2019/01/NAPX-I2-15v1.svg 130 indent3 vpad\n\n<br>How much liquid did Carlos pour into the cup?"},{"varval":"Since the flask now has 400 mL,\n\n\r\nsm_nogap Volume of water poured out\n\n<div class=\"aligned\">\n\n>>||\n|-|\n|= 460 $−$ 400|\n|= {{{correctAnswer}}}|\n\n</div>"}]},{"vars":[{"varval":"Timothy started with 53 millilitres of oil in a flask.\n\r\nThe picture below shows the flask after he poured some unused oil from a test tube back into it.\n\nsm_img https://teacher.smartermaths.com.au/wp-content/uploads/2021/04/q30.png 450 indent3 vpad\n\n<br>How much oil was poured back into the flask?"},{"varval":"<div class=\"aligned\">\n\n|||\n|-|-|\n|Oil in the test tube|= New volume − Original volume|\n||= 80 $−$ 53|\n||= 27 millilitres|\n\n</div>"}]},{"vars":[{"varval":"A chemist started with 685 millilitres of a solution.\n\r\nHe then poured some of the solution into a test tube.\n\n<br>\n\nsm_img https://teacher.smartermaths.com.au/wp-content/uploads/2021/04/tube.png 280 indent3 vpad\n\n<br>How much of the chemical solution was poured into the test tube?"},{"varval":"<div class=\"aligned\">\n\n|||\n|-|-|\n|Solution in test tube|= New volume − Original volume|\n||= 685 $−$ 500|\n||= {{{correctAnswer}}}|\n\n</div>"}]},{"vars":[{"varval":"At the start of an experiment, Stephanie has 215 millilitres of chemical solution in a flask.\n\nStephanie then used some of the solution in the experiment.\n\nThe image below shows the volume of the chemical solution left in the flask.\n\n<br>\n\nsm_img https://teacher.smartermaths.com.au/wp-content/uploads/2021/05/Raph11-14-MC.svg 200 indent3 vpad\n\n<br>How much of the solution was used?"},{"varval":"<div class=\"aligned\">\n\n|||\n|-|-|\n|Volume used|= 215 $-$ 100|\n||= {{{correctAnswer}}}|\n\n</div>"}]},{"vars":[{"varval":"Rosa started with 227 millitres of chemical solution in a flask.\n\n\r\n\r\nShe then poured some of the chemical solution into a test tube.\n\n\r\n\r\nThe level of chemical solution left in the flask is shown in the image below.\n\n<br>\n\nsm_img https://teacher.smartermaths.com.au/wp-content/uploads/2021/05/Raph11-13-MC.svg 200 indent3 vpad\n\n<br>How much solution did Rosa pour into the test tube?"},{"varval":"<div class=\"aligned\">\n\n|||\n|-|-|\n|Volume used|= 227 $-$ 200|\n||= {{{correctAnswer}}}|\n\n</div>"}]}]

  328. <div class="sm_mode"> {{{question}}} </div>

    [{"vars":[{"varval":"Chris is 120 centimetres tall.\n\n<br>\n\nsm_img https://teacher.smartermaths.com.au/wp-content/uploads/2018/12/NAPX-K2-15v2.svg 240 indent3 vpad\n\n<br>About how tall is his big sister Maddie?"},{"varval":"Since Chris is 120 cm tall\n\n\r\n→ Each mark = 20 cm\n\n<div class=\"aligned\">\n\n|||\n|-|-|\n|$\\therefore$ Maddie's height|= 9 × 20|\n||= {{{correctAnswer}}}|\n\n</div>"}]},{"vars":[{"varval":"Colby is 150 centimetres tall.\n\n<br>\n\nsm_img https://teacher.smartermaths.com.au/wp-content/uploads/2018/12/NAPX-K2-15v1rev.svg 240 indent3 vpad\n\n<br>About how tall is his little sister Peach?\n"},{"varval":"Since Colby is 150 cm tall\n\n\r\n→ Each mark = 30 cm\n\n<div class=\"aligned\">\n\n|||\n|-|-|\n|$\\therefore$ Peach's height|= 3 × 30|\n||= {{{correctAnswer}}}|\n\n</div>"}]}]

  329. Measurement, NAPX-p124624v04 SJ Added v2-5

    <div class="sm_mode"> {{{question}}} </div>

    [{"vars":[{"varval":"Avril bought half a kilogram of ham at the butchers.\n\n\r\n\r\nHow many grams of ham did Avril buy?"},{"varval":"1 kilogram = 1000 grams\n\n<div class=\"aligned\">\n\n|||\n|-|-|\n|$\\therefore \\ \\dfrac{1}{2}$ kilogram|= 1000 ÷ 2|\n||= {{{correctAnswer}}} grams|\n\n</div>"}]},{"vars":[{"varval":"Barnsy owned a carp that was one quarter of a metre long.\n\n\r\n\r\nHow long was Barnsy's carp in millimetres?"},{"varval":"1 metre = 1000 millimetres\n\n<div class=\"aligned\">\n\n|||\n|-|-|\n|$\\dfrac{1}{4}$ metre|= $\\dfrac{1}{4}\\ \\times$ 1000|\n||= {{{correctAnswer}}} millimetres|\n\n</div>"}]},{"vars":[{"varval":"Harvey missed the bus and walked half a kilometre to school?\n\nHow many metres did Harvey walk?"},{"varval":"1 kilometre = 1000 metres\n\n<div class=\"aligned\">\n\n|||\n|-|-|\n|$\\dfrac{1}{2}$ kilometre|= $\\dfrac{1}{2}\\ \\times$ 1000|\n||= {{{correctAnswer}}} metres|\n\n</div>"}]},{"vars":[{"varval":"Michelle bought two and a half litres of milk.\n\nHow many millilitres did she buy?"},{"varval":"1 litre = 1000 millilitres\n\n<div class=\"aligned\">\n\n|||\n|-|-|\n|$\\dfrac{1}{2}$ litre|= $\\dfrac{1}{2}\\ \\times$ 1000|\n||= 500 millilitres|\n\n</div>\n\n<div class=\"aligned\">\n\n|||\n|-|-|\n|2$\\dfrac{1}{2}$ litre|= ${2}\\ \\times$ 1000 + 500|\n||= {{{correctAnswer}}} millilitres|\n\n</div>"}]},{"vars":[{"varval":"Roxanne needed three quarters of a metre of blue material to finish her halloween costume.\n\nHow many centimetres of material did she need?"},{"varval":"1 metre = 100 centimetres\n\n<div class=\"aligned\">\n\n|||\n|-|-|\n|$\\dfrac{1}{4}$ metre|= $\\dfrac{1}{4}\\ \\times$ 100|\n||= 250 centimetres|\n\n</div>\n\n<div class=\"aligned\">\n\n|||\n|-|-|\n|$\\dfrac{3}{4}$ metre|= ${3}\\ \\times$ 250|\n||= {{{correctAnswer}}} centimetres|\n\n</div>"}]},{"vars":[{"varval":"A supermarket purchased a quarter of a tonne of pumpkins.\n\nHow many kilograms of pumpkins did the supermarket purchase?"},{"varval":"1 tonne = 1000 kilograms\n\n<div class=\"aligned\">\n\n|||\n|-|-|\n|$\\dfrac{1}{4}$ tonne|= $\\dfrac{1}{4}\\ \\times$ 1000|\n||= {{{correctAnswer}}} kilograms|\n\n</div>"}]}]

  330. <div class="sm_mode"> {{{question}}} </div>

    [{"vars":[{"varval":"The table below shows the session times of movies.\n\n<br>\n\n<div class=\"sm-table col1-color3 row1-color3 top-left-cell-hidden\">\n\n>| | Frozen | Tangled | Peter Pan | Cars |\n|:-:|:-:|:-:|:-:|:-:|\n| Start Time| 9:45 am| 1:00 pm| 1:30 pm| 2:10 pm|\n| Finish Time | 11:45 am| 3:50 pm|4:00 pm|4:50 pm|\n\n</div>\n\n<br>Which of these movies lasts two and a half hours?"},{"varval":"1:30 pm to 4:00 pm\n \r\n>> = 2 hours and 30 minutes\n\n<br>\n\r\n$\\therefore$ {{{correctAnswer}}}"}]},{"vars":[{"varval":"The table below shows the session times of movies.\n\n<br>\n\n<div class=\"sm-table col1-color2 row1-color2 top-left-cell-hidden\">\n\n>| | Evil Dead | Ed Wood | Fight Club | Blood Brothers |\n|:-:|:-:|:-:|:-:|:-:|\n| Start Time| 2:00 pm| 3:10 pm| 3:45 pm| 4:30 pm|\n| Finish Time | 4:50 am| 5:40 pm|5:15 pm|7:20 pm|\n\n</div>\n\n<br>Which of these movies lasts two and a half hours?"},{"varval":"3:10 pm to 5:40 pm = 2 hours and 30 minutes\n\n$\\therefore$ Ed Wood"}]}]

  331. <div class="sm_mode"> Chaucer has 83 match sticks. He uses 6 match sticks to make two small triangles. <br> sm_img https://teacher.smartermaths.com.au/wp-content/uploads/2018/12/NAP-J2-28-v1.svg 300 indent vpad <br>What is the largest number of small triangles like this that Chaucer can make with his 83 match sticks? </div>

    [{"vars":null}]

  332. <div class="sm_mode"> The lights below make a pattern. sm_img https://teacher.smartermaths.com.au/wp-content/uploads/2017/04/NAP-C1-14.png 350 indent vpad Which of these shows the order of the next four lights? </div>

    [{"vars":null}]

  333. Number, NAPX-p110532v02

    <div class="sm_mode"> {{{question}}} </div>

    [{"vars":[{"varval":"Leo puts $15 into his piggy bank to start saving money.\r\n\r\nHe then puts the same amount into the piggy bank at the end of each month.\r\n\r\nThe total amount of money in his piggy bank at the end of each month will be which of the following:"},{"varval":"Starting amount = $15 (odd)\r\n\nEnd of 1st month = 15 + 15 = $30 (even)\r\n\nEnd of 2nd month = 30 + 15 = $45 (odd)\r\n\n$\\therefore$ Total amount will be either an odd or even number."}]},{"vars":[{"varval":"Ana recycles 3 kilograms of plastic bottles in her first week of recycling.\r\n\r\nShe then recycles twice as much as she recycled the previous week for the next 5 weeks.\r\n\r\nThe total weight, in kilograms, of the plastic bottles she recycles in any given week will be which of the following:"},{"varval":"\r\n1st week: 3 kg (odd)\r\n\n2nd week: 3 + 6 = 9 kg (odd)\r\n\n3rd week: 9 + 18 = 27 kg (odd)\r\n\nAny week: odd + even = odd\n\n∴ Total amount will be always an odd number."}]}]

  334. <div class="sm_mode"> This repeating pattern is made by turning a square tile. sm_img https://teacher.smartermaths.com.au/wp-content/uploads/2017/04/naplan-Y3-2013-10mc.png 500 indent vpad Which of these comes next in this pattern? </div>

    [{"vars":null}]

  335. <div class="sm_mode"> Rasputin cuts a pizza into 6 smaller pieces along the lines shown. <br> sm_img https://teacher.smartermaths.com.au/wp-content/uploads/2019/01/NAPX-K2-21v3.svg 280 indent3 vpad <br>How many pieces are in one-third of the whole pizza? </div>

    [{"vars":null}]

  336. <div class="sm_mode"> {{{question}}} </div>

    [{"vars":[{"varval":"Cameron writes a number pattern where each number is three more than the previous number.\n\nWhat number should Cameron write next?"},{"varval":"<div class=\"aligned\">\n\n| | |\n| ------------------------------- | ---------------------- |\n| <span class=\"sm_box\"> ` ` </span>| \\= 13 + 3 |\n| | \\= {{{correctAnswer0}}} |\n\n</div>"}]},{"vars":[{"varval":"Jevin writes a number pattern where each number is four more than the previous number.\n\nWhat number should Jevin write next?"},{"varval":"<div class=\"aligned\">\n\n| | |\n| ------------------------------- | ---------------------- |\n| <span class=\"sm_box\"> ` ` </span>| \\= 21 + 4 |\n| | \\= {{{correctAnswer0}}} |\n\n</div>"}]},{"vars":[{"varval":"Chardae writes a number pattern where each number is three less than the previous number.\n\nWhat number should Chardae write next?"},{"varval":"<div class=\"aligned\">\n\n| | |\n| ------------------------------- | ---------------------- |\n| <span class=\"sm_box\"> ` ` </span>| \\= 11 - 3 |\n| | \\= {{{correctAnswer0}}} |\n\n</div>"}]},{"vars":[{"varval":"Benson writes a number pattern where each number is five more than the previous number.\n\nWhat number should Benson write next?"},{"varval":"<div class=\"aligned\">\n\n| | |\n| ------------------------------- | ---------------------- |\n| <span class=\"sm_box\"> ` ` </span>| \\= 17 + 5 |\n| | \\= {{{correctAnswer0}}} |\n\n</div>"}]},{"vars":[{"varval":"Misty writes a number pattern where each number is four less than the previous number.\n\nWhat number should Misty write next?"},{"varval":"<div class=\"aligned\">\n\n| | |\n| ------------------------------- | ---------------------- |\n| <span class=\"sm_box\"> ` ` </span>| \\= 21 - 4 |\n| | \\= {{{correctAnswer0}}} |\n\n</div>"}]}]

  337. This question didn't have the angles in italics in the solution, the letters were too small and the worked solution said "aandc are both obtuse....." I have added 4 more variants to this question.

    <div class="sm_mode"> {{{question}}} </div>

    [{"vars":[{"varval":"sm_img https://teacher.smartermaths.com.au/wp-content/uploads/2018/08/NAPX-G3-CA17.svg 260 indent vpad\n\nWhich of the angles in this shape is closest to 80$\\degree$?"},{"varval":"By inspection:\n\n\r\nAngles $\\large a$ and $\\large c$ are both obtuse (greater than 90$\\degree$)\n\n\r\nAngle $\\large b$ is close to 45$\\degree$.\n\n\r\n$\\therefore$ Angle {{{correctAnswer}}} ≈ 80$\\degree$\n"}]},{"vars":[{"varval":"sm_img https://teacher.smartermaths.com.au/wp-content/uploads/2022/11/Geom_NAPX-G3-CA17_v1.svg 120 indent vpad\n\nWhich of the angles in this shape is closest to 60$\\degree$?"},{"varval":"By inspection:\n\n\r\nAngles $\\large b$ and $\\large d$ are both obtuse (greater than 90$\\degree$)\n\n\r\nAngle $\\large a$ is close to 45$\\degree$.\n\n\r\n$\\therefore$ Angle {{{correctAnswer}}} ≈ 60$\\degree$\n"}]},{"vars":[{"varval":"sm_img https://teacher.smartermaths.com.au/wp-content/uploads/2018/08/NAPX-G3-CA17.svg 260 indent vpad\n\nWhich of the angles in this shape is closest to 45$\\degree$?"},{"varval":"By inspection:\n\nAngles $\\large a$ and $\\large c$ are both obtuse (greater than 90$\\degree$)\n\n\r\nAngle $\\large d$ is bigger than 45$\\degree$.\n\n\r\n$\\therefore$ Angle {{{correctAnswer}}} ≈ 45$\\degree$\n"}]},{"vars":[{"varval":"sm_img https://teacher.smartermaths.com.au/wp-content/uploads/2022/11/Geom_NAPX-G3-CA17_v1.svg 120 indent vpad\n\nWhich of the angles in this shape is closest to 30$\\degree$?"},{"varval":"By inspection:\n\nAngles $\\large b$ and $\\large d$ are both obtuse (greater than 90$\\degree$)\n\n\r\nAngle $\\large c$ ≈ 60$\\degree$.\n\n\r\n$\\therefore$ Angle {{{correctAnswer}}} ≈ 30$\\degree$\n"}]},{"vars":[{"varval":"sm_img https://teacher.smartermaths.com.au/wp-content/uploads/2022/11/Geom_NAPX-G3-CA17_v4.svg 360 indent vpad\n\nWhich of the angles in this shape is closest to 20$\\degree$?"},{"varval":"By inspection:\n\nAngle $\\large d$ is reflex (greater than 180$\\degree$ less than 360$\\degree$)\n\n\r\nAngle $\\large a$ ≈ 70$\\degree$\n\nAngle $\\large b$ ≈ 40$\\degree$\n\n$\\therefore$ Angle {{{correctAnswer}}} ≈ 20$\\degree$\n"}]}]

  338. Added categories. Corrected wording in var0. Added var2-4 with staging_suejones in categories.

    <div class="sm_mode"> {{{question}}} </div>

    [{"vars":[{"varval":"A one metre tall person casts a 4 metre shadow on level ground.\n\nsm_img https://teacher.smartermaths.com.au/wp-content/uploads/2021/04/Math-Job-Q35.svg 300 indent vpad\n\nIf a 1.8 metre tall person stands at the same position, how long will the shadow be?"},{"varval":"1 metre height → 4 metre shadow\n\n<div class=\"aligned\">\n\n|||\n|-|-|\n|$\\therefore$ 1.8 m height shadow|= 1.8 × 4|\n||= {{{correctAnswer}}}|\n\n</div>"}]},{"vars":[{"varval":"A one metre tall person casts a 2 metre shadow on level ground.\n\nsm_img https://teacher.smartermaths.com.au/wp-content/uploads/2021/04/Math-Job-Q35.svg 300 indent vpad\n\nIf a 1.4 metre tall person stands at the same position, how long will the shadow be?"},{"varval":"1 metre height → 2 metre shadow\n\n<div class=\"aligned\">\n\n|||\n|-|-|\n|$\\therefore$ 1.4 m height shadow|= 1.4 × 2|\n||= {{{correctAnswer}}}|\n\n</div>"}]},{"vars":[{"varval":"A one metre tall person casts a 4 metre shadow on level ground.\n\nsm_img https://teacher.smartermaths.com.au/wp-content/uploads/2021/04/Math-Job-Q35.svg 300 indent vpad\n\nIf a 0.8 metre tall person stands at the same position, how long will the shadow be?"},{"varval":"1 metre height → 4 metre shadow\n\n<div class=\"aligned\">\n\n|||\n|-|-|\n|$\\therefore$ 0.8 m height shadow|= 0.8 × 4|\n||= {{{correctAnswer}}}|\n\n</div>"}]},{"vars":[{"varval":"A one metre tall person casts a 5 metre shadow on level ground.\n\nsm_img https://teacher.smartermaths.com.au/wp-content/uploads/2021/04/Math-Job-Q35.svg 300 indent vpad\n\nIf a 2.1 metre tall person stands at the same position, how long will the shadow be?"},{"varval":"1 metre height → 5 metre shadow\n\n<div class=\"aligned\">\n\n|||\n|-|-|\n|$\\therefore$ 2.1 m height shadow|= 2.1 × 5|\n||= {{{correctAnswer}}}|\n\n</div>"}]},{"vars":[{"varval":"A one metre tall adult casts a 3 metre shadow on level ground.\n\nsm_img https://teacher.smartermaths.com.au/wp-content/uploads/2021/04/Math-Job-Q35.svg 300 indent vpad\n\nIf a 0.6 metre tall child stands at the same position, how long will the shadow be? "},{"varval":"1 metre height → 3 metre shadow\n\n<div class=\"aligned\">\n\n|||\n|-|-|\n|$\\therefore$ 0.6 m height shadow|= 0.6 × 3|\n||= {{{correctAnswer}}}|\n\n</div>"}]}]

  339. Can't find any errors in this question.

    <div class="sm_mode"> Henry got lost driving to his uncle’s house and made 3 U-turns before arriving. In total, how many degrees does Henry turn through when making U-turns on his trip? </div>

    [{"vars":null}]

  340. The decagon did not look like a regular decagon so I redid the images. Otherwise the question looks OK.

    <div class="sm_mode"> {{{question}}} </div>

    [{"vars":[{"varval":"A regular decagon is folded in half along the dotted line.\n\nsm_img https://teacher.smartermaths.com.au/wp-content/uploads/2022/11/Geom_NAPX-p168433v01_v0.svg 160 indent vpad\n\nWhat name is given to is the folded figure?"},{"varval":"The folded shape has 6 sides → {{{correctAnswer}}}.\n\nsm_img https://teacher.smartermaths.com.au/wp-content/uploads/2022/11/Geom_NAPX-p168433v01_v0_ws.svg 90 indent vpad"}]}]

  341. Geometry, NAPX-G3-NC02

    <div class="sm_mode"> {{{question}}} </div>

    [{"vars":[{"varval":"Which picture shows that the book is opened at an angle of 180°?"},{"varval":"sm_img https://teacher.smartermaths.com.au/wp-content/uploads/2021/04/RAPH10-Q9-SOL.png 320 indent vpad\n\nThis book is opened at an angle of 180∘"}]},{"vars":[{"varval":"Which angle below is closest to 60°?"},{"varval":"sm_img https://teacher.smartermaths.com.au/wp-content/uploads/2017/01/NAP-G3-NC02a.png 120 indent vpad"}]},{"vars":[{"varval":"A floor plan marked an angle that measured 35°.\n\r\nWhich of these shows an angle closest to the angle on the floor plan?"},{"varval":"{{{correctAnswer}}}"}]}]

  342. <div class="sm_mode"> A scale diagram is shown below and represents the distances between 3 cities. sm_img https://teacher.smartermaths.com.au/wp-content/uploads/2021/05/RAPH10_82.svg 260 indent vpad The distance from City A to City B is 84 kilometres. What is the distance from City B to City C? </div>

    [{"vars":null}]

  343. Can't see any problems with this question.

    <div class="sm_mode"> Penelope is walking due south. She makes a quarter turn to her left and keeps walking. Penelope then makes another quarter turn to her left and stops. What direction is she now facing? </div>

    [{"vars":null}]

  344. <div class="sm_mode"> A building's floor plan is pictured below. <br> sm_img https://teacher.smartermaths.com.au/wp-content/uploads/2017/03/naplan-Y5-2015-32mc-new.png 410 indent2 vpad <br>Misty enters the building through the southern door and leaves through the eastern door. What is the smallest number of rooms she must walk through? </div>

    [{"vars":null}]

  345. Not sure of the problem. Maybe it's that the question asks about the direction of the arrow and there is the North arrow and the one on the street.

    <div class="sm_mode"> Alexa was walking on Ruby Street in the direction of the arrow shown below. sm_img https://teacher.smartermaths.com.au/wp-content/uploads/2021/04/RAPH9-3.svg 200 indent vpad In what direction was Alexa heading? </div>

    [{"vars":null}]

  346. <div class="sm_mode"> {{{question}}} </div>

    [{"vars":[{"varval":"Shawn has a bag that holds one blue ball, one red ball and one green ball.\r\n\r\nShawn picks one ball without looking, records its colour and puts it back in the bag.\r\n\r\nHe repeated this process 60 times and summarises the results in the table below.\n\n<br> \n\n<div class=\"outline\">\n\n>>> |Colour |Number of Times|\n|:-:|:-:|\n|Red|23|\n|Blue|19|\n|Green|18|\n\n</div>\n\n<br>\n\nWhat is the difference between the expected and actual number of red balls picked?\n"},{"varval":"Probability of grabbing a red ball = $\\dfrac{1}{3}$\n\n<div class=\"aligned\">\r\n\r\n| | |\r\n| ------------- | ---------- |\r\n| Expected number of red balls | \\= $\\dfrac{1}{3} \\ \\times 60$ |\r\n| | \\= 20 |\r\n\r\n</div>\r\n\n<br>\n\n<div class=\"aligned\">\r\n\r\n| | |\r\n| ------------- | ---------- |\r\n| $\\therefore$ Difference | \\= $23 \\ − \\ 20$ |\r\n| | \\= {{{correctAnswer0}}} |\r\n\r\n</div>\r\n"}]},{"vars":[{"varval":"Elaine rolled a regular die 100 times.\r\n\r\nShe recorded if an even or odd number appeared each time.\r\n\r\nThe table below represents the recorded data.\n\n<br> \n\n<div class=\"outline\">\n\n>>> |Result |Number of Times|\n|:-:|:-:|\n|Even|58|\n|Odd|42|\n\n</div>\n\n<br>\n\nWhat is the difference between the expected number of odd rolls to the actual odd rolls recorded?\n"},{"varval":"Probability of odd number = $\\dfrac{3}{6}$ = 50%\n\n<div class=\"aligned\">\r\n\r\n| | |\r\n| ------------- | ---------- |\r\n| Expected number of odd rolls | \\= 50% $\\times$ 100 |\r\n| | \\= 50 |\r\n\r\n</div>\r\n\n<br>\n\n<div class=\"aligned\">\r\n\r\n| | |\r\n| ------------- | ---------- |\r\n| $\\therefore$ Difference | \\= $50 \\ − \\ 42$ |\r\n| | \\= {{{correctAnswer0}}} |\r\n\r\n</div>\r\n"}]},{"vars":[{"varval":"Geoff has a bag that holds one orange ball, one red ball, one white ball and one green ball.\r\n\r\nGeoff picks one ball without looking, records its colour and puts it back in the bag.\r\n\r\nHe repeated this process 60 times and summarises the results in the table below.\n\n<br> \n\n<div class=\"outline\">\n\n>>> |Colour |Number of Times|\n|:-:|:-:|\n|Orange|19|\n|Red|12|\n|White|15|\n|Green|14|\n\n</div>\n\n<br>\n\nWhat is the difference between the expected and actual number of orange balls picked?"},{"varval":"Probability of grabbing an orange ball = $\\dfrac{1}{4}$\n\n<div class=\"aligned\">\r\n\r\n| | |\r\n| ------------- | ---------- |\r\n| Expected number of orange balls | \\= $\\dfrac{1}{4} \\ \\times 60$ |\r\n| | \\= 15 |\r\n\r\n</div>\r\n\n<br>\n\n<div class=\"aligned\">\r\n\r\n| | |\r\n| ------------- | ---------- |\r\n| $\\therefore$ Difference | \\= $19 \\ − \\ 15$ |\r\n| | \\= {{{correctAnswer0}}} |\r\n\r\n</div>\r"}]},{"vars":[{"varval":"Tammy rolls a fair dice with six faces numbered 1 to 6.\n\r\nShe repeated this process 30 times and the results are summarised in the table below.\n\n<br> \n\n<div class=\"outline\">\n\n>>> |Number|Number of Times|\n|:-:|:-:|\n|1|5|\n|2|3|\n|3|8|\n|4|6|\n|5|3|\n|6|5|\n\n</div>\n\n<br>\n\nWhat is the difference between the expected and actual number of times that number 3 was rolled?"},{"varval":"Probability of rolling a 3 = $\\dfrac{1}{6}$\n\n<div class=\"aligned\">\r\n\r\n| | |\r\n| ------------- | ---------- |\r\n| Expected number of 3's | \\= $\\dfrac{1}{6} \\ \\times 30$ |\r\n| | \\= 5 |\r\n\r\n</div>\r\n\n<br>\n\n<div class=\"aligned\">\r\n\r\n| | |\r\n| ------------- | ---------- |\r\n| $\\therefore$ Difference | \\= $8 \\ − \\ 5$ |\r\n| | \\= {{{correctAnswer0}}} |\r\n\r\n</div>\r"}]},{"vars":[{"varval":"Keiron rolls a fair dice with six faces numbered 1 to 6.\n\r\nHe repeated this process 42 times and the results are summarised in the table below.\n\n<br> \n\n<div class=\"outline\">\n\n>>> |Number|Number of Times|\n|:-:|:-:|\n|1|7|\n|2|5|\n|3|11|\n|4|7|\n|5|6|\n|6|6|\n\n</div>\n\n<br>\n\nWhat is the difference between the expected and actual number of times that number 3 was rolled?"},{"varval":"Probability of rolling a 3 = $\\dfrac{1}{6}$\n\n<div class=\"aligned\">\r\n\r\n| | |\r\n| ------------- | ---------- |\r\n| Expected number of 3's | \\= $\\dfrac{1}{6} \\ \\times 42$ |\r\n| | \\= 7 |\r\n\r\n</div>\r\n\n<br>\n\n<div class=\"aligned\">\r\n\r\n| | |\r\n| ------------- | ---------- |\r\n| $\\therefore$ Difference | \\= $11 \\ − \\ 7$ |\r\n| | \\= {{{correctAnswer0}}} |\r\n\r\n</div>\r"}]},{"vars":[{"varval":"Dolly rolls a fair dice with six faces numbered 1 to 6.\n\r\nShe repeated this process 48 times and the results are summarised in the table below.\n\n<br> \n\n<div class=\"outline\">\n\n>>> |Number|Number of Times|\n|:-:|:-:|\n|1|8|\n|2|5|\n|3|12|\n|4|10|\n|5|5|\n|6|8|\n\n</div>\n\n<br>\n\nWhat is the difference between the expected and actual number of times that number 4 was rolled?"},{"varval":"Probability of rolling a 4 = $\\dfrac{1}{6}$\n\n<div class=\"aligned\">\r\n\r\n| | |\r\n| ------------- | ---------- |\r\n| Expected number of 4's | \\= $\\dfrac{1}{6} \\ \\times 48$ |\r\n| | \\= 8 |\r\n\r\n</div>\r\n\n<br>\n\n<div class=\"aligned\">\r\n\r\n| | |\r\n| ------------- | ---------- |\r\n| $\\therefore$ Difference | \\= $10 \\ − \\ 8$ |\r\n| | \\= {{{correctAnswer0}}} |\r\n\r\n</div>\r"}]}]

  347. var: Probability, NAPX-p167310v02 Probability, NAPX-p167310v01 NOTE: Probability, NAPX-p167310v04 (Teacher Edition) solution variables doesn't match the question specified?

    <div class="sm_mode"> {{{question}}} </div>

    [{"vars":[{"varval":"A man spins a wheel with 5 different coloured sections and records which colour it lands on each time.\r\n\r\nHe repeats the process multiple times.\r\n\r\nThe table below shows the results.\n\n<br>\n\n<div class=\"outline color2\">\n\n>> | WHITE | YELLOW | RED | BLUE | GREEN |\n> | :-----------: | :----------: | :------------: | :----------: | :------------: |\n> | 42 | 24 | 38 | 27 | 37 |\n\n</div>\n\n<br>\n\n<br>Using the table, what is the probability that the next spin will be yellow?"},{"varval":"<div class=\"aligned\">\r\n\r\n| | |\r\n| ------------: | ---------- |\r\n| $P$(yellow) | \\= $\\dfrac{\\text{Number of yellow}}{\\text{Total number of throws}}$ |\n| | |\n| | \\= $\\dfrac{24}{42+24+38+27+37}$ |\n| | |\n| | \\= $\\dfrac{24}{168}$ |\r\n| | |\n| | \\= {{{correctAnswer}}} |\r\n\r\n</div>\r\n"}]},{"vars":[{"varval":"Marie grabs a coloured ball from a bag and records the colour. She then puts the ball back into the bag and repeats this process 96 times.\r\n\r\nThe results are shown in the table below.\n\n<div class=\"outline color5\">\n\n>> | Orange | Blue | Red | Green| White | Black | Yellow |\n> | :-----------: | :----------: | :------------: | :----------: | :------------: | :----------: | :------------: |\n> | 13 | 20 | 18 | 9 | 12 | 14 | 10 |\n\n</div>\n\n<br>\n\nUsing the table, what is the probability that the next ball picked out by Marie will be yellow?\n"},{"varval":"<div class=\"aligned\">\r\n\r\n| | |\r\n| ------------: | ---------- |\r\n| $P$(yellow) | \\= $\\dfrac{\\text{Number of yellow}}{\\text{Total number}}$ |\n| | \\= $\\dfrac{10}{96}$ |\r\n| | \\= {{{correctAnswer}}} |\r\n\r\n</div>\r\n"}]},{"vars":[{"varval":"A biased die has 6 faces numbered from 1 to 6.\r\n\r\nRaven throws the die 50 times and recorded the results in the table below.\n\n<br>\n\n<div class=\"outline color3\">\n\n>> | Number | 1 | 2 | 3| 4 | 5 | 6 |\n> | :-----------: | :----------: | :------------: | :----------: | :------------: | :----------: | :------------: |\n> | **Times**| 11 | 8 | 9 | 10 | 6 | 6 |\n\n</div>\n\n\n<br>\n\nUsing the table, what is the probability that Raven will be throwing a 5 on her next throw?"},{"varval":"<div class=\"aligned\">\r\n\r\n| | |\r\n| ------------: | ---------- |\r\n| $P$(5) | \\= $\\dfrac{\\text{Number of 5's}}{\\text{Total number of throws}}$ |\n| | |\n| | \\= $\\dfrac{6}{50}$ |\r\n| | |\n| | \\= {{{correctAnswer}}} |\r\n\r\n</div>\r\n"}]},{"vars":[{"varval":"Peggy picks a marble from a bag and records its colour before placing it back in the bag.\r\n\r\nShe repeats this 20 times.\r\n\r\nHer results are in the table below.\n\n<br>\n\n<div class=\"outline color8\">\n\n>> | Black | Green | Yellow | Blue | Red |\n> | :-----------: | :----------: | :------------: | :----------: | :------------: |\n> | 4 | 5 | 1 | 8 | 2 |\n\n</div>\n\n<br>\n\n<br>Using the table, what is the probability that the next marble Peggy picks will be black?"},{"varval":"<div class=\"aligned\">\r\n\r\n| | |\r\n| ------------- | ---------- |\r\n| $P$(Black) | \\= $\\dfrac{\\text{number of black picked}}{\\text{total picks}}$ |\r\n| | \\= $\\dfrac{4}{20}$ |\n| | \\= {{{correctAnswer}}} |\r\n\r\n</div>\r"}]},{"vars":[{"varval":"Klaus picks a ball from a bag and records the colour. \n\nHe then puts the ball back to the bag and repeats this process 70 times.\r\n\r\nThe results are shown in the table below.\n\n<div class=\"outline color7\">\n\n>> | Puple | Blue | Red | Green| White | Pink |\n> | :-----------: | :----------: | :------------: | :----------: | :------------: | :----------: | \n> | 9 | 12 | 21 | 15 |5 | 8 |\n\n</div>\n\n<br>\n\nUsing the table, what is the probability that the next ball picked up by Klaus will be white?"},{"varval":"<div class=\"aligned\">\r\n\r\n| | |\r\n| ------------: | ---------- |\r\n| $P$(white) | \\= $\\dfrac{\\text{Number of white}}{\\text{Total number}}$ |\n| | \\= $\\dfrac{5}{70}$ |\r\n| | \\= {{{correctAnswer}}} |\r\n\r\n</div>\r"}]},{"vars":[{"varval":"A biased die has 6 faces numbered from 1 to 6.\r\n\r\nJackson throws the die 60 times and recorded the results in the table below.\n\n<br>\n\n<div class=\"outline color6\">\n\n>> | Number | 1 | 2 | 3| 4 | 5 | 6 |\n> | :-----------: | :----------: | :------------: | :----------: | :------------: | :----------: | :------------: |\n> | **Times**| 8 | 14 | 9 | 13 | 7 | 9 |\n\n</div>\n\n\n<br>\n\nUsing the table, what is the probability that Jackson will be throwing a 2 on his next throw?"},{"varval":"<div class=\"aligned\">\r\n\r\n| | |\r\n| ------------: | ---------- |\r\n| $P$(2) | \\= $\\dfrac{\\text{Number of 2's}}{\\text{Total number of throws}}$ |\n| | |\n| | \\= $\\dfrac{14}{60}$ |\r\n| | |\n| | \\= {{{correctAnswer}}} |\r\n\r\n</div>\r"}]}]

  348. <div class="sm_mode"> A bag contains different coloured balls. It contains 4 red balls, 8 green balls, 3 pink balls, 6 grey balls and 11 yellow balls. Nathalie picks a ball from the bag without looking. What is the chance that she draws a grey ball? </div>

    [{"vars":null}]

  349. NOTE: Teacher Edition question has 2 therefores in worked solution.

    <div class="sm_mode"> In a standard deck of 52 playing cards, Ian draws one without looking. sm_img https://teacher.smartermaths.com.au/wp-content/uploads/2021/05/RAPH10_Q56.svg 350 indent vpad Which card is most likely to be drawn? </div>

    [{"vars":null}]

  350. var: Probability, NAPX-p169473v01 NOTE: Probability, NAPX-p169473v02 woked solution in teacher edition has "P(white)" instead of "blue". Might be a typing error.

    <div class="sm_mode"> {{{question}}} </div>

    [{"vars":[{"varval":"Noel has a bowl full of red chewing gum balls and blue chewing gum balls.\r\n\r\nThe chance of randomly picking a red chewing gum ball is 85%.\r\n\r\nWhat is the probability of randomly picking a blue chewing gum ball?"},{"varval":"sm_nogap $P$(red) + $P$(blue) = 100%\n\n<div class=\"aligned\">\r\n\r\n| | |\r\n| ------------: | ---------- |\r\n| $\\therefore\\ P$(blue) | \\= $100 \\ − \\ 85$ |\r\n| | = {{{correctAnswer0}}}{{{suffix0}}} |\r\n\r\n</div>"}]},{"vars":[{"varval":"Tristan's laundry has a lost clothing basket that contains only black and white socks.\r\n\r\nThe probability of randomly picking a black sock from the basket is 35%.\r\n\r\nWhat is the probability of randomly picking a white sock?\n"},{"varval":"sm_nogap $P$(white) + $P$(black) = 100%\n\n<div class=\"aligned\">\r\n\r\n| | |\r\n| ------------: | ---------- |\r\n| $\\therefore\\ P$(white) | \\= $100 \\ − \\ 35$ |\r\n| | \\= {{{correctAnswer0}}}{{{suffix0}}} |\r\n\r\n</div>"}]}]

  351. <div class="sm_mode"> {{{question}}} </div>

    [{"vars":[{"varval":"sm_img https://teacher.smartermaths.com.au/wp-content/uploads/2018/12/nap-K2-28.svg 200 indent vpad\r\n\nRoderick rolls a standard dice once.\r\n\r\nWhich of the following results is least likely?"},{"varval":"Consider the chance of each option:\r\n\n$P$(rolling an odd number) = $\\dfrac{3}{6}$\r\n\n$P$(rolling an even number) = $\\dfrac{3}{6}$\r\n\n$P$(rolling a 2 or 4) = $\\dfrac{2}{6}$\r\n\n$P$(rolling a 6) = $\\dfrac{1}{6}$\n\n$\\therefore$ {{{correctAnswer}}} is the least likely."}]},{"vars":[{"varval":"Rocky rolls a standard 6-sided dice once.\r\n\r\nWhich of the following is Rocky most likely to roll?"},{"varval":"Consider the probability of each option:\r\n\n$P$(greater than or equal to 3) = $\\dfrac{4}{6}$ = $\\dfrac{2}{3}$\r\n\n$P$(odd) = $\\dfrac{3}{6}$ = $\\dfrac{1}{2}$\r\n\n$P$(less than 4) = $\\dfrac{3}{6}$ = $\\dfrac{1}{2}$\r\n\n$P$(4 or 6)= $\\dfrac{2}{6}$ = $\\dfrac{1}{3}$\r\n \r\n\r\n$\\therefore$ {{{correctAnswer}}} is the most likely."}]}]

  352. <div class="sm_mode"> A standard deck of 52 cards is made up of four suits - Hearts, Diamonds, Clubs and Spades. Each suit contains 13 cards that include an Ace, King, Queen and Jack, together with numbered cards from 2 to 10. Lara has a standard deck of cards and without looking, picks a number 7 and returns it to the deck. She repeats this three times and draws a number 7 each time. If she draws a 4th card without looking, which of the following is true? </div>

    [{"vars":null}]

  353. var: Probability, NAPX7-TLA-4 v2

    <div class="sm_mode"> {{{question}}} </div>

    [{"vars":[{"varval":"The probability of rolling a number below 3 on a standard six-sided die is"},{"varval":"<div class=\"aligned\">\r\n\r\n| | |\r\n| ------------: | ---------- |\r\n| $P$(rolling less than 3) | \\= $\\dfrac{\\text{favorable events}}{\\text{total possible events}}$ |\r\n| | \\= $\\dfrac{2}{6}$ |\n| | \\= {{{correctAnswer}}} |\r\n\r\n</div>\r\n"}]},{"vars":[{"varval":"The probability of rolling a number greater than 2 on a standard six-sided die is"},{"varval":"<div class=\"aligned\">\r\n\r\n| | |\r\n| ------------: | ---------- |\r\n| $P$(rolling greater than 2) | \\= $\\dfrac{\\text{favorable events}}{\\text{total possible events}}$ |\r\n| | \\= $\\dfrac{4}{6}$ |\n| | \\= {{{correctAnswer}}} |\r\n\r\n</div>\r\n"}]},{"vars":[{"varval":"The probability of rolling an odd number on a standard six-sided dice is"},{"varval":"<div class=\"aligned\">\r\n\r\n| | |\r\n| ------------: | ---------- |\r\n| $P$(rolling an odd) | \\= $\\dfrac{\\text{favorable events}}{\\text{total possible events}}$ |\r\n| | \\= $\\dfrac{3}{6}$ |\n| | \\= {{{correctAnswer}}} |\r\n\r\n</div>\r"}]}]

  354. <div class="sm_mode"> {{{question}}} </div>

    [{"vars":[{"varval":"Marie rolls a fair dice 60 times.\r\n\r\nWhich result is most likely?"},{"varval":"The expected result is 10 rolls for each number.\r\n\n$\\therefore$ Most likely result is {{{correctAnswer}}}."}]},{"vars":[{"varval":"Rooster rolls a fair dice 24 times.\r\n\r\nWhich result is most likely?"},{"varval":"The expected result is 4 rolls land on each number.\r\n\n$\\therefore$ Most likely result is {{{correctAnswer}}}."}]}]

  355. <div class="sm_mode"> {{{question}}} </div>

    [{"vars":[{"varval":"A charity calculates its total donations every month.\r\n\r\nOver a six month period, the amounts they calculate are as follows:\r\n\r\n\n> > $1520, $1270, $1600, $1310, $1430, $1288\r\n\r\n\n<br>\n\nWhat is the mean monthly donation the charity received over this 6 month period?"},{"varval":"<div class=\"aligned\">\r\n\r\n| | |\r\n| ------------: | ---------- |\r\n| Mean | \\= $\\dfrac{1520+1270+1600+1310+1430+1288}{6}$ |\n| | \\= $\\dfrac{8418}{6}$|\r\n| | \\= {{{prefix0}}}{{{correctAnswer0}}} |\r\n\r\n</div>"}]},{"vars":[{"varval":"A carwash calculates its total receipts every month.\r\n\r\nOver a six month period, the amounts they calculate are as follows:\r\n\r\n\n> > $31 450, $29 465, $24 385, $25 460, $28 700, $26 344\r\n\r\n\n<br>\n\nWhat is the mean monthly amount the carwash received over this 6 month period?"},{"varval":"<div class=\"aligned\">\r\n\r\n| | |\r\n| ------------: | ---------- |\r\n| Mean | \\= $\\dfrac{31\\ 450+29\\ 464+24\\ 385+25\\ 460+28\\ 700+26\\ 344}{6}$ |\n| | \\= $\\dfrac{165\\ 804}{6}$|\r\n| | \\= $27 634 |\r\n\r\n</div>"}]},{"vars":[{"varval":"Gerwyn is recycling bottles and cans and calculates the amount he receives at the recycling depot.\r\n\r\nOver a six week period, the amounts he calculates are as follows:\r\n\r\n\n> > $16.70, $12.70, $21.40, $14.10, $18.30, $12.80\r\n\r\n\n<br>\n\nWhat is the mean weekly recycling amount Gerwyn received over this 6 week period?"},{"varval":"<div class=\"aligned\">\r\n\r\n| | |\r\n| ------------: | ---------- |\r\n| Mean | \\= $\\dfrac{16.70+12.70+21.40+14.10+18.30+12.80}{6}$ |\n| | \\= $\\dfrac{96}{6}$|\r\n| | \\= {{{prefix0}}}{{{correctAnswer0}}} |\r\n\r\n</div>"}]},{"vars":[{"varval":"Bexley owns a delivery truck and calculates his delivery charges every month.\n\n\r\n\r\nOver a six month period, the amounts he calculated are as follows:\r\n\r\n\n> > $2452, $3400, $4510, $3567, $1243, $890\r\n\r\n\n<br>\n\nWhat is the mean monthly amount Bexley received over this 6 month period?"},{"varval":"<div class=\"aligned\">\r\n\r\n| | |\r\n| ------------: | ---------- |\r\n| Mean | \\= $\\dfrac{2452+3400+4510+3567+1243+890}{6}$ |\n| | \\= $\\dfrac{16\\ 062}{6}$|\r\n| | \\= {{{prefix0}}}{{{correctAnswer0}}} |\r\n\r\n</div>"}]},{"vars":[{"varval":"A parking station records the total number of cars parking there every day.\r\n\r\nOver a seven day period, the numbers of cars parking are recorded as follows:\r\n\r\n\n> > 846, 465, 786, 345, 682, 987, 782\r\n\r\n\n<br>\n\nWhat is the mean number of cars parking over this 7 day period?"},{"varval":"<div class=\"aligned\">\r\n\r\n| | |\r\n| ------------: | ---------- |\r\n| Mean | \\= $\\dfrac{846+465+786+345+682+987+782}{7}$ |\n| | \\= $\\dfrac{4893}{7}$|\r\n| | \\= {{{correctAnswer0}}} {{{suffix0}}} |\r\n\r\n</div>"}]},{"vars":[{"varval":"A landscaping company calculates the total number of kilograms of topsoil it sells every day.\n\r\n\r\n\nOver a seven day period, the amount of topsoil sold is calculated as follows:\r\n\r\n\n> > 345, 476, 256, 360, 481, 950, 240,\r\n\r\n\n<br>\n\nWhat is the mean amount of topsoil sold over this 7 day period?"},{"varval":"<div class=\"aligned\">\r\n\r\n| | |\r\n| ------------: | ---------- |\r\n| Mean | \\= $\\dfrac{345+476+256+360+481+950+240}{7}$ |\n| | \\= $\\dfrac{3108}{7}$|\r\n| | \\= {{{correctAnswer0}}} {{{suffix0}}} |\r\n\r\n</div>"}]}]

  356. <div class="sm_mode"> {{{question}}} </div>

    [{"vars":[{"varval":"This graph shows the number of men and women that registered to vote before a council election on different days of the week.\n\n<br>\n\nsm_img https://teacher.smartermaths.com.au/wp-content/uploads/2018/12/nap-K2-35rev.svg 630 indent2 vpad\r\n\nOn which day is the difference between the number of men and women registering closest to 55?"},{"varval":"Each interval = 20 people\r\n\nDifference needs to be slightly smaller than 3 intervals.\n\n$\\Rarr$ {{{correctAnswer}}} is closest."}]},{"vars":[{"varval":"This graph shows the megalitres of E10 and premium 98 fuel sold in NSW for the last six months of 2021.\n\n<br>\n\nsm_img https://teacher.smartermaths.com.au/wp-content/uploads/2022/09/Stat_NAPX-K2-35_v1_1.svg 630 indent2 vpad\r\n\nDuring which month is the difference between the megalitres of E10 and premium 98 fuel closest to 8 megalitres?"},{"varval":"Each interval = 5 megalitres\r\n\nDifference needs to be slightly smaller than 2 intervals.\n\n$\\Rarr$ {{{correctAnswer}}} is closest."}]},{"vars":[{"varval":"This graph shows the number of goals scored in basketball games by Mathew and Max over 5 games.\n\n<br>\n\nsm_img https://teacher.smartermaths.com.au/wp-content/uploads/2022/09/Stat_NAPX-K2-35_v2.svg 580 indent2 vpad\r\n\nOn which game is the difference between the number of goals scored by Mathew and Max more that 5 but less than 10?"},{"varval":"Each interval = 5 goals\r\n\nDifference needs to be more than 1 and less than 2 intervals.\n\n$\\Rarr$ {{{correctAnswer}}}"}]},{"vars":[{"varval":"This graph shows the comparison between the highest marks and the lowest marks over 5 mathematics tests for a Year 8 class.\n\n<br>\n\nsm_img https://teacher.smartermaths.com.au/wp-content/uploads/2022/09/Stat_NAPX-K2-35_v3_1.svg 550 indent vpad\r\n\nIn which test is the difference between the highest and lowest marks closest to 25?"},{"varval":"Each interval = 2.5 marks\r\n\nDifference needs to be around than 10 intervals.\n\n$\\Rarr$ {{{correctAnswer}}} is closest."}]},{"vars":[{"varval":"This graph shows the number of men and women that attended the emergency department of a regional hospital on different days of the week.\n\n<br>\n\nsm_img https://teacher.smartermaths.com.au/wp-content/uploads/2022/09/Stat_NAPX-K2-35_v4.svg 630 indent vpad\r\n\nOn which 2 days of the week is the difference between the number of men and women attending the emergency department the same?"},{"varval":"Each interval = 5 people\r\n\nDifference needs to be exactly the same.\n\n$\\Rarr$ {{{correctAnswer}}} difference = 5 people"}]},{"vars":[{"varval":"This graph shows the number of 17 year olds and 18 year olds getting their P-plates on different days of the week.\n\n<br>\n\nsm_img https://teacher.smartermaths.com.au/wp-content/uploads/2022/09/Stat_NAPX-K2-35_v5.svg 700 indent2 vpad\r\n\nOn which day is the difference between the number 17 year olds and 18 year olds getting their P-plates closest to 20?"},{"varval":"Each interval = 5 people\r\n\nDifference needs to be closest to 4 intervals.\n\n$\\Rarr$ {{{correctAnswer}}} is closest."}]}]

  357. <div class="sm_mode"> A bag of coloured balls contains 12 blue balls, 10 white balls, 6 black balls, 2 red balls, and 8 orange balls. Kimberly grabs a ball from the bag without looking. What is the probability that she grabs an orange ball? </div>

    [{"vars":null}]

  358. <div class="sm_mode"> The time spent by Mark playing video games on his computer is recorded in a table. <br> <div class="sm-table row1-color1 heading-color1"> >>Time Spent Playing Games >>| Day | Time | |:-:|:-:| | Monday | 2 hours | | Tuesday | 35 minutes | | Wednesday | 40 minutes | | Thursday | 55 minutes | | Friday | 63 minutes | | Saturday | 29 minutes | </div> <br>What was the average time per day that Mark spent playing video games over this period? </div>

    [{"vars":null}]

  359. <div class="sm_mode"> A store sells second hand mobile phones. The graph below shows the price of 2 similar second-hand phones. <br> sm_img https://teacher.smartermaths.com.au/wp-content/uploads/2021/04/Math-Job-Q32.svg 230 indent3 vpad <br>Which of the following is true based on the graph shown? </div>

    [{"vars":null}]

  360. Geometry, NAPX-p110293v01

    <div class="sm_mode"> {{{question}}} </div>

    [{"vars":[{"varval":"A quadrilateral FGHI is moved to a new position labelled LMJK.\n\nsm_img https://teacher.smartermaths.com.au/wp-content/uploads/2021/05/RAPH10_70.svg 340 indent vpad\n\nWhich of these processes resulted in the new position of the quadrilateral?"},{"varval":"sm_img https://teacher.smartermaths.com.au/wp-content/uploads/2021/05/RAPH10_70-sol.svg 340 indent vpad\n\nReflection in the $x$-axis\n\n\r\n$FGHI → F\\prime G\\prime H\\prime I\\prime$\n\n\r\nTranslate the triangle 9 units to the left\n\n\r\n$F\\prime G\\prime H\\prime I\\prime → KJML$"}]},{"vars":[{"varval":"The triangle ABC is moved to the new position labelled XYZ.\n\nsm_img https://teacher.smartermaths.com.au/wp-content/uploads/2021/05/RAPH10_69.svg 350 indent vpad\n\nWhich of these processes resulted in the new position of the triangle?"},{"varval":"sm_img https://teacher.smartermaths.com.au/wp-content/uploads/2021/05/RAPH10_69-sol.svg 350 indent vpad\n\nReflection in the $y$-axis:\n\n\r\n$ABC → A\\prime B\\prime C\\prime$\n\n\r\nTranslate the triangle 5 units downward:\n\n\r\n$A\\prime B\\prime C\\prime → XYZ$"}]}]

  361. I removed the italics from the multiple choice answers to match the image. Aside from that the only other problem is that the faces containing the B and D are not the right size. But they're close enough to get the idea as the question is about location of the triangle not the size of the shape.

    <div class="sm_mode"> Milo is drawing the net of this box. sm_img https://teacher.smartermaths.com.au/wp-content/uploads/2018/06/NAPX-E4-NC09-SA_1.svg 200 indent vpad Where should he draw the $\large \blacktriangle$ shape on the box? sm_img https://teacher.smartermaths.com.au/wp-content/uploads/2018/06/NAPX-E4-NC09-SA.svg 350 indent vpad </div>

    [{"vars":null}]

  362. Image in question did not match image in solution. Also there were 2 solutions that created a symmetrical pattern A and C with the original question.

    <div class="sm_mode"> Some shapes are missing in this pattern. sm_img https://teacher.smartermaths.com.au/wp-content/uploads/2022/11/Geom_NAPX-p107263v02_q.svg 230 indent3 vpad When completed, the pattern has one line of symmetry. Which of these could be the missing part of the pattern? </div>

    [{"vars":null}]

  363. Geometry, NAPX-p109101v01 Geometry, NAPX-F3-CA03 Var0 figure is red and blue was in solution. Var2 figure is blue and red was in question and solution.

    <div class="sm_mode"> {{{question}}} </div>

    [{"vars":[{"varval":"Troy built a solid figure using cubes.\n\nsm_img https://teacher.smartermaths.com.au/wp-content/uploads/2021/04/RAPH10-q38.svg 120 indent vpad\n\nHe paints all the outer sides red, including the base, and then separates the cubes.\n\n\r\n\r\nHow many faces are painted red?"},{"varval":"Number of faces painted red (top down, back to front)\n\n\r\n>>= 5 + 5 + 3 + 5 + 4 + 4 + 4\n\n\n\r\n>>= 30"}]},{"vars":[{"varval":"Sarah creates a solid figure using five cubes.\n\nsm_img https://teacher.smartermaths.com.au/wp-content/uploads/2021/04/RAPH10-q37.svg 120 indent vpad\n\nShe paints all the outer sides lilac, including the base, and then separates the cubes.\n\n\r\n\r\nHow many faces are painted lilac?"},{"varval":"Number of faces painted lilac (top down, back to front)\n\n\r\n>>= 5 + 4 + 4 + 4 + 5\n\n\r\n>>= 22"}]},{"vars":[{"varval":"There are six cubes in this 3D puzzle.\n\nsm_img https://teacher.smartermaths.com.au/wp-content/uploads/2020/09/NAPX-F3-CA03_1.svg 160 indent vpad\n\nThe puzzle is completely dipped into blue paint.\n\n\r\n\r\nWhen the cubes are separated, how many faces will be blue?"},{"varval":"The number of blue faces on each cube is:\n\nsm_img https://teacher.smartermaths.com.au/wp-content/uploads/2018/08/NAPX-F3-CA03-Answer1.svg 180 indent vpad\n\nsm_nogap $\\therefore$ Total faces blue\n\r\n\n>>= 5 + 5 + 3 + 4 + 4 + 5\r\n\n>>= 26 "}]}]

  364. Billy followed these instructions to create a drawing: <div style="background-color: lightyellow; padding: 5px 25px 20px 15px; border: 2px solid; width: max-content; margin-left: 30px" > * Draw a rectangle. * Use one of the sides to draw a triangle with 2 equal sides. * Draw a line of symmetry through your completed drawing. </div> <br>Which of these did he draw?

    [{"vars":null}]

  365. Could not really see anything wrong with this question/solution. Changed to "cubic units" in all options in worked solution from "cubes" eg 30 cubes -> 30 cubic units.

    <div class="sm_mode"> {{{question}}} </div>

    [{"vars":[{"varval":"Clarence made these solid prisms out of identical cubes.\n\n\r\n\r\nWhich prism has the largest volume?"},{"varval":"Volume of each prism:\n\n\r\nOption 1: 5 × 2 × 3 = 30 cubic units\n\n\r\nOption 2: 7 × 1 × 4 = 28 cubic units\n\n\r\nOption 3: 7 × 2 × 2 = 28 cubic units\n\n\r\nOption 4: 3 × 3 × 3 = 27 cubic units\r\n \r\n\r\n$\\therefore$ The prism with the largest volume is\n\n{{{correctAnswer}}}\n"}]}]

  366. <div class="sm_mode"> Charity has the picture of a panda on her phone. sm_img https://teacher.smartermaths.com.au/wp-content/uploads/2019/01/NAPX-G2-22.svg 200 indent vpad She makes the picture half as wide and twice as high. How will the picture look after she does that? </div>

    [{"vars":null}]

  367. <div class="sm_mode"> Romeo has a square piece of paper. He folds it in half twice and then cuts a piece off as shown below. <br> sm_img https://teacher.smartermaths.com.au/wp-content/uploads/2019/01/NAPX-G2-19v1.svg 320 indent vpad What will the square piece of paper look like after Romeo unfolds it? </div>

    [{"vars":null}]

  368. <div class="sm_mode"> {{{question}}} </div>

    [{"vars":[{"varval":"Mary stuck some different shapes onto the back window of her car.\n\r\nThey look like this from the outside.\n\n<br>\n\nsm_img https://teacher.smartermaths.com.au/wp-content/uploads/2019/01/NAPX-G2-16v1.svg 200 indent vpad\n\n<br>What do they look like from inside the car?"},{"varval":"Consider the triangle from outside the car:\n\nThe triangle is under each of the other 3 shapes.\n\nFrom the inside of the car:\n\nThe triangle will be visible in full. \n\n<br>\n\n{{{correctAnswer}}}\n"}]},{"vars":[{"varval":"Maeve stuck some different shapes onto the back window of her car.\n\r\nThey look like this from the outside.\n\n<br>\n\nsm_img https://teacher.smartermaths.com.au/wp-content/uploads/2022/10/Geom_NAPX-G2-16_v1.svg 250 indent vpad\n\n<br>What do they look like from inside the car?"},{"varval":"Consider the rectangle and hexagon from outside the car: \n\nThey are partially hidden by other shapes.\n\nFrom the inside of the car:\n\nThe rectangle and hexagon will be fully visible and in opposite positions.\n\n<br>\n\n{{{correctAnswer}}}\n"}]},{"vars":[{"varval":"Mitchell stuck some different shapes onto the back window of his car.\n\r\nThey look like this from the outside. \n\n<br>\n\nsm_img https://teacher.smartermaths.com.au/wp-content/uploads/2022/10/Geom_NAPX-G2-16_v2q.svg 250 indent vpad\n\n<br>What do they look like from inside the car?"},{"varval":"Consider the triangle from outside the car:\n\nThe triangle is fully visible.\n\nFrom the inside of the car:\n\nThe triangle will be partially covered by the star and the rectangle and flipped to the other side.\n\n<br>\n\n{{{correctAnswer}}}\n"}]},{"vars":[{"varval":"Mick stuck some different shapes onto the back window of his car.\n\r\nThey look like this from the outside.\n\n<br>\n\nsm_img https://teacher.smartermaths.com.au/wp-content/uploads/2022/10/Geom_NAPX-G2-16_v3q.svg 250 indent vpad\n\n<br>What do they look like from inside the car?"},{"varval":"Consider the arrows from outside the car:\n\nThe arrows are visible in full.\n\nFrom the inside of the car:\n\nThe arrows will be covered by both the shapes it overlaps and flipped to the other side.\n\n<br>\n\n{{{correctAnswer}}}\n"}]},{"vars":[{"varval":"Minnie stuck some different shapes onto the back window of her car.\n\r\nThey look like this from the outside.\n\n<br>\n\nsm_img https://teacher.smartermaths.com.au/wp-content/uploads/2022/10/Geom_NAPX-G2-16_v4q.svg 250 indent vpad\n\n<br>What do they look like from inside the car?"},{"varval":"Consider the triangle from outside the car:\n\nThe triangle is under both of the shapes it overlaps.\n\nFrom the inside of the car:\n\nThe triangle will be visible in full and all shapes will be in reverse order from left to right. \n\n<br>\n\n{{{correctAnswer}}}\n"}]},{"vars":[{"varval":"Montgomery stuck some different shapes onto the back window of his car.\n\r\nThey look like this from the outside.\n\n<br>\n\nsm_img https://teacher.smartermaths.com.au/wp-content/uploads/2022/10/Geom_NAPX-G2-16_v5q.svg 200 indent vpad\n\n<br>What do they look like from inside the car?"},{"varval":"Consider the oval from outside the car: \n\nThe oval is under each of the other 3 shapes.\n\nFrom the inside of the car:\n\nThe oval will be visible in full and all shapes will be in reverse order from left to right. \n\n<br>\n\n{{{correctAnswer}}}\n"}]}]

  369. <div class="sm_mode"> Inline svg test <svg xmlns:dc="http://purl.org/dc/elements/1.1/" xmlns:cc="http://creativecommons.org/ns#" xmlns:rdf="http://www.w3.org/1999/02/22-rdf-syntax-ns#" xmlns:svg="http://www.w3.org/2000/svg" xmlns="http://www.w3.org/2000/svg" xmlns:sodipodi="http://sodipodi.sourceforge.net/DTD/sodipodi-0.dtd" xmlns:inkscape="http://www.inkscape.org/namespaces/inkscape" width="51.983917mm" height="26.326054mm" viewBox="0 0 51.983917 26.326054" version="1.1" id="svg8" inkscape:version="0.92.2 (5c3e80d, 2017-08-06)" sodipodi:docname="nap-b4-nc12-b.svg"> <defs id="defs2"/> <sodipodi:namedview id="base" pagecolor="#ffffff" bordercolor="#666666" borderopacity="1.0" inkscape:pageopacity="0.0" inkscape:pageshadow="2" inkscape:zoom="1.979899" inkscape:cx="116.59832" inkscape:cy="41.738749" inkscape:document-units="mm" inkscape:current-layer="layer3" showgrid="false" inkscape:window-width="1366" inkscape:window-height="715" inkscape:window-x="-8" inkscape:window-y="22" inkscape:window-maximized="1"/> <metadata id="metadata5"> <rdf:RDF> <cc:Work rdf:about=""> <dc:format>image/svg+xml</dc:format> <dc:type rdf:resource="http://purl.org/dc/dcmitype/StillImage"/> <dc:title/> </cc:Work> </rdf:RDF> </metadata> <g inkscape:groupmode="layer" id="layer2" inkscape:label="Layer 2" transform="translate(-105.40175,-38.901821)" style="display:inline"> <g inkscape:label="Layer 1" inkscape:groupmode="layer" id="layer1"> <circle style="opacity:1;fill:none;fill-opacity:1;fill-rule:nonzero;stroke:#000000;stroke-width:0.39687499;stroke-miterlimit:4;stroke-dasharray:none;stroke-dashoffset:0;stroke-opacity:1" id="path10" cx="54.617561" cy="49.614582" r="16.819941"/> <rect style="opacity:1;fill:none;fill-opacity:1;fill-rule:nonzero;stroke:none;stroke-width:0.39687499;stroke-miterlimit:4;stroke-dasharray:none;stroke-dashoffset:0;stroke-opacity:1" id="rect862" width="37.618126" height="36.995808" x="36.045307" y="30.90476"/> <rect style="opacity:1;fill:none;fill-opacity:1;fill-rule:nonzero;stroke:none;stroke-width:0.39687499;stroke-miterlimit:4;stroke-dasharray:none;stroke-dashoffset:0;stroke-opacity:1" id="rect864" width="51.983921" height="26.326052" x="105.40175" y="38.901821"/> </g> <g id="g826" transform="translate(41.955357,-8.6934522)"> <circle r="11.358954" cy="60.867371" cx="76.038177" id="path817" style="opacity:1;fill:none;fill-opacity:1;fill-rule:nonzero;stroke:#000000;stroke-width:0.39687499;stroke-miterlimit:4;stroke-dasharray:none;stroke-dashoffset:0;stroke-opacity:1"/> <circle style="opacity:1;fill:none;fill-opacity:1;fill-rule:nonzero;stroke:#000000;stroke-width:0.39687499;stroke-miterlimit:4;stroke-dasharray:none;stroke-dashoffset:0;stroke-opacity:1" id="circle819" cx="102.49651" cy="60.867371" r="11.358954"/> <rect y="49.508415" x="75.904549" height="22.651093" width="27.061039" id="rect821" style="opacity:1;fill:none;fill-opacity:1;fill-rule:nonzero;stroke:#000000;stroke-width:0.39687499;stroke-miterlimit:4;stroke-dasharray:none;stroke-dashoffset:0;stroke-opacity:1"/> </g> <rect style="opacity:1;fill:#ffffff;fill-opacity:1;fill-rule:nonzero;stroke:none;stroke-width:0.40035436;stroke-miterlimit:4;stroke-dasharray:none;stroke-dashoffset:0;stroke-opacity:1" id="rect829" width="28.012352" height="22.26721" x="117.38788" y="41.015831"/> <ellipse style="opacity:1;fill:#ffffff;fill-opacity:1;fill-rule:nonzero;stroke:#000000;stroke-width:0.39687499;stroke-miterlimit:4;stroke-dasharray:none;stroke-dashoffset:0;stroke-opacity:1" id="path852" cx="53.519558" cy="99.176598" rx="23.786989" ry="12.027128"/> <rect style="opacity:1;fill:#ffffff;fill-opacity:1;fill-rule:nonzero;stroke:#000000;stroke-width:0.39687499;stroke-miterlimit:4;stroke-dasharray:none;stroke-dashoffset:0;stroke-opacity:1" id="rect860" width="48.002975" height="24.190477" x="114.52678" y="86.845238"/> </g> <g inkscape:groupmode="layer" id="layer3" inkscape:label="Layer 3" style="display:inline" transform="translate(1.3427734e-6)"> <g id="g833"> <path sodipodi:nodetypes="cssscsc" inkscape:connector-curvature="0" id="path25" d="m 12.402411,2.3770818 c 0,0 23.224594,-0.4459326 27.706328,0 3.652261,0.3634001 10.121947,3.4068306 9.9792,11.3607082 -0.132115,7.361406 -6.324614,10.104952 -9.9792,10.444667 -1.927976,0.179217 -27.706328,0 -27.706328,0 0,0 -10.7300455,-0.702431 -10.7061602,-10.902688 C 1.719654,3.285371 12.402411,2.3770818 12.402411,2.3770818" style="fill:#ffffff;fill-opacity:1;stroke:none;stroke-width:0.260391px;stroke-linecap:butt;stroke-linejoin:miter;stroke-opacity:1"/> </g> <rect style="opacity:1;fill:none;fill-opacity:1;fill-rule:nonzero;stroke:none;stroke-width:0.39687499;stroke-miterlimit:4;stroke-dasharray:none;stroke-dashoffset:0;stroke-opacity:1" id="rect835" width="51.983917" height="26.326054" x="-1.3427734e-006" y="1.3647461e-007"/> </g> </svg> </div>

    [{"vars":null}]

  370. Geometry, NAPX-p122209v01

    <div class="sm_mode"> {{{question}}} </div>

    [{"vars":[{"varval":"Kwan uses some blocks to make a cube and a rectangular prism.\n\n<br>\n\nsm_img https://teacher.smartermaths.com.au/wp-content/uploads/2019/01/NAPX-K2-17v3.svg 300 indent3 vpad\n\n<br>How many blocks does Kwan use altogether?"},{"varval":"Blocks in cube = 3 × 3 × 3 = 27\n\n\r\nBlocks in rectangular prism = 2 × 3 × 3 = 18\n\n<div class=\"aligned\">\n\n|||\n|-|-|\n|$\\therefore$ Total blocks|= 27 + 18|\n||= 45|\n\r\n\n</div>"}]},{"vars":[{"varval":"Kelly uses some blocks to make the two prisms below.\n\n<br>\n\nsm_img https://teacher.smartermaths.com.au/wp-content/uploads/2019/01/NAPX-K2-17v2.svg 300 indent3 vpad\n\n<br>How many blocks does Kelly use altogether?"},{"varval":"Blocks in cube = 3 × 3 × 3 = 27\n\n\r\nBlocks in 2nd object (step) = 5 × 3 = 15\n\n<div class=\"aligned\">\n\n|||\n|-|-|\n|$\\therefore$ Total blocks|= 27 + 15|\n||= 42|\n\r\n\n</div>"}]},{"vars":[{"varval":"Faraji uses some blocks to make the two prisms below.\n\n<br>\n\nsm_img https://teacher.smartermaths.com.au/wp-content/uploads/2022/10/Geom_NAPX-p122209v02_v2.svg 320 indent3 vpad\n\n<br>How many blocks does Faraji use altogether?"},{"varval":"Blocks in cube = 4 × 4 × 4 = 64\n\n\r\nBlocks in 2nd object (step) = 8 × 3 = 24\n\n<div class=\"aligned\">\n\n|||\n|-|-|\n|$\\therefore$ Total blocks|= 64 + 24|\n||= 88|\n\r\n\n</div>"}]},{"vars":[{"varval":"Taraji uses some blocks to make the two prisms below.\n\n<br>\n\nsm_img https://teacher.smartermaths.com.au/wp-content/uploads/2022/10/Geom_NAPX-p122209v02_v3q.svg 300 indent3 vpad\n\n<br>How many blocks does Taraji use altogether?"},{"varval":"Blocks in rectangular prism = 4 × 3 × 3 = 36\n\n\r\nBlocks in prism (U-shaped) = 10 × 3 = 30\n\n<div class=\"aligned\">\n\n|||\n|-|-|\n|$\\therefore$ Total blocks|= 36 + 30|\n||= 66|\n\r\n\n</div>"}]},{"vars":[{"varval":"Calista uses some blocks to make the two prisms below.\n\n\n<br>\n\nsm_img https://teacher.smartermaths.com.au/wp-content/uploads/2022/10/Geom_NAPX-p122209v02_v4.svg 300 indent3 vpad\n\n<br>How many blocks does Calista use altogether?"},{"varval":"Blocks in first prism (E-shaped) = 8 × 4 = 32\n\n\r\nBlocks in rectangular prism = 3 × 6 × 4 = 72\n\n<div class=\"aligned\">\n\n|||\n|-|-|\n|$\\therefore$ Total blocks|= 32 + 72|\n||= 104|\n\r\n\n</div>"}]},{"vars":[{"varval":"Moses uses some blocks to make a cube and an F-shaped prism.\n\n<br>\n\nsm_img https://teacher.smartermaths.com.au/wp-content/uploads/2022/10/Geom_NAPX-p122209v02_v5.svg 370 indent3 vpad\n\n<br>How many blocks does Moses use altogether?"},{"varval":"Blocks in cube = 3 × 3 × 3 = 27\n\n\r\nBlocks in F-shaped prism = 15 × 2 = 30\n\n<div class=\"aligned\">\n\n|||\n|-|-|\n|$\\therefore$ Total blocks|= 27 + 30|\n||= 57|\n\r\n\n</div>"}]}]

  371. <div class="sm_mode"> Celine sticks a pin in a fishing hook pictured below. <br> sm_img https://teacher.smartermaths.com.au/wp-content/uploads/2018/12/NAPX-K2-13v2.svg 160 indent2 vpad <br>She then rotates the fishing hook a quarter turn anti-clockwise. Which of the following shows Celine's hook after the rotation? </div>

    [{"vars":null}]

  372. Corrected first sentence.

    <div class="sm_mode"> A shape of an arrow head is folded in half along the dotted line. sm_img https://teacher.smartermaths.com.au/wp-content/uploads/2021/04/RAPH10-q41.svg 200 indent vpad What geometrical figure is the folded shape? </div>

    [{"vars":null}]

  373. <div class="sm_mode"> <div class="sm_img_inline"> <img src="https://teacher.smartermaths.com.au/wp-content/uploads/2021/04/RAPH10_shape2.png" style="height: 30px; width: 30px;"> = <img src="https://teacher.smartermaths.com.au/wp-content/uploads/2021/04/RAPH10_shape2.png" style="height: 30px; width: 30px;"> x 3 - 2 </div> </div>

    [{"vars":null}]

  374. <div class="sm_mode"> {{{question}}} </div>

    [{"vars":[{"varval":"The picture graph shows how many tonnes of concrete are needed for 4 jobs.\n\n<br>\n\nsm_img https://teacher.smartermaths.com.au/wp-content/uploads/2019/01/NAPX-K2-23v3.svg 350 indent vpad\r\n\n<br>How many more tonnes of concrete does Job 1 need than Job 3?"},{"varval":"\nJob 1 needs 9 tonnes\r.\n\nJob 3 needs 7 tonnes.\n\n<div class=\"aligned\">\r\n\r\n| | |\r\n| ------------- | ---------- |\r\n| $\\therefore$ Extra tonnes | \\= $9 \\ − \\ 7$ |\r\n| | \\= {{{correctAnswer0}}} |\r\n\r\n</div>\r"}]},{"vars":[{"varval":"The goals scored by 4 players in a season of soccer were recorded in the graph below.\n\n<br>\n\nsm_img https://teacher.smartermaths.com.au/wp-content/uploads/2018/12/NAPX-H2-26-SA-v2.svg 250 indent2 vpad\r\n\n\n<br>Simon scored 6 goals in the season.\n\nGigi scored 3 goals.\n\nHow many **more** goals did Henry score than Fiona?\n"},{"varval":"Since Simon scored 6 goals\n\n$\\rightarrow$ 1 soccer ball = 2 goals\n\nHenry scored 8 goals.\n\nFi scored 1 goal.\n\n$\\therefore$ Henry scored {{{correctAnswer0}}} more goals than Fiona."}]},{"vars":[{"varval":"The goals scored by 4 players in a season of soccer were recorded in the graph below.\n\n<br>\n\nsm_img https://teacher.smartermaths.com.au/wp-content/uploads/2018/12/NAPX-H2-26-SA.svg 250 indent2 vpad\r\n\n<br>Fiona scored 8 goals in the season.\n\nSimon scored 5 goals.\n\nHow many **more** goals did Henry score than Gigi?"},{"varval":"Since Fiona scored 8 goals\n\n$\\rightarrow$ 1 soccer ball = 2 goals\n\nHenry scored 10 goals.\n\nGigi scored 5 goals.\n\n$\\therefore$ Henry scored {{{correctAnswer0}}} more goals than Gigi."}]}]

  375. <div class="sm_mode"> {{{question}}} </div>

    [{"vars":[{"varval":"Harley has bag that contains 32 balls that are coloured either black or white.\r\n\r\nHarley chose a ball without looking and had the chance of 3 in 8 of choosing a white ball.\r\n\r\nHow many black balls are in Harley's bag?"},{"varval":"<div class=\"aligned\">\r\n\r\n| | |\r\n| ------------: | ---------- |\r\n| Number of white balls | \\= $\\dfrac{3}{8} \\ \\times 32$ |\r\n| | \\= 12 |\r\n\r\n</div>\r\n\n<br>\n\n<div class=\"aligned\">\r\n\r\n| | |\r\n| ------------- | ---------- |\r\n| Number of black balls | \\= Total balls – white balls |\n| | \\= $32 \\ − \\ 12$ |\r\n| | \\= {{{correctAnswer0}}} |\r\n\r\n</div>\r"}]},{"vars":[{"varval":"Ginger has a box of 48 ice-blocks in her fridge. Some ice-blocks have a prize on their paddle pop stick.\n\r\nGinger chooses one without looking and has a one in six chance of winning a prize.\n\r\nHow many ice-blocks in the box do not have a prize?"},{"varval":"<div class=\"aligned\">\r\n\r\n| | |\r\n| ------------: | ---------- |\r\n| Ice-blocks with prizes | \\= $\\dfrac{1}{6} \\ \\times 48$ |\r\n| | \\= 8 |\r\n\r\n</div>\r\n\n<br>\n\n<div class=\"aligned\">\r\n\r\n| | |\r\n| ------------- | ---------- |\r\n| Ice-blocks with no prize | \\= $48 \\ − \\ 8$ |\r\n| | \\= {{{correctAnswer0}}} |\r\n\r\n</div>\r"}]},{"vars":[{"varval":"Dave buys tickets in the meat raffle at the RSL club.\n\nA total of 56 tickets were sold and Dave has a 2 in 7 chance of winning a prize.\n\r\nHow many tickets sold will not win a prize?"},{"varval":"<div class=\"aligned\">\r\n\r\n| | |\r\n| ------------: | ---------- |\r\n| Number of winning tickets| \\= $\\dfrac{2}{7} \\ \\times 56$ |\r\n| | \\= 16 |\r\n\r\n</div>\r\n\n<br>\n\n<div class=\"aligned\">\r\n\r\n| | |\r\n| ------------- | ---------- |\r\n| Losing tickets | \\= Total tickets – winning tickets|\n| | \\= $56 \\ − \\ 16$ |\r\n| | \\= {{{correctAnswer0}}} |\r\n\r\n</div>\r"}]},{"vars":[{"varval":"Karen collects 81 grasshoppers for a research project.\n\nIf Karen chooses a grasshopper at random, there is a 2 in 3 chance it is a female.\n\r\nHow many male grasshoppers did Karen collect?"},{"varval":"<div class=\"aligned\">\r\n\r\n| | |\r\n| ------------: | ---------- |\r\n| Number of females| \\= $\\dfrac{2}{3} \\ \\times 81$ |\r\n| | \\= 54 |\r\n\r\n</div>\r\n\n<br>\n\n<div class=\"aligned\">\r\n\r\n| | |\r\n| ------------- | ---------- |\r\n| Number of males | \\= 81 – 54|\n| | \\= {{{correctAnswer0}}} |\r\n\r\n</div>\r"}]},{"vars":[{"varval":"Murray exhibits 54 of his best photos at an art gallery. His photos are either landscape photos or portraits.\n\nIf a visitor goes to the exhibition and looks at one of Murray's photos, there is a 2 in 9 chance it is a portrait.\n\r\nHow many landscape photos is Murray exhibiting at the gallery?"},{"varval":"<div class=\"aligned\">\r\n\r\n| | |\r\n| ------------: | ---------- |\r\n| Number of portraits| \\= $\\dfrac{2}{9} \\ \\times 54$ |\r\n| | \\= 12 |\r\n\r\n</div>\r\n\n<br>\n\n<div class=\"aligned\">\r\n\r\n| | |\r\n| ------------- | ---------- |\r\n| Number landscapes | \\= 54 – 12|\n| | \\= {{{correctAnswer0}}} |\r\n\r\n</div>\r"}]}]

  376. <div class="sm_mode"> The table below shows the prices of different kinds of fruit sold at a supermarket. <br> sm_img https://teacher.smartermaths.com.au/wp-content/uploads/2021/05/Math-Job-11-27.svg 620 indent vpad <br>Which type of fruit is the most expensive per kilogram? </div>

    [{"vars":null}]

  377. Number, NAPX-p107088v01 SA

    <div class="sm_mode"> {{{question}}} </div>

    [{"vars":[{"varval":"The product of three different positive whole numbers is 48.\n\nWhat is the smallest possible sum of the three different numbers?"},{"varval":"Testing possible combinations:\n\n48 = 24 × 2 × 1, sum = 27\n\n48 = 12 × 4 × 1, sum = 17\n\n48 = 8 × 6 × 1, sum = 15\n\n48 = 8 × 3 × 2, sum = 13\r\n\n48 = 6 × 4 × 2, sum = 12\n\n \r\n$\\therefore$ The smallest possible sum is {{{correctAnswer0}}}, made up of 6, 4 and 2 OR ."}]},{"vars":[{"varval":"The product of three different positive whole numbers is 60.\n\n\r\n\r\nWhat is smallest possible sum of the three different numbers?"},{"varval":"Testing some possible combinations:\n\n\r\n60 = 30 × 2 × 1, sum = 33\n\n\r60 = 10 × 3 × 2, sum = 15\n\n60 = 6 × 5 × 2, sum = 13\n\n60 = 5 × 4 × 3, sum = 12\r\n\n\r\n$\\therefore$ The smallest possible numbers are 5, 4 and 3, with a sum of {{{correctAnswer0}}}."}]},{"vars":[{"varval":"The product of three different positive whole numbers is 56.\n\n\r\n\r\nWhat is smallest possible sum of the three different numbers?"},{"varval":"Testing some possible combinations:\n\n\r\n56 = 28 × 2 × 1, sum = 31\n\n56 = 14 × 4 × 1, sum = 19\n\n56 = 8 × 7 × 1, sum = 16\n\n56 = 7 × 4 × 2, sum = 13\r\n \r\n\r\n$\\therefore$ The smallest possible numbers are 7, 4 and 2, with a sum of {{{correctAnswer0}}}."}]},{"vars":[{"varval":"The product of three different positive whole numbers is 42.\n\n\r\n\r\nWhat is smallest possible sum of the three different numbers?"},{"varval":"Testing some possible combinations:\n\n\r\n42 = 21 × 2 × 1, sum = 24\n\n\r42 = 14 × 3 × 1, sum = 18\n\n42 = 7 × 6 × 1, sum = 14\n\n42 = 7 × 3 × 2, sum = 12\r\n \r\n\r\n$\\therefore$ The smallest possible numbers are 7, 3 and 2, with a sum of {{{correctAnswer0}}}."}]},{"vars":[{"varval":"The product of three different positive whole numbers is 36.\n\n\r\n\r\nWhat is smallest possible sum of the three different numbers?"},{"varval":"Testing some possible combinations:\n\n36 = 18 × 2 × 1, sum = 21\n\n36 = 12 × 3 × 1, sum = 16\n\n36 = 9 × 4 × 1, sum = 14\n\n36 = 6 × 3 × 2, sum = 11\n\n$\\therefore$ The smallest possible numbers are 6, 3 and 2, with a sum of {{{correctAnswer0}}}."}]},{"vars":[{"varval":"The product of three different positive whole numbers is 40.\n\n\r\n\r\nWhat is smallest possible sum of the three different numbers?"},{"varval":"Testing some possible combinations:\n\n\r\n40 = 20 × 2 × 1, sum = 23\n\n40 = 10 × 4 × 1, sum = 15\n\n\r40 = 8 × 5 × 1, sum = 14\n\n40 = 5 × 4 × 2, sum = 11\n\n\n$\\therefore$ The smallest possible numbers are 5, 4 and 2, with a sum of {{{correctAnswer0}}}."}]}]

  378. var: Algebra, NAPX-p109338v01

    <div class="sm_mode"> {{{question}}} </div>

    [{"vars":[{"varval":"<div class=\"sm_img_inline\">\n\nThe table shown below represents a rule of changing each ![](https://teacher.smartermaths.com.au/wp-content/uploads/2021/05/RAPH10_Q54-i.svg) into a ![](https://teacher.smartermaths.com.au/wp-content/uploads/2021/05/RAPH10_Q54-ii.svg).\n</div>\n\n<br>\n\n>>sm_img https://teacher.smartermaths.com.au/wp-content/uploads/2021/05/RAPH10_Q54.svg 230 indent vpad\r\n\nWhich rule is used?"},{"varval":"Consider option 1:\r\n\n$5 \\times 4\\ −\\ 1 = 19$ &nbsp;$\\checkmark$\n\n$7 \\times 4\\ −\\ 1 = 27$ &nbsp;$\\checkmark$\n\n$9 \\times 4\\ −\\ 1 = 35$ &nbsp;$\\checkmark$\n\n{{{correctAnswer}}}"}]},{"vars":[{"varval":"<div class=\"sm_img_inline\" style=\"height: 20px\">\n\nThe table below shows the results of a rule that changes each \r<img src=\"https://teacher.smartermaths.com.au/wp-content/uploads/2021/04/RAPH10_shape1.png\" style=\"height: 40px;\"> into a <img src=\"https://teacher.smartermaths.com.au/wp-content/uploads/2021/04/RAPH10_shape2.png\" style=\"height: 50px;\">.\n\n</div>\n\r\n<br>\n\n>>sm_img https://teacher.smartermaths.com.au/wp-content/uploads/2021/04/RAPH10_Q53.svg 240 indent vpad\r\n\nWhich rule is used?"},{"varval":"Consider option 1:\r\n\n$2 \\times 4\\ −\\ 4 = 4$ &nbsp;$\\checkmark$\n\n$4 \\times 4\\ −\\ 4 = 12$ &nbsp;x\n\n<br>\n\nConsider option 2:\r\n\n$2 \\times 2\\ −\\ 2 = 4$ &nbsp;x\n\n<br>\n\nConsider option 3:\r\n\n$2 \\times 3\\ −\\ 2 = 4$ &nbsp;$\\checkmark$\n\n$4 \\times 3\\ −\\ 2 = 10$ &nbsp;$\\checkmark$\n\n$6 \\times 3\\ −\\ 2 = 16$ &nbsp;$\\checkmark$\n\n<br>\n\n\n{{{correctAnswer}}}"}]}]

  379. <div class="sm_mode"> {{{question}}} </div>

    [{"vars":[{"varval":"Which of these numbers has 3 in the hundredths place?"},{"varval":"Digits in the decimal places are (in order):\n\nTenths, hundredths, thousandths, ...\n\n$\\therefore$ 12.43 is correct."}]},{"vars":[{"varval":"Which of these numbers has 7 in the thousandths place?"},{"varval":"Digits in the decimal places are (in order):\n\nTenths, hundredths, thousandths, ...\n\n$\\therefore$ 12.537 is correct."}]},{"vars":[{"varval":"Which of these numbers has 4 in the tenths place?"},{"varval":"Digits in the decimal places are (in order):\n\nTenths, hundredths, thousandths, ...\n\n$\\therefore$ 2.401 is correct."}]},{"vars":[{"varval":"Which of these numbers has 2 in the thousandths place?"},{"varval":"Digits in the decimal places are (in order):\n\nTenths, hundredths, thousandths, ...\n\n$\\therefore$ 83.4128 is correct."}]},{"vars":[{"varval":"Which of these numbers has 1 in the hundredths place?"},{"varval":"Digits in the decimal places are (in order):\n\nTenths, hundredths, thousandths, ...\n\n$\\therefore$ 83.4128 is correct."}]},{"vars":[{"varval":"Which of these numbers has 9 in the ten thoundandths place?"},{"varval":"Digits in the decimal places are (in order):\n\nTenths, hundredths, thousandths, ten thousandths ...\n\n$\\therefore$ 2.08497 is correct."}]}]

  380. <div class="sm_mode"> The parallelogram shown below has width ($W$) and length ($L$). <br> sm_img https://teacher.smartermaths.com.au/wp-content/uploads/2021/04/RAPH10-q47.svg 300 indent3 vpad <br>Which expression can be used as a perimeter of this figure? </div>

    [{"vars":null}]

  381. variants: Algebra, NAPX-K2-24

    <div class="sm_mode"> {{{question}}} </div>

    [{"vars":[{"varval":"\r\nA netball club receives 6 boxes containing 42 netballs each.\r\n\r\nIt then gives 7 balls to every team in the club and there are no balls left over.\r\n\r\nHow many teams are in the club?"},{"varval":"\r\nOne box (42 balls) supplies 6 teams with 7 balls each.\n\nsm_nogap $\\therefore$ Teams supplied by 6 boxes\n\n<div class=\"aligned\">\r\n\r\n>>| | |\r\n| ------------- | ---------- |\r\n| | \\= $6 \\times 6$ |\r\n| | \\= {{{correctAnswer}}} |\r\n\r\n</div>\r\n"}]},{"vars":[{"varval":"A soccer club receives 7 boxes containing 32 soccer balls each.\r\n\r\nIt then gives 8 soccer balls to every team in the club and there are no balls left over.\r\n\r\nHow many teams are in the club?"},{"varval":"One box (32 balls) supplies 4 teams with\r 8 balls each.\n\nsm_nogap $\\therefore$ Teams supplied by 7 boxes\n\n<div class=\"aligned\">\r\n\r\n>>| | |\r\n| ------------- | ---------- |\r\n| | \\= $7 \\times 4$ |\r\n| | \\= {{{correctAnswer}}} |\r\n\r\n</div>\r\n"}]}]

  382. Number, NAPX-p115611v03 SA Number, NAPX-p115611v01 SA

    <div class="sm_mode"> {{{question}}} </div>

    [{"vars":[{"varval":"A 5-cent coin has a mass of 3.1 grams.\n\n\r\n\r\nA 20-cent coin has a mass of 5.3 grams.\n\n\r\n\r\nJonathan has 198.4 grams of 5-cent coins.\n\n\r\n\r\nKevin has 964.6 grams of 20-cent coins.\n\n\r\n\r\nHow much more worth of coins does Kevin have than Jonathan?"},{"varval":"<div class=\"aligned\">\n\n|||\n|-|-|\n|Jonathan's total|= $\\dfrac{\\text{Total mass of coins}}{\\text{mass of 1 coin}} \\times$ 5 cents|\n| | |\n||= $\\dfrac{198.4}{3.1} \\times 5$|\n||= 64 × 5|\n||= 320 cents|\n\n</div>\n\n<br>\n\n<div class=\"aligned\">\n\n|||\n|-|-|\n|Kevin's total|= = $\\dfrac{964.6}{5.3} \\times 20$|\n||= 182 × 20|\n||= 3640 cents|\n\n</div>\n\n<br>\n\n<div class=\"aligned\">\n\n|||\n|-|-|\n|$\\therefore$ Difference|= 3640 $-$ 320|\n||= 3320 cents|\n||= {{{prefix0}}}{{{correctAnswer0}}}|\n\n</div>"}]},{"vars":[{"varval":"A 10-cent coin has a mass of 6.2 grams.\n\n\r\n\r\nA 5-cent coin has a mass of 2.9 grams.\n\n\r\n\r\nRico has 347.2 grams of 10-cent coins.\n\n\r\n\r\nJester has 301.6 grams of 5-cent coins.\n\n\r\n\r\nHow much more worth of coins does Rico have than Jester, in cents?\n"},{"varval":"<div class=\"aligned\">\n\n|||\n|-|-|\n|Rico's total|= $\\dfrac{\\text{Total mass of coins}}{\\text{mass of 1 coin}} \\times$ 10 cents|\n| | |\n||= $\\dfrac{347.2}{6.2} \\times 10$|\n||= 56 × 10|\n||= 560 cents|\n\n</div>\n\n<br>\n\n<div class=\"aligned\">\n\n|||\n|-|-|\n|Jester's total|= $\\dfrac{301.6}{2.9} \\times 5$|\n||= 104 × 5|\n||= 520 cents|\n\n</div>\n\n<br>\n\n<div class=\"aligned\">\n\n|||\n|-|-|\n|$\\therefore$ Difference|= 560 $-$ 520|\n||= 40 cents|\n\n</div>"}]},{"vars":[{"varval":"A 20-cent coin has a mass of 12.5 grams.\n\n\r\n\r\nA 5-cent coin has a mass of 2.9 grams.\r\n\r\nHarold has 487.5 grams of 20-cent coins.\n\n\r\n\r\nRick has 179.8 grams of 5-cent coins.\n\n\r\n\r\nHow much more worth of coins does Harold have than Rick, in cents?\n"},{"varval":"<div class=\"aligned\">\n\n|||\n|-|-|\nHarold's total|= $\\dfrac{\\text{Total mass}}{\\text{mass 1 coin}} \\times$ 20 cents|\n| | |\n||= $\\dfrac{487.5}{12.5} \\times 20$|\n||= 39 × 20|\n||= 780 cents|\n\n</div>\n\n<br>\n\n<div class=\"aligned\">\n\n|||\n|-|-|\n|Rick's total|= $\\dfrac{179.8}{2.9} \\times 5$|\n||= 62 × 5|\n||= 310 cents|\n\n</div>\n\n<br>\n\n<div class=\"aligned\">\n\n|||\n|-|-|\n|$\\therefore$ Extra money|= 780 $-$ 310|\n||= 470 cents|\n\n</div>"}]}]

  383. <div class="sm_mode"> {{{question}}} </div>

    [{"vars":[{"varval":"Norman started cycling to stay fit.\r\n\r\nThe table below shows the distance he cycles on his rides.\n\n<br>\n\n<div class=\"sm-table col1-color1\">\n\n>>| **Bike Ride**| 1|2|3|4|5|\n|:-:|:-:|:-:|:-:|:-:|:-:|\n| **Distance (km)** | 4|6|10|18|**?**|\n\n</div>\n\n<br>The distance he cycles increases each ride and follows the rule:\r\n\r\n>> Double the last distance and deduct 2.\n\n<br>\n\n\r\n\r\nWhat is the distance travelled by Norman on his 5th ride?"},{"varval":"Using the rule:\n\n<div class=\"aligned\">\r\n\r\n| | |\r\n| ------------- | ---------- |\r\n| Distance of 5th ride | \\= $(18 \\times 2) \\ - \\ 2$ |\r\n| | \\= 36 $-$ 2 |\n| | \\= {{{correctAnswer}}} |\r\n\r\n</div>\r"}]},{"vars":[{"varval":"Deek runs to stay fit.\r\n\r\nThe table below shows Deek's longest run each month over a period of 5 months.\n\n<br>\n\n<div class=\"sm-table col1-color1\">\n\n>>| **Month**| 1|2|3|4|5|\n|:-:|:-:|:-:|:-:|:-:|:-:|\n| **Longest distance (km)** | 5|6|8|12|**?**|\n\n</div>\n\n<br>Deek's longest run each month follows the rule:\r\n\r\n>> Double the last month's longest run and deduct 4.\n\n<br>\n\n\r\n\r\nWhat was Deek's longest run in month 5?"},{"varval":"Using the rule:\n\n<div class=\"aligned\">\r\n\r\n| | |\r\n| ------------- | ---------- |\r\n| Longest run in month 5 | \\= $(12 \\times 2) \\ - \\ 4$ |\r\n| | \\= 24 $-$ 4 |\n| | \\= {{{correctAnswer}}} |\r\n\r\n</div>\r"}]},{"vars":[{"varval":"Susie is training to swim the English channel.\n\r\nThe table below shows Susie's longest swim each month over a period of 5 months.\n\n<br>\n\n<div class=\"sm-table col1-color1\">\n\n>>| **Month**| 1|2|3|4|5|\n|:-:|:-:|:-:|:-:|:-:|:-:|\n| **Longest swim (km)** | 2|3|6|15|**?**|\n\n</div>\n\n<br>Susie's longest swim each month follows the rule:\r\n\r\n>> Triple last month's longest swim and deduct 3.\n\n<br>\n\r\nWhat distance was Susie's longest swim in month 5?"},{"varval":"Using the rule:\n\n<div class=\"aligned\">\r\n\r\n| | |\r\n| ------------- | ---------- |\r\n| Longest swim in month 5 | \\= $(15 \\times 3) \\ - \\ 3$ |\r\n| | \\= 45 $-$ 3 |\n| | \\= {{{correctAnswer}}} |\r\n\r\n</div>\r"}]}]

  384. <div class="sm_mode"> Lucy sees this signpost at Snoopy World. <br> sm_img https://teacher.smartermaths.com.au/wp-content/uploads/2019/01/NAPX-J2-41-SA-v1.svg 410 indent vpad How many metres is it from the car park to Linus Lane? </div>

    [{"vars":null}]

  385. <div class="sm_mode"> {{{question}}} </div>

    [{"vars":[{"varval":"A cricket kit has white, pink and red cricket balls.\r\n\r\nThere are:\n\n\r\n\r\n* 4 more white balls than pink\n\n\r\n* 9 fewer pink balls than red\n\n\r\n* 11 red balls\n\n\r\nHow many balls, in total, are in the kit?"},{"varval":"Red balls = 11 (given)\n\n\r\n⇒ Pink balls = 11 − 9 = 2\n\n\r\n⇒ White balls = 2 + 4 = 6\n\n<div class=\"aligned\">\n\n|||\n|-|-|\n|$\\therefore$ Total balls|= 11 + 2 + 6|\n||= {{{correctAnswer}}}|\n\n</div>"}]},{"vars":[{"varval":"\nSergio's golf bag contains white, pink and yellow golf balls.\n\nThere are:\n\n* 6 more yellow balls than pink balls\r\n* 8 fewer pink balls than white balls\r\n* 13 white balls\n\n\r\nHow many golf balls, in total, are in Sergio's bag?"},{"varval":"13 white balls (given)\n\n\r\n⇒ 13 − 8 = 5 pink balls\n\n\r\n⇒ 5 + 6 = 11 yellow balls\n\n<div class=\"aligned\">\n\n|||\n|-|-|\n|$\\therefore$ Total balls| = 13 + 5 + 11|\n||= {{{correctAnswer}}}|\n\n</div>"}]}]

  386. Number, NAPX-p168435v01

    <div class="sm_mode"> {{{question}}} </div>

    [{"vars":[{"varval":"\r\nA student needs 12 folder dividers for each subject and is enrolled in 5 subjects.\n\nA school supply store sells the folder dividers in packets of 8.\n\nHow many packets should the student buy?"},{"varval":"Dividers required = 12 × 5 = 60\n\n<div class=\"aligned\">\n\n|||\n|-|-|\n|$\\therefore$ Packets required| = $\\dfrac{60}{8}$|\n||= 7 remainder 4|\n||= 8 packets (round up) |\n\n</div>"}]},{"vars":[{"varval":"William needs 4 eggs for each cake he will bake.\n\n\r\n\r\nHe wants to make 12 cakes.\n\n\r\n\r\nA certain store sells eggs in bags of 5.\n\n\r\n\r\nHow many bags must he buy in order to make 12 cakes?"},{"varval":"Eggs required = 4 × 12 = 48\n\n<div class=\"aligned\">\n\n|||\n|-|-|\n|$\\therefore$ Bags required| = $\\dfrac{48}{5}$|\n||= 9 remainder 3|\n||= 10 packets (round up) |\n\n</div>"}]}]

  387. <div class="sm_mode"> Five people had sticks of different lengths. Each person measured an 18 metre distance by counting how many stick lengths it took. The results were put into the table below. <br> <div class="sm-table row1-color8"> >>| Person | Stick lengths | |:-:|:-:| | John | 14| | Caroline | 18| | Paul | 36| | Simon | 42| | Ken | 45| </div> <br>Which person had a stick that was 40 centimetres long? </div>

    [{"vars":null}]

  388. <div class="sm_mode"> Eisha won a jar of candy with 366 pieces of candy in it at her school fete. She gave a hand full each to five of her friends. Eisha then counted the remaining candy and found she had 232 pieces left. How many pieces of candy did Eisha give to her friends? </div>

    [{"vars":null}]

  389. variants: Number, NAPX-p111573v01 Algebra, NAPX-p168204v02

    <div class="sm_mode"> {{{question}}} </div>

    [{"vars":[{"varval":"320 $\\div$ <span class=\"sm-text color3\">?</span> = 40\r\n\n\nWhich of the following numbers make the number sentence above correct?"},{"varval":"Check each option:\r\n\nOption 1 - $320 \\div 8=40$ &nbsp;$\\checkmark$\n\nOption 2 - $320 \\div 16=20$\r &nbsp;x\n\nOption 3 - $320 \\div 80=4$\r &nbsp;x\n\nOption 4 - $320 \\div 20=16$\r &nbsp;x\n \r\n\r\n$\\therefore$ 8 makes the number sentence correct."}]},{"vars":[{"varval":"7 $\\times$ <span class=\"sm-text color3\">?</span>\n= 161\r\n\nWhat number will make the number sentence correct?"},{"varval":"Check each option\r\n\nOption 1: &nbsp;$7 \\times 13=91$\r &nbsp;x\n\nOption 2: &nbsp;$7 \\times 23=161$ &nbsp;$\\checkmark$\n\nOption 3: &nbsp;$7 \\times 154=1078$\r &nbsp;x\n\nOption 4: &nbsp;$7 \\times 161=1127$ &nbsp;x"}]}]

  390. <div class="sm_mode"> {{{question}}} </div>

    [{"vars":[{"varval":"Ziggy throws darts at a board split into 4 sections and records which section they hit.\n\n<br>\n\nsm_img https://teacher.smartermaths.com.au/wp-content/uploads/2018/04/NAPX-J4-CA34.svg 380 indent vpad\r\n\n<br>Based on Ziggy's data, what is the probability that a dart he throws will land in section 1 or 2?\r\n\r\nExpress your answer as a percentage rounded to the nearest tenth of a percent."},{"varval":"<div class=\"aligned\">\r\n\r\n| | |\r\n| ------------: | ---------- |\r\n| Number of throws | \\= 45 + 22 + 13 + 9 |\r\n| | \\= 89 |\r\n\r\n</div>\r\n\n<br>\n\n<div class=\"aligned\">\r\n\r\n| | |\r\n| ------------: | ---------- |\r\n| $\\therefore$ Probability (1 or 2) | \\= $\\dfrac{(45 + 22)}{89}$ |\r\n| | \\= $\\dfrac{67}{89}$ |\n| | \\= 0.7528 … |\n| | = {{{correctAnswer0}}}{{{suffix0}}} (nearest tenth) |\r\n\r\n</div>\r"}]},{"vars":[{"varval":"Jarrod throws darts at a board split into 4 sections and records which section they hit.\n\n<br>\n\nsm_img https://teacher.smartermaths.com.au/wp-content/uploads/2018/04/NAPX-J4-CA34.svg 380 indent vpad\r\n\n<br>Based on Jarrod's data, what is the probability that a dart he throws will land in section 2 or 4?\r\n\r\nExpress your answer as a percentage rounded to the nearest tenth of a percent."},{"varval":"<div class=\"aligned\">\r\n\r\n| | |\r\n| ------------: | ---------- |\r\n| Number of throws | \\= 45 + 22 + 13 + 9 |\r\n| | \\= 89 |\r\n\r\n</div>\r\n\n<br>\n\n<div class=\"aligned\">\r\n\r\n| | |\r\n| ------------: | ---------- |\r\n| $\\therefore$ Probability (2 or 4) | \\= $\\dfrac{(22 + 9)}{89}$ |\r\n| | \\= $\\dfrac{31}{89}$ |\n| | \\= 0.3483...|\n| | = {{{correctAnswer0}}}{{{suffix0}}} (nearest tenth) |\r\n\r\n</div>\r"}]},{"vars":[{"varval":"Sigmund throws darts at a board split into 4 sections and records which section they hit.\n\n<br>\n\nsm_img https://teacher.smartermaths.com.au/wp-content/uploads/2018/04/NAPX-J4-CA34.svg 380 indent vpad\r\n\n<br>Based on Sigmund's data, what is the probability that a dart he throws will land in section 3 or 4?\r\n\r\nExpress your answer as a percentage rounded to the nearest tenth of a percent."},{"varval":"<div class=\"aligned\">\r\n\r\n| | |\r\n| ------------: | ---------- |\r\n| Number of throws | \\= 45 + 22 + 13 + 9 |\r\n| | \\= 89 |\r\n\r\n</div>\r\n\n<br>\n\n<div class=\"aligned\">\r\n\r\n| | |\r\n| ------------: | ---------- |\r\n| $\\therefore$ Probability (3 or 4) | \\= $\\dfrac{(13 + 9)}{89}$ |\r\n| | \\= $\\dfrac{22}{89}$ |\n| | \\= 0.2471...|\n| | = {{{correctAnswer0}}}{{{suffix0}}} (nearest tenth) |\r\n\r\n</div>\r"}]}]

  391. <div class="sm_mode"> {{{question}}} </div>

    [{"vars":[{"varval":"A standard six-sided dice is rolled once.\r\n\r\nWhat is the probability that the number on the top face is a factor of 6?"},{"varval":"Factors of 6 are: 6, 1, 3, 2\r\n\nsm_nogap $\\therefore$ P(rolling a factor of 6)\n\n<div class=\"aligned\">\r\n\r\n>| | |\r\n| ------------- | ---------- |\r\n| | \\= $\\dfrac{4}{6}$|\r\n| | \\= {{{correctAnswer}}} |\r\n\r\n</div>\r"}]},{"vars":[{"varval":"A standard six-sided die is rolled once.\r\n\r\nWhat is the probability that the number on the top face is a factor of 12?"},{"varval":"Factors of 12 are: 12, 1, 6, 2\r, 3, 4\n\nsm_nogap $\\therefore$ P(rolling a factor of 12)\n\n<div class=\"aligned\">\r\n\r\n>| | |\r\n| ------------- | ---------- |\r\n| | \\= {{{correctAnswer}}} |\r\n\r\n</div>\r"}]},{"vars":[{"varval":"Felix has 8 buttons in his pocket numbered 1 to 8.\n\nHe picks one of the buttons without looking.\n\nWhat is the probability that the button is a factor of 12?"},{"varval":"Factors of 12 are: 12, 1, 6, 2\r, 3, 4\n\nsm_nogap $\\therefore$ P(choosing a factor of 12)\n\n<div class=\"aligned\">\r\n\r\n>| | |\r\n| ------------- | ---------- |\r\n| | \\= {{{correctAnswer}}} |\r\n\r\n</div>\r"}]},{"vars":[{"varval":"A standard six-sided dice is rolled once.\r\n\r\nWhat is the probability that the number on the top face is a factor of 4?"},{"varval":"Factors of 4 are: 4, 1, 2\r\n\nsm_nogap $\\therefore$ P(rolling a factor of 4)\n\n<div class=\"aligned\">\r\n\r\n>| | |\r\n| ------------- | ---------- |\r\n| | \\= $\\dfrac{3}{6}$|\r\n| | \\= {{{correctAnswer}}} |\r\n\r\n</div>\r"}]},{"vars":[{"varval":"Bobby has 10 discs in a bag that are numbered 1 to 10.\n\nHe picks one of the discs out of the bag without looking.\n\nWhat is the probability that the button is a factor of 6?"},{"varval":"Factors of 6 are: 6, 1, 2\r 3\n\nsm_nogap $\\therefore$ P(choosing a factor of 6)\n\n<div class=\"aligned\">\r\n\r\n>| | |\r\n| ------------- | ---------- |\r\n||= $\\dfrac{4}{10}$ |\n|||\n| | \\= {{{correctAnswer}}} |\r\n\r\n</div>\r"}]},{"vars":[{"varval":"Veronique has 10 discs in a bag that are numbered 1 to 10.\n\nShe picks one of the discs out of the bag without looking.\n\nWhat is the probability that the button is a factor of 12?"},{"varval":"Factors of 12 are: 12, 1, 6, 2, 4,\r3\n\nsm_nogap $\\therefore$ P(choosing a factor of 6)\n\n<div class=\"aligned\">\r\n\r\n>| | |\r\n| ------------- | ---------- |\r\n||= $\\dfrac{5}{10}$ |\n|||\n| | \\= {{{correctAnswer}}} |\r\n\r\n</div>\r"}]}]

  392. <div class="sm_mode"> {{{question}}} </div>

    [{"vars":[{"varval":"Michael rolled a standard dice 76 times.\r\n\r\nHe wrote down if he rolled an odd or even number each time, and recorded the results in the table below.\n\n<br>\n\n<div class=\"sm-table row1-color8\">\n\n>>| Result| Number of times |\n|:-:|:-:|\n| Odd | 32|\n| Even| 44|\n\n</div>\n\n<br>What is the difference between the expected number of odd numbers and the actual number recorded?"},{"varval":"Probability of odd = $\\dfrac{3}{6}$ = $\\dfrac{1}{2}$\r\n\nsm_nogap Expected number of odd numbers\n\n<div class=\"aligned\">\r\n\r\n>| | |\r\n| ------------- | ---------- |\r\n| | \\= $\\dfrac{1}{2} \\times 76$ |\r\n| | \\= 38 |\r\n\r\n</div>\n\n<br>\n\n<div class=\"aligned\">\r\n\r\n| | |\r\n| ------------: | ---------- |\r\n| $\\therefore$ Difference | \\= 38 $-$ 32 |\r\n| | \\= {{{correctAnswer0}}} |\r\n\r\n</div>\r"}]},{"vars":[{"varval":"Penelope flipped a fair coin 128 times.\r\n\r\nShe wrote whether the toss was a head or tail each time, and recorded the results in the table below.\n\n<br>\n\n<div class=\"sm-table row1-color2\">\n\n>>| Result| Number of times |\n|:-:|:-:|\n| Head | 53|\n| Tail| 75|\n\n</div>\n\n<br>What is the difference between the expected number of tails and the actual number recorded?"},{"varval":"Probability of tail = $\\dfrac{1}{2}$\r\n\nsm_nogap Expected number of tails\n\n<div class=\"aligned\">\r\n\r\n>| | |\r\n| ------------- | ---------- |\r\n| | \\= $\\dfrac{1}{2} \\times 128$ |\r\n| | \\= 64 |\r\n\r\n</div>\n\n<br>\n\n<div class=\"aligned\">\r\n\r\n| | |\r\n| ------------: | ---------- |\r\n| $\\therefore$ Difference | \\= 75 $-$ 64 |\r\n| | \\= {{{correctAnswer0}}} |\r\n\r\n</div>\r"}]},{"vars":[{"varval":"Jevin rolled a standard dice 93 times.\r\n\r\nHe wrote down if he rolled a number above two each time, and recorded the results in the table below.\n\n<br>\n\n<div class=\"sm-table row1-color2\">\n\n>>| Result| Number of times |\n|:-:|:-:|\n| Above 2 | 56|\n| Not above 2| 37|\n\n</div>\n\n<br>What is the difference between the expected number of rolls that produced a number above two and the actual number recorded?"},{"varval":"Possible results = 1, 2, 3, 4, 5 or 6\n\nNumber of results above 2 = 4\n\nProbability of above 2 = $\\dfrac{4}{6}$ = $\\dfrac{2}{3}$\r\n\nsm_nogap Expected number rolls above 2\n\n<div class=\"aligned\">\r\n\r\n>| | |\r\n| ------------- | ---------- |\r\n| | \\= $\\dfrac{2}{3} \\times 93$ |\r\n| | \\= {{{correctAnswer0}}} |\r\n\r\n</div>\n\n<br>\n\n<div class=\"aligned\">\r\n\r\n| | |\r\n| ------------: | ---------- |\r\n| $\\therefore$ Difference | \\= 62 $-$ 56 |\r\n| | \\= {{{correctAnswer0}}} |\r\n\r\n</div>\r"}]},{"vars":[{"varval":"Campbell rolled a standard dice 15 times.\r\n\r\nHe wrote down what number he rolled each time, and recorded the results in the table below.\n\n<br>\n\n<div class=\"sm-table row1-color8\">\n\n>>| Result| Number of times |\n|:-:|:-:|\n| 1 or 2| 3|\n| 3 or 4| 5|\n| 5 or 6| 7|\n\n</div>\n\n<br>What is the difference between the expected number of times he rolled a 5 or 6, and the actual number of times recorded?"},{"varval":"Probability of 5 or 6 = $\\dfrac{2}{6}$ = $\\dfrac{1}{3}$\r\n\nsm_nogap Expected number of rolls\n\n<div class=\"aligned\">\r\n\r\n>| | |\r\n| ------------- | ---------- |\r\n| | \\= $\\dfrac{1}{3} \\times 15$ |\r\n| | \\= 5 |\r\n\r\n</div>\n\n<br>\n\n<div class=\"aligned\">\r\n\r\n| | |\r\n| ------------: | ---------- |\r\n| $\\therefore$ Difference | \\= 7 $-$ 5 |\r\n| | \\= {{{correctAnswer0}}} |\r\n\r\n</div>\r"}]},{"vars":[{"varval":"James flipped a fair coin 42 times.\r\n\r\nHe wrote down if it was a head or tail each time, and recorded the results in the table below.\n\n<br>\n\n<div class=\"sm-table row1-color3\">\n\n>>| Result| Number of times |\n|:-:|:-:|\n| Head| 17|\n| Tail| 25|\n\n</div>\n\n<br>What is the difference between the expected number of tails and the actual number recorded?"},{"varval":"Probability of tail = $\\dfrac{1}{2}$\r\n\nsm_nogap Expected number of tails\n\n<div class=\"aligned\">\r\n\r\n>| | |\r\n| ------------- | ---------- |\r\n| | \\= $\\dfrac{1}{2} \\times 42$ |\r\n| | \\= 21 |\r\n\r\n</div>\n\n<br>\n\n<div class=\"aligned\">\r\n\r\n| | |\r\n| ------------: | ---------- |\r\n| $\\therefore$ Difference | \\= 25 $-$ 21 |\r\n| | \\= {{{correctAnswer0}}} |\r\n\r\n</div>\r"}]},{"vars":[{"varval":"Yoshi rolled a standard dice 26 times.\r\n\r\nHe wrote down if he rolled an odd or an even number each time, and recorded the results in the table below.\n\n<br>\n\n<div class=\"sm-table row1-color2\">\n\n>>| Result| Number of times |\n|:-:|:-:|\n| Odd | 16|\n| Even| 10|\n\n</div>\n\n<br>What is the difference between the expected number of rolls that produced an even number and the actual number recorded?"},{"varval":"Probability of even = $\\dfrac{3}{6}$ = $\\dfrac{1}{2}$\r\n\nsm_nogap Expected number of even rolls\n\n<div class=\"aligned\">\r\n\r\n>| | |\r\n| ------------- | ---------- |\r\n| | \\= $\\dfrac{1}{2} \\times 26$ |\r\n| | \\= 13 |\r\n\r\n</div>\n\n<br>\n\n<div class=\"aligned\">\r\n\r\n| | |\r\n| ------------: | ---------- |\r\n| $\\therefore$ Difference | \\= 13 $-$ 10 |\r\n| | \\= {{{correctAnswer0}}} |\r\n\r\n</div>\r"}]}]

  393. <div class="sm_mode"> {{{question}}} </div>

    [{"vars":[{"varval":"A group of 85 people were asked if they have their ears pierced.\r\n\r\nThis table shows the results.\n\n<br>\n\n<div class=\"sm-table col1-color3 row1-color3 top-left-cell-hidden\">\n\n>>| | Pierced| Not Pierced | Total|\n|:-:|:-:|:-:|:-:|\n| Men| 10| 25|35|\n| Women | 35| 15|50|\n|Total|45|40|85|\n\n</div>\n\n<br>A man was selected at random.\r\n\r\nWhat is the probability that he does not have his ears pierced?"},{"varval":"sm_nogap $P$(man has ears pierced)\n\n<div class=\"aligned\">\r\n\r\n>>| | |\r\n| ------------- | ---------- |\r\n| | \\= $\\dfrac{\\text{number of men not pierced}}{\\text{total men}}$ |\n| | \\= $\\dfrac{25}{35}$ |\r\n| | \\= {{{correctAnswer}}} |\r\n\r\n</div>\r"}]},{"vars":[{"varval":"A group of 65 kindergarten students were asked if they watched the cartoon Bluey.\n\r\nThis table shows the results.\n\n<br>\n\n<div class=\"sm-table col1-color3 row1-color3 top-left-cell-hidden\">\n\n>>| | Watched Bluey| Did Not Watch Bluey| Total|\n|:-:|:-:|:-:|:-:|\n| Boys| 21| 11|32|\n| Girls| 27| 6|33|\n|Total|48|17|65|\n\n</div>\n\n<br>One of the girls was selected at random.\r\n\r\nWhat is the probability that she watches Bluey?"},{"varval":"sm_nogap $P$(watches Bluey)\n\n<div class=\"aligned\">\r\n\r\n>>| | |\r\n| ------------- | ---------- |\r\n| | \\= $\\dfrac{\\text{number of girls that watch Bluey}}{\\text{total girls}}$ |\n| | \\= $\\dfrac{27}{33}$ |\r\n| | \\= {{{correctAnswer}}} |\r\n\r\n</div>\r"}]},{"vars":[{"varval":"A group of 55 outback students were asked if they owned a horse.\n\r\nThis table shows the results.\n\n<br>\n\n<div class=\"sm-table col1-color3 row1-color3 top-left-cell-hidden\">\n\n>>| | Own a horse| Do not own a horse| Total|\n|:-:|:-:|:-:|:-:|\n| Boys| 21| 5|26|\n| Girls| 23| 6|29|\n|Total|44|11|55|\n\n</div>\n\n<br>One of the boys was selected at random.\r\n\r\nWhat is the probability that he does not own a horse?"},{"varval":"sm_nogap $P$(does not own a horse)\n\n<div class=\"aligned\">\r\n\r\n>>| | |\r\n| ------------- | ---------- |\r\n| | \\= $\\dfrac{\\text{number of boys with no horse}}{\\text{total boys}}$ |\n| | \\= $\\dfrac{5}{26}$ |\r\n| | \\= {{{correctAnswer}}} |\r\n\r\n</div>\r"}]},{"vars":[{"varval":"A group of 48 mainland students were surveyed if they have visited Tasmania or not.\n\r\nThis table shows the results.\n\n<br>\n\n<div class=\"sm-table col1-color3 row1-color3 top-left-cell-hidden\">\n\n>>| | Visited Tasmania| Not Visited Tasmania| Total|\n|:-:|:-:|:-:|:-:|\n| Boys| 12| 6|18|\n| Girls| 18| 12|30|\n|Total|30|18|48|\n\n</div>\n\n<br>One of the girls was selected at random.\r\n\r\nWhat is the probability that she has not visited Tasmania?"},{"varval":"sm_nogap $P$(not travelled to Tasmania)\n\n<div class=\"aligned\">\r\n\r\n>>| | |\r\n| ------------- | ---------- |\r\n| | \\= $\\dfrac{\\text{number of girls not visited}}{\\text{total girls}}$ |\n| | \\= $\\dfrac{12}{30}$ |\r\n| | \\= {{{correctAnswer}}} |\r\n\r\n</div>\r"}]},{"vars":[{"varval":"A researcher collected 60 live insect specimens for a study. The insects were either cicadas or wasps.\n\r\nThe researcher counted how many females and males of each type, and recorded the results in the table below.\n\n<br>\n\n<div class=\"sm-table col1-color3 row1-color3 top-left-cell-hidden\">\n\n>>| | Cicada| Wasp| Total|\n|:-:|:-:|:-:|:-:|\n| Male| 17| 19|36|\n| Female| 14| 10|24|\n|Total|31|29|60|\n\n</div>\n\n<br>One of the wasps was selected at random.\r\n\r\nWhat is the probability that it was a female?"},{"varval":"sm_nogap $P$(wasp is a female)\n\n<div class=\"aligned\">\r\n\r\n>>| | |\r\n| ------------- | ---------- |\r\n| | \\= $\\dfrac{\\text{number of female wasps}}{\\text{total females}}$ |\n| | \\= $\\dfrac{10}{29}$ |\r\n| | \\= {{{correctAnswer}}} |\r\n\r\n</div>\r"}]},{"vars":[{"varval":"Joe owns a private zoo that has 52 big cats made up of tigers and lions.\n\r\nJoe counted how many females and males of each type, and recorded the results in the table below.\n\n<br>\n\n<div class=\"sm-table col1-color3 row1-color3 top-left-cell-hidden\">\n\n>>| | Lions| Tigers| Total|\n|:-:|:-:|:-:|:-:|\n| Male| 8| 5|13|\n| Female| 25| 14|39|\n|Total|33|19|52|\n\n</div>\n\n<br>One of the female big cats was selected at random.\r\n\r\nWhat is the probability that it was a tiger?"},{"varval":"sm_nogap $P$(female is a tiger)\n\n<div class=\"aligned\">\r\n\r\n>>| | |\r\n| ------------- | ---------- |\r\n| | \\= $\\dfrac{\\text{number of female tigers}}{\\text{total females}}$ |\n| | \\= $\\dfrac{14}{39}$ |\r\n| | \\= {{{correctAnswer}}} |\r\n\r\n</div>\r"}]}]

  394. <div class="sm_mode"> {{{question}}} </div>

    [{"vars":[{"varval":"Two fair 50 cent coins are tossed at the same time.\r\n\r\nWhat is the probability that a head and a tail will result?"},{"varval":"Possible outcomes are: \r\n\n>>HH, HT, TH, TT.\r\n\n<br>\n\n$\\therefore$ $P$(head and tail) = $\\dfrac{2}{4}$ = {{{correctAnswer}}}"}]},{"vars":[{"varval":"Two fair $1 coins are tossed at the same time.\r\n\r\nWhat is the probability that two heads will result?"},{"varval":"Possible outcomes are: \r\n\n>>HH, HT, TH, TT.\r\n\n<br>\n\n$\\therefore$ $P$(two heads) = {{{correctAnswer}}}"}]},{"vars":[{"varval":"A fair 20-cent coin was tossed two times.\r\n\r\nWhat is the probability that two tails will result?"},{"varval":"<div class=\"sm-table row1-color8 heading-color8\">\n\n>>Possible Outcomes\n\n>>| 1st toss | 2nd toss |\n|:-:|:-:|\n| Head | Head |\n| Head | Tail |\n| Tail| Head|\n| Tail| Tail|\n\n</div>\n\n<br>\n\n$\\therefore$ $P$(two heads) = {{{correctAnswer}}}"}]}]

  395. <div class="sm_mode"> {{{question}}} </div>

    [{"vars":[{"varval":"Cedar surveyed 42 families about what brand of car they own.\r\n\r\nThe Venn diagram shows the results.\n\nsm_img https://teacher.smartermaths.com.au/wp-content/uploads/2018/04/NAPX-I4-CA19rev.svg 300 indent vpad\n\nWhat is the probability that a family randomly selected from the group owns both a Toyota and a VW, rounded to three decimal places?"},{"varval":"sm_nogap $P$(own both Toyota and VW)\n\n<div class=\"aligned\">\r\n\r\n>| | |\r\n| ------------- | ---------- |\r\n| | \\= $\\dfrac{6}{42}$ |\n| | \\= 0.1428 … |\r\n| | \\= {{{correctAnswer}}} (to 3 d.p.) |\r\n\r\n</div>\r"}]},{"vars":[{"varval":"Ben surveyed 42 families about what brands of car they own.\r\n\r\nThe Venn diagram shows the results.\n\nsm_img https://teacher.smartermaths.com.au/wp-content/uploads/2018/04/NAPX-I4-CA19rev.svg 300 indent vpad\n\nWhat is the probability that a family randomly selected from the group owns a Toyota, rounded to three decimal places?"},{"varval":"sm_nogap $P$(owns a Toyota)\n\n<div class=\"aligned\">\r\n\r\n>| | |\r\n| ------------- | ---------- |\r\n| | \\= $\\dfrac{13 + 6}{42}$ |\n| | \\= $\\dfrac{19}{42}$ |\n| | \\= 0.4523...|\r\n| | \\= {{{correctAnswer}}} (to 3 d.p.) |\r\n\r\n</div>\r"}]},{"vars":[{"varval":"Imogen surveyed 42 families about what brands of car they own.\r\n\r\nThe Venn diagram shows the results.\n\nsm_img https://teacher.smartermaths.com.au/wp-content/uploads/2018/04/NAPX-I4-CA19rev.svg 300 indent vpad\n\nWhat is the probability that a family randomly selected from the group owns a VW or a Toyota, but not both?"},{"varval":"sm_nogap $P$(owns only a Toyota or VW)\n\n<div class=\"aligned\">\r\n\r\n>| | |\r\n| ------------- | ---------- |\r\n| | \\= $\\dfrac{13 + 7}{42}$ |\n| | \\= $\\dfrac{20}{42}$ |\n| | \\= 0.4761...|\r\n| | \\= {{{correctAnswer}}} (to 3 d.p.) |\r\n\r\n</div>\r"}]}]

  396. <div class="sm_mode"> {{{question}}} </div>

    [{"vars":[{"varval":"Billy has a sewing bag full of buttons where 65% of his buttons are red.\r\n\r\nBilly takes a black button from his bag and sews it onto his shirt.\r\n\r\nBilly then randomly takes a button from his sewing bag without looking.\r\n\r\nWhat are the chances it is red?"},{"varval":"After taking a black button, the sewing bag has same\r number of red and less non-red coloured buttons.\n\n$\\therefore$ Chances of a red are now {{{correctAnswer}}}"}]},{"vars":[{"varval":"Novak has a basket full of tennis balls to practice with where 75% of the balls are Slazenger brand and 25% are Wilson brand.\n\r\nNovak picks a Slazenger ball from the basket and practices his service.\n\r\nIf Novak immediately picks a second ball from the basket without looking, what are the chances it is a Slazenger ball?"},{"varval":"After serving with the first Slazenger ball, the basket now has same\r number of Wilson balls but one less Slazenger ball.\n\n$\\therefore$ Chances of the second ball being a Slazenger is {{{correctAnswer}}}"}]}]

  397. <div class="sm_mode"> Nerrili spins the arrow in a board game. The arrow has an equal chance of landing in each section. <br> sm_img https://teacher.smartermaths.com.au/wp-content/uploads/2021/02/NAP9-A-10_.svg 200 indent3 vpad <br>What is the probability that Nerrili lands on a vowel? </div>

    [{"vars":null}]

  398. Number, NAPX-p115150v01 NAPX-G3-NC04

    <div class="sm_mode"> {{{question}}} </div>

    [{"vars":[{"varval":"Sheryl has 90 cupcakes to sell to her customers.\n\n\r\n\r\nShe packed the cupcakes in boxes that fit 4 cupcakes.\n\n\r\n\r\nHow many full boxes of cupcakes can she sell?"},{"varval":"sm_nogap Number of boxes\n\n<div class=\"aligned\">\n\n>>||\n|-|\n|= 90 ÷ 4|\n|= 22 remainder 2 |\n\n</div>\n\n<br>\n\n$\\therefore$ Sheryl can sell 22 boxes full of cupcakes."}]},{"vars":[{"varval":"Darren bought 141 eggs from the market.\n\n\r\n\r\nHe decided to put the eggs in cartons of 12.\n\n\r\n\r\nHow many full cartons of eggs does he have?"},{"varval":"sm_nogap Number of cartons of eggs\n\n<div class=\"aligned\">\n\n>>||\n|-|\n|= 141 ÷ 12|\n|= 11 remainder 9 |\n\n</div>\n\n<br>\n\n$\\therefore$ Darren can make 11 full cartons of eggs."}]},{"vars":[{"varval":"Korin collects 150 carrots from his vegetable garden.\n\nHe puts the carrots in bunches of 12 carrots.\n\nHow many full bunches can he make?"},{"varval":"sm_nogap Number of full bunches\n\n<div class=\"aligned\">\n\n>>||\n|-|\n|= 150 ÷ 12|\n|= 12 remainder 6|\n\n</div>\n\n<br>\n\r\n$\\therefore$ Korin can make {{{correctAnswer}}} full bunches."}]}]

  399. <div class="sm_mode"> Mr Newman asked his class a question and recorded the results in the table below. <br> sm_img https://teacher.smartermaths.com.au/wp-content/uploads/2018/10/NAPX-J2-13-v1.svg 400 indent vpad <br>Which question could Mr Newman have asked? </div>

    [{"vars":null}]

  400. <div class="sm_mode"> A backwards running group in Europe have a ratio of men to women of &nbsp;$7:4$. There are 12 more men than women in this group. How many people are in this group altogether? </div>

    [{"vars":null}]

  401. <div class="sm_mode"> Luigi packs boxes with different types of pasta. He packs one box at a time. Each box contains 5 packets of spaghetti, 2 packets of fettucini and 1 packet of penne pasta. Every hour Luigi packs 400 packets of pasta in total. How many packets of spaghetti does he pack into boxes in one hour. </div>

    [{"vars":null}]

  402. Statistics, NAPX-I3-NC04, NAPX-I2-12 Statistics, NAPX-p110558v02

    <div class="sm_mode"> {{{question}}} </div>

    [{"vars":[{"varval":"Patrick counts the number of moths he has collected.\r\n\r\n* 9 Bogon moths\r\n* 3 Atlas moths\r\n* 6 Gypsy moths\n\n<div class=\"sm_img_inline\">\r\n\nIn each picture graph below, ![](https://teacher.smartermaths.com.au/wp-content/uploads/2019/01/NAPX-J2-29-v1.svg) = 3 moths\r\n\r\nSelect the picture graph that shows the number of moths Patrick counts.\n\n</div>"},{"varval":"<div class=\"sm_img_inline\">\r\n\n![](https://teacher.smartermaths.com.au/wp-content/uploads/2019/01/NAPX-J2-29-v1.svg) = 3 moths\n\n![](https://teacher.smartermaths.com.au/wp-content/uploads/2019/01/NAPX-J2-29-v1.svg)![](https://teacher.smartermaths.com.au/wp-content/uploads/2019/01/NAPX-J2-29-v1.svg) = 6 moths\n\n![](https://teacher.smartermaths.com.au/wp-content/uploads/2019/01/NAPX-J2-29-v1.svg)![](https://teacher.smartermaths.com.au/wp-content/uploads/2019/01/NAPX-J2-29-v1.svg)![](https://teacher.smartermaths.com.au/wp-content/uploads/2019/01/NAPX-J2-29-v1.svg) = 9 moths\n\n<br>\n\n{{{correctAnswer}}}\n\n</div>"}]},{"vars":[{"varval":"A charity event was held over three weeks. \n\nIn the first week $5000 was raised, in the second week $3500 was raised and in the last week $2000 was raised.\r\n\r\nIn the tables below, O = $500\r\n\r\nWhich table correctly shows the amount of donations raised in the 3 weeks?"},{"varval":"\"O\" represents $500 in donations.\r\n\n\"O\" symbols required\n\n1st week: &nbsp;5000 $\\div$ 500 = 10\r\n\n2nd week: &nbsp;3500 $\\div$ 500 = 7\r\n\n3rd week: &nbsp;2000 $\\div$ 500 = 4\r\n\n<br>\n\nsm_img https://teacher.smartermaths.com.au/wp-content/uploads/2021/04/RAPH8-Q24-A.svg 250 indent vpad"}]},{"vars":[{"varval":"Tim was recording the colour of cars that drove past his school in 1 hour.\n\nIn the hour, Tim saw 14 blue, 8 silver and 6 white cars.\n\nIn the tables below, $X$ = 2 cars.\n\nWhich table correctly shows the number of cars that Tim saw?"},{"varval":"sm_img https://teacher.smartermaths.com.au/wp-content/uploads/2018/06/NAPX-I3-NC04b.svg 220 indent vpad\n\n<div class=\"sm_mode\">\n\n<div class=\"no-margin-bottom\">\n\nSince &nbsp;$X$ = 2 cars,\n\n</div>\n\n<div class=\"aligned\">\n\n| | |\n| ------------- | ------------ |\n| 14 Blue cars | \\= 7$$X$$ |\n| 8 Silver cars | \\= 4$$X$$ |\n| 6 White cars | \\= 3$$X$$ |\n\n</div>\n\n</div>"}]}]

  403. <div class="sm_mode"> A menswear store recoded the number of items it sold in four different categories. What category makes up 60% of the total number of items sold? <br> sm_img https://teacher.smartermaths.com.au/wp-content/uploads/2021/04/raph10-q8.svg 300 indent3 vpad </div>

    [{"vars":null}]

  404. Number, NAPX-p109430v01 SA

    <div class="sm_mode"> {{{question}}} </div>

    [{"vars":[{"varval":"Alejandro makes sangria for his Spanish restaurant.\n\n\r\n\r\nA glass of sangria has a mass of 240 grams and contains 120 millilitres of sangria.\n\n\r\n\r\nWhat is the total mass of the glasses that contain 3.6 litres of sangria?\n"},{"varval":"1 litre = 1000 mL\n\n\r\n3.6 litres = 3600 mL\n\n<div class=\"aligned\">\n\n|||\n|-|-|\n|Number of glasses|= $\\dfrac{3600}{120}$|\n||= 30|\n\n</div>\n\n<br>\n\nsm_nogap Since every glass weighs 240 grams,\n\n<div class=\"aligned\">\n\n|||\n|-|-|\n|Total mass|= 30 × 240|\n||= 7200 grams|\n||= {{{correctAnswer0}}} {{{suffix0}}}|\n\n</div>"}]},{"vars":[{"varval":"Fillon produces fresh milk.\n\n\r\n\r\nA bottle of milk contains 500 millilitres and has a mass of 600 grams.\n\n\r\n\r\nHow many litres of milk has Fillon produced if the bottles have a total mass of 15 kilograms?"},{"varval":"1 kilogram = 1000 grams\n\n\r\n15 kilograms = 15 000 grams\n\n<div class=\"aligned\">\n\n|||\n|-|-|\n|Number of bottles|= $\\dfrac{15 000}{600}$|\n||= 25|\n\n</div>\n\n<br>\n\nsm_nogap Since there is 500 mL in each bottle,\n\n<div class=\"aligned\">\n\n|||\n|-|-|\n|Volume|= 25 × 500|\n||= 12 500 mL|\n||= {{{correctAnswer0}}} {{{suffix0}}}|\n\n</div>"}]},{"vars":[{"varval":"Ralph makes an energy breakfast juice for his cafe.\n\n\r\n\r\nOne bottle of the juice has a mass of 225 grams and contains 150 millilitres of juice.\n\n\r\n\r\nWhat is the total mass of the bottles that contain 6 litres of juice?"},{"varval":"1 litre = 1000 mL\n\n\r\n6 litres = 6000 mL\n\n<div class=\"aligned\">\n\n|||\n|-|-|\n|Number of glasses|= $\\dfrac{6000}{150}$|\n||= 40|\n\n</div>\n\n<br>\n\nsm_nogap Since every glass weighs 225 grams,\n\n<div class=\"aligned\">\n\n|||\n|-|-|\n|Total mass|= 40 × 225|\n||= 9000 grams|\n||= {{{correctAnswer0}}} {{{suffix0}}}|\n\n</div>"}]},{"vars":[{"varval":"Gill makes avocado juice that he sells to health cafes.\n\n\r\n\r\nA bottle of avocado juice contains 325 millilitres and has a mass of 450 grams.\n\n\r\n\r\nHow many litres of avocado juice has Gill made if the bottles have a total mass of 36.9 kilograms?"},{"varval":"1 kilogram = 1000 grams\n\n\r\n36.9 kilograms = 36 900 grams\n\n<div class=\"aligned\">\n\n|||\n|-|-|\n|Number of bottles|= $\\dfrac{36\\ 900}{450}$|\n||= 82|\n\n</div>\n\n<br>\n\nsm_nogap Since there is 325 mL in each bottle,\n\n<div class=\"aligned\">\n\n|||\n|-|-|\n|Volume|= 82 × 325|\n||= 26 650 mL|\n||= {{{correctAnswer0}}} {{{suffix0}}}|\n\n</div>"}]},{"vars":[{"varval":"Jeremiah makes a vegetable soup that he sells in bottles to vegan restaurants.\n\nOne bottle has a mass of 675 grams and contains 600 millilitres of vegetable soup.\n\nWhat is the total mass of the bottles that contain 8.4 litres of vegetable soup?"},{"varval":"1 litre = 1000 mL\n\n\r\n8.4 litres = 8400 mL\n\n<div class=\"aligned\">\n\n|||\n|-|-|\n|Number of bottles|= $\\dfrac{8400}{600}$|\n||= 14|\n\n</div>\n\n<br>\n\nsm_nogap Since every bottle weighs 675 grams,\n\n<div class=\"aligned\">\n\n|||\n|-|-|\n|Total mass|= 14 × 675|\n||= 9450 grams|\n||= {{{correctAnswer0}}} {{{suffix0}}}|\n\n</div>"}]},{"vars":[{"varval":"Luigi makes pasta sauce for his Italian restaurant.\n\n\r\n\r\nOne bottle of pasta sauce has a mass of 250 grams and contains 200 millilitres of pasta sauce.\n\n\r\n\r\nWhat is the total mass of the bottles that contain 5 litres of pasta sauce?"},{"varval":"1 litre = 1000 mL\n\n\r\n5 litres = 5000 mL\n\n<div class=\"aligned\">\n\n|||\n|-|-|\n|Number of bottles|= $\\dfrac{5000}{200}$|\n||= 25|\n\n</div>\n\n<br>\n\nsm_nogap Since every bottle weighs 250 grams,\n\n<div class=\"aligned\">\n\n|||\n|-|-|\n|Total mass|= 25 × 250|\n||= 6250 grams|\n||= {{{correctAnswer0}}} {{{suffix0}}}|\n\n</div>"}]},{"vars":[{"varval":"Greg produces special oil mix for racing cars that he sells in tins.\n\n\r\n\r\nOne tin of oil contains 500 millilitres of oil and has a mass of 750 grams.\n\n\r\n\r\nHow many litres of oil has Greg produced if the tins have a total mass of 30 kilograms?"},{"varval":"1 kilogram = 1000 grams\n\n30 kilograms = 30 000 grams\n\n<div class=\"aligned\">\n\n|||\n|-|-|\n|Number of bottles|= $\\dfrac{30\\ 000}{750}$|\n||= 40|\n\n</div>\n\n<br>\n\nsm_nogap Since there is 500 mL in each bottle,\n\n<div class=\"aligned\">\n\n|||\n|-|-|\n|Volume|= 40 × 500|\n||= 20 000 mL|\n||= {{{correctAnswer0}}} {{{suffix0}}}|\n\n</div>"}]}]

  405. Can't find anything wrong with this question/solution.

    <div class="sm_mode"> {{{question}}} </div>

    [{"vars":[{"varval":"Oggy's gym is 3 kilometres away from his apartment.\r\n\r\nHe jogs there at a constant speed of 10 kilometres per hour.\n\n\r\n\r\nHow many minutes does it take for Oggy to get to the gym from his apartment?"},{"varval":"<div class=\"aligned\">\n\n|||\n|-|-|\n|Time|= $\\dfrac{\\text{Distance}}{\\text{Speed}}$|\n||= $\\dfrac{3}{10}$ hr|\n||= $\\dfrac{3}{10}\\ \\times$ 60 minues|\n||= {{{correctAnswer}}} minutes|\n\n</div>"}]}]

  406. <div class="sm_mode"> A chemist makes 70 millilitres of a compound by mixing 20 millilitres of Chemical A and 50 millilitres of Chemical B. If 250 millilitres of Chemical B is used to make a larger batch of the compound, how much Chemical A is needed? </div>

    [{"vars":null}]

  407. Changed acres to hectares.

    <div class="sm_mode"> {{{question}}} </div>

    [{"vars":[{"varval":"On Farmer Jim's property, he allocates 2 hectares of land to one cow. \n\n\rHow many cows would Farmer Jim allocate to a 12 hectare paddock?"},{"varval":"<div class=\"aligned\">\n\n|||\n|-|-|\n|Cows|= $\\dfrac{12}{2}$|\n||= {{{correctAnswer}}}|\n\n</div>"}]},{"vars":[{"varval":"On a fish farm, 3 square metres of a pond is allocated to one large adult fish. \n\nHow many large adult fish could be allocated to a 15 square metre pond?"},{"varval":"<div class=\"aligned\">\n\n|||\n|-|-|\n|Large adult fish|= $\\dfrac{15}{3}$|\n||= {{{correctAnswer}}}|\n\n</div>"}]},{"vars":[{"varval":"On Yvette's market garden, she allocates 3 square metres of garden to one pumpkin vine. \n\nHow many pumpkin vines could Yvette allocate to a 36 square metre garden?"},{"varval":"<div class=\"aligned\">\n\n|||\n|-|-|\n|Pumpkin vines|= $\\dfrac{36}{3}$|\n||= {{{correctAnswer}}}|\n\n</div>"}]},{"vars":[{"varval":"At a garden nursery, pallets are used to stack pots. \n\nThe nursery allocates 2 square metres of floor space to each pallet.\n\nsm_img https://teacher.smartermaths.com.au/wp-content/uploads/2023/06/euro-pallet-161949.svg 250 indent3 vpad\n\nHow many pallets would be allocated to a 28 square metre floor space?"},{"varval":"<div class=\"aligned\">\n\n|||\n|-|-|\n|Pallets|= $\\dfrac{28}{2}$|\n||= {{{correctAnswer}}}|\n\n</div>"}]},{"vars":[{"varval":"In a shopping centre carpark, each car is allocated 9 square metres of parking space.\n\nsm_img https://teacher.smartermaths.com.au/wp-content/uploads/2023/06/automobile-1300467.svg 250 indent3 vpad\n\nHow many cars would be allocated to a 54 square metre section of carpark?"},{"varval":"<div class=\"aligned\">\n\n|||\n|-|-|\n|Cars|= $\\dfrac{54}{9}$|\n||= {{{correctAnswer}}}|\n\n</div>"}]},{"vars":[{"varval":"On his farm, Neville allocates 1.5 hectares of land to one alpaca.\n\nHow many alpacas would be allocated to a 15 hectare property?"},{"varval":"<div class=\"aligned\">\n\n|||\n|-|-|\n|Alpacas|= $\\dfrac{15}{1.5}$|\n||= {{{correctAnswer}}}|\n\n</div>"}]}]

  408. var: Algebra, NAPX-p115542v02

    <div class="sm_mode"> {{{question}}} </div>

    [{"vars":[{"varval":"The corners of a certain parallelogram are shown below.\n\nsm_img https://teacher.smartermaths.com.au/wp-content/uploads/2021/05/36s.svg 350 indent vpad\r\n\nWhich point in the graph represents the fourth point?"},{"varval":"sm_img https://teacher.smartermaths.com.au/wp-content/uploads/2021/05/36.svg 350 indent vpad\n\n\r\nConsider the top left point on the graph (– 1, – 1),\r\n\nThe point below is 3 grid lines down and three left.\r\n\nConsider the top right point on the graph (4, – 1),\r\n\nThe point {{{correctAnswer}}} is 3 grid lines down and three left."}]},{"vars":[{"varval":"Shane is drawing a repeated pattern in the grid.\n\nsm_img https://teacher.smartermaths.com.au/wp-content/uploads/2021/04/Math-Job-Q42.svg 450 indent vpad\n\nWhich of these points will be part of Shane’s pattern?\r\n"},{"varval":"If the pattern continues:\n\nsm_img https://teacher.smartermaths.com.au/wp-content/uploads/2021/04/Math-Job-Q42Answer.svg 450 indent vpad\r\n\nThe point {{{correctAnswer}}} will be part of the pattern."}]},{"vars":[{"varval":"Ricardo is drawing a repeating pattern on this grid.\n\nsm_img https://teacher.smartermaths.com.au/wp-content/uploads/2018/08/NAPX-F3-CA19.svg 500 indent vpad\n\nWhich of these points will be part of Ricardo's pattern?"},{"varval":"sm_img https://teacher.smartermaths.com.au/wp-content/uploads/2018/08/NAPX-F3-CA19-Answer.svg 500 indent vpad"}]}]

  409. <div class="sm_mode"> {{{question}}} </div>

    [{"vars":[{"varval":"Three corners of a rectangle are shown below.\n\nsm_img https://teacher.smartermaths.com.au/wp-content/uploads/2021/05/35.svg 350 indent vpad\r\n\nWhich point represents the fourth corner of the rectangle?"},{"varval":"sm_img https://teacher.smartermaths.com.au/wp-content/uploads/2021/05/35s.svg 350 indent vpad\r\n\nConsider the top left point on the graph (-1, 1)\r.\n\nThe point below is 3 grid lines down.\r\n\nConsider the top right point on the graph (3, 1),\r\n\nThe point {{{correctAnswer}}} is 3 grid lines down."}]},{"vars":[{"varval":"Three corners of a parallelogram are shown below.\n\nsm_img https://teacher.smartermaths.com.au/wp-content/uploads/2021/05/35.svg 350 indent vpad\r\n\nWhich point could represent the fourth corner of the parallelogram?"},{"varval":"sm_img https://teacher.smartermaths.com.au/wp-content/uploads/2022/01/35-var-1.svg 350 indent vpad\n\nConsider the bottom left point on the graph $(-1, -2)$:\n\nOne point it joins is 4 grids to the right and 3 grids up.\n\nConsider the top left point on the graph $(-1, 1)$:\n\nThe last point {{{correctAnswer}}} is 4 grids to the right and 3 grids up."}]},{"vars":[{"varval":"Three corners of a parallelogram are shown below.\n\nsm_img https://teacher.smartermaths.com.au/wp-content/uploads/2021/05/35.svg 350 indent vpad\r\n\nWhich point could represent the fourth corner of the parallelogram?"},{"varval":"sm_img https://teacher.smartermaths.com.au/wp-content/uploads/2022/01/35-var-2.svg 350 indent vpad\n\nConsider the top right point on the graph $(3, 1)$:\n\nOne point it joins is 3 grids down and 4 grids to the left.\n\nConsider the top left point on the graph $(-1, 1)$:\n\nThe last point {{{correctAnswer}}} 3 grids down and 4 grids to the left."}]}]

  410. var: Algebra, NPAX-p167453v02

    <div class="sm_mode"> {{{question}}} </div>

    [{"vars":[{"varval":"Jason creates the following figures using toothpicks.\n\n<br>\n\nsm_img https://teacher.smartermaths.com.au/wp-content/uploads/2021/04/RAPH9-18.svg 580 indent vpad\r\n\n<br>How many toothpicks will figure 9 have?"},{"varval":"Each succeeding figure adds 5 extra toothpicks.\r\n\nFigure 4: 21 toothpicks\r\n\nFigure 5: 26 toothpicks\r\n\nFigure 6: 31 toothpicks\r\n\nFigure 7: 36 toothpicks\r\n\nFigure 8: 41 toothpicks\r\n\n$\\therefore$ Figure 9 will have 46 toothpicks"}]},{"vars":[{"varval":"Andrew produced a pattern of small stars and diamonds.\n\n<br>\n\nsm_img https://teacher.smartermaths.com.au/wp-content/uploads/2021/04/RAPH9-22.svg 580 indent vpad\r\n\n<br>If this pattern continues how many diamonds will the 9th figure contain?\n"},{"varval":"Each succeeding figure adds 2 diamonds\r\n\nNumber of diamonds\r\n\nFigure 4 = 7 diamonds\r\n\nFigure 5 = 9 diamonds\r \n\nFigure 8 = 15 diamonds\r\n\n$\\therefore$ Figure 9 contains {{{correctAnswer}}} diamonds."}]}]

  411. <div class="sm_mode"> Amelia is making a pattern using flowers. <br> sm_img https://teacher.smartermaths.com.au/wp-content/uploads/2021/04/raph10-q21.svg 580 indent vpad <br>The table below shows the number of flowers she needs for the pattern to be complete. <br> <div class="sm-table col1-color2"> >>| Number Pattern | 1 |2|3|4|5| |:-:|:-:|:-:|:-:|:-:|:-:| | Number of Flowers| 2|4|6|8|**?**| </div> <br>How many flowers will appear in pattern #5? </div>

    [{"vars":null}]

  412. var: Algebra, NAPX-K2-08

    <div class="sm_mode"> {{{question}}} </div>

    [{"vars":[{"varval":"Krusty is using matchsticks to make a pattern of rectangles.\n\n<br>\n\nsm_img https://teacher.smartermaths.com.au/wp-content/uploads/2020/03/ALGEBRANAPX-K2-08V1-AAa-1.svg 450 indent vpad\n\n\r\n<br>How many matchsticks will Krusty need to make Rectangle 5?"},{"varval":"\r\nEach rectangle has 4 extra matchsticks.\n\nRectangle $4 = 14 + 4 = 18$\r\n\nRectangle $5 = 18 + 4 = 22$"}]},{"vars":[{"varval":"Lynx is using matchsticks to make a pattern of triangles.\n\n<br>\n\nsm_img https://teacher.smartermaths.com.au/wp-content/uploads/2018/12/NAPX-K2-08.svg 600 indent vpad\r\n\n<br>How many matchsticks will Lynx need to make Pattern 6?"},{"varval":"Method 1: \n\nEach pattern has 2 matchsticks added\n\n>>Pattern 4 = 7 + 2 = 9\n\n>>Pattern 5 = 9 + 2 = 11\n\n>>Pattern 6 = 11 + 2 = {{{correctAnswer}}} matchsticks\n\n<br>Method 2: (Advanced)\n\n>>Pattern 1 = $3 \r+2 \\times 0 = 3$\n\n>>Pattern 2 = $3 + 2 \\times 1 = 5\r$\n\n>>Pattern 3 = $3 + 2 \\times 2 = 7\r$\n\n>>Pattern 4 = $3 + 2 \\times 3 = 9$\n\n<br>$\\therefore$ following this pattern\n\n>>Pattern $6 =3+2 \\times 5={{{correctAnswer}}}$ matchsticks"}]}]

  413. <div class="sm_mode"> {{{question}}} </div>

    [{"vars":[{"varval":"A food delivery company provides 3 levels of service: Regular, Moderate and Premium.\r\n\r\nRegular service costs $15.50 less than Moderate service.\r\n\r\nRegular service costs $45.80 less than Premium service.\r\n\r\nWhich of these equations correctly represents the cost of Moderate service?"},{"varval":"sm_nogap Extra cost of Premium service over Moderate service\n\n<div class=\"aligned\">\r\n\r\n>>| |\r\n| ---------- |\r\n| \\= $45.80 \\ − \\ 15.50$ |\n| \\= $30.30 |\r\n\r\n\r\n</div>\r\n\n<br>\n\n$\\therefore$ {{{correctAnswer}}}"}]},{"vars":[{"varval":"A delivery company provides 3 levels of service: Regular, Moderate and Premium.\r\n\r\nRegular service costs $10.20 less than Moderate service.\r\n\r\nRegular service costs $25.80 less than Premium service.\r\n\r\nWhich of these equations correctly represents the cost of Moderate service?"},{"varval":"sm_nogap Extra cost of Premium service over Moderate service\n\n<div class=\"aligned\">\r\n\r\n>>| |\r\n| ---------- |\r\n| \\= $25.80 \\ − \\ 10.20$ |\n| \\= $15.60 |\r\n\r\n\r\n</div>\r\n\n<br>\n\n$\\therefore$ {{{correctAnswer}}}"}]},{"vars":[{"varval":"A catering company provides grazing boxes in 3 sizes: Small, Medium and Large.\r\n\r\nA Small box costs $8.50 less than the Medium box.\r\n\r\nA Small box costs $24.30 less than the Large box.\r\n\r\nWhich of these equations correctly represents the cost of Medium box?"},{"varval":"sm_nogap Extra cost of a Large box over a Medium box\n\n<div class=\"aligned\">\r\n\r\n>>| |\r\n| ---------- |\r\n| \\= $24.30 \\ − \\ 8.50$ |\n| \\= $15.80 |\r\n\r\n\r\n</div>\r\n\n<br>\n\n$\\therefore$ {{{correctAnswer}}}"}]},{"vars":[{"varval":"The remaining tickets for \"Disney on Ice\" come in 3 price levels: Front Row, Premium and VIP.\r\n\r\nA VIP ticket costs $12.00 less than a Premium ticket.\r\n\r\nA VIP ticket costs $56.00 less than a Front Row ticket.\r\n\r\nWhich of these equations correctly represents the cost of a Premium ticket?"},{"varval":"sm_nogap Extra cost of a Front Row ticket over a Premium ticket\n\n<div class=\"aligned\">\r\n\r\n>>| |\r\n| ---------- |\r\n| \\= $56.00 \\ − \\ 12.00$ |\n| \\= $44.00 |\r\n\r\n\r\n</div>\r\n\n<br>\n\n$\\therefore$ {{{correctAnswer}}}"}]},{"vars":[{"varval":"Jocasta is headed to New Zealand for her birthday and is searching flights. \n\nIn order of cost, Qantas had the most expensive tickets, then Air New Zealand followed by Jetstar as the cheapest.\r\n\r\nA Jetstar ticket costs $75.00 less than an Air New Zealand ticket. \r\n\r\nA Jetstar ticket costs $118.00 less than a Qantas ticket.\r\n\r\nWhich of these equations correctly represents the cost of an Air New Zealand ticket?"},{"varval":"sm_nogap Extra cost of a Qantas ticket over Air New Zealand ticket\n\n<div class=\"aligned\">\r\n\r\n>>| |\r\n| ---------- |\r\n| \\= $118.00 \\ − \\ 75.00$ |\n| \\= $43.00 |\r\n\r\n\r\n</div>\r\n\n<br>\n\n$\\therefore$ {{{correctAnswer}}}"}]},{"vars":[{"varval":"A cinema complex provides 3 levels of movie tickets: Gold Class, V-Max and Original.\r\n\r\nOriginal tickets cost $5.00 less than V-Max tickets.\r\n\r\nOriginal tickets cost $21.50 less than Gold Class tickets.\r\n\r\nWhich of these equations correctly represents the cost of a V-Max ticket?"},{"varval":"sm_nogap Extra cost of Gold Class tickets over V-Max tickets\n\n<div class=\"aligned\">\r\n\r\n>>| |\r\n| ---------- |\r\n| \\= $21.50 \\ − \\ 5.00$ |\n| \\= $16.50 |\r\n\r\n\r\n</div>\r\n\n<br>\n\n$\\therefore$ {{{correctAnswer}}}"}]}]

  414. var: Algebra, NAPX-p109130v02

    <div class="sm_mode"> {{{question}}} </div>

    [{"vars":[{"varval":"A college marching band is made up of 102 people.\r\n\r\nThere are 24 more men than women.\r\n\r\nHow many women are there?"},{"varval":"Strategy 1\r\n\nCheck each option:\r\n\n$30+(30+24)=84$ &nbsp;x\n\n$39+(39+24)=102$ &nbsp;$\\checkmark$\n\n$\\therefore$ There are 39 women.\n\n<br>\n\nStrategy 2\n\nThere are 24 more men than women.\r\n\nLet $\\ W$ = number of women\n\n<div class=\"aligned\">\r\n\r\n| | |\r\n| ------------: | ---------- |\r\n| $W+(W+24)$ | \\= 102 |\n| $2W$ | \\= 78 |\r\n| $\\therefore W$ | \\= {{{correctAnswer}}} |\r\n\r\n</div>\r"}]},{"vars":[{"varval":"\r\nOn a given day, the RSPCA had 87 lost dogs and cats in its care.\r\n\r\nIf there was 11 more dogs than cats, how many dogs were there?"},{"varval":"Strategy 1\r\n\nCheck each option:\r\n\n$38+(38\\ −\\ 11)=65$ &nbsp;x\n\n$49+(49\\ −\\ 11)=87$ &nbsp;$\\checkmark$\n\n$\\therefore$ There are 49 dogs.\n\n<br>\n\nStrategy 2\n\nThere are 11 more dogs than cats.\r\n\nLet $\\ D$ = number of dogs\n\n<div class=\"aligned\">\r\n\r\n| | |\r\n| ------------: | ---------- |\r\n| $D+(D\\ − \\ 11)$ | \\= 87 |\n| $2D$ | \\= 98 |\r\n| $\\therefore D$ | \\= {{{correctAnswer}}} |\r\n\r\n</div>\r"}]},{"vars":[{"varval":"An AFL club has 68 junior players registered.\n\nThere are 10 more girls registered than boys.\n\nHow many boys are registered with the club?"},{"varval":"<div class=\"sm_mode\">\n\nStrategy 1\n\nBy trial and error:\n\nIf number of boys = 29,\n\nsm_nogap Total players\n\n<div class=\"aligned\">\n\n>| |\n| -------------------- |\n| = 29 + 29 + 10 | \n| = 68 &nbsp$\\checkmark$| \n\n\n</div>\n\n<br>\n\n\nStrategy 2\n\nsm_nogap Let $\\ \\large n$ = number of boys registered\n\n<div class=\"aligned\">\n\n| | |\n| --------------------: | -------------- |\n| $\\large n + n$ + 10 | \\= 68 |\n| 2$\\large n$ | \\= 58 |\n| $\\large n$ | \\= 29 |\n\n</div>\n\n<br>\n\n$\\therefore$ {{correctAnswer}} boys are registered.\n\n</div>"}]},{"vars":[{"varval":"At the fishing club weigh-in there were 45 fish weighed in.\n\nThere were 17 more flathead weighed in than bream.\n\nHow many bream were weighed in?"},{"varval":"<div class=\"sm_mode\">\n\nStrategy 1\n\nsm_nogap Check each option:\n\n<div class=\"aligned\">\n\n>| |\n| -------------------- |\n| 12 + 12 + 17 = 41 &nbsp;x |\n| 13 + 13 + 17 = 43 &nbsp;x |\n| 14 + 14 + 17 = 45 &nbsp$\\checkmark$| \n\n\n</div>\n\n<br>\n\n\nStrategy 2\n\nsm_nogap Let $\\ \\large n$ = number of bream weighed in\n\n<div class=\"aligned\">\n\n| | |\n| --------------------: | -------------- |\n| $\\large n + n$ + 17 | \\= 45 |\n| 2$\\large n$ | \\= 28 |\n| $\\large n$ | \\= 14 |\n\n</div>\n\n<br>\n\n$\\therefore$ {{correctAnswer}} bream are weighed in.\n\n</div>"}]},{"vars":[{"varval":"In a fruit bowl containing apples and bananas, there are 37 pieces of fruit.\n\nThere are 5 more apples than bananas.\n\nHow many bananas are there?"},{"varval":"<div class=\"sm_mode\">\n\nStrategy 1\n\nsm_nogap Check each option:\n\n<div class=\"aligned\">\n\n>| |\n| -------------------- |\n| 42 + 42 + 5 = 89 &nbsp;x |\n| 32 + 32 + 5 = 69 &nbsp;x |\n| 16 + 16 + 5 = 37 &nbsp$\\checkmark$| \n\n\n</div>\n\n<br>\n\n\nStrategy 2\n\nsm_nogap Let $\\ \\large n$ = number of bananas\n\n<div class=\"aligned\">\n\n| | |\n| --------------------: | -------------- |\n| $\\large n + n$ + 5 | \\= 37 |\n| 2$\\large n$ | \\= 32 |\n| $\\large n$ | \\= 16 |\n\n</div>\n\n<br>\n\n$\\therefore$ {{correctAnswer}} bananas are in the fruit bowl.\n\n</div>"}]},{"vars":[{"varval":"A manufacturing plant has made 238 cars. All the cars are either hatch-backs or SUVs.\n\nThere are 74 more hatch-backs than SUVs.\n\nHow many hatch-backs are there?"},{"varval":"<div class=\"sm_mode\">\n\nStrategy 1\n\nsm_nogap Check each option:\n\n<div class=\"aligned\">\n\n>| |\n| -------------------- |\n| 82 + (82 − 74) = 90 &nbsp;x |\n| 156 + (156 − 74) = 238 &nbsp$\\checkmark$| \n\n\n</div>\n\n<br>\n\n\nStrategy 2\n\nsm_nogap Let $\\ \\large n$ = number of hatch-backs \n\n<div class=\"aligned\">\n\n| | |\n| --------------------: | -------------- |\n| $\\large n$ + ($\\large n$ − 74) | \\= 238 |\n| 2$\\large n$ | \\= 312 |\n| $\\large n$ | \\= 156 |\n\n</div>\n\n<br>\n\n$\\therefore$ {{correctAnswer}} hatch-backs in the carpark.\n\n</div>"}]}]

  415. <div class="sm_mode"> {{{question}}} </div>

    [{"vars":[{"varval":"A salesman earns $84 for every 7 litres of paint he sells.\r\n\r\nWhich expression shows how much he would earn for selling $\\large x$ litres?"},{"varval":"84 $\\rarr$ 7 litres \r\n\n84 $\\div$ 7 $\\rarr$ 1 litre\n\n$\\therefore$ {{{correctAnswer}}} $\\rarr \\ \\large x$ litres\n"}]},{"vars":[{"varval":"A fitness instructor earns $90 for every 4 hours of instruction she provides.\r\n\r\nWhich expression shows how much she would earn for providing $\\large x$ hours of instruction?"},{"varval":"90 $\\rarr$ 4 hours \r\n\n90 $\\div$ 4 $\\rarr$ 1 hour\n\n$\\therefore$ {{{correctAnswer}}} $\\rarr \\ \\large x$ hours\n"}]},{"vars":[{"varval":"A quad bike hire company charges $280 for every 5 hours of hire.\r\n\r\nWhich expression shows how much would would be charged for a hire of $\\large x$ hours?"},{"varval":"280 $\\rarr$ 5 hours \r\n\n280 $\\div$ 5 $\\rarr$ 1 hour \n\n$\\therefore$ {{{correctAnswer}}} $\\rarr \\ \\large x$ hours\n"}]},{"vars":[{"varval":"A pet shop charges $120 for every 8 kilograms of dog biscuits it sells.\r\n\r\nWhich expression shows how much would be charged for a sale of $\\large x$ kilograms?"},{"varval":"120 $\\rarr$ 8 kilograms \r\n\n120 $\\div$ 8 $\\rarr$ 1 kilogram\n\n$\\therefore$ {{{correctAnswer}}} $\\rarr \\ \\large x$ kilograms\n"}]},{"vars":[{"varval":"A market stall holder charges $36 for every 4 jars of honey they sell.\r\n\r\nWhich expression shows how much would be charged for a sale of $\\large x$ jars?"},{"varval":"36 $\\rarr$ 4 jars\n\n36 $\\div$ 4 $\\rarr$ 1 jar\n\n$\\therefore$ {{{correctAnswer}}} $\\rarr \\ \\large x$ jars\n"}]},{"vars":[{"varval":"Jason paid $102.60 for 45 litres of petrol.\r\n\r\nWhich expression shows how much he would pay for $\\large x$ litres of petrol?"},{"varval":"102.60 $\\rarr$ 45 litres\n\n102.60 $\\div$ 45 $\\rarr$ 1 litre\n\n$\\therefore$ {{{correctAnswer}}} $\\rarr \\ \\large x$ litres\n"}]}]

  416. <div class="sm_mode"> In a card game, Herbert gets 4 cards and earns points. <br> sm_img https://teacher.smartermaths.com.au/wp-content/uploads/2019/01/NAPX-G2-34.svg 350 indent vpad <br>Each white triangle is worth double the points of each black triangle. How many points is a white triangle worth? </div>

    [{"vars":null}]

  417. <div class="sm_mode"> {{{question}}} </div>

    [{"vars":[{"varval":"Jahangir buys a squash racquet, originally marked $40, that is discounted by 20%.\n\n\r\n\r\nHe also has a loyalty card that gives him another 25% off the sale price.\n\n\r\n\r\nWhat is the final cost of the squash racquet?"},{"varval":"Solution 1\n\n<div class=\"aligned\">\n\n|||\n|-|-|\n|Sale price|= 40 $-$ (20% × 40)|\n||= $32|\n\n</div>\n\n<br>\n\n<div class=\"aligned\">\n\n|||\n|-|-|\n|$\\therefore$ Final cost|= 32 $-$ (25% × 32)|\n||= {{{prefix0}}}{{{correctAnswer0}}}|\n\n</div>\n\n<br>\n\r\nSolution 2 (advanced)\n\n<div class=\"aligned\">\n\n|||\n|-|-|\n|Final cost|= 40 × 0.80 × 0.75|\n||= {{{prefix0}}}{{{correctAnswer0}}}|\n\n</div>"}]},{"vars":[{"varval":"Mervin buys a hockey stick, originally marked $48, that is discounted by 25% on sale.\n\n\r\n\r\nHe also has a loyalty card that gives him another 25% off the sale price.\n\n\r\n\r\nWhat is the final cost of the hockey stick?"},{"varval":"Solution 1\n\n<div class=\"aligned\">\n\n|||\n|-|-|\n|Sale price|= 48 $-$ (25% × 48)|\n||= 48 $-$ 12|\n||= $36|\n\n</div>\n\n<br>\n\n<div class=\"aligned\">\n\n|||\n|-|-|\n|$\\therefore$ Final cost|= 36 $-$ (25% × 36)|\n||= {{{prefix0}}}{{{correctAnswer0}}}|\n\n</div>\n\n<br>\n\r\nSolution 2 (advanced)\n\n<div class=\"aligned\">\n\n|||\n|-|-|\n|Final cost|= 48 × 0.75 × 0.75|\n||= {{{prefix0}}}{{{correctAnswer0}}}|\n\n</div>"}]},{"vars":[{"varval":"Mariah buys a Christmas tree, originally marked $120, that is discounted by 25% for a sale.\n\r\nShe also has a coupon that gives her another 20% off the sale price.\n\nWhat is the final amount that Mariah must pay for the Christmas tree?"},{"varval":"Solution 1\n\n<div class=\"aligned\">\n\n|||\n|-|-|\n|Sale price|= 120 $-$ (25% × 120)|\n||= 120 $-$ 30|\n||= $90|\n\n</div>\n\n<br>\n\n<div class=\"aligned\">\n\n|||\n|-|-|\n|$\\therefore$ Final cost|= 90 $-$ (20% × 90)|\n||= 90 $-$ 18|\n||= {{{prefix0}}}{{{correctAnswer0}}}|\n\n</div>\n\n<br>\n\r\nSolution 2 (advanced)\n\n<div class=\"aligned\">\n\n|||\n|-|-|\n|Final cost|= 120 × 0.75 × 0.80|\n||= {{{prefix0}}}{{{correctAnswer0}}}|\n\n</div>"}]},{"vars":[{"varval":"Barry buys a muffler, originally marked $80, that is discounted by 25% for a sale.\n\nHe also has a loyalty card that gives him another 10% off the sale price.\n\n\rWhat is the final price that Barry pays for the muffler?"},{"varval":"Solution 1\n\n<div class=\"aligned\">\n\n|||\n|-|-|\n|Sale price|= 80 $-$ (25% × 80)|\n||= 80 $-$ 20|\n||= $60|\n\n</div>\n\n<br>\n\n<div class=\"aligned\">\n\n|||\n|-|-|\n|$\\therefore$ Final cost|= 60 $-$ (10% × 60)|\n||= 60 $-$ 6|\n||= {{{prefix0}}}{{{correctAnswer0}}}|\n\n</div>\n\n<br>\n\r\nSolution 2 (advanced)\n\n<div class=\"aligned\">\n\n|||\n|-|-|\n|Final cost|= 80 × 0.75 × 0.9|\n||= {{{prefix0}}}{{{correctAnswer0}}}|\n\n</div>"}]},{"vars":[{"varval":"Tommy buys a bowling ball, originally marked $50, that is discounted by 10% for a sale.\n\nHe also has a loyalty card that gives him another 20% off the sale price.\n\nWhat is the final cost of the bowling ball?"},{"varval":"Solution 1\n\n<div class=\"aligned\">\n\n|||\n|-|-|\n|Sale price|= 50 $-$ (10% × 50)|\n||= 50 $-$ 5|\n||= $45|\n\n</div>\n\n<br>\n\n<div class=\"aligned\">\n\n|||\n|-|-|\n|$\\therefore$ Final cost|= 45 $-$ (20% × 45)|\n||= 45 $-$ 9|\n||= {{{prefix0}}}{{{correctAnswer0}}}|\n\n</div>\n\n<br>\n\r\nSolution 2 (advanced)\n\n<div class=\"aligned\">\n\n|||\n|-|-|\n|Final cost|= 50 × 0.9 × 0.8|\n||= {{{prefix0}}}{{{correctAnswer0}}}|\n\n</div>"}]},{"vars":[{"varval":"Jordan buys a sleeping bag, originally marked $70, that is discounted by 10% for a sale.\n\nHe also benefits from an end of financial year special that gives him another 30% off the sale price.\n\nWhat is the final cost of the sleeping bag?"},{"varval":"Solution 1\n\n<div class=\"aligned\">\n\n|||\n|-|-|\n|Sale price|= 70 $-$ (10% × 70)|\n||= 70 $-$ 7|\n||= $63|\n\n</div>\n\n<br>\n\n<div class=\"aligned\">\n\n|||\n|-|-|\n|$\\therefore$ Final cost|= 63 $-$ (30% × 63)|\n||= 63 $-$ 18.90|\n||= {{{prefix0}}}{{{correctAnswer0}}}|\n\n</div>\n\n<br>\n\r\nSolution 2 (advanced)\n\n<div class=\"aligned\">\n\n|||\n|-|-|\n|Final cost|= 70 × 0.9 × 0.7|\n||= {{{prefix0}}}{{{correctAnswer0}}}|\n\n</div>"}]}]

  418. <div class="sm_mode"> {{{question}}} </div>

    [{"vars":[{"varval":"Sachin's cricket bat is pictured below.\n\n<br>\n\nsm_img https://teacher.smartermaths.com.au/wp-content/uploads/2018/07/NAPX-F4-CA18.svg 220 indent3 vpad\n\n<br>The handle of the bat is 29 cm in length.\n\nWhich of the following show the length of the handle as a percentage of the total length of the bat?\n"},{"varval":"sm_nogap Length of the handle as a percentage\n\n<div class=\"aligned\">\n\n>>||\n|-|\n|= $\\dfrac{29}{89.1} \\times$ 100|\n|= 32.54...%|\n\n</div>\n\n<br>\n\n$\\therefore$ {{{correctAnswer}}} is the closest.\n"}]},{"vars":[{"varval":"Ricky's cricket bat is pictured below.\n\n<br>\n\nsm_img https://teacher.smartermaths.com.au/wp-content/uploads/2018/07/NAPX-F4-CA18.svg 220 indent3 vpad\n\n<br>The handle of the bat is 31 cm in length.\n\nWhich of the following show the length of the handle as a percentage of the total length of the bat?\n"},{"varval":"sm_nogap Length of the handle as a percentage\n\n<div class=\"aligned\">\n\n>>||\n|-|\n|= $\\dfrac{31}{89.1} \\times$ 100|\n|= 34.79...%|\n\n</div>\n\n<br>\n\n$\\therefore$ {{{correctAnswer}}} is the closest.\n"}]},{"vars":[{"varval":"Greg's cricket bat is pictured below.\n\n<br>\n\nsm_img https://teacher.smartermaths.com.au/wp-content/uploads/2018/07/NAPX-F4-CA18.svg 220 indent3 vpad\n\n<br>The handle of the bat is 30 cm in length.\n\nWhich of the following show the length of the handle as a percentage of the total length of the bat?\n"},{"varval":"sm_nogap Length of the handle as a percentage\n\n<div class=\"aligned\">\n\n>>||\n|-|\n|= $\\dfrac{30}{89.1} \\times$ 100|\n|= 33.67...%|\n\n</div>\n\n<br>\n\n$\\therefore$ {{{correctAnswer}}} is the closest.\n"}]},{"vars":[{"varval":"Mithali's cricket bat is pictured below.\n\n<br>\n\nsm_img https://teacher.smartermaths.com.au/wp-content/uploads/2018/07/NAPX-F4-CA18.svg 220 indent3 vpad\n\n<br>The handle of the bat is 28.7 cm in length.\n\nWhich of the following show the length of the handle as a percentage of the total length of the bat?\n"},{"varval":"sm_nogap Length of the handle as a percentage\n\n<div class=\"aligned\">\n\n>>||\n|-|\n|= $\\dfrac{28.7}{89.1} \\times$ 100|\n|= 32.21...%|\n\n</div>\n\n<br>\n\n$\\therefore$ {{{correctAnswer}}} is the closest.\n"}]},{"vars":[{"varval":"Charlotte's cricket bat is pictured below.\n\n<br>\n\nsm_img https://teacher.smartermaths.com.au/wp-content/uploads/2018/07/NAPX-F4-CA18.svg 220 indent3 vpad\n\n<br>The handle of the bat is 32 cm in length.\n\nWhich of the following show the length of the handle as a percentage of the total length of the bat?\n"},{"varval":"sm_nogap Length of the handle as a percentage\n\n<div class=\"aligned\">\n\n>>||\n|-|\n|= $\\dfrac{32}{89.1} \\times$ 100|\n|= 35.91...%|\n\n</div>\n\n<br>\n\n$\\therefore$ {{{correctAnswer}}} is the closest.\n"}]},{"vars":[{"varval":"Alyssa's cricket bat is pictured below.\n\n<br>\n\nsm_img https://teacher.smartermaths.com.au/wp-content/uploads/2018/07/NAPX-F4-CA18.svg 220 indent3 vpad\n\n<br>The handle of the bat is 26 cm in length.\n\nWhich of the following show the length of the handle as a percentage of the total length of the bat?"},{"varval":"sm_nogap Length of the handle as a percentage\n\n<div class=\"aligned\">\n\n>>||\n|-|\n|= $\\dfrac{26}{89.1} \\times$ 100|\n|= 29.18..%|\n\n</div>\n\n<br>\n\n$\\therefore$ {{{correctAnswer}}} is the closest.\n"}]}]

  419. <div class="sm_mode"> {{{question}}} </div>

    [{"vars":[{"varval":"A class of 48 students is surveyed. 36 students said they owned a mobile phone.\n\n\r\n\r\nWhat percentage of the students surveyed do not own a mobile phone?\n"},{"varval":"sm_nogap Number of students who don't own mobile\n\n<div class=\"aligned\">\n\n>>||\n|-|\n|= 48 $-$ 36|\n|= 12|\n\n</div>\n\n<div class=\"aligned\">\n\n|||\n|-|-|\n|$\\therefore$ Percentage|= $\\dfrac{12}{48}$ × 100|\n| |= {{{correctAnswer0}}}{{{suffix0}}}|\n\n</div>"}]},{"vars":[{"varval":"A group of 75 soccer players is surveyed. 60 players said they had scored at least 1 goal this season.\n\n\r\n\r\nWhat percentage of the soccer players surveyed did not score at least 1 goal this season?"},{"varval":"sm_nogap Number of soccer players who did not score at least 1 goal\n\n<div class=\"aligned\">\n\n>>||\n|-|\n|= 75 $-$ 60|\n|= 15|\n\n</div>\n\n<div class=\"aligned\">\n\n|||\n|-|-|\n|$\\therefore$ Percentage|= $\\dfrac{15}{75}$ × 100|\n| |= {{{correctAnswer0}}}{{{suffix0}}}|\n\n</div>"}]},{"vars":[{"varval":"An animal shelter has 84 animals. 45 of the animals are dogs.\n\nWhat percentage of the animals at the shelter are not dogs?\n\nGive your answer correct to one decimal place."},{"varval":"sm_nogap Number of animals that are not dogs\n\n<div class=\"aligned\">\n\n>>||\n|-|\n|= 84 $-$ 45|\n|= 39|\n\n</div>\n\n<div class=\"aligned\">\n\n|||\n|-|-|\n|$\\therefore$ Percentage|= $\\dfrac{39}{84}$ × 100|\n| |$\\approx$ {{{correctAnswer0}}}{{{suffix0}}}|\n\n</div>"}]},{"vars":[{"varval":"A group of 96 students is surveyed. 72 students said they travel to school by bus.\n\n\r\n\r\nWhat percentage of the students surveyed do not travel to school by bus?\n"},{"varval":"sm_nogap Number of students who don't own mobile\n\n<div class=\"aligned\">\n\n>>||\n|-|\n|= 96 $-$ 72|\n|= 24|\n\n</div>\n\n<div class=\"aligned\">\n\n|||\n|-|-|\n|$\\therefore$ Percentage|= $\\dfrac{24}{96}$ × 100|\n| |= {{{correctAnswer0}}}{{{suffix0}}}|\n\n</div>"}]},{"vars":[{"varval":"A gym surveyed 120 members. 45 of the members said they go to the gym 5 or more times a week.\n\n\r\n\r\nWhat percentage of the members surveyed do not go to the gym 5 or more times a week?"},{"varval":"sm_nogap Number of gym members who don't train 5 or more times a week\n\n<div class=\"aligned\">\n\n>>||\n|-|\n|= 120 $-$ 45|\n|= 75|\n\n</div>\n\n<div class=\"aligned\">\n\n|||\n|-|-|\n|$\\therefore$ Percentage|= $\\dfrac{75}{120}$ × 100|\n| |= {{{correctAnswer0}}}{{{suffix0}}}|\n\n</div>"}]},{"vars":[{"varval":"An aquarium contains 240 fish. 108 of the fish are guppies.\n\n\r\n\r\nWhat percentage of the fish are not guppies?\n"},{"varval":"sm_nogap Number of fish that are not guppies\n\n<div class=\"aligned\">\n\n>>||\n|-|\n|= 240 $-$ 108|\n|= 132|\n\n</div>\n\n<div class=\"aligned\">\n\n|||\n|-|-|\n|$\\therefore$ Percentage|= $\\dfrac{132}{240}$ × 100|\n| |= {{{correctAnswer0}}}{{{suffix0}}}|\n\n</div>"}]}]

  420. Number, NAPX-p169627v01

    <div class="sm_mode"> {{{question}}} </div>

    [{"vars":[{"varval":"A box of fruit contains 12 apples, 16 oranges, and 13 pears.\n\n\r\n\r\nAbout what percentage of the fruit in the box are apples?"},{"varval":"<div class=\"aligned\">\n\n|||\n|-|-|\n|Percentage of apples|= $\\dfrac{\\text{number of apples}}{\\text{total pieces of fruit}}$|\n|||\n||= $\\dfrac{12}{12 + 16 + 13}$|\n||= 0.292|\n|| $\\approx$ {{{correctAnswer}}}|\n\n</div>"}]},{"vars":[{"varval":"In a classroom there are 24 boys and 36 girls.\n\n\r\n\r\nWhat percentage of the students in the classroom are girls?"},{"varval":"<div class=\"aligned\">\n\n|||\n|-|-|\n|Percentage of girls|= $\\dfrac{\\text{number of girls}}{\\text{total students}}$|\n|||\n||= $\\dfrac{36}{24 + 36}$|\n||= 0.6|\n||= {{{correctAnswer}}}|\n\n</div>"}]},{"vars":[{"varval":"At a dog show there are 15 labradors, 25 poodles and 20 malamutes.\n\r\n\r\nAbout what percentage of the dogs at the show are poodles?"},{"varval":"<div class=\"aligned\">\n\n|||\n|-|-|\n|Percentage of poodles|= $\\dfrac{\\text{number of poodles}}{\\text{total dogs}}$|\n|||\n||= $\\dfrac{25}{15 + 25 + 20}$|\n||= 0.416...|\n|| $\\approx$ {{{correctAnswer}}}|\n\n</div>"}]},{"vars":[{"varval":"A bag contains 22 red marbles, 40 green marbles and 16 yellow marbles.\n\r\n\r\nAbout what percentage of the marbles in the bag are yellow?"},{"varval":"<div class=\"aligned\">\n\n|||\n|-|-|\n|Percentage of yellow marbles|= $\\dfrac{\\text{number of yellow}}{\\text{total marbles}}$|\n|||\n||= $\\dfrac{16}{22 + 40 + 16}$|\n||= 0.205128...|\n|| $\\approx$ {{{correctAnswer}}}|\n\n</div>"}]},{"vars":[{"varval":"A fishing boat returns with 16 flathead, 7 bream and 9 flounder.\n\r\n\r\nAbout what percentage of the fish are bream?"},{"varval":"<div class=\"aligned\">\n\n|||\n|-|-|\n|Percentage of bream|= $\\dfrac{\\text{number of bream}}{\\text{total fish}}$|\n|||\n||= $\\dfrac{7}{16 + 7 + 9}$|\n||= 0.21875|\n|| $\\approx$ {{{correctAnswer}}}|\n\n</div>"}]},{"vars":[{"varval":"A cafe receives a milkshake order for 2 chocolate, 4 strawberry, 4 caramel and 3 vanilla milkshakes.\n\r\n\r\nAbout what percentage of the milkshakes are chocolate?"},{"varval":"<div class=\"aligned\">\n\n|||\n|-|-|\n|Percentage of chocolate|= $\\dfrac{\\text{number of chocolate}}{\\text{total milkshakes}}$|\n|||\n||= $\\dfrac{2}{2 + 4 + 4 + 3}$|\n||= 0.15384...|\n|| $\\approx$ {{{correctAnswer}}}|\n\n</div>"}]}]

  421. Number, NAPX-p110751v01 SA Number, NAPX-I3-NC29 SA

    <div class="sm_mode"> {{{question}}} </div>

    [{"vars":[{"varval":"In a bulk food supermarket, a kilogram of pepper costs $14.\n\n\r\n\r\nNancy bought 2.65 kilograms of pepper for her restaurant.\n\n\r\n\r\nHow much did Nancy pay for the pepper?\n"},{"varval":"Cost of 1 kg = $14 \n\n\r\nCost of 2 kg = 2 × 14 = $28\n\n\r\nCost of 0.1 kg = 0.1 × $14 = $1.40\n\n<div class=\"aligned\">\n\n|||\n|-|-|\n|Cost of 0.65 kg|= 6 × 1.4 + 0.70|\n||= $9.10|\n\n</div>\n\n<br>\n\n<div class=\"aligned\">\n\n|||\n|-|-|\n|$\\therefore$ Cost of 2.65 kg|= 28 + 9.1|\n||= {{{prefix0}}}{{{correctAnswer0}}}|\n\n</div>"}]},{"vars":[{"varval":"A kilogram of apples cost $8.\n\n\r\n\r\nDanica bought 6.35 kilograms of apples.\n\n\r\n\r\nHow much did Danica pay for the apples?\n"},{"varval":"Cost of 1 kg = $8 \n\n\r\nCost of 6 kg = 6 × 8 = $48\n\n\r\nCost of 0.1 kg = 0.1 × 8 = $0.80\n\n<div class=\"aligned\">\n\n|||\n|-|-|\n|Cost of 0.35 kg|= 3 × 0.8 + 0.4|\n||= $2.80|\n\n</div>\n\n<br>\n\n<div class=\"aligned\">\n\n|||\n|-|-|\n|$\\therefore$ Cost of 6.35 kg|= 48 + 2.80|\n||= {{{prefix0}}}{{{correctAnswer0}}}|\n\n</div>"}]},{"vars":[{"varval":"Bananas cost $5 per kilogram.\n\n\r\n\r\nMagilla buys 5.45 kilograms of bananas.\n\n\r\n\r\nHow much does Magilla pay for the bananas? "},{"varval":"\r\nCost of 5 kg = 5 × $5 = $25\n\n\r\nCost of 0.1 kg = 0.1 × $5 =50 cents\n\n<div class=\"aligned\">\n\n|||\n|-|-|\n|$\\rArr$ Cost of 0.45 kg|= 4.5 × 50 cents|\n||= $2.25|\n\n</div>\n\n<br>\n\n<div class=\"aligned\">\n\n|||\n|-|-|\n|$\\therefore$ Cost of 5.45 kg|= 25 + 2.25|\n||= {{{prefix0}}}{{{correctAnswer0}}}|\n\n</div>"}]}]

  422. The weights on the borders are inconsistent for Var1 to Var5 ie some look bold and others are not. I'm not sure how to fix this. This might be browser specific. All same width my side but will check with a live exam.

    <div class="sm_mode"> {{{question}}} </div>

    [{"vars":[{"varval":"Candice buys art supplies for her art studio.\n\r\nShe rounds each price to the nearest dollar to estimate the total cost of the supplies.\n\n<br>\n\nsm_img https://teacher.smartermaths.com.au/wp-content/uploads/2019/01/NAPX-K2-30.svg 220 indent3 vpad\n\n<br>How much is Candice's estimate?"},{"varval":"Rounding each item to the nearest dollar:\n\npaint = $7\n\npencil = $2\n\ncanvas = $13\n\nchalk = $7\n\nglue = $4\n\n<div class=\"aligned\">\n\n|||\n|-|-|\n|Candice's Estimate| = 7 + 2 + 13 + 7 + 4|\n|| = {{{prefix0}}}{{{correctAnswer0}}}|\n\r\n</div>"}]},{"vars":[{"varval":"John is buying electrical supplies for a job.\n\nHe rounds each price in the table below to the nearest dollar to estimate the total cost of his purchase.\n\n<br>\n\n<div class=\"sm-table row1-color1 heading-color1\">\n\n>>Electrical Supplies\n\n>>| Item | Cost |\n|:-:|-:|\n| 5-watt lightbulb | $7.55 |\n| 7-watt lighbulb | $8.40 |\n| pliars | $15.75 |\n| electrical tape | $2.95 |\n| capacitor| $4.80|\n\n</div>\n\n<br>How much is Johns's estimate?"},{"varval":"Rounding each item to the nearest dollar:\n\n5-watt lightbulb = $8\n\n7-watt lightbulb = $8\n\npliars = $16\n\nelectrical tape = $3\n\ncapacitor = $5\n\n<div class=\"aligned\">\n\n|||\n|-|-|\n|John's Estimate| = 8 + 8 + 16 + 3 + 5|\n|| = {{{prefix0}}}{{{correctAnswer0}}}|\n\r\n</div>"}]},{"vars":[{"varval":"Kate is buying materials for making a dress.\n\nShe rounds each price in the table below to the nearest dollar to estimate the total cost of the materials.\n\n<br>\n\n<div class=\"sm-table row1-color1 heading-color2\">\n\n>>Materials\n\n>>| Item | Cost |\n|:-:|-:|\n| buttons | $3.35 |\n| zipper | $6.65 |\n| material | $23.45 |\n| sewing thread| $2.60 |\n| scissors| $4.95|\n\n</div>\n\n<br>How much is Kate's estimate?"},{"varval":"Rounding each item to the nearest dollar:\n\nbuttons = $3\n\nzipper = $7\n\nmaterial = $23\n\nsewing thread = $3\n\nscissors = $5\n\n<div class=\"aligned\">\n\n|||\n|-|-|\n|Kate's Estimate| = 3 + 7 + 23 + 3 + 5|\n|| = {{{prefix0}}}{{{correctAnswer0}}}|\n\r\n</div>"}]},{"vars":[{"varval":"Byron is buying materials for repairing his hot air balloon.\n\nHe rounds each price in the table below to the nearest dollar to estimate the total cost of the materials.\n\n<br>\n\n<div class=\"sm-table row1-color7 heading-color8\">\n\n>>Materials\n\n>>| Item | Cost |\n|:-:|-:|\n| pliars | $7.55 |\n| zipper | $4.80 |\n| screws | $11.35 |\n| thread| $4.90 |\n| box cutter| $2.15|\n\n</div>\n\n<br>How much is Byron's estimate?"},{"varval":"Rounding each item to the nearest dollar:\n\npliars = $8\n\nzipper = $5\n\nscrews = $11\n\nthread = $5\n\nbox cutter = $2\n\n<div class=\"aligned\">\n\n|||\n|-|-|\n|Byron's Estimate| = 8 + 5 + 11 + 5 + 2|\n|| = {{{prefix0}}}{{{correctAnswer0}}}|\n\r\n</div>"}]},{"vars":[{"varval":"Phil is buying materials for repairing a BMX bike.\n\nHe rounds each price in the table below to the nearest dollar to estimate the total cost of the materials.\n\n<br>\n\n<div class=\"sm-table row1-color1 heading-color2\">\n\n>>Materials\n\n>>| Item | Cost |\n|:-:|-:|\n| oil | $4.65 |\n| chain | $22.95 |\n| pliars | $3.45 |\n| tyre| $8.60 |\n| rag | $1.85|\n\n</div>\n\n<br>How much is Phil's estimate?"},{"varval":"Rounding each item to the nearest dollar:\n\noil = $5\n\nchain = $23\n\npliars = $3\n\ntyre = $9\n\nrag = $2\n\n<div class=\"aligned\">\n\n|||\n|-|-|\n|Phil's Estimate| = 5 + 23 + 3 + 9 + 2|\n|| = {{{prefix0}}}{{{correctAnswer0}}}|\n\r\n</div>"}]},{"vars":[{"varval":"Shannon is buying materials for making a bookcase.\n\nShe rounds each price in the table below to the nearest dollar to estimate the total cost of the materials.\n\n<br>\n\n<div class=\"sm-table row1-color3 heading-color6\">\n\n>>**Materials**\n\n>>| Item | Cost |\n|:-:|-:|\n| hammer | $5.65 |\n| glue | $3.95 |\n| wood | $11.55 |\n| nails| $1.60 |\n| stain| $3.85|\n\n</div>\n\n<br>How much is Shannon's estimate?"},{"varval":"Rounding each item to the nearest dollar:\n\nhammer = $6\n\nglue = $4\n\nwood = $12\n\nnails = $2\n\nstain = $4\n\n<div class=\"aligned\">\n\n|||\n|-|-|\n|Shannon's Estimate| = 6 + 4 + 12 + 2 + 4|\n|| = {{{prefix0}}}{{{correctAnswer0}}}|\n\r\n</div>"}]}]

  423. <div class="sm_mode"> Nick had tests to check his health that are summarised in the table below. sm_img https://teacher.smartermaths.com.au/wp-content/uploads/2018/08/NAPX-E3-CA06.svg 460 indent vpad For which tests were Nick's results within the normal range? </div>

    [{"vars":null}]

  424. <div class="sm_mode"> {{{question}}} </div>

    [{"vars":[{"varval":"The result of a 100-metre dash was recorded in the table shown below.\n\n\n<div class=\"sm-table col1-color8\">\n\n>>| 1st Place| 13.55 seconds|\n|:-:|:-:|\n| 2nd Place| 14.29 seconds|\n| 3rd Place| ?| \n| 4th Place| 14.84 seconds| \n\n</div>\n\n<br>What could be the time of the runner in 3rd place?"},{"varval":"The time of the 3rd runner must be between 14.29 and 14.84 seconds.\n\n\r\n$\\therefore$ The time of the 3rd runner = {{{correctAnswer}}}."}]},{"vars":[{"varval":"The result of a 400-metre time trial was recorded in the table shown below.\n\n\n<div class=\"sm-table col1-color2\">\n\n>>| 1st Place| 47.89 seconds|\n|:-:|:-:|\n| 2nd Place| 48.27 seconds|\n| 3rd Place| ?| \n| 4th Place| 48.76 seconds| \n\n</div>\n\n<br>What could be the time of the runner in 3rd place?"},{"varval":"The time of the 3rd runner must be between 48.27 and 48.76 seconds.\n\n\r\n$\\therefore$ The time of the 3rd runner = {{{correctAnswer}}} seconds."}]},{"vars":[{"varval":"The result of a 50-metre freestyle swimming final was recorded in the table shown below.\n\n\n<div class=\"sm-table col1-color3\">\n\n>>| 1st Place| 32.69 seconds|\n|:-:|:-:|\n| 2nd Place| ?|\n| 3rd Place| 33.54 seconds| \n| 4th Place| 33.91 seconds| \n\n</div>\n\n<br>What could be the time of the swimmer in 2nd place?"},{"varval":"The time of the 2nd swimmer must be between 32.69 and 33.54 seconds.\n\n\r\n$\\therefore$ The time of the 2nd swimmer = {{{correctAnswer}}} seconds."}]},{"vars":[{"varval":"The result of a drag racing sprint was recorded in the table shown below.\n\n\n<div class=\"sm-table col1-color3\">\n\n>>| 1st Place| 12.74 seconds|\n|:-:|:-:|\n| 2nd Place| 13.38 seconds|\n| 3rd Place| ?| \n| 4th Place| 13.82 seconds| \n\n</div>\n\n<br>What could be the time of the drag car in 3rd place?"},{"varval":"The time of the drag car in 3rd place must be between 13.38 and 13.82 seconds.\n\n\r\n$\\therefore$ The time of the 2nd swimmer = {{{correctAnswer}}} seconds."}]}]

  425. <div class="sm_mode"> The image below is the plan of a farming plot. sm_img https://teacher.smartermaths.com.au/wp-content/uploads/2021/04/q5.png 200 indent vpad Squares containing plants represent land that is growing crops. Which fraction shows the amount of land that is used for growing crops? </div>

    [{"vars":null}]

  426. <div class="sm_mode"> {{{question}}} </div>

    [{"vars":[{"varval":"A tube of tennis balls normally cost $11.50.\n\nDuring a sale, the sports store took 10% off the regular price of the balls.\n\nWhat is the new sale price of the tube of tennis balls?"},{"varval":"<div class=\"aligned\">\n\n|||\n|-|-|\n|10% discount|= $\\dfrac{11.50}{10}$|\n||= $1.15|\n\n</div>\n\n<br>\n\n<div class=\"aligned\">\n\n|||\n|-|-|\n|$\\therefore$ Sale price|= 11.50 $-$ 1.15|\n||= {{{prefix0}}}{{{correctAnswer0}}}|\n\n</div>"}]},{"vars":[{"varval":"Cleo wants to buy a doll that normally costs $19.50.\n\nDuring a sale, the toy store took 10% off the regular price of the doll.\n\nWhat is the new sale price of the doll?"},{"varval":"<div class=\"aligned\">\n\n|||\n|-|-|\n|10% discount|= $\\dfrac{19.50}{10}$|\n||= $1.95|\n\n</div>\n\n<br>\n\n<div class=\"aligned\">\n\n|||\n|-|-|\n|$\\therefore$ Sale price|= 19.50 $-$ 1.95|\n||= {{{prefix0}}}{{{correctAnswer0}}}|\n\n</div>"}]},{"vars":[{"varval":"Humphrey wants to buy a jar of honey that normally costs $8.50.\n\nA supermarket reduces the price of the jar of honey by 10% during a sale.\n\nWhat is the new sale price of the jar of honey?"},{"varval":"<div class=\"aligned\">\n\n|||\n|-|-|\n|10% discount|= $\\dfrac{8.50}{10}$|\n||= $0.85|\n\n</div>\n\n<br>\n\n<div class=\"aligned\">\n\n|||\n|-|-|\n|$\\therefore$ Sale price|= 8.50 $-$ 0.85|\n||= {{{prefix0}}}{{{correctAnswer0}}}|\n\n</div>"}]},{"vars":[{"varval":"Kaylan wants to buy a football that normally costs $23.50.\n\nA sports store reduces the price of the football by 10% during a sale.\n\nWhat is the new sale price of the football?"},{"varval":"<div class=\"aligned\">\n\n|||\n|-|-|\n|10% discount|= $\\dfrac{23.50}{10}$|\n||= $2.35|\n\n</div>\n\n<br>\n\n<div class=\"aligned\">\n\n|||\n|-|-|\n|$\\therefore$ Sale price|= 23.50 $-$ 2.35|\n||= {{{prefix0}}}{{{correctAnswer0}}}|\n\n</div>"}]}]

  427. Number, NAPX-p111991v01 SA Number, NAPX-H3-NC25 SA

    <div class="sm_mode"> {{{question}}} </div>

    [{"vars":[{"varval":"Justin went to a lolly shop to buy some sweets and spent $12.\n\n\r\n\r\nHe bought a chocolate bar that cost $5 and 7 pieces of liquorice that cost 40 cents each.\n\n\r\n\r\nThe rest of his money was spent on musk sticks that costs $0.30 each.\n\n\r\n\r\nHow many musk sticks did he buy?\n"},{"varval":"<div class=\"aligned\">\n\n|||\n|-|-|\n|Money left|= $12 - 5 - (7 × 0.40)$|\n||= $7 - 2.80$|\n||= $4.20|\n\n</div>\n\n<br>\n\n<div class=\"aligned\">\n\n|||\n|-|-|\n|$\\therefore$ Musk sticks|= 4.2 ÷ 0.3|\n||= {{{correctAnswer0}}}|\n\n</div>"}]},{"vars":[{"varval":"Joanna went to the supermarket and spent $52.\n\n\r\n\r\nShe bought $15 worth of meat, 10 energy drinks costing $1.90 each and some bottles of water that cost $1.20 each.\n\n\rHow many bottles of water did she buy?"},{"varval":"<div class=\"aligned\">\n\n|||\n|-|-|\n|Money left for water|= $52 - 15 - (10 × 1.9)$|\n||= $18|\n\n</div>\n\n<br>\n\n<div class=\"aligned\">\n\n|||\n|-|-|\n|$\\therefore$ Bottles of water|= 18 ÷ 1.2|\n||= {{{correctAnswer0}}}|\n\n</div>"}]},{"vars":[{"varval":"Nico spent $54 at the post office.\n\nHe bought a large box of envelopes for $17.\n\nWith the remaining money he bought 10 stamps at $1.30 each and some more stamps at 80 cents each.\n\nHow many 80-cent stamps did Nico buy?"},{"varval":"Calculate money left for stamps:\n\n\r\n$54 − $17 = $37\n\n$37 − (10 × 1.30) = $24\n\nsm_nogap $\\therefore$ Number of stamps\n\n<div class=\"aligned\">\n\n>>||\n|-|\n|= 24 ÷ 0.8|\n|= 30|\n\n</div>"}]}]

  428. Number, NAPX-p109843v01

    <div class="sm_mode"> {{{question}}} </div>

    [{"vars":[{"varval":"Murray sells sausage rolls for $1.50 each.\n\nMurray sells a total of 60 sausage rolls.\n\nHow much money does Murray collect for selling the sausage rolls?"},{"varval":"<div class=\"aligned\">\n\n|||\n|-|-|\n|Money collected|= 60 × $1.50|\n||= (60 × $1) + (60 × $0.50)|\n||= $60 + $30|\n||= {{{prefix0}}}{{{correctAnswer0}}}|\n\n</div>"}]},{"vars":[{"varval":"Barry is selling pies.\n\n\r\n\r\nEach pie costs $2.50.\n\n\r\n\r\nBarry sells a total of 50 pies.\n\nHow much money does Barry collect for selling the pies?"},{"varval":"<div class=\"aligned\">\n\n|||\n|-|-|\n|Money collected|= 50 × $2.50|\n||= (50 × $2) + (50 × $0.50)|\n||= $100 + $25|\n||= {{{prefix0}}}{{{correctAnswer0}}}|\n\n</div>"}]}]

  429. <div class="sm_mode"> Soda cans are sold in three different ways at the supermarket. <br> sm_img https://teacher.smartermaths.com.au/wp-content/uploads/2019/01/NAPX-H2-39-SA-300x143.png 280 indent2 vpad <br>What is the cheapest price that someone can buy 14 cans for? </div>

    [{"vars":null}]

  430. <div class="sm_mode"> Butch has only 5-cent, 10-cent, 20-cent and 50-cent coins in his pocket. He buys a doughnut that costs him $2.65 and pays the exact price. What is the smallest number of coins he could use to pay for his doughnut? </div>

    [{"vars":null}]

  431. Number, NAPX-p111587v01

    <div class="sm_mode"> {{{question}}} </div>

    [{"vars":[{"varval":"Lorenzo had a $10 note.\n\nHe decided to buy 13 tokens that are worth 60 cents each to play in the arcade.\n\nHow much change will he get?"},{"varval":"<div class=\"aligned\">\n\n|||\n|-|-|\n|$\\therefore$ Change|= 10 $-$ (13 × 0.60)|\n||= 10 $-$ 7.80|\n||= $2.20|\n\n</div>"}]},{"vars":[{"varval":"Jillian has $25 for buying some groceries.\n\n\r\n\r\nAt the supermarket, she bought 10 oranges that cost $0.25 each and 8 sweet potatoes that cost $1.50 each.\n\n\r\n\r\nHow much money did Jillian have left?"},{"varval":"<div class=\"aligned\">\n\n|||\n|-|-|\n|Total cost|= (10 × 0.25) + (8 × 1.50)|\n||= 2.50 + 12.00|\n||= $14.50|\n\n</div>\n\n<br>\n\n<div class=\"aligned\">\n\n|||\n|-|-|\n|$\\therefore$ Change|= 25.00 $-$ 14.50|\n||= $10.50|\n\n</div>"}]}]

  432. Number, NAPX-p169733v01

    <div class="sm_mode"> {{{question}}} </div>

    [{"vars":[{"varval":"Hilary wants to buy second hand furniture for her new flat.\n\n\r\n\r\nShe went to a charity store that sold furniture with the pricing listed in the table below.\n\n<br>\n\nsm_img https://teacher.smartermaths.com.au/wp-content/uploads/2021/05/p169733v02-300x147.png 270 indent3 vpad\n\n<br>She decided to buy one of each furniture item listed.\n\n\r\n\r\nWhich of these gives the correct range of the total cost of the furniture that she bought?"},{"varval":"sm_nogap Estimate the price of each furniture:\n\n<div class=\"aligned\">\n\n>>||\n|-|\n|$69.20 ≈ $70|\n|$298.75 ≈ $300|\n|$79.50 ≈ $80|\n|$112.30 ≈ $110|\n\r\n\n</div>\n\n<br>\n\n\n<div class=\"aligned\">\n\n|||\n|-|-|\n|Total cost of furniture|= 70 + 300 + 80 + 110|\n| |= $560|\n\r\n\n</div>\n\n<br>\n\n$\\therefore$ The range of the total cost is around $550 – $575."}]},{"vars":[{"varval":"Nicolas wants to build his own desktop gaming setup.\n\nThe shop’s prices for different gaming components are as follows:\n\n<br>\n\nsm_img https://teacher.smartermaths.com.au/wp-content/uploads/2021/05/p169733v01-300x137.png 320 indent3 vpad\n\n<br>Nicolas bought all 4 components from the shop.\n\n\r\n\r\nWhich of these gives the correct range of the total cost of his purchase?"},{"varval":"sm_nogap Estimate the price of each furniture:\n\n<div class=\"aligned\">\n\n>>||\n|-|\n|$54.30 ≈ $55|\n|$23.50 ≈ $25|\n|$248.60 ≈ $250|\n|$158.25 ≈ $160|\n\r\n\n</div>\n\n<br>\n\n<div class=\"aligned\">\n\n|||\n|-|-|\n|Total cost of furniture|= 55 + 25 + 250 + 160|\n| |= $490|\n\r\n\n</div>\n\n<br>\n\n$\\therefore$ The range of the total cost is around $475 – $500."}]}]

  433. Number, NAPX-p115333v01

    <div class="sm_mode"> {{{question}}} </div>

    [{"vars":[{"varval":"What is the value of $16^2$?"},{"varval":"<div class=\"aligned\">\n\n|||\n|-|-|\n|$16^2$|= 16 × 16|\n||= 256|\n\n</div>"}]},{"vars":[{"varval":"What is the value of $24^2$?"},{"varval":"<div class=\"aligned\">\n\n|||\n|-|-|\n|$24^2$|= 24 × 24|\n||= {{{correctAnswer}}}|\n\n</div>"}]},{"vars":[{"varval":"What is the value of $15^2$?"},{"varval":"<div class=\"aligned\">\n\n|||\n|-|-|\n|$15^2$|= 15 × 15|\n||= {{{correctAnswer}}}|\n\n</div>"}]},{"vars":[{"varval":"What is the value of $17^2$?"},{"varval":"<div class=\"aligned\">\n\n|||\n|-|-|\n|$17^2$|= 17 × 17|\n||= {{{correctAnswer}}}|\n\n</div>"}]},{"vars":[{"varval":"What is the value of $31^2$?"},{"varval":"<div class=\"aligned\">\n\n|||\n|-|-|\n|$31^2$|= 31 × 31|\n||= {{{correctAnswer}}}|\n\n</div>"}]},{"vars":[{"varval":"What is the value of $62^2$?"},{"varval":"<div class=\"aligned\">\n\n|||\n|-|-|\n|$62^2$|= 62 × 62|\n||= {{{correctAnswer}}}|\n\n</div>"}]}]

  434. Changed the values in the answers so they were more clearly in or out of the boundary arrows. Otherwise can't find any errors.

    <div class="sm_mode"> {{{question}}} </div>

    [{"vars":[{"varval":"Two arrows point to numbers on a number line.\n\nsm_img https://teacher.smartermaths.com.au/wp-content/uploads/2018/05/NAPX-G4-CA07.svg 300 indent vpad\n\nWhich one of these numbers lies between the two arrows?"},{"varval":"First arrow ≈ 125\n\n\r\nsecond arrow ≈ 220\n\n\r\n$\\therefore$ {{{correctAnswer}}} lies between the arrows."}]}]

  435. <div class="sm_mode"> {{{question}}} </div>

    [{"vars":[{"varval":"Charlize just turned 7 years old.\n\nRambo is 6 years younger than three times Charlize's age.\n\nHow old is Rambo?\n"},{"varval":"<div class=\"aligned\">\n\n|||\n|-|-|\n|Rambo's age|= (3 $\\times$ 7) $-$ 6|\n||= 21 $-$ 6|\n||= {{{correctAnswer}}} years|\n\n</div>\n"}]},{"vars":[{"varval":"Bruno just turned 20 years old.\n\nBluey is 5 years older than two times Bruno's age.\n\nHow old is Bluey?\n"},{"varval":"<div class=\"aligned\">\n\n|||\n|-|-|\n|Bluey's age|= (2 $\\times$ 20) $+$ 5|\n||= 40 $+$ 5|\n||= {{{correctAnswer}}} years|\n\n</div>\n"}]},{"vars":[{"varval":"Maverick just turned 37 years old.\n\nHis grandmother is 15 years younger than three times Maverick's age.\n\nHow old is Maverick's grandmother?\n"},{"varval":"<div class=\"aligned\">\n\n|||\n|-|-|\n|Grandmother's age|= (3 $\\times$ 37) $-$ 15|\n||= 111 $-$ 15|\n||= {{{correctAnswer}}} years|\n\n</div>\n"}]},{"vars":[{"varval":"Elise just turned 7 years old.\n\nHer brother is 2 years older than two times Elise's age.\n\nHow old is Elise's brother?\n"},{"varval":"<div class=\"aligned\">\n\n|||\n|-|-|\n|Brother's age|= (2 $\\times$ 7) $+$ 2|\n||= 14 $+$ 2|\n||= {{{correctAnswer}}} years|\n\n</div>"}]},{"vars":[{"varval":"Jamal just turned 17 years old.\n\nLatika is 12 years younger than two times Jamal's age.\n\nHow old is Latika?\n"},{"varval":"<div class=\"aligned\">\n\n|||\n|-|-|\n|Latika's age|= (2 $\\times$ 17) $-$ 12|\n||= 34 $-$ 12|\n||= {{{correctAnswer}}} years|\n\n</div>"}]},{"vars":[{"varval":"Bailey scored 19 goals in Futsal last season.\n\nParis scored 25 less goals than three times Bailey's total.\n\nHow many goals did Paris score?\n"},{"varval":"<div class=\"aligned\">\n\n|||\n|-|-|\n|Paris's goals|= (3 $\\times$ 19) $-$ 25|\n||= 57 $-$ 25|\n||= {{{correctAnswer}}} goals|\n\n</div>"}]}]

  436. Algebra, NAPX-p122731v02 Algebra, NAPX-K2-22 SA

    <div class="sm_mode"> {{{question}}} </div>

    [{"vars":[{"varval":"Penny runs a chicken shop.\n\n\r\n\r\nAt the start of one day, she has 402 BBQ chickens for sale.\n\n\r\n\r\nAt the end of the same day, she has 32 BBQ chickens not sold.\n\n\r\n\r\nHow may BBQ chickens did Penny sell on that day?"},{"varval":"<div class=\"aligned\">\n\nsm_nogap Chickens sold\n\n>>||\n|-|\n|= 402 $−$ 32|\n|= 400 $−$ 30|\n|= {{{correctAnswer0}}}|\n\r\n\r\n\r\n\n</div>"}]},{"vars":[{"varval":"George runs a large restaurant.\n\n\r\n\r\nAt the start of one day, George has 506 eggs in his kitchen.\n\n\r\n\r\nAt the end of the day, George has 46 eggs left.\n\n\r\n\r\nHow may eggs, in total, were used on that day?\n"},{"varval":"<div class=\"aligned\">\n\nsm_nogap Eggs used in day\n\n>>||\n|-|\n|= 506 $−$ 46|\n|= 500 $−$ 40|\n|= {{{correctAnswer0}}}|\n\r\n\r\n\r\n\n</div>"}]},{"vars":[{"varval":"Andrew collected football cards. He had collected 64 cards.\n\n\r\n\r\nAt school, Andrew and his friends put all their football cards together.\n\n\r\n\r\nThere was a total of 404 cards.\n\n\r\n\r\nHow many football cards, in total, had Andrew's friends collected?\n"},{"varval":"<div class=\"aligned\">\n\nsm_nogap Cards collected by Andrew's friends\n\n>>||\n|-|\n|= 404 $−$ 64|\n|= 400 $−$ 60|\n|= {{{correctAnswer0}}}|\n\r\n\r\n\r\n\n</div>"}]}]

  437. Number, NAPX-p116689v05

    <div class="sm_mode"> {{{question}}} </div>

    [{"vars":[{"varval":"In one month, a factory makes two thousand, nine hundred and eight tennis balls.\n\n\r\nWrite this as a numbers in the box below:"}]},{"vars":[{"varval":"The exact length of great white shark is measured as five thousand and ninety six millimetres.\n\nWrite this as a number in the box below."}]},{"vars":[{"varval":"The tallest living giraffe is measured at five thousand, seven hundred and eight millimetres tall.\n\n\r\n\r\nWrite this as a number in the box below."},{"varval":"5708 millimetres"}]},{"vars":[{"varval":"Candice goes for a run. The exact distance she runs is ten thousand, one hundred and eight metres.\n\n\r\n\r\nWrite this as a numbers in the box below."}]}]

  438. <div class="sm_mode"> {{{question}}} </div>

    [{"vars":[{"varval":"Laura plays golf.\n\r\nDuring one round, she found 13 golf balls and lost 6.\n\r\nAt the end of the round, Laura had a total of 14 golf balls.\n\n<br>\n\nsm_img https://teacher.smartermaths.com.au/wp-content/uploads/2019/01/NAPX-K2-18v1.svg 360 indent vpad\n\n<br>How many golf balls did Laura start with?"},{"varval":"Working backwards:\n\n\r\n⇒ Balls if none lost\n\n\r\n>>14 + 6 = 20\n\n<br>\n\n\r\n⇒ Balls if none found\n\n\r\n>>20 $−$ 13 = 7\r\n\n<br>\n\r\n\r\n$\\therefore$ Laura started with {{{correctAnswer}}}."}]},{"vars":[{"varval":"Sevie plays golf.\n\nDuring one round, he found 5 golf balls and lost 9.\n\nAt the end of the round, Sevie had a total of 8 golf balls.\n\n<br>\n\nsm_img https://teacher.smartermaths.com.au/wp-content/uploads/2019/01/NAPX-K2-18v2.svg 350 indent3 vpad\n\n<br>How many golf balls did Sevie start with?"},{"varval":"Working backwards: \n\n$\\Rightarrow$ If no balls were lost\n\n>>8 + 9 = 17\n\n<br>$\\Rightarrow$ If no balls were found\n\n>>17 − 5 = 12\n \n<br>\n\n\n$\\therefore$ Sevie started with {{{correctAnswer}}}."}]}]

  439. <div class="sm_mode"> A company has 3208 exercise books in a warehouse. Another 731 exercise books are delivered to the warehouse. Which of these could be used to calculate the correct number of exercise books in the warehouse? </div>

    [{"vars":[]}]

  440. <div class="sm_mode"> {{{question}}} </div>

    [{"vars":[{"varval":"What number makes this number sentence correct?"},{"varval":"Deduct 12 and 15 from both sides:\n\n<div class=\"aligned\">\n\n|||\n|-|-|\n|<span class=\"sm_box\"> ` ` ` ` </span>|= $48 - 12 - 15$|\n||= 48 $-$ 27|\n||= 21|\n\n</div>"}]},{"vars":[{"varval":"What number makes this number sentence correct?"},{"varval":"Deduct 16 and 9 from both sides:\n\n<div class=\"aligned\">\n\n|||\n|-|-|\n|<span class=\"sm_box\"> ` ` ` ` </span>|= $44 - 16 - 9$|\n||= 44 $-$ 25|\n||= 19|\n\n</div>"}]},{"vars":[{"varval":"What number makes this number sentence correct?"},{"varval":"Deduct 6 and 19 from both sides:\n\n<div class=\"aligned\">\n\n|||\n|-|-|\n|<span class=\"sm_box\"> ` ` </span>| = 52 $-$ 6 $-$ 19|\n||= 52 $-$ 25|\n||= 27|\n\n</div>"}]},{"vars":[{"varval":"Complete the number sentence."},{"varval":"\rDeduct 12 and 16 from both sides:\n\n<div class=\"aligned\">\n\n|||\n|-|-|\n|<span class=\"sm_box\"> ` ` </span>| = 61 $-$ 12 $-$ 16|\n||= 61 $-$ 28|\n||= {{{correctAnswer0}}}|\n\n</div>"}]}]

  441. Number, NAPX-p169217v01 SA

    <div class="sm_mode"> {{{question}}} </div>

    [{"vars":[{"varval":"Karl picked 102 apples from his family orchard and packed them into small boxes.\n\nEach box can hold 12 apples.\n\nWhat is the smallest number of boxes Karl needs to make sure all the apples are packed?"},{"varval":"<div class=\"aligned\">\n\n|||\n|-|-|\n|Boxese needed|= 102 $\\div$ 12|\n||= 8 remainder 6|\n||= 9 boxes (round up)|\n\n</div>"}]},{"vars":[{"varval":"George has 84 toy train sets in his collection.\n\n\r\n\r\nHe wanted to pack them into containers which can hold 5 train sets each.\n\n\r\n\r\nWhat is the smallest number of containers George needs to make sure all the toy train sets are packed away?"},{"varval":"<div class=\"aligned\">\n\n|||\n|-|-|\n|Containers needed|= 84 $\\div$ 5|\n||= 16 remainder 4|\n||$\\approx$ 17 boxes (round up)|\n\n</div>"}]}]

  442. <div class="sm_mode"> Richard has a bookshelf that he splits into 8 different categories. He has a total of 152 books. He has the same number of books in each category. How many books in each category does he own? </div>

    [{"vars":null}]

  443. Number, NAPX-p146458v08 Number, NAPX-p146458v07 SA

    <div class="sm_mode"> {{{question}}} </div>

    [{"vars":[{"varval":"What is the largest number that can be made using two of these cards?\n\n<br>\n\nsm_img https://teacher.smartermaths.com.au/wp-content/uploads/2020/03/NAPX-K2-03-V1-SA-figure.svg 240 indent vpad\n\n<br>Write the number in the box below."},{"varval":"87"}]},{"vars":[{"varval":"What is the highest number that can be made using two of these cards?\r\n\nsm_img https://teacher.smartermaths.com.au/wp-content/uploads/2021/04/number-napx5-TLA-7-v2_.svg 300 indent vpad\n\nWrite the number in the box below."},{"varval":"75"}]},{"vars":[{"varval":"What is the smallest number that can be made using two of these cards?\n\nsm_img https://teacher.smartermaths.com.au/wp-content/uploads/2021/04/number-napx5-TLA-7-v1.svg 280 indent vpad\n\nWrite the number in the box."},{"varval":"14"}]}]

  444. <div class="sm_mode"> {{{question}}} </div>

    [{"vars":[{"varval":"A return trip from your house to the grocery store is 9.82 kilometres.\n\nIf you go to the store 23 times, how far do you need to travel?"},{"varval":"<div class=\"aligned\">\n\n|||\n|-|-|\n|$\\therefore$ Distance| = 23 × 9.82 km|\n||= {{{correctAnswer}}}|\n\n</div>"}]},{"vars":[{"varval":"A return trip from your house to the beach is 24.3 kilometres.\n\nIf you go to the beach 15 times during the holidays, how far do you need to travel?"},{"varval":"<div class=\"aligned\">\n\n|||\n|-|-|\n|$\\therefore$ Distance| = 15 × 24.3 km|\n||= {{{correctAnswer}}}|\n\n</div>"}]},{"vars":[{"varval":"A return trip from your house to the bait store is 4.58 kilometres.\n\nIf you go to the store 8 times, how far do you need to travel?"},{"varval":"<div class=\"aligned\">\n\n|||\n|-|-|\n|$\\therefore$ Distance| = 8 × 4.58 km|\n||= {{{correctAnswer}}}|\n\n</div>"}]},{"vars":[{"varval":"A return trip from your house to school is 11.44 kilometres.\n\nIf you do the return trip to school 9 times in a fortnight, how far do you need to travel?"},{"varval":"<div class=\"aligned\">\n\n|||\n|-|-|\n|$\\therefore$ Distance| = 9 × 11.44 km|\n||= {{{correctAnswer}}}|\n\n</div>"}]},{"vars":[{"varval":"A return trip from your house to the pool is 22.18 kilometres.\n\nIf you go to the pool 12 times in the holidays, how far do you need to travel?"},{"varval":"<div class=\"aligned\">\n\n|||\n|-|-|\n|$\\therefore$ Distance| = 12 × 22.18 km|\n||= {{{correctAnswer}}}|\n\n</div>"}]},{"vars":[{"varval":"A return trip from your house to the skate park is 4.38 kilometres.\n\nIf you go to the skate park 16 times, how far do you need to travel?"},{"varval":"<div class=\"aligned\">\n\n|||\n|-|-|\n|$\\therefore$ Distance| = 16 × 4.38 km|\n||= {{{correctAnswer}}}|\n\n</div>"}]}]

  445. 8072af47-f42a-46bb-9ed4-092609d23ed4 a1a7c342-4296-47d4-838f-acd7ce952de1

    <div class="sm_mode"> {{{question}}} </div>

    [{"vars":[{"varval":"At the start of school, every child chooses one sport.\n\nThe table shows how many children chose each sport.\n\n<br>\n\n<div class=\"sm-table col1-color2 row1-color2 top-left-cell-hidden\">\n\n>>| | Athletics | Soccer |AFL|\n|:-:|:-:|:-:|:-:|\n| Girls | 40| 35|15|\n| Boys| 20| 45|25|\n\n</div>\n\n<br>Select the statement that is true.\n"},{"varval":"<div class=\"sm_mode\">\n\nThe true statement is:\n\nThe same number of boys and girls are at the school. \n\n</div>"}]},{"vars":[{"varval":"On a school camp, each child chose their meal for dinner.\n\nThe table shows how many children chose each meal.\n\n<br>\n\n<div class=\"sm-table col1-color3 row1-color3 top-left-cell-hidden\">\n\n>>| | Hamburger | Chicko Roll |Nuggets|\n|:-:|:-:|:-:|:-:|\n| Girls | 30| 5|25|\n| Boys| 20| 15|15|\n\n</div>\n\n<br>Select the statement that is true.\n"},{"varval":"The true statement is:\n\nLess than half the boys chose the chicko roll."}]}]

  446. 5493b8d6-a600-41b1-82e5-e4cb46c36105 f0e5aa24-23f4-43e1-aea0-1c4a9d82a35d

    <div class="sm_mode"> {{{question}}} </div>

    [{"vars":[{"varval":"A survey was conducted to determine the number of children who have a dog or a cat as a pet.\n\nThe results are shown below.\n\n<div class=\"sm-table col1-color4 row1-color4 top-left-cell-hidden\">\n\n>>| | Dog | No Dog |\n|:-:|:-:|:-:|\n| Cat | 60| 42|\n| No Cat | 34| 71|\n\n</div>\n\n<br>How many children have a cat as pet, but not a dog?"},{"varval":"\t\r\n42"}]},{"vars":[{"varval":"A survey was conducted to determine the number of children who have a pool or a trampoline at home.\n\nThe results are shown below.\n\n<br>\n\n<div class=\"sm-table col1-color5 row1-color5 top-left-cell-hidden\">\n\n>>| | Trampoline | No Trampoline |\n|:-:|:-:|:-:|\n| Pool | 44| 34|\n| No Pool | 21| 23|\n\n</div>\n\n<br>How many children have a pool at home, but not a trampoline?"},{"varval":"34"}]}]

  447. ba7b6c5d-0b46-4b1e-9454-d175d899f65e 86ddace3-2f13-49a5-b1c0-60c1697988a3

    <div class="sm_mode"> {{{question}}} </div>

    [{"vars":[{"varval":"This graph shows the number of cockatoos in a gum tree at 15 minute intervals over 4 hours.\n\n<br>\n\nsm_img https://teacher.smartermaths.com.au/wp-content/uploads/2018/06/NAPX-F4-NC02.svg 546 indent vpad\n\n<br>\n\nAt which time were the highest number of cockatoos in the gum tree?\n"},{"varval":"The highest data point is two intervals past 6:00 pm.\n\n$\\therefore$ The highest number were in the tree at 6:30 pm."}]},{"vars":[{"varval":"This graph shows the number of customers in a supermarket at 10-minute intervals over 3 hours.\n\n<br>\n\nsm_img https://teacher.smartermaths.com.au/wp-content/uploads/2018/08/NAPX-F3-NC03.svg 515 indent vpad\n\n<br>\n\nAt which of these times was the greatest number of customers in the store?"},{"varval":"$11:50$ (by inspection)"}]},{"vars":[{"varval":"The graph below shows the number of people in a supermarket at 15-minute intervals during a 4 hour period.\n\n<br>\n\nsm_img https://teacher.smartermaths.com.au/wp-content/uploads/2021/04/RAPH9-8.svg 500 indent vpad\r\n\n<br>What time were the least amount of people in the supermarket?"},{"varval":"Each data point represents 15 minutes.\r\n\n$\\therefore$ The lowest data point in the graph is at {{{correctAnswer}}}"}]}]

  448. f55df63b-3299-444d-a973-f8648cd0816e f46bb9f3-170a-4f2f-b660-3b850e104526

    <div class="sm_mode"> {{{question}}} </div>

    [{"vars":[{"varval":"Mike and Georgia saved $90 together.\n\n\rGeorgia saved twice as much money as Mike.\n\r\nHow much money did Georgia save?\n"},{"varval":"Solution 1\n\nTest each option:\n\n>>Option 1 $=2 \\times 15 + 15 = 45$\n\n>>Option 2 $=2 \\times 25 + 25 = 75$\n\n>>Option 3 $=2 \\times 30 + 30 = 90$ $\\checkmark$\n\n>>Option 4 $=2 \\times 90 + 90 = 75$\n\n\n<br>$\\therefore$ Georgia saved {{{correctAnswer}}} \n\n<br>\n\nSolution 2 (advanced)\n\nLet $\\large x$ = Mike's savings\n\n<div class=\"aligned\">\n\n>| | |\n| --------------------: | -------------- |\n| $\\large x$ + $2\\large x$ | \\= 90 |\n| $3\\large x$ | \\= 90 |\n| $\\large x$ | \\= $30 |\n\n</div>\n\n<br>\n\n$\\therefore$ Georgia saved $60 \n"}]},{"vars":[{"varval":"Starsky spent twice as much money as Hutch.\n\nIf they spent a total of \\$120, how much did Hutch spend?\n"},{"varval":"Solution 1\n\nTest each option:\n\n>>Option 1 $=2 \\times 20 + 20 = 60$\n\n>>Option 2 $=2 \\times 40 + 40 = 120$ $\\checkmark$\n\n>>Option 3 $=2 \\times 80 + 80 = 240$ \n\n>>Option 4 $=2 \\times 240 + 240 = 720$\n\n\n<br>$\\therefore$ Hutch spent {{{correctAnswer}}} \n\n<br>\n\nSolution 2 (advanced)\n\n<div class=\"aligned\">\n\n| | |\n| --------------------: | -------------- |\n| $\\text{Let}\\ \\ \\large x$ | \\= Amount Hutch spent |\n| $2\\large x + x$ | \\= 120 |\n| $3 \\large x$ | \\= 120 |\n| $\\large x$ | \\= {{{correctAnswer}}}|\n\n</div>"}]},{"vars":[{"varval":"Bill spent twice as much money as Hans.\n\nIf they spent a total of \\$210, how much did Hans spend?"},{"varval":"Solution 1\n\nTest each option:\n\n>>Option 1 $=2 \\times 420 + 420 = 1260$\n\n>>Option 2 $=2 \\times 140 + 140 = 420$\n\n>>Option 3 $=2 \\times 70 + 70 = 210$ $\\checkmark$\n\n>>Option 4 $=2 \\times 35 + 35 = 105$\n\n\n<br>$\\therefore$ Hans spent {{{correctAnswer}}} \n\n<br>\n\nSolution 2 (advanced)\n\n<div class=\"aligned\">\n\n| | |\n| --------------------: | -------------- |\n| $\\text{Let}\\ \\ \\large x$ | \\= Amount Hans spent |\n| $2\\large x + x$ | \\= 210 |\n| $3 \\large x$ | \\= 210 |\n| $\\large x$ | \\= {{{correctAnswer}}}|\n\n</div>"}]},{"vars":[{"varval":"Wilma spent three times as much money as Betty.\n\nIf they spent a total of \\$200, how much did Betty spend?"},{"varval":"Solution 1\n\nTest each option:\n\n>>Option 1 $=3 \\times 10 + 10 = 40$\n\n>>Option 2 $=3 \\times 40 + 40 = 160$ \n\n>>Option 3 $=3 \\times 50 + 50 = 200$ $\\checkmark$\n\n>>Option 4 $=3 \\times 100 + 100 = 400$\n\n\n<br>$\\therefore$ Betty spent {{{correctAnswer}}} \n\n<br>\n\nSolution 2 (advanced)\n\n<div class=\"aligned\">\n\n| | |\n| --------------------: | -------------- |\n| $\\text{Let}\\ \\ \\large x$ | \\= Amount Betty spent |\n| $3\\large x + x$ | \\= 200 |\n| $4 \\large x$ | \\= 200 |\n| $\\large x$ | \\= {{{correctAnswer}}}|\n\n</div>"}]},{"vars":[{"varval":"Kerry spent four times as much money as Nathan.\n\nIf they spent a total of \\$600, how much did Kerry spend?"},{"varval":"Solution 1\n\nTest each option:\n\n>>Option 1 $=4 \\times 120 + 120 = 480 + 120 =600$ $\\checkmark$ \n\n>>Option 2 $=4 \\times 40 + 40 = 160 + 40 = 200$ \n\n>>Option 3 $=4 \\times 30 + 30 = 120 + 30 = 150$ \n\n>>Option 4 $=4 \\times 20 + 20 = 80 + 20 = 100$\n\n\n<br>$\\therefore$ Kerry spent {{{correctAnswer}}} \n\n\n\nSolution 2 (advanced)\n\n<div class=\"aligned\">\n\n| | |\n| --------------------: | -------------- |\n| $\\text{Let}\\ \\ \\large x$ | \\= Amount Nathan spent |\n| $4\\large x + x$ | \\= 600 |\n| $5 \\large x$ | \\= 600 |\n| $\\large x$ | \\= $120|\n\n$\\therefore$ Kerry spent 4 $\\times$ $120 = {{{correctAnswer}}}\n\n</div>"}]},{"vars":[{"varval":"Bo spent three times as much money as Derek.\n\nIf they spent a total of \\$220, how much did Derek spend?"},{"varval":"Solution 1\n\nTest each option:\n\n>>Option 1 $=3 \\times 20 + 20 = 80$\n\n>>Option 2 $=3 \\times 35 + 35 = 140$\n\n>>Option 3 $=3 \\times 50 + 50 = 200$ \n\n>>Option 4 $=3 \\times 55 + 55 = 220$ $\\checkmark$\n\n\n<br>$\\therefore$ Derek spent {{{correctAnswer}}} \n\n<br>\n\nSolution 2 (advanced)\n\n<div class=\"aligned\">\n\n| | |\n| --------------------: | -------------- |\n| $\\text{Let}\\ \\ \\large x$ | \\= Amount Derek spent |\n| $3\\large x + x$ | \\= 220 |\n| $4 \\large x$ | \\= 220 |\n| $\\large x$ | \\= {{{correctAnswer}}}|\n\n</div>\n"}]}]

  449. 1281e633-5048-432d-87d4-589b56d5c006 bd0e8cb6-2a75-4887-8c13-cdd2f14544c1 Can't find anything wrong with these questions/solutions.

    <div class="sm_mode"> {{{question}}} </div>

    [{"vars":[{"varval":"<div class=\"sm_mode\">\n\nJeeves buys 3 water bottles for \\$1.10 each.\n\nHe pays for these water bottles with a \\$5 note.\n\nHow much change should Jeeves receive?\n\n</div>\n"},{"varval":"<div class=\"sm_mode\">\n\n<div class=\"aligned\">\n\n| | |\n| --------------------- | -------------------------------------------- |\n| Change | = \\$5 $-$ (3 $\\times$ \\$1.10) |\n| | = \\$5 $-$ \\$3.30 |\n| | = $1.70 |\n\n</div>\n\n</div>"}]},{"vars":[{"varval":"<div class=\"sm_mode\">\n\nSam buys 5 water bottles for \\$1.50 each.\n\nHe pays for these water bottles with a \\$10 note.\n\nHow much change should Sam receive?\n\n</div>\n"},{"varval":"<div class=\"sm_mode\">\n\n<div class=\"aligned\">\n\n| | |\n| --------------------- | -------------------------------------------- |\n| Change | = \\$10 $-$ (5 $\\times$ \\$1.50) |\n| | = \\$10 $-$ \\$7.50 |\n| | = $2.50 |\n\n</div>\n\n</div>"}]}]

  450. 7e57c1a7-13f1-4c0e-97a6-d66f4db8f9fd f030b5a7-9238-4c32-a417-994c0dded6c3

    <div class="sm_mode"> {{{question}}} </div>

    [{"vars":[{"varval":"Romy plans to hike from Cronulla Beach to Whale Beach in four days.\n\n\r\n\r\nThe walk is 68 kilometres.\n\n\r\n\r\nOn the first day, she walks 24 500 metres to Surry Hills.\n\n\r\n\r\nThe next day, she walks 17 800 metres to Brookvale.\r\n\r\nHow many kilometres does Romy have left to complete this hike?\n\n\r\n\r\nGive your answer in kilometres, to one decimal place."},{"varval":"<div class=\"aligned\">\n\n| | |\n| --------------------- | -------------- |\n| $\\therefore$ Distance left | \\= 68 000 − 24 500 − 17 800 |\n| | \\= 25 700 m |\n|| \\= {{{correctAnswer0}}} {{{suffix0}}}|\n\n</div>"}]},{"vars":[{"varval":"<div class=\"sm_mode\">\n\nAlicia and Renee plan to complete the Very Long Walk from Belmont to Newcastle Beach in three days.\n\n\r\n\r\nThe walk is 35 kilometres in length.\n\n\r\n\r\nOn the first day, they walk 12 400 metres to Dudley.\n\n\r\n\r\nThe next day, they walk 14 900 metres to Burwood.\n\n\r\n\r\nHow many kilometres remain for them to complete this walk?\n\n\r\n\r\nGive your answer in kilometres, to one decimal place.\n\n</div>"},{"varval":"<div class=\"aligned\">\n\n| | |\n| --------------------- | -------------- |\n|$\\therefore$ Distance left | \\= 35 000 − 12 400 − 14 900 |\n| | \\= 7700 m |\n|| \\= {{{correctAnswer0}}} {{{suffix0}}}|\n\n</div>"}]},{"vars":[{"varval":"Julius plans to complete the Sydney Marathon in three hours.\n\n\r\n\r\nThe marathon is 42 kilometres in length.\n\n\r\n\r\nIn the first hour, he runs 15 100 metres.\n\n\r\n\r\nIn the second hour, he runs 14 700 metres.\n\n\r\n\r\nHow many kilometres remain for Julius to complete the Sydney Marathon in three hours?\n\n\r\n\r\nGive your answer in kilometres, to one decimal place."},{"varval":"<div class=\"aligned\">\n\n| | |\n| --------------------- | -------------- |\n| $\\therefore$ Distance left | \\= 42 000 − 15 100 − 14 700 |\n| | \\= 12 200 m |\n|| \\= {{{correctAnswer0}}} {{{suffix0}}}|\n\n</div>"}]},{"vars":[{"varval":"Jack and Jill have booked a hiking holiday in Tasmania's Cradle Mountain District.\n\n\r\n\r\nThe hike takes three days and is 34 kilometres long.\n\n\r\n\r\nOn the first day, they hike 9400 metres along the Ronny Creek trail.\n\n\r\n\r\nThe next day, they hike 12 800 metres in Cradle Mountain National Park.\n\n\r\n\r\nHow many kilometres do they hike on the last day?\n\n\r\n\r\nGive your answer in kilometres, to one decimal place."},{"varval":"<div class=\"aligned\">\n\n| | |\n| --------------------- | -------------- |\n| $\\therefore$ Distance left | \\= 34 000 − 9 400 − 12 800 |\n| | \\= 11 800 m |\n|| \\= {{{correctAnswer0}}} {{{suffix0}}}|\n\n</div>"}]},{"vars":[{"varval":"Cadel is riding his bike in a charity event over three days.\n\n\r\n\r\nThe ride is 154 kilometres long.\n\n\r\n\r\nDuring the first day, he rides 34 400 metres.\n\n\r\n\r\nOn the next day, he rides 54 800 metres.\n\n\r\n\r\nHow many kilometres does he ride on the last day?\n\n\r\n\r\nGive your answer in kilometres, to one decimal place."},{"varval":"<div class=\"aligned\">\n\n| | |\n| --------------------- | -------------- |\n| $\\therefore$ Distance left | \\= 154 000 − 34 400 − 54 800 |\n| | \\= 64 800 m |\n|| \\= {{{correctAnswer0}}} {{{suffix0}}}|\n\n</div>"}]},{"vars":[{"varval":"Anna and her friends are walking the Thredbo to Kosciuszko trail in the December school holidays.\n\n\r\n\r\nThe hike is 13 kilometres long.\n\n\r\n\r\nDuring the first leg, they walk 4 200 metres.\n\n\r\n\r\nOn the next leg, they walk 3 700 metres.\n\n\r\n\r\nHow many kilometres do they walk on the final leg?\n\n\r\n\r\nGive your answer in kilometres, to one decimal place."},{"varval":"<div class=\"aligned\">\n\n| | |\n| --------------------- | -------------- |\n| $\\therefore$ Distance left | \\= 13 000 − 4 200 − 3 700 |\n| | \\= 5 100 m |\n|| \\= {{{correctAnswer0}}} {{{suffix0}}}|\n\n</div>"}]}]

  451. a1152ac2-bc91-471b-a7c8-9ccd3a6bd509 a7ce27a5-e0f0-408e-9978-dbb246a7e598 f98a09c3-5e3a-4047-bbd3-b96019dfc7c8

    <div class="sm_mode"> {{{question}}} </div>

    [{"vars":[{"varval":"Dusty wants to find the number of people who play soccer but do not play Aussie rules.\n\nWhich shaded region in the Venn diagrams represents the information Dusty is looking for?"},{"varval":"sm_img https://teacher.smartermaths.com.au/wp-content/uploads/2018/04/NAPX-I4-NC04c-rev.svg 231 indent vpad"}]},{"vars":[{"varval":"Murray wants to find the number of boys in his class that play AFL or soccer but don't play both.\n\n\r\n\r\nWhich shaded region in the Venn diagrams represents the boys that Murray is seeking?"},{"varval":"sm_img https://teacher.smartermaths.com.au/wp-content/uploads/2020/04/NAPX-LA-CA08-o2-d.svg 234 indent vpad"}]},{"vars":[{"varval":"Tilba needs to find out the number of students studying both Biology and Physics.\n\n\r\n\r\nWhich shaded region in the Venn Diagrams represents the students that Tilba is looking for?"},{"varval":"sm_img https://teacher.smartermaths.com.au/wp-content/uploads/2020/04/NAPX-LA-CA08-o1-d.svg 216 indent vpad"}]}]

  452. b7df2fc9-ba02-4032-8e2e-d2a6d4f38ef5 060c2332-bc21-4d50-a371-f416ab2e4030 060c2332-bc21-4d50-a371-f416ab2e4030

    <div class="sm_mode"> {{{question}}} </div>

    [{"vars":[{"varval":"<div class=\"sm_mode\">\n\nShiver bought 13 staplers for her art class.\n\nShe bought:\n\n* one box of 9 staplers for $45.90, and\n* one box of 4 staplers for $21.05.\n\nWhat is the mean price of the 13 staplers?\n\n</div>"},{"varval":"<div class=\"sm_mode\">\n\n<div class=\"aligned\">\n\n| | |\n| --------------------: | -------------- |\n| Total spent | \\= 45.90 + 21.05 |\n| | \\= $66.95 |\n\n| | |\n| ----------------------------: | -------------- |\n| $\\therefore$ Mean price | \\= $\\dfrac{66.95}{13}$ |\n| | \\= $5.15 |\n\n</div>\n</div>"}]},{"vars":[{"varval":"<div class=\"sm_mode\">\n\nColin bought 12 pencils.\n\nHe bought:\n* one packet of 10 pencils for \\$23.00, and\n* another packet of 2 pencils for \\$5.20.\n\nWhat is the mean price of the 12 pencils?\n\n</div>"},{"varval":"<div class=\"sm_mode\">\n\n<div class=\"aligned\">\n\n| | |\n| --------------------: | -------------- |\n| Total spent | \\= 23.00 + 5.20 |\n| | \\= \\$28.20 |\n\n| | |\n| ----------------------------: | -------------- |\n| $\\therefore$ Mean price | \\= $\\dfrac{28.20}{12}$ |\n| | \\= $2.35 |\n\n</div>\n\n</div>"}]},{"vars":[{"varval":"Tinky-Winky bought 12 doughnuts.\n\nHe bought:\n\n* one box of 9 doughnuts for $38.70, and\n\n* another box of 3 doughnuts for $16.50.\n\nWhat is the mean price of the 12 doughnuts?\n"},{"varval":"<div class=\"sm_mode\">\n\n<div class=\"aligned\">\n\n| | |\n| --------------------: | -------------- |\n| Total spent | = 38.70 + 16.50 |\n| | = $55.20 |\n\n| | |\n| ----------------------------: | -------------- |\n| $\\therefore$ Mean price | \\= $\\dfrac{55.20}{12}$ |\n| | = {{{correctAnswer}}} |\n\n</div>\n\n</div>"}]}]

  453. <div class="sm_mode"> {{{question}}} </div>

    [{"vars":[{"varval":"In the diagram $PQ$ is a straight line.\n\n<br>\n\nsm_img https://teacher.smartermaths.com.au/wp-content/uploads/2018/04/NAPX-H4-CA30-SA.svg 330 indent3 vpad\n\n<br>What is the size of the angle marked $\\large a$$\\degree$?"},{"varval":"The bottom left triangle is equilateral.\n\n<div class=\"aligned\">\n\n|||\n|-|-|\n|$\\therefore \\large a$$\\degree$|= $180 - (90 + 30)$|\n||= {{{correctAnswer0}}}{{{suffix0}}}|\n\n</div>\n\n<br>\n\nsm_img https://teacher.smartermaths.com.au/wp-content/uploads/2018/04/NAPX-H4-CA30-SA-Answer.svg 290 indent vpad\n\n"}]},{"vars":[{"varval":"In the diagram $XY$ is a straight line.\n\n<br>\n\nsm_img https://teacher.smartermaths.com.au/wp-content/uploads/2022/10/Geom_-NAPX-H4-CA30-SA-NAPX-H3-CA32-SA.svg 300 indent3 vpad\n\n<br>What is the size of the angle marked $\\large a$$\\degree$?"},{"varval":"The bottom left triangle is equilateral.\n\n<div class=\"aligned\">\n\n|||\n|-|-|\n|$\\therefore \\large a$$\\degree$|= $90 - 60$|\n||= {{{correctAnswer0}}}{{{suffix0}}}|\n\n</div>\n\n<br>\n\nsm_img https://teacher.smartermaths.com.au/wp-content/uploads/2022/10/Geom_-NAPX-H4-CA30-SA-NAPX-H3-CA32-SA_v1ws.svg 290 indent vpad\n\n"}]}]

  454. <div class="sm_mode"> {{{question}}} </div>

    [{"vars":[{"varval":"\nThorfinn was designing a viking shield using 9 identical isosceles triangles, as shown in the diagram below.\n\nsm_img https://teacher.smartermaths.com.au/wp-content/uploads/2018/06/NAPX-E4-NC30-SA.svg 280 indent vpad\n\nHow many degrees is the angle marked $\\large x$?\n"},{"varval":"sm_nogap Angles at centre of circle\n\n<div class=\"aligned\">\n\n>>||\n|-|\n|= $\\dfrac{360}{9}$|\n|= 40$\\degree$|\n\n</div>\n\n<br>\n\nsm_nogap Since triangles are isosceles,\n\n<div class=\"aligned\">\n\n|||\n|-:|-|\n|180|= 40 + 2$\\large x$|\n|2$\\large x$|= 140|\n|$\\large x$|= {{{correctAnswer0}}}{{{suffix0}}}|\n\n</div>\n\n"}]},{"vars":[{"varval":"\nTerra was designing a new frisbee using 3 identical isosceles triangles, as shown in the diagram below.\n\nsm_img https://teacher.smartermaths.com.au/wp-content/uploads/2022/10/Geom_NAPX-E4-NC30-SA_v4.svg 150 indent2 vpad\n\nHow many degrees is the angle marked $\\large x$?\n"},{"varval":"sm_nogap Angles at centre of circle\n\n<div class=\"aligned\">\n\n>>||\n|-|\n|= $\\dfrac{360}{3}$|\n|= 120$\\degree$|\n\n</div>\n\n<br>\n\nsm_nogap Since triangles are isosceles,\n\n<div class=\"aligned\">\n\n|||\n|-:|-|\n|180|= 120 + 2$\\large x$|\n|2$\\large x$|= 60|\n|$\\large x$|= {{{correctAnswer0}}}{{{suffix0}}}|\n\n</div>\n\n"}]},{"vars":[{"varval":"\nGlory was designing a new logo for her catering business using 5 identical isosceles triangles, as shown in the diagram below.\n\nsm_img https://teacher.smartermaths.com.au/wp-content/uploads/2022/10/Geom_NAPX-E4-NC30-SA_v3.svg 160 indent vpad\n\nHow many degrees is the angle marked $\\large p$?\n"},{"varval":"sm_nogap Angles at centre of circle\n\n<div class=\"aligned\">\n\n>>||\n|-|\n|= $\\dfrac{360}{5}$|\n|= 72$\\degree$|\n\n</div>\n\n<br>\n\nsm_nogap Since triangles are isosceles,\n\n<div class=\"aligned\">\n\n|||\n|-:|-|\n|180|= 72 + 2$\\large p$|\n|2$\\large p$|= 108|\n|$\\large p$|= {{{correctAnswer0}}}{{{suffix0}}}|\n\n</div>\n\n"}]},{"vars":[{"varval":"\nBergan was designing a chicken coup using 8 identical isosceles triangles, as shown in the diagram below. \n\nsm_img https://teacher.smartermaths.com.au/wp-content/uploads/2022/10/Geom_NAPX-E4-NC30-SA_v2.svg 220 indent vpad\n\nHow many degrees is the angle marked $\\large m$?\n"},{"varval":"sm_nogap Angles at centre of circle\n\n<div class=\"aligned\">\n\n>>||\n|-|\n|= $\\dfrac{360}{8}$|\n|= 45$\\degree$|\n\n</div>\n\n<br>\n\nsm_nogap Since triangles are isosceles,\n\n<div class=\"aligned\">\n\n|||\n|-:|-|\n|180|= 45 + 2$\\large m$|\n|2$\\large m$|= 135|\n|$\\large m$|= {{{correctAnswer0}}}{{{suffix0}}}|\n\n</div>\n\n"}]},{"vars":[{"varval":"\nBilbo was designing a new thatched roof for his cottage using 6 identical isosceles triangles, as shown in the diagram below.\n\nsm_img https://teacher.smartermaths.com.au/wp-content/uploads/2022/10/Geom_NAPX-E4-NC30-SA_v1.svg 180 indent vpad\n\nHow many degrees is the angle marked $\\large y$?\n"},{"varval":"sm_nogap Angles at centre of circle\n\n<div class=\"aligned\">\n\n>>||\n|-|\n|= $\\dfrac{360}{6}$|\n|= 60$\\degree$|\n\n</div>\n\n<br>\n\nsm_nogap Since triangles are isosceles,\n\n<div class=\"aligned\">\n\n|||\n|-:|-|\n|180|= 60 + 2$\\large y$|\n|2$\\large y$|= 120|\n|$\\large y$|= {{{correctAnswer0}}}{{{suffix0}}}|\n\n</div>\n\n"}]},{"vars":[{"varval":"\nWalt was designing a ferris wheel for his theme park using 12 identical isosceles triangles, as shown in the diagram below. \n\nsm_img https://teacher.smartermaths.com.au/wp-content/uploads/2022/10/Geom_NAPX-E4-NC30-SA_v5.svg 260 indent vpad\n\nHow many degrees is the angle marked $\\large w$?\n"},{"varval":"sm_nogap Angles at centre of circle\n\n<div class=\"aligned\">\n\n>>||\n|-|\n|= $\\dfrac{360}{12}$|\n|= 30$\\degree$|\n\n</div>\n\n<br>\n\nsm_nogap Since triangles are isosceles,\n\n<div class=\"aligned\">\n\n|||\n|-:|-|\n|180|= 30 + 2$\\large w$|\n|2$\\large w$|= 150|\n|$\\large w$|= {{{correctAnswer0}}}{{{suffix0}}}|\n\n</div>\n\n"}]}]

  455. <div class="sm_mode"> {{{question}}} </div>

    [{"vars":[{"varval":"Six identical triangles are used to make the figure below.\n\nsm_img https://teacher.smartermaths.com.au/wp-content/uploads/2018/06/NAPX-F4-CA27-SA.svg 160 indent3 vpad\n\nWhat is the size of the angle marked $\\large x \\degree$?\n\n"},{"varval":"Consider the centre angles of each triangle, $\\large c\\degree$\n\nsm_img https://teacher.smartermaths.com.au/wp-content/uploads/2018/06/NAPX-F4-CA27-SA-Answer.svg 90 indent3 vpad\n\n<div class=\"aligned\">\n\n|||\n|-:|-|\n|6 $\\times\\ \\large c$$\\degree$|= 180$\\degree$|\n|$\\large c$$\\degree$|= 30$\\degree$|\n|$\\therefore \\large x$$\\degree$|= $180 - (90 + 30)$|\n||= {{{correctAnswer0}}}{{{suffix0}}}|\n\n</div>\n"}]},{"vars":[{"varval":"Three identical triangles are used to make the figure below.\n\nsm_img https://teacher.smartermaths.com.au/wp-content/uploads/2022/10/Geom_NAPX-F4-CA27-SA_v1.svg 120 indent3 vpad\n\nWhat is the size of the angle marked $\\large x \\degree$?\n\n"},{"varval":"Consider the centre angles of each triangle, $\\large c\\degree$\n\nsm_img https://teacher.smartermaths.com.au/wp-content/uploads/2018/06/NAPX-F4-CA27-SA-Answer.svg 100 indent3 vpad\n\n<div class=\"aligned\">\n\n|||\n|-:|-|\n|3 $\\times\\ \\large c$$\\degree$|= 180$\\degree$|\n|$\\large c$$\\degree$|= 60$\\degree$|\n|$\\therefore \\large x$$\\degree$|= $180 - (90 + 60)$|\n||= {{{correctAnswer0}}}{{{suffix0}}}|\n\n</div>\n"}]},{"vars":[{"varval":"Four identical triangles are used to make the figure below. \n\nsm_img https://teacher.smartermaths.com.au/wp-content/uploads/2022/10/Geom_NAPX-F4-CA27-SA_v2.svg 130 indent3 vpad\n\nWhat is the size of the angle marked $\\large y \\degree$?\n\n"},{"varval":"Consider the centre angles of each triangle, $\\large c\\degree$\n\nsm_img https://teacher.smartermaths.com.au/wp-content/uploads/2018/06/NAPX-F4-CA27-SA-Answer.svg 90 indent3 vpad\n\n<div class=\"aligned\">\n\n|||\n|-:|-|\n|4 $\\times\\ \\large c$$\\degree$|= 180$\\degree$|\n|$\\large c$$\\degree$|= 45$\\degree$|\n|$\\therefore \\large y$$\\degree$|= $180 - (90 + 45)$|\n||= {{{correctAnswer0}}}{{{suffix0}}}|\n\n</div>\n"}]},{"vars":[{"varval":"Five identical triangles are used to make the figure below.\n\nsm_img https://teacher.smartermaths.com.au/wp-content/uploads/2022/10/Geom_NAPX-F4-CA27-SA_v3.svg 140 indent3 vpad\n\nWhat is the size of the angle marked $\\large m \\degree$?\n\n"},{"varval":"Consider the centre angles of each triangle, $\\large c\\degree$\n\nsm_img https://teacher.smartermaths.com.au/wp-content/uploads/2018/06/NAPX-F4-CA27-SA-Answer.svg 90 indent3 vpad\n\n<div class=\"aligned\">\n\n|||\n|-:|-|\n|5 $\\times\\ \\large c$$\\degree$|= 180$\\degree$|\n|$\\large c$$\\degree$|= 36$\\degree$|\n|$\\therefore \\large m$$\\degree$|= $180 - (90 + 36)$|\n||= {{{correctAnswer0}}}{{{suffix0}}}|\n\n</div>\n"}]},{"vars":[{"varval":"Twelve identical triangles are used to make the figure below.\n\nsm_img https://teacher.smartermaths.com.au/wp-content/uploads/2022/10/Geom_NAPX-F4-CA27-SA_v4.svg 180 indent3 vpad\n\nWhat is the size of the angle marked $\\large x \\degree$?\n\n"},{"varval":"Consider the centre angles of each triangle, $\\large c\\degree$\n\nsm_img https://teacher.smartermaths.com.au/wp-content/uploads/2018/06/NAPX-F4-CA27-SA-Answer.svg 90 indent3 vpad\n\n<div class=\"aligned\">\n\n|||\n|-:|-|\n|12 $\\times\\ \\large c$$\\degree$|= 180$\\degree$|\n|$\\large c$$\\degree$|= 15$\\degree$|\n|$\\therefore \\large x$$\\degree$|= $180 - (90 + 15)$|\n||= {{{correctAnswer0}}}{{{suffix0}}}|\n\n</div>\n"}]}]

  456. VAR1-5 ... don't change all but students have a very simple calculator and no ability to do square roots (makes the pythagorean triples v difficult). - no need to change all, but can you simplify a couple of the triples to easier ones (with multiples of 10) like 30,40,50 and 60,80,100 - on these posts, the font on the images was much smaller than the writing font on my browser (i have adjusted) - worth checking what you see I've made most of them easier. Also have made the font bigger.

    <div class="sm_mode"> {{{question}}} </div>

    [{"vars":[{"varval":"\nShanti travels directly from from $P$ to $R$ as shown in the diagram below.\n\n<br>\n\nsm_img https://teacher.smartermaths.com.au/wp-content/uploads/2018/05/NAPX-I4-NC31.svg 290 indent vpad\n\n<br>If she takes the route $P → Q → R$, how far extra does she travel?\n\n"},{"varval":"sm_nogap Using Pythagoras:\n\n<div class=\"aligned\">\n\n|||\n|-:|-|\n|$PR^2$|= $QR^2 + QP^2$|\n||= $50^2 + 120^2$|\n||= 130$^2$|\n|$PR$|= 130 m|\n\n</div>\n\n<br>\n\nsm_nogap $\\therefore$ Extra distance travelled\n\n>>= 120 + 50 $-$ 130\n\n>>= {{{correctAnswer0}}} {{{suffix0}}}\n"}]},{"vars":[{"varval":"\nPortia travels directly from from $X$ to $Z$ as shown in the diagram below.\n\n<br>\n\nsm_img https://teacher.smartermaths.com.au/wp-content/uploads/2022/11/Geom_NAPX-I4-NC31-SA_v5b.svg 330 indent vpad\n\n<br>If she takes the route $X → Y → Z$, how far extra does she travel?\n\n"},{"varval":"sm_nogap Using Pythagoras:\n\n<div class=\"aligned\">\n\n|||\n|-:|-|\n|$XZ^2$|= $XY^2 + YZ^2$|\n||= $7^2 + 24^2$|\n||= 625|\n|$XZ$|= $25^2$|\n|$XZ$|= 25 km|\n\n</div>\n\n<br>\n\nsm_nogap $\\therefore$ Extra distance travelled\n\n>>= 7 + 24 $-$ 25\n\n>>= {{{correctAnswer0}}} {{{suffix0}}}\n"}]},{"vars":[{"varval":"\nChev travels directly from from $M$ to $L$ as shown in the diagram below.\n\n<br>\n\nsm_img https://teacher.smartermaths.com.au/wp-content/uploads/2022/11/Geom_NAPX-I4-NC31-SA_v3a.svg 230 indent vpad\n\n<br>If he takes the route $M → N → L$, how far extra does he travel?\n\n"},{"varval":"sm_nogap Using Pythagoras:\n\n<div class=\"aligned\">\n\n|||\n|-:|-|\n|$ML^2$|= $MN^2 + NL^2$|\n||= $80^2 + 60^2$\n||= 100$^2$|\n|$ML$|= 100 m\n\n</div>\n\n<br>\n\nsm_nogap $\\therefore$ Extra distance travelled\n\n>>= 60 + 80 $-$ 100\n\n>>= {{{correctAnswer0}}} {{{suffix0}}}\n"}]},{"vars":[{"varval":"\nSean travels directly from from $X$ to $Y$ as shown in the diagram below.\n\n<br>\n\nsm_img https://teacher.smartermaths.com.au/wp-content/uploads/2022/11/Geom_NAPX-I4-NC31-SA_v1b.svg 270 indent vpad\n\n<br>If he takes the route $X → Z → Y$, how far extra does he travel?\n\n"},{"varval":"sm_nogap Using Pythagoras:\n\n<div class=\"aligned\">\n\n|||\n|-:|-|\n|$XY^2$|= $YZ^2 + ZX^2$|\n||= $12^2 + 5^2$\n||= 13$^2$|\n|$XY$|= 13 km|\n\n</div>\n\n<br>\n\nsm_nogap $\\therefore$ Extra distance travelled\n\n>>= 12 + 5 $-$ 13\n\n>>= {{{correctAnswer0}}} {{{suffix0}}}\n"}]},{"vars":[{"varval":"\nPenny travels directly from from $A$ to $C$ as shown in the diagram below.\n\n<br>\n\nsm_img https://teacher.smartermaths.com.au/wp-content/uploads/2022/11/Geom_NAPX-I4-NC31-SA_v4a.svg 240 indent vpad\n\n<br>If she takes the route $A → B → C$, how far extra does she travel?\n\n"},{"varval":"sm_nogap Using Pythagoras:\n\n<div class=\"aligned\">\n\n|||\n|-:|-|\n|$AC^2$|= $AB^2 + BC^2$|\n||= $6^2 + 8^2$|\n||= 10$^2$|\n|$AC$|= 10 km|\n\n</div>\n\n<br>\n\nsm_nogap $\\therefore$ Extra distance travelled\n\n>>= 6 + 8 $-$ 10\n\n>>= {{{correctAnswer0}}} {{{suffix0}}}\n"}]},{"vars":[{"varval":"\nBjorn travels directly from from $B$ to $C$ as shown in the diagram below.\n\n<br>\n\nsm_img https://teacher.smartermaths.com.au/wp-content/uploads/2022/11/Geom_NAPX-I4-NC31-SA_v2a.svg 290 indent vpad\n\n<br>If he takes the route $C → A → B$, how far extra does he travel?\n\n"},{"varval":"sm_nogap Using Pythagoras:\n\n<div class=\"aligned\">\n\n|||\n|-:|-|\n|$BC^2$|= $AC^2 + AB^2$|\n||= $30^2 + 40^2$|\n||= 50$^2$|\n|$BC$|= 50 m|\n\n</div>\n\n<br>\n\nsm_nogap $\\therefore$ Extra distance travelled\n\n>>= 30 + 40 $-$ 50\n\n>>= {{{correctAnswer0}}} {{{suffix0}}}\n"}]}]

  457. <div class="sm_mode"> {{{question}}} </div>

    [{"vars":[{"varval":"A regular pentagon can be divided into three triangles from one vertex, as shown below.\n\n<br>\n\nsm_img https://teacher.smartermaths.com.au/wp-content/uploads/2020/09/NAPX-I4-CA25rev.svg 160 indent3 vpad\n\n<br>What is the size of the internal angle $\\large x$$\\degree$?"},{"varval":"sm_nogap Sum of interior angles\n\n<div class=\"aligned\">\n\n>|||\n|-|-|\n||= 3 × 180|\n||= 540$\\degree$|\n\n</div>\n\n<br>\n\n<div class=\"aligned\">\n\n|||\n|-|-|\n|$\\therefore \\large x$$\\degree$|= 540 ÷ 5|\n||= {{{correctAnswer0}}}{{{suffix0}}}|\n\n</div>\n\n"}]},{"vars":[{"varval":"A regular hexagon can be divided into four triangles from one vertex, as shown below.\n\n<br>\n\nsm_img https://teacher.smartermaths.com.au/wp-content/uploads/2022/10/Geom_NAPX-I4-CA25-SA_v1.svg 160 indent3 vpad\n\n<br>What is the size of the internal angle $\\large x$$\\degree$?"},{"varval":"sm_nogap Sum of interior angles\n\n<div class=\"aligned\">\n\n>|||\n|-|-|\n||= 4 × 180|\n||= 720$\\degree$|\n\n</div>\n\n<br>\n\n<div class=\"aligned\">\n\n|||\n|-|-|\n|$\\therefore \\large x$$\\degree$|= 720 ÷ 6|\n||= {{{correctAnswer0}}}{{{suffix0}}}|\n\n</div>\n\n"}]},{"vars":[{"varval":"A regular octagon can be divided into six triangles from one vertex, as shown below.\n\n<br>\n\nsm_img https://teacher.smartermaths.com.au/wp-content/uploads/2022/10/Geom_NAPX-I4-CA25-SA_v2.svg 260 indent3 vpad\n\n<br>What is the size of the internal angle $\\large x$$\\degree$?"},{"varval":"sm_nogap Sum of interior angles\n\n<div class=\"aligned\">\n\n>|||\n|-|-|\n||= 6 × 180|\n||= 1080$\\degree$|\n\n</div>\n\n<br>\n\n<div class=\"aligned\">\n\n|||\n|-|-|\n|$\\therefore \\large x$$\\degree$|= 1080 ÷ 8|\n||= {{{correctAnswer0}}}{{{suffix0}}}|\n\n</div>\n\n"}]},{"vars":[{"varval":"A regular decagon can be divided into eight triangles from one vertex, as shown below.\n\n<br>\n\nsm_img https://teacher.smartermaths.com.au/wp-content/uploads/2022/10/Geom_NAPX-I4-CA25-SA_v3.svg 260 indent3 vpad\n\n<br>What is the size of the internal angle $\\large x$$\\degree$?"},{"varval":"sm_nogap Sum of interior angles\n\n<div class=\"aligned\">\n\n>|||\n|-|-|\n||= 8 × 180|\n||= 1440$\\degree$|\n\n</div>\n\n<br>\n\n<div class=\"aligned\">\n\n|||\n|-|-|\n|$\\therefore \\large x$$\\degree$|= 1440 ÷ 10|\n||= {{{correctAnswer0}}}{{{suffix0}}}|\n\n</div>\n\n"}]},{"vars":[{"varval":"A regular nonagon can be divided into seven triangles from one vertex, as shown below.\n\n<br>\n\nsm_img https://teacher.smartermaths.com.au/wp-content/uploads/2022/10/Geom_NAPX-I4-CA25-SA_v4.svg 260 indent3 vpad\n\n<br>What is the size of the internal angle $\\large x$$\\degree$?"},{"varval":"sm_nogap Sum of interior angles\n\n<div class=\"aligned\">\n\n>|||\n|-|-|\n||= 7 × 180|\n||= 1260$\\degree$|\n\n</div>\n\n<br>\n\n<div class=\"aligned\">\n\n|||\n|-|-|\n|$\\therefore \\large x$$\\degree$|= 1260 ÷ 9|\n||= {{{correctAnswer0}}}{{{suffix0}}}|\n\n</div>\n\n"}]},{"vars":[{"varval":"A regular dodecagon can be divided into ten triangles from one vertex, as shown below.\n\n<br>\n\nsm_img https://teacher.smartermaths.com.au/wp-content/uploads/2022/10/Geom_NAPX-I4-CA25-SA_v5.svg 280 indent3 vpad\n\n<br>What is the size of the internal angle $\\large x$$\\degree$?"},{"varval":"sm_nogap Sum of interior angles\n\n<div class=\"aligned\">\n\n>|||\n|-|-|\n||= 10 × 180|\n||= 1800$\\degree$|\n\n</div>\n\n<br>\n\n<div class=\"aligned\">\n\n|||\n|-|-|\n|$\\therefore \\large x$$\\degree$|= 1800 ÷ 12|\n||= {{{correctAnswer0}}}{{{suffix0}}}|\n\n</div>\n\n"}]}]

  458. VAR1-4 ... isosceles triangle dimensions don't work in these examples (thinking it will only work for a hexagon where you can make equilateral triangles)

    <div class="sm_mode"> {{{question}}} </div>

    [{"vars":[{"varval":"A regular hexagon has a perimeter of 30 cm. \n\nIma cuts it into 6 equilateral triangles.\n\n<br>\n\nsm_img https://teacher.smartermaths.com.au/wp-content/uploads/2020/09/NAPX-H4-NC27.svg 260 indent2 vpad\n\n<br>What is the perimeter of one of the triangles?\n"},{"varval":"sm_nogap Length of one side of hexagon\n\n<div class=\"aligned\">\n\n>|||\n|-|-|\n||= 30 $\\div$ 5|\n||= 5 cm|\n\n</div>\n\n<br>\n\nsm_img https://teacher.smartermaths.com.au/wp-content/uploads/2020/09/NAPX-H4-NC27-Answer1.svg 110 indent3 vpad\n\n<div class=\"aligned\">\n\n|||\n|-|-|\n|$\\therefore$ Perimeter|= 5 + 5 + 5|\n||= {{{correctAnswer}}} |\n\n</div>\n"}]},{"vars":[{"varval":"A regular pentagon has a perimeter of 30 cm. \n\nIan cuts it into 5 isosceles triangles.\n\n<br>\n\nsm_img https://teacher.smartermaths.com.au/wp-content/uploads/2022/10/Geom_NAPX-H4-NC27_v1q.svg 260 indent2 vpad\n\n<br>What is the perimeter of one of the isosceles triangles if the equal sides are 5 cm in length?\n"},{"varval":"sm_nogap Length of one side of hexagon\n\n<div class=\"aligned\">\n\n>|||\n|-|-|\n||= 30 $\\div$ 5|\n||= 6 cm|\n\n</div>\n\n<br>\n\nsm_img https://teacher.smartermaths.com.au/wp-content/uploads/2022/10/Geom_NAPX-H4-NC27_v1aws.svg 140 indent3 vpad\n\n<div class=\"aligned\">\n\n|||\n|-|-|\n|$\\therefore$ Perimeter|= 6 + 5 + 5|\n||= {{{correctAnswer}}} |\n\n</div>\n"}]},{"vars":[{"varval":"A regular octagon has a perimeter of 96 cm.\n\nUma cuts it into 8 isosceles triangles.\n\n<br>\n\nsm_img https://teacher.smartermaths.com.au/wp-content/uploads/2022/10/Geom_NAPX-H4-NC27_v2aq.svg 340 indent2 vpad\n\n<br>What is the perimeter of one of the isosceles triangles if the equal sides are 10 cm in length?\n"},{"varval":"sm_nogap Length of one side of hexagon\n\n<div class=\"aligned\">\n\n>|||\n|-|-|\n||= 96 $\\div$ 8|\n||= 12 cm|\n\n</div>\n\n<br>\n\nsm_img https://teacher.smartermaths.com.au/wp-content/uploads/2022/10/Geom_NAPX-H4-NC27_v2ws.svg 130 indent3 vpad\n\n<div class=\"aligned\">\n\n|||\n|-|-|\n|$\\therefore$ Perimeter|= 12 + 10 + 10|\n||= {{{correctAnswer}}} |\n\n</div>\n"}]},{"vars":[{"varval":"A regular dodecagon has a perimeter of 108 cm.\n\nIggy cuts it into 12 isosceles triangles.\n\n<br>\n\nsm_img https://teacher.smartermaths.com.au/wp-content/uploads/2022/10/Geom_NAPX-H4-NC27_v4s.svg 340 indent2 vpad\n\n<br>What is the perimeter of one of the isosceles triangles if the equal sides are 15 cm in length?\n"},{"varval":"sm_nogap Length of one side of dodecagon \n\n<div class=\"aligned\">\n\n>|||\n|-|-|\n||= 108 $\\div$ 12|\n||= 9 cm|\n\n</div>\n\n<br>\n\nsm_img https://teacher.smartermaths.com.au/wp-content/uploads/2022/10/Geom_NAPX-H4-NC27_v4ws.svg 120 indent3 vpad\n\n<div class=\"aligned\">\n\n|||\n|-|-|\n|$\\therefore$ Perimeter|= 9 + 15 + 15|\n||= {{{correctAnswer}}} |\n\n</div>\n"}]},{"vars":[{"varval":"A regular decagon has a perimeter of 240 cm. \n\nJorja cuts it into 10 isosceles triangles.\n\n<br>\n\nsm_img https://teacher.smartermaths.com.au/wp-content/uploads/2022/10/Geom_NAPX-H4-NC27_v3s.svg 320 indent2 vpad\n\n<br>What is the perimeter of one of the isosceles triangles if the equal sides are 20 cm in length?"},{"varval":"sm_nogap Length of one side of decagon\n\n<div class=\"aligned\">\n\n>|||\n|-|-|\n||= 240 $\\div$ 10|\n||= 24 cm|\n\n</div>\n\n<br>\n\nsm_img https://teacher.smartermaths.com.au/wp-content/uploads/2022/10/Geom_NAPX-H4-NC27_v3sa.svg 130 indent3 vpad\n\n<div class=\"aligned\">\n\n|||\n|-|-|\n|$\\therefore$ Perimeter|= 24 + 20 + 20|\n||= {{{correctAnswer}}} |\n\n</div>\n"}]}]

  459. <div class="sm_mode"> {{{question}}} </div>

    [{"vars":[{"varval":"Which statement is always true?"},{"varval":"{{{correctAnswer}}}\n\nsm_img https://teacher.smartermaths.com.au/wp-content/uploads/2018/04/NAPX-H4-CA26.svg 160 indent vpad\n"}]},{"vars":[{"varval":"Which statement is always true?"},{"varval":"{{{correctAnswer}}}\n\nsm_img https://teacher.smartermaths.com.au/wp-content/uploads/2022/10/Geom_NAPX-H4-CA26_v1.svg 200 indent vpad\n"}]},{"vars":[{"varval":"Which statement is always true?"},{"varval":"{{{correctAnswer}}}\n\nsm_img https://teacher.smartermaths.com.au/wp-content/uploads/2022/10/Geom_NAPX-H4-CA26_v6.svg 140 indent2 vpad\n"}]},{"vars":[{"varval":"Which statement is always true?"},{"varval":"{{{correctAnswer}}}\n\nsm_img https://teacher.smartermaths.com.au/wp-content/uploads/2022/10/Geom_NAPX-H4-CA26_v3.svg 160 indent vpad\n"}]},{"vars":[{"varval":"Which statement is always true?"},{"varval":"{{{correctAnswer}}}\n\nsm_img https://teacher.smartermaths.com.au/wp-content/uploads/2022/10/Geom_NAPX-H4-CA26_v4.svg 120 indent2 vpad\n"}]},{"vars":[{"varval":"Which statement is always true?"},{"varval":"{{{correctAnswer}}}\n\nsm_img https://teacher.smartermaths.com.au/wp-content/uploads/2022/10/Geom_NAPX-H4-CA26_v5.svg 210 indent2 vpad\n"}]}]

  460. VAR3 ... the original version stated "dodecahedron" but the image only had 10 sides. I changed the question back to match the image until you can update the image (var3 is now the same shape as var0)

    <div class="sm_mode"> {{{question}}} </div>

    [{"vars":[{"varval":"Five lines of symmetry are drawn on a regular decagon pictured below.\n\nsm_img https://teacher.smartermaths.com.au/wp-content/uploads/2018/08/NAPX-G3-CA29-SA.svg 200 indent vpad\n\nWhat is the size of the angle between each line of symmetry at the centre of the decagon?\n"},{"varval":"sm_nogap There are 10 angles at the centre of the decagon of the same size.\n\n<div class=\"aligned\">\n\n|||\n|-|-|\n|$\\therefore$ Size of angle|= 360 ÷ 10|\n||= {{{correctAnswer0}}}{{{suffix0}}}|\n\n</div>"}]},{"vars":[{"varval":"Three lines of symmetry are drawn on a regular hexagon pictured below. \n\nsm_img https://teacher.smartermaths.com.au/wp-content/uploads/2022/10/Geom_NAPX-G3-CA29-SA_v1.svg 180 indent vpad\n\nWhat is the size of the angle between each line of symmetry at the centre of the hexagon?\n"},{"varval":"sm_nogap There are 6 angles at the centre of the hexagon of the same size.\n\n<div class=\"aligned\">\n\n|||\n|-|-|\n|$\\therefore$ Size of angle|= 360 ÷ 6|\n||= {{{correctAnswer0}}}{{{suffix0}}}|\n\n</div>"}]},{"vars":[{"varval":"Four lines of symmetry are drawn on a regular octagon pictured below.\n\nsm_img https://teacher.smartermaths.com.au/wp-content/uploads/2022/10/Geom_NAPX-G3-CA29-SA_v2.svg 180 indent vpad\n\nWhat is the size of the angle between each line of symmetry at the centre of the octagon?\n"},{"varval":"sm_nogap There are 8 angles at the centre of the octagon of the same size.\n\n<div class=\"aligned\">\n\n|||\n|-|-|\n|$\\therefore$ Size of angle|= 360 ÷ 8|\n||= {{{correctAnswer0}}}{{{suffix0}}}|\n\n</div>"}]},{"vars":[{"varval":"Six lines of symmetry are drawn on a regular dodecagon pictured below. \n\nsm_img https://teacher.smartermaths.com.au/wp-content/uploads/2022/11/Geom_NAPX-G3-CA29-SA_v3b.svg 300 indent vpad\n\nWhat is the size of the angle between each line of symmetry at the centre of the dodecagon?\n"},{"varval":"sm_nogap There are 12 angles at the centre of the dodecagon of the same size.\n\n<div class=\"aligned\">\n\n|||\n|-|-|\n|$\\therefore$ Size of angle|= 360 ÷ 12|\n||= {{{correctAnswer0}}}{{{suffix0}}}|\n\n</div>"}]},{"vars":[{"varval":"Five lines of symmetry are drawn on a regular pentagon pictured below.\n\nsm_img https://teacher.smartermaths.com.au/wp-content/uploads/2022/10/Geom_NAPX-G3-CA29-SA_v3.svg 200 indent vpad\n\nWhat is the size of the angle between each line of symmetry at the centre of the pentagon?\n"},{"varval":"sm_nogap There are 10 angles at the centre of the pentagon of the same size.\n\n<div class=\"aligned\">\n\n|||\n|-|-|\n|$\\therefore$ Size of angle|= 360 ÷ 10|\n||= {{{correctAnswer0}}}{{{suffix0}}}|\n\n</div>"}]},{"vars":[{"varval":"Three lines of symmetry are drawn on a equilateral triangle pictured below.\n\nsm_img https://teacher.smartermaths.com.au/wp-content/uploads/2022/10/Geom_NAPX-G3-CA29-SA_v4.svg 200 indent vpad\n\nWhat is the size of the angle between each line of symmetry at the centre of the equilateral triangle?\n"},{"varval":"sm_nogap There are 6 angles at the centre of the equilateral triangle of the same size.\n\n<div class=\"aligned\">\n\n|||\n|-|-|\n|$\\therefore$ Size of angle|= 360 ÷ 6|\n||= {{{correctAnswer0}}}{{{suffix0}}}|\n\n</div>"}]}]

  461. <div class="sm_mode"> {{{question}}} </div>

    [{"vars":[{"varval":"The diagonals of which shape below cross at right-angles?"},{"varval":"A rhombus has diagonals that cross at right angles.\n\n{{{correctAnswer}}}\n\n"}]},{"vars":[{"varval":"The diagonals of which shape below bisect each other?"},{"varval":"A rhombus has diagonals that bisect each other.\n\n{{{correctAnswer}}}\n"}]},{"vars":[{"varval":"Which shape below has two pairs of equal adjacent sides?"},{"varval":"A kite has two pairs of equal adjacent sides.\n\n{{{correctAnswer}}}\n"}]},{"vars":[{"varval":"The opposite angles of which shape below are equal?"},{"varval":"The opposite angles of a parallelogram are equal.\n\n{{{correctAnswer}}}\n"}]},{"vars":[{"varval":"The diagonals of which shape below **do not** intersect at right angles?"},{"varval":"The diagonals of a rectangle **do not** intersect at right angles.\n\n{{{correctAnswer}}}\n"}]},{"vars":[{"varval":"Which shape below **does not** have opposite sides equal and parallel?"},{"varval":"A trapezium **does not** have opposite sides equal and parallel.\n\n{{{correctAnswer}}}\n\n"}]}]

  462. <div class="sm_mode"> {{{question}}} </div>

    [{"vars":[{"varval":"Which one of the following triangles has an area of 16 cm$^2$ ?\n\n\r\n\r\n(Triangles are not drawn to scale)\n\n"},{"varval":"{{{correctAnswer}}}\n\n<div class=\"aligned\">\n\n|||\n|-|-|\n|Area|= $\\dfrac{1}{2} \\times \\large b$ × $\\large h$|\n||= $\\dfrac{1}{2} \\times 8 \\times 4$|\n||= 16 cm$^2$|\n\n</div>"}]},{"vars":[{"varval":"Which one of the following triangles has an area of 20 cm$^2$ ?\n\n\r\n\r\n(Triangles are not drawn to scale)\n\n"},{"varval":"{{{correctAnswer}}}\n\n<div class=\"aligned\">\n\n|||\n|-|-|\n|Area|= $\\dfrac{1}{2} \\times \\large b$ × $\\large h$|\n||= $\\dfrac{1}{2} \\times 5 \\times 8$|\n||= 20 cm$^2$|\n\n</div>"}]},{"vars":[{"varval":"Which one of the following triangles has an area of 10 cm$^2$ ?\n\n\r\n\r\n(Triangles are not drawn to scale)\n\n"},{"varval":"{{{correctAnswer}}}\n\n<div class=\"aligned\">\n\n|||\n|-|-|\n|Area|= $\\dfrac{1}{2} \\times \\large b$ × $\\large h$|\n||= $\\dfrac{1}{2} \\times 5 \\times 4$|\n||= 10 cm$^2$|\n\n</div>"}]},{"vars":[{"varval":"Which one of the following triangles has an area of 15 cm$^2$ ?\n\n\r\n\r\n(Triangles are not drawn to scale)\n\n"},{"varval":"{{{correctAnswer}}}\n\n<div class=\"aligned\">\n\n|||\n|-|-|\n|Area|= $\\dfrac{1}{2} \\times \\large b$ × $\\large h$|\n||= $\\dfrac{1}{2} \\times 5 \\times 6$|\n||= 15 cm$^2$|\n\n</div>"}]},{"vars":[{"varval":"Which one of the following triangles has an area of 12 cm$^2$ ?\n\n\r\n\r\n(Triangles are not drawn to scale)\n\n"},{"varval":"{{{correctAnswer}}}\n\n<div class=\"aligned\">\n\n|||\n|-|-|\n|Area|= $\\dfrac{1}{2} \\times \\large b$ × $\\large h$|\n||= $\\dfrac{1}{2} \\times 12 \\times 2$|\n||= 12 cm$^2$|\n\n</div>"}]},{"vars":[{"varval":"Which one of the following triangles has an area of 100 cm$^2$ ?\n\n\r\n\r\n(Triangles are not drawn to scale)\n\n"},{"varval":"{{{correctAnswer}}}\n\n<div class=\"aligned\">\n\n|||\n|-|-|\n|Area|= $\\dfrac{1}{2} \\times \\large b$ × $\\large h$|\n||= $\\dfrac{1}{2} \\times 20 \\times 10$|\n||= 100 cm$^2$|\n\n</div>"}]}]

  463. <div class="sm_mode"> $A, B$ and $C$ are vertices on the cube below. sm_img https://teacher.smartermaths.com.au/wp-content/uploads/2018/06/NAPX-G4-NC23_1.svg 240 indent vpad <br>What is the best description of $\Delta ABC$? </div>

    [{"vars":null}]

  464. <div class="sm_mode"> Bo draws and labels 3 points, on a Cartesian plane. <br> sm_img https://teacher.smartermaths.com.au/wp-content/uploads/2018/05/NAPX-H4-NC18.svg 320 indent vpad He wants to add another point so that all 4 points can be joined to make a parallelogram. Which of these is a possible coordinate for the 4th point? </div>

    [{"vars":null}]

  465. <div class="sm_mode"> sm_img https://teacher.smartermaths.com.au/wp-content/uploads/2018/06/NAPX-G4-NC18_1.svg 220 indent vpad Two triangles, $AOB$ and $DOC$ are drawn in a circle with centre $O$. The two triangles are best described as: </div>

    [{"vars":null}]

  466. <div class="sm_mode"> {{{question}}} </div>

    [{"vars":[{"varval":"Lines $AB$ and $CD$ are parallel.\n\n\r\nLine $EF$ intersects lines $AB$ and $CD$ as shown.\n\n<br>\n\nsm_img https://teacher.smartermaths.com.au/wp-content/uploads/2018/04/NAPX-I4-CA20.svg 220 indent3 vpad\n\n<br>Which pair of angles are equal?\n"},{"varval":"sm_img https://teacher.smartermaths.com.au/wp-content/uploads/2018/04/NAPX-I4-CA20ans.svg 220 indent vpad\n\n{{{correctAnswer}}}\n\n\r\n(Alternate angles)"}]},{"vars":[{"varval":"Lines $AB$ and $CD$ are parallel.\n\n\r\nLine $EF$ intersects lines $AB$ and $CD$ as shown.\n\n<br>\n\nsm_img https://teacher.smartermaths.com.au/wp-content/uploads/2022/10/Geom_NAPX-I4-CA20_NAPX-I3-CA28_v1.svg 220 indent3 vpad\n\n<br>Which pair of angles are equal?\n"},{"varval":"sm_img https://teacher.smartermaths.com.au/wp-content/uploads/2022/10/Geom_NAPX-I4-CA20_NAPX-I3-CA28_v1_ws.svg 220 indent vpad\n\n{{{correctAnswer}}}\n\n\r\n(Alternate angles)"}]},{"vars":[{"varval":"Lines $AB$ and $CD$ are parallel.\n\n\r\nLine $XY$ intersects lines $AB$ and $CD$ as shown.\n\n<br>\n\nsm_img https://teacher.smartermaths.com.au/wp-content/uploads/2022/10/Geom_NAPX-I4-CA20_NAPX-I3-CA28_v2.svg 220 indent3 vpad\n\n<br>Which pair of angles are equal?\n"},{"varval":"sm_img https://teacher.smartermaths.com.au/wp-content/uploads/2022/10/Geom_NAPX-I4-CA20_NAPX-I3-CA28_v2_ws.svg 220 indent vpad\n\n{{{correctAnswer}}}\n\n\r\n(Alternate angles)"}]},{"vars":[{"varval":"Lines $AB$ and $CD$ are parallel.\n\n\r\nLine $XY$ intersects lines $AB$ and $CD$ as shown.\n\n<br>\n\nsm_img https://teacher.smartermaths.com.au/wp-content/uploads/2022/10/Geom_NAPX-I4-CA20_NAPX-I3-CA28_v2.svg 220 indent3 vpad\n\n<br>Which pair of angles are equal?\n"},{"varval":"sm_img https://teacher.smartermaths.com.au/wp-content/uploads/2022/10/Geom_NAPX-I4-CA20_NAPX-I3-CA28_v3_ws.svg 220 indent vpad\n\n{{{correctAnswer}}}\n\n\r\n(Alternate angles)"}]},{"vars":[{"varval":"Lines $EF$ and $GH$ are parallel.\n\n\r\nLine $XY$ intersects lines $EF$ and $GH$ as shown.\n\n<br>\n\nsm_img https://teacher.smartermaths.com.au/wp-content/uploads/2022/10/Geom_NAPX-I4-CA20_NAPX-I3-CA28_v4.svg 220 indent3 vpad\n\n<br>Which pair of angles are equal?\n"},{"varval":"sm_img https://teacher.smartermaths.com.au/wp-content/uploads/2022/10/Geom_NAPX-I4-CA20_NAPX-I3-CA28_v4_ws.svg 220 indent vpad\n\n{{{correctAnswer}}}\n\n\r\n(Alternate angles)"}]},{"vars":[{"varval":"Lines $EF$ and $GH$ are parallel.\n\n\r\nLine $XY$ intersects lines $EF$ and $GH$ as shown.\n\n<br>\n\nsm_img https://teacher.smartermaths.com.au/wp-content/uploads/2022/10/Geom_NAPX-I4-CA20_NAPX-I3-CA28_v4.svg 220 indent3 vpad\n\n<br>Which pair of angles are equal?\n"},{"varval":"sm_img https://teacher.smartermaths.com.au/wp-content/uploads/2022/10/Geom_NAPX-I4-CA20_NAPX-I3-CA28_v5_ws.svg 220 indent vpad\n\n{{{correctAnswer}}}\n\n\r\n(Alternate angles)"}]}]

  467. <div class="sm_mode"> {{{question}}} </div>

    [{"vars":[{"varval":"sm_img https://teacher.smartermaths.com.au/wp-content/uploads/2018/05/NAPX-I4-NC19.svg 170 indent3 vpad\n\nThe square $ABDC$ is cut into two triangles along the diagonal $AD$.\n\nWhich of the following best describes triangle $ABD$?\n"},{"varval":"sm_img https://teacher.smartermaths.com.au/wp-content/uploads/2018/05/NAPX-I4-NC19-ans.svg 170 indent vpad\n\n{{{correctAnswer}}}\n"}]},{"vars":[{"varval":"sm_img https://teacher.smartermaths.com.au/wp-content/uploads/2022/11/Geom_NAPX-I4-NC19_NAPX-I3-NC22_v1.svg 250 indent3 vpad\n\nThe rectangle $ABCD$ is cut into two triangles along the diagonal $AC$.\n\nWhich of the following best describes triangle $ABC$?\n"},{"varval":"sm_img https://teacher.smartermaths.com.au/wp-content/uploads/2022/11/Geom_NAPX-I4-NC19_NAPX-I3-NC22_v1_ws.svg 250 indent vpad\n\n{{{correctAnswer}}}\n"}]},{"vars":[{"varval":"sm_img https://teacher.smartermaths.com.au/wp-content/uploads/2022/11/Geom_NAPX-I4-NC19_NAPX-I3-NC22_v2.svg 190 indent3 vpad\n\nThe regular pentagon $ABCDE$ is cut in two along the line $BE$.\n\nWhich of the following best describes triangle $AEB$?\n"},{"varval":"sm_img https://teacher.smartermaths.com.au/wp-content/uploads/2022/11/Geom_NAPX-I4-NC19_NAPX-I3-NC22_v2_ws.svg 190 indent vpad\n\n{{{correctAnswer}}}\n"}]},{"vars":[{"varval":"sm_img https://teacher.smartermaths.com.au/wp-content/uploads/2022/11/Geom_NAPX-I4-NC19_NAPX-I3-NC22_v3.svg 190 indent3 vpad\n\nThe regular hexagon $ABCDEF$ is cut in two along the line $DF$.\n\nWhich of the following best describes triangle $DEF$?\n"},{"varval":"sm_img https://teacher.smartermaths.com.au/wp-content/uploads/2022/11/Geom_NAPX-I4-NC19_NAPX-I3-NC22_v3_ws.svg 110 indent2 vpad\n\n{{{correctAnswer}}}\n"}]},{"vars":[{"varval":"sm_img https://teacher.smartermaths.com.au/wp-content/uploads/2022/11/Geom_NAPX-I4-NC19_NAPX-I3-NC22_v4.svg 250 indent3 vpad\n\nThe regular octagon $ABCDEFGH$ is cut in two along the line $AC$.\n\nWhich of the following best describes triangle $ABC$?\n"},{"varval":"sm_img https://teacher.smartermaths.com.au/wp-content/uploads/2022/11/Geom_NAPX-I4-NC19_NAPX-I3-NC22_v4_ws.svg 210 indent2 vpad\n\n{{{correctAnswer}}}\n"}]}]

  468. <div class="sm_mode"> {{{question}}} </div>

    [{"vars":[{"varval":"A regular hexagon has six internal angles.\n\nWhat type of angle is each internal angle?\n"},{"varval":"90$\\degree$ < {{{correctAnswer}}} $< 180\\degree$\n\nsm_img https://teacher.smartermaths.com.au/wp-content/uploads/2021/03/NAPX9-TLD-18-3-293x300.png 200 indent vpad"}]},{"vars":[{"varval":"A regular pentagon has five internal angles.\n\nWhat type of angle is each internal angle?\n"},{"varval":"90$\\degree$ < {{{correctAnswer}}} $< 180\\degree$\n\nsm_img https://teacher.smartermaths.com.au/wp-content/uploads/2022/11/Geom_NAPX9-TLD-CA18-v3_v1.svg 200 indent vpad"}]},{"vars":[{"varval":"A regular octagon has eight internal angles.\n\nWhat type of angle is each internal angle?\n"},{"varval":"90$\\degree$ < {{{correctAnswer}}} $< 180\\degree$\n\nsm_img https://teacher.smartermaths.com.au/wp-content/uploads/2022/11/Geom_NAPX9-TLD-CA18-v3_v2.svg 200 indent vpad"}]},{"vars":[{"varval":"A regular decagon has ten internal angles.\n\nWhat type of angle is each internal angle?\n"},{"varval":"90$\\degree$ < {{{correctAnswer}}} $< 180\\degree$\n\nsm_img https://teacher.smartermaths.com.au/wp-content/uploads/2022/11/Geom_NAPX9-TLD-CA18-v3_v3a.svg 200 indent vpad"}]},{"vars":[{"varval":"A regular dodecagon has twelve internal angles.\n\nWhat type of angle is each internal angle?\n"},{"varval":"90$\\degree$ < {{{correctAnswer}}} $< 180\\degree$\n\nsm_img https://teacher.smartermaths.com.au/wp-content/uploads/2022/11/Geom_NAPX9-TLD-CA18-v3_v4.svg 200 indent vpad"}]},{"vars":[{"varval":"A regular nonagon has nine internal angles.\n\nWhat type of angle is each internal angle?\n"},{"varval":"90$\\degree$ < {{{correctAnswer}}} $< 180\\degree$\n\nsm_img https://teacher.smartermaths.com.au/wp-content/uploads/2022/11/Geom_NAPX9-TLD-CA18-v3_v5.svg 200 indent vpad\n"}]}]

  469. <div class="sm_mode"> {{{question}}} </div>

    [{"vars":[{"varval":"A parallelogram is drawn below.\n\n<br>\n\nsm_img https://teacher.smartermaths.com.au/wp-content/uploads/2021/03/NAPX9-TLE-39-2.svg 275 indent2 vpad\n\n<br>What is the size of <span class=\"sm_katex-font\">$\\angle$</span>$BAD$?"},{"varval":"sm_nogap Since diagonally opposite angles are equal:\n\n<div class=\"aligned\">\n\n|||\n|-|-|\n|$\\therefore$ <span class=\"sm_katex-font\">$\\angle$</span>$BAD$|= $\\dfrac{1}{2} (360 - (2 \\times 130))$|\n||= $\\dfrac{1}{2} \\times 100$|\n||= {{{correctAnswer}}}|\n\n</div>"}]},{"vars":[{"varval":"A parallelogram is drawn below.\n\n<br>\n\nsm_img https://teacher.smartermaths.com.au/wp-content/uploads/2022/12/Geom_NAPX9-TLE-39-v3_1.svg 275 indent2 vpad\n\n<br>What is the size of <span class=\"sm_katex-font\">$\\angle$</span>$BAD$?"},{"varval":"sm_nogap Since diagonally opposite angles are equal:\n\n<div class=\"aligned\">\n\n|||\n|-|-|\n|$\\therefore$ <span class=\"sm_katex-font\">$\\angle$</span>$BAD$|= $\\dfrac{1}{2} (360 - (2 \\times 50))$|\n||= $\\dfrac{1}{2} \\times 260$|\n||= {{{correctAnswer}}}|\n\n</div>"}]},{"vars":[{"varval":"A parallelogram is drawn below.\n\n<br>\n\nsm_img https://teacher.smartermaths.com.au/wp-content/uploads/2022/12/Geom_NAPX9-TLE-39-v3_2.svg 275 indent2 vpad\n\n<br>What is the size of <span class=\"sm_katex-font\">$\\angle$</span>$WXY$?"},{"varval":"sm_nogap Since diagonally opposite angles are equal:\n\n<div class=\"aligned\">\n\n|||\n|-|-|\n|$\\therefore$ <span class=\"sm_katex-font\">$\\angle$</span>$WXY$|= $\\dfrac{1}{2} (360 - (2 \\times 120))$|\n||= $\\dfrac{1}{2} \\times 120$|\n||= {{{correctAnswer}}}|\n\n</div>"}]},{"vars":[{"varval":"A parallelogram is drawn below.\n\n<br>\n\nsm_img https://teacher.smartermaths.com.au/wp-content/uploads/2022/12/Geom_NAPX9-TLE-39-v3_3.svg 355 indent2 vpad\n\n<br>What is the size of <span class=\"sm_katex-font\">$\\angle$</span>$MPO$?"},{"varval":"sm_nogap Since diagonally opposite angles are equal:\n\n<div class=\"aligned\">\n\n|||\n|-|-|\n|$\\therefore$ <span class=\"sm_katex-font\">$\\angle$</span>$MPO$|= $\\dfrac{1}{2} (360 - (2 \\times 40))$|\n||= $\\dfrac{1}{2} \\times 280$|\n||= {{{correctAnswer}}}|\n\n</div>"}]},{"vars":[{"varval":"A parallelogram is drawn below.\n\n<br>\n\nsm_img https://teacher.smartermaths.com.au/wp-content/uploads/2022/12/Geom_NAPX9-TLE-39-v3_4.svg 175 indent2 vpad\n\n<br>What is the size of <span class=\"sm_katex-font\">$\\angle$</span>$QRS$?"},{"varval":"sm_nogap Since diagonally opposite angles are equal:\n\n<div class=\"aligned\">\n\n|||\n|-|-|\n|$\\therefore$ <span class=\"sm_katex-font\">$\\angle$</span>$QRS$|= $\\dfrac{1}{2} (360 - (2 \\times 105))$|\n||= $\\dfrac{1}{2} \\times 150$|\n||= {{{correctAnswer}}}|\n\n</div>"}]},{"vars":[{"varval":"A parallelogram is drawn below.\n\n<br>\n\nsm_img https://teacher.smartermaths.com.au/wp-content/uploads/2022/12/Geom_NAPX9-TLE-39-v3_5.svg 175 indent2 vpad\n\n<br>What is the size of <span class=\"sm_katex-font\">$\\angle$</span>$PQR$?"},{"varval":"sm_nogap Since diagonally opposite angles are equal:\n\n<div class=\"aligned\">\n\n|||\n|-|-|\n|$\\therefore$ <span class=\"sm_katex-font\">$\\angle$</span>$PQR$|= $\\dfrac{1}{2} (360 - (2 \\times 77))$|\n||= $\\dfrac{1}{2} \\times 206$|\n||= {{{correctAnswer}}}|\n\n</div>"}]}]

  470. <div class="sm_mode"> {{{question}}} </div>

    [{"vars":[{"varval":"What is the size of the angle marked $\\large x$$\\degree$ in the diagram below?\n\n<br>\n\nsm_img https://teacher.smartermaths.com.au/wp-content/uploads/2018/04/NAPX-J4-CA15.svg 200 indent3 vpad\n"},{"varval":"sm_nogap Since 360$\\degree$ in a quadrilateral.\n\n<div class=\"aligned\">\n\n|||\n|-|-|\n|$\\large x$$\\degree$|= $360 - (103 + 88 + 62)$|\n||= $360 - 253$|\n||= {{{correctAnswer0}}}{{{suffix0}}}|\n\n</div>"}]},{"vars":[{"varval":"What is the size of the angle marked $\\large x$$\\degree$ in the diagram below?\n<br>\n\nsm_img https://teacher.smartermaths.com.au/wp-content/uploads/2022/12/Geom_NAPX-J4-CA15-SA_v1.svg 200 indent2 vpad"},{"varval":"sm_nogap Since 360$\\degree$ in a quadrilateral.\n\n<div class=\"aligned\">\n\n|||\n|-|-|\n|$\\large x$$\\degree$|= $360 - (124 + 77 + 82)$|\n||= $360 - 283$|\n||= {{{correctAnswer0}}}{{{suffix0}}}|\n\n</div>"}]},{"vars":[{"varval":"What is the size of the angle marked $\\large x$$\\degree$ in the diagram below?\n<br>\n\nsm_img https://teacher.smartermaths.com.au/wp-content/uploads/2022/12/Geom_NAPX-J4-CA15-SA_v2.svg 200 indent3 vpad\n"},{"varval":"sm_nogap Since 360$\\degree$ in a quadrilateral.\n\n<div class=\"aligned\">\n\n|||\n|-|-|\n|$\\large x$$\\degree$|= $360 - (68 + 97 + 79)$|\n||= $360 - 244$|\n||= {{{correctAnswer0}}}{{{suffix0}}}|\n\n</div>"}]},{"vars":[{"varval":"What is the size of the angle marked $\\large x$$\\degree$ in the diagram below?\n<br>\n\nsm_img https://teacher.smartermaths.com.au/wp-content/uploads/2022/12/Geom_NAPX-J4-CA15-SA_v3.svg 250 indent2 vpad\n"},{"varval":"sm_nogap Since 360$\\degree$ in a quadrilateral.\n\n<div class=\"aligned\">\n\n|||\n|-|-|\n|$\\large x$$\\degree$|= $360 - (28 + 100 + 30)$|\n||= $360 - 158$|\n||= {{{correctAnswer0}}}{{{suffix0}}}|\n\n</div>"}]},{"vars":[{"varval":"What is the size of the angle marked $\\large x$$\\degree$ in the diagram below?\n<br>\n\nsm_img https://teacher.smartermaths.com.au/wp-content/uploads/2022/12/Geom_NAPX-J4-CA15-SA_v4.svg 140 indent3 vpad\n"},{"varval":"sm_nogap Since 360$\\degree$ in a quadrilateral.\n\n<div class=\"aligned\">\n\n|||\n|-|-|\n|$\\large x$$\\degree$|= $360 - (46 + 95 + 112)$|\n||= $360 - 253$|\n||= {{{correctAnswer0}}}{{{suffix0}}}|\n\n</div>"}]},{"vars":[{"varval":"What is the size of the angle marked $\\large x$$\\degree$ in the diagram below?\n<br>\n\nsm_img https://teacher.smartermaths.com.au/wp-content/uploads/2022/12/Geom_NAPX-J4-CA15-SA_v5.svg 200 indent2 vpad\n"},{"varval":"sm_nogap Since 360$\\degree$ in a quadrilateral.\n\n<div class=\"aligned\">\n\n|||\n|-|-|\n|$\\large x$$\\degree$|= $360 - (55 + 47 + 18)$|\n||= $360 - 120$|\n||= {{{correctAnswer0}}}{{{suffix0}}}|\n\n</div>"}]}]

  471. <div class="sm_mode"> sm_img https://teacher.smartermaths.com.au/wp-content/uploads/2018/05/NAPX-H4-NC12.svg 250 indent3 vpad <br>$ABCD$ is a trapezium. Which of these must be a property of $ABCD$? </div>

    [{"vars":null}]

  472. <div class="sm_mode"> {{{question}}} </div>

    [{"vars":[{"varval":"sm_img https://teacher.smartermaths.com.au/wp-content/uploads/2018/04/NAPX-I4-CA16.svg 380 indent2 vpad\n\nWhat is the size of angle $\\large x$$\\degree$?"},{"varval":"sm_nogap Base angles of the isosceles triangle both = 32$\\degree$\n\n<div class=\"aligned\">\n\n|||\n|-|-|\n|$\\therefore \\large x$$\\degree$|= $180 - 32$|\n||= {{{correctAnswer0}}}{{{suffix0}}}|\n\n</div>"}]},{"vars":[{"varval":"sm_img https://teacher.smartermaths.com.au/wp-content/uploads/2022/12/Geom_NAPX-I4-CA16-SA_v5a.svg 340 indent2 vpad\n\nWhat is the size of angle $\\large x$$\\degree$?"},{"varval":"sm_nogap Base angles of the isosceles triangle both = 45$\\degree$\n\n<div class=\"aligned\">\n\n|||\n|-|-|\n|$\\therefore \\large x$$\\degree$|= $180 - 45$|\n||= {{{correctAnswer0}}}{{{suffix0}}}|\n\n</div>"}]},{"vars":[{"varval":"sm_img https://teacher.smartermaths.com.au/wp-content/uploads/2022/12/Geom_NAPX-I4-CA16-SA_v1.svg 300 indent2 vpad\n\nWhat is the size of angle $\\large x \\degree$?"},{"varval":"sm_nogap Base angles of the isosceles triangle both = 63$\\degree$\n\n<div class=\"aligned\">\n\n|||\n|-|-|\n|$\\therefore \\large x$$\\degree$|= $180 - 63$|\n||= {{{correctAnswer0}}}{{{suffix0}}}|\n\n</div>"}]},{"vars":[{"varval":"sm_img https://teacher.smartermaths.com.au/wp-content/uploads/2022/12/Geom_NAPX-I4-CA16-SA_v2.svg 400 indent2 vpad\n\nWhat is the size of angle $\\large x$$\\degree$?"},{"varval":"sm_nogap Base angles of the isosceles triangle both = 36$\\degree$\n\n<div class=\"aligned\">\n\n|||\n|-|-|\n|$\\therefore \\large x$$\\degree$|= $180 - 36$|\n||= {{{correctAnswer0}}}{{{suffix0}}}|\n\n</div>"}]},{"vars":[{"varval":"sm_img https://teacher.smartermaths.com.au/wp-content/uploads/2022/12/Geom_NAPX-I4-CA16-SA_v6.svg 80 indent2 vpad\n\nWhat is the size of angle $\\large x$$\\degree$?"},{"varval":"sm_nogap Base angles of the isosceles triangle both = 31$\\degree$\n\n<div class=\"aligned\">\n\n|||\n|-|-|\n|$\\therefore \\large x$$\\degree$|= $180 - 31$|\n||= {{{correctAnswer0}}}{{{suffix0}}}|\n\n</div>"}]},{"vars":[{"varval":"sm_img https://teacher.smartermaths.com.au/wp-content/uploads/2022/12/Geom_NAPX-I4-CA16-SA_v3.svg 180 indent2 vpad\n\nWhat is the size of angle $\\large x$$\\degree$?"},{"varval":"sm_nogap Base angles of the isosceles triangle both = 69$\\degree$\n\n<div class=\"aligned\">\n\n|||\n|-|-|\n|$\\therefore \\large x$$\\degree$|= $180 - 69$|\n||= {{{correctAnswer0}}}{{{suffix0}}}|\n\n</div>"}]}]

  473. <div class="sm_mode"> {{{question}}} </div>

    [{"vars":[{"varval":"Triangle $PQR$ is an isosceles triangle.\n\nsm_img https://teacher.smartermaths.com.au/wp-content/uploads/2018/04/NAPX-H4-CA121.svg 200 indent vpad\n\nWhat is the size of the angle <span class=\"sm_katex-font\">$\\angle$</span>$PQR$?"},{"varval":"Since 180$\\degree$ in $\\Delta$,\n\n<div class=\"aligned\">\n\n|||\n|-|-|\n|<span class=\"sm_katex-font\">$\\angle$</span>$PQR$|= $180 - (74 + 74)$|\n||= {{{correctAnswer}}}|\n\n</div>"}]},{"vars":[{"varval":"Triangle $PQR$ is an isosceles triangle.\n\nsm_img https://teacher.smartermaths.com.au/wp-content/uploads/2023/01/Geom_NAPX-H4-CA12_v4.svg 120 indent2 vpad\n\nWhat is the size of the angle <span class=\"sm_katex-font\">$\\angle$</span>$PQR$?"},{"varval":"Since 180$\\degree$ in a $\\Delta$,\n\n<div class=\"aligned\">\n\n|||\n|-|-|\n|<span class=\"sm_katex-font\">$\\angle$</span>$PQR$|= $180 - (2 \\times39)$|\n||= {{{correctAnswer}}}|\n\n</div>"}]},{"vars":[{"varval":"Triangle $PQR$ is an isosceles triangle.\n\nsm_img https://teacher.smartermaths.com.au/wp-content/uploads/2023/01/Geom_NAPX-H4-CA12_v5.svg 180 indent vpad\n\nWhat is the size of the angle <span class=\"sm_katex-font\">$\\angle$</span>$QPR$?"},{"varval":"Since 180$\\degree$ in a $\\Delta$,\n\n<div class=\"aligned\">\n\n|||\n|-|-|\n|<span class=\"sm_katex-font\">$\\angle$</span>$QPR$|= $180 - (2 \\times73)$|\n||= {{{correctAnswer}}}|\n\n</div>"}]},{"vars":[{"varval":"Triangle $PQR$ is an isosceles triangle.\n\nsm_img https://teacher.smartermaths.com.au/wp-content/uploads/2023/01/Geom_NAPX-H4-CA12_v3.svg 200 indent vpad\n\nWhat is the size of the angle <span class=\"sm_katex-font\">$\\angle$</span>$PRQ$?"},{"varval":"Since 180$\\degree$ in a $\\Delta$,\n\n<div class=\"aligned\">\n\n|||\n|-|-|\n|<span class=\"sm_katex-font\">$\\angle$</span>$PRQ$|= $180 - (2\\times 81)$|\n||= {{{correctAnswer}}}|\n\n</div>"}]},{"vars":[{"varval":"Triangle $PQR$ is an isosceles triangle.\n\nsm_img https://teacher.smartermaths.com.au/wp-content/uploads/2023/01/Geom_NAPX-H4-CA12_v2.svg 220 indent vpad\n\nWhat is the size of the angle <span class=\"sm_katex-font\">$\\angle$</span>$QPR$?"},{"varval":"Since 180$\\degree$ in a $\\Delta$,\n\n<div class=\"aligned\">\n\n|||\n|-|-|\n|<span class=\"sm_katex-font\">$\\angle$</span>$QPR$|= $180 - (2\\times 61)$|\n||= {{{correctAnswer}}}|\n\n</div>"}]},{"vars":[{"varval":"Triangle $PQR$ is an isosceles triangle.\n\nsm_img https://teacher.smartermaths.com.au/wp-content/uploads/2023/01/Geom_NAPX-H4-CA12_v1.svg 280 indent vpad\n\nWhat is the size of the angle <span class=\"sm_katex-font\">$\\angle$</span>$PRQ$?"},{"varval":"Since 180$\\degree$ in $\\Delta$,\n\n<div class=\"aligned\">\n\n|||\n|-|-|\n|<span class=\"sm_katex-font\">$\\angle$</span>$PRQ$|= $180 - (2\\times 76)$|\n||= {{{correctAnswer}}}|\n\n</div>"}]}]

  474. <div class="sm_mode"> {{{question}}} </div>

    [{"vars":[{"varval":"Rob creates a design that is made up of 3 rectangles and 2 straight lines, as shown below.\n\n<br>\n\nsm_img https://teacher.smartermaths.com.au/wp-content/uploads/2018/06/NAPX-I3-CA26-SA.svg 280 indent3 vpad\n\n<br>What is the size of angle $\\large x \\degree$?\n"},{"varval":"sm_img https://teacher.smartermaths.com.au/wp-content/uploads/2018/06/NAPX-I3-CA26-SA-Answer.svg 280 indent3 vpad\n\n<br>Base angles of isosceles triangle = 30$\\degree$\n\n\r\nSince 180$\\degree$ in a straight line:\n\n<div class=\"aligned\">\n\n|||\n|-|-|\n|$\\large a$$\\degree$|= $180 - (90 + 30)$|\n||= 60$\\degree$|\n\n</div>\n\n<div class=\"aligned\">\n\n|||\n|-|-|\n|$\\therefore \\large x$|= $180 - 60$|\n||= {{{correctAnswer0}}}{{{suffix0}}}|\n\n</div>\n\n"}]},{"vars":[{"varval":"Chloe creates a design that is made up of 3 rectangles and 2 straight lines, as shown below.\n\n<br>\n\nsm_img https://teacher.smartermaths.com.au/wp-content/uploads/2022/10/Geom_NAPX-I3-CA26-SA_v1.svg 240 indent3 vpad\n\n<br>What is the size of angle $\\large x \\degree$?\n"},{"varval":"sm_img https://teacher.smartermaths.com.au/wp-content/uploads/2022/10/Geom_NAPX-I3-CA26-SA_v1_ws.svg 260 indent3 vpad\n\n<br>Base angles of isosceles triangle = 55$\\degree$\n\n\r\nSince 180$\\degree$ in a straight line:\n\n<div class=\"aligned\">\n\n|||\n|-|-|\n|$\\large a$$\\degree$|= $180 - (90 + 55)$|\n||= 35$\\degree$|\n\n</div>\n\n<div class=\"aligned\">\n\n|||\n|-|-|\n|$\\therefore \\large x$|= $180 - 35$|\n||= {{{correctAnswer0}}}{{{suffix0}}}|\n\n</div>\n\n"}]},{"vars":[{"varval":"Dudley creates a design that is made up of 3 rectangles and 2 straight lines, as shown below.\n\n<br>\n\nsm_img https://teacher.smartermaths.com.au/wp-content/uploads/2022/10/Geom_NAPX-I3-CA26-SA_v2a.svg 220 indent3 vpad\n\n<br>What is the size of angle $\\large x \\degree$?\n"},{"varval":"sm_img https://teacher.smartermaths.com.au/wp-content/uploads/2022/10/Geom_NAPX-I3-CA26-SA_v2a_ws.svg 220 indent3 vpad\n\n<br>Base angles of isosceles triangle = 45$\\degree$\n\n\r\nSince 180$\\degree$ in a straight line:\n\n<div class=\"aligned\">\n\n|||\n|-|-|\n|$\\large a$$\\degree$|= $180 - (90 + 45)$|\n||= 45$\\degree$|\n\n</div>\n\n<div class=\"aligned\">\n\n|||\n|-|-|\n|$\\therefore \\large x$$\\degree$|= $180 - 45$|\n||= {{{correctAnswer0}}}{{{suffix0}}}|\n\n</div>\n\n"}]},{"vars":[{"varval":"Lidija creates a design that is made up of 3 rectangles and 2 straight lines, as shown below.\n\n<br>\n\nsm_img https://teacher.smartermaths.com.au/wp-content/uploads/2022/10/Geom_NAPX-I3-CA26-SA_v3.svg 260 indent3 vpad\n\n<br>What is the size of angle $\\large x \\degree$?\n"},{"varval":"sm_img https://teacher.smartermaths.com.au/wp-content/uploads/2022/10/Geom_NAPX-I3-CA26-SA_v3_ws.svg 260 indent3 vpad\n\n<br>Base angles of isosceles triangle = 51$\\degree$\n\n\r\nSince 180$\\degree$ in a straight line:\n\n<div class=\"aligned\">\n\n|||\n|-|-|\n|$\\large a$$\\degree$|= $180 - (90 + 51)$|\n||= 39$\\degree$|\n\n</div>\n\n<div class=\"aligned\">\n\n|||\n|-|-|\n|$\\therefore \\large x$$\\degree$|= $180 - 39$|\n||= {{{correctAnswer0}}}{{{suffix0}}}|\n\n</div>\n\n"}]},{"vars":[{"varval":"Bernie creates a design that is made up of 3 rectangles and 2 straight lines, as shown below.\n\n<br>\n\nsm_img https://teacher.smartermaths.com.au/wp-content/uploads/2022/10/Geom_NAPX-I3-CA26-SA_v4.svg 310 indent3 vpad\n\n<br>What is the size of angle $\\large x \\degree$?\n"},{"varval":"sm_img https://teacher.smartermaths.com.au/wp-content/uploads/2022/10/Geom_NAPX-I3-CA26-SA_v4_ws.svg 310 indent3 vpad\n\n<br>Base angles of isosceles triangle = 27$\\degree$\n\n\r\nSince 180$\\degree$ in a straight line:\n\n<div class=\"aligned\">\n\n|||\n|-|-|\n|$\\large a$$\\degree$|= $180 - (90 + 27)$|\n||= 63$\\degree$|\n\n</div>\n\n<div class=\"aligned\">\n\n|||\n|-|-|\n|$\\therefore \\large x$$\\degree$|= $180 - 63$|\n||= {{{correctAnswer0}}}{{{suffix0}}}|\n\n</div>\n\n"}]}]

  475. <div class="sm_mode"> {{{question}}} </div>

    [{"vars":[{"varval":"Which of these is an octagon?"},{"varval":"{{{correctAnswer}}}\n\nAn octagon has 8 sides"}]},{"vars":[{"varval":"Which of these is a pentagon?"},{"varval":"{{{correctAnswer}}}\n\nA pentagon has 5 sides"}]},{"vars":[{"varval":"Which of these is a regular hexagon?"},{"varval":"{{{correctAnswer}}}\n\nA regular hexagon has 6 equal sides"}]},{"vars":[{"varval":"Which of these is a decagon?"},{"varval":"{{{correctAnswer}}}\n\nA decagon has 10 sides"}]},{"vars":[{"varval":"Which of these is a trapezium?"},{"varval":"{{{correctAnswer}}}\n\nA trapezium is a quadrilateral with one pair of parallel sides"}]},{"vars":[{"varval":"Which of these is a rhombus?"},{"varval":"{{{correctAnswer}}}\n\nA rhombus is a quadrilateral with four equal sides"}]}]

  476. <div class="sm_mode"> Floyd draws a sketch of his back paddock which is in the shape of a quadrilateral. <br> sm_img https://teacher.smartermaths.com.au/wp-content/uploads/2021/03/NAPX9-TLE-28-v3.svg 260 indent3 vpad <br>What is the size of angle $\large x\degree$? </div>

    [{"vars":null}]

  477. <div class="sm_mode"> A ramp rises 1 metre for every 12 metres of its base length. <br> sm_img https://teacher.smartermaths.com.au/wp-content/uploads/2020/09/naplan-2014-12mc-300x132-1.svg 280 indent3 vpad <br>If the ramp rises 0.8 metres, what is its base length? </div>

    [{"vars":null}]

  478. <div class="sm_mode"> Sarah plotted the points $A − F$ on grid paper, as shown below. sm_img https://teacher.smartermaths.com.au/wp-content/uploads/2021/03/NAPX-TL-B-18-2.svg 420 indent3 vpad She then joined some of the points together with lines. Which of these pairs of lines are parallel? </div>

    [{"vars":null}]

  479. <div class="sm_mode"> Magnolia and Talia hold retreats on their respective properties. The size of each property is shown in the table below: <br> <div class="sm-table row1-color8"> >>| Property Owner | Property Size| |:-|:-:| | Magnolia | 2 square kilometres| | Talia | 40 000 square metres| </div> <br>What is the ratio of the land area of Magnolia's property to Talia's property. </div>

    [{"vars":null}]

  480. <div class="sm_mode"> Two circles have the same centre, $O$, as shown in the diagram below. sm_img https://teacher.smartermaths.com.au/wp-content/uploads/2018/06/NAPX-E4-NC32-SA.svg 240 indent vpad The radius of the small circle is $\dfrac{3}{4}$ the radius of the large circle. Arc $CD$ is 24 mm and the angle between the lines $AC$ and BD is 30$\degree$. What is the length of the arc $RS$ in millimetres? </div>

    [{"vars":null}]

  481. <div class="sm_mode"> Zappa is making a scale model of his music studio. * The height of the studio in the scale model is 5 centimetres. * The height of Zappa's actual studio is 2 metres. What does 1 centimetre in Zappa's scale model represent in his real studio? </div>

    [{"vars":null}]

  482. <div class="sm_mode"> Chisholm has a photo which is 15 cm high and 20 cm wide. <br> sm_img https://teacher.smartermaths.com.au/wp-content/uploads/2020/09/NAPX-L4-13-ver1.svg 300 indent3 vpad <br>He wants to enlarge it to make a photo with a height of 30 cm. What will be the width of the poster? </div>

    [{"vars":[]}]

  483. <div class="sm_mode"> Willow placed the foot of her ladder 2.5 metres from the base of the wall. The ratio of the vertical height of the ladder to its distance from the wall on the ground is &nbsp;$6:1$. <br> sm_img https://teacher.smartermaths.com.au/wp-content/uploads/2020/09/NAPX-I4-NC07new.svg 330 indent3 vpad <br>What is the vertical height of the ladder? </div>

    [{"vars":null}]

  484. <div class="sm_mode"> Bruce has a photo which is 15 cm high and 10 cm wide. <br> sm_img https://teacher.smartermaths.com.au/wp-content/uploads/2020/09/NAPX-L4-13-ver2r.svg 180 indent3 vpad <br>Bruce wants to enlarge it to make a poster with a width of 30 cm. What will be the height of the poster? </div>

    [{"vars":null}]

  485. <div class="sm_mode"> 5 towns are drawn in the diagram below and labelled T1 to T5. The range of travel times between towns is shown on the diagram. <br> sm_img https://teacher.smartermaths.com.au/wp-content/uploads/2020/09/NAPX-F4-CA30-SA.svg 350 indent3 vpad <br>What is the **shortest possible** travel time from T1 to T5? </div>

    [{"vars":null}]

  486. <div class="sm_mode"> Peter takes a photo of a skateboard ramp from the east. <br> sm_img https://teacher.smartermaths.com.au/wp-content/uploads/2020/09/NAPX-F4-NC26.svg 200 indent3 vpad <br>Which view could be from the south? </div>

    [{"vars":null}]

  487. <div class="sm_mode"> Aaron’s house is 50 metres directly west of Ciana's house. Barry’s house is 50 metres due south of Aaron's house. Ciana is at her house facing directly east and turns anticlockwise until she is facing in the exact direction of Barry’s house. How many degrees does Ciana turn through? </div>

    [{"vars":null}]

  488. <div class="sm_mode"> Wyatt enters a park and wants to get to the pond. <br> sm_img https://teacher.smartermaths.com.au/wp-content/uploads/2020/09/NAPX-G4-NC21.svg 620 indent1 vpad <br>Which route is the shortest distance Wyatt can take? </div>

    [{"vars":null}]

  489. <div class="sm_mode"> Xavier draws a scale map of Australia on a coordinate grid. <br> sm_img https://teacher.smartermaths.com.au/wp-content/uploads/2020/09/NAPX-I4-NC15.svg 440 indent3 vpad <br>He knows that the actual distance between Arno Bay and Sydney is 1782 kilometres. Which scale value should Xavier use for his map? </div>

    [{"vars":null}]

  490. <div class="sm_mode"> Grant is a town planner and needs to know the angles between streets in the diagram below. <br> sm_img https://teacher.smartermaths.com.au/wp-content/uploads/2020/09/NAPX-L4-15-ver1.svg 300 indent vpad <br>Grant knows that Dooley Street and Fittler Street are parallel. What is the size of the shaded angle on the map? </div>

    [{"vars":null}]

  491. <div class="sm_mode"> The distance between Sydney and Dubbo on the map below is 7 cm. <br> sm_img https://teacher.smartermaths.com.au/wp-content/uploads/2020/09/NAPX-F4-CA02.svg 350 indent vpad <br>The scale of the map is 1 cm = 50 km. What is the actual distance between Sydney and Dubbo? </div>

    [{"vars":null}]

  492. <div class="sm_mode"> {{{question}}} </div>

    [{"vars":[{"varval":"This is a street map.\n\n<br>\n\nsm_img https://teacher.smartermaths.com.au/wp-content/uploads/2020/09/NAPX-H3-CA01_1.svg 500 indent3 vpad\n\n<br>About how far is it from Ali's house to Zilda's house?"},{"varval":"The distance is approximately 3.5 times the width of the scale interval.\n\n<div class=\"aligned\">\n\n|||\n|-|-|\n|$\\therefore$ Distance|≈ 3.5 × 60|\n||≈ {{{correctAnswer}}}|\n\n</div>"}]},{"vars":[{"varval":"This is a street map.\n\n<br>\n\nsm_img https://teacher.smartermaths.com.au/wp-content/uploads/2017/01/NAP-H3-011.png 450 indent3 vpad\n\n<br>\n\nAbout how far is it from Matt's house to Ali's house?"},{"varval":"The distance between Matt and Ali's house is about 2.5 times the width of the scale interval of 40 metres.\n\n<div class=\"aligned\">\n\n| | |\n| --- | ---------------------- |\n| $\\therefore$ Distance | $\\approx$ 2.5 $\\times$ 40 |\n| | $\\approx$ {{{correctAnswer}}} |\n\n</div>"}]}]

  493. <div class="sm_mode"> This spreadsheet shows the names of students in three classes. <br> sm_img https://teacher.smartermaths.com.au/wp-content/uploads/2020/09/NAPX-F3-NC02.svg 350 indent2 vpad <br>Which student's name is in cell B3? </div>

    [{"vars":null}]

  494. <div class="sm_mode"> Darrell put 4 points on a grid and labelled them $V$ to $Y$, as shown on the diagram below. <br> sm_img https://teacher.smartermaths.com.au/wp-content/uploads/2018/06/NAPX-E4-CA28.svg 290 indent2 vpad <br>Point $V$ is 56 millimetres from point $X$. Darrell adds a fifth point, $Z$ so that the arrangement of points has one line of symmetry. How far is point $Z$ from point $Y$? </div>

    [{"vars":null}]

  495. <div class="sm_mode"> {{{question}}} </div>

    [{"vars":[{"varval":"The object below is a prism.\n\n<br>\n\nsm_img https://teacher.smartermaths.com.au/wp-content/uploads/2018/07/NAPX-F4-NC22.svg 170 indent3 vpad\n\n<br>How many **edges** does the object have?\n"},{"varval":"Edges on front face = 12\n \r\nEdges on back face = 12\n\n\r\nEdges on side = 12\n\n\r\n$\\therefore$ {{{correctAnswer0}}} edges\n"}]},{"vars":[{"varval":"The object below is a prism.\n\n<br>\n\nsm_img https://teacher.smartermaths.com.au/wp-content/uploads/2022/09/Geom_NAPX-F4-NC22-SA_v1.svg 170 indent3 vpad\n\n<br>How many **edges** does the object have?\n"},{"varval":"Edges on front face = 12\n \r\nEdges on back face = 12\n\n\r\nEdges on side = 12\n\n\r\n$\\therefore$ {{{correctAnswer0}}} edges\n"}]},{"vars":[{"varval":"The object below is a prism.\n\n<br>\n\nsm_img https://teacher.smartermaths.com.au/wp-content/uploads/2022/09/Geom_NAPX-F4-NC22-SA_v2.svg 170 indent3 vpad\n\n<br>How many **edges** does the object have?\n"},{"varval":"Edges on top face = 12\n \r\nEdges on bottom face = 12\n\n\r\nEdges on side = 12\n\n\r\n$\\therefore$ {{{correctAnswer0}}} edges\n"}]},{"vars":[{"varval":"The object below is a prism.\n\n<br>\n\nsm_img https://teacher.smartermaths.com.au/wp-content/uploads/2022/09/Geom_NAPX-F4-NC22-SA_v3.svg 190 indent3 vpad\n\n<br>How many **edges** does the object have?"},{"varval":"Edges on top face = 20\n \r\nEdges on bottom face = 20\n\n\r\nEdges on side = 20\n\n\r\n$\\therefore$ {{{correctAnswer0}}} edges\n"}]},{"vars":[{"varval":"The object below is a prism.\n\n<br>\n\nsm_img https://teacher.smartermaths.com.au/wp-content/uploads/2022/09/Geom_NAPX-F4-NC22-SA_v4.svg 270 indent3 vpad\n\n<br>How many **edges** does the object have?"},{"varval":"Edges on top face = 24\n \r\nEdges on bottom face = 24\n\n\r\nEdges on side = 24\n\n\r\n$\\therefore$ {{{correctAnswer0}}} edges\n"}]},{"vars":[{"varval":"The object below is a prism.\n\n<br>\n\nsm_img https://teacher.smartermaths.com.au/wp-content/uploads/2022/09/Geom_NAPX-F4-NC22-SA_v5.svg 350 indent3 vpad\n\n<br>How many **edges** does the object have?"},{"varval":"Edges on top face = 28\n \r\nEdges on bottom face = 28\n\n\r\nEdges on side = 28\n\n\r\n$\\therefore$ {{{correctAnswer0}}} edges\n"}]}]

  496. <div class="sm_mode"> {{{question}}} </div>

    [{"vars":[{"varval":"A rectangular piece of wood is cut out of the prism below.\n\n<br>\n\nsm_img https://teacher.smartermaths.com.au/wp-content/uploads/2018/08/NAPX-I3-NC32-SA.svg 400 indent3 vpad\n\n<br>The rectangular piece creates an opening at the front and back of the prism.\n\n\r\n\r\nHow many edges does the original piece of wood have after the rectangular prism is removed?\n"},{"varval":"Edges on front face = 16\n\n\r\nEdges on back face = 16\n\n\r\nEdges on side (including the hole)\r = 16\n\n<div class=\"aligned\">\n\n|||\n|-|-|\n|$\\therefore$ Total edges| = 16 + 16 + 16|\n||= {{{correctAnswer0}}}|\n\n</div>"}]},{"vars":[{"varval":"A rectangular piece of wood is cut out of the prism below.\n\n<br>\n\nsm_img https://teacher.smartermaths.com.au/wp-content/uploads/2022/09/Geom_NAPX-I3-NC32-SA_v1.svg 360 indent3 vpad\n\n<br>The rectangular piece creates an opening at the front and back of the prism.\n\n\r\n\r\nHow many edges does the original piece of wood have after the rectangular prism is removed?\n"},{"varval":"Edges on front face = 14\n\n\r\nEdges on back face = 14\n\n\r\nEdges on side (including the hole)\r = 14\n\n<div class=\"aligned\">\n\n|||\n|-|-|\n|$\\therefore$ Total edges| = 14 + 14 + 14|\n||= {{{correctAnswer0}}}|\n\n</div>"}]},{"vars":[{"varval":"A rectangular piece of wood is cut out of the prism below.\n\n<br>\n\nsm_img https://teacher.smartermaths.com.au/wp-content/uploads/2022/09/Geom_NAPX-I3-NC32-SA_v2.svg 420 indent3 vpad\n\n<br>The rectangular piece creates an opening at the front and back of the prism.\n\n\r\n\r\nHow many edges does the original piece of wood have after the rectangular prism is removed?"},{"varval":"Edges on front face = 24\n\n\r\nEdges on back face = 24\n\n\r\nEdges on side (including the hole)\r = 24\n\n<div class=\"aligned\">\n\n|||\n|-|-|\n|$\\therefore$ Total edges| = 24 + 24 + 24|\n||= {{{correctAnswer0}}}|\n\n</div>"}]},{"vars":[{"varval":"A rectangular piece of wood is cut out of the prism below.\n\n<br>\n\nsm_img https://teacher.smartermaths.com.au/wp-content/uploads/2022/09/Geom_NAPX-I3-NC32-SA_v3.svg 300 indent3 vpad\n\n<br>The rectangular piece creates an opening at the front and back of the prism.\n\n\r\n\r\nHow many edges does the original piece of wood have after the rectangular prism is removed?"},{"varval":"Edges on front face = 20\n\n\r\nEdges on back face = 20\n\n\r\nEdges on side (including the hole)\r = 20\n\n<div class=\"aligned\">\n\n|||\n|-|-|\n|$\\therefore$ Total edges| = 20 + 20 + 20|\n||= {{{correctAnswer0}}}|\n\n</div>"}]},{"vars":[{"varval":"A rectangular piece of wood is cut out of the prism below.\n\n<br>\n\nsm_img https://teacher.smartermaths.com.au/wp-content/uploads/2022/09/Geom_NAPX-I3-NC32-SA_v4.svg 300 indent3 vpad\n\n<br>The rectangular piece creates an opening at the front and back of the prism.\n\n\r\n\r\nHow many edges does the original piece of wood have after the rectangular prism is removed?"},{"varval":"Edges on top face = 22\n\n\r\nEdges on bottom face = 22\n\n\r\nEdges on side (including the hole)\r = 22\n\n<div class=\"aligned\">\n\n|||\n|-|-|\n|$\\therefore$ Total edges| = 22 + 22 + 22|\n||= {{{correctAnswer0}}}|\n\n</div>"}]},{"vars":[{"varval":"A rectangular piece of wood is cut out of the prism below.\n\n<br>\n\nsm_img https://teacher.smartermaths.com.au/wp-content/uploads/2022/09/Geom_NAPX-I3-NC32-SA_v5.svg 350 indent3 vpad\n\n<br>The rectangular piece creates an opening at the front and back of the prism.\n\n\r\n\r\nHow many edges does the original piece of wood have after the rectangular prism is removed?"},{"varval":"Edges on front face = 16\n\n\r\nEdges on back face = 16\n\n\r\nEdges on side (including the hole)\r = 16\n\n<div class=\"aligned\">\n\n|||\n|-|-|\n|$\\therefore$ Total edges| = 16 + 16 + 16|\n||= {{{correctAnswer0}}}|\n\n</div>"}]}]

  497. <div class="sm_mode"> {{{question}}} </div>

    [{"vars":[{"varval":"This shape is reflected across the line and then rotated 90° anti-clockwise.\n\n<br>\n\nsm_img https://teacher.smartermaths.com.au/wp-content/uploads/2018/04/NAPX-J4-CA27.svg 150 indent3 vpad\n\n<br>Which image shows the appearance of the shape after these transformations?\n"},{"varval":"{{{correctAnswer}}}"}]},{"vars":[{"varval":"This shape is rotated 90° anti-clockwise and then reflected in the line.\n\n<br>\n\nsm_img https://teacher.smartermaths.com.au/wp-content/uploads/2018/04/NAPX-J4-CA27.svg 150 indent3 vpad\n\n<br>Which image shows the appearance of the shape after these transformations?\n"},{"varval":"{{{correctAnswer}}}"}]},{"vars":[{"varval":"This shape is reflected in the line and then rotated 90° clockwise.\n\n<br>\n\nsm_img https://teacher.smartermaths.com.au/wp-content/uploads/2022/09/Geom_NAPX-J4-CA27_NAPX-J3-CA29v3.svg 140 indent3 vpad\n\n<br>Which image shows the appearance of the shape after these transformations?"},{"varval":"{{{correctAnswer}}}"}]},{"vars":[{"varval":"This shape is reflected in the line and then rotated 90° anti-clockwise.\n\n<br>\n\nsm_img https://teacher.smartermaths.com.au/wp-content/uploads/2022/09/Geom_NAPX-J4-CA27_NAPX-J3-CA29v3.svg 140 indent3 vpad\n\n<br>Which image shows the appearance of the shape after these transformations?"},{"varval":"{{{correctAnswer}}}"}]},{"vars":[{"varval":"This shape is rotated 90° clockwise and then reflected in the line.\n\n<br>\n\nsm_img https://teacher.smartermaths.com.au/wp-content/uploads/2022/09/Geom_NAPX-J4-CA27_NAPX-J3-CA29v3.svg 140 indent3 vpad\n\n<br>Which image shows the appearance of the shape after these transformations?"},{"varval":"{{{correctAnswer}}}"}]},{"vars":[{"varval":"This shape is reflected in the line and then rotated 90° anticlockwise.\n\n<br>\n\nsm_img https://teacher.smartermaths.com.au/wp-content/uploads/2022/09/Geom_NAPX-J4-CA27_NAPX-J3-CA29v5.svg 140 indent3 vpad\n\n<br>Which image shows the appearance of the shape after these transformations?"},{"varval":"{{{correctAnswer}}}"}]}]

  498. <div class="sm_mode"> {{{question}}} </div>

    [{"vars":[{"varval":"A H-shaped prism is pictured below.\n\n<br>\n\nsm_img https://teacher.smartermaths.com.au/wp-content/uploads/2018/07/NAPX-E4-NC21.svg 150 indent3 vpad\n\n<br>How many edges does the prism have?\n"},{"varval":"Edges on front face = 12\n\n\r\nEdges on back face = 12\n\n\r\nEdges connecting front and back = 12\n\n<div class=\"aligned\">\n\n|||\n|-|-|\n|$\\therefore$ Total edges|= 12 + 12 + 12|\n||= {{{correctAnswer}}}|\n\n</div>\n"}]},{"vars":[{"varval":"An F-shaped prism is pictured below.\n\n<br>\n\nsm_img https://teacher.smartermaths.com.au/wp-content/uploads/2022/09/Geom_NAPX-E4-NC21_v1.svg 120 indent3 vpad\n\n<br>How many edges does the prism have?\n"},{"varval":"Edges on front face = 10\n\n\r\nEdges on back face = 10\n\n\r\nEdges connecting front and back = 10\n\n<div class=\"aligned\">\n\n|||\n|-|-|\n|$\\therefore$ Total edges|= 10 + 10 + 10|\n||= {{{correctAnswer}}}|\n\n</div>\n"}]},{"vars":[{"varval":"An E-shaped prism is pictured below.\n\n<br>\n\nsm_img https://teacher.smartermaths.com.au/wp-content/uploads/2022/09/Geom_NAPX-E4-NC21_v2.svg 80 indent3 vpad\n\n<br>How many edges does the prism have?"},{"varval":"Edges on front face = 12\n\n\r\nEdges on back face = 12\n\n\r\nEdges connecting front and back = 12\n\n<div class=\"aligned\">\n\n|||\n|-|-|\n|$\\therefore$ Total edges|= 12 + 12 + 12|\n||= {{{correctAnswer}}}|\n\n</div>\n"}]},{"vars":[{"varval":"An S-shaped prism is pictured below.\n\n<br>\n\nsm_img https://teacher.smartermaths.com.au/wp-content/uploads/2022/09/Geom_NAPX-E4-NC21_v3.svg 220 indent3 vpad\n\n<br>How many edges does the prism have?"},{"varval":"Edges on top face = 16\n\n\r\nEdges on bottom face = 16\n\n\r\nEdges connecting top and bottom = 16\n\n<div class=\"aligned\">\n\n|||\n|-|-|\n|$\\therefore$ Total edges|= 16 + 16 + 16|\n||= {{{correctAnswer}}}|\n\n</div>"}]},{"vars":[{"varval":"A P-shaped prism is pictured below.\n\n<br>\n\nsm_img https://teacher.smartermaths.com.au/wp-content/uploads/2022/09/Geom_NAPX-E4-NC21_v4.svg 100 indent3 vpad\n\n<br>How many edges does the prism have?"},{"varval":"Edges on front face = 10\n\n\r\nEdges on back face = 10\n\n\r\nEdges connecting front and back = 10\n\n<div class=\"aligned\">\n\n|||\n|-|-|\n|$\\therefore$ Total edges|= 10 + 10 + 10|\n||= {{{correctAnswer}}}|\n\n</div>"}]},{"vars":[{"varval":"An R-shaped prism is pictured below.\n\n<br>\n\nsm_img https://teacher.smartermaths.com.au/wp-content/uploads/2022/09/Geom_NAPX-E4-NC21_v5.svg 130 indent3 vpad\n\n<br>How many edges does the prism have?"},{"varval":"Edges on front face = 15\n\n\r\nEdges on back face = 15\n\n\r\nEdges connecting front and back = 15\n\n<div class=\"aligned\">\n\n|||\n|-|-|\n|$\\therefore$ Total edges|= 15 + 15 + 15|\n||= {{{correctAnswer}}}|\n\n</div>"}]}]

  499. The answer to this question doesn't seem correct as the lines do NOT look longer than a "4"

    <div class="sm_mode"> Hugh is using a computer graphics program to draw the following shapes. Three of his commands and the resulting shapes are drawn below: <br> sm_img https://teacher.smartermaths.com.au/wp-content/uploads/2021/06/Geometry-NAPX-H4-NC22.svg 480 indent vpad <br>Which shape would be drawn by the command: shape (2, 6, 135)? </div>

    [{"vars":null}]

  500. <div class="sm_mode"> {{{question}}} </div>

    [{"vars":[{"varval":"The pictures show the front, side and top views of an object made of cubes.\n\n<br>\n\nsm_img https://teacher.smartermaths.com.au/wp-content/uploads/2018/08/NAPX-G3-NC31.svg 480 indent vpad\n\n<br>How many cubes are needed to make the object?\n"},{"varval":"sm_img https://teacher.smartermaths.com.au/wp-content/uploads/2018/08/NAPX-G3-NC31rev.svg 180 indent vpad\n\nUsing the top view\n\n\r\n$\\rArr$ 7 cubes on base.\r\n \r\n\r\nUsing the front view\n\n\r\n$\\rArr$ Stacked cubes only along the middle row.\r\n \r\n\r\nUsing the side view\n\n\r\n$\\rArr$ Stacked levels are 3 cubes high.\r\n \r\n\r\n$\\therefore$ Total cubes in the object = {{{correctAnswer0}}}\n"}]},{"vars":[{"varval":"The pictures show the front, side and top views of an object made of cubes.\n\n<br>\n\nsm_img https://teacher.smartermaths.com.au/wp-content/uploads/2022/09/Geom_NAPX-G3-NC31-SA_NAPX-G2-40_v1.svg 540 indent vpad\n\n<br>How many cubes are needed to make the object?\n"},{"varval":"sm_img https://teacher.smartermaths.com.au/wp-content/uploads/2022/09/Geom_NAPX-G3-NC31-SA_NAPX-G2-40_v1ws1.svg 250 indent vpad\n\nUsing the top view\n\n\r\n$\\rArr$ 7 cubes on base.\r\n \r\n\r\nUsing the front view\n\n\r\n$\\rArr$ Stacked cubes only along the second row.\r\n \r\n\r\nUsing the side view\n\n\r\n$\\rArr$ Stacked levels are 1 cube high.\r\n \r\n\r\n$\\therefore$ Total cubes in the object = {{{correctAnswer0}}}\n"}]},{"vars":[{"varval":"The pictures show the front, side and top views of an object made of cubes.\n\n<br>\n\nsm_img https://teacher.smartermaths.com.au/wp-content/uploads/2022/09/Geom_NAPX-G3-NC31-SA_NAPX-G2-40_v2.svg 480 indent vpad\n\n<br>How many cubes are needed to make the object?\n"},{"varval":"sm_img https://teacher.smartermaths.com.au/wp-content/uploads/2022/09/Geom_NAPX-G3-NC31-SA_NAPX-G2-40_v2ws1.svg 200 indent vpad\n\nUsing the front view\n\n\r\n$\\rArr$ 8 cubes on front.\n \r\n\r\nUsing the right side view\n\n\r\n$\\rArr$ Cubes are added on the top and second rows\n \r\n\r\nUsing the top view\n\n\r\n$\\rArr$ Confirms 1 cube only is added to the top and second rows identified in the right side view.\n \r\n\r\n$\\therefore$ Total cubes in the object = {{{correctAnswer0}}}\n"}]},{"vars":[{"varval":"The pictures show the front, side and top views of an object made of cubes.\n\n<br>\n\nsm_img https://teacher.smartermaths.com.au/wp-content/uploads/2022/09/Geom_NAPX-G3-NC31-SA_NAPX-G2-40_v3.svg 480 indent vpad\n\n<br>How many cubes are needed to make the object?\n"},{"varval":"sm_img https://teacher.smartermaths.com.au/wp-content/uploads/2022/09/Geom_NAPX-G3-NC31-SA_NAPX-G2-40_v3ws.svg 300 indent vpad\n\nUsing the top view\n\n\r\n$\\rArr$ 7 cubes on base.\r\n \r\n\r\nUsing the front view\n\n\r\n$\\rArr$ Stacked cubes are two high on the left hand side and three high on right hand side. \n \r\n\r\nUsing the side view\n\n\r\n$\\rArr$ Stacked levels are all on the back row. 1 cube high \ron the left back and 2 cubes high on the right back. \n \r\n\r\n$\\therefore$ Total cubes in the object = {{{correctAnswer0}}}\n"}]},{"vars":[{"varval":"The pictures show the front, side and top views of an object made of cubes.\n\n<br>\n\nsm_img https://teacher.smartermaths.com.au/wp-content/uploads/2022/09/Geom_NAPX-G3-NC31-SA_NAPX-G2-40_v4.svg 520 indent vpad\n\n<br>How many cubes are needed to make the object?"},{"varval":"sm_img https://teacher.smartermaths.com.au/wp-content/uploads/2022/09/Geom_NAPX-G3-NC31-SA_NAPX-G2-40_v4ws.svg 260 indent vpad\n\nUsing the top view\n\n\r\n$\\rArr$ 8 cubes on base.\r\n \r\n\r\nUsing the front view\n\n\r\n$\\rArr$ Stacked cubes only along the middle row\n \r\n\r\nUsing the side view\n\n\r\n$\\rArr$ Stacked levels are 3 cubes high \ron the 2nd row\n \r\n\r\n$\\therefore$ Total cubes in the object = {{{correctAnswer0}}}"}]},{"vars":[{"varval":"The pictures show the front, side and top views of an object made of cubes.\n\n<br>\n\nsm_img https://teacher.smartermaths.com.au/wp-content/uploads/2022/09/Geom_NAPX-G3-NC31-SA_NAPX-G2-40_v5.svg 500 indent vpad\n\n<br>How many cubes are needed to make the object?\n"},{"varval":"sm_img https://teacher.smartermaths.com.au/wp-content/uploads/2022/09/Geom_NAPX-G3-NC31-SA_NAPX-G2-40_v5ws.svg 300 indent vpad\n\nUsing the top view\n\n\r\n$\\rArr$ 7 cubes on base.\r\n \r\n\r\nUsing the front view\n\n\r\n$\\rArr$ Stacked cubes are only along the first row.\n \r\n\r\nUsing the right side view\n\n\r\n$\\rArr$ Stacked levels are 2 cubes high \ron the first row and 3 cubes high on the middle row.\n \r\n\r\n$\\therefore$ Total cubes in the object = {{{correctAnswer0}}}\n"}]}]

  501. <div class="sm_mode"> {{{question}}} </div>

    [{"vars":[{"varval":"Which of the following nets cannot be folded to make a cube?"},{"varval":"{{{correctAnswer}}}\n"}]},{"vars":[{"varval":"Which of the following nets cannot be folded to make a cube?"},{"varval":"{{{correctAnswer}}}\n"}]},{"vars":[{"varval":"Which of the following nets cannot be folded to make a cube?"},{"varval":"{{{correctAnswer}}}\n"}]},{"vars":[{"varval":"Which of the following nets cannot be folded to make a cube?"},{"varval":"{{{correctAnswer}}}\n"}]},{"vars":[{"varval":"Which of the following nets cannot be folded to make a cube?"},{"varval":"{{{correctAnswer}}}\n"}]},{"vars":[{"varval":"Which of the following nets cannot be folded to make a cube?"},{"varval":"{{{correctAnswer}}}\n"}]}]

  502. <div class="sm_mode"> {{{question}}} </div>

    [{"vars":[{"varval":"A sculpture is cut into 3 objects, as shown below.\n\n<br>\n\nsm_img https://teacher.smartermaths.com.au/wp-content/uploads/2018/08/NAPX-G3-NC25-1-1.svg 400 indent2 vpad\n\n<br>What is the total number of faces of the three pieces after the sculpture is cut, including the bases?\n"},{"varval":"<div class=\"aligned\">\n\n|||\n|-|-|\n|Total faces|= 10 + (2 $\\times$ 6)|\n||= {{{correctAnswer0}}}|\n\n</div>"}]},{"vars":[{"varval":"A sculpture is cut into 3 objects, as shown below.\n\n<br>\n\nsm_img https://teacher.smartermaths.com.au/wp-content/uploads/2022/09/Geom_NAPX-G3-NC25-SA_NAPX-G2-37_v1.svg 460 indent2 vpad\n\n<br>What is the total number of faces of the three pieces after the sculpture is cut, including the bases?\n"},{"varval":"<div class=\"aligned\">\n\n|||\n|-|-|\n|Total faces|= 14 + (2 $\\times$ 6)|\n||= {{{correctAnswer0}}}|\n\n</div>"}]},{"vars":[{"varval":"A sculpture is cut into 3 objects, as shown below.\n\n<br>\n\nsm_img https://teacher.smartermaths.com.au/wp-content/uploads/2022/09/Geom_NAPX-G3-NC25-SA_NAPX-G2-37_v2.svg 500 indent2 vpad\n\n<br>What is the total number of faces of the three pieces after the sculpture is cut, including the bases?"},{"varval":"<div class=\"aligned\">\n\n|||\n|-|-|\n|Total faces|= 11 + (2 $\\times$ 8)|\n||= {{{correctAnswer0}}}|\n\n</div>"}]},{"vars":[{"varval":"A sculpture is cut into 3 objects, as shown below.\n\n<br>\n\nsm_img https://teacher.smartermaths.com.au/wp-content/uploads/2022/09/Geom_NAPX-G3-NC25-SA_NAPX-G2-37_v3.svg 440 indent vpad\n\n<br>What is the total number of faces of the three pieces after the sculpture is cut, including the bases?"},{"varval":"<div class=\"aligned\">\n\n|||\n|-|-|\n|Total faces|= 10 + (2 $\\times$ 8)|\n||= {{{correctAnswer0}}}|\n\n</div>"}]},{"vars":[{"varval":"A sculpture is cut into 3 objects, as shown below.\n\n<br>\n\nsm_img https://teacher.smartermaths.com.au/wp-content/uploads/2022/09/Geom_NAPX-G3-NC25-SA_NAPX-G2-37_v4.svg 530 indent vpad\n\n<br>What is the total number of faces of the three pieces after the sculpture is cut, including the bases?"},{"varval":"<div class=\"aligned\">\n\n|||\n|-|-|\n|Total faces|= 18 + (2 $\\times$ 10)|\n||= {{{correctAnswer0}}}|\n\n</div>"}]},{"vars":[{"varval":"A sculpture is cut into 3 objects, as shown below.\n\n<br>\n\nsm_img https://teacher.smartermaths.com.au/wp-content/uploads/2022/09/Geom_NAPX-G3-NC25-SA_NAPX-G2-37_v5.svg 450 indent2 vpad\n\n<br>What is the total number of faces of the three pieces after the sculpture is cut, including the bases?"},{"varval":"<div class=\"aligned\">\n\n|||\n|-|-|\n|Total faces|= 8 + (2 $\\times$ 14)|\n||= {{{correctAnswer0}}}|\n\n</div>"}]}]

  503. <div class="sm_mode"> {{{question}}} </div>

    [{"vars":[{"varval":"Igor drew a shape with more than four lines of symmetry.\n\n\r\n\r\nWhich of these could be Igor's shape?\n"},{"varval":"sm_img https://teacher.smartermaths.com.au/wp-content/uploads/2018/08/NAPX-E3-CA19-Answer.svg 200 indent3 vpad\n\nThe regular hexagon has 6 lines of symmetry.\n"}]},{"vars":[{"varval":"Bethany drew a shape with more than four lines of symmetry.\n\n\r\n\r\nWhich of these could be Bethany's shape?\n"},{"varval":"sm_img https://teacher.smartermaths.com.au/wp-content/uploads/2022/09/Geom_NAPX-E3-CA19_v1ws.svg 200 indent3 vpad\n\nThe regular pentagon has 5 lines of symmetry.\n"}]},{"vars":[{"varval":"Ivy drew a shape with exactly four lines of symmetry.\n\n\r\n\r\nWhich of these could **NOT** be Ivy's shape?\n"},{"varval":"sm_img https://teacher.smartermaths.com.au/wp-content/uploads/2022/09/Geom_NAPX-E3-CA19_v2ws.svg 200 indent3 vpad\n\nThe rhombus has 2 lines of symmetry.\n"}]},{"vars":[{"varval":"Optimus drew a shape with more than seven lines of symmetry.\n\n\r\n\r\nWhich of these could be Optimus's shape?\n"},{"varval":"sm_img https://teacher.smartermaths.com.au/wp-content/uploads/2022/09/Geom_NAPX-E3-CA19_v3ws.svg 200 indent3 vpad\n\nThe regular octagon has 8 lines of symmetry.\n"}]},{"vars":[{"varval":"Bron drew a shape with more than two lines of symmetry.\n\n\r\n\r\nWhich of these could be Bron's shape?\n"},{"varval":"sm_img https://teacher.smartermaths.com.au/wp-content/uploads/2022/09/Geom_NAPX-E3-CA19_v4ws.svg 200 indent3 vpad\n\nThis shape has 4 lines of symmetry.\n"}]},{"vars":[{"varval":"Col drew a shape with more than 6 lines of symmetry.\n\n\r\n\r\nWhich of these could be Col's shape?\n"},{"varval":"sm_img https://teacher.smartermaths.com.au/wp-content/uploads/2022/09/Geom_NAPX-E3-CA19_v5ws.svg 220 indent3 vpad\n\nThe regular heptagon has 7 lines of symmetry.\n"}]}]

  504. Var3 ... question image - points T, U and V are missing Correct file imported now

    <div class="sm_mode"> {{{question}}} </div>

    [{"vars":[{"varval":"Lance drew a trapezium $ABCD$ with 3 equal sides.\n\n\r\n\r\nHe folds it along BD.\n\nsm_img https://teacher.smartermaths.com.au/wp-content/uploads/2018/06/NAPX-G4-NC12_1.svg 280 indent3 vpad\n\nAfter folding, where will point $C$ finish?"},{"varval":"Folding creates a reflection in the line of the fold.\n\nsm_img https://teacher.smartermaths.com.au/wp-content/uploads/2018/06/NAPX-G4-NC12-Answer.svg 280 indent3 vpad\n\n$\\therefore$ Point $C$ finishes at point {{{correctAnswer}}}.\n"}]},{"vars":[{"varval":"Lulu drew a trapezium $ABCD$ with 3 equal sides.\n\n\r\n\r\nShe folds it along BD.\n\nsm_img https://teacher.smartermaths.com.au/wp-content/uploads/2022/09/Geom_NAPX-G4-NC12_v1.svg 350 indent3 vpad\n\nAfter folding, where will point $C$ finish?"},{"varval":"Folding creates a reflection in the line of the fold.\n\nsm_img https://teacher.smartermaths.com.au/wp-content/uploads/2022/09/Geom_NAPX-G4-NC12_v1ws.svg 350 indent3 vpad\n\n$\\therefore$ Point $C$ finishes at point {{{correctAnswer}}}.\n"}]},{"vars":[{"varval":"Lincoln drew a trapezium $ABCD$ with 3 equal sides.\n\n\r\n\r\nHe folds it along AC.\n\nsm_img https://teacher.smartermaths.com.au/wp-content/uploads/2022/09/Geom_NAPX-G4-NC12_v2.svg 350 indent3 vpad\n\nAfter folding, where will point $B$ finish?"},{"varval":"Folding creates a reflection in the line of the fold.\n\nsm_img https://teacher.smartermaths.com.au/wp-content/uploads/2022/09/Geom_NAPX-G4-NC12_v2ws.svg 350 indent3 vpad\n\n$\\therefore$ Point $B$ finishes at point {{{correctAnswer}}}.\n"}]},{"vars":[{"varval":"Lucy drew a trapezium $WXYZ$ with 3 equal sides.\n\n\r\n\r\nShe folds it along WY.\n\nsm_img https://teacher.smartermaths.com.au/wp-content/uploads/2022/09/Geom_NAPX-G4-NC12_v3q.svg 350 indent3 vpad\n\nAfter folding, where will point $Z$ finish?"},{"varval":"Folding creates a reflection in the line of the fold. Extend the fold line so that a perpendicular bisector can be drawn across it.\n\nsm_img https://teacher.smartermaths.com.au/wp-content/uploads/2022/09/Geom_NAPX-G4-NC12_v3ws.svg 350 indent3 vpad\n\n$\\therefore$ Point $Z$ finishes at point {{{correctAnswer}}}."}]}]

  505. <div class="sm_mode"> {{{question}}} </div>

    [{"vars":[{"varval":"Renee is shading squares in a grid. \n\n<br>\n\nsm_img https://teacher.smartermaths.com.au/wp-content/uploads/2022/09/Geom_NAPX-G4-CA10_v1aws.svg 220 indent3 vpad\n\n<br>Renee needs to shade some extra squares so that the dotted line is a line of symmetry.\n\n\r\n\r\nWhat is the smallest number of squares she has to shade?"},{"varval":"{{{correctAnswer}}} squares need to be shaded (see below).\n\nsm_img https://teacher.smartermaths.com.au/wp-content/uploads/2022/09/Geom_NAPX-G4-CA10_v1q.svg 200 indent vpad"}]},{"vars":[{"varval":"Rufus is shading squares in a grid.\n\n<br>\n\nsm_img https://teacher.smartermaths.com.au/wp-content/uploads/2018/07/NAPX-G4-CA10.svg 240 indent3 vpad\n\n<br>Rufus needs to shade some extra squares so that the dotted line is a line of symmetry.\n\n\r\n\r\nWhat is the smallest number of squares he has to shade?"},{"varval":"{{{correctAnswer}}} squares need to be shaded (see below).\n\nsm_img https://teacher.smartermaths.com.au/wp-content/uploads/2018/07/NAPX-G4-CA10-Answer.svg 200 indent vpad"}]},{"vars":[{"varval":"Rocky is shading squares in a grid.\n\n<br>\n\nsm_img https://teacher.smartermaths.com.au/wp-content/uploads/2022/09/Geom_NAPX-G4-CA10_v2q.svg 240 indent3 vpad\n\n<br>Rocky needs to shade some extra squares so that the dotted line is a line of symmetry.\n\n\r\n\r\nWhat is the smallest number of squares he has to shade?"},{"varval":"{{{correctAnswer}}} squares need to be shaded (see below).\n\nsm_img https://teacher.smartermaths.com.au/wp-content/uploads/2022/09/Geom_NAPX-G4-CA10_v2ws.svg 200 indent vpad"}]},{"vars":[{"varval":"Riley is shading squares in a grid.\n\n<br>\n\nsm_img https://teacher.smartermaths.com.au/wp-content/uploads/2022/09/Geom_NAPX-G4-CA10_v3q.svg 240 indent3 vpad\n\n<br>Riley needs to shade some extra squares so that the dotted line is a line of symmetry.\n\n\r\n\r\nWhat is the smallest number of squares she has to shade?"},{"varval":"{{{correctAnswer}}} squares need to be shaded (see below).\n\nsm_img https://teacher.smartermaths.com.au/wp-content/uploads/2022/09/Geom_NAPX-G4-CA10_v3ws.svg 200 indent vpad"}]},{"vars":[{"varval":"Red is shading squares in a grid.\n\n<br>\n\nsm_img https://teacher.smartermaths.com.au/wp-content/uploads/2022/09/Geom_NAPX-G4-CA10_v4q.svg 240 indent3 vpad\n\n<br>Red needs to shade some extra squares so that the dotted line is a line of symmetry.\n\n\r\n\r\nWhat is the smallest number of squares he has to shade?"},{"varval":"{{{correctAnswer}}} squares need to be shaded (see below).\n\nsm_img https://teacher.smartermaths.com.au/wp-content/uploads/2022/09/Geom_NAPX-G4-CA10_v4ws.svg 200 indent vpad"}]},{"vars":[{"varval":"Roxy is shading squares in a grid.\n<br>\n\nsm_img https://teacher.smartermaths.com.au/wp-content/uploads/2022/09/Geom_NAPX-G4-CA10_v5q.svg 240 indent3 vpad\n\n<br>Roxy needs to shade some extra squares so that the dotted line is a line of symmetry.\n\n\r\n\r\nWhat is the smallest number of squares she has to shade?"},{"varval":"{{{correctAnswer}}} squares need to be shaded (see below).\n\nsm_img https://teacher.smartermaths.com.au/wp-content/uploads/2022/09/Geom_NAPX-G4-CA10_v5ws.svg 200 indent vpad"}]}]

  506. <div class="sm_mode"> {{{question}}} </div>

    [{"vars":[{"varval":"Kylie is drawing a 3-dimensional square pyramid on an isometric grid.\n\nsm_img https://teacher.smartermaths.com.au/wp-content/uploads/2021/03/NAPX9-TLE-27-3-1.png 300 indent vpad\n\nOn which point does the 4th vertex of the pyramid's base lie?\n"},{"varval":"{{{correctAnswer}}}\n\nsm_img https://teacher.smartermaths.com.au/wp-content/uploads/2021/03/NAPX9-TLE-27-3-2.png 300 indent vpad"}]},{"vars":[{"varval":"Kurt is drawing a 3-dimensional square pyramid on an isometric grid.\n\nsm_img https://teacher.smartermaths.com.au/wp-content/uploads/2022/09/algebra_NAPX9-TLE-27v3-v1_q.svg 230 indent vpad\n\nOn which point does the 4th vertex of the pyramid's base lie?\n"},{"varval":"{{{correctAnswer}}}\n\nsm_img https://teacher.smartermaths.com.au/wp-content/uploads/2022/09/algebra_NAPX9-TLE-27v3-v1_ws.svg 230 indent vpad"}]},{"vars":[{"varval":"Kate is drawing a 3-dimensional square pyramid on an isometric grid.\n\nsm_img https://teacher.smartermaths.com.au/wp-content/uploads/2022/09/algebra_NAPX9-TLE-27v3-v2q.svg 360 indent vpad\n\nOn which point does the 4th vertex of the pyramid's base lie?\n"},{"varval":"{{{correctAnswer}}}\n\nsm_img https://teacher.smartermaths.com.au/wp-content/uploads/2022/09/algebra_NAPX9-TLE-27v3-v2ws.svg 360 indent vpad"}]},{"vars":[{"varval":"Krinash is drawing a 3-dimensional square pyramid on an isometric grid.\n\nsm_img https://teacher.smartermaths.com.au/wp-content/uploads/2022/09/algebra_NAPX9-TLE-27v3-v3q.svg 450 indent vpad\n\nOn which point does the 4th vertex of the pyramid's base lie?"},{"varval":"{{{correctAnswer}}}\n\nsm_img https://teacher.smartermaths.com.au/wp-content/uploads/2022/09/algebra_NAPX9-TLE-27v3-v3ws.svg 450 indent vpad"}]},{"vars":[{"varval":"Kalinda is drawing a 3-dimensional square pyramid on an isometric grid.\n\nsm_img https://teacher.smartermaths.com.au/wp-content/uploads/2022/09/algebra_NAPX9-TLE-27v3-v4q.svg 550 indent vpad\n\nOn which point does the 4th vertex of the pyramid's base lie?"},{"varval":"{{{correctAnswer}}}\n\nsm_img https://teacher.smartermaths.com.au/wp-content/uploads/2022/09/algebra_NAPX9-TLE-27v3-v4ws.svg 550 indent vpad"}]}]

  507. <div class="sm_mode"> {{{question}}} </div>

    [{"vars":[{"varval":"Wally drew the net of a rectangular prism.\n\nsm_img https://teacher.smartermaths.com.au/wp-content/uploads/2018/04/NAPX-H4-NC06.svg 200 indent vpad\n\nOne of the prism's faces is drawn incorrectly.\n\n\r\n\r\nWhich face did Wally draw incorrectly?"},{"varval":"Side {{{correctAnswer}}} is drawn incorrectly. It should be \rthe same width as side $B$."}]},{"vars":[{"varval":"Bryson drew the net of a rectangular prism.\n\nsm_img https://teacher.smartermaths.com.au/wp-content/uploads/2022/10/Geom_NAPX-H4-NC06_NAPX-H3-NC11_v5.svg 280 indent vpad\n\nOne of the prism's faces is drawn incorrectly.\n\n\r\n\r\nWhich face did Bryson draw incorrectly?"},{"varval":"Side {{{correctAnswer}}} is drawn incorrectly. The height of $C$ should be equal to the width of side $B$."}]},{"vars":[{"varval":"Gretchen drew the net of a triangular prism.\n\nsm_img https://teacher.smartermaths.com.au/wp-content/uploads/2022/10/Geom_NAPX-H4-NC06_NAPX-H3-NC11_v4.svg 300 indent vpad\n\nOne of the prism's faces is drawn incorrectly.\n\n\r\n\r\nWhich face did Gretchen draw incorrectly?"},{"varval":"Side {{{correctAnswer}}} is drawn incorrectly. The height of $B$ should be equal to the slant length of $D$."}]},{"vars":[{"varval":"Wendy drew the net of a rectangular prism.\n\nsm_img https://teacher.smartermaths.com.au/wp-content/uploads/2022/10/Geom_NAPX-H4-NC06_NAPX-H3-NC11_v3a.svg 200 indent vpad\n\nOne of the prism's faces is drawn incorrectly.\n\n\r\n\r\nWhich face did Wendy draw incorrectly?"},{"varval":"Side {{{correctAnswer}}} is drawn incorrectly. The dimensions of side $B$ should be the same as side $D$."}]},{"vars":[{"varval":"Ken drew the net of a rectangular prism.\n\nsm_img https://teacher.smartermaths.com.au/wp-content/uploads/2022/10/Geom_NAPX-H4-NC06_NAPX-H3-NC11_v2.svg 200 indent vpad\n\nOne of the prism's faces is drawn incorrectly.\n\n\r\n\r\nWhich face did draw Ken incorrectly?"},{"varval":"Side {{{correctAnswer}}} is drawn incorrectly. It should have the same dimensions as side $D$."}]},{"vars":[{"varval":"Betty drew the net of a triangular prism.\n\nsm_img https://teacher.smartermaths.com.au/wp-content/uploads/2022/10/Geom_NAPX-H4-NC06_NAPX-H3-NC11_v1.svg 300 indent vpad\n\nOne of the prism's faces is drawn incorrectly.\n\n\r\n\r\nWhich face did Betty draw incorrectly?"},{"varval":"Side {{{correctAnswer}}} is drawn incorrectly. The height of side $B$ should be equal to the slant length of side $A$."}]}]

  508. <div class="sm_mode"> {{{question}}} </div>

    [{"vars":[{"varval":"sm_img https://teacher.smartermaths.com.au/wp-content/uploads/2018/08/NAPX-G3-NC20-1.svg 220 indent3 vpad\n\n<br>A flag is rotated from Position 1 to Position 2.\n\n\r\n\r\nWhich of these could describe the rotation?\n"},{"varval":"Rotation is {{{correctAnswer}}}\n"}]},{"vars":[{"varval":"sm_img https://teacher.smartermaths.com.au/wp-content/uploads/2022/09/Geom_NAPX-G3-NC20_v1a.svg 310 indent3 vpad\n\n<br>A flag is rotated from Position 1 to Position 2.\n\n\r\n\r\nWhich of these could describe the rotation?\n"},{"varval":"Rotation is {{{correctAnswer}}}\n"}]},{"vars":[{"varval":"sm_img https://teacher.smartermaths.com.au/wp-content/uploads/2022/09/Geom_NAPX-G3-NC20_v2.svg 270 indent3 vpad\n\n<br>A flag is rotated from Position 1 to Position 2.\n\n\r\n\r\nWhich of these could describe the rotation?\n"},{"varval":"Rotation is {{{correctAnswer}}}\n"}]},{"vars":[{"varval":"sm_img https://teacher.smartermaths.com.au/wp-content/uploads/2022/09/Geom_NAPX-G3-NC20_v3.svg 340 indent3 vpad\n\n<br>A flag is rotated from Position 1 to Position 2.\n\n\r\n\r\nWhich of these could describe the rotation?\n"},{"varval":"Rotation is {{{correctAnswer}}}\n"}]},{"vars":[{"varval":"sm_img https://teacher.smartermaths.com.au/wp-content/uploads/2022/09/Geom_NAPX-G3-NC20_v4.svg 380 indent vpad\n\n<br>A flag is rotated from Position 1 to Position 2.\n\n\r\n\r\nWhich of these could describe the rotation?\n"},{"varval":"Rotation is {{{correctAnswer}}}\n"}]},{"vars":[{"varval":"sm_img https://teacher.smartermaths.com.au/wp-content/uploads/2022/09/Geom_NAPX-G3-NC20_v5a.svg 290 indent3 vpad\n\n<br>A flag is rotated from Position 1 to Position 2.\n\n\r\n\r\nWhich of these could describe the rotation?\n"},{"varval":"Rotation is {{{correctAnswer}}}\n"}]}]

  509. <div class="sm_mode"> {{{question}}} </div>

    [{"vars":[{"varval":"The rectangle below will be folded along the dashed line, $BD$.\n\n<br>\n\nsm_img https://teacher.smartermaths.com.au/wp-content/uploads/2018/08/NAPX-G3-NC18.svg 200 indent vpad\n\n<br>Where will point $C$ move to?\n"},{"varval":"sm_img https://teacher.smartermaths.com.au/wp-content/uploads/2018/08/NAPX-G3-NC18ans.svg 200 indent vpad\n\n$\\therefore C$ moves to {{{correctAnswer}}}\n"}]},{"vars":[{"varval":"The rectangle below will be folded along the dashed line, $BD$.\n\n<br>\n\nsm_img https://teacher.smartermaths.com.au/wp-content/uploads/2022/09/Geom_NAPX-G3-NC18_v1q.svg 250 indent3 vpad\n\n<br>Where will point $A$ move to?\n"},{"varval":"sm_img https://teacher.smartermaths.com.au/wp-content/uploads/2022/09/Geom_NAPX-G3-NC18_v1aws.svg 250 indent3 vpad\n\n$\\therefore A$ moves to {{{correctAnswer}}}\n"}]},{"vars":[{"varval":"The rectangle below will be folded along the dashed line, $MO$\n\n<br>\n\nsm_img https://teacher.smartermaths.com.au/wp-content/uploads/2022/09/Geom_NAPX-G3-NC18_v2.svg 210 indent3 vpad\n\n<br>Where will point $P$ move to?\n"},{"varval":"sm_img https://teacher.smartermaths.com.au/wp-content/uploads/2022/09/Geom_NAPX-G3-NC18_v2a.svg 210 indent3 vpad\n\n$\\therefore P$ moves to {{{correctAnswer}}}\n"}]}]

  510. <div class="sm_mode"> {{{question}}} </div>

    [{"vars":[{"varval":"Which object is a prism?\n"},{"varval":"{{{correctAnswer}}}\n\nNote that a cylinder is not a prism because all its surfaces are not flat. \n"}]},{"vars":[{"varval":"Which object is a prism?\n"},{"varval":"{{{correctAnswer}}}\n\nNote that a cylinder is not a prism because all its surfaces are not flat. \n"}]},{"vars":[{"varval":"Which object is a pyramid?\n"},{"varval":"{{{correctAnswer}}}\n\nNote that a cone is not a pyramid because all its surfaces are not flat. \n"}]},{"vars":[{"varval":"Which object is a pyramid?\n"},{"varval":"{{{correctAnswer}}}\n\nNote that a cone is not a pyramid because all its surfaces are not flat. \n\n"}]},{"vars":[{"varval":"Which object is a pyramid?\n"},{"varval":"{{{correctAnswer}}}\n\nNote that a cone is not a pyramid because all its surfaces are not flat. \n"}]},{"vars":[{"varval":"Which object is a prism?\n"},{"varval":"{{{correctAnswer}}}\n\nNote that a cylinder is not a prism because all its surfaces are not flat. \n"}]}]

  511. <div class="sm_mode"> {{{question}}} </div>

    [{"vars":[{"varval":"Which two lines below are lines of symmetry?\n\nsm_img https://teacher.smartermaths.com.au/wp-content/uploads/2018/06/NAPX-F4-NC06.svg 240 indent vpad\n"},{"varval":"{{{correctAnswer}}}"}]},{"vars":[{"varval":"Which two lines below are lines of symmetry?\n\nsm_img https://teacher.smartermaths.com.au/wp-content/uploads/2022/10/Geom_NAPX-F4-NC06_v1.svg 200 indent vpad\n"},{"varval":"{{{correctAnswer}}}"}]},{"vars":[{"varval":"Which two lines below are lines of symmetry?\n\nsm_img https://teacher.smartermaths.com.au/wp-content/uploads/2022/10/Geom_NAPX-F4-NC06_v3.svg 220 indent vpad"},{"varval":"{{{correctAnswer}}}"}]},{"vars":[{"varval":"Which two lines below are lines of symmetry?\n\nsm_img https://teacher.smartermaths.com.au/wp-content/uploads/2022/10/Geom_NAPX-F4-NC06_v5.svg 220 indent vpad"},{"varval":"{{{correctAnswer}}}"}]},{"vars":[{"varval":"Which two lines below are lines of symmetry?\n\nsm_img https://teacher.smartermaths.com.au/wp-content/uploads/2022/10/Geom_NAPX-F4-NC06_v4A.svg 220 indent vpad"},{"varval":"{{{correctAnswer}}}"}]},{"vars":[{"varval":"Which two lines below are lines of symmetry?\n\nsm_img https://teacher.smartermaths.com.au/wp-content/uploads/2022/10/Geom_NAPX-F4-NC06_v5-1.svg 220 indent vpad"},{"varval":"{{{correctAnswer}}}"}]}]

  512. <div class="sm_mode"> {{{question}}} </div>

    [{"vars":[{"varval":"Which of these shapes has exactly five faces?"},{"varval":"\n{{{correctAnswer}}} has five faces.\n\nsm_img https://teacher.smartermaths.com.au/wp-content/uploads/2021/03/NAPX9-TLC-27-2-300x282.png 130 indent3 vpad"}]},{"vars":[{"varval":"Which of these shapes has exactly six faces?"},{"varval":"\nPentagonal pyramid $\\rightarrow$ 5 triangular faces and a pentagonal base.\n{{{correctAnswer}}} has six faces.\n\nsm_img https://teacher.smartermaths.com.au/wp-content/uploads/2022/10/Geom_NAPX9-TLC-26v3_v1.svg 130 indent3 vpad"}]},{"vars":[{"varval":"Which of these shapes has exactly seven faces?"},{"varval":"\nHexagonal pyramid $\\rightarrow$ 6 triangular faces and a hexagonal base\n\n{{{correctAnswer}}} has seven faces.\n\nsm_img https://teacher.smartermaths.com.au/wp-content/uploads/2022/10/Geom_NAPX9-TLC-26v3_v2.svg 130 indent3 vpad"}]},{"vars":[{"varval":"Which of these shapes has exactly eight faces?"},{"varval":"\nHeptagonal pyramid $\\rightarrow$ 7 triangular faces and a heptagonal base\n\nA {{{correctAnswer}}} has eight faces.\n\nsm_img https://teacher.smartermaths.com.au/wp-content/uploads/2022/10/Geom_NAPX9-TLC-26v3_v3.svg 130 indent3 vpad"}]},{"vars":[{"varval":"Which of these shapes has exactly nine faces?"},{"varval":"\nOctagonal pyramid $\\rightarrow$ 8 triangular faces and an octagonal base\n\n{{{correctAnswer}}} has nine faces.\n\nsm_img https://teacher.smartermaths.com.au/wp-content/uploads/2022/10/Geom_NAPX9-TLC-26v3_v4.svg 130 indent3 vpad"}]},{"vars":[{"varval":"Which of these shapes has exactly ten faces?"},{"varval":"Octagonal prism $\\rightarrow$ 8 rectangular faces and 2 octagonal faces.\n\n{{{correctAnswer}}} has ten faces.\n\nsm_img https://teacher.smartermaths.com.au/wp-content/uploads/2022/10/Geom_NAPX9-TLC-26v3_v5.svg 250 indent3 vpad"}]}]

  513. <div class="sm_mode"> {{{question}}} </div>

    [{"vars":[{"varval":"Some tiles are missing from this pattern of tiles.\n\n<br>\n\nsm_img https://teacher.smartermaths.com.au/wp-content/uploads/2020/09/NAPX-H4-NC04.svg 230 indent3 vpad\n\n<br>When complete the pattern has two lines of symmetry.\n\n\r\n\r\nWhich of these could be the missing part of the pattern?"},{"varval":"{{{correctAnswer}}}\n"}]},{"vars":[{"varval":"Some tiles are missing from this pattern of tiles.\n\n<br>\n\nsm_img https://teacher.smartermaths.com.au/wp-content/uploads/2022/10/Geom_NAPX-H4-NC04_H3-NC10_v1q1.svg 240 indent3 vpad\n\n<br>When complete the pattern has two lines of symmetry.\n\n\r\n\r\nWhich of these could be the missing part of the pattern?"},{"varval":"{{{correctAnswer}}}\n"}]},{"vars":[{"varval":"Some tiles are missing from this pattern of tiles.\n\n<br>\n\nsm_img https://teacher.smartermaths.com.au/wp-content/uploads/2022/10/Geom_NAPX-H4-NC04_H3-NC10_v2q.svg 230 indent3 vpad\n\n<br>When complete the pattern has two lines of symmetry.\n\n\r\n\r\nWhich of these could be the missing part of the pattern?"},{"varval":"{{{correctAnswer}}}\n"}]},{"vars":[{"varval":"Some tiles are missing from this pattern of tiles.\n\n<br>\n\nsm_img https://teacher.smartermaths.com.au/wp-content/uploads/2022/10/Geom_NAPX-H4-NC04_H3-NC10_v3q.svg 230 indent3 vpad\n\n<br>When complete the pattern has two lines of symmetry.\n\n\r\n\r\nWhich of these could be the missing part of the pattern?"},{"varval":"{{{correctAnswer}}}\n"}]},{"vars":[{"varval":"Some tiles are missing from this pattern of tiles.\n\n<br>\n\nsm_img https://teacher.smartermaths.com.au/wp-content/uploads/2022/10/Geom_NAPX-H4-NC04_H3-NC10_v4q.svg 230 indent3 vpad\n\n<br>When complete the pattern has two lines of symmetry.\n\n\r\n\r\nWhich of these could be the missing part of the pattern?"},{"varval":"{{{correctAnswer}}}"}]},{"vars":[{"varval":"Some tiles are missing from this pattern of tiles.\n\n<br>\n\nsm_img https://teacher.smartermaths.com.au/wp-content/uploads/2022/10/Geom_NAPX-H4-NC04_H3-NC10_v5q.svg 230 indent3 vpad\n\n<br>When complete the pattern has two lines of symmetry.\n\n\r\n\r\nWhich of these could be the missing part of the pattern?"},{"varval":"{{{correctAnswer}}}\n"}]}]

  514. Corrected error in solution. Added var1. Added categories.

    <div class="sm_mode"> {{{question}}} </div>

    [{"vars":[{"varval":"sm_img https://teacher.smartermaths.com.au/wp-content/uploads/2020/09/NAPX-J4-CA09.svg 200 indent vpad\n\nWhich of these is the front view of this object made from cubes?​"},{"varval":"{{{correctAnswer}}}\n"}]},{"vars":[{"varval":"sm_img https://teacher.smartermaths.com.au/wp-content/uploads/2020/09/NAPX-J4-CA09.svg 200 indent vpad\n\nWhich of these is the left view of this object made from cubes?​ "},{"varval":"{{{correctAnswer}}}\n"}]}]

  515. <div class="sm_mode"> Lily's right mitten is pictured below. <br> sm_img https://teacher.smartermaths.com.au/wp-content/uploads/2021/03/NAPX9-TLC-29-2-1.png 100 indent3 vpad <br>Her left mitten is a mirror image of her right mitten. Which of these is Lily's left mitten? </div>

    [{"vars":null}]

  516. <div class="sm_mode"> {{{question}}} </div>

    [{"vars":[{"varval":"Which of these is **not** a net of a square pyramid?"},{"varval":"{{{correctAnswer}}}"}]},{"vars":[{"varval":"Which of these is **not** a net of a square pyramid?"},{"varval":"{{{correctAnswer}}}"}]},{"vars":[{"varval":"Which of these is **not** a net of a square pyramid?"},{"varval":"{{{correctAnswer}}}"}]},{"vars":[{"varval":"Which of these is **not** a net of a square pyramid?"},{"varval":"{{{correctAnswer}}}"}]},{"vars":[{"varval":"Which of these is **not** a net of a square pyramid?"},{"varval":"{{{correctAnswer}}}"}]},{"vars":[{"varval":"Which of these is **not** a net of a square pyramid?"},{"varval":"{{{correctAnswer}}}"}]}]

  517. <div class="sm_mode"> {{{question}}} </div>

    [{"vars":[{"varval":"sm_img https://teacher.smartermaths.com.au/wp-content/uploads/2017/02/NAPX-F3-CA04.svg 290 indent3 vpad\n\nThis diagram is the net of a\n"},{"varval":"{{{correctAnswer}}}"}]},{"vars":[{"varval":"sm_img https://teacher.smartermaths.com.au/wp-content/uploads/2022/10/Geom_NAPX-p120861v01_v4a.svg 290 indent3 vpad\n\nThis diagram is the net of a\n"},{"varval":"Face of prism has 6 sides $\\rightarrow$ hexa\n\n{{{correctAnswer}}}"}]},{"vars":[{"varval":"sm_img https://teacher.smartermaths.com.au/wp-content/uploads/2022/10/Geom_NAPX-p120861v01_v4b.svg 290 indent3 vpad\n\nThis diagram is the net of a\n"},{"varval":"Base has 5 sides $\\rightarrow$ penta\n\n{{{correctAnswer}}}"}]},{"vars":[{"varval":"sm_img https://teacher.smartermaths.com.au/wp-content/uploads/2022/10/Geom_NAPX-G3-CA12_v1c.svg 290 indent3 vpad\n\nThis diagram is the net of a\n"},{"varval":"{{{correctAnswer}}}"}]},{"vars":[{"varval":"sm_img https://teacher.smartermaths.com.au/wp-content/uploads/2022/10/Geom_NAPX-F3-CA04_v4.svg 290 indent3 vpad\n\nThis diagram is the net of a\n"},{"varval":"Base has 6 sides $\\rightarrow$ hexa\n\n{{{correctAnswer}}}"}]},{"vars":[{"varval":"sm_img https://teacher.smartermaths.com.au/wp-content/uploads/2022/10/Geom_NAPX-F3-CA04_v5.svg 290 indent3 vpad\n\nThis diagram is the net of a\n"},{"varval":"Prism will have 5 sides $\\rightarrow$ penta\n\n{{{correctAnswer}}}"}]}]

  518. Geometry, NAPX-L4-CA06 v1

    <div class="sm_mode"> {{{question}}} </div>

    [{"vars":[{"varval":"Ruby draws a design by reflecting this shape across a mirror line.\n\n<br>\n\nsm_img https://teacher.smartermaths.com.au/wp-content/uploads/2020/09/nap-L4-06-ver3.svg 300 indent3 vpad\n\n<br>Which design shows the correct reflection?"}]},{"vars":[{"varval":"Ruby draws a design by reflecting this shape across a mirror line.\n\n<br>\n\nsm_img https://teacher.smartermaths.com.au/wp-content/uploads/2020/09/nap-L4-06-ver2.svg 300 indent vpad\n\n<br>Which design shows the correct reflection?"}]}]

  519. <div class="sm_mode"> Justine painted the letter L on a paper square. She folded the paper square along the dotted line while the paint was still wet. <br> sm_img https://teacher.smartermaths.com.au/wp-content/uploads/2018/08/NAPX-F3-NC06rev2.svg 160 indent3 vpad <br>She then unfolded the paper square. What was on the square? </div>

    [{"vars":null}]

  520. Worked solution images need dimensions altered to look more like square bases - redone 16/8

    <div class="sm_mode"> {{{question}}} </div>

    [{"vars":[{"varval":"Shaz uses a rectangular prism pictured below as a basic building block.\n\nsm_img https://teacher.smartermaths.com.au/wp-content/uploads/2018/06/NAPX-E4-NC31-SA.svg 280 indent3 vpad\n\nShe stacks the blocks and makes a solid cube.\n\n\r\n\r\nWhat is the **smallest** number of blocks needed to make a cube?\n"},{"varval":"The smallest cube will have the smallest base:\n\n<br>\n\nsm_img https://teacher.smartermaths.com.au/wp-content/uploads/2022/08/Measurement-NAPX-E4-NC31-SA_v0.svg 440 indent3 vpad\n\n<br>Base will have 15 blocks.\n\n\rCube will have 15 stacked levels.\n\nsm_nogap $\\therefore$ Blocks needed for the smallest cube\n\n<div class=\"aligned\">\n\n>>||\n|-|\n|= 15 $\\times$ 15|\n|= {{{correctAnswer0}}}|\n\n</div>\n"}]},{"vars":[{"varval":"Brad uses a rectangular prism pictured below as a basic building block.\n\nsm_img https://teacher.smartermaths.com.au/wp-content/uploads/2022/08/Measurement_NAPX-E4-NC31-SA_v1q_q.svg 250 indent3 vpad\n\nHe stacks the blocks and makes a solid cube.\n\n\r\n\r\nWhat is the **smallest** number of blocks needed to make a cube?\n"},{"varval":"The smallest cube will have the smallest base:\n\n<br>\n\nsm_img https://teacher.smartermaths.com.au/wp-content/uploads/2022/08/Measurement-NAPX-E4-NC31-SA_v1.svg 380 indent3 vpad\n\n<br>Base will have 6 blocks.\n\n\rCube will have 6 stacked levels.\n\nsm_nogap $\\therefore$ Blocks needed for the smallest cube\n\n<div class=\"aligned\">\n\n>>||\n|-|\n|= 6 $\\times$ 6|\n|= {{{correctAnswer0}}}|\n\n</div>"}]},{"vars":[{"varval":"Shirley uses a rectangular prism pictured below as a basic building block.\n\nsm_img https://teacher.smartermaths.com.au/wp-content/uploads/2022/08/Measurement_NAPX-E4-NC31-SA_v2q1.svg 260 indent3 vpad\n\nShe stacks the blocks and makes a solid cube.\n\n\r\n\r\nWhat is the **smallest** number of blocks needed to make a cube?\n"},{"varval":"The smallest cube will have the smallest base:\n\n<br>\n\nsm_img https://teacher.smartermaths.com.au/wp-content/uploads/2022/08/Measurement-NAPX-E4-NC31-SA_v2.svg 370 indent3 vpad\n\n<br>Base will have 10 blocks.\n\n\rCube will have 10 stacked levels.\n\nsm_nogap $\\therefore$ Blocks needed for the smallest cube\n\n<div class=\"aligned\">\n\n>>||\n|-|\n|= 10 $\\times$ 10|\n|= {{{correctAnswer0}}}|\n\n</div>\n"}]}]

  521. <div class="sm_mode"> {{{question}}} </div>

    [{"vars":[{"varval":"An ancient civilisation created the following pattern in a large square field.\n\n<br>\n\nsm_img https://teacher.smartermaths.com.au/wp-content/uploads/2018/06/NAPX-F4-CA32-SA.svg 300 indent3 vpad\n\n<br>What is the size of the shaded area in the pattern?"},{"varval":"sm_nogap Total area of the largest square\n\n<div class=\"aligned\">\n\n>>||\n|-|\n|= 80 $\\times$ 80|\n|= 6400 m$^2$|\n\n</div>\n\r\n\r<br>\n \r\n\r\nArea of next smallest square\r = 3200 m$^2$\n\nArea of next smallest square = 1600 m$^2$\n\n<br>\n\n<div class=\"aligned\">\n\n|||\n|-|-|\n|$\\therefore$ Shaded area|= $\\dfrac{3}{4}\\times$ 1600|\n||= {{{correctAnswer0}}} {{{suffix0}}}|\n\n</div>"}]},{"vars":[{"varval":"An ancient civilisation created the following pattern in a large square field.\n\n<br>\n\nsm_img https://teacher.smartermaths.com.au/wp-content/uploads/2022/08/aebca96b-b270-4f44-80d6-e3a689067e0cv0_V3.svg 360 indent3 vpad\n\n<br>What is the size of the shaded area in the pattern?"},{"varval":"sm_nogap Total area of the largest square\n\n<div class=\"aligned\">\n\n>>||\n|-|\n|= 120 $\\times$ 120|\n|= 14 400 m$^2$|\n\n</div>\n\r\n\r<br>\n \r\n\r\nArea of next smallest square\r = 7200 m$^2$\n\nArea of next smallest square = 3600 m$^2$\n\n<br>\n\n<div class=\"aligned\">\n\n|||\n|-|-|\n|$\\therefore$ Shaded area|= $\\dfrac{1}{4}\\times$ 7200 + $\\dfrac{1}{2}\\times$ 3600|\n||= 1800 + 1800|\n||= {{{correctAnswer0}}} {{{suffix0}}}|\n\n</div>"}]},{"vars":[{"varval":"An ancient civilisation created the following pattern in a large square field.\n\n<br>\n\nsm_img https://teacher.smartermaths.com.au/wp-content/uploads/2022/08/aebca96b-b270-4f44-80d6-e3a689067e0cv0_V1.svg 400 indent3 vpad\n\n<br>What is the size of the shaded area in the pattern?"},{"varval":"sm_nogap Total area of the largest square\n\n<div class=\"aligned\">\n\n>>||\n|-|\n|= 80 $\\times$ 80|\n|= 6400 m$^2$|\n\n</div>\n\r\n\r<br>\n \r\n\r\nArea of next smallest square\r = 3200 m$^2$\n\nArea of next smallest square = 1600 m$^2$\n\n<br>\n\n<div class=\"aligned\">\n\n|||\n|-|-|\n|$\\therefore$ Shaded area|= $\\dfrac{1}{2}\\times$ 3200 + $\\dfrac{1}{4}\\times$ 1600|\n||= 1600 + 400|\n||= {{{correctAnswer0}}} {{{suffix0}}}|\n\n</div>"}]},{"vars":[{"varval":"An ancient civilisation created the pattern below on a large mural.\n\n<br>\n\nsm_img https://teacher.smartermaths.com.au/wp-content/uploads/2022/08/aebca96b-b270-4f44-80d6-e3a689067e0cv0_V1-1.svg 400 indent3 vpad\n\n<br>What is the size of the shaded area in the pattern?"},{"varval":"sm_nogap Total area of the largest square\n\n<div class=\"aligned\">\n\n>>||\n|-|\n|= 120 $\\times$ 120|\n|= 14 400 m$^2$|\n\n</div>\n\r\n\r<br>\n \r\n\r\nArea of next smallest square\r = 7200 m$^2$\n\nArea of next smallest square = 3600 m$^2$\n\n<br>\n\n<div class=\"aligned\">\n\n|||\n|-|-|\n|$\\therefore$ Shaded area|= $\\dfrac{1}{2}\\times$ 14 400 + $\\dfrac{3}{4}\\times$ 3600|\n||= 7200 + 2700|\n||= {{{correctAnswer0}}} {{{suffix0}}}|\n\n</div>"}]},{"vars":[{"varval":"A gardener was paving a large courtyard and created the following pattern.\n<br>\n\nsm_img https://teacher.smartermaths.com.au/wp-content/uploads/2022/08/aebca96b-b270-4f44-80d6-e3a689067e0cv0_V5.svg 400 indent3 vpad\n\n<br>What is the size of the shaded area in the pattern?"},{"varval":"sm_nogap Total area of the largest square\n\n<div class=\"aligned\">\n\n>>||\n|-|\n|= 40 $\\times$ 40|\n|= 1600 m$^2$|\n\n</div>\n\r\n\r<br>\n \r\n\r\nArea of next smallest square\r = 800 m$^2$\n\nArea of next smallest square = 400 m$^2$\n\n<br>\n\n<div class=\"aligned\">\n\n|||\n|-|-|\n|$\\therefore$ Shaded area|= $\\dfrac{1}{4}\\times$ 1600 + $\\dfrac{1}{2}\\times$ 800 + + $\\dfrac{1}{4}\\times$ 400|\n||= 400 + 400 + 100|\n||= {{{correctAnswer0}}} {{{suffix0}}}|\n\n</div>"}]},{"vars":[{"varval":"A gardener was paving a large courtyard and created the following pattern.\n\n<br>\n\nsm_img https://teacher.smartermaths.com.au/wp-content/uploads/2022/08/aebca96b-b270-4f44-80d6-e3a689067e0cv0_V1-1.svg 400 indent3 vpad\n\n<br>What is the size of the shaded area in the pattern?"},{"varval":"sm_nogap Total area of the largest square\n\n<div class=\"aligned\">\n\n>>||\n|-|\n|= 120 $\\times$ 120|\n|= 14 400 m$^2$|\n\n</div>\n\r\n\r<br>\n \r\n\r\nArea of next smallest square\r = 7200 m$^2$\n\nArea of next smallest square = 3600 m$^2$\n\n<br>\n\n<div class=\"aligned\">\n\n|||\n|-|-|\n|$\\therefore$ Shaded area|= $\\dfrac{1}{2}\\times$ 14 400 + $\\dfrac{3}{4}\\times$ 3600|\n||= 7200 + 2700|\n||= {{{correctAnswer0}}} {{{suffix0}}}|\n\n</div>"}]}]

  522. <div class="sm_mode"> Two views of a trapezoidal prism are shown below. <br> sm_img https://teacher.smartermaths.com.au/wp-content/uploads/2018/06/NAPX-E4-CA32i.svg 400 indent3 vpad <br>Each square on this grid has an area of one square centimetre. The vertical edges of the prism are 5 centimetres. What is the volume of the prism in cubic centimetres? </div>

    [{"vars":null}]

  523. <div class="sm_mode"> {{{question}}} </div>

    [{"vars":[{"varval":"Mickey cuts a quarter circle out of a full circle, as shown below.\n\n<br>\n\nsm_img https://teacher.smartermaths.com.au/wp-content/uploads/2018/04/NAPX-H4-CA32.svg 200 indent vpad\n\n<br>If the circle's radius is 7 cm, what is the perimeter of the shape, to the nearest centimetre?"},{"varval":"<div class=\"aligned\">\n\n|||\n|-|-|\n|Perimeter|= arc + 2 × radii|\n||= $\\bigg( \\dfrac{3}{4} \\times 2 \\times \\large \\pi$ $\\times\\ \\large r$ $\\bigg) + 2\\large r$|\n||= $\\bigg(\\dfrac{3}{4} \\times 2 \\times \\large \\pi$ $\\times\\ 7 \\bigg) + 2 \\times 7$|\n||= 46.986...|\n||= {{{correctAnswer0}}} {{{suffix0}}} (nearest cm)|\n\n</div>\n"}]},{"vars":[{"varval":"Vinnie cuts away two-thirds of a circle, leaving the shape shown below.\n\n<br>\n\nsm_img https://teacher.smartermaths.com.au/wp-content/uploads/2022/08/Measurement_J-NAPX-H4-CA32-SA_1.svg 200 indent vpad\n\n<br>If the circles' radius is 10 cm, what is the perimeter of the remaining shape, to the nearest centimetre?"},{"varval":"<div class=\"aligned\">\n\n|||\n|-|-|\n|Perimeter|= arc + 2 × radii|\n||= $\\bigg( \\dfrac{1}{3} \\times 2 \\times \\large \\pi$ $\\times\\ \\large r$ $\\bigg) + 2\\large r$|\n||= $\\bigg(\\dfrac{1}{3} \\times 2 \\times \\large \\pi$ $\\times\\ 10 \\bigg) + 2 \\times 10$|\n||= 40.9439...|\n||= {{{correctAnswer0}}} {{{suffix0}}} (nearest cm)|\n\n</div>\n"}]},{"vars":[{"varval":"Milo cuts away one third of a circle, leaving the shape shown below.\n\n<br>\n\nsm_img https://teacher.smartermaths.com.au/wp-content/uploads/2022/08/Measurement_J-NAPX-H4-CA32-SA_2.svg 300 indent vpad\n\n<br>If the circle's radius is 5 cm, what is the perimeter of the remaining shape, to the nearest centimetre?"},{"varval":"<div class=\"aligned\">\n\n|||\n|-|-|\n|Perimeter|= arc + 2 × radii|\n||= $\\bigg( \\dfrac{2}{3} \\times 2 \\times \\large \\pi$ $\\times\\ \\large r$ $\\bigg) + 2\\large r$|\n||= $\\bigg(\\dfrac{2}{3} \\times 2 \\times \\large \\pi$ $\\times\\ 5 \\bigg) + 2 \\times 5$|\n||= 30.94...|\n||= {{{correctAnswer0}}} {{{suffix0}}} (nearest cm)|\n\n</div>\n"}]},{"vars":[{"varval":"Morris cuts a sector measuring 30 degrees from a full circle, as shown below.\n\n<br>\n\nsm_img https://teacher.smartermaths.com.au/wp-content/uploads/2022/08/Measurement_J-NAPX-H4-CA32-SA_3.svg 240 indent vpad\n\n<br>If the circle's radius is 20 cm, what is the perimeter of the shape, to the nearest centimetre?"},{"varval":"<div class=\"aligned\">\n\n|||\n|-|-|\n|Perimeter|= arc + 2 × radii|\n||= $\\bigg( \\dfrac{30}{360} \\times 2 \\times \\large \\pi$ $\\times\\ \\large r$ $\\bigg) + 2\\large r$|\n||= $\\bigg(\\dfrac{1}{12} \\times 2 \\times \\large \\pi$ $\\times\\ 20 \\bigg) + 2 \\times 20$|\n||= 50.471...|\n||= {{{correctAnswer0}}} {{{suffix0}}} (nearest cm)|\n\n</div>\n"}]},{"vars":[{"varval":"Morris cuts a sector measuring 60 degrees from a full circle, as shown below.\n\n<br>\n\nsm_img https://teacher.smartermaths.com.au/wp-content/uploads/2022/08/Measurement_J-NAPX-H4-CA32-SA_5.svg 210 indent vpad\n\n<br>If the circle's radius is 36 cm, what is the perimeter of the shape, to the nearest centimetre?"},{"varval":"<div class=\"aligned\">\n\n|||\n|-|-|\n|Perimeter|= arc + 2 × radii|\n||= $\\bigg( \\dfrac{60}{360} \\times 2 \\times \\large \\pi$ $\\times\\ \\large r$ $\\bigg) + 2\\large r$|\n||= $\\bigg(\\dfrac{1}{6} \\times 2 \\times \\large \\pi$ $\\times\\ 36 \\bigg) + 2 \\times 36$|\n||= 109.699...|\n||= {{{correctAnswer0}}} {{{suffix0}}} (nearest cm)|\n\n</div>\n"}]},{"vars":[{"varval":"Monica cuts a right-angled sector from a full circle, as shown below.\n\n<br>\n\nsm_img https://teacher.smartermaths.com.au/wp-content/uploads/2022/08/Measurement_J-NAPX-H4-CA32-SA_4.svg 170 indent vpad\n\n<br>If the circle's radius is 25 cm, what is the perimeter of the shape, to the nearest centimetre?"},{"varval":"<div class=\"aligned\">\n\n|||\n|-|-|\n|Perimeter|= arc + 2 × radii|\n||= $\\bigg( \\dfrac{1}{4} \\times 2 \\times \\large \\pi$ $\\times\\ \\large r$ $\\bigg) + 2\\large r$|\n||= $\\bigg(\\dfrac{1}{4} \\times 2 \\times \\large \\pi$ $\\times\\ 25 \\bigg) + 2 \\times 25$|\n||= 89.2699...|\n||= {{{correctAnswer0}}} {{{suffix0}}} (nearest cm)|\n\n</div>\n"}]}]

  524. <div class="sm_mode"> {{{question}}} </div>

    [{"vars":[{"varval":"\nA rectangular prism is made by stacking 6 identical triangular prisms, as pictured below.\n\n<br>\n\nsm_img https://teacher.smartermaths.com.au/wp-content/uploads/2018/04/NAPX-J4-CA39.svg 320 indent3 vpad\n\n<br>What is the volume of one triangular prism, in cubic centimetres?"},{"varval":"sm_nogap Volume of rectangular prism\n\n<div class=\"aligned\">\n\n>>||\n|-|\n|= 42 × 48 × 82|\n|= 165 312 cm$^3$|\n\n</div>\n\n<br>\n\nsm_nogap $\\therefore$ Volume of 1 triangular prism\n\n<div class=\"aligned\">\n\n>>||\n|-|\n|= 165 312 ÷ 6|\n|= 27 552 {{{suffix0}}}|\n\n</div>"}]},{"vars":[{"varval":"\nA rectangular prism is made by stacking 8 identical triangular prisms, as pictured below.\n\n<br>\n\nsm_img https://teacher.smartermaths.com.au/wp-content/uploads/2022/08/Measurement_J-NAPX-J4-CA39-SA_1.svg 400 indent vpad\n\n<br>What is the volume of one triangular prism, in cubic centimetres?"},{"varval":"sm_nogap Volume of rectangular prism\n\n<div class=\"aligned\">\n\n>>||\n|-|\n|= 18 × 24 × 44|\n|= 19 008 cm$^3$|\n\n</div>\n\n<br>\n\nsm_nogap $\\therefore$ Volume of 1 triangular prism\n\n<div class=\"aligned\">\n\n>>||\n|-|\n|= 19 008 ÷ 8|\n|= {{{correctAnswer0}}} {{{suffix0}}}|\n\n</div>"}]},{"vars":[{"varval":"A rectangular prism is made by stacking 12 identical triangular prisms, as pictured below.\n\n<br>\n\nsm_img https://teacher.smartermaths.com.au/wp-content/uploads/2022/08/Measurement^J-NAPX-J4-CA39-SA_2.svg 400 indent vpad\n\n<br>What is the volume of one triangular prism, in cubic centimetres?"},{"varval":"sm_nogap Volume of rectangular prism\n\n<div class=\"aligned\">\n\n>>||\n|-|\n|= 84 × 62 × 123|\n|= 640 584 cm$^3$|\n\n</div>\n\n<br>\n\nsm_nogap $\\therefore$ Volume of 1 triangular prism\n\n<div class=\"aligned\">\n\n>>||\n|-|\n|= 640 584 ÷ 12|\n|= 53 382 {{{suffix0}}}|\n\n</div>"}]},{"vars":[{"varval":"A rectangular prism is made by stacking 6 identical triangular prisms, as pictured below.\n\n<br>\n\nsm_img https://teacher.smartermaths.com.au/wp-content/uploads/2022/08/Measurement^J-NAPX-J4-CA39-SA_3.svg 450 indent vpad\n\n<br>What is the volume of one triangular prism, in cubic centimetres?"},{"varval":"sm_nogap Volume of rectangular prism\n\n<div class=\"aligned\">\n\n>>||\n|-|\n|= 264 × 125 × 85|\n|= 2 805 000 cm$^3$|\n\n</div>\n\n<br>\n\nsm_nogap $\\therefore$ Volume of 1 triangular prism\n\n<div class=\"aligned\">\n\n>>||\n|-|\n|= 2 805 000 ÷ 6|\n|= 467 500 {{{suffix0}}}|\n\n</div>"}]},{"vars":[{"varval":"A rectangular prism is made by stacking 8 identical triangular prisms, as pictured below.\n\n<br>\n\nsm_img https://teacher.smartermaths.com.au/wp-content/uploads/2022/08/Measurement^J-NAPX-J4-CA39-SA_4.svg 440 indent3 vpad\n\n<br>What is the volume of one triangular prism, in cubic centimetres?"},{"varval":"sm_nogap Volume of rectangular prism\n\n<div class=\"aligned\">\n\n>>||\n|-|\n|= 80 × 35 × 64|\n|= 179 200 cm$^3$|\n\n</div>\n\n<br>\n\nsm_nogap $\\therefore$ Volume of 1 triangular prism\n\n<div class=\"aligned\">\n\n>>||\n|-|\n|= 179 200 ÷ 8|\n|= 22 400 {{{suffix0}}}|\n\n</div>"}]},{"vars":[{"varval":"A rectangular prism is made by stacking 10 identical triangular prisms, as pictured below.\n\n<br>\n\nsm_img https://teacher.smartermaths.com.au/wp-content/uploads/2022/08/Measurement^J-NAPX-J4-CA39-SA_5.svg 370 indent3 vpad\n\n<br>What is the volume of one triangular prism, to the nearest **cubic millimetre**?"},{"varval":"sm_nogap Volume of rectangular prism\n\n<div class=\"aligned\">\n\n>>||\n|-|\n|= 118 × 102 × 188|\n|= 2 262 768 mm$^3$|\n\n</div>\n\n<br>\n\nsm_nogap $\\therefore$ Volume of 1 triangular prism\n\n<div class=\"aligned\">\n\n>>||\n|-|\n|= 2 262 768 ÷ 10|\n|= 226 277 {{{suffix0}}}|\n\n</div>"}]}]

  525. <div class="sm_mode"> {{{question}}} </div>

    [{"vars":[{"varval":"A farmer uses an existing stone wall and fencing to create a large grazing paddock for his sheep, as shown in the diagram below.\n\r\nThe fencing has 3-strand wiring which is also shown below.\n\n<br>\n\nsm_img https://teacher.smartermaths.com.au/wp-content/uploads/2018/06/NAPX-F4-NC31.svg 540 indent vpad\n\n<br>How many kilometres of wire will be required?\n"},{"varval":"<div class=\"aligned\">\n\n|||\n|-|-|\n|Wire needed|= 3 × (250 + 700 + 550 + 400 + 800)|\n||= 3 × 2700|\n||= 8100 m|\n||= {{{correctAnswer0}}} {{{suffix0}}}|\n\n</div>"}]},{"vars":[{"varval":"A farmer uses the river bordering his property and fencing to create a large grazing paddock for his horses, as shown in the diagram below.\n\r\nThe fencing has 4-strand wiring which is also shown below.\n\n\n\n<br>\n\nsm_img https://teacher.smartermaths.com.au/wp-content/uploads/2022/08/Measurement_NAPX-F4-NC31-SA-NAPX-F3-NC32-SA_v1b.svg 600 indent vpad\n\n<br>Given that fencing is not required on the river boundary, how many kilometres of wire will be required?\n"},{"varval":"<div class=\"aligned\">\n\n|||\n|-|-|\n|Wire needed|= 4 × (700 + 200 + 300 + 1100 + 200 + 200 + 1200)|\n||= 4 × 3900|\n||= 15 600 m|\n||= {{{correctAnswer0}}} {{{suffix0}}}|\n\n</div>"}]},{"vars":[{"varval":"A farmer is planning to build a holding pen for his cows. He will use an existing barn as one border and complete the remaining borders using wire fencing as shown in the diagram below.\n\r\nThe fencing has 4-strand wiring which is also shown below.\n\n<br>\n\nsm_img https://teacher.smartermaths.com.au/wp-content/uploads/2022/08/Measurement_NAPX-F4-NC31-SA-NAPX-F3-NC32-SA_v2b.svg 780 indent vpad\n\nGiven that fencing is not required on the barn boundary, how many kilometres of wire will be required?"},{"varval":"<div class=\"aligned\">\n\n|||\n|-|-|\n|Wire needed|= 4 × (170 + 25 + 60 + 100 + 60 + 40 + 130 + 80)|\n||= 4 × 665|\n||= 2660 m|\n||= {{{correctAnswer0}}} {{{suffix0}}}|\n\n</div>"}]},{"vars":[{"varval":"Kai uses an existing sandstone wall and fencing to create a boundary for a community herb garden as shown in the diagram below.\n\r\nThe fencing has 3-strand wiring which is also shown below.\n\n<br>\n\nsm_img https://teacher.smartermaths.com.au/wp-content/uploads/2022/08/Measurement_NAPX-F4-NC31-SA-NAPX-F3-NC32-SA_v3.svg 550 indent vpad\n\n<br>How many metres of wire will be required?\n"},{"varval":"Using &nbsp;1 m = 100 cm\n\n<div class=\"aligned\">\n\n|||\n|-|-|\n|Wire needed|= 3 × (360 + 70 + 190 + 180 + 190 + 50 + 360)|\n||= 3 × 1400|\n||= 4200 cm|\n||= {{{correctAnswer0}}} {{{suffix0}}}|\n\n</div>"}]},{"vars":[{"varval":"Brenda uses an existing brick wall and fencing to create a boundary for her small market garden as shown in the diagram below.\n\r\nThe fencing has 3-strand wiring which is also shown below.\n\n<br>\n\nsm_img https://teacher.smartermaths.com.au/wp-content/uploads/2022/08/Measurement_NAPX-F4-NC31-SA-NAPX-F3-NC32-SA_v4.svg 590 indent vpad\n\n<br>How many metres of wire will be required?\n"},{"varval":"<div class=\"aligned\">\n\n|||\n|-|-|\n|Wire needed|= 3 × (2 × 8 + 4 × 5 + 2.2 + 2.7 + 2.1 + 2.6 + 2.3)|\n||= 3 × 47.9|\n||= {{{correctAnswer0}}} {{{suffix0}}}|\n\n</div>"}]}]

  526. <div class="sm_mode"> {{{question}}} </div>

    [{"vars":[{"varval":"The octagon below is divided into identical squares and identical triangles.\n\n<br>\n\nsm_img https://teacher.smartermaths.com.au/wp-content/uploads/2017/01/NAP-E4-NC29-150x150.png 160 indent3 vpad\n\n<br>The area of the octagon is a multiple of the area of one of the squares.\n\n\r\n\r\nThe area of the octagon is 112 cm$^2$.\n\n\r\n\r\nWhat is the side length of each square?"},{"varval":"sm_nogap Area of octagon in squares\n\n\r\n>> = 7 squares\n\nsm_nogap Area of 1 square\n\n\r\n>> = 112 ÷ 7\n\n\r\n>> = 16 cm$^2$\r\n \r\n\r\nsm_nogap $\\therefore$ Side length of each square\r\n\n>>= $\\sqrt{16}$\r\n\n>>= {{{correctAnswer0}}} {{{suffix0}}}"}]},{"vars":[{"varval":"The octagon below is divided into identical squares and identical triangles.\n\n<br>\n\nsm_img https://teacher.smartermaths.com.au/wp-content/uploads/2022/08/Measurement_NAPX-E4-NC29-SA_v1.svg 180 indent3 vpad\n\n<br>The area of the octagon is a multiple of the area of one of the squares.\n\n\r\n\r\nThe area of the octagon is 360 cm$^2$.\n\n\r\n\r\nWhat is the side length of each square?"},{"varval":"sm_nogap Area of octagon in squares\n\n\r\n>> = 10 squares\n\nsm_nogap Area of 1 square\n\n\r\n>> = 360 ÷ 10\n\n\r\n>> = 36 cm$^2$\r\n \r\n\r\nsm_nogap $\\therefore$ Side length of each square\n\r\n\n>>= $\\sqrt{36}$\r\n\n>>= {{{correctAnswer0}}} {{{suffix0}}}"}]},{"vars":[{"varval":"The octagon below is divided into identical squares and identical triangles.\n\n<br>\n\nsm_img https://teacher.smartermaths.com.au/wp-content/uploads/2022/08/Measurement_NAPX-E4-NC29-SA_v2.svg 180 indent3 vpad\n\n<br>The area of the octagon is a multiple of the area of one of the squares.\n\n\r\n\r\nThe area of the octagon is 686 cm$^2$.\n\n\r\n\r\nWhat is the side length of each square?"},{"varval":"sm_nogap Area of octagon in squares\n\n\r\n>> = 14 squares\n\nsm_nogap Area of 1 square\n\n\r\n>> = 686 ÷ 14\n\n\r\n>> = 49 cm$^2$\r\n \r\n\r\nsm_nogap $\\therefore$ Side length of each square\n\r\n\n>>= $\\sqrt{49}$\r\n\n>>= {{{correctAnswer0}}} {{{suffix0}}}"}]},{"vars":[{"varval":"The octagon below is divided into identical squares and identical triangles.\n\n<br>\n\nsm_img https://teacher.smartermaths.com.au/wp-content/uploads/2022/08/Measurement_NAPX-E4-NC29-SA_v3.svg 270 indent3 vpad\n\n<br>The area of the octagon is a multiple of the area of one of the squares.\n\n\r\n\r\nThe area of the octagon is 450 cm$^2$.\n\n\r\n\r\nWhat is the side length of each square?"},{"varval":"sm_nogap Area of octagon in squares\n\n\r\n>> = 18 squares\n\nsm_nogap Area of 1 square\n\n\r\n>> = 450 ÷ 18\n\n\r\n>> = 25 cm$^2$\r\n \r\n\r\nsm_nogap $\\therefore$ Side length of each square\n\r\n\n>>= $\\sqrt{25}$\r\n\n>>= {{{correctAnswer0}}} {{{suffix0}}}"}]},{"vars":[{"varval":"The octagon below is divided into identical squares and identical triangles.\n\n<br>\n\nsm_img https://teacher.smartermaths.com.au/wp-content/uploads/2022/08/Measurement_NAPX-E4-NC29-SA_v4.svg 300 indent3 vpad\n\n<br>The area of the octagon is a multiple of the area of one of the squares.\n\nThe area of the octagon is 1782 cm$^2$.\n\nWhat is the side length of each square?"},{"varval":"sm_nogap Area of octagon in squares\n\n\r\n>> = 22 squares\n\nsm_nogap Area of 1 square\n\n\r\n>> = 1782 ÷ 22\n\n\r\n>> = 81 cm$^2$\r\n \r\n\r\nsm_nogap $\\therefore$ Side length of each square\n\r\n\n>>= $\\sqrt{81}$\r\n\n>>= {{{correctAnswer0}}} {{{suffix0}}}"}]},{"vars":[{"varval":"The octagon below is divided into identical squares and identical triangles.\n\n<br>\n\nsm_img https://teacher.smartermaths.com.au/wp-content/uploads/2022/08/Measurement_NAPX-E4-NC29-SA_v5.svg 300 indent3 vpad\n\n<br>The area of the octagon is a multiple of the area of one of the squares.\n\nThe area of the octagon is 1792 cm$^2$.\n\nWhat is the side length of each square?"},{"varval":"sm_nogap Area of octagon in squares\n\n\r\n>> = 28 squares\n\nsm_nogap Area of 1 square\n\n\r\n>> = 1792 ÷ 28\n\n\r\n>> = 64 cm$^2$\r\n \r\n\r\nsm_nogap $\\therefore$ Side length of each square\n\r\n\n>>= $\\sqrt{64}$\r\n\n>>= {{{correctAnswer0}}} {{{suffix0}}}"}]}]

  527. <div class="sm_mode"> {{{question}}} </div>

    [{"vars":[{"varval":"Lux is a carpenter. He cuts a block of wood in the shape of a rectangular prism along the cut line shown below.\n\n<br>\n\nsm_img https://teacher.smartermaths.com.au/wp-content/uploads/2018/05/NAPX-G4-CA31-SA.svg 350 indent vpad\n\n<br>Lux keeps the larger piece of wood which is in the shape of a trapezoidal prism.\n\nWhat is the volume of the larger piece of wood in cubic metres?\n\nGive your answer to 2 decimal places."},{"varval":"sm_nogap Area of trapezoid (face)\n\n<div class=\"aligned\">\n\n>>||\n|-|\n|= $\\dfrac{1}{2}\\large h$($\\large a$ + $\\large b)$|\n|= $\\dfrac{1}{2}$ × 0.7 × (0.4 + 0.76)|\n|= 0.406 m$^2$|\n\n</div>\n\n<br>\n\n<div class=\"aligned\">\n\n|||\n|-|-|\n|Volume\t|= $A\\large h$|\n||= 0.406 × 0.8|\n||= 0.3248|\n||= {{{correctAnswer0}}} {{{suffix0}}} (to 2 d.p.)|\n\n</div>"}]},{"vars":[{"varval":"Manny is a carpenter. He cuts a block of wood in the shape of a rectangular prism along the cut line shown below.\n\n<br>\n\nsm_img https://teacher.smartermaths.com.au/wp-content/uploads/2022/08/Measurement_NAPX-G4-CA31-SA_1.svg 500 indent vpad\n\n<br>Manny keeps the larger piece of wood which is in the shape of a trapezoidal prism.\n\nWhat is the volume of the larger piece of wood in cubic metres?\n\nGive your answer to 2 decimal places."},{"varval":"sm_nogap Area of trapezoid (face)\n\n<div class=\"aligned\">\n\n>>||\n|-|\n|= $\\dfrac{1}{2}\\large h$($\\large a$ + $\\large b)$|\n|= $\\dfrac{1}{2}$ × 1.5 × (0.6 + 0.72)|\n|= 0.99 m$^2$|\n\n</div>\n\n<br>\n\n<div class=\"aligned\">\n\n|||\n|-|-|\n|Volume\t|= $A\\large h$|\n||= 0.99 × 0.5|\n||= 0.495|\n||= {{{correctAnswer0}}} {{{suffix0}}} (to 2 d.p.)|\n\n</div>"}]},{"vars":[{"varval":"Bree is a carpenter. She cuts a block of wood in the shape of a rectangular prism along the cut line shown below.\n\n<br>\n\nsm_img https://teacher.smartermaths.com.au/wp-content/uploads/2022/08/Measurement_NAPX-G4-CA31-SA_2.svg 550 indent vpad\n\n<br>Bree keeps the larger piece of wood which is in the shape of a trapezoidal prism.\n\nWhat is the volume of the larger piece of wood in cubic metres?\n\nGive your answer to 2 decimal places."},{"varval":"sm_nogap Area of trapezoid (face)\n\n<div class=\"aligned\">\n\n>>||\n|-|\n|= $\\dfrac{1}{2}\\large h$($\\large a$ + $\\large b)$|\n|= $\\dfrac{1}{2}$ × 0.4 × (0.12 + 0.17)|\n|= 0.058 m$^2$|\n\n</div>\n\n<br>\n\n<div class=\"aligned\">\n\n|||\n|-|-|\n|Volume\t|= $A\\large h$|\n||= 0.058 × 0.5|\n||= 0.029|\n||= {{{correctAnswer0}}} {{{suffix0}}} (to 2 d.p.)|\n\n</div>"}]},{"vars":[{"varval":"Carrie is a carpenter. She cuts a block of wood in the shape of a rectangular prism along the cut line shown below.\n\n<br>\n\nsm_img https://teacher.smartermaths.com.au/wp-content/uploads/2022/08/Measurement_NAPX-G4-CA31-SA_3.svg 550 indent vpad\n\n<br>Carrie keeps the larger piece of wood which is in the shape of a trapezoidal prism.\n\nWhat is the volume of the larger piece of wood in cubic metres?\n\nGive your answer to 2 decimal places."},{"varval":"sm_nogap Area of trapezoid (face)\n\n<div class=\"aligned\">\n\n>>||\n|-|\n|= $\\dfrac{1}{2}\\large h$($\\large a$ + $\\large b)$|\n|= $\\dfrac{1}{2}$ × 0.35 × (0.175 + 0.19)|\n|= 0.063875 m$^2$|\n\n</div>\n\n<br>\n\n<div class=\"aligned\">\n\n|||\n|-|-|\n|Volume\t|= $A\\large h$|\n||= 0.063875 × 0.4|\n||= 0.02555|\n||= {{{correctAnswer0}}} {{{suffix0}}}|\n\n</div>"}]},{"vars":[{"varval":"Blaze is a carpenter. He cuts a block of wood in the shape of a rectangular prism along the cut line shown below.\n\n<br>\n\nsm_img https://teacher.smartermaths.com.au/wp-content/uploads/2022/08/Measurement_NAPX-G4-CA31-SA_4.svg 500 indent vpad\n\n<br>Blaze keeps the larger piece of wood which is in the shape of a trapezoidal prism.\n\nWhat is the volume of the larger piece of wood in cubic metres?\n\nGive your answer to 2 decimal places."},{"varval":"sm_nogap Area of **larger** trapezoid (face)\n\n<div class=\"aligned\">\n\n>>||\n|-|\n|= $\\dfrac{1}{2}\\large h$($\\large a$ + $\\large b)$|\n|= $\\dfrac{1}{2}$ × 0.5 × (0.15 + 0.5)|\n|= 0.1625 m$^2$|\n\n</div>\n\n<br>\n\n<div class=\"aligned\">\n\n|||\n|-|-|\n|Volume\t|= $A\\large h$|\n||= 0.1625 × 0.55|\n||=0.089375|\n||= {{{correctAnswer0}}} {{{suffix0}}} (to 2 d.p.)|\n\n</div>"}]},{"vars":[{"varval":"Olive is a carpenter. She cuts a block of wood in the shape of a rectangular prism along the cut line shown below.\n\n<br>\n\nsm_img https://teacher.smartermaths.com.au/wp-content/uploads/2022/08/Measurement_NAPX-G4-CA31-SA_5.svg 550 indent vpad\n\n<br>Olive keeps the larger piece of wood which is in the shape of a trapezoidal prism.\n\nWhat is the volume of the larger piece of wood in cubic metres?"},{"varval":"sm_nogap Area of **larger** trapezoid (face)\n\n<div class=\"aligned\">\n\n>>||\n|-|\n|= $\\dfrac{1}{2}\\large h$($\\large a$ + $\\large b)$|\n|= $\\dfrac{1}{2}$ × 0.8 × (0.6 + 1.4)|\n|= 0.8 m$^2$|\n\n</div>\n\n<br>\n\n<div class=\"aligned\">\n\n|||\n|-|-|\n|Volume\t|= $A\\large h$|\n||= 0.8 × 0.5|\n||= {{{correctAnswer0}}} {{{suffix0}}}|\n\n</div>"}]}]

  528. <div class="sm_mode"> {{{question}}} </div>

    [{"vars":[{"varval":"A box in the shape of a triangular prism is used to store smaller triangular prism pieces as shown below.\n\nsm_img https://teacher.smartermaths.com.au/wp-content/uploads/2018/04/NAPX-I4-CA30.svg 370 indent vpad\n\nEach smaller triangular prism is 78 cm³ in size.\n\nsm_img https://teacher.smartermaths.com.au/wp-content/uploads/2018/04/NAPX-I4-CA301.svg 260 indent vpad\n\nWhat is the maximum number of smaller triangular prisms that can fit inside the box?\n"},{"varval":"The dimensions of the smaller triangular prism fit.\n\n\r\nFind the smaller triangular prism's width ($\\large w$):\n\n<div class=\"aligned\">\n\n|||\n|-:|-|\n|$A \\times \\large w$|= 78|\n|$\\dfrac{1}{2} \\times 8.8 \\times 12 \\times \\large w$|= 78|\n|$\\large w$|= $\\dfrac{78}{52.8}$|\n||= 1.477... cm|\n\n</div>\n\n<br>\n\nsm_nogap $\\therefore$ Maximum triangular prisms that fit\n\n<div class=\"aligned\">\n\n>>||\n|-|\n|= $\\dfrac{40}{1.477...}$|\n|= 27.07...|\n|= {{{correctAnswer0}}} {{{suffix0}}}|\n\n</div>\n"}]},{"vars":[{"varval":"A box in the shape of a triangular prism is used to store smaller triangular prism pieces as shown below.\n\nsm_img https://teacher.smartermaths.com.au/wp-content/uploads/2022/08/Measurement-–-NAPX-I4-CA30-SA_0_a.svg 500 indent vpad\n\nEach smaller triangular prism is 95 cm³ in size.\n\nsm_img https://teacher.smartermaths.com.au/wp-content/uploads/2022/08/Measurement-–-NAPX-I4-CA30-SA_0_b.svg 320 indent vpad\n\nWhat is the maximum number of smaller triangular prisms that can fit inside the box?\n"},{"varval":"The dimensions of the smaller triangular prism fit into the larger one.\n\n\r\nFind the smaller triangular prism's width ($\\large w$):\n\n<div class=\"aligned\">\n\n|||\n|-:|-|\n|$A \\times \\large w$|= 95|\n|$\\dfrac{1}{2} \\times 9.8 \\times 15 \\times \\large w$|= 95|\n|$\\large w$|= $\\dfrac{95}{73.5}$|\n||= 1.292... cm|\n\n</div>\n\n<br>\n\nsm_nogap $\\therefore$ Maximum triangular prisms that fit\n\n<div class=\"aligned\">\n\n>>||\n|-|\n|= $\\dfrac{60}{1.2925...}$|\n|= 46.42...|\n|= {{{correctAnswer0}}} {{{suffix0}}}|\n\n</div>\n\n"}]},{"vars":[{"varval":"A box in the shape of a triangular prism is used to store smaller triangular prism pieces as shown below.\n\nsm_img https://teacher.smartermaths.com.au/wp-content/uploads/2022/08/Measurement-–-NAPX-I4-CA30-SA_2_a-2.svg 480 indent vpad\n\nEach smaller triangular prism is 80 cm³ in size.\n\nsm_img https://teacher.smartermaths.com.au/wp-content/uploads/2022/08/Measurement-–-NAPX-I4-CA30-SA_2_b.svg 300 indent vpad\n\nWhat is the maximum number of smaller triangular prisms that can fit inside the box?\n"},{"varval":"The dimensions of the smaller triangular prism fit into the larger one.\n\n\r\nFind the smaller triangular prism's width ($\\large w$):\n\n<div class=\"aligned\">\n\n|||\n|-:|-|\n|$A \\times \\large w$|= 80|\n|$\\dfrac{1}{2} \\times 6.2 \\times 8 \\times \\large w$|= 80|\n|$\\large w$|= $\\dfrac{80}{24.8}$|\n||= 3.225... cm|\n\n</div>\n\n<br>\n\nsm_nogap $\\therefore$ Maximum triangular prisms that fit\n\n<div class=\"aligned\">\n\n>>||\n|-|\n|= $\\dfrac{60}{3.2258...}$|\n|= 18.6|\n|= {{{correctAnswer0}}} {{{suffix0}}}|\n\n</div>\n\n"}]},{"vars":[{"varval":"A box in the shape of a triangular prism is used to store smaller triangular prism pieces as shown below.\n\nsm_img https://teacher.smartermaths.com.au/wp-content/uploads/2022/08/Measurement-–-NAPX-I4-CA30-SA_3_c.svg 480 indent vpad\n\nEach smaller triangular prism is 200 cm³ in size.\n\nsm_img https://teacher.smartermaths.com.au/wp-content/uploads/2022/08/Measurement-–-NAPX-I4-CA30-SA_3_b.svg 300 indent vpad\n\nWhat is the maximum number of smaller triangular prisms that can fit inside the box?\n"},{"varval":"The dimensions of the smaller triangular prism fit into the larger one.\n\n\r\nFind the smaller triangular prism's width ($\\large w$):\n\n<div class=\"aligned\">\n\n|||\n|-:|-|\n|$A \\times \\large w$|= 200|\n|$\\dfrac{1}{2} \\times 18 \\times 11 \\times \\large w$|= 200|\n|$\\large w$|= $\\dfrac{200}{99}$|\n||= 2.02... cm|\n\n</div>\n\n<br>\n\nsm_nogap $\\therefore$ Maximum triangular prisms that fit\n\n<div class=\"aligned\">\n\n>>||\n|-|\n|= $\\dfrac{100}{2.02...}$|\n|= 49.5...|\n|= {{{correctAnswer0}}} {{{suffix0}}}|\n\n</div>\n\n"}]},{"vars":[{"varval":"A box in the shape of a triangular prism is used to store smaller triangular prism pieces as shown below.\n\nsm_img https://teacher.smartermaths.com.au/wp-content/uploads/2022/08/Measurement-–-NAPX-I4-CA30-SA_4_c.svg 480 indent vpad\n\nEach smaller triangular prism is 330 cm³ in size.\n\nsm_img https://teacher.smartermaths.com.au/wp-content/uploads/2022/08/Measurement-–-NAPX-I4-CA30-SA_4_b.svg 300 indent vpad\n\nWhat is the maximum number of smaller triangular prisms that can fit inside the box?\n"},{"varval":"The dimensions of the smaller triangular prism fit into the larger one.\n\n\r\nFind the smaller triangular prism's width ($\\large w$):\n\n<div class=\"aligned\">\n\n|||\n|-:|-|\n|$A \\times \\large w$|= 330|\n|$\\dfrac{1}{2} \\times 10.8 \\times 12 \\times \\large w$|= 330|\n|$\\large w$|= $\\dfrac{330}{64.8}$|\n||= 5.09... cm|\n\n</div>\n\n<br>\n\nsm_nogap $\\therefore$ Maximum triangular prisms that fit\n\n<div class=\"aligned\">\n\n>>||\n|-|\n|= $\\dfrac{50}{5.09...}$|\n|= 9.8181...|\n|= {{{correctAnswer0}}} {{{suffix0}}}|\n\n</div>\n"}]},{"vars":[{"varval":"A box in the shape of a triangular prism is used to store smaller triangular prism pieces as shown below.\n\nsm_img https://teacher.smartermaths.com.au/wp-content/uploads/2022/08/Measurement-–-NAPX-I4-CA30-SA_5_a.svg 480 indent vpad\n\nEach smaller triangular prism is 660 cm³ in size.\n\nsm_img https://teacher.smartermaths.com.au/wp-content/uploads/2022/08/Measurement-–-NAPX-I4-CA30-SA_5_b.svg 300 indent vpad\n\nWhat is the maximum number of smaller triangular prisms that can fit inside the box?\n"},{"varval":"The dimensions of the smaller triangular prism fit into the larger one.\n\n\r\nFind the smaller triangular prism's width ($\\large w$):\n\n<div class=\"aligned\">\n\n|||\n|-:|-|\n|$A \\times \\large w$|= 660|\n|$\\dfrac{1}{2} \\times 15 \\times 20 \\times \\large w$|= 660|\n|$\\large w$|= $\\dfrac{660}{150}$|\n||= 4.4 cm|\n\n</div>\n\n<br>\n\nsm_nogap $\\therefore$ Maximum triangular prisms that fit\n\n<div class=\"aligned\">\n\n>>||\n|-|\n|= $\\dfrac{80}{4.4}$|\n|= 18.1818...|\n|= {{{correctAnswer0}}} {{{suffix0}}}|\n\n</div>\n"}]}]

  529. <div class="sm_mode"> {{{question}}} </div>

    [{"vars":[{"varval":"A cube has a surface area of 150 square centimetres.\n\n\r\n\r\nWhat is the volume of the cube?"},{"varval":"sm_nogap Cube has 6 faces.\n\n<div class=\"aligned\">\n\n|||\n|-|-|\n|Area of 1 face|= 150 ÷ 6|\n||= 25 cm$^2$|\n\n</div>\n\n<div class=\"aligned\">\n\n|||\n|-|-|\n|Side length|= $\\sqrt{25}$|\n||= 5 cm|\n\n</div>\n\n<br>\n\n<div class=\"aligned\">\n\n|||\n|-|-|\n|$\\therefore$ Volume|= $5^3$|\n||= {{{correctAnswer0}}} {{{suffix0}}}|\n\n</div>"}]},{"vars":[{"varval":"A cube has a surface area of 294 square centimetres.\n\n\r\n\r\nWhat is the volume of the cube?"},{"varval":"sm_nogap Cube has 6 faces.\n\n<div class=\"aligned\">\n\n|||\n|-|-|\n|Area of 1 face|= 294 ÷ 6|\n||= 49 cm$^2$|\n\n</div>\n\n<div class=\"aligned\">\n\n|||\n|-|-|\n|Side length|= $\\sqrt{49}$|\n||= 7 cm|\n\n</div>\n\n<br>\n\n<div class=\"aligned\">\n\n|||\n|-|-|\n|$\\therefore$ Volume|= $7^3$|\n||= {{{correctAnswer0}}} {{{suffix0}}}|\n\n</div>"}]},{"vars":[{"varval":"A cube has a surface area of 600 square centimetres.\n\n\r\n\r\nWhat is the volume of the cube?"},{"varval":"sm_nogap Cube has 6 faces.\n\n<div class=\"aligned\">\n\n|||\n|-|-|\n|Area of 1 face|= 600 ÷ 6|\n||= 100 cm$^2$|\n\n</div>\n\n<div class=\"aligned\">\n\n|||\n|-|-|\n|Side length|= $\\sqrt{100}$|\n||= 10 cm|\n\n</div>\n\n<br>\n\n<div class=\"aligned\">\n\n|||\n|-|-|\n|$\\therefore$ Volume|= $10^3$|\n||= {{{correctAnswer0}}} {{{suffix0}}}|\n\n</div>"}]},{"vars":[{"varval":"A cube has a surface area of 384 square centimetres.\n\n\r\n\r\nWhat is the volume of the cube?"},{"varval":"sm_nogap Cube has 6 faces.\n\n<div class=\"aligned\">\n\n|||\n|-|-|\n|Area of 1 face|= 384 ÷ 6|\n||= 64 cm$^2$|\n\n</div>\n\n<div class=\"aligned\">\n\n|||\n|-|-|\n|Side length|= $\\sqrt{64}$|\n||= 8 cm|\n\n</div>\n\n<br>\n\n<div class=\"aligned\">\n\n|||\n|-|-|\n|$\\therefore$ Volume|= $8^3$|\n||= {{{correctAnswer0}}} {{{suffix0}}}|\n\n</div>"}]},{"vars":[{"varval":"A cube has a surface area of 96 square centimetres.\n\n\r\n\r\nWhat is the volume of the cube?"},{"varval":"sm_nogap Cube has 6 faces.\n\n<div class=\"aligned\">\n\n|||\n|-|-|\n|Area of 1 face|= 96 ÷ 6|\n||= 16 cm$^2$|\n\n</div>\n\n<div class=\"aligned\">\n\n|||\n|-|-|\n|Side length|= $\\sqrt{16}$|\n||= 4 cm|\n\n</div>\n\n<br>\n\n<div class=\"aligned\">\n\n|||\n|-|-|\n|$\\therefore$ Volume|= $4^3$|\n||= {{{correctAnswer0}}} {{{suffix0}}}|\n\n</div>"}]},{"vars":[{"varval":"A cube has a surface area of 54 square centimetres.\n\n\r\n\r\nWhat is the volume of the cube?"},{"varval":"sm_nogap Cube has 6 faces.\n\n<div class=\"aligned\">\n\n|||\n|-|-|\n|Area of 1 face|= 54 ÷ 6|\n||= 9 cm$^2$|\n\n</div>\n\n<div class=\"aligned\">\n\n|||\n|-|-|\n|Side length|= $\\sqrt{9}$|\n||= 3 cm|\n\n</div>\n\n<br>\n\n<div class=\"aligned\">\n\n|||\n|-|-|\n|$\\therefore$ Volume|= $3^3$|\n||= {{{correctAnswer0}}} {{{suffix0}}}|\n\n</div>"}]}]

  530. <div class="sm_mode"> {{{question}}} </div>

    [{"vars":[{"varval":"\nAn archery target has the following dimensions.\n\n<br>\n\nsm_img https://teacher.smartermaths.com.au/wp-content/uploads/2018/04/NAPX-I4-CA29.svg 420 indent3 vpad\n\n<br>What is the area of the entire target?\n\nUse &nbsp;$\\large \\pi$ = 3.14 &nbsp;and round your answer to the nearest square centimetre."},{"varval":"<div class=\"aligned\">\n\n|||\n|-|-|\n|Area|= $\\large \\pi r$$^2$|\n||= $\\large \\pi$ $\\times\\ (4 + 3.5 \\times 4)^2$|\n||= 3.14 $\\times\\ 18^2$|\n||= 1017.36...|\n||= {{{correctAnswer0}}} {{{suffix0}}}|\n\n</div>"}]},{"vars":[{"varval":"\nA small plate has the following dimensions.\n\n<br>\n\nsm_img https://teacher.smartermaths.com.au/wp-content/uploads/2022/08/Measurement-NAPX-I4-CA29-SA_1nts.svg 500 indent1 vpad\n\n<br>What is the area of the entire plate?\n\nUse &nbsp;$\\large \\pi$ = 3.14 &nbsp;and round your answer to the nearest square centimetre."},{"varval":"<div class=\"aligned\">\n\n|||\n|-|-|\n|Area|= $\\large \\pi r$$^2$|\n||= $\\large \\pi$ $\\times\\ (3.5 + 3 + 4 + 5)^2$|\n||= 3.14 $\\times\\ 15.5^2$|\n||= 754.385|\n||= {{{correctAnswer0}}} {{{suffix0}}}|\n\n</div>"}]},{"vars":[{"varval":"The base of a cat basket has the following dimensions.\n\n<br>\n\nsm_img https://teacher.smartermaths.com.au/wp-content/uploads/2022/08/Measurement-NAPX-I4-CA29-SA_2nts.svg 500 indent1 vpad\n\n<br>What is the area of the entire base?\n\nUse &nbsp;$\\large \\pi$ = 3.14 &nbsp;and round your answer to the nearest square centimetre."},{"varval":"<div class=\"aligned\">\n\n|||\n|-|-|\n|Area|= $\\large \\pi r$$^2$|\n||= $\\large \\pi$ $\\times\\ (10 + 8 \\times 4)^2$|\n||= 3.14 $\\times\\ 42^2$|\n||= 5538.96|\n||= {{{correctAnswer0}}} {{{suffix0}}}|\n\n</div>"}]},{"vars":[{"varval":"A placemat has the following dimensions.\n\n<br>\n\nsm_img https://teacher.smartermaths.com.au/wp-content/uploads/2022/08/Measurement-NAPX-I4-CA29-SA_3nts.svg 500 indent1 vpad\n\n<br>What is the area of the entire placemat?\n\nUse &nbsp;$\\large \\pi$ = 3.14 &nbsp;and round your answer to the nearest square centimetre."},{"varval":"<div class=\"aligned\">\n\n|||\n|-|-|\n|Area|= $\\large \\pi r$$^2$|\n||= $\\large \\pi$ $\\times\\ (7.5 + 6.5 \\times 4)^2$|\n||= 3.14 $\\times\\ 33.5^2$|\n||= 3523.865|\n||= {{{correctAnswer0}}} {{{suffix0}}}|\n\n</div>"}]},{"vars":[{"varval":"A large circular sheet has the following dimensions.\n\n<br>\n\nsm_img https://teacher.smartermaths.com.au/wp-content/uploads/2022/08/Measurement-NAPX-I4-CA29-SA_4nts.svg 500 indent1 vpad\n\n<br>What is the area of the entire sheet?\n\nUse &nbsp;$\\large \\pi$ = 3.14 &nbsp;and round your answer to the nearest square metre."},{"varval":"<div class=\"aligned\">\n\n|||\n|-|-|\n|Area|= $\\large \\pi r$$^2$|\n||= $\\large \\pi$ $\\times\\ (0.75 + 0.8 \\times 4)^2$|\n||= 3.14 $\\times\\ 3.95^2$|\n||= 48.99185|\n||= {{{correctAnswer0}}} {{{suffix0}}}|\n\n</div>"}]},{"vars":[{"varval":"A baby's play mat has the following dimensions.\n\n<br>\n\nsm_img https://teacher.smartermaths.com.au/wp-content/uploads/2022/08/Measurement-NAPX-I4-CA29-SA_5nts.svg 500 indent1 vpad\n\n<br>What is the area of the entire play mat?\n\nUse &nbsp;$\\large \\pi$ = 3.14 &nbsp;and round your answer to the nearest tenth of a square metre."},{"varval":"Radius of inner circle = 25 cm = 0.25 m\n\n<div class=\"aligned\">\n\n|||\n|-|-|\n|Area|= $\\large \\pi r$$^2$|\n||= $\\large \\pi$ $\\times\\ (0.25 + 0.2 \\times 4)^2$|\n||= 3.14 $\\times\\ 1.05^2$|\n||= 3.46185|\n||= {{{correctAnswer0}}} {{{suffix0}}}|\n\n</div>"}]}]

  531. <div class="sm_mode"> {{{question}}} </div>

    [{"vars":[{"varval":"The edges of a $\\ 4 × 4×4$ &nbsp;cube are formed using blocks as shown below:\n\n<br>\n\nsm_img https://teacher.smartermaths.com.au/wp-content/uploads/2016/12/NAP-G4-NC281-259x300.png 200 indent3 vpad\n\n<br>In total, 32 blocks are used.\n\n\r\n\r\nHow many blocks would be needed to make the edges of a $\\ 6 × 6 × 6$ &nbsp;cube in the same way?\n"},{"varval":"Blocks in top layer = 20\n\n\r\nBlocks in bottom layer = 20\n\n\r\nOther blocks (pillars) = 4 × 4 = 16\n\n\r\n$\\therefore$ Total blocks = {{{correctAnswer0}}}\n"}]},{"vars":[{"varval":"The edges of a $\\ 3 × 3 × 3$ &nbsp;cube are formed using blocks as shown below:\n\n<br>\n\nsm_img https://teacher.smartermaths.com.au/wp-content/uploads/2022/08/Measurement_NAPX-G4-NC28-SA_1.svg 200 indent3 vpad\n\n<br>In total, 20 blocks are used.\n\n\r\n\r\nHow many blocks would be needed to make the edges of a $\\ 4 × 4 × 4$ &nbsp;cube in the same way?\n"},{"varval":"Blocks in top layer = 12\n\n\r\nBlocks in bottom layer = 12\n\n\r\nOther blocks (pillars) = 4 × 2 = 8\n\n\r\n$\\therefore$ Total blocks = {{{correctAnswer0}}}\n"}]},{"vars":[{"varval":"The edges of a $\\ 5 × 5 × 5$ &nbsp;cube are formed using blocks as shown below:\n\n<br>\n\nsm_img https://teacher.smartermaths.com.au/wp-content/uploads/2022/08/Measurement_NAPX-G4-NC28-SA_2-1.svg 200 indent3 vpad\n\n<br>In total, 44 blocks are used.\n\n\r\n\r\nHow many blocks would be needed to make the edges of a $\\ 7 × 7 × 7$ &nbsp;cube in the same way?\n"},{"varval":"Blocks in top layer = 24\n\n\r\nBlocks in bottom layer = 24\n\n\r\nOther blocks (pillars) = 4 × 5 = 20\n\n\r\n$\\therefore$ Total blocks = {{{correctAnswer0}}}\n"}]},{"vars":[{"varval":"The edges of a $\\ 6 × 6 × 6$ &nbsp;cube are formed using blocks as shown below:\n\n<br>\n\nsm_img https://teacher.smartermaths.com.au/wp-content/uploads/2022/08/Measurement_NAPX-G4-NC28-SA_3.svg 200 indent3 vpad\n\n<br>In total, 56 blocks are used.\n\n\r\n\r\nHow many blocks would be needed to make the edges of a $\\ 8 × 8 × 8$ &nbsp;cube in the same way?\n"},{"varval":"Blocks in top layer = 28\n\n\r\nBlocks in bottom layer = 28\n\n\r\nOther blocks (pillars) = 4 × 6 = 24\n\n\r\n$\\therefore$ Total blocks = {{{correctAnswer0}}}\n"}]},{"vars":[{"varval":"The edges of a $\\ 7 × 7 × 7$ &nbsp;cube are formed using blocks as shown below:\n\n<br>\n\nsm_img https://teacher.smartermaths.com.au/wp-content/uploads/2022/08/Measurement_NAPX-G4-NC28-SA_4.svg 200 indent3 vpad\n\n<br>In total, 68 blocks are used.\n\n\r\n\r\nHow many blocks would be needed to make the edges of a $\\ 5 × 5 × 5$ &nbsp;cube in the same way?\n"},{"varval":"Blocks in top layer = 16\n\n\r\nBlocks in bottom layer = 16\n\n\r\nOther blocks (pillars) = 4 × 3 = 12\n\n\r\n$\\therefore$ Total blocks = {{{correctAnswer0}}}\n"}]},{"vars":[{"varval":"The edges of a $\\ 8 × 8 × 8$ &nbsp;cube are formed using blocks as shown below:\n\n<br>\n\nsm_img https://teacher.smartermaths.com.au/wp-content/uploads/2022/08/Measurement_NAPX-G4-NC28-SA_5.svg 200 indent3 vpad\n\n<br>In total, 80 blocks are used.\n\n\r\n\r\nHow many blocks would be needed to make the edges of a $\\ 10 × 10 × 10$ &nbsp;cube in the same way?\n"},{"varval":"Blocks in top layer = 36\n\n\r\nBlocks in bottom layer = 36\n\n\r\nOther blocks (pillars) = 4 × 8 = 32\n\n\r\n$\\therefore$ Total blocks = {{{correctAnswer0}}}\n"}]}]

  532. var2 and var 3 ... h in image should be in italics - fixed var4 and var5 ... w should replace "h" for width in the question image (then the last worked solution paragraph should read "w = " changed to length and "l"

    <div class="sm_mode"> {{{question}}} </div>

    [{"vars":[{"varval":"Three identical cubes are placed on top of each other to form a rectangular prism as shown in the diagram below.\n\n<br>\n\nsm_img https://teacher.smartermaths.com.au/wp-content/uploads/2018/06/NAPX-F4-CA31-SA.svg 170 indent3 vpad\n\n<br>If the prism has a volume of 1029 cubic centimetres, what it its height?"},{"varval":"sm_nogap Let dimensions of one cube = $\\large a$ × $\\large a$ × $\\large a$\n\n<div class=\"aligned\">\n\n|||\n|-:|-|\n|3$\\large a$$^3$|= 1029|\n|$\\large a$$^3$|= 343|\n|$\\large a$|= $\\sqrt[3]{343}$|\n||= 7 cm|\n\n</div>\n\n<br>\n\n<div class=\"aligned\">\n\n|||\n|-|-|\n|$\\therefore \\large h$|= 3 × 7 |\n| |= {{{correctAnswer0}}} {{{suffix0}}}|\n\n</div>"}]},{"vars":[{"varval":"Three identical cubes are placed on top of each other to form a rectangular prism as shown in the diagram below.\n\n<br>\n\nsm_img https://teacher.smartermaths.com.au/wp-content/uploads/2022/08/Measurement_NAPX-F4-CA31-SA_1.svg 170 indent3 vpad\n\n<br>If the prism has a volume of 5184 cubic centimetres, what it its height?"},{"varval":"sm_nogap Let dimensions of one cube = $\\large a$ × $\\large a$ × $\\large a$\n\n<div class=\"aligned\">\n\n|||\n|-:|-|\n|3$\\large a$$^3$|= 5184|\n|$\\large a$$^3$|= 1728|\n|$\\large a$|= $\\sqrt[3]{1728}$|\n||= 12 cm|\n\n</div>\n\n<br>\n\n<div class=\"aligned\">\n\n|||\n|-|-|\n|$\\therefore \\large h$|= 3 × 12 |\n| |= {{{correctAnswer0}}} {{{suffix0}}}|\n\n</div>"}]},{"vars":[{"varval":"Four identical cubes are placed on top of each other to form a rectangular prism as shown in the diagram below.\n\n<br>\n\nsm_img https://teacher.smartermaths.com.au/wp-content/uploads/2022/08/Measurement-NAPX-F4-CA31-SA_v2.svg 170 indent3 vpad\n\n<br>If the prism has a volume of 500 cubic centimetres, what it its height?"},{"varval":"sm_nogap Let dimensions of one cube = $\\large a$ × $\\large a$ × $\\large a$\n\n<div class=\"aligned\">\n\n|||\n|-:|-|\n|4$\\large a$$^3$|= 500|\n|$\\large a$$^3$|= 125|\n|$\\large a$|= $\\sqrt[3]{125}$|\n||= 5 cm|\n\n</div>\n\n<br>\n\n<div class=\"aligned\">\n\n|||\n|-|-|\n|$\\therefore \\large h$|= 4 × 5 |\n| |= {{{correctAnswer0}}} {{{suffix0}}}|\n\n</div>"}]},{"vars":[{"varval":"Four identical cubes are placed on top of each other to form a rectangular prism as shown in the diagram below.\n\n<br>\n\nsm_img https://teacher.smartermaths.com.au/wp-content/uploads/2022/08/Measurement-NAPX-F4-CA31-SA_v3.svg 170 indent3 vpad\n\n<br>If the prism has a volume of 2048 cubic centimetres, what it its height?"},{"varval":"sm_nogap Let dimensions of one cube = $\\large a$ × $\\large a$ × $\\large a$\n\n<div class=\"aligned\">\n\n|||\n|-:|-|\n|4$\\large a$$^3$|= 2048|\n|$\\large a$$^3$|= 512|\n|$\\large a$|= $\\sqrt[3]{512}$|\n||= 8 cm|\n\n</div>\n\n<br>\n\n<div class=\"aligned\">\n\n|||\n|-|-|\n|$\\therefore \\large h$|= 4 × 8 |\n| |= {{{correctAnswer0}}} {{{suffix0}}}|\n\n</div>"}]},{"vars":[{"varval":"Five identical cubes are placed side by side to form a rectangular prism as shown in the diagram below.\n\n<br>\n\nsm_img https://teacher.smartermaths.com.au/wp-content/uploads/2022/08/Measurement-NAPX-F4-CA31-SA_v4.svg 400 indent3 vpad\n\n<br>If the prism has a volume of 6655 cubic centimetres, what it its length?"},{"varval":"sm_nogap Let dimensions of one cube = $\\large a$ × $\\large a$ × $\\large a$\n\n<div class=\"aligned\">\n\n|||\n|-:|-|\n|5$\\large a$$^3$|= 6655|\n|$\\large a$$^3$|= 1331|\n|$\\large a$|= $\\sqrt[3]{1331}$|\n||= 11 cm|\n\n</div>\n\n<br>\n\n<div class=\"aligned\">\n\n|||\n|-|-|\n|$\\therefore \\large l$|= 5 × 11 |\n| |= {{{correctAnswer0}}} {{{suffix0}}}|\n\n</div>"}]},{"vars":[{"varval":"Six identical cubes are placed side by side to form a rectangular prism as shown in the diagram below.\n\n<br>\n\nsm_img https://teacher.smartermaths.com.au/wp-content/uploads/2022/08/Measurement-NAPX-F4-CA31-SA_v5.svg 400 indent3 vpad\n\n<br>If the prism has a volume of 4374 cubic centimetres, what it its length?"},{"varval":"sm_nogap Let dimensions of one cube = $\\large a$ × $\\large a$ × $\\large a$ \n\n<div class=\"aligned\">\n\n|||\n|-:|-|\n|6$\\large a$$^3$|= 4374|\n|$\\large a$$^3$|= 729|\n|$\\large a$|= $\\sqrt[3]{729}$|\n||= 9 cm|\n\n</div>\n\n<br>\n\n<div class=\"aligned\">\n\n|||\n|-|-|\n|$\\therefore \\large l$|= 6 × 9 |\n| |= {{{correctAnswer0}}} {{{suffix0}}}|\n\n</div>"}]}]

  533. <div class="sm_mode"> {{{question}}} </div>

    [{"vars":[{"varval":"\nA museum has a glass entrance that is made up of four equal triangles and a square, as shown in the diagram below.\n\n<br>\n\nsm_img https://teacher.smartermaths.com.au/wp-content/uploads/2018/04/NAPX-I4-CA272.svg 440 indent vpad\n\n<br>Belou is hired to clean the interior sides of the entrance, not including the floor.\n\n\r\n\r\nWhat is the area that Belou will need to clean? Answer to the nearest square metre.\n"},{"varval":"sm_nogap Area to clean\n\n<div class=\"aligned\">\n\n>>||\n|-|\n|= 4 × $\\dfrac{1}{2}\\large bh$ |\n|= 4 × $\\dfrac{1}{2}$ × 18 × 17.7|\n|= 637.2 m²|\n|= {{{correctAnswer0}}} {{{suffix0}}} (nearest m²)|\n\r\n\n</div>"}]},{"vars":[{"varval":"A light fitting is made up of four equal triangles and a square, as shown in the diagram below\n\n<br>\n\nsm_img https://teacher.smartermaths.com.au/wp-content/uploads/2022/08/Measurement_NAPX-I4-CA27-SA_1.svg 450 indent vpad\n\n<br>Chrissy is spray painting the outside of the light fitting, not including the base.\n\n\r\n\r\nWhat is the area that Chrissy will need to paint?\n"},{"varval":"sm_nogap Area to paint\n\n<div class=\"aligned\">\n\n>>||\n|-|\n|= 4 × $\\dfrac{1}{2}\\large bh$ |\n|= 4 × $\\dfrac{1}{2}$ × 25 × 46|\n|= {{{correctAnswer0}}} {{{suffix0}}} |\n\r\n\n</div>"}]},{"vars":[{"varval":"A large glamping tent is made up of four equal triangles and a square, as shown in the diagram below\n\n<br>\n\nsm_img https://teacher.smartermaths.com.au/wp-content/uploads/2022/08/Measurement_NAPX-I4-CA27-SA_5.svg 330 indent vpad\n\n<br>Bruce is covering the outside of the tent with waterproofing spray, not including the base.\n\n\r\n\r\nWhat is the area that Bruce will need to spray? Answer to the nearest square metre.\n\n"},{"varval":"sm_nogap Area to spray\n\n<div class=\"aligned\">\n\n>>||\n|-|\n|= 4 × $\\dfrac{1}{2}\\large bh$ |\n|= 4 × $\\dfrac{1}{2}$ × 4.5 × 3.6|\n|= 32.4 m²|\n|= {{{correctAnswer0}}} {{{suffix0}}} (nearest m²)|\n</div>"}]},{"vars":[{"varval":"A two person tent is made up of four equal triangles and a square, as shown in the diagram below\n\n<br>\n\nsm_img https://teacher.smartermaths.com.au/wp-content/uploads/2022/08/Measurement_NAPX-I4-CA27-SA_4.svg 330 indent vpad\n\n<br>Jennifer is covering the outside of the tent with mosquito netting, not including the base.\n\n\r\n\r\nWhat is the area that Jennifer will need to cover? Answer to the nearest square metre.\n"},{"varval":"sm_nogap Area to cover\n\n<div class=\"aligned\">\n\n>>||\n|-|\n|= 4 × $\\dfrac{1}{2}\\large bh$ |\n|= 4 × $\\dfrac{1}{2}$ × 2.6 × 2.1|\n|= 10.92 m²|\n|= {{{correctAnswer0}}} {{{suffix0}}} (nearest m²)|\n</div>"}]},{"vars":[{"varval":"A Christmas decoration is made up of four equal triangles and a square, as shown in the diagram below\n\n<br>\n\nsm_img https://teacher.smartermaths.com.au/wp-content/uploads/2022/08/Measurement_NAPX-I4-CA27-SA_3.svg 350 indent vpad\n\n<br>Klaus is covering the outside of the decoration with glitter spray, not including the base.\n\n\r\n\r\nWhat is the area that Klaus will need to cover? Answer to the nearest square centimetre."},{"varval":"sm_nogap Area to cover\n\n<div class=\"aligned\">\n\n>>||\n|-|\n|= 4 × $\\dfrac{1}{2}\\large bh$ |\n|= 4 × $\\dfrac{1}{2}$ × 5.6 × 9.1|\n|= 101.92 cm²|\n|= {{{correctAnswer0}}} {{{suffix0}}} (nearest cm²)|\n</div>"}]},{"vars":[{"varval":"A small ancient pyramid has been excavated. The pyramid is made up of four equal triangles and a square, as shown in the diagram below\n\n<br>\n\nsm_img https://teacher.smartermaths.com.au/wp-content/uploads/2022/08/Measurement_NAPX-I4-CA27-SA_2.svg 300 indent vpad\n\n<br>Della is calculating the outside surface area of the pyramid, not including the base.\n\n\r\n\r\nWhat is the area that Della will calculate? Answer to the nearest square metre."},{"varval":"sm_nogap Area to calculate\n\n<div class=\"aligned\">\n\n>>||\n|-|\n|= 4 × $\\dfrac{1}{2}\\large bh$ |\n|= 4 × $\\dfrac{1}{2}$ × 15.2 × 12.7|\n|= 386.08 m²|\n|= {{{correctAnswer0}}} {{{suffix0}}} (nearest m²)|\n</div>"}]}]

  534. <div class="sm_mode"> {{{question}}} </div>

    [{"vars":[{"varval":"A dog bath is in the shape of a rectangular prism.\r\n\r\nIts measurements can be seen below:\n\n<br>\n\nsm_img https://teacher.smartermaths.com.au/wp-content/uploads/2018/05/NAPX-H4-NC29-SA.svg 280 indent3 vpad\n\n<br>One cubic metre holds 1000 litres of water.\n\n\r\n\r\nHow many litres of water will fill the dog bath?\n"},{"varval":"sm_nogap Volume of dog bath\n\n<div class=\"aligned\">\n\n>>||\n|-|\n|= 4 × 1.5 × 0.75|\n|= 4.5 m$^3$|\n|= {{{correctAnswer0}}} {{{suffix0}}}|\n\r\n\r\n\n</div>"}]},{"vars":[{"varval":"A water trough is in the shape of a rectangular prism.\r\n\r\nIts measurements can be seen below:\n\n<br>\n\nsm_img https://teacher.smartermaths.com.au/wp-content/uploads/2022/08/Measurement_NAPX-H4-NC29-SA_v1.svg 350 indent3 vpad\n\n<br>One cubic metre holds 1000 litres of water.\n\n\r\n\r\nHow many litres of water will it take to fill the water trough?\n"},{"varval":"sm_nogap Volume of water trough\n\n<div class=\"aligned\">\n\n>>||\n|-|\n|= 2.4 × 0.9 × 1.1|\n|= 2.376 m$^3$|\n|= {{{correctAnswer0}}} {{{suffix0}}}|\n\r\n\r\n\n</div>"}]},{"vars":[{"varval":"A shipping container is in the shape of a rectangular prism is to be converted into a swimming pool.\n\n\r\n\r\nIts measurements can be seen below:\n\n\n\nsm_img https://teacher.smartermaths.com.au/wp-content/uploads/2022/08/Measurement_NAPX-H4-NC29-SA_v2a.svg 240 indent3 vpad\n\n<br>One cubic metre holds 1000 litres of water.\n\nHow many litres of water will it take to fill the converted shipping container?\n"},{"varval":"sm_nogap Volume of converted shipping container\n\n<div class=\"aligned\">\n\n>>||\n|-|\n|= 9.2 × 2.6 × 2.4|\n|= 56.784 m$^3$|\n|= 56 784 litres|\n\r\n\r\n\n</div>"}]},{"vars":[{"varval":"A fish tank is in the shape of a rectangular prism.\n\n\r\n\r\nIts measurements can be seen below:\n\n<br>\n\nsm_img https://teacher.smartermaths.com.au/wp-content/uploads/2022/08/Measurement_NAPX-H4-NC29-SA_v3.svg 340 indent3 vpad\n\n<br>One cubic metre holds 1000 litres of water.\n\nHow many litres of water will it take to fill the fish tank?\n"},{"varval":"sm_nogap Volume of fish tank\n\n<div class=\"aligned\">\n\n>>||\n|-|\n|= 0.6 × 0.8 × 0.6|\n|= 0.288 m$^3$|\n|= {{{correctAnswer0}}} {{{suffix0}}}|\n\r\n\r\n\n</div>"}]},{"vars":[{"varval":"A kitchen sink is in the shape of a rectangular prism.\n\n\r\n\r\nIts measurements can be seen below:\n\n<br>\n\nsm_img https://teacher.smartermaths.com.au/wp-content/uploads/2022/08/Measurement_NAPX-H4-NC29-SA-v4.svg 300 indent3 vpad\n\n<br>One cubic metre holds 1000 litres of water.\n\nHow many litres of water, to the nearest litre, will it take to fill the kitchen sink?\n"},{"varval":"sm_nogap Volume of kitchen sink\n\n<div class=\"aligned\">\n\n>>||\n|-|\n|= 0.24 × 0.36 × 0.44|\n|= 0.038016 m$^3$|\n|= 38.016 litres|\n|= {{{correctAnswer0}}} {{{suffix0}}}|\n\r\n\r\n\n</div>"}]},{"vars":[{"varval":"A water tank is in the shape of a rectangular prism.\n\n\r\n\r\nIts measurements can be seen below:\n\n<br>\n\nsm_img https://teacher.smartermaths.com.au/wp-content/uploads/2022/08/Measurement_NAPX-H4-NC29-SA-v5.svg 400 indent vpad\n\n<br>One cubic metre holds 1000 litres of water.\n\nHow many litres of water will it take to fill the water tank?\n"},{"varval":"sm_nogap Volume of water tank\n\n<div class=\"aligned\">\n\n>>||\n|-|\n|= 2.1 × 4 × 0.8|\n|= 6.72 m$^3$|\n|= {{{correctAnswer0}}} {{{suffix0}}}|\n\r\n\r\n\n</div>"}]}]

  535. <div class="sm_mode"> {{{question}}} </div>

    [{"vars":[{"varval":"Minerva is tiling her back deck which is rectangular and measures 3.6 m × 4.2 m.\n\r\nShe decides to use the square tiles shown below.\n\n<br>\n\nsm_img https://teacher.smartermaths.com.au/wp-content/uploads/2018/05/NAPX-G4-CA27-SA.svg 290 indent3 vpad\n\n<br>How many boxes of these square tiles does Minerva need to order?\n"},{"varval":"sm_nogap Area of back deck in the number of tiles\n\n<div class=\"aligned\">\n\n>>||\n|-|\n|= $\\dfrac{3.6}{0.3} \\times \\dfrac{4.2}{0.3}$|\n|= 12 tiles $\\times$ 14 tiles|\n|= 168 tiles|\n\n</div>\n\n<br>\n\nsm_nogap $\\therefore$ Number of boxes to order\n\n<div class=\"aligned\">\n\n>>||\n|-|\n|= $\\dfrac{168}{8}$|\n|= {{{correctAnswer0}}} {{{suffix0}}}|\n\n</div>\n"}]},{"vars":[{"varval":"Jake is tiling his bbq area which is rectangular and measures 4.0 m × 6.4 m.\n\r\nHe decides to use the square tiles shown below.\n\n<br>\n\nsm_img https://teacher.smartermaths.com.au/wp-content/uploads/2022/08/Measurement_NAPX-G4-CA27-SA_NAPX-G3-CA30-SA_1b.svg 330 indent3 vpad\n\n<br>How many boxes of these square tiles does Jake need to order?\n"},{"varval":"sm_nogap Area of bbq area in the number of tiles\n\n<div class=\"aligned\">\n\n>>||\n|-|\n|= $\\dfrac{4.0}{0.4} \\times \\dfrac{6.4}{0.4}$|\n|= 10 tiles $\\times$ 16 tiles|\n|= 160 tiles|\n\n</div>\n\n<br>\n\nsm_nogap $\\therefore$ Number of boxes to order\n\n<div class=\"aligned\">\n\n>>||\n|-|\n|= $\\dfrac{160}{10}$|\n|= {{{correctAnswer0}}} {{{suffix0}}}|\n\n</div>\n"}]},{"vars":[{"varval":"Sienna is tiling the splashback in her new kitchen which is rectangular and measures 0.6 m × 2.4 m.\n\r\nShe decides to use the square tiles shown below.\n\n<br>\n\nsm_img https://teacher.smartermaths.com.au/wp-content/uploads/2022/08/Measurement_NAPX-G4-CA27-SA_NAPX-G3-CA30-SA_2_1.svg 360 indent3 vpad\n\n<br>How many boxes of these square tiles does Sienna need to order?"},{"varval":"sm_nogap Area of splashback in the number of tiles\n\n<div class=\"aligned\">\n\n>>||\n|-|\n|= $\\dfrac{0.6}{0.1} \\times \\dfrac{2.4}{0.1}$|\n|= 6 tiles $\\times$ 24 tiles|\n|= 144 tiles|\n\n</div>\n\n<br>\n\nsm_nogap $\\therefore$ Number of boxes to order\n\n<div class=\"aligned\">\n\n>>||\n|-|\n|= $\\dfrac{144}{36}$|\n|= {{{correctAnswer0}}} {{{suffix0}}}|\n\n</div>\n"}]},{"vars":[{"varval":"Bryson is tiling his patio which is rectangular and measures 6.3 m × 3.0 m.\n\r\nHe decides to use the square tiles shown below.\n\n<br>\n\nsm_img https://teacher.smartermaths.com.au/wp-content/uploads/2022/08/Measurement_NAPX-G4-CA27-SA_NAPX-G3-CA30-SA_3_a.svg 360 indent3 vpad\n\n<br>How many boxes of these square tiles does Bryson need to order?"},{"varval":"sm_nogap Area of patio in the number of tiles\n\n<div class=\"aligned\">\n\n>>||\n|-|\n|= $\\dfrac{6.0}{0.15} \\times \\dfrac{3.0}{0.15}$|\n|= 42 tiles $\\times$ 20 tiles|\n|= 840 tiles|\n\n</div>\n\n<br>\n\nsm_nogap $\\therefore$ Number of boxes to order\n\n<div class=\"aligned\">\n\n>>||\n|-|\n|= $\\dfrac{840}{20}$|\n|= {{{correctAnswer0}}} {{{suffix0}}}|\n\n</div>\n"}]},{"vars":[{"varval":"Donna is tiling one wall in her butler's pantry which is rectangular and measu 4.5 m × 1.2 m.\n\r\nShe decides to use the square tiles shown below.\n\n<br>\n\nsm_img https://teacher.smartermaths.com.au/wp-content/uploads/2022/08/Measurement_NAPX-G4-CA27-SA_NAPX-G3-CA30-SA_2_a.svg 360 indent3 vpad\n\n<br>How many boxes of these square tiles does Donna need to order?"},{"varval":"sm_nogap Area of walls in the number of tiles\n\n<div class=\"aligned\">\n\n>>||\n|-|\n|= $\\dfrac{4.5}{0.10} \\times \\dfrac{1.2}{0.10}$|\n|= 45 tiles $\\times$ 12 tiles|\n|= 540 tiles|\n\n</div>\n\n<br>\n\nsm_nogap $\\therefore$ Number of boxes to order\n\n<div class=\"aligned\">\n\n>>||\n|-|\n|= $\\dfrac{540}{90}$|\n|= {{{correctAnswer0}}} {{{suffix0}}}|\n\n</div>"}]},{"vars":[{"varval":"Matt is using rubber tiles to tile a playground floor which is rectangular and measures 13.2 m × 9.0 m.\n\r\nHe decides to use the square tiles shown below.\n\n<br>\n\nsm_img https://teacher.smartermaths.com.au/wp-content/uploads/2022/08/Measurement_NAPX-G4-CA27-SA_NAPX-G3-CA30-SA_4_a.svg 360 indent3 vpad\n\n<br>How many boxes of these square tiles does Matt need to order?"},{"varval":"sm_nogap Area of playground floor in the number of tiles\n\n<div class=\"aligned\">\n\n>>||\n|-|\n|= $\\dfrac{13.2}{0.60} \\times \\dfrac{9.0}{0.60}$|\n|= 22 tiles $\\times$ 15 tiles|\n|= 330 tiles|\n\n</div>\n\n<br>\n\nsm_nogap $\\therefore$ Number of boxes to order\n\n<div class=\"aligned\">\n\n>>||\n|-|\n|= $\\dfrac{330}{10}$|\n|= {{{correctAnswer0}}} {{{suffix0}}}|\n\n</div>"}]}]

  536. <div class="sm_mode"> {{{question}}} </div>

    [{"vars":[{"varval":"A wheelchair ramp is pictured below.\n\n<br>\n\nsm_img https://teacher.smartermaths.com.au/wp-content/uploads/2018/06/NAPX-F4-NC29.svg 360 indent3 vpad\n\n<br>The ramp is in the shape of a triangular prism?\n\nWhat is the volume of the ramp?"},{"varval":"<div class=\"aligned\">\n\n|||\n|-|-|\n|Volume| = $A\\large h$|\n||= $\\bigg( \\dfrac{1}{2} \\times 7 \\times 0.4 \\bigg) \\times 2$|\n||= 1.4 $\\times$ 2|\n||= {{{correctAnswer}}}|\n\n</div>"}]},{"vars":[{"varval":"A boat ramp is pictured below.\n\n<br>\n\nsm_img https://teacher.smartermaths.com.au/wp-content/uploads/2022/08/Measurement_NAPX-F4-NC29_v1.svg 550 vpad\n\n<br>The ramp is in the shape of a triangular prism?\n\nWhat is the volume of the ramp?"},{"varval":"<div class=\"aligned\">\n\n|||\n|-|-|\n|Volume| = $A\\large h$|\n||= $\\bigg( \\dfrac{1}{2} \\times 10 \\times 0.3 \\bigg) \\times 5$|\n||= 1.5 $\\times$ 5|\n||= {{{correctAnswer}}}|\n\n</div>"}]},{"vars":[{"varval":"A large access ramp is pictured below.\n\n<br>\n\nsm_img https://teacher.smartermaths.com.au/wp-content/uploads/2022/08/Measurement_NAPX-F4-NC29_v2.svg 350 vpad\n\n<br>The ramp is in the shape of a triangular prism?\n\nWhat is the volume of the ramp?"},{"varval":"<div class=\"aligned\">\n\n|||\n|-|-|\n|Volume| = $A\\large h$|\n||= $\\bigg( \\dfrac{1}{2} \\times 3 \\times 0.7 \\bigg) \\times 1.5$|\n||= 1.05 $\\times$ 1.5|\n||= {{{correctAnswer}}}|\n\n</div>"}]},{"vars":[{"varval":"An timber sleeper in the shape of a rectangular prism is to be cut into to 2 equal triangular prisms as pictured below.\n\n\nsm_img https://teacher.smartermaths.com.au/wp-content/uploads/2022/08/Measurement_NAPX-F4-NC29_v3_a.svg 450 indent1 vpad\n\n<br>\n\nWhat is the volume in cubic centimetres, of one of the triangular prism shaped pieces of timber?\n"},{"varval":"Using &nbsp;1 m = 100 cm\n\n<div class=\"aligned\">\n\n|||\n|-|-|\n|Volume| = $A\\large h$|\n||= $\\bigg( \\dfrac{1}{2} \\times 100 \\times 10 \\bigg) \\times 20$|\n||= 500 $\\times$ 20|\n||= {{{correctAnswer}}}|\n\n</div>"}]},{"vars":[{"varval":"An timber door wedge is pictured below.\n\n\nsm_img https://teacher.smartermaths.com.au/wp-content/uploads/2022/08/Measurement_NAPX-F4-NC29_v4.svg 600 indent vpad\n\n<br>The wedge is in the shape of a triangular prism?\n\nWhat is the volume of the wedge in cubic centimetres?\n"},{"varval":"Using &nbsp;1 cm = 10 mm\n\n<div class=\"aligned\">\n\n|||\n|-|-|\n|Volume| = $A\\large h$|\n||= $\\bigg( \\dfrac{1}{2} \\times 10 \\times 3 \\bigg) \\times 2.5$|\n||= 15 $\\times$ 2.5|\n||= {{{correctAnswer}}}|\n\n</div>"}]},{"vars":[{"varval":"A rubber access ramp is pictured below.\n\nsm_img https://teacher.smartermaths.com.au/wp-content/uploads/2022/08/Measurement_NAPX-F4-NC29_v_5.svg 400 indent3 vpad\n\n<br>The access ramp is in the shape of a triangular prism?\n\nWhat is the volume of the access ramp?"},{"varval":"<div class=\"aligned\">\n\n|||\n|-|-|\n|Volume| = $A\\large h$|\n||= $\\bigg( \\dfrac{1}{2} \\times 30 \\times 10 \\bigg) \\times 90$|\n||= 150 $\\times$ 90|\n||= {{{correctAnswer}}}|\n\n</div>"}]}]

  537. <div class="sm_mode"> {{{question}}} </div>

    [{"vars":[{"varval":"Rodin makes a large sculpture in the shape of a cube.\n\n\r\n\r\nThe total length of all of its edges is 72 metres.\n\n\r\n\r\nWhat is the area of one of the cube's faces?\n"},{"varval":"Total edges of a cube = 12\n\n\r\nsm_nogap Length of 1 edge\n\n\r\n>>= 72 ÷ 12\n\n\r\n>>= 6 metres\r\n \r\n\r\n<br>\n\nsm_nogap $\\therefore$ Area of one face\n\n\r\n>> = 6 × 6\n\n\r\n>> = {{{correctAnswer0}}} {{{suffix0}}}\n"}]},{"vars":[{"varval":"Dwayne cuts a sandstone block in the shape of a cube.\n\n\r\n\r\nThe total length of all of its edges is 48 metres.\n\n\r\n\r\nWhat is the area of one of the cube's faces?\n"},{"varval":"Total edges of a cube = 12\n\n\r\nsm_nogap Length of 1 edge\n\n\r\n>>= 48 ÷ 12\n\n\r\n>>= 4 metres\r\n \r\n\r\n<br>\n\nsm_nogap $\\therefore$ Area of one face\n\n\r\n>> = 4 × 4\n\n\r\n>> = {{{correctAnswer0}}} {{{suffix0}}}\n"}]},{"vars":[{"varval":"Liv builds a climbing gym in the shape of a cube.\n\n\r\n\r\nThe total length of all of its edges is 24 metres.\n\n\r\n\r\nWhat is the area of one of the cube's faces?\n"},{"varval":"Total edges of a cube = 12\n\n\r\nsm_nogap Length of 1 edge\n\n\r\n>>= 24 ÷ 12\n\n\r\n>>= 2 metres\r\n \r\n\r\n<br>\n\nsm_nogap $\\therefore$ Area of one face\n\n\r\n>> = 2 × 2\n\n\r\n>> = {{{correctAnswer0}}} {{{suffix0}}}"}]},{"vars":[{"varval":"Levy makes a birthday cake in the shape of a cube.\n\n\r\n\r\nThe total length of all of its edges is 180 centimetres.\n\n\r\n\r\nWhat is the area of one of the cube's faces?"},{"varval":"Total edges of a cube = 12\n\n\r\nsm_nogap Length of 1 edge\n\n\r\n>>= 180 ÷ 12\n\n\r\n>>= 15 centimetres\r\n \r\n\r\n<br>\n\nsm_nogap $\\therefore$ Area of one face\n\n\r\n>> = 15 × 15\n\n\r\n>> = {{{correctAnswer0}}} {{{suffix0}}}"}]},{"vars":[{"varval":"Mishy makes a Lego house in the shape of a cube.\n\n\r\n\r\nThe total length of all of its edges is 192 centimetres.\n\n\r\n\r\nWhat is the area of one of the cube's faces?"},{"varval":"Total edges of a cube = 12\n\n\r\nsm_nogap Length of 1 edge\n\n\r\n>>= 192 ÷ 12\n\n\r\n>>= 16 centimetres\r\n \r\n\r\n<br>\n\nsm_nogap $\\therefore$ Area of one face\n\n\r\n>> = 16 × 16\n\n\r\n>> = {{{correctAnswer0}}} {{{suffix0}}}"}]},{"vars":[{"varval":"Anthony trims a hedge in the shape of a cube.\n\n\r\n\r\nThe total length of all of its edges is 9.60 metres.\n\n\r\n\r\nWhat is the area of one of the cube's faces in square centimetres?"},{"varval":"Total edges of a cube = 12\n\n\r\nsm_nogap Length of 1 edge\n\n\r\n>>= 960 ÷ 12\n\n\r\n>>= 80 centimetres\r\n \r\n\r\n<br>\n\nsm_nogap $\\therefore$ Area of one face\n\n\r\n>> = 80 × 80\n\n\r\n>> = {{{correctAnswer0}}} {{{suffix0}}}"}]}]

  538. <div class="sm_mode"> {{{question}}} </div>

    [{"vars":[{"varval":"A builder is tiling a floor that is 240 cm wide and 360 cm long.\n\r\nShe uses the triangular floor tile that is drawn below.\n\n<br>\n\nsm_img https://teacher.smartermaths.com.au/wp-content/uploads/2018/06/NAPX-E4-CA26.svg 240 indent vpad\n\n<br>She uses all of her tiles and has no gaps between them.\n\nHow many tiles does she need?"},{"varval":"Strategy 1\n\n2 tiles form a $\\ 4 × 12$ cm &nbsp;rectangle.\n\n\r\nFitting rectangles into floor plan:\n\n\r\nWidth = $\\dfrac{240}{4}$ = 60 rectangles\n\n\r\nLength = $\\dfrac{360}{12}$ = 30 rectangles\n\nsm_nogap $\\therefore$ Total tiles\n\n\r\n>>= 2 × (30 × 60)\n\n\r\n>>= {{{correctAnswer}}}\n\n<br>\n\nStrategy 2\n\n<div class=\"aligned\">\n\n| | |\n| --------------------- | -------------------------------------------- |\n| Area of 1 triangle | = $\\dfrac{1}{2} \\times 4 \\times 12$ |\n| | = 24 cm$^2$|\n\n</div>\n\n<br>\n\n<div class=\"aligned\">\n\n| | |\n| --------------------- | -------------------------------------------- |\n| $\\therefore$ Tiles needed | = $(240 \\times 360) \\div 24$ |\n| | = {{{correctAnswer}}} |\n\n</div>\n"}]},{"vars":[{"varval":"A landscaper is tiling a patio that is 360 cm wide and 450 cm long.\n\r\nHe uses the triangular floor tile that is drawn below.\n\n<br>\n\nsm_img https://teacher.smartermaths.com.au/wp-content/uploads/2022/08/Measurement_NAPX-E4-CA26_5.svg 450 indent vpad\n\n<br>He uses all of his tiles and has no gaps between them.\n\nHow many tiles does he need?"},{"varval":"Strategy 1\n\n2 tiles form a $\\ 12 × 5$ cm &nbsp;rectangle.\n\n\r\nFitting rectangles into floor plan:\n\n\r\nWidth = $\\dfrac{360}{12}$ = 30 rectangles\n\n\r\nLength = $\\dfrac{450}{5}$ = 90 rectangles\n\nsm_nogap $\\therefore$ Total tiles\n\n\r\n>>= 2 × (30 × 90)\n\n\r\n>>= {{{correctAnswer}}}\n\n<br>\n\nStrategy 2\n\n<div class=\"aligned\">\n\n| | |\n| --------------------- | -------------------------------------------- |\n| Area of 1 triangle | = $\\dfrac{1}{2} \\times 12 \\times 5$ |\n| | = 30 cm$^2$|\n\n</div>\n\n<br>\n\n<div class=\"aligned\">\n\n| | |\n| --------------------- | -------------------------------------------- |\n| $\\therefore$ Tiles needed | = $(360 \\times 450) \\div 30$ |\n| | = {{{correctAnswer}}} |\n\n</div>\n"}]},{"vars":[{"varval":"A pastry chef is cutting pastries from a sheet of pastry that is 96 cm wide and 55 cm long.\n\r\nHe cuts out pastry triangles in the shape shown below.\n\n<br>\n\nsm_img https://teacher.smartermaths.com.au/wp-content/uploads/2022/08/Measurement_NAPX-E4-CA26_4.svg 270 indent3 vpad\n\n<br>He uses the whole pastry sheet and has no pieces left over.\n\nHow many pastry triangles does he cut?"},{"varval":"Strategy 1\n\n2 pastries form a $\\ 6 × 5$ cm &nbsp;rectangle.\n\n\r\nCutting pastries from the pastry sheet:\n\n\r\nWidth = $\\dfrac{96}{6}$ = 16 rectangles\n\n\r\nLength = $\\dfrac{55}{5}$ = 11 rectangles\n\nsm_nogap $\\therefore$ Total pastries\n\n\r\n>>= 2 × (16 × 11)\n\n\r\n>>= {{{correctAnswer}}}\n\n<br>\n\nStrategy 2\n\n<div class=\"aligned\">\n\n| | |\n| --------------------- | -------------------------------------------- |\n| Area of 1 triangle | = $\\dfrac{1}{2} \\times 6 \\times 5$ |\n| | = 15 cm$^2$|\n\n</div>\n\n<br>\n\n<div class=\"aligned\">\n\n| | |\n| --------------------- | -------------------------------------------- |\n| $\\therefore$ Pastries cut | = $(96 \\times 55) \\div 15$ |\n| | = {{{correctAnswer}}} |\n\n</div>\n"}]},{"vars":[{"varval":"Jeff is tiling his bathroom that is 2.4 m wide and 2.1 m long.\n\r\nHe uses the triangular floor tile that is drawn below.\n\n<br>\n\nsm_img https://teacher.smartermaths.com.au/wp-content/uploads/2022/08/Measurement_NAPX-E4-CA26_3.svg 270 indent vpad\n\n<br>He uses all of his tiles and has no gaps between them.\n\nHow many tiles does he need?"},{"varval":"Strategy 1\n\n2 tiles form a $\\ 8 × 3$ cm &nbsp;rectangle.\n\n\r\nFitting tiles into floor plan:\n\n\r\nWidth = $\\dfrac{240}{8}$ = 30 rectangles\n\n\r\nLength = $\\dfrac{210}{3}$ = 70 rectangles\n\nsm_nogap $\\therefore$ Total tiles\n\n\r\n>>= 2 × (30 × 70)\n\n\r\n>>= {{{correctAnswer}}}\n\n<br>\n\nStrategy 2\n\n<div class=\"aligned\">\n\n| | |\n| --------------------- | -------------------------------------------- |\n| Area of 1 triangle | = $\\dfrac{1}{2} \\times 8 \\times 3$ |\n| | = 12 cm$^2$|\n\n</div>\n\n<br>\n\n<div class=\"aligned\">\n\n| | |\n| --------------------- | -------------------------------------------- |\n| $\\therefore$ Tiles needed | = $(240 \\times 210) \\div 12$ |\n| | = {{{correctAnswer}}} |\n\n</div>\n"}]},{"vars":[{"varval":"Carmen is cutting material for a patchwork quilt. The finished quilt will measure 180 cm wide and 196 cm long.\n\r\nShe uses the triangular template that is drawn below.\n\n<br>\n\nsm_img https://teacher.smartermaths.com.au/wp-content/uploads/2022/08/Measurement_NAPX-E4-CA26_2.svg 400 indent vpad\n\n<br>She covers the area exactly and has no gaps between the pieces.\n\nHow many triangular pieces does she need?"},{"varval":"Strategy 1\n\n2 triangles form a $\\ 12 × 7$ cm &nbsp;rectangle.\n\n\r\nFitting triangles into quilt measurements:\n\n\r\nWidth = $\\dfrac{180}{12}$ = 15 rectangles\n\n\r\nLength = $\\dfrac{196}{7}$ = 28 rectangles\n\nsm_nogap $\\therefore$ Total tiles\n\n\r\n>>= 2 × (15 × 28)\n\n\r\n>>= {{{correctAnswer}}}\n\n<br>\n\nStrategy 2\n\n<div class=\"aligned\">\n\n| | |\n| --------------------- | -------------------------------------------- |\n| Area of 1 triangle | = $\\dfrac{1}{2} \\times 12 \\times 7$ |\n| | = 42 cm$^2$|\n\n</div>\n\n<br>\n\n<div class=\"aligned\">\n\n| | |\n| --------------------- | -------------------------------------------- |\n| $\\therefore$ Tiles needed | = $(180 \\times 196) \\div 42$ |\n| | = {{{correctAnswer}}} |\n\n</div>\n"}]},{"vars":[{"varval":"Bernice is tiling a mural that is 330 cm wide and 450 cm high.\n\r\nShe uses the triangular wall tile that is drawn below.\n\n<br>\n\nsm_img https://teacher.smartermaths.com.au/wp-content/uploads/2022/08/Measurement-NAPX-E4-CA26_1.svg 400 indent vpad\n\n<br>She uses all of her tiles and has no gaps between them.\n\nHow many tiles does she need?"},{"varval":"Strategy 1\n\n2 tiles form a $\\ 15 × 5$ cm &nbsp;rectangle.\n\n\r\nFitting tiles into floor plan:\n\n\r\nWidth = $\\dfrac{330}{15}$ = 22 rectangles\n\n\r\nLength = $\\dfrac{450}{5}$ = 90 rectangles\n\nsm_nogap $\\therefore$ Total tiles\n\n\r\n>>= 2 × (22 × 90)\n\n\r\n>>= {{{correctAnswer}}}\n\n<br>\n\nStrategy 2\n\n<div class=\"aligned\">\n\n| | |\n| --------------------- | -------------------------------------------- |\n| Area of 1 triangle | = $\\dfrac{1}{2} \\times 15 \\times 5$ |\n| | = 37.5 cm$^2$|\n\n</div>\n\n<br>\n\n<div class=\"aligned\">\n\n| | |\n| --------------------- | -------------------------------------------- |\n| $\\therefore$ Tiles needed | = $(330 \\times 450) \\div 37.5$ |\n| | = {{{correctAnswer}}} |\n\n</div>\n"}]}]

  539. <div class="sm_mode"> {{{question}}} </div>

    [{"vars":[{"varval":"sm_img https://teacher.smartermaths.com.au/wp-content/uploads/2018/06/NAPX-F4-NC24.svg 350 indent3 vpad\n\n<br>\n\nWhat is the area of the shaded part of the figure?\n"},{"varval":"sm_nogap Area of large triangle\n\n<div class=\"aligned\">\n\n>>||\n|-|\n|= $\\dfrac{1}{2} \\times 12 \\times 12$|\n|= 72 cm$^2$|\n\n</div>\n\n<br>\n\nsm_nogap Area of smaller triangle\n\n<div class=\"aligned\">\n\n>>||\n|-|\n|= $\\dfrac{1}{2} \\times 4 \\times 4$|\n|= 8 cm$^2$|\n\n</div>\n\n<br>\n\nsm_nogap $\\therefore$ Shaded area\n\n<div class=\"aligned\">\n\n>>||\n|-|\n|= 72 − (2 × 8)|\n|= {{{correctAnswer0}}} {{{suffix0}}}|\n\n</div>\n"}]},{"vars":[{"varval":"sm_img https://teacher.smartermaths.com.au/wp-content/uploads/2022/08/Measurement_NAPX-F4-NC24-SA_NAPX-F3-NC27-SA_v1.svg 400 indent3 vpad\n\n<br>\n\nWhat is the area of the shaded part of the figure?\n"},{"varval":"sm_nogap Area of large triangle\n\n<div class=\"aligned\">\n\n>>||\n|-|\n|= $\\dfrac{1}{2} \\times 16 \\times 16$|\n|= 128 cm$^2$|\n\n</div>\n\n<br>\n\nsm_nogap Area of smaller triangle\n\n<div class=\"aligned\">\n\n>>||\n|-|\n|= $\\dfrac{1}{2} \\times 6 \\times 6$|\n|= 18 cm$^2$|\n\n</div>\n\n<br>\n\nsm_nogap $\\therefore$ Shaded area\n\n<div class=\"aligned\">\n\n>>||\n|-|\n|= 128 − (2 × 18)|\n|= {{{correctAnswer0}}} {{{suffix0}}}|\n\n</div>\n"}]},{"vars":[{"varval":"sm_img https://teacher.smartermaths.com.au/wp-content/uploads/2022/08/Measurement_NAPX-F4-NC24-SA_NAPX-F3-NC27-SA_v2.svg 400 indent3 vpad\n\n<br>\n\nWhat is the area of the shaded part of the figure?\n"},{"varval":"sm_nogap Area of large triangle\n\n<div class=\"aligned\">\n\n>>||\n|-|\n|= $\\dfrac{1}{2} \\times 10 \\times 10$|\n|= 50 cm$^2$|\n\n</div>\n\n<br>\n\nsm_nogap Area of smaller triangle\n\n<div class=\"aligned\">\n\n>>||\n|-|\n|= $\\dfrac{1}{2} \\times 3 \\times 3$|\n|= 4.5 cm$^2$|\n\n</div>\n\n<br>\n\nsm_nogap $\\therefore$ Shaded area\n\n<div class=\"aligned\">\n\n>>||\n|-|\n|= 50 − (2 × 4.5)|\n|= {{{correctAnswer0}}} {{{suffix0}}}|\n\n</div>\n"}]},{"vars":[{"varval":"sm_img https://teacher.smartermaths.com.au/wp-content/uploads/2022/08/Measurement_NAPX-F4-NC24-SA_NAPX-F3-NC27-SA_v3.svg 400 indent3 vpad\n\n<br>\n\nWhat is the area of the shaded part of the figure?\n"},{"varval":"sm_nogap Area of large triangle\n\n<div class=\"aligned\">\n\n>>||\n|-|\n|= $\\dfrac{1}{2} \\times 20 \\times 20$|\n|= 200 cm$^2$|\n\n</div>\n\n<br>\n\nsm_nogap Area of smaller triangle\n\n<div class=\"aligned\">\n\n>>||\n|-|\n|= $\\dfrac{1}{2} \\times 8 \\times 8$|\n|= 32 cm$^2$|\n\n</div>\n\n<br>\n\nsm_nogap $\\therefore$ Shaded area\n\n<div class=\"aligned\">\n\n>>||\n|-|\n|= 200 − (2 × 32)|\n|= {{{correctAnswer0}}} {{{suffix0}}}|\n\n</div>\n"}]},{"vars":[{"varval":"sm_img https://teacher.smartermaths.com.au/wp-content/uploads/2022/08/Measurement_NAPX-F4-NC24-SA_NAPX-F3-NC27-SA_v4.svg 400 indent3 vpad\n\n<br>\n\nWhat is the area of the shaded part of the figure?\n"},{"varval":"sm_nogap Area of large triangle\n\n<div class=\"aligned\">\n\n>>||\n|-|\n|= $\\dfrac{1}{2} \\times 20 \\times 20$|\n|= 200 cm$^2$|\n\n</div>\n\n<br>\n\nsm_nogap Area of smaller triangle\n\n<div class=\"aligned\">\n\n>>||\n|-|\n|= $\\dfrac{1}{2} \\times 5 \\times 5$|\n|= 12.5 cm$^2$|\n\n</div>\n\n<br>\n\nsm_nogap $\\therefore$ Shaded area\n\n<div class=\"aligned\">\n\n>>||\n|-|\n|= 200 − (2 × 12.5)|\n|= {{{correctAnswer0}}} {{{suffix0}}}|\n\n</div>\n"}]},{"vars":[{"varval":"sm_img https://teacher.smartermaths.com.au/wp-content/uploads/2022/08/Measurement_NAPX-F4-NC24-SA_NAPX-F3-NC27-SA_v5.svg 400 indent3 vpad\n\n<br>\n\nWhat is the area of the shaded part of the figure?\n"},{"varval":"sm_nogap Area of large triangle\n\n<div class=\"aligned\">\n\n>>||\n|-|\n|= $\\dfrac{1}{2} \\times 12 \\times 12$|\n|= 72 cm$^2$|\n\n</div>\n\n<br>\n\nsm_nogap Area of smaller triangle\n\n<div class=\"aligned\">\n\n>>||\n|-|\n|= $\\dfrac{1}{2} \\times 3.6 \\times 3.6$|\n|= 6.48 cm$^2$|\n\n</div>\n\n<br>\n\nsm_nogap $\\therefore$ Shaded area\n\n<div class=\"aligned\">\n\n>>||\n|-|\n|= 72 − (2 × 6.48)|\n|= {{{correctAnswer0}}} {{{suffix0}}}|\n\n</div>\n"}]}]

  540. <div class="sm_mode"> {{{question}}} </div>

    [{"vars":[{"varval":"A square has an area of 196 square centimetres.\n\n\r\n\r\nWhat is the perimeter?\n"},{"varval":"sm_nogap Let &nbsp;$\\large x$ = length of 1 side \n\n<div class=\"aligned\">\n\n|||\n|-:|-|\n|$\\large x$$^2$|= 196|\n|$\\large x$|= $\\sqrt{196}$|\n||= 14 cm|\n\n</div>\n\n<br>\n\r\n<div class=\"aligned\">\n\n|||\n|-:|-|\n|$\\therefore$ Perimeter|= 4 $\\times$ 14|\n| |= {{{correctAnswer0}}} {{{suffix0}}}|\n\n\n</div>"}]},{"vars":[{"varval":"A square has an area of 225 square centimetres.\n\n\r\n\r\nWhat is the perimeter?\n"},{"varval":"sm_nogap Let &nbsp;$\\large x$ = length of 1 side \n\n<div class=\"aligned\">\n\n|||\n|-:|-|\n|$\\large x$$^2$|= 225|\n|$\\large x$|= $\\sqrt{225}$|\n||= 15 cm|\n\n</div>\n\n<br>\n\r\n<div class=\"aligned\">\n\n|||\n|-:|-|\n|$\\therefore$ Perimeter|= 4 $\\times$ 15|\n| |= {{{correctAnswer0}}} {{{suffix0}}}|\n\n\n</div>"}]},{"vars":[{"varval":"A square has an area of 576 square centimetres.\n\n\r\n\r\nWhat is the perimeter?\n"},{"varval":"sm_nogap Let &nbsp;$\\large x$ = length of 1 side \n\n<div class=\"aligned\">\n\n|||\n|-:|-|\n|$\\large x$$^2$|= 576|\n|$\\large x$|= $\\sqrt{576}$|\n||= 24 cm|\n\n</div>\n\n<br>\n\r\n<div class=\"aligned\">\n\n|||\n|-:|-|\n|$\\therefore$ Perimeter|= 4 $\\times$ 24|\n| |= {{{correctAnswer0}}} {{{suffix0}}}|\n\n\n</div>"}]},{"vars":[{"varval":"A square has an area of 324 square centimetres.\n\n\r\n\r\nWhat is the perimeter?\n"},{"varval":"sm_nogap Let &nbsp;$\\large x$ = length of 1 side \n\n<div class=\"aligned\">\n\n|||\n|-:|-|\n|$\\large x$$^2$|= 324|\n|$\\large x$|= $\\sqrt{324}$|\n||= 18 cm|\n\n</div>\n\n<br>\n\r\n<div class=\"aligned\">\n\n|||\n|-:|-|\n|$\\therefore$ Perimeter|= 4 $\\times$ 18|\n| |= {{{correctAnswer0}}} {{{suffix0}}}|\n\n\n</div>"}]},{"vars":[{"varval":"A square has an area of 1.44 square metres.\n\n\r\n\r\nWhat is the perimeter?\n"},{"varval":"sm_nogap Let &nbsp;$\\large x$ = length of 1 side \n\n<div class=\"aligned\">\n\n|||\n|-:|-|\n|$\\large x$$^2$|= 1.44|\n|$\\large x$|= $\\sqrt{1.44}$|\n||= 1.2 m|\n\n</div>\n\n<br>\n\r\n<div class=\"aligned\">\n\n|||\n|-:|-|\n|$\\therefore$ Perimeter|= 4 $\\times$ 1.2|\n| |= {{{correctAnswer0}}} {{{suffix0}}}|\n\n\n</div>"}]},{"vars":[{"varval":"A square has an area of 3.61 square metres.\n\n\r\n\r\nWhat is the perimeter?\n"},{"varval":"sm_nogap Let &nbsp;$\\large x$ = length of 1 side \n\n<div class=\"aligned\">\n\n|||\n|-:|-|\n|$\\large x$$^2$|= 3.61|\n|$\\large x$|= $\\sqrt{3.61}$|\n||= 1.9 m|\n\n</div>\n\n<br>\n\r\n<div class=\"aligned\">\n\n|||\n|-:|-|\n|$\\therefore$ Perimeter|= 4 $\\times$ 1.9|\n| |= {{{correctAnswer0}}} {{{suffix0}}}|\n\n\n</div>"}]}]

  541. <div class="sm_mode"> {{{question}}} </div>

    [{"vars":[{"varval":"A logo is shaped like a hexagon.\n\nThe dimensions are given from the top view of the logo.\n\n<br>\n\nsm_img https://teacher.smartermaths.com.au/wp-content/uploads/2021/09/NAPX9-TLF-32.svg 320 indent3 vpad\n\n<br>What is the total area of the logo?\n"},{"varval":"<div class=\"aligned\">\n\n|||\n|-|-|\n|Area|= Area of rectangle + 2 × Area of triangle|\n||= (13 × 22) + 2 × $\\bigg( \\dfrac{1}{2} × 22 × 3 \\bigg)$|\n||= 286 + 66|\n||= {{{correctAnswer0}}} {{{suffix0}}}|\n\n</div>"}]},{"vars":[{"varval":"A logo is shaped like a hexagon.\n\nThe dimensions are given from the top view of the logo.\n\n<br>\n\nsm_img https://teacher.smartermaths.com.au/wp-content/uploads/2022/08/Measurement_NAPX9-TLF-CA32-v3_v1.svg 350 indent3 vpad\n\n<br>What is the total area of the logo?\n"},{"varval":"<div class=\"aligned\">\n\n|||\n|-|-|\n|Area|= Area of rectangle + 2 × Area of triangle|\n||= (40 × 19) + 2 × $\\bigg( \\dfrac{1}{2} × 40 × 7 \\bigg)$|\n||= 760 + 280|\n||= {{{correctAnswer0}}} {{{suffix0}}}|\n\n</div>"}]},{"vars":[{"varval":"A resort's swimming pool is shaped like a hexagon.\n\nThe dimensions are given from the top view of the swimming pool.\n\n<br>\n\nsm_img https://teacher.smartermaths.com.au/wp-content/uploads/2022/08/Measurement_NAPX9-TLF-CA32-v3_v2.svg 420 indent vpad\n\n<br>What is the total area of the swimming pool?\n"},{"varval":"<div class=\"aligned\">\n\n|||\n|-|-|\n|Area|= Area of rectangle + 2 × Area of triangle|\n||= (3 × 4) + 2 × $\\bigg( \\dfrac{1}{2} × 3 × 5 \\bigg)$|\n||= 12 + 15|\n||= {{{correctAnswer0}}} {{{suffix0}}}|\n\n</div>"}]},{"vars":[{"varval":"An earring is shaped like a hexagon.\n\nThe dimensions are given from the top view of the earring.\n\n<br>\n\nsm_img https://teacher.smartermaths.com.au/wp-content/uploads/2022/08/Measurement_NAPX9-TLF-CA32-v3_v3.svg 270 indent3 vpad\n\n<br>What is the total area of the earring?\n"},{"varval":"<div class=\"aligned\">\n\n|||\n|-|-|\n|Area|= Area of rectangle + 2 × Area of triangle|\n||= (6 × 2) + 2 × $\\bigg( \\dfrac{1}{2} × 6 × 1 \\bigg)$|\n||= 12 + 6|\n||= {{{correctAnswer0}}} {{{suffix0}}}|\n\n</div>"}]},{"vars":[{"varval":"A holiday unit is shaped like a hexagon.\n\nThe dimensions of its floor plan are shown below.\n\n<br>\n\nsm_img https://teacher.smartermaths.com.au/wp-content/uploads/2022/08/Measurement_NAPX9-TLF-CA32-v3_v4.svg 480 indent vpad\n\n<br>What is the total area of the holiday unit?\n"},{"varval":"<div class=\"aligned\">\n\n|||\n|-|-|\n|Area|= Area of rectangle + 2 × Area of triangle|\n||= (14 × 9) + 2 × $\\bigg( \\dfrac{1}{2} × 9 × 3 \\bigg)$|\n||= 126 + 27|\n||= {{{correctAnswer0}}} {{{suffix0}}}|\n\n</div>"}]},{"vars":[{"varval":"A place mat is shaped like a hexagon.\n\nThe dimensions are given from the top view of the place mat.\n\n<br>\n\nsm_img https://teacher.smartermaths.com.au/wp-content/uploads/2022/08/Measurement_NAPX9-TLF-CA32-v3_v5.svg 380 indent vpad\n\n<br>What is the total area of the place mat?\n"},{"varval":"<div class=\"aligned\">\n\n|||\n|-|-|\n|Area|= Area of rectangle + Area of triangle 1 + Area of triangle 2|\n||= (24 × 27) + $\\bigg( \\dfrac{1}{2} × 27 × 6 \\bigg)$ + $\\bigg( \\dfrac{1}{2} × 27 × 5 \\bigg)$||\n||= 648 + 81 + 67.5|\n||= {{{correctAnswer0}}} {{{suffix0}}}|\n\n</div>"}]}]

  542. Measurement, NAPX9-TLD-CA22 v2

    <div class="sm_mode"> {{{question}}} </div>

    [{"vars":[{"varval":"The length of this rectangle is three times its height.\n\n<br>\n\nsm_img https://teacher.smartermaths.com.au/wp-content/uploads/2022/08/Measurement-NAPX9-TLD-CA22-v3_0_1_a.svg 400 indent vpad\n\n<br>The perimeter of the rectangle is 32 centimetres.\n\n\r\n\r\nWhat is the area of the rectangle?"},{"varval":"<div class=\"aligned\">\n\n|||\n|-:|-|\n|Let $\\ \\large h$| = height|\n|$3\\large h$|= length|\n\n</div>\n\n<div class=\"aligned\">\n\n|||\n|-:|-|\n|$2 \\times (\\large h$ + 3$\\large h$)| = 32|\n|$8\\large h$|= 32|\n|$\\large h$|= 4|\n\n</div>\n\n<div class=\"aligned\">\n\n|||\n|-|-|\n|$\\therefore$ Area| = 4 $\\times$ 12|\n| |= {{{correctAnswer0}}} {{{suffix0}}}|\n\n</div>"}]},{"vars":[{"varval":"The length of this rectangle is twice its height.\n\n<br>\n\nsm_img https://teacher.smartermaths.com.au/wp-content/uploads/2022/08/Measurement-NAPX9-TLD-CA22-v3_0_1_a.svg 400 indent3 vpad\n\n<br>The perimeter of the rectangle is 30 centimetres.\n\n\r\n\r\nWhat is the area of the rectangle?"},{"varval":"<div class=\"aligned\">\n\n|||\n|-:|-|\n|Let $\\ \\large h$| = height|\n|$2\\large h$|= length|\n\n</div>\n\n<div class=\"aligned\">\n\n|||\n|-:|-|\n|$2 \\times (\\large h$ + 2$\\large h$)| = 30|\n|$6\\large h$|= 30|\n|$\\large h$|= 5|\n\n</div>\n\n<div class=\"aligned\">\n\n|||\n|-|-|\n|$\\therefore$ Area| = 5 $\\times$ 10|\n| |= {{{correctAnswer0}}} {{{suffix0}}}|\n\n</div>"}]},{"vars":[{"varval":"The length of this rectangle is one third its height.\n\n<br>\n\nsm_img https://teacher.smartermaths.com.au/wp-content/uploads/2022/08/Measurement-NAPX9-TLD-CA22-v3_2_3.svg 240 indent3 vpad\n\n<br>The perimeter of the rectangle is 72 centimetres.\n\n\r\n\r\nWhat is the area of the rectangle?"},{"varval":"<div class=\"aligned\">\n\n|||\n|-:|-|\n|Let $\\ \\large l$| = length|\n|$3\\large l$|= height|\n\n</div>\n\n<div class=\"aligned\">\n\n|||\n|-:|-|\n|$2 \\times (\\large l$ + 3$\\large l$)| = 72|\n|$8\\large l$|= 72|\n|$\\large l$|= 9|\n\n</div>\n\n<div class=\"aligned\">\n\n|||\n|-|-|\n|$\\therefore$ Area| = 9 $\\times$ 27|\n| |= {{{correctAnswer0}}} {{{suffix0}}}|\n\n</div>"}]},{"vars":[{"varval":"The length of this rectangle is one-quarter its height.\n\n<br>\n\nsm_img https://teacher.smartermaths.com.au/wp-content/uploads/2022/08/Measurement-NAPX9-TLD-CA22-v3_2_3.svg 240 indent3 vpad\n\n<br>The perimeter of the rectangle is 50 centimetres.\n\n\r\n\r\nWhat is the area of the rectangle?"},{"varval":"<div class=\"aligned\">\n\n|||\n|-:|-|\n|Let $\\ \\large l$| = length|\n|$4\\large l$|= height|\n\n</div>\n\n<div class=\"aligned\">\n\n|||\n|-:|-|\n|$2 \\times (\\large l$ + 4$\\large l$)| = 50|\n|$10\\large l$|= 50|\n|$\\large l$|= 5|\n\n</div>\n\n<div class=\"aligned\">\n\n|||\n|-|-|\n|$\\therefore$ Area| = 5 $\\times$ 20|\n| |= {{{correctAnswer0}}} {{{suffix0}}}|\n\n</div>"}]},{"vars":[{"varval":"The length of this rectangle is one and a half times its height.\n\n<br>\n\nsm_img https://teacher.smartermaths.com.au/wp-content/uploads/2022/08/Measurement-NAPX9-TLD-CA22-v3_4_5.svg 300 indent vpad\n\n<br>The perimeter of the rectangle is 60 centimetres.\n\n\r\n\r\nWhat is the area of the rectangle?"},{"varval":"<div class=\"aligned\">\n\n|||\n|-:|-|\n|Let $\\ \\large h$| = height|\n|$1.5\\large h$|= length|\n\n</div>\n\n<div class=\"aligned\">\n\n|||\n|-:|-|\n|$2 \\times (\\large h$ + 1.5$\\large h$)| = 60|\n|$5\\large h$|= 60|\n|$\\large h$|= 12|\n\n</div>\n\n<div class=\"aligned\">\n\n|||\n|-|-|\n|$\\therefore$ Area| = 12 $\\times$ 18|\n| |= {{{correctAnswer0}}} {{{suffix0}}}|\n\n</div>"}]},{"vars":[{"varval":"The length of this rectangle is two and half times its height.\n\n<br>\n\nsm_img https://teacher.smartermaths.com.au/wp-content/uploads/2022/08/Measurement-NAPX9-TLD-CA22-v3_4_5.svg 300 indent vpad\n\n<br>The perimeter of the rectangle is 42 centimetres.\n\n\r\n\r\nWhat is the area of the rectangle?"},{"varval":"<div class=\"aligned\">\n\n|||\n|-:|-|\n|Let $\\ \\large h$| = height|\n|$2.5\\large h$|= length|\n\n</div>\n\n<div class=\"aligned\">\n\n|||\n|-:|-|\n|$2 \\times (\\large h$ + 2.5$\\large h$)| = 42|\n|$7\\large h$|= 42|\n|$\\large h$|= 6|\n\n</div>\n\n<div class=\"aligned\">\n\n|||\n|-|-|\n|$\\therefore$ Area| = 6 $\\times$ 15|\n| |= {{{correctAnswer0}}} {{{suffix0}}}|\n\n</div>"}]}]

  543. <div class="sm_mode"> {{{question}}} </div>

    [{"vars":[{"varval":"Petal did a survey of her class, asking everyone what their favourite season is.\r\n\r\nThis table below shows the results.\n\n<br>\n\n<div class=\"sm-table row1-color3\">\n\n>>| Season | Number of Classmates|\n|:-:|:-:|\n| Summer | 17|\n| Autumn | 8|\n|Winter|2|\n|Spring| 10|\n\n</div>\n\n<br>What is the probability that a randomly selected classmate's favourite season is Spring?\r\n\r\nRound your answer to the nearest hundredth."},{"varval":"sm_nogap $P$(Spring is favourite)\n\n<div class=\"aligned\">\r\n\r\n>| | |\r\n| ------------- | ---------- |\r\n| | \\= $\\dfrac{\\text{number who chose Spring}}{\\text{total classmates}}$ |\r\n|||\n| | \\= $\\dfrac{10}{17 + 8 + 2 + 10}$ |\n|||\n| | \\= $\\dfrac{10}{37}$ |\n| | \\= 0.270... |\r\n| | \\= {{{correctAnswer}}} (nearest hundredth)|\r\n\r\n</div>\r"}]},{"vars":[{"varval":"Jenny did a survey of her kindergarten class, asking everyone who their favourite Wiggle was.\r\n\r\nThis table below shows the results.\n\n<br>\n\n<div class=\"sm-table row1-color3\">\n\n>>| Wiggle | Number of Students|\n|:-:|:-:|\n| Red | 5|\n| Blue | 6|\n|Yellow|9|\n|Purple| 3|\n\n</div>\n\n<br>What is the probability that a randomly selected classmate's favourite Wiggle is Yellow Wiggle?\r\n\r\nRound your answer to the nearest hundredth."},{"varval":"sm_nogap $P$(Yellow Wiggle favourite)\n\n<div class=\"aligned\">\r\n\r\n>| | |\r\n| ------------- | ---------- |\r\n| | \\= $\\dfrac{\\text{number who chose Yellow Wiggle}}{\\text{total classmates}}$ |\r\n|||\n| | \\= $\\dfrac{9}{5 + 6 + 9 + 3}$ |\n|||\n| | \\= $\\dfrac{9}{23}$ |\n| | \\= 0.391... |\r\n| | \\= {{{correctAnswer}}} (nearest hundredth) |\r\n\r\n</div>\r"}]},{"vars":[{"varval":"Haggs did a survey of his kindergarten class, asking everyone who their favourite Teletubby was.\r\n\r\nThis table below shows the results.\n\n<br>\n\n<div class=\"sm-table row1-color3\">\n\n>>| **Teletubby** | **Number of Students**|\n|:-:|:-:|\n| Dipsy | 3|\n| Laa-Laa |5|\n|Tinky-Winky| 11|\n|Po| 2 |\n\n</div>\n\n<br>What is the probability that a randomly selected classmate's favourite Teletubby is Tinky-Winky?\r\n\r\nRound your answer to the nearest hundredth."},{"varval":"sm_nogap $P$(Tinky-Winky favourite)\n\n<div class=\"aligned\">\r\n\r\n>| | |\r\n| ------------- | ---------- |\r\n| | \\= $\\dfrac{\\text{number who chose Tinky-Winky}}{\\text{total classmates}}$ |\r\n|||\n| | \\= $\\dfrac{11}{3 + 5 + 11 + 2}$ |\n|||\n| | \\= $\\dfrac{11}{21}$ |\n| | \\= 0.523... |\r\n| | \\= {{{correctAnswer}}} (nearest hundredth) |\r\n\r\n</div>\r"}]}]

  544. <div class="sm_mode"> Kooger spins the arrow on two identical spinners. The arrow on each spinner is equally likely to land on 1, 2 or 3. <br> sm_img https://teacher.smartermaths.com.au/wp-content/uploads/2016/12/NAP-149-NC13.png 300 indent3 vpad <br>If Kooger adds up the two results, which total is he least likely to get? </div>

    [{"vars":null}]

  545. <div class="sm_mode"> {{{question}}} </div>

    [{"vars":[{"varval":"\nTwo identical spinners are spun at the same time and the two numbers they land on are added together.\n\n<br>\n\n\n\nsm_img https://teacher.smartermaths.com.au/wp-content/uploads/2018/04/NAPX-H4-NC031.svg 320 indent vpad\n\n<br>Which total is most likely?\n"},{"varval":"Seven can be achieved in 6 different ways.\n\n<div class=\"sm-table col1-color1\">\n\n>| Spinner 1| 1|2|3|4|5|6|\n|:-:|:-:|:-:|:-:|:-:|:-:|:-:|\n| Spinner 2| 6|5|4|3|2|1|\n\n\n</div>\n\n<br>All other totals have less possible combinations.\n\n$\\therefore$ {{{correctAnswer}}} is the most likely.\n"}]},{"vars":[{"varval":"Two identical spinners are spun at the same time and the two numbers they land on are added together.\n\n<br>\n\n\n\nsm_img https://teacher.smartermaths.com.au/wp-content/uploads/2018/04/NAPX-H4-NC031.svg 320 indent vpad\n\n<br>Which are the two least likely totals?\n\n\n"},{"varval":"The totals when the numbers are added together are shown in the grid below.\n\n<div class=\"sm-table col1-color5 row1-color5 \">\n\n>>|+| 1|2|3|4|5|6|\n|:-:|:-:|:-:|:-:|:-:|:-:|:-:|\n| 1| 2|3|4|5|6|7|\n| 2| 3|4|5|6|7|8|\n| 3| 4|5|6|7|8|9|\n| 4| 5|6|7|8|9|10|\n| 5| 6|7|8|9|10|11|\n| 6| 7|8|9|10|11|12|\n\n\n\n</div>\n\n<br>All totals except {{{correctAnswer}}} occur more than once.\n\n$\\therefore$ {{{correctAnswer}}} are the two least likely totals."}]}]

  546. <div class="sm_mode"> {{{question}}} </div>

    [{"vars":[{"varval":"Dennis is a fast bowler. In a cricket game, the chance of him getting a wicket on a given ball is unlikely.\r\n\r\nWhich probability best describes Dennis' chance of getting a wicket from one particular ball?"},{"varval":"Unlikely means the probability is\r close to zero.\n\n$\\therefore$ &nbsp;{{{correctAnswer}}} &nbsp;is the best probability."}]},{"vars":[{"varval":"Isaac holds an apple in his outstretched hand and releases it.\r\n\r\nWhich number below represents the probability that it will travel towards the ground?"},{"varval":"A certain event has a probability of {{{correctAnswer}}}"}]}]

  547. <div class="sm_mode"> {{{question}}} </div>

    [{"vars":[{"varval":"A biased dice is rolled.\r\n\r\nThe probability of rolling an even number is 70%.\r\n\r\n\nWhat is the probability of rolling an odd number?"},{"varval":"sm_nogap $P$(odd number)\n\n<div class=\"aligned\">\r\n\r\n>| |\r\n| ---------- |\r\n| \\= 1 $− \\ P$(even number)|\n| \\= $1\\ − \\ 0.70$|\n| \\= 0.30 |\r\n| \\= {{{correctAnswer0}}}{{{suffix0}}} |\r\n\r\n</div>\r"}]},{"vars":[{"varval":"A biased dice is rolled.\r\n\r\nThe probability of rolling an odd number is 40%.\r\n\r\n\nWhat is the probability of rolling an even number?"},{"varval":"sm_nogap $P$(even number)\n\n<div class=\"aligned\">\r\n\r\n>| |\r\n| ---------- |\r\n| \\= 1 $− \\ P$(odd number)|\n| \\= $1\\ − \\ 0.40$|\n| \\= 0.60 |\r\n| \\= {{{correctAnswer0}}}{{{suffix0}}} |\r\n\r\n</div>\r"}]},{"vars":[{"varval":"A biased dice is rolled.\r\n\r\nThe probability of rolling an odd number is 55%.\r\n\r\n\nWhat is the probability of rolling an even number?"},{"varval":"sm_nogap $P$(even number)\n\n<div class=\"aligned\">\r\n\r\n>| |\r\n| ---------- |\r\n| \\= 1 $− \\ P$(odd number)|\n| \\= $1\\ − \\ 0.55%$|\n| \\= 0.45 |\r\n| \\= {{{correctAnswer0}}}{{{suffix0}}} |\r\n\r\n</div>\r"}]}]

  548. <div class="sm_mode"> {{{question}}} </div>

    [{"vars":[{"varval":"Albert runs a pet store that sells pure breed dogs.\r\n\r\nHe has a total of 36 dogs in the shop from different breeds.\n\n<br>\n\n<div class=\"sm-table row1-color8\">\n\n>>| **Breed** | **Number of dogs** |\n|:-:|:-:|\n| Pug| 4|\n| Labrador| 16|\n| Bulldog| 6|\n| Chihuahua| 5|\n| Husky | 5|\n\n</div>\n\n<br>If one of Albert's dogs is chosen at random, what is the probability it will be either a labrador or a bulldog?"},{"varval":"\n<div class=\"aligned\">\r\n\r\n| | |\r\n| ------------- | ---------- |\r\n| $P$(L or B) | \\= $\\dfrac{\\text{Number of L and B}}{\\text{Total puppies}}$ |\n| | \\= $\\dfrac{16 + 6}{36}$ |\r\n| | \\= {{{correctAnswer}}} |\r\n\r\n</div>\r\n\n"}]},{"vars":[{"varval":"Nerida breeds 4 different types of hounds.\r\n\r\nLast year her hounds produced a total of 32 puppies.\n\n<br>\n\n<div class=\"sm-table row1-color8\">\n\n>>| **Hound Type**| **Number of puppies** |\n|:-:|:-:|\n| Beagle| 6|\n| Ridgeback| 8|\n| Bloodhound| 12|\n| Dachshund| 6|\n\n</div>\n\n<br>If one puppy is chosen at random, what is the probability it will be either a Ridgeback or a Dachshund?"},{"varval":"<div class=\"aligned\">\r\n\r\n| | |\r\n| ------------: | ---------- |\r\n| $P$(R or D) \t | \\= $\\dfrac{\\text{Number of R and D}}{\\text{Total puppies}}$ |\n| | \\= $\\dfrac{8 + 6}{32}$ |\r\n| | \\= {{{correctAnswer}}} |\r\n\r\n</div>\r"}]}]

  549. <div class="sm_mode"> {{{question}}} </div>

    [{"vars":[{"varval":"An ice hockey competition has six teams playing on the same day.\r\n\r\nThe table below shows the game order.\n\n<br>\n\n<div class=\"sm-table row1-color6\">\n\n>>| Game | Teams |\n|:-:|:-:|\n| 1 | Kings vs Oilers|\n| 2 | Flames vs Penguins|\n| 3 | Canucks vs Rangers|\n\n</div>\n\n<br>If no match ends in a draw, which of the following is possible?"},{"varval":"Since the Penguins and Canucks do not play each other, both can lose.\n\n$\\therefore$ Possible result: {{{correctAnswer}}}"}]},{"vars":[{"varval":"A soccer gala day has six teams playing in round one.\r\n\nMatches can end in a draw.\n\r\nThe table below shows the game order.\n\n<br>\n\n<div class=\"sm-table row1-color1\">\n\n>>| Game | Teams |\n|:-:|:-:|\n| 1 | Magpies vs Yabbies|\n| 2 | Devils vs Red Mist|\n| 3 | Lillies vs Mangroves|\n\n</div>\n\n<br>Which of the following results is not possible?"},{"varval":"Since the Mangroves and Lillies play each other, both cannot win.\n\n$\\therefore$ Impossible result: {{{correctAnswer}}}"}]}]

  550. <div class="sm_mode"> A spinner can land in any of 4 sections, labelled 1 to 4. The spinner is spun 100 times and the results are recorded in the bar chart below. <br> sm_img https://teacher.smartermaths.com.au/wp-content/uploads/2019/01/NAPX-G2-25.svg 350 indent vpad <br> Which of these spinners is most likely to give results shown in the graph? </div>

    [{"vars":null}]

  551. Internal test for element alignment

    <div class="sm_mode"> This one is not vertically alligned ... ![](/img/napx-h4-nc08-sa1.svg) + ![](/img/napx-h4-nc08-sa.svg) <div class="sm_img_inline"> But this one is since it is in the special div ... ![](/img/napx-h4-nc08-sa1.svg) + ![](/img/napx-h4-nc08-sa.svg) + ![](/img/napx-h4-nc08-sa.svg) = ![](/img/napx-h4-nc08-sa.svg) Please check the body of this post to see how it's done. The images, are standard markdown image formats...in this case the images are local, but they can be images from teacher.smartermaths.com.au. In this case, I've picked a png, and need to change it's default size, which you would do with the following html (not markdown) syntax (that should still be in the spacial sm_img_inline div): And here is another example specifying the size ... <img src="https://teacher.smartermaths.com.au/wp-content/uploads/2021/04/RAPH10_shape1.png" alt="" width="50"/> </div> </div>

    [{"vars":null}]

  552. <div class="sm_mode"> {{{question}}} </div>

    [{"vars":[{"varval":"A plan of Petal's garden is shown below.\n<br>\n\nsm_img https://teacher.smartermaths.com.au/wp-content/uploads/2018/05/NAPX-I4-NC22.svg 240 indent3 vpad\n\nPetal wants to irrigate her garden and needs to know the total area.\n\nWhat is the total area of Petal's garden, in square metres?\n"},{"varval":"sm_img https://teacher.smartermaths.com.au/wp-content/uploads/2018/05/NAPX-I4-NC22ans.svg 220 indent vpad\n\n<div class=\"aligned\">\n\n|||\n|-|-|\n| Area| = square + triangle|\n||= (6 × 5) + $\\bigg( \\dfrac{1}{2} \\times 6 \\times 3 \\bigg)$|\n||= 30 + 9|\n||= {{{correctAnswer0}}} {{{suffix0}}}|\n\n</div>"}]},{"vars":[{"varval":"A plan of Regina's backyard is shown below.\n\nsm_img https://teacher.smartermaths.com.au/wp-content/uploads/2022/08/Measurement_NAPX-I4-NC22-SA_v1.svg 430 indent3 vpad\n\nRegina wants to lay turf to cover the whole of her backyard and needs to know the total area.\n\n\r\nWhat is the total area of Regina's backyard, in square metres?\n"},{"varval":"sm_img https://teacher.smartermaths.com.au/wp-content/uploads/2022/08/Measurement_NAPX-I4-NC22-SA_v1_ws.svg 430 indent3 vpad\n\n<div class=\"aligned\">\n\n|||\n|-|-|\n| Area| = square + triangle|\n||= (10 × 12) + $\\bigg( \\dfrac{1}{2} \\times 10 \\times 8 \\bigg)$|\n||= 120 + 40|\n||= {{{correctAnswer0}}} {{{suffix0}}}|\n\n</div>"}]},{"vars":[{"varval":"A plan of Ulk's Tibetan wall hanging is shown below.\n\n\nsm_img https://teacher.smartermaths.com.au/wp-content/uploads/2022/08/Measurement_NAPX-I4-NC22-SA_v3.svg 450 indent vpad\n\nWhat is the total area of the wall hanging, in square centimetres?"},{"varval":"sm_img https://teacher.smartermaths.com.au/wp-content/uploads/2022/08/Measurement_NAPX-I4-NC22-SA_v3_ws.svg 480 indent vpad\n\n<div class=\"aligned\">\n\n|||\n|-|-|\n| Area| = square + triangle|\n||= (9 × 8) + $\\bigg( \\dfrac{1}{2} \\times 8 \\times 5 \\bigg)$|\n||= 72 + 20|\n||= {{{correctAnswer0}}} {{{suffix0}}}|\n\n</div>"}]},{"vars":[{"varval":"A plan of a floor tile is shown below.\n\nsm_img https://teacher.smartermaths.com.au/wp-content/uploads/2022/08/Measurement_NAPX-I4-NC22-SA_v4.svg 330 indent vpad\n\nPo wants to tile his bathroom using these tiles and needs to know the total area of one tile.\n\r\nWhat is the total area of Po's floor tile, in square centimetres?"},{"varval":"sm_img https://teacher.smartermaths.com.au/wp-content/uploads/2022/08/Measurement_NAPX-I4-NC22-SA_v4_ws.svg 350 indent vpad\n\n<div class=\"aligned\">\n\n|||\n|-|-|\n| Area| = square + triangle|\n||= (18 × 6) + $\\bigg( \\dfrac{1}{2} \\times 18 \\times 3 \\bigg)$|\n||= 108 + 27|\n||= {{{correctAnswer0}}} {{{suffix0}}}|\n\n</div>"}]},{"vars":[{"varval":"A plan of a children's playground at a shopping centre is shown below.\n\nsm_img https://teacher.smartermaths.com.au/wp-content/uploads/2022/08/Measurement_NAPX-I4-NC22-SA_v2.svg 400 indent vpad\n\nThe centre wants to lay rubber matting in the playground and needs to know the total area.\n\nWhat at is the total area of the shopping centre's playground, in square metres?"},{"varval":"sm_img https://teacher.smartermaths.com.au/wp-content/uploads/2022/08/Measurement_NAPX-I4-NC22-SA_v2_ws.svg 400 indent3 vpad\n\n<div class=\"aligned\">\n\n|||\n|-|-|\n| Area| = square + triangle|\n||= (10 × 7) + $\\bigg( \\dfrac{1}{2} \\times 10 \\times 4 \\bigg)$|\n||= 70 + 20|\n||= {{{correctAnswer0}}} {{{suffix0}}}|\n\n</div>"}]},{"vars":[{"varval":"A plan of a first place pennant is shown below.\n\nsm_img https://teacher.smartermaths.com.au/wp-content/uploads/2022/08/Measurement_NAPX-I4-NC22-SA_v5.svg 600 indent vpad\n\nJax wants to make these pennants for the local pony club and needs to know the total area.\n\n\r\n\r\nWhat is the total area of the first place pennant, in square centimetres?"},{"varval":"sm_img https://teacher.smartermaths.com.au/wp-content/uploads/2022/08/Measurement_NAPX-I4-NC22-SA_v5_ws_1.svg 600 indent vpad\n\n<div class=\"aligned\">\n\n|||\n|-|-|\n| Area| = square + triangle|\n||= (5 × 20) + $\\bigg( \\dfrac{1}{2} \\times 5 \\times 7 \\bigg)$|\n||= 100 + 17.5|\n||= {{{correctAnswer0}}} {{{suffix0}}}|\n\n</div>"}]}]

  553. Awaiting full svg images for v1-5

    <div class="sm_mode"> {{{question}}} </div>

    [{"vars":[{"varval":"Jerry cut a golf ball into two halves.\n\n<br>\n\nsm_img https://teacher.smartermaths.com.au/wp-content/uploads/2018/07/NAPX-G4-CA22-SA_1.svg 200 indent3 vpad\n\nThe following calculation gives the approximate volume of one half of the ball in cm$^3$.\n\n>>Volume = $\\dfrac{1}{2} \\times \\dfrac{4}{3} \\large \\pi$ $\\times\\ \\large r$$^3,$ &nbsp&nbsp;where &nbsp;$\\large \\pi$ = 3.14\n\n<br>\n\nWhat volume does the calculation give, to the nearest cm³, where $\\large r$ is the radius of the ball?\n"},{"varval":"sm_nogap Radius = 2 cm\n\n<br>\n<div class=\"aligned\">\n\n|||\n|-|-|\n|$\\therefore V$| = $\\dfrac{1}{2} \\times \\dfrac{4}{3} \\times \\large \\pi$ $\\times\\ 2^3$|\n||= 16.74...|\n||$\\approx$ {{{correctAnswer0}}} {{{suffix0}}}|\n\n</div>"}]},{"vars":[{"varval":"Patty cut a basketball into two halves.\n\n<br>\n\nsm_img https://teacher.smartermaths.com.au/wp-content/uploads/2022/08/Measurement_NAPX-G4-CA22-SA_basketball_v1.svg 190 indent3 vpad\n\nThe following calculation gives the approximate volume of one half of the ball in cm$^3$.\n\n>>Volume = $\\dfrac{1}{2} \\times \\dfrac{4}{3} \\large \\pi$ $\\times\\ \\large r$$^3,$ &nbsp&nbsp;where &nbsp;$\\large \\pi$ = 3.14\n\n<br>\n\nWhat volume does the calculation give, to the nearest cm³, where $\\large r$ is the radius of the ball?\n"},{"varval":"sm_nogap Radius = 12 cm \n\n<br>\n<div class=\"aligned\">\n\n|||\n|-|-|\n|$\\therefore V$| = $\\dfrac{1}{2} \\times \\dfrac{4}{3} \\times \\large \\pi$ $\\times\\ 12^3$|\n||= 3617.28|\n||$\\approx$ {{{correctAnswer0}}} {{{suffix0}}}|\n\n</div>"}]},{"vars":[{"varval":"Ricky cut a cricket ball into two halves.\n\n<br>\n\nsm_img https://teacher.smartermaths.com.au/wp-content/uploads/2022/08/Measurement_NAPX-G4-CA22-SA_cricketball_1.svg 200 indent3 vpad\n\nThe following calculation gives the approximate volume of one half of the ball in mm$^3$.\n\n>>Volume = $\\dfrac{1}{2} \\times \\dfrac{4}{3} \\large \\pi$ $\\times\\ \\large r$$^3,$ &nbsp&nbsp;where &nbsp;$\\large \\pi$ = 3.14\n\n<br>\n\nWhat volume does the calculation give, to the nearest cm³, where $\\large r$ is the radius of the ball?\n"},{"varval":"Using &nbsp;1 cm = 10 mm\n\nsm_nogap Radius = 3.6 cm \n\n<br>\n<div class=\"aligned\">\n\n|||\n|-|-|\n|$\\therefore V$| = $\\dfrac{1}{2} \\times \\dfrac{4}{3} \\times \\large \\pi$ $\\times\\ 3.6^3$|\n||= 97.66...|\n||$\\approx$ {{{correctAnswer0}}} {{{suffix0}}}|\n\n</div>"}]},{"vars":[{"varval":"Becky imagined cutting a beach ball into two halves.\n\n<br>\n\nsm_img https://teacher.smartermaths.com.au/wp-content/uploads/2022/08/Measurement_NAPX-G4-CA22-SA_beachball_1a.svg 170 indent3 vpad\n\nThe following calculation gives the approximate volume of one half of the ball in cm$^3$.\n\n>>Volume = $\\dfrac{1}{2} \\times \\dfrac{4}{3} \\large \\pi$ $\\times\\ \\large r$$^3,$ &nbsp&nbsp;where &nbsp;$\\large \\pi$ = 3.14\n\n<br>\n\nWhat volume does the calculation give, to the nearest cm³, where $\\large r$ is the radius of the ball?\n"},{"varval":"sm_nogap Radius = 25 cm \n\n<br>\n<div class=\"aligned\">\n\n|||\n|-|-|\n|$\\therefore V$| = $\\dfrac{1}{2} \\times \\dfrac{4}{3} \\times \\large \\pi$ $\\times\\ 25^3$|\n||= $32\\ 708.33...$|\n||$\\approx$ $32\\ 708$ cm$^3$|\n\n</div>"}]},{"vars":[{"varval":"Diego cut a soccer ball into two halves.\n\n<br>\n\nsm_img https://teacher.smartermaths.com.au/wp-content/uploads/2022/08/Measurement_NAPX-G4-CA22-SA_soccerball_1.svg 170 indent3 vpad\n\nThe following calculation gives the approximate volume of one half of the ball in cm$^3$.\n\n>>Volume = $\\dfrac{1}{2} \\times \\dfrac{4}{3} \\large \\pi$ $\\times\\ \\large r$$^3,$ &nbsp&nbsp;where &nbsp;$\\large \\pi$ = 3.14\n\n<br>\n\nWhat volume does the calculation give, to the nearest cm³, where $\\large r$ is the radius of the ball?"},{"varval":"sm_nogap Radius = 11 cm \n\n<br>\n<div class=\"aligned\">\n\n|||\n|-|-|\n|$\\therefore V$| = $\\dfrac{1}{2} \\times \\dfrac{4}{3} \\times \\large \\pi$ $\\times\\ 11^3$|\n||= 2786.22...|\n||$\\approx$ {{{correctAnswer0}}} {{{suffix0}}}|\n\n</div>"}]},{"vars":[{"varval":"Cher cut a mirror ball into two halves.\n\n<br>\n\nsm_img https://teacher.smartermaths.com.au/wp-content/uploads/2022/08/Measurement_NAPX-G4-CA22-SA_mirrorball_1.svg 170 indent3 vpad\n\nThe following calculation gives the approximate volume of one half of the ball in cm$^3$.\n\n>>Volume = $\\dfrac{1}{2} \\times \\dfrac{4}{3} \\large \\pi$ $\\times\\ \\large r$$^3,$ &nbsp&nbsp;where &nbsp;$\\large \\pi$ = 3.14\n\n<br>\n\nWhat volume does the calculation give, to the nearest cm³, where $\\large r$ is the radius of the ball?"},{"varval":"sm_nogap Radius = 40 cm \n\n<br>\n<div class=\"aligned\">\n\n|||\n|-|-|\n|$\\therefore V$| = $\\dfrac{1}{2} \\times \\dfrac{4}{3} \\times \\large \\pi$ $\\times\\ 40^3$|\n||= 133 973.33...|\n||= 133 973 {{{suffix0}}}|\n\n</div>"}]}]

  554. <div class="sm_mode"> {{{question}}} </div>

    [{"vars":[{"varval":"Brigit made a shape using 6 identical squares.\n\n<br>\n\nsm_img https://teacher.smartermaths.com.au/wp-content/uploads/2018/06/NAPX-J3-CA34-SA.svg 170 indent3 vpad\n\n<br>What is the perimeter of the shape?\n"},{"varval":"<div class=\"aligned\">\n\n| | |\n| --------------------- | -------------- |\n| Side length of 1 square | \\= $\\dfrac{24}{3}$ |\n| | \\= 8 cm |\n\n</div>\n\n<br>\n\n<div class=\"aligned\">\n\n| | |\n| --------------------- | -------------- |\n| Perimeter | \\= (10 × 8) + (4 × 4)|\n| | \\= {{{correctAnswer0}}} {{{suffix0}}} |\n\n</div>"}]},{"vars":[{"varval":"Carl made a shape using 10 identical squares.\n\n<br>\n\nsm_img https://teacher.smartermaths.com.au/wp-content/uploads/2022/08/Measurement_NAPX-J3-CA34-SA_v1.svg 300 indent3 vpad\n\n<br>What is the perimeter of the shape?\n"},{"varval":"<div class=\"aligned\">\n\n| | |\n| --------------------- | -------------- |\n| Side length of 1 square | \\= $\\dfrac{44}{4}$ |\n| | \\= 11 cm |\n\n</div>\n\n<br>\n\n<div class=\"aligned\">\n\n| | |\n| --------------------- | -------------- |\n| Perimeter | \\= (13 × 11) + (6 × 5.5)|\n| | \\= {{{correctAnswer0}}} {{{suffix0}}} |\n\n</div>"}]},{"vars":[{"varval":"Adele made a shape using 15 identical squares.\n\n<br>\n\nsm_img https://teacher.smartermaths.com.au/wp-content/uploads/2022/08/Measurement_NAPX-J3-CA34-SA_v2.svg 290 indent3 vpad\n\n<br>What is the perimeter of the shape?\n"},{"varval":"<div class=\"aligned\">\n\n| | |\n| --------------------- | -------------- |\n| Side length of 1 square | \\= $\\dfrac{90}{5}$ |\n| | \\= 18 cm |\n\n</div>\n\n<br>\n\n<div class=\"aligned\">\n\n| | |\n| --------------------- | -------------- |\n| Perimeter | \\= (16 × 18) + (8 × 9)|\n| | \\= {{{correctAnswer0}}} {{{suffix0}}} |\n\n</div>"}]},{"vars":[{"varval":"Michael made a shape using 21 identical squares.\n\n<br>\n\nsm_img https://teacher.smartermaths.com.au/wp-content/uploads/2022/08/Measurement_NAPX-J3-CA34-SA_v3.svg 320 indent3 vpad\n\n<br>What is the perimeter of the shape?\n"},{"varval":"<div class=\"aligned\">\n\n| | |\n| --------------------- | -------------- |\n| Side length of 1 square | \\= $\\dfrac{54}{6}$ |\n| | \\= 9 cm |\n\n</div>\n\n<br>\n\n<div class=\"aligned\">\n\n| | |\n| --------------------- | -------------- |\n| Perimeter | \\= (19 × 9) + (10 × 4.5)|\n| | \\= {{{correctAnswer0}}} {{{suffix0}}} |\n\n</div>"}]},{"vars":[{"varval":"Elton made a shape using 10 identical squares.\n\n<br>\n\nsm_img https://teacher.smartermaths.com.au/wp-content/uploads/2022/08/Measurement_NAPX-J3-CA34-SA_v4.svg 270 indent3 vpad\n\n<br>What is the perimeter of the shape?"},{"varval":"<div class=\"aligned\">\n\n| | |\n| --------------------- | -------------- |\n| Side length of 1 square | \\= $\\dfrac{16.8}{4}$ |\n| | \\= 4.2 cm |\n\n</div>\n\n<br>\n\n<div class=\"aligned\">\n\n| | |\n| --------------------- | -------------- |\n| Perimeter | \\= (13 × 4.2) + (6 × 2.1)|\n| | \\= {{{correctAnswer0}}} {{{suffix0}}} |\n\n</div>"}]},{"vars":[{"varval":"Dione made a shape using 10 identical squares.\n\n<br>\n\nsm_img https://teacher.smartermaths.com.au/wp-content/uploads/2022/08/Measurement_NAPX-J3-CA34-SA_v5.svg 270 indent2 vpad\n\n<br>What is the perimeter of the shape?"},{"varval":"<div class=\"aligned\">\n\n| | |\n| --------------------- | -------------- |\n| Side length of 1 square | \\= $\\dfrac{12.6}{3}$ |\n| | \\= 4.2 cm |\n\n</div>\n\n<br>\n\n<div class=\"aligned\">\n\n| | |\n| --------------------- | -------------- |\n| Perimeter | \\= (10 × 4.2) + (4 × 2.1)|\n| | \\= {{{correctAnswer0}}} {{{suffix0}}} |\n\n</div>"}]}]

  555. <div class="sm_mode"> {{{question}}} </div>

    [{"vars":[{"varval":"The rectangular prism, pictured below, has a volume of 19 000 cm³ and a height of 25 cm.\n\n<br>\n\nsm_img https://teacher.smartermaths.com.au/wp-content/uploads/2018/06/NAPX-I3-CA252.svg 240 indent3 vpad\n\n<br>What could be the length and width of the prism?\n"},{"varval":"<div class=\"aligned\">\n\n| | |\n| --------------------: | -------------- |\n| $V$ | \\= $\\large l$ $\\times\\ \\large w$ $\\times\\ \\large h$|\n| 19 000 | \\= $\\large l$ $\\times\\ \\large w$ $\\times\\ 25$|\n| 760| \\= $\\large l$ $\\times\\ \\large w$|\n|| \\= 40 $\\times$ 19 $\\checkmark$ (check options)|\n\n</div>\n\n<br>\n\n$\\therefore$ {{{correctAnswer}}}\n"}]},{"vars":[{"varval":"The rectangular prism, pictured below, has a volume of 13 056 cm³ and a height of 32 cm.\n\n<br>\n\nsm_img https://teacher.smartermaths.com.au/wp-content/uploads/2022/08/Measurement_NAPX-I3-CA25_v1.svg 300 indent3 vpad\n\n<br>What could be the length and width of the prism?\n"},{"varval":"<div class=\"aligned\">\n\n| | |\n| --------------------: | -------------- |\n| $V$ | \\= $\\large l$ $\\times\\ \\large w$ $\\times\\ \\large h$|\n| 13 056 | \\= $\\large l$ $\\times\\ \\large w$ $\\times\\ 32$|\n| 408| \\= $\\large l$ $\\times\\ \\large w$|\n|| \\= 24 $\\times$ 17 $\\checkmark$ (check options)|\n\n\n</div>\n\n<br>\n\n$\\therefore$ {{{correctAnswer}}}\n"}]},{"vars":[{"varval":"The rectangular prism, pictured below, has a volume of 6555 cm³ and a length of 23 cm.\n\n<br>\n\nsm_img https://teacher.smartermaths.com.au/wp-content/uploads/2022/08/Measurement_NAPX-I3-CA25_v2.svg 260 indent3 vpad\n\n<br>What could be the height and width of the prism?\n"},{"varval":"<div class=\"aligned\">\n\n| | |\n| --------------------: | -------------- |\n| $V$ | \\= $\\large l$ $\\times\\ \\large w$ $\\times\\ \\large h$|\n| 6555 | \\= 23 $\\times\\ \\large w$ $\\times\\large h$|\n| 285| \\= $\\large w$ $\\times\\ \\large h$|\n|| \\= 19 $\\times$ 15 $\\checkmark$ (check options)|\n\n\n</div>\n\n<br>\n\n$\\therefore$ {{{correctAnswer}}}\n"}]},{"vars":[{"varval":"The rectangular prism, pictured below, has a volume of 56 848 m³ and a height of 22 m.\n\n<br>\n\nsm_img https://teacher.smartermaths.com.au/wp-content/uploads/2022/08/Measurement_NAPX-I3-CA25_v3.svg 420 indent3 vpad\n\n<br>What could be the length and width of the prism?\n"},{"varval":"<div class=\"aligned\">\n\n| | |\n| --------------------: | -------------- |\n| $V$ | \\= $\\large l$ $\\times\\ \\large w$ $\\times\\ \\large h$|\n| 56 848 | \\= $\\large l$ $\\times\\ \\large w$ $\\times\\ 22$|\n| 2584| \\= $\\large l$ $\\times\\ \\large w$|\n|| \\= 65 $\\times$ 38 $\\checkmark$ (check options)|\n\n\n</div>\n\n<br>\n\n$\\therefore$ {{{correctAnswer}}}\n"}]},{"vars":[{"varval":"The rectangular prism, pictured below, has a volume of 1200 m³ and a length of 10 m.\n\n<br>\n\nsm_img https://teacher.smartermaths.com.au/wp-content/uploads/2022/08/Measurement_NAPX-I3-CA25_v4.svg 250 indent3 vpad\n\n<br>What could be the width and height of the prism?\n"},{"varval":"<div class=\"aligned\">\n\n| | |\n| --------------------: | -------------- |\n| $V$ | \\= $\\large l$ $\\times\\ \\large w$ $\\times\\ \\large h$|\n| 1200 | \\= 10 $\\times\\ \\large w$ $\\times\\ \\large h$|\n| 120| \\= $\\large w$ $\\times\\ \\large h$|\n|| \\= 8 $\\times$ 15 $\\checkmark$ (check options)|\n\n\n</div>\n\n<br>\n\n$\\therefore$ {{{correctAnswer}}}\n"}]},{"vars":[{"varval":"The rectangular prism, pictured below, has a volume of 392 mm³ and a length of 14 mm.\n\n<br>\n\nsm_img https://teacher.smartermaths.com.au/wp-content/uploads/2022/08/Measurement_NAPX-I3-CA25_v5.svg 370 indent3 vpad\n\n<br>What could be the width and height of the prism?\n"},{"varval":"<div class=\"aligned\">\n\n| | |\n| --------------------: | -------------- |\n| $V$ | \\= $\\large l$ $\\times\\ \\large w$ $\\times\\ \\large h$|\n| 392 | \\= 14 $\\times\\ \\large w$ $\\times\\ \\large h$|\n| 28| \\= $\\large w$ $\\times\\ \\large h$|\n|| \\= 7 $\\times$ 4 $\\checkmark$ (check options)|\n\n\n</div>\n\n<br>\n\n$\\therefore$ {{{correctAnswer}}}\n"}]}]

  556. <div class="sm_mode"> {{{question}}} </div>

    [{"vars":[{"varval":"A cube has a side length of 8 cm.\n\r\nTwo smaller cubes of side length 4 cm are attached to the larger cube as shown in the diagram below.\n\n<br>\n\nsm_img https://teacher.smartermaths.com.au/wp-content/uploads/2017/01/naplan-2013-21mc-300x207.png 240 indent vpad\n\n<br>Including the base, what is the surface area of the new object?"},{"varval":"One strategy:\n\n\r\nCalculate the surface area (S.A.) of each object\r\nthen deduct the faces not showing.\r\n \r\n\r\nS.A. (large cube) = 6 × 8 × 8 = 384 cm$^2$\n\n\r\nS.A. (small cube) = 6 × 4 × 4 = 96 cm$^2$\n\n\r\nS.A. (sides not showing) = 6 × (4 × 4) = 96 cm$^2$\r\n \r\n\r\nsm_nogap $\\therefore$ Total S.A.\n\n<div class=\"aligned\">\n\n>>||\n|-|\n|= 384 + (2 × 96) − 96|\n|= {{{correctAnswer}}}|\n\r\n\r\n\n</div>"}]},{"vars":[{"varval":"A cube has a side length of 6 cm.\n\r\nFour smaller cubes of side length 3 cm are attached to the larger cube as shown in the diagram below.\n\n<br>\n\nsm_img https://teacher.smartermaths.com.au/wp-content/uploads/2022/08/Measurement_NAPX-F4-CA21_v1.svg 240 indent vpad\n\n<br>Including the base, what is the surface area of the new object?"},{"varval":"One strategy:\n\n\r\nCalculate the surface area (S.A.) of each object\r\nthen deduct the faces not showing.\r\n \r\n\r\nS.A. (large cube) = 6 × 6 × 6 = 216 cm$^2$\n\n\r\nS.A. (small cube) = 6 × 3 × 3 = 54 cm$^2$\n\n\r\nS.A. (sides not showing) = 12 × (3 × 3) = 108 cm$^2$\r\n \r\n\r\nsm_nogap $\\therefore$ Total S.A.\n\n<div class=\"aligned\">\n\n>>||\n|-|\n|= 216 + (4 × 54) − 108|\n|= {{{correctAnswer}}}|\n\r\n\r\n\n</div>"}]},{"vars":[{"varval":"A cube has a side length of 20 cm.\n\r\nTwo smaller cubes of side length 10 cm are attached to the larger cube as shown in the diagram below.\n\n<br>\n\nsm_img https://teacher.smartermaths.com.au/wp-content/uploads/2022/08/Measurement_NAPX-F4-CA21_v2.svg 240 indent vpad\n\n<br>Including the base, what is the surface area of the new object?"},{"varval":"One strategy:\n\n\r\nCalculate the surface area (S.A.) of each object\r\nthen deduct the faces not showing.\r\n \r\n\r\nS.A. (large cube) = 6 × 20 × 20 = 2400 cm$^2$\n\n\r\nS.A. (small cube) = 6 × 10 × 10 = 600 cm$^2$\n\n\r\nS.A. (sides not showing) = 6 × (10 × 10) = 600 cm$^2$\r\n \r\n\r\nsm_nogap $\\therefore$ Total S.A.\n\n<div class=\"aligned\">\n\n>>||\n|-|\n|= 2400 + (2 × 600) − 600|\n|= {{{correctAnswer}}}|\n\r\n\r\n\n</div>"}]},{"vars":[{"varval":"A cube has a side length of 10 cm.\n\r\nFour smaller cubes of side length 5 cm are attached to the larger cube as shown in the diagram below.\n\n<br>\n\nsm_img https://teacher.smartermaths.com.au/wp-content/uploads/2022/08/Measurement_NAPX-F4-CA21_v3.svg 240 indent vpad\n\n<br>Including the base, what is the surface area of the new object?"},{"varval":"One strategy:\n\n\r\nCalculate the surface area (S.A.) of each object\r\nthen deduct the faces not showing.\r\n \r\n\r\nS.A. (large cube) = 6 × 10 × 10 = 600 cm$^2$\n\n\r\nS.A. (small cube) = 6 × 5 × 5 = 150 cm$^2$\n\n\r\nS.A. (sides not showing) = 12 × (5 × 5) = 300 cm$^2$\r\n \r\n\r\nsm_nogap $\\therefore$ Total S.A.\n\n<div class=\"aligned\">\n\n>>||\n|-|\n|= 600 + (4 × 150) − 300|\n|= {{{correctAnswer}}}|\n\r\n\r\n\n</div>"}]},{"vars":[{"varval":"A cube has a side length of 14 cm.\n\r\nTwo smaller cubes of side length 7 cm are attached to the larger cube as shown in the diagram below.\n\n<br>\n\nsm_img https://teacher.smartermaths.com.au/wp-content/uploads/2022/08/Measurement_NAPX-F4-CA21_v4.svg 240 indent vpad\n\n<br>Including the base, what is the surface area of the new object?"},{"varval":"One strategy:\n\n\r\nCalculate the surface area (S.A.) of each object\r\nthen deduct the faces not showing.\r\n \r\n\r\nS.A. (large cube) = 6 × 14 × 14 = 1176 cm$^2$\n\n\r\nS.A. (small cube) = 6 × 7 × 7 = 294 cm$^2$\n\n\r\nS.A. (sides not showing) = 4 × (7 × 7) = 196 cm$^2$\r\n \r\n\r\nsm_nogap $\\therefore$ Total S.A.\n\n<div class=\"aligned\">\n\n>>||\n|-|\n|= 1176 + (2 × 294) − 196|\n|= {{{correctAnswer}}}|\n\r\n\r\n\n</div>"}]},{"vars":[{"varval":"A cube has a side length of 8 cm.\n\r\nThree smaller cubes of side length 4 cm are attached to the larger cube as shown in the diagram below.\n\n<br>\n\nsm_img https://teacher.smartermaths.com.au/wp-content/uploads/2022/08/Measurement_NAPX-F4-CA21_v5.svg 240 indent vpad\n\n<br>Including the base, what is the surface area of the new object?"},{"varval":"One strategy:\n\n\r\nCalculate the surface area (S.A.) of each object\r\nthen deduct the faces not showing.\r\n \r\n\r\nS.A. (large cube) = 6 × 8 × 8 = 384 cm$^2$\n\n\r\nS.A. (small cube) = 6 × 4 × 4 = 96 cm$^2$\n\n\r\nS.A. (sides not showing) = 6 × (4 × 4) = 96 cm$^2$\r\n \r\n\r\nsm_nogap $\\therefore$ Total S.A.\n\n<div class=\"aligned\">\n\n>>||\n|-|\n|= 384 + (3 × 96) − 96|\n|= {{{correctAnswer}}}|\n\r\n\r\n\n</div>"}]}]

  557. <div class="sm_mode"> {{{question}}} </div>

    [{"vars":[{"varval":"A child's bicycle wheel has a radius of 260 mm.\n\nsm_img https://teacher.smartermaths.com.au/wp-content/uploads/2018/04/NAPX-I4-CA17.svg 200 indent vpad\n\nThe wheel turns 8 times.\n\n\r\n\r\nUsing &nbsp;$\\large \\pi$ = 3.14, approximately how many metres has the bicycle travelled?\n"},{"varval":"Using &nbsp;1 m = 1000 mm\n\nsm_nogap Distance (8 circumferences)\n\n<div class=\"aligned\">\n\n>>||\n|-|\n|= 8 × (2$\\large \\pi r$)|\n|= 8 × 2 × $\\large \\pi$ × 0.26|\n|= 13.06... m|\n|= {{{correctAnswer}}} (nearest metre)|\n\n</div>"}]},{"vars":[{"varval":"A child's bicycle wheel has a radius of 365 mm.\n\nsm_img https://teacher.smartermaths.com.au/wp-content/uploads/2022/08/Measurement_NAPX-I4-CA17_v1.svg 200 indent vpad\n\nThe wheel turns 5 times.\n\n\r\n\r\nUsing &nbsp;$\\large \\pi$ = 3.14, approximately how many metres has the bicycle travelled?\n"},{"varval":"Using &nbsp;1 m = 1000 mm\n\nsm_nogap Distance (5 circumferences)\n\n<div class=\"aligned\">\n\n>>||\n|-|\n|= 5 × (2$\\large \\pi r$)|\n|= 5 × 2 × $\\large \\pi$ × 0.365|\n|= 11.461 m|\n|= {{{correctAnswer}}} (nearest metre)|\n\n</div>"}]},{"vars":[{"varval":"A child's scooter wheel has a radius of 130 mm.\n\nsm_img https://teacher.smartermaths.com.au/wp-content/uploads/2022/08/Measurement_NAPX-I4-CA17_v2.svg 200 indent vpad\n\nThe wheel turns 15 times.\n\n\r\n\r\nUsing &nbsp;$\\large \\pi$ = 3.14, approximately how many metres has the scooter travelled?\n"},{"varval":"Using &nbsp;1m = 1000 mm\n\nsm_nogap Distance (15 circumferences)\n\n<div class=\"aligned\">\n\n>>||\n|-|\n|= 15 × (2$\\large \\pi r$)|\n|= 15 × 2 × $\\large \\pi$ × 0.13|\n|= 12.246 m|\n|= {{{correctAnswer}}} (nearest metre)|\n\n</div>"}]},{"vars":[{"varval":"A wheelchair has a wheel that has a radius of 400 mm.\n\nsm_img https://teacher.smartermaths.com.au/wp-content/uploads/2022/08/Measurement_NAPX-I4-CA17_v3.svg 200 indent vpad\n\nThe wheel turns 125 times.\n\n\r\n\r\nApproximately how many metres has the wheelchair travelled?\n"},{"varval":"Using &nbsp;1m = 1000 mm\n\nsm_nogap Distance (125 circumferences)\n\n<div class=\"aligned\">\n\n>>||\n|-|\n|= 125 × (2$\\large \\pi r$)|\n|= 125 × 2 × $\\large \\pi$ × 0.4|\n|= {{{correctAnswer}}} |\n\n</div>"}]},{"vars":[{"varval":"A mountain bike wheel has a radius of 330 mm.\r\n\r\n\nsm_img https://teacher.smartermaths.com.au/wp-content/uploads/2022/08/Measurement_NAPX-I4-CA17_v4.svg 200 indent vpad\n\nThe wheel turns 48 times.\n\n\r\n\r\nUsing &nbsp;$\\large \\pi$ = 3.14, approximately how many metres has the mountain bike travelled?\n"},{"varval":"sm_nogap Distance (48 circumferences)\n\n<div class=\"aligned\">\n\n>>||\n|-|\n|= 48 × (2$\\large \\pi r$)|\n|= 48 × 2 × $\\large \\pi$ × 0.33|\n|= 99.4752 m|\n|= {{{correctAnswer}}} (nearest metre)|\n\n</div>"}]},{"vars":[{"varval":"A road bike has a wheel that has a radius of 360 mm.\n\nsm_img https://teacher.smartermaths.com.au/wp-content/uploads/2022/08/Measurement_NAPX-I4-CA17_v5.svg 200 indent vpad\n\nThe wheel turns 100 times.\n\n\r\n\r\nUsing &nbsp;$\\large \\pi$ = 3.14, approximately how many metres has the road bike travelled?\n"},{"varval":"Using &nbsp;1m = 1000 mm\n\nsm_nogap Distance (100 circumferences)\n\n<div class=\"aligned\">\n\n>>||\n|-|\n|= 100 × (2$\\large \\pi r$)|\n|= 100 × 2 × $\\large \\pi$ × 0.36|\n|= 226.08 m|\n|= {{{correctAnswer}}} (nearest metre)|\n\n</div>"}]}]

  558. Measurement, NAPX-L4-CA14 o1

    <div class="sm_mode"> {{{question}}} </div>

    [{"vars":[{"varval":"Clarence used the tile pictured below to pave his garden path.\n\r\nAltogether, he used 1000 tiles.\n\n<br>\n\nsm_img https://teacher.smartermaths.com.au/wp-content/uploads/2022/08/Measurement_NAPX-L4-CA14-o2_v0_nts.svg 250 indent vpad\n\n<br>What is the total area of Clarence's garden path in square metres?"},{"varval":"<div class=\"sm_mode\">\n\nConvert cm to metres:\n\n\r\n10 cm = 0.1 m\n\n\r\n5 cm = 0.05 m\n\r\n\n<div class=\"aligned\">\n\n|||\n|-|-|\n|Area of 1 tile| = 0.1$^2$ − 0.05$^2$|\n||= 0.0075 m$^2$|\n\n</div>\n\n<br>\n\n<div class=\"aligned\">\n\n|||\n|-|-|\n|$\\therefore$ Area of garden path| = 0.0075 × 1000|\n||= {{{correctAnswer}}}|\n\n</div>\n\n</div>\n\r\n\r\n\r"}]},{"vars":[{"varval":"Aaron used the tile pictured below to pave his back deck.\n\r\nAltogether, he used 1000 tiles.\n\n<br>\n\nsm_img https://teacher.smartermaths.com.au/wp-content/uploads/2022/08/Measurement_NAPX-L4-CA14-o2_v1_nts.svg 250 indent vpad\n\n<br>What is the total area of Aaron's back deck in square metres?"},{"varval":"<div class=\"sm_mode\">\n\nConvert cm to metres:\n\n\r\n14 cm = 0.14 m\n\n\r\n7 cm = 0.07 m\n\r\n\n<div class=\"aligned\">\n\n|||\n|-|-|\n|Area of 1 tile| = 0.14$^2$ − 0.07$^2$|\n||= 0.0147 m$^2$|\n\n</div>\n\n<br>\n\n<div class=\"aligned\">\n\n|||\n|-|-|\n|$\\therefore$ Area of deck| = 0.0147 × 1000|\n||= {{{correctAnswer}}}|\n\n</div>\n\n</div>\n\n\r\n\r\n\r"}]},{"vars":[{"varval":"Polly used the tile pictured below to tile her loungeroom floor.\n\r\nAltogether, she used 150 tiles.\n\n<br>\n\nsm_img https://teacher.smartermaths.com.au/wp-content/uploads/2022/08/Measurement_NAPX-L4-CA14-o2_v2_nts.svg 370 indent vpad\n\n<br>What is the total area of Polly's loungeroom in square metres?"},{"varval":"<div class=\"sm_mode\">\n\nConvert cm to metres:\n\n\r\n30 cm = 0.3 m\n\n\r\n15 cm = 0.15 m\n\r\n\n<div class=\"aligned\">\n\n|||\n|-|-|\n|Area of 1 tile| = 0.3$^2$ − 0.15$^2$|\n||= 0.0675 m$^2$|\n\n</div>\n\n<br>\n\n<div class=\"aligned\">\n\n|||\n|-|-|\n|$\\therefore$ Area of loungeroom | = 0.0675 × 150|\n||= {{{correctAnswer}}}|\n\n</div>\n\n</div>"}]},{"vars":[{"varval":"Ainsley used the tile pictured below to tile her kitchen floor.\n\r\nAltogether, she used 1500 tiles.\n\n<br>\n\nsm_img https://teacher.smartermaths.com.au/wp-content/uploads/2022/08/Measurement_NAPX-L4-CA14-o2_v3_nts.svg 320 indent vpad\n\n<br>What is the total area of Ainsley's kitchen floor in square metres?"},{"varval":"<div class=\"sm_mode\">\n\nConvert cm to metres:\n\n\r\n8 cm = 0.08 m\n\n\r\n4 cm = 0.04 m\n\r\n\n<div class=\"aligned\">\n\n|||\n|-|-|\n|Area of 1 tile| = 0.08$^2$ − 0.04$^2$|\n||= 0.0048 m$^2$|\n\n</div>\n\n<br>\n\n<div class=\"aligned\">\n\n|||\n|-|-|\n|$\\therefore$ Area of kitchen | = 0.0048 × 1500|\n||= {{{correctAnswer}}}|\n\n</div>\n\n</div>"}]},{"vars":[{"varval":"Bora used the tile pictured below to tile his tool shed floor.\n\r\nAltogether, he used 1200 tiles.\n\n<br>\n\nsm_img https://teacher.smartermaths.com.au/wp-content/uploads/2022/08/Measurement_NAPX-L4-CA14-o2_v4_nts.svg 370 indent vpad\n\n<br>What is the total area of Bora's tool shed floor in square metres?"},{"varval":"<div class=\"sm_mode\">\n\nConvert cm to metres:\n\n\r\n6 cm = 0.06 m\n\n\r\n3 cm = 0.03 m\n\r\n\n<div class=\"aligned\">\n\n|||\n|-|-|\n|Area of 1 tile| = 0.06$^2$ − 0.03$^2$|\n||= 0.0027 m$^2$|\n\n</div>\n\n<br>\n\n<div class=\"aligned\">\n\n|||\n|-|-|\n|$\\therefore$ Area of tool shed | = 0.0027 × 1200|\n||= {{{correctAnswer}}}|\n\n</div>\n\n</div>"}]},{"vars":[{"varval":"Binky used the paver pictured below to pave her pool area.\n\r\nAltogether, she used 50 tiles.\n\n<br>\n\nsm_img https://teacher.smartermaths.com.au/wp-content/uploads/2022/08/Measurement_NAPX-L4-CA14-o2_v5_nts.svg 300 indent3 vpad\n\n<br>What is the total area of Binky's pool area in square metres?"},{"varval":"<div class=\"sm_mode\">\n\nConvert cm to metres:\n\n\r\n60 cm = 0.6 m\n\n\r\n30 cm = 0.3 m\n\r\n\n<div class=\"aligned\">\n\n|||\n|-|-|\n|Area of 1 paver| = 0.6$^2$ − 0.3$^2$|\n||= 0.27 m$^2$|\n\n</div>\n\n<br>\n\n<div class=\"aligned\">\n\n|||\n|-|-|\n|$\\therefore$ Area of pool area | = 0.27 × 50|\n||= {{{correctAnswer}}}|\n\n</div>\n\n</div>"}]}]

  559. Measurement, NAPX-L4-CA13 o1

    <div class="sm_mode"> {{{question}}} </div>

    [{"vars":[{"varval":"Kellogg creates a rectangular prism by stacking 8 identical triangular prisms together.\n\n<br>\n\nsm_img https://teacher.smartermaths.com.au/wp-content/uploads/2020/04/NAPX-LA-CA13-o2.svg 180 indent3 vpad\n\n<br>What is the volume of one triangular prism, in cubic centimetres?"},{"varval":"<div class=\"aligned\">\n\n| | |\n| --------:| -------------- |\n| Total volume\t | \\= 28 × 13 × 18 |\n| | \\= 6552 |\n\n</div>\n\n<br>\n\n<div class=\"aligned\">\n\n| | |\n| ---------------------: | -------------- |\n| $\\therefore$ Volume of 1 prism | \\= $\\dfrac{6552}{8}$ |\n| | \\= {{{correctAnswer0}}} {{{suffix0}}}|\n\n</div>"}]},{"vars":[{"varval":"Ellie creates a rectangular prism by stacking 8 identical triangular prisms together.\n\n<br>\n\nsm_img https://teacher.smartermaths.com.au/wp-content/uploads/2020/04/NAPX-LA-CA13-o1.svg 180 indent3 vpad\n\n<br>What is the volume of one triangular prism, in cubic centimetres?"},{"varval":"<div class=\"aligned\">\n\n| | |\n| --------------------- | -------------- |\n| Total volume\t | \\= 24 × 20 × 30 |\n| | \\= 14 400 cm$^3$ |\n\n</div>\n\n<br>\n\n<div class=\"aligned\">\n\n| | |\n| --------------------- | -------------- |\n| $\\therefore$ Volume of 1 prism | \\= $\\dfrac{14\\ 400}{8}$ |\n| | \\= {{{correctAnswer0}}} {{{suffix0}}} |\n\n</div>"}]},{"vars":[{"varval":"Groot creates a rectangular prism by stacking 12 identical triangular prisms together.\n\n<br>\n\nsm_img https://teacher.smartermaths.com.au/wp-content/uploads/2022/08/Measurement_NAPX-L4-CA13-o2.svg 320 indent vpad\n\n<br>What is the volume of one triangular prism, in cubic centimetres?"},{"varval":"<div class=\"aligned\">\n\n| | |\n| --------:| -------------- |\n| Total volume\t | \\= 9 × 4 × 5 |\n| | \\= 180 |\n\n</div>\n\n<br>\n\n<div class=\"aligned\">\n\n| | |\n| ---------------------: | -------------- |\n| $\\therefore$ Volume of 1 prism | \\= $\\dfrac{180}{12}$ |\n| | \\= {{{correctAnswer0}}} {{{suffix0}}}|\n\n</div>"}]},{"vars":[{"varval":"Sienna creates a rectangular prism by stacking 16 identical triangular prisms together.\n\n\n\nsm_img https://teacher.smartermaths.com.au/wp-content/uploads/2022/08/Measurement_NAPX-L4-CA13-o2_v3.svg 320 indent2 vpad\n\nWhat is the volume of one triangular prism, in cubic centimetres?"},{"varval":"<div class=\"aligned\">\n\n| | |\n| --------:| -------------- |\n| Total volume\t | \\= 36 × 8 × 12 |\n| | \\= 3456 |\n\n</div>\n\n<br>\n\n<div class=\"aligned\">\n\n| | |\n| ---------------------: | -------------- |\n| $\\therefore$ Volume of 1 prism | \\= $\\dfrac{3456}{16}$ |\n| | \\= {{{correctAnswer0}}} {{{suffix0}}}|\n\n</div>"}]},{"vars":[{"varval":"Simone creates a rectangular prism by stacking 24 identical triangular prisms together.\n\n<br>\n\nsm_img https://teacher.smartermaths.com.au/wp-content/uploads/2022/08/Measurement_NAPX-L4-CA13-o2_v4.svg 530 indent vpad\n\nWhat is the volume of one triangular prism, in cubic centimetres?"},{"varval":"<div class=\"aligned\">\n\n| | |\n| --------:| -------------- |\n| Total volume\t | \\= 14 × 21 × 10 |\n| | \\= 2940 |\n\n</div>\n\n<br>\n\n<div class=\"aligned\">\n\n| | |\n| ---------------------: | -------------- |\n| $\\therefore$ Volume of 1 prism | \\= $\\dfrac{2940}{24}$ |\n| | \\= {{{correctAnswer0}}} {{{suffix0}}}|\n\n</div>"}]},{"vars":[{"varval":"Celeste creates a rectangular prism by stacking 10 identical triangular prisms together.\n\n<br>\n\nsm_img https://teacher.smartermaths.com.au/wp-content/uploads/2022/08/Measurement_NAPX-L4-CA13-o2_v5a.svg 440 indent vpad\n\nWhat is the volume of one triangular prism, in cubic centimetres?"},{"varval":"<div class=\"aligned\">\n\n| | |\n| --------:| -------------- |\n| Total volume\t | \\= 7 × 28 × 12 |\n| | \\= 2352 |\n\n</div>\n\n<br>\n\n<div class=\"aligned\">\n\n| | |\n| ---------------------: | -------------- |\n| $\\therefore$ Volume of 1 prism | \\= $\\dfrac{2352}{10}$ |\n| | \\= {{{correctAnswer0}}} {{{suffix0}}}|\n\n</div>"}]}]

  560. <div class="sm_mode"> {{{question}}} </div>

    [{"vars":[{"varval":"Wyatt is designing a rectangular lawn for a landscaping project.\n\n\r\n\r\nHe wants the lawn to be at least 400 square metres.\n\n\r\n\r\nHe has marked out two sides, each 25.5 metres in length.\n\n\r\n\r\nWhat is the smallest possible length of each of the other two sides, rounded to one decimal place?"},{"varval":"sm_nogap Smallest possible length\n\n<div class=\"aligned\">\n\n>>||\n|-|\n|= 400 ÷ 25.5|\n|= 15.68…|\n|= {{{correctAnswer}}}|\n\n</div>\n\r\n\r\n \r<br>\n\r\nNote: this distance must be rounded up due to the\r\ncontext of the question (area “at least” 400 m$^2$)."}]},{"vars":[{"varval":"Ruth is designing a rectangular quilt for a sewing project.\n\n\r\n\r\nShe wants the quilt to be at least 83 700 square centimetres.\n\n\r\n\r\nShe has marked out two sides, each 274 centimetres in length.\n\n\r\n\r\nWhat is the smallest possible length of each of the other two sides, rounded to one decimal place?"},{"varval":"sm_nogap Smallest possible length\n\n<div class=\"aligned\">\n\n>>||\n|-|\n|= 83 700 ÷ 274|\n|= 305.47…|\n|= {{{correctAnswer}}}|\n\n</div>\n\r\n\r\n \r<br>\n\r\nNote: this distance must be rounded up due to the\r\ncontext of the question (area “at least” 83 700 cm$^2$)."}]},{"vars":[{"varval":"Jim is making a rectangular cut of plywood for a carpentry project.\n\n\r\n\r\nHe wants the plywood to be at least 7.36 square metres.\n\n\r\n\r\nHe has marked out two sides, each 2.4 metres in length.\n\n\r\n\r\nWhat is the smallest possible length of each of the other two sides, rounded to one decimal place?"},{"varval":"sm_nogap Smallest possible length\n\n<div class=\"aligned\">\n\n>>||\n|-|\n|= 7.36 ÷ 2.4|\n|= 3.066…|\n|= {{{correctAnswer}}}|\n\n</div>\n\r\n\r\n \r<br>\n\r\nNote: this distance must be rounded up due to the\r\ncontext of the question (area “at least” 7.36 m$^2$)."}]},{"vars":[{"varval":"Bryn is marking out a rectangular area for his stall at the markets.\n\n\r\n\r\nHe wants the area to be at least 80 square metres.\n\n\r\n\r\nHe has marked out two sides, each 12 metres in length.\n\n\r\n\r\nWhat is the smallest possible length of each of the other two sides, rounded to one decimal place?"},{"varval":"sm_nogap Smallest possible length\n\n<div class=\"aligned\">\n\n>>||\n|-|\n|= 80 ÷ 12|\n|= 6.66…|\n|= {{{correctAnswer}}}|\n\n</div>\n\r\n\r\n \r<br>\n\r\nNote: this distance must be rounded up due to the\r\ncontext of the question (area “at least” 80 m$^2$)."}]},{"vars":[{"varval":"Emile is marking out a rectangular area for his new pool.\n\n\r\n\r\nHe wants the area to be at least 45 square metres.\n\n\r\n\r\nHe has marked out two sides, each 9.5 metres in length.\n\n\r\n\r\nWhat is the smallest possible length of each of the other two sides, rounded to one decimal place?"},{"varval":"sm_nogap Smallest possible length\n\n<div class=\"aligned\">\n\n>>||\n|-|\n|= 45 ÷ 9.5|\n|= 4.73…|\n|= {{{correctAnswer}}}|\n\n</div>\n\r\n\r\n \r<br>\n\r\nNote: this distance must be rounded up due to the\r\ncontext of the question (area “at least” 45 m$^2$)."}]},{"vars":[{"varval":"Monique is painting a rectangular mural on a wall and is marking out its dimensions.\n\n\r\n\r\nShe wants the area of the mural to be at least 60 square metres.\n\n\r\n\r\nShe has marked out two sides, each 18 metres in length.\n\n\r\n\r\nWhat is the smallest possible length of each of the other two sides, rounded to one decimal place?"},{"varval":"sm_nogap Smallest possible length\n\n<div class=\"aligned\">\n\n>>||\n|-|\n|= 60 ÷ 18|\n|= 3.33…|\n|= {{{correctAnswer}}}|\n\n</div>\n\r\n\r\n \r<br>\n\r\nNote: this distance must be rounded up due to the\r\ncontext of the question (area “at least” 60 m$^2$)."}]}]

  561. <div class="sm_mode"> {{{question}}} </div>

    [{"vars":[{"varval":"A trapezium is constructed on a grid of 10 rectangles.\n\r\nEach rectangle measures 3 cm × 7 cm.\n\n<br>\n\nsm_img https://teacher.smartermaths.com.au/wp-content/uploads/2018/06/NAPX-F4-CA08.svg 240 indent3 vpad\n\n<br>What is the area of the trapezium?"},{"varval":"<div class=\"aligned\">\n\nStrategy 1\n\n|||\n|-|-|\n|Area 1 rectangle|= 3 × 7|\n||= 21 cm$^2$|\n\n</div>\n\n<br>\n\n<div class=\"aligned\">\n\n|||\n|-|-|\n|$\\therefore$ Total Area|= (6 × 21) + 2 triangles|\n||= 126 + 2 × $\\bigg( \\dfrac{1}{2} \\times 3 \\times 14 \\bigg)$|\n||= 126 + 42|\n||= {{{correctAnswer}}}|\n\n</div>\n\n<br>\n\nStrategy 2 (advanced)\n\n<div class=\"aligned\">\n\n|||\n|-|-|\n|Area of trapezium|= $\\dfrac{1}{2} \\times$ height $\\times\\ (\\large a$ + $\\large b$)|\n||= $\\dfrac{1}{2} \\times 14 \\times (9+15)$|\n||= $7 \\times 24$|\n||= {{{correctAnswer}}}|\n\n</div>\n\r\n"}]},{"vars":[{"varval":"A trapezium is constructed on a grid of 28 rectangles.\n\r\nEach rectangle measures 2 cm × 6 cm.\n\n<br>\n\nsm_img https://teacher.smartermaths.com.au/wp-content/uploads/2022/08/Measurement_NAPX-F4-CA08_v1.svg 240 indent3 vpad\n\n<br>What is the area of the trapezium?"},{"varval":"<div class=\"aligned\">\n\nStrategy 1\n\n|||\n|-|-|\n|Area 1 rectangle|= 2 × 6|\n||= 12 cm$^2$|\n\n</div>\n\n<br>\n\n<div class=\"aligned\">\n\n|||\n|-|-|\n|$\\therefore$ Total Area|= (12 × 12) + 2 triangles|\n||= 144 + 2 × $\\bigg( \\dfrac{1}{2} \\times 4 \\times 24 \\bigg)$|\n||= 144 + 96|\n||= {{{correctAnswer}}}|\n\n</div>\n\n<br>\n\nStrategy 2 (advanced)\n\n<div class=\"aligned\">\n\n|||\n|-|-|\n|Area of trapezium|= $\\dfrac{1}{2} \\times$ height $\\times\\ (\\large a$ + $\\large b$)|\n||= $\\dfrac{1}{2} \\times 24 \\times (14+6)$|\n||= $12 \\times 20$|\n||= {{{correctAnswer}}}|\n\n</div>\n\r\n"}]},{"vars":[{"varval":"A trapezium is constructed on a grid of 16 rectangles.\n\r\nEach rectangle measures 3 cm × 5 cm.\n\n<br>\n\nsm_img https://teacher.smartermaths.com.au/wp-content/uploads/2022/08/Measurement_NAPX-F4-CA08_v3.svg 240 indent3 vpad\n\n<br>What is the area of the trapezium?"},{"varval":"<div class=\"aligned\">\n\nStrategy 1\n\n|||\n|-|-|\n|Area 1 rectangle|= 5 × 3|\n||= 15 cm$^2$|\n\n</div>\n\n<br>\n\n<div class=\"aligned\">\n\n|||\n|-|-|\n|$\\therefore$ Total Area|= (8 × 15) + 2 triangles|\n||= 120 + 2 × $\\bigg( \\dfrac{1}{2} \\times 5 \\times 12 \\bigg)$|\n||= 120 + 60|\n||= {{{correctAnswer}}}|\n\n</div>\n\n<br>\n\nStrategy 2 (advanced)\n\n<div class=\"aligned\">\n\n|||\n|-|-|\n|Area of trapezium|= $\\dfrac{1}{2} \\times$ height $\\times\\ (\\large a$ + $\\large b$)|\n||= $\\dfrac{1}{2} \\times 12 \\times (20+10)$|\n||= $6 \\times 30$|\n||= {{{correctAnswer}}}|\n\n</div>\n\r\n"}]},{"vars":[{"varval":"A trapezium is constructed on a grid of 24 rectangles.\n\r\nEach rectangle measures 4 cm × 7 cm.\n\n<br>\n\nsm_img https://teacher.smartermaths.com.au/wp-content/uploads/2022/08/Measurement_NAPX-F4-CA08_v2.svg 240 indent3 vpad\n\n<br>What is the area of the trapezium?"},{"varval":"<div class=\"aligned\">\n\nStrategy 1\n\n|||\n|-|-|\n|Area 1 rectangle|= 4 × 7|\n||= 28 cm$^2$|\n\n</div>\n\n<br>\n\n<div class=\"aligned\">\n\n|||\n|-|-|\n|$\\therefore$ Total Area|= (18 × 28) + 2 triangles|\n||= 504 + 2 × $\\bigg( \\dfrac{1}{2} \\times 4 \\times 21 \\bigg)$|\n||= 504 + 84|\n||= {{{correctAnswer}}}|\n\n</div>\n\n<br>\n\nStrategy 2 (advanced)\n\n<div class=\"aligned\">\n\n|||\n|-|-|\n|Area of trapezium|= $\\dfrac{1}{2} \\times$ height $\\times\\ (\\large a$ + $\\large b$)|\n||= $\\dfrac{1}{2} \\times 21 \\times (32+24)$|\n||= $10.5 \\times 56$|\n||= {{{correctAnswer}}}|\n\n</div>\n\r\n"}]},{"vars":[{"varval":"A trapezium is constructed on a grid of 21 rectangles.\n\r\nEach rectangle measures 4 cm × 10 cm.\n\n<br>\n\nsm_img https://teacher.smartermaths.com.au/wp-content/uploads/2022/08/Measurement_NAPX-F4-CA08_v4.svg 240 indent3 vpad\n\n<br>What is the area of the trapezium?"},{"varval":"<div class=\"aligned\">\n\nStrategy 1\n\n<br>\n\nsm_img https://teacher.smartermaths.com.au/wp-content/uploads/2022/08/Measurement_NAPX-F4-CA08_v4_ws.svg 260 indent3 vpad\n\n|||\n|-|-|\n|Area 1 rectangle|= 4 × 10|\n||= 40 cm$^2$|\n\n</div>\n\n<br>\n\n<div class=\"aligned\">\n\n|||\n|-|-|\n|$\\therefore$ Total Area|= (9 × 40) + triangle 1 + triangle 2|\n||= 9 × 40 + $\\bigg( \\dfrac{1}{2} \\times 4 \\times 30 \\bigg)$ + $\\bigg( \\dfrac{1}{2} \\times 12 \\times 30 \\bigg)$|\n||= 360 + 60 + 180|\n||= {{{correctAnswer}}}|\n\n</div>\n\n<br>\n\nStrategy 2 (advanced)\n\n<div class=\"aligned\">\n\n|||\n|-|-|\n|Area of trapezium|= $\\dfrac{1}{2} \\times$ height $\\times\\ (\\large a$ + $\\large b$)|\n||= $\\dfrac{1}{2} \\times 30 \\times (28+12)$|\n||= $15 \\times 40$|\n||= {{{correctAnswer}}}|\n\n</div>\n\r\n"}]},{"vars":[{"varval":"A trapezium is constructed on a grid of 28 rectangles.\n\r\nEach rectangle measures 3 cm × 5 cm.\n\n<br>\n\nsm_img https://teacher.smartermaths.com.au/wp-content/uploads/2022/08/Measurement_NAPX-F4-CA08_v5.svg 240 indent3 vpad\n\n<br>What is the area of the trapezium?"},{"varval":"<div class=\"aligned\">\n\nStrategy 1\n<br>\n\nsm_img https://teacher.smartermaths.com.au/wp-content/uploads/2022/08/Measurement_NAPX-F4-CA08_v5_ws.svg 260 indent3 vpad\n\n|||\n|-|-|\n|Area 1 rectangle|= 3 × 5|\n||= 15 cm$^2$|\n\n</div>\n\n<br>\n\n<div class=\"aligned\">\n\n|||\n|-|-|\n|$\\therefore$ Total Area|= (8 × 15) + triangle 1 + triangle 2|\n||= 8 × 15 + $\\bigg( \\dfrac{1}{2} \\times 9 \\times 20 \\bigg)$ + $\\bigg( \\dfrac{1}{2} \\times 6 \\times 20 \\bigg)$|\n||= 120 + 90 + 60|\n||= {{{correctAnswer}}}|\n\n</div>\n\n<br>\n\nStrategy 2 (advanced)\n\n<div class=\"aligned\">\n\n|||\n|-|-|\n|Area of trapezium|= $\\dfrac{1}{2} \\times$ height $\\times\\ (\\large a$ + $\\large b$)|\n||= $\\dfrac{1}{2} \\times 20 \\times (21+6)$|\n||= $10 \\times 27$|\n||= {{{correctAnswer}}}|\n\n</div>\n\r\n"}]}]

  562. <div class="sm_mode"> A large box is square on the bottom and twice as high as it is long. <br> sm_img https://teacher.smartermaths.com.au/wp-content/uploads/2018/06/NAPX-H4-CA071.svg 220 indent3 vpad <br>Which of these is the closest estimate for the maximum number of smaller cubes that could be packed into the large box? </div>

    [{"vars":null}]

  563. <div class="sm_mode"> {{{question}}} </div>

    [{"vars":[{"varval":"Two circles have radii of different lengths.\n\n<br>\n\nsm_img https://teacher.smartermaths.com.au/wp-content/uploads/2021/05/RAPH10_62.svg 280 indent vpad\n\n<br>The larger circle’s radius is 3.5 times the radius of the smaller circle.\n\n\r\n\r\nWhich of the following is the circumference of the larger circle?\n"},{"varval":"<div class=\"aligned\">\n\n| | |\n| --------------------- | -------------- |\n| Large radius\t | \\= 6 × 3.5 |\n| | \\= 21 cm |\n\n</div>\n\n<div class=\"aligned\">\n\n| | |\n| --------------------- | -------------- |\n| $\\therefore$ Circumference| \\= 2 × $\\large \\pi$ × $R$ |\n| | \\= 2 × $\\large \\pi$ × 21 |\n|| \\= {{{correctAnswer}}}|\n\n</div>\n"}]},{"vars":[{"varval":"Two circles have radii of different lengths.\n\n<br>\n\nsm_img https://teacher.smartermaths.com.au/wp-content/uploads/2022/08/Measurement_NAPX-p169635v01_v1a.svg 330 indent3 vpad\n\n\n<br>The larger circle’s radius is 1.5 times the radius of the smaller circle.\n\n\r\n\r\nWhich of the following is the circumference of the larger circle?\n"},{"varval":"<div class=\"aligned\">\n\n| | |\n| --------------------- | -------------- |\n| Large radius\t | \\= 4 × 1.5 |\n| | \\= 6 cm |\n\n</div>\n\n<div class=\"aligned\">\n\n| | |\n| --------------------- | -------------- |\n| $\\therefore$ Circumference| \\= 2 × $\\large \\pi$ × $R$ |\n| | \\= 2 × $\\large \\pi$ × 6 |\n|| \\= {{{correctAnswer}}}|\n\n</div>\n"}]},{"vars":[{"varval":"Two circles have radii of different lengths.\n\n<br>\n\nsm_img https://teacher.smartermaths.com.au/wp-content/uploads/2022/08/Measurement_NAPX-p169635v01_v2a.svg 400 indent vpad\n\n<br>The larger circle’s radius is 2.5 times the radius of the smaller circle.\n\n\r\n\r\nWhich of the following is the circumference of the larger circle?\n"},{"varval":"<div class=\"aligned\">\n\n| | |\n| --------------------- | -------------- |\n| Large radius\t | \\= 10 × 2.5 |\n| | \\= 25 m |\n\n</div>\n\n<div class=\"aligned\">\n\n| | |\n| --------------------- | -------------- |\n| $\\therefore$ Circumference| \\= 2 × $\\large \\pi$ × $R$ |\n| | \\= 2 × $\\large \\pi$ × 25 |\n|| \\= {{{correctAnswer}}}|\n\n</div>\n"}]},{"vars":[{"varval":"Two circles have radii of different lengths.\n\n<br>\n\nsm_img https://teacher.smartermaths.com.au/wp-content/uploads/2022/08/Measurement_NAPX-p169635v01_v3a_2.svg 330 indent vpad\n\n<br>The smaller circle’s radius is one quarter the radius of the larger circle.\n\n\r\n\r\nWhich of the following is the circumference of the smaller circle?\n"},{"varval":"<div class=\"aligned\">\n\n| | |\n| --------------------- | -------------- |\n| Large radius\t | \\= 12 × 0.25 |\n| | \\= 3 cm |\n\n</div>\n\n<div class=\"aligned\">\n\n| | |\n| --------------------- | -------------- |\n| $\\therefore$ Circumference| \\= 2 × $\\large \\pi$ × $r$ |\n| | \\= 2 × $\\large \\pi$ × 3 |\n|| \\= {{{correctAnswer}}}|\n\n</div>\n"}]},{"vars":[{"varval":"Two circles have radii of different lengths.\n\n<br>\n\nsm_img https://teacher.smartermaths.com.au/wp-content/uploads/2022/08/Measurement_NAPX-p169635v01_v4a_2.svg 360 indent vpad\n\n<br>The smaller circle’s radius is two-thirds the radius of the larger circle.\n\n\r\n\r\nWhich of the following is the circumference of the smaller circle?\n\n"},{"varval":"<div class=\"aligned\">\n\n| | |\n| --------------------- | -------------- |\n| Large radius\t | \\= 15 × $\\dfrac{2}{3}$ |\n| | \\= 10 cm |\n\n</div>\n\n<div class=\"aligned\">\n\n| | |\n| --------------------- | -------------- |\n| $\\therefore$ Circumference| \\= 2 × $\\large \\pi$ × $\\large r$ |\n| | \\= 2 × $\\large \\pi$ × 10 |\n|| \\= {{{correctAnswer}}}|\n\n</div>\n"}]},{"vars":[{"varval":"Two concentric circles are shown below.\n\n<br>\n\nsm_img https://teacher.smartermaths.com.au/wp-content/uploads/2022/08/Measurement_NAPX-p169635v01_v5a.svg 330 indent vpad\n\n<br>The larger circle’s radius is one and one-third times the radius of the smaller circle.\n\n\r\n\r\nWhich of the following is the circumference of the larger circle?\n"},{"varval":"<div class=\"aligned\">\n\n| | |\n| --------------------- | -------------- |\n| Large radius\t | \\= 21 × $\\dfrac{4}{3}$ |\n| | \\= 28 cm |\n\n</div>\n\n<div class=\"aligned\">\n\n| | |\n| --------------------- | -------------- |\n| $\\therefore$ Circumference| \\= 2 × $\\large \\pi$ × $R$ |\n| | \\= 2 × $\\large \\pi$ × 28 |\n|| \\= {{{correctAnswer}}}|\n\n</div>\n"}]}]

  564. <div class="sm_mode"> {{{question}}} </div>

    [{"vars":[{"varval":"A hexagon with equal sides is drawn below.\n\nsm_img https://teacher.smartermaths.com.au/wp-content/uploads/2018/06/NAPX-I3-CA11.svg 100 indent vpad\n\n5 identical hexagons are connected as shown in the diagram below.\n\nsm_img https://teacher.smartermaths.com.au/wp-content/uploads/2017/01/naplan-Y7-2016-11mci-223x300.png 240 indent vpad\n\nWhat is the perimeter of the larger shape?\n"},{"varval":"<div class=\"aligned\">\n\n| | |\n| --------------------- | -------------- |\n| Perimeter | \\= Number of sides × 9.5 |\n| | \\= 22 × 9.5 |\n|| \\= {{{correctAnswer}}}|\n\n</div>"}]},{"vars":[{"varval":"A octagon with equal sides is drawn below.\n\nsm_img https://teacher.smartermaths.com.au/wp-content/uploads/2022/08/Measurement_NAPX-I3-CA11_v1a.svg 155 indent vpad\n\n6 identical octagons are connected as shown in the diagram below.\n\nsm_img https://teacher.smartermaths.com.au/wp-content/uploads/2022/08/Measurement_NAPX-I3-CA11_v1.svg 320 indent vpad\n\nWhat is the perimeter of the larger shape?\n"},{"varval":"<div class=\"aligned\">\n\n| | |\n| --------------------- | -------------- |\n| Perimeter | \\= Number of sides × 7.5 |\n| | \\= 38 × 7.5 |\n|| \\= {{{correctAnswer}}}|\n\n</div>"}]},{"vars":[{"varval":"A pentagon with equal sides is drawn below.\n\nsm_img https://teacher.smartermaths.com.au/wp-content/uploads/2022/08/Measurement_NAPX-I3-CA11_v2b.svg 100 indent3 vpad\n\n5 identical pentagons are connected as shown in the diagram below.\n\nsm_img https://teacher.smartermaths.com.au/wp-content/uploads/2022/08/Measurement_NAPX-I3-CA11_v2.svg 180 indent3 vpad\n\nWhat is the perimeter of the larger shape?\n"},{"varval":"<div class=\"aligned\">\n\n| | |\n| --------------------- | -------------- |\n| Perimeter | \\= Number of sides × 3.5 |\n| | \\= 17 × 3.5 |\n|| \\= {{{correctAnswer}}}|\n\n</div>"}]},{"vars":[{"varval":"A decagon with equal sides is drawn below.\n\nsm_img https://teacher.smartermaths.com.au/wp-content/uploads/2022/08/Measurement_NAPX-I3-CA11_v3a.svg 160 indent3 vpad\n\n5 identical decagons are connected as shown in the diagram below.\n\nsm_img https://teacher.smartermaths.com.au/wp-content/uploads/2022/08/Measurement_NAPX-I3-CA11_v3.svg 340 indent3 vpad\n\nWhat is the perimeter of the larger shape?\n"},{"varval":"<div class=\"aligned\">\n\n| | |\n| --------------------- | -------------- |\n| Perimeter | \\= Number of sides × 4.5 |\n| | \\= 42 × 4.5 |\n|| \\= {{{correctAnswer}}}|\n\n</div>"}]},{"vars":[{"varval":"A hexagon with equal sides and an equilateral triangle are drawn below.\n\nsm_img https://teacher.smartermaths.com.au/wp-content/uploads/2022/08/Measurement_NAPX-I3-CA11_v4a.svg 150 indent3 vpad\n\n3 identical hexagons and 8 identical equilateral triangles are connected as shown in the diagram below.\n\nsm_img https://teacher.smartermaths.com.au/wp-content/uploads/2022/08/Measurement_NAPX-I3-CA11_v4.svg 300 indent3 vpad\n\nWhat is the perimeter of the larger shape?\n"},{"varval":"<div class=\"aligned\">\n\n| | |\n| --------------------- | -------------- |\n| Perimeter | \\= Number of sides × 8.5 |\n| | \\= 22 × 8.5 |\n|| \\= {{{correctAnswer}}}|\n\n</div>"}]},{"vars":[{"varval":"A heptagon with equal sides is drawn below.\n\nsm_img https://teacher.smartermaths.com.au/wp-content/uploads/2022/08/Measurement_NAPX-I3-CA11_v5a.svg 100 indent3 vpad\n\n6 identical heptagons are connected as shown in the diagram below.\n\nsm_img https://teacher.smartermaths.com.au/wp-content/uploads/2022/08/Measurement_NAPX-I3-CA11_v5.svg 400 indent3 vpad\n\nWhat is the perimeter of the larger shape?\n"},{"varval":"<div class=\"aligned\">\n\n| | |\n| --------------------- | -------------- |\n| Perimeter | \\= Number of sides × 5.5 |\n| | \\= 32 × 5.5 |\n|| \\= {{{correctAnswer}}}|\n\n</div>"}]}]

  565. <div class="sm_mode"> A junior soccer oval is marked out with the dimensions shown below. sm_img https://teacher.smartermaths.com.au/wp-content/uploads/2018/08/NAPX-E3-CA03.svg 520 indent vpad What is the area of the field? </div>

    [{"vars":null}]

  566. <div class="sm_mode"> {{{question}}} </div>

    [{"vars":[{"varval":"Zachary has a bag of marbles.\r\n\r\nThe number of marbles of each colour is recorded in the table below.\n\n<br>\n\n<div class=\"sm-table row1-color3\">\n\n>>| Colour | Number of marbles |\n|:-:|:-:|\n| green | 13|\n| blue | 2|\n| white | 3|\n| red | 9|\n\n</div>\n\n<br>Zachary randomly takes 1 marble out of his bag without looking.\r\n\r\nWhat is the chance it is red?"},{"varval":"<div class=\"aligned\">\r\n\r\n| | |\r\n| ------------- | ---------- |\r\n| $P$(red) | \\= $\\dfrac{\\text{Number of red marbles}}{\\text{Total marbles}}$ |\n| | \\= $\\dfrac{9}{27}$ |\r\n| | \\= {{{correctAnswer}}} |\r\n\r\n</div>\r"}]},{"vars":[{"varval":"Blinky is blowing up balloons for a birthday party.\r\n\r\nThe number of blown up balloons of each colour is recorded in the table below.\n\n<br>\n\n<div class=\"sm-table row1-color3\">\n\n>>| Colour | Number of balloons |\n|:-:|:-:|\n| white | 11|\n| purple | 7|\n| orange | 6|\n| yellow | 9|\n\n</div>\n\n<br>Blinky picks one balloon without looking and gives it to the first person who arrives at the party.\n\r\nWhat is the chance it is white?"},{"varval":"<div class=\"aligned\">\r\n\r\n| | |\r\n| ------------- | ---------- |\r\n| $P$(white) | \\= $\\dfrac{\\text{Number of white balloons}}{\\text{Total balloons}}$ |\n| | \\= $\\dfrac{11}{33}$ |\r\n| | \\= {{{correctAnswer}}} |\r\n\r\n</div>\r"}]}]

  567. <div class="sm_mode"> Bennett creates a game with the spinner shown below. <br> sm_img https://teacher.smartermaths.com.au/wp-content/uploads/2017/01/NAP-I3-CA07.svg 200 indent3 vpad <br>If the spinner lands on a 2, he wins a prize. What is the probability that Bennett will win a prize on his next spin? </div>

    [{"vars":null}]

  568. Added categories

    <div class="sm_mode"> {{{question}}} </div>

    [{"vars":[{"varval":"Brent has a jar full of cookies.\r\n\r\nThe jar contains M & M cookies, choc chip cookies and gingernuts.\r\n\r\nBrent closes his eyes and picks one cookie from the jar.\r\n\r\nWhich of the following could be the probability that Brent picked a choc chip cookie?"},{"varval":"Any probability must be between 0 and 1.\r\n\n$\\therefore$ Only possibility is {{{correctAnswer}}}\n"}]},{"vars":[{"varval":"Billy has a box of apples.\n\nThe box contains Pink Lady apples, Delicious apples and Royal Gala apples.\n\nBilly closes his eyes and picks one apple from the box.\n\nWhich of the following could be the probability that Billy picked a Royal Gala apple?"},{"varval":"Any probability must be between 0 and 1.\n\n$\\therefore$ Only possibility is {{{correctAnswer}}}"}]},{"vars":[{"varval":"Magnus has a box full of coloured crystals.\n\nThe box contains green crystals, pink crystals and blue crystals.\n\nMagnus closes his eyes and picks one crystal from the box.\n\nWhich of the following could be the probability that Magnus picked a blue crystal?"},{"varval":"Any probability must be between 0 and 1.\n\n$\\therefore$ Only possibility is {{{correctAnswer}}}"}]},{"vars":[{"varval":"Bjorn has a bag full of tennis racquets.\n\nThe bag contains Wilson, Yonex and Prince brands of racquets.\n\nBjorn closes his eyes and picks one racquet from the bag.\n\nWhich of the following could be the probability that Bjorn picked a Yonex racquet?"},{"varval":"Any probability must be between 0 and 1.\n\n$\\therefore$ Only possibility is {{{correctAnswer}}}"}]},{"vars":[{"varval":"Gollum has a bag full of rings.\n\nThe bag contains gold rings, silver rings and titanium rings.\n\nGollum closes his eyes and picks one ring from the bag.\n\nWhich of the following could be the probability that Gollum picked a gold ring?"},{"varval":"Any probability must be between 0 and 1.\n\n$\\therefore$ Only possibility is {{{correctAnswer}}}"}]},{"vars":[{"varval":"Misty has a box of old comics.\n\nThe box contains Marvel comics, DC comics and Phantom comics.\n\nMisty closes her eyes and picks one comic from the box.\n\nWhich of the following could be the probability that Misty picked a DC comic?\n"},{"varval":"Any probability must be between 0 and 1.\n\n$\\therefore$ Only possibility is {{{correctAnswer}}}"}]}]

  569. <div class="sm_mode"> {{{question}}} </div>

    [{"vars":[{"varval":"Ricky has a bag of orange, white, blue and green marbles.\n\n<br>\n\nsm_img https://teacher.smartermaths.com.au/wp-content/uploads/2017/10/NAP-J3-CA19_1.png 150 indent3 vpad\n\n<br>Ricky picks one marble from the bag.\r\n\r\nWhich of the following could be the probability that the marble he picks is green?"},{"varval":"Any probability must be between 0 and 1.\n\n$\\therefore$ Only possibility is {{{correctAnswer}}}"}]},{"vars":[{"varval":"Angus has a bag full of lollies.\n\nThe bag contains jelly snakes, maltesers and sour worms.\n\nAngus closes his eyes and picks one lolly from the bag.\n\nWhich of the following could be the probability that Angus picked a sour worm?"},{"varval":"Any probability must be between 0 and 1.\n\n$\\therefore$ Only possibility is {{{correctAnswer}}}"}]},{"vars":[{"varval":"Bess has a bag of orange, grey and blue marbles.\n\n<br>\n\nsm_img https://teacher.smartermaths.com.au/wp-content/uploads/2023/07/balls-6-coloured-min.svg 150 indent3 vpad\n\n<br>Bess picks one marble from the bag.\n\nWhich of the following could be the probability that the marble she picks is grey?"},{"varval":"Any probability must be between 0 and 1.\n\n$\\therefore$ Only possibility is {{{correctAnswer}}}"}]},{"vars":[{"varval":"Ringo has a bag of gold and silver coins.\n\n<br>\n\nsm_img https://teacher.smartermaths.com.au/wp-content/uploads/2023/07/coin-bag-min.svg 150 indent3 vpad\n\n<br>Ringo picks one coin from the bag.\n\nWhich of the following could be the probability that the coin he picks is silver?"},{"varval":"Any probability must be between 0 and 1.\n\n$\\therefore$ Only possibility is {{{correctAnswer}}}"}]},{"vars":[{"varval":"Kuki has a box full of balls.\n\nThe balls are red, purple and white.\n\nKuki closes her eyes and picks one ball from the box.\n\nWhich of the following could be the probability that Kuki picked a purple ball?"},{"varval":"Any probability must be between 0 and 1.\n\n$\\therefore$ Only possibility is {{{correctAnswer}}}"}]},{"vars":[{"varval":"Min has a bag of table tennis balls.\n\nThe bag contains white and orange balls.\n\nMin closes his eyes and picks one ball from the bag.\n\nWhich of the following could be the probability that Min picked a white ball?"},{"varval":"Any probability must be between 0 and 1.\n\n$\\therefore$ Only possibility is {{{correctAnswer}}}"}]}]

  570. SJ v1 - v7

    <div class="sm_mode"> {{{question}}} </div>

    [{"vars":[{"varval":"Enigma uses this net to make a dice.\n\n<br>\n\nsm_img https://teacher.smartermaths.com.au/wp-content/uploads/2019/12/nap-L4-08-ver3.svg 200 indent3 vpad\n\n<br>He rolls the dice once.\n\nWhat is the chance that Enigma will roll a 4? "},{"varval":"<div class=\"aligned\">\n\n| | |\n| ------------: | ---------- |\n| $P$(4) | \\= $\\dfrac{\\text{number of 4's}}{\\text{total possibilities}}$ |\n| | \\= $\\dfrac{2}{6}$ |\n| | \\= {{{correctAnswer}}} |\n\n</div>\n\n"}]},{"vars":[{"varval":"Tran uses this net to make a dice.\n\n<br>\n\nsm_img https://teacher.smartermaths.com.au/wp-content/uploads/2019/12/nap-L4-08-ver3.svg 200 indent3 vpad\n\n<br>He rolls the dice once.\n\nWhat is the chance that Tran will roll a 2? "},{"varval":"<div class=\"aligned\">\n\n| | |\n| ------------: | ---------- |\n| $P$(2) | \\= $\\dfrac{\\text{number of 2's}}{\\text{total possibilities}}$ |\n| | \\= $\\dfrac{3}{6}$ |\n| | \\= {{{correctAnswer}}} |\n\n</div>\n"}]},{"vars":[{"varval":"Toni uses this net to make a dice.\n\n<br>\n\nsm_img https://teacher.smartermaths.com.au/wp-content/uploads/2019/12/nap-L4-08-ver3.svg 200 indent3 vpad\n\n<br>She rolls the dice once.\n\nWhat is the chance that Toni will roll a 1? "},{"varval":"<div class=\"aligned\">\n\n| | |\n| ------------: | ---------- |\n| $P$(1) | \\= $\\dfrac{\\text{number of 1's}}{\\text{total possibilities}}$ |\n| | \\= {{{correctAnswer}}} |\n\n</div>\n"}]},{"vars":[{"varval":"Blyss uses this net to make a dice.\n\n<br>\n\nsm_img https://teacher.smartermaths.com.au/wp-content/uploads/2023/08/dice-biassed.svg 150 indent3 vpad\n\n<br>She rolls the dice once.\n\nWhat is the chance that Blyss will roll a 1? "},{"varval":"<div class=\"aligned\">\n\n| | |\n| ------------: | ---------- |\n| $P$(1) | \\= $\\dfrac{\\text{number of 1's}}{\\text{total possibilities}}$ |\n| | \\= $\\dfrac{2}{6}$ |\n| | \\= {{{correctAnswer}}} |\n\n</div>\n"}]},{"vars":[{"varval":"Chusi uses this net to make a dice.\n\n<br>\n\nsm_img https://teacher.smartermaths.com.au/wp-content/uploads/2023/08/dice-biassed-1.svg 200 indent3 vpad\n\n<br>She rolls the dice once.\n\nWhat is the chance that Chusi will roll a 1? "},{"varval":"<div class=\"aligned\">\n\n| | |\n| ------------: | ---------- |\n| $P$(1) | \\= $\\dfrac{\\text{number of 1's}}{\\text{total possibilities}}$ |\n| | \\= $\\dfrac{3}{6}$ |\n| | \\= {{{correctAnswer}}} |\n\n</div>\n"}]},{"vars":[{"varval":"Aldo uses this net to make a dice.\n\n<br>\n\nsm_img https://teacher.smartermaths.com.au/wp-content/uploads/2023/08/dice-biassed-2.svg 200 indent3 vpad\n\n<br>He rolls the dice once.\n\nWhat is the chance that Aldo will roll a 3? "},{"varval":"<div class=\"aligned\">\n\n| | |\n| ------------: | ---------- |\n| $P$(3) | \\= $\\dfrac{\\text{number of 3's}}{\\text{total possibilities}}$ |\n| | \\= $\\dfrac{4}{6}$ |\n| | \\= {{{correctAnswer}}} |\n\n</div>\n"}]},{"vars":[{"varval":"Ari uses this net to make a dice.\n\n<br>\n\nsm_img https://teacher.smartermaths.com.au/wp-content/uploads/2023/08/dice-biassed-3.svg 200 indent3 vpad\n\n<br>He rolls the dice once.\n\nWhat is the chance that Ari will roll a 2? "},{"varval":"<div class=\"aligned\">\n\n| | |\n| ------------: | ---------- |\n| $P$(2) | \\= $\\dfrac{\\text{number of 2's}}{\\text{total possibilities}}$ |\n| | \\= $\\dfrac{3}{6}$ |\n| | \\= {{{correctAnswer}}} |\n\n</div>\n"}]},{"vars":[{"varval":"Soula uses this net to make a dice.\n\n<br>\n\nsm_img https://teacher.smartermaths.com.au/wp-content/uploads/2023/08/dice-biassed-3.svg 200 indent3 vpad\n\n<br>She rolls the dice once.\n\nWhat is the chance that Soula will roll a 5? "},{"varval":"<div class=\"aligned\">\n\n| | |\n| ------------: | ---------- |\n| $P$(5) | \\= $\\dfrac{\\text{number of 5's}}{\\text{total possibilities}}$ |\n| | \\= {{{correctAnswer}}} |\n\n</div>\n"}]}]

  571. var1: Statistics and Probability, NAPX-J4-CA*38, NAPX-J3-CA*37

    <div class="sm_mode"> {{{question}}} </div>

    [{"vars":[{"varval":"The Strikers and the Hurricanes are playing cricket in a 20 over competition.\r\n\r\nThe stem-and-leaf plots show the number of runs each side has scored in their last 15 games.\n\n<br>\n\nsm_img https://teacher.smartermaths.com.au/wp-content/uploads/2018/06/NAPX-J4-CA381.svg 340 indent3 vpad\n\n\r<br>Select the true statement about the data."},{"varval":"Striker's range = $148\\ −\\ 110$ = 38\r\n\nHurricane's range = $144\\ −\\ 103$ = 41\r\n\n$\\therefore$ Striker's range < Hurricane's range"}]},{"vars":[{"varval":"The Strikers and the Hurricanes are playing cricket in a 20 over competition.\r\n\nThe stem-and-leaf plots show the number of runs each side has scored in their last 15 games.\n\n<br>\n\nsm_img https://teacher.smartermaths.com.au/wp-content/uploads/2018/07/NAPX-J3-CA37.svg 340 indent3 vpad\r\n\n<br>Which statement about the data below is true?"},{"varval":"Consider each statement:\r\n\nLow scores: $S=108, \\ H=107$ &nbspx \t\r\n\nHigh scores: $S=146, \\ H=147$ &nbspx\t\r\n\nScores $> 132: S=6, \\ H=7$ &nbspx \t\r\n\nMedian: $S=127, \\ H=129 \\ \\ \\checkmark$ \t\r\n\nRange: $S=146\\ −\\ 108=38, \\ H=147\\ −\\ 107=40$ &nbspx\t "}]},{"vars":[{"varval":"The Sydney Swifts and the Melbourne Vixens are playing netball in the Super Netball competition.\r\n\r\nThe stem-and-leaf plots show the number of goals each side has scored in their last 14 games.\n\n<br>\n\nsm_img https://teacher.smartermaths.com.au/wp-content/uploads/2022/08/Stat_Prob_NAPX-J4-CA38_NAPX-J3-CA37_v2_q.svg 360 indent3 vpad\n\n\r<br>Select the true statement about the data."},{"varval":"Consider each statement:\r\n\nMost goals: $SS=75, \\ MV=75$ &nbspx \t\r\n\nRange: $SS=75\\ −\\ 45=30, \\ MV=75\\ −\\ 51=24$ &nbspx\t\n\nMedian: $SS=58, \\ MV=62 \\ \\ \\checkmark$ \n\nLowest scores: $SS=45, \\ MV=51$ &nbspx\t\r\n\nScores $> 60: SS=5, \\ MV=7$ &nbspx \t\r\n\n\t\r\n\n"}]},{"vars":[{"varval":"The West Coast Fever and the Netball Giants are playing netball in the Super Netball competition.\r\n\r\nThe stem-and-leaf plots show the number of goals each side has scored in their last 14 games.\n\n<br>\n\nsm_img https://teacher.smartermaths.com.au/wp-content/uploads/2022/08/Stat_Prob_NAPX-J4-CA38_NAPX-J3-CA37_v3.svg 360 indent3 vpad\n\n\r<br>Select the true statement about the data."},{"varval":"Consider each statement:\r\n\nMedian: $WCF=73, \\ NG=65.5$ &nbspx\n\nLowest scores: $WCF=60, \\ NG=43$ &nbspx\t\n\nScores $> 70: WCF=8, \\ NG=5$ &nbspx \t\n\nMost goals: $WCF=86, \\ NG=82$ &nbspx\t\r\n\nRange: $WCF=86\\ −\\ 60=26, \\ NG=82\\ −\\ 43=39 \\ \\ \\checkmark$ \t\n\n\r\n\n\r\n"}]},{"vars":[{"varval":"The Sea Eagles and the Panthers are playing rugby league in the Telstra Premiership.\r\n\r\nThe stem-and-leaf plots show the number of points scored by each team in the first 15 rounds of the 2022 premiership.\n<br>\n\nsm_img https://teacher.smartermaths.com.au/wp-content/uploads/2022/08/Stat_Prob_NAPX-J4-CA38_NAPX-J3-CA37_v4.svg 360 indent3 vpad\n\n\r<br>Select the true statement about the data."},{"varval":"Consider each statement:\r\n\nLowest scores: $SE=0, \\ P=18$ &nbspx\n\nMedian: $SE=22, \\ P=32 \\ \\ \\checkmark$\t\n\nScores $> 20: SE=9, \\ P=12$ &nbspx \t\n\nHighest Score: $SE=44, \\ P=42$ | Lowest Score: $SE=0, \\ P=8$&nbspx\t\r\n\nRange: $SE=44\\ −\\ 0=44, \\ P=42\\ −\\ 18=24$ &nbspx \t\n"}]},{"vars":[{"varval":"The Knights and the Rabbitohs are playing rugby league in the Telstra Premiership.\r\n\r\nThe stem-and-leaf plots show the number of points scored by each team in the first 15 rounds of the premiership.\n<br>\n\nsm_img https://teacher.smartermaths.com.au/wp-content/uploads/2022/08/Stat_Prob_NAPX-J4-CA38_NAPX-J3-CA37_v5.svg 360 indent3 vpad\n\n\r<br>Select the true statement about the data."},{"varval":"Consider each statement:\r\n\nScores > 12: Knights = 9, Rabbitohs = 10 &nbspx \n\nMedian: Knights = 16, Rabbitohs = 24 &nbspx\n\nMedian Difference: $24\\ −\\ 16=8$ &nbspx\n\nHighest Score: Rabbitohs = 44, Lowest Score: Knights = 0 &nbspx\n\n\r\nRange: Rabbitohs = 44 − 4 = 40, 2 $\\times$ Median: Knights = $2\\ \\times$ 16 = 32 $\\ \\ \\checkmark$\t\t\n"}]}]

  572. <div class="sm_mode"> {{{question}}} </div>

    [{"vars":[{"varval":"Manou was cooking a defrosted chicken in his oven and checking its temperature every 7 minutes.\r\n\r\n\nThe first temperature taken was −8.2°C.\r\n\r\n\nThe second temperature was 4.2°C.\r\n\r\n\nThe third measurement showed that the temperature had increased one and a half times the previous increase.\r\n\r\n\nWhat was the third temperature?"},{"varval":"<div class=\"aligned\">\r\n\r\n| | |\r\n| -------------: | ---------- |\r\n| 1st increase | \\= $4.2\\ −\\ (−8.2)$ |\r\n| | \\= $4.2+8.2$ |\n| | \\= 12.4 |\r\n\r\n</div>\n\r\n\n<br>\n\n<div class=\"aligned\">\r\n\r\n| | |\r\n| ------------: | ---------- |\r\n| 2nd increase | \\= $1.5×12.4$ |\n| | \\= $1×12.4+0.5×12.4$ |\n| | \\= $12.4+6.2$ |\n| | \\= 18.6 |\n\r\n\r\n\r\n</div>\n\n<br>\n\n<div class=\"aligned\">\r\n\r\n| | |\r\n| ------------: | ---------- |\r\n| $\\therefore$ Third temp | \\= $4.2+18.6$ |\r\n| | \\= {{{correctAnswer0}}}{{{suffix0}}} |\n\r\n\r\n</div>\n"}]},{"vars":[{"varval":"Marion was cooking a defrosted Viking steak in her oven and checking its temperature every 10 minutes.\r\n\r\n\nThe first temperature taken was −2.8°C.\r\n\r\n\nThe second temperature was 13.6°C.\r\n\r\n\nThe third measurement showed that the temperature had increased one and a quarter times the previous increase.\r\n\r\n\nWhat was the third temperature?"},{"varval":"<div class=\"aligned\">\r\n\r\n| | |\r\n| -------------: | ---------- |\r\n| 1st increase | \\= $13.6\\ −\\ (−2.8)$ |\r\n| | \\= $13.6+2.8$ |\n| | \\= 16.4 |\r\n\r\n</div>\n\r\n\n<br>\n\n<div class=\"aligned\">\r\n\r\n| | |\r\n| ------------: | ---------- |\r\n| 2nd increase | \\= $1.25×16.4$ |\r\n| | \\= $1×16.4+0.25×16.4$ |\n| | \\= $16.4+4.1$ |\n| | \\= 20.5 |\r\n\r\n</div>\n\n<br>\n\n<div class=\"aligned\">\r\n\r\n| | |\r\n| ------------: | ---------- |\r\n| $\\therefore$ Third temp | \\= $13.6+20.5$ |\r\n| | \\= {{{correctAnswer0}}}{{{suffix0}}} |\n\r\n\r\n</div>\n"}]},{"vars":[{"varval":"Adam was cooking a partly frozen lasagne in his oven and checking its temperature every 5 minutes.\r\n\r\n\nThe first temperature taken was −31.4°C.\r\n\r\n\nThe second temperature was −13.4°C.\r\n\r\n\nThe third measurement showed that the temperature had increased two and a one tenth times the previous increase.\r\n\r\n\nWhat was the third temperature?"},{"varval":"<div class=\"aligned\">\r\n\r\n| | |\r\n| -------------: | ---------- |\r\n| 1st increase | \\= $−13.4\\ −\\ (−31.4)$ |\r\n| | \\= $−13.4+31.4$ |\n| | \\= 18 |\r\n\r\n</div>\n\r\n\n<br>\n\n<div class=\"aligned\">\r\n\r\n| | |\r\n| ------------: | ---------- |\r\n| 2nd increase | \\= $2.1×18$ |\r\n| | \\= $2×18+0.1×18$ |\n| | \\= $36+1.8$ |\n| | \\= 37.8 |\r\n\r\n</div>\n\n<br>\n\n<div class=\"aligned\">\r\n\r\n| | |\r\n| ------------: | ---------- |\r\n| $\\therefore$ Third temp | \\= $−13.4+37.8$ |\r\n| | \\= {{{correctAnswer0}}}{{{suffix0}}} |\n\r\n\r\n</div>\n"}]},{"vars":[{"varval":"Einstein heated a beaker filled with ice water using a Bunsen burner. He measured the temperature every 30 seconds.\r\n\r\n\nThe third temperature taken was −31.1°C.\r\n\r\n\nThe fourth temperature was −9.9°C.\r\n\r\n\nThe fifth measurement showed that the temperature had increased three and a quarter times the previous increase.\r\n\r\n\nWhat was the fifth temperature?"},{"varval":"<div class=\"aligned\">\r\n\r\n| | |\r\n| -------------: | ---------- |\r\n| 1st increase | \\= $−9.9\\ −\\ (−31.1)$ |\r\n| | \\= $−9.9+31.1$ |\n| | \\= 21.2 |\r\n\r\n</div>\n\r\n\n<br>\n\n<div class=\"aligned\">\r\n\r\n| | |\r\n| ------------: | ---------- |\r\n| 2nd increase | \\= $3.25×21.2$ |\r\n| | \\= $3×21.2+0.25×21.2$ |\n| | \\= $63.6+5.3$ |\n| | \\= 68.9 |\r\n\r\n</div>\n\n<br>\n\n<div class=\"aligned\">\r\n\r\n| | |\r\n| ------------: | ---------- |\r\n| $\\therefore$ Fifth temp | \\= $−9.9+68.9$ |\r\n| | \\= {{{correctAnswer0}}}{{{suffix0}}} |\n\r\n\r\n</div>"}]},{"vars":[{"varval":"Donna had baked a cheesecake and put it in the freezer to cool. She checked the temperature of the cheesecake every 45 minutes.\r\n\r\n\nThe first temperature taken was 180.8°C.\r\n\r\n\nThe second temperature was 84.6°C.\r\n\r\n\nThe third measurement showed that the temperature had decreased one and a half times the previous decrease.\r\n\r\n\nWhat was the third temperature?"},{"varval":"<div class=\"aligned\">\r\n\r\n| | |\r\n| -------------: | ---------- |\r\n| 1st decrease| \\= $180.8\\ −\\ 84.6$ |\r\n| | \\= 96.2 |\r\n\r\n</div>\n\r\n\n<br>\n\n<div class=\"aligned\">\r\n\r\n| | |\r\n| ------------: | ---------- |\r\n| 2nd decrease | \\= $1.5×96.2$ |\r\n| | \\= $1×96.2+0.5×96.2$ |\n| | \\= $96.2+48.1$ |\n| | \\= 144.3 |\r\n\r\n</div>\n\n<br>\n\n<div class=\"aligned\">\r\n\r\n| | |\r\n| ------------: | ---------- |\r\n| $\\therefore$ Third temp | \\= 84.6 − 144.3 |\r\n| | \\= {{{correctAnswer0}}}{{{suffix0}}} |\n\r\n\r\n</div>\n"}]},{"vars":[{"varval":"George was testing the temperature of his melting chocolate icecream every 30 seconds.\r\n\r\n\nThe first temperature taken was −18.7°C.\r\n\r\n\nThe second temperature was −15.3°C.\r\n\r\n\nThe third measurement showed that the temperature had increased one and a half times the previous increase.\r\n\r\n\nWhat was the third temperature?"},{"varval":"<div class=\"aligned\">\r\n\r\n| | |\r\n| -------------: | ---------- |\r\n| 1st increase | \\= $−15.3\\ −\\ (−18.7)$ |\r\n| | \\= $−15.3+18.7$ |\n| | \\= 3.4 |\r\n\r\n</div>\n\r\n\n<br>\n\n<div class=\"aligned\">\r\n\r\n| | |\r\n| ------------: | ---------- |\r\n| 2nd increase | \\= $1.5×3.4$ |\r\n| | \\= $1×3.4+0.5×3.4$ |\n| | \\= $3.4+1.7$ |\n| | \\= 5.1 |\r\n\r\n</div>\n\n<br>\n\n<div class=\"aligned\">\r\n\r\n| | |\r\n| ------------: | ---------- |\r\n| $\\therefore$ Third temp | \\= $−15.3+5.1$ |\r\n| | \\= {{{correctAnswer0}}}{{{suffix0}}} |\n\r\n\r\n</div>"}]}]

  573. <div class="sm_mode"> {{{question}}} </div>

    [{"vars":[{"varval":"A random sample of people were asked what is their favourite winter sport.\r\n\r\nThe table below recorded the results.\n\n\n\n<div class=\"sm-table row1-color8\">\n\n>>| Sport | Number of People|\n|:-:|:-:|\n| Netball | 56|\n| Aussie Rules | 69|\n| Rugby League | 42|\n| Hockey | 17|\n\n</div>\n\n<br>\n\nUsing the data from the survey, predict how many people would choose Aussie Rules if 2000 people were surveyed?"},{"varval":"sm_nogap Total people surveyed\n\n<div class=\"aligned\">\r\n\r\n>>| |\r\n| ---------- |\r\n| \\= $56+69+42+17$ |\r\n| \\= 184 |\r\n\r\n</div>\n\n<br>\n\nsm_nogap $\\therefore$ Predicted number to choose Aussie Rules\n\n<div class=\"aligned\">\r\n\r\n>>| |\r\n| ---------- |\r\n| \\= $\\dfrac{69}{184} \\times 2000$ |\r\n| \\= {{{correctAnswer0}}} |\r\n\r\n</div>"}]},{"vars":[{"varval":"A random sample of people were surveyed on the brand of their smartphone.\r\n\r\nThe table below recorded the results.\n\n\n\n<div class=\"sm-table row1-color2\">\n\n>>| Brand of Phone| Number of People|\n|:-:|:-:|\n| Nokia| 27|\n| Apple| 108|\n| Samsung| 71|\n| Google| 10 |\n\n</div>\n\n<br>\n\nUsing the data from the survey, predict how many people would have a Nokia phone if 4000 people were surveyed?"},{"varval":"sm_nogap Total people surveyed\n\n<div class=\"aligned\">\r\n\r\n>>| |\r\n| ---------- |\r\n| \\= 27 + 108 + 71 + 10 |\r\n| \\= 216 |\r\n\r\n</div>\n\n<br>\n\nsm_nogap $\\therefore$ Predicted number to own Nokia\n\n<div class=\"aligned\">\r\n\r\n>>| |\r\n| ---------- |\r\n| \\= $\\dfrac{27}{216} \\times 4000$ |\r\n| \\= {{{correctAnswer0}}} |\r\n\r\n</div>"}]},{"vars":[{"varval":"A random sample of people were surveyed on what is their favourite ice cream flavour.\r\n\r\nThe table below recorded the results.\n\n\n\n<div class=\"sm-table row1-color3\">\n\n>>| Ice Cream Flavour| Number of People|\n|:-:|:-:|\n| Vanilla| 49|\n| Bubblegum| 28|\n| Passionfruit| 35|\n| Chocolate| 59 |\n| Caramel | 29 |\n\n</div>\n\n<br>\n\nUsing the data from the survey, predict how many people would choose chocolate if 3000 people were surveyed?"},{"varval":"sm_nogap Total people surveyed\n\n<div class=\"aligned\">\r\n\r\n>>| |\r\n| ---------- |\r\n| \\= 49 + 28 + 35 + 59 + 29 |\r\n| \\= 200 |\r\n\r\n</div>\n\n<br>\n\nsm_nogap $\\therefore$ Predicted number to choose chocolate\n\n<div class=\"aligned\">\r\n\r\n>>| |\r\n| ---------- |\r\n| \\= $\\dfrac{59}{200} \\times 3000$ |\r\n| \\= {{{correctAnswer0}}} |\n</div>"}]},{"vars":[{"varval":"A random sample of people were surveyed on what is their favourite winter snow sport.\r\n\r\nThe table below recorded the results.\n\n\n\n<div class=\"sm-table row1-color5\">\n\n>>| Snow Sport | Number of People|\n|:-:|:-:|\n| Skiing | 102|\n| Snow Boarding | 228|\n| Cross Country | 85|\n| Fat Biking | 25|\n\n</div>\n\n<br>\n\nUsing the data from the survey, predict how many people would choose Skiing if 5500 people were surveyed?"},{"varval":"sm_nogap Total people surveyed\n\n<div class=\"aligned\">\r\n\r\n>>| |\r\n| ---------- |\r\n| \\= 102 + 228 + 85 + 25 |\r\n| \\= 440 |\r\n\r\n</div>\n\n<br>\n\nsm_nogap $\\therefore$ Predicted number to choose Snow Skiing\n\n<div class=\"aligned\">\r\n\r\n>>| |\r\n| ---------- |\r\n| \\= $\\dfrac{102}{440} \\times 5500$ |\r\n| \\= {{{correctAnswer0}}} |\n</div>"}]},{"vars":[{"varval":"A random sample of people were surveyed on their favourite streaming channel.\r\n\r\nThe table below recorded the results.\n\n\n\n<div class=\"sm-table row1-color4\">\n\n>>| Streaming Channel| Number of People|\n|:-:|:-:|\n| Amazon Prime| 37|\n|Disney+| 64|\n| Google TV| 29|\n| Netflix| 85 |\n|Paramount | 24 |\n|Stan| 81 |\n\n\n</div>\n\n<br>\n\nUsing the data from the survey, predict how many people would choose Amazon Prime if 2560 people were surveyed?"},{"varval":"sm_nogap Total people surveyed\n\n<div class=\"aligned\">\r\n\r\n>>| |\r\n| ---------- |\r\n| \\= 37 + 64 + 29 + 85 + 24 + 81|\r\n| \\= 320 |\r\n\r\n</div>\n\n<br>\n\nsm_nogap $\\therefore$ Predicted number to choose Amazon Prime\n\n<div class=\"aligned\">\r\n\r\n>>| |\r\n| ---------- |\r\n| \\= $\\dfrac{37}{320} \\times 2560$ |\r\n| \\= {{{correctAnswer0}}} |\n</div>"}]},{"vars":[{"varval":"A random sample of people were surveyed on their favourite breed of dog.\r\n\r\nThe table below recorded the results.\n\n\n\n<div class=\"sm-table row1-color1\">\n\n>>| Breed of Dog| Number of People|\n|:-:|:-:|\n| Border Collie| 65|\n| Cavoodle| 22 |\n| Golden Retriever| 125|\n| Labrador| 75 | \n| Maltese Cross| 36 |\n| Staffordshire Bull Terrier| 52|\n\n</div>\n\n<br>\n\nUsing the data from the survey, predict how many people would choose a Cavoodle if 3000 people were surveyed?"},{"varval":"sm_nogap Total people surveyed\n\n<div class=\"aligned\">\r\n\r\n>>| |\r\n| ---------- |\r\n| \\= 65 + 22 + 125 +75 + 36 + 52 |\r\n| \\= 375 |\r\n\r\n</div>\n\n<br>\n\nsm_nogap $\\therefore$ Predicted number to choose a Cavoodle\n\n<div class=\"aligned\">\r\n\r\n>>| |\r\n| ---------- |\r\n| \\= $\\dfrac{22}{375} \\times 3000$ |\r\n| \\= {{{correctAnswer0}}} |\n</div>"}]}]

  574. <div class="sm_mode"> Ranch is tiling his bathroom. <br> sm_img https://teacher.smartermaths.com.au/wp-content/uploads/2018/06/NAPX-F4-NC25.svg 400 indent vpad <br>He follows this pattern to make a line of tiles that is 60 cm long. How many tiles are in this line? </div>

    [{"vars":null}]

  575. <div class="sm_mode"> {{{question}}} </div>

    [{"vars":[{"varval":"Fifty football fans were surveyed about modes of transport they took to get to the game.\r\n\r\n\n* 22 caught a train\r\n* 21 caught a bus\r\n* 20 didn't take a train or a bus.\r\n\nThe diagram is missing some information.\n\n<br>\n\nsm_img https://teacher.smartermaths.com.au/wp-content/uploads/2018/04/NAPX-J4-CA24.svg 250 indent3 vpad\r\n\n<br>How many caught both a train and a bus to get to the game?\n"},{"varval":"Interpreting the Venn diagram:\r\n\nsm_nogap Since fifty fans surveyed,\n\n\r\n* 20 didn't take either train or bus\r\n* 9 caught train only\r\n* 8 caught bus only\n\nsm_nogap $\\therefore$ Number that took both train and bus\n\n<div class=\"aligned\">\r\n\r\n>>| |\r\n| ---------- |\r\n| \\= $50\\ −\\ (20 + 9 + 8)$ |\r\n| \\= {{{correctAnswer0}}} |\r\n\r\n</div>"}]},{"vars":[{"varval":"Eighty students were surveyed about instruments they play.\r\n\r\n\n* 39 play the piano\r\n* 18 play the guitar\r\n* 30 didn't play either instrument\r\n\nThe diagram is missing some information.\n\n<br>\n\nsm_img https://teacher.smartermaths.com.au/wp-content/uploads/2022/08/Stat_Prob_NAPX-J4-CA24-SA_v1.svg 250 indent3 vpad\r\n\n<br>How many students play both the piano and the guitar?\n"},{"varval":"Interpreting the Venn diagram:\r\n\nsm_nogap Since eighty students surveyed,\n\n\r\n* 30 didn't play either piano or guitar\r\n* 32 play piano only\r\n* 11 play guitar only\n\nsm_nogap $\\therefore$ Number that play both piano and guitar\n\n<div class=\"aligned\">\r\n\r\n>>| |\r\n| ---------- |\r\n| \\= $80\\ −\\ (30 + 32 + 11)$ |\r\n| \\= {{{correctAnswer0}}} |\r\n\r\n</div>"}]},{"vars":[{"varval":"Ninety six athletes were surveyed about the physiotherapist they had used during the season.\r\n\r\n\n* 45 used physio 1 \r\n* 30 used physio 2\r\n* 26 didn't use a physio\r\n\nThe diagram is missing some information.\n\n<br>\n\nsm_img https://teacher.smartermaths.com.au/wp-content/uploads/2022/08/Stat_Prob_NAPX-J4-CA24-SA_v2.svg 250 indent3 vpad\r\n\n<br>How many athletes used both physio 1 and physio 2?\n"},{"varval":"Interpreting the Venn diagram:\r\n\nsm_nogap Ninety six athletes surveyed (given)\n\n\r\n* 26 didn't use physio 1 or physio 2\r\n* 40 used physio 1 only\r\n* 25 used physio 2 only\n\nsm_nogap $\\therefore$ Number that used both physio 1 and 2\n\n<div class=\"aligned\">\r\n\r\n>>| |\r\n| ---------- |\r\n| \\= $96\\ −\\ (26 + 40 + 25)$ |\r\n| \\= {{{correctAnswer0}}} |\r\n\r\n</div>"}]},{"vars":[{"varval":"Eighty eight shoppers at a department store were surveyed about the perfume they like.\r\n\r\n\n* 27 liked perfume 1\r\n* 33 liked perfume 2\r\n* 37 didn't like either perfume\r\n\nThe diagram is missing some information.\n\n<br>\n\nsm_img https://teacher.smartermaths.com.au/wp-content/uploads/2022/08/Stat_Prob_NAPX-J4-CA24-SA_v3.svg 300 indent3 vpad\r\n\n<br>How many shoppers liked both the perfume A and perfume B?\n"},{"varval":"Interpreting the Venn diagram:\r\n\nsm_nogap Eighty eight shoppers were surveyed (given)\n\n\r\n* 37 didn't like either perfume 1 or perfume 2\r\n* 18 liked perfume 1 only\r\n* 24 liked perfume 2 only\n\nsm_nogap $\\therefore$ Number that like both perfume 1 and perfume 2\n\n<div class=\"aligned\">\r\n\r\n>>| |\r\n| ---------- |\r\n| \\= $88\\ −\\ (37 + 18 + 24)$ |\r\n| \\= {{{correctAnswer0}}} |\r\n\r\n</div>"}]},{"vars":[{"varval":"Two hundred people were surveyed about the fruits they like.\r\n\r\n\n* 73 like bananas\r\n* 60 like apples\r\n* 83 didn't like either bananas or apples\r\n\nThe diagram is missing some information.\n\n<br>\n\nsm_img https://teacher.smartermaths.com.au/wp-content/uploads/2022/08/Stat_Prob_NAPX-J4-CA24-SA_v4.svg 300 indent3 vpad\r\n\n<br>How many people like both bananas and apples?\n"},{"varval":"Interpreting the Venn diagram:\r\n\nsm_nogap Two hundred people were surveyed (given)\n\n\r\n* 83 didn't like either bananas or apples\r\n* 57 like bananas only\r\n* 44 like apples only\n\nsm_nogap $\\therefore$ Number that like both bananas and apples\n\n<div class=\"aligned\">\r\n\r\n>>| |\r\n| ---------- |\r\n| \\= $200\\ −\\ (57 + 44 + 83)$ |\r\n| \\= {{{correctAnswer0}}} |\r\n\r\n</div>"}]},{"vars":[{"varval":"Three hundred people were surveyed about the movie cinemas they have attended in the last 6 months.\r\n\r\n\n* 160 have attended Events\r\n* 140 have attended Hoyts\r\n* 50 didn't attend either cinema\r\n\nThe diagram is missing some information.\n\n<br>\n\nsm_img https://teacher.smartermaths.com.au/wp-content/uploads/2022/08/Stat_Prob_NAPX-J4-CA24-SA_v5.svg 300 indent3 vpad\r\n\n<br>How many people had attended both Events and Hoyts cinemas?\n"},{"varval":"Interpreting the Venn diagram:\r\n\nsm_nogap Three hundred people were surveyed (given)\n\n\r\n* 50 didn't attend either Events or Hoyts\r\n* 110 attended Events only\r\n* 90 attended Hoyts only\n\nsm_nogap $\\therefore$ Number that attended both Events and Hoyts\n\n<div class=\"aligned\">\r\n\r\n>>| |\r\n| ---------- |\r\n| \\= $300\\ −\\ (50 + 110 + 90)$ |\r\n| \\= {{{correctAnswer0}}} |\r\n\r\n</div>"}]}]

  576. <div class="sm_mode"> {{{question}}} </div>

    [{"vars":[{"varval":"Steph randomly picks one disk from a group of white and coloured disks, pictured below.\n\n<br>\n\nsm_img https://teacher.smartermaths.com.au/wp-content/uploads/2018/07/NAPX-H3-CA05.svg 300 indent3 vpad\n\n<br>What is the probability Steph chooses a coloured disk?"},{"varval":"<div class=\"aligned\">\r\n\r\n| | |\r\n| ------------- | ---------- |\r\n| $P$(Coloured disk) | \\= $\\dfrac{\\text{Number of coloured disks}}{\\text{Total disks}}$ |\r\n| | \\= {{{correctAnswer}}} |\r\n\r\n</div>\r"}]},{"vars":[{"varval":"Clare randomly picks one disk from a group of white and coloured disks, pictured below.\n\n<br>\n\nsm_img https://teacher.smartermaths.com.au/wp-content/uploads/2018/07/NAPX-H3-CA05.svg 300 indent3 vpad\n\n<br>What is the probability Clare chooses a white disk?"},{"varval":"<div class=\"aligned\">\r\n\r\n| | |\r\n| ------------- | ---------- |\r\n| $P$(White disk) | \\= $\\dfrac{\\text{Number of white disks}}{\\text{Total disks}}$ |\r\n| | \\= {{{correctAnswer}}} |\r\n\r\n</div>\r"}]}]

  577. <div class="sm_mode"> {{{question}}} </div>

    [{"vars":[{"varval":"A new museum records the number of visitors over 5 days.\n\n<br>\n\n<div class=\"sm-table row1-color8\">\n\n>>| Day | Visitors |\n|:-:|:-:|\n| 1 | 220| \n| 2 | 200| \n| 3 | 230| \n| 4 | 220| \n| 5 | 180| \n\n</div>\n\n<br>The cost of entry for each visitor was $12.\r\n\r\nWhat was the mean total entry fee collected per day?"},{"varval":"sm_nogap Mean number of visitors\n\n<div class=\"aligned\">\r\n\r\n>>| |\r\n| ---------- |\r\n| \\= $(220+200+230+220+180) \\div 5$ |\r\n| \\= 210 |\r\n\r\n</div>\n\n\r\n<br>\n\n<div class=\"aligned\">\r\n\r\n| | |\r\n| ------------- | ---------- |\r\n| $\\therefore$ Mean entry fees per day | \\= $210 \\times 12$ |\r\n| | \\= {{{prefix0}}}{{{correctAnswer0}}} |\r\n\r\n</div>\r\n"}]},{"vars":[{"varval":"A bowling rink records the number of visitors over 5 days.\n\n<br>\n\n<div class=\"sm-table row1-color3\">\n\n>>| Day | Visitors |\n|:-:|:-:|\n| 1 | 120| \n| 2 | 80| \n| 3 | 100| \n| 4 | 60| \n| 5 | 140| \n\n</div>\n\n<br>The cost of entry for each visitor was $21.\r\n\r\nWhat was the mean total entry fee collected per day?"},{"varval":"sm_nogap Mean number of visitors\n\n<div class=\"aligned\">\r\n\r\n>>| |\r\n| ---------- |\r\n| \\= $(120+80+100+60+140) \\div 5$ |\r\n| \\= 100 |\r\n\r\n</div>\n\n\r\n<br>\n\n<div class=\"aligned\">\r\n\r\n| | |\r\n| ------------- | ---------- |\r\n| $\\therefore$ Mean entry fees per day | \\= $100 \\times 21$ |\r\n| | \\= {{{prefix0}}}{{{correctAnswer0}}} |\r\n\r\n</div>\r\n"}]},{"vars":[{"varval":"An aquarium records the number of visitors over 7 days.\n\n<br>\n\n<div class=\"sm-table row1-color5\">\n\n>>| Day | Visitors |\n|:-:|:-:|\n| 1 | 540| \n| 2 | 660| \n| 3 | 400| \n| 4 | 320| \n| 5 | 450|\n| 6 | 600|\n| 7 | 880| \n\n</div>\n\n<br>The cost of entry for each visitor was $40.\r\n\r\nWhat was the mean total entry fee collected per day?"},{"varval":"sm_nogap Mean number of visitors\n\n<div class=\"aligned\">\r\n\r\n>>| |\r\n| ---------- |\r\n| \\= $(540+660+400+320+450+600+880) \\div 7$ |\r\n| \\= 550 |\r\n\r\n</div>\n\n\r\n<br>\n\n<div class=\"aligned\">\r\n\r\n| | |\r\n| ------------- | ---------- |\r\n| $\\therefore$ Mean entry fees per day | \\= $$40 \\times 550$ |\r\n| | \\= $22 000 |\r\n\r\n</div>\r\n"}]},{"vars":[{"varval":"A zoo records the number of visitors it receives over 5 days.\n\n<br>\n\n<div class=\"sm-table row1-color4\">\n\n>>| Day | Visitors |\n|:-:|:-:|\n| 1 | 560| \n| 2 | 880| \n| 3 | 1500| \n| 4 | 2400| \n| 5 | 2900| \n\n</div>\n\n<br>The cost of entry for each visitor was $32.\r\n\r\nWhat was the mean total entry fee collected per day?"},{"varval":"sm_nogap Mean number of visitors\n\n<div class=\"aligned\">\r\n\r\n>>| |\r\n| ---------- |\r\n| \\= $(560+880+1500+2400+2900) \\div 5$ |\r\n| \\= 1648 |\r\n\r\n</div>\n\n\r\n<br>\n\n<div class=\"aligned\">\r\n\r\n| | |\r\n| ------------- | ---------- |\r\n| $\\therefore$ Mean entry fees per day | \\= $$32 \\times 1648$ |\r\n| | \\= $52 736 |\r\n\r\n</div>\r\n"}]},{"vars":[{"varval":"A Universal Studios Los Angeles records the number of visitors it receives over 7 days.\n\n<br>\n\n<div class=\"sm-table row1-color2\">\n\n>>| Day | Visitors |\n|:-:|:-:|\n| 1 | 19 657| \n| 2 | 21 300| \n| 3 | 20 900| \n| 4 | 18 568| \n| 5 | 15 700|\n| 6 | 24 600|\n| 7 | 18 449| \n\n</div>\n\n<br>The cost of entry for each visitor was $45.\r\n\r\nWhat was the mean total entry fee collected per day?"},{"varval":"sm_nogap Mean number of visitors\n\n<div class=\"aligned\">\r\n\r\n>>| |\r\n| ---------- |\r\n| \\= $(19\\ 657+21\\ 300+20\\ 900+18\\ 568+15\\ 700+24\\ 600+18\\ 449) \\div 7$ |\r\n| \\= 19 882 |\r\n\r\n</div>\n\n\r\n<br>\n\n<div class=\"aligned\">\r\n\r\n| | |\r\n| ------------- | ---------- |\r\n| $\\therefore$ Mean entry fees per day | \\= $$45 \\times 19882$ |\r\n| | \\= $894 690 |\r\n\r\n</div>\r\n"}]},{"vars":[{"varval":"A ninja park records the number of visitors over 5 days.\n\n<br>\n\n<div class=\"sm-table row1-color6\">\n\n>>| Day | Visitors |\n|:-:|:-:|\n| 1 | 72| \n| 2 | 68| \n| 3 | 46| \n| 4 | 84| \n| 5 | 110| \n\n</div>\n\n<br>The cost of entry for each visitor was $12.\r\n\r\nWhat was the mean total entry fee collected per day?"},{"varval":"sm_nogap Mean number of visitors\n\n<div class=\"aligned\">\r\n\r\n>>| |\r\n| ---------- |\r\n| \\= $(72+68+46+84+110) \\div 5$ |\r\n| \\= 76 |\r\n\r\n</div>\n\n\r\n<br>\n\n<div class=\"aligned\">\r\n\r\n| | |\r\n| ------------- | ---------- |\r\n| $\\therefore$ Mean entry fees per day | \\= $$12 \\times 76$ |\r\n| | \\= {{{prefix0}}}{{{correctAnswer0}}} |\r\n\r\n</div>\r\n"}]}]

  578. <div class="sm_mode"> {{{question}}} </div>

    [{"vars":[{"varval":"Angelica made a dot plot to show the distances she has run in her training for a marathon.\n\nsm_img https://teacher.smartermaths.com.au/wp-content/uploads/2018/04/NAPX-I4-CA211.svg 400 indent vpad\r\n\nWhat is the range of the distances Angelica has run?\n"},{"varval":"\n<div class=\"aligned\">\r\n\r\n| | |\r\n| ------------- | ---------- |\r\n| Range | \\= high − low |\n| | \\= $25 - 18$ |\r\n| | \\= {{{correctAnswer}}} km |\r\n\r\n</div>\r\n"}]},{"vars":[{"varval":"Eve made a dot plot to show the sales of pet mince, in kilograms, she made during the day.\n\nsm_img https://teacher.smartermaths.com.au/wp-content/uploads/2022/08/Stats_Prob_NAPX-I4-CA21_v1a.svg 310 indent vpad\r\n\nWhat is the range of the kilograms of pet mince she sold?\n"},{"varval":"\n<div class=\"aligned\">\r\n\r\n| | |\r\n| ------------- | ---------- |\r\n| Range | \\= high − low |\n| | \\= $6 - 1$ |\r\n| | \\= {{{correctAnswer}}} kg |\r\n\r\n</div>\r\n"}]},{"vars":[{"varval":"Brandon made a dot plot to show the hours he worked over the last 16 weeks.\n\n<br>\n\nsm_img https://teacher.smartermaths.com.au/wp-content/uploads/2022/08/Stats_Prob_NAPX-I4-CA21_v2.svg 370 indent vpad\r\n\nWhat is the median number of hours that Brandon worked?\n"},{"varval":"\n<div class=\"aligned\">\r\n\r\n| | |\r\n| ------------- | ---------- |\r\n| Median | \\= Middle score when arranged in order |\n| | \\= Average of the 8th and 9th scores|\n| | \\= $\\dfrac{15+16}{2}$|\r\n| | \\= {{{correctAnswer}}} hours |\r\n\r\n</div>\r\n"}]},{"vars":[{"varval":"Leila made a dot plot to show the hours she walked each day whilst trekking in Nepal.\n\n<br>\n\nsm_img https://teacher.smartermaths.com.au/wp-content/uploads/2022/08/Stats_Prob_NAPX-I4-CA21_v3.svg 420 indent vpad\r\n\nWhat is the range of the hours that Leila trekked?"},{"varval":"<div class=\"aligned\">\r\n\r\n| | |\r\n| ------------- | ---------- |\r\n| Range | \\= 10 $-$ 0 |\n| | \\= {{{correctAnswer}}} hours |\r\n\r\n</div>"}]},{"vars":[{"varval":"Jennifer made a dot plot to show her maximum heart rate during exercise, in beats per minute, over 20 workouts.\n\n<br>\n\nsm_img https://teacher.smartermaths.com.au/wp-content/uploads/2022/08/Stats_Prob_NAPX-I4-CA21_v4.svg 580 indent vpad\r\n\nWhat is her median maximum heart rate?\n"},{"varval":"\n<div class=\"aligned\">\r\n\r\n| | |\r\n| ------------- | ---------- |\r\n| Median | \\= Middle score when arranged in order |\n| | \\= Average of the 10th and 11th scores|\n| | \\= $\\dfrac{169+171}{2}$|\r\n| | \\= {{{correctAnswer}}} beats per minute |\r\n\r\n</div>"}]},{"vars":[{"varval":"Allen made a dot plot to show the number of eggs he collected from his chickens over the last 14 days.\n\n<br>\n\nsm_img https://teacher.smartermaths.com.au/wp-content/uploads/2022/08/Stats_Prob_NAPX-I4-CA21_v5a.svg 310 indent3 vpad\r\n\nWhat is the mean number of eggs Allen collected?\n"},{"varval":"\n<div class=\"aligned\">\r\n\r\n| | |\r\n| ------------- | ---------- |\r\n| Mean | \\= Average of the scores |\n| | \\= $\\dfrac{1+1+1+1+1+3+3+4+4+4+4+5+5+5}{14}$|\r\n| | \\= $\\dfrac{42}{14}$|\n| | \\= {{{correctAnswer}}} eggs |\r\n\r\n</div>"}]}]

  579. <div class="sm_mode"> {{{question}}} </div>

    [{"vars":[{"varval":"Suzie asked all the students in her primary school how many brothers and sisters they have.\r\n\r\nShe used the results to create the bar chart below but left off some labels.\n\n<br>\n\nsm_img https://teacher.smartermaths.com.au/wp-content/uploads/2018/08/NAPX-G3-CA27-SA.svg 300 indent3 vpad\r\n\n<br>Suzie's primary school has 100 students.\r\n\r\nHow many students have over 4 brothers and sisters?\n"},{"varval":"Since there are 100 students and 5 columns, the\r average number of students per column is 20.\r\n\n$\\rArr$ Each horizontal interval = 4 students\r\n \r\n\r\nsm_nogap $\\therefore$ Students with more than 4\n\n<div class=\"aligned\">\r\n\r\n>>| |\r\n| ---------- |\r\n| \\= $5.5 \\times 4$ |\r\n| \\= {{{correctAnswer0}}} |\r\n\r\n</div>"}]},{"vars":[{"varval":"Kevin asked all the people at a pet show how many pets they have.\r\n\r\nHe used the results to create the bar chart below but left off some labels.\n\n<br>\n\nsm_img https://teacher.smartermaths.com.au/wp-content/uploads/2022/08/Stat_Prob_NAPX-G3-CA27-SA_v1_1.svg 320 indent3 vpad\r\n\n<br>There were 172 people at the pet show.\r\n\r\nHow many people at the pet show have over 4 pets?\n"},{"varval":"There are 172 people at the pet show.\n\nCalculate the number of people in 1 interval (vertical) of the graph:\n\nsm_nogap $\\therefore$ Number of people per interval\n\n<div class=\"aligned\">\r\n\r\n>>| |\r\n| ---------- |\r\n| \\= $172 \\div (4 + 3.5 + 5 + 3.5 + 2.5 + 3)$ |\r\n| \\= $172 \\div 21.5$ |\n| \\= 8 |\r\n\r\n</div>\n\n\n\r\n\r\nsm_nogap $\\therefore$ People with more than 4 pets\n\n<div class=\"aligned\">\r\n\r\n>>| |\r\n| ---------- |\r\n| \\= $3 \\times 8$ |\r\n| \\= {{{correctAnswer0}}} |\r\n\r\n</div>\n"}]},{"vars":[{"varval":"Jesse kept track of the goals he scored in the 2022 NPL soccer season.\r\n\r\nHe used the results to create the bar chart below but left off some labels.\n\n<br>\n\nsm_img https://teacher.smartermaths.com.au/wp-content/uploads/2022/08/Stat_Prob_NAPX-G3-CA27-SA_v2.svg 320 indent3 vpad\r\n\n<br>Jesse scored 72 goals in the 2022 season.\r\n\r\nOn how many occasions did he score exactly 2 goals?\n"},{"varval":"Jesse scored 72 goals in the 2022 season.\n\nCalculate the number of goals in 1 interval (vertical) of the graph:\n\nsm_nogap $\\therefore$ Number of goals per interval\n\n\n<div class=\"aligned\">\r\n\r\n>>| |\r\n| ---------- |\r\n| \\= $72 \\div (4 + 3.5 + 5 + 3.5 + 2)$ |\r\n| \\= $72 \\div 18$ |\n| \\= 4 |\r\n\r\n</div>\n\n\n\r\n\r\nsm_nogap $\\therefore$ Times Jesse scored exactly 2 goals\n\n<div class=\"aligned\">\r\n\r\n>>| |\r\n| ---------- |\r\n| \\= $5 \\times 4$ |\r\n| \\= {{{correctAnswer0}}} |\r\n\r\n</div>"}]},{"vars":[{"varval":"Ashleigh recorded the number of times the members of her gym visited the gym each week.\n\n\r\n\r\nShe used the results to create the bar chart below but left off some labels.\n\n<br>\n\nsm_img https://teacher.smartermaths.com.au/wp-content/uploads/2022/08/Stat_Prob_NAPX-G3-CA27-SA_v3_1.svg 320 indent3 vpad\r\n\n<br>Ashleigh's gym has 616 members.\n\n\r\n\r\nHow many gym members attend 4 or more times per week?\n"},{"varval":"Ashleigh has 616 gym members.\n\nCalculate the number of visits in 1 interval (vertical) of the graph:\n\nsm_nogap $\\therefore$ Number of visits per interval\n\n\n<div class=\"aligned\">\r\n\r\n>>| |\r\n| ---------- |\r\n| \\= $616 \\div (5 + 4.5 + 3 + 4 + 2.5 + 3.5 + 5.5)$ |\r\n| \\= $616 \\div 28$ |\n| \\= 22 |\r\n\r\n</div>\n\n\n\r\n\r\nsm_nogap $\\therefore$ People who attended the gym 4 or more times\n\n<div class=\"aligned\">\r\n\r\n>>| |\r\n| ---------- |\r\n| \\= $(2.5 + 3.5 +5.5) \\times 22$ |\r\n| \\= $11.5 \\times 22$ |\n| \\= {{{correctAnswer0}}} |\r\n\r\n</div>"}]},{"vars":[{"varval":"Martin recorded the sleep patterns of the residents at an aged care facility on a single night.\n\n\r\n\r\nHe used the results to create the bar chart below but left off some labels.\n\n<br>\n\nsm_img https://teacher.smartermaths.com.au/wp-content/uploads/2022/08/Stat_Prob_NAPX-G3-CA27-SA_v4.svg 330 indent3 vpad\r\n\n<br>There are 174 residents in the aged care facility.\n\n\r\n\r\nHow many residents sleep less than 6 hours per night?\n"},{"varval":"There are 174 residents in the aged care facility.\n\nCalculate the number of residents in 1 interval (vertical) of the graph:\n\nsm_nogap $\\therefore$ Number of residents per interval\n\n<div class=\"aligned\">\r\n\r\n>>| |\r\n| ---------- |\r\n| \\= $174 \\div (2 + 4 + 3.5 + 5.5 + 6 + 4.5 + 3.5)$ |\r\n| \\= $174 \\div 29$ |\n| \\= 6 |\r\n\r\n</div>\n\n\n\r\n\r\nsm_nogap $\\therefore$ Residents who sleep less than 6 hours per night\n\n<div class=\"aligned\">\r\n\r\n>>| |\r\n| ---------- |\r\n| \\= $(2 + 4 + 3.5) \\times 6$ |\r\n| \\= $9.5 \\times 6$ |\n| \\= {{{correctAnswer0}}} |\r\n\r\n</div>"}]},{"vars":[{"varval":"Susan recorded the number of cups of coffee consumed by a group of teachers at her school in a week.\n\n\r\n\r\nShe used the results to create the bar chart below but left off some labels.\n\n<br>\n\nsm_img https://teacher.smartermaths.com.au/wp-content/uploads/2022/08/Stat_Prob_NAPX-G3-CA27-SA_v5_1.svg 330 indent3 vpad\r\n\n<br>In total, there were 128 teachers who took part in Susan's survey.\n\n\r\n\r\nHow many teachers consumed 4 or more cups of coffee per day?\n"},{"varval":"There are 128 teachers taking part in the survey.\n\nCalculate the number of teachers in 1 interval (vertical) of the graph:\n\nsm_nogap $\\therefore$ Number of teachers per interval\n\n<div class=\"aligned\">\r\n\r\n>>| |\r\n| ---------- |\r\n| \\= $128 \\div (3 + 4 + 8 + 7 + 4.5 + 3.5 + 2)$ |\r\n| \\= $128 \\div 32$ |\n| \\= 4 |\r\n\r\n</div>\n\n\n\r\n\r\nsm_nogap $\\therefore$ Number of teachers who consumed 4 or more cups of coffee per day\n\n<div class=\"aligned\">\r\n\r\n>>| |\r\n| ---------- |\r\n| \\= $(4.5 + 3.5 + 2) \\times 4$ |\r\n| \\= $10 \\times 4$ |\n| \\= {{{correctAnswer0}}} |\r\n\r\n</div>"}]}]

  580. <div class="sm_mode"> {{{question}}} </div>

    [{"vars":[{"varval":"Dippa arrived at Toorak station at 11:00 am and caught the next train to Southbank.\n\n<br>\n\n<div class=\"sm-table col1-color3 row1-color3 top-left-cell-hidden\">\n\n>>| | Train A | Train B |Train C|Train D|\n|:-|:-:|:-:|:-:|:-:|\n| Kooyong| 10:44| 10:58|11:10| $-$|\n| Toorak | 10:51| 11:05|$-$|11:19|\n| South Yarra| 11:08| $-$|11:24|11:36|\n| Botanic Gardens | 11:18| 11:22|$-$|11:46|\n| Southbank | 11:30| $-$|11:46|11:58|\n\n</div>\n\n<br>At what time did Dippa arrive at Southbank station?"},{"varval":"Arriving at Toorak at 11:00 am, \r\n\n$\\Rightarrow$ Train A has already left\r\n\n$\\Rightarrow$ Train B doesn't stop at Southbank\r\n\n$\\Rightarrow$ Train C doesn't stop at Toorak\r\n\n$\\Rightarrow$ Train D arrives at 11:58 am\r\n\n$\\therefore$ He arrives at {{{correctAnswer}}} am."}]},{"vars":[{"varval":"Novak arrived at Toorak station at 2:35 pm and caught the next train to Melbourne Park station.\n\n<br>\n\n<div class=\"sm-table col1-color3 row1-color3 top-left-cell-hidden\">\n\n>>| | Train A | Train B |Train C|Train D|\n|:-|:-:|:-:|:-:|:-:|\n| Kooyong| 2:24 pm| 2:44 pm|3:04 pm| $-$|\n| Toorak | 2:31 pm| $-$|3:11 pm|3:23 pm|\n| South Yarra| 2:46 pm| $-$|3:26 pm|3:38 pm|\n| Botanic Gardens | 2:56 pm| 3:16 pm|3:36 pm|3:48 pm|\n| Melbourne Park | 3:08 pm| 3:28 pm|$-$ |4:00 pm|\n\n</div>\n\n<br>At what time did Novak arrive at Melbourne Park station?"},{"varval":"Arriving at Toorak at 2:35 pm, \r\n\n$\\Rightarrow$ Train A has already left\r\n\n$\\Rightarrow$ Train B doesn't stop at Toorak\n\n$\\Rightarrow$ Train C doesn't stop at Melbourne Park\n\n$\\Rightarrow$ Train D arrives at 4:00 pm\n\n$\\therefore$ Novak arrives at {{{correctAnswer}}}."}]}]

  581. <div class="sm_mode"> {{{question}}} </div>

    [{"vars":[{"varval":"The median mass of a group of emperor penguins at the zoo is 23 kg.\r\n\r\nThe mean mass of the same group is 25 kg.\r\n\r\nIf a new 29 kg penguin joins the group, which of these is possible?"},{"varval":"It is possible for the mean mass to increase and\r\nthe median mass to stay the same.\n"}]},{"vars":[{"varval":"The median high jump height of primary school students competing at the state carnival is 1.1 metres.\r\n\r\nThe mean high jump height of the same group is 0.9 metres.\r\n\r\nIf a new competitor then records a jump of 0.8 metres, which of these is possible?"},{"varval":"It is possible for the mean height to decrease and\r\nthe median height to stay the same.\n"}]},{"vars":[{"varval":"The mean diameter of a batch of cookies is 8.5 centimetres.\r\n\r\nThe median diameter of the same batch of cookies is 8.9 centimetres.\r\n\r\nIf a new cookie with a diameter of 7 centimetres is added to the batch, which of these is possible?"},{"varval":"It is possible for the mean diameter to decrease and\r\nthe median diameter to stay the same.\n"}]},{"vars":[{"varval":"The median wingspan of a group of wedge tailed eagles is 2.6 metres.\r\n\r\nThe mean wingspan of the same group is 2.5 metres.\r\n\r\nIf a new wedge tailed eagle joins the group with a wingspan of 2.7 metres, which of these is possible?"},{"varval":"{{{correctAnswer}}}"}]},{"vars":[{"varval":"The mean age of a group of friends is 26 years.\r\n\r\nThe median age of the same group is 28 years.\r\n\r\nIf an 18 year old joins the group, which of these is possible?"},{"varval":"It is possible for the mean age to decrease and the median age decreases"}]},{"vars":[{"varval":"The median price of houses sold in a suburb is $850 000.\r\n\r\nThe mean price of the same group of houses is $925 000.\r\n\r\nIf another house in the suburb sells for $1.2 million, which of these is possible?"},{"varval":"It is possible for the mean house price to increase and the median house price to stay the same."}]}]

  582. var 1: Statistics and Probability, NAPX-L4-CA20 v1 var2: Statistics and Probability, NAPX-L4-CA20 Var3-5 ... can we get a lighter tone of each colour and see 9if the font can be closer to question writing. See colour palette here: c9383723-23a7-4f92-9f7f-2061efecd181 - This is this question? I've toned down the colours and the font I've used is Segoe UI Variable Display which is what comes up when I highlight and copy the questions into word. if you can let me know the correct font I can change.

    <div class="sm_mode"> {{{question}}} </div>

    [{"vars":[{"varval":"The stem-and-leaf plot shows the number of lunch diners a small cafe had each day for the last 2 weeks.\n\n<br>\n\nsm_img https://teacher.smartermaths.com.au/wp-content/uploads/2020/04/NAPX-L4-20-ver3.svg 410 indent3 vpad\r\n\n<br>On how many days were there at least 25 people dining at lunch?"},{"varval":"The plot shows 7 data points that are at least 25.\n\n$\\therefore$ 7 days"}]},{"vars":[{"varval":"The stem-and-leaf plot shows the number of movie tickets sold to a foreign film over 15 screenings.\n\n<br>\n\nsm_img https://teacher.smartermaths.com.au/wp-content/uploads/2020/04/NAPX-L4-20-ver2.svg 410 indent3 vpad\n\n\r\n<br>In how many screenings were there at least 30 tickets sold?"},{"varval":"The plot shows 8 data points that are 30 or above.\r\n\n$\\therefore$ 8 screenings"}]},{"vars":[{"varval":"The stem-and-leaf plot shows the number of nippers who attended surf club training each day for the last two weeks.\n\n<br>\n\nsm_img https://teacher.smartermaths.com.au/wp-content/uploads/2020/04/NAPX-L4-20-ver1.svg 410 indent3 vpad\r\n\n<br>On how many days were there at least 40 nippers at training?"},{"varval":"The plot shows 4 data points that are 40 or above.\r\n\n$\\therefore$ 4 days"}]},{"vars":[{"varval":"The stem-and-leaf plot shows the number of buckets of strawberries picked by visitors to a strawberry farm each day for the last two weeks.\n\n<br>\n\nsm_img https://teacher.smartermaths.com.au/wp-content/uploads/2022/09/Stat_Prob_NAPX-J4-CA38_NAPX-J3-CA37_2_v3.svg 410 indent3 vpad\r\n\n<br>On how many days were there at least 25 buckets picked?"},{"varval":"The plot shows 10 data points that are 25 or above.\r\n\n$\\therefore$ 10 days"}]},{"vars":[{"varval":"The stem-and-leaf plot shows the highest points per game scored by LeBron James in the NBA seasons from 2003 to 2021.\n\n<br>\n\nsm_img https://teacher.smartermaths.com.au/wp-content/uploads/2022/09/Stat_Prob_NAPX-J4-CA38_NAPX-J3-CA37_2_v4.svg 500 indent3 vpad\r\n\n<br>In how many seasons was his highest game score at least 76?"},{"varval":"The plot shows 11 data points that are 76 or above.\r\n\n$\\therefore$ 11 seasons"}]},{"vars":[{"varval":"The stem-and-leaf plot shows the number of boats launched at the local boat ramp during two weeks of the summer holidays.\n\n<br>\n\nsm_img https://teacher.smartermaths.com.au/wp-content/uploads/2022/09/Stat_Prob_NAPX-J4-CA38_NAPX-J3-CA37_2_v5.svg 420 indent3 vpad\r\n\n<br>On how many days were there at least 12 but less than 31 boats launched?"},{"varval":"The plot shows 7 data points that are at least 12 but less than 31.\r\n\n$\\therefore$ 7 days"}]}]

  583. <div class="sm_mode"> {{{question}}} </div>

    [{"vars":[{"varval":"The table below shows the relationship between the width of a tree's trunk in centimetres and how old it is in months.\n\n<br>\n\n<div class=\"sm-table col1-color1\">\n\n>>| Age in months | 6|9|12|15|18|\n|:-:|:-:|:-:|:-:|:-:|:-:|\n| Width of trunk (cm) | 6$\\frac{1}{8}$|6$\\frac{1}{4}$|6$\\frac{3}{8}$|6$\\frac{1}{2}$|6$\\frac{5}{8}$|\n\n</div>\n\n<br>If this pattern continues, how old would you expect a tree with a 7 cm wide trunk to be?"},{"varval":"Continuing the pattern adding $\\frac{1}{8}$:\n\n<div class=\"sm-table col1-color1\">\n\n>>| Age in months | 18|21|24|27|\n|:-:|:-:|:-:|:-:|:-:|\n| Width of trunk (cm) | 6$\\frac{5}{8}$|6$\\frac{3}{4}$|6$\\frac{7}{8}$|7|\n\n</div>\n\n<br>$\\therefore$ Expected age = {{{correctAnswer0}}} {{{suffix0}}}"}]},{"vars":[{"varval":"The table below shows the relationship between the length of a gecko in centimetres and how old it is in months.\n\n<br>\n\n<div class=\"sm-table col1-color3\">\n\n>>| Age in months | 4|6|8|10|12|\n|:-:|:-:|:-:|:-:|:-:|:-:|\n| Length (cm) | 8$\\frac{3}{8}$|8$\\frac{1}{2}$|8$\\frac{5}{8}$|8$\\frac{3}{4}$|8$\\frac{7}{8}$|\n\n</div>\n\n<br>If this pattern continues, how old would you expect a 9.5 cm gecko to be?"},{"varval":"Continuing the pattern adding $\\frac{1}{8}$:\n\n<div class=\"sm-table col1-color3\">\n\n>>| Age in months | 12|14|16|18|20|22|\n|:-:|:-:|:-:|:-:|:-:|:-:|:-:|\n| Length (cm) | 8$\\frac{7}{8}$|9|9$\\frac{1}{8}$|9$\\frac{1}{4}$|9$\\frac{3}{8}$|9$\\frac{1}{2}$|\n\n</div>\n\n<br>$\\therefore$ Expected age = {{{correctAnswer0}}} {{{suffix0}}}"}]},{"vars":[{"varval":"The table below shows the relationship between the length of a garden worm in centimetres and how old it is in months.\n\n<br>\n\n<div class=\"sm-table col1-color2\">\n\n>>| Age in months | 7|10|13|16|19|\n|:-:|:-:|:-:|:-:|:-:|:-:|\n| Length (cm) | 2$\\frac{3}{5}$|2$\\frac{4}{5}$|3|3$\\frac{1}{5}$|3$\\frac{2}{5}$|\n\n</div>\n\n<br>If this pattern continues, how old would you expect a 4.4 cm worm to be?"},{"varval":"Continuing the pattern adding $\\frac{1}{5}$:\n\n<div class=\"sm-table col1-color2\">\n\n>>| Age in months | 19|22|25|28|31|34|\n|:-:|:-:|:-:|:-:|:-:|:-:|:-:|\n| Length (cm) | 3$\\frac{2}{5}$|3$\\frac{3}{5}$|3$\\frac{4}{5}$|4|4$\\frac{1}{5}$|4$\\frac{2}{5}$|\n\n</div>\n\n<br>$\\therefore$ Expected age = {{{correctAnswer0}}} {{{suffix0}}}"}]},{"vars":[{"varval":"The table below shows the relationship between the weight of a sea slug in grams and how old it is in days.\n\n<br>\n\n<div class=\"sm-table col1-color4\">\n\n>>| Age in days| 4|8|12|16|20|\n|:-:|:-:|:-:|:-:|:-:|:-:|\n| Weight in grams (g) | 19$\\frac{4}{5}$|20$\\frac{1}{10}$|20$\\frac{2}{5}$|20$\\frac{7}{10}$|21|\n\n</div>\n\n<br>If this pattern continues, how old would you expect a sea slug with a weight of 22.2 g to be?"},{"varval":"Continuing the pattern adding $\\frac{3}{10}$:\n\n<div class=\"sm-table col1-color4\">\n\n>>| Age in days| 20|24|28|32|36|\n|:-:|:-:|:-:|:-:|:-:|:-:|\n| Weight in grams (g) | 21|21$\\frac{3}{10}$|21$\\frac{3}{5}$|21$\\frac{9}{10}$|22$\\frac{1}{5}$|\n\n</div>\n\n<br>$\\therefore$ Expected age = {{{correctAnswer0}}} {{{suffix0}}}"}]},{"vars":[{"varval":"The table below shows the relationship between the height of a seedling in millimetres and how old it is in days.\n\n<br>\n\n<div class=\"sm-table col1-color5\">\n\n>>| Age in days| 20|24|28|32|36|\n|:-:|:-:|:-:|:-:|:-:|:-:|\n| Height of seedling (mm) | 40$\\frac{7}{8}$|41$\\frac{1}{4}$|41$\\frac{5}{8}$|42|42$\\frac{3}{8}$|\n\n</div>\n\n<br>If this pattern continues, how old would you expect a seedling with a height of 43.5 mm to be?"},{"varval":"Continuing the pattern adding $\\frac{3}{8}$:\n\n<div class=\"sm-table col1-color5\">\n\n>>| Age in days| 36|40|44|48|\n|:-:|:-:|:-:|:-:|:-:|\n| Height of seedling (mm) | 42$\\frac{3}{8}$|42$\\frac{3}{4}$|43$\\frac{1}{8}$|43$\\frac{1}{2}$|\n\n</div>\n\n<br>$\\therefore$ Expected age = {{{correctAnswer0}}} {{{suffix0}}}"}]}]

  584. Var2 answer was incorrect (0.425) changed to 0.4225 and adjusted MC answers

    <div class="sm_mode"> {{{question}}} </div>

    [{"vars":[{"varval":"A square table top has an area of 9025 square centimetres.\n\r\nWhat is the area of the table top in square millimetres?"},{"varval":"Method 1\n\n\r\nDimensions of square with area 9025 cm$^2$ is:\n\n\r\n95 cm × 95 cm\n\n\r\nConverting to millimetres:\n\n\r\n950 mm × 950 mm = {{{correctAnswer}}}\n\n<br>\n\n\r\n\r\n\r\nMethod 2\n\n\r\nScale factor of converting cm to mm = 10\n\n\r\nScale factor of converting cm$^2$ to mm$^2$ = 10 × 10 = 100\n\n\r\n$\\therefore$ 9025 cm$^2$ × 100 = {{{correctAnswer}}}"}]},{"vars":[{"varval":"A large square feature tile has an area of 15 625 square centimetres.\n\r\nWhat is the area of the feature tile in square millimetres?"},{"varval":"Dimensions of square with area 15 625 cm$^2$ is:\n\n125 cm × 125 cm\n\nConverting to millimetres:\n\n\r\n1250 mm × 1250 mm = {{{correctAnswer}}}\n\n<br>\n\n\r\n\r\n\r\nMethod 2\n\n\r\nScale factor of converting cm to mm = 10\n\n\r\nScale factor of converting cm$^2$ to mm$^2$ = 10 × 10 = 100\n\n\r\n$\\therefore$ 15 625 cm$^2$ × 100 = {{{correctAnswer}}}"}]},{"vars":[{"varval":"A square table top has an area of 4225 square centimetres.\n\r\nWhat is the area of the table top in square metres?"},{"varval":"Dimensions of square with area 4225 cm$^2$ is:\n\n65 cm × 65 cm\n\nConverting to metres:\n\n\r\n0.65 m × 0.65 m = {{{correctAnswer}}}\n\n<br>\n\n\r\n\r\n\r\nMethod 2\n\n\r\nScale factor of converting cm to m = $\\dfrac{1}{100}$\n\n\r\nScale factor of converting cm$^2$ to mm$^2$\n\n>= $\\dfrac{1}{100}\\ \\times\\ \\dfrac{1}{100}$\n\n<br>\n\n>= $\\dfrac{1}{10\\ 000}$\n\n<br>\n\n\r\n$\\therefore$ 4225 cm$^2$ × $\\dfrac{1}{10\\ 000}$ = {{{correctAnswer}}}"}]},{"vars":[{"varval":"A square fridge magnet has an area of 900 square millimetres.\n\r\nWhat is the area of the fridge magnet in square centimetres?"},{"varval":"Dimensions of square with area 900 mm$^2$ is:\n\n30 mm × 30 mm\n\nConverting to centimetres:\n\n\r\n3 cm × 3 cm = {{{correctAnswer}}}\n\n<br>\n\n\r\n\r\n\r\nMethod 2\n\n\r\nScale factor of converting mm to cm = $\\dfrac{1}{10}$\n\n\r\nScale factor of converting mm$^2$ to cm$^2$\n\n>= $\\dfrac{1}{10}\\ \\times\\ \\dfrac{1}{10}$\n\n<br>\n\n>= $\\dfrac{1}{100}$\n\n<br>\n\n\r\n$\\therefore$ 900 mm$^2$ × $\\dfrac{1}{100}$ = {{{correctAnswer}}}"}]},{"vars":[{"varval":"Jeremy's back deck is square in shape and has an area of 16 square metres.\n\r\nWhat is the area of the deck in square centimetres?"},{"varval":"Method 1\n\n\r\nDimensions of square with area 16 m$^2$ is:\n\n\r\n4 m × 4 m\n\n\r\nConverting to centimetres:\n\n\r\n400 cm × 400 cm = {{{correctAnswer}}}\n\n<br>\n\n\r\n\r\n\r\nMethod 2\n\n\r\nScale factor of converting m to cm = 100\n\n\r\nScale factor of converting m$^2$ to cm$^2$ = 100 × 100 = 10 000\n\n\r\n$\\therefore$ 16 m$^2$ × 10 000 = {{{correctAnswer}}}"}]},{"vars":[{"varval":"Vlad has a giant square chess board that has an area of 4 m$^2$.\n\nWhat is the area of the chess board in square centimetres?"},{"varval":"Method 1\n\n\r\nDimensions of square with area 4 m$^2$ is:\n\n\r\n2 m × 2 m\n\n\r\nConverting to centiimetres:\n\n\r\n200 cm × 200 cm = {{{correctAnswer}}}\n\n<br>\n\n\r\n\r\n\r\nMethod 2\n\n\r\nScale factor of converting cm to m = 100\n\n\r\nScale factor of converting cm$^2$ to m$^2$ = 100 × 100 = 10 000\n\n\r\n$\\therefore$ 4 m$^2$ × 10 000 = {{{correctAnswer}}}"}]}]

  585. <div class="sm_mode"> {{{question}}} </div>

    [{"vars":[{"varval":"Ben bought a dog mat with an area of 0.5 square metres.\n\n\r\n\r\nWhat is the area of the dog mat in square centimetres?\n"},{"varval":"<div class=\"aligned\">\n\n| | |\n| --------------------- | -------------- |\n| 1 m$^2$ | \\= 100 cm × 100 cm |\n| | \\= 10 000 cm$^2$ |\n\n</div>\n\n<br>\n\n<div class=\"aligned\">\n\n| | |\n| --------------------- | -------------- |\n| $\\therefore$ 0.5 m$^2$ | \\= 0.5 × 10 000 |\n| | \\= {{{correctAnswer}}} |\n\n</div>"}]},{"vars":[{"varval":"Julia bought a kitchen rug with an area of 0.85 square metres.\n\n\r\n\r\nWhat is the area of the kitchen rug in square centimetres?\n"},{"varval":"<div class=\"aligned\">\n\n| | |\n| --------------------- | -------------- |\n| 1 m$^2$ | \\= 100 cm × 100 cm |\n| | \\= 10 000 cm$^2$ |\n\n</div>\n\n<br>\n\n<div class=\"aligned\">\n\n| | |\n| --------------------- | -------------- |\n| $\\therefore$ 0.85 m$^2$ | \\= 0.85 × 10 000 |\n| | \\= {{{correctAnswer}}} |\n\n</div>"}]},{"vars":[{"varval":"Nigel bought a wall hanging with an area of 0.4 square metres.\n\n\r\n\r\nWhat is the area of the wall hanging in square centimetres?\n"},{"varval":"<div class=\"aligned\">\n\n| | |\n| --------------------- | -------------- |\n| 1 m$^2$ | \\= 100 cm × 100 cm |\n| | \\= 10 000 cm$^2$ |\n\n</div>\n\n<br>\n\n<div class=\"aligned\">\n\n| | |\n| --------------------- | -------------- |\n| $\\therefore$ 0.4 m$^2$ | \\= 0.4 × 10 000 |\n| | \\= {{{correctAnswer}}} |\n\n</div>"}]},{"vars":[{"varval":"Louis bought a picnic rug with an area of 1.2 square metres.\n\n\r\n\r\nWhat is the area of the picnic rug in square centimetres?\n"},{"varval":"<div class=\"aligned\">\n\n| | |\n| --------------------- | -------------- |\n| 1 m$^2$ | \\= 100 cm × 100 cm |\n| | \\= 10 000 cm$^2$ |\n\n</div>\n\n<br>\n\n<div class=\"aligned\">\n\n| | |\n| --------------------- | -------------- |\n| $\\therefore$ 1.2 m$^2$ | \\= 1.2 × 10 000 |\n| | \\= {{{correctAnswer}}} |\n\n</div>"}]},{"vars":[{"varval":"Mandy bought a table cloth with an area of 1.8 square metres.\n\n\r\n\r\nWhat is the area of the table cloth in square centimetres?\n"},{"varval":"<div class=\"aligned\">\n\n| | |\n| --------------------- | -------------- |\n| 1 m$^2$ | \\= 100 cm × 100 cm |\n| | \\= 10 000 cm$^2$ |\n\n</div>\n\n<br>\n\n<div class=\"aligned\">\n\n| | |\n| --------------------- | -------------- |\n| $\\therefore$ 1.8 m$^2$ | \\= 1.8 × 10 000 |\n| | \\= {{{correctAnswer}}} |\n\n</div>"}]},{"vars":[{"varval":"Gough bought a canvas with an area of 0.2 square metres.\n\n\r\n\r\nWhat is the area of the canvas in square centimetres?\n"},{"varval":"<div class=\"aligned\">\n\n| | |\n| --------------------- | -------------- |\n| 1 m$^2$ | \\= 100 cm × 100 cm |\n| | \\= 10 000 cm$^2$ |\n\n</div>\n\n<br>\n\n<div class=\"aligned\">\n\n| | |\n| --------------------- | -------------- |\n| $\\therefore$ 0.2 m$^2$ | \\= 0.2 × 10 000 |\n| | \\= {{{correctAnswer}}} |\n\n</div>"}]}]

  586. <div class="sm_mode"> {{{question}}} </div>

    [{"vars":[{"varval":"Kate is freezing chicken stock liquid as a treat for her dog in summer.\n\n\r\n\r\nOne container requires 0.65 litres of liquid to fill it.\n\n\r\n\r\nHow many millilitres of liquid chicken stock would Kate need to fill up one container?"},{"varval":"1 litre = 1000 millilitres\n\n<div class=\"aligned\">\n\n| | |\n| --------------------- | -------------- |\n| Liquid required | \\= 0.65 x 1000 |\n| | \\= {{{correctAnswer0}}} {{{suffix0}}} |\n\n</div>"}]},{"vars":[{"varval":"Shelly is freezing red cordial for ice-block treats in summer.\n\nOne ice-block container requires 0.45 litres of red cordial to fill it.\n\nHow many millilitres of red cordial would it take to fill up 2 containers?"},{"varval":"1 litre = 1000 millilitres\n\n<div class=\"aligned\">\n\n| | |\n| --------------------- | -------------- |\n| Red cordial required | \\= 2 x 0.45 x 1000 |\n| | \\= 2 x 450 |\n| | \\= {{{correctAnswer0}}} {{{suffix0}}} |\n\n</div>"}]},{"vars":[{"varval":"In a science experiment, Joanna pours 0.25 litres of solution into a container.\n\nShe repeats this 3 times.\n\nHow many millilitres of solution would it take Joanna to complete this task?"},{"varval":"1 litre = 1000 millilitres\n\n<div class=\"aligned\">\n\n| | |\n| --------------------- | -------------- |\n| Solution required | \\= 3 x 0.25 x 1000 |\n| | \\= 3 x 250 |\n| | \\= {{{correctAnswer0}}} {{{suffix0}}} |\n\n</div>"}]},{"vars":[{"varval":"Fred has a full 1-litre container of milk.\n\nHe pours 0.35 litres of milk onto his cereal for breakfast.\n\nHow many millilitres of milk are left in the container?"},{"varval":"1 litre = 1000 millilitres\n\n<div class=\"aligned\">\n\n| | |\n| --------------------- | -------------- |\n| Milk left | \\= 1000 $-$ 350 |\n| | \\= {{{correctAnswer0}}} {{{suffix0}}} |\n\n</div>"}]},{"vars":[{"varval":"Renee and Leisa are pouring liquid into an empty glass for a science experiment.\n\nRenee pours 100 millilitres into the glass.\n\nLeisa then pours 0.45 of a litre into the same glass.\n\nHow many millilitres of liquid are now in the glass?"},{"varval":"1 litre = 1000 millilitres\n\n<div class=\"aligned\">\n\n| | |\n| --------------------- | -------------- |\n| Liquid in glass | \\= 100 + 0.45 x 1000 |\n| | \\= 100 + 450 |\n| | \\= {{{correctAnswer0}}} {{{suffix0}}} |\n\n</div>"}]},{"vars":[{"varval":"Tango and Cash are pouring liquid into an empty glass for a science experiment.\n\nTango pours 200 millilitres into the glass.\n\nCash then pours 0.45 of a litre into the same glass.\n\nHow many millilitres of liquid are now in the glass?"},{"varval":"1 litre = 1000 millilitres\n\n<div class=\"aligned\">\n\n| | |\n| --------------------- | -------------- |\n| Liquid in glass | \\= 200 + 0.45 x 1000 |\n| | \\= 200 + 450 |\n| | \\= {{{correctAnswer0}}} {{{suffix0}}} |\n\n</div>"}]}]

  587. Added categories.

    <div class="sm_mode"> {{{question}}} </div>

    [{"vars":[{"varval":"Abigail has a 0.65 metre length of rope.\n\n\r\n\r\nShe cuts a 350 millimetre piece off it.\n\n\r\n\r\nHow long is the remaining length of rope?\n"},{"varval":"<div class=\"aligned\">\n\n| | |\n| --------------------- | -------------- |\n| Length remaining | \\= (0.65 × 1000) − 350 |\n| | \\= 650 − 350 |\n|| \\= {{{correctAnswer0}}} {{{suffix0}}}|\n\n</div>"}]}]

  588. <div class="sm_mode"> Claire asked her classmates what their favourite food. The results were recorded in the pie chart below. <br> sm_img https://teacher.smartermaths.com.au/wp-content/uploads/2022/01/NAPX9-TLC-37-1.svg 400 indent vpad <br>Which food is the favourite of around a quarter of her classmates? </div>

    [{"vars":null}]

  589. <div class="sm_mode"> {{{question}}} </div>

    [{"vars":[{"varval":"Clinton designs a line of tiles that repeats 6 different tiles in the same order, as shown below.\n\n<br>\n\nsm_img https://teacher.smartermaths.com.au/wp-content/uploads/2018/05/NAPX-E4-CA19.svg 650 indent vpad\n\n<br>Each tile costs $7.50 and Clinton uses $240 worth of tiles.\r\n\r\nWhich of these is the last tile in his line of tiles?"},{"varval":"<div class=\"aligned\">\r\n\r\n| | |\r\n| ------------: | ---------- |\r\n| Total number of tiles used | = $\\dfrac{240}{7.50}$ |\r\n| |= 32 |\r\n\r\n</div>\r\n\n\n<br>\n\n∴ Five full patterns of 6 tiles + 2 (i.e. the last tile is the second in the pattern)\n\n{{{correctAnswer}}}"}]},{"vars":[{"varval":"Mr Earp designs a line of tiles that repeats 6 different tiles in the same order, as shown below.\n\n<br>\n\nsm_img https://teacher.smartermaths.com.au/wp-content/uploads/2018/05/NAPX-E4-CA19.svg 650 indent vpad\n\n<br>Each tile costs $13.70 and Mr Earp uses $548 worth of tiles.\r\n\r\nWhich of these is the last tile in his line of tiles?"},{"varval":"<div class=\"aligned\">\r\n\r\n| | |\r\n| ------------: | ---------- |\r\n| Total number of tiles used | = $\\dfrac{548}{13.70}$ |\r\n| |= 40 |\r\n\r\n</div>\r\n\n<br>\n\n∴ Six full patterns of 6 tiles + 4 (i.e. the last tile is the\r 4th in the pattern)\n\n{{{correctAnswer}}}"}]},{"vars":[{"varval":"Klim decorated his fence with a line of tiles that repeats 6 different tiles in the same order, as shown below.\n\n<br>\n\nsm_img https://teacher.smartermaths.com.au/wp-content/uploads/2018/05/NAPX-E4-CA19.svg 650 indent vpad\n\n<br>Each tile costs $9.50 and Klim uses $256.50 worth of tiles.\r\n\r\nWhich of these is the last tile in his line of tiles?"},{"varval":"<div class=\"aligned\">\r\n\r\n| | |\r\n| ------------: | ---------- |\r\n| Total number of tiles used | = $\\dfrac{256.50}{9.50}$ |\r\n| |= 27 |\r\n\r\n</div>\r\n\n<br>\n\n∴ Four full patterns of 6 tiles + 3 (i.e. the last tile is the 3rd in the pattern)\n\n{{{correctAnswer}}}"}]},{"vars":[{"varval":"Danny decorated his patio with a line of tiles that repeats 6 different tiles in the same order, as shown below.\n\n<br>\n\nsm_img https://teacher.smartermaths.com.au/wp-content/uploads/2023/08/Algebra-NAPX-E4-CA19-v3-min.svg 650 indent vpad\n\n<br>Each tile costs $11.50 and Danny uses $448.50 worth of tiles.\n\nWhich of these is the last tile in his line of tiles?"},{"varval":"<div class=\"aligned\">\n\n| | |\n| ------------: | ---------- |\n| Total number of tiles used | = $\\dfrac{448.50}{11.50}$ |\n| |= 39 |\n| Total number of repititions | = $\\dfrac{39}{6}$ |\n| |= 6 remainder 3 |\n\n</div>\n\n<br>\n\n∴ Six full patterns of 6 tiles + 3 \n\n∴ The last tile is the 3rd tile in the pattern\n\n{{{correctAnswer}}}"}]},{"vars":[{"varval":"Shayna decorated her ensuite with a line of tiles that repeats 6 different tiles in the same order, as shown below.\n\n<br>\n\nsm_img https://teacher.smartermaths.com.au/wp-content/uploads/2023/08/Algebra-NAPX-E4-CA19-v3-min.svg 650 indent vpad\n\n<br>Each tile costs $14.20 and Shayna uses $1263.80 worth of tiles.\n\nWhich of these is the last tile in her line of tiles?"},{"varval":"<div class=\"aligned\">\n\n| | |\n| ------------: | ---------- |\n| Total number of tiles used | = $\\dfrac{1263.80}{14.20}$ |\n| |= 89 |\n| Total number of repititions | = $\\dfrac{89}{6}$ |\n| |= 14 remainder 5 |\n\n</div>\n\n<br>\n\n∴ 14 full patterns of 6 tiles + 5\n \n∴ The last tile is the 5th tile in the pattern\n\n{{{correctAnswer}}}"}]},{"vars":[{"varval":"Lincoln decorated his outdoor barbecue area with a line of tiles that repeats 6 different tiles in the same order, as shown below.\n\n<br>\n\nsm_img https://teacher.smartermaths.com.au/wp-content/uploads/2023/08/Algebra-NAPX-E4-CA19-v3-min.svg 650 indent vpad\n\n<br>Each tile costs $18.35 and Lincoln uses $1137.70 worth of tiles.\n\nWhich of these is the last tile in his line of tiles?"},{"varval":"<div class=\"aligned\">\n\n| | |\n| ------------: | ---------- |\n| Total number of tiles used | = $\\dfrac{1137.70}{18.35}$ |\n| |= 62 |\n| Total number of repititions | = $\\dfrac{62}{6}$ |\n| |= 10 remainder 2 |\n\n</div>\n\n<br>\n\n∴ 10 full patterns of 6 tiles + 2 \n∴ The last tile is the 2nd tile in the pattern\n\n{{{correctAnswer}}}"}]}]

  590. <div class="sm_mode"> {{{question}}} </div>

    [{"vars":[{"varval":"A red blood cell and a white blood cell have a combined total mass of $4.2 × 10^{−11}$ grams.\n\n\r\n\r\nIf the red blood cell has a mass of $2 × 10^{−12}$ grams, what is the mass of the white blood cell?\n"},{"varval":"$4.2 × 10^{−11}\\ −\\ 2 × 10^{−12}$\n\n\r\n>> = {{{correctAnswer}}}"}]},{"vars":[{"varval":"A red blood cell and a white blood cell have a combined total mass of $5.3 × 10^{−9}$ grams.\n\n\r\n\r\nIf the red blood cell has a mass of $3 × 10^{−10}$ grams, what is the mass of the white blood cell?\n"},{"varval":"\n$5.3 × 10^{−9}\\ −\\ 3 × 10^{−10}$\n\n\r\n>> = {{{correctAnswer}}}"}]},{"vars":[{"varval":"Two mico-organisms have a combined total mass of $6.4 × 10^{−11}$ grams.\n\n\r\n\r\nIf one of the mico-organisms has a mass of $4 × 10^{−12}$ grams, what is the mass of the other micro-organism?"},{"varval":"$6.4 × 10^{−11}\\ −\\ 4 × 10^{−12}$\n\n\r\n>> = {{{correctAnswer}}}"}]},{"vars":[{"varval":"In a single grain of salt, sodium and chlorine have a combined total mass of $3.8 × 10^{−12}$ grams.\n\n\r\n\r\nIf the sodium has a mass of $8 × 10^{−13}$ grams, what is the mass of the chlorine?\n"},{"varval":"$3.8 × 10^{−12}\\ −\\ 8 × 10^{−13}$\n\n\r\n>> = {{{correctAnswer}}}"}]},{"vars":[{"varval":"The active ingredients in a tablet, iodine and fluoride, have a combined total mass of $4.9 × 10^{−10}$ grams.\n\n\r\n\r\nIf the iodine has a mass of $9 × 10^{−11}$ grams, what is the mass of the fluoride?"},{"varval":"$4.9 × 10^{−10}\\ −\\ 9 × 10^{−11}$\n\n\r\n>> = {{{correctAnswer}}}"}]}]

  591. <div class="sm_mode"> Kyle and Jackie are standing in a line, waiting to go into their classroom. There are: * 7 people between Kyle and Jackie. * 15 people behind Jackie including Kyle. * 12 people in front of Kyle including Jackie. In total, how many people are in the line? </div>

    [{"vars":null}]

  592. <div class="sm_mode"> Daisy is in high school and her age this year is both a prime number and a factor of 51. In two years' time, Daisy's age will again be a prime number. What is Daisy's age this year? </div>

    [{"vars":null}]

  593. <div class="sm_mode"> The bottles in Leisa's fridge are pictured below. <br> sm_img https://teacher.smartermaths.com.au/wp-content/uploads/2018/06/NAPX-J3-CA17_1-768x253.png 500 indent2 vpad <br>Leisa decides to make a graph where each bar represents one type of bottle in her fridge. <br> sm_img https://teacher.smartermaths.com.au/wp-content/uploads/2018/06/NAPX-J3-CA17.svg 500 indent2 vpad <br>Leisa makes an error when creating the graph. What should Leisa do to correct the error? </div>

    [{"vars":null}]

  594. <div class="sm_mode"> {{{question}}} </div>

    [{"vars":[{"varval":"\nThe heights, in centimetres, of Coriander's netball side are as follows:\n \n\n>> 143, 150, 147, 142, 147, 153, 148, 142, 147\n\n\n<br>\n \nSelect the dot plot that correctly displays the data.\n"},{"varval":"{{{correctAnswer}}}"}]},{"vars":[{"varval":"\nThe distances, in kilometres, of Klaus's training runs over a ten day period are as follows:\n \n\n>> 8, 5, 6, 8, 9, 10, 12, 6, 10, 6\n\n\n<br>\n \nSelect the dot plot that correctly displays the data.\n"},{"varval":"{{{correctAnswer}}}"}]},{"vars":[{"varval":"The test marks from a recent Statistics test out of 36 are as follows:\n \n\n>> 27, 25, 33, 29, 27, 30, 28, 29, 35, 33, 29, 28\n\n\n<br>\n \nSelect the dot plot that correctly displays the data.\n"},{"varval":"{{{correctAnswer}}}"}]},{"vars":[{"varval":"The heights, in centimetres, of David's hockey side are as follows:\n \n\n>> 172, 177, 171, 174, 174, 176, 173, 177, 174, 172, 174,\n\n\n<br>\n \nSelect the dot plot that correctly displays the data.\n"},{"varval":"{{{correctAnswer}}}"}]}]

  595. <div class="sm_mode"> {{{question}}} </div>

    [{"vars":[{"varval":"Jasper used this rule to make a number pattern.\r\n\r\n\n* Start with 3.\r\n\n\r\n* To get the next number, multiply by 3 then deduct 5.\n\n<br>\r\n\r\n\nThe first four numbers in his pattern were 3, 4, 7, 16.\r\n\r\n\nWhat was the sixth number in his pattern?"},{"varval":"Fifth number\n\n>16 × 3 − 5 = 48 − 5 = 43\n\n<br>\n\n\nSixth number\r\n\n>43 × 3 − 5 = 129 − 5 = {{{correctAnswer}}}"}]},{"vars":[{"varval":"Felicity used this rule to make a number pattern.\r\n\r\n\n* Start with 7.\r\n\n\r\n* To get the next number, multiply by 2 then deduct 4.\n\n<br>\r\n\r\n\nThe first four numbers in Felicity's pattern were 7, 10, 16, 28.\r\n\r\n\nWhat was the sixth number in her pattern?"},{"varval":"Fifth number:\n\n>28 × 2 − 4 = 56 − 4 = 52\n\n<br>Sixth number\r:\n\n>52 × 2 − 4 = 104 − 4 = {{{correctAnswer}}}"}]}]

  596. <div class="sm_mode"> Raven makes the following pattern of figures using sticks. sm_img https://teacher.smartermaths.com.au/wp-content/uploads/2018/06/NAPX-E4-NC05.svg 550 indent vpad How many sticks will Figure 6 require? </div>

    [{"vars":null}]

  597. Var2 has no border around tyhe graph ... can the border be removed off Var1,3,4? Done Var 4 ... questioin image - on Safari, the "Months" label of the x-axis has moved up and is written ove rthe months of "Jun" and "Aug" Not sure how that happened but it's fixed now.

    <div class="sm_mode"> {{{question}}} </div>

    [{"vars":[{"varval":"This graph shows the number of people in Australia, in millions, who wear watches on their wrist, from 1998 to 2018.\n\n<br>\n\nsm_img https://teacher.smartermaths.com.au/wp-content/uploads/2020/03/NAPX-L4-16-ver1.svg 500 indent vpad\r\n\n<br>Which of these statements about the graph is true?\n"},{"varval":"People wearing watches in 2013 ≈ 3.1 million \r\n\nPeople wearing watches in 2018 ≈ 2.3 million\r\n\n$\\therefore$ Decreased by more than 500 000 between 2013 and 2018.\n"}]},{"vars":[{"varval":"This graph shows the number of businesses in New South Wales for the Years 2016 to 2020.\n\n<br>\n\nsm_img https://teacher.smartermaths.com.au/wp-content/uploads/2022/09/Stats_Prob_NAPX-L4-CA16-v1_v1_a.svg 520 indent vpad\r\n\n<br>Which of these statements about the graph is true?\n"},{"varval":"Businesses in 2016 ≈ 729 000\r\n\nBusinesses in 2017 ≈ 752 000\n\nBusinesses in 2018 ≈ 781 000\n\nBusinesses in 2019 ≈ 802 000\n\nBusinesses in 2020 ≈ 820 000\r\n\n$\\therefore$ The biggest increase of approximately 29 000 businesses occurs from 2017 to 2018.\n"}]},{"vars":[{"varval":"This graph shows the number of building approvals in Victoria for the first 6 months of 2022.\n\n<br>\n\nsm_img https://teacher.smartermaths.com.au/wp-content/uploads/2022/09/Stats_Prob_NAPX-L4-CA16-v1_v2b.svg 600 indent vpad\r\n\n<br>Which of these statements about the graph is true?\n"},{"varval":"\nArrange the scores in order:\n\n>>3000, 4500, 5000, 5500, 5750, 6000\n\n<br>\n\n<div class=\"aligned\">\r\n\r\n| | |\r\n| ------------- | ---------- |\r\n| Median | \\= Middle score when arranged in order |\n| | \\= Average of the 3rd and 4th scores|\n| | \\= $\\dfrac{5000+5500}{2}$|\r\n| | \\= 5250 approvals|\r\n\r\n</div>\n<br>\n\n$\\therefore$ The median number of building approvals during the first 6 months of 2022 was 5250 approvals.\n\n\r\n"}]},{"vars":[{"varval":"This graph shows the estimated resident population of Tasmania from 2016 to 2021.\n\n<br>\n\nsm_img https://teacher.smartermaths.com.au/wp-content/uploads/2022/09/Stats_Prob_NAPX-L4-CA16-v1_v3a.svg 450 indent vpad\r\n\n<br>Which of these statements about the graph is true?"},{"varval":"Consider option 1:\n\nIncrease in population from 2020 to 2021 = 5 000 people \r\n\nIncrease in population from 2019 to 2020 = 10 000 people \r\n\n$\\therefore$ {{{correctAnswer}}}"}]},{"vars":[{"varval":"This graph shows a company's monthly profits for a 12 month period.\n\n\n\nsm_img https://teacher.smartermaths.com.au/wp-content/uploads/2022/09/Stats_Prob_NAPX-L4-CA16-v1_v4_b.svg 500 indent vpad\r\n\nWhich of these statements about the graph is true?"},{"varval":"<div class=\"aligned\">\r\n\r\n| | |\r\n| ------------- | ---------- |\r\n| Range | \\= high − low |\n| | \\= $320\\ 000 - 100\\ 000$ |\r\n| | \\= $220 000 |\r\n\r\n</div>\r\n<br>\n\n$\\therefore$ The range of monthly profits for the 12 month period is $220 000."}]}]

  598. Var5 graph ... the x-axis values of $1,000 and $1,100 overlap on Safari. I would make the Axis label "Sales Achieved ($)" and make 1,000 = 1000 etc... to make extra room ***Fixed

    <div class="sm_mode"> {{{question}}} </div>

    [{"vars":[{"varval":"The results of a general knowledge test are shown in the graph below.\n\n\nsm_img https://teacher.smartermaths.com.au/wp-content/uploads/2022/09/Stat_NAPX9-TLB-16v1_v0.svg 300 indent vpad\r\n\nHow many students scored 70 or less?\n"},{"varval":"sm_nogap Number of students who scored 70 or less\n\n<div class=\"aligned\">\r\n\r\n>| |\r\n ---------- |\r\n| \\= $8+4+10+8$ |\r\n| \\= {{{correctAnswer}}} |\r\n\r\n</div>\r\n"}]},{"vars":[{"varval":"The results of a spelling quiz are shown in the graph below.\n\n\nsm_img https://teacher.smartermaths.com.au/wp-content/uploads/2022/09/Stat_NAPX9-TLB-16v1_v1.svg 300 indent vpad\r\n\nHow many students scored 35 or more?\n"},{"varval":"sm_nogap Number of students who scored 35 or more\n\n<div class=\"aligned\">\r\n\r\n>| |\r\n ---------- |\r\n| \\= $12+14+10+6$ |\r\n| \\= {{{correctAnswer}}} |\r\n\r\n</div>\r\n"}]},{"vars":[{"varval":"The results of a mathematics test are shown in the graph below.\n\n\n\nsm_img https://teacher.smartermaths.com.au/wp-content/uploads/2022/09/Stat_NAPX9-TLB-16v1_v2.svg 300 indent vpad\r\n\nIf the test was out of 30, how many students scored less than 50%?\n"},{"varval":"50% $\\times$ 30 = 15\n\nsm_nogap Number of students who scored less than 15\n\n<div class=\"aligned\">\r\n\r\n>| |\r\n ---------- |\r\n| \\= $6+8+9$ |\r\n| \\= {{{correctAnswer}}} |\r\n\r\n</div>\r\n"}]},{"vars":[{"varval":"The results of a reading test are shown in the graph below.\n\nsm_img https://teacher.smartermaths.com.au/wp-content/uploads/2022/09/Stat_NAPX9-TLB-16v1_v3a.svg 300 indent vpad\r\n\nIf the test was out of 10, how many students scored 70% or more?\n"},{"varval":"70% $\\times$ 10 = 7\n\nsm_nogap Number of students who scored 7, 8, 9 or 10\n\n<div class=\"aligned\">\r\n\r\n>| |\r\n ---------- |\r\n| \\= $5+7+6+4$ |\r\n| \\= {{{correctAnswer}}} |\r\n\r\n</div>\r\n"}]},{"vars":[{"varval":"The position of riders in an 80 kilometre race was tracked after 2 hours and the results are show in the graph below.\n\n\nsm_img https://teacher.smartermaths.com.au/wp-content/uploads/2022/09/Stat_NAPX9-TLB-16v1_v4.svg 300 indent vpad\r\n\nFrom the graph, how many riders are still more than half way from home after 2 hours?\n"},{"varval":"$\\dfrac{1}{2} \\times\\ 80 = 40$\n\nsm_nogap Number of riders more than 40 kilometres from home\n\n<div class=\"aligned\">\r\n\r\n>| |\r\n ---------- |\r\n| \\= $7+4$ |\n| \\= {{{correctAnswer}}} |\r\n\r\n</div>\r\n"}]},{"vars":[{"varval":"The sales targets of employees at a company are shown in the graph below.\n\n\nsm_img https://teacher.smartermaths.com.au/wp-content/uploads/2022/09/Stat_NAPX9-TLB-16v1_v5a.svg 440 indent vpad\r\n\nHow many employees reached the sales target of at least $800 in sales?\n"},{"varval":"sm_nogap Number of employees who reached the target of at least $800\n\n<div class=\"aligned\">\r\n\r\n>| |\r\n ---------- |\r\n| \\= $16+21+22+26$ |\r\n| \\= {{{correctAnswer}}} |\r\n\r\n</div>\r\n"}]}]

  599. var1: Statistics, NAPX9-TLC-26 v3

    <div class="sm_mode"> {{{question}}} </div>

    [{"vars":[{"varval":"Oliver is doing an inventory of his fruit shop.\r\n\r\nThe cost and quantity of some of the inventory is recorded in the table below.\n\n<br>\n\n<div class=\"outline color1\">\n\n>> | Product | Cost/kg | Quantity (kgs) | Total Cost |\n> | :-----------: | :----------: | :------------: | :------------: |\n> | **Bananas** | $1.20 | 29 | $34.80 |\n> | **Apples** | $1.80 | 20 | $36.00 |\n> | **Cherries** | $1.40 | 40 | $56.00 |\n> | **Dates** | $2.10 | 27 | $56.70 |\n\n</div>\n\n<br>How did Oliver order the data in this table?"},{"varval":"The data on total cost is arranged from lowest to highest.\r\n\n$\\therefore$ Data is ordered by total cost."}]},{"vars":[{"varval":"Jason took a survey of four friends in his class and recorded the results in the table below.\n\n<br>\n\n<div class=\"outline color1\">\n\n>> | NAME | AGE | HEIGHT (cm) | WEIGHT (kg) | Math grade (%) |\n> | :-----------: | :----------: | :------------: | :------------: | :------------: |\n> | **Jack** | 16 | 160 | 50 | 90 |\n> | **Remy** | 17 | 172 | 53 | 85 |\n> | **Charlize** | 17 | 163 | 56 |92|\n> | **Leo** | 16 | 168 | 62 | 88 |\n\n</div>\n\n<br>How did Jason order his data?"},{"varval":"The data on weight is arranged from lightest to heaviest.\r\n\n$\\therefore$ Data is ordered by weight."}]},{"vars":[{"varval":"Tony took a survey of participants in his triathlon group and recorded the results in the table below.\r\n\r\n\n\n<br>\n\n<div class=\"outline color1\">\n\n>> | NAME | AGE | RUN (5 km) | SWIM (750 m) | CYCLE (20 km) |\n> | :-----------: | :----------: | :------------: | :------------: | :------------: |\n> | **Amy** | 24 | 18 min 20 sec | 13 min 10 sec | 19 min 40 sec |\n> | **John** | 30 | 20 min 10 sec | 17 min 4 sec | 19 min 55 sec |\n> | **Mike** | 28 | 23 min 45 sec| 16 min 12 sec |23 min 5 sec|\n> | **Maxine** | 18 | 26 min 40 sec | 15 min 34 sec | 22 min 15 sec |\n\n</div>\n\n<br>How did Tony order his data?"},{"varval":"The data on run (5 km) is arranged from lowest to highest.\r\n\n$\\therefore$ Data is ordered by run."}]},{"vars":[{"varval":"Holly is doing an inventory of her Christmas decoration shop.\r\n\r\nThe cost and quantity of some of the inventory is recorded in the table below.\n\n<br>\n\n<div class=\"outline color1\">\n\n>> | Product | Cost | Quantity | Total Cost |\n> | :-----------: | :----------: | :------------: | :------------: |\n> | **Baubles** | $4.80 each | 10 | $48.00 |\n> | **Tinsel** | $5.60 each | 20 | $112.00 |\n> | **Lights** | $6.80 each | 13 | $88.40 |\n> | **Trees** | $14.50 each| 15 | $217.50 |\n\n</div>\n\n<br>How did Holly order the data in this table?"},{"varval":"The data on cost is arranged from lowest to highest.\r\n\n$\\therefore$ Data is ordered by cost."}]},{"vars":[{"varval":"Marco is doing an inventory of his pizza sales over the weekend.\r\n\r\nThe cost and quantity of some of the inventory is recorded in the table below.\n\n<br>\n\n<div class=\"outline color1\">\n\n>> | Product | Cost/pizza | Quantity Sold | Total Cost |\n> | :-----------: | :----------: | :------------: | :------------: |\n> | **Margarita** | $5.40 | 23 | $124.20 |\n> | **Pepperoni** | $6.50 | 20 | $130.00 |\n> | **Supreme** | $7.00 | 15 | $105.00 |\n> | **Vegetarian** | $4.80 | 12 | $57.60 |\n\n</div>\n\n<br>How did Marco order the data in this table?"},{"varval":"The data on quantity sold is arranged from highest to lowest.\r\n\n$\\therefore$ Data is ordered by quantity sold."}]},{"vars":[{"varval":"April is doing an inventory of her coffee van sales on Monday.\r\n\r\nThe cost and quantity of some of the inventory is recorded in the table below.\n\n<br>\n\n<div class=\"outline color1\">\n\n>> | Product | Cost/cup | Quantity | Total Cost |\n> | :-----------: | :----------: | :------------: | :------------: |\n> | **Cappuccino** | $5.40 | 23 | $124.20 |\n> | **Latte** | $5.20 | 24 | $124.80 |\n> | **Flat White** | $5.00 | 27 | $135.00 |\n> | **Hot Chocolate** | $5.30 | 26 | $137.80 |\n\n</div>\n\n<br>How did April order the data in this table?"},{"varval":"The data on total cost is arranged from lowest to highest.\r\n\n$\\therefore$ Data is ordered by total cost."}]}]

  600. <div class="sm_mode"> {{{question}}} </div>

    [{"vars":[{"varval":"A miniature engine contains two types of ball bearings.\r\n\r\nThere are two times as many small ball bearings as large ball bearings.\r\n\r\nThe mass of a small ball bearing is 190 milligrams and the mass of a large ball bearing is 300 milligrams.\r\n\r\nThe total mass of all the ball bearings in the engine is 238 grams.\r\n\r\nHow many ball bearings in total are used in the engine?"},{"varval":"<div class=\"aligned\">\r\n\r\n| | |\r\n| ------------: | ---------- |\r\n| Let $\\ 2\\large x$ | \\= number of small bearings |\r\n| $\\large x$ | \\= number of large bearings |\r\n\r\n</div>\n\r\n\n<br>\n\n<div class=\"aligned\">\r\n\r\n| | |\r\n| ------------: | ---------- |\r\n| $2\\large x$(0.190) + $\\large x$(0.300) | \\= 238 |\n| $0.38\\large x$ + 0.3$\\large x$ | \\= 238 |\n| $0.68\\large x$ | \\= 238 |\n| $\\therefore \\ \\large x$ | \\= $\\dfrac{238}{0.68}$ |\n| | \\= 350 |\r\n\r\n\r\n</div>\r\n\n<br>\n\n<div class=\"aligned\">\r\n\r\n| | |\r\n| ------------: | ---------- |\r\n| $\\therefore$ Total ball bearings | \\= (2 $\\times$ 350) + 350 |\n| | \\= {{{correctAnswer0}}} |\n\n\r\n\r\n</div>"}]},{"vars":[{"varval":"A recipe contains two types of dry ingredients.\r\n\r\nThere is two times as much almond meal as plain flour.\r\n\r\nThe mass of a cup of almond meal is 84 grams and the mass of a cup of plain flour is 120 grams.\r\n\r\nThe total mass of all the dry ingredients in the recipe is 2.304 kilograms.\r\n\r\nHow many cups of dry ingredients in total are used in the recipe?"},{"varval":"<div class=\"aligned\">\r\n\r\n| | |\r\n| ------------: | ---------- |\r\n| Let $\\ 2\\large x$ | \\= amount of almond meal |\r\n| $\\large x$ | \\= amount of plain flour |\r\n\r\n</div>\n\r\n\n<br>\n\n<div class=\"aligned\">\r\n\r\n| | |\r\n| ------------: | ---------- |\r\n| $2\\large x$(0.084) + $\\large x$(0.120) | \\= 2.304 |\n| $0.168\\large x$ + 0.120$\\large x$ | \\= 2.304 |\n| $0.288\\large x$ | \\= 2.304 |\n| $\\therefore \\ \\large x$ | \\= $\\dfrac{2.304}{0.288}$ |\n| | \\= 8 |\r\n\r\n\r\n</div>\r\n\n<br>\n\n<div class=\"aligned\">\r\n\r\n| | |\r\n| ------------: | ---------- |\r\n| $\\therefore$ Total cups of dry ingredients | \\= (2 $\\times$ 8) + 8 |\n| | \\= {{{correctAnswer0}}} |\n\n\r\n\r\n</div>"}]},{"vars":[{"varval":"A BMX race track is made up of straight sections and hill climb sections.\r\n\nThere are three times as many straight sections as hill climb sections.\r\n\r\nThe length of each straight section is 2380 metres and the length of each hill climb section is 860 metres.\r\n\r\nThe total length of all sections is 24 kilometres.\r\n\r\nHow many racing sections in total make up the BMX track?"},{"varval":"<div class=\"aligned\">\r\n\r\n| | |\r\n| ------------: | ---------- |\r\n| Let $\\ 3\\large x$ | \\= number of straight sections |\r\n| $\\large x$ | \\= number of hill climb sections |\r\n\r\n</div>\n\r\n\n<br>\n\n<div class=\"aligned\">\r\n\r\n| | |\r\n| ------------: | ---------- |\r\n| $3\\large x$(2.38) + $\\large x$(0.86) | \\= 24 |\n| $7.14\\large x$ + 0.86$\\large x$ | \\= 24 |\n| $8\\large x$ | \\= 24 |\n| $\\therefore \\ \\large x$ | \\= $\\dfrac{24}{8}$ |\n| | \\= 3 |\r\n\r\n\r\n</div>\r\n\n<br>\n\n<div class=\"aligned\">\r\n\r\n| | |\r\n| ------------: | ---------- |\r\n| $\\therefore$ Total racing sections | \\= (3 $\\times$ 3) + 3 |\n| | \\= {{{correctAnswer0}}} |\n\n\r\n\r\n</div>"}]},{"vars":[{"varval":"A herb garden contains two types of herbs.\r\n\r\nThere are four times as many parsley plants as oregano plants.\r\n\r\nThe parsley plant requires 120 millilitres of liquid fertiliser per week and an oregano plant requires 150 millilitres of liquid fertiliser per week.\r\n\r\nThe total number of litres of liquid fertiliser used in the herb garden each week is 50.4 litres.\r\n\r\nHow many herb plants in total are in the garden?"},{"varval":"<div class=\"aligned\">\r\n\r\n| | |\r\n| ------------: | ---------- |\r\n| Let $\\ 4\\large x$ | \\= number of parsley plants|\r\n| $\\large x$ | \\= number of oregano plants |\r\n\r\n</div>\n\r\n\n<br>\n\n<div class=\"aligned\">\r\n\r\n| | |\r\n| ------------: | ---------- |\r\n| $4\\large x$(0.120) + $\\large x$(0.150) | \\= 50.4 |\n| $0.48\\large x$ + 0.15$\\large x$ | \\= 50.4 |\n| $0.63\\large x$ | \\= 50.4 |\n| $\\therefore \\ \\large x$ | \\= $\\dfrac{50.4}{0.63}$ |\n| | \\= 80 |\r\n\r\n\r\n</div>\r\n\n<br>\n\n<div class=\"aligned\">\r\n\r\n| | |\r\n| ------------: | ---------- |\r\n| $\\therefore$ Total herb plants | \\= (4 $\\times$ 80) + 80 |\n| | \\= {{{correctAnswer0}}} |\n\n\r\n\r\n</div>"}]},{"vars":[{"varval":"A diamond necklace contains two types of diamonds.\r\n\r\nThere are three times as many square cut diamonds as round cut diamonds.\r\n\r\nThe size of each square cut diamond is 0.9 carats and the size of each round cut diamond is 0.75 carats.\r\n\r\nThe total mass, in carats, of all the diamonds in the necklace is 51.75.\r\n\r\nHow many diamonds in total are used in the necklace?"},{"varval":"<div class=\"aligned\">\r\n\r\n| | |\r\n| ------------: | ---------- |\r\n| Let $\\ 3\\large x$ | \\= number of square cut diamonds |\r\n| $\\large x$ | \\= number of round cut diamonds |\r\n\r\n</div>\n\r\n\n<br>\n\n<div class=\"aligned\">\r\n\r\n| | |\r\n| ------------: | ---------- |\r\n| $3\\large x$(0.90) + $\\large x$(0.75) | \\= 51.75 |\n| $2.7\\large x$ + 0.75$\\large x$ | \\= 51.75 |\n| $3.45\\large x$ | \\= 51.75 |\n| $\\therefore \\ \\large x$ | \\= $\\dfrac{51.75}{3.45}$ |\n| | \\= 15 |\r\n\r\n\r\n</div>\r\n\n<br>\n\n<div class=\"aligned\">\r\n\r\n| | |\r\n| ------------: | ---------- |\r\n| $\\therefore$ Total diamonds | \\= (3 $\\times$ 15) + 15 |\n| | \\= {{{correctAnswer0}}} |\n\n\r\n\r\n</div>"}]},{"vars":[{"varval":"A coffee shop sells two types of muffins.\r\n\r\nThere are two times as many blueberry muffins as choc chip muffins.\r\n\r\nThe energy, in kilojoules, of a blueberry muffin is 2140 kilojoules and the energy of a choc chip muffin is 2470 kilojoules.\r\n\r\nThe total energy of all the muffins in the coffee shop is $101\\ 250$ kilojoules.\r\n\r\nHow many muffins in total are for sale in the coffee shop?"},{"varval":"<div class=\"aligned\">\r\n\r\n| | |\r\n| ------------: | ---------- |\r\n| Let $\\ 2\\large x$ | \\= number of blueberry muffins |\r\n| $\\large x$ | \\= number of choc chip muffins |\r\n\r\n</div>\n\r\n\n<br>\n\n<div class=\"aligned\">\r\n\r\n| | |\r\n| ------------: | ---------- |\r\n| $2\\large x$(2140) + $\\large x$(2470) | \\= 101 250 |\n| $4280\\large x$ + 2470$\\large x$ | \\= 101 250 |\n| $6750\\large x$ | \\= $101\\ 250$ |\n| $\\therefore \\ \\large x$ | \\= $\\dfrac{101\\ 250}{6750}$ |\n| | \\= 15 |\r\n\r\n\r\n</div>\r\n\n<br>\n\n<div class=\"aligned\">\r\n\r\n| | |\r\n| ------------: | ---------- |\r\n| $\\therefore$ Total muffins | \\= (2 $\\times$ 15) + 15 |\n| | \\= {{{correctAnswer0}}} |\n\n\r\n\r\n</div>"}]}]

  601. <div class="sm_mode"> {{{question}}} </div>

    [{"vars":[{"varval":"May created liquid sanitiser by mixing the four liquids: water, alcohol, solution A and solution B.\r\n\r\nShe makes 650 litres of the sanitiser according to the following instructions:\r\n\r\n\n* 60% of the mixture is water.\r\n\n* Solution A is used in the same volume as Solution B.\r\n\n* The volume of alcohol is three times the volume of Solution B.\r\n\n\nHow many litres of Solution A are required to make 650 litres of the sanitiser?"},{"varval":"<div class=\"aligned\">\r\n\r\n| | |\r\n| ------------: | ---------- |\r\n| Volume of water | \\= $60\\% \\times 650$|\r\n| | \\= 390 litres |\r\n\r\n</div>\r\n\n<br>\n\nsm_nogap Volume of remaining liquids\n\n<div class=\"aligned\">\r\n\r\n>>| |\r\n| ---------- |\r\n| \\= 650 $-$ 390 |\r\n| \\= 260 litres |\r\n\r\n</div>\r\n\n<br>\n\nsm_nogap Let $\\ \\large x$ = volume of Solution A\n\n<div class=\"aligned\">\r\n\r\n| | |\r\n| ------------: | ---------- |\r\n| $\\large x$ + $\\large x$ + 3$\\large x$ | \\= 260 |\n| $5\\large x$ | \\= 260 |\n| $\\therefore \\ \\large x$ | \\= {{{correctAnswer0}}} {{{suffix0}}} |\r\n\r\n</div>"}]},{"vars":[{"varval":"Justine created garden soil by mixing the four ingredients: compost, peat moss, cow manure and vermiculite.\r\n\r\nShe makes 450 kilograms of the garden soil according to the following instructions:\r\n\r\n\n* 30% of the mixture is compost.\r\n\n* peat moss is used in the same amount as vermiculite.\r\n\n* The amount of cow manure is one-third of the amount of the peat moss.\r\n\n\nHow many kilograms of peat moss are required to make 450 kilograms of the garden soil?"},{"varval":"<div class=\"aligned\">\r\n\r\n| | |\r\n| ------------: | ---------- |\r\n| Amount of compost | \\= $30\\% \\times 450$|\r\n| | \\= 135 kilograms |\r\n\r\n</div>\r\n\n<br>\n\nsm_nogap Amount of remaining ingredients\n\n<div class=\"aligned\">\r\n\r\n>>| |\r\n| ---------- |\r\n| \\= 450 $-$ 135 |\r\n| \\= 315 kilograms |\r\n\r\n</div>\r\n\n<br>\n\nsm_nogap Let $\\ \\large x$ = amount of peat moss\n\n<div class=\"aligned\">\r\n\r\n| | |\r\n| ------------: | ---------- |\r\n| $\\large x$ + $\\large x$ + $\\dfrac{1}{3}\\large x$ | \\= 315 |\n| $\\dfrac{7}{3}\\large x$ | \\= 315 |\n| $\\large x$ | = 315 $\\times\\ \\dfrac{3}{7}$ |\n| $\\therefore \\ \\large x$ | \\= {{{correctAnswer0}}} {{{suffix0}}} |\r\n\r\n</div>"}]},{"vars":[{"varval":"Mel made a jug of green smoothies for her friends by mixing the four ingredients: kale, avocado, ice and almond milk.\r\n\r\nShe makes 4 litres of the green smoothie according to the following instructions:\r\n\r\n\n* 50% of the mixture is avocado.\r\n\n* Almond milk is used in the same volume as ice.\r\n\n* The volume of kale is two times the volume of ice.\r\n\n\nHow many millilitres of almond milk are required to make 4 litres of the green smoothie?"},{"varval":"<div class=\"aligned\">\r\n\r\n| | |\r\n| ------------: | ---------- |\r\n| Volume of avocado | \\= $50\\% \\times 4$|\r\n| | \\= 2 litres |\r\n\r\n</div>\r\n\n<br>\n\nsm_nogap Volume of remaining ingredients\n\n<div class=\"aligned\">\r\n\r\n>>| |\r\n| ---------- |\r\n| \\= 4 $-$ 2 |\r\n| \\= 2 litres |\n| \\= 2000 millilitres |\r\n\r\n</div>\r\n\n<br>\n\nsm_nogap Let $\\ \\large x$ = volume of almond milk\n\n<div class=\"aligned\">\r\n\r\n| | |\r\n| ------------: | ---------- |\r\n| $\\large x$ + $\\large x$ + 2$\\large x$ | \\= 2000 |\n| $4\\large x$ | \\= 2000 |\n| $\\therefore \\ \\large x$ | \\= {{{correctAnswer0}}} {{{suffix0}}} |\r\n\r\n</div>"}]},{"vars":[{"varval":"Jerry was preparing to lay a small cement slab in his back yard.\n\nHe mixed the four ingredients: cement, rock, sand and water.\n\n\r\n\r\nJerry makes 50 kilograms of the cement mix according to the following instructions:\r\n\r\n\n* 40% of the mixture is rock.\r\n\n* the same mass of sand and water is used.\r\n\n* The mass of cement is half the mass of sand.\r\n\n\nHow many kilograms of sand are required to make 50 kilograms of the cement mix?"},{"varval":"<div class=\"aligned\">\r\n\r\n| | |\r\n| ------------: | ---------- |\r\n| Mass of rock | \\= $40\\% \\times 50$|\r\n| | \\= 20 kilograms |\r\n\r\n</div>\r\n\n<br>\n\nsm_nogap Mass of remaining ingredients\n\n<div class=\"aligned\">\r\n\r\n>>| |\r\n| ---------- |\r\n| \\= 50 $-$ 20 |\r\n| \\= 30 kilograms |\r\n\r\n</div>\r\n\n<br>\n\nsm_nogap Let $\\ \\large x$ = mass of sand\n\n<div class=\"aligned\">\r\n\r\n| | |\r\n| ------------: | ---------- |\r\n| $\\large x$ + $\\large x$ + $\\dfrac{1}{2}\\large x$ | \\= 30 |\n| $\\dfrac{5}{2}\\large x$ | \\= 30 |\n| $\\large x$ | = 30 $\\times\\ \\dfrac{2}{5}$ |\n| $\\therefore \\ \\large x$ | \\= {{{correctAnswer0}}} {{{suffix0}}} |\r\n\r\n</div>"}]},{"vars":[{"varval":"Julie is creating an exercise program.\n\nShe includes the four exercises: burpees, squats, push ups and box jumps.\n\n\r\n\r\nThe program requires 420 individual repetitions of the exercises given the following instructions:\r\n\r\n\n* 20% of the repetitions are push ups.\r\n\n* squat repetitions are double burpee repetitions.\r\n\n* box jump repetitions are half burpee repetitions.\n\nHow many repetitions of burpees are included in Julie's 420 repetition exercise program?"},{"varval":"<div class=\"aligned\">\r\n\r\n| | |\r\n| ------------: | ---------- |\r\n| Push up repetitions | \\= $20\\% \\times 420$|\r\n| | \\= 84 repetitions |\r\n\r\n</div>\r\n\n<br>\n\nsm_nogap Repetitions of remaining exercises\n\n<div class=\"aligned\">\r\n\r\n>>| |\r\n| ---------- |\r\n| \\= 420 $-$ 84 |\r\n| \\= 336 repetitions |\r\n\r\n</div>\r\n\n<br>\n\nsm_nogap Let $\\ \\large x$ = burpee repetitions\n\n<div class=\"aligned\">\r\n\r\n| | |\r\n| ------------: | ---------- |\r\n| $2\\large x$ + $\\large x$ + $\\dfrac{1}{2}\\large x$ | \\= 336 |\n| $\\dfrac{7}{2}\\large x$ | \\= 336 |\n| $\\large x$ | = 336 $\\times\\ \\dfrac{2}{7}$ |\n| $\\therefore \\ \\large x$ | \\= {{{correctAnswer0}}} {{{suffix0}}} |\r\n\r\n</div>"}]},{"vars":[{"varval":"Kira is organising her school's athletics carnival.\n\nIn the morning session she rotates the Year 9 participants through 4 activities: 100 metres sprint, discus, shot put and high jump.\n\n\r\n\r\nThe morning session is 2 hours long and time is allocated using the following constraints:\r\n\r\n\n* 40% of the total time allocation is taken up by high jump.\r\n\n* discus and shot put are allocated an equal amount time.\r\n\n* The 100 metres sprint is allocated one quarter the amount of time of shot put.\r\n\nWhat is the amount of time, in minutes, allocated to shot put during the 2 hour morning session?"},{"varval":"2 hours $\\Rightarrow$ 120 minures\n\n<div class=\"aligned\">\r\n\r\n| | |\r\n| ------------: | ---------- |\r\n| High jump time allocation | \\= $40\\% \\times 120$|\r\n| | \\= 48 minutes |\r\n\r\n</div>\r\n\n<br>\n\nsm_nogap Time allocation of remaining activities\n\n<div class=\"aligned\">\r\n\r\n>>| |\r\n| ---------- |\r\n| \\= 120 $-$ 48 |\r\n| \\= 72 minutes |\r\n\r\n</div>\r\n\n<br>\n\nsm_nogap Let $\\ \\large x$ = shot put time allocation\n\n<div class=\"aligned\">\r\n\r\n| | |\r\n| ------------: | ---------- |\r\n| $\\large x$ + $\\large x$ + $\\dfrac{1}{4}\\large x$ | \\= 72 |\n| $\\dfrac{9}{4}\\large x$ | \\= 72 |\n| $\\large x$ | = 72 $\\times\\ \\dfrac{4}{9}$ |\n| $\\therefore \\ \\large x$ | \\= {{{correctAnswer0}}} {{{suffix0}}} |\r\n\r\n</div>"}]}]

  602. <div class="sm_mode"> {{{question}}} </div>

    [{"vars":[{"varval":"\nBellarose collects insects.\r\n\r\n$\\dfrac{1}{4}$ of her collection are beetles and $\\dfrac{1}{6}$ of her collection are bees.\r\n\r\nBellarose has 6 more beetles than bees in her collection.\r\n\r\nHow many insects are in Bellarose's collection in total?\n"},{"varval":"Solution 1\n\nsm_nogap Let $\\ \\large x$ = total number of insects\n\n<div class=\"aligned\">\r\n\r\n| | |\r\n| ------------: | ---------- |\r\n| $\\dfrac{1}{4}\\large x$ $-$ $\\dfrac{1}{6}\\large x$ | \\= 6 |\n| $\\dfrac{3}{12}\\large x$ $-$ $\\dfrac{2}{12}\\large x$ | \\= 6 |\r\n| $\\dfrac{1}{12}\\large x$ | \\= 6 |\r\n| $\\large x$| \\= {{{correctAnswer}}} |\r\n\r\n</div>\r\n\n<br>\n\nSolution 2\r\n\nBy trial and error:\n\n$\\dfrac{1}{4} \\times 72=18$ beetles\r\n\n$\\dfrac{1}{6} \\times 72=12$ bees\n\nbeetles\r $-$ bees = 18 $-$ 12 = 6\n\n$\\therefore$ {{{correctAnswer}}} insects in total."}]},{"vars":[{"varval":"Edgell grows vegetables in a market garden.\r\n\r\n$\\dfrac{1}{5}$ of his crop are carrots and $\\dfrac{1}{4}$ of his crop are potatoes.\r\n\r\nEdgell has 95 more potato plants than carrots in his market garden.\r\n\r\nHow many plants are in Edgell's market garden in total?\n"},{"varval":"Solution 1\n\nsm_nogap Let $\\ \\large x$ = total number of plants\n\n<div class=\"aligned\">\r\n\r\n| | |\r\n| ------------: | ---------- |\r\n| $\\dfrac{1}{4}\\large x$ $-$ $\\dfrac{1}{5}\\large x$ | \\= 95 |\n| $\\dfrac{5}{20}\\large x$ $-$ $\\dfrac{4}{20}\\large x$ | \\= 95 |\r\n| $\\dfrac{1}{20}\\large x$ | \\= 95 |\r\n| $\\large x$| \\= {{{correctAnswer}}} |\r\n\r\n</div>\r\n\n<br>\n\nSolution 2\r\n\nBy trial and error:\n\n$\\dfrac{1}{4} \\times 1900=475$ potatoes\r\n\n$\\dfrac{1}{5} \\times 1900=380$ carrots\n\npotatoes $-$ carrots = 475 $-$ 380 = 95\n\n$\\therefore$ {{{correctAnswer}}} plants in total."}]},{"vars":[{"varval":"Sage collects sea shells on the beach.\r\n\r\n$\\dfrac{1}{2}$ of her collection are scallop shells and $\\dfrac{1}{3}$ of her collection are cowrie shells.\r\n\r\nSage has 16 more scallop shells than cowrie shells in her collection.\r\n\r\nHow many sea shells are in Sage's collection in total?\n"},{"varval":"Solution 1\n\nsm_nogap Let $\\ \\large x$ = total number of sea shells\n\n<div class=\"aligned\">\r\n\r\n| | |\r\n| ------------: | ---------- |\r\n| $\\dfrac{1}{2}\\large x$ $-$ $\\dfrac{1}{3}\\large x$ | \\= 16 |\n| $\\dfrac{3}{6}\\large x$ $-$ $\\dfrac{2}{6}\\large x$ | \\= 16 |\r\n| $\\dfrac{1}{6}\\large x$ | \\= 16 |\r\n| $\\large x$| \\= {{{correctAnswer}}} |\r\n\r\n</div>\r\n\n<br>\n\nSolution 2\r\n\nBy trial and error:\n\n$\\dfrac{1}{2} \\times 96=48$ scallop shells\n\n$\\dfrac{1}{3} \\times 96=32$ cowrie shells\n\nscallop shells $-$ cowrie shells = 48 $-$ 32 = 16\n\n$\\therefore$ {{{correctAnswer}}} sea shells in total."}]},{"vars":[{"varval":"Sydney writes backing music for advertisements.\r\n\r\n$\\dfrac{3}{5}$ of the money he has earned came from music he wrote in 2019 and $\\dfrac{1}{2}$ of the money he has earned came from music he wrote in 2020.\r\n\r\nSydney earned $24 000 more in 2019 than in 2020.\r\n\r\nHow much money has Sydney earned in total?\n"},{"varval":"Solution 1\n\nsm_nogap Let $\\ \\large x$ = total amount earned\n\n<div class=\"aligned\">\r\n\r\n| | |\r\n| ------------: | ---------- |\r\n| $\\dfrac{3}{5}\\large x$ $-$ $\\dfrac{1}{2}\\large x$ | \\= $24 000 |\n| $\\dfrac{6}{10}\\large x$ $-$ $\\dfrac{5}{10}\\large x$ | \\= $24 000 |\r\n| $\\dfrac{1}{10}\\large x$ | \\= $24 000 |\r\n| $\\large x$| \\= {{{correctAnswer}}} |\r\n\r\n</div>\r\n\n<br>\n\nSolution 2\r\n\nBy trial and error:\n\n$\\dfrac{3}{5} \\times 240\\ 000=144 \\ 000$\n\n$\\dfrac{1}{2} \\times 240\\ 000 = 120 \\ 000$\n\n2019 earnings $-$ 2020 earnings = 144 000 − 120 000 = $24 000\n\n$\\therefore$ {{{correctAnswer}}} earnings in total."}]},{"vars":[{"varval":"A bag contains coloured discs.\r\n\r\n$\\dfrac{2}{3}$ of the discs are purple and $\\dfrac{1}{6}$ of the discs are orange.\r\n\r\nThe bag has 18 more purple discs than orange discs.\r\n\r\nHow many discs are in the bag in total?\n"},{"varval":"Solution 1\n\nsm_nogap Let $\\ \\large x$ = total number of discs\n\n<div class=\"aligned\">\r\n\r\n| | |\r\n| ------------: | ---------- |\r\n| $\\dfrac{2}{3}\\large x$ $-$ $\\dfrac{1}{6}\\large x$ | \\= 18 |\n| $\\dfrac{4}{6}\\large x$ $-$ $\\dfrac{1}{6}\\large x$ | \\= 18 |\r\n| $\\dfrac{1}{2}\\large x$ | \\= 18 |\r\n| $\\large x$| \\= {{{correctAnswer}}} |\r\n\r\n</div>\r\n\n<br>\n\nSolution 2\r\n\nBy trial and error:\n\n$\\dfrac{2}{3} \\times 36=24$ purple discs\r\n\n$\\dfrac{1}{6} \\times 36=6$ orange discs\n\npurple\r $-$ orange = 24 $-$ 6 = 18\n\n$\\therefore$ {{{correctAnswer}}} discs in total."}]},{"vars":[{"varval":"iWatch streams a selection of movies, series and documentaries.\r\n\r\n$\\dfrac{3}{5}$ of the shows are movies and $\\dfrac{3}{20}$ of the shows are documentaries.\r\n\r\niWatch is currently streaming 36 more movies than documentaries.\r\n\r\nHow many shows are being streamed in total?"},{"varval":"Solution 1\n\nsm_nogap Let $\\ \\large x$ = total number of shows being streamed\n\n<div class=\"aligned\">\r\n\r\n| | |\r\n| ------------: | ---------- |\r\n| $\\dfrac{3}{5}\\large x$ $-$ $\\dfrac{3}{20}\\large x$ | \\= 36 |\n| $\\dfrac{12}{20}\\large x$ $-$ $\\dfrac{3}{20}\\large x$ | \\= 36 |\n| $\\dfrac{9}{20}\\large x$ | \\= 36 |\n| $\\large x$ | \\= $\\dfrac{20}{9} \\times 36$ |\r\n| $\\large x$| \\= {{{correctAnswer}}} |\r\n\r\n</div>\r\n\n<br>\n\nSolution 2\r\n\nBy trial and error:\n\n$\\dfrac{3}{5} \\times 80=48$ movies\r\n\n$\\dfrac{3}{20} \\times 80=12$ documentaries\n\nmovies\r $-$ documentaries = 48 $-$ 12 = 36\n\n$\\therefore$ {{{correctAnswer}}} shows in total."}]}]

  603. <div class="sm_mode"> {{{question}}} </div>

    [{"vars":[{"varval":"sm_img https://teacher.smartermaths.com.au/wp-content/uploads/2018/04/NAPX-H4-CA292.svg 300 indent3 vpad\r\n\nWhich of these equations represents the line in the graph?"},{"varval":"\n\n<div class=\"aligned\">\r\n\r\n| | |\r\n| ------------: | ---------- |\r\n| $\\large y$ intercept | \\= 5 |\n| Gradient | \\= $\\dfrac{\\text{rise}}{\\text{run}}$ |\n| | \\= $\\dfrac{-5}{1.25}$ |\n| | \\= $-4$ |\r\n\r\n</div>\r\n\n<br>\n\n$\\therefore$ Equation is &nbsp;{{{correctAnswer}}}\n\n"}]}]

  604. <div class="sm_mode"> {{{question}}} </div>

    [{"vars":[{"varval":"In this inequality $\\large n$ is a whole number.\r\n\r\n\n>$\\dfrac{5}{\\large n} < \\dfrac{3}{5}$\r\n\n<br>What is the smallest possible value for $\\large n$ to make this inequality true?"},{"varval":"<div class=\"aligned\">\r\n\r\n| | |\r\n| -------------: | ---------- |\r\n| $\\dfrac{5}{\\large n}$ | < $\\dfrac{3}{5}$ |\n| $3\\large n$ | > 25 |\n| $\\large n$ | > $\\dfrac{25}{3}$ |\r\n| $\\large n$ | > 8.33... |\r\n\r\n</div>\r\n\n<br>\n\n$\\therefore$ Smallest $\\ \\large n$ = {{{correctAnswer0}}}"}]},{"vars":[{"varval":"In this inequality $\\large x$ is a whole number.\r\n\r\n\n>$\\dfrac{7}{\\large x} < \\dfrac{4}{3}$\r\n\n<br>What is the smallest possible value for $\\large x$ to make this inequality true?"},{"varval":"<div class=\"aligned\">\r\n\r\n| | |\r\n| -------------: | ---------- |\r\n| $\\dfrac{7}{\\large x}$ | < $\\dfrac{4}{3}$ |\n| $4\\large x$ | > 21 |\n| $\\large x$ | > $\\dfrac{21}{4}$ |\r\n| $\\large x$ | > 5.25 |\r\n\r\n</div>\r\n\n<br>\n\n$\\therefore$ Smallest $\\ \\large x$ = {{{correctAnswer0}}}"}]},{"vars":[{"varval":"In this inequality $\\large n$ is a whole number.\r\n\r\n\n>$\\dfrac{9}{\\large n} < \\dfrac{7}{4}$\r\n\n<br>What is the smallest possible value for $\\large n$ to make this inequality true?"},{"varval":"<div class=\"aligned\">\r\n\r\n| | |\r\n| -------------: | ---------- |\r\n| $\\dfrac{9}{\\large n}$ | < $\\dfrac{7}{4}$ |\n| $7\\large n$ | > 36 |\n| $\\large n$ | > $\\dfrac{36}{7}$ |\r\n| $\\large n$ | > 5.142... |\r\n\r\n</div>\r\n\n<br>\n\n$\\therefore$ Smallest $\\ \\large n$ = {{{correctAnswer0}}}"}]},{"vars":[{"varval":"In this inequality $\\large m$ is a whole number.\r\n\r\n\n>$\\dfrac{11}{\\large m} < \\dfrac{3}{4}$\r\n\n<br>What is the smallest possible value for $\\large m$ to make this inequality true?"},{"varval":"<div class=\"aligned\">\r\n\r\n| | |\r\n| -------------: | ---------- |\r\n| $\\dfrac{11}{\\large m}$ | < $\\dfrac{3}{4}$ |\n| $3\\large m$ | > 44 |\n| $\\large m$ | > $\\dfrac{44}{3}$ |\r\n| $\\large m$ | > 14.66... |\r\n\r\n</div>\r\n\n<br>\n\n$\\therefore$ Smallest $\\ \\large m$ = {{{correctAnswer0}}}"}]},{"vars":[{"varval":"In this inequality $\\large q$ is a whole number.\r\n\r\n\n>$\\dfrac{7}{5} > \\dfrac{9}{\\large q}$\r\n\n<br>What is the smallest possible value for $\\large q$ to make this inequality true?"},{"varval":"<div class=\"aligned\">\r\n\r\n| | |\r\n| -------------: | ---------- |\r\n| $\\dfrac{7}{5}$ | > $\\dfrac{9}{\\large q}$ |\n| $7\\large q$ | > 45 |\n| $\\large q$ | > $\\dfrac{45}{7}$ |\r\n| $\\large q$ | > 6.428... |\r\n\r\n</div>\r\n\n<br>\n\n$\\therefore$ Smallest $\\ \\large q$ = {{{correctAnswer0}}}"}]},{"vars":[{"varval":"In this inequality $\\large d$ is a whole number.\r\n\r\n\n>$\\dfrac{11}{23} > \\dfrac{5}{\\large d}$\r\n\n<br>What is the smallest possible value for $\\large d$ to make this inequality true?"},{"varval":"<div class=\"aligned\">\r\n\r\n| | |\r\n| -------------: | ---------- |\r\n| $\\dfrac{11}{23}$ | > $\\dfrac{5}{\\large d}$ |\n| $11\\large d$ | > 115 |\n| $\\large d$ | > $\\dfrac{115}{11}$ |\r\n| $\\large d$ | > 10.45... |\r\n\r\n</div>\r\n\n<br>\n\n$\\therefore$ Smallest $\\ \\large d$ = {{{correctAnswer0}}}"}]}]

  605. <div class="sm_mode"> {{{question}}} </div>

    [{"vars":[{"varval":"$8 \\large x > \\large x^2$\n\nWhat is the largest whole number $\\large x$ can be that makes this expression correct?"},{"varval":"sm_nogap Equating the expression\n\r\n<div class=\"aligned\">\r\n\r\n>| | |\r\n| ------------: | ---------- |\r\n| 8$\\large x$ | \\= $\\large x$$^2$ |\r\n| $\\large x$ | \\= 8 |\r\n\r\n</div>\r\n\n<br>\n\n$\\therefore$ Largest $\\ \\large x$ = {{{correctAnswer0}}}"}]},{"vars":[{"varval":"$3 \\large x > \\large x^2$\n\nWhat is the largest whole number $\\large x$ can be that makes this expression correct?"},{"varval":"sm_nogap Equating the expression\n\r\n<div class=\"aligned\">\r\n\r\n>| | |\r\n| ------------: | ---------- |\r\n| 3$\\large x$ | \\= $\\large x$$^2$|\r\n| $\\large x$ | \\= 3 |\r\n\r\n</div>\r\n\n<br>\n\n$\\therefore$ Largest $\\ \\large x$ = {{{correctAnswer0}}}"}]},{"vars":[{"varval":"$7 \\large x > \\large x^2$\n\nWhat is the largest whole number $\\large x$ can be that makes this expression correct?"},{"varval":"sm_nogap Equating the expression\n\r\n<div class=\"aligned\">\r\n\r\n>| | |\r\n| ------------: | ---------- |\r\n| 7$\\large x$ | \\= $\\large x$$^2$|\r\n| $\\large x$ | \\= 7 |\r\n\r\n</div>\r\n\n<br>\n\n$\\therefore$ Largest $\\ \\large x$ = {{{correctAnswer0}}}"}]},{"vars":[{"varval":"$4 \\large x$ > $2\\large x^2$\n\nWhat is the largest whole number $\\large x$ can be that makes this expression correct?"},{"varval":"sm_nogap Equating the expression\n\r\n<div class=\"aligned\">\r\n\r\n>| | |\r\n| ------------: | ---------- |\r\n| 4$\\large x$ | \\= $2\\large x$$^2$|\n| $2\\large x$ | \\= 4 |\r\n| $\\large x$ | \\= 2 |\r\n\r\n</div>\r\n\n<br>\n\n$\\therefore$ Largest $\\ \\large x$ = {{{correctAnswer0}}}"}]},{"vars":[{"varval":"$15 \\large x$ > $5\\large x^2$\n\nWhat is the largest whole number $\\large x$ can be that makes this expression correct?"},{"varval":"Equating the expression\n\n<div class=\"aligned\">\n\n>| | |\n| ------------: | ---------- |\n| 15$\\large x$ | \\= $5\\large x$$^2$|\n| $5\\large x$ | \\= 15 |\n| $\\large x$ | \\= 3 |\n\n</div>\n\n<br>\n\n$\\therefore$ Largest $\\ \\large x$ = {{{correctAnswer0}}}"}]},{"vars":[{"varval":"$35 \\large x$ > $7\\large x^2$\n\nWhat is the largest whole number $\\large x$ can be that makes this expression correct?"},{"varval":"sm_nogap Equating the expression\n\r\n<div class=\"aligned\">\r\n\r\n>| | |\r\n| ------------: | ---------- |\r\n| 35$\\large x$ | \\= $7\\large x$$^2$|\r\n| $7\\large x$ | \\= 35 |\n| $\\large x$ | \\= 5 |\r\n\r\n</div>\r\n\n<br>\n\n$\\therefore$ Largest $\\ \\large x$ = {{{correctAnswer0}}}"}]}]

  606. <div class="sm_mode"> {{{question}}} </div>

    [{"vars":[{"varval":"\n$\\dfrac{2}{50} = \\dfrac{2}{55} + \\dfrac{1}{\\large x}$\n\nFind the value of $\\large x$?"},{"varval":"<div class=\"aligned\">\r\n\r\n| | |\r\n| -------------: | ---------- |\r\n| $\\dfrac{1}{\\large x}$ | \\= $\\dfrac{2}{50} - \\dfrac{2}{55}$ |\r\n| | \\= $\\dfrac{110}{2750}\\ -\\ \\dfrac{100}{2750}$ | \n| | \\= $\\dfrac{10}{2750}$ | \n| | \\= $\\dfrac{1}{275}$ | \n| $\\therefore \\ \\large x$ | \\= {{{correctAnswer0}}} |\r\n\r\n</div>"}]},{"vars":[{"varval":"\n$\\dfrac{2}{20} = \\dfrac{2}{25} + \\dfrac{1}{\\large x}$\n\nFind the value of $\\large x$?"},{"varval":"<div class=\"aligned\">\r\n\r\n| | |\r\n| -------------: | ---------- |\r\n| $\\dfrac{1}{\\large x}$ | \\= $\\dfrac{2}{20} - \\dfrac{2}{25}$ |\r\n| | \\= $\\dfrac{50}{500}\\ -\\ \\dfrac{40}{500}$ | \n| | \\= $\\dfrac{10}{500}$ | \n| | \\= $\\dfrac{1}{50}$ | \n| $\\therefore \\ \\large x$ | \\= {{{correctAnswer0}}} |\r\n\r\n</div>"}]},{"vars":[{"varval":"$\\dfrac{2}{10} = \\dfrac{2}{15} + \\dfrac{1}{\\large x}$\n\nFind the value of $\\large x$?"},{"varval":"<div class=\"aligned\">\r\n\r\n| | |\r\n| -------------: | ---------- |\r\n| $\\dfrac{1}{\\large x}$ | \\= $\\dfrac{2}{10} - \\dfrac{2}{15}$ |\r\n| | \\= $\\dfrac{30}{150}\\ -\\ \\dfrac{20}{150}$ | \n| | \\= $\\dfrac{10}{150}$ | \n| | \\= $\\dfrac{1}{15}$ | \n| $\\therefore \\ \\large x$ | \\= {{{correctAnswer0}}} |\r\n\r\n</div>"}]},{"vars":[{"varval":"$\\dfrac{2}{30} = \\dfrac{2}{35} + \\dfrac{1}{\\large x}$\n\nFind the value of $\\large x$?"},{"varval":"<div class=\"aligned\">\r\n\r\n| | |\r\n| -------------: | ---------- |\r\n| $\\dfrac{1}{\\large x}$ | \\= $\\dfrac{2}{30} - \\dfrac{2}{35}$ |\r\n| | \\= $\\dfrac{70}{1050}\\ -\\ \\dfrac{60}{1050}$ | \n| | \\= $\\dfrac{10}{1050}$ | \n| | \\= $\\dfrac{1}{105}$ | \n| $\\therefore \\ \\large x$ | \\= {{{correctAnswer0}}} |\r\n\r\n</div>"}]},{"vars":[{"varval":"$\\dfrac{2}{40} = \\dfrac{2}{45} + \\dfrac{1}{\\large x}$\n\nFind the value of $\\large x$?"},{"varval":"<div class=\"aligned\">\r\n\r\n| | |\r\n| -------------: | ---------- |\r\n| $\\dfrac{1}{\\large x}$ | \\= $\\dfrac{2}{40} - \\dfrac{2}{45}$ |\r\n| | \\= $\\dfrac{90}{1800}\\ -\\ \\dfrac{80}{1800}$ | \n| | \\= $\\dfrac{10}{1800}$ | \n| | \\= $\\dfrac{1}{180}$ | \n| $\\therefore \\ \\large x$ | \\= {{{correctAnswer0}}} |\r\n\r\n</div>"}]},{"vars":[{"varval":"$\\dfrac{2}{60} = \\dfrac{2}{65} + \\dfrac{1}{\\large x}$\n\nFind the value of $\\large x$?"},{"varval":"<div class=\"aligned\">\r\n\r\n| | |\r\n| -------------: | ---------- |\r\n| $\\dfrac{1}{\\large x}$ | \\= $\\dfrac{2}{60} - \\dfrac{2}{65}$ |\r\n| | \\= $\\dfrac{130}{3900}\\ -\\ \\dfrac{120}{3900}$ | \n| | \\= $\\dfrac{10}{3900}$ | \n| | \\= $\\dfrac{1}{390}$ | \n| $\\therefore \\ \\large x$ | \\= {{{correctAnswer0}}} |\r\n\r\n</div>"}]}]

  607. <div class="sm_mode"> {{{question}}} </div>

    [{"vars":[{"varval":"Joe is $\\dfrac{3}{4}$ the age of Nick.\r\n\r\nKevin is $\\dfrac{2}{3}$ the age of Nick.\r\n\r\nJoe is 3 years older than Kevin.\r\n\r\nHow old is Nick in years?"},{"varval":"<div class=\"aligned\">\r\n\r\n| | |\r\n| ------------- | ---------- |\r\n| $J$ | \\= $\\dfrac{3}{4} N \\ ... \\ (1)$ |\r\n| $K$ | \\= $\\dfrac{2}{3} N \\ ... \\ (2)$ |\r\n\r\n</div>\r\n\n\n<br>\n\nSince Joe is 3 years older than Kevin:\n\n<div class=\"aligned\">\r\n\r\n| | |\r\n| ------------: | ---------- |\r\n| $J \\ - \\ K$ | \\= 3 |\n| $\\dfrac{3}{4} N \\ - \\ \\dfrac{2}{3} N$ | \\= 3 |\n| $N \\bigg( \\dfrac{9}{12} \\ - \\ \\dfrac{8}{12} \\bigg)$ | \\= 3 |\n| $N \\times \\dfrac{1}{12}$ | \\= 3 |\r\n| $N$ | \\= {{{correctAnswer0}}} |\r\n\r\n</div>\n\n\r\n<br>\n\n$\\therefore$ Nick is {{{correctAnswer0}}} years old"}]},{"vars":[{"varval":"Mork is $\\dfrac{2}{5}$ the age of Conrad.\r\n\r\nOrson is $\\dfrac{1}{3}$ the age of Conrad.\r\n\r\nMork is 5 years older than Orson.\r\n\r\nHow old is Conrad in years?"},{"varval":"<div class=\"aligned\">\r\n\r\n| | |\r\n| ------------- | ---------- |\r\n| $M$ | \\= $\\dfrac{2}{5} C \\ ... \\ (1)$ |\r\n| $O$ | \\= $\\dfrac{1}{3} C \\ ... \\ (2)$ |\r\n\r\n</div>\r\n\n\n<br>\n\nSince Mork is 5 years older than Orson:\n\n<div class=\"aligned\">\r\n\r\n| | |\r\n| ------------: | ---------- |\r\n| $M \\ - \\ O$ | \\= 5 |\n| $\\dfrac{2}{5} C \\ - \\ \\dfrac{1}{3} C$ | \\= 5 |\n| $C \\bigg( \\dfrac{6}{15} \\ - \\ \\dfrac{5}{15} \\bigg)$ | \\= 5 |\n| $C \\times \\dfrac{1}{15}$ | \\= 5 |\r\n| $N$ | \\= {{{correctAnswer0}}} |\r\n\r\n</div>\n\n\r\n<br>\n\n$\\therefore$ Conrad is {{{correctAnswer0}}} years old"}]},{"vars":[{"varval":"Han is $\\dfrac{1}{2}$ the age of Darth.\r\n\r\nLuke is $\\dfrac{1}{3}$ the age of Darth.\r\n\r\nHan is 6 years older than Luke.\r\n\r\nHow old is Darth in years?"},{"varval":"<div class=\"aligned\">\r\n\r\n| | |\r\n| ------------- | ---------- |\r\n| $H$ | \\= $\\dfrac{1}{2} D \\ ... \\ (1)$ |\r\n| $L$ | \\= $\\dfrac{1}{3} D \\ ... \\ (2)$ |\r\n\r\n</div>\r\n\n\n<br>\n\nSince Han is 6 years older than Luke:\n\n<div class=\"aligned\">\r\n\r\n| | |\r\n| ------------: | ---------- |\r\n| $H \\ - \\ L$ | \\= 6 |\n| $\\dfrac{1}{2} D \\ - \\ \\dfrac{1}{3} D$ | \\= 6 |\n| $D \\bigg( \\dfrac{3}{6} \\ - \\ \\dfrac{2}{6} \\bigg)$ | \\= 6 |\n| $D \\times \\dfrac{1}{6}$ | \\= 6 |\r\n| $D$ | \\= {{{correctAnswer0}}} |\r\n\r\n</div>\n\n\r\n<br>\n\n$\\therefore$ Darth is {{{correctAnswer0}}} years old"}]},{"vars":[{"varval":"Max is $\\dfrac{4}{7}$ the age of Chief.\r\n\r\nBarbara is $\\dfrac{5}{8}$ the age of Chief.\r\n\r\nBarbara is 3 years older than Max.\r\n\r\nHow old is Chief in years?"},{"varval":"<div class=\"aligned\">\r\n\r\n| | |\r\n| ------------- | ---------- |\r\n| $M$ | \\= $\\dfrac{4}{7} C \\ ... \\ (1)$ |\r\n| $B$ | \\= $\\dfrac{5}{8} C \\ ... \\ (2)$ |\r\n\r\n</div>\r\n\n\n<br>\n\nSince Barbara is 3 years older than Max:\n\n<div class=\"aligned\">\r\n\r\n| | |\r\n| ------------: | ---------- |\r\n| $B \\ - \\ M$ | \\= 3|\n| $\\dfrac{5}{8} C \\ - \\ \\dfrac{4}{7} C$ | \\= 3 |\n| $C \\bigg( \\dfrac{35}{56} \\ - \\ \\dfrac{32}{56} \\bigg)$ | \\= 3 |\n| $C \\times \\dfrac{3}{56}$ | \\= 3 |\r\n|$3C$ | \\= 168 |\n|$C$ | \\= {{{correctAnswer0}}} |\r\n\r\n</div>\n\n\r\n<br>\n\n$\\therefore$ Chief is {{{correctAnswer0}}} years old"}]},{"vars":[{"varval":"Minn is $\\dfrac{4}{9}$ the age of Jae.\r\n\r\nAri is $\\dfrac{5}{12}$ the age of Jae.\r\n\r\nMinn is 2 years older than Ari.\r\n\r\nHow old is Jae in years?"},{"varval":"<div class=\"aligned\">\r\n\r\n| | |\r\n| ------------- | ---------- |\r\n| $M$ | \\= $\\dfrac{4}{9} J \\ ... \\ (1)$ |\r\n| $A$ | \\= $\\dfrac{5}{12} J \\ ... \\ (2)$ |\r\n\r\n</div>\r\n\n\n<br>\n\nSince Minn is 2 years older than Ari:\n\n<div class=\"aligned\">\r\n\r\n| | |\r\n| ------------: | ---------- |\r\n| $M \\ - \\ A$ | \\= 2 |\n| $\\dfrac{4}{9} J \\ - \\ \\dfrac{5}{12} J$ | \\= 2 |\n| $J \\bigg( \\dfrac{16}{36} \\ - \\ \\dfrac{15}{36} \\bigg)$ | \\= 2 |\n| $J \\times \\dfrac{1}{36}$ | \\= 2 |\r\n| $J$ | \\= {{{correctAnswer0}}} |\r\n\r\n</div>\n\n\r\n<br>\n\n$\\therefore$ Jae is {{{correctAnswer0}}} years old"}]},{"vars":[{"varval":"Fatima is $\\dfrac{3}{5}$ the age of Ahmed.\r\n\r\nDina is $\\dfrac{1}{2}$ the age of Ahmed.\r\n\r\nFatima is 5 years older than Dina.\r\n\r\nHow old is Ahmed in years?"},{"varval":"<div class=\"aligned\">\r\n\r\n| | |\r\n| ------------- | ---------- |\r\n| $F$ | \\= $\\dfrac{3}{5} A \\ ... \\ (1)$ |\r\n| $D$ | \\= $\\dfrac{1}{2} A \\ ... \\ (2)$ |\r\n\r\n</div>\r\n\n\n<br>\n\nSince Fatima is 5 years older than Dina:\n\n<div class=\"aligned\">\r\n\r\n| | |\r\n| ------------: | ---------- |\r\n| $F \\ - \\ D$ | \\= 5 |\n| $\\dfrac{3}{5} A \\ - \\ \\dfrac{1}{2} A$ | \\= 5 |\n| $A \\bigg( \\dfrac{6}{10} \\ - \\ \\dfrac{5}{10} \\bigg)$ | \\= 5 |\n| $A \\times \\dfrac{1}{10}$ | \\= 5 |\r\n| $A$ | \\= {{{correctAnswer0}}} |\r\n\r\n</div>\n\n\r\n<br>\n\n$\\therefore$ Ahmed is {{{correctAnswer0}}} years old"}]}]

  608. <div class="sm_mode"> {{{question}}} </div>

    [{"vars":[{"varval":"What is the value of $\\large x$ given $\\ \\dfrac{7}{2}\\large x$ + 7 = 4$\\large x$ $−$ 5?"},{"varval":"<div class=\"aligned\">\r\n\r\n| | |\r\n| ------------: | ---------- |\r\n| $\\dfrac{7}{2}\\large x$ + 7 | \\= $4\\large x$ $-$ 5 |\n| $7\\large x$ + 14 | \\= $8\\large x$ $-$ 10 |\r\n| $\\therefore \\large x$| \\= {{{correctAnswer0}}} |\r\n\r\n</div>"}]},{"vars":[{"varval":"What is the value of $\\large x$ given $\\ \\dfrac{4}{3}\\large x$ + 7 = 4$\\large x$ $−$ 17?"},{"varval":"<div class=\"aligned\">\r\n\r\n| | |\r\n| ------------: | ---------- |\r\n| $\\dfrac{4}{3}\\large x$ + 7 | \\= $4\\large x$ $-$ 17 |\n| $4\\large x$ + 21 | \\= $12\\large x$ $-$ 51 |\n| $8\\large x$ | \\= 72 |\r\n| $\\therefore \\large x$| \\= {{{correctAnswer0}}} |\r\n\r\n</div>"}]},{"vars":[{"varval":"What is the value of $\\large x$ given $\\ \\dfrac{5}{2}\\large x$ $-$ 7 = $\\large x$ $−$ 4?"},{"varval":"<div class=\"aligned\">\r\n\r\n| | |\r\n| ------------: | ---------- |\r\n| $\\dfrac{5}{2}\\large x$ $-$ 7 | \\= $\\large x$ $-$ 4 |\n| $5\\large x$ $-$ 14 | \\= $2\\large x$ $-$ 8 |\r\n| $3\\large x$ | \\= 6 |\n| $\\therefore \\large x$| \\= {{{correctAnswer0}}} |\r\n\r\n</div>"}]},{"vars":[{"varval":"What is the value of $\\large x$ given $\\ \\dfrac{5}{3}\\large x$ + 3 = 2$\\large x$ + 4?"},{"varval":"<div class=\"aligned\">\r\n\r\n| | |\r\n| ------------: | ---------- |\r\n| $\\dfrac{5}{3}\\large x$ + 3 | \\= $2\\large x$ + 4 |\n| $5\\large x$ + 9 | \\= $6\\large x$ + 12 |\n| $\\therefore \\large x$| \\= {{{correctAnswer0}}} |\r\n\r\n</div>"}]},{"vars":[{"varval":"What is the value of $\\large x$ given $\\ \\dfrac{5}{6}\\large x$ + 3 = 3$\\large x$ $−$ 10?"},{"varval":"<div class=\"aligned\">\r\n\r\n| | |\r\n| ------------: | ---------- |\r\n| $\\dfrac{5}{6}\\large x$ + 3 | \\= $3\\large x$ $-$ 10 |\n| $5\\large x$ + 18 | \\= $18\\large x$ $-$ 60 |\n| $13\\large x$ | \\= 78 |\r\n| $\\therefore \\large x$| \\= {{{correctAnswer0}}} |\r\n\r\n</div>"}]},{"vars":[{"varval":"What is the value of $\\large x$ given $\\ \\dfrac{2}{5}\\large x$ $-$ 3 = $\\large x$ $−$ 6?"},{"varval":"<div class=\"aligned\">\r\n\r\n| | |\r\n| ------------: | ---------- |\r\n| $\\dfrac{2}{5}\\large x$ $-$ 3 | \\= $\\large x$ $-$ 6 |\n| $2\\large x$ $-$ 15 | \\= $5\\large x$ $-$ 30 |\n| $3\\large x$ | \\= 15 |\r\n| $\\therefore \\large x$| \\= {{{correctAnswer0}}} |\r\n\r\n</div>"}]}]

  609. <div class="sm_mode"> {{{question}}} </div>

    [{"vars":[{"varval":"Sue and Belinda have total combined savings of $32.\r\n\r\nIf $\\dfrac{1}{5}$ of Sue's savings equals $\\dfrac{1}{3}$ of Belinda's savings, how much has Belinda saved?"},{"varval":"<div class=\"aligned\">\r\n\r\n| | |\r\n| ------------: | ---------- |\r\n| $\\dfrac{1}{5} S$ | \\= $\\dfrac{1}{3} B$ |\n| $S$ | \\= $\\dfrac{5}{3} B \\ ... \\ (1)$ |\r\n| $S + B$ | \\= 32 ... (2) |\r\n\r\n</div>\r\n\n<br>\n\nSubstitute (1) into (2)\n\n<div class=\"aligned\">\r\n\r\n| | |\r\n| ------------: | ---------- |\r\n| $\\dfrac{5}{3} B + B$ | \\= 32 |\n| $\\dfrac{8}{3} B$ | \\= 32 |\n| $B$| \\= 32 $\\times \\dfrac{3}{8}$|\n| | \\= {{{prefix0}}}{{{correctAnswer0}}} |\r\n\r\n</div>"}]},{"vars":[{"varval":"Blake and Ryan have total combined savings of $5400.\r\n\r\nIf $\\dfrac{1}{6}$ of Blake's savings equals $\\dfrac{1}{4}$ of Ryan's savings, how much has Ryan saved?"},{"varval":"<div class=\"aligned\">\r\n\r\n| | |\r\n| ------------: | ---------- |\r\n| $\\dfrac{1}{6} B$ | \\= $\\dfrac{1}{4} R$ |\n| $B$ | \\= $\\dfrac{6}{4} R$ |\n| $B$ | \\= $\\dfrac{3}{2} R \\ ... \\ (1)$ |\r\n| $B + R$ | \\= 5400 ... (2) |\r\n\r\n</div>\r\n\n<br>\n\nSubstitute (1) into (2)\n\n<div class=\"aligned\">\r\n\r\n| | |\r\n| ------------: | ---------- |\r\n| $\\dfrac{3}{2} R + R$ | \\= 5400 |\n| $\\dfrac{5}{2} R$ | \\= 5400 |\n| $R$| \\= 5400 $\\times \\dfrac{2}{5}$|\n| | \\= {{{prefix0}}}{{{correctAnswer0}}} |\r\n\r\n</div>"}]},{"vars":[{"varval":"Ellen and Jocelyn have total combined savings of $123 500.\r\n\r\nIf $\\dfrac{2}{3}$ of Ellen's savings equals $\\dfrac{3}{5}$ of Jocelyn's savings, how much has Jocelyn saved?"},{"varval":"<div class=\"aligned\">\r\n\r\n| | |\r\n| ------------: | ---------- |\r\n| $\\dfrac{2}{3} E$ | \\= $\\dfrac{3}{5} J$ |\n| $E$ | \\= $\\dfrac{9}{10} J \\ ... \\ (1)$ |\r\n| $E + J$ | \\= 123 500 ... (2) |\r\n\r\n</div>\r\n\n<br>\n\nSubstitute (1) into (2)\n\n<div class=\"aligned\">\r\n\r\n| | |\r\n| ------------: | ---------- |\r\n| $\\dfrac{9}{10} J + J$ | \\= 123 500 |\n| $\\dfrac{19}{10} J$ | \\= 123 500 |\n| $J$| \\= 123 500 $\\times \\dfrac{10}{19}$|\n| | \\= 65 000 |\r\n\r\n</div>"}]},{"vars":[{"varval":"Will and Ben have total combined savings of $12 810.\r\n\r\nIf $\\dfrac{4}{5}$ of Will's savings equals $\\dfrac{1}{4}$ of Ben's savings, how much has Ben saved?"},{"varval":"<div class=\"aligned\">\r\n\r\n| | |\r\n| ------------: | ---------- |\r\n| $\\dfrac{4}{5} W$ | \\= $\\dfrac{1}{4} B$ |\n| $W$ | \\= $\\dfrac{5}{16} B \\ ... \\ (1)$ |\r\n| $W + B$ | \\= 12 810 ... (2) |\r\n\r\n</div>\r\n\n<br>\n\nSubstitute (1) into (2)\n\n<div class=\"aligned\">\r\n\r\n| | |\r\n| ------------: | ---------- |\r\n| $\\dfrac{5}{16} B + B$ | \\= 12 810 |\n| $\\dfrac{21}{16} B$ | \\= 12 810 |\n| $B$| \\= 12 810 $\\times \\dfrac{16}{21}$|\n| | \\= {{{prefix0}}}{{{correctAnswer0}}} |\r\n\r\n</div>"}]},{"vars":[{"varval":"Batman and Robin have total combined savings of $1 776 000.\r\n\r\nIf $\\dfrac{1}{3}$ of Batman's savings equals $\\dfrac{9}{10}$ of Robin's savings, how much has Robin saved?"},{"varval":"<div class=\"aligned\">\r\n\r\n| | |\r\n| ------------: | ---------- |\r\n| $\\dfrac{1}{3} B$ | \\= $\\dfrac{9}{10} R$ |\n| $B$ | \\= $\\dfrac{27}{10} R \\ ... \\ (1)$ |\r\n| $B + R$ | \\= 1 776 000 ... (2) |\r\n\r\n</div>\r\n\n<br>\n\nSubstitute (1) into (2)\n\n<div class=\"aligned\">\r\n\r\n| | |\r\n| ------------: | ---------- |\r\n| $\\dfrac{27}{10} R + R$ | \\= 1 776 000 |\n| $\\dfrac{37}{10} R$ | \\= 1 776 000 |\n| $R$| \\= 1 776 000 $\\times \\dfrac{10}{37}$|\n| | \\= 480 000 |\r\n\r\n</div>"}]},{"vars":[{"varval":"Anna and Kristoff have total combined savings of $45.\r\n\r\nIf $\\dfrac{1}{5}$ of Anna's savings equals $\\dfrac{4}{5}$ of Kristoff's savings, how much has Kristoff saved?"},{"varval":"<div class=\"aligned\">\r\n\r\n| | |\r\n| ------------: | ---------- |\r\n| $\\dfrac{1}{5} A$ | \\= $\\dfrac{4}{5} K$ |\n| $A$ | \\= 4K ... (1) |\r\n| $A + K$ | \\= 45 ... (2) |\r\n\r\n</div>\r\n\n<br>\n\nSubstitute (1) into (2)\n\n<div class=\"aligned\">\r\n\r\n| | |\r\n| ------------: | ---------- |\r\n| $4K + K$ | \\= 45 |\n| $5 K$ | \\= 45 |\n| $K$| = {{{prefix0}}}{{{correctAnswer0}}} |\r\n\r\n</div>"}]}]

  610. <div class="sm_mode"> {{{question}}} </div>

    [{"vars":[{"varval":"A teacher is choosing two students from a group of 3 to ring the school bell.\r\n\r\nThere are a total of 3 different combinations that are possible.\r\n\r\nThe formula below gives the total number of combinations $C$ if the teacher is choosing from $S$ students.\r\n\r\n>> $C=0.5S (S − 1)$\r\n \r\n\r\n<br>What is the value of $S$ if the total possible combinations $C$ is 153? \n"},{"varval":"Strategy 1\r\n\nBy trial and error:\r\n\nIf $\\ S=12,\\ C=0.5×12×11=66$\r\n\nIf $\\ S=16,\\ C=0.5×16×15=120$\r\n\nIf $\\ S=18,\\ C=0.5×18×17=153$\r $\\checkmark$\n\n<br>\n\nStrategy 2 (advanced)\r\n\n$C=0.5S(S\\ −\\ 1)$\r\n\n$153=0.5S^{2}\\ −\\ 0.5S$\r\n\n$S^{2}\\ −\\ S\\ −\\ 306=0$\r\n\n$(S\\ −\\ 18)(S+17)=0$\r\n\n$\\therefore \\ S$ = {{{correctAnswer0}}} , $\\ S>0$"}]},{"vars":[{"varval":"A coach is choosing two players from a group of 3 to play in the goals in the upcoming netball semi-final.\r\n\r\nThere are a total of 3 different combinations that are possible.\r\n\r\nThe formula below gives the total number of combinations $C$ if the coach is choosing from $P$ players.\r\n\r\n>> $C=0.5P (P − 1)$\r\n \r\n\r\n<br>What is the value of $P$ if the total possible combinations $C$ is 91? \n"},{"varval":"Strategy 1\r\n\nBy trial and error:\r\n\nIf $\\ P=10,\\ C=0.5×10×9=45$\r\n\nIf $\\ P=12,\\ C=0.5×12×11=66$\r\n\nIf $\\ P=14,\\ C=0.5×14×13=91$\r $\\checkmark$\n\n<br>\n\nStrategy 2 (advanced)\r\n\n$C=0.5P(P\\ −\\ 1)$\r\n\n$91=0.5P^{2}\\ −\\ 0.5P$\r\n\n$P^{2}\\ −\\ P\\ −\\ 182=0$\r\n\n$(P\\ −\\ 14)(P+13)=0$\r\n\n$\\therefore \\ P$ = {{{correctAnswer0}}} , $\\ P>0$"}]},{"vars":[{"varval":"A teacher is choosing two students from a group of 3 to be class captains.\r\n\r\nThere are a total of 3 different combinations that are possible.\r\n\r\nThe formula below gives the total number of combinations $C$ if the teacher is choosing from $S$ students.\r\n\r\n>> $C=0.5S (S − 1)$\r\n \r\n\r\n<br>What is the value of $S$ if the total possible combinations $C$ is 406? "},{"varval":"Strategy 1\r\n\nBy trial and error:\r\n\nIf $\\ S=25,\\ C=0.5×25×24=300$\r\n\nIf $\\ S=27,\\ C=0.5×27×26=351$\r\n\nIf $\\ S=29,\\ C=0.5×29×28=406$\r $\\checkmark$\n\n<br>\n\nStrategy 2 (advanced)\r\n\n$C=0.5S(S\\ −\\ 1)$\r\n\n$406=0.5S^{2}\\ −\\ 0.5S$\r\n\n$S^{2}\\ −\\ S\\ −\\ 812=0$\r\n\n$(S\\ −\\ 29)(S+28)=0$\r\n \r\n$\\therefore \\ S$ = {{{correctAnswer0}}} , $\\ S>0$"}]},{"vars":[{"varval":"A gymnastics coach is choosing two gymnasts from a group of 3 to compete in the parallel bars event.\r\n\r\nThere are a total of 3 different combinations that are possible.\r\n\r\nThe formula below gives the total number of combinations $C$ if the coach is choosing from $G$ gymnasts.\r\n\r\n>> $C=0.5G (G − 1)$\r\n \r\n\r\n<br>What is the value of $G$ if the total possible combinations $C$ is 15? "},{"varval":"Strategy 1\r\n\nBy trial and error:\r\n\nIf $\\ G=4,\\ C=0.5×4×3=6$\r\n\nIf $\\ G=5,\\ C=0.5×5×4=10$\r\n\nIf $\\ G=6,\\ C=0.5×6×5=15$\r $\\checkmark$\n\n<br>\n\nStrategy 2 (advanced)\r\n\n$C=0.5G(G\\ −\\ 1)$\r\n\n$15=0.5G^{2}\\ −\\ 0.5G$\r\n\n$G^{2}\\ −\\ G\\ −\\ 30=0$\r\n\n$(G\\ −\\ 6)(G+5)=0$\r\n \r\n$\\therefore \\ G$ = {{{correctAnswer0}}} , $\\ G>0$"}]},{"vars":[{"varval":"A teacher is choosing two students from a group of 3 to compete in the javelin event at the zone athletics carnival.\r\n\r\nThere are a total of 3 different combinations that are possible.\r\n\r\nThe formula below gives the total number of combinations $C$ if the teacher is choosing from $S$ students.\r\n\r\n>> $C=0.5S (S − 1)$\r\n \r\n\r\n<br>What is the value of $S$ if the total possible combinations $C$ is 78? \n"},{"varval":"Strategy 1\r\n\nBy trial and error:\r\n\nIf $\\ S=9,\\ C=0.5×9×8=36$\r\n\nIf $\\ S=11,\\ C=0.5×11×10=55$\r\n\nIf $\\ S=13,\\ C=0.5×13×12=78$\r $\\checkmark$\n\n<br>\n\nStrategy 2 (advanced)\r\n\n$C=0.5S(S\\ −\\ 1)$\r\n\n$78=0.5S^{2}\\ −\\ 0.5S$\r\n\n$S^{2}\\ −\\ S\\ −\\ 156=0$\r\n\n$(S\\ −\\ 13)(S+12)=0$\r\n \r\n$\\therefore \\ S$ = {{{correctAnswer0}}} , $\\ S>0$"}]},{"vars":[{"varval":"A teacher is choosing two students from a group of 3 to compete in the regional debating competition.\r\n\r\nThere are a total of 3 different combinations that are possible.\r\n\r\nThe formula below gives the total number of combinations $C$ if the teacher is choosing from $S$ students.\r\n\r\n>> $C=0.5S (S − 1)$\r\n \r\n\r\n<br>What is the value of $S$ if the total possible combinations $C$ is 300? "},{"varval":"Strategy 1\r\n\nBy trial and error:\r\n\nIf $\\ S=21,\\ C=0.5×21×20=210$\r\n\nIf $\\ S=23,\\ C=0.5×23×22=253$\r\n\nIf $\\ S=25,\\ C=0.5×25×24=300$\r $\\checkmark$\n\n<br>\n\nStrategy 2 (advanced)\r\n\n$C=0.5S(S\\ −\\ 1)$\r\n\n$300=0.5S^{2}\\ −\\ 0.5S$\r\n\n$S^{2}\\ −\\ S\\ −\\ 600=0$\r\n\n$(S\\ −\\ 25)(S+24)=0$\r\n \r\n$\\therefore \\ S$ = {{{correctAnswer0}}} , $\\ S>0$"}]}]

  611. <div class="sm_mode"> {{{question}}} </div>

    [{"vars":[{"varval":"Korin is saving to buy a Spiderman suit.\r\n\r\nHe created a table to keep track of his savings.\n\n<br> \n\n<div class=\"sm-table col1-color2\">\n\n>| Number of months ($\\large x$) |3|6|8|10|\n|:-:|:-:|:-:|:-:|:-:|\n| Amount in savings account ($\\large y$)|250|340|400|460|\n\n</div>\n\n<br>Complete the rule for the linear relationship between the amount of money in Korin's savings account and the number of months of saving."},{"varval":"When $\\ \\large x$ = 3, &nbsp;$\\large y$ = 250\n\n<div class=\"aligned\">\r\n\r\n| | |\r\n| ------------: | ---------- |\r\n| 250 | \\= <span class=\"sm_box\"> ` ` </span> × 3 + 160 |\n| | |\r\n| | |\r\n| <span class=\"sm_box\"> ` ` </span> | \\= $\\dfrac{250-160}{3}$ |\r\n|||\n|<span class=\"sm_box\"> ` ` </span> | \\= {{{correctAnswer0}}} |\r\n\r\n</div>\r"}]},{"vars":[{"varval":"Hans is saving to buy a new lederhosen for Octoberfest celebrations.\n\r\nHe created a table to keep track of his savings.\n\n<br> \n\n<div class=\"sm-table col1-color3\">\n\n>| Number of months ($\\large x$) |2|5|8|10|\n|:-:|:-:|:-:|:-:|:-:|\n| Amount in savings account ($\\large y$)|230|395|560|670|\n\n</div>\n\n<br>Complete the rule for the linear relationship between the amount of money in Hans' savings account and the number of months of saving."},{"varval":"When $\\ \\large x$ = 2, &nbsp;$\\large y$ = 230\n\n<div class=\"aligned\">\r\n\r\n| | |\r\n| ------------: | ---------- |\r\n| 230 | \\= <span class=\"sm_box\"> ` ` </span> × 2 + 120 |\n| | |\r\n| | |\r\n| <span class=\"sm_box\"> ` ` </span> | \\= $\\dfrac{230-120}{2}$ |\r\n|||\n|<span class=\"sm_box\"> ` ` </span> | \\= {{{correctAnswer0}}} |\r\n\r\n</div>\r"}]},{"vars":[{"varval":"Sara is saving up to purchase a new set of cloggs.\n\r\nShe created a table to keep track of her savings.\n\n<br> \n\n<div class=\"sm-table col1-color6\">\n\n>| Number of months ($\\large x$) |3|5|8|10|\n|:-:|:-:|:-:|:-:|:-:|\n| Amount in savings account ($\\large y$)|46.75|63.25|88.00|104.50|\n\n</div>\n\n<br>Complete the rule for the linear relationship between the amount of money in Sara's savings account and the number of months of saving."},{"varval":"When $\\ \\large x$ = 3, &nbsp;$\\large y$ = 46.75\n\n<div class=\"aligned\">\r\n\r\n| | |\r\n| ------------: | ---------- |\r\n| 46.75 | \\= <span class=\"sm_box\"> ` ` </span> × 3 + 22 |\n| | |\r\n| | |\r\n| <span class=\"sm_box\"> ` ` </span> | \\= $\\dfrac{46.75-22}{3}$ |\r\n|||\n|<span class=\"sm_box\"> ` ` </span> | \\= {{{correctAnswer0}}} |\r\n\r\n</div>\r"}]},{"vars":[{"varval":"Tiger is saving up to purchase a new set of golf clubs.\n\r\nHe created a table to keep track of his savings.\n\n<br> \n\n<div class=\"sm-table col1-color4\">\n\n>| Number of months ($\\large x$) |2|4|7|9|\n|:-:|:-:|:-:|:-:|:-:|\n| Amount in savings account ($\\large y$)|420|660|1020|1260|\n\n</div>\n\n<br>Complete the rule for the linear relationship between the amount of money in Tiger's savings account and the number of months of saving."},{"varval":"When $\\ \\large x$ = 2, &nbsp;$\\large y$ = 420\n\n<div class=\"aligned\">\r\n\r\n| | |\r\n| ------------: | ---------- |\r\n| 420 | \\= <span class=\"sm_box\"> ` ` </span> × 2 + 180 |\n| | |\r\n| | |\r\n| <span class=\"sm_box\"> ` ` </span> | \\= $\\dfrac{420-180}{2}$ |\r\n|||\n|<span class=\"sm_box\"> ` ` </span> | \\= {{{correctAnswer0}}} |\r\n\r\n</div>"}]},{"vars":[{"varval":"Axl is saving up to purchase a new piano.\n\r\nHe created a table to keep track of his savings.\n\n<br> \n\n<div class=\"sm-table col1-color5\">\n\n>| Number of months ($\\large x$) |3|6|8|10|\n|:-:|:-:|:-:|:-:|:-:|\n| Amount in savings account ($\\large y$)|1706|3512|4716|5920|\n\n</div>\n\n<br>Complete the rule for the linear relationship between the amount of money in Axl's savings account and the number of months of saving."},{"varval":"When $\\ \\large x$ = 3, &nbsp;$\\large y$ = 1706\n\n<div class=\"aligned\">\r\n\r\n| | |\r\n| ------------: | ---------- |\r\n| 1706 | \\= <span class=\"sm_box\"> ` ` </span> × 3 $-$ 100 |\n| | |\r\n| | |\r\n| <span class=\"sm_box\"> ` ` </span> | \\= $\\dfrac{1706+100}{3}$ |\r\n|||\n|<span class=\"sm_box\"> ` ` </span> | \\= {{{correctAnswer0}}} |\r\n\r\n</div>"}]},{"vars":[{"varval":"Jet is saving up to travel overseas.\n\r\nHe created a table to keep track of his savings.\n\n<br> \n\n<div class=\"sm-table col1-color1\">\n\n>| Number of months ($\\large x$) |2|4|7|10|\n|:-:|:-:|:-:|:-:|:-:|\n| Amount in savings account ($\\large y$)|381|812|1458.50|2105|\n\n</div>\n\n<br>Complete the rule for the linear relationship between the amount of money in Jet's savings account and the number of months of saving."},{"varval":"When $\\ \\large x$ = 2, &nbsp;$\\large y$ = 381\n\n<div class=\"aligned\">\r\n\r\n| | |\r\n| ------------: | ---------- |\r\n| 381 | \\= <span class=\"sm_box\"> ` ` </span> × 2 $-$ 50 |\n| | |\r\n| | |\r\n| <span class=\"sm_box\"> ` ` </span> | \\= $\\dfrac{381+50}{2}$ |\r\n|||\n|<span class=\"sm_box\"> ` ` </span> | \\= {{{correctAnswer0}}} |\r\n\r\n</div>"}]}]

  612. <div class="sm_mode"> {{{question}}} </div>

    [{"vars":[{"varval":"$\\dfrac{\\large x}{4} = \\dfrac{5}{7}$\n\nWhat is the value of $\\large x$?"},{"varval":"<div class=\"aligned\">\r\n\r\n| | |\r\n| ------------: | ---------- |\r\n| $\\dfrac{\\large x}{4}$ | \\= $\\dfrac{5}{7}$ |\n| $\\large x$ | \\= $\\dfrac{5 \\times 4}{7}$ |\r\n| | \\= {{{correctAnswer}}} |\r\n\r\n</div>\r"}]},{"vars":[{"varval":"$\\dfrac{\\large y}{4} = \\dfrac{4}{9}$\n\nWhat is the value of $\\large y$?"},{"varval":"<div class=\"aligned\">\r\n\r\n| | |\r\n| ------------: | ---------- |\r\n| $\\dfrac{\\large y}{4}$ | \\= $\\dfrac{4}{9}$ |\n| $\\large y$ | \\= $\\dfrac{4 \\times 4}{9}$ |\r\n| | \\= {{{correctAnswer}}} |\r\n\r\n</div>\r"}]},{"vars":[{"varval":"$\\dfrac{\\large n}{7} = \\dfrac{5}{9}$\n\nWhat is the value of $\\large n$?"},{"varval":"<div class=\"aligned\">\r\n\r\n| | |\r\n| ------------: | ---------- |\r\n| $\\dfrac{\\large n}{7}$ | \\= $\\dfrac{5}{9}$ |\n| $\\large n$ | \\= $\\dfrac{7 \\times 5}{9}$ |\r\n| | \\= {{{correctAnswer}}} |\r\n\r\n</div>\r"}]},{"vars":[{"varval":"$\\dfrac{\\large m}{11} = \\dfrac{2}{5}$\n\nWhat is the value of $\\large m$?"},{"varval":"<div class=\"aligned\">\r\n\r\n| | |\r\n| ------------: | ---------- |\r\n| $\\dfrac{\\large m}{11}$ | \\= $\\dfrac{2}{5}$ |\n| $\\large m$ | \\= $\\dfrac{11 \\times 2}{5}$ |\r\n| | \\= {{{correctAnswer}}} |\r\n\r\n</div>"}]},{"vars":[{"varval":"$\\dfrac{3\\large p}{8} = \\dfrac{11}{7}$\n\nWhat is the value of $\\large p$?"},{"varval":"<div class=\"aligned\">\r\n\r\n| | |\r\n| ------------: | ---------- |\r\n| $\\dfrac{3\\large p}{8}$ | \\= $\\dfrac{11}{7}$ |\n| $3 \\large p$ | \\= $\\dfrac{11 \\times 8}{7}$ |\r\n| $\\large p$ | \\= $\\dfrac{88}{7 \\times 3}$ |\r\n| | \\= {{{correctAnswer}}} |\r\n\r\n</div>"}]},{"vars":[{"varval":"$\\dfrac{2\\large q}{3} = \\dfrac{3}{5}$\n\nWhat is the value of $\\large q$?"},{"varval":"<div class=\"aligned\">\r\n\r\n| | |\r\n| ------------: | ---------- |\r\n| $\\dfrac{2\\large q}{3}$ | \\= $\\dfrac{3}{5}$ |\n| $2\\large q$ | \\= $\\dfrac{3 \\times 3}{5}$ |\r\n| $\\large q$ | \\= $\\dfrac{9}{5 \\times 2}$ |\r\n| | \\= {{{correctAnswer}}} |\r\n\r\n</div>"}]}]

  613. Awaiting images from Myca. Sent 28/7.

    <div class="sm_mode"> {{{question}}} </div>

    [{"vars":[{"varval":"The mass of a trout fish is 16.9 kilograms.\r\n\r\nIts length is 1.24 metres.\n\nsm_img https://teacher.smartermaths.com.au/wp-content/uploads/2018/07/NAPX-F4-CA29-SA_1.svg 330 indent3 vpad\r\n\nA rule that can be used to approximately predict the length of a trout from its mass is\r\n\n<br>\n\r\n\n>>$\\large l$ = $\\sqrt{ \\dfrac{m}{10}}$\r\n\n<br>where $\\large l$ is the length of the trout in metres and $\\large m$ is the mass in kilograms.\r\n\r\nWhat is the difference in centimetres between the trout's actual length and the length predicted by the rule?\n"},{"varval":"Actual length = 1.24 m = 124 cm\r\n\nPredicted length\n\n>> = $\\sqrt{\\dfrac{16.9}{10}}$\n\n>> = $\\sqrt{1.69}$\n\n>>= 1.3 m\n\n>>= 130 cm\n\n<br>\n\n<div class=\"aligned\">\r\n\r\n| | |\r\n| ------------: | ---------- |\r\n| $\\therefore$ Difference\t | \\= 130 $-$ 124 |\r\n| | \\= {{{correctAnswer0}}} {{{suffix0}}} |\r\n\r\n</div>\r\n\n"}]},{"vars":[{"varval":"The mass of a trout fish is 12.1 kilograms.\r\n\r\nIts length is 1.05 metres.\n\nsm_img https://teacher.smartermaths.com.au/wp-content/uploads/2022/08/NAPX-F4-CA29-SA_var1.svg 330 indent3 vpad\r\n\nA rule that can be used to approximately predict the length of a trout from its mass is\r\n\n<br>\n\r\n\n>>$\\large l$ = $\\sqrt{ \\dfrac{m}{10}}$\r\n\n<br>where $\\large l$ is the length of the trout in metres and $\\large m$ is the mass in kilograms.\r\n\r\nWhat is the difference in centimetres between the trout's actual length and the length predicted by the rule?\n"},{"varval":"Actual length = 1.05 m = 105 cm\r\n\nPredicted length\n\n>> = $\\sqrt{\\dfrac{12.1}{10}}$\n\n>> = $\\sqrt{1.21}$\n\n>>= 1.1 m\n\n>>= 110 cm\n\n<br>\n\n<div class=\"aligned\">\r\n\r\n| | |\r\n| ------------: | ---------- |\r\n| $\\therefore$ Difference\t | \\= 110 $-$ 105 |\r\n| | \\= {{{correctAnswer0}}} {{{suffix0}}} |\r\n\r\n</div>\r\n\n"}]},{"vars":[{"varval":"The mass of a trout fish is 6.4 kilograms.\r\n\r\nIts length is 0.87 metres.\n\nsm_img https://teacher.smartermaths.com.au/wp-content/uploads/2022/08/NAPX-F4-CA29-SA_var2.svg 330 indent3 vpad\r\n\nA rule that can be used to approximately predict the length of a trout from its mass is\r\n\n<br>\n\r\n\n>>$\\large l$ = $\\sqrt{ \\dfrac{m}{10}}$\r\n\n<br>where $\\large l$ is the length of the trout in metres and $\\large m$ is the mass in kilograms.\r\n\r\nWhat is the difference in centimetres between the trout's actual length and the length predicted by the rule?\n"},{"varval":"Actual length = 0.87 m = 87 cm\r\n\nPredicted length\n\n>> = $\\sqrt{\\dfrac{6.4}{10}}$\n\n>> = $\\sqrt{0.64}$\n\n>>= 0.80 m\n\n>>= 80 cm\n\n<br>\n\n<div class=\"aligned\">\r\n\r\n| | |\r\n| ------------: | ---------- |\r\n| $\\therefore$ Difference\t | \\= 87 $-$ 80 |\r\n| | \\= {{{correctAnswer0}}} {{{suffix0}}} |\r\n\r\n</div>\r\n"}]},{"vars":[{"varval":"The mass of a trout fish is 14.4 kilograms.\r\n\r\nIts length is 1.18 metres.\n\nsm_img https://teacher.smartermaths.com.au/wp-content/uploads/2022/08/NAPX-F4-CA29-SA_var3.svg 330 indent3 vpad\r\n\nA rule that can be used to approximately predict the length of a trout from its mass is\r\n\n<br>\n\r\n\n>>$\\large l$ = $\\sqrt{ \\dfrac{m}{10}}$\r\n\n<br>where $\\large l$ is the length of the trout in metres and $\\large m$ is the mass in kilograms.\r\n\r\nWhat is the difference in centimetres between the trout's actual length and the length predicted by the rule?\n"},{"varval":"Actual length = 1.18 m = 118 cm\r\n\nPredicted length\n\n>> = $\\sqrt{\\dfrac{14.4}{10}}$\n\n>> = $\\sqrt{1.44}$\n\n>>= 1.20 m\n\n>>= 120 cm\n\n<br>\n\n<div class=\"aligned\">\r\n\r\n| | |\r\n| ------------: | ---------- |\r\n| $\\therefore$ Difference\t | \\= 120 $-$ 118 |\r\n| | \\= {{{correctAnswer0}}} {{{suffix0}}} |\r\n\r\n</div>\r\n"}]},{"vars":[{"varval":"The mass of a trout fish is 8.1 kilograms.\r\n\r\nIts length is 0.87 metres.\n\nsm_img https://teacher.smartermaths.com.au/wp-content/uploads/2022/08/NAPX-F4-CA29-SA_var4.svg 330 indent3 vpad\r\n\nA rule that can be used to approximately predict the length of a trout from its mass is\r\n\n<br>\n\r\n\n>>$\\large l$ = $\\sqrt{ \\dfrac{m}{10}}$\r\n\n<br>where $\\large l$ is the length of the trout in metres and $\\large m$ is the mass in kilograms.\r\n\r\nWhat is the difference in centimetres between the trout's actual length and the length predicted by the rule?\n"},{"varval":"Actual length = 0.87 m = 87 cm\r\n\nPredicted length\n\n>> = $\\sqrt{\\dfrac{8.1}{10}}$\n\n>> = $\\sqrt{0.81}$\n\n>>= 0.90 m\n\n>>= 90 cm\n\n<br>\n\n<div class=\"aligned\">\r\n\r\n| | |\r\n| ------------: | ---------- |\r\n| $\\therefore$ Difference\t | \\= 90 $-$ 87 |\r\n| | \\= {{{correctAnswer0}}} {{{suffix0}}} |\r\n\r\n</div>\r\n"}]},{"vars":[{"varval":"The mass of a trout fish is 4.9 kilograms.\r\n\r\nIts length is 0.64 metres.\n\nsm_img https://teacher.smartermaths.com.au/wp-content/uploads/2022/08/NAPX-F4-CA29-SA_var5.svg 330 indent3 vpad\r\n\nA rule that can be used to approximately predict the length of a trout from its mass is\r\n\n<br>\n\r\n\n>>$\\large l$ = $\\sqrt{ \\dfrac{m}{10}}$\r\n\n<br>where $\\large l$ is the length of the trout in metres and $\\large m$ is the mass in kilograms.\r\n\r\nWhat is the difference in centimetres between the trout's actual length and the length predicted by the rule?\n"},{"varval":"Actual length = 0.64 m = 64 cm\r\n\nPredicted length\n\n>> = $\\sqrt{\\dfrac{4.9}{10}}$\n\n>> = $\\sqrt{0.49}$\n\n>>= 0.70 m\n\n>>= 70 cm\n\n<br>\n\n<div class=\"aligned\">\r\n\r\n| | |\r\n| ------------: | ---------- |\r\n| $\\therefore$ Difference\t | \\= 70 $-$ 64 |\r\n| | \\= {{{correctAnswer0}}} {{{suffix0}}} |\r\n\r\n</div>\r\n"}]}]

  614. <div class="sm_mode"> {{{question}}} </div>

    [{"vars":[{"varval":"sm_img https://teacher.smartermaths.com.au/wp-content/uploads/2022/08/Algebra_NAPX-G4-CA25-SA_v0.svg 470 indent vpad\n\nThe rule for the graph is\r\n\r\n\n>>$A= \\dfrac{\\large c^2}{\\large k}$\n\n<br>\n\r\n\n$A$ is the area of bushland in square kilometres.\r\n\r\n$\\large c$ is the estimated population of feral cats.\r\n\r\nThe graph shows that a 40 km² area of bushland is estimated to have a feral cat population of 80.\r\n\r\nUse this to calculate the value of $\\large k$.\n"},{"varval":"(80, 40) represents one point on the graph.\n\nSubstituting into the given formula:\n\n<div class=\"aligned\">\r\n\r\n| | |\r\n| ------------: | ---------- |\r\n| 40 | \\= $\\dfrac{80^2}{\\large k}$ |\n| | |\r\n| $\\therefore \\large k$ | \\= $\\dfrac{80^2}{40}$ |\r\n| | \\= {{{correctAnswer0}}} |\r\n\r\n</div>"}]},{"vars":[{"varval":"sm_img https://teacher.smartermaths.com.au/wp-content/uploads/2022/08/Algebra_NAPX-G4-CA25-SA_v1.svg 450 indent vpad\n\nThe rule for the graph is\r\n\r\n\n>>$A= \\dfrac{\\large c^2}{\\large k}$\n\n<br>\n\r\n\n$A$ is the area of bushland in square kilometres.\r\n\r\n$\\large c$ is the estimated population of feral cats.\r\n\r\nThe graph shows that a 30 km² area of bushland is estimated to have a feral cat population of 60.\r\n\r\nUse this to calculate the value of $\\large k$.\n"},{"varval":"(60, 30) represents one point on the graph.\n\nSubstituting into the given formula:\n\n<div class=\"aligned\">\r\n\r\n| | |\r\n| ------------: | ---------- |\r\n| 30 | \\= $\\dfrac{60^2}{\\large k}$ |\n| | |\r\n| $\\therefore \\large k$ | \\= $\\dfrac{60^2}{30}$ |\r\n| | \\= {{{correctAnswer0}}} |\r\n\r\n</div>"}]},{"vars":[{"varval":"sm_img https://teacher.smartermaths.com.au/wp-content/uploads/2022/08/Algebra_NAPX-G4-CA25-SA_v2.svg 420 indent vpad\n\nThe rule for the graph is\r\n\r\n\n>>$A= \\dfrac{\\large c^2}{\\large k}$\n\n<br>\n\r\n\n$A$ is the area of bushland in square kilometres.\r\n\r\n$\\large c$ is the estimated population of feral cats.\r\n\r\nThe graph shows that a 40 km² area of bushland is estimated to have a feral cat population of 60.\r\n\r\nUse this to calculate the value of $\\large k$.\n"},{"varval":"(60, 40) represents one point on the graph.\n\nSubstituting into the given formula:\n\n<div class=\"aligned\">\r\n\r\n| | |\r\n| ------------: | ---------- |\r\n| 40 | \\= $\\dfrac{60^2}{\\large k}$ |\n| | |\r\n| $\\therefore \\large k$ | \\= $\\dfrac{60^2}{40}$ |\r\n| | \\= {{{correctAnswer0}}} |\r\n\r\n</div>"}]},{"vars":[{"varval":"sm_img https://teacher.smartermaths.com.au/wp-content/uploads/2022/08/Algebra_NAPX-G4-CA25-SA_v3.svg 400 indent vpad\n\nThe rule for the graph is\r\n\r\n\n>>$A= \\dfrac{\\large c^2}{\\large k}$\n\n<br>\n\r\n\n$A$ is the area of bushland in square kilometres.\r\n\r\n$\\large c$ is the estimated population of feral cats.\r\n\r\nThe graph shows that a 20 km² area of bushland is estimated to have a feral cat population of 40.\r\n\r\nUse this to calculate the value of $\\large k$.\n"},{"varval":"(40, 20) represents one point on the graph.\n\nSubstituting into the given formula:\n\n<div class=\"aligned\">\r\n\r\n| | |\r\n| ------------: | ---------- |\r\n| 20 | \\= $\\dfrac{40^2}{\\large k}$ |\n| | |\r\n| $\\therefore \\large k$ | \\= $\\dfrac{40^2}{20}$ |\r\n| | \\= {{{correctAnswer0}}} |\r\n\r\n</div>"}]},{"vars":[{"varval":"sm_img https://teacher.smartermaths.com.au/wp-content/uploads/2022/08/Algebra_NAPX-G4-CA25-SA_v4.svg 500 indent vpad\n\nThe rule for the graph is\r\n\r\n\n>>$A= \\dfrac{\\large c^2}{\\large k}$\n\n<br>\n\r\n\n$A$ is the area of bushland in square kilometres.\r\n\r\n$\\large c$ is the estimated population of feral cats.\r\n\r\nThe graph shows that a 40 km² area of bushland is estimated to have a feral cat population of 100.\r\n\r\nUse this to calculate the value of $\\large k$.\n"},{"varval":"(100, 40) represents one point on the graph.\n\nSubstituting into the given formula:\n\n<div class=\"aligned\">\r\n\r\n| | |\r\n| ------------: | ---------- |\r\n| 40 | \\= $\\dfrac{100^2}{\\large k}$ |\n| | |\r\n| $\\therefore \\large k$ | \\= $\\dfrac{100^2}{40}$ |\r\n| | \\= {{{correctAnswer0}}} |\r\n\r\n</div>"}]},{"vars":[{"varval":"sm_img https://teacher.smartermaths.com.au/wp-content/uploads/2022/08/Algebra_NAPX-G4-CA25-SA_v5.svg 500 indent vpad\n\nThe rule for the graph is\r\n\r\n\n>>$A= \\dfrac{\\large c^2}{\\large k}$\n\n<br>\n\r\n\n$A$ is the area of bushland in square kilometres.\r\n\r\n$\\large c$ is the estimated population of feral cats.\r\n\r\nThe graph shows that a 20 km² area of bushland is estimated to have a feral cat population of 60.\r\n\r\nUse this to calculate the value of $\\large k$.\n"},{"varval":"(60, 20) represents one point on the graph.\n\nSubstituting into the given formula:\n\n<div class=\"aligned\">\r\n\r\n| | |\r\n| ------------: | ---------- |\r\n| 20 | \\= $\\dfrac{60^2}{\\large k}$ |\n| | |\r\n| $\\therefore \\large k$ | \\= $\\dfrac{60^2}{20}$ |\r\n| | \\= {{{correctAnswer0}}} |\r\n\r\n</div>"}]}]

  615. <div class="sm_mode"> {{{question}}} </div>

    [{"vars":[{"varval":"Jim and Andy each bought a large order of plums from the farmers' market.\r\n\r\nJim bought three-quarters of the quantity that Andy bought.\r\n\r\nThe total cost of Jim and Andy's plums was $158.55.\r\n\r\nWhat was the cost of Andy's plums?"},{"varval":"<div class=\"aligned\">\r\n\r\n| | |\r\n| ------------: | ---------- |\r\n| Let $\\ \\large a$ | \\= cost of Andy's plums |\r\n| $\\dfrac{3}{4} \\large a$ | \\= cost of Jim's plums |\r\n\r\n</div>\r\n\n<br>\n\n<div class=\"aligned\">\r\n\r\n| | |\r\n| ------------: | ---------- |\r\n| $\\large a$ + $\\dfrac{3}{4} \\large a$ | \\= 158.55 |\n| $\\dfrac{7}{4}\\large a$ | \\= 158.55 |\n| $\\therefore \\large a$ | \\= 158.55 $\\div \\ \\dfrac{7}{4}$ |\r\n| | \\= {{{prefix0}}}{{{correctAnswer0}}} |\r\n\r\n</div>"}]},{"vars":[{"varval":"Bob and Chris each bought a large order of nails from the hardware store.\r\n\r\nBob bought four fifths of the quantity that Chris bought.\r\n\r\nThe total cost of Bob and Chris's nails was $62.55.\r\n\r\nWhat was the cost of Chris's nails?"},{"varval":"<div class=\"aligned\">\r\n\r\n| | |\r\n| ------------: | ---------- |\r\n| Let $\\ \\large c$ | \\= cost of Chris's nails |\r\n| $\\dfrac{4}{5} \\large c$ | \\= cost of Bob's nails |\r\n\r\n</div>\r\n\n<br>\n\n<div class=\"aligned\">\r\n\r\n| | |\r\n| ------------: | ---------- |\r\n| $\\large c$ + $\\dfrac{4}{5} \\large c$ | \\= 62.55 |\n| $\\dfrac{9}{5}\\large c$ | \\= 62.55 |\n| $\\therefore \\large c$ | \\= 62.55 $\\div \\ \\dfrac{9}{5}$ |\r\n| | \\= {{{prefix0}}}{{{correctAnswer0}}} |\r\n\r\n</div>"}]},{"vars":[{"varval":"Sally and Greta own adjoining blocks of land.\r\n\r\nSally's land is five-eighths the size of Greta's.\r\n\r\nThe combined area of Sally and Greta's land is 3380 square metres.\r\n\r\nWhat is the area of Greta's block in square metres?"},{"varval":"<div class=\"aligned\">\r\n\r\n| | |\r\n| ------------: | ---------- |\r\n| Let $\\ \\large g$ | \\= area of Greta's land |\r\n| $\\dfrac{5}{8} \\large g$ | \\= area of Sally's land |\r\n\r\n</div>\r\n\n<br>\n\n<div class=\"aligned\">\r\n\r\n| | |\r\n| ------------: | ---------- |\r\n| $\\large g$ + $\\dfrac{5}{8} \\large g$ | \\= 3380 |\n| $\\dfrac{13}{8}\\large g$ | \\= 3380 |\n| $\\therefore \\large g$ | \\= 3380 $\\div \\ \\dfrac{13}{8}$ |\r\n| | \\= {{{correctAnswer0}}} {{{suffix0}}} |\r\n\r\n</div>"}]},{"vars":[{"varval":"Bert and Ernie each bought a large order of cookies from the supermarket.\r\n\r\nBert bought two-thirds of the quantity that Ernie bought.\r\n\r\nThe total cost of Bert and Ernie's cookies was $64.50.\r\n\r\nWhat was the cost of Ernie's cookies?"},{"varval":"<div class=\"aligned\">\r\n\r\n| | |\r\n| ------------: | ---------- |\r\n| Let $\\ \\large e$ | \\= cost of Ernie's cookies |\r\n| $\\dfrac{2}{3} \\large e$ | \\= cost of Bert's cookies |\r\n\r\n</div>\r\n\n<br>\n\n<div class=\"aligned\">\r\n\r\n| | |\r\n| ------------: | ---------- |\r\n| $\\large e$ + $\\dfrac{2}{3} \\large e$ | \\= 64.50 |\n| $\\dfrac{5}{3}\\large e$ | \\= 64.50 |\n| $\\therefore \\large e$ | \\= 64.50 $\\div \\ \\dfrac{5}{3}$ |\r\n| | \\= {{{prefix0}}}{{{correctAnswer0}}} |\r\n\r\n</div>"}]},{"vars":[{"varval":"Kramer and Elaine each bought a large order of soup from the soup kitchen.\r\n\r\nKramer bought four-ninths of the quantity that Elaine bought.\r\n\r\nThe total cost of Kramer and Elaine's soup was $72.93.\r\n\r\nWhat was the cost of Elaine's soup?"},{"varval":"<div class=\"aligned\">\r\n\r\n| | |\r\n| ------------: | ---------- |\r\n| Let $\\ \\large e$ | \\= cost of Elaine's soup |\r\n| $\\dfrac{4}{9} \\large e$ | \\= cost of Kramer's soup |\r\n\r\n</div>\r\n\n<br>\n\n<div class=\"aligned\">\r\n\r\n| | |\r\n| ------------: | ---------- |\r\n| $\\large e$ + $\\dfrac{4}{9} \\large e$ | \\= 72.93 |\n| $\\dfrac{13}{9}\\large e$ | \\= 72.93 |\n| $\\therefore \\large e$ | \\= 72.93 $\\div \\ \\dfrac{13}{9}$ |\r\n| | \\= {{{prefix0}}}{{{correctAnswer0}}} |\r\n\r\n</div>"}]},{"vars":[{"varval":"April and May buy their coffee beans from the local cafe.\r\n\r\nLast month, April bought three-eighths of the quantity of coffee beans that May bought.\r\n\r\nThe total cost of April and May's coffee beans last month was $107.80.\r\n\r\nWhat was the cost of May's coffee beans over this time?"},{"varval":"<div class=\"aligned\">\r\n\r\n| | |\r\n| ------------: | ---------- |\r\n| Let &nbsp;$\\ \\large m$ | \\= cost of May's coffee beans |\r\n| $\\dfrac{3}{8} \\large m$ | \\= cost of April's coffee beans |\r\n\r\n</div>\r\n\n<br>\n\n<div class=\"aligned\">\r\n\r\n| | |\r\n| ------------: | ---------- |\r\n| $\\large m$ + $\\dfrac{3}{8} \\large m$ | \\= 107.80 |\n| $\\dfrac{11}{8}\\large m$ | \\= 107.80 |\n| $\\therefore \\large m$ | \\= 107.80 $\\div \\ \\dfrac{11}{8}$ |\r\n| | \\= {{{prefix0}}}{{{correctAnswer0}}} |\r\n\r\n</div>"}]}]

  616. Should this question have 4 options for the answers? Added categories.

    <div class="sm_mode"> {{{question}}} </div>

    [{"vars":[{"varval":"A nursery sells potting mix in two sizes.\r\n\r\n\nA 30 kilogram bag costs $5.50 and a 60 kilogram bag costs $11.00.\r\n\r\n\nWhich graph could show the relationship between the amount of potting mix and its cost?​"},{"varval":"sm_img https://teacher.smartermaths.com.au/wp-content/uploads/2018/06/NAPX-I4-CA09-rev-ans.svg 220 indent vpad"}]}]

  617. <div class="sm_mode"> {{{question}}} </div>

    [{"vars":[{"varval":"A restaurant sells pizza for $5 each and bottles of iced tea for $2 each.\r\n\r\nPaula wrote an equation $\\ 5\\large p$ + 2$\\large t$ = 38 &nbsp;to calculate the number of pizzas and bottles of iced tea she could buy.\r\n\r\nHow many pizzas and bottles of iced tea could Paula buy for $38?"},{"varval":"By trial and error:\r\n\nsm_nogap Cost of 6 pizzas and 4 bottles of iced tea\r\n\n>>= (6 × $5) + (4 × $2)\r\n\n>>= $38\n\r\n\r\n\r\n\n<br>\n\n$\\therefore$ 6 pizzas and 4 bottles of iced tea."}]},{"vars":[{"varval":"A restaurant sells bowls of soup for $8 each and bottles of sparkling water for $3 each.\r\n\r\nPaula wrote an equation $\\ 8\\large s$ + 3$\\large w$ = 61 &nbsp;to calculate the number of bowls of soup and bottles of sparkling water she could buy.\r\n\r\nHow many bowls of soup and bottles of sparkling water could Paula buy for $61?"},{"varval":"By trial and error:\r\n\nsm_nogap Cost of 5 bowls of soup and 7 bottles of sparkling water \r\n\n>>= (5 × $8) + (7 × $3)\r\n\n>>= $61\n\r\n\r\n\r\n\n<br>\n\n$\\therefore$ 5 bowls of soup and 7 bottles of sparkling water \r\n"}]},{"vars":[{"varval":"A cafe sells muffins for $4 each and cookies for $3 each.\r\n\r\nBrian wrote an equation $\\ 4\\large m$ + 3$\\large c$ = 64 &nbsp;to calculate the number of muffins and cookies he could buy.\r\n\r\nHow many muffins and cookies could Brian buy for $64?"},{"varval":"By trial and error:\r\n\nsm_nogap Cost of 10 muffins and 8 cookies\r\n\n>>= (10 × $4) + (8 × $3)\r\n\n>>= $64\n\r\n\r\n\r\n\n<br>\n\n$\\therefore$ 10 muffins and 8 cookies.\r\n"}]},{"vars":[{"varval":"A cafe sells large coffees for $5 each and small coffees for $4 each.\r\n\r\nJean wrote an equation $\\ 5\\large l$ + 4$\\large s$ = 57 &nbsp;to calculate the number of large coffees and small coffees she could buy.\r\n\r\nHow many large coffees and small coffees could Jean buy for $57?"},{"varval":"By trial and error:\r\n\nsm_nogap Cost of 9 large coffees and 3 small coffees\r\n\n>>= (9 × $5) + (3 × $4)\r\n\n>>= $57\n\r\n\r\n\r\n\n<br>\n\n$\\therefore$ 9 large coffees and 3 small coffees."}]},{"vars":[{"varval":"A movie theatre sells boxes of popcorn for $6 each and cans of soft drink for $3 each.\r\n\r\nBrad wrote an equation $\\ 6\\large p$ + 3$\\large d$ = 45 &nbsp;to calculate the number of boxes of popcorn and cans of soft drink he could buy.\r\n\r\nHow many boxes of popcorn and cans of soft drink could Brad buy for $45?"},{"varval":"By trial and error:\r\n\nsm_nogap 7 boxes of popcorn and 1 can of soft drink\r\n\n>>= (7 × $6) + (1 × $3)\r\n\n>>= $45\n\r\n\r\n\r\n\n<br>\n\n$\\therefore$ 7 boxes of popcorn and 1 can of soft drink."}]},{"vars":[{"varval":"A school canteen sells apples for $1.50 each and sausage rolls for $4 each.\r\n\r\nAnna wrote an equation $\\ 1.50\\large a$ + 4$\\large s$ = 25 &nbsp;to calculate the number of apples and sausage rolls she could buy.\r\n\r\nHow many apples and sausage rolls could Anna buy for $25?"},{"varval":"By trial and error:\r\n\nsm_nogap Cost of 6 apples and 4 sausage rolls\r\n\n>>= (6 × $1.50) + (4 × $4)\r\n\n>>= $25\n\r\n\r\n\r\n\n<br>\n\n$\\therefore$ 6 apples and 4 sausage rolls.\r\n"}]}]

  618. <div class="sm_mode"> {{{question}}} </div>

    [{"vars":[{"varval":"Tamaki owns a Japanese garden.\r\n\r\nHer garden pond has twice as many carp as bream, and four times as many bream as frogs.\r\n\r\nThe pond has 24 more bream than frogs.\r\n\r\nHow many carp does Tamaki's garden pond have?"},{"varval":"Let $\\ C$ = number of carp\n\nLet $\\ B$ = number of bream\n\nLet $\\ F$ = number of frogs \n\nsm_nogap Express the information in 3 equations:\n\n<div class=\"aligned\">\r\n\r\n>| | |\r\n| ------------: | ---------- |\r\n| $C$ | \\= $2B\\ ... \\ (1)$ |\n| $B$ | \\= $4F \\ … \\ (2)$ |\r\n| 24 | \\= $B - F$ ... $\\ (3)$ |\r\n\r\n</div>\r\n\n<br>\n\nsm_nogap Substitute $\\ B = 4F$ &nbsp;into (3)\n\n<div class=\"aligned\">\r\n\r\n>| | |\r\n| -------------: | ---------- |\r\n| 24| \\= $4F - F$ |\n| $3F$ | \\= 24 |\r\n| $F$ | \\= 8 |\r\n\r\n</div>\n\r\n\n<br>\n\nNumber of frogs = 8 \n\nsm_nogap Number of bream = $4 \\times 8 =32$\n\n<div class=\"aligned\">\r\n\r\n| | |\r\n| -------------: | ---------- |\r\n| $\\therefore$ Number of carp | \\= $2 \\times 32$ |\r\n| | \\= {{{correctAnswer0}}} |\r\n\r\n</div>"}]},{"vars":[{"varval":"Rosemary owns a fragrant garden.\r\n\r\nHer fragrant garden has twice as many roses as lavender plants, and three times as many lavender plants as jasmine.\r\n\r\nThe garden has 60 more lavender plants than jasmine.\r\n\r\nHow many roses does Rosemary's fragrant garden have?"},{"varval":"Let $\\ L$ = number of lavender plants\n\nLet $\\ R$ = number of roses\n\nLet $\\ J$ = number of jasmine \n\nsm_nogap Express the information in 3 equations:\n\n<div class=\"aligned\">\r\n\r\n>| | |\r\n| ------------: | ---------- |\r\n| $R$ | \\= $2L\\ ... \\ (1)$ |\n| $L$ | \\= $3J \\ … \\ (2)$ |\r\n| 60 | \\= $L - J$ ... $\\ (3)$ |\r\n\r\n</div>\r\n\n<br>\n\nsm_nogap Substitute $\\ L = 3J$ &nbsp;into (3)\n\n<div class=\"aligned\">\r\n\r\n>| | |\r\n| -------------: | ---------- |\r\n| 60| \\= $3J - J$ |\n| $2J$ | \\= 60 |\r\n| $J$ | \\= 30 |\r\n\r\n</div>\n\r\n\n<br>\n\nNumber of jasmine = 30\n\nNumber of lavender = $3 \\times 30 =90$\n\n<div class=\"aligned\">\r\n\r\n| | |\r\n| ------------- | ---------- |\r\n| $\\therefore$ Number of roses | \\= $2 \\times 90$ |\r\n| | \\= {{{correctAnswer0}}} |\r\n\r\n</div>"}]},{"vars":[{"varval":"\r\n\r\nGene has been shopping at the farmers' market to buy fruit.\n\nHe bought twice as many apples as pears, and 5 times as many pears as bananas.\r\n\r\n\nHe has 8 more pears than bananas.\r\n\r\n\nHow many apples did he buy?"},{"varval":"Let $\\ A$ = number of apples\n\nLet $\\ P$ = number of pears\n\nLet $\\ B$ = number of bananas\n\nsm_nogap Express the information in 3 equations:\n\n<div class=\"aligned\">\r\n\r\n>| | |\r\n| ------------: | ---------- |\r\n| $A$ | \\= $2P\\ ... \\ (1)$ |\n| $P$ | \\= $5B \\ … \\ (2)$ |\r\n| 8 | \\= $P - B$ ... $\\ (3)$ |\r\n\r\n</div>\r\n\n<br>\n\nsm_nogap Substitute $\\ P = 5B$ &nbsp;into (3)\n\n<div class=\"aligned\">\r\n\r\n>| | |\r\n| -------------: | ---------- |\r\n| 8| \\= $5B - B$ |\n| $4B$ | \\= 8 |\r\n| $B$ | \\= 2 |\r\n\r\n</div>\n\r\n\n<br>\n\nNumber of bananas = 2\n\nNumber of pears = $5 \\times 2 =10$\n\n<div class=\"aligned\">\r\n\r\n| | |\r\n| ------------- | ---------- |\r\n| $\\therefore$ Number of apples | \\= $2 \\times 10$ |\r\n| | \\= {{{correctAnswer0}}} |\r\n\r\n</div>"}]},{"vars":[{"varval":"Christian collects Formula One model racing cars.\n\nHis collection has three times as many Ferraris as McLarens, and two times as many McLarens as Alfa Romeos.\r\n\r\n\nChristian's collection has 3 more McLarens than Alfa Romeos.\r\n\r\n\nHow many Formula One model racing cars are in Christian's collection?"},{"varval":"Let $\\ F$ = number of Ferraris\n\nLet $\\ M$ = number of McLarens\n\nLet $\\ A$ = number of Alfa Romeos\n\nsm_nogap Express the information in 3 equations:\n\n<div class=\"aligned\">\r\n\r\n>| | |\r\n| ------------: | ---------- |\r\n| $F$ | \\= $3M\\ ... \\ (1)$ |\n| $M$ | \\= $2A \\ … \\ (2)$ |\r\n| 3 | \\= $M - A$ ... $\\ (3)$ |\r\n\r\n</div>\r\n\n<br>\n\nsm_nogap Substitute $\\ M = 2A$ &nbsp;into (3)\n\n<div class=\"aligned\">\r\n\r\n>| | |\r\n| -------------: | ---------- |\r\n| 3| \\= $2A - A$ |\n| $A$| \\= 3 |\n\r\n\r\n\r\n</div>\n\r\n\n<br>\n\nNumber of Alfa Romeos = 3\n\nNumber of McLarens = $3 \\times 2 = 6$\n\nNumber of Ferraris = $3 \\times 6 = 18$\n\n<div class=\"aligned\">\r\n\r\n| | |\r\n| ------------- | ---------- |\r\n| $\\therefore$ Total Formula One cars in collection | \\= 3 + 6 + 18 |\r\n| | \\= {{{correctAnswer0}}} |\r\n\r\n</div>"}]},{"vars":[{"varval":"A bag contains coloured marbles.\r\n\r\nThe bag has four times as many red marbles as blue marbles, and half as many blue marbles as green marbles.\r\n\r\nThe bag has 20 more green marbles than blue marbles.\r\n\r\nHow many marbles are in the bag?"},{"varval":"Let $\\ R$ = number of red marbles\n\nLet $\\ B$ = number of blue marbles\n\nLet $\\ G$ = number of green marbles \n\nsm_nogap Express the information in 3 equations:\n\n<div class=\"aligned\">\r\n\r\n>| | |\r\n| ------------: | ---------- |\r\n| $R$ | \\= $4B\\ ... \\ (1)$ |\n| $B$ | \\= $\\dfrac{1}{2}G … \\ (2)$ |\r\n| 20 | \\= $G - B$ ... $\\ (3)$ |\r\n\r\n</div>\r\n\n<br>\n\n\n\nsm_nogap Substitute $\\ B$ \\= $\\dfrac{1}{2}G$ &nbsp;into (3)\n\n<div class=\"aligned\">\r\n\r\n>| | |\r\n| -------------: | ---------- |\r\n| 20| \\= $G - \\dfrac{1}{2}G$ |\n| $\\dfrac{1}{2}G$ | \\= 20 |\r\n| $G$ | \\= 40 |\r\n\r\n</div>\n\r\n\n<br>\n\nNumber of green marbles = 40\n\nNumber of blue marbles = $\\dfrac{1}{2} \\times 40 =20$\n\nNumber of red marbles = $\\ 4 \\times 20 =80$\n\n<div class=\"aligned\">\r\n\r\n| | |\r\n| ------------- | ---------- |\r\n| $\\therefore$ Total marbles in bag | \\= $40 + 20 + 80$ |\r\n| | \\= {{{correctAnswer0}}} |\r\n\r\n</div>"}]}]

  619. <div class="sm_mode"> {{{question}}} </div>

    [{"vars":[{"varval":"Ember is six years younger than Keenak.\r\n\r\nStarla is thirteen years older than twice Keenak's age.\r\n\r\nThe sum of all three ages is 63.\r\n\r\nHow old is Starla?"},{"varval":"Strategy one:\r\n\nTry some educated guesses:\n\nIf Ember is 7,\r\n\nTotal of ages = 7 + 13 + 39 = 59\r\n\nIf Ember is 8,\r\n\nTotal of ages = 8 + 14 + 41 = 63\r $\\checkmark$\n \r\n\r<br>\n\nStrategy two (using algebra):\r\n\nExpress the information into 3 equations,\n\n<div class=\"aligned\">\r\n\r\n>| | |\r\n| ------------: | ---------- |\r\n| $E$ | \\= $K \\ −\\ 6 \\ ... \\ (1)$ |\n| $S$ | \\= $2K+13 \\ … \\ (2)$ |\r\n| $E+K+S$ | \\= 63 ... $\\ (3)$ |\r\n\r\n</div>\r\n\n<br>\n\nSubstitute (1) and (2) into (3)\n\n<div class=\"aligned\">\r\n\r\n>| | |\r\n| -------------: | ---------- |\r\n| $K\\ −\\ 6+K+2K+13$ | \\= 63 |\n| $4K$ | \\= 56 |\r\n| $K$ | \\= 14 |\r\n\r\n</div>\n\r\n\n<br>\n\n<div class=\"aligned\">\r\n\r\n| | |\r\n| ------------- | ---------- |\r\n| $\\therefore$ Starla's age | \\= $2×14+13$ |\r\n| | \\= {{{correctAnswer0}}} |\r\n\r\n</div>"}]},{"vars":[{"varval":"Michaela is four years younger than Brian.\r\n\r\nCormac is ten years older than twice Brian's age.\r\n\r\nThe sum of all three ages is 86.\r\n\r\nHow old is Cormac?"},{"varval":"Strategy one:\r\n\nTry some educated guesses:\n\nIf Michaela is 14,\r\n\nTotal of ages = 14 + 18 + 46 = 68\r\n\nIf Michaela is 16,\r\n\nTotal of ages = 16 + 20 + 50 = 86\r $\\checkmark$\n \r\n\r<br>\n\nStrategy two (using algebra):\r\n\nExpress the information into 3 equations,\n\n<div class=\"aligned\">\r\n\r\n>| | |\r\n| ------------: | ---------- |\r\n| $M$ | \\= $B \\ −\\ 4 \\ ... \\ (1)$ |\n| $C$ | \\= $2B+10 \\ … \\ (2)$ |\r\n| $M+B+C$ | \\= 86 ... $\\ (3)$ |\r\n\r\n</div>\r\n\n<br>\n\nSubstitute (1) and (2) into (3)\n\n<div class=\"aligned\">\r\n\r\n>| | |\r\n| -------------: | ---------- |\r\n| $B\\ −\\ 4+B+2B+10$ | \\= 86 |\n| $4B$ | \\= 80 |\r\n| $B$ | \\= 20 |\r\n\r\n</div>\n\r\n\n<br>\n\n<div class=\"aligned\">\r\n\r\n| | |\r\n| ------------- | ---------- |\r\n| $\\therefore$ Cormac's age | \\= $2×20+10$ |\r\n| | \\= {{{correctAnswer0}}} |\r\n\r\n</div>"}]},{"vars":[{"varval":"Abbie is five years younger than Bert.\r\n\r\nErnie is twenty years younger than three times Bert's age.\r\n\r\nThe sum of all three ages is 50.\r\n\r\nHow old is Ernie?"},{"varval":"Strategy one:\r\n\nTry some educated guesses:\n\nIf Abbie is 5,\r\n\nTotal of ages = 5 + 10 + 10 = 25\r\n\nIf Abbie is 10,\r\n\nTotal of ages = 10 + 15 + 25 = 50\r $\\checkmark$\n \r\n\r<br>\n\nStrategy two (using algebra):\r\n\nExpress the information into 3 equations,\n\n<div class=\"aligned\">\r\n\r\n>| | |\r\n| ------------: | ---------- |\r\n| $A$ | \\= $B \\ −\\ 5 \\ ... \\ (1)$ |\n| $E$ | \\= $3B\\ \\ − \\ 20 \\ … \\ (2)$ |\r\n| $A+B+E$ | \\= 50 ... $\\ (3)$ |\r\n\r\n</div>\r\n\n<br>\n\nSubstitute (1) and (2) into (3)\n\n<div class=\"aligned\">\r\n\r\n>| | |\r\n| -------------: | ---------- |\r\n| $B\\ −\\ 5+B+3B \\ − \\ 20$ | \\= 50 |\n| $5B$ | \\= 75 |\r\n| $B$ | \\= 15 |\r\n\r\n</div>\n\r\n\n<br>\n\n<div class=\"aligned\">\r\n\r\n| | |\r\n| ------------- | ---------- |\r\n| $\\therefore$ Ernie's age | \\= $3×15 \\ − \\ 20$ |\r\n| | \\= {{{correctAnswer0}}} |\r\n\r\n</div>"}]},{"vars":[{"varval":"Grace is ten years older than Kelly.\r\n\r\nCaroline is four years younger than three times Kelly's age.\r\n\r\nThe sum of all three ages is 76.\r\n\r\nHow old is Caroline?"},{"varval":"Strategy one:\r\n\nTry some educated guesses:\n\nIf Grace is 20,\r\n\nTotal of ages = 20 + 10 + 26 = 56\r\n\nIf Grace is 24,\r\n\nTotal of ages = 24 + 14 + 38 = 76\r $\\checkmark$\n \r\n\r<br>\n\nStrategy two (using algebra):\r\n\nExpress the information into 3 equations,\n\n<div class=\"aligned\">\r\n\r\n>| | |\r\n| ------------: | ---------- |\r\n| $G$ | \\= $K + 10 \\ ... \\ (1)$ |\n| $C$ | \\= $3K \\ −\\ 4 \\ … \\ (2)$ |\r\n| $G+K+C$ | \\= 76 ... $\\ (3)$ |\r\n\r\n</div>\r\n\n<br>\n\nSubstitute (1) and (2) into (3)\n\n<div class=\"aligned\">\r\n\r\n>| | |\r\n| -------------: | ---------- |\r\n| $K + 10+K+3K \\ − \\ 4$ | \\= 76 |\n| $5K$ | \\= 70 |\r\n| $K$ | \\= 14 |\r\n\r\n</div>\n\r\n\n<br>\n\n<div class=\"aligned\">\r\n\r\n| | |\r\n| ------------- | ---------- |\r\n| $\\therefore$ Caroline's age | \\= $3×14 \\ −\\ 4$ |\r\n| | \\= {{{correctAnswer0}}} |\r\n\r\n</div>"}]},{"vars":[{"varval":"Byron is twelve years older than Tennyson.\r\n\r\nEyre is one year younger than twice Tennyson's age.\r\n\r\nThe sum of all three ages is 23.\r\n\r\nHow old is Eyre?"},{"varval":"Strategy one:\r\n\nTry some educated guesses:\n\nIf Byron is 14,\r\n\nTotal of ages = 14 + 2 + 3 = 19\n\nIf Byron is 15,\r\n\nTotal of ages = 15 + 3 + 5 = 23\r $\\checkmark$\n \r\n\r<br>\n\nStrategy two (using algebra):\r\n\nExpress the information into 3 equations,\n\n<div class=\"aligned\">\r\n\r\n>| | |\r\n| ------------: | ---------- |\r\n| $B$ | = $T$ + 12 ... (1) |\n| $E$ | = $2T\\ −$ 1 ... (2) |\r\n| $B + T + E$ | = 23 ... (3) |\r\n\r\n</div>\r\n\n<br>\n\nSubstitute (1) and (2) into (3)\n\n<div class=\"aligned\">\r\n\r\n>| | |\r\n| -------------: | ---------- |\r\n| $T + 12 + T + 2T\\ −\\ 1$ | = 23 |\n| $4T$ | = 12 |\r\n| $T$ | = 3 |\r\n\r\n</div>\n\r\n\n<br>\n\n<div class=\"aligned\">\r\n\r\n| | |\r\n| ------------- | ---------- |\r\n| $\\therefore$ Eyre's age | \\= 2 $\\times$ 3 $-$ 1 |\r\n| | \\= {{{correctAnswer0}}} |\r\n\r\n</div>"}]},{"vars":[{"varval":"Harry is ten years younger than half Voldermort's age.\r\n\r\nDumbledore is three years younger than twice Voldermort's age.\r\n\r\nThe sum of all three ages is 162.\r\n\r\nHow old is Dumbledore?"},{"varval":"Strategy one:\r\n\nTry some educated guesses:\n\nIf Voldermort is 40,\r\n\nTotal of ages = 40 + 10 + 77 = 127\r\n\nIf Voldermort is 50,\r\n\nTotal of ages = 50 + 15 + 97 = 162\r $\\checkmark$\n \r\n\r<br>\n\nStrategy two (using algebra):\r\n\nExpress the information into 3 equations,\n\n<div class=\"aligned\">\r\n\r\n>| | |\r\n| ------------: | ---------- |\r\n| $H$ | = $O.5V\\ −\\ 10$ ... (1) |\n| $D$ | \\= $2V − \\ 3 \\ … \\ (2)$ |\r\n| $H+V+D$ | \\= 162 ... $\\ (3)$ |\r\n\r\n</div>\r\n\n<br>\n\nSubstitute (1) and (2) into (3)\n\n<div class=\"aligned\">\r\n\r\n>| | |\r\n| -------------: | ---------- |\r\n| $0.5V\\ − \\ 10+V+2V \\ −\\ 3$ | \\= 162 |\n| $3.5V$ | \\= 175 |\r\n| $V$ | \\= 50 |\r\n\r\n</div>\n\r\n\n<br>\n\n<div class=\"aligned\">\r\n\r\n| | |\r\n| ------------- | ---------- |\r\n| $\\therefore$ Dumbledore's age | \\= $2×50\\ −\\ 3$ |\r\n| | \\= {{{correctAnswer0}}} |\r\n\r\n</div>"}]}]

  620. <div class="sm_mode"> {{{question}}} </div>

    [{"vars":[{"varval":"A Greek wedding reception area has pillars that are $\\large x$ cm wide and the gap between pillars is $\\large y$ cm.\n\n<br>\n\nsm_img https://teacher.smartermaths.com.au/wp-content/uploads/2018/04/NAPX-J4-CA32.svg 240 indent2 vpad\n\n<br>If the reception area has 9 pillars on one side, the length of that side can be represented by which expression?"},{"varval":"sm_nogap The length of the side\r\n\n>> = width of 9 pillars + width of 8 gaps\r\n\n>> = {{{correctAnswer}}}\n"}]},{"vars":[{"varval":"A Greek wedding reception area has pillars that are $\\large x$ cm wide and the gap between pillars is $\\large y$ cm.\n\n<br>\n\nsm_img https://teacher.smartermaths.com.au/wp-content/uploads/2018/04/NAPX-J4-CA32.svg 240 indent2 vpad\n\n<br>If the reception area has 12 pillars on one side, the length of that side can be represented by which expression?"},{"varval":"sm_nogap The length of the side\r\n\n>> = width of 12 pillars + width of 11 gaps\r\n\n>> = {{{correctAnswer}}}\n"}]},{"vars":[{"varval":"A Greek wedding reception area has pillars that are $\\large x$ cm wide and the gap between pillars is $\\large y$ cm.\n\n<br>\n\nsm_img https://teacher.smartermaths.com.au/wp-content/uploads/2018/04/NAPX-J4-CA32.svg 240 indent2 vpad\n\n<br>If the reception area has 16 pillars on one side, the length of that side can be represented by which expression?"},{"varval":"sm_nogap The length of the side\r\n\n>> = width of 16 pillars + width of 15 gaps\r\n\n>> = {{{correctAnswer}}}\n"}]},{"vars":[{"varval":"A Greek wedding reception area has pillars that are $\\large x$ cm wide and the gap between pillars is $\\large y$ cm.\n\n<br>\n\nsm_img https://teacher.smartermaths.com.au/wp-content/uploads/2018/04/NAPX-J4-CA32.svg 240 indent2 vpad\n\n<br>If the reception area has 7 pillars on one side, the length of that side can be represented by which expression?"},{"varval":"sm_nogap The length of the side\r\n\n>> = width of 7 pillars + width of 6 gaps\r\n\n>> = {{{correctAnswer}}}\n"}]},{"vars":[{"varval":"A Greek wedding reception area has pillars that are $\\large x$ cm wide and the gap between pillars is $\\large y$ cm.\n\n<br>\n\nsm_img https://teacher.smartermaths.com.au/wp-content/uploads/2018/04/NAPX-J4-CA32.svg 240 indent2 vpad\n\n<br>If the reception area has 6 pillars on each of the 2 opposite walls, the total length of the pillar sections on both walls can be represented by which expression?"},{"varval":"sm_nogap The length of each side\r\n\n>> = width of 6 pillars + width of 5 gaps\n\r\n\nsm_nogap The length of both sides\n\n>> = 2 x (width of 6 pillars + width of 5 gaps)\r\n\n>> = 2 x (6$\\large x$ + 5$\\large y$)\r\n\n>> = {{{correctAnswer}}}\n"}]},{"vars":[{"varval":"A Greek wedding reception area has pillars that are $\\large x$ cm wide and the gap between pillars is $\\large y$ cm.\n\n<br>\n\nsm_img https://teacher.smartermaths.com.au/wp-content/uploads/2018/04/NAPX-J4-CA32.svg 240 indent2 vpad\n\n<br>If the reception area has 8 pillars on each of the 2 opposite walls, the total length of the pillar sections on both walls can be represented by which expression?"},{"varval":"sm_nogap The length of each side\r\n\n>> = width of 8 pillars + width of 7 gaps\n\r\n\nsm_nogap The length of both sides\n\n>> = 2 x (width of 8 pillars + width of 7 gaps)\r\n\n>> = 2 x (8$\\large x$ + 7$\\large y$)\r\n\n>> = {{{correctAnswer}}}\n"}]}]

  621. <div class="sm_mode"> Rory has 25 lollies that he shares with 3 other friends. He gave Steve 2 more than he kept for himself. He gave Delilah 3 less that lollies of Steve. Kelly has twice as many as Steve. How many lollies did Rory keep for himself? </div>

    [{"vars":null}]

  622. <div class="sm_mode"> A piece of wood is leaning against four stacks of bricks. sm_img https://teacher.smartermaths.com.au/wp-content/uploads/2018/08/NAPX-F3-CA24-SA.svg 250 indent vpad There are 31 bricks in total. How many bricks are in the tallest stack? </div>

    [{"vars":null}]

  623. <div class="sm_mode"> Alota and Bea are sisters and are setting up their study desks. They want their desks positioned the same distance from the centre of the room. <br> sm_img https://teacher.smartermaths.com.au/wp-content/uploads/2018/05/NAPX-H4-NC15.svg 570 indent vpad <br>What is the distance $\large x$ from the edge of each desk to the middle of the room? </div>

    [{"vars":null}]

  624. <div class="sm_mode"> {{{question}}} </div>

    [{"vars":[{"varval":"Marty is mining for gold in Papua New Guinea.\n\r\nHis mine aims to produce 13 kilograms of gold in the first 3 months of mining.\n\nIn the first month, the mine produces 2400 grams of gold.\n\nIn the second month, it produces 2900 grams of gold.\n\nHow much gold does the mine need to produce in the third month to reach Marty's target of 13 kilograms?\n\nGive your answer in kilograms, to one decimal place."},{"varval":"1 kilogram = 1000 grams\n\n<div class=\"aligned\">\n\n| | |\n| --------------------- | -------------- |\n|$\\therefore$ Gold needed in 3rd month | = 13 000 $−$ 2400 $−$ 2900 |\n| | \\= 7700 grams |\n|| \\= {{{correctAnswer0}}} {{{suffix0}}}|\n\n</div>"}]},{"vars":[{"varval":"Manny is making a vegetable soup for his vegan restaurant.\n\nHe wants to make 8 litres of soup in 3 batches.\n\nIn the first batch, the makes 2400 millilitres of soup.\n\nIn the second batch, he makes 3800 millilitres.\n\nHow much soup does Manny need to make in the 3rd batch to reach his target of 8 litres?\n\nGive your answer in litres, to one decimal place."},{"varval":"8 litres = 8000 millilitres\n\n<div class=\"aligned\">\n\n| | |\n| --------------------- | -------------- |\n|$\\therefore$ Soup needed in 3rd batch | = 8000 $−$ 2400 $−$ 3800 |\n| | \\= 1800 millilitres |\n|| \\= {{{correctAnswer0}}} {{{suffix0}}}|\n\n</div>"}]},{"vars":[{"varval":"Justine supplied a chocolate shop and needed to make 17 kilograms of Rocky Road for the shop over 3 days.\n\nOn the first day, she made 3500 grams of Rocky Road.\n\nOn the second day she made 5700 grams of Rock Road.\n\nHow much Rocky Road does she need to make on day 3 to reach her target of 17 kilograms?\n\nGive your answer in kilograms, to one decimal place."},{"varval":"<div class=\"aligned\">\n\n| | |\n| --------------------- | -------------- |\n|$\\therefore$ Rocky Road needed | = 17 000 $−$ 3500 $−$ 5700 |\n| | \\= 7800 grams |\n|| \\= {{{correctAnswer0}}} {{{suffix0}}}|\n\n</div>"}]},{"vars":[{"varval":"Delvine is making an energy juice drink for her cafe.\n\nShe wants to make 12 litres of juice in one hour and a half.\n\nIn the first half hour, she makes 1200 millilitres of juice.\n\nIn the second half hour, she makes 3300 millilitres.\n\nHow much juice does Delvine need to make in the third half hour to reach her target of 12 litres?\n\nGive your answer in litres, to one decimal place."},{"varval":"12 litres = 12 000 millilitres\n\n<div class=\"aligned\">\n\n| | |\n| --------------------- | -------------- |\n|$\\therefore$ Juice needed | = 12 000 $−$ 1200 $−$ 3300 |\n| | \\= 7500 millilitres |\n|| \\= {{{correctAnswer0}}} {{{suffix0}}}|\n\n</div>"}]},{"vars":[{"varval":"Andrew peels the prawns in a commercial restaurant and needs to prepare 16 kilograms of peeled prawns in 1 hour.\n\nIn the first 20 minutes, he peels 4400 grams of prawns.\n\nIn the second 20 minutes, he peels 5900 grams of prawns.\n\nWhat mass of prawns does Andrew need to peel in the third 20 minutes to reach his target of 16 kilograms?\n\nGive your answer in kilograms, to one decimal place."},{"varval":"1 kilogram = 1000 grams \n\n<div class=\"aligned\">\n\n| | |\n| --------------------- | -------------- |\n|$\\therefore$ Prawns needed | = 16 000 $−$ 4400 $−$ 5900 |\n| | \\= 5700 grams |\n|| \\= {{{correctAnswer0}}} {{{suffix0}}}|\n\n</div>"}]},{"vars":[{"varval":"Bert captures cane toads and aims to capture 32 kilograms during a 3 hour operation.\n\nIn the first hour, his captured cane toads weigh 8700 grams.\n\nIn the second hour, he captures an extra 9400 grams of cane toads.\n\nWhat mass of cane toads does Bert need to catch in the third hour in order to reach his target capture of 32 kilograms?\n\nGive your answer in kilograms, to one decimal place."},{"varval":"1 kilogram = 1000 grams\n\n<div class=\"aligned\">\n\n| | |\n| --------------------- | -------------- |\n|$\\therefore$ Cane toads needed | = 32 000 $−$ 8700 $−$ 9400 |\n| | \\= 13 900 grams |\n|| \\= {{{correctAnswer0}}} {{{suffix0}}}|\n\n</div>"}]}]

  625. <div class="sm_mode"> {{{question}}} </div>

    [{"vars":[{"varval":"A positive number $\\large x$ is multiplied by 6, then 12 is added.\r\n\r\nWhich of the following operations give the same result?\n"},{"varval":"Number = $6\\large x$ + 12\r\n \r\n\r\nConsider each option:\r\n\n>>$(\\large x$ + 2) × 12 = 12$\\large x$ + 24\t \r\n\n>>$(\\large x$ + 2) × 6 = 6$\\large x$ + 12 $\\ \\ \\checkmark$ \r\n\n>>$(\\large x$ + 6) × 2 = 2$\\large x$ + 12\r\n\n>>$(\\large x$ + 12) × 6 = 6$\\large x$ + 72\r\n\n\r\n\r\n<br>\n\n$\\therefore$ Add 2 to $\\large x$, then multiply by 6.\n"}]},{"vars":[{"varval":"A positive number $\\large x$ is multiplied by 5, then 10 is added.\r\n\r\nWhich of the following operations give the same result?\n"},{"varval":"Number = $5\\large x$ + 10\r\n \r\n\r\nConsider each option:\r\n\n>>$(\\large x$ + 2) × 5 = 5$\\large x$ + 10 $\\ \\ \\checkmark$ \n\n>>$(\\large x$ + 5) × 2 = 2$\\large x$ + 10\t \r\n\n\r\n>>$(\\large x$ + 10) × 5 = 5$\\large x$ + 50\r\n\n>>$(\\large x$ + 3) × 2 = 2$\\large x$ + 6\n\n\r\n\r\n<br>\n\n$\\therefore$ Add 2 to $\\large x$, then multiply by 5.\n"}]},{"vars":[{"varval":"A positive number $\\large x$ is multiplied by 3, then 6 is added.\r\n\r\nWhich of the following operations give the same result?\n"},{"varval":"Number = $3\\large x$ + 6\r\n \r\n\r\nConsider each option:\r\n\n>>$(\\large x$ + 3) × 2 = 2$\\large x$ + 6\r\n\n>>$(\\large x$ + 6) × 3 = 6$\\large x$ + 18\n\n\r\n>>$(\\large x$ + 2) × 3 = 3$\\large x$ + 6 $\\ \\ \\checkmark$\n\n>>$(\\large x$ + 2) × 6 = 6$\\large x$ + 12\t \r\n\n\n\n\r\n\r\n<br>\n\n$\\therefore$ Add 2 to $\\large x$, then multiply by 3.\n"}]},{"vars":[{"varval":"A positive number $\\large x$ is multiplied by 4, then 8 is added.\r\n\r\nWhich of the following operations give the same result?"},{"varval":"Number = $4\\large x$ + 8\r\n \r\n\r\nConsider each option:\r\n\n>>$(\\large x$ + 2) × 8 = 8$\\large x$ + 16\t \r\n\n>>$(\\large x$ + 8) × 2 = 2$\\large x$ + 16 \r\n\n>>$(\\large x$ + 4) × 2 = 2$\\large x$ + 8\r\n\n>>$(\\large x$ + 2) × 4 = 4$\\large x$ + 8 $\\ \\ \\checkmark$\r\n\n\r\n\r\n<br>\n\n$\\therefore$ Add 2 to $\\large x$, then multiply by 6.\n"}]},{"vars":[{"varval":"A positive number $\\large x$ is multiplied by 6, then 12 is subtracted.\r\n\r\nWhich of the following operations give the same result?\n"},{"varval":"Number = $6\\large x$ $-$ 12\r\n \r\n\r\nConsider each option:\r\n\n>>$(\\large x$ $-$ 2) × 12 = 12$\\large x$ $-$ 24\t \r\n\n>>$(\\large x$ $-$ 2) × 6 = 6$\\large x$ $-$ 12 $\\ \\ \\checkmark$ \r\n\n>>$(\\large x$ $-$ 6) × 2 = 2$\\large x$ $-$ 12\r\n\n>>$(\\large x$ $-$ 12) × 6 = 6$\\large x$ $-$ 72\r\n\n\r\n\r\n<br>\n\n$\\therefore$ Subtract 2 from $\\large x$, then multiply by 6.\n"}]},{"vars":[{"varval":"A positive number $\\large x$ is multiplied by 5, then 10 is subtracted.\r\n\r\nWhich of the following operations give the same result?\n"},{"varval":"Number = $5\\large x$ $-$ 10\r\n \r\n\r\nConsider each option:\r\n\n>>$(\\large x$ $-$ 2) × 5 = 5$\\large x$ $-$ 10 $\\ \\ \\checkmark$ \n\n>>$(\\large x$ $-$ 5) × 2 = 2$\\large x$ $-$ 10\t \r\n\n\r\n>>$(\\large x$ $-$ 10) × 5 = 5$\\large x$ $-$ 50\r\n\n>>$(\\large x$ $-$ 5) × 5 = 5$\\large x$ $-$ 25\r\n\n\r\n\r\n<br>\n\n$\\therefore$ Subtract 2 from $\\large x$, then multiply by 5."}]}]

  626. <div class="sm_mode"> {{{question}}} </div>

    [{"vars":[{"varval":"Vivian throws a cricket ball from a height of 2 metres above the ground.\r\n\r\nThe graph below shows the height of the ball above ground level against the time after Vivian releases the ball.\n\n<br>\n\nsm_img https://teacher.smartermaths.com.au/wp-content/uploads/2018/06/NAPX-F4-CA20.svg 390 indent vpad\n\n<br>How long does the cricket ball travel at or above 3 metres?\n"},{"varval":"Ball initially hits 3 m at 0.6 seconds.\r\n\nBall drops to 3 m at 3.2 seconds.\r\n\n$\\therefore$ Ball is at or above 3 m for {{{correctAnswer}}}.\n\n"}]},{"vars":[{"varval":"Ian throws a cricket ball from a height of 2 metres above the ground.\r\n\r\nThe graph below shows the height of the ball above ground level against the time after Ian releases the ball.\n\n<br>\n\nsm_img https://teacher.smartermaths.com.au/wp-content/uploads/2018/06/NAPX-F4-CA20.svg 390 indent vpad\n\n<br>How long does the cricket ball travel at or above 1.8 metres?\n"},{"varval":"Ball starts at 2 m so is above 1.8 m from 0 seconds.\r\n\nBall drops to 1.8 m at 3.6 seconds.\r\n\n$\\therefore$ Ball is at or above 1.8 m for {{{correctAnswer}}}."}]},{"vars":[{"varval":"Greg throws a cricket ball from a height of 2 metres above the ground.\r\n\r\nThe graph below shows the height of the ball above ground level against the time after Greg releases the ball.\n\n<br>\n\nsm_img https://teacher.smartermaths.com.au/wp-content/uploads/2018/06/NAPX-F4-CA20.svg 390 indent vpad\n\n<br>What is the difference between the balls maximum height and its starting height above the ground?\n"},{"varval":"Initial height of the ball is 2 m.\r\n\nMaximum height of the ball is 4.2 m.\r\n\n$\\therefore$ Difference between maximum and starting height is {{{correctAnswer}}}."}]},{"vars":[{"varval":"Shane throws a cricket ball **straight up** from a height of 2 metres above the ground.\r\n\r\nThe graph below shows the height of the ball above ground level against the time after Shane releases the ball.\n\n<br>\n\nsm_img https://teacher.smartermaths.com.au/wp-content/uploads/2018/06/NAPX-F4-CA20.svg 390 indent vpad\n\n<br>What is the total distance travelled by the ball in 4 seconds?\n"},{"varval":"Initial height of the ball is 2 m.\r\n\nMaximum height of the ball is 4.2 m.\r\n\nDifference between maximum and starting heights is &nbsp;4.2 $-$ 2 = 2.2 m\n\nThe ball then travels 4.2 metres to the ground.\n\n$\\therefore$ The total distance travelled by the ball is 4.2 + 2.2 = {{{correctAnswer}}}"}]},{"vars":[{"varval":"Ricky throws a cricket ball **straight up** from a height of 2 metres above the ground.\r\n\r\nThe graph below shows the height of the ball above ground level against the time after Ricky releases the ball.\n\n<br>\n\nsm_img https://teacher.smartermaths.com.au/wp-content/uploads/2018/06/NAPX-F4-CA20.svg 390 indent vpad\n\n<br>What is the total distance travelled by the ball in the first 3.8 seconds?\n"},{"varval":"Inital height of the ball is 2 m.\r\n\nAfter 2 seconds the ball reaches its maximum height of 4.2 m.\r\n\nDifference between maximum and starting heights is &nbsp;4.2 $-$ 2 = 2.2 m\n\nIn the next 1.8 seconds the ball travels from a height of 4.2 metres to 1 metre above the ground, or &nbsp;4.2 $-$ 1 = 3.2 m.\n\n$\\therefore$ Total distance = 2.2 + 3.2 = {{{correctAnswer}}} "}]},{"vars":[{"varval":"Imran throws a cricket ball from a height of 2 metres above the ground.\r\n\r\nThe graph below shows the height of the ball above ground level against the time after Imran releases the ball.\n\n<br>\n\nsm_img https://teacher.smartermaths.com.au/wp-content/uploads/2018/06/NAPX-F4-CA20.svg 390 indent vpad\n\n<br>How long does the cricket ball travel at or above 1 metre?\n"},{"varval":"Ball starts at 2 metres above the ground, so already above 1 metre.\r\n\nThe ball again drops to 1 m after 3.8 seconds.\r\n\n$\\therefore$ Ball is at or above 1 m for {{{correctAnswer}}}.\n\n"}]}]

  627. <div class="sm_mode"> Marigold strikes a tennis ball 1 metre above ground level and lobs the ball over her opponent. The tennis ball reaches a maximum height of 3 metres and lands 2 seconds after being hit. Which graph represents the path of Marigold's tennis ball? </div>

    [{"vars":null}]

  628. <div class="sm_mode"> {{{question}}} </div>

    [{"vars":[{"varval":"The number of registered vehicles in five countries are recorded in the table below.\n\n<br>\n\n<div class=\"outline\">\n\n>>> |Country|Registered Vehicles|\n|:-:|:-:|\n|Australia|18 337 225|\n|USA|263 520 104|\n|China|248 621 907|\n|India|73 818 963|\n|Norway|5 986 254|\n\n</div>\n\n<br>How many more registered vehicles are there in the USA than Australia and Norway combined?\n"},{"varval":"sm_nogap Extra cars in USA\n\n<div class=\"aligned\">\n\n>>||\n|-|\n|= 263 520 104 − (18 337 225 + 5 986 254)|\n|= 239 196 625|\n\n</div>\n"}]},{"vars":[{"varval":"The number of registered vehicles in five countries are recorded in the table below.\n\n<br>\n\n<div class=\"outline\">\n\n>>> |Country|Registered Vehicles|\n|:-:|:-:|\n|Australia|18 337 225|\n|USA|263 520 104|\n|China|248 621 907|\n|India|73 818 963|\n|Norway|5 986 254|\n\n</div>\n\n<br>How many more registered vehicles are there in China than Australia and India combined?"},{"varval":"sm_nogap Extra cars in China\n\n<div class=\"aligned\">\n\n>>||\n|-|\n|= 248 621 907 − (18 337 225 + 73 818 963)|\n|= 156 465 719|\n\n</div>\n"}]},{"vars":[{"varval":"The number of registered vehicles in five countries are recorded in the table below.\n\n<br>\n\n<div class=\"outline\">\n\n>>> |Country|Registered Vehicles|\n|:-:|:-:|\n|Australia|18 337 225|\n|USA|263 520 104|\n|China|248 621 907|\n|India|73 818 963|\n|Norway|5 986 254|\n\n</div>\n\n<br>How many more registered vehicles are there in India than Australia and Norway combined?\n"},{"varval":"sm_nogap Extra cars in India\n\n<div class=\"aligned\">\n\n>>||\n|-|\n|= 73 818 963 − (18 337 225 + 5 986 254)|\n|= 49 495 484|\n\n</div>"}]},{"vars":[{"varval":"The number of registered vehicles in five countries are recorded in the table below.\n\n<br>\n\n<div class=\"outline\">\n\n>>> |Country|Registered Vehicles|\n|:-:|:-:|\n|Australia|18 337 225|\n|USA|263 520 104|\n|China|248 621 907|\n|India|73 818 963|\n|Norway|5 986 254|\n\n</div>\n\n<br>How many more registered vehicles are there in the USA than China and Norway combined?\n"},{"varval":"sm_nogap Extra cars in USA\n\n<div class=\"aligned\">\n\n>>||\n|-|\n|= 263 520 104 − (248 621 907 + 5 986 254)|\n|= {{{correctAnswer0}}}|\n\n</div>\n"}]},{"vars":[{"varval":"The number of registered vehicles in five countries are recorded in the table below.\n\n<br>\n\n<div class=\"outline\">\n\n>>> |Country|Registered Vehicles|\n|:-:|:-:|\n|Australia|18 337 225|\n|USA|263 520 104|\n|China|248 621 907|\n|India|73 818 963|\n|Norway|5 986 254|\n\n</div>\n\n<br>How many more registered vehicles are there in China than India and Norway combined?\n"},{"varval":"sm_nogap Extra cars in China\n\n<div class=\"aligned\">\n\n>>||\n|-|\n|= 248 621 907 − (73 818 963 + 5 986 254)|\n|= 168 816 690|\n\n</div>"}]},{"vars":[{"varval":"The number of registered vehicles in five countries are recorded in the table below.\n\n<br>\n\n<div class=\"outline\">\n\n>>> |Country|Registered Vehicles|\n|:-:|:-:|\n|Australia|18 337 225|\n|USA|263 520 104|\n|China|248 621 907|\n|India|73 818 963|\n|Norway|5 986 254|\n\n</div>\n\n<br>How many more registered vehicles are there in the USA than India and Australia combined?\n"},{"varval":"sm_nogap Extra cars in USA\n\n<div class=\"aligned\">\n\n>>||\n|-|\n|= 263 520 104 − (73 818 963 + 18 337 225)|\n|= 171 363 916|\n\n</div>"}]}]

  629. <div class="sm_mode"> The lowest point in Asia is Ayoling Lake at &nbsp;−154 metres, which is below sea level. Mount Everest, also in Asia, is 8848 metres above sea level. What is the difference, in metres, between the highest and lowest points in Asia? </div>

    [{"vars":null}]

  630. (sometimes my browser can look differrent to yours, but this is what I see on Safari) var1 and var2 ... the width of the borders look inconsistent - in var1, looks like some shadow is on certain lines - in var2, some borders much bolder than other lines and second square not lined up 1st square in last two pattern images Have fixed var 1 and var 2. I've learned a lot about images since I made these.

    <div class="sm_mode"> {{{question}}} </div>

    [{"vars":[{"varval":"Matilda created a shape using equilateral triangles and squares.\r\n\r\n\nShe begins by using a triangle with side length 1 cm.\r\n\r\n\nShe then adds a square, then another triangle, then a square etc... until she has a shape with a perimeter of 24 cm.\n\n<br>\n\nsm_img https://teacher.smartermaths.com.au/wp-content/uploads/2016/12/NAP-H4-NC31.png 600 indent vpad\n\n<br>What is the total number of triangles and squares combined that Matilda has used?\n"},{"varval":"Perimeter of 1 triangle = 3 cm.\r\n\nAdd a square, perimeter increases by 2 cm\r\n\nAdd a triangle, perimeter increases by 1 cm\r\n \r\n\r\n$\\Rightarrow$ Total perimeter increases as follows:\r\n>>$3,5,6,8,9,11,12,14,15,17,18,20,21,23,24$\r\n\n<br>\n\n\r\n\r\n$\\therefore$ {{{correctAnswer0}}} triangles and squares (combined) are used.\n"}]},{"vars":[{"varval":"Bella created a shape using squares and regular pentagons.\r\n\r\n\nShe begins by using a square with side length 1 cm.\r\n\r\n\nShe then adds a pentagon, then another square, then a pentagon etc... until she has a shape with a perimeter of 22 cm.\n\n<br>\n\nsm_img https://teacher.smartermaths.com.au/wp-content/uploads/2022/08/Algebra_NAPX-H4-NC31-SA_v1.svg 500 indent vpad\n\n<br>What is the total number of squares and pentagons combined that Bella has used?\n"},{"varval":"Perimeter of 1 square = 4 cm.\r\n\nAdd a pentagon, perimeter increases by 3 cm\r\n\nAdd a square, perimeter increases by 2 cm\r\n \r\n\r\n$\\Rightarrow$ Total perimeter increases as follows:\r\n>>$4,7,9,12,14,17,19,22$\r\n\n<br>\n\n\r\n\r\n$\\therefore$ {{{correctAnswer0}}} squares and pentagons (combined) are used.\n"}]},{"vars":[{"varval":"Barney created a shape using squares and equilateral triangles.\r\n\r\n\nHe begins by using a square with side length 1 cm.\r\n\r\n\nHe then adds two triangles, then another square, then two triangles etc... until he has a shape with a perimeter of 22 cm.\n\n<br>\n\nsm_img https://teacher.smartermaths.com.au/wp-content/uploads/2022/08/Algebra_NAPX-H4-NC31-SA_v2.svg 500 indent vpad\n\n<br>What is the total number of squares and triangles combined that Barney has used?"},{"varval":"Perimeter of 1 square = 4 cm.\r\n\nAdd 2 triangles, perimeter increases by 2 cm\r\n\nAdd a square, perimeter increases by 2 cm\r\n \r\n\r\n$\\Rightarrow$ Total perimeter increases as follows:\r\n>>$4,6,8,10,12,14,16,18,20,22$\n<br>\n\n<br>\n<div class=\"sm-table col1-color2\">\n\n>>| Perimeter | 4 |6|8|...|20|22|\n|:-:|:-:|:-:|:-:|:-:|:-:|:-:|\n| Total squares and triangles| 1|3|4|...|13|15|\n\n</div>\n\n\r\n\n<br>\n\n\r\n\r\n$\\therefore$ {{{correctAnswer0}}} squares and triangles (combined) are used.\n"}]},{"vars":[{"varval":"Asti created a shape using regular hexagons and squares.\r\n\r\n\nShe begins by using a regular hexagon with side length 1 cm.\r\n\r\n\nShe then adds a square, then another hexagon, then a square etc... until she has a shape with a perimeter of 36 cm.\n\n<br>\n\nsm_img https://teacher.smartermaths.com.au/wp-content/uploads/2022/08/Algebra_J-NAPX-H4-NC31-SA_3.svg 600 indent vpad\n\n<br>What is the total number of hexagons and squares combined that Asti has used?\n"},{"varval":"Perimeter of 1 hexagon = 6 cm.\r\n\nAdd a square, perimeter increases by 2 cm\r\n\nAdd a hexagon, perimeter increases by 4 cm\r\n \r\n\r\n$\\Rightarrow$ Total perimeter increases as follows:\r\n>>$6,8,12,14,18,20,24,26,30,32,36$\r\n\n<br>\n\n\r\n\r\n$\\therefore$ {{{correctAnswer0}}} hexagons and squares (combined) are used.\n"}]},{"vars":[{"varval":"Carter created a shape using regular pentagons and squares.\r\n\r\n\nHe begins by using a regular pentagon with side length 1 cm.\r\n\r\n\nHe then adds a square, then another regular pentagon, then a square etc... until he has a shape with a perimeter of 40 cm.\n\n<br>\n\nsm_img https://teacher.smartermaths.com.au/wp-content/uploads/2022/08/Algebra_J-NAPX-H4-NC31-SA_4.svg 660 indent vpad\n\n<br>What is the total number of regular pentagons and squares combined that Carter has used?"},{"varval":"Perimeter of 1 pentagon = 5 cm.\r\n\nAdd a square, perimeter increases by 2 cm\r\n\nAdd a pentagon, perimeter increases by 3 cm\r\n\n \r\n\r\n$\\Rightarrow$ Total perimeter increases as follows:\n\r\n\n\n\n>>$5,7,10,12,15,17,20,22,25,27,30,32,35,37,40,42$\r\n\n<br>\n\n\r\n\r\n$\\therefore$ {{{correctAnswer0}}} pentagons and squares (combined) are used.\n\n\n"}]}]

  631. <div class="sm_mode"> {{{question}}} </div>

    [{"vars":[{"varval":"Rearrange the equation $\\ 6\\large y$ + 7 = $\\large x$ &nbsp;so that $\\large y$ is the subject.\r\n\r\nWhich of these correctly gives $\\large y$ as the subject?\n"},{"varval":"<div class=\"aligned\">\r\n\r\n| | |\r\n| ------------: | ---------- |\r\n| $6\\large y$ + 7 | \\= $\\large x$ |\n| $6\\large y$ | \\= $\\large x$ $-$ 7 |\r\n| $\\large y$ | \\= $\\dfrac{x-7}{6}$ |\r\n\r\n</div>"}]},{"vars":[{"varval":"Rearrange the equation $\\ 3\\large m$ + 4 = $\\large n$ &nbsp;so that $\\large m$ is the subject.\r\n\r\nWhich of these correctly gives $\\large m$ as the subject?\n"},{"varval":"<div class=\"aligned\">\r\n\r\n| | |\r\n| ------------: | ---------- |\r\n| $3\\large m$ + 4 | \\= $\\large n$ |\n| $3\\large m$ | \\= $\\large n$ $-$ 4 |\r\n| $\\large m$ | \\= $\\dfrac{n-4}{3}$ |\r\n\r\n</div>"}]},{"vars":[{"varval":"Rearrange the equation $\\ 4\\large y$ $-$ 3 = $\\large x$ &nbsp;so that $\\large y$ is the subject.\r\n\r\nWhich of these correctly gives $\\large y$ as the subject?\n"},{"varval":"<div class=\"aligned\">\r\n\r\n| | |\r\n| ------------: | ---------- |\r\n| $4\\large y$ $-$ 3 | \\= $\\large x$ |\n| $4\\large y$ | \\= $\\large x$ + 3 |\r\n| $\\large y$ | \\= $\\dfrac{x + 3}{4}$ |\r\n\r\n</div>"}]},{"vars":[{"varval":"Rearrange the equation $\\ 5\\large y$ $-$ 7 = $2\\large x$ &nbsp;so that $\\large y$ is the subject.\r\n\r\nWhich of these correctly gives $\\large y$ as the subject?\n"},{"varval":"<div class=\"aligned\">\r\n\r\n| | |\r\n| ------------: | ---------- |\r\n| $5\\large y$ $-$ 7 | \\= $2\\large x$ |\n| $5\\large y$ | \\= $2\\large x$ + 7 |\r\n| $\\large y$ | \\= $\\dfrac{2x+7}{5}$ |\r\n\r\n</div>"}]},{"vars":[{"varval":"Rearrange the equation $\\ 6\\large y$ + $\\large x$ = 5 &nbsp;so that $\\large y$ is the subject.\r\n\r\nWhich of these correctly gives $\\large y$ as the subject?\n"},{"varval":"<div class=\"aligned\">\r\n\r\n| | |\r\n| ------------: | ---------- |\r\n| $\\ 6\\large y$ + $\\large x$ | \\= 5 |\n| $6\\large y$ | \\= 5 $-$ $\\large x$ |\r\n| $\\large y$ | \\= $\\dfrac{5-\\large x}{6}$ |\r\n\r\n</div>"}]},{"vars":[{"varval":"Rearrange the equation $\\ 6\\large y$ $-$ $\\large x$ $-$ 2 = 0 &nbsp;so that $\\large y$ is the subject.\r\n\r\nWhich of these correctly gives $\\large y$ as the subject?\n"},{"varval":"<div class=\"aligned\">\r\n\r\n| | |\r\n| ------------: | ---------- |\r\n| $\\ 6\\large y$ $-$ $\\large x$ $-$ 2 | \\= 0 |\n| $6\\large y$ | \\= $\\large x$ $+$ 2 |\r\n| $\\large y$ | \\= $\\dfrac{x+2}{6}$ |\r\n\r\n</div>"}]}]

  632. <div class="sm_mode"> {{{question}}} </div>

    [{"vars":[{"varval":"Rex is a lettuce grower and the cost of fertilising different crop areas is shown in the table below.\n\n<br>\n\n<div class=\"sm-table col1-color1\">\n\n>>| size of crop area (m$^2$) | 20 |60|80|90|100|\n|:-:|:-:|:-:|:-:|:-:|:-:|\n| cost of fertilising ($) | 1800|5400|7200|8100|9000\n\n</div>\n\n\n\n<br>Complete the rule that agrees with the values in the table."},{"varval":"Taking the values in the 1st column:\n\n<div class=\"aligned\">\r\n\r\n| | |\r\n| ------------: | ---------- |\r\n| Cost of fertilising\t | \\= cost of 1 m² × area of crop |\n| | \\= $\\dfrac{1800}{20}$ × area of crop |\r\n| | \\= {{{correctAnswer0}}} × area of crop |\r\n\r\n</div>\r"}]},{"vars":[{"varval":"Elie grows saffron commercially. The cost of harvesting different crop areas is summarised in the table below.\n\n<br>\n\n<div class=\"sm-table col1-color3\">\n\n>>| size of crop area (m$^2$) | 30 |50|80|100|\n|:-:|:-:|:-:|:-:|:-:|\n| cost of harvesting ($) | 2550|4250|6800|8500\n\n</div>\n\n\n\n<br>Complete the rule that agrees with the values in the table."},{"varval":"Taking the values in the 1st column:\n\n<div class=\"aligned\">\r\n\r\n| | |\r\n| ------------: | ---------- |\r\n| Cost of harvesting\t | \\= cost of 1 m² × area of crop |\n| | \\= $\\dfrac{2550}{30}$ × area of crop |\r\n| | \\= {{{correctAnswer0}}} × area of crop |\r\n\r\n</div>\r"}]},{"vars":[{"varval":"Denise grows pumpkins and recorded the cost of fertilising different crop areas in the table below.\n\n<br>\n\n<div class=\"sm-table col1-color8\">\n\n>>| size of crop area (m$^2$) | 20|50|90|120|\n|:-:|:-:|:-:|:-:|:-:|\n| cost of fertilising ($) |1300|3250|5850|7800|\n\n</div>\n\n\n\n<br>Complete the rule that agrees with the values in the table."},{"varval":"Taking the values in the 1st column:\n\n<div class=\"aligned\">\r\n\r\n| | |\r\n| ------------: | ---------- |\r\n| Cost of fertilising | \\= cost of 1 m² × area of crop |\n| | \\= $\\dfrac{1300}{20}$ × area of crop |\r\n| | \\= {{{correctAnswer0}}} × area of crop |\r\n\r\n</div>\r"}]},{"vars":[{"varval":"Bertie is a market gardener and the cost of cultivating different crop areas is shown in the table below.\n\n<br>\n\n<div class=\"sm-table col1-color5\">\n\n>>| size of crop area (m$^2$) | 15 |30|50|80|110|\n|:-:|:-:|:-:|:-:|:-:|:-:|\n| cost of cultivating ($) | 1530|3060|5100|8160|11220\n\n</div>\n\n\n\n<br>Complete the rule that agrees with the values in the table."},{"varval":"Taking the values in the 1st column:\n\n<div class=\"aligned\">\r\n\r\n| | |\r\n| ------------: | ---------- |\r\n| Cost of cultivating | \\= cost of 1 m² × area of crop |\n| | \\= $\\dfrac{1530}{15}$ × area of crop |\r\n| | \\= {{{correctAnswer0}}} × area of crop |\r\n\r\n</div>"}]},{"vars":[{"varval":"Genie is a macadamia nut grower and the cost of harvesting different crop areas is shown in the table below.\n\n<br>\n\n<div class=\"sm-table col1-color7\">\n\n>>| size of crop area (m$^2$) | 25 |40|65|90|\n|:-:|:-:|:-:|:-:|:-:|\n| cost of harvesting ($) | 4125|6600|10 725|14 850|\n\n</div>\n\n\n\n<br>Complete the rule that agrees with the values in the table."},{"varval":"Taking the values in the 1st column:\n\n<div class=\"aligned\">\r\n\r\n| | |\r\n| ------------: | ---------- |\r\n| Cost of harvesting\t | \\= cost of 1 m² × area of crop |\n| | \\= $\\dfrac{4125}{25}$ × area of crop |\r\n| | \\= {{{correctAnswer0}}} × area of crop |\r\n\r\n</div>"}]},{"vars":[{"varval":"Bella is a blueberry grower and the cost of fertilising different crop areas is shown in the table below.\n\n<br>\n\n<div class=\"sm-table col1-color8\">\n\n>>| size of crop area (m$^2$) | 80 |110|150|160|\n|:-:|:-:|:-:|:-:|:-:|\n| cost of fertilising ($) | 1680|2310|3150|3360|\n\n</div>\n\n\n\n<br>Complete the rule that agrees with the values in the table."},{"varval":"Taking the values in the 1st column:\n\n<div class=\"aligned\">\r\n\r\n| | |\r\n| ------------: | ---------- |\r\n| Cost of fertilising\t | \\= cost of 1 m² × area of crop |\n| | \\= $\\dfrac{1680}{80}$ × area of crop |\r\n| | \\= {{{correctAnswer0}}} × area of crop |\r\n\r\n</div>"}]}]

  633. <div class="sm_mode"> {{{question}}} </div>

    [{"vars":[{"varval":"Paul and Simon are saving money so they can visit their grandmother on a holiday.\r\n\r\nPaul has $200 and plans to save $50 each week.\r\n\r\nSimon has $300 and plans to save $25 each week.\r\n\r\nAfter how many weeks will Paul and Simon have saved the same amount?"},{"varval":"After $\\large w$ weeks, \r\n\nPaul has saved: $200+50\\large w$\r\n\nSimon has saved: $300+25\\large w$\n\nsm_nogap Savings are equal when:\n\n<div class=\"aligned\">\r\n\r\n| | |\r\n| ------------: | ---------- |\r\n| $200+50\\large w$ | \\= $300+25\\large w$ |\n| $25\\large w$ | \\= 100 |\r\n| $\\large w$ | \\= {{{correctAnswer0}}} |\r\n\r\n</div>\r\n\n<br>\n\n$\\therefore$ Amounts are equal after {{{correctAnswer0}}} weeks."}]},{"vars":[{"varval":"Bracken and Fern are saving money so they can build a granny flat in the backyard.\r\n\r\nBracken has $1500 and plans to save $450 each week.\r\n\r\nFern has $4500 and plans to save $250 each week.\r\n\r\nAfter how many weeks will Bracken and Fern have saved the same amount?"},{"varval":"After $\\large w$ weeks, \r\n\nBracken has saved: $1500+450\\large w$\r\n\nFern has saved: $4500+250\\large w$\n\nsm_nogap Savings are equal when:\n\n<div class=\"aligned\">\r\n\r\n| | |\r\n| ------------: | ---------- |\r\n| $1500+450\\large w$ | \\= $4500+250\\large w$ |\n| $200\\large w$ | \\= 3000 |\r\n| $\\large w$ | \\= {{{correctAnswer0}}} |\r\n\r\n</div>\r\n\n<br>\n\n$\\therefore$ Amounts are equal after {{{correctAnswer0}}} weeks."}]},{"vars":[{"varval":"Ribena and Oban are saving money so they can stock their greenhouse with plants.\r\n\r\nRibena has $550 and plans to save $25 each week.\r\n\r\nOban has $750 and plans to save $15 each week.\r\n\r\nAfter how many weeks will Ribena and Oban have saved the same amount?"},{"varval":"After $\\large w$ weeks, \r\n\nRibena has saved: $550+25\\large w$\r\n\nOban has saved: $750+15\\large w$\n\nsm_nogap Savings are equal when:\n\n<div class=\"aligned\">\r\n\r\n| | |\r\n| ------------: | ---------- |\r\n| $550+25\\large w$ | \\= $750+15\\large w$ |\n| $10\\large w$ | \\= 200 |\r\n| $\\large w$ | \\= {{{correctAnswer0}}} |\r\n\r\n</div>\r\n\n<br>\n\n$\\therefore$ Amounts are equal after {{{correctAnswer0}}} weeks."}]},{"vars":[{"varval":"Meg and Ryan are saving money for their wedding next year.\r\n\r\nMeg has $2600 and plans to save $120 each week.\r\n\r\nRyan has $3800 and plans to save $100 each week.\r\n\r\nAfter how many weeks will Meg and Ryan have saved the same amount?"},{"varval":"After $\\large w$ weeks, \r\n\nMeg has saved: $2600+120\\large w$\r\n\nRyan has saved: $3800+100\\large w$\n\nsm_nogap Savings are equal when:\n\n<div class=\"aligned\">\r\n\r\n| | |\r\n| ------------: | ---------- |\r\n| $2600+120\\large w$ | \\= $3800+100\\large w$ |\n| $20\\large w$ | \\= 1200 |\r\n| $\\large w$ | \\= {{{correctAnswer0}}} |\r\n\r\n</div>\r\n\n<br>\n\n$\\therefore$ Amounts are equal after {{{correctAnswer0}}} weeks."}]},{"vars":[{"varval":"Ketut and Made are saving money so they can stock their new online handmade jewellery store.\r\n\r\nKetut has $540 and plans to save $20 each week.\r\n\r\nMade has $660 and plans to save $15 each week.\r\n\r\nAfter how many weeks will Ketut and Made have saved the same amount?"},{"varval":"After $\\large w$ weeks, \r\n\nKetut has saved: $540+20\\large w$\r\n\nMade has saved: $660+15\\large w$\n\nsm_nogap Savings are equal when:\n\n<div class=\"aligned\">\r\n\r\n| | |\r\n| ------------: | ---------- |\r\n| $540+20\\large w$ | \\= $660+15\\large w$ |\n| $5\\large w$ | \\= 120 |\r\n| $\\large w$ | \\= {{{correctAnswer0}}} |\r\n\r\n</div>\r\n\n<br>\n\n$\\therefore$ Amounts are equal after {{{correctAnswer0}}} weeks."}]},{"vars":[{"varval":"Diego and Santiago are saving money so they can buy a fishing boat.\r\n\r\nDiego has $4580 and plans to save $55 each week.\r\n\r\nSantiago has $5640 and plans to save $45 each week.\r\n\r\nAfter how many weeks will Diego and Santiago have saved the same amount?"},{"varval":"After $\\large w$ weeks, \r\n\nDiego has saved: $4580+55\\large w$\r\n\nSantiago has saved: $5640+45\\large w$\n\nsm_nogap Savings are equal when:\n\n<div class=\"aligned\">\r\n\r\n| | |\r\n| ------------: | ---------- |\r\n| $4580+55\\large w$ | \\= $5640+45\\large w$ |\n| $10\\large w$ | \\= 1060 |\r\n| $\\large w$ | \\= {{{correctAnswer0}}} |\r\n\r\n</div>\r\n\n<br>\n\n$\\therefore$ Amounts are equal after {{{correctAnswer0}}} weeks."}]}]

  634. <div class="sm_mode"> {{{question}}} </div>

    [{"vars":[{"varval":"Which of the following has the same value as $\\ \\large a$$^{−2}$?\n"},{"varval":">{{{correctAnswer}}}"}]},{"vars":[{"varval":"Which of the following has the same value as $2 \\large a$$^{−1}$?\n"},{"varval":">{{{correctAnswer}}}"}]},{"vars":[{"varval":"Which of the following has the same value as $\\ \\large b$$^{−3}$?"},{"varval":"{{{correctAnswer}}}"}]},{"vars":[{"varval":"Which of the following has the same value as $\\ 5\\large a$$^{−4}$?"},{"varval":"{{{correctAnswer}}}"}]},{"vars":[{"varval":"Which of the following has the same value as $-4\\large a$$^{−2}$?\n"},{"varval":"{{{correctAnswer}}}"}]},{"vars":[{"varval":"Which of the following has the same value as $\\\\$ $\\dfrac{2}{3}\\large a$$^{−2}$?\n"},{"varval":"{{{correctAnswer}}}"}]}]

  635. <div class="sm_mode"> Jen drew the line $\ \large y$ = 5 $−\ \large x$ on a grid. She then drew $\ \large y$ = $\large x$ $−\ 1$ on the same grid. What are the coordinates of the intersection point of these two lines? </div>

    [{"vars":null}]

  636. <div class="sm_mode"> {{{question}}} </div>

    [{"vars":[{"varval":"Liz and Elena baked some cookies.\r\n\r\nThe total number of cookies they baked is 35.\r\n\r\nLiz baked 11 more cookies than Elena.\r\n\r\nHow many cookies does Liz bake?"},{"varval":"Solution 1 (Trial and error)\r\n\nIf Liz baked 19 ⇒ Elena baked 8 (27 total)\r\n\nIf Liz baked 21 ⇒ Elena baked 10 (31 total)\r\n\nIf Liz baked 23 ⇒ Elena baked 12 (35 total)\r$\\checkmark$\n\n$\\therefore$ Liz baked {{{correctAnswer0}}} cookies.\r\n \r\n\r\n<br>\n\nSolution 2 (Algebra)\r\n\nLet $\\ \\large n$ = number of cookies Elena baked\n\n$\\large n$ + 11 = number of cookies Liz baked\n\n<div class=\"aligned\">\r\n\r\n| | |\r\n| ------------: | ---------- |\r\n| $\\large n$ + $\\large n$ + 11 | \\= 35 |\n| $2\\large n$ | \\= 24 |\n| $\\large n$ | \\= 12 |\r\n\r\n</div>\r\n\n<br>$\\therefore$ Liz bakes = 12 + 11 = {{{correctAnswer0}}} {{{suffix0}}} \n"}]},{"vars":[{"varval":"Kane and Abel spread some topsoil over their lawn.\r\n\r\nThe total number of cubic metres of topsoil they spread is 17.\r\n\r\nKane spread 5 more cubic metres than Abel.\r\n\r\nHow many cubic metres does Kane spread?"},{"varval":"Solution 1 (Trial and error)\r\n\nIf Kane spread 8 ⇒ Abel spread 3 (11 total)\r\n\nIf Kane spread 10 ⇒ Abel spread 5 (15 total)\r\n\nIf Kane spread 11 ⇒ Abel spread 6 (17 total)\r$\\checkmark$\n\n$\\therefore$ Kane spread {{{correctAnswer0}}} metres.\r\n \r\n\r\n<br>\n\nSolution 2 (Algebra)\r\n\nLet $\\ \\large n$ = number of metres Abel spread\n\n$\\large n$ + 5 = number of metres Kane spread\n\n<div class=\"aligned\">\r\n\r\n| | |\r\n| ------------: | ---------- |\r\n| $\\large n$ + $\\large n$ + 5 | \\= 17 |\n| $2\\large n$ | \\= 12 |\n| $\\large n$ | \\= 6 |\r\n\r\n</div>\r\n\n<br> $\\therefore$ Kane spread = 6 + 5 = {{{correctAnswer0}}} {{{suffix0}}}\n"}]},{"vars":[{"varval":"Gunther and Gertrude are filling jars with marmalade.\r\n\r\nThe total number of jars of marmalade they have filled is 68.\r\n\r\nGunther fills 16 more jars than Gertrude.\r\n\r\nHow many jars does Gunther fill?"},{"varval":"Solution 1 (Trial and error)\r\n\nIf Gunther fills 36 ⇒ Gertrude fills 20 (56 total)\r\n\nIf Gunther fills 40 ⇒ Gertrude fills 24 (64 total)\r\n\nIf Gunther fills 42 ⇒ Gertrude fills 26 (68 total) $\\checkmark$\n\n$\\therefore$ Gunther fills {{{correctAnswer0}}} {{{suffix0}}}.\r\n \r\n\r\n<br>\n\nSolution 2 (Algebra)\r\n\nLet $\\ \\large n$ = number of jars Gertrude fills\n\n$\\large n$ + 16 = number of jars Gunther fills\n\n<div class=\"aligned\">\r\n\r\n>| | |\r\n| ------------: | ---------- |\r\n| $\\large n$ + $\\large n$ + 16 | \\= 68 |\n| $2\\large n$ | \\= 52 |\n| $\\large n$ | \\= 26 |\r\n\r\n</div>\n\n<br>$\\therefore$ Gunther fills = 26 + 16 = {{{correctAnswer0}}} {{{suffix0}}} |\r"}]},{"vars":[{"varval":"Johnny and Evie are picking strawberries.\r\n\r\nThe total number of buckets of strawberries they have picked is 104.\r\n\r\nJohnny picks 42 more buckets than Evie.\r\n\r\nHow many buckets does Johnny pick?"},{"varval":"Solution 1 (Trial and error)\r\n\nIf Johnny picks 70 ⇒ Evie picks 28 (98 total)\r\n\nIf Johnny picks 72 ⇒ Evie picks 30 (102 total)\r\n\nIf Johnny picks 73 ⇒ Evie picks 31 (104 total)\r$\\checkmark$\n\n$\\therefore$ Johnny picks {{{correctAnswer0}}} {{{suffix0}}}.\r\n \r\n\r\n<br>\n\nSolution 2 (Algebra)\r\n\nLet $\\ \\large n$ = number of buckets Evie picks \n\n$\\large n$ + 42 = number of buckets Johnny picks \n\n<div class=\"aligned\">\r\n\r\n>| | |\r\n| ------------: | ---------- |\r\n| $\\large n$ + $\\large n$ + 42 | \\= 104 |\n| $2\\large n$ | \\= 62 |\n| $\\large n$ | \\= 31 |\r\n\r\n</div>\n\n<br>$\\therefore$ Johnny picks = 31 + 42 = {{{correctAnswer0}}} {{{suffix0}}}"}]},{"vars":[{"varval":"Charlie and Linus are counting sheep.\r\n\r\nThe total number of sheep they have counted is 741.\r\n\r\nCharlie counts 181 more sheep than Linus.\r\n\r\nHow many sheep does Charlie count?"},{"varval":"Solution 1 (Trial and error)\r\n\nIf Charlie counts 481 ⇒ Linus counts 300 (781 total)\r\n\nIf Charlie counts 471 ⇒ Linus counts 290 (761 total)\r\n\nIf Charlie counts 461 ⇒ Linus counts 280 (741 total)\r$\\checkmark$\n\n$\\therefore$ Charlie counts {{{correctAnswer0}}} {{{suffix0}}}.\r\n \r\n\r\n<br>\n\nSolution 2 (Algebra)\r\n\nLet $\\ \\large n$ = number of sheep Linus counts \n\n$\\large n$ + 181 = number of sheep Charlie counts \n\n<div class=\"aligned\">\r\n\r\n| | |\r\n| ------------: | ---------- |\r\n| $\\large n$ + $\\large n$ + 181 | \\= 741 |\n| $2\\large n$ | \\= 560 |\n| $\\large n$ | \\= 280 |\r\n\r\n</div>\n\n<br>$\\therefore$ Charlie counts = 280 + 181 = {{{correctAnswer0}}} {{{suffix0}}}"}]},{"vars":[{"varval":"Dakota and Duke are mustering cattle.\r\n\r\nThe total number of cattle they have mustered is 2500.\r\n\r\nDakota musters 420 more cattle than Duke.\r\n\r\nHow many cattle does Dakota muster?"},{"varval":"Solution 1 (Trial and error)\r\n\nIf Dakota musters 1400 ⇒ Duke musters 980 (2380 total)\r\n\nIf Dakota musters 1430 ⇒ Duke musters 1010 (2440 total)\r\n\nIf Dakota musters 1460 ⇒ Duke musters 1040 (2500 total)\r\n\n$\\therefore$ Dakota musters {{{correctAnswer0}}} {{{suffix0}}}.\r\n \r\n\r\n<br>\n\nSolution 2 (Algebra)\r\n\nLet $\\ \\large n$ = number of cattle Duke musters \n\n$\\large n$ + 420 = number of cattle Dakota musters \n\n<div class=\"aligned\">\r\n\r\n| | |\r\n| ------------: | ---------- |\r\n| $\\large n$ + $\\large n$ + 420 | \\= 2500 |\n| $2\\large n$ | \\= 2080 |\n| $\\large n$ | \\= 1040 |\r\n\r\n</div>\n\n<br>$\\therefore$ Dakota musters = 1040 + 420 = {{{correctAnswer0}}} {{{suffix0}}}"}]}]

  637. Var2 ... graph goes slightly below the (50,300) intersection used Var5 ... graph "x"-axis doesn't have the 5,10,15 etc... aligning with the vertical lines (this may be Safari specific) Have replaced the above graphs with the correct ones Take #3: I redid Var0 so it was consistent with the other graphs I removed all the dots at the beginning of the lines I moved the y-axis label "Charge ($)" further away from the numbers

    <div class="sm_mode"> {{{question}}} </div>

    [{"vars":[{"varval":"Indigo is a lawyer who charges $250 to look at a case and then a fixed price for each minute she spends on the case.\r\n\r\n\nThe graph below shows Indigo's charge based on the number of minutes she spends on the case.\n\n<br>\n\nsm_img https://teacher.smartermaths.com.au/wp-content/uploads/2022/08/Algebra_NAPX-J4-CA29-SA_v0_1.svg 300 indent vpad\n\n<br>How much will Indigo charge for a case that takes 60 minutes of her time?\n"},{"varval":"Initial charge = $250\r\n\nFrom the graph,\r\n\n⇒ 50 minute case costs $400\n\nCost per minute =$\\dfrac{400 - 250}{50}$ = $3\r\n\nsm_nogap $\\therefore$ Cost of 60 minute case\n\n>> $=250+60×3$\r\n\n>> = {{{prefix0}}}{{{correctAnswer0}}}\n"}]},{"vars":[{"varval":"Brendon is a lawyer who charges $150 to look at a case and then a fixed price for each minute he spends on the case.\r\n\r\n\nThe graph below shows Brendon's charge based on the number of minutes he spends on the case.\n\n<br>\n\nsm_img https://teacher.smartermaths.com.au/wp-content/uploads/2022/08/Algebra_NAPX-J4-CA29-SA_v1_1.svg 320 indent vpad\n\n<br>How much will Brendon charge for a case that takes 70 minutes of his time?\n"},{"varval":"Initial charge = $150\r\n\nFrom the graph,\r\n\n⇒ 50 minute case costs $300\r\n\n\nCost per minute =$\\dfrac{300 - 150}{50}$ = $3\r\n\nsm_nogap $\\therefore$ Cost of 70 minute case\n\n>> $=150+70×3$\r\n\n>> = {{{prefix0}}}{{{correctAnswer0}}}\n"}]},{"vars":[{"varval":"Tahlia is a paralegal who charges $50 to look at a case and then a fixed price for each minute she spends on the case.\r\n\r\n\nThe graph below shows Tahlia's charge based on the number of minutes she spends on the case.\n\n\nsm_img https://teacher.smartermaths.com.au/wp-content/uploads/2022/08/Algebra_NAPX-J4-CA29-SA_v2_2.svg 280 indent vpad\n\n<br>How much will Tahlia charge for a case that takes 80 minutes of her time?"},{"varval":"Initial charge = $50\r\n\nFrom the graph,\r\n\n⇒ 50 minute case costs $300\r\n\n<br>\n\nCost per minute =$\\dfrac{300 - 50}{50}$ = $5\r\n\nsm_nogap $\\therefore$ Cost of 80 minute case\n\n>> $=50+80×5$\r\n\n>> = {{{prefix0}}}{{{correctAnswer0}}}\n"}]},{"vars":[{"varval":"Geordie wants to hire a stand-up paddle board. The hire company charges an hourly rate plus a flat fee of $25 for insurance.\r\n\r\n\nThe graph below shows the cost of board hire based on the number of hours the board is hired.\n\n<br>\n\nsm_img https://teacher.smartermaths.com.au/wp-content/uploads/2022/08/Algebra_NAPX-J4-CA29-SA_v3_1.svg 350 indent vpad\n\n<br>How much will Geordie pay for a stand-up paddle board hire of 5 hours?"},{"varval":"Initial fee = $25\r\n\nFrom the graph,\r\n\n⇒ 1 hour hire costs $40\r\n\nCost per hour = 40 $-$ 25 = $15\r\n\nsm_nogap $\\therefore$ Cost of 5 hour hire\n\n>> $=25+15×5$\r\n\n>> = {{{prefix0}}}{{{correctAnswer0}}}\n"}]},{"vars":[{"varval":"MaxiCab taxis charge a hire fee of $4 plus a charge per kilometre travelled.\r\n\r\n\nThe graph below shows the total charge of a ride based on the number of kilometres travelled.\n\n<br>\n\nsm_img https://teacher.smartermaths.com.au/wp-content/uploads/2022/08/Algebra_NAPX-J4-CA29-SA_v4_1.svg 320 indent vpad\n\n<br>How much will Preshan pay if she hires a MaxiCab to take her a distance of 35 kilometres from the airport to her home?\n"},{"varval":"Hire fee = $4\r\n\nFrom the graph,\r\n\n⇒ 10 kilometre trip costs $24\r\n\nCost per kilometre =$\\dfrac{24 - 4}{10}$ = $2\r\n\nsm_nogap $\\therefore$ Cost of 35 kilometre trip\n\n>> $=4+35×2$\r\n\n>> = {{{prefix0}}}{{{correctAnswer0}}}\n"}]},{"vars":[{"varval":"Mavis' Hire Cars charge a hire fee of $6 plus a charge per kilometre travelled.\r\n\r\n\nThe graph below shows the total charge of a ride based on the number of kilometres travelled.\n\n<br>\n\nsm_img https://teacher.smartermaths.com.au/wp-content/uploads/2022/08/Algebra_NAPX-J4-CA29-SA_v5_1.svg 340 indent vpad\n\n<br>How much will Mahlia be charged if she rents a car and returns it after travelling 30 kilometres?\n"},{"varval":"Initial charge = $6\r\n\nFrom the graph,\r\n\n⇒ 20 kilometre trip costs $36\r\n\nCost per kilometre =$\\dfrac{36 - 6}{20}$ = $1.50\r\n\nsm_nogap $\\therefore$ Cost of 30 kilometre trip\n\n>> $=6+30×1.50$\r\n\n>> = {{{prefix0}}}{{{correctAnswer0}}}\n"}]}]

  638. <div class="sm_mode"> Pearl uses matchsticks to make a pattern of figures. <br> sm_img https://teacher.smartermaths.com.au/wp-content/uploads/2018/04/NAPX-H4-CA24-SA.svg 560 indent vpad <br>How many sticks will the Shape Number 6 have? </div>

    [{"vars":null}]

  639. question with image in between texts

    <div class="sm_mode"> {{{question}}} </div>

    [{"vars":[{"varval":"Herb made a number pattern using this rule:\n\n<div class=\"sm_img_inline\">\n\nnext number = previous number × ![](https://teacher.smartermaths.com.au/wp-content/uploads/2018/05/NAPX-H4-NC24_4.svg) $-$ ![](https://teacher.smartermaths.com.au/wp-content/uploads/2018/05/NAPX-H4-NC24_3.svg) \n\n![](https://teacher.smartermaths.com.au/wp-content/uploads/2018/05/NAPX-H4-NC24_4.svg) and ![](https://teacher.smartermaths.com.au/wp-content/uploads/2018/05/NAPX-H4-NC24_3.svg) are whole numbers.\n\n</div>\n\n<br>\n\nHis number pattern is:\n\n> > $4, 3, 1, −3, −11$\n\n<br>\n\nWhat is the next number in Herb's pattern?"},{"varval":"Using the rule and first 3 terms:\n\nLet $\\ \\large x$ = ![](https://teacher.smartermaths.com.au/wp-content/uploads/2018/05/NAPX-H4-NC24_4.svg) and $\\ \\large y$ = ![](https://teacher.smartermaths.com.au/wp-content/uploads/2018/05/NAPX-H4-NC24_3.svg)\n\n</div>\n\n<br>\n\n$3=4\\large x$ − $\\large y$\t… (1)\n\n$1=3\\large x$ − $\\large y$\t… (2)\n\nSubtract (1) $−$ (2),\n\n> > $\\large x$ = 2\n\n<br>\n\nSubstitute x = 2 into (1)\n\n> > $\\large y$ = 5\n\n<br>\n\n<div class=\"aligned\">\n\n| | |\n| ----------- | ---------------------- |\n| ∴ Next number | \\= $−11×2\\ −\\ 5$ |\n| | \\= {{{correctAnswer}}} |\n\n</div>"}]},{"vars":[{"varval":"Jen made a number pattern using this rule:\n\n<div class=\"sm_img_inline\">\n\nnext number = previous number × ![](https://teacher.smartermaths.com.au/wp-content/uploads/2018/05/NAPX-H4-NC24_4.svg) $-$ ![](https://teacher.smartermaths.com.au/wp-content/uploads/2018/05/NAPX-H4-NC24_3.svg) \n\n![](https://teacher.smartermaths.com.au/wp-content/uploads/2018/05/NAPX-H4-NC24_4.svg) and ![](https://teacher.smartermaths.com.au/wp-content/uploads/2018/05/NAPX-H4-NC24_3.svg) are whole numbers.\n\n</div>\n\n<br>\n\nHer number pattern is:\n\n> > $2, 1, -2, −11$\n\n<br>\n\nWhat is the next number in Jen's pattern?"},{"varval":"Using the rule and first 3 terms:\n\nLet $\\ \\large x$ = ![](https://teacher.smartermaths.com.au/wp-content/uploads/2018/05/NAPX-H4-NC24_4.svg) and $\\ \\large y$ = ![](https://teacher.smartermaths.com.au/wp-content/uploads/2018/05/NAPX-H4-NC24_3.svg)\n\n</div>\n\n<br>\n\n$1=2\\large x$ − $\\large y$\t… (1)\n\n$-2=1\\large x$ − $\\large y$\t… (2)\n\nSubtract (1) $−$ (2),\n\n> > $\\large x$ = 3\n\n<br>\n\nSubstitute x = 3 into (1)\n\n> > $\\large y$ = 5\n\n<br>\n\n<div class=\"aligned\">\n\n| | |\n| ----------- | ---------------------- |\n| ∴ Next number | \\= $−11×3\\ −\\ 5$ |\n| | \\= {{{correctAnswer}}} |\n\n</div>"}]}]

  640. <div class="sm_mode"> Moonjava stacks up identical cups. <br> sm_img https://teacher.smartermaths.com.au/wp-content/uploads/2018/06/NAPX-F4-CA28.svg 300 indent vpad <br>A stack of 3 cups is 18 cm high. A stack of 6 cups is 28.5 cm high. What height will a stack of 12 identical cups be? </div>

    [{"vars":null}]

  641. <div class="sm_mode"> {{{question}}} </div>

    [{"vars":[{"varval":"David is going on a golfing trip.\r\n\r\n\nHe wants to purchase balls before the trip, which are sold in boxes of a dozen balls.\r\n\r\n\nThe more boxes of balls that David buys, the less it costs per box, as shown in the table below.\n\n<div class=\"sm-table row1-color1 heading-color2\">\n\n>>Golf ball pricing\n\n>>| Number of boxes | Total Cost |\n|:-:|:-:|\n| 1 | $12.50 |\n| 2 | $24.50 |\n| 3 | $36.00 |\n|4|$47.00|\n\n</div>\n\n<br>If this price pattern continues, how much will it cost for 6 boxes of balls?"},{"varval":"Additional boxes reduce in cost by 50 cents.\n\n⇒ 4th box costs an extra $11.00\r\n\n⇒ 5th box costs an extra $10.50\r\n\n⇒ 6th box costs an extra $10.00\n\n<div class=\"aligned\">\r\n\r\n| | |\r\n| ------------- | ---------- |\r\n| $\\therefore$ Total cost of 6 boxes | \\= $47.00+10.50+10.00$ |\r\n| | \\= {{{prefix0}}}{{{correctAnswer0}}} |\r\n\r\n</div>\r"}]},{"vars":[{"varval":"Gary is purchasing tins of paint for a job he is working on.\n\r\n\nThe more tins of paint Gary buys, the less it costs per tin of paint, as shown in the table below.\n\n<div class=\"sm-table row1-color1 heading-color2\">\n\n>>Pricing of paint\n\n>>| Number of tins | Total Cost |\n|:-:|:-:|\n| 1 | $16.50 |\n| 2 | $32.50 |\n| 3 | $48.00 |\n|4|$63.00|\n\n</div>\n\n<br>If this price pattern continues, how much will it cost Gary to buy 6 tins of paint?"},{"varval":"Each extra tin of paint costs 50 cents\r less than the previous tin.\r\n\n⇒ 4th tin costs an extra $15.00\r\n\n⇒ 5th tin costs an extra $14.50\r\n\n⇒ 6th tin costs an extra $14.00\n\n<div class=\"aligned\">\r\n\r\n| | |\r\n| ------------- | ---------- |\r\n| $\\therefore$ Total cost of 6 tins | \\= $63.00 + 14.50 + 14.00$ |\r\n| | \\= {{{prefix0}}}{{{correctAnswer0}}} |\r\n\r\n</div>\r"}]}]

  642. <div class="sm_mode"> {{{question}}} </div>

    [{"vars":[{"varval":"Jeremy uses the formula below to estimate the population of Amazonian River Dolphins over a three year period.\n\n>Year 1 = 2000\r\n\n<br>\n\n>Year 2 = Year 1 + $\\dfrac{ \\text{Year 1}}{20}$\r\n\n<br>\n\n>Year 3 = Year 2 + $\\dfrac{ \\text{Year 2}}{20}$\n\n<br>\n\nEstimate the population of Amazonian River Dolphins in Year 3?"},{"varval":"Year 1 = 2000\r\n\nYear 2 = 2000 + $\\dfrac{2000}{20}$ = 2100\r\n\nYear 3 = 2100 + $\\dfrac{2100}{20}$ = {{{correctAnswer0}}}"}]},{"vars":[{"varval":"Will uses the formula below to estimate the population of koalas in a national park for three years after a bush fire.\n\n>Year 1 = 1200\r\n\n<br>\n\n>Year 2 = Year 1 + $\\dfrac{ \\text{Year 1}}{10}$\r\n\n<br>\n\n>Year 3 = Year 2 + $\\dfrac{ \\text{Year 2}}{10}$\n\n<br>\n\nEstimate the koala population in Year 3?"},{"varval":"Year 1 = 1200\n\nYear 2 = 1200 + $\\dfrac{1200}{10}$ = 1320\r\n\nYear 3 = 1320 + $\\dfrac{1320}{10}$ = {{{correctAnswer0}}}"}]}]

  643. <div class="sm_mode"> Pepper uses matchsticks to make a pattern of shapes, as shown in the table below. <br> sm_img https://teacher.smartermaths.com.au/wp-content/uploads/2018/06/NAPX-F4-NC21.svg 630 indent vpad <br>The equation used to show the relationship between $T$ and $N$ is </div>

    [{"vars":null}]

  644. <div class="sm_mode"> The number of dots in each pattern increases by the same amount each time. <br> sm_img https://teacher.smartermaths.com.au/wp-content/uploads/2018/08/NAPX-G3-CA23-SA.svg 300 indent vpad <br>How many dots are in Pattern 15? </div>

    [{"vars":null}]

  645. <div class="sm_mode"> Jane accidentally tore her receipt from her office Christmas party at a lobster restaurant. <br> sm_img https://teacher.smartermaths.com.au/wp-content/uploads/2021/03/NAPX9-TLE-30-v3-768x353.png 400 indent vpad <br>Her credit card statement showed that she spent $314.00. How much did one lobster cost? </div>

    [{"vars":null}]

  646. <div class="sm_mode"> Ben and Megs were playing a game where they both started with zero points. Ben's final score was 550 points and Meg's final score was −70. How many more points did Ben have than Megs? </div>

    [{"vars":null}]

  647. <div class="sm_mode"> It takes Charles 2.4 minutes to ride his bike one full lap around a lake track. <br> sm_img https://teacher.smartermaths.com.au/wp-content/uploads/2018/04/NAPX-J4-CA02.svg 320 indent vpad <br>How many laps does Charles complete in 12 minutes? </div>

    [{"vars":null}]

  648. <div class="sm_mode"> {{{question}}} </div>

    [{"vars":[{"varval":"Crisco takes part in a walkathon to raise money for charity.\n\nHis grandmother sponsors him $30.\n\nHis sister sponsors him $1.50 for each kilometre he walks.\n\nIn order to raise a total of $120, how many kilometres does Crisco need to walk?"},{"varval":"Solution 1\n\nTest each answer option:\n\n60 laps $\\times$ $1.50 = $90\n\n90 + 30 = $\\$120\\ \\checkmark$\n\n<br>\n\nStrategy 2 (advanced)\n\nsm_nogap Let $\\ \\large x$ = number of kilometres Crisco walks\n\n<div class=\"aligned\">\n\n|||\n|-:|-|\n|120|= 1.5$\\large x$ + 30|\n|1.5$\\large x$|= 90|\n|$\\therefore \\large x$|= $\\dfrac{90}{1.5}$|\n||= {{{correctAnswer}}} laps|\n\r\n\n</div>"}]},{"vars":[{"varval":"Daxx participates in a walkathon at the local cross country track.\n\nHis grandfather donates $30.\n\nHis neighbor donates $15 for each kilometre that Daxx walks.\n\n\r\n\r\nHow many kilometers does Daxx need to walk to raise $150 in total?"},{"varval":"Strategy 1\n\n1 km = 30 + 15 = $45\n\n2 km = 45 + 15 = $60\n\n3 km = 60 + 15 = $75\n\nsm_nogap >⋮\n\n8 km = 135 + 15 = $\\$150\\ \\checkmark$\n\n<br>\n \r\nStrategy 2 (more advanced)\n\n<br>Let $\\ \\large n$ = number of kilometers walked.\n\n\r\nsm_nogap Total money raised = 30 + 15 × $\\large n$\n\n<div class=\"aligned\">\n\n| | |\n| --------------------: | -------------- |\n| 30 + 15$\\large n$ | \\= 150 |\n| 15$\\large n$ | \\= 120 |\n| $\\therefore \\large n$| \\= {{{correctAnswer}}}|\n\n</div>"}]}]

  649. <div class="sm_mode"> {{{question}}} </div>

    [{"vars":[{"varval":"Carmen has three blocks of chocolate that are identical.\n\n\r\n\r\nShe also has a 4th smaller block of chocolate that weighs 175 grams.\n\n\r\n\r\nCarmen has 1480 grams of chocolate in total.\n\n\r\n\r\nWhich expression can be used to calculate the mass of one of the large blocks of chocolate?\n"},{"varval":"sm_nogap Mass of the 3 larger blocks\n\n\r\n>>= 1480 − 175\r\n\nsm_nogap $\\therefore$ Mass of one larger block\n\n>>= {{{correctAnswer}}}\n"}]},{"vars":[{"varval":"Stuart has three bottles of soft drink that are identical.\n\n\r\n\r\nHe also has a 4th smaller bottle of soft drink that contains 375 millilitres.\n\n\r\n\r\nStuart has 4125 millilitres of soft drink in total.\n\n\r\n\r\nWhich expression can be used to calculate the volume of one of the large bottles of soft drink?\n"},{"varval":"sm_nogap Total volume of the 3 larger bottles\n\n>>= 4125 − 375\r\n\n\nsm_nogap $\\therefore$ Volume of one larger bottle\n\n>>= {{{correctAnswer}}}\n"}]},{"vars":[{"varval":"Gareth has three containers of engine oil that are identical.\n\n\r\n\r\nHe also has a 4th smaller container of engine oil that contains 450 millilitres.\n\n\r\n\r\nGareth has 4950 millilitres of engine oil in total.\n\n\r\n\r\nWhich expression can be used to calculate the volume of one of the large containers of engine oil?\n"},{"varval":"sm_nogap Total volume of the 3 larger containers\n\r\n>>= 4950 − 450\r\n \r\n\r\nsm_nogap $\\therefore$ Volume of one larger container\n\n>>= {{{correctAnswer}}}\n"}]},{"vars":[{"varval":"Jute has four bunches of flowers that are identical.\n\n\r\n\r\nShe also has a 5th smaller bunch of flowers that is made up of 15 flowers.\n\n\r\n\r\nJute has used 153 flowers in total.\n\n\r\n\r\nWhich expression can be used to calculate the number of flowers in one of the larger bunches?\n"},{"varval":"sm_nogap Number of flowers in 4 larger bunches\n\n>>= 153 − 15\r\n\nsm_nogap $\\therefore$ Flowers in one larger bunch\n\n>>= {{{correctAnswer}}}\n"}]},{"vars":[{"varval":"Akira has five trucks that hold identical quantities of garden soil.\n\n\r\n\r\nHe also has a 6th smaller truck that holds 4 cubic metres of garden soil.\n\n\r\n\r\nAkira's six trucks hold 59 cubic metres of garden soil in total.\n\n\r\n\r\nWhich expression can be used to calculate the number of cubic metres of garden soil one of the large truck holds?\n"},{"varval":"sm_nogap Cubic metres in the 5 larger trucks\n\n\r\n>>= 59 − 4\r\n\nsm_nogap $\\therefore$ Cubic Metres in one the larger trucks\n\n>>= {{{correctAnswer}}}\n\n"}]},{"vars":[{"varval":"Louisa has four packets of cereal that are identical.\n\n\r\n\r\nShe also has a 5th smaller packet of cereal that weighs 375 grams.\n\n\r\n\r\nLouisa has 2455 grams of cereal in total.\n\n\r\n\r\nWhich expression can be used to calculate the mass of one of the large packets of cereal?\n"},{"varval":"sm_nogap Mass of the 4 larger packets\n\n>>= 2455 − 375\r\n\nsm_nogap $\\therefore$ Mass of one larger packet\n\n>>= {{{correctAnswer}}}\n"}]}]

  650. <div class="sm_mode"> Leila has $87. <br> sm_img https://teacher.smartermaths.com.au/wp-content/uploads/2018/08/NAPX-G3-NC03.svg 250 indent vpad <br>How much more money does she need to buy the dress? </div>

    [{"vars":null}]

  651. Number, NAPX9-TLC-30 v2

    <div class="sm_mode"> {{{question}}} </div>

    [{"vars":[{"varval":"Jacob has 4 dogs.\n\n\r\n\r\nEach dog eats 7 biscuits a week.\n\n\r\n\r\nJacob has no biscuits when he goes to the shop.\n\n\r\n\r\nIf the shop sells its biscuits in packets of 6, how many packets does Jacob need to buy for 1 week?"},{"varval":"sm_nogap Biscuits eaten in 1 week\n\n<div class=\"aligned\">\n\n>>||\n|-|\n|= 4 $\\times$ 7|\n|= 28|\n\n</div>\n\n<br>\n\n<div class=\"aligned\">\n\n|||\n|-|-|\n|$\\therefore$ Packets required|= 28 $\\div$ 6|\n||= 4 remainder 4|\n||= 5 full packets|\n\n</div>"}]},{"vars":[{"varval":"Carl wants to cook 20 chicken curry pies.\n\n\r\n\r\nEach chicken curry pie needs 2 eggs. He has no eggs left and went to the store.\n \r\n\r\nIf the store sells eggs in cartons of 12, how many cartons does Carl need?"},{"varval":"sm_nogap Number of eggs needed\n\n<div class=\"aligned\">\n\n>>||\n|-|\n|= 20 $\\times$ 2|\n|= 40|\n\n</div>\n\n<br>\n\n<div class=\"aligned\">\n\n|||\n|-|-|\n|$\\therefore$ Number of cartons needed|= 40 $\\div$ 12|\n||= 3 remainder 4|\n||= 4 cartons|\n\n</div>"}]}]

  652. <div class="sm_mode"> Basil cut a diagonal into a rectangular piece of wood. <br> sm_img https://teacher.smartermaths.com.au/wp-content/uploads/2018/08/NAPX-F3-CA12.svg 380 indent3 vpad <br>What is the missing length? </div>

    [{"vars":null}]

  653. <div class="sm_mode"> {{{question}}} </div>

    [{"vars":[{"varval":"The Sydney Opera House began construction in 1959.\r\n\r\nIn 2023, approximately how old would the Sydney Opera House have been?"},{"varval":"2023 - 1959 = 64 years.\n\n\r\n$\\therefore$ Closest approximation is {{{correctAnswer}}} years.\n"}]},{"vars":[{"varval":"Luna Park Sydney was opened on 4 October, 1935.\n\n\r\n\r\nIn 2019, approximately how long ago would the opening have been?"},{"varval":"2019 - 1935 = 84 years \n\n\r\n$\\therefore$ Closest approximation is {{{correctAnswer}}} years.\n"}]},{"vars":[{"varval":"Construction on the Eiffel Tower in France began in 1889.\n\n\r\n\r\nIn 2020, approximately how long ago did construction begin?"},{"varval":"2020 - 1889 = 131 years.\n\n\r\n$\\therefore$ Closest approximation is {{{correctAnswer}}} years.\n"}]},{"vars":[{"varval":"Construction on the Empire State Building in the USA was completed on 28 October, 1886.\n\n\r\n\r\nIn 2025, approximately how many years ago was it completed?"},{"varval":"2025 - 1886 = 139 years.\n\n\r\n$\\therefore$ Closest approximation is {{{correctAnswer}}} years.\n"}]},{"vars":[{"varval":"Mary's great grandfather was born in England in 1913.\n\n\r\n\r\nIn 2022, approximately how many years ago was he born?"},{"varval":"2022 - 1913 = 109 years.\n\n\r\n$\\therefore$ Closest approximation is {{{correctAnswer}}} years.\n"}]},{"vars":[{"varval":"Sir Edmund Hilary and Tenzing Norgay reached the summit of Mount Everest on May 29, 1953.\n\n\r\n\r\nIn 2024, approximately how many years ago did they reach the summit?"},{"varval":"2024 - 1953 = 71 years.\n\n\r\n$\\therefore$ Closest approximation is {{{correctAnswer}}} years.\n"}]}]

  654. <div class="sm_mode"> {{{question}}} </div>

    [{"vars":[{"varval":"Kami uses the number sentence &nbsp;9 × 12 = 108 &nbsp;to solve a problem.\n\n\r\n\r\nWhich of the following could be the problem?\n"},{"varval":"$9 per book × 12 books = $108\n\n$\\therefore$ {{{correctAnswer}}}\n"}]},{"vars":[{"varval":"Amaranta uses the number sentence &nbsp;4 × 24 = 96 &nbsp;to solve a problem.\n\n\r\n\r\nWhich of the following could be the problem?\n"},{"varval":"$24 per hour × 4 hours = $96\n\n$\\therefore$ {{{correctAnswer}}}\n"}]},{"vars":[{"varval":"Danillo uses the number sentence &nbsp;42 $\\div$ 6 = 7 &nbsp;to solve a problem.\n\n\r\n\r\nWhich of the following could be the problem?\n"},{"varval":"$$42\\ \\div$ 6 hours = $7 per hour\n\n$\\therefore$ {{{correctAnswer}}}\n"}]},{"vars":[{"varval":"Max uses the number sentence &nbsp;20 × 17 = 340 &nbsp;to solve a problem.\n\n\r\n\r\nWhich of the following could be the problem?\n"},{"varval":"$17 per ticket × 20 tickets = $340\n\n$\\therefore$ {{{correctAnswer}}}\n"}]},{"vars":[{"varval":"Belle uses the number sentence &nbsp;52 × 800 = 41 600 &nbsp;to solve a problem.\n\n\r\n\r\nWhich of the following could be the problem?\n"},{"varval":"52 weeks × 800 per week = $41 600 annual salary\n\n$\\therefore$ {{{correctAnswer}}}\n"}]},{"vars":[{"varval":"James uses the number sentence &nbsp;2.12 × 50 = 106 &nbsp;to solve a problem.\n\n\r\n\r\nWhich of the following could be the problem?\n"},{"varval":"$2.12 × 50 = $106\n\n$\\therefore$ {{{correctAnswer}}}\n"}]}]

  655. <div class="sm_mode"> {{{question}}} </div>

    [{"vars":[{"varval":"Pamela is a teacher and buys pencils for her class.\n\nShe purchases the pencils in two different sized packets.\n\n<br>\n\nsm_img https://teacher.smartermaths.com.au/wp-content/uploads/2017/09/NAP-J1-CA09-283x300.png 300 indent3 vpad\n\n<br>Pamela buys 6 packet A's and 4 packet B's.\n\n\r\n\r\nShe then divides all the pencils equally among her 6 students.\n\n\r\n\r\nHow many pencils does each student receive?"},{"varval":"<div class=\"aligned\">\n\n|||\n|-|-|\n|Total pencils| = (6 × 6) + (4 × 3)|\n|| = 48|\n\n</div>\n\n<div class=\"aligned\">\n\n|||\n|-|-|\n|$\\therefore$ Pencils per student| = 48 ÷ 6|\n|| = {{{correctAnswer}}}|\n\n</div>"}]},{"vars":[{"varval":"John bought bread rolls that came in bags of 4 and bags of 6.\n\n<br>\n\nsm_img https://teacher.smartermaths.com.au/wp-content/uploads/2021/04/RAPH10-q45.svg 350 indent3 vpad\n\n<br>He bought 5 Bags A's and 10 Bag B's.\n\nJohn then divided the bread rolls equally among 20 families.\n\nHow many bread rolls did each family receive?"},{"varval":"<div class=\"aligned\">\n\n|||\n|-|-|\n|Total bread rolls|= (4 × 5) + (6 × 10)|\n||= 20 + 60|\n||= 80|\n\n</div>\n\n<br>\n\n<div class=\"aligned\">\n\n|||\n|-|-|\n|$\\therefore$ Bread rolls per family|= $\\dfrac{80}{20}$|\n||= {{{correctAnswer}}}|\n\n</div>"}]}]

  656. <div class="sm_mode"> A stack of 9 flower pots is 560 mm high. A stack of 5 flower pots is 360 mm high. <br> sm_img https://teacher.smartermaths.com.au/wp-content/uploads/2021/03/NAPX9-TLE-40-2-768x618.png 350 indent vpad <br>Which rule can be used to work out the height, in millimetres, of a stack of $\large p$ flower pots? </div>

    [{"vars":null}]

  657. <div class="sm_mode"> Phoenix drew a straight line through the points (−2, 2) and (3,0) as shown in the diagram below. <br> sm_img https://teacher.smartermaths.com.au/wp-content/uploads/2018/04/NAPX-J4-CA22.svg 310 indent2 vpad <br>What is the gradient of the line that Phoenix drew? </div>

    [{"vars":null}]

  658. <div class="sm_mode"> Three corners of a parallelogram are drawn below. sm_img https://teacher.smartermaths.com.au/wp-content/uploads/2018/06/NAPX-F4-NC16.svg 300 indent vpad Which point below is the fourth corner? </div>

    [{"vars":null}]

  659. <div class="sm_mode"> The rules of a number game are shown in the flowchart below. <br> sm_img https://teacher.smartermaths.com.au/wp-content/uploads/2018/05/NAPX-E4-CA10.svg 550 indent vpad <br>The output number is 28. What are two possible values for the input number? </div>

    [{"vars":null}]

  660. 2 var question

    <div class="sm_mode"> {{q1}} {{{image}}} {{q2}} </div>

    [{"vars":[{"varval":"A surfboard can be hired for different numbers of hours, as shown in the table below.\r\n "},{"varval":"<div class=\"sm-table col1-color4\">\n\n>>| Number of hours| 1|2|3|4|5|\n|:-:|:-:|:-:|:-:|:-:|:-:|\n| Cost in dollars| 35|50|65|80|95|\n\n</div>\n\n<br>"},{"varval":"Select the answer that shows how to calculate the cost of hiring a surfboard."},{"varval":"By trial and error:\n\n$15 for every hour plus an extra $20\r\n\nTest:\r\n\n1 hour = 1 × 15 + 20 = $35 ✔\r\n\r\n3 hours = 3 × 15 + 20 = $65 ✔"}]},{"vars":[{"varval":"A stand up paddle board can be hired for different numbers of hours, as shown in the table below."},{"varval":"<div class=\"sm-table col1-color3\">\n\n>>| Number of hours| 1|2|3|4|5|\n|:-:|:-:|:-:|:-:|:-:|:-:|\n| Cost in dollars| 25|40|55|70|85|\n\n</div>\n\n<br>"},{"varval":"Select the answer that shows how to calculate the cost of hiring a stand up paddle board."},{"varval":"By trial and error:\n\n$15 for every hour plus an extra $10\r\n\nTest:\r\n\n1 hour = 1 × 15 + 10 = $25 ✔\r\n\r\n4 hours = 4 × 15 + 10 = $70 ✔"}]}]

  661. SJ Added v3, 4, 5

    <div class="sm_mode"> {{{question}}} </div>

    [{"vars":[{"varval":"Patrick is saving his money to buy a second hand drum kit.\r\n\r\nIn week 1, he has $9.\r\n\r\nEvery week after week 1, he contributes the same amount to his saving.\n\n<br>\n\n<div class=\"sm-table col1-color6\">\n\n>>| **Week** | 1|2|3|4|5|\n|:-|:-:|:-:|:-:|:-:|:-:|\n| **Total amount saved** | $9|$16|$23|$30|$37|\n\n\n</div>\n\n<br>How much will Patrick have saved by the end of week 9?"},{"varval":"Weekly saving = 16 $-$ 9 = $7\n\nTotal savings at end of:\r\n\nWeek 6 = 37 + 7 = $44\r\n\nWeek 7 = 44 + 7= $51\r\n\nWeek 8 = 51 + 7 = $58\r\n\nWeek 9 = 58 + 7 = {{{prefix0}}}{{{correctAnswer0}}}"}]},{"vars":[{"varval":"Bernard is saving his money to buy V-bux for his gaming.\r\n\r\nIn week 1, he has $7.\r\n\r\nEvery week after week 1, he contributes the same amount to his saving.\n\n<br>\n\n<div class=\"sm-table col1-color8\">\n\n>>| **Week** | 1|2|3|4|5|\n|:-|:-:|:-:|:-:|:-:|:-:|\n| **Total amount saved** | $7|$19|$31|$43|$55|\n\n\n</div>\n\n<br>How much will Bernard have saved by the end of week 8?"},{"varval":"Weekly saving = 19 $-$ 7 = $12\n\nTotal savings at end of:\r\n\nWeek 6 = 55 + 12 = $67\n\nWeek 7 = 67 + 12 = $79\r\n\nWeek 8 = 79 + 12 = {{{prefix0}}}{{{correctAnswer0}}}"}]},{"vars":[{"varval":"Jill is saving her money to buy a new AFL football.\r\n\r\nIn week 1, she has $8.\r\n\r\nEvery week after week 1, she contributes the same amount to her saving.\n\n<br>\n\n<div class=\"sm-table col1-color2\">\n\n>>| **Week** | 1|2|3|4|5|\n|:-|:-:|:-:|:-:|:-:|:-:|\n| **Total amount saved** | $8|$12|$16|$20|$24|\n\n\n</div>\n\n<br>How much will Jill have saved by the end of week 9?"},{"varval":"Weekly saving = 12 $-$ 8 = $4\n\nTotal savings at end of:\r\n\nWeek 6 = 24 + 4 = $28\n\nWeek 7 = 28 + 4 = $32\n\nWeek 8 = 32 + 4 = $36\n\nWeek 9 = 36 + 4 = {{{prefix0}}}{{{correctAnswer0}}}"}]},{"vars":[{"varval":"Matilda is saving her money to buy a new soccer ball.\n\nIn week 1, she has $7.\n\nEvery week after week 1, she contributes the same amount to her saving.\n\n<br>\n\n<div class=\"sm-table col1-color4\">\n\n>>| **Week** | 1|2|3|4|5|\n|:-|:-:|:-:|:-:|:-:|:-:|\n| **Total amount saved** | $7|$17|$27|$37|$47|\n\n\n</div>\n\n<br>How much will Matilda have saved by the end of week 8?"},{"varval":"Weekly saving = 17 $-$ 7 = $10\n\nTotal savings at end of:\n\nWeek 6 = 47 + 10 = $57\n\nWeek 7 = 57 + 10 = $67\n\nWeek 8 = 67 + 10 = {{{prefix0}}}{{{correctAnswer0}}}"}]},{"vars":[{"varval":"Marty is saving his money to buy a new cricket bat.\n\nIn week 1, he has $36.\n\nEvery week after week 1, he contributes the same amount to his saving.\n\n<br>\n\n<div class=\"sm-table col1-color5\">\n\n>>| **Week** | 1|2|3|4|5|\n|:-|:-:|:-:|:-:|:-:|:-:|\n| **Total amount saved** | $36|$48|$60|$72|$84|\n\n\n</div>\n\n<br>How much will Marty have saved by the end of week 9?"},{"varval":"Weekly saving = 48 $-$ 36 = $12\n\nTotal savings at end of:\n\nWeek 6 = 84 + 12 = $96\n\nWeek 7 = 96 + 12 = $108\n\nWeek 8 = 108 + 12 = $120\n\nWeek 9 = 120 + 12 = {{{prefix0}}}{{{correctAnswer0}}}"}]},{"vars":[{"varval":"Milo is saving his money to buy a new kayak.\n\nIn week 1, he has $150.\n\nEvery week after week 1, he contributes the same amount to his saving.\n\n<br>\n\n<div class=\"sm-table col1-color1\">\n\n>>| **Week** | 1|2|3|4|5|\n|:-|:-:|:-:|:-:|:-:|:-:|\n| **Total amount saved** | $150|$190|$230|$270|$310|\n\n\n</div>\n\n<br>How much will Milo have saved by the end of week 8?"},{"varval":"Weekly saving = 190 $-$ 150 = $40\n\nTotal savings at end of:\n\nWeek 6 = 310 + 40 = $350\n\nWeek 7 = 350 + 40 = $390\n\nWeek 8 = 390 + 40 = {{{prefix0}}}{{{correctAnswer0}}}"}]}]

  662. <div class="sm_mode"> Lief uses sticks to make a pattern. He starts with 2 sticks for Design 1. sm_img https://teacher.smartermaths.com.au/wp-content/uploads/2018/08/NAPX-G3-NC08.svg 725 indent vpad How many sticks does Lief need for Design 5? </div>

    [{"vars":null}]

  663. <div class="sm_mode"> Heather is making a pattern with coloured tiles and white tiles, as shown below. sm_img https://teacher.smartermaths.com.au/wp-content/uploads/2017/01/naplan-Y7-2016-3mc.png 350 indent vpad </div> If the pattern continues, how many coloured tiles will there be in Design 9?

    [{"vars":null}]

  664. <div class="sm_mode"> {{{question}}} </div>

    [{"vars":[{"varval":"The table below has a pattern where the top and bottom numbers are connected by a rule.\n\n<br>\n\n<div class=\"sm-table col1-color3\">\n\n>>| Top number| 1 |2|3|4|...|**?**|\n|:-:|:-:|:-:|:-:|:-:|:-:|:-:|\n| Bottom number| 6|12|18|24| ...| 42|\n\n</div>\n\n<br>What is the top number when the bottom number is 42?"},{"varval":"The rule is:\r\n\nTop number × 6 = Bottom number\r\n\nIf Top number × 6 = 42,\n\n<div class=\"aligned\">\r\n\r\n| | |\r\n| ------------- | ---------- |\r\n| ∴ Top number | \\= 42 ÷ 6 |\r\n| | \\= {{{correctAnswer}}} |\r\n\r\n</div>"}]},{"vars":[{"varval":"The table below has a pattern where the top and bottom numbers are connected by a rule.\n\n<br>\n\n<div class=\"sm-table col1-color3\">\n\n>>| Top number| 1 |2|3|4|...|**?**|\n|:-:|:-:|:-:|:-:|:-:|:-:|:-:|\n| Bottom number| 9|18|27|36| ...| 54|\n\n</div>\n\n<br>What is the top number when the bottom number is 54?"},{"varval":"The rule is:\r\n\nTop number × 9 = Bottom number\r\n\nIf Top number × 9 = 54,\n\n<div class=\"aligned\">\r\n\r\n| | |\r\n| ------------- | ---------- |\r\n| ∴ Top number | \\= 54 ÷ 9 |\r\n| | \\= {{{correctAnswer}}} |\r\n\r\n</div>"}]},{"vars":[{"varval":"The table below has a pattern where the top and bottom numbers are connected by a rule.\n\n<br>\n\n<div class=\"sm-table col1-color3\">\n\n>>| Top number| 1 |2|3|4|...|**?**|\n|:-:|:-:|:-:|:-:|:-:|:-:|:-:|\n| Bottom number| 8|16|24|32| ...| 72|\n\n</div>\n\n<br>What is the top number when the bottom number is 72?"},{"varval":"The rule is:\r\n\nTop number × 8 = Bottom number\r\n\nIf Top number × 8 = 72,\n\n<div class=\"aligned\">\r\n\r\n| | |\r\n| ------------- | ---------- |\r\n| ∴ Top number | \\= 72 ÷ 8 |\r\n| | \\= {{{correctAnswer}}} |\r\n\r\n</div>"}]}]

  665. 2 variant: Algebra, NAPX-L4-CA02 SA v2 Algebra, NAPX-L4-CA02 SA v1

    <div class="sm_mode"> {{q1}} {{{image}}} <br> {{q2}} </div>

    [{"vars":[{"varval":"The table shows the distances Marvin runs on four consecutive days."},{"varval":"<div class=\"sm-table col1-color2\">\n\n>| **Day**| Tuesday| Wednesday|Thursday|Friday|\n|:-:|:-:|:-:|:-:|:-:|\n| **Distance (km)**| 7.3|7.6|7.9|8.2|\n\n</div>"},{"varval":"Every day, Marvin increases the distance he runs by the same amount.\r\n\r\nWhat distance will Marvin run on Saturday?"},{"varval":"Each day, Marvin runs an extra 0.3 km."},{"varval":"On Saturday,"},{"varval":"8.2"},{"varval":"0.3"},{"varval":"Distance"}]},{"vars":[{"varval":"The table shows the distances Paulo swims on four different days."},{"varval":"<div class=\"sm-table col1-color8\">\n\n>| **Day**| Tuesday| Wednesday|Thursday|Friday|\n|:-:|:-:|:-:|:-:|:-:|\n| **Distance (km)**| 2.3|2.5|2.7|2.9|\n\n</div>"},{"varval":"Every day, Paulo increases the distance he swims by the same amount.\r\n\r\n\nWhat distance will Paulo swim on Tuesday?"},{"varval":"Each day, Paulo swims an extra 0.2 km."},{"varval":"On Tuesday,"},{"varval":"2.9"},{"varval":"0.2"},{"varval":"Distance"}]},{"vars":[{"varval":"The table shows the total area of a growing mould in an experiment."},{"varval":"<div class=\"sm-table col1-color2\">\n\n>| **Day**| Thursday| Friday|Saturday|Sunday|\n|:-:|:-:|:-:|:-:|:-:|\n| **Area (cm$^2$)**| 0.7|1.3|1.9|2.5|\n\n</div>"},{"varval":"Every day, the mould grows by the same amount.\r\n\r\n\nWhat area will the mould cover on Tuesday?"},{"varval":"Each day, the mould grows an extra 0.6 cm$^2$."},{"varval":"On Tuesday,"},{"varval":"2.5"},{"varval":"$0.6 \\times 2$"},{"varval":"Total Area"}]}]

  666. <div class="sm_mode"> {{{question}}} </div>

    [{"vars":[{"varval":"Using the table, a school teacher allocates pieces of cardboard to class groups depending on the number of students in each group.\n\n<br>\n\n<div class=\"sm-table row1-color8\">\n\n>>| Number of <br> students in group | Pieces of <br> cardboard | \n|:-:|:-:|\n| 2 | 6|\n| 4 | 12|\n| 6 | **?**|\n|8|24|\n\n</div>\n\n<br>Using the pattern in the table, how many pieces of cardboard should a group of 6 students receive?"},{"varval":"The pattern shows that each student receives\r 3 pieces of cardboard.\r\n\n∴ A group of 6 will be given {{{correctAnswer0}}} pieces."}]},{"vars":[{"varval":"Using the table, a school teacher allocates dominos to class groups depending on the number of students in each group.\n\n<br>\n\n<div class=\"sm-table row1-color8\">\n\n>>| Number of <br> students in group | Number of <br> dominos | \n|:-:|:-:|\n| 3 | 18|\n| 5 | 30|\n| 7 | **?**|\n|9|54|\n\n</div>\n\n<br>Using the pattern in the table, how many dominos should a group of 7 students receive?"},{"varval":"The pattern shows that each student receives\r 6 dominos each.\r\n\n∴ A group of 7 will be given {{{correctAnswer0}}} dominos."}]},{"vars":[{"varval":"Using the table, an art teacher allocates paint brushes to class groups depending on the number of students in each group.\n\n<br>\n\n<div class=\"sm-table row1-color8\">\n\n>>| Number of <br> students in group | Number of <br> paint brushes | \n|:-:|:-:|\n| 3 | 21|\n| 5 | 35|\n| 7 | **?**|\n|8|56|\n\n</div>\n\n<br>Using the pattern in the table, how many paint brushes should a group of 7 students receive?"},{"varval":"The pattern shows that each student receives\r 7 paint brushes each.\r\n\n∴ A group of 7 will be given {{{correctAnswer0}}} paint brushes."}]}]

  667. <div class="sm_mode"> {{{question}}} </div>

    [{"vars":[{"varval":"Harley solved the following equation:\r\n\r\n\n$4\\large x$ + 3 = 12\r\n\nWhich of the following could be two lines of her solution?"},{"varval":"<div class=\"aligned\">\r\n\r\n\n| | |\r\n| -------------: | ---------- |\r\n| $4\\large x$ + 3 | \\= 12 |\r\n| $4\\large x$ | \\= 9 |\n| $\\large x$| \\= $\\dfrac{9}{4}$ |\n\r\n\r\n\n</div>"}]},{"vars":[{"varval":"Charli solved the following equation:\r\n\r\n\n $3\\large x$ + 8 = 15\r\n\nWhich of the following could be two lines of her solution?"},{"varval":"<div class=\"aligned\">\r\n\r\n\n| | |\r\n| -------------: | ---------- |\r\n| $3\\large x$ + 8 | \\= 15 |\r\n| $3\\large x$ | \\= 7 |\n| $\\large x$| \\= $\\dfrac{7}{3}$ |\n\r\n\r\n\n</div>"}]},{"vars":[{"varval":"Blinky solved the following equation:\r\n\r\n\n$2\\large x$ $-$ 7 = 4\r\n\nWhich of the following could be two lines of her solution?"},{"varval":"<div class=\"aligned\">\r\n\r\n\n| | |\r\n| -------------: | ---------- |\r\n| $2\\large x$ $-$ 7 | \\= 4 |\r\n| $2\\large x$ | \\= 11 |\n| $\\large x$| \\= $\\dfrac{11}{2}$ |\n\r\n\r\n\n</div>"}]},{"vars":[{"varval":"Joanna solved the following equation:\r\n\r\n\n$5\\large x$ $-$ 8 = $-$ 5\r\n\nWhich of the following could be two lines of her solution?"},{"varval":"<div class=\"aligned\">\r\n\r\n\n| | |\r\n| -------------: | ---------- |\r\n| $5\\large x$ $-$ 8 | \\= $-$ 5 |\r\n| $5\\large x$ | \\= 3 |\n| $\\large x$| \\= $\\dfrac{3}{5}$ |\n\r\n\r\n\n</div>"}]},{"vars":[{"varval":"Sheldon solved the following equation:\r\n\r\n\n$7\\large x$ + 8 = $-$ 2\r\n\nWhich of the following could be two lines of her solution?"},{"varval":"<div class=\"aligned\">\r\n\r\n\n| | |\r\n| -------------: | ---------- |\r\n| $7\\large x$ + 8 | \\= $-$ 2 |\r\n| $7\\large x$ | \\= $-$ 10 |\n| $\\large x$| \\= $-$ $\\dfrac{10}{7}$ |\n\r\n\r\n\n</div>"}]},{"vars":[{"varval":"Joyce solved the following equation:\r\n\r\n\n$-$ $3\\large x$ + 5 = 3\r\n\nWhich of the following could be two lines of her solution?"},{"varval":"<div class=\"aligned\">\r\n\r\n\n| | |\r\n| -------------: | ---------- |\r\n| $-$ $3\\large x$ + 5 | \\= 3 |\r\n| $-$ $3\\large x$ | \\= $-$ 2 |\n| $\\large x$| \\= $\\dfrac{2}{3}$ |\n\r\n\r\n\n</div>"}]}]

  668. <div class="sm_mode"> Daniel is in a 2-day rally car race. On day 1, Daniel takes 1 hour and 20 minutes to travel 115 km. On day 2, Daniel takes 2 hours to travel 175 km. What is Daniel's average speed over the 2 days. </div>

    [{"vars":null}]

  669. <div class="sm_mode"> $P, Q, R$ and $S$ are towns that lie on a straight train track as shown below. sm_img https://teacher.smartermaths.com.au/wp-content/uploads/2018/06/NAPX-F4-NC28.svg 300 indent vpad The distance between $P$ and $Q$ is half the distance from $P$ to $S$. The distance from $R$ to $S$ is one-fifth the distance from $P$ to $S$. The distance from $Q$ to $R$ is 21 km. What is the distance between $P$ and $S$? </div>

    [{"vars":null}]

  670. <div class="sm_mode"> {{{question}}} </div>

    [{"vars":[{"varval":"Sisko keeps a tin of lollipops on his desk at home.\n\n\r\n\r\nHis son Tardis takes 1 lollipop out every 4 days.\n\n\r\n\r\nSisko adds a new lollipop to the tin every 5 days.\n\nThe lollipops in Sisko's tin are decreasing by 1 lollipop every $\\large x$ days.\n\nWhat is the value of $\\large x$?\n"},{"varval":"Expressing the information as an equation:\n\n<div class=\"aligned\">\r\n\r\n| | |\r\n| ------------- :| ---------- |\r\n| $\\dfrac{\\large x}{4} - \\dfrac{\\large x}{5}$| \\= 1 |\r\n| $5 \\large x$ $-$ $4\\large x$| \\= 20 |\r\n| $\\therefore \\large x$ | \\= {{{correctAnswer0}}} {{{suffix0}}} |\r\n\r\n</div>"}]},{"vars":[{"varval":"John keeps a tin of highlighters on his desk at work.\n\n\r\n\r\nHis work colleagues take 1 highlighter out of the tin every 3 days.\n\n\r\n\r\nJohn adds a new highlighter to the tin every 4 days.\n\nThe highlighters in John's tin are decreasing by 1 highlighter every $\\large x$ days.\n\nWhat is the value of $\\large x$?\n"},{"varval":"Expressing the information as an equation:\n\n<div class=\"aligned\">\r\n\r\n| | |\r\n| ------------- :| ---------- |\r\n| $\\dfrac{\\large x}{3} - \\dfrac{\\large x}{4}$| \\= 1 |\r\n| $4 \\large x$ $-$ $3\\large x$| \\= 12 |\r\n| $\\therefore \\large x$ | \\= {{{correctAnswer0}}} {{{suffix0}}} |\r\n\r\n</div>"}]},{"vars":[{"varval":"May has a pot of daisies in her garden.\n\n\r\n\r\nThe snails in her garden eat 1 daisy plant every 6 days.\n\n\r\n\r\nMay adds a new daisy plant to the garden every 5 days.\n\nThe daisy plants in May's garden are increasing by 1 daisy plant every $\\large x$ days.\n\nWhat is the value of $\\large x$?\n"},{"varval":"Expressing the information as an equation:\n\n<div class=\"aligned\">\r\n\r\n| | |\r\n| ------------- :| ---------- |\r\n| $\\dfrac{\\large x}{5} - \\dfrac{\\large x}{6}$| \\= 1 |\r\n| $6 \\large x$ $-$ $5\\large x$| \\= 30 |\r\n| $\\therefore \\large x$ | \\= {{{correctAnswer0}}} {{{suffix0}}} |\r\n\r\n</div>"}]},{"vars":[{"varval":"Jake is serving drinks at a function.\n\nHe takes 7 drinks from the refrigerator every minute and gives them to guests.\n\nRob is adding 6 drinks to the same refrigerator every minute to restock it. \n\nThe number of drinks in the refrigerator is decreasing by 1 drink every $\\large x$ minutes.\n\nWhat is the value of $\\large x$?\n"},{"varval":"Expressing the information as an equation:\n\n<div class=\"aligned\">\r\n\r\n| | |\r\n| ------------- :| ---------- |\r\n| $\\dfrac{\\large x}{6} - \\dfrac{\\large x}{7}$| \\= 1 |\r\n| $7 \\large x$ $-$ $6\\large x$| \\= 42 |\r\n| $\\therefore \\large x$ | \\= {{{correctAnswer0}}} {{{suffix0}}} |\r\n\r\n</div>"}]},{"vars":[{"varval":"Clarice has a tub of ice for cooling drinks at her party.\n\n\r\n\r\nThe ice is melting at a rate of 1 litre every 2 hours.\n\n\r\n\r\nClarice adds 1 litre of ice to the tub every 3 hours.\n\nThe ice in Clarice's tub is decreasing by 1 litre every $\\large x$ hours.\n\nWhat is the value of $\\large x$?"},{"varval":"Expressing the information as an equation:\n\n<div class=\"aligned\">\r\n\r\n| | |\r\n| ------------- :| ---------- |\r\n| $\\dfrac{\\large x}{2} - \\dfrac{\\large x}{3}$| \\= 1 |\r\n| $3 \\large x$ $-$ $2\\large x$| \\= 6 |\r\n| $\\therefore \\large x$ | \\= {{{correctAnswer0}}} {{{suffix0}}} |\r\n\r\n</div>"}]},{"vars":[{"varval":"Bill is adding 5 drinks a minute to a triathlon drinks table during an event.\n\nSix drinks are being taken from the table every minute by competitors.\n\nThe number of drinks on the table is decreasing by 1 drink every $\\large x$ minutes.\n\nWhat is the value of $\\large x$?"},{"varval":"Expressing the information as an equation:\n\n<div class=\"aligned\">\r\n\r\n| | |\r\n| ------------- :| ---------- |\r\n| $\\dfrac{\\large x}{5} - \\dfrac{\\large x}{6}$| \\= 1 |\r\n| $6 \\large x$ $-$ $5\\large x$| \\= 30 |\r\n| $\\therefore \\large x$ | \\= {{{correctAnswer0}}} {{{suffix0}}} |\r\n\r\n</div>"}]}]

  671. Awaiting correct images for Variants1-5

    <div class="sm_mode"> {{{question}}} </div>

    [{"vars":[{"varval":"The graph below can be used to approximately convert kilograms to pounds.\n\n<br>\n\nsm_img https://teacher.smartermaths.com.au/wp-content/uploads/2022/08/Number-NAPX-F4-NC27-va0.svg 310 indent vpad\n\n<br>Using the graph, a mass of 2 kilograms is closest to\n"},{"varval":"Since 5 kg = 11 pounds\n\n\r\n1 kg = $\\dfrac{11}{5}$ = 2.2 pounds\n\n<div class=\"aligned\">\n\n| | |\n| --------------------- | -------------- |\n|$\\therefore$ 2 kg | \\= 2 × 2.2 |\n| | \\= {{{correctAnswer}}} |\n\r\n\n</div>\n"}]},{"vars":[{"varval":"The graph below can be used to approximately convert kilograms to pounds.\n\n<br>\n\nsm_img https://teacher.smartermaths.com.au/wp-content/uploads/2022/08/Number-NAPX-F4-NC27-va1.svg 310 indent vpad\n\n<br>Using the graph, a mass of 4 kilograms is closest to\n"},{"varval":"Since 5 kg = 11 pounds\n\n\r\n1 kg = $\\dfrac{11}{5}$ = 2.2 pounds\n\n<div class=\"aligned\">\n\n| | |\n| --------------------- | -------------- |\n|$\\therefore$ 4 kg | \\= 4 × 2.2 |\n| | \\= {{{correctAnswer}}} |\n\r\n\n</div>\n"}]},{"vars":[{"varval":"The graph below can be used to approximately convert kilograms to pounds.\n\n<br>\n\nsm_img https://teacher.smartermaths.com.au/wp-content/uploads/2022/08/Number-NAPX-F4-NC27-va2.svg 310 indent vpad\n\n<br>Using the graph, a mass of 3 kilograms is closest to\n"},{"varval":"Since 5 kg = 11 pounds\n\n\r\n1 kg = $\\dfrac{11}{5}$ = 2.2 pounds\n\n<div class=\"aligned\">\n\n| | |\n| --------------------- | -------------- |\n|$\\therefore$ 3 kg | \\= 3 × 2.2 |\n| | \\= {{{correctAnswer}}} |\n\r\n\n</div>\n"}]},{"vars":[{"varval":"The graph below can be used to approximately convert kilograms to pounds.\n\n<br>\n\nsm_img https://teacher.smartermaths.com.au/wp-content/uploads/2022/08/Number-NAPX-F4-NC27-va3.svg 310 indent vpad\n\n<br>Using the graph, a mass of 6 kilograms is closest to\n"},{"varval":"Since 5 kg = 11 pounds\n\n\r\n1 kg = $\\dfrac{11}{5}$ = 2.2 pounds\n\n<div class=\"aligned\">\n\n| | |\n| --------------------- | -------------- |\n|$\\therefore$ 6 kg | \\= 6 × 2.2 |\n| | \\= {{{correctAnswer}}} |\n\r\n\n</div>\n"}]},{"vars":[{"varval":"The graph below can be used to approximately convert kilograms to pounds.\n\n<br>\n\nsm_img https://teacher.smartermaths.com.au/wp-content/uploads/2022/08/Number-NAPX-F4-NC27-va4.svg 310 indent vpad\n\n<br>Using the graph, a mass of 1.5 kilograms is closest to\n"},{"varval":"Since 5 kg = 11 pounds\n\n\r\n1 kg = $\\dfrac{11}{5}$ = 2.2 pounds\n\n<div class=\"aligned\">\n\n| | |\n| --------------------- | -------------- |\n|$\\therefore$ 1.5 kg | \\= 1.5 × 2.2 |\n| | \\= {{{correctAnswer}}} |\n\r\n\n</div>\n"}]},{"vars":[{"varval":"The graph below can be used to approximately convert kilograms to pounds.\n\n<br>\n\nsm_img https://teacher.smartermaths.com.au/wp-content/uploads/2022/08/Number-NAPX-F4-NC27-va5.svg 310 indent vpad\n\n<br>Using the graph, a mass of 8 pounds is closest to\n"},{"varval":"Since 11 pounds = 5 kg\n\n\r\n1 pound = $\\dfrac{5}{11}$ kg\n\n<div class=\"aligned\">\n\n| | |\n| --------------------- | -------------- |\n|$\\therefore$ 8 pounds | \\= 8 × $\\dfrac{5}{11}$ |\n| | \\= 3.6363.... |\n| | $\\approx$ {{{correctAnswer}}} |\n\r\n\n</div>\n"}]}]

  672. <div class="sm_mode"> The sand in an egg timer takes 3 minutes to completely flow through. The egg timer contains 145 grams of sand. Each millilitre of sand weighs 1.25 grams. sm_img https://teacher.smartermaths.com.au/wp-content/uploads/2018/04/NAPX-H4-CA22.svg 100 indent3 vpad How many millilitres of sand flow through the egg timer per minute? </div>

    [{"vars":null}]

  673. <div class="sm_mode"> Jaco has a square billboard poster with an area of 9 m². He enlarges the billboard poster to have an area of 81 m². The enlargement multiplied the side lengths of the original billboard poster by a factor of </div>

    [{"vars":null}]

  674. <div class="sm_mode"> Caleb is travelling from Brisbane to Toowoomba. The journey is 135 kilometres. His car uses 9.45 litres of fuel per 100 kilometres. How much fuel will Caleb need to make the journey? Round your answer to the nearest litre. </div>

    [{"vars":null}]

  675. <div class="sm_mode"> {{{question}}} </div>

    [{"vars":[{"varval":"Shinji used 8 litres of paint to paint a wall.\n\r\nThe wall was a square with sides 4 metres long.\n\r\nHow many litres of paint would he need to paint a rectangular wall which is 3 metres high and 10 metres wide?"},{"varval":"sm_nogap Area of smaller wall\n\n<div class=\"aligned\">\n\n>>||\n|-|\n|= 4 × 4|\n|= 16 m$^2$|\n\n</div>\n\n<br>\n\nsm_nogap Area of larger wall\n\n<div class=\"aligned\">\n\n>>||\n|-|\n|= 3 × 10|\n|= 30 m$^2$|\n\n</div>\n\n<br>\n\r\nsm_nogap $\\therefore$ Paint needed for larger wall\r\n\n<div class=\"aligned\">\n\n>>||\n|-|\n|= $\\dfrac{30}{16}$ × 8|\n|= {{{correctAnswer0}}} {{{suffix0}}}|\n\n</div>"}]},{"vars":[{"varval":"Bobby used 4 litres of varnish to paint the loungeroom floor.\n\n\rThe floor was a square with sides 4 metres long.\n\r\nHow many litres of varnish would he need to paint a rectangular floor which is 2.5 metres long and 8 metres wide?"},{"varval":"sm_nogap Area of smaller floor\n\n<div class=\"aligned\">\n\n>>||\n|-|\n|= 4 × 4|\n|= 16 m$^2$|\n\n</div>\n\n<br>\n\nsm_nogap Area of larger floor\n\n<div class=\"aligned\">\n\n>>||\n|-|\n|= 2.5 × 8|\n|= 20 m$^2$|\n\n</div>\n\n<br>\n\r\nsm_nogap $\\therefore$ Varnish needed for larger floor\r\n\n<div class=\"aligned\">\n\n>>||\n|-|\n|= $\\dfrac{20}{16}$ × 4|\n|= {{{correctAnswer0}}} {{{suffix0}}}|\n\n</div>"}]},{"vars":[{"varval":"Bliss used 10 litres of paint to paint a mural.\n\n\r\n\r\nThe mural was a square with sides 5 metres long.\n\n\r\n\r\nHow many litres of paint would she need to paint a rectangular mural which is 4 metres high and 15 metres wide?"},{"varval":"sm_nogap Area of smaller mural\n\n<div class=\"aligned\">\n\n>>||\n|-|\n|= 5 × 5|\n|= 25 m$^2$|\n\n</div>\n\n<br>\n\nsm_nogap Area of larger mural\n\n<div class=\"aligned\">\n\n>>||\n|-|\n|= 4 × 15|\n|= 60 m$^2$|\n\n</div>\n\n<br>\n\r\nsm_nogap $\\therefore$ Paint needed for larger mural\r\n\n<div class=\"aligned\">\n\n>>||\n|-|\n|= $\\dfrac{60}{25}$ × 10|\n|= {{{correctAnswer0}}} {{{suffix0}}}|\n\n</div>"}]},{"vars":[{"varval":"Jordan used 15 litres of sealant to reseal his driveway.\n\n\r\n\r\nThe driveway was a square with sides 4.5 metres long.\n\n\r\n\r\nHow many litres of sealant would he need to reseal a rectangular driveway which is 6 metres wide and 9 metres long?"},{"varval":"sm_nogap Area of smaller driveway\n\n<div class=\"aligned\">\n\n>>||\n|-|\n|= 4.5 × 4.5|\n|= 20.25 m$^2$|\n\n</div>\n\n<br>\n\nsm_nogap Area of larger driveway\n\n<div class=\"aligned\">\n\n>>||\n|-|\n|= 6 × 9|\n|= 54 m$^2$|\n\n</div>\n\n<br>\n\r\nsm_nogap $\\therefore$ Sealant needed for larger driveway\r\n\n<div class=\"aligned\">\n\n>>||\n|-|\n|= $\\dfrac{54}{20.25}$ × 15|\n|= {{{correctAnswer0}}} {{{suffix0}}}|\n\n</div>"}]},{"vars":[{"varval":"Wendy used 8 litres of fertiliser to fertilise her garden.\n\n\r\n\r\nThe garden was a square with sides 16 metres long.\n\n\r\n\r\nHow many litres of fertiliser would she need to fertilise a rectangular garden which is 40 metres wide and 12 metres long?"},{"varval":"sm_nogap Area of smaller garden\n\n<div class=\"aligned\">\n\n>>||\n|-|\n|= 16 × 16|\n|= 256 m$^2$|\n\n</div>\n\n<br>\n\nsm_nogap Area of larger garden\n\n<div class=\"aligned\">\n\n>>||\n|-|\n|= 40 × 12|\n|= 480 m$^2$|\n\n</div>\n\n<br>\n\r\nsm_nogap $\\therefore$ Fertiliser needed for larger garden\r\n\n<div class=\"aligned\">\n\n>>||\n|-|\n|= $\\dfrac{480}{256}$ × 8|\n|= {{{correctAnswer0}}} {{{suffix0}}}|\n\n</div>"}]},{"vars":[{"varval":"Giles used 15 litres of pesticide to treat pests in his orchard.\n\n\r\n\r\nThe orchard was a square with sides 25 metres long.\n\n\r\n\r\nHow many litres of pesticide would he need to treat a rectangular orchard which is 50 metres wide and 30 metres long?"},{"varval":"sm_nogap Area of smaller orchard\n\n<div class=\"aligned\">\n\n>>||\n|-|\n|= 25 × 25|\n|= 625 m$^2$|\n\n</div>\n\n<br>\n\nsm_nogap Area of larger orchard\n\n<div class=\"aligned\">\n\n>>||\n|-|\n|= 50 × 30|\n|= 1500 m$^2$|\n\n</div>\n\n<br>\n\r\nsm_nogap $\\therefore$ Pesticide needed for larger orchard\n\n<div class=\"aligned\">\n\n>>||\n|-|\n|= $\\dfrac{1500}{625}$ × 15|\n|= {{{correctAnswer0}}} {{{suffix0}}}|\n\n</div>"}]}]

  676. <div class="sm_mode"> Ron is renting a room in a dormitory at $400 per month. His landlord offers him an upgraded room with its own bathroom. His rent for the upgraded room increases to $6600 for 12 months. What is the extra **monthly cost** of the upgraded room? </div>

    [{"vars":null}]

  677. <div class="sm_mode"> {{{question}}} </div>

    [{"vars":[{"varval":"Misty made 21 shakes at her friend's birthday party. Some were kale and some were spinach.\n\nBarry randomly chose one shake and had a 1 in 7 chance of choosing a kale shake.\n\nHow many spinach shakes did Misty make?"},{"varval":"sm_nogap Number of kale shakes\n\n<div class=\"aligned\">\n\n>>||\n|-|\n|= $\\dfrac{1}{7} \\times 21$|\n|= 3|\n\n</div>\n\n<br>\n\nsm_nogap $\\therefore$ Number of spinach shakes\n\n<div class=\"aligned\">\n\n>>||\n|-|\n|= 21 $-$ 3|\n|= {{{correctAnswer0}}}|\n\n</div>"}]},{"vars":[{"varval":"Kellog made 120 cupcakes for his daughter's birthday party. Some were vanilla and the rest were chocolate.\n\nIf Kenny randomly chooses one cupcake, he has a 1 in 5 chance of choosing a chocolate cupcake.\n\nHow many vanilla cupcakes did Kellog make?"},{"varval":"sm_nogap Number of chocolate cupcakes\n\n<div class=\"aligned\">\n\n>>||\n|-|\n|= $\\dfrac{1}{5} \\times 120$|\n|= 24|\n\n</div>\n\n<br>\n\nsm_nogap $\\therefore$ Number of vanilla cupcakes\n\n<div class=\"aligned\">\n\n>>||\n|-|\n|= 120 $-$ 24|\n|= {{{correctAnswer0}}}|\n\n</div>"}]},{"vars":[{"varval":"A lottery awarded 48 cash prizes that were vouchers for $25 or $50.\n\nIf Ronald won a prize, he had a 1 in 3 chance of winning a $50 voucher.\n\nHow many $25 voucher prizes were there?"},{"varval":"sm_nogap Number of $50 vouchers\n\n<div class=\"aligned\">\n\n>>||\n|-|\n|= $\\dfrac{1}{3} \\times 48$|\n|= 16|\n\n</div>\n\n<br>\n\nsm_nogap $\\therefore$ Number of $25 vouchers\n\n<div class=\"aligned\">\n\n>>||\n|-|\n|= 48 $-$ 16|\n|= {{{correctAnswer0}}}|\n\n</div>"}]},{"vars":[{"varval":"A dog breeder has 36 groodle puppies for sale. The puppies are either standard groodles or miniature groodles.\n\nIf Sheena randomly chooses one puppy, she has a 1 in 9 chance of choosing a miniature groodle.\n\nHow many standard groodle puppies does the breeder have for sale?"},{"varval":"sm_nogap Number of miniature groodles\n\n<div class=\"aligned\">\n\n>>||\n|-|\n|= $\\dfrac{1}{9} \\times 36$|\n|= 4|\n\n</div>\n\n<br>\n\nsm_nogap $\\therefore$ Number of standard groodles\n\n<div class=\"aligned\">\n\n>>||\n|-|\n|= 36 $-$ 4|\n|= {{{correctAnswer0}}}|\n\n</div>"}]},{"vars":[{"varval":"A research study of one Powerful Owl found that it killed 72 possums over a 3 month period. The possums it killed were either Honey Possums or Brushtail Possums.\n\nIf the researcher looked at any individual kill, there was a 1 in 8 chance of it being a Honey Possum.\n\nHow many Brushtail Possums did the Powerful Owl kill over the course of this study?"},{"varval":"sm_nogap Number of Honey Possums\n\n<div class=\"aligned\">\n\n>>||\n|-|\n|= $\\dfrac{1}{8} \\times 72$|\n|= 9|\n\n</div>\n\n<br>\n\nsm_nogap $\\therefore$ Number of Brushtail Possums\n\n<div class=\"aligned\">\n\n>>||\n|-|\n|= 72 $-$ 9|\n|= {{{correctAnswer0}}}|\n\n</div>"}]},{"vars":[{"varval":"Jessie caught 24 canetoads. Some were male and the rest were female.\n\nIf Jessie randomly inspected one of the canetoads, it had a 1 in 3 chance of being a male.\n\nHow many female canetoads did Jessie catch?"},{"varval":"sm_nogap Number of males\n\n<div class=\"aligned\">\n\n>>||\n|-|\n|= $\\dfrac{1}{3} \\times 24$|\n|= 8|\n\n</div>\n\n<br>\n\nsm_nogap $\\therefore$ Number of females\n\n<div class=\"aligned\">\n\n>>||\n|-|\n|= 24 $-$ 8|\n|= {{{correctAnswer0}}}|\n\n</div>"}]}]

  678. <div class="sm_mode"> Scientists have discovered that light travels at approximately 290 000 kilometres per second. Which estimate, in kilometres, is closest to the distance that light travels in **two** minutes? </div>

    [{"vars":null}]

  679. <div class="sm_mode"> Jeff walks across a football field. It takes him 48 strides to walk across. His stride length is 80 centimetres long. Jeff's daughter, Deloris, has a stride length of 60 centimetres. How many strides does it take Deloris to walk across the same football field? </div>

    [{"vars":null}]

  680. <div class="sm_mode"> Chandler, Ross and Joey are picking grapes to earn money. Their pay is based on the number of tonnes of grapes they pick. <br> <div class="sm-table col1-color4 row1-color4 top-left-cell-hidden"> >>| | Tonnes picked| |:-:|:-:| | Chandler| 2| | Ross | 3| | Joey | 4| </div> <br>Their total pay is $1080. How much does Chandler earn? </div>

    [{"vars":null}]

  681. <div class="sm_mode"> Dianne was making a scale drawing of a pipeline on grid paper. The full length of the pipeline is drawn to scale on the grid below. It consists of 6 identical sections joined together (not shown on the diagram). sm_img https://teacher.smartermaths.com.au/wp-content/uploads/2018/07/NAPX-H3-NC27-SA.svg 400 indent vpad What is the length of each section of pipeline? </div>

    [{"vars":null}]

  682. <div class="sm_mode"> Ten nails have a mass of 18 grams. <br> sm_img https://teacher.smartermaths.com.au/wp-content/uploads/2018/08/NAPX-G3-CA26-SA.png 450 indent3 vpad <br>How many nails would have a mass of 540 grams? </div>

    [{"vars":null}]

  683. <div class="sm_mode"> Columbo had a full bucket of water. <br> sm_img https://teacher.smartermaths.com.au/wp-content/uploads/2017/01/NAP-E4-NC1311-293x300.png 150 indent vpad <br>He put two holes in the bottom and the water leaked out. After a few minutes, Columbo closed off one of the holes in the bucket and the water poured out more slowly. Which graph below best shows the depth of water in the bucket against time? </div>

    [{"vars":null}]

  684. <div class="sm_mode"> sm_img https://teacher.smartermaths.com.au/wp-content/uploads/2021/03/NAPX9-TLE-35-1-768x136.png 480 indent vpad The distance from A to B is two times longer than the distance from B to C. The distance from A to C is 210 metres. What is the distance from A to B in metres? </div>

    [{"vars":null}]

  685. <div class="sm_mode"> A hospital purchases 24 litres of hand sanitizer at a total cost of $99.60. The hand sanitiser can also be purchased in litre bottles at the same price per litre. How much would 7 litres of sanitiser cost? </div>

    [{"vars":null}]

  686. <div class="sm_mode"> {{{question}}} </div>

    [{"vars":[{"varval":"The sale price of a second hand car is $4800.\n\nZilda buys the car for 25% off the sale price.\n\nZilda has a discount voucher that gives her a further $240 off the sale price.\n\nWhat percentage of the original price does Zilda pay for the car?"},{"varval":"<div class=\"aligned\">\n\n| | |\n| --------------------- | -------------- |\n| Sale price | \\= $4800 − (25% × 4800) |\n| | \\= 75% × 4800 |\n|| \\= $3600|\n\n</div>\n\n<br>\n\n<div class=\"aligned\">\n\n| | |\n| --------------------- | -------------- |\n| Purchase price | \\= 3600 − 240 |\n| | \\= $3360 |\n\n</div>\n\n<br>\n\r\n\nsm_nogap $\\therefore$ Percentage of original price\n\n<div class=\"aligned\">\n\n>>||\n|-|\n|= $\\dfrac{3360}{4800} \\times 100$|\n|= {{{correctAnswer0}}}{{{suffix0}}}|\n\n</div>"}]},{"vars":[{"varval":"The sale price of an electric guitar is $600.\n\nKerry buys the guitar for 25% off the sale price.\n\nKerry has a discount voucher that gives her a further $90 off the sale price.\n\nWhat percentage of the original price does Kerry pay for the guitar?"},{"varval":"<div class=\"aligned\">\n\n| | |\n| --------------------- | -------------- |\n| Sale price | \\= 600 − (25% × 600) |\n| | \\= 600 $-$ 150 |\n|| \\= $450|\n\n</div>\n\n<br>\n\n<div class=\"aligned\">\n\n| | |\n| --------------------- | -------------- |\n| Purchase price | \\= 450 − 90 |\n| | \\= $360 |\n\n</div>\n\n<br>\n\r\n\nsm_nogap $\\therefore$ Percentage of original price\n\n<div class=\"aligned\">\n\n>>||\n|-|\n|= $\\dfrac{360}{600} \\times 100$|\n|= {{{correctAnswer0}}}{{{suffix0}}}|\n\n</div>"}]},{"vars":[{"varval":"The price of a hamburger is $12.00.\n\nOn Tuesday at lunchtime, all customers receive 25% off any order.\n\nLlandra has a discount voucher that gives her an extra $1.80 off.\n\nWhat percentage of the original hamburger price does Llandra pay?"},{"varval":"<div class=\"aligned\">\n\n| | |\n| --------------------- | -------------- |\n| Sale price | \\= $12 − (25% × 12) |\n| | \\= $12 - 3$ |\n|| \\= $9.00|\n\n</div>\n\n<br>\n\n<div class=\"aligned\">\n\n| | |\n| --------------------- | -------------- |\n| Purchase price | \\= 9.00 $−$ 1.80 |\n| | \\= $7.20 |\n\n</div>\n\n<br>\n\r\n\nsm_nogap $\\therefore$ Percentage of original price\n\n<div class=\"aligned\">\n\n>>||\n|-|\n|= $\\dfrac{7.20}{12.00} \\times 100$|\n|= $\\dfrac{6 \\times\\ 1.20}{10 \\times\\ 1.20} \\times 100$|\n|= {{{correctAnswer0}}}{{{suffix0}}}|\n\n</div>"}]},{"vars":[{"varval":"The sale price of a trampoline is $2400.\n\nBevan buys the trampoline for 25% off the sale price.\n\nBevan has a store voucher that gives him a further $840 off the sale price.\n\nWhat percentage of the original price does Bevan pay for the trampoline?"},{"varval":"<div class=\"aligned\">\n\n| | |\n| --------------------- | -------------- |\n| Sale price | \\= $2400 − (25% × 2400) |\n| | \\= 2400 $-$ 600 |\n|| \\= $1800|\n\n</div>\n\n<br>\n\n<div class=\"aligned\">\n\n| | |\n| --------------------- | -------------- |\n| Purchase price | \\= 1800 $−$ 840 |\n| | \\= $960 |\n\n</div>\n\n<br>\n\r\n\nsm_nogap $\\therefore$ Percentage of original price\n\n<div class=\"aligned\">\n\n>>||\n|-|\n|= $\\dfrac{960}{2400} \\times 100$|\n|= $\\dfrac{4 \\times 240}{10 \\times 240} \\times 100$|\n|= {{{correctAnswer0}}}{{{suffix0}}}|\n\n</div>"}]},{"vars":[{"varval":"The sale price of a Labradoodle puppy is $3500.\n\nThe pet store gives Veronica 10% off the sale price.\n\nVeronica has a pet store discount voucher that gives her a further $350 off the sale price.\n\nWhat percentage of the original price does Veronica pay for the Labradoodle puppy?"},{"varval":"<div class=\"aligned\">\n\n| | |\n| --------------------- | -------------- |\n| Sale price | \\= $3500 − (10% × 3500) |\n| | \\= 3500 $-$ 350 |\n|| \\= $3150|\n\n</div>\n\n<br>\n\n<div class=\"aligned\">\n\n| | |\n| --------------------- | -------------- |\n| Purchase price | \\= 3150 − 350 |\n| | \\= $2800 |\n\n</div>\n\n<br>\n\r\n\nsm_nogap $\\therefore$ Percentage of original price\n\n<div class=\"aligned\">\n\n>>||\n|-|\n|= $\\dfrac{2800}{3500} \\times 100$|\n|= $\\dfrac{4 \\times 700}{5 \\times 700} \\times 100$|\n|= $\\dfrac{4}{5} \\times 100$|\n|= {{{correctAnswer0}}}{{{suffix0}}}|\n\n</div>"}]},{"vars":[{"varval":"The sale price of a parrot is $400.\n\nThe pet store gives Albert 25% off the sale price.\n\nAlbert has a pet store discount voucher that gives him a further $60 off the sale price.\n\nWhat percentage of the original price does Albert pay for the parrot?"},{"varval":"<div class=\"aligned\">\n\n| | |\n| --------------------- | -------------- |\n| Sale price | \\= $400 − (25% × 400) |\n| | \\= 400 $-$ 100 |\n|| \\= $300|\n\n</div>\n\n<br>\n\n<div class=\"aligned\">\n\n| | |\n| --------------------- | -------------- |\n| Purchase price | \\= 300 − 60 |\n| | \\= $240 |\n\n</div>\n\n<br>\n\r\n\nsm_nogap $\\therefore$ Percentage of original price\n\n<div class=\"aligned\">\n\n>>||\n|-|\n|= $\\dfrac{240}{400} \\times 100$|\n|= $\\dfrac{6 \\times 40}{10 \\times 40} \\times 100$|\n|= $\\dfrac{6}{10} \\times 100$|\n|= {{{correctAnswer0}}}{{{suffix0}}}|\n\n</div>"}]}]

  687. <div class="sm_mode"> {{{question}}} </div>

    [{"vars":[{"varval":"A candle maker increases the price of a candle from $20 to $25.\n\n\r\n\r\nWhat is the percentage increase in his price?\n"},{"varval":"sm_nogap Price increase = 25 $−$ 20 = $5\n\n<div class=\"aligned\">\n\n| | |\n| --------------------- | -------------- |\n| $\\therefore$ Percentage increase | \\= $\\dfrac{5}{20}$ × 100 |\n| | \\= 0.25 × 100 |\n|| \\= {{{correctAnswer0}}}{{{suffix0}}}|\n\n</div>\n"}]},{"vars":[{"varval":"The price of a new car increases from $40 000 to $45 000.\n\nWhat is the percentage increase in the price?"},{"varval":"sm_nogap Price increase = 45 000 $−$ 40 000 = $5 000\n\n<div class=\"aligned\">\n\n| | |\n| --------------------- | -------------- |\n| $\\therefore$ Percentage increase | \\= $\\dfrac{5000}{40000}$ × 100 |\n| | \\= 0.125 × 100 |\n|| \\= {{{correctAnswer0}}}{{{suffix0}}}|\n\n</div>"}]},{"vars":[{"varval":"A coffee shop increases the price of a caramel latte from $4.80 to $6.00.\n\n\r\n\r\nWhat is the percentage increase in the price?\n"},{"varval":"sm_nogap Price increase = 6 $−$ 4.80 = $1.20\n\n<div class=\"aligned\">\n\n| | |\n| --------------------- | -------------- |\n| $\\therefore$ Percentage increase | \\= $\\dfrac{1.2}{4.8}$ × 100 |\n| | \\= 0.25 × 100 |\n|| \\= {{{correctAnswer0}}}{{{suffix0}}}|\n\n</div>\n"}]},{"vars":[{"varval":"A clothing store increases the price of a pair of jeans from $85 to $93.50.\n\nWhat is the percentage increase in the price?"},{"varval":"sm_nogap Price increase = 93.50 $−$ 85.00 = $8.50\n\n<div class=\"aligned\">\n\n| | |\n| --------------------- | -------------- |\n| $\\therefore$ Percentage increase | \\= $\\dfrac{8.5}{85}$ × 100 |\n| | \\= 0.1 × 100 |\n|| \\= {{{correctAnswer0}}}{{{suffix0}}}|\n\n</div>\n"}]},{"vars":[{"varval":"A hamburger increases in price from $7.80 to $8.19.\n\nWhat is the percentage increase in price?"},{"varval":"sm_nogap Price increase = 8.19 $−$ 7.80 = $0.39\n\n<div class=\"aligned\">\n\n| | |\n| --------------------- | -------------- |\n| $\\therefore$ Percentage increase | \\= $\\dfrac{0.39}{7.8}$ × 100 |\n| | \\= 0.05 × 100 |\n|| \\= {{{correctAnswer0}}}{{{suffix0}}}|\n\n</div>\n"}]},{"vars":[{"varval":"A price of a new caravan has increased from $75 480 to $84 537.60\n\nWhat is the percentage increase in price?"},{"varval":"sm_nogap Price increase = 84 537.60 $−$ 75 480 = $9057.60\n\n<div class=\"aligned\">\n\n| | |\n| --------------------- | -------------- |\n| $\\therefore$ Percentage increase | \\= $\\dfrac{9057.60}{75 480}$ × 100 |\n| | \\= 0.12 × 100 |\n|| \\= {{{correctAnswer0}}}{{{suffix0}}}|\n\n</div>\n"}]}]

  688. <div class="sm_mode"> {{{question}}} </div>

    [{"vars":[{"varval":"A loaf of bread is on sale with a 15% discount.\n\n\r\n\r\nIn the last hour of trading, the bread is reduced a further 20% on the already discounted price.\n\n\r\n\r\nWhat is the overall percentage discount on the bread?\n"},{"varval":"Strategy 1\n\n\r\nAssume the bread costs $1.00 (say)\n\n\r\nCost after 15% discount = $0.85\n\n\r\nsm_nogap Cost after further 20% discount\n\n<div class=\"aligned\">\n\n>>||\n|-|\n|= 0.85 − (20% × 0.85)|\n|= 0.85 − 0.17|\n|= $0.68|\n\r\n\r\n</div>\n\n<br>\n\n<div class=\"aligned\">\n\n| | |\n| --------------------- | -------------- |\n| $\\therefore$ Overall discount | \\= (1 − 0.68) × 100 |\n| | \\= {{{correctAnswer}}} |\n\n</div>\n\n<br>\n\r\n\r\r\n\r\n\nsm_nogap Strategy 2 (advanced)\n\n<div class=\"aligned\">\n\n| | |\n| --------------------- | -------------- |\n| Overall discount | \\= 1 − (0.85 × 0.80) |\n| | \\= 1 − 0.68 |\n|| \\= 0.32|\n|| \\= {{{correctAnswer}}}|\n\n</div>"}]},{"vars":[{"varval":"A container of salad is on sale with a 10% discount.\n\n\r\n\r\nOne hour before the supermarket closes, the salad is reduced a further 20% on the already discounted price.\n\n\r\n\r\nWhat is the overall percentage discount on the salad?\n"},{"varval":"Strategy 1\n\n\r\nAssume the salad costs $1.00 (say)\n\n\r\nCost after 10% discount = $0.90\n\n\r\nsm_nogap Cost after further 20% discount\n\n<div class=\"aligned\">\n\n>>||\n|-|\n|= 0.90 − (20% × 0.90)|\n|= 0.90 − 0.18|\n|= $0.72|\n\r\n\r\n</div>\n\n<br>\n\n<div class=\"aligned\">\n\n| | |\n| --------------------- | -------------- |\n| $\\therefore$ Overall discount | \\= (1 − 0.72) × 100 |\n| | \\= {{{correctAnswer}}} |\n\n</div>\n\n<br>\n\r\n\r\r\n\r\n\nsm_nogap Strategy 2 (advanced)\n\n<div class=\"aligned\">\n\n| | |\n| --------------------- | -------------- |\n| Overall discount | \\= 1 − (0.90 × 0.80) |\n| | \\= 1 − 0.72 |\n|| \\= 0.28|\n|| \\= {{{correctAnswer}}}|\n\n</div>"}]},{"vars":[{"varval":"A car is offered for sale with a 15% discount.\n\n\r\n\r\nIn the end of financial year sale, the car is reduced a further 10% on the already discounted price.\n\n\r\n\r\nWhat is the overall percentage discount on the car?\n"},{"varval":"Strategy 1\n\n\r\nAssume the car costs $1.00 (say)\n\n\r\nCost after 15% discount = $0.85\n\n\r\nsm_nogap Cost after further 10% discount\n\n<div class=\"aligned\">\n\n>>||\n|-|\n|= 0.85 − (10% × 0.85)|\n|= 0.85 − 0.085|\n|= $0.765|\n\r\n\r\n</div>\n\n<br>\n\n<div class=\"aligned\">\n\n| | |\n| --------------------- | -------------- |\n| $\\therefore$ Overall discount | \\= (1 − 0.765) × 100 |\n| | \\= {{{correctAnswer}}} |\n\n</div>\n\n<br>\n\r\n\r\r\n\r\n\nsm_nogap Strategy 2 (advanced)\n\n<div class=\"aligned\">\n\n| | |\n| --------------------- | -------------- |\n| Overall discount | \\= 1 − (0.85 × 0.90) |\n| | \\= 1 − 0.765 |\n|| \\= 0.235|\n|| \\= {{{correctAnswer}}}|\n\n</div>"}]},{"vars":[{"varval":"A punnet of blueberries is on sale with a 20% discount.\n\n\r\n\r\nIn the last hour of trading, the blueberries are reduced a further 20% on the already discounted price.\n\n\r\n\r\nWhat is the overall percentage discount on the blueberries?\n"},{"varval":"Strategy 1\n\n\r\nAssume the blueberries cost $1.00 (say)\n\n\r\nCost after 20% discount = $0.80\n\n\r\nsm_nogap Cost after further 20% discount\n\n<div class=\"aligned\">\n\n>>||\n|-|\n|= 0.80 − (20% × 0.80)|\n|= 0.80 − 0.16|\n|= $0.64|\n\r\n\r\n</div>\n\n<br>\n\n<div class=\"aligned\">\n\n| | |\n| --------------------- | -------------- |\n| $\\therefore$ Overall discount | \\= (1 − 0.64) × 100 |\n| | \\= {{{correctAnswer}}} |\n\n</div>\n\n<br>\n\r\n\r\r\n\r\n\nsm_nogap Strategy 2 (advanced)\n\n<div class=\"aligned\">\n\n| | |\n| --------------------- | -------------- |\n| Overall discount | \\= 1 − (0.80 × 0.80) |\n| | \\= 1 − 0.64 |\n|| \\= 0.36|\n|| \\= {{{correctAnswer}}}|\n\n</div>"}]},{"vars":[{"varval":"A gas heater is on sale with a 15% discount.\n\n\r\n\r\nAt the end of winter, the heater is reduced a further 25% on the already discounted price.\n\n\r\n\r\nWhat is the overall percentage discount on the heater?\n"},{"varval":"Strategy 1\n\n\r\nAssume the heater costs $1.00 (say)\n\n\r\nCost after 15% discount = $0.85\n\n\r\nsm_nogap Cost after further 25% discount\n\n<div class=\"aligned\">\n\n>>||\n|-|\n|= 0.85 − (25% × 0.85)|\n|= 0.85 − 0.2125|\n|= $0.6375|\n\r\n\r\n</div>\n\n<br>\n\n<div class=\"aligned\">\n\n| | |\n| --------------------- | -------------- |\n| $\\therefore$ Overall discount | \\= (1 − 0.6375) × 100 |\n| | \\= {{{correctAnswer}}} |\n\n</div>\n\n<br>\n\r\n\r\r\n\r\n\nsm_nogap Strategy 2 (advanced)\n\n<div class=\"aligned\">\n\n| | |\n| --------------------- | -------------- |\n| Overall discount | \\= 1 − (0.85 × 0.75) |\n| | \\= 1 − 0.6375 |\n|| \\= 0.3625|\n|| \\= {{{correctAnswer}}}|\n\n</div>"}]},{"vars":[{"varval":"A fan is on sale with a 12% discount.\n\n\r\n\r\nAt the end of summer, the fan is reduced a further 16% on the already discounted price.\n\n\r\n\r\nWhat is the overall percentage discount on the heater?\n"},{"varval":"Strategy 1\n\n\r\nAssume the fan costs $1.00 (say)\n\n\r\nCost after 12% discount = $0.88\n\n\r\nsm_nogap Cost after further 16% discount\n\n<div class=\"aligned\">\n\n>>||\n|-|\n|= 0.88 − (16% × 0.88)|\n|= 0.88 − 0.1408|\n|= $0.7392|\n\r\n\r\n</div>\n\n<br>\n\n<div class=\"aligned\">\n\n| | |\n| --------------------- | -------------- |\n| $\\therefore$ Overall discount | \\= (1 − 0.7392) × 100 |\n| | \\= {{{correctAnswer}}} |\n\n</div>\n\n<br>\n\r\n\r\r\n\r\n\nsm_nogap Strategy 2 (advanced)\n\n<div class=\"aligned\">\n\n| | |\n| --------------------- | -------------- |\n| Overall discount | \\= 1 − (0.88 × 0.84) |\n| | \\= 1 − 0.7392 |\n|| \\= 0.2608|\n|| \\= {{{correctAnswer}}}|\n\n</div>"}]}]

  689. <div class="sm_mode"> {{{question}}} </div>

    [{"vars":[{"varval":"Blart bought a second hand bike at a cost of $440.\n\n\r\n\r\nHe had to spend 55% of his savings to buy the bike.\n\n\r\n\r\nHow much did Blart have in his savings after buying the bike?\n"},{"varval":"sm_nogap 55% × Savings = $440\n\n<div class=\"aligned\">\n\n| | |\n| --------------------- | -------------- |\n| Savings | \\= $\\dfrac{440}{0.55}$ |\n| | \\= $800 |\n\n</div>\n\n<br>\n\n<div class=\"aligned\">\n\n| | |\n| --------------------- | -------------- |\n| $\\therefore$ Savings left | \\= 800 $-$ 440|\n| | \\= {{{prefix0}}}{{{correctAnswer0}}} |\n\n</div>\n"}]},{"vars":[{"varval":"Veronica bought a second hand car at a cost of $12 840.\n\n\r\n\r\nShe had to spend 60% of her savings to buy the car.\n\n\r\n\r\nHow much did Veronica have in her savings after buying the car?\n"},{"varval":"sm_nogap 60% × Savings = $12 840\n\n<div class=\"aligned\">\n\n| | |\n| --------------------- | -------------- |\n| Savings | \\= $\\dfrac{12\\ 840}{0.60}$ |\n| | \\= $21 400 |\n\n</div>\n\n<br>\n\n<div class=\"aligned\">\n\n| | |\n| --------------------- | -------------- |\n| $\\therefore$ Savings left | \\= 21 400 $-$ 12 840|\n| | \\= {{{prefix0}}}{{{correctAnswer0}}} |\n\n</div>\n"}]},{"vars":[{"varval":"Joseph bought a coat at a cost of $395.\n\n\r\n\r\nHe had to spend 40% of his savings to buy the coat.\n\n\r\n\r\nHow much did Joseph have in his savings after buying the coat?\n"},{"varval":"sm_nogap 40% × Savings = $395\n\n<div class=\"aligned\">\n\n| | |\n| --------------------- | -------------- |\n| Savings | \\= $\\dfrac{395}{0.40}$ |\n| | \\= $987.50 |\n\n</div>\n\n<br>\n\n<div class=\"aligned\">\n\n| | |\n| --------------------- | -------------- |\n| $\\therefore$ Savings left | \\= 987.50 $-$ 395|\n| | \\= {{{prefix0}}}{{{correctAnswer0}}} |\n\n</div>\n"}]},{"vars":[{"varval":"Logan bought an Xbox console at a cost of $449.\n\n\r\n\r\nHe had to spend 20% of his savings to buy the Xbox.\n\n\r\n\r\nHow much did Logan have in his savings after buying the Xbox?"},{"varval":"sm_nogap 20% × Savings = $449\n\n<div class=\"aligned\">\n\n| | |\n| --------------------- | -------------- |\n| Savings | \\= $\\dfrac{449}{0.20}$ |\n| | \\= $2245|\n\n</div>\n\n<br>\n\n<div class=\"aligned\">\n\n| | |\n| --------------------- | -------------- |\n| $\\therefore$ Savings left | \\= 2245 $-$ 449|\n| | \\= {{{prefix0}}}{{{correctAnswer0}}} |\n\n</div>\n"}]},{"vars":[{"varval":"Davinia bought an Alaskan malamute puppy at a cost of $3200.\n\n\r\n\r\nShe had to spend 20% of her savings to buy the puppy.\n\n\r\n\r\nHow much did Davinia have in her savings after buying the puppy?\n"},{"varval":"sm_nogap 20% × Savings = $3200\n\n<div class=\"aligned\">\n\n| | |\n| --------------------- | -------------- |\n| Savings | \\= $\\dfrac{3200}{0.20}$ |\n| | \\= $16 000|\n\n</div>\n\n<br>\n\n<div class=\"aligned\">\n\n| | |\n| --------------------- | -------------- |\n| $\\therefore$ Savings left | \\= 16 000 $-$ 3200|\n| | \\= $12 800 |\n\n</div>\n"}]}]

  690. <div class="sm_mode"> {{{question}}} </div>

    [{"vars":[{"varval":"A dog breeder records the number of puppies born in litters to four of her dogs.\n\nShe also records the percentage of puppies in each litter that were sick.\n\n<br>\n\n<div class=\"sm-table row1-color3\">\n\n>>| Dogs | Number of puppies | Percentage sick |\n|:-:|:-:|:-:|\n| 1 | 12| 25%|\n| 2 | 4| 50%|\n| 3 | 5| 20%|\n| 4 | 4| 50%|\n\n</div>\n\n<br>\n\nWhat percentage of the total number of puppies were sick?\n"},{"varval":"sm_nogap Total sick puppies\n\n<div class=\"aligned\">\n\n>>||\n|-|\n|= 12 × 0.25 + 4 × 0.5 + 5 × 0.2 + 4 × 0.5|\n|= 3 + 2 + 1 + 2|\n|= 8|\n\n</div>\n\nsm_nogap $\\therefore$ Percentage of sick puppies\n\n<div class=\"aligned\">\n\n>>||\n|-|\n|= $\\dfrac{\\text{number of sick puppies}}{\\text{total puppies}} \\times 100$|\n|||\n|= $\\dfrac{8}{(12 + 4 + 5 + 4)} \\times 100$|\n|||\n|= $\\dfrac{8}{25} \\times 100$|\n|= {{{correctAnswer0}}}{{{suffix0}}}|\n\n</div>"}]},{"vars":[{"varval":"A dog breeder records the number of puppies born in litters to four of her dogs.\n\n\r\n\r\nShe also records the percentage of puppies in each litter that were sick.\n\n<br>\n\n<div class=\"sm-table row1-color3\">\n\n>>| Dogs | Number of puppies | Percentage sick |\n|:-:|:-:|:-:|\n| 1 | 12| 25%|\n| 2 | 4| 50%|\n| 3 | 5| 20%|\n| 4 | 4| 50%|\n\n</div>\n\n<br>\n\nWhat percentage of the total number of puppies were NOT sick?\n"},{"varval":"sm_nogap Total puppies\n\n<div class=\"aligned\">\n\n>>||\n|-|\n|= 12 + 4 + 5 + 4|\n|= 25|\n\n</div>\n\nsm_nogap Total sick puppies\n\n<div class=\"aligned\">\n\n>>||\n|-|\n|= 12 × 0.25 + 4 × 0.5 + 5 × 0.2 + 4 × 0.5|\n|= 3 + 2 + 1 + 2|\n|= 8|\n\n</div>\n\nsm_nogap $\\therefore$ Number of puppies NOT sick\n\n<div class=\"aligned\">\n\n>>||\n|-|\n|= 25 $-$ 8|\n|= 17|\n\n</div>\n\nsm_nogap $\\therefore$ Percentage of puppies NOT sick\n\n<div class=\"aligned\">\n\n>>||\n|-|\n|= $\\dfrac{\\text{number of puppies NOT sick}}{\\text{total puppies}} \\times 100$|\n|||\n|= $\\dfrac{17}{25} \\times 100$|\n|||\n|= {{{correctAnswer0}}}{{{suffix0}}}|\n\n</div>"}]},{"vars":[{"varval":"A farmer records the number of eggs laid by 4 chickens in his coup in a week.\n\nHe also records the percentage of eggs laid that were fertilised.\n\n<br>\n\n<div class=\"sm-table row1-color2\">\n\n>>| Chicken | Eggs laid | Percentage fertilised |\n|:-:|:-:|:-:|\n| 1 | 5| 20%|\n| 2 | 6| 50%|\n| 3 | 4| 75%|\n| 4 | 10| 30%|\n\n</div>\n\n<br>\n\nWhat percentage of the total number of eggs were fertilised?"},{"varval":"sm_nogap Total eggs\n\n<div class=\"aligned\">\n\n>>||\n|-|\n|= 5 + 6 + 4 + 10|\n|= 25|\n\n</div>\n\nsm_nogap Total eggs fertilised\n\n<div class=\"aligned\">\n\n>>||\n|-|\n|= 5 × 0.2 + 6 × 0.5 + 4 × 0.75 + 10 × 0.3|\n|= 1 + 3 + 3 + 3|\n|= 10|\n\n</div>\n\n\nsm_nogap $\\therefore$ Percentage of eggs fertilised\n\n<div class=\"aligned\">\n\n>>||\n|-|\n|= $\\dfrac{\\text{eggs fertilised}}{\\text{total eggs}} \\times 100$|\n|||\n|= $\\dfrac{10}{25} \\times 100$|\n|||\n|= {{{correctAnswer0}}}{{{suffix0}}}|\n\n</div>"}]},{"vars":[{"varval":"Three Ridgeback dog breeders record the number of Ridgeback puppies they sell in a month.\n\nThe breeders also record the percentage of the puppies sold that are liver-nosed.\n\n<br>\n\n<div class=\"sm-table row1-color2\">\n\n>>| Breeder | Ridgeback puppies sold | Percentage liver-nosed |\n|:-:|:-:|:-:|\n| 1 | 8| 75%|\n| 2 | 2| 50%|\n| 3 | 15| 60%|\n\n</div>\n\n<br>\n\nWhat percentage of the total number of puppies sold were liver-nosed?"},{"varval":"sm_nogap Total Ridgeback puppies\n\n<div class=\"aligned\">\n\n>>||\n|-|\n|= 8 + 2 + 15|\n|= 25|\n\n</div>\n\nsm_nogap Total liver-nosed puppies\n\n<div class=\"aligned\">\n\n>>||\n|-|\n|= 8 × 0.75 + 2 × 0.5 + 15 × 0.6|\n|= 6 + 1 + 9|\n|= 16|\n\n</div>\n\n\nsm_nogap $\\therefore$ Percentage of liver-nosed\n\n<div class=\"aligned\">\n\n>>||\n|-|\n|= $\\dfrac{\\text{total liver-nosed}}{\\text{total puppies}} \\times 100$|\n|||\n|= $\\dfrac{16}{25} \\times 100$|\n|||\n|= {{{correctAnswer0}}}{{{suffix0}}}|\n\n</div>"}]},{"vars":[{"varval":"A researcher records the number of koalas she tags in three expeditions.\n\nShe also records the percentage of tagged koalas that were female.\n\n<br>\n\n<div class=\"sm-table row1-color8\">\n\n>>| Expedition | Koalas tagged | Percentage female |\n|:-:|:-:|:-:|\n| 1 | 8| 25%|\n| 2 | 2| 50%|\n| 3 | 10| 40%|\n\n</div>\n\n<br>\n\nWhat percentage of the total number of tagged koalas were female?"},{"varval":"sm_nogap Total koalas\n\n<div class=\"aligned\">\n\n>>||\n|-|\n|= 8 + 2 + 10|\n|= 20|\n\n</div>\n\nsm_nogap Total female koalas\n\n<div class=\"aligned\">\n\n>>||\n|-|\n|= 8 × 0.25 + 2 × 0.5 + 10 × 0.4|\n|= 2 + 1 + 4|\n|= 7|\n\n</div>\n\n\nsm_nogap $\\therefore$ Percentage of females\n\n<div class=\"aligned\">\n\n>>||\n|-|\n|= $\\dfrac{\\text{total females}}{\\text{total koalas}} \\times 100$|\n|||\n|= $\\dfrac{7}{20} \\times 100$|\n|||\n|= {{{correctAnswer0}}}{{{suffix0}}}|\n\n</div>"}]},{"vars":[{"varval":"A researcher records the number of elephants in the largest 3 zoos in his country.\n\nHe also records the percentage of the elephants in each zoo that were African.\n\n<br>\n\n<div class=\"sm-table row1-color8\">\n\n>>| Zoo | Number of elephants | Percentage of African elephants |\n|:-:|:-:|:-:|\n| 1 | 12| 25%|\n| 2 | 4| 75%|\n| 3 | 4| 50%|\n\n</div>\n\n<br>\n\nWhat percentage of the total number of elephants are African?"},{"varval":"sm_nogap Total elephants\n\n<div class=\"aligned\">\n\n>>||\n|-|\n|= 12 + 4 + 4|\n|= 20|\n\n</div>\n\nsm_nogap Total African elephants\n\n<div class=\"aligned\">\n\n>>||\n|-|\n|= 12 × 0.25 + 4 × 0.75 + 4 × 0.5|\n|= 3 + 3 + 2|\n|= 8|\n\n</div>\n\n\nsm_nogap $\\therefore$ Percentage of African elephants\n\n<div class=\"aligned\">\n\n>>||\n|-|\n|= $\\dfrac{\\text{total African elphants}}{\\text{total elephants}} \\times 100$|\n|||\n|= $\\dfrac{8}{20} \\times 100$|\n|||\n|= {{{correctAnswer0}}}{{{suffix0}}}|\n\n</div>"}]}]

  691. <div class="sm_mode"> {{{question}}} </div>

    [{"vars":[{"varval":"Candy is saving her money to buy some new riding boots.\n\n\r\n\r\nAfter 1 week, she had saved 35% of the cost of the boots.\n\n\r\n\r\nAfter 2 weeks, she had saved another $60 and now had 65% of the cost of the boots.\n\n\r\n\r\nHow much do the riding boots cost?\n"},{"varval":"<div class=\"aligned\">\n\n| | |\n| --------------------: | -------------- |\n| 65% − 35% | \\= $60 |\n| 30% | \\= $60 |\n| 1%| \\= $2|\n\n</div>\n\n<br>\n\n<div class=\"aligned\">\n\n| | |\n| --------------------: | -------------- |\n| $\\therefore$ Cost | \\= 100 × 2 |\n| | \\= {{{prefix0}}}{{{correctAnswer0}}} |\n\n</div>\n"}]},{"vars":[{"varval":"Jonas is saving his money to buy a guitar.\n\n\r\n\r\nAfter 1 month, he had saved 45% of the cost of the guitar.\n\n\r\n\r\nAfter 2 months, he had saved another $350 and now had 70% of the cost of the guitar.\n\n\r\n\r\nHow much does the guitar cost?\n"},{"varval":"<div class=\"aligned\">\n\n| | |\n| --------------------: | -------------- |\n| 70% − 45% | \\= $350 |\n| 25% | \\= $350 |\n| 1%| \\= $14|\n\n</div>\n\n<br>\n\n<div class=\"aligned\">\n\n| | |\n| --------------------: | -------------- |\n| $\\therefore$ Cost | \\= 100 × 14 |\n| | \\= {{{prefix0}}}{{{correctAnswer0}}} |\n\n</div>\n"}]},{"vars":[{"varval":"Brutus is saving his money to buy a campervan.\n\n\r\n\r\nAfter 6 months, he had saved 15% of the cost of the campervan.\n\n\r\n\r\nAfter 12 months, he had saved another $15 400 and now had 85% of the cost of the campervan.\n\n\r\n\r\nHow much does the campervan cost?"},{"varval":"<div class=\"aligned\">\n\n| | |\n| --------------------: | -------------- |\n| 85% − 15% | \\= $15 400 |\n| 70% | \\= $15 400 |\n| 1%| \\= $220|\n\n</div>\n\n<br>\n\n<div class=\"aligned\">\n\n| | |\n| --------------------: | -------------- |\n| $\\therefore$ Cost | \\= 100 × 220 |\n| | \\= $22 000 |\n\n</div>"}]},{"vars":[{"varval":"Jani is saving her money to buy a laptop.\n\n\r\n\r\nAfter 2 months, she had saved 55% of the cost of the laptop.\n\n\r\n\r\nAfter 3 months, she had saved another $480 and now had 95% of the cost of the laptop.\n\n\r\n\r\nHow much does the laptop cost?"},{"varval":"<div class=\"aligned\">\n\n| | |\n| --------------------: | -------------- |\n| 95% − 55% | \\= $480 |\n| 40% | \\= $480 |\n| 1%| \\= $12|\n\n</div>\n\n<br>\n\n<div class=\"aligned\">\n\n| | |\n| --------------------: | -------------- |\n| $\\therefore$ Cost | \\= 100 × 12 |\n| | \\= {{{prefix0}}}{{{correctAnswer0}}} |\n\n</div>"}]},{"vars":[{"varval":"Lucia is saving her money to buy a jet ski.\n\n\r\n\r\nAfter 3 months, she had saved 40% of the cost of the jet ski.\n\n\r\n\r\nAfter 5 months, she had saved another $1200 and now had 60% of the cost of the jet ski.\n\n\r\n\r\nHow much does the jet ski cost?"},{"varval":"<div class=\"aligned\">\n\n| | |\n| --------------------: | -------------- |\n| 60% − 40% | \\= $1200 |\n| 20% | \\= $1200 |\n| 1%| \\= $60|\n\n</div>\n\n<br>\n\n<div class=\"aligned\">\n\n| | |\n| --------------------: | -------------- |\n| $\\therefore$ Cost | \\= 100 × 60 |\n| | \\= {{{prefix0}}}{{{correctAnswer0}}} |\n\n</div>"}]},{"vars":[{"varval":"Thor is saving his money to buy a new hammer.\n\n\r\n\r\nAfter 6 months, he had saved 35% of the cost of the hammer.\n\n\r\n\r\nAfter 1 year, he had saved another $40 000 and now had 55% of the cost of the hammer.\n\n\r\n\r\nHow much does the hammer cost?"},{"varval":"<div class=\"aligned\">\n\n| | |\n| --------------------: | -------------- |\n| 55% − 35% | \\= $40 000 |\n| 20% | \\= $40 000 |\n| 1%| \\= $2000|\n\n</div>\n\n<br>\n\n<div class=\"aligned\">\n\n| | |\n| --------------------: | -------------- |\n| $\\therefore$ Cost | \\= 100 × 2000 |\n| | \\= $200 000 |\n\n</div>"}]}]

  692. <div class="sm_mode"> {{{question}}} </div>

    [{"vars":[{"varval":"Fargo is doing an intensive swim program over the summer holidays.\n\n\r\n\r\nIn his first session, he swims 50 laps.\n\n\r\n\r\nIn his second session, he swims 10% more laps than his first session.\n\n\r\n\r\nIn his third session, he swims 20% more laps than his second session.\n\n\r\n\r\nHow many laps does Fargo swim at his third practice?"},{"varval":"sm_nogap 1st session = 50 laps\n\n<div class=\"aligned\">\n\n| | |\n| --------------------- | -------------- |\n| 2nd session\t | \\= 50 + (10% × 50) |\n| | \\= 50 + 5 |\n|| \\= 55 laps|\n\n</div>\n\n<br>\n\n<div class=\"aligned\">\n\n| | |\n| --------------------- | -------------- |\n| 3rd session\t | \\= 55 + (20% × 55) |\n| | \\= 55 + 11 |\n|| \\= {{{correctAnswer0}}} laps|\n\n</div>\n"}]},{"vars":[{"varval":"Sandra is doing an intensive running program over the summer holidays.\n\n\r\n\r\nIn her first session, she runs 15 kilometres.\n\n\r\n\r\nIn her second session, she runs 15% more kilometres than her first session.\n\n\r\n\r\nIn her third session, she runs 20% more kilometres than her second session.\n\n\r\n\r\nHow many kilometres does Sandra run at her third session?"},{"varval":"sm_nogap 1st session = 15 kilometres\n\n<div class=\"aligned\">\n\n| | |\n| --------------------- | -------------- |\n| 2nd session\t | \\= 15 + (15% × 15) |\n| | \\= 15 + 2.25 |\n|| \\= 17.25 kilometres|\n\n</div>\n\n<br>\n\n<div class=\"aligned\">\n\n| | |\n| --------------------- | -------------- |\n| 3rd session\t | \\= 17.25 + (20% × 17.25) |\n| | \\= 17.25 + 3.45 |\n|| \\= {{{correctAnswer0}}} kilometres|\n\n</div>\n"}]},{"vars":[{"varval":"Jamieson is doing a 3 day walk for charity at the local park.\n\n\r\n\r\nOn his first day, he walks 120 laps of the park.\n\n\r\n\r\nOn his second day, he walks 25% more laps than on the first day.\n\n\r\n\r\nOn his third day, he walks 10% more laps than on the second day.\n\n\r\n\r\nHow many laps does Jamieson walk on the third day?"},{"varval":"sm_nogap 1st day = 120 laps\n\n<div class=\"aligned\">\n\n| | |\n| --------------------- | -------------- |\n| 2nd day\t | \\= 120 + (25% × 120) |\n| | \\= 120 + 30 |\n|| \\= 150 laps|\n\n</div>\n\n<br>\n\n<div class=\"aligned\">\n\n| | |\n| --------------------- | -------------- |\n| 3rd day\t | \\= 150 + (10% × 150) |\n| | \\= 150 + 15 |\n|| \\= {{{correctAnswer0}}} laps|\n\n</div>\n"}]},{"vars":[{"varval":"Kale is raising money for cancer research over a 3 day period.\n\n\r\n\r\nOn his first day, he raises $3200.\n\n\r\n\r\nOn his second day, he raises 30% more money than on the first day.\n\n\r\n\r\nOn his third day, he raises 40% more money than on the second day.\n\n\r\n\r\nHow much money does Kale raise on the third day?"},{"varval":"sm_nogap 1st day = $3200\n\n<div class=\"aligned\">\n\n| | |\n| --------------------- | -------------- |\n| 2nd day\t | \\= $3200 + (30% × $3200) |\n| | \\= $3200 + $960 |\n|| \\= $4160|\n\n</div>\n\n<br>\n\n<div class=\"aligned\">\n\n| | |\n| --------------------- | -------------- |\n| 3rd day\t | \\= $4160 + (40% × $4160) |\n| | \\= $4160 + $1664 |\n|| \\= ${{{correctAnswer0}}}|\n\n</div>\n"}]},{"vars":[{"varval":"Buddy the border collie is competing in sheep dog trials over a 3 day period.\n\n\r\n\r\nOn his first day, he rounds up 150 sheep.\n\n\r\n\r\nOn his second day, he rounds up 20% more sheep than on the first day.\n\n\r\n\r\nOn his third day, he rounds up 50% more sheep than on the second day.\n\n\r\n\r\nHow many sheep does Buddy round up on the third day?"},{"varval":"sm_nogap 1st session = 150 sheep\n\n<div class=\"aligned\">\n\n| | |\n| --------------------- | -------------- |\n| 2nd day\t | \\= 150 + (20% × 150) |\n| | \\= 150 + 30 |\n|| \\= 180 sheep|\n\n</div>\n\n<br>\n\n<div class=\"aligned\">\n\n| | |\n| --------------------- | -------------- |\n| 3rd day\t | \\= 180 + (50% × 180) |\n| | \\= 180 + 90 |\n|| \\= {{{correctAnswer0}}} sheep|\n\n</div>"}]},{"vars":[{"varval":"Stefka is competing in the Olympic high jump which has 3 jumps.\n\n\r\n\r\nOn her first jump, she jumps 1.6 metres.\n\n\r\n\r\nOn her second jump, she jumps 25% higher than on the first jump.\n\n\r\n\r\nOn her third jump, she jumps 4% higher than on the second jump.\n\n\r\n\r\nHow high does Stefka jump on the third jump?"},{"varval":"sm_nogap 1st session = 1.6 metres\n\n<div class=\"aligned\">\n\n| | |\n| --------------------- | -------------- |\n| 2nd jump\t | \\= 1.6 + (25% × 1.6) |\n| | \\= 1.6 + 0.4 |\n|| \\= 2 metres|\n\n</div>\n\n<br>\n\n<div class=\"aligned\">\n\n| | |\n| --------------------- | -------------- |\n| 3rd jump\t | \\= 2 + (4% × 2) |\n| | \\= 2 + 0.08 |\n|| \\= {{{correctAnswer0}}} metres|\n\n</div>"}]}]

  693. This question is the same as 50095. Slightly different worked solution method. Question adjusted so it is an hours to minutes then back to hours question - I have ticked up the difficulty as well.

    <div class="sm_mode"> {{{question}}} </div>

    [{"vars":[{"varval":"Kate studied 2 hours and 24 minutes for her History exam.\n\nBrianna studied 15% longer than Kate for the same exam.\n\nApproximately how long did Brianna study for the History exam?\n"},{"varval":"Kate's study time = 2 $\\times$ 60 + 24 = 144 minutes\n\n\n<div class=\"aligned\">\n\n| | |\n| --------------------- | -------------- |\n| Brianna's study time\t | \\= 144 + $\\dfrac{15}{100}$ × 144 |\n| | \\= 144 + 21.6 |\n| | \\= 165.6 minutes|\n| | $\\approx$ {{{correctAnswer}}} |\n\n</div>\n"}]},{"vars":[{"varval":"Elsie studied 2 hours and 24 minutes for her Hospitality exam.\n\nIsla studied 20% longer than Elsie for the same exam.\n\nApproximately how long did Isla study for the Hospitality exam?\n"},{"varval":"Elsie's study time = 2 $\\times$ 60 + 24 = 144 minutes\n\n\n<div class=\"aligned\">\n\n| | |\n| --------------------- | -------------- |\n| Isla's study time\t | \\= 144 + $\\dfrac{20}{100}$ × 144 |\n| | \\= 144 + 28.8 |\n| | \\= 172.8 minutes|\n| | $\\approx$ {{{correctAnswer}}} |\n\n</div>\n"}]},{"vars":[{"varval":"Judd studied 3 hours and 10 minutes for his Physics exam.\n\nJerry studied 5% longer than Judd for the same exam.\n\nApproximately how long did Jerry study for the Physics exam?\n"},{"varval":"Judd's study time = 3 $\\times$ 60 + 10 = 190 minutes\n\n\n<div class=\"aligned\">\n\n| | |\n| --------------------- | -------------- |\n| Jerry's study time\t | \\= 190 + $\\dfrac{5}{100}$ × 190 |\n| | \\= 190 + 9.5 |\n| | \\= 199.5 minutes|\n| | $\\approx$ {{{correctAnswer}}} |\n\n</div>\n"}]},{"vars":[{"varval":"Curtis studied 1 hour and 56 minutes for his Japanese exam.\n\nCarol studied 30% longer than Curtis for the same exam.\n\nApproximately how long did Carol study for the Japanese exam?\n"},{"varval":"Curtis's study time = 1 $\\times$ 60 + 56 = 116 minutes\n\n\n<div class=\"aligned\">\n\n| | |\n| --------------------- | -------------- |\n| Carol's study time\t | \\= 116 + $\\dfrac{30}{100}$ × 116 |\n| | \\= 116 + 34.8|\n| | \\= 150.8 minutes|\n| | $\\approx$ {{{correctAnswer}}} |\n\n</div>\n"}]},{"vars":[{"varval":"Lewis studied 2 hour and 48 minutes for his Biology exam.\n\nLeanne studied 18% longer than Lewis for the same exam.\n\nApproximately how long did Leanne study for the Biology exam?\n"},{"varval":"Lewis's study time = 2 $\\times$ 60 + 48 = 168 minutes\n\n\n<div class=\"aligned\">\n\n| | |\n| --------------------- | -------------- |\n| Leanne's study time | \\= 168 + $\\dfrac{18}{100}$ × 168 |\n| | \\= 168 + 30.24|\n| | \\= 198.24 minutes|\n| | $\\approx$ {{{correctAnswer}}} |\n\n</div>\n"}]},{"vars":[{"varval":"Ken studied 4 hour and 5 minutes for his Mathematics exam.\n\nKylie studied for 10% less time than Ken for the same exam.\n\nApproximately how long did Kylie study for the Mathematics exam?\n"},{"varval":"Kens's study time = 4 $\\times$ 60 + 5 = 245 minutes\n\n\n<div class=\"aligned\">\n\n| | |\n| --------------------- | -------------- |\n| Kylie's study time\t | \\= 245 $-$ $\\dfrac{10}{100}$ × 245 |\n| | \\= 245 $-$ 24.5|\n| | \\= 220.5 minutes|\n| | $\\approx$ {{{correctAnswer}}} |\n\n</div>\n"}]}]

  694. <div class="sm_mode"> {{{question}}} </div>

    [{"vars":[{"varval":"An African wildlife reserve has 480 elephants.\n\n\r\n\r\nOf the 480 elephants, $\\dfrac{5}{8}$ of them weigh more than 4000 kilograms.\n\n\r\n\r\nOf the elephants that weigh more than 4000 kilograms, $\\dfrac{1}{4}$ weigh more than 6000 kilograms.\n\n\r\n\r\nHow many elephants in the reserve weigh more than 6000 kilograms?"},{"varval":"sm_nogap Number of elephants > 4000 kg\n\n<div class=\"aligned\">\n\n>>||\n|-|\n|= $\\dfrac{5}{8}$ × 480|\n|= 300|\n\n</div>\n\n<br>\n\nsm_nogap Number of elephants > 6000 kg\n\n<div class=\"aligned\">\n\n>>||\n|-|\n|= $\\dfrac{1}{4} \\times 300$|\n|= {{{correctAnswer0}}} |\n\n</div>\n\r"}]},{"vars":[{"varval":"25 000 humpback whales migrate up the east coast of Australia every winter.\n\nOf the 25 000 humpbacks, $\\dfrac{3}{5}$ weigh more than 30 tonnes.\n\nOf those that weigh more than 30 tonnes, $\\dfrac{1}{6}$ weigh more than 40 tonnes.\n\nHow many humpbacks in the migration weigh more than 40 tonnes?\n"},{"varval":"sm_nogap Number of humpbacks > 30 tonnes\n\n<div class=\"aligned\">\n\n>>||\n|-|\n|= $\\dfrac{3}{5}$ × 25 000|\n|= 15 000|\n\n</div>\n\n<br>\n\nsm_nogap Number of humpbacks > 40 tonnes\n\n<div class=\"aligned\">\n\n>>||\n|-|\n|= $\\dfrac{1}{6}$ × 15 000|\n|= {{{correctAnswer0}}} |\n\n</div>"}]},{"vars":[{"varval":"A soccer team plays 24 games in a season.\n\nWet weather resulted in $\\dfrac{2}{3}$ of the games being rescheduled as the grounds were saturated.\n\n\r\n\r\n\r\nOf the rescheduled games, $\\dfrac{3}{8}$ were rescheduled again due to wet weather.\n\n\r\n\r\nHow many games were rescheduled more than once?"},{"varval":"sm_nogap Number of games rescheduled \n\n<div class=\"aligned\">\n\n>>||\n|-|\n|= $\\dfrac{2}{3}$ × 24|\n|= 16|\n\n</div>\n\n<br>\n\nsm_nogap Number of games rescheduled more than once\n\n\n<div class=\"aligned\">\n\n>>||\n|-|\n|= $\\dfrac{3}{8} \\times 16$|\n|= {{{correctAnswer0}}} |\n\n</div>"}]},{"vars":[{"varval":"Last week a hospital emergency department attended to 243 patients.\n\n\r\n\r\nOf the 243 patients, $\\dfrac{1}{3}$ of them waited longer than 30 minutes to see a doctor.\n\n\r\n\r\nOf the patients that waited longer than 30 minutes to see a doctor, $\\dfrac{2}{9}$ waited longer than one hour.\n\n\r\n\r\nHow many patients in the emergency department waited longer than one hour to see a doctor?"},{"varval":"sm_nogap Number of patients waiting > 30 minutes\n\n<div class=\"aligned\">\n\n>>||\n|-|\n|= $\\dfrac{1}{3}$ × 243|\n|= 81|\n\n</div>\n\n<br>\n\nsm_nogap Number of patients waiting > 1 hour\n\n<div class=\"aligned\">\n\n>>||\n|-|\n|= $\\dfrac{2}{9} \\times 81$|\n|= {{{correctAnswer0}}} |\n\n</div>"}]},{"vars":[{"varval":"An inner city tram has a capacity of 270 passengers with some seated and the rest standing.\n\nThere are seats for $\\dfrac{2}{9}$ of the 270 passengers.\n\nOf the seats provided, $\\dfrac{1}{5}$ are priority seats for less mobile passengers.\n\nHow many of the seats are priority seats?"},{"varval":"sm_nogap Number of seats provided\n\n<div class=\"aligned\">\n\n>>||\n|-|\n|= $\\dfrac{2}{9}$ × 270|\n|= 60|\n\n</div>\n\n<br>\n\nsm_nogap Number of priority seats\n\n<div class=\"aligned\">\n\n>>||\n|-|\n|= $\\dfrac{1}{5} \\times 60$|\n|= {{{correctAnswer0}}} |\n\n</div>"}]},{"vars":[{"varval":"A school bus leaves school carrying 60 seated students and 15 standing students.\n\n$\\dfrac{2}{3}$ of the students remain on the bus for more than 5 stops.\n\nOf the students on the bus for more than 5 stops, $\\dfrac{3}{10}$ are on the bus for more than 7 stops.\n\n\r\n\r\nHow many of the students are on the bus for more than 7 stops?"},{"varval":"sm_nogap Total students = 60 + 15 = 75\n\nsm_nogap Number of students > 5 stops\n\n<div class=\"aligned\">\n\n>>||\n|-|\n|= $\\dfrac{2}{3}$ × 75|\n|= 50|\n\n</div>\n\n<br>\n\nsm_nogap Number of passengers > 7 stops\n\n\n<div class=\"aligned\">\n\n>>||\n|-|\n|= $\\dfrac{3}{10} \\times 50$|\n|= {{{correctAnswer0}}} |\n\n</div>"}]}]

  695. <div class="sm_mode"> {{{question}}} </div>

    [{"vars":[{"varval":"Xavier bought 2 identical tubs of liquid chlorine for his swimming pool.\n\n\r\n\r\nAfter he used $\\dfrac{3}{4}$ of one tub in the pool, he had a total of 30 litres of chlorine left.\n\n\r\n\r\nHow many litres of chlorine were in one full tub?"},{"varval":"sm_nogap Let $\\ V$ = volume of 1 tub\n\n<div class=\"aligned\">\n\n| | |\n| --------------------: | -------------- |\n| $\\dfrac{1}{4} V + V$ | \\= 30|\n| $\\dfrac{5}{4} V$ | \\= 30 |\n| $\\therefore V$| \\= $\\dfrac{30 \\times 4}{5}$|\n|| \\= {{{correctAnswer0}}} {{{suffix0}}}|\n\n</div>"}]},{"vars":[{"varval":"Simpson bought 2 identical cans of paint for his mural.\n\n\r\n\r\nAfter he used $\\dfrac{1}{4}$ of one can, he had a total of 14 litres of paint left.\n\n\r\n\r\nHow many litres of paint were in one full can?"},{"varval":"sm_nogap Let $\\ V$ = volume of 1 can\n\n<div class=\"aligned\">\n\n| | |\n| --------------------: | -------------- |\n| $\\dfrac{3}{4} V + V$ | \\= 14|\n| $\\dfrac{7}{4} V$ | \\= 14 |\n| $\\therefore V$| \\= $\\dfrac{14 \\times 4}{7}$|\n|| \\= {{{correctAnswer0}}} {{{suffix0}}}|\n\n</div>"}]},{"vars":[{"varval":"Roger bought 2 identical tubs of acid for his plastics factory.\n\n\r\n\r\nAfter he used $\\dfrac{1}{3}$ of one tub, he had a total of 35 litres of acid left.\n\n\r\n\r\nHow many litres of acid were in one full tub?"},{"varval":"sm_nogap Let $\\ V$ = volume of 1 tub\n\n<div class=\"aligned\">\n\n| | |\n| --------------------: | -------------- |\n| $\\dfrac{2}{3} V + V$ | \\= 35|\n| $\\dfrac{5}{3} V$ | \\= 35 |\n| $\\therefore V$| \\= $\\dfrac{35 \\times 3}{5}$|\n|| \\= {{{correctAnswer0}}} {{{suffix0}}}|\n\n</div>"}]},{"vars":[{"varval":"Susan bought 2 identical cylinders of liquid hydrogen for her energy plant.\n\n\r\n\r\nAfter she used $\\dfrac{2}{3}$ of one cylinder, she had a total of 8 litres of liquid hydrogen left.\n\n\r\n\r\nHow many litres of liquid hydrogen were in one full cylinder?"},{"varval":"sm_nogap Let $\\ V$ = volume of 1 cylinder\n\n<div class=\"aligned\">\n\n| | |\n| --------------------: | -------------- |\n| $\\dfrac{1}{3} V + V$ | \\= 8|\n| $\\dfrac{4}{3} V$ | \\= 8 |\n| $\\therefore V$| \\= $\\dfrac{8 \\times 3}{4}$|\n|| \\= {{{correctAnswer0}}} {{{suffix0}}}|\n\n</div>"}]},{"vars":[{"varval":"Brett bought 3 identical cans of paint for his mural.\n\n\r\n\r\nAfter he used $\\dfrac{1}{2}$ of one can of paint, he had a total of 20 litres of paint left.\n\n\r\n\r\nHow many litres of paint are in one full can?"},{"varval":"sm_nogap Let $\\ V$ = volume of 1 can\n\n<div class=\"aligned\">\n\n| | |\n| --------------------: | -------------- |\n| $\\dfrac{1}{2} V + 2V$ | \\= 20|\n| $\\dfrac{5}{2} V$ | \\= 20 |\n| $\\therefore V$| \\= $\\dfrac{20 \\times 2}{5}$|\n|| \\= {{{correctAnswer0}}} {{{suffix0}}}|\n\n</div>"}]}]

  696. 2 variants: Algebra, NAPX-F4-NC18 Algebra, NAPX-I4-NC21

    <div class="sm_mode"> {{question}} </div>

    [{"vars":[{"varval":"What expression is equivalent to $−(8\\ −\\ \\large p)$?"},{"varval":"<div class=\"sm_mode\">\r\n\r\n\n<div class=\"aligned\">\r\n\r\n\n>| | |\r\n| -------------: | ---------- |\r\n| $−(8\\ −\\ \\large p)$| \\= $−\\ 8 + \\large p$ |\r\n| | \\= $\\large p\\ -$ 8 |\r\n\r\n\n</div>\r\n\r\n\r\n\n</div>\r\n"}]},{"vars":[{"varval":"Which of the following is always equal to $\\ −\\ \\large p$ + $\\large q$?"},{"varval":"$\\ −\\ \\large p$ + $\\large q$ = $\\large q$ $−$ $\\large p$"}]},{"vars":[{"varval":"Which of the following is always equal to $\\ −\\ \\large p$ − $\\large q$?"},{"varval":"$\\ −\\ \\large p$ − $\\large q$ = $−$ ($\\large q$ + $\\large p$)"}]},{"vars":[{"varval":"Which of the following is always equal to $\\ −\\ 2\\large m$ + $\\large n$?"},{"varval":"$\\ −\\ 2\\large m$ + $\\large n$ = $−\\ 2\\large m$ $−$ ($−\\ \\large n$)"}]},{"vars":[{"varval":"What expression is equivalent to $−(\\large x\\ −$ 2$\\large y)$?"},{"varval":"<div class=\"aligned\">\r\n\r\n\n>| | |\r\n| -------------: | ---------- |\r\n| $−(\\large x\\ −$ 2$\\large y)$| \\= $−\\ \\large x\\ +$ 2$\\large y$ |\r\n| | \\= $2\\large y\\ -\\ x$ |\r\n\r\n\n</div>"}]},{"vars":[{"varval":"Which of the following is always equal to $−(5\\large v$ + $7 \\large t)$?"},{"varval":"$−(5\\large v$ + $7 \\large t)$ = $−5\\large v$ $-$ $7 \\large t$"}]}]

  697. <div class="sm_mode"> {{{question}}} </div>

    [{"vars":[{"varval":"Isaac dropped an apple from the top of a bridge into the water below.\n\nHe uses the following equation to calculate the distance the apple travels in metres,\n\n` ` *distance = 4.9 x (time)$^2$*\n\nwhere time is the number of seconds it takes the apple to reach the water.\n\nWhich of these is closest to the distance, if it took the apple a time of 3.3 seconds to reach the water?\n"},{"varval":"<div class=\"aligned\">\r\n\r\n| | |\r\n| ------------- | ---------- |\r\n| Distance | \\= $4.9×(3.3)^2$ |\n| | \\= $4.9×10.89$ |\n| | \\= $53.361$ |\r\n| | ≈ {{{correctAnswer}}} |\r\n\r\n</div>\r\n"}]},{"vars":[{"varval":"Golem dropped a rock from the top of a tower onto the ground below.\n\nHe uses the following equation to calculate the distance the rock travels in metres,\n\n` ` *distance = 4.9 x (time)$^2$*\n\nwhere time is the number of seconds it takes the rock to reach the ground.\n\nWhich of these is closest to the distance, if it took the rock a time of 4.2 seconds to reach the water?\n"},{"varval":"<div class=\"aligned\">\r\n\r\n| | |\r\n| ------------- | ---------- |\r\n| Distance | \\= $4.9×(4.2)^2$ |\n| | \\= $4.9×17.64$ |\n| | \\= $86.436$ |\r\n| | ≈ {{{correctAnswer}}} |\r\n\r\n</div>\r\n"}]},{"vars":[{"varval":"A rescue worker dropped a warning beacon from a helicopter into the ocean below.\n\nHe uses the following equation to calculate the distance the warning beacon travels in metres,\n\n` ` *distance = 4.9 x (time)$^2$*\n\nwhere time is the number of seconds it takes the warning beacon to reach the ocean.\n\nWhich of these is closest to the distance, if it took the warning beacon a time of 2.1 seconds to reach the ocean?\n"},{"varval":"<div class=\"aligned\">\r\n\r\n| | |\r\n| ------------- | ---------- |\r\n| Distance | \\= $4.9×(2.1)^2$ |\n| | \\= $4.9×4.41$ |\n| | \\= $21.609$ |\r\n| | ≈ {{{correctAnswer}}} |\r\n\r\n</div>\r\n"}]},{"vars":[{"varval":"A building is damaged when a metal bar falls from a crane onto its roof.\n\nThe investigation team uses the following equation to calculate the distance the metal bar fell in metres,\n\n` ` *distance = 4.9 x (time)$^2$*\n\nwhere time is the number of seconds it takes the metal bar to reach the roof.\n\nWhich of these is closest to the distance, if it took the metal bar a time of 3 seconds to reach the roof?\n"},{"varval":"<div class=\"aligned\">\r\n\r\n| | |\r\n| ------------- | ---------- |\r\n| Distance | \\= $4.9×(3)^2$ |\n| | \\= $4.9×9$ |\n| | \\= $44.1$ |\r\n| | ≈ {{{correctAnswer}}} |\r\n\r\n</div>\r\n"}]},{"vars":[{"varval":"A bird egg fell from a nest to the ground.\n\nThe following equation can be used to calculate the distance the egg fell in metres,\n\n` ` *distance = 4.9 x (time)$^2$*\n\nwhere time is the number of seconds it takes the egg to reach the ground.\n\nWhich of these is closest to the distance, if it took the egg a time of 1.1 seconds to reach the ground?\n"},{"varval":"<div class=\"aligned\">\r\n\r\n| | |\r\n| ------------- | ---------- |\r\n| Distance | \\= $4.9×(1.1)^2$ |\n| | \\= $4.9×1.21$ |\n| | \\= $5.929$ |\r\n| | ≈ {{{correctAnswer}}} |\r\n\r\n</div>\r\n"}]},{"vars":[{"varval":"A coconut fell from the top of a coconut palm to the ground.\n\nThe following equation can be used to calculate the distance the coconut fell in metres,\n\n` ` *distance = 4.9 x (time)$^2$*\n\nwhere time is the number of seconds it takes the coconut to reach the ground.\n\nWhich of these is closest to the distance, if it took the coconut a time of 2.2 seconds to reach the ground?"},{"varval":"<div class=\"aligned\">\r\n\r\n| | |\r\n| ------------- | ---------- |\r\n| Distance | \\= $4.9×(2.2)^2$ |\n| | \\= $4.9×4.84$ |\n| | \\= $23.716$ |\r\n| | ≈ {{{correctAnswer}}} |\r\n\r\n</div>\r\n"}]}]

  698. <div class="sm_mode"> {{{question}}} </div>

    [{"vars":[{"varval":"$\\large b$ is a positive whole number.\n\nWhich of the equations below is **not** true?\n\n"},{"varval":"Test each option:\n\n<div class=\"aligned\">\r\n\r\n| | |\r\n| ------------- | ---------- |\r\n| $\\large b$ $\\times\\ \\large b$ $\\times\\ \\large b$ $\\times\\ 3$ | \\= $3\\large b$$^3$ |\r\n| | ≠ $3\\large b$ |\r\n\r\n</div>\r\n"}]},{"vars":[{"varval":"$\\large a$ is a positive whole number.\n\nWhich of the equations below is **not** true?\n"},{"varval":"Test each option:\n\n<div class=\"aligned\">\r\n\r\n| | |\r\n| ------------- | ---------- |\r\n| $2\\large a$ × $3\\large a$ | \\= $6\\large a$$^2$ |\r\n| | ≠ $5\\large a$$^2$ |\n\r\n\r\n</div>\r\n"}]},{"vars":[{"varval":"$\\large x$ is a positive whole number.\n\nWhich of the equations below is **not** true?\n"},{"varval":"Test each option:\n\n<div class=\"aligned\">\r\n\r\n| | |\r\n| ------------- | ---------- |\r\n| $\\large x$ $-$ 3$\\large x$ $-$ 2$\\large x$ | \\= $-$$4\\large x$ |\r\n| | ≠ $-$$6\\large x$ |\r\n\r\n</div>\r\n"}]},{"vars":[{"varval":"$\\large x$ and $\\large y$ are positive whole numbers.\n\nWhich of the equations below is **not** true?\n"},{"varval":"Test each option:\n\n<div class=\"aligned\">\r\n\r\n| | |\r\n| ------------- | ---------- |\r\n| $\\dfrac{3\\large x^2y}{2\\large x}$ | \\= $\\dfrac{3\\large xy}{2}$ |\r\n| | ≠ $\\dfrac{3\\large y}{2\\large x}$ |\n\n\r\n\r\n</div>\n"}]},{"vars":[{"varval":"$\\large m$ is a positive whole number.\n\nWhich of the equations below is **not** true?\n\n"},{"varval":"Test each option:\n\n<div class=\"aligned\">\r\n\r\n| | |\r\n| ------------- | ---------- |\r\n| $−$($2\\large m$)$^2$ |= $-$ ($2\\large m$) × ($2\\large m$) |\n| |= $-$ $4\\large m$$^2$ |\r\n| | ≠ 4 $\\large m$$^2$ |\n\n\r\n\r\n</div>"}]},{"vars":[{"varval":"$\\large a$ is a positive whole number.\n\nWhich of the equations below is **not** true?\n"},{"varval":"Test each option:\n\n<div class=\"aligned\">\r\n\r\n| | |\r\n| ------------- | ---------- |\r\n| 2($2\\large a$ + 3) | \\= $2$ × $2\\large a$ + $2$ × $3$ |\r\n| | = $4\\large a$ + $6$ |\n| | ≠ $4\\large a$ + 3 |\n\r\n\r\n</div>\r\n"}]}]

  699. <div class="sm_mode"> {{{question}}} </div>

    [{"vars":[{"varval":"Alex has a job selling bikes. Her weekly salary, $C$ dollars, is calculated using the rule below:\r\n\r\n\n` `$C=400+0.06W$\r\n\nwhere $W$ is the total value in dollars of the bikes she sells that week.\r\n\r\nAlex sold $24 000 worth of bikes in a given week.\r\n\r\nWhat was Alex's salary in the week?"},{"varval":"<div class=\"aligned\">\r\n\r\n| | |\r\n| ------------- | ---------- |\r\n| $C$ | \\= 400 + (0.06 $\\times$ 24 000) |\n| | \\= {{{correctAnswer}}} |\r\n\r\n</div>\r"}]},{"vars":[{"varval":"Zac works in the electronics department of JB HiFi. \n\nHis weekly salary, $D$ dollars, is calculated using the rule below:\r\n\r\n\n` `$D=500+0.03E$\r\n\nwhere $E$ is the total value in dollars of the electronics products he sells that week.\r\n\r\nZac sold $52 000 worth of electronics in a given week.\r\n\r\nWhat was Zac's salary in the week?"},{"varval":"<div class=\"aligned\">\r\n\r\n| | |\r\n| ------------- | ---------- |\r\n| $D$ | \\= 500 + (0.03 $\\times$ 52 000) |\n| | \\= 500 + 1560 |\n| | \\= {{{correctAnswer}}} |\r\n\r\n</div>\r"}]},{"vars":[{"varval":"Lilli works in clothes shop. \n\nHer weekly salary, $W$ dollars, is calculated using the rule below:\r\n\r\n\n` `$W=300+0.08S$\r\n\nwhere $S$ is the total value in dollars of the clothes she sells that week.\r\n\r\nLilli earned $860 in a given week.\r\n\r\nWhat was the value of the clothes that Lilli sold in the week?"},{"varval":"<div class=\"aligned\">\r\n\r\n| | |\r\n| -------------: | ---------- |\r\n| $W$ | \\= 300 + (0.08 $\\times$ $S$) |\n| 860 | \\= 300 + 0.08$S$ |\n| 0.08$S$| \\= 560 |\n| $S$ | \\= $\\dfrac{560}{0.08}$|\n| | \\= {{{correctAnswer}}} |\r\n\r\n</div>\r"}]},{"vars":[{"varval":"Trevor works in dive shop. \n\nHis weekly salary, $D$ dollars, is calculated using the rule below:\r\n\r\n\n` `$D=480+0.06E$\r\n\nwhere $E$ is the total value in dollars of the diving equipment he sells that week.\r\n\r\nTrevor earned $1200 in a given week.\r\n\r\nWhat was the value of the diving equipment Trevor sold in the week?"},{"varval":"<div class=\"aligned\">\r\n\r\n| | |\r\n| -------------: | ---------- |\r\n| $D$ | \\= 480 + (0.06 $\\times$ $E$) |\n| 1200| \\= 480 + 0.06$E$ |\n| 0.06$E$| \\= 720 |\n| $E$ | \\= $\\dfrac{720}{0.06}$|\n| | \\= {{{correctAnswer}}} |\r\n\r\n</div>\r"}]}]

  700. <div class="sm_mode"> {{{question}}} </div>

    [{"vars":[{"varval":"Dawn had a bag of different coloured marbles.\n\n\r\n\r\n$\\dfrac{1}{4}$ of the marbles were clear and $\\dfrac{1}{3}$ of the marbles were black.\n\n\r\n\r\nHer bag contained 5 more black marbles than clear marbles.\n\n\r\n\r\nHow many marbles were in Dawn's bag?"},{"varval":"Strategy 1\n\nTest each option:\n\n$\\dfrac{1}{4}\\ \\times 60$ = 15\n\n$\\dfrac{1}{3}\\ \\times 60$ = 20\n\nDifference = 20 $-$ 15 = 5\n\n$\\therefore$ 60 marbles are in the bag.\n\n<br>\n\nStrategy 2 (advanced)\n\nLet $\\ \\large n$ = number of marbles in bag\n\n<div class=\"aligned\">\n\n| | |\n| ---------------------: | -------------- |\n| $\\dfrac{1}{3}\\large n$ $-$ $\\dfrac{1}{4}\\large n$ | \\= 5 |\n| $\\dfrac{4}{12}\\large n$ $-$ $\\dfrac{3}{12}\\large n$ | \\= 5 |\n| $\\dfrac{1}{12}\\large n$| = 5|\n|$\\therefore \\large n$| \\= {{{correctAnswer}}}|\n\n</div>"}]},{"vars":[{"varval":"Mica had a barrel of different coloured discs.\n\n\r\n\r\n$\\dfrac{1}{5}$ of the discs were yellow and $\\dfrac{1}{4}$ of the discs were green.\n\n\r\n\r\nHis barrel contained 7 more green discs than yellow.\n\n\r\n\r\nHow many discs were in Mica's barrel?"},{"varval":"Strategy 1\n\nTest each option:\n\n$\\dfrac{1}{5}\\ \\times 140$ = 28\n\n$\\dfrac{1}{4}\\ \\times 140$ = 35\n\nDifference = 35 $-$ 28 = 7\n\n$\\therefore$ 140 discs are in the barrel.\n\n<br>\n\nStrategy 2 (advanced)\n\nLet $\\ \\large n$ = number of discs in barrel\n\n<div class=\"aligned\">\n\n| | |\n| ---------------------: | -------------- |\n| $\\dfrac{1}{4}\\large n$ $-$ $\\dfrac{1}{5}\\large n$ | \\= 7 |\n| $\\dfrac{5}{20}\\large n$ $-$ $\\dfrac{4}{20}\\large n$ | \\= 7 |\n| $\\dfrac{1}{20}\\large n$| = 7|\n|$\\therefore \\large n$| \\= {{{correctAnswer}}}|\n\n</div>"}]},{"vars":[{"varval":"Ed had a hobby farm growing two different varieties of cabbages.\n\n\r\n\r\n$\\dfrac{1}{3}$ of the cabbages were purple and the rest were green.\n\n\r\n\r\nLast year Ed planted 120 more green cabbages than purple.\n\n\r\n\r\nHow many cabbages did Ed plant last year?"},{"varval":"Strategy 1\n\n$\\dfrac{1}{3}$ of cabbages are purple\n\n$\\dfrac{2}{3}$ of cabbages are green\n\nTest each option:\n\n$\\dfrac{1}{3}\\ \\times 360$ = 120\n\n$\\dfrac{2}{3}\\ \\times 360$ = 240\n\nDifference = 240 $-$ 120 = 120\n\n$\\therefore$ 360 cabbages were planted last year.\n\n<br>\n\nStrategy 2 (advanced)\n\nLet $\\ \\large n$ = number of cabbages planted\n\n<div class=\"aligned\">\n\n| | |\n| ---------------------: | -------------- |\n| $\\dfrac{2}{3}\\large n$ $-$ $\\dfrac{1}{3}\\large n$ | \\= 120 |\n| $\\dfrac{1}{3}\\large n$| = 120|\n|$\\therefore \\large n$| \\= {{{correctAnswer}}}|\n\n</div>"}]},{"vars":[{"varval":"The local animal shelter has a number of different animals for adoption.\n\n\r\n\r\n$\\dfrac{1}{6}$ of the animals are cats and $\\dfrac{1}{3}$ of the animals are dogs.\n\n\r\n\r\nThere are currently 25 more dogs than cats in the shelter.\n\n\r\n\r\nHow many animals are currently available for adoption in the shelter?"},{"varval":"Strategy 1\n\nTest each option:\n\n$\\dfrac{1}{6}\\ \\times 150$ = 25\n\n$\\dfrac{1}{3}\\ \\times 150$ = 50\n\nDifference = 50 $-$ 25 = 25\n\n$\\therefore$ 150 animals were in the shelter.\n\n<br>\n\nStrategy 2 (advanced)\n\nLet $\\ \\large n$ = number of animals in the shelter\n\n<div class=\"aligned\">\n\n| | |\n| ---------------------: | -------------- |\n| $\\dfrac{1}{3}\\large n$ $-$ $\\dfrac{1}{6}\\large n$ | \\= 25 |\n| $\\dfrac{2}{6}\\large n$ $-$ $\\dfrac{1}{6}\\large n$ | \\= 25 |\n| $\\dfrac{1}{6}\\large n$| = 25|\n|$\\therefore \\large n$| \\= {{{correctAnswer}}}|\n\n</div>"}]},{"vars":[{"varval":"At a swimming carnival there were two grandstands. \n\nStudents were seated in the grandstands based on whether they were competitors or non-competitors. \n\r\n\r\n\n$\\dfrac{5}{8}$ of the students were seated in the non-competitors' grandstand and the rest were seated in the competitors' grandstand.\n\n\r\nThere were 60 more non-competitors than competitors at the carnival.\n\n\r\nHow many students, in total, were at the swimming carnival?"},{"varval":"Strategy 1\n\n$\\dfrac{5}{8}$ are non-competitors\n\n$\\dfrac{3}{8}$ are competitors\n\nTest each option:\n\n$\\dfrac{5}{8}\\ \\times 240$ = 150\n\n$\\dfrac{3}{8}\\ \\times 240$ = 90\n\n\n\nDifference = 150 $-$ 90 = 60\n\n$\\therefore$ 240 students were at the carnival.\n\n<br>\n\nStrategy 2 (advanced)\n\nLet $\\ \\large n$ = number of students at the carnival\n\n<div class=\"aligned\">\n\n| | |\n| ---------------------: | -------------- |\n| $\\dfrac{5}{8}\\large n$ $-$ $\\dfrac{3}{8}\\large n$ | \\= 60 |\n| $\\dfrac{2}{8}\\large n$| = 60|\n| $\\dfrac{1}{4}\\large n$| = 60|\n|$\\therefore \\large n$| \\= {{{correctAnswer}}}|\n\n</div>"}]}]

  701. <div class="sm_mode"> sm_img https://teacher.smartermaths.com.au/wp-content/uploads/2018/08/NAPX-G3-NC27.svg 230 indent vpad Which fraction is halfway between $\dfrac{1}{3}$ and $\dfrac{3}{7}$ on the number line? </div>

    [{"vars":null}]

  702. <div class="sm_mode"> {{{question}}} </div>

    [{"vars":[{"varval":"The table below shows all the people at Vamp's birthday party.\n\n<br>\n\n<div class=\"sm-table col1-color1 row1-color1 top-left-cell-hidden\">\n\n>>| | Male | Female |\n|:-:|:-:|:-:|\n| Adult| 35| 15 |\n| Child| 30| 20|\n\n</div>\n\n\n<br>What fraction of the children at the party are female?"},{"varval":"<div class=\"aligned\">\n\n| | |\n| --------------------- | -------------- |\n| Fraction | \\= $\\dfrac{\\text{female children}}{\\text{total children}}$ |\n| | | \n| | \\= $\\dfrac{20}{(30 + 20)}$ |\n| | | \n|| \\= {{{correctAnswer}}}|\n\n</div>"}]},{"vars":[{"varval":"The table below shows all the members in a tennis club.\n\n<br>\n\n<div class=\"sm-table col1-color1 row1-color1 top-left-cell-hidden\">\n\n>>| | Male | Female |\n|:-:|:-:|:-:|\n| Left-handed| 15| 10 |\n| Right-handed| 45| 30|\n\n</div>\n\n\n<br>What fraction of the left-handed players in the club are female?"},{"varval":"<div class=\"aligned\">\n\n| | |\n| --------------------- | -------------- |\n| Fraction | \\= $\\dfrac{\\text{female left-handers}}{\\text{total left-handers}}$ |\n| | | \n| | \\= $\\dfrac{10}{(15 + 10)}$ |\n| | | \n|| \\= {{{correctAnswer}}}|\n\n</div>"}]},{"vars":[{"varval":"The table below shows all the members in a junior AFL club.\n\n<br>\n\n<div class=\"sm-table col1-color3 row1-color3 top-left-cell-hidden\">\n\n>>| | Male | Female |\n|:-:|:-:|:-:|\n| Right-footed| 45| 30 |\n| Left-footed| 20| 5|\n\n</div>\n\n\n<br>What fraction of the right-footed players in the club are female?"},{"varval":"<div class=\"aligned\">\n\n| | |\n| --------------------- | -------------- |\n| Fraction | \\= $\\dfrac{\\text{female right-footers}}{\\text{total right-footers}}$ |\n| | | \n| | \\= $\\dfrac{30}{(45 + 30)}$ |\n| | | \n|| \\= {{{correctAnswer}}}|\n\n</div>"}]},{"vars":[{"varval":"The table below shows all the people watching the movie \"Minions: The Rise of Gru\" in Cinemas 1 and 2 at 2:30 pm at the local movie complex.\n\n<br>\n\n<div class=\"sm-table col1-color1 row1-color1 top-left-cell-hidden\">\n\n>>| | Cinema 1| Cinema 2|\n|:-:|:-:|:-:|\n| Adult| 80| 120 |\n| Child| 90| 100 |\n\n</div>\n\n\n<br>What fraction of the adults were in Cinema 2?"},{"varval":"<div class=\"aligned\">\n\n| | |\n| --------------------- | -------------- |\n| Fraction | \\= $\\dfrac{\\text{adults in Cinema 2}}{\\text{total adults}}$ |\n| | | \n| | \\= $\\dfrac{120}{(80 + 120)}$ |\n| | | \n|| \\= {{{correctAnswer}}}|\n\n</div>"}]},{"vars":[{"varval":"Two Year 9 classes, 9A and 9B, attended a Careers Expo. Each student was given an orange or a purple wrist band.\n\n The table below summarises this information.\n\n<br>\n\n<div class=\"sm-table col1-color1 row1-color1 top-left-cell-hidden\">\n\n>> | | Orange Wrist Band |Purple Wrist Band|\n|:-:|:-:|:-:|\n| Students in 9A| 9| 21 |\n| Students in 9B| 16| 12 |\n\n</div>\n\n\n<br>What fraction of the total number of students were given orange wrist bands?"},{"varval":"<div class=\"aligned\">\n\n| | |\n| --------------------- | -------------- |\n| Fraction | \\= $\\dfrac{\\text{total orange wrist bands}}{\\text{total students}}$ |\n| | | \n| | \\= $\\dfrac{9+16}{(9+16+21+12)}$ |\n| | | \n|| \\= {{{correctAnswer}}}|\n\n</div>\n"}]}]

  703. <div class="sm_mode"> {{{question}}} </div>

    [{"vars":[{"varval":"Peter measured the diameter of 2 coins and recorded his measurements in the table below.\n\n<div class=\"sm-table row1-color3\">\n\n>>| Coin | Diameter|\n|:-:|:-:|\n| $1 | 25 millimetres|\n| 20c | $1\\frac{1}{8}$ inch|\n\n</div>\n\n<br>One inch equals 25.4 millimetres.\n\n\r\n\r\nWhat is the difference in millimetres between the diameters of the two coins?\n\n\r\n\r\nGive your answer to three decimal places."},{"varval":"<div class=\"aligned\">\n\n| | |\n| --------------------- | -------------- |\n| Diameter of 20c\t | \\= $1 \\dfrac{1}{8} \\ \\times 25.4$ |\n| | \\= 28.575 mm |\n\n</div>\n\n<br>\n\n<div class=\"aligned\">\n\n| | |\n| --------------------- | -------------- |\n| $\\therefore$ Difference | \\= 28.575 − 25 |\n| | \\= {{{correctAnswer0}}} {{{suffix0}}} |\n\n</div>"}]},{"vars":[{"varval":"Fidel measured the diameter of 2 coins and recorded his measurements in the table below.\n\n<div class=\"sm-table row1-color3\">\n\n>>| Coin | Diameter|\n|:-:|:-:|\n| $1 | 25 millimetres|\n| 10c | $\\frac{7}{8}$ inch|\n\n</div>\n\n<br>One inch equals 25.4 millimetres.\n\n\r\n\r\nWhat is the difference in millimetres between the diameters of the two coins?\n\n\r\n\r\nGive your answer to three decimal places."},{"varval":"<div class=\"aligned\">\n\n| | |\n| --------------------- | -------------- |\n| Diameter of 10c\t | \\= $\\dfrac{7}{8} \\times 25.4$ |\n| | \\= 22.225 mm |\n\n</div>\n\n<br>\n\n<div class=\"aligned\">\n\n| | |\n| --------------------- | -------------- |\n| $\\therefore$ Difference | \\= 25 $-$ 22.225|\n| | \\= {{{correctAnswer0}}} {{{suffix0}}} |\n\n</div>"}]},{"vars":[{"varval":"Sandra measured the diameter of 2 tokens and recorded her measurements in the table below.\n\n<div class=\"sm-table row1-color3\">\n\n>>| Token | Diameter|\n|:-:|:-:|\n| $5 | 30 millimetres|\n| $1 | $\\frac{5}{8}$ inch|\n\n</div>\n\n<br>One inch equals 25.4 millimetres.\n\n\r\n\r\nWhat is the difference in millimetres between the diameters of the two tokens?\n\n\r\n\r\nGive your answer to three decimal places."},{"varval":"<div class=\"aligned\">\n\n| | |\n| --------------------- | -------------- |\n| Diameter of $1\t | \\= $\\dfrac{5}{8} \\times 25.4$ |\n| | \\= 15.875 mm |\n\n</div>\n\n<br>\n\n<div class=\"aligned\">\n\n| | |\n| --------------------- | -------------- |\n| $\\therefore$ Difference | \\= 30 $-$ 15.875|\n| | \\= {{{correctAnswer0}}} {{{suffix0}}} |\n\n</div>"}]},{"vars":[{"varval":"India measured the diameter of 2 tokens and recorded her measurements in the table below.\n\n<div class=\"sm-table row1-color3\">\n\n>>| Token | Diameter|\n|:-:|:-:|\n| $2 | 25 millimetres|\n| $1 | $\\frac{4}{5}$ inch|\n\n</div>\n\n<br>One inch equals 25.4 millimetres.\n\n\r\n\r\nWhat is the difference in millimetres between the diameters of the two tokens?\n\n\r\n\r\nGive your answer to two decimal places."},{"varval":"<div class=\"aligned\">\n\n| | |\n| --------------------- | -------------- |\n| Diameter of $1\t | \\= $\\dfrac{4}{5} \\times 25.4$ |\n| | \\= 20.32 mm |\n\n</div>\n\n<br>\n\n<div class=\"aligned\">\n\n| | |\n| --------------------- | -------------- |\n| $\\therefore$ Difference | \\= 25 $-$ 20.32|\n| | \\= {{{correctAnswer0}}} {{{suffix0}}} |\n\n</div>"}]},{"vars":[{"varval":"Jonathon measured the diameter of 2 coloured discs and recorded his measurements in the table below.\n\n<div class=\"sm-table row1-color3\">\n\n>>| Disc| Diameter|\n|:-:|:-:|\n| Red | 18 millimetres|\n| Blue | $1 \\frac{3}{8}$ inch|\n\n</div>\n\n<br>One inch equals 25.4 millimetres.\n\n\r\n\r\nWhat is the difference in millimetres between the diameters of the two discs?\n\n\r\n\r\nGive your answer to three decimal places."},{"varval":"<div class=\"aligned\">\n\n| | |\n| --------------------- | -------------- |\n| Diameter of Blue\t | \\= $1 \\dfrac{3}{8} \\ \\times 25.4$ |\n| | \\= 34.925 mm |\n\n</div>\n\n<br>\n\n<div class=\"aligned\">\n\n| | |\n| --------------------- | -------------- |\n| $\\therefore$ Difference | \\= 34.925 $-$ 18|\n| | \\= {{{correctAnswer0}}} {{{suffix0}}} |\n\n</div>"}]}]

  704. <div class="sm_mode"> {{{question}}} </div>

    [{"vars":[{"varval":"Which expression is equivalent to $\\ 9\\large x\\ −$ 3 + 3$\\large x$ + 1?"},{"varval":"$9\\large x\\ −$ 3 + 3$\\large x$ + 1\n\n>= {{{correctAnswer}}}"}]},{"vars":[{"varval":"Which expression is equivalent to $\\ 2\\large x\\ −$ 7 + 8$\\large x$ + 6?"},{"varval":"$\\ 2\\large x\\ −$ 7 + 8$\\large x$ + 6\n\n>= {{{correctAnswer}}}"}]},{"vars":[{"varval":"Which expression is equivalent to $\\ 5\\large x\\ +$ 2 − 3$\\large x$ + 3?"},{"varval":"$\\ 5\\large x\\ +$ 2 − 3$\\large x$ + 3\n\n>= {{{correctAnswer}}}"}]},{"vars":[{"varval":"Which expression is equivalent to $\\ 4\\large x\\ −$ 2 − 3$\\large x$ − 1?"},{"varval":"$\\ 4\\large x\\ −$ 2 − 3$\\large x$ − 1\n\n>= {{{correctAnswer}}}"}]},{"vars":[{"varval":"Which expression is equivalent to $\\ 3\\large x\\ +$ 2 − 5$\\large x$ − 4?"},{"varval":"$\\ 3\\large x\\ +$ 2 − 5$\\large x$ − 4\n\n>= {{{correctAnswer}}}"}]},{"vars":[{"varval":"Which expression is equivalent to $\\ \\large x\\ −$ 1 − 5$\\large x$ + 3?"},{"varval":"$\\ \\large x\\ −$ 1 − 5$\\large x$ + 3\n\n>= {{{correctAnswer}}}"}]}]

  705. <div class="sm_mode"> {{{question}}} </div>

    [{"vars":[{"varval":"$300 = 3000 - \\dfrac{\\large b}{9}$\r\n\nWhat is the value of $\\large b$?"},{"varval":"Strategy 1\n\nBy trial and error using given options:\n\n<div class=\"aligned\">\r\n\r\n>| | |\r\n| -------------: | ---------- |\r\n| 300 | = 3000 $-$ $\\dfrac{24\\ 300}{9}$ |\n| | = 3000 $-$ 2700 |\r\n| | = 300 &nbsp;$\\checkmark$ |\r\n\r\n</div>\r\n\n<br>\n\n$\\therefore\\ \\large b$ = {{{correctAnswer}}}\n\nStrategy 2 (advanced)\n\n\n<div class=\"aligned\">\r\n\r\n>| | |\r\n| -------------: | ---------- |\r\n| 300 | = 3000 $-$ $\\dfrac{\\large b}{9}$ |\n| $\\dfrac{\\large b}{9}$ | \\= 3000 $-$ 300 |\n| $\\large b$ | \\= 9 $\\times$ 2700 |\r\n| | \\= {{{correctAnswer}}} |\r\n\r\n</div>\r"}]},{"vars":[{"varval":"$150 = 1000 - \\dfrac{\\large x}{6}$\r\n\nWhat is the value of $\\large x$?"},{"varval":"Strategy 1\n\nBy trial and error using given options:\n\n<div class=\"aligned\">\r\n\r\n>| | |\r\n| -------------: | ---------- |\r\n| 150 | = 1000 $-$ $\\dfrac{5100}{6}$ |\n| | = 1000 $-$ 850 |\r\n| | = 150 &nbsp;$\\checkmark$ |\r\n\n<br>\n\n$\\therefore \\large x$ = {{{correctAnswer}}}\n\r\n</div>\r\n\n<br>\n\nStrategy 2 (advanced)\n\n<div class=\"aligned\">\r\n\r\n>| | |\r\n| -------------: | ---------- |\r\n| 150 | = 1000 $-$ $\\dfrac{\\large x}{6}$ |\n| $\\dfrac{\\large x}{6}$ | \\= 1000 $-$ 150 |\n| $\\large x$ | \\= 6 $\\times$ 850 |\r\n| | \\= {{{correctAnswer}}} |\r\n\r\n</div>\r"}]},{"vars":[{"varval":"$200 = 2000 - \\dfrac{\\large n}{5}$\r\n\nWhat is the value of $\\large n$?"},{"varval":"Strategy 1\n\nBy trial and error using given options:\n\n<div class=\"aligned\">\r\n\r\n>| | |\r\n| -------------: | ---------- |\r\n| 200 | = 2000 $-$ $\\dfrac{9000}{5}$ |\n| | = 2000 $-$ 1800 |\r\n| | = 200 &nbsp;$\\checkmark$ |\r\n\n<br>\n\n$\\therefore \\large n$ = {{{correctAnswer}}}\n\r\n</div>\r\n\n<br>\n\nStrategy 2 (advanced)\n\n<div class=\"aligned\">\r\n\r\n>| | |\r\n| -------------: | ---------- |\r\n| 200 | = 2000 $-$ $\\dfrac{\\large n}{5}$ |\n| $\\dfrac{\\large n}{5}$ | \\= 2000 $-$ 200 |\n| $\\large n$ | \\= 5 $\\times$ 1800 |\r\n| | \\= {{{correctAnswer}}} |\r\n\r\n</div>\r"}]},{"vars":[{"varval":"$180 = 2000 - \\dfrac{\\large y}{6}$\r\n\nWhat is the value of $\\large y$?"},{"varval":"Strategy 1\n\nBy trial and error using given options:\n\n<div class=\"aligned\">\r\n\r\n>| | |\r\n| -------------: | ---------- |\r\n| 180 | = 2000 $-$ $\\dfrac{10\\ 920}{6}$ |\n| | = 2000 $-$ 1820 |\r\n| | = 180 &nbsp;$\\checkmark$ |\r\n\n<br>\n\n$\\therefore \\large y$ = {{{correctAnswer}}}\n\r\n</div>\r\n\n<br>\n\nStrategy 2 (advanced)\n\n<div class=\"aligned\">\r\n\r\n>| | |\r\n| -------------: | ---------- |\r\n| 180 | = 2000 $-$ $\\dfrac{\\large y}{6}$ |\n| $\\dfrac{\\large y}{6}$ | \\= 2000 $-$ 180 |\n| $\\large y$ | \\= 6 $\\times$ 1820 |\r\n| | \\= {{{correctAnswer}}} |\r\n\r\n</div>"}]},{"vars":[{"varval":"$250 = 5000 - \\dfrac{\\large p}{10}$\r\n\nWhat is the value of $\\large p$?"},{"varval":"Strategy 1\n\nBy trial and error using given options:\n\n<div class=\"aligned\">\r\n\r\n>| | |\r\n| -------------: | ---------- |\r\n| 250 | = 5000 $-$ $\\dfrac{47\\ 500}{10}$ |\n| | = 5000 $-$ 4750 |\r\n| | = 250 &nbsp;$\\checkmark$ |\r\n\n<br>\n\n$\\therefore \\large p$ = {{{correctAnswer}}}\n\r\n</div>\r\n\n<br>\n\nStrategy 2 (advanced)\n\n<div class=\"aligned\">\r\n\r\n>| | |\r\n| -------------: | ---------- |\r\n| 250 | = 5000 $-$ $\\dfrac{\\large p}{10}$ |\n| $\\dfrac{\\large p}{10}$ | \\= 5000 $-$ 250 |\n| $\\large p$ | \\= 10 $\\times$ 4750 |\r\n| | \\= {{{correctAnswer}}} |\r\n\r\n</div>"}]},{"vars":[{"varval":"$90 = 200 - \\dfrac{\\large m}{6}$\r\n\nWhat is the value of $\\large m$?"},{"varval":"Strategy 1\n\nBy trial and error using given options:\n\n<div class=\"aligned\">\r\n\r\n>| | |\r\n| -------------: | ---------- |\r\n| 90 | = 200 $-$ $\\dfrac{660}{6}$ |\n| | = 200 $-$ 110 |\r\n| | = 90 &nbsp;$\\checkmark$ |\r\n\n<br>\n\n$\\therefore \\large m$ = {{{correctAnswer}}}\n\r\n</div>\r\n\n<br>\n\nStrategy 2 (advanced)\n\n<div class=\"aligned\">\r\n\r\n>| | |\r\n| -------------: | ---------- |\r\n| 90 | = 200 $-$ $\\dfrac{\\large m}{6}$ |\n| $\\dfrac{\\large m}{6}$ | \\= 200 $-$ 90 |\n| $\\large m$ | \\= 6 $\\times$ 110 |\r\n| | \\= {{{correctAnswer}}} |\r\n\r\n</div>"}]}]

  706. <div class="sm_mode"> {{{question}}} </div>

    [{"vars":[{"varval":"<div class=\"sm_img_inline\">\n\n![](https://teacher.smartermaths.com.au/wp-content/uploads/2018/04/NAPX-H4-NC08-SA1.svg) = 8\n\n![](https://teacher.smartermaths.com.au/wp-content/uploads/2018/04/NAPX-H4-NC08-SA1.svg) $\\times$ ![](https://teacher.smartermaths.com.au/wp-content/uploads/2018/04/NAPX-H4-NC08-SA.svg) = (![](https://teacher.smartermaths.com.au/wp-content/uploads/2018/04/NAPX-H4-NC08-SA1.svg) $\\times$ 3 ) + ![](https://teacher.smartermaths.com.au/wp-content/uploads/2018/04/NAPX-H4-NC08-SA.svg) + ![](https://teacher.smartermaths.com.au/wp-content/uploads/2018/04/NAPX-H4-NC08-SA.svg)\n\nWhat is the value of ![](https://teacher.smartermaths.com.au/wp-content/uploads/2018/04/NAPX-H4-NC08-SA.svg)?\n\n</div>\n"},{"varval":"Strategy 1\n\nTest each option:\n\n<div class=\"sm_img_inline\">\n\nIf ![](https://teacher.smartermaths.com.au/wp-content/uploads/2018/04/NAPX-H4-NC08-SA.svg) = 4,\n\n</div>\n\n<div class=\"aligned\">\n\n| | |\n| ---------------------: | -------------- |\n| 8 $\\times$ 4 | \\= 8 $\\times$ 3 + 4 + 4 |\n| 32 | \\= 32 &nbsp$\\checkmark$ |\n\n</div>\n\n<br>\n\nStrategy 2 (more advanced)\n\n<div class=\"sm_img_inline\">\n\n8 $\\times$ ![](https://teacher.smartermaths.com.au/wp-content/uploads/2018/04/NAPX-H4-NC08-SA.svg) = 8 $\\times$ 3 + ![](https://teacher.smartermaths.com.au/wp-content/uploads/2018/04/NAPX-H4-NC08-SA.svg) + ![](https://teacher.smartermaths.com.au/wp-content/uploads/2018/04/NAPX-H4-NC08-SA.svg)\n\n6 x ![](https://teacher.smartermaths.com.au/wp-content/uploads/2018/04/NAPX-H4-NC08-SA.svg) = 24\n\n$\\therefore$ &nbsp;\r ![](https://teacher.smartermaths.com.au/wp-content/uploads/2018/04/NAPX-H4-NC08-SA.svg) = {{{correctAnswer}}}\n\n</div>\n"}]},{"vars":[{"varval":"<div class=\"sm_img_inline\">\n\n![](https://teacher.smartermaths.com.au/wp-content/uploads/2018/04/NAPX-H4-NC08-SA1.svg) = 3\n\n![](https://teacher.smartermaths.com.au/wp-content/uploads/2018/04/NAPX-H4-NC08-SA1.svg) $\\times$ ![](https://teacher.smartermaths.com.au/wp-content/uploads/2018/04/NAPX-H4-NC08-SA.svg) = (![](https://teacher.smartermaths.com.au/wp-content/uploads/2018/04/NAPX-H4-NC08-SA1.svg) $\\times$ 3 ) + ![](https://teacher.smartermaths.com.au/wp-content/uploads/2018/04/NAPX-H4-NC08-SA.svg) + ![](https://teacher.smartermaths.com.au/wp-content/uploads/2018/04/NAPX-H4-NC08-SA.svg)\n\nWhat is the value of ![](https://teacher.smartermaths.com.au/wp-content/uploads/2018/04/NAPX-H4-NC08-SA.svg)?\n\n</div>\n"},{"varval":"Strategy 1\n\nTest each option:\n\n<div class=\"sm_img_inline\">\n\nIf ![](https://teacher.smartermaths.com.au/wp-content/uploads/2018/04/NAPX-H4-NC08-SA.svg) = 9,\n\n</div>\n\n<div class=\"aligned\">\n\n| | |\n| ---------------------: | -------------- |\n| 3 $\\times$ 9 | \\= 3 $\\times$ 3 + 9 + 9 |\n| 27 | \\= 27 &nbsp$\\checkmark$ |\n\n</div>\n\n<br>\n\nStrategy 2 (more advanced)\n\n<div class=\"sm_img_inline\">\n\n3 $\\times$ ![](https://teacher.smartermaths.com.au/wp-content/uploads/2018/04/NAPX-H4-NC08-SA.svg) = 3 $\\times$ 3 + ![](https://teacher.smartermaths.com.au/wp-content/uploads/2018/04/NAPX-H4-NC08-SA.svg) + ![](https://teacher.smartermaths.com.au/wp-content/uploads/2018/04/NAPX-H4-NC08-SA.svg)\n\n1 x ![](https://teacher.smartermaths.com.au/wp-content/uploads/2018/04/NAPX-H4-NC08-SA.svg) = 9\n\n$\\therefore$ &nbsp;\r ![](https://teacher.smartermaths.com.au/wp-content/uploads/2018/04/NAPX-H4-NC08-SA.svg) = {{{correctAnswer}}}\n\n</div>"}]},{"vars":[{"varval":"<div class=\"sm_img_inline\">\n\n![](https://teacher.smartermaths.com.au/wp-content/uploads/2018/04/NAPX-H4-NC08-SA1.svg) = 6\n\n![](https://teacher.smartermaths.com.au/wp-content/uploads/2018/04/NAPX-H4-NC08-SA1.svg) $\\times$ ![](https://teacher.smartermaths.com.au/wp-content/uploads/2018/04/NAPX-H4-NC08-SA.svg) = (![](https://teacher.smartermaths.com.au/wp-content/uploads/2018/04/NAPX-H4-NC08-SA1.svg) $\\times$ 2 ) + ![](https://teacher.smartermaths.com.au/wp-content/uploads/2018/04/NAPX-H4-NC08-SA.svg) + ![](https://teacher.smartermaths.com.au/wp-content/uploads/2018/04/NAPX-H4-NC08-SA.svg)\n\nWhat is the value of ![](https://teacher.smartermaths.com.au/wp-content/uploads/2018/04/NAPX-H4-NC08-SA.svg)?\n\n</div>\n"},{"varval":"Strategy 1\n\nTest each option:\n\n<div class=\"sm_img_inline\">\n\nIf ![](https://teacher.smartermaths.com.au/wp-content/uploads/2018/04/NAPX-H4-NC08-SA.svg) = 3,\n\n</div>\n\n<div class=\"aligned\">\n\n| | |\n| ---------------------: | -------------- |\n| 6 $\\times$ 3 | \\= 6 $\\times$ 2 + 3 + 3 |\n| 18 | \\= 18 &nbsp$\\checkmark$ |\n\n</div>\n\n<br>\n\nStrategy 2 (more advanced)\n\n<div class=\"sm_img_inline\">\n\n6 $\\times$ ![](https://teacher.smartermaths.com.au/wp-content/uploads/2018/04/NAPX-H4-NC08-SA.svg) = 6 $\\times$ 2 + ![](https://teacher.smartermaths.com.au/wp-content/uploads/2018/04/NAPX-H4-NC08-SA.svg) + ![](https://teacher.smartermaths.com.au/wp-content/uploads/2018/04/NAPX-H4-NC08-SA.svg)\n\n4 x ![](https://teacher.smartermaths.com.au/wp-content/uploads/2018/04/NAPX-H4-NC08-SA.svg) = 12\n\n$\\therefore$ &nbsp;\r ![](https://teacher.smartermaths.com.au/wp-content/uploads/2018/04/NAPX-H4-NC08-SA.svg) = {{{correctAnswer}}}\n\n</div>"}]},{"vars":[{"varval":"<div class=\"sm_img_inline\">\n\n![](https://teacher.smartermaths.com.au/wp-content/uploads/2018/04/NAPX-H4-NC08-SA1.svg) = 4\n\n![](https://teacher.smartermaths.com.au/wp-content/uploads/2018/04/NAPX-H4-NC08-SA1.svg) $\\times$ ![](https://teacher.smartermaths.com.au/wp-content/uploads/2018/04/NAPX-H4-NC08-SA.svg) = (![](https://teacher.smartermaths.com.au/wp-content/uploads/2018/04/NAPX-H4-NC08-SA1.svg) $\\times$ 5 ) + ![](https://teacher.smartermaths.com.au/wp-content/uploads/2018/04/NAPX-H4-NC08-SA.svg) + ![](https://teacher.smartermaths.com.au/wp-content/uploads/2018/04/NAPX-H4-NC08-SA.svg)\n\nWhat is the value of ![](https://teacher.smartermaths.com.au/wp-content/uploads/2018/04/NAPX-H4-NC08-SA.svg)?\n\n</div>\n"},{"varval":"Strategy 1\n\nTest each option:\n\n<div class=\"sm_img_inline\">\n\nIf ![](https://teacher.smartermaths.com.au/wp-content/uploads/2018/04/NAPX-H4-NC08-SA.svg) = 10,\n\n</div>\n\n<div class=\"aligned\">\n\n| | |\n| ---------------------: | -------------- |\n| 4 $\\times$ 10 | \\= 4 $\\times$ 5 + 10 + 10 |\n| 40 | \\= 40 &nbsp$\\checkmark$ |\n\n</div>\n\n<br>\n\nStrategy 2 (more advanced)\n\n<div class=\"sm_img_inline\">\n\n4 $\\times$ ![](https://teacher.smartermaths.com.au/wp-content/uploads/2018/04/NAPX-H4-NC08-SA.svg) = 4 $\\times$ 5 + ![](https://teacher.smartermaths.com.au/wp-content/uploads/2018/04/NAPX-H4-NC08-SA.svg) + ![](https://teacher.smartermaths.com.au/wp-content/uploads/2018/04/NAPX-H4-NC08-SA.svg)\n\n2 x ![](https://teacher.smartermaths.com.au/wp-content/uploads/2018/04/NAPX-H4-NC08-SA.svg) = 20\n\n$\\therefore$ &nbsp;\r ![](https://teacher.smartermaths.com.au/wp-content/uploads/2018/04/NAPX-H4-NC08-SA.svg) = {{{correctAnswer}}}\n\n</div>"}]},{"vars":[{"varval":"<div class=\"sm_img_inline\">\n\n![](https://teacher.smartermaths.com.au/wp-content/uploads/2018/04/NAPX-H4-NC08-SA1.svg) = 9\n\n![](https://teacher.smartermaths.com.au/wp-content/uploads/2018/04/NAPX-H4-NC08-SA1.svg) $\\times$ ![](https://teacher.smartermaths.com.au/wp-content/uploads/2018/04/NAPX-H4-NC08-SA.svg) = (![](https://teacher.smartermaths.com.au/wp-content/uploads/2018/04/NAPX-H4-NC08-SA1.svg) $\\times$ 2 ) + ![](https://teacher.smartermaths.com.au/wp-content/uploads/2018/04/NAPX-H4-NC08-SA.svg) + ![](https://teacher.smartermaths.com.au/wp-content/uploads/2018/04/NAPX-H4-NC08-SA.svg) + ![](https://teacher.smartermaths.com.au/wp-content/uploads/2018/04/NAPX-H4-NC08-SA.svg)\n\nWhat is the value of ![](https://teacher.smartermaths.com.au/wp-content/uploads/2018/04/NAPX-H4-NC08-SA.svg)?\n\n</div>\n"},{"varval":"Strategy 1\n\nTest each option:\n\n<div class=\"sm_img_inline\">\n\nIf ![](https://teacher.smartermaths.com.au/wp-content/uploads/2018/04/NAPX-H4-NC08-SA.svg) = 3,\n\n</div>\n\n<div class=\"aligned\">\n\n| | |\n| ---------------------: | -------------- |\n| 9 $\\times$ 3 | \\= 9 $\\times$ 2 + 3 + 3 + 3 |\n| 27 | \\= 27 &nbsp$\\checkmark$ |\n\n</div>\n\n<br>\n\nStrategy 2 (more advanced)\n\n<div class=\"sm_img_inline\">\n\n9 $\\times$ ![](https://teacher.smartermaths.com.au/wp-content/uploads/2018/04/NAPX-H4-NC08-SA.svg) = 9 $\\times$ 2 + ![](https://teacher.smartermaths.com.au/wp-content/uploads/2018/04/NAPX-H4-NC08-SA.svg) + ![](https://teacher.smartermaths.com.au/wp-content/uploads/2018/04/NAPX-H4-NC08-SA.svg) + ![](https://teacher.smartermaths.com.au/wp-content/uploads/2018/04/NAPX-H4-NC08-SA.svg)\n\n6 x ![](https://teacher.smartermaths.com.au/wp-content/uploads/2018/04/NAPX-H4-NC08-SA.svg) = 18\n\n$\\therefore$ &nbsp;\r ![](https://teacher.smartermaths.com.au/wp-content/uploads/2018/04/NAPX-H4-NC08-SA.svg) = {{{correctAnswer}}}\n\n</div>"}]},{"vars":[{"varval":"<div class=\"sm_img_inline\">\n\n![](https://teacher.smartermaths.com.au/wp-content/uploads/2018/04/NAPX-H4-NC08-SA1.svg) = 2\n\n![](https://teacher.smartermaths.com.au/wp-content/uploads/2018/04/NAPX-H4-NC08-SA1.svg) $\\times$ ![](https://teacher.smartermaths.com.au/wp-content/uploads/2018/04/NAPX-H4-NC08-SA.svg) = (![](https://teacher.smartermaths.com.au/wp-content/uploads/2018/04/NAPX-H4-NC08-SA1.svg) $\\times$ 3 ) + ![](https://teacher.smartermaths.com.au/wp-content/uploads/2018/04/NAPX-H4-NC08-SA.svg)\n\nWhat is the value of ![](https://teacher.smartermaths.com.au/wp-content/uploads/2018/04/NAPX-H4-NC08-SA.svg)?\n\n</div>\n"},{"varval":"Strategy 1\n\nTest each option:\n\n<div class=\"sm_img_inline\">\n\nIf ![](https://teacher.smartermaths.com.au/wp-content/uploads/2018/04/NAPX-H4-NC08-SA.svg) = 6,\n\n</div>\n\n<div class=\"aligned\">\n\n| | |\n| ---------------------: | -------------- |\n| 2 $\\times$ 6 | \\= 2 $\\times$ 3 + 6 |\n| 12 | \\= 12 &nbsp$\\checkmark$ |\n\n</div>\n\n<br>\n\nStrategy 2 (more advanced)\n\n<div class=\"sm_img_inline\">\n\n2 $\\times$ ![](https://teacher.smartermaths.com.au/wp-content/uploads/2018/04/NAPX-H4-NC08-SA.svg) = 2 $\\times$ 3 + ![](https://teacher.smartermaths.com.au/wp-content/uploads/2018/04/NAPX-H4-NC08-SA.svg) \n\n1 x ![](https://teacher.smartermaths.com.au/wp-content/uploads/2018/04/NAPX-H4-NC08-SA.svg) = 6\n\n$\\therefore$ &nbsp;\r ![](https://teacher.smartermaths.com.au/wp-content/uploads/2018/04/NAPX-H4-NC08-SA.svg) = {{{correctAnswer}}}\n\n</div>"}]}]

  707. <div class="sm_mode"> {{{question}}} </div>

    [{"vars":[{"varval":"This shape is made up of 9 squares.\n\n<br>\n\nsm_img https://teacher.smartermaths.com.au/wp-content/uploads/2018/05/NAPX-G4-NC19.svg 220 indent3 vpad\n\n<br>What fraction of the shape is unshaded?"},{"varval":"Dividing each square into 4 triangles:\r\n\nNumber of unshaded triangles = 10\n\n<div class=\"aligned\">\n\n| | |\n| --------------------- | -------------- |\n| $\\therefore$ Fraction unshaded | \\= $\\dfrac{10}{9 \\times 4}$ |\n|||\n| | \\= $\\dfrac{10}{36}$ |\n|||\n| | \\= {{{correctAnswer}}} |\n\n</div>"}]},{"vars":[{"varval":"This shape is made up of 9 squares.\n\n<br>\n\nsm_img https://teacher.smartermaths.com.au/wp-content/uploads/2018/05/NAPX-G4-NC19.svg 220 indent3 vpad\n\n<br>What fraction of the shape is shaded?"},{"varval":"Dividing each square into 4 triangles:\r\n\nNumber of shaded triangles = 26\n\n<div class=\"aligned\">\n\n| | |\n| --------------------- | -------------- |\n| $\\therefore$ Fraction shaded | \\= $\\dfrac{26}{9 \\times 4}$ |\n|||\n| | \\= $\\dfrac{26}{36}$ |\n|||\n| | \\= {{{correctAnswer}}} |\n\n</div>"}]},{"vars":[{"varval":"Mick has decided to tile his bathroom floor using shaded and unshaded tiles, as shown in the pattern below.\n\n<br>\n\nsm_img https://teacher.smartermaths.com.au/wp-content/uploads/2022/07/NAPX-G4-NC19-var2.svg 220 indent3 vpad\n\n<br>What fraction of the pattern is unshaded?"},{"varval":"Dividing each square into 4 triangles:\r\n\nNumber of unshaded triangles = 12\n\n<div class=\"aligned\">\n\n| | |\n| --------------------- | -------------- |\n| $\\therefore$ Fraction white | \\= $\\dfrac{12}{9 \\times 4}$ |\n|||\n| | \\= $\\dfrac{12}{36}$ |\n|||\n| | \\= {{{correctAnswer}}} |\n\n</div>"}]},{"vars":[{"varval":"Charlie has decided to tile his bathroom floor using shaded and unshaded tiles in the pattern below. \n<br>\n\nsm_img https://teacher.smartermaths.com.au/wp-content/uploads/2022/07/NAPX-G4-NC19-var3.svg 220 indent3 vpad\n\n<br>What fraction of the finished floor is shaded?"},{"varval":"Dividing each square into 4 triangles:\r\n\nNumber of unshaded triangles = 16\n\n<div class=\"aligned\">\n\n| | |\n| --------------------- | -------------- |\n| $\\therefore$ Fraction unshaded | \\= $\\dfrac{18}{9 \\times 4}$ |\n|||\n| | \\= $\\dfrac{16}{36}$ |\n|||\n| | \\= {{{correctAnswer}}} |\n\n</div>"}]},{"vars":[{"varval":"Juliet has decided to tile her balcony floor using shaded and unshaded tiles in the pattern below. \n<br><br>\n\nsm_img https://teacher.smartermaths.com.au/wp-content/uploads/2022/07/NAPX-G4-NC19-var4.svg 220 indent3 vpad\n\n\n<br>What fraction of the tiles will be unshaded?"},{"varval":"Dividing each square into 4 triangles:\r\n\nNumber of unshaded triangles = 20\n\n<div class=\"aligned\">\n\n| | |\n| --------------------- | -------------- |\n| $\\therefore$ Fraction unshaded | \\= $\\dfrac{20}{9 \\times 4}$ |\n|||\n| | \\= $\\dfrac{20}{36}$ |\n|||\n| | \\= {{{correctAnswer}}} |\n\n</div>"}]},{"vars":[{"varval":"Jamie has decided to tile his back deck using shaded and unshaded tiles in the pattern below. \n<br><br>\n\nsm_img https://teacher.smartermaths.com.au/wp-content/uploads/2022/07/NAPX-G4-NC19-var2.svg 220 indent3 vpad\n\n<br>What fraction of the tiles will be shaded?"},{"varval":"Dividing each square into 4 triangles:\r\n\nNumber of shaded triangles = 24\n\n<div class=\"aligned\">\n\n| | |\n| --------------------- | -------------- |\n| $\\therefore$ Fraction shaded | \\= $\\dfrac{24}{9 \\times 4}$ |\n|||\n| | \\= $\\dfrac{24}{36}$ |\n|||\n| | \\= {{{correctAnswer}}} |\n\n</div>"}]}]

  708. <div class="sm_mode"> {{{question}}} </div>

    [{"vars":[{"varval":"Vanessa has a basket of fruit that contains plums, oranges and peaches.\n\n\r\n\r\n$\\dfrac{1}{3}$ of the basket of fruit are oranges and $\\dfrac{1}{5}$ are peaches.\n\n\r\n\r\nWhat fraction of the fruit are plums?"},{"varval":"<div class=\"aligned\">\n\n| | |\n| --------------------- | -------------- |\n| <div style=\"vertical-align: text-bottom;\">Plums</div> | \\= 1 $-$ $\\bigg( \\dfrac{1}{3} + \\dfrac{1}{5} \\bigg)$ |\n| | \\= 1 $-$ $\\bigg( \\dfrac{5}{15} + \\dfrac{3}{15} \\bigg)$ |\n|| \\= {{{correctAnswer}}}|\n\n</div>"}]},{"vars":[{"varval":"A pet shop has fish, guinea pigs and rabbits for sale.\n\n\r\n\r\n$\\dfrac{1}{6}$ of the animals are guinea pigs and $\\dfrac{1}{8}$ are rabbits.\n\n\r\n\r\nWhat fraction of the animals are fish?"},{"varval":"<div class=\"aligned\">\n\n| | |\n| --------------------- | -------------- |\n| <div style=\"vertical-align: text-bottom;\">Fish</div> | \\= 1 $-$ $\\bigg( \\dfrac{1}{6} + \\dfrac{1}{8} \\bigg)$ |\n| | \\= 1 $-$ $\\bigg( \\dfrac{4}{24} + \\dfrac{3}{24} \\bigg)$ |\n|| \\= {{{correctAnswer}}}|\n\n</div>\n"}]},{"vars":[{"varval":"A kitchen utensils draw contains knives, forks and spoons.\n\n\r\n\r\n$\\dfrac{1}{3}$ of the utensils are knives and $\\dfrac{1}{8}$ are forks.\n\n\r\n\r\nWhat fraction of the utensils are spoons?"},{"varval":"<div class=\"aligned\">\n\n| | |\n| --------------------- | -------------- |\n| <div style=\"vertical-align: text-bottom;\">Spoons</div> | \\= 1 $-$ $\\bigg( \\dfrac{1}{3} + \\dfrac{1}{8} \\bigg)$ |\n| | \\= 1 $-$ $\\bigg( \\dfrac{8}{24} + \\dfrac{3}{24} \\bigg)$ |\n| | = {{correctAnswer}} |\n\n</div>\n\n\n"}]},{"vars":[{"varval":"A bag contains red, orange and blue coloured discs.\n\n\r\n\r\n$\\dfrac{1}{3}$ of the discs are red and $\\dfrac{2}{5}$ are orange.\n\n\r\n\r\nWhat fraction of the discs are blue?"},{"varval":"<div class=\"aligned\">\n\n| | |\n| --------------------- | -------------- |\n| <div style=\"vertical-align: text-bottom;\">Blue discs</div> | \\= 1 $-$ $\\bigg( \\dfrac{1}{3} + \\dfrac{2}{5} \\bigg)$ |\n| | \\= 1 $-$ $\\bigg( \\dfrac{5}{15} + \\dfrac{6}{15} \\bigg)$ |\n| | = {{correctAnswer}} |\n\n</div>\n\n"}]},{"vars":[{"varval":"Jonas has a hobby farm consisting of goats, sheep and cows.\n\n\r\n\r\n$\\dfrac{3}{7}$ of the animals are goats and $\\dfrac{2}{5}$ are cows.\n\n\r\n\r\nWhat fraction of the animals are sheep?"},{"varval":"<div class=\"aligned\">\n\n| | |\n| --------------------- | -------------- |\n| <div style=\"vertical-align: text-bottom;\">Sheep</div> | \\= 1 $-$ $\\bigg( \\dfrac{3}{7} + \\dfrac{2}{5} \\bigg)$ |\n| | \\= 1 $-$ $\\bigg( \\dfrac{15}{35} + \\dfrac{14}{35} \\bigg)$ |\n| | = {{correctAnswer}} |\n\n</div>\n"}]},{"vars":[{"varval":"Bailey watches Netflix most nights of the week. \n\nHe watches a mixture of movies, cartoons and documentaries.\n\n\r\n\r\nLast week, $\\dfrac{1}{8}$ of the shows he watched were cartoons and $\\dfrac{2}{3}$ were documentaries.\n\n\r\n\r\nWhat fraction of shows that he watched were movies?"},{"varval":"<div class=\"aligned\">\n\n| | |\n| --------------------- | -------------- |\n| <div style=\"vertical-align: text-bottom;\">Movies</div> | \\= 1 $-$ $\\bigg( \\dfrac{1}{8} + \\dfrac{2}{3} \\bigg)$ |\n| | \\= 1 $-$ $\\bigg( \\dfrac{3}{24} + \\dfrac{16}{24} \\bigg)$ |\n| | = {{correctAnswer}} |\n\n</div>\n"}]}]

  709. <div class="sm_mode"> {{{question}}} </div>

    [{"vars":[{"varval":"Brett has three 6-litre cans of paint.\n\nHe divides the paint up equally between his 24 art students.\n\nHow much paint will each student receive?\n\nWrite your answer as a decimal."},{"varval":"<div class=\"aligned\">\n\n| | |\n| ----------- | ------------ |\n| Total paint | \\= 3 × 6 |\n| | \\= 18 litres |\n\n</div>\n\n<br>\n\n<div class=\"aligned\">\n\n| | |\n| ------------------------------ | ------------------------------------- |\n| $\\therefore$ Paint per student | \\= $\\dfrac{18}{24}$ |\n| | \\= $\\dfrac{3}{4}$ |\n| | \\= {{{correctAnswer0}}} {{{suffix0}}} |\n\n</div>"}]},{"vars":[{"varval":"Dimitri has four 8-litre containers of olive oil.\n\nHe divides the olive oil equally into 80 bottles.\n\nHow much olive oil will each bottle contain?\n\nWrite your answer as a decimal."},{"varval":"<div class=\"aligned\">\n\n| | |\n| --------------- | ------------ |\n| Total olive oil | \\= 4 × 8 |\n| | \\= 32 litres |\n\n</div>\n\n<br>\n\n<div class=\"aligned\">\n\n| | |\n| ------------------------------ | ------------------------------------- |\n| $\\therefore$ Paint per student | \\= $\\dfrac{32}{80}$ |\n| | \\= $\\dfrac{2}{5}$ |\n| | \\= {{{correctAnswer0}}} {{{suffix0}}} |\n\n</div>"}]},{"vars":[{"varval":"Daniel has eight 7-kilogram boxes of lobster meat.\n\nHe divides the lobster meat equally amongst his 64 work colleagues.\n\nHow many kilograms of lobster meat will each work colleague receive?\n\nWrite your answer as a decimal."},{"varval":"<div class=\"aligned\">\n\n| | |\n| -------------- | --------------- |\n| Lobster meat | \\= 8 × 7 |\n| | \\= 56 kilograms |\n\n</div>\n\n<br>\n\n<div class=\"aligned\">\n\n| | |\n| --------------------------------------- | ------------------------------------- |\n| $\\therefore$ Lobster per work colleague | \\= $\\dfrac{56}{64}$ |\n| | \\= $\\dfrac{7}{8}$ |\n| | \\= {{{correctAnswer0}}} {{{suffix0}}} |\n\n</div>"}]},{"vars":[{"varval":"Samantha has twelve 3-metre lengths of ribbon.\n\nShe divides the ribbon equally amongst her 30 students.\n\nHow many metres of ribbon will each student receive?\n\nWrite your answer as a decimal."},{"varval":"<div class=\"aligned\">\n\n| | |\n| ------------ | ------------ |\n| Total ribbon | \\= 12 × 3 |\n| | \\= 36 metres |\n\n</div>\n\n<br>\n\n<div class=\"aligned\">\n\n| | |\n| ------------------------------- | ------------------------------------- |\n| $\\therefore$ Ribbon per student | \\= $\\dfrac{36}{30}$ |\n| | \\= $\\dfrac{6}{5}$ |\n| | \\= {{{correctAnswer0}}} {{{suffix0}}} |\n\n</div>"}]},{"vars":[{"varval":"James has fifteen 16-kilogram hay bales.\n\nHe divides the hay equally amongst his 64 cows.\n\nHow many kilograms of hay will each cow receive?\n\nWrite your answer as a decimal."},{"varval":"<div class=\"aligned\">\n\n| | |\n| ------------ | ---------------- |\n| Total hay | \\= 15 × 16 |\n| | \\= 240 kilograms |\n\n</div>\n\n<br>\n\n<div class=\"aligned\">\n\n| | |\n| ------------------------ | -------------------------------------- |\n| $\\therefore$ Hay per cow | \\= $\\dfrac{240}{64}$ |\n| | \\= $\\dfrac{15}{4}$ |\n| | \\= {{{correctAnswer0}}} {{{suffix0}}} |\n\n</div>"}]}]

  710. <div class="sm_mode"> {{{question}}} </div>

    [{"vars":[{"varval":"Darlene records the sprint times of her greyhound dogs in seconds, to three decimal places.\n\n\r\n\r\nHer fastest dog records a sprint time of 12.587 seconds.\n\n\r\n\r\nDarlene records this time in her journal, correct to two decimal places.\n\n\r\n\r\nWhat time, in seconds, does Darlene record?"},{"varval":"12.587 = {{{correctAnswer}}} (to 2 d.p.)"}]},{"vars":[{"varval":"Paul's stopwatch records his swimmers' lap times to three decimal places.\n\n\r\n\r\nPaul's fastest swimmer swam her freestyle lap in 26.386 seconds.\n\n\r\n\r\nPaul recorded this time in his notepad correct to two decimal places.\n\n\r\n\r\nWhat time, in seconds, does Paul record?"},{"varval":"26.386 = {{{correctAnswer}}} (to 2 decimal places)"}]},{"vars":[{"varval":"Kerry records the times of her one kilometre sprints in minutes, to three decimal places.\n\n\r\n\r\nHer fastest one kilometre a sprint time was 4.456 minutes.\n\n\r\n\r\nKerry records this time in her training app, correct to two decimal places.\n\n\r\n\r\nWhat time, in minutes, does Kerry record?"},{"varval":"4.456 = {{{correctAnswer}}} (to 2 d.p.)"}]},{"vars":[{"varval":"Jenny's scale weighs her watermelons in kilograms to three decimal places.\n\n\r\n\r\nJenny's largest watermelon weighs 10.498 kilograms.\n\n\r\n\r\nJenny recorded this weight correct to two decimal places.\n\n\r\n\r\nWhat weight, in kilograms, does Jenny record?"},{"varval":"10.498 = {{{correctAnswer}}} (to 2 d.p.)"}]},{"vars":[{"varval":"The 750 metre team sprint is an Olympic track cycling event.\n\nThe current men's world record time of 41.225 seconds is held by the Netherlands team.\n\nWhat time, in seconds, is the world record time correct to two decimal places?"},{"varval":"41.225 = {{{correctAnswer}}} (to 2 d.p.)"}]},{"vars":[{"varval":"The 500 metre time trial is an Olympic track cycling event.\n\nAnna Meares holds the current Australian women's 500 metre time trial record with a time of 32.836 seconds.\n\nWhat time, in seconds, is Anna's Australian record correct to two decimal places?"},{"varval":"32.836 = {{{correctAnswer}}} (to 2 d.p.)"}]}]

  711. <div class="sm_mode"> Curtis makes 14 pecan pies for his restaurant's dessert menu. Each pie uses $\dfrac{1}{4}$ kg of pecans. After he makes each pecan pie, he counts the total weight in pecans he has used. sm_img https://teacher.smartermaths.com.au/wp-content/uploads/2020/01/nap-L4-17-ver1.svg 210 indent vpad What is the last number he will count? </div>

    [{"vars":null}]

  712. <div class="sm_mode"> {{{question}}} </div>

    [{"vars":[{"varval":"This shape is made with three regular hexagons and three rhombuses.\n\n<br>\n\nsm_img https://teacher.smartermaths.com.au/wp-content/uploads/2018/08/NAPX-G3-NC24.svg 200 indent3 vpad\n\n<br>What fraction of the shape is shaded?"},{"varval":"The three shaded smaller rhombuses can be joined to form a regular hexagon.\n\nTherefore there is the equivalent of 4 whole hexagons in the shape.\n\n\r\n$\\therefore$ Fraction shaded is {{{correctAnswer}}}."}]},{"vars":[{"varval":"This shape is made with three regular hexagons and three rhombuses.\n\n<br>\n\nsm_img https://teacher.smartermaths.com.au/wp-content/uploads/2018/08/NAPX-G3-NC24.svg 200 indent3 vpad\n\n<br>What fraction of the shape is unshaded?"},{"varval":"The three shaded smaller rhombuses can be joined to form a regular hexagon.\n\nSo there is the equivalent of 2 shaded and 2 unshaded hexagons.\n\n\r\n$\\therefore$ Fraction unshaded is {{{correctAnswer}}}."}]},{"vars":[{"varval":"This shape is made with three regular hexagons and three rhombuses.\n\n<br>\n\nsm_img https://teacher.smartermaths.com.au/wp-content/uploads/2022/10/Number_NAPX-G3-NC24_v2.svg 200 indent3 vpad\n\n<br>What fraction of the shape is unshaded?"},{"varval":"The three hexagons can be divided into 9 rhombuses.\n\nSo there is the equivalent of 12 rhombuses in the whole shape.\n\nTherefore there is the equivalent of 5 shaded rhombuses and 7 unshaded rhombuses.\n\n\r\n$\\therefore$ Fraction unshaded is {{{correctAnswer}}}.\n\n\n"}]},{"vars":[{"varval":"This shape is made with three regular hexagons and three rhombuses.\n\n<br>\n\nsm_img https://teacher.smartermaths.com.au/wp-content/uploads/2022/10/Number_NAPX-G3-NC24_v2.svg 200 indent3 vpad\n\n<br>What fraction of the shape is shaded?"},{"varval":"The three hexagons can be divided into 9 rhombuses.\n\nSo there is the equivalent of 12 rhombuses in the whole shape.\n\nTherefore there is the equivalent of 5 shaded rhombuses and 7 unshaded rhombuses.\n\n\r\n$\\therefore$ Fraction shaded is {{{correctAnswer}}}.\n"}]},{"vars":[{"varval":"This shape is made with nine regular hexagons and nine rhombuses.\n\n<br>\n\nsm_img https://teacher.smartermaths.com.au/wp-content/uploads/2022/10/Number_NAPX-G3-NC24_v4_5.svg 240 indent3 vpad\n\n<br>What fraction of the shape is unshaded?"},{"varval":"Three rhombuses can be joined to form one regular hexagon.\n\nThere are 9 rhombuses and 9 hexagons.\n\nSo there is the equivalent of 12 hexagons in total.\n\nThe equivalent of 5 hexagons are shaded and 7 are unshaded\n\n\r\n$\\therefore$ Fraction unshaded is {{{correctAnswer}}}."}]},{"vars":[{"varval":"This shape is made with nine regular hexagons and nine rhombuses.\n\n<br>\n\nsm_img https://teacher.smartermaths.com.au/wp-content/uploads/2022/10/Number_NAPX-G3-NC24_v4_5.svg 240 indent3 vpad\n\n<br>What fraction of the shape is unshaded?"},{"varval":"Three rhombuses can be joined to form one regular hexagon.\n\nThere are 9 rhombuses and 9 hexagons.\n\nSo there is the equivalent of 12 hexagons in total.\n\nThe equivalent of 5 hexagons are shaded and 7 are unshaded\n\n\r\n$\\therefore$ Fraction shaded is {{{correctAnswer}}}."}]}]

  713. <div class="sm_mode"> {{{question}}} </div>

    [{"vars":[{"varval":"<span class=\"sm-text color5\">?</span> + $\\dfrac{4}{7} = 2$\n\nWhat makes this number sentence correct?\n"},{"varval":"<div class=\"aligned\">\n\n| | |\n| ---------------------: | -------------- |\n| <span class=\"sm-text color5\">?</span> + $\\dfrac{4}{7}$ | \\= 2\n| | |\n| | |\n| <span class=\"sm-text color5\">?</span> | \\= $\\dfrac{14}{7} - \\dfrac{4}{7}$ |\n| | |\n| <span class=\"sm-text color5\">?</span> | \\= {{{correctAnswer}}} |\n\n</div>"}]},{"vars":[{"varval":"<span class=\"sm-text color5\">?</span> + $\\dfrac{2}{3} = 3$\n\nWhat makes this number sentence correct?\n"},{"varval":"<div class=\"aligned\">\n\n| | |\n| ---------------------: | -------------- |\n| <span class=\"sm-text color5\">?</span> + $\\dfrac{2}{3}$ | \\= 3\n| | |\n| | |\n| <span class=\"sm-text color5\">?</span> | \\= $\\dfrac{9}{3} - \\dfrac{2}{3}$ |\n| | |\n| <span class=\"sm-text color5\">?</span> | \\= {{{correctAnswer}}} |\n\n</div>"}]},{"vars":[{"varval":"<span class=\"sm-text color5\">?</span> + $\\dfrac{3}{8} = 2$\n\nWhat makes this number sentence correct?\n"},{"varval":"<div class=\"aligned\">\n\n| | |\n| ---------------------: | -------------- |\n| <span class=\"sm-text color5\">?</span> + $\\dfrac{3}{8}$ | \\= 2\n| | |\n| | |\n| <span class=\"sm-text color5\">?</span> | \\= $\\dfrac{16}{8} - \\dfrac{3}{8}$ |\n| | |\n| <span class=\"sm-text color5\">?</span> | \\= {{{correctAnswer}}} |\n\n</div>\n"}]},{"vars":[{"varval":"<span class=\"sm-text color5\">?</span> + $\\dfrac{4}{11} = 1$\n\nWhat makes this number sentence correct?"},{"varval":"<div class=\"aligned\">\n\n| | |\n| ---------------------: | -------------- |\n| <span class=\"sm-text color5\">?</span> + $\\dfrac{4}{11}$ | \\= 1\n| | |\n| | |\n| <span class=\"sm-text color5\">?</span> | \\= $\\dfrac{11}{11} - \\dfrac{4}{11}$ |\n| | |\n| <span class=\"sm-text color5\">?</span> | \\= {{{correctAnswer}}} |\n\n</div>"}]},{"vars":[{"varval":"<span class=\"sm-text color5\">?</span> + $\\dfrac{7}{10} = 4$\n\nWhat makes this number sentence correct?"},{"varval":"<div class=\"aligned\">\n\n| | |\n| ---------------------: | -------------- |\n| <span class=\"sm-text color5\">?</span> + $\\dfrac{7}{10}$ | \\= 4\n| | |\n| | |\n| <span class=\"sm-text color5\">?</span> | \\= $\\dfrac{40}{10} - \\dfrac{7}{10}$ |\n| | |\n| <span class=\"sm-text color5\">?</span> | \\= $\\dfrac{33}{10}$ |\n| <span class=\"sm-text color5\">?</span> | \\= {{{correctAnswer}}} |\n\n</div>"}]},{"vars":[{"varval":"<span class=\"sm-text color5\">?</span> + $\\dfrac{11}{12} = 3$\n\nWhat makes this number sentence correct?"},{"varval":"<div class=\"aligned\">\n\n| | |\n| ---------------------: | -------------- |\n| <span class=\"sm-text color5\">?</span> + $\\dfrac{11}{12}$ | \\= 3\n| | |\n| | |\n| <span class=\"sm-text color5\">?</span> | \\= $\\dfrac{36}{12} - \\dfrac{11}{12}$ |\n| | |\n| <span class=\"sm-text color5\">?</span> | \\= $\\dfrac{25}{12}$ |\n| <span class=\"sm-text color5\">?</span> | \\= {{{correctAnswer}}} |\n\n</div>"}]}]

  714. <div class="sm_mode"> sm_img https://teacher.smartermaths.com.au/wp-content/uploads/2018/08/NAPX-F3-CA14.svg 450 indent vpad What number is added to $3\dfrac{1}{2}$ to equal $4\dfrac{1}{4}$? </div>

    [{"vars":null}]

  715. <div class="sm_mode"> Darwin has 10 teaspoons of sugar to use for making 2 cookies. He used $4\dfrac{1}{2}$ teaspoons of sugar in the first cookie and $4\dfrac{1}{4}$ teaspoons in the second one. How many teaspoons of sugar did Darwin have left? </div>

    [{"vars":null}]

  716. <div class="sm_mode"> {{{question}}} </div>

    [{"vars":[{"varval":"A couple decided to get married at a resort and sent out invitations to 120 guests.\n\nIf &nbsp;$\\dfrac{3}{5}$ &nbsp;responded that they could attend, how many guests would be at the wedding?"},{"varval":"<div class=\"aligned\">\n\n|||\n|-|-|\n|Guests Attending|= $\\dfrac{3}{5} \\times 120$|\n||= $3 \\times 24$|\n||= {{{correctAnswer0}}}|\n\n</div>"}]},{"vars":[{"varval":"Bill races pigeons and owns 80.\n\nA structure he built can shelter &nbsp;$\\dfrac{2}{5}$ &nbsp;of the pigeons.\n\nHow many pigeons can be sheltered under the structure?"},{"varval":"<div class=\"aligned\">\n\n|||\n|-|-|\n|Pigeons sheltered|= $\\dfrac{2}{5} \\times 80$|\n||= $2 \\times 16$|\n||= {{{correctAnswer0}}}|\n\n</div>"}]},{"vars":[{"varval":"Alvin and Vivian were batting in a cricket match where Vivian made 160 runs.\n\nIf Alvin made &nbsp;$\\dfrac{2}{5}$ &nbsp;of the runs that Vivian made, how many runs did Alvin score?"},{"varval":"<div class=\"aligned\">\n\n|||\n|-|-|\n|Alvin's score|= $\\dfrac{2}{5} \\times 160$|\n||= $2 \\times 32$|\n||= {{{correctAnswer0}}}|\n\n</div>"}]},{"vars":[{"varval":"Cadel cycled 140 kilometres on a long training ride.\n\nLeisa cycled &nbsp;$\\dfrac{3}{5}$ &nbsp;of the distance that Cadel rode.\n\nHow many kilometres did Leisa cycle?"},{"varval":"<div class=\"aligned\">\n\n|||\n|-|-|\n|Leisa's distance (km)|= $\\dfrac{3}{5} \\times 140$|\n||= $3 \\times 28$|\n||= {{{correctAnswer0}}}|\n\n</div>"}]}]

  717. In Variant 2 worked solution had ∴ Cost for 60 minutes. Should have been ∴ Cost for 45 minutes

    <div class="sm_mode"> {{{question}}} </div>

    [{"vars":[{"varval":"Ziggy rents a hang glider from a hire company.\n\n\r\n\r\nThe cost to rent the hang glider is directly proportional to the number of minutes between the original hire time and when it is returned.\n\n\r\n\r\nZiggy's rental cost for 3 hours and 20 minutes is $92.50.\n\n\r\n\r\nWhat will be Ziggy's rental cost if he returns the hang glider after 60 minutes?"},{"varval":"sm_nogap Rental cost per minute\n\n<div class=\"aligned\">\n\n>>|||\n|-|-|\n||= $\\dfrac{92.50}{200}$|\n||= 46.25 cents|\n\n\r\n</div>\n\n<br>\n\n\r\nsm_nogap $\\therefore$ Cost for 60 minutes\n\n<div class=\"aligned\">\n\n>>|||\n|-|-|\n||= 60 × 46.25|\n||= {{{prefix0}}}{{{correctAnswer0}}}|\n\n\r\n\r\n\r\n</div>"}]},{"vars":[{"varval":"Brett rents a sailing boat from a boat rental shop.\n\nThe cost to rent the sailing boat is directly proportional to the number of minutes between the original hire time and when it is returned.\n\nBrett's rental cost for 1 hours and 50 minutes is $82.50.\n\n\r\nWhat will be Brett's rental cost if he returns the sailing boat after 60 minutes?"},{"varval":"sm_nogap Rental cost per minute\n\n<div class=\"aligned\">\n\n>>|||\n|-|-|\n||= $\\dfrac{82.50}{110}$|\n||= 75 cents|\n\n\r\n</div>\n\n<br>\n\n\r\nsm_nogap $\\therefore$ Cost for 60 minutes\n\n<div class=\"aligned\">\n\n>>|||\n|-|-|\n||= 60 × 0.75|\n||= {{{prefix0}}}{{{correctAnswer0}}}|\n\n\r\n\r\n\r\n</div>"}]},{"vars":[{"varval":"Rhonda rents a stand-up paddle board from a surf shop.\n\nThe cost to rent the stand-up paddle board is directly proportional to the number of minutes between the original hire time and when it is returned.\n\nRhonda's rental cost for 1 hours and 10 minutes is $31.50\n\n\r\nWhat will be Rhonda's rental cost if she returns the stand-up paddle board after 45 minutes?"},{"varval":"sm_nogap Rental cost per minute\n\n<div class=\"aligned\">\n\n>>|||\n|-|-|\n||= $\\dfrac{31.50}{70}$|\n||= 45 cents|\n\n\r\n</div>\n\n<br>\n\n\r\nsm_nogap $\\therefore$ Cost for 45 minutes\n\n<div class=\"aligned\">\n\n>>|||\n|-|-|\n||= 45 × 0.45|\n||= {{{prefix0}}}{{{correctAnswer0}}}|\n\n\r\n\r\n\r\n</div>"}]},{"vars":[{"varval":"Donald rents an e-scooter on the Gold Coast.\n\nThe cost to rent the e-scooter is directly proportional to the number of minutes Donald uses it.\n\nDonald's rental cost for 2 hours and 10 minutes is $45.50.\n\n\r\nWhat will be Donald's rental cost if he uses the e-scooter for only 30 minutes?"},{"varval":"sm_nogap Rental cost per minute\n\n<div class=\"aligned\">\n\n>>|||\n|-|-|\n||= $\\dfrac{45.50}{130}$|\n||= 35 cents|\n\n\r\n</div>\n\n<br>\n\n\r\nsm_nogap $\\therefore$ Cost for 30 minutes\n\n<div class=\"aligned\">\n\n>>|||\n|-|-|\n||= 30 × 0.35|\n||= {{{prefix0}}}{{{correctAnswer0}}}|\n\n\r\n\r\n\r\n</div>"}]},{"vars":[{"varval":"Duff rents an e-bike in Dubbo to explore the town.\n\nThe cost to rent the e-bike is directly proportional to the number of minutes Duff uses it.\n\nDuff's rental cost for 3 hours and 30 minutes is $100.80.\n\n\r\nWhat will be Duff's rental cost if he uses the e-bike for only 60 minutes?"},{"varval":"sm_nogap Rental cost per minute\n\n<div class=\"aligned\">\n\n>>|||\n|-|-|\n||= $\\dfrac{100.80}{210}$|\n||= 48 cents|\n\n\r\n</div>\n\n<br>\n\n\r\nsm_nogap $\\therefore$ Cost for 60 minutes\n\n<div class=\"aligned\">\n\n>>|||\n|-|-|\n||= 60 × 0.48|\n||= {{{prefix0}}}{{{correctAnswer0}}}|\n\n\r\n\r\n\r\n</div>"}]},{"vars":[{"varval":"Boonie parks in Launceston while he visits a cricket bat factory.\n\nThe cost to park is directly proportional to the number of minutes Boonie leaves his car there.\n\nBoonie's rental cost for 1 hour and 20 minutes is $5.20.\n\n\r\nWhat would Boonie's parking cost have been if he parked for only 50 minutes?"},{"varval":"sm_nogap Parking cost per minute\n\n<div class=\"aligned\">\n\n>>|||\n|-|-|\n||= $\\dfrac{5.20}{80}$|\n||= 6.5 cents|\n\n\r\n</div>\n\n<br>\n\n\r\nsm_nogap $\\therefore$ Cost for 50 minutes\n\n<div class=\"aligned\">\n\n>>|||\n|-|-|\n||= 50 × 0.065|\n||= {{{prefix0}}}{{{correctAnswer0}}}|\n\n\r\n\r\n\r\n</div>"}]}]

  718. This will be a pain in the backside, but if possible: Var1 ... can you make colours on each side more of shades of a single colour, and the dimensions of first row look a bit too rectangular Var4 ... dimensions of RHS looks a bit too rectangular Var5 ... dimensions of RHS looks a bit too rectangular - if changes take ages, I can live with the current images

    <div class="sm_mode"> {{{question}}} </div>

    [{"vars":[{"varval":"\nThe rectangular prism, shown below, is cut into 16 identical cubes.\n\n<br>\n\nsm_img https://teacher.smartermaths.com.au/wp-content/uploads/2022/08/Measurement_NAPX9-TLF-CA35-SA-v3_0-1.svg 300 indent2 vpad\n\n<br>What is the ratio of the surface area of the rectangular prism to the surface area of one of the cubes?\n"},{"varval":"sm_nogap S.A. of rectangular prism\r\n\n>> = 2 × (2$\\large s$ × 2$\\large s$) + 4 × (4$\\large s$ × 2$\\large s$)\n\n>> = 8$\\large s$$^2$ + 32$\\large s$$^2$\r\n\n>> = 40$\\large s$$^2$\r\n\n<br>\n \r\n\r\nsm_nogap S.A. of cube\r\n\n>> = $6 × (\\large s$ × $\\large s)$\r\n\n>> = 6$\\large s$$^2$\r\n\n<br>\n \r\n\r\nsm_nogap $\\therefore$ Ratio of prism to cube\n\n\r\n>> = 40$\\large s$$^2$ : 6$\\large s$$^2$\n\n\r\n>> = {{{correctAnswer}}}\n"}]},{"vars":[{"varval":"The rectangular prism, shown below, is cut into 20 identical cubes.\n\n<br>\n\nsm_img https://teacher.smartermaths.com.au/wp-content/uploads/2022/08/Measurement_NAPX9-TLF-CA35-SA-v3_1.svg 300 indent2 vpad\n\n<br>What is the ratio of the surface area of the rectangular prism to the surface area of one of the cubes?\n"},{"varval":"sm_nogap S.A. of rectangular prism\r\n\n>> = 2 × (2$\\large s$ × 2$\\large s$) + 4 × (5$\\large s$ × 2$\\large s$)\n\n>> = 8$\\large s$$^2$ + 40$\\large s$$^2$\r\n\n>> = 48$\\large s$$^2$\r\n\n<br>\n \r\n\r\nsm_nogap S.A. of cube\r\n\n>> = $6 × (\\large s$ × $\\large s)$\r\n\n>> = 6$\\large s$$^2$\r\n\n<br>\n \r\n\r\nsm_nogap $\\therefore$ Ratio of prism to cube\n\n\r\n>> = 48$\\large s$$^2$ : 6$\\large s$$^2$\n\n\r\n>> = {{{correctAnswer}}}"}]},{"vars":[{"varval":"The rectangular prism, shown below, is cut into 12 identical cubes.\n\n<br>\n\nsm_img https://teacher.smartermaths.com.au/wp-content/uploads/2022/08/Measurement_NAPX9-TLF-CA35-SA-v3_2.svg 300 indent2 vpad\n\n<br>What is the ratio of the surface area of the rectangular prism to the surface area of one of the cubes?\n"},{"varval":"sm_nogap S.A. of rectangular prism\r\n\n>> = 2 × (2$\\large s$ × 2$\\large s$) + 4 × (3$\\large s$ × 2$\\large s$)\n\n>> = 8$\\large s$$^2$ + 24$\\large s$$^2$\r\n\n>> = 32$\\large s$$^2$\r\n\n<br>\n \r\n\r\nsm_nogap S.A. of cube\r\n\n>> = $6 × (\\large s$ × $\\large s)$\r\n\n>> = 6$\\large s$$^2$\r\n\n<br>\n \r\n\r\nsm_nogap $\\therefore$ Ratio of prism to cube\n\n\r\n>> = 32$\\large s$$^2$ : 6$\\large s$$^2$\n\n\r\n>> = {{{correctAnswer}}}"}]},{"vars":[{"varval":"The rectangular prism, shown below, is cut into 24 identical cubes.\n\n<br>\n\nsm_img https://teacher.smartermaths.com.au/wp-content/uploads/2022/08/Measurement_NAPX9-TLF-CA35-SA-v3_3.svg 330 indent2 vpad\n\n<br>What is the ratio of the surface area of the rectangular prism to the surface area of one of the cubes?\n"},{"varval":"sm_nogap S.A. of rectangular prism\r\n\n>> = 2 × (3$\\large s$ × 2$\\large s$) + 2 × (4$\\large s$ × 2$\\large s$) + 2 × (4$\\large s$ × 3$\\large s$)\n\n>> = 12$\\large s$$^2$ + 16$\\large s$$^2$ + 24$\\large s$$^2$\r\n\n>> = 52$\\large s$$^2$\r\n\n<br>\n \r\n\r\nsm_nogap S.A. of cube\r\n\n>> = $6 × (\\large s$ × $\\large s)$\r\n\n>> = 6$\\large s$$^2$\r\n\n<br>\n \r\n\r\nsm_nogap $\\therefore$ Ratio of prism to cube\n\n\r\n>> = 52$\\large s$$^2$ : 6$\\large s$$^2$\n\n\r\n>> = {{{correctAnswer}}}\n"}]},{"vars":[{"varval":"The rectangular prism, shown below, is cut into 60 identical cubes.\n\n<br>\n\nsm_img https://teacher.smartermaths.com.au/wp-content/uploads/2022/08/Measurement_NAPX9-TLF-CA35-SA-v3_4.svg 400 indent2 vpad\n\n<br>What is the ratio of the surface area of the rectangular prism to the surface area of one of the cubes?\n"},{"varval":"sm_nogap S.A. of rectangular prism\r\n\n>> = 2 × (5$\\large s$ × 3$\\large s$) + 2 × (4$\\large s$ × 3$\\large s$) + 2 × (5$\\large s$ × 4$\\large s$)\n\n>> = 30$\\large s$$^2$ + 24$\\large s$$^2$ + 40$\\large s$$^2$\r\n\n>> = 94$\\large s$$^2$\r\n\n<br>\n \r\n\r\nsm_nogap S.A. of cube\r\n\n>> = $6 × (\\large s$ × $\\large s)$\r\n\n>> = 6$\\large s$$^2$\r\n\n<br>\n \r\n\r\nsm_nogap $\\therefore$ Ratio of prism to cube\n\n\r\n>> = 94$\\large s$$^2$ : 6$\\large s$$^2$\n\n\r\n>> = {{{correctAnswer}}}\n"}]},{"vars":[{"varval":"\nThe rectangular prism, shown below, is cut into 24 identical cubes.\n\n<br>\n\nsm_img https://teacher.smartermaths.com.au/wp-content/uploads/2022/08/Measurement_NAPX9-TLF-CA35-SA-v3_5.svg 300 indent2 vpad\n\n<br>What is the ratio of the surface area of the rectangular prism to the surface area of one of the cubes?\n"},{"varval":"sm_nogap S.A. of rectangular prism\r\n\n>> = 2 × (2$\\large s$ × 2$\\large s$) + 4 × (6$\\large s$ × 2$\\large s$)\n\n>> = 8$\\large s$$^2$ + 48$\\large s$$^2$\r\n\n>> = 56$\\large s$$^2$\r\n\n<br>\n \r\n\r\nsm_nogap S.A. of cube\r\n\n>> = $6 × (\\large s$ × $\\large s)$\r\n\n>> = 6$\\large s$$^2$\r\n\n<br>\n \r\n\r\nsm_nogap $\\therefore$ Ratio of prism to cube\n\n\r\n>> = 56$\\large s$$^2$ : 6$\\large s$$^2$\n\n\r\n>> = {{{correctAnswer}}}\n"}]}]

  719. <div class="sm_mode"> {{{question}}} </div>

    [{"vars":[{"varval":"A cement slab is laid in Yvette's backyard that forms an 8 m × 4 m rectangle.\n\n<br>\n\nsm_img https://teacher.smartermaths.com.au/wp-content/uploads/2018/06/NAPX-H4-CA28-SA2.svg 600 indent vpad\n\n<br>Yvette is going to create a full perimeter around the outside of her slab using 250 mm × 250 mm plastic tiles.\n\n\r\n\r\nShe will use 50% black tiles costing $6 per tile and 50% white tiles costing $7 per tile.\n\n\r\n\r\nWhat is the total cost of the tiles she used?"},{"varval":"sm_nogap Tiles used on length (including extra width)\n\n>> = (8 ÷ 0.25) + 2\n\n>> = 34\n \r\n\r\nsm_nogap Tiles used on width\n\n>> = (4 ÷ 0.25)\n\n>> = 16\r\n\n\n \r\n\r\nsm_nogap Total tiles used\r\n\n>> = 2 × (34 + 16)\r\n\n>> = 100\n\n\n \r\n\r\nsm_nogap $\\therefore$ Total cost of tiles\r\n\n>> = 50 × 6 + 50 × 7\r\n\n>> = {{{prefix0}}}{{{correctAnswer0}}}"}]},{"vars":[{"varval":"Luke builds a wooden deck in his backyard that forms a 12 m × 5 m rectangle.\n\n<br>\n\n\nsm_img https://teacher.smartermaths.com.au/wp-content/uploads/2022/06/NAPX-H4-CA28-SA2-v1.svg 600 indent vpad\n\n<br>He is going to create a full perimeter around the outside of his deck using 500 mm × 500 mm stone pavers.\n\n\r\n\r\nHe will use 50% black pavers costing $10 per paver and 50% white pavers costing $8 per paver.\n\n\r\n\r\nWhat is the total cost of the pavers he used?"},{"varval":"sm_nogap Pavers used on length (including extra width)\n\n>> = (12 ÷ 0.5) + 2\n\n>> = 26\n \r\n\r\nsm_nogap Pavers used on width\n\n>> = (5 ÷ 0.5)\n\n>> = 10\r\n\n \r\n\r\nsm_nogap Total tiles used\r\n\n>> = 2 × (26 + 10)\r\n\n>> = 72\n \r\n\r\nsm_nogap $\\therefore$ Total cost of tiles\r\n\n>> = 36 × 10 + 36 × 8\n\n>> = {{{prefix0}}}{{{correctAnswer0}}}"}]},{"vars":[{"varval":"A cement slab is laid in Pandora's backyard that forms an 6 m × 5 m rectangle.\n\n<br>\n\nsm_img https://teacher.smartermaths.com.au/wp-content/uploads/2022/06/NAPX-H4-CA28-SA2-v2.svg 490 indent vpad\n\n<br>Pandora is going to create a full perimeter around the outside of her slab using 250 mm × 250 mm plastic tiles.\n\n\r\n\r\nShe will use 50% blue tiles costing $11 per tile and 50% white tiles costing $9 per tile.\n\n\r\n\r\nWhat is the total cost of the tiles she used?"},{"varval":"sm_nogap Tiles used on length (including extra width)\n\n>> = (6 ÷ 0.25) + 2\n\n>> = 26\n \r\n\r\nsm_nogap Tiles used on width\n\n>> = (5 ÷ 0.25)\n\n>> = 20\r\n\n\n \r\n\r\nsm_nogap Total tiles used\r\n\n>> = 2 × (26 + 20)\r\n\n>> = 92\n\n\n \r\n\r\nsm_nogap $\\therefore$ Total cost of tiles\r\n\n>> = 46 × 11 + 46 × 9\n\n>> = {{{prefix0}}}{{{correctAnswer0}}}"}]},{"vars":[{"varval":"A cement foundation for a deck is laid in Priscilla's backyard that forms an 6 m × 3 m rectangle.\n\n<br>\n\n\nsm_img https://teacher.smartermaths.com.au/wp-content/uploads/2022/06/NAPX-H4-CA28-SA2-v3.svg 610 indent vpad\n\n<br>Priscilla is going to create a full perimeter around the outside of the deck foundation using 750 mm × 750 mm plastic tiles.\n\n\r\n\r\nShe will use 50% teal tiles costing $14 per tile and 50% blue tiles costing $11 per tile.\n\n\r\n\r\nWhat is the total cost of the tiles she used?"},{"varval":"sm_nogap Tiles used on length (including extra width)\n\n>> = (6 ÷ 0.75) + 2\n\n>> = 10\n \r\n\r\nsm_nogap Tiles used on width\n\n>> = (3 ÷ 0.75)\n\n>> = 4\n\n \r\n\r\nsm_nogap Total tiles used\r\n\n>> = 2 × (10 + 4)\r\n\n>> = 28\n\n \r\n\r\nsm_nogap $\\therefore$ Total cost of tiles\r\n\n>> = 14 × 14 + 14 × 11\n\n>> = {{{prefix0}}}{{{correctAnswer0}}}"}]},{"vars":[{"varval":"A cement slab is laid in Carmen's backyard that forms an 9 m × 9 m square.\n\n<br>\n\n\nsm_img https://teacher.smartermaths.com.au/wp-content/uploads/2022/06/NAPX-H4-CA28-SA2-v4.svg 500 indent vpad\n\n<br>Carmen is going to create a full perimeter around the outside of her slab using 300 mm × 300 mm plastic tiles.\n\n\r\n\r\nShe will use 50% black tiles costing $8 per tile and 50% red tiles costing $10 per tile.\n\n\r\n\r\nWhat is the total cost of the tiles she used?"},{"varval":"sm_nogap Tiles used on top and bottom (including extra width)\n\n>> = (9 ÷ 0.3) + 2\n\n>> = 32\n\n \r\n\r\nsm_nogap Tiles used on sides\n\n>> = (9 ÷ 0.3)\n\n>> = 30\n\n \r\n\r\nsm_nogap Total tiles used\r\n\n>> = 2 × (32 + 30)\r\n\n>> = 124\n\n \r\n\r\nsm_nogap $\\therefore$ Total cost of tiles\r\n\n>> = 62 × 8 + 62 × 10\n\n>> = {{{prefix0}}}{{{correctAnswer0}}}"}]}]

  720. <div class="sm_mode"> Joplin bought a hat that cost $27.50 before the Goods and Services Tax (GST) was added. sm_img https://teacher.smartermaths.com.au/wp-content/uploads/2018/05/NAPX-I4-NC28rev.svg 200 indent3 vpad How much did Joplin pay for the hat after adding the 10% GST? </div>

    [{"vars":null}]

  721. <div class="sm_mode"> {{{question}}} </div>

    [{"vars":[{"varval":"During a night shift, Dan worked for 4 hours at his normal pay rate, 4 hours at one and a half times his normal pay rate, and another 3 hours at twice his normal pay rate.\n\n\r\n\r\nDan was paid $452 for that night shift.\n\n\r\n\r\nHow much is Dan's normal pay rate?"},{"varval":"sm_nogap Total normal hours\n\n<div class=\"aligned\">\n\n>>|||\n|-|-|\n||= 4 + (4 × 1.5) + (3 × 2)|\n||= 16 hours|\n\n</div>\n\r\n<br>\n\r\nsm_nogap $\\therefore$ Normal pay rate\r\n\n<div class=\"aligned\">\n\n>>|||\n|-|-|\n||= $\\dfrac{452}{16}$|\n||= {{{prefix0}}}{{{correctAnswer0}}} per hour.|\n\n</div>"}]},{"vars":[{"varval":"During a music festival, Carly worked for 4 hours at her normal pay rate, 2 hours at one and a half times her normal pay rate, and another 3 hours at twice her normal pay rate.\n\n\r\n\r\nCarly was paid $357.50 for that shift.\n\n\r\n\r\nHow much is Carly's normal pay rate?"},{"varval":"sm_nogap Total normal hours\n\n<div class=\"aligned\">\n\n>>|||\n|-|-|\n||= 4 + (2 × 1.5) + (3 × 2)|\n||= 13 hours|\n\n</div>\n\r\n<br>\n\r\nsm_nogap $\\therefore$ Normal pay rate\r\n\n<div class=\"aligned\">\n\n>>|||\n|-|-|\n||= $\\dfrac{357.50}{13}$|\n||= {{{prefix0}}}{{{correctAnswer0}}} per hour.|\n\n</div>"}]},{"vars":[{"varval":"During her shift at the bowling alley, Ginger worked for 6 hours at her normal pay rate, 4 hours at one and a half times her normal pay rate, and another 2 hours at twice her normal pay rate.\n\r\nGinger was paid $476 for that shift.\n\r\nHow much is Ginger's normal pay rate?"},{"varval":"sm_nogap Total normal hours\n\n<div class=\"aligned\">\n\n>>|||\n|-|-|\n||= 6 + (4 × 1.5) + (2 × 2)|\n||= 16 hours|\n\n</div>\n\r\n<br>\n\r\nsm_nogap $\\therefore$ Normal pay rate\r\n\n<div class=\"aligned\">\n\n>>|||\n|-|-|\n||= $\\dfrac{476}{16}$|\n||= {{{prefix0}}}{{{correctAnswer0}}} per hour.|\n\n</div>"}]},{"vars":[{"varval":"During a night shift at the hospital, Sarah worked for 7 hours at her normal pay rate, 3 hours at one and a half times her normal pay rate, and another 3 hours at twice her normal pay rate.\n\n\r\n\r\nSarah was paid $745.50 for that shift.\n\n\r\n\r\nHow much is Sarah's normal pay rate?"},{"varval":"sm_nogap Total normal hours\n\n<div class=\"aligned\">\n\n>>|||\n|-|-|\n||= 7 + (3 × 1.5) + (3 × 2)|\n||= 17.5 hours|\n\n</div>\n\r\n<br>\n\r\nsm_nogap $\\therefore$ Normal pay rate\r\n\n<div class=\"aligned\">\n\n>>|||\n|-|-|\n||= $\\dfrac{745.50}{17.5}$|\n||= {{{prefix0}}}{{{correctAnswer0}}} per hour.|\n\n</div>"}]},{"vars":[{"varval":"Fi teaches learn to swim classes. During one shift, she worked for 3 hours at her normal pay rate, 2 hours at one and a half times her normal pay rate, and another 2 hours at twice her normal pay rate.\n\n\r\n\r\nFi was paid $289 for that shift.\n\n\r\n\r\nHow much is Fi's normal pay rate?"},{"varval":"sm_nogap Total normal hours\n\n<div class=\"aligned\">\n\n>>|||\n|-|-|\n||= 3 + (2 × 1.5) + (2 × 2)|\n||= 10 hours|\n\n</div>\n\r\n<br>\n\r\nsm_nogap $\\therefore$ Normal pay rate\r\n\n<div class=\"aligned\">\n\n>>|||\n|-|-|\n||= $\\dfrac{289}{10}$|\n||= {{{prefix0}}}{{{correctAnswer0}}} per hour.|\n\n</div>"}]},{"vars":[{"varval":"John works as a doctor in the emergency department at the hospital. During one shift, he works for 7 hours at his normal pay rate, 4 hours at one and a half times his normal pay rate, and another 3 hours at twice his normal pay rate.\n\r\nJohn was paid $4674 for that shift.\n\r\nHow much is John's normal pay rate?"},{"varval":"sm_nogap Total normal hours\n\n<div class=\"aligned\">\n\n>>|||\n|-|-|\n||= 7 + (4 × 1.5) + (3 × 2)|\n||= 19 hours|\n\n</div>\n\r\n<br>\n\r\nsm_nogap $\\therefore$ Normal pay rate\r\n\n<div class=\"aligned\">\n\n>>|||\n|-|-|\n||= $\\dfrac{4674}{19}$|\n||= {{{prefix0}}}{{{correctAnswer0}}} per hour.|\n\n</div>"}]}]

  722. <div class="sm_mode"> {{{question}}} </div>

    [{"vars":[{"varval":"Mr. Soros put $2000 into a simple interest account for a year.\n\n\r\n\r\nHe did not take any money out or add any money to the account.\n\n\r\n\r\nAt the end of the year he had $2080 in the account.\n\n\r\n\r\nWhat was the annual percentage interest rate?"},{"varval":"<div class=\"aligned\">\n\n| | |\n| --------------------- | -------------- |\n| Interest earned | \\= 2080 − 2000 |\n| | \\= $80 |\n\n</div>\n\n<br>\n\nsm_nogap $\\therefore$ Annual interest rate\n\n<div class=\"aligned\">\n\n>>|||\n|-|-|\n||= $\\dfrac{80}{2000} \\ \\times$ 100|\n||= {{{correctAnswer0}}}{{{suffix0}}}|\n\n</div>"}]},{"vars":[{"varval":"Ms Reinhart put $3000 into a simple interest account for a year.\n\n\r\n\r\nShe did not take any money out or add any money to the account.\n\n\r\n\r\nAt the end of the year she had $3090 in the account.\n\n\r\n\r\nWhat was the annual percentage interest rate?"},{"varval":"<div class=\"aligned\">\n\n| | |\n| --------------------- | -------------- |\n| Interest earned | \\= 3090 − 3000 |\n| | \\= $90 |\n\n</div>\n\n<br>\n\nsm_nogap $\\therefore$ Annual interest rate\n\n<div class=\"aligned\">\n\n>>|||\n|-|-|\n||= $\\dfrac{90}{3000} \\ \\times$ 100|\n||= {{{correctAnswer0}}}{{{suffix0}}}|\n\n</div>"}]},{"vars":[{"varval":"Mr. Musk put $4000 into a simple interest account for a year.\n\n\r\n\r\nHe did not take any money out or add any money to the account.\n\n\r\n\r\nAt the end of the year he had $4100 in the account.\n\n\r\n\r\nWhat was the annual percentage interest rate?"},{"varval":"<div class=\"aligned\">\n\n| | |\n| --------------------- | -------------- |\n| Interest earned | \\= 4100 − 4000 |\n| | \\= $100 |\n\n</div>\n\n<br>\n\nsm_nogap $\\therefore$ Annual interest rate\n\n<div class=\"aligned\">\n\n>>|||\n|-|-|\n||= $\\dfrac{100}{4000} \\ \\times$ 100|\n||= {{{correctAnswer0}}}{{{suffix0}}}|\n\n</div>"}]},{"vars":[{"varval":"Penelope put $2500 into a simple interest account for a year.\n\n\r\n\r\nShe did not take any money out or add any money to the account.\n\n\r\n\r\nAt the end of the year she had $2600 in the account.\n\n\r\n\r\nWhat was the annual percentage interest rate?"},{"varval":"<div class=\"aligned\">\n\n| | |\n| --------------------- | -------------- |\n| Interest earned | \\= 2600 − 2500 |\n| | \\= $100 |\n\n</div>\n\n<br>\n\nsm_nogap $\\therefore$ Annual interest rate\n\n<div class=\"aligned\">\n\n>>|||\n|-|-|\n||= $\\dfrac{100}{2500} \\ \\times$ 100|\n||= {{{correctAnswer0}}}{{{suffix0}}}|\n\n</div>"}]},{"vars":[{"varval":"Mr. Packer put $8000 into a simple interest account for a year.\n\n\r\n\r\nHe did not take any money out or add any money to the account.\n\n\r\n\r\nAt the end of the year he had $8280 in the account.\n\n\r\n\r\nWhat was the annual percentage interest rate?"},{"varval":"<div class=\"aligned\">\n\n| | |\n| --------------------- | -------------- |\n| Interest earned | \\= 8280 − 8000 |\n| | \\= $280 |\n\n</div>\n\n<br>\n\nsm_nogap $\\therefore$ Annual interest rate\n\n<div class=\"aligned\">\n\n>>|||\n|-|-|\n||= $\\dfrac{280}{8000} \\ \\times$ 100|\n||= {{{correctAnswer0}}}{{{suffix0}}}|\n\n</div>"}]},{"vars":[{"varval":"Kate put $7500 into a simple interest account for a year.\n\n\r\n\r\nShe did not take any money out or add any money to the account.\n\n\r\n\r\nAt the end of the year she had $7743.75 in the account.\n\n\r\n\r\nWhat was the annual percentage interest rate?"},{"varval":"<div class=\"aligned\">\n\n| | |\n| --------------------- | -------------- |\n| Interest earned | \\= 7743.75 − 7500 |\n| | \\= $243.75 |\n\n</div>\n\n<br>\n\nsm_nogap $\\therefore$ Annual interest rate\n\n<div class=\"aligned\">\n\n>>|||\n|-|-|\n||= $\\dfrac{243.75}{7500} \\ \\times$ 100|\n||= {{{correctAnswer0}}}{{{suffix0}}}|\n\n</div>"}]}]

  723. <div class="sm_mode"> {{{question}}} </div>

    [{"vars":[{"varval":"Skye sells 18 mini-cakes of soap at the farmers market for a total of $9.90.\n\n\r\n\r\nDharma sells 30 cakes of soap at the same price per cake of soap as Skye.\n\n\r\n\r\nIn total, how much more does Dharma make than Skye?"},{"varval":"<div class=\"aligned\">\n\n| | |\n| --------------------- | -------------- |\n| Cost of 1 soap | \\= 9.90 ÷ 18 |\n| | \\= $0.55 |\n\n</div>\n\n<div class=\"aligned\">\n\n| | |\n| --------------------- | -------------- |\n| $\\therefore$ Extra sales | \\= (30 $-$ 18) $\\times$ 0.55|\n| | \\= {{{prefix0}}}{{{correctAnswer0}}} |\n\n</div>"}]},{"vars":[{"varval":"Levi sells 8 cob lettuces at the farmers market for a total of $89.20.\n\nHannah sells 15 iceberg lettuces at the same price per lettuce as Levi.\n\r\nIn total, how much more does Hannah make than Levi?"},{"varval":"<div class=\"aligned\">\n\n| | |\n| --------------------- | -------------- |\n| Cost of 1 lettuce | \\= 89.20 ÷ 8 |\n| | \\= $11.15 |\n\n</div>\n\n<div class=\"aligned\">\n\n| | |\n| --------------------- | -------------- |\n| $\\therefore$ Extra sales | \\= (15 $-$ 8) $\\times$ 11.15 |\n| | \\= {{{prefix0}}}{{{correctAnswer0}}} |\n\n</div>"}]},{"vars":[{"varval":"Vanilla sells 16 birthday cards at the farmers market for a total of $74.40\n\n\rRamsay sells 7 birthday cards at the same price per card as Vanilla.\n\r\nIn total, how much more does Vanilla make than Ramsay?"},{"varval":"<div class=\"aligned\">\n\n| | |\n| --------------------- | -------------- |\n| Cost of 1 card | \\= 74.40 ÷ 16 |\n| | \\= $4.65 |\n\n</div>\n\n<div class=\"aligned\">\n\n| | |\n| --------------------- | -------------- |\n| $\\therefore$ Extra sales | \\= (16 $-$ 7) $\\times$ 4.65 |\n| | \\= {{{prefix0}}}{{{correctAnswer0}}} |\n\n</div>"}]},{"vars":[{"varval":"Zac sells 23 glasses of lemonade from a stall in his front yard for a total of $50.60\n\nZac's sister Maya sells 17 glasses of barley water at the same cost per glass as Zac.\n\r\nIn total, how much more does Zac make than Maya?"},{"varval":"<div class=\"aligned\">\n\n| | |\n| --------------------- | -------------- |\n| Cost of 1 glass | \\= 50.60 ÷ 23 |\n| | \\= $2.20 |\n\n</div>\n\n<div class=\"aligned\">\n\n| | |\n| --------------------- | -------------- |\n| $\\therefore$ Extra sales | \\= (23 $-$ 17) $\\times$ 2.20 |\n| | \\= {{{prefix0}}}{{{correctAnswer0}}} |\n\n</div>"}]},{"vars":[{"varval":"Dave returns from a fishing trip and sells 32 brim to the fishing coop for a total of $187.20\n\nRichard sells 21 salmon to the fishing coop for the same price per fish as Dave.\n\r\nIn total, how much more does Dave make than Richard?"},{"varval":"<div class=\"aligned\">\n\n| | |\n| --------------------- | -------------- |\n| Cost of 1 fish | \\= 187.20 ÷ 32 |\n| | \\= $5.85 |\n\n</div>\n\n<div class=\"aligned\">\n\n| | |\n| --------------------- | -------------- |\n| $\\therefore$ Extra sales | \\= (32 $-$ 21) $\\times$ 5.85 |\n| | \\= {{{prefix0}}}{{{correctAnswer0}}} |\n\n</div>"}]},{"vars":[{"varval":"Sandy sells 12 kilograms of prawns at the fish market for a total of $270.\n\nChristy sells 7 kilograms for the same price per kilogram as Sandy.\n\r\nIn total, how much more does Sandy make than Christy?"},{"varval":"<div class=\"aligned\">\n\n| | |\n| --------------------- | -------------- |\n| Cost of 1 kilogram | \\= 270 ÷ 12 |\n| | \\= $22.50 |\n\n</div>\n\n<div class=\"aligned\">\n\n| | |\n| --------------------- | -------------- |\n| $\\therefore$ Extra sales | \\= (12 $-$ 7) $\\times$ 22.50 |\n| | \\= {{{prefix0}}}{{{correctAnswer0}}} |\n\n</div>"}]}]

  724. <div class="sm_mode"> {{{question}}} </div>

    [{"vars":[{"varval":"Huckleberry opens a savings account and deposits $1200 into it.\n\nHe makes no more deposits and earns simple interest on his original deposit at 2.5% each year.\n\nHow much interest will Huckleberry earn after 3 years?"},{"varval":"<div class=\"aligned\">\n\n| | |\n| --------------------- | -------------- |\n| Interest earned | \\= $P \\large rn$ |\n| | \\= 1200 $\\times \\dfrac{2.5}{100} \\times$ 3 |\n|| \\= {{{prefix0}}}{{{correctAnswer0}}}|\n\n</div>"}]},{"vars":[{"varval":"Harvey opens a savings account and deposits $1500 into it.\n\nHe makes no more deposits and earns simple interest on his original deposit at 1.5% each year.\n\nHow much interest will Harvey earn after 2 years?"},{"varval":"<div class=\"aligned\">\n\n| | |\n| --------------------- | -------------- |\n| Interest earned | \\= $P \\large rn$ |\n| | \\= 1500 $\\times \\dfrac{1.5}{100} \\times$ 2 |\n|| \\= {{{prefix0}}}{{{correctAnswer0}}}|\n\n</div>"}]},{"vars":[{"varval":"Elizabeth opens a savings account and deposits $12 000 into it.\n\nShe makes no more deposits and earns simple interest on her original deposit at 3.5% each year.\n\nHow much interest will Elizabeth earn after 3 years?"},{"varval":"<div class=\"aligned\">\n\n| | |\n| --------------------- | -------------- |\n| Interest earned | \\= $P \\large rn$ |\n| | \\= 12 000 $\\times \\dfrac{3.5}{100} \\times$ 3 |\n|| \\= {{{prefix0}}}{{{correctAnswer0}}}|\n\n</div>"}]},{"vars":[{"varval":"Sylvia opens a savings account and deposits $8200 into it.\n\nShe makes no more deposits and earns simple interest on her original deposit at 2.25% each year.\n\nHow much interest will Sylvia earn after 2 years?"},{"varval":"<div class=\"aligned\">\n\n| | |\n| --------------------- | -------------- |\n| Interest earned | \\= $P \\large rn$ |\n| | \\= 8200 $\\times \\dfrac{2.25}{100} \\times$ 2 |\n|| \\= {{{prefix0}}}{{{correctAnswer0}}}|\n\n</div>"}]}]

  725. <div class="sm_mode"> {{{question}}} </div>

    [{"vars":[{"varval":"Laurel and Hardy own a shed business.\n\n\r\n\r\nThey charge $4150 to build a shed.\n\n\r\n\r\nThey pay 5 workers $44 per hour for 7 hours each to construct the shed.\n\n\r\n\r\nThe materials used for the shed cost $985.\n\n\r\n\r\nThe remaining money is divided equally between Laurel and Hardy.\n\n\r\n\r\nHow much will they each receive?"},{"varval":"Shed sale price = $4150\n\nLabour cost = 5 × 44 × 7 = $1540\n\nMaterials cost = $985\n\n<div class=\"aligned\">\n\n| | |\n| --------------------- | -------------- |\n| Remaining money | \\= 4150 − (1540 + 985) |\n| | \\= $1625 |\n\n</div>\n\r\r\n \r<br>\n\r\n$\\therefore$ Each receives {{{prefix0}}}{{{correctAnswer0}}}"}]},{"vars":[{"varval":"Kochie and Mel own a garage business.\n\n\r\n\r\nThey charge $9500 to build a garage.\n\n\r\n\r\nThey pay 8 workers $48 per hour for 12 hours each to construct the garage.\n\n\r\n\r\nThe materials used for the garage cost $1180.\n\n\r\n\r\nThe remaining money is divided equally between Kochie and Mel.\n\n\r\n\r\nHow much will they each receive?"},{"varval":"Garage sale price = $9500\n\nLabour cost = 8 × 48 × 12 = $4608\n\nMaterials cost = $1180\n\n<div class=\"aligned\">\n\n| | |\n| --------------------- | -------------- |\n| Remaining money | \\= 9500 − (4608 + 1180) |\n| | \\= $3712 |\n\n</div>\n\r\r\n \r<br>\n\r\n$\\therefore$ Each receives {{{prefix0}}}{{{correctAnswer0}}}"}]},{"vars":[{"varval":"Nicole and Keith build childrens' playgrounds.\n\n\r\n\r\nThey charge $7250 to build a complete playground.\n\n\r\n\r\nThey pay 4 workers $54 per hour for 7 hours each to construct the playground.\n\n\r\n\r\nThe materials used for the playground cost $2040.\n\n\r\n\r\nThe remaining money is divided equally between Nicole and Keith.\n\n\r\n\r\nHow much will they each receive?"},{"varval":"Playground sale price = $7250\n\nLabour cost = 4 × 54 × 7 = $1512\n\nMaterials cost = $2040\n\n<div class=\"aligned\">\n\n| | |\n| --------------------- | -------------- |\n| Remaining money | \\= 7250 − (1512 + 2040) |\n| | \\= $3698 |\n\n</div>\n\r\r\n \r<br>\n\r\n$\\therefore$ Each receives {{{prefix0}}}{{{correctAnswer0}}}"}]},{"vars":[{"varval":"Brad and Angelina own a kitchen business.\n\nThey charge $9750 to build a kitchen.\n\nThey pay 4 workers $62 per hour for 11 hours each to construct a kitchen.\n\n\r\n\r\nThe materials used for the kitchen cost $3160.\n\n\r\n\r\nThe remaining money is divided equally between Brad and Angelina.\n\n\r\n\r\nHow much will they each receive?"},{"varval":"Kitchen sale price = $9750\n\nLabour cost = 4 × 62 × 11 = $2728\n\nMaterials cost = $3160\n\n<div class=\"aligned\">\n\n| | |\n| --------------------- | -------------- |\n| Remaining money | \\= 9750 − (2728 + 3160) |\n| | \\= $3862 |\n\n</div>\n\r\r\n \r<br>\n\r\n$\\therefore$ Each receives {{{prefix0}}}{{{correctAnswer0}}}"}]},{"vars":[{"varval":"French and Saunders own a tree house business.\n\nThey charge $4750 to build a tree house.\n\n\r\n\r\nThey pay 3 workers $36 per hour for 6 hours each to construct the tree house.\n\n\r\n\r\nThe materials used for the tree house cost $1780.\n\n\r\n\r\nThe remaining money is divided equally between French and Saunders.\n\n\r\n\r\nHow much will they each receive?"},{"varval":"Tree house sale price = $4750\n\nLabour cost = 3 × 36 × 6 = $648\n\nMaterials cost = $1780\n\n<div class=\"aligned\">\n\n| | |\n| --------------------- | -------------- |\n| Remaining money | \\= 4750 − (648 + 1780) |\n| | \\= $2322 |\n\n</div>\n\r\r\n \r<br>\n\r\n$\\therefore$ Each receives {{{prefix0}}}{{{correctAnswer0}}}"}]},{"vars":[{"varval":"Kate and Jot own a business that restores vintage cars.\n\r\nThey charge $6800 to restore one particular car.\n\nThey pay 3 workers $68 per hour for 9 hours each to restore the car.\n\nThe materials used for the restoration cost $2980.\n\nThe remaining money is divided equally between Kate and Jot.\n\n\r\n\r\nHow much will they each receive?"},{"varval":"Restoration price = $6800\n\nLabour cost = 3 × 68 × 9 = $1836\n\nMaterials cost = $2980\n\n<div class=\"aligned\">\n\n| | |\n| --------------------- | -------------- |\n| Remaining money | \\= 6800 − (1836 + 2980) |\n| | \\= $1984 |\n\n</div>\n\r\r\n \r<br>\n\r\n$\\therefore$ Each receives {{{prefix0}}}{{{correctAnswer0}}}"}]}]

  726. Weights of borders in tables inconsistent; some are randomly bold. Var4: Corrected answer from $78.00 to $78.10.

    <div class="sm_mode"> {{{question}}} </div>

    [{"vars":[{"varval":"A farmers' market sells potatoes in four different sizes.\n\n<div class=\"sm-table col1-color3\">\n\n>>| Size| 1 kg | 2 kg| 3 kg | 5 kg|\n|:-:|:-:|:-:|:-:|:-:|\n| Price| $3.40|$6.10|$9.35|$15.75|\n\n</div>\n\n<br>What is the lowest price a customer can pay for 8 kg of potatoes?"},{"varval":"sm_nogap Calculate the cost per kg for each size:\n\n\r\n> 1 kg: &nbsp;$3.40 per kg\n\n\r\n> 2 kg: &nbsp;6.10 ÷ 2 = $3.05 per kg\n\n\r\n> 3 kg: &nbsp;9.35 ÷ 3 ≈ $3.12 per kg\n\n\r\n> 5 kg: &nbsp;15.75 ÷ 5 = $3.15 per kg\r\n\n<br>\n\r\nsm_nogap $\\therefore$ Cheapest price to buy 8 kg\n\n\r\n>> = 4 × 6.10\n\n\r\n>> = {{{prefix0}}}{{{correctAnswer0}}}"}]},{"vars":[{"varval":"A fish market sells prawns in four different sizes.\n\n<div class=\"sm-table col1-color2\">\n\n>>| Size| 0.5 kg | 1 kg| 2 kg | 4 kg|\n|:-:|:-:|:-:|:-:|:-:|\n| Price| $8.25|$15.75|$34.00|$65.50|\n\n</div>\n\n<br>What is the lowest price a customer can pay for 6 kg of prawns?"},{"varval":"sm_nogap Calculate the cost per kg for each size:\n\n\r\n> 0.5 kg: &nbsp;8.25 $\\times$ 2 = $16.50 per kg\n\n\r\n> 1 kg: &nbsp;$15.75 per kg\n\n\r\n> 2 kg: &nbsp;34.00 ÷ 2 = $17.00 per kg\n\n\r\n> 4 kg: &nbsp;65.50 ÷ 4 ≈ $16.38 per kg\r\n\n<br>\n\r\nsm_nogap $\\therefore$ Cheapest price to buy 6 kg\n\n\r\n>> = 6 × 15.75\n\n\r\n>> = {{{prefix0}}}{{{correctAnswer0}}}"}]},{"vars":[{"varval":"A farmers' market sells olive oil in four different sizes.\n\n<div class=\"sm-table col1-color3\">\n\n>>| Size| 0.5 litre | 1 litre| 1.5 litres | 3 litres|\n|:-:|:-:|:-:|:-:|:-:|\n| Price| $3.75|$7.90|$11.70|$24.00|\n\n</div>\n\n<br>What is the lowest price a customer can pay for 6 litres of olive oil?"},{"varval":"sm_nogap Calculate the cost per litre for each size:\n\n\r\n> 0.5L: &nbsp; 3.75 $\\times$ 2 = $7.50 per litre\n\n\r\n> 1L: &nbsp; $7.90 per litre\n\n\r\n> 1.5L: &nbsp;11.70 ÷ 1.5 = $7.80 per litre\n\n\r\n> 3L: &nbsp;24.00 ÷ 3 = $8.00 per litre\n\n<br>\n\r\nsm_nogap $\\therefore$ Cheapest price to buy 6 litres\n\n\r\n>> = 12 × 3.75\n\n\r\n>> = {{{prefix0}}}{{{correctAnswer0}}}"}]},{"vars":[{"varval":"An avocado farmer sells her avocados in four different sizes.\n\n<div class=\"sm-table col1-color2\">\n\n>>| Size| 1 kg | 2 kg| 3 kg | 4 kg|\n|:-:|:-:|:-:|:-:|:-:|\n| Price| $6.35|$13.00|$19.00|$25.10|\n\n</div>\n\n<br>What is the lowest price a customer can pay for 8 kg of avocados?"},{"varval":"sm_nogap Calculate the cost per kg for each size:\n\n\r\n> 1 kg: &nbsp;6.35 per kg\n\n\r\n> 2 kg: &nbsp;13.00 ÷ 2 = $6.50 per kg\n\n\r\n> 3 kg: &nbsp;19.00 ÷ 3 = $6.33 per kg\n\n\r\n> 4 kg: &nbsp;25.10 ÷ 4 ≈ $6.28 per kg\r\n\n<br>\n\r\nsm_nogap $\\therefore$ Cheapest price to buy 8 kg\n\n\r\n>> = 2 × 25.10\n\n\r\n>> = {{{prefix0}}}{{{correctAnswer0}}}"}]},{"vars":[{"varval":"An oyster farm sells bags of oysters in four different sizes.\n\n<div class=\"sm-table col1-color3\">\n\n>>| Bag Size| 1 kg | 2 kg| 3 kg | 5 kg|\n|:-:|:-:|:-:|:-:|:-:|\n| Price| $12.00|$23.10|$37.00|$55.00|\n\n</div>\n\n<br>What is the lowest price a customer can pay for 7 kg of oysters?"},{"varval":"sm_nogap Calculate the cost per kg for each size:\n\n\r\n> 1 kg: &nbsp;$12.00 per kg\n\n\r\n> 2 kg: &nbsp;23.10 ÷ 2 = $11.55 per kg\n\n\r\n> 3 kg: &nbsp;37.00 ÷ 3 ≈ $12.33 per kg\n\n\r\n> 5 kg: &nbsp;55 ÷ 5 = $11.00 per kg\r\n\n<br>\n\r\nsm_nogap $\\therefore$ Cheapest price to buy 7 kg\n\n\r\n>> = 1 × 55.00 + 1 × 23.10\n\n\r\n>> = {{{prefix0}}}{{{correctAnswer0}}}"}]},{"vars":[{"varval":"A vegetable farmer sells cucumbers in four different sizes.\n\n<div class=\"sm-table col1-color3\">\n\n>>| Size| 1 kg | 2 kg| 3 kg | 5 kg|\n|:-:|:-:|:-:|:-:|:-:|\n| Price| $5.20|$10.90|$16.80|$25.50|\n\n</div>\n\n<br>What is the lowest price a customer can pay for 7 kilograms of cucumbers?"},{"varval":"sm_nogap Calculate the cost per kg for each size:\n\n\r\n> 1 kg: &nbsp;$5.20 per kg\n\n\r\n> 2 kg: &nbsp;10.90 ÷ 2 = $5.45 per kg\n\n\r\n> 3 kg: &nbsp;16.80 ÷ 3 = $5.60 per kg\n\n\r\n> 5 kg: &nbsp;25.50 ÷ 5 = $5.10 per kg\r\n\n<br>\n\r\nsm_nogap $\\therefore$ Cheapest price to buy 7 kg\n\n\r\n>> = 1 × 25.50 + 2 × 5.20\n\n\r\n>> = {{{prefix0}}}{{{correctAnswer0}}}"}]}]

  727. <div class="sm_mode"> Raphael sells 60 cakes at the market. Each cake costs him $4.50 to make. He sells 50 cakes for $6.00 each. In the last hour of the market, he sells the last 10 cakes for $1.50 each. What is the total profit or loss of the 60 cakes? </div>

    [{"vars":null}]

  728. <div class="sm_mode"> {{{question}}} </div>

    [{"vars":[{"varval":"$9 + (13\\ −\\ 4\\ \\times 2 + 3)$ = <span class=\"sm-text color8\">?</span>\n\n<br>\n\nWhat number does <span class=\"sm-text color8\">?</span> represent?"},{"varval":"<div class=\"aligned\">\n\n| | |\n| --------------------: | -------------- |\n| 9 + (13 $-\\ 4\\ \\times$ 2 + 3) | \\= <span class=\"sm-text color8\">?</span> |\n| | |\n| 9 + (13 $-$ 8 + 3) | \\= <span class=\"sm-text color8\">?</span> |\n| | |\n|9 + 8| \\= <span class=\"sm-text color8\">?</span> |\n| | |\n|17| \\= <span class=\"sm-text color8\">?</span> |\n\n</div>"}]},{"vars":[{"varval":"7 + (13 − 4 × 3 + 1) = <span class=\"sm-text color8\">?</span>\n\n<br>\n\nWhat number does <span class=\"sm-text color8\">?</span> represent?"},{"varval":"<div class=\"aligned\">\n\n| | |\n| --------------------: | -------------- |\n| 7 + (13 $-$ 4 $\\times$ 3 + 1) | \\= <span class=\"sm-text color8\">?</span> |\n| | |\n| 7 + (13 $-$ 12 + 1) | \\= <span class=\"sm-text color8\">?</span> |\n| | |\n|7 + 2| \\= <span class=\"sm-text color8\">?</span> |\n| | |\n|9| \\= <span class=\"sm-text color8\">?</span> |\n\n</div>"}]}]

  729. <div class="sm_mode"> {{{question}}} </div>

    [{"vars":[{"varval":"$90 - \\dfrac{\\sqrt{18^2 + 80^2}}{4}$ = <span class=\"sm-text color8\">?</span>\n\nCalculate the exact value of &nbsp;<span class=\"sm-text color8\">?</span>."},{"varval":"$18^2 + 80^2 = 6724$\n\n$\\sqrt{6742} = 82$ (by trial and error)\n\n<div class=\"aligned\">\n\n|||\n|-:|-|\n|$\\therefore\\ 90 - \\dfrac{\\sqrt{18^2 + 80^2}}{4}$|= 90 $- \\dfrac{82}{4}$|\n||= {{{correctAnswer0}}}|\n\n</div>"}]},{"vars":[{"varval":"$73 - \\dfrac{\\sqrt{16^2 + 30^2}}{4}$ = <span class=\"sm-text color8\">?</span>\n\nCalculate the exact value of &nbsp;<span class=\"sm-text color8\">?</span>."},{"varval":"$16^2 + 30^2 = 1156$\n\n$\\sqrt{1156} = 34$ (by trial and error)\n\n<div class=\"aligned\">\n\n|||\n|-:|-|\n|$73 - \\dfrac{\\sqrt{16^2 + 30^2}}{4}$|= 73 $- \\dfrac{34}{4}$|\n||= {{{correctAnswer0}}}|\n\n</div>"}]},{"vars":[{"varval":"$110 - \\dfrac{\\sqrt{8^2 + 15^2}}{2}$ = <span class=\"sm-text color8\">?</span>\n\nCalculate the exact value of &nbsp;<span class=\"sm-text color8\">?</span>."},{"varval":"$8^2 + 15^2 = 289$\n\n$\\sqrt{289} = 17$ (by trial and error)\n\n<div class=\"aligned\">\n\n|||\n|-:|-|\n|$\\therefore\\ 110 - \\dfrac{\\sqrt{8^2 + 15^2}}{2}$|= 110 $- \\dfrac{17}{2}$|\n||= {{{correctAnswer0}}}|\n\n</div>"}]},{"vars":[{"varval":"$108 - \\dfrac{\\sqrt{16^2 + 12^2}}{8}$ = <span class=\"sm-text color8\">?</span>\n\nCalculate the exact value of &nbsp;<span class=\"sm-text color8\">?</span>."},{"varval":"$16^2 + 12^2 = 400$\n\n$\\sqrt{400} = 20$ (by trial and error)\n\n<div class=\"aligned\">\n\n|||\n|-:|-|\n|$108 - \\dfrac{\\sqrt{16^2 + 12^2}}{8}$|= 108 $- \\dfrac{20}{8}$|\n||= {{{correctAnswer0}}}|\n\n</div>"}]},{"vars":[{"varval":"$47 - \\dfrac{\\sqrt{10^2 + 24^2}}{4}$ = <span class=\"sm-text color8\">?</span>\n\nCalculate the exact value of &nbsp;<span class=\"sm-text color8\">?</span>."},{"varval":"$10^2 + 24^2 = 676$\n\n$\\sqrt{676} = 26$ (by trial and error)\n\n<div class=\"aligned\">\n\n|||\n|-:|-|\n|$47 - \\dfrac{\\sqrt{10^2 + 24^2}}{4}$|= 47 $- \\dfrac{26}{4}$|\n||= {{{correctAnswer0}}}|\n\n</div>"}]},{"vars":[{"varval":"$83 - \\dfrac{\\sqrt{21^2 + 28^2}}{4}$ = <span class=\"sm-text color8\">?</span>\n\nCalculate the exact value of &nbsp;<span class=\"sm-text color8\">?</span>."},{"varval":"$21^2 + 28^2 = 1225$\n\n$\\sqrt{1225} = 35$ (by trial and error)\n\n<div class=\"aligned\">\n\n|||\n|-:|-|\n|$83 - \\dfrac{\\sqrt{21^2 + 28^2}}{4}$|= 83 $- \\dfrac{35}{4}$|\n||= {{{correctAnswer0}}}|\n\n</div>"}]}]

  730. <div class="sm_mode"> {{{question}}} </div>

    [{"vars":[{"varval":"The weight of a gold bar is 1.0247 kg.\n\n<br>\n\nsm_img https://teacher.smartermaths.com.au/wp-content/uploads/2018/04/NAPX-J4-CA18-300x206.png 220 indent vpad\n\n<br>\n\nWhat is the weight of the gold bar rounded to the nearest hundredth of a kilogram?\n"},{"varval":"Digits to the right of the decimal place:\n\n$\\Rightarrow$ tenths, hundredths, thousandths\n\nNearest hundredth = {{{correctAnswer}}}\n"}]},{"vars":[{"varval":"The weight of a gold bar is 1.0247 kg.\n\n<br>\n\nsm_img https://teacher.smartermaths.com.au/wp-content/uploads/2018/04/NAPX-J4-CA18-300x206.png 220 indent vpad\n\n<br>\n\nWhat is the weight of the gold bar rounded to the nearest thousandth of a kilogram?\n"},{"varval":"Digits to the right of the decimal place:\n\n$\\Rightarrow$ tenths, hundredths, thousandths\n\nNearest thousandth = {{{correctAnswer}}}"}]},{"vars":[{"varval":"The weight of a gold bar is 1.0247 kg.\n\n<br>\n\nsm_img https://teacher.smartermaths.com.au/wp-content/uploads/2018/04/NAPX-J4-CA18-300x206.png 220 indent vpad\n\n<br>\n\nWhat is the weight of the gold bar rounded to the nearest tenth of a kilogram?\n"},{"varval":"Digits to the right of the decimal place:\n\n$\\Rightarrow$ tenths, hundredths, thousandths\n\nNearest tenth = {{{correctAnswer}}}"}]},{"vars":[{"varval":"The weight of a gold bar is 1.0247 kg.\n\nGina has ten of these gold bars.\n\n<br>\n\nsm_img https://teacher.smartermaths.com.au/wp-content/uploads/2018/04/NAPX-J4-CA18-300x206.png 220 indent vpad\n\n<br>\n\nWhat is the weight of the ten gold bars rounded to the nearest hundredth of a kilogram?\n"},{"varval":"Weight of 10 gold bars = 1.0247 × 10 = 10.247 kg\n\nDigits to the right of the decimal place:\n\n$\\Rightarrow$ tenths, hundredths, thousandths\n\n$\\therefore$ Nearest hundredth = {{{correctAnswer}}}"}]}]

  731. <div class="sm_mode"> Which arrows show the approximate locations of $\sqrt{5}$ and $\large \pi$ on the number line? sm_img https://teacher.smartermaths.com.au/wp-content/uploads/2018/07/NAPX-F4-CA09_1.svg 400 indent vpad </div>

    [{"vars":null}]

  732. <div class="sm_mode"> {{{question}}} </div>

    [{"vars":[{"varval":"One billion is one thousand million.\n\n\r\n\r\nWhich of the following is 300 billion?\n"},{"varval":"sm_nogap Using the description:\n\n<div class=\"aligned\">\n\n| | |\n| --------------------- | -------------- |\n| 300 billion | \\= 300 × 1000 × 1 000 000 |\n|| \\= 300 000 000 000|\n| | \\= {{{correctAnswer}}} |\n\n</div>"}]},{"vars":[{"varval":"One billion is one thousand million.\n\n\r\n\r\nWhich of the following is 24 billion?\n"},{"varval":"sm_nogap Using the description:\n\n<div class=\"aligned\">\n\n| | |\n| --------------------- | -------------- |\n| 24 billion | \\= 24 × 1000 × 1 000 000 |\n|| \\= 24 000 000 000|\n| | \\= {{{correctAnswer}}} |\n\n</div>"}]},{"vars":[{"varval":"One billion is one thousand million.\n\n\r\n\r\nWhich of the following is 900 billion?\n"},{"varval":"sm_nogap Using the description:\n\n<div class=\"aligned\">\n\n| | |\n| --------------------- | -------------- |\n| 900 billion | \\= 900 × 1000 × 1 000 000 |\n|| \\= 900 000 000 000|\n| | \\= {{{correctAnswer}}} |\n\n</div>"}]},{"vars":[{"varval":"One billion is one thousand million.\n\n\r\n\r\nWhich of the following is 50 billion?\n"},{"varval":"sm_nogap Using the description:\n\n<div class=\"aligned\">\n\n| | |\n| --------------------- | -------------- |\n| 50 billion | \\= 50 × 1000 × 1 000 000 |\n|| \\= 50 000 000 000|\n| | \\= {{{correctAnswer}}} |\n\n</div>"}]},{"vars":[{"varval":"One billion is one thousand million.\n\n\r\n\r\nWhich of the following is 565 billion?\n"},{"varval":"sm_nogap Using the description:\n\n<div class=\"aligned\">\n\n| | |\n| --------------------- | -------------- |\n| 565 billion | \\= 565 × 1000 × 1 000 000 |\n|| \\= 565 000 000 000|\n| | \\= {{{correctAnswer}}} |\n\n</div>"}]},{"vars":[{"varval":"One billion is one thousand million.\n\n\r\n\r\nWhich of the following is 4.5 billion?\n"},{"varval":"sm_nogap Using the description:\n\n<div class=\"aligned\">\n\n| | |\n| --------------------- | -------------- |\n| 4.5 billion | \\= 4.5 × 1000 × 1 000 000 |\n|| \\= 4 500 000 000|\n| | \\= {{{correctAnswer}}} |\n\n</div>"}]}]

  733. <div class="sm_mode"> {{{question}}} </div>

    [{"vars":[{"varval":"What is the value of ` ` $\\dfrac{80.3 + 16.28}{7.1 \\times 1.8}$\n"},{"varval":"<div class=\"aligned\">\n\n| | |\n| --------------------- | -------------- |\n|$\\dfrac{80.3 + 16.28}{7.1 \\times 1.8}$| = $\\dfrac{96.58}{12.78}$ |\n| | \\= 7.557... |\n| | \\= {{{correctAnswer}}} |\n\n</div>"}]},{"vars":[{"varval":"What is the value of ` ` $\\dfrac{19.2 + 49.56}{2.4 \\times 3.6}$\n"},{"varval":"<div class=\"aligned\">\n\n| | |\n| --------------------- | -------------- |\n||$\\dfrac{19.2 + 49.56}{2.4 \\times 3.6}$ |\n| | \\= 7.958... |\n| | \\= {{{correctAnswer}}} |\n\n</div>"}]},{"vars":[{"varval":"What is the value of ` ` $\\dfrac{21.2 + 45.67}{9.2 \\times 4.8}$"},{"varval":"<div class=\"aligned\">\n\n| | |\n| --------------------- | -------------- |\n|$\\dfrac{21.2 + 45.67}{9.2 \\times 4.9}$| = $\\dfrac{66.87}{45.08}$ |\n| | \\= 1.514... |\n| | \\= {{{correctAnswer}}} |\n\n</div>"}]},{"vars":[{"varval":"What is the value of ` ` $\\dfrac{112.3 \\ − \\ 45.6}{2.6 \\times 3.1}$\n"},{"varval":"<div class=\"aligned\">\n\n| | |\n| --------------------- | -------------- |\n|$\\dfrac{112.3 \\ − \\ 45.6}{2.6 \\times 3.1}$| = $\\dfrac{66.7}{8.06}$ |\n| | \\= 8.275... |\n| | \\= {{{correctAnswer}}} |\n\n</div>"}]},{"vars":[{"varval":"What is the value of ` ` $\\dfrac{21.7 \\ − \\ 8.2}{0.3 \\times 12.8}$\n"},{"varval":"<div class=\"aligned\">\n\n| | |\n| --------------------- | -------------- |\n|$\\dfrac{21.7 \\ − \\ 8.2}{0.3 \\times 12.8}$| = $\\dfrac{13.5}{3.84}$ |\n| | \\= 3.515... |\n| | \\= {{{correctAnswer}}} |\n\n</div>"}]},{"vars":[{"varval":"What is the value of ` ` $\\dfrac{97.4 \\ − \\ 28.3}{0.8 \\times 4.9}$\n"},{"varval":"<div class=\"aligned\">\n\n| | |\n| --------------------- | -------------- |\n|$\\dfrac{97.4 \\ − \\ 28.3}{0.8 \\times 4.9}$| = $\\dfrac{69.1}{3.92}$ |\n| | \\= 17.627... |\n| | \\= {{{correctAnswer}}} |\n\n</div>"}]}]

  734. <div class="sm_mode"> {{{question}}} </div>

    [{"vars":[{"varval":"$8^3$ is equal to which of the following?\n"},{"varval":"<div class=\"aligned\">\n\n| | |\n| --------------------- | -------------- |\n| $8^3$ | \\= 8 × 8 × 8 |\n| | \\= {{{correctAnswer}}} |\n\n</div>"}]},{"vars":[{"varval":"$15^3$ is equal to which of the following?"},{"varval":"<div class=\"aligned\">\n\n| | |\n| --------------------- | -------------- |\n| $15^3$ | \\= 15 × 15 × 15 |\n| | \\= {{{correctAnswer}}} |\n\n</div>"}]},{"vars":[{"varval":"$21^4$ is equal to which of the following?"},{"varval":"<div class=\"aligned\">\n\n| | |\n| --------------------- | -------------- |\n| $21^4$ | \\= 21 × 21 × 21 × 21|\n| | \\= {{{correctAnswer}}} |\n\n</div>"}]},{"vars":[{"varval":"Which of the following is equal to $12^2$?"},{"varval":"<div class=\"aligned\">\n\n| | |\n| --------------------- | -------------- |\n| $12^2$ | \\= 12 × 12|\n||= 4 × 3 × 4 × 3|\n||= {{{correctAnswer}}}\n\n</div>\n"}]},{"vars":[{"varval":"Which of the following is NOT equal to $4^3$?"},{"varval":"<div class=\"aligned\">\n\n| | |\n| --------------------- | -------------- |\n| $4^3$ | \\= 4 × 4 × 4|\n| | \\= 64 |\n\n</div>\n<div class=\"sm_mode\">\n\n\n\nsm_nogap Check each option:\n\n<div class=\"aligned\">\n\n>| |\n| -------------------- |\n| $4^2$ × $4^1$ = 64 &nbsp$\\checkmark$ |\n| 2 × 2 × 2 × 2 × 2 × 2 = 64 &nbsp$\\checkmark$|\n| 8 × 3 × 2= 48 &nbsp;x | \n| $4^4 \\div (2 × 2)$ = 64 &nbsp$\\checkmark$ |\n\n\n</div>\n\n<br>\n\n$\\therefore$ {{correctAnswer}} is NOT equal to $4^3$.\n\n</div>\n\n"}]},{"vars":[{"varval":"Which of the following is NOT equal to $2^8$?"},{"varval":"<div class=\"aligned\">\n\n| | |\n| --------------------- | -------------- |\n| $2^8$ | \\= 2 × 2 × 2 × 2 × 2 × 2 × 2 × 2|\n| | \\= 256 |\n\n</div>\n<div class=\"sm_mode\">\n\n\n\nsm_nogap Check each option:\n\n<div class=\"aligned\">\n\n>| |\n| -------------------- |\n| $16^2$ = 256 &nbsp$\\checkmark$ |\n| 2 × 2 × 4 × 4 × 4 × 4 = 1024 &nbsp;x|\n| $2^2$ × $2^2$ × $2^2$ × $2^2$= 256 &nbsp$\\checkmark$ | \n| $2^10 \\div (2 × 2)$ = 64 &nbsp$\\checkmark$ |\n\n\n</div>\n\n<br>\n\n$\\therefore$ {{correctAnswer}} is NOT equal to $2^8$.\n\n</div>\n"}]}]

  735. <div class="sm_mode"> {{{question}}} </div>

    [{"vars":[{"varval":"What is the value of the expression\n\n\r\n\r\n6 + (−12) − (−5)"},{"varval":"<div class=\"aligned\">\n\n| | |\n| --------------------- | -------------- |\n| 6 + (−12) − (−5)\t | \\= 6 − 12 + 5 |\n| | \\= {{{correctAnswer}}} |\n\n</div>"}]},{"vars":[{"varval":"What is the value of the expression\n\n\r\n\r\n−12 + (−3) − (−5)"},{"varval":"<div class=\"aligned\">\n\n| | |\n| --------------------- | -------------- | \n| −12 + (−3) − (−5)\t | \\= −12 − 3 + 5 |\n| | \\= {{{correctAnswer}}} |\n\n</div>"}]},{"vars":[{"varval":"What is the value of the expression\n\n\r\n\r\n4 − (−1) − (−7)"},{"varval":"<div class=\"aligned\">\n\n| | |\n| --------------------- | -------------- | \n| 4 − (−1) − (−7)\t | \\= 4 + 1 + 7 |\n| | \\= {{{correctAnswer}}} |\n\n</div>"}]},{"vars":[{"varval":"What is the value of the expression\n\n\r\n\r\n8 + (−2) − (−3)"},{"varval":"<div class=\"aligned\">\n\n| | |\n| --------------------- | -------------- |\n| 8 + (−2) − (−3)\t | \\= 8 − 2 + 3 |\n| | \\= {{{correctAnswer}}} |\n\n</div>"}]},{"vars":[{"varval":"What is the value of the expression\n\n\r\n\r\n−5 + (−10) − (−2)"},{"varval":"<div class=\"aligned\">\n\n| | |\n| --------------------- | -------------- |\n| −5 + (−10) − (−2)\t | \\= −5 − 10 + 2 |\n| | \\= {{{correctAnswer}}} |\n\n</div>"}]},{"vars":[{"varval":"What is the value of the expression\n\n\r\n\r\n−1 − (−3) + (−5)"},{"varval":"<div class=\"aligned\">\n\n| | |\n| --------------------- | -------------- |\n| −1 − (−3) + (−5)\t | \\= −1 + 3 − 5 |\n| | \\= {{{correctAnswer}}} |\n\n</div>"}]}]

  736. <div class="sm_mode"> {{{question}}} </div>

    [{"vars":[{"varval":"Karl draws a rectangle on a number plane as shown below.\n\n<br>\n\nsm_img https://teacher.smartermaths.com.au/wp-content/uploads/2020/03/NAP-L4-CA05.svg 365 indent3 vpad\n\n<br>\n\nWhat are the coordinates of point $R$?\n"},{"varval":"<div class=\"aligned\">\n\n| | |\n| --------------------- | -------------- |\n| Coordinates | \\= (x, y) |\n| | \\= (8, –3) |\n\n</div>"}]},{"vars":[{"varval":"Janice draws a rectangle on a number plane as shown below.\n\n<br>\n\nsm_img https://teacher.smartermaths.com.au/wp-content/uploads/2020/03/NAP-L4-CA05.svg 365 indent3 vpad\n\n<br>\n\nWhat are the coordinates of point $S$?\n"},{"varval":"<div class=\"aligned\">\n\n| | |\n| ----------- | ---------------------- |\n| Coordinates | \\= (x, y) |\n| | \\= (–2, –3) |\n\n</div>"}]}]

  737. <div class="sm_mode"> {{{question}}} </div>

    [{"vars":[{"varval":"A child's bike tyre has a circumference of 90 cm.\n\nWhich of these is closest to the radius of the circle?"},{"varval":"<div class=\"aligned\">\n\n| | |\n| ----: | ------------------------------ |\n| $C$ | \\= 2$\\large \\pi r$ |\n| 90 | \\= 2 $\\times\\ \\large \\pi\\ \\times \\large r$ |\n| $\\large r$ | \\= $\\dfrac{90}{2\\large \\pi}$ |\n| | $\\approx \\dfrac{90}{2\\times 3.14}$ |\n| | $\\approx \\dfrac{90}{6.3}$ |\n| | $\\approx$ {{{correctAnswer}}} (closest answer) |\n\n</div>"}]},{"vars":[{"varval":"A child's bike tyre has a circumference of 64 cm.\n\nWhich of these is closest to the radius of the tyre?"},{"varval":"<div class=\"aligned\">\n\n| | |\n| ----------------: | ----------------------- |\n| $C$ | \\= 2$\\large \\pi \\large r$ |\n| 64 | \\= 2$\\large \\pi \\large r$ |\n| $\\therefore\\ \\large r$ | \\= $\\dfrac{64}{2\\large \\pi}$ |\n| | $\\approx \\dfrac{64}{2\\times 3.14}$ |\n| | \\= {{{correctAnswer}}} |\n\n</div>"}]}]

  738. <div class="sm_mode"> {{{question}}} </div>

    [{"vars":[{"varval":"A triangle is drawn on grid paper.\n\nsm_img https://teacher.smartermaths.com.au/wp-content/uploads/2018/06/NAPX-J3-CA11_1.svg 220 indent3 vpad\n\nWhat is the area of the triangle?"},{"varval":"<div class=\"indent2\">\n\nsm_img https://teacher.smartermaths.com.au/wp-content/uploads/2018/06/NAPX-J3-CA11.svg 220 indent vpad\n\n</div>\n\n<div class=\"aligned\">\n\n| | |\n| --------------------- | ------------------------------------------- |\n| Area | \\= $\\dfrac{1}{2}\\ \\times \\text{base}\\ \\times\\ \\text{height}$ |\n| | \\= $\\dfrac{1}{2}\\ \\times\\ 6\\ \\times\\ 5$ |\n| | \\= {{{correctAnswer}}} |\n\n</div>"}]},{"vars":[{"varval":"A triangle is drawn on grid paper.\n\nsm_img https://teacher.smartermaths.com.au/wp-content/uploads/2017/09/NAP-J1-CA11_1.png 300 indent3 vpad\n\nWhat is the area of the triangle?\n"},{"varval":"<div class=\"indent2\">\n\nsm_img https://teacher.smartermaths.com.au/wp-content/uploads/2022/08/Measurement_60003_v1_ws.svg 300 indent vpad\n\n</div>\n\n\n<div class=\"aligned\">\n\n| | |\n| --------------------- | -------------------------------------------- |\n| Area | = $\\dfrac{1}{2}\\ \\times\\ \\text{base}\\ \\times\\ \\text{height}$ |\n| | = $\\dfrac{1}{2}\\ \\times\\ 4\\ \\times\\ 4$ |\n| | = {{{correctAnswer}}} |\n\n</div>"}]},{"vars":[{"varval":"A triangle is drawn on grid paper.\n\nsm_img https://teacher.smartermaths.com.au/wp-content/uploads/2022/08/Measurement_60003_v2.svg 320 indent3 vpad\n\nWhat is the area of the triangle?"},{"varval":"<div class=\"indent2\">\n\nsm_img https://teacher.smartermaths.com.au/wp-content/uploads/2022/08/Measurement_60003_v2_ws.svg 320 indent vpad\n\n</div>\n\n<div class=\"aligned\">\n\n| | |\n| --------------------- | ------------------------------------------- |\n| Area | \\= $\\dfrac{1}{2}\\ \\times \\text{base}\\ \\times\\ \\text{height}$ |\n| | \\= $\\dfrac{1}{2}\\ \\times\\ 10\\ \\times\\ 7$ |\n| | \\= {{{correctAnswer}}} |\n\n</div>"}]},{"vars":[{"varval":"A triangle is drawn on grid paper.\n\nsm_img https://teacher.smartermaths.com.au/wp-content/uploads/2022/08/Measurement_60003_v3.svg 360 indent vpad\n\nWhat is the area of the triangle?"},{"varval":"<div class=\"indent\">\n\nsm_img https://teacher.smartermaths.com.au/wp-content/uploads/2022/08/Measurement_60003_v3_ws.svg 360 indent vpad\n\n</div>\n\n<div class=\"aligned\">\n\n| | |\n| --------------------- | ------------------------------------------- |\n| Area | \\= $\\dfrac{1}{2}\\ \\times \\text{base}\\ \\times\\ \\text{height}$ |\n| | \\= $\\dfrac{1}{2}\\ \\times\\ 6\\ \\times\\ 5$ |\n| | \\= {{{correctAnswer}}} |\n\n</div>"}]},{"vars":[{"varval":"A triangle is drawn on grid paper.\n\nsm_img https://teacher.smartermaths.com.au/wp-content/uploads/2022/08/Measurement_60003_v4.svg 360 indent3 vpad\n\nWhat is the area of the triangle?"},{"varval":"<div class=\"indent2\">\n\nsm_img https://teacher.smartermaths.com.au/wp-content/uploads/2022/08/Measurement_60003_v4_ws.svg 360 indent vpad\n\n</div>\n\n<div class=\"aligned\">\n\n| | |\n| --------------------- | ------------------------------------------- |\n| Area | \\= $\\dfrac{1}{2}\\ \\times \\text{base}\\ \\times\\ \\text{height}$ |\n| | \\= $\\dfrac{1}{2}\\ \\times\\ 3\\ \\times\\ 5$ |\n| | \\= {{{correctAnswer}}} |\n\n</div>"}]},{"vars":[{"varval":"A triangle is drawn on grid paper.\n\nsm_img https://teacher.smartermaths.com.au/wp-content/uploads/2022/08/Measurement_60003_v5.svg 350 indent3 vpad\n\nWhat is the area of the triangle?"},{"varval":"<div class=\"indent2\">\n\nsm_img https://teacher.smartermaths.com.au/wp-content/uploads/2022/08/Measurement_60003_v5_ws.svg 350 indent vpad\n\n</div>\n\n<div class=\"aligned\">\n\n| | |\n| --------------------- | ------------------------------------------- |\n| Area | \\= $\\dfrac{1}{2}\\ \\times \\text{base}\\ \\times\\ \\text{height}$ |\n| | \\= $\\dfrac{1}{2}\\ \\times\\ 2\\ \\times\\ 4$ |\n| | \\= {{{correctAnswer}}} |\n\n</div>"}]}]

  739. <div class="sm_mode"> {{{question}}} </div>

    [{"vars":[{"varval":"Which expression is equivalent to $12\\large x$ + 24?"},{"varval":"sm_nogap Check each option:\n\n<div class=\"aligned\">\r\n\r\n| | |\r\n| ------------- | ---------- |\r\n| {{{correctAnswer}}} | \\= ($3 \\times 4\\large x$) + (3 $\\times\\ 8$) |\r\n| |= $12\\large x$ + 24 &nbsp;$\\checkmark$ |\r\n\r\n</div>"}]},{"vars":[{"varval":"Which expression is equivalent to $24\\large m$ $-$ 16?"},{"varval":"sm_nogap Check each option:\n\n<div class=\"aligned\">\r\n\r\n| | |\r\n| ------------- | ---------- |\r\n| $4(6\\large m$ $-$ 20) | \\= ($4 \\times 6\\large m$) $-$ (4 $\\times\\ 20$) |\r\n| |= $24\\large m$ $-$ 80 &nbsp;x |\r\n\r\n</div>\n<br>\n<div class=\"aligned\">\r\n\r\n| | |\r\n| ------------- | ---------- |\r\n| {{{correctAnswer}}} | \\= ($4 \\times 6\\large m$) $-$ (4 $\\times\\ 4$) |\r\n| |= $24\\large m$ $-$ 16 &nbsp;$\\checkmark$ |\r\n\r\n</div>"}]},{"vars":[{"varval":"Which expression is equivalent to $15\\large y$ + 25?"},{"varval":"sm_nogap Check each option:\n\n<div class=\"aligned\">\r\n\r\n| | |\r\n| ------------- | ---------- |\r\n| $15(\\large y$ + 5) | \\= ($15 \\times \\large y$) + (15 $\\times\\ 5$) |\r\n| |= $15\\large y$ + 75 &nbsp;x |\r\n\r\n</div>\n<br>\n<div class=\"aligned\">\r\n\r\n| | |\r\n| ------------- | ---------- |\r\n| $15(\\large y$ + 1) | \\= ($15 \\times \\large y$) + (15 $\\times\\ 1$) |\r\n| |= $15\\large y$ + 15 &nbsp;x |\r\n\r\n</div>\n<br>\n<div class=\"aligned\">\r\n\r\n| | |\r\n| ------------- | ---------- |\r\n| {{{correctAnswer}}} | \\= ($5 \\times 3\\large y$) + (5 $\\times\\ 5$) |\r\n| |= $15\\large y$ + 25 &nbsp;$\\checkmark$ |\r\n\r\n</div>"}]},{"vars":[{"varval":"Which expression is equivalent to $18\\large y$ + 36?"},{"varval":"Check each option\n\n<div class=\"aligned\">\r\n\r\n| | |\r\n| ------------- | ---------- |\r\n| $18(\\large y$ + 36) | \\= ($18 \\times \\large y$) + (18 $\\times\\ 36$) |\r\n| |= $18\\large y$ + 648 &nbsp;x |\r\n\r\n</div>\n<br>\n<div class=\"aligned\">\r\n\r\n| | |\r\n| ------------- | ---------- |\r\n| $6(6\\large y$ + 3) | \\= ($6 \\times 6\\large y$) + (6 $\\times\\ 3$) |\r\n| |= $36\\large y$ + 18 &nbsp;x |\r\n\r\n</div>\n<br>\n<div class=\"aligned\">\r\n\r\n| | |\r\n| ------------- | ---------- |\r\n| $3(12\\large y$ + 12) | \\= ($3 \\times 12\\large y$) + (3 $\\times\\ 12$) |\r\n| |= $36\\large y$ + 36 &nbsp;x |\r\n\r\n</div>\n<br>\n<div class=\"aligned\">\r\n\r\n| | |\r\n| ------------- | ---------- |\r\n| {{{correctAnswer}}} | \\= ($6 \\times 3\\large y$) + (6 $\\times\\ 6$) |\r\n| |= $18\\large y$ + 36 &nbsp;$\\checkmark$ |\r\n\r\n</div>\n"}]},{"vars":[{"varval":"Which expression is equivalent to $20\\large p$ $-$ 48?"},{"varval":"sm_nogap Check each option:\n\n<div class=\"aligned\">\r\n\r\n| | |\r\n| ------------- | ---------- |\r\n| {{{correctAnswer}}} | \\= ($2 \\times 10\\large p$) $-$ (2 $\\times\\ 24$) |\r\n| |= $20\\large p$ $-$ 48 &nbsp;$\\checkmark$ |\r\n\r\n</div>"}]},{"vars":[{"varval":"Which expression is equivalent to $21\\large m$ $-$ 35?"},{"varval":"sm_nogap Check each option:\n\n<div class=\"aligned\">\r\n\r\n| | |\r\n| ------------- | ---------- |\r\n| $3(7\\large m$ $-$ 15) | \\= ($3 \\times 7\\large m$) $-$ (3 $\\times\\ 15$) |\r\n| |= $21\\large m$ $-$ 45 &nbsp;x |\r\n\r\n</div>\n<br>\n<div class=\"aligned\">\r\n\r\n| | |\r\n| ------------- | ---------- |\r\n| {{{correctAnswer}}} | \\= ($7 \\times 3\\large m$) $-$ (7 $\\times\\ 5$) |\r\n| |= $21\\large m$ $-$ 35 &nbsp;$\\checkmark$ |\r\n\r\n</div>"}]}]

  740. <div class="sm_mode"> {{{question}}} </div>

    [{"vars":[{"varval":"Coral bought 8 bags of mandarins from the market.\r\n\r\nEach bag had the same number of mandarins and a total of 56 mandarins were purchased.\r\n\r\nWhich equation shows the average number of mandarins, $\\large m$, in each bag?\n"},{"varval":"<div class=\"aligned\">\r\n\r\n| | |\r\n| ------------: | ---------- |\r\n| Mandarins per bag | \\= $\\dfrac{ \\text{total mandarins}}{\\text{number of bags}}$ |\r\n|||\n| $\\large m$ | $= \\dfrac{56}{8}$ |\n| $\\large m$ $\\times\\ 8$| \\= 56|\n\r\n\r\n</div>"}]},{"vars":[{"varval":"Jessie bought 7 boxes of chocolates from the shop.\r\n\r\nEach box had the same number of chocolates and a total of 84 chocolates were purchased.\r\n\r\nWhich equation shows the average number of chocolates, $\\large c$, in each box?\n"},{"varval":"<div class=\"aligned\">\r\n\r\n| | |\r\n| ------------: | ---------- |\r\n| Chocolates per box | \\= $\\dfrac{ \\text{total chocolates }}{\\text{number of boxes}}$ |\r\n|||\n| $\\large c$ | $= \\dfrac{84}{7}$ |\n| $\\large c$ $\\times\\ 7$| \\= 84|\n\r\n\r\n</div>"}]},{"vars":[{"varval":"Vernon bought 6 pallets of pavers from PaverCity.\r\n\r\nEach pallet had the same number of pavers and a total of 126 pavers were purchased.\r\n\r\nWhich equation shows the average number of pavers, $\\large p$, on each pallet?\n"},{"varval":"<div class=\"aligned\">\r\n\r\n| | |\r\n| ------------: | ---------- |\r\n| pavers per pallet | \\= $\\dfrac{ \\text{total pavers}}{\\text{number of pallets}}$ |\r\n|||\n| $\\large p$ | $= \\dfrac{126}{6}$ |\n| $\\large p$ $\\times\\ 6$| \\= 126|\n\r\n\r\n</div>"}]},{"vars":[{"varval":"Hope bought 9 boxes of pens from Office Supplies.\r\n\r\nEach box had the same number of pens and a total of 189 pens were purchased.\r\n\r\nWhich equation shows the average number of pens, $\\large p$, in each box?\n"},{"varval":"<div class=\"aligned\">\r\n\r\n| | |\r\n| ------------: | ---------- |\r\n| pens per box | \\= $\\dfrac{ \\text{total pens}}{\\text{number of boxes}}$ |\r\n|||\n| $\\large p$ | $= \\dfrac{189}{9}$ |\n| $\\large p$ $\\times\\ 9$| \\= 189|\n\r\n\r\n</div>"}]},{"vars":[{"varval":"Mel bought 24 packets of lollies from the supermarket.\r\n\r\nEach packet had the same number of lollies and a total of 360 lollies were purchased.\r\n\r\nWhich equation shows the average number of lollies, $\\large l$, in each packet?\n"},{"varval":"<div class=\"aligned\">\r\n\r\n| | |\r\n| ------------: | ---------- |\r\n| lollies per packet | \\= $\\dfrac{ \\text{total lollies}}{\\text{number of packets}}$ |\r\n|||\n| $\\large l$ | $= \\dfrac{360}{24}$ |\n| $\\large l$ $\\times\\ 24$| \\= 360|\n\r\n\r\n</div>"}]},{"vars":[{"varval":"Skylar bought 500 boxes of nails from the hardware store.\r\n\r\nEach box had the same number of nails and a total of 7000 nails were purchased.\r\n\r\nWhich equation shows the average number of nails, $\\large n$, in each box?\n"},{"varval":"<div class=\"aligned\">\r\n\r\n| | |\r\n| ------------: | ---------- |\r\n| nails per box | \\= $\\dfrac{ \\text{total nails}}{\\text{number of boxes}}$ |\r\n|||\n| $\\large n$ | $= \\dfrac{7000}{500}$ |\n| $\\large n$ $\\times\\ 500$| \\= 7000|\n\r\n\r\n</div>"}]}]

  741. <div class="sm_mode"> A square has a side length $S$. sm_img https://teacher.smartermaths.com.au/wp-content/uploads/2018/07/NAPX-F4-NC12new.svg 200 indent vpad Which expression cannot be used for the perimeter? </div>

    [{"vars":null}]

  742. <div class="sm_mode"> {{{question}}} </div>

    [{"vars":[{"varval":"The running time of a documentary is 148 minutes.\n\n\r\n\r\nPaul starts watching the documentary at 2:13 pm.\n\n\r\n\r\nWhat time will the documentary end in 24-hour time?"},{"varval":"<div class=\"aligned\">\n\n| | |\n| --------------------- | -------------- |\n| Running time | \\= 148 minutes |\n| | \\= 2 hours 28 minutes |\n\n</div>\n\n<br>\n\n<div class=\"aligned\">\n\n| | |\n| --------------------- | -------------- |\n| $\\therefore$ Finish time | \\= 2:13 pm plus 2 hours 28 minutes |\n| | \\= 4:41 pm |\n|| = {{{correctAnswer}}}|\n\n</div>"}]},{"vars":[{"varval":"The running time of a documentary is 157 minutes.\n\n\r\n\r\nMichael starts watching the documentary at 12:07 pm.\n\n\r\n\r\nWhat time will the documentary end in 24-hour time?"},{"varval":"<div class=\"aligned\">\n\n| | |\n| --------------------- | -------------- |\n| Running time | \\= 157 minutes |\n| | \\= 2 hours 37 minutes |\n\n</div>\n\n<br>\n\n<div class=\"aligned\">\n\n| | |\n| --------------------- | -------------- |\n| $\\therefore$ Finish time | \\= 12:07 pm plus 2 hours 37 minutes |\n| | \\= 2:44 pm |\n|| = 14:44|\n\n</div>"}]},{"vars":[{"varval":"The running time of a movie at the cinema is 148 minutes.\n\r\nPaul starts watching the movie at 2:13 pm.\n\n\r\n\r\nWhat time will the movie end in 24-hour time?"},{"varval":"<div class=\"aligned\">\n\n|||\n|-|-|\n|Running time|= 148 minutes|\n||= 2 hours 28 minutes |\n\n</div>\n\n<br>\n\n<div class=\"aligned\">\n\n|||\n|-|-|\n|$\\therefore$ Finish time|= 2:13 pm plus 2 hours 28 minutes|\n||= 4:41 pm|\n||= {{{correctAnswer}}}|\n\n</div>"}]}]

  743. <div class="sm_mode"> {{{question}}} </div>

    [{"vars":[{"varval":"An unknown number is added to −3.\r\n\r\nThe result is multiplied by 2 to give an answer of 6.\r\n\r\nWhich of these is the unknown number?"},{"varval":"Strategy 1\n\nTrial each option:\n\nIf unknown number = 6\n\n>6 $-$ 3 = 3\n\n<br>\n\n>2 $\\times$ 3 = 6 (correct)\n\n<br>\n\nStrategy 2 (advanced)\n\nLet &nbsp;$\\large n$ = unknown number\n\nWorking backwards:\n\n>3 $\\times$ 2 = 6\n\n<br>\n\n>$\\Rightarrow\\ \\large n$ + ($-$ 3) = 3\n\n<br>\n\n$\\therefore\\ \\large n$ = {{{correctAnswer}}}"}]},{"vars":[{"varval":"An unknown number is added to −5.\r\n\r\nThe result is multiplied by 3 to give an answer of 9.\r\n\r\nWhich of these is the unknown number?"},{"varval":"Solution 1\n\nTrial each option:\n\nIf unknown number = 8\n\n8 $-$ 5 = 3\n\n3 $\\times$ 3 = 9 (correct)\n\n<br>\n\nSolution 2 (advanced)\n\nLet &nbsp;$\\large n$ = unknown number\n\nWorking backwards:\n\n3 $\\times$ 3 = 9\n\n$\\Rightarrow\\ \\large n$ + ($-$ 5) = 3\n\n$\\therefore\\ \\large n$ = {{{correctAnswer}}}"}]},{"vars":[{"varval":"An unknown number is added to 2.\r\n\r\nThe result is multiplied by 4 to give an answer of $-$ 4.\r\n\r\nWhich of these is the unknown number?"},{"varval":"Solution 1\n\nTrial each option:\n\nIf unknown number = $-$ 3 \n\n$-$ 3 + 2 = $-$ 1\n\n4 $\\times$ $-$ 1 = $-$ 4 (correct)\n\n<br>\n\nSolution 2 (advanced)\n\nLet &nbsp;$\\large n$ = unknown number\n\nWorking backwards:\n\n4 $\\times$ $-$ 1 = $-$ 4\n\n$\\Rightarrow\\ \\large n$ + 2 = $-$ 1\n\n$\\therefore\\ \\large n$ = {{{correctAnswer}}}"}]},{"vars":[{"varval":"An unknown number is added to 5.\r\n\r\nThe result is multiplied by 3 to give an answer of $-$ 6.\r\n\r\nWhich of these is the unknown number?"},{"varval":"Solution 1\n\nTrial each option:\n\nIf unknown number = $-$ 7 \n\n$-$ 7 + 5 = $-$ 2\n\n3 $\\times$ $-$ 2 = $-$ 6 (correct)\n\n<br>\n\nSolution 2 (advanced)\n\nLet &nbsp;$\\large n$ = unknown number\n\nWorking backwards:\n\n3 $\\times$ $-$ 2 = $-$ 6\n\n$\\Rightarrow\\ \\large n$ + 5 = $-$ 2\n\n$\\therefore\\ \\large n$ = {{{correctAnswer}}}"}]},{"vars":[{"varval":"An unknown number is added to −1.\r\n\r\nThe result is multiplied by 2 to give an answer of −8.\r\n\r\nWhich of these is the unknown number?"},{"varval":"Solution 1\n\nTrial each option:\n\nIf unknown number = $-$ 3\n\n$-$ 3 + ($-$ 1) = $-$ 4\n\n2 $\\times$ $-$ 4 = $-$ 8 (correct)\n\n<br>\n\nSolution 2 (advanced)\n\nLet &nbsp;$\\large n$ = unknown number\n\nWorking backwards:\n\n2 $\\times$ $-$ 4 = $-$ 8\n\n$\\Rightarrow\\ \\large n$ + ($-$ 1) = $-$ 4\n\n$\\therefore\\ \\large n$ = {{{correctAnswer}}}"}]},{"vars":[{"varval":"An unknown number is added to −2.\r\n\r\nThe result is multiplied by 4 to give an answer of $-$ 6.\r\n\r\nWhich of these is the unknown number?"},{"varval":"Solution 1\n\nTrial each option:\n\nIf unknown number = $\\dfrac{1}{2}$\n\n$-$ 2 + $\\dfrac{1}{2}$ = $-$ $1\\dfrac{1}{2}$\n\n4 $\\times$ $-$ $1\\dfrac{1}{2}$ = $-$ 6 (correct)\n\n<br>\n\nSolution 2 (advanced)\n\nLet &nbsp;$\\large n$ = unknown number\n\nWorking backwards:\n\n4 $\\times$ $-$ $1\\dfrac{1}{2}$ = $-$ 86\n\n$\\Rightarrow\\ \\large n$ + ($-$ 2) = $-$ $1\\dfrac{1}{2}$\n\n$\\therefore\\ \\large n$ = {{{correctAnswer}}}"}]}]

  744. <div class="sm_mode"> {{{question}}} </div>

    [{"vars":[{"varval":"$\\large y$ = $2\\large x$$^2\\ −\\ 1.5$\n\n\r\nWhat is the value of $\\large y$ when &nbsp;$\\large x$ = 3.5\n"},{"varval":"\n<div class=\"aligned\">\r\n\r\n| | |\r\n| ------------: | ---------- |\r\n| $\\large y$| \\= 2$\\large x$$^2\\ −\\ 1.5$ |\n| | \\= $2(3.5)^2\\ −\\ 1.5$ |\r\n| | \\= {{{correctAnswer}}} |\r\n\r\n</div>"}]},{"vars":[{"varval":"$\\large y$ $=3\\large x$$^2\\ −\\ 2.5$\n\n\r\nWhat is the value of $\\large y$ when &nbsp;$\\large x$ $=3.5$\n"},{"varval":"<div class=\"aligned\">\n\n| | |\n| ------------: | ---------- |\n| $\\large y$ | \\= 3$\\large x$$^2\\ −\\ 2.5$ |\n| | \\= $3(3.5)^2\\ −\\ 2.5$ |\n| | \\= {{{correctAnswer}}} |\n\n</div>"}]},{"vars":[{"varval":"$\\large y$ $=5\\large x$$^2\\ −\\ 4.5$\n\n\r\nWhat is the value of $\\large y$ when &nbsp;$\\large x$ $=0.5$\n"},{"varval":"<div class=\"aligned\">\r\n\r\n| | |\r\n| ------------: | ---------- |\r\n| $\\large y$ | \\= 5$\\large x$$^2\\ −\\ 4.5$ |\n| | \\= $5(0.5)^2\\ −\\ 4.5$ |\r\n| | \\= {{{correctAnswer}}} |\r\n\r\n</div>"}]},{"vars":[{"varval":"$y=\\large x$$^2\\ − \\ \\large x$ $-$ 1.5\n\n\r\nWhat is the value of $\\large y$ when &nbsp;$\\large x$ $=1.5$\n"},{"varval":"<div class=\"aligned\">\r\n\r\n| | |\r\n| ------------: | ---------- |\r\n| $\\large y$ | \\= $\\large x$$^2 \\ − \\ \\large x$ $-$ 1.5\n |\n| | \\= $(1.5)^2\\ −\\ 1.5\\ −\\ 1.5$ |\r\n| | \\= {{{correctAnswer}}} |\r\n\r\n</div>"}]},{"vars":[{"varval":"$\\large y$ $=\\large x$$^2\\ − \\ \\large x$ $-$ 4\n\n\r\nWhat is the value of $\\large y$ when &nbsp;$\\large x$ = $-$ 1\n"},{"varval":"<div class=\"aligned\">\r\n\r\n| | |\r\n| ------------: | ---------- |\r\n| $\\large y$ | \\= $\\large x$$^2 \\ − \\ \\large x$ $-$ 4\n |\n| | \\= ($-$ 1)$^2 \\ \\ −\\ (-1 )\\ − \\ 4$ |\r\n| | \\= {{{correctAnswer}}} |\r\n\r\n</div>"}]},{"vars":[{"varval":"$\\large y$ $=\\large x$$^2\\ + \\ 2\\large x$ + 3\n\n\r\nWhat is the value of $\\large y$ when &nbsp;$\\large x$ = $-$ 1\n"},{"varval":"<div class=\"aligned\">\r\n\r\n| | |\r\n| ------------: | ---------- |\r\n| $\\large y$ | \\= $\\large x$$^2 \\ + \\ 2\\large x$ $+$ 3\n |\n| | \\= ($-$ 1)$^2 \\ \\ +2\\times\\ (-1 )\\ + \\ 3$ |\r\n| | \\= {{{correctAnswer}}} |\r\n\r\n</div>"}]}]

  745. <div class="sm_mode"> {{{question}}} </div>

    [{"vars":[{"varval":"Which expression is equal to $4\\large r$$^3$?"},{"varval":"$4\\large r$$^3$ = {{{correctAnswer}}}"}]},{"vars":[{"varval":"Which expression is equal to $3\\large p$$^2$?"},{"varval":"$3\\large p$$^2$ = {{{correctAnswer}}}"}]}]

  746. SJ v1-v5

    <div class="sm_mode"> {{{question}}} </div>

    [{"vars":[{"varval":"$1.5$ × <span class=\"sm-text color2\">?</span> &nbsp;= $1.2$\n\n<br>\n\nFind the value of &nbsp;<span class=\"sm-text color2\">?</span> &nbsp;that makes this number sentence correct."},{"varval":"<div class=\"aligned\">\n\n> | | |\n> | ------------: | ---------------------- |\n> | $1.5$ × <span class=\"sm-text color2\">?</span>| = 1.2 |\n| | |\n| | |\n> | <span class=\"sm-text color2\">?</span>| = $\\dfrac{1.2}{1.5}$|\n> | | \\= {{{correctAnswer}}} |\n\n</div>\n\n"}]},{"vars":[{"varval":"$2.5$ × <span class=\"sm-text color3\">?</span> &nbsp;= $1.5$\n\n<br>\n\nFind the value of &nbsp;<span class=\"sm-text color3\">?</span> &nbsp;that makes this number sentence correct."},{"varval":"<div class=\"aligned\">\n\n> | | |\n> | ------------: | ---------------------- |\n> | $2.5$ × <span class=\"sm-text color3\">?</span>| = 1.5 |\n| | |\n| | |\n> | <span class=\"sm-text color3\">?</span>| = $\\dfrac{1.5}{2.5}$|\n> | | \\= {{{correctAnswer}}} |\n\n</div>\n"}]},{"vars":[{"varval":"$3.2$ × <span class=\"sm-text color4\">?</span> &nbsp;= $4.8$\n\n<br>\n\nFind the value of &nbsp;<span class=\"sm-text color4\">?</span> &nbsp;that makes this number sentence correct."},{"varval":"<div class=\"aligned\">\n\n> | | |\n> | ------------: | ---------------------- |\n> | $3.2$ × <span class=\"sm-text color4\">?</span>| = 4.8 |\n| | |\n| | |\n> | <span class=\"sm-text color4\">?</span>| = $\\dfrac{4.8}{3.2}$|\n> | | \\= {{{correctAnswer}}} |\n\n</div>\n"}]},{"vars":[{"varval":"$2.4$ × <span class=\"sm-text color5\">?</span> &nbsp;= $1.8$\n\n<br>\n\nFind the value of &nbsp;<span class=\"sm-text color5\">?</span> &nbsp;that makes this number sentence correct."},{"varval":"<div class=\"aligned\">\n\n> | | |\n> | ------------: | ---------------------- |\n> | $2.4$ × <span class=\"sm-text color5\">?</span>| = 1.8 |\n| | |\n| | |\n> | <span class=\"sm-text color5\">?</span>| = $\\dfrac{1.8}{2.4}$|\n> | | \\= {{{correctAnswer}}} |\n\n</div>\n"}]},{"vars":[{"varval":"$3.5$ × <span class=\"sm-text color6\">?</span> &nbsp;= $2.1$\n\n<br>\n\nFind the value of &nbsp;<span class=\"sm-text color6\">?</span> &nbsp;that makes this number sentence correct."},{"varval":"<div class=\"aligned\">\n\n> | | |\n> | ------------: | ---------------------- |\n> | $3.5$ × <span class=\"sm-text color6\">?</span>| = 2.1 |\n| | |\n| | |\n> | <span class=\"sm-text color6\">?</span>| = $\\dfrac{2.1}{3.5}$|\n> | | \\= {{{correctAnswer}}} |\n\n</div>\n"}]},{"vars":[{"varval":"$4.20$ × <span class=\"sm-text color9\">?</span> &nbsp;= $3.15$\n\n<br>\n\nFind the value of &nbsp;<span class=\"sm-text color9\">?</span> &nbsp;that makes this number sentence correct."},{"varval":"<div class=\"aligned\">\n\n> | | |\n> | ------------: | ---------------------- |\n> | $4.20$ × <span class=\"sm-text color9\">?</span>| = 3.15 |\n| | |\n| | |\n> | <span class=\"sm-text color9\">?</span>| = $\\dfrac{3.15}{4.20}$|\n> | | \\= {{{correctAnswer}}} |\n\n</div>\n"}]}]

  747. <div class="sm_mode"> {{{question}}} </div>

    [{"vars":[{"varval":"<div class=\"sm_mode\">\n\nA school's chess club has 6 girls and 5 boys.\r\n\r\n2 girls and some more boys then joined the club.\r\n\r\nThe number of boys in the club was now double the number of girls.\r\n\r\nHow many boys joined the club?\n\n</div>\n"},{"varval":"<div class=\"sm_mode\">\n\n\n<div class=\"aligned\">\n\n| | |\n| --------------------: | -------------- |\n| Girls in club | = 6 + 2 |\n| | = 8 |\n\n</div>\n\n\n<br>\n\n\n<div class=\"aligned\">\n\n| | |\n| --------------------: | -------------- |\n| $\\Rightarrow$ Total boys required | = 2 $\\times$ 8 |\n| | = 16 |\n\n</div>\n\n\n<br>\n\n\n<div class=\"aligned\">\n\n| | |\n| --------------------: | -------------- |\n| $\\therefore$ Boys that joined | = 16 $-$ 5 |\n| | = 11 |\n\n</div>\n\n\n</div>"}]},{"vars":[{"varval":"<div class=\"sm_mode\">\n\nA school's garden club has 4 girls and 6 boys.\r\n\r\nTwo boys and some girls then joined the club.\r\n\r\nThe number of girls in the club was now double the number of boys.\r\n\r\nHow many girls joined the club?\n\n</div>\n"},{"varval":"<div class=\"sm_mode\">\n\n\n<div class=\"aligned\">\n\n| | |\n| --------------------: | -------------- |\n| Boys in club | = 6 + 2 |\n| | = 8 |\n\n</div>\n\n\n<br>\n\n\n<div class=\"aligned\">\n\n| | |\n| --------------------: | -------------- |\n| $\\Rightarrow$ Total girls required | = 2 $\\times$ 8 |\n| | = 16 |\n\n</div>\n\n\n<br>\n\n\n<div class=\"aligned\">\n\n| | |\n| --------------------: | -------------- |\n| $\\therefore$ Girls that joined | = 16 $-$ 4 |\n| | = 12 |\n\n</div>\n\n\n</div>"}]},{"vars":[{"varval":"A school's drama club has 7 girls and 9 boys.\n\n\r\n\r\n4 girls and some boys then joined the club.\n\n\r\n\r\nThe number of boys in the club is now double the number of girls.\n\n\r\n\r\nHow many boys joined the club?"},{"varval":"<div class=\"aligned\">\n\n| | |\n| --------------------- | -------------- |\n| Girls in club\t | \\= 7 + 4 |\n| | \\= 11 |\n\n</div>\n\n<div class=\"aligned\">\n\n| | |\n| --------------------- | -------------- |\n| ⇒ Total boys required\t\t | \\= 2 $\\times$ 11 |\n| | \\= 22 |\n\n</div>\n\n<div class=\"aligned\">\n\n| | |\n| --------------------- | -------------- |\n| $\\therefore$ Boys that joined\t\t | \\= 22 $-$ 9 |\n| | \\= 13 |\n\n</div>"}]}]

  748. Number, NAPX-L4-CA04 v2 Number, NAPX-L4-CA04 v1 Added categories.

    <div class="sm_mode"> {{{question}}} </div>

    [{"vars":[{"varval":"In one year, $\\dfrac{3}{10}$ of the puppies that a dog breeder sells are ridgebacks.\n\nWhat is $\\dfrac{3}{10}$ as percentage?\n"},{"varval":"$\\dfrac{3}{10} = \\dfrac{30}{100}$ = 30%"}]},{"vars":[{"varval":"In one term, $\\dfrac{7}{10}$ of students in a school had at least 1 day off sick.\n\nWhat is $\\dfrac{7}{10}$ as percentage?\n"},{"varval":"$\\dfrac{7}{10} = \\dfrac{70}{100}$ = 70%"}]}]

  749. <div class="sm_mode"> {{question}} </div>

    [{"vars":[{"varval":"At midday, the temperature is 30°C.\n\nAt midnight, the temperature is 6°C.\n\nWhich calculation below can be used to work out how many degrees cooler it is at midnight than at midday? "},{"varval":"Degrees cooler = 30 $-$ 6"}]},{"vars":[{"varval":"At 6 am the temperature is 3°C.\n\nAt 2 pm the temperature is 27°C.\n\nWhich calculation below can be used to work out how many degrees warmer it is at 2 pm than at 6 am? "},{"varval":"Degrees warmer = 27 $−$ 3"}]},{"vars":[{"varval":"At 6 am the temperature in San Diego was 12.7°C.\n\nAt 2 pm it was 7.5°C warmer.\n\nAt 8 pm it was 9.6°C cooler than at 2 pm.\n\nWhat was the temperature at 8 pm?"},{"varval":"<div class=\"aligned\">\n\n\n>| | |\n| --------------------: | -------------- |\n| Temperature at 8 pm| = 12.7 + 7.5 $-$ 9.6 |\n| |= 10.6$\\degree$C |\n\n</div>\n\r\r"}]},{"vars":[{"varval":"Karen arrived home and turned on the heater.\n\nBefore turning on the heater, the temperature in the room was –5°C.\n\nAfter fifteen minutes, the temperature in the room was 12°C.\n\nWhat was the change in temperature in the room?"},{"varval":"<div class=\"aligned\">\n\n>| | |\n| --------------------: | -------------- |\n| Change in temperature | = 12 $−\\ (−5)$ |\n| | = 17°C (increase) |\n\n</div>"}]}]

  750. <div class="sm_mode"> {{{question}}} </div>

    [{"vars":[{"varval":"Which of the following is equal to 48?"},{"varval":"<div class=\"aligned\">\n\n>| | |\n| ----------- :| -------------------------------------- |\n| $2^4 + 2^5$| \\= ($2 \\times 2\\times 2\\times 2) + (2 \\times 2\\times 2\\times 2\\times 2)$|\n| | = 16 + 32|\n| | = 48 |\n\n</div>"}]},{"vars":[{"varval":"Which of the following is equal to 108?"},{"varval":"<div class=\"aligned\">\n\n>| | |\n| ----------- :| -------------------------------------- |\n| $3^3 + 3^4$| \\= ($3 \\times 3\\times 3) + (3 \\times 3\\times 3\\times 3)$|\n| | = 27 + 81|\n| | = 108 |\n\n</div>"}]},{"vars":[{"varval":"Which of the following is equal to 80?"},{"varval":"<div class=\"aligned\">\n\n>| | |\n| ----------- :| -------------------------------------- |\n| $4^3 + 4^2$| \\= ($4 \\times 4\\times 4) + (4 \\times 4)$|\n| | = 64 + 16|\n| | = 80 |\n\n</div>"}]},{"vars":[{"varval":"Which of the following is equal to 500?"},{"varval":"<div class=\"aligned\">\n\n>| | |\n| ----------- :| -------------------------------------- |\n| $5^4 \\ -\\ 5^3$| \\= ($5 \\times 5\\times 5\\times 5) \\ - \\ (5 \\times 5\\times 5)$|\n| | = $625 \\ - \\ 125$|\n| | = 500 |\n\n</div>"}]},{"vars":[{"varval":"Which of the following is equal to 45?"},{"varval":"<div class=\"aligned\">\n\n>| | |\n| ----------- :| -------------------------------------- |\n| $2^3 \\ + \\ 4^3\\ -\\ 3^3$| \\= ($2 \\times 2\\times 2$) + ($4 \\times 4\\times 4) \\ - \\ (3 \\times 3\\times 3$)|\n| | = $8 \\ + \\ 64 \\ - \\ 27$|\n| | = 45 |\n\n</div>"}]}]

  751. Added a Trial and Error worked solution option.

    <div class="sm_mode"> Which number is missing from this number sentence? > > (153.8 $-$ 11.1) $\times$ <span class="sm-text color8">?</span> = 549.395 </div>

    [{"vars":null}]

  752. Corrected error in solution. Added categories Needs difficulty

    <div class="sm_mode"> {{{question}}} </div>

    [{"vars":[{"varval":"Zeus has 21 chickens, from 3 different breeds.\n\n<div class=\"outline\">\n\n> | Breed | Number of Chickens | \n> | :-----------: | :----------: |\n> | Leghorn | 7 | \n> | Polish | 5 | \n> | Dorking | 9 | \n\n</div>\n\n<br>What is the probability that a chicken selected randomly will be either a Dorking chicken or a Leghorn chicken?"},{"varval":"<div class=\"aligned\">\r\n\r\n| | |\r\n| ------------- | ---------- |\r\n| $P$ (Leg horn or Dorking) | \\= $\\dfrac{\\text{Number of Leghorn + Number of Dorking}}{\\text{Total number of chickens}}$ |\n| | \\= $\\dfrac{7 + 9}{21}$|\r\n| | \\= {{{correctAnswer}}} |\r\n\r\n</div>\r\n"}]},{"vars":[{"varval":"Wendy has 32 different flowers as shown in the table below. \n\n<div class=\"outline\">\n\n> | Kind of flower | Number of flowers | \n> | :-----------: | :----------: |\n> | Carnations | 8 | \n> | Tulips | 6 | \n> | Dandelions | 11 | \n> | Roses | 7 |\n\n</div>\n\n<br>What is the probability that a flower selected randomly will be either a carnation or a tulip?"},{"varval":"<div class=\"aligned\">\r\n\r\n| | |\r\n| ------------- | ---------- |\r\n| $P$ (Carnation or Tulip) | \\= $\\dfrac{\\text{Number of Carnations + Number of Tulips}}{\\text{Total number of flowers}}$ |\n| | \\= $\\dfrac{8 + 6}{32}$|\n|| \\= $\\dfrac{14}{32}$ |\r\n| | \\= {{{correctAnswer}}} (lowest term) |\r\n\r\n</div>\r\n"}]}]

  753. Corrected grammar Added categories Needs difficulty

    <div class="sm_mode"> {{{question}}} </div>

    [{"vars":[{"varval":"Tom has a dice and rolls it repeatedly 54 times, each time recording which side faces up.\t\r\n\nHow many times should he expect to see the side four coming up?\n"},{"varval":"\r\nProbability of getting number four on a dice = $\\dfrac{1}{6}$\n\n<div class=\"aligned\">\r\n\r\n| | |\r\n| ------------- | ---------- |\r\n| $P$ (4 on a die) | \\= $\\dfrac{1}{6} \\ \\times \\ 54$ |\r\n| | \\= {{{correctAnswer0}}} |\r\n\r\n</div>\r\n"}]},{"vars":[{"varval":"Maybelle has a coin and she tosses it and records which side is facing up when it lands.\n\nShe repeats this process 68 times. \t\r\n\nHow many times should she expect that the coin will land with heads \rfacing up?\n"},{"varval":"Probability of getting heads on a coin = $\\dfrac{1}{2}$\n\n<div class=\"aligned\">\r\n\r\n| | |\r\n| ------------- | ---------- |\r\n| $P$ (heads) | \\= $\\dfrac{1}{2} \\ \\times \\ 68$ |\r\n| | \\= {{{correctAnswer0}}} |\r\n\r\n</div>"}]}]

  754. Changed variable to workedSolution so solution visible Added categories

    <div class="sm_mode"> {{{question}}} </div>

    [{"vars":[{"varval":"Brian drew a shape.\n\nsm_img https://teacher.smartermaths.com.au/wp-content/uploads/2021/06/RAPH13-q1.svg 350 indent vpad\n\nAngle B measures 235°. \n\nWhat is the size of angle A?\n"},{"varval":"One full rotation = 360$\\degree$\n\n<div class=\"aligned\">\n\n| | |\n| --------------------: | -------------- |\n| Angle A | \\= 360$\\degree$ $-$ 235$\\degree$ |\n| | \\= 125$\\degree$ |\n\n</div>\n"}]},{"vars":[{"varval":"A shape was drawn by Moby.\n\nsm_img https://teacher.smartermaths.com.au/wp-content/uploads/2021/06/RAPH13-q2.svg 250 indent vpad\n\n\nAngle R measures 147°. \n\nWhat is the size of angle Q?\n"},{"varval":"One full rotation = 360$\\degree$\n\n<div class=\"aligned\">\n\n| | |\n| --------------------: | -------------- |\n| Angle Q | \\= 360$\\degree$ $-$ 147$\\degree$ |\n| | \\= 213$\\degree$ |\n\n</div>\n"}]}]

  755. NAP-F2-23

    <div class="sm_mode"> {{{question}}} </div>

    [{"vars":[{"varval":"Julie looked at the figure made up of cubes from the direction of the arrow, as shown in the image below.\n\n<br>\n\nsm_img https://teacher.smartermaths.com.au/wp-content/uploads//2021/08/41.svg 250 indent3 vpad\n\nWhich of the following is Julie’s view of the cubes?"},{"varval":"Julie's view depends on the highest stack of cubes in each column (check all rows).\n\n{{{correctAnswer}}}"}]},{"vars":[{"varval":"Tom was looking at a stack cubes from the direction of the arrow, as shown below.\n\n<br>\n\nsm_img https://teacher.smartermaths.com.au/wp-content/uploads//2021/08/42.svg 250 indent3 vpad\n\n<br>Which of the following is Tom’s view of the cubes?"},{"varval":"Tom's view depends on the highest stack of cubes in each column (check all rows).\n\n{{{correctAnswer}}}"}]},{"vars":[{"varval":"Johan looked at the figure made up of cubes from the direction of the arrow, as shown in the image below.\n\n<br>\n\nsm_img https://teacher.smartermaths.com.au/wp-content/uploads/2022/09/Geom_RAPH12-Q41-42_v2.svg 250 indent3 vpad\n\nWhich of the following is Johan’s view of the cubes?"},{"varval":"Johan's view depends on the highest stack of cubes in each column (check all rows).\n\n{{{correctAnswer}}}"}]},{"vars":[{"varval":"Jori looked at the figure made up of cubes from the direction of the arrow, as shown in the image below.\n\n<br>\n\nsm_img https://teacher.smartermaths.com.au/wp-content/uploads/2022/09/Geom_RAPH12-Q41-42_v-3.svg 250 indent3 vpad\n\nWhich of the following is Jori’s view of the cubes?"},{"varval":"Jori's view depends on the highest stack of cubes in each column (check all rows).\n\n{{{correctAnswer}}}"}]},{"vars":[{"varval":"Jimmy looked at the figure made up of cubes from the direction of the arrow, as shown in the image below.\n\n<br>\n\nsm_img https://teacher.smartermaths.com.au/wp-content/uploads/2022/09/Geom_RAPH12-Q41-42_v4.svg 250 indent3 vpad\n\nWhich of the following is Jimmy’s view of the cubes?"},{"varval":"Jimmy's view depends on the highest stack of cubes in each column (check all rows).\n\n{{{correctAnswer}}}"}]},{"vars":[{"varval":"Jamal looked at the figure made up of cubes from the direction of the arrow, as shown in the image below. \n\n<br>\n\nsm_img https://teacher.smartermaths.com.au/wp-content/uploads/2022/09/Geom_RAPH12-Q41-42_v5.svg 250 indent3 vpad\n\nWhich of the following is Jamal’s view of the cubes?"},{"varval":"Jamal's view depends on the highest stack of cubes in each column (check all rows).\n\n{{{correctAnswer}}}"}]}]

  756. NAP-F2-24

    <div class="sm_mode"> {{{question}}} </div>

    [{"vars":[{"varval":"4 angles are labeled on the image below.\n\n<br>\n\nsm_img https://teacher.smartermaths.com.au/wp-content/uploads//2021/07/39.svg 250 indent3 vpad\n\n<br>Which angle is closest to 30$\\degree$?"},{"varval":"sm_nogap Consider each angle:\n\n>Angle X is close to 60$\\degree$\r\n\n>Angle Y is close to 30$\\degree$\r\n\n>Angle W is 90$\\degree$\r\n\n>Angle Z $\\approx$ Angle X\r\n\n<br>\n\nTherefore, angle {{{correctAnswer}}} is closest to 30$\\degree$\r\n"}]},{"vars":[{"varval":"Four angles are labeled in the quadrilateral shown below.\n\n<br>\n\nsm_img https://teacher.smartermaths.com.au/wp-content/uploads//2021/07/40.svg 200 indent3 vpad\n\n<br>Which angle is closest to 120$\\degree$?"},{"varval":"sm_nogap Consider each angle:\n\n>Angles A and B are acute angles\r (less than 90$\\degree$)\n\n\n>Angle D is greater than 90° but less than 120$\\degree$\r\n\n>Angle C is an obtuse angle with a size $\\approx$ 120$\\degree$\r\n\n<br>\n\nTherefore, {{{correctAnswer}}} is the angle closest to 120$\\degree$\r.\n"}]}]

  757. NAP-F2-25

    <div class="sm_mode"> {{{question}}} </div>

    [{"vars":[{"varval":"Using the table shown below, the distance from Bryton to Clifton is 28 kilometres\r.\n\r\n\r\n\n<div class=\"outline\">\r\n<table>\r\n <tr>\r\n <th align= \"center\" rowspan=\"2\" colspan=\"5\">Distance Between Cities (km)</th>\r\n </tr>\r\n <tr>\r\n </tr>\r\n <tr>\r\n <td>Bryton</td>\r\n <td rowspan=\"1\" colspan=\"5\"></td>\r\n </tr>\r\n <tr align= \"center\">\r\n <td align= \"center\">32</td>\r\n <td>Virginia</td>\r\n <td rowspan=\"1\" colspan=\"5\"></td>\r\n </tr>\r\n <tr>\r\n <td align= \"center\">28</td>\r\n <td align= \"center\">26</td>\r\n <td>Clifton</td>\r\n <td rowspan=\"1\" colspan=\"5\"></td>\r\n </tr>\r\n <tr>\r\n <td align= \"center\">14</td>\r\n <td align= \"center\">30</td>\r\n <td align= \"center\">21</td>\r\n <td>Chelsea</td>\r\n <td></td>\r\n </tr>\r\n <tr>\r\n <td align= \"center\">22</td>\r\n <td align= \"center\">31</td>\r\n <td align= \"center\">19</td>\r\n <td align= \"center\">8</td>\r\n <td>Binks</td>\r\n\n </tr>\r\n</table>\r\n</div>\n\n\r\n<br>What is the distance between Virginia and Chelsea?"},{"varval":"Go down the \"Virginia\" column in the table until the \"Chelsea\" row.\n\nTherefore, the distance between Virginia and Chelsea is {{{correctAnswer}}}.\r\n"}]},{"vars":[{"varval":"Using the table shown below, the distance from Maine to Grove is 24 kilometres.\n\n<div class=\"outline\">\r\n<table>\r\n <tr>\r\n <th align= \"center\" rowspan=\"2\" colspan=\"5\">Distance between Towns (km)</th>\r\n </tr>\r\n <tr>\r\n </tr>\r\n <tr>\r\n <td>Hampshire</td>\r\n <td rowspan=\"1\" colspan=\"5\"></td>\r\n </tr>\r\n <tr align= \"center\">\r\n <td align= \"center\">30</td>\r\n <td>Maine</td>\r\n <td rowspan=\"1\" colspan=\"5\"></td>\r\n </tr>\r\n <tr>\r\n <td align= \"center\">12</td>\r\n <td align= \"center\">32</td>\r\n <td>Carolina</td>\r\n <td rowspan=\"1\" colspan=\"5\"></td>\r\n </tr>\r\n <tr>\r\n <td align= \"center\">26</td>\r\n <td align= \"center\">24</td>\r\n <td align= \"center\">41</td>\r\n <td>Grove</td>\r\n <td></td>\r\n </tr>\n <tr>\r\n <td align= \"center\">14</td>\r\n <td align= \"center\">28</td>\r\n <td align= \"center\">19</td>\r\n <td align= \"center\">23</td>\r\n <td>Franklin</td>\r\n </tr>\n\r\n</table>\r\n</div>\n\n<br>What is the distance between Grove and Hampshire?"},{"varval":"Go down the \"Hampshire\" column in the table until the \"Grove\" row.\n\nTherefore, the distance between Hampshire and Grove is {{{correctAnswer}}}.\r\n"}]}]

  758. NAP-F2-26 SA

    <div class="sm_mode"> {{{question}}} </div>

    [{"vars":[{"varval":"Gabriel and Henry earn money every week for delivering pamphlets in their neighbourhood.\r\n\nHenry earns $12 each week while Gabriel earns $15.\r\n\nAfter 8 weeks, how much more money will Gabriel have made compared to Henry?"},{"varval":"Strategy one:\n\nGabriel earns $3 extra each week.\n\nsm_nogap Extra money after 8 weeks\n\n>> = 8 $\\times$ 3\n\n>> = {{{prefix0}}}{{{correctAnswer0}}}\n\n<br>\n\nStrategy two:\n\nGabriel's earnings = 8 $\\times$ 15 = $120\n\nHenry's earnings = 8 $\\times$ 12 = $96\n\nGabriel's extra earnings = 120 $-$ 96 = {{{prefix0}}}{{{correctAnswer0}}}"}]},{"vars":[{"varval":"Clarise and Hannibal are saving money each week for their overseas holiday.\r\n\nHannibal saves $25 each week while Clarise saves $30.\r\n\nAfter 14 weeks, how much more money will Clarise have saved compared to Hannibal?\r\n"},{"varval":"Strategy one:\n\nClarise saves $5 extra each week.\n\nsm_nogap Extra money after 14 weeks\n\n>> = 14 $\\times$ 5\n\n>> = {{{prefix0}}}{{{correctAnswer0}}}\n\n<br>\n\nStrategy two:\n\nClarise's savings = 14 $\\times$ 30 = $420\n\nHannibal's savings = 14 $\\times$ 25 = $350\n\nClarice's extra savings = 420 $-$ 350 = {{{prefix0}}}{{{correctAnswer0}}}\n"}]}]

  759. <div class="sm_mode"> {{{question}}} </div>

    [{"vars":[{"varval":"Which of the following expressions has the value closest to 15?"},{"varval":"Check each option:\r\n\n6 $\\dfrac{2}{3}$ + 7 $\\dfrac{1}{3}$ = 14\r\n\r\n\n7 $\\dfrac{1}{4}$ + 8 $\\dfrac{1}{4}$ = 15 $\\dfrac{1}{2}$ $\\checkmark$\r\n\n\r\n8 $\\dfrac{1}{2}$ + 7 $\\dfrac{3}{4}$ = 16 $\\dfrac{1}{4}$\r\n\r\n\n7 $\\dfrac{1}{2}$ + 8 $\\dfrac{1}{4}$ = 15 $\\dfrac{3}{4}$\r\n\nTherefore, the expression 7 $\\dfrac{1}{4}$ + 8 $\\dfrac{1}{4}$ has the value which is closest to 15."}]},{"vars":[{"varval":"Which of the following expressions has the value closest to 9?"},{"varval":"Check each option:\n\n5 $\\dfrac{2}{3}$ + 4 $\\dfrac{1}{3}$ = 10\r\n\n6 $\\dfrac{1}{4}$ + 3 $\\dfrac{1}{2}$ = 9 $\\dfrac{3}{4}$\r\n\n10 $\\dfrac{3}{4}$ $-$ 1 $\\dfrac{1}{4}$ = 9 $\\dfrac{1}{2}$\n\n\r\n3 $\\dfrac{1}{4}$ + 5 $\\dfrac{1}{2}$ = 8 $\\dfrac{3}{4}$ $\\checkmark$\n\n$\\therefore$ {{{correctAnswer}}} is closest to 9."}]}]

  760. NAP-F2-28 Var5 ... can you make the correct answer closer to a shape with a line of symmetry. A variation of the correct answer in var0 would make the question more challenging Fixed.

    <div class="sm_mode"> {{{question}}} </div>

    [{"vars":[{"varval":"There are four shapes drawn below.\n\n\r\nWhich of these shapes has no line of symmetry?\r\n"},{"varval":"This shape has no line of symmetry.\n\n{{{correctAnswer}}}"}]},{"vars":[{"varval":"Peter drew 4 shapes.\n\n\r\nWhich of them has no line of symmetry?\r\n"},{"varval":"This shape has no line of symmetry.\n\n{{{correctAnswer}}}"}]},{"vars":[{"varval":"Portia drew 4 shapes.\n\n\r\nWhich of them has no line of symmetry?\r\n"},{"varval":"This shape has no line of symmetry.\n\n{{{correctAnswer}}}"}]},{"vars":[{"varval":"Bryn drew 4 shapes. \n\n\r\nWhich of them has no line of symmetry?\r\n"},{"varval":"This shape has no line of symmetry.\n\n{{{correctAnswer}}}"}]},{"vars":[{"varval":"Guido drew 4 shapes.\n\n\r\nWhich of them has no line of symmetry?\r\n"},{"varval":"This shape has no line of symmetry.\n\n{{{correctAnswer}}}"}]},{"vars":[{"varval":"Becky drew 4 shapes.\n\n\r\nWhich of them has no line of symmetry?\r\n"},{"varval":"This shape has no line of symmetry.\n\n{{{correctAnswer}}}"}]}]

  761. NAP-F2-34 SA

    <div class="sm_mode"> {{{question}}} </div>

    [{"vars":[{"varval":"Complete the expression below:\r"},{"varval":"Since the decimal moved two places to the left, the number is divided by 100.\n\n296 ÷ {{{correctAnswer0}}} = 2.96"}]},{"vars":[{"varval":"Complete the expression below:\n"},{"varval":"The decimal moved one place to the left $\\rightarrow$ number is divided by 10.\n\n38.4 ÷ {{{correctAnswer0}}} = 3.84"}]}]

  762. NAP-F2-35 SA Var0 ... can you redo the old images so they look like yours (I much prefer the "finer" lines in your images Var1 ... OK Var2-5 ... all good

    <div class="sm_mode"> {{{question}}} </div>

    [{"vars":[{"varval":"Victoria cut a piece of paper in half as shown below.\n\nsm_img https://teacher.smartermaths.com.au/wp-content/uploads//2021/07/17.svg 228 indent3 vpad\n\nShe then placed the one half on top of the other half, as shown below.\n\nsm_img https://teacher.smartermaths.com.au/wp-content/uploads//2021/07/17_1.svg 210 indent3 vpad\n\nWhat is the perimeter of the newly formed shape in millimetres?"},{"varval":"Calculating the perimeter clockwise from the top:\n\nsm_img https://teacher.smartermaths.com.au/wp-content/uploads//2021/07/17_s.svg 200 indent3 vpad\n\n<div class=\"aligned\">\n\n| | |\n| --------- | ---------------------------------------- |\n| Perimeter | \\= 60 + 40 + 10 + 20 + 40 + 20 + 10 + 40 |\n| | \\= {{{correctAnswer0}}} {{{suffix0}}} |\n\n</div>"}]},{"vars":[{"varval":"Kevin cut a piece of paper in half.\n\nsm_img https://teacher.smartermaths.com.au/wp-content/uploads//2021/07/18.svg 250 indent3 vpad\n\nHe then placed the one half on top of the other half, as shown below.\n\nsm_img https://teacher.smartermaths.com.au/wp-content/uploads//2021/07/18_1.svg 250 indent3 vpad\n\nWhat is the perimeter of the newly formed shape in centimetres?"},{"varval":"Calculating the perimeter clockwise from the right:\n\nsm_img https://teacher.smartermaths.com.au/wp-content/uploads//2021/07/18_s.svg 250 indent3 vpad\n\n<div class=\"aligned\">\r\n\r\n| | |\r\n| ------------- | ---------- |\r\n| Perimeter | \\= 26 + 26 + 4 + 22 + 22 + 4 |\r\n| | \\= {{{correctAnswer0}}} {{{suffix0}}} |\r\n\r\n</div>\r\n"}]},{"vars":[{"varval":"Edward cut a piece of paper in half as shown below.\n\nsm_img https://teacher.smartermaths.com.au/wp-content/uploads/2022/08/Measurement_Raph12-17-18_v2a.svg 270 indent2 vpad\n\nHe then placed the one half on top of the other half, as shown below.\n\nsm_img https://teacher.smartermaths.com.au/wp-content/uploads/2022/08/Measurement_Raph12-17-18_v1b.svg 230 indent3 vpad\n\nWhat is the perimeter of the newly formed shape in millimetres?"},{"varval":"Calculating the perimeter clockwise from the top:\n\nsm_img https://teacher.smartermaths.com.au/wp-content/uploads/2022/08/Measurement_Raph12-17-18_v1ws.svg 300 indent3 vpad\n\n<div class=\"aligned\">\n\n| | |\n| --------- | ---------------------------------------- |\n| Perimeter | \\= 50 + 20 + 10 + 50 + 70 + 50 + 10 + 20 |\n| | \\= {{{correctAnswer0}}} {{{suffix0}}} |\n\n</div>"}]},{"vars":[{"varval":"Martha cut a piece of material in half as shown below.\n\nsm_img https://teacher.smartermaths.com.au/wp-content/uploads/2022/08/Measurement_Raph12-17-18_v3aa.svg 250 indent3 vpad\n\nShe then placed the one half on top of the other half, as shown below.\n\nsm_img https://teacher.smartermaths.com.au/wp-content/uploads/2022/08/Measurement_Raph12-17-18_v3b.svg 270 indent3 vpad\n\nWhat is the perimeter of the newly formed shape in millimetres?"},{"varval":"Calculating the perimeter clockwise from the top:\n\nsm_img https://teacher.smartermaths.com.au/wp-content/uploads/2022/08/Measurement_Raph12-17-18_v3ws.svg 320 indent3 vpad\n\n<div class=\"aligned\">\n\n| | |\n| --------- | ---------------------------------------- |\n| Perimeter | \\= 30 + 50 + 25 + 30 + 80 + 30 + 25 + 50 |\n| | \\= {{{correctAnswer0}}} {{{suffix0}}} |\n\n</div>"}]},{"vars":[{"varval":"Tracy cut a piece of ribbon in half.\n\nsm_img https://teacher.smartermaths.com.au/wp-content/uploads/2022/08/Measurement_Raph12-17-18_v4a.svg 300 indent3 vpad\n\nShe then placed the one half on top of the other half, as shown below.\n\nsm_img https://teacher.smartermaths.com.au/wp-content/uploads/2022/08/Measurement_Raph12-17-18_v4b.svg 280 indent3 vpad\n\nWhat is the perimeter of the newly formed shape in millimetres?"},{"varval":"Calculating the perimeter clockwise from the top:\n\nsm_img https://teacher.smartermaths.com.au/wp-content/uploads/2022/08/Measurement_Raph12-17-18_v4ws.svg 330 indent3 vpad\n\n<div class=\"aligned\">\r\n\r\n| | |\r\n| ------------- | ---------- |\r\n| Perimeter | \\= 250 + 250 + 25 + 225 + 225 + 25 |\r\n| | \\= {{{correctAnswer0}}} {{{suffix0}}} |\r\n\r\n</div>\r\n"}]},{"vars":[{"varval":"Jill cut a piece of binding in half.\n\nsm_img https://teacher.smartermaths.com.au/wp-content/uploads/2022/08/Measurement_Raph12-17-18_v5a.svg 280 indent3 vpad\n\nShe then placed the one half on top of the other half, as shown below.\n\nsm_img https://teacher.smartermaths.com.au/wp-content/uploads/2022/08/Measurement_Raph12-17-18_v5b.svg 280 indent3 vpad\n\nWhat is the perimeter of the newly formed shape in millimetres?"},{"varval":"Calculating the perimeter clockwise from the top:\n\nsm_img https://teacher.smartermaths.com.au/wp-content/uploads/2022/08/Measurement_Raph12-17-18_v5wsa.svg 330 indent3 vpad\n\n<div class=\"aligned\">\r\n\r\n| | |\r\n| ------------- | ---------- |\r\n| Perimeter | \\= 400 + 18 + 382 + 382 + 18 + 400 |\r\n| | \\= {{{correctAnswer0}}} {{{suffix0}}} |\r\n\r\n</div>\r\n"}]}]

  763. NAP-F2-36 SA

    <div class="sm_mode"> {{{question}}} </div>

    [{"vars":[{"varval":"Cedric has $20 and bought some chocolates for Valentine’s day.\n\nEach chocolate he bought costs $2.25.\r\n\nIf received $2 change, how many chocolates did he buy?\r\n"},{"varval":"<div class=\"aligned\">\r\n\r\n| | |\r\n| ------------- | ---------- |\r\n| Total cost of chocolates| \\= 20 – 2 |\r\n| | \\= $18 |\r\n\r\n</div>\n\n<br>\n\n\r\n<div class=\"aligned\">\r\n\r\n| | |\r\n| ------------- | ---------- |\r\n| Number of chocolates | \\= 18 $\\div$ 2.25 |\r\n| | \\= {{{correctAnswer0}}} {{{suffix0}}} |\r\n\r\n</div>\r\n"}]},{"vars":[{"varval":"Brendan has $85 and bought some books at a sale.\r\n\nEach book in the store sells for $11.75\r.\n\nIf Brendan received $2.75 in change, how many books did he buy?\r\n"},{"varval":"<div class=\"aligned\">\r\n\r\n| | |\r\n| ------------- | ---------- |\r\n| Total cost of books | = 85 – 2.75 |\r\n| | = 82.25 |\r\n\r\n</div>\n\n<br>\n\n\r\n<div class=\"aligned\">\r\n\r\n| | |\r\n| ------------- | ---------- |\r\n| Number of chocolates | = 82.25 $\\div$ 11.75 |\r\n| | \\= {{{correctAnswer0}}} {{{suffix0}}} |\r\n\r\n</div>\r\n"}]}]

  764. NAP-F2-37 SA

    <div class="sm_mode"> {{{question}}} </div>

    [{"vars":[{"varval":"The picture shown below is one face of a prism.\n\n<br>\n\nsm_img https://teacher.smartermaths.com.au/wp-content/uploads//2021/07/13.svg 150 indent3 vpad\n\n<br>How many edges does it have?"},{"varval":"Edges on front/end faces = 2 $\\times$ 5 = 10\n\nEdges joining faces = 5\r\n\n<div class=\"aligned\">\r\n\r\n| | |\r\n| ------------- | ---------- |\r\n| Total edges |= 10 + 5 |\r\n| | \\= {{{correctAnswer0}}} |\r\n\r\n</div>\r\n"}]},{"vars":[{"varval":"The picture shown below is one face of a prism.\n\n<br>\n\nsm_img https://teacher.smartermaths.com.au/wp-content/uploads//2021/07/14.svg 150 indent3 vpad\n\n<br>\n\nHow many edges does it have?"},{"varval":"Edges on front/back faces = 2 $\\times$ 8 = 16\n\nEdges joining front/back faces = 8\r\n\n<div class=\"aligned\">\r\n\r\n| | |\r\n| ------------- | ---------- |\r\n| Total edges | \\= 16 + 8 |\r\n| | \\= {{{correctAnswer0}}} |\r\n\r\n</div>\r\n"}]},{"vars":[{"varval":"The picture shown below is one face of a prism.\n\n<br>\n\nsm_img https://teacher.smartermaths.com.au/wp-content/uploads/2022/09/Geom_Raph12-13_14_v2.svg 180 indent3 vpad\n\n<br>\n\nHow many edges does it have?"},{"varval":"Edges on front/back faces = 2 $\\times$ 6 = 12\n\nEdges joining front/back faces = 6\r\n\n<div class=\"aligned\">\r\n\r\n| | |\r\n| ------------- | ---------- |\r\n| Total edges | \\= 12 + 6 |\r\n| | \\= {{{correctAnswer0}}} |\r\n\r\n</div>\r\n"}]},{"vars":[{"varval":"The picture shown below is one face of a prism.\n\n<br>\n\nsm_img https://teacher.smartermaths.com.au/wp-content/uploads/2022/09/Geom_Raph12-13_14_v3.svg 200 indent3 vpad\n\n<br>\n\nHow many edges does it have?"},{"varval":"Edges on front/back faces = 2 $\\times$ 7 = 14\n\nEdges joining front/back faces = 7\r\n\n<div class=\"aligned\">\r\n\r\n| | |\r\n| ------------- | ---------- |\r\n| Total edges | \\= 14 + 7 |\r\n| | \\= {{{correctAnswer0}}} |\r\n\r\n</div>\r\n"}]},{"vars":[{"varval":"The picture shown below is one face of a prism.\n\n<br>\n\nsm_img https://teacher.smartermaths.com.au/wp-content/uploads/2022/09/Geom_Raph12-13_14_v4.svg 200 indent3 vpad\n\n<br>\n\nHow many edges does it have?"},{"varval":"Edges on front/back faces = 2 $\\times$ 9 = 18\n\nEdges joining front/back faces = 9\r\n\n<div class=\"aligned\">\r\n\r\n| | |\r\n| ------------- | ---------- |\r\n| Total edges | \\= 18 + 9 |\r\n| | \\= {{{correctAnswer0}}} |\r\n\r\n</div>\r\n"}]},{"vars":[{"varval":"The picture shown below is one face of a prism.\n\n<br>\n\nsm_img https://teacher.smartermaths.com.au/wp-content/uploads/2022/09/Geom_Raph12-13_14_v5.svg 180 indent3 vpad\n\n<br>\n\nHow many edges does it have?"},{"varval":"Edges on front/back faces = 2 $\\times$ 10 = 20\n\nEdges joining front/back faces = 10\n\n<div class=\"aligned\">\r\n\r\n| | |\r\n| ------------- | ---------- |\r\n| Total edges | \\= 20 + 10 |\r\n| | \\= {{{correctAnswer0}}} |\r\n\r\n</div>\r\n"}]}]

  765. NAP-F2-38 SA

    <div class="sm_mode"> {{{question}}} </div>

    [{"vars":[{"varval":"Carlo is building a rectangular prism using identical triangular blocks as shown below.\n\n<br>\n\nsm_img https://teacher.smartermaths.com.au/wp-content/uploads/2021/07/11.svg 150 indent3 vpad\n\nThe solid figure made has 6 faces.\n\n<br>\n\nsm_img https://teacher.smartermaths.com.au/wp-content/uploads/2021/07/11_s.svg 250 indent3 vpad\n\nHow many triangular blocks did Carlo use to build this figure?"},{"varval":"Blocks in the top row = 8\r\n\r\n\nThere are an equal number of triangular blocks in each row.\r\n\n<div class=\"aligned\">\r\n\r\n\n| | |\r\n| ------------- | ---------- |\r\n| Total blocks | \\= 8 $\\times$ 4 |\r\n| | \\= {{{correctAnswer0}}} |\r\n\r\n\n</div>\r\n\r\n"}]},{"vars":[{"varval":"Francis is building a rectangular prism using identical blocks as shown below.\n\nsm_img https://teacher.smartermaths.com.au/wp-content/uploads//2021/07/12.svg 50 indent3 vpad\n\nThe solid figure made has 6 faces.\n\n<br>\n\nsm_img https://teacher.smartermaths.com.au/wp-content/uploads//2021/07/12_s.svg 250 indent3 vpad\n\nHow many blocks did Francis use to build this figure?"},{"varval":"Blocks in facing row = 4 + 6 + 6 = 16\r\n\r\n\nThere are an equal number of rectangular blocks in each row (going back).\r\n\n<div class=\"aligned\">\r\n\r\n\n| | |\r\n| ------------- | ---------- |\r\n| Total blocks | \\= 16 $\\times$ 4 |\r\n| | \\= {{{correctAnswer0}}} |\r\n\r\n\n</div>\r\n\r\n"}]},{"vars":[{"varval":"Luca is building a rectangular prism using identical triangular blocks as shown below.\n\nsm_img https://teacher.smartermaths.com.au/wp-content/uploads/2022/08/Measurement_RAPH12_Q11-12_v2a.svg 100 indent3 vpad\n\nThe solid figure made has 6 faces.\n\n<br>\n\nsm_img https://teacher.smartermaths.com.au/wp-content/uploads/2022/08/Measurement_RAPH12_Q11-12_v2b.svg indent3 vpad\n\nHow many triangular blocks did Luca use to build this figure?"},{"varval":"Blocks in top third of prism = 4 + 16 + 4 + 8 = 32\r\n\r\n\nThere are an equal number of triangular blocks in each third (going down).\r\n\n<div class=\"aligned\">\r\n\r\n\n| | |\r\n| ------------- | ---------- |\r\n| Total blocks | \\= 32 $\\times$ 3 |\r\n| | \\= {{{correctAnswer0}}} |\r\n\r\n\n</div>"}]},{"vars":[{"varval":"Paris is building a rectangular prism using identical triangular blocks as shown below.\n\nsm_img https://teacher.smartermaths.com.au/wp-content/uploads/2022/08/Measurement_RAPH12_Q11-12_v3a.svg 100 indent3 vpad\n\nThe solid figure made has 6 faces.\n\n<br>\n\nsm_img https://teacher.smartermaths.com.au/wp-content/uploads/2022/08/Measurement_RAPH12_Q11-12_v3_b.svg indent3 vpad\n\nHow many triangular blocks did Paris use to build this figure?"},{"varval":"Blocks in top third of prism = 4 + 16 + 4 = 24\r\n\r\n\nThere are an equal number of triangular blocks in each third (going down).\r\n\n<div class=\"aligned\">\r\n\r\n\n| | |\r\n| ------------- | ---------- |\r\n| Total blocks | \\= 24 $\\times$ 3 |\r\n| | \\= {{{correctAnswer0}}} |\r\n\r\n\n</div>"}]},{"vars":[{"varval":"Boston is building a 3-dimensional figure using identical blocks as shown below.\n\nsm_img https://teacher.smartermaths.com.au/wp-content/uploads/2022/08/Measurement_RAPH12_Q11-12_v4a.svg 70 indent3 vpad\n\nThe solid figure made has 6 faces.\n\n<br>\n\nsm_img https://teacher.smartermaths.com.au/wp-content/uploads/2022/08/Measurement_RAPH12_Q11-12_v4b.svg 300 indent3 vpad\n\nHow many blocks did Boston use to build this figure?"},{"varval":"Blocks in top third = 3 + 6 + 3 + 6 + 3 + 6 = 27\r\n\r\n\nThere are an equal number of rectangular blocks in each third (going down).\r\n\n<div class=\"aligned\">\r\n\r\n\n| | |\r\n| ------------- | ---------- |\r\n| Total blocks | \\= 27 $\\times$ 3 |\r\n| | \\= {{{correctAnswer0}}} |\r\n\r\n\n</div>"}]}]

  766. NAP-F2-29

    <div class="sm_mode"> {{{question}}} </div>

    [{"vars":[{"varval":"The grid map below shows the names of a number of cities in Japan, and the roads that connect them.\r\n\nThe distance between Yokohama to Tokyo is about 30 kilometres by road.\n\nsm_img https://teacher.smartermaths.com.au/wp-content/uploads/2021/07/29.svg 450 indent vpad\n\r\n\nWhich of these is closest to the distance from Oyama to Nikko Park along the road?"},{"varval":"Yokohama to Tokyo $\\approx$ 30 km.\n\nOyama to Nikko Park $\\approx$ twice the distance of Yokohama to Tokyo.\n\r\n\n<div class=\"aligned\">\n\n| | |\n| --------------------- | -------------- |\n| $\\therefore$ Oyama to Nikko Park | $\\approx 2\\ \\times$ 30 |\n| | $\\approx$ {{{correctAnswer}}} |\n\n</div>"}]},{"vars":[{"varval":"The grid map below shows the names and locations of a number of country towns, and the roads that join them.\n\nThe distance between Farras Creek and Tanbar is around 100 kilometres.\n\nsm_img https://teacher.smartermaths.com.au/wp-content/uploads//2021/07/30.svg 500 indent vpad\n\nWhich of these is closest to the distance from Birdsville to Tanbar along the connecting road?"},{"varval":"Tanbar to Farras Creek $\\approx$ 100 km.\n\nBirdsville to Tanbar is about twice the distance of Tanbar to Farras.\n\n\r\n\r\n<div class=\"aligned\">\r\n\r\n| | |\r\n| --------------------- | -------------- |\r\n| $\\therefore$ Birdsville to Tanbar | $\\approx$ 2 x 100 |\r\n| | = {{{correctAnswer}}} |\r\n\r\n</div>"}]}]

  767. NAP-F2-39 SA

    <div class="sm_mode"> {{{question}}} </div>

    [{"vars":[{"varval":"Hillary needs 1.7 kilograms of apples to make apple pies for a family celebration.\r\n\nShe places a box of apples on the scale shown\r.\n\n<br>\n\nsm_img https://teacher.smartermaths.com.au/wp-content/uploads/2021/05/RAPH12-Q9.svg 220 indent3 vpad\n\n<br>Neglecting the weight of the box, what amount of apples, in grams, does she still need?\n"},{"varval":"Reading the scale:\r\n\n4 intervals = 1 kg = 1000 grams\r\n\n1 interval = 250 grams\r\n\n<div class=\"aligned\">\r\n\r\n| | |\r\n| ------------- | ---------- |\r\n| Scale reading | \\= 3 $\\times$ 250 |\r\n| | \\= 750 grams |\r\n\r\n</div>\r\n\n<br>\n\n<div class=\"aligned\">\r\n\r\n| | |\r\n| ------------- | ---------- |\r\n| Weight needed | \\= 1700 – 750 |\r\n| | \\= {{{correctAnswer0}}} {{{suffix0}}} |\r\n\r\n</div>\r\n\n\n\r\n\r\r\n"}]},{"vars":[{"varval":"Samuel purchases 2.5 kilograms of salt at the market.\r\n\nPart of his purchase is weighed and shown on the scale below\r.\n\n<br>\n\nsm_img https://teacher.smartermaths.com.au/wp-content/uploads/2021/08/Raph12-10.png 220 indent3 vpad\n\n\n<br>How many more grams of salt does he need?\n"},{"varval":"Reading the scale:\r\n\n4 intervals = 1 kg = 1000 grams\r\n\n1 interval = 250 grams\r\n\n<div class=\"aligned\">\r\n\r\n| | |\r\n| ------------- | ---------- |\r\n| Scale reading | \\= 1000 + 2 $\\times$ 250 |\r\n| | \\= 1500 grams |\r\n\r\n</div>\r\n\n<br>\n\n<div class=\"aligned\">\r\n\r\n| | |\r\n| ------------- | ---------- |\r\n| Grams needed | \\= 2500 – 1500 |\r\n| | \\= {{{correctAnswer0}}} {{{suffix0}}} |\r\n\r\n</div>\r\n"}]}]

  768. NAP-F2-30

    <div class="sm_mode"> {{{question}}} </div>

    [{"vars":[{"varval":"10 charities share 5250 kilograms of rice in equal shares.\r\n\nEach of them distributes 510 kilograms and puts the remaining rice in storage.\r\n\nHow many kilograms, in total, were put in storage?\r\n\r\n"},{"varval":"<div class=\"aligned\">\n\n| | |\n| --------------------- | -------------- |\n|Amount distributed | \\= 510 $\\times$ 10 |\n| | \\= 5100 kilograms |\n\n</div>\n\n<br>\n\r\n<div class=\"aligned\">\n\n| | |\n| --------------------- | -------------- |\n| Rice in storage | \\= 5250 – 5100 |\n| | \\= {{{correctAnswer}}} |\n\n</div>\n"}]},{"vars":[{"varval":"A group of 10 people won a lottery prize of $2750.\r\n\nEach of\tthem received $250.\r\n\nThe balance of the prize was donated to charity.\r\n\nHow much was donated?\r\n"},{"varval":"<div class=\"aligned\">\n\n| | |\n| --------------------- | -------------- |\n| Amount received | \\= 10 x 250 |\n| | \\= $2500 |\n\n</div>\n\n\r<br>\n\n<div class=\"aligned\">\n\n| | |\n| --------------------- | -------------- |\n| Donation | \\= 2750 – 2500 |\n| | \\= $250 |\n\n</div>\n"}]}]

  769. NAP-F2-31

    <div class="sm_mode"> {{{question}}} </div>

    [{"vars":[{"varval":"Sarah buys a bread roll for 75 cents.\n\nShe pays the exact amount with 4 coins.\r\n\nWhich coin was **not** used to pay for the bread roll?\r\n"},{"varval":"4 coins that total exactly 75 cents:\n\r\n1 x 50 cent = 50 cents\r\n\n2 x 10 cent = 20 cents\r\n\n1 x 5 cent = 5 cents\r\n\n$\\therefore$ A {{{correctAnswer}}} coin was not used.\r\n"}]},{"vars":[{"varval":"Robert buys a drink that costs $1.15.\r\n\nHe has 4 coins in his man-bag and uses them all to pay the exact amount.\r\n\nWhich coin could **not** be in his man-bag?\r\n"},{"varval":"Combination 1: 1 x $1, 3 x 5 cents\n\nCombination 2: 2 x 50 cent, 1 x 10 cent, 1 x 5 cent\n\n$\\therefore$ A {{{correctAnswer}}} coin could not be in his man-bag.\r"}]}]

  770. NAP-F2-32

    <div class="sm_mode"> {{{question}}} </div>

    [{"vars":[{"varval":"A green grocer sells boxes of apples.\n\nEach box contains 9 apples and costs $5.\n\nsm_img https://teacher.smartermaths.com.au/wp-content/uploads//2021/05/RAPH12-Q23.svg 200 indent3 vpad\n\n\nWhat is the largest number of apples that can be bought for $36?\n\r\n"},{"varval":"<div class=\"aligned\">\n\n| | |\n| --------------------- | -------------- |\n| Number of boxes | \\= 36 $\\div$ 5 |\n| | \\= 7 remainder 1|\n\n</div>\n\n<br>\n\n<div class=\"aligned\">\n\n| | |\n| --------------------- | -------------- |\n| Number of apples | \\= 7 $\\times$ 9 |\n| | \\= 63 apples |\n\n</div>\n"}]},{"vars":[{"varval":"A carton of eggs is sold for $3.\r\n\nEach carton contains 12 eggs.\n\r\n\nsm_img https://teacher.smartermaths.com.au/wp-content/uploads/2021/11/RAPH12-Q24.png 200 indent vpad\n\nWhat is the largest number of eggs that can be bought for $25?"},{"varval":"<div class=\"aligned\">\n\n| | |\n| --------------------- | -------------- |\n| Number of cartons | \\= 25 $\\div$ 3 |\n| | \\= 8 remainder 1 |\n\n</div>\n\n<br>\n\n<div class=\"aligned\">\n\n| | |\n| --------------------- | -------------- |\n| Number of eggs | \\= 8 $\\times$ 12 |\n| | \\= {{{correctAnswer}}} eggs|\n\n</div>\n"}]}]

  771. <div class="sm_mode"> {{{question}}} </div>

    [{"vars":[{"varval":"A crowd of 3852 people attend a basketball game.\n\nOne quarter of the audience supports the away team.\n\nWhich of these is closest to the number of people who support the away team?\r\n"},{"varval":"Rounding up: 3852 ≈ 4000\n\n<div class=\"aligned\">\n\n| | |\n| ------ | ----------------- |\n| Away team supporters| ≈ $\\dfrac{1}{4}\\ \\times$ 4000 |\n| | ≈ {{{correctAnswer}}} |\n\n</div>"}]},{"vars":[{"varval":"In a supermarket, the total number of products in stock is 6325.\n\nOne eighth of the products supplied are made overseas.\n\nWhich of these is closest to the number of products in stock that are made overseas?\r\n"},{"varval":"Rounding up: 6325 ≈ 6400\n\n<div class=\"aligned\">\n\n| | |\n| ------ | ----------------- |\n| Made overseas| ≈ $\\dfrac{1}{8}\\ \\times$ 6400 |\n| | ≈ {{{correctAnswer}}} |\n\n</div>"}]}]

  772. VAR1-VAR5 ... The pie charts are good size but can we make the graph keys larger font? Also, worth checking greyscale (for monotone printing) ... I think VAR0,1,3,4,5 will need colour change - the best grey scale colours are VAR2 (I would just use these in every graph unless you have another good combination)

    <div class="sm_mode"> {{{question}}} </div>

    [{"vars":[{"varval":"Louise collected data on the fruit picked on a certain farm.\r\n\nThe graph below shows the amount of each fruit picked on the farm, in tonnes.\n\n<br>\n\nsm_img https://teacher.smartermaths.com.au/wp-content/uploads/2021/05/RAPH12-Q7.svg 330 indent2 vpad\n\n<br> There were 10 more tonnes of oranges picked than bananas.\n\nIn total, how many tonnes of fruit were picked?\r\n"},{"varval":"Total sections in graph = 16\n\n6 sections $\\rightarrow$ oranges\n\n4 sections $\\rightarrow$ bananas\r\n\n<div class=\"aligned\">\r\n\r\n| | |\r\n| -------------: | ---------- |\r\n| 2 sections | \\= 10 tonnes (given) |\n| 1 section | \\= 5 tonnes |\n\r\n\r\n\r\n</div>\n\n<br>\n\nSince all sections are equal\n\nTotal tonnes picked = 5 $\\times$ 16 = {{{correctAnswer0}}} {{{suffix0}}}\r\n"}]},{"vars":[{"varval":"Alexis collected data on the number of vehicles that used a certain road over a 30-minute period.\r\n\nThe graph below shows her results.\n\n<br>\n\r\nsm_img https://teacher.smartermaths.com.au/wp-content/uploads/2021/05/RAPH12-Q8.svg 330 indent3 vpad\n\n\n<br> There were 40 more cars than motorcycles that used the road during the 30-minute period.\r\n\nHow many vehicles, in total, used the road in the 30-minute period?\r\n\n"},{"varval":"\nTotal sections in graph = 16\n\n5 sections $\\rightarrow$ cars\n\n3 sections $\\rightarrow$ motorcycles\n\n<div class=\"aligned\">\r\n\r\n| | |\r\n| -------------: | ---------- |\r\n| 2 sections | \\= 40 vehicles (given) |\n| 1 section | \\= 20 vehicles |\n\r\n\r\n\r\n</div>\n\n<br>\n\nSince all sections are equal:\n\nTotal vehicles = 20 $\\times$ 16 = {{{correctAnswer0}}} {{{suffix0}}}"}]},{"vars":[{"varval":"A fast food restaurant collected data on the number of customers using the drive through service during a 4 hour period.\r\n\nThe graph below shows the results.\n\n<br>\n\nsm_img https://teacher.smartermaths.com.au/wp-content/uploads/2022/08/Stat_Prob_RAPH12-Q7-8_v2.svg 380 indent2 vpad\n\n<br> There were 87 more drive through customers between the hour 11am - 12pm than 2pm - 3 pm.\n\nIn total, how many customers used the drive through service during the 4 hour period?\r\n"},{"varval":"Total sections in graph = 16\n\n5 sections $\\rightarrow$ 11am - 12pm\n\n2 sections $\\rightarrow$ 2pm - 3 pm\r\n\n<div class=\"aligned\">\r\n\r\n| | |\r\n| -------------: | ---------- |\r\n| 3 sections | \\= 87 customers (given) |\n| 1 section | \\= 29 customers|\n\r\n\r\n\r\n</div>\n\n<br>\n\nSince all sections are equal\n\nTotal customers served = 29 $\\times$ 16 = {{{correctAnswer0}}} {{{suffix0}}}\r\n"}]},{"vars":[{"varval":"Gilligan collected data on the four most popular preferences of customers when booking accommodation through his online booking service.\r\n\nThe graph below shows the number of people using each type of accommodation.\n\n<br>\n\nsm_img https://teacher.smartermaths.com.au/wp-content/uploads/2022/08/Stat_Prob_RAPH12-Q7-8_v3.svg 400 indent2 vpad\n\n<br> There were 42 more people who chose a Resort than a Holiday Home.\n\nIn total, how many people chose one of the four types of accommodation when booking through Gilligan's booking service?\r\n"},{"varval":"Total sections in graph = 16\n\n5 sections $\\rightarrow$ Resort\n\n3 sections $\\rightarrow$ Holiday Home\r\n\n<div class=\"aligned\">\r\n\r\n| | |\r\n| -------------: | ---------- |\r\n| 2 sections | \\= 42 people (given) |\n| 1 section | \\= 21 people |\n\r\n\r\n\r\n</div>\n\n<br>\n\nSince all sections are equal\n\nTotal people = 21 $\\times$ 16 = {{{correctAnswer0}}} {{{suffix0}}}\r\n"}]},{"vars":[{"varval":"Jonah collected data on the seafood he had delivered to the fish market.\r\n\nThe graph below shows the amount of each seafood delivered, in kilograms.\n\n<br>\n\nsm_img https://teacher.smartermaths.com.au/wp-content/uploads/2022/09/Stat_Prob_RAPH12-Q7-8_v4a.svg 330 indent2 vpad\n\n<br> There were 110 more kilograms of prawns delivered than snapper.\n\nIn total, how many kilograms of seafood were delivered?\r\n"},{"varval":"Total sections in graph = 16\n\n5 sections $\\rightarrow$ Prawns\n\n3 sections $\\rightarrow$ Snapper\r\n\n<div class=\"aligned\">\r\n\r\n| | |\r\n| -------------: | ---------- |\r\n| 2 sections | \\= 110 kilograms (given) |\n| 1 section | \\= 55 kilograms |\n\r\n\r\n\r\n</div>\n\n<br>\n\nSince all sections are equal\n\nTotal kilograms delivered = 55 $\\times$ 16 = {{{correctAnswer0}}} {{{suffix0}}}\r\n"}]},{"vars":[{"varval":"Warwick collected data on the garden supplies sold at his nursery\r\n\nThe graph below shows the amount of each of the garden supplies sold, in kilograms.\n\n<br>\n\nsm_img https://teacher.smartermaths.com.au/wp-content/uploads/2022/09/Stat_Prob_RAPH12-Q7-8_v5a.svg 330 indent2 vpad\n\n<br> There were 1800 more kilograms of potting mix sold than compost.\n\nIn total, how many kilograms of garden supplies were sold?\r\n"},{"varval":"Total sections in graph = 12\n\n5 sections $\\rightarrow$ potting mix\n\n1 sections $\\rightarrow$ compost\r\n\n<div class=\"aligned\">\r\n\r\n| | |\r\n| -------------: | ---------- |\r\n| 4 sections | \\= 1800 kilograms (given) |\n| 1 section | \\= 450 kilograms |\n\r\n\r\n\r\n</div>\n\n<br>\n\nSince all sections are equal,\n\nTotal kilograms sold = 450 $\\times$ 12 = {{{correctAnswer0}}} \r\n"}]}]

  773. <div class="sm_mode"> {{{question}}} </div>

    [{"vars":[{"varval":"Felix asked some of his classmates what their favourite subject was.\n\nHe recorded their responses in the table shown below but forgot to label them.\n\n<br>\n\nsm_img https://teacher.smartermaths.com.au/wp-content/uploads/2021/05/RAPH12-Q5.svg 300 indent3 vpad\n\n<br>Felix remembers that:\n\n* Mathematics was the most popular subject\n \n* Physics was the least favourite subject\n \n* English was the favourite subject of more students than History\n \n\nHow many of Felix’s classmates chose History as their favourite subject?"},{"varval":"Using the data in the table given:\r\n\n12 people chose Mathematics\n\n8 people chose Physics\n\n\n$\\therefore$ 11 chose English while {{{correctAnswer0}}} chose History.\r\n"}]},{"vars":[{"varval":"Claire asked a group of students what their favourite sport was.\r\n\nTheir responses are shown in the table below but no labels were recorded.\n\n<br>\n\nsm_img https://teacher.smartermaths.com.au/wp-content/uploads/2021/05/RAPH12-Q6.svg 300 indent3 vpad\n\n<br>Claire who recorded the data shown remembers that:\r\n\n* Basketball received the most votes\n\n* Volleyball was the least favourite sport\r\n\n* Football was the favourite sport of more students than Tennis\r\n\nHow many of the students liked Football the most?\r\n"},{"varval":"Using the data in the table:\r\n\n16 people chose Basketball\n\r\n12 people chose Volleyball\r\n\n$\\therefore$ {{{correctAnswer0}}} chose Football while 14 chose Tennis.\r\n"}]}]

  774. <div class="sm_mode"> {{{question}}} </div>

    [{"vars":[{"varval":"George and Mae both bought a new keyboard and mouse for their computers.\n\nThey spent the same amount of money.\n\nGeorge’s mouse costs $38.85 and his keyboard costs $31.15.\n\nMae’s mouse costs $32.50.\n\nHow much did Mae’s keyboard cost?"},{"varval":"<div class=\"aligned\">\n\n| | |\n| --------------------: | -------------- |\n| George's total| \\= 38.85 + 31.15 |\n| | \\= $70 |\n\n</div>\n\nsm_nogap Since they spent the same amount of money:\n\n<div class=\"aligned\">\n\n| | |\n| --------------------: | -------------- |\n| Mae’s keyboard cost | \\= Total cost $–$ cost of mouse |\n| | \\= 70 $-$ 32.50 |\n| | \\= $37.50 |\n\n</div>\n\n"}]},{"vars":[{"varval":"Joyce and Michelle both bought a shirt and pants to wear on an excursion.\n\nEach of them spent the same amount of money.\n\nJoyce’s shirt costs $7.25 and her pants costs $10.75.\n\nMichelle’s pants costs $12.50.\n\nHow much did Michelle’s shirt cost?\r\n"},{"varval":"<div class=\"aligned\">\n\n| | |\n| --------------------: | -------------- |\n| Joyce's total | \\= 7.25 + 10.75 |\n| | \\= $18 |\n\n</div>\n\nsm_nogap Since they have spent the same amount of money:\n\n<div class=\"aligned\">\n\n| | |\n| --------------------: | -------------- |\n| Michelle's shirt cost | \\= Total cost $–$ cost of pants |\n| | \\= 18 $-$ 12.50 |\n| | \\= $5.50 |\n\n</div>\n\n"}]}]

  775. This uses a katex extension, so you can use the chemistry equations just like maths equations, in tables, indented, etc.. * For chemistry equations equations see https://mhchem.github.io/MathJax-mhchem/ * For molecule diagrams, use https://pubchem.ncbi.nlm.nih.gov/ (click on "draw structure") then export as svg.

    <div class="sm_mode"> $$\underrightarrow{AB}$$ Note: * For chemistry equations equations see https://mhchem.github.io/MathJax-mhchem/ * For molecule diagrams, use https://pubchem.ncbi.nlm.nih.gov/ (click on "draw structure") then export as svg. $\ce{CO2 + C -> 2 CO}$ $\ce{Hg^2+ ->[I-] HgI2 ->[I-] [Hg^{II}I4]^2-}$ $C_p[\ce{H2O(l)}] = \pu{75.3 J // mol K}$ $\ce{x Na(NH4)HPO4 ->[\Delta] (NaPO3)_x + x NH3 ^ + x H2O}$ $$\ce{Zn^2+ <=>[+ 2OH-][+ 2H+] $\underset{\text{amphoteres Hydroxid}}{\ce{Zn(OH)2 v}}$ <=>[+ 2OH-][+ 2H+] $\underset{\text{Hydroxozikat}}{\ce{[Zn(OH)4]^2-}}$}$$ $\ce{SO4^2- + Ba^2+ -> BaSO4 v}$ </div> --- $$\ce{Zn^2+ <=>[+ 2OH-][+ 2H+] $\underset{\text{amphoteres Hydroxid}}{\ce{Zn(OH)2 v}}$ <=>[+ 2OH-][+ 2H+] $\underset{\text{Hydroxozikat}}{\ce{[Zn(OH)4]^2-}}$}$$

    [{"vars":null}]

  776. RAPH - JOB #5 - Q18 Numbers 86-90

    <div class="sm_mode"> {{question}} </div>

    [{"vars":[{"varval":"A horse canters for 4 hours at an average speed of 12 km/h. \n\nHow far did the horse travel?"},{"varval":"<div class=\"aligned\">\r\n\r\n\n| | |\r\n| --: | ------- |\r\n| Distance travelled | = 4 $\\times$ 12 |\r\n| | = 48 km |\r\n\n\r\n\r\n</div>"}]},{"vars":[{"varval":"Lisa jogged for 3 hours at 18 km/h. \n\nHow far did she travel?"},{"varval":"<div class=\"aligned\">\r\n\r\n\n| | |\r\n| --: | ------- |\r\n| Distance travelled | = 3 $\\times$ 18 |\r\n| | = 54 km |\r\n\n\r\n\r\n</div>"}]},{"vars":[{"varval":"A light aircraft was flying at a speed of 60 metres per second. \n\nHow far did it travel in 5 seconds?\r\n"},{"varval":"<div class=\"aligned\">\r\n\r\n\n| | |\r\n| --: | ------- |\r\n| Distance travelled | = 5 $\\times$ 60 |\r\n| | = 300 |\r\n\n\r\n\r\n</div>"}]},{"vars":[{"varval":"A catamaran sails for 3 hours at 16 km/hr. \n\nHow far did it travel?"},{"varval":"<div class=\"aligned\">\r\n\r\n\n| | |\r\n| --: | ------- |\r\n| Distance travelled | = 3 $\\times$ 16 |\r\n| | = 48 km |\r\n\n\r\n\r\n</div>"}]},{"vars":[{"varval":"Water is being pumped out of a pool for 3 mins at 24 litres per minute. \n\nHow much water has been removed?"},{"varval":"<div class=\"aligned\">\r\n\r\n\n| | |\r\n| --: | ------- |\r\n| Water removed | = 3 $\\times$ 24 |\r\n| | = 72 litres |\r\n\n\r\n\r\n</div>"}]}]

  777. RAPH - JOB #5 - Q17 Numbers 81-85

    <div class="sm_mode"> {{question}} </div>

    [{"vars":[{"varval":"The total cost of 3 apples and 2 bananas is $1.50.\n\nThe total cost of 4 tangerines is $1.20. \n\nWhat is the average price of each piece of fruit?"},{"varval":"<div class=\"aligned\">\r\n\r\n\n| | |\r\n| -------: | -- |\r\n| Total cost | = 1.50 + 1.20 |\r\n| | = $2.70 |\r\n\r\n\n\r\n\r\n</div>\n\n<br>\n\n<div class=\"aligned\">\r\n\r\n\n| | |\r\n| -------: | -- |\r\n| Total pieces of fruit | = 3 + 2 + 4 |\r\n| | = 9 |\r\n\r\n\n\r\n\r\n</div>\n\n<br>\n\n<div class=\"aligned\">\r\n\r\n\n| | |\r\n| -------: | -- |\r\n| Average price | = $\\dfrac{2.70}{9}$ |\r\n| | = $0.30 |\r\n\r\n\n\r\n\r\n</div>"}]},{"vars":[{"varval":"The total weight of 3 goats and 2 sheep is 1000 kg. \n\nThe total weight of 3 pigs is 600 kg. \n\nWhat is the average weight of each animal?\r\n"},{"varval":"<div class=\"aligned\">\r\n\r\n\n| | |\r\n| -------: | -- |\r\n| Total weight| = 1000 + 600 |\r\n| | = 1600 kg |\r\n\r\n\n\r\n\r\n</div>\n\n<br>\n\n<div class=\"aligned\">\r\n\r\n\n| | |\r\n| -------: | -- |\r\n| Number of animals | = 3 + 2 + 3 |\r\n| | = 8 |\r\n\r\n\n\r\n\r\n</div>\n\n<br>\n\n<div class=\"aligned\">\r\n\r\n\n| | |\r\n| -------: | -- |\r\n| Average weight | = $\\dfrac{1600}{8}$ |\r\n| | = 200 kg |\r\n\r\n\n\r\n\r\n</div>"}]},{"vars":[{"varval":"The total length of 5 cane toads and 4 Surinam toads is 194 cm. \n\nThe total length of 3 common toads is 46 cm. \n\nWhat is the average length of the toads?"},{"varval":"<div class=\"aligned\">\r\n\r\n\n| | |\r\n| -------: | -- |\r\n| Total length of all toads | = 194 + 46 |\r\n| | = 240 |\r\n\r\n\n\r\n\r\n</div>\n\n<br>\n\n<div class=\"aligned\">\r\n\r\n\n| | |\r\n| -------: | -- |\r\n| Number of toads | = 5 + 4 + 3 |\r\n| | = 12 |\r\n\r\n\n\r\n\r\n</div>\n\n<br>\n\n<div class=\"aligned\">\r\n\r\n\n| | |\r\n| -------: | -- |\r\n| Average length | = $\\dfrac{240}{12}$ |\r\n| | = 20 cm |\r\n\r\n\n\r\n\r\n</div>"}]},{"vars":[{"varval":"Bryan measures the head widths of racquets in his sporting cupboard.\n\nThe total head width of 1 tennis racquet and 4 squash racquets is 112 cm. \n\nThe total head width of 2 badminton racquets is 42 cm. \n\nWhat is the average head width of all the racquets in his cupboard?"},{"varval":"<div class=\"aligned\">\r\n\r\n\n| | |\r\n| -------: | -- |\r\n| Total of all head widths | = 112 + 42 |\r\n| | = 154 cm |\r\n\r\n\n\r\n\r\n</div>\n\n<br>\n\n<div class=\"aligned\">\r\n\r\n\n| | |\r\n| -------: | -- |\r\n| Number of racquets | = 1 + 4 + 2 |\r\n| | = 7 |\r\n\r\n\n\r\n\r\n</div>\n\n<br>\n\n<div class=\"aligned\">\r\n\r\n\n| | |\r\n| -------: | -- |\r\n| Average head width | = $\\dfrac{154}{7}$ |\r\n| | = 22 cm |\r\n\r\n\n\r\n\r\n</div>"}]},{"vars":[{"varval":"The total capacity of 6 cups and 1 jug is 1.8 litres. \n\nThe total capacity of 5 glasses is 1.2 litres. \n\nWhat is the average capacity of all the items?"},{"varval":"<div class=\"aligned\">\r\n\r\n\n| | |\r\n| -------: | -- |\r\n| Total capacity of all items | = 1.8 + 1.2 |\r\n| | = 3 litres |\n| | = 3000 mL |\r\n\r\n\n\r\n\r\n</div>\n\n<br>\n\n<div class=\"aligned\">\r\n\r\n\n| | |\r\n| -------: | -- |\r\n| Number of items | = 6 + 1 + 5 |\r\n| | = 12 |\r\n\r\n\n\r\n\r\n</div>\n\n<br>\n\n<div class=\"aligned\">\r\n\r\n\n| | |\r\n| -------: | -- |\r\n| Average capacity | = $\\dfrac{3000}{12}$ |\r\n| | = 250 mL |\r\n\r\n\n\r\n\r\n</div>"}]}]

  778. RAPH - JOB #5 - Q16 Numbers 76-80

    <div class="sm_mode"> {{question}} </div>

    [{"vars":[{"varval":"3 + 9 × 2 + 5 ="},{"varval":"sm_nogap Use MDAS rule (solve the different operations in order of Multiplication, Division, Addition and Subtraction):\n\nsm_nogap 3 + 9 × 2 + 5 \n\r\n\r\n\n<div class=\"aligned\">\r\n\r\n\n> > | |\r\n> > | ------- |\r\n> > | = 3 + 18 + 5 |\r\n> > | = 26 |\r\n\r\n\r\n\n\r\n\r\n</div>"}]},{"vars":[{"varval":"6 ÷ 2 + 8 ÷ 4 ="},{"varval":"sm_nogap Use MDAS rule (solve the different operations in order of Multiplication, Division, Addition and Subtraction):\n\nsm_nogap 6 ÷ 2 + 8 ÷ 4\n\r\n\r\n\n<div class=\"aligned\">\r\n\r\n\n> > | |\r\n> > | ------- |\r\n> > | = 3 + 2 |\r\n> > | = 5 |\r\n\r\r\n\n\r\n\r\n</div>"}]},{"vars":[{"varval":"24 ÷ 4 + 8 × 2 ="},{"varval":"sm_nogap Use MDAS rule (solve the different operations in order of Multiplication, Division, Addition and Subtraction):\n\nsm_nogap 24 ÷ 4 + 8 × 2 \n\r\n\r\n\n<div class=\"aligned\">\r\n\r\n\n> > | |\r\n> > | ------- |\r\n> > | = 24 ÷ 4 + 16 |\r\n> > | = 6 + 16 |\r\n> > | = 22 |\r\r\n\n\r\n\r\n</div>"}]},{"vars":[{"varval":"24 $-$ 10 ÷ 2 + 8 ="},{"varval":"sm_nogap Use MDAS rule (solve the different operations in order of Multiplication, Division, Addition and Subtraction):\n\nsm_nogap 24 $-$ 10 ÷ 2 + 8\n\r\n\r\n\n<div class=\"aligned\">\r\n\r\n\n> > | |\r\n> > | ------- |\r\n> > | = 24 $-$ 5 + 8 |\r\n> > | = 27 |\r\n\r\r\n\n\r\n\r\n</div>"}]},{"vars":[{"varval":"5 + 9 $-$ 3 × 2 ="},{"varval":"sm_nogap Use MDAS rule (solve the different operations in order of Multiplication, Division, Addition and Subtraction):\n\nsm_nogap 5 + 9 $-$ 3 × 2\n\r\n\r\n\n<div class=\"aligned\">\r\n\r\n\n> > | |\r\n> > | ------- |\r\n> > | = 5 + 9 $-$ 6 |\r\n> > | = 8 |\r\n\r\r\n\n\r\n\r\n</div>"}]}]

  779. RAPH - JOB #5 - Q15 Numbers 71-75

    <div class="sm_mode"> {{question}} </div>

    [{"vars":[{"varval":"At a market stall, lemonade is sold in 250 mL cups. If 5 litres of lemonade is sold in total, how many cups of lemonade were sold?"},{"varval":"sm_nogap Convert litres to mL:\n\n\r\n\r\n<div class=\"aligned\">\n\n\r\n\r\n| | |\r\n| :| - |\r\n| 5 L | = 5 $\\times$ 1000 |\r\n| | = 5000 mL|\r\n\r\n\r\n\n</div>\n\n<br>\n\r\n\r\n\nsm_nogap Convert to 250 mL cups:\n\n<div class=\"aligned\">\r\n\r\n\n| | |\r\n| :| - |\r\n| $\\dfrac{5000}{250}$ | = $\\dfrac{500}{25}$ |\r\n| | = 20|\n\n\r\n\r\n\r\n\n</div>"}]},{"vars":[{"varval":"80 cm of string is required to tie up one package to send in the mail. \n\nIf a spool contains 4 metres of string, how many packages can be sent in the mail?"},{"varval":"sm_nogap Convert metres to centimetres:\n\n\r\n\r\n<div class=\"aligned\">\n\n\r\n\r\n| | |\r\n| :| - |\r\n| 4 metres | = 4 $\\times$ 100 |\r\n| | = 400 cm|\r\n\r\n\r\n\n</div>\n\n<br>\n\r\n\r\n\nsm_nogap Number of 80cm pieces:\n\n<div class=\"aligned\">\r\n\r\n\n| | |\r\n| :| - |\r\n| $\\dfrac{400}{80}$ | = $\\dfrac{40}{8}$ |\r\n| | = 5|\n\n\r\n\r\n\r\n\n</div>"}]},{"vars":[{"varval":"Peter wants to buy 2 kilograms of bananas.\n\nIf a single banana weighs 125 grams, how many bananas will Peter need to buy?"},{"varval":"sm_nogap Convert kilograms to grams:\n\n\r\n\r\n<div class=\"aligned\">\n\n\r\n\r\n| | |\r\n| :| - |\r\n| 2 kilograms | = 2 $\\times$ 1000 |\r\n| | = 2000 grams|\r\n\r\n\r\n\n</div>\n\n<br>\n\r\n\r\n\nsm_nogap Number of bananas to purchase:\n\n<div class=\"aligned\">\r\n\r\n\n| | |\r\n| :| - |\r\n| $\\dfrac{2000}{125}$ | = $\\dfrac{4000}{250}$ |\n| | = $\\dfrac{400}{25}$ |\r\n| | = 16|\n\n\r\n\r\n\r\n\n</div>"}]},{"vars":[{"varval":"The battery of a medical device lasts for exactly 20 hours. \n\nHow many batteries are required to run the device for a 5-day period?"},{"varval":"sm_nogap Convert days to hours:\n\n\r\n\r\n<div class=\"aligned\">\n\n\r\n\r\n| | |\r\n| :| - |\r\n| 5 days | = 5 $\\times$ 24 |\r\n| | = 120 hours|\r\n\r\n\r\n\n</div>\n\n<br>\n\r\n\r\n\nsm_nogap Number of batteries required:\n\n<div class=\"aligned\">\r\n\r\n\n| | |\r\n| - :| - |\r\n| $\\dfrac{120}{20}$ | = $\\dfrac{12}{2}$ |\n| | = 6 |\n\r\n\r\n\r\n\r\n\n</div>"}]},{"vars":[{"varval":"A cross country running circuit is measured at 800 metres. \n\nA cross country running event requires competitors to run a distance of two kilometres. \n\nHow many laps of the circuit would runners need to complete?"},{"varval":"sm_nogap Convert kilometres to metres:\n\n\r\n\r\n<div class=\"aligned\">\n\n\r\n\r\n| | |\r\n| :| - |\r\n| 2 km | = 2 $\\times$ 1000 |\r\n| | = 2000 metres|\r\n\r\n\r\n\n</div>\n\n<br>\n\r\n\r\n\nsm_nogap Number of laps:\n\n<div class=\"aligned\">\r\n\r\n\n| | |\r\n| - :| - |\r\n| $\\dfrac{2000}{800}$ | = $\\dfrac{20}{8}$ |\n| | = 2.5 |\n\r\n\r\n\r\n\r\n\n</div>"}]}]

  780. RAPH - JOB #5 - Q14 Numbers 66-70

    <div class="sm_mode"> {{question}} </div>

    [{"vars":[{"varval":"Joseph had a weekly allowance of $100. In one week, he spent $40 on food, $20 on transportation and another $20 on clothes. \n\nWhat percentage of his allowance did he spend?\r\n"},{"varval":"sm_nogap Total spending\r\n\r\n\n<div class=\"aligned\">\r\n\r\n\n> > | |\r\n> > | ------- |\r\n> > | = 40 + 20 + 20 |\r\n> > | = $80 |\r\n\r\n\r\n\n</div>\n\n<br>\n\nsm_nogap $\\therefore$ Percentage of allowance spent\r\n\r\n\n<div class=\"aligned\">\r\n\r\n\n> > | |\r\n> > | ------- |\r\n> > | = $\\dfrac{80}{100} \\times 100%$ |\r\n> > | = 80% |\r\n\r\n\r\n\n</div>"}]},{"vars":[{"varval":"Lyn had 8 kg of sugar. She used 3 kg to bake cupcakes, 1 kg to make lemonade and 2 kg to make jam. \n\nWhat percentage of her total sugar did Lyn use?"},{"varval":"sm_nogap Total sugar used\r\n\r\n\n<div class=\"aligned\">\r\n\r\n\n> > | |\r\n> > | ------- |\r\n> > | = 3 + 1 + 2 |\r\n> > | = 6 kg|\r\n\r\n\r\n\n</div>\n\n<br>\n\nsm_nogap Percentage of sugar used\r\n\r\n\n<div class=\"aligned\">\r\n\r\n\n> > | |\r\n> > | ------- |\r\n> > | = $\\dfrac{6}{8} \\times 100$ |\r\n> > | = $\\dfrac{3}{4} \\times$ 100 |\n> > | = 75% |\r\n\r\n\r\n\n</div>"}]},{"vars":[{"varval":"A builder purchased 50 bags of cement to build an outdoor shed. \n\nHe used 10 bags for the footings, 15 bags for the slab and 5 bags for the path. \n\nWhat percentage of the cement bags purchased did the builder use?\r\n"},{"varval":"sm_nogap Total bags of cement used\r\n\r\n\n<div class=\"aligned\">\r\n\r\n\n> > | |\r\n> > | ------- |\r\n> > | = 10 + 15 + 5 |\r\n> > | = 30 |\r\n\r\n\r\n\n</div>\n\n<br>\n\nsm_nogap Percentage of cement bags used\r\n\r\n\n<div class=\"aligned\">\r\n\r\n\n> > | |\r\n> > | ------- |\r\n> > | = $\\dfrac{30}{50} \\times 100$ |\r\n> > | = 0.6 $\\times$ 100 |\n> > | = 60% |\r\n\r\n\r\n\n</div>"}]},{"vars":[{"varval":"Barry weighed 80 kg and started doing triathlons.\n\nHe lost 2.8 kg in the 1st month, 3 kg in the 2nd month and 2.2 kg in the 3rd month. \n\nWhat percentage of his original weight did Barry lose?"},{"varval":"sm_nogap Total kilograms lost\r\n\r\n\n<div class=\"aligned\">\r\n\r\n\n> > | |\r\n> > | ------- |\r\n> > | = 2.8 + 3 + 2.2 |\r\n> > | = 8.0 |\r\n\r\n\r\n\n</div>\n\n<br>\n\nsm_nogap Percentage of original weight lost\r\n\r\n\n<div class=\"aligned\">\r\n\r\n\n> > | |\r\n> > | ------- |\r\n> > | = $\\dfrac{8}{80} \\times 100$ |\r\n> > | = 0.1 $\\times$ 100 |\n> > | = 10% |\r\n\r\n\r\n\n</div>"}]},{"vars":[{"varval":"Maurice bought a family size pizza cut into 12 equal slices. \n\nHe ate 4 slices, his brother Barry ate 2 slices and his other brother Robin ate 3 slices. \n\nWhat percentage of Maurice’s pizza was eaten?"},{"varval":"sm_nogap Total pizza slices eaten\r\n\r\n\n<div class=\"aligned\">\r\n\r\n\n> > | |\r\n> > | ------- |\r\n> > | = 4 + 2 + 3 |\r\n> > | = 9 |\r\n\r\n\r\n\n</div>\n\n<br>\n\nsm_nogap Percentage of pizza eaten\r\n\r\n\n<div class=\"aligned\">\r\n\r\n\n> > | |\r\n> > | ------- |\r\n> > | = $\\dfrac{9}{12} \\times 100$ |\n> > | = $\\dfrac{3}{4} \\times 100$ |\r\n> > | = 75% |\r\n\r\n\r\n\n</div>"}]}]

  781. RAPH - JOB #5 - Q13 Numbers 61-65

    <div class="sm_mode"> {{question}} </div>

    [{"vars":[{"varval":"10 ÷ 2 + 3 × 3 ="},{"varval":"Use MDAS rule (solve the different operations in order of Multiplication, Division, Addition and Subtraction):\n\nsm_nogap 10 ÷ 2 + 3 × 3 \n\n<div class=\"aligned\">\n\n> > | |\n> > | ----- |\n> > | = 5 + 9 |\n> > | = {{{correctAnswer}}} |\n\n</div>\n"}]},{"vars":[{"varval":"18 ÷ 6 + 3 × 2 ="},{"varval":"Use MDAS rule (solve the different operations in order of Multiplication, Division, Addition and Subtraction):\n\nsm_nogap 18 ÷ 6 + 3 × 2 \n\n<div class=\"aligned\">\n\n> > | |\n> > | ----- |\n> > | = 3 + 6 |\n> > | = {{{correctAnswer}}} |\n\n</div>"}]},{"vars":[{"varval":"5 × 5 + 5 ÷ 5 =\r\n"},{"varval":"Use MDAS rule (solve the different operations in order of Multiplication, Division, Addition and Subtraction):\n\nsm_nogap 5 × 5 + 5 ÷ 5 \n\n<div class=\"aligned\">\n\n> > | |\n> > | ----- |\n> > | = 25 + 1 |\n> > | = {{{correctAnswer}}} |\n\n</div>"}]},{"vars":[{"varval":"8 + 4 ÷ 2 × 2 ="},{"varval":"Use MDAS rule (solve the different operations in order of Multiplication, Division, Addition and Subtraction):\n\nsm_nogap 8 + 4 ÷ 2 × 2 \n\n<div class=\"aligned\">\n\n> > | |\n> > | ----- |\n> > | = 8 + 2 × 2 |\n> > | = 8 + 4 |\n> > | = {{{correctAnswer}}} |\n\n</div>"}]},{"vars":[{"varval":"18 $-$ 6 × 6 ÷ 3 ="},{"varval":"Use MDAS rule (solve the different operations in order of Multiplication, Division, Addition and Subtraction):\n\nsm_nogap 18 - 6 × 6 ÷ 3 \n\n<div class=\"aligned\">\n\n> > | |\n> > | ----- |\n> > | = 18 $-$ 36 ÷ 3 |\n> > | = 18 $-$ 12 |\n> > | = {{{correctAnswer}}} |\n\n</div>"}]}]

  782. RAPH - JOB #5 - Q11 Numbers 51-55 **Corrected spelling of "octogon" to octagon

    <div class="sm_mode"> {{question}} </div>

    [{"vars":[{"varval":"Identify the pair of shapes below where both have at least one pair of parallel sides?\r\n"},{"varval":"A parallelogram has 2 sets of parallel sides.\n\nA trapezium has 1 set of parallel sides.\r\n"}]},{"vars":[{"varval":"Identify the pair of shapes below where both have at least one pair of parallel sides?"},{"varval":"A regular hexagon has 3 sets of parallel sides.\n\n\r\nA trapezium has 1 set of parallel sides.\r\n"}]},{"vars":[{"varval":"Identify the pair of shapes below where both have at least one pair of parallel sides?"},{"varval":"A regular hexagon has 3 sets of parallel sides.\n\nA rhombus has 2 sets of parallel sides.\r\n\r\n"}]},{"vars":[{"varval":"Identify the pair of shapes below where both have at least one pair of parallel sides?"},{"varval":"A trapezium has 1 set of parallel sides.\n\nA regular octagon has 4 sets of parallel sides."}]},{"vars":[{"varval":"Identify the pair of shapes below where both have at least one pair of parallel sides?\r\n"},{"varval":"A regular hexagon has 3 sets of parallel sides.\n\nA rhombus has 2 sets of parallel sides. \n\r"}]}]

  783. RAPH - JOB #5 - Q10 Numbers 46-50 Added categories. Corrected var4 solution. Triangle A and B solutions were in wrong place.

    <div class="sm_mode"> {{{question}}} </div>

    [{"vars":[{"varval":"Which of the triangles listed in the table below are right-angled?\n\n<br>\n\n<div class=\"outline\">\n\n> > | | Side 1 | Side 2 | Side 3 |\n> > | ---------- | ------ | ------ | ------ |\n> > | Triangle A | 6 cm | 8 cm | 10 cm |\n> > | Triangle B | 8 cm | 10 cm | 12 cm |\n\n</div>"},{"varval":"sm_nogap Use Pythagoras to verify if each triangle is right-angled:\r\n\nsm_nogap Triangle A\n\n<div class=\"aligned\">\n\n| | | |\n|--| -- | -- |\n| $6^2$ + $8^2$ | = $10^2$ | |\n| 36 + 64 | = 100 | $\\checkmark$ |\n\n</div>\n\n\r\n\nsm_nogap Triangle B\n\n<div class=\"aligned\">\n\n| | | |\n|--| -- | -- |\n| $8^2$ + $10^2$ | = $12^2$ | |\n| 64 + 100 | = 144 | $\\Chi$ |\n\n</div>\n\n<br>\n\n$\\therefore$ Only Triangle A is right-angled."}]},{"vars":[{"varval":"Which of the triangles listed in the table below are right-angled?\n\n<br>\n\n<div class=\"outline\">\n\n> > | | Side 1 | Side 2 | Side 3 |\n> > | ---------- | ------ | ------ | ------ |\n> > | Triangle A | 1 cm | 3 cm | 2 cm |\n> > | Triangle B | 3 cm | 5 cm | 4 cm |\n\n</div>\r\n"},{"varval":"Use Pythagoras to verify if each triangle is right-angled:\r\n\nsm_nogap Triangle A\n\n<div class=\"aligned\">\n\n| | | |\n|-- :| -- | -- |\n| $1^2$ + $2^2$ | = $3^2$ | |\n| 1 + 4 | = 9 | $\\Chi$ |\n\n</div>\n\n\r\n\nsm_nogap Triangle B\n\n<div class=\"aligned\">\n\n| | | |\n|-- :| -- | -- |\n| $3^2$ + $4^2$ | = $5^2$ | |\n| 9 + 16 | = 25 | $\\checkmark$ |\n\n</div>\n\n<br>\n\n$\\therefore$ Only Triangle B is right-angled."}]},{"vars":[{"varval":"Which of the triangles listed in the table below are right-angled?\n\n<br>\n\n<div class=\"outline\">\n\n> > | | Side 1 | Side 2 | Side 3 |\n> > | ---------- | ------ | ------ | ------ |\n> > | Triangle A | 2 cm | 5 cm | 10 cm |\n> > | Triangle B | 4 cm | 10 cm | 12 cm |\n\n</div>\r\n\r"},{"varval":"Use Pythagoras to verify if each triangle is right-angled:\r\n\nsm_nogap Triangle A\n\n<div class=\"aligned\">\n\n| | | |\n|-- :| -- | -- |\n| $2^2$ + $5^2$ | = $10^2$ | |\n| 4 + 25 | = 100 | $\\Chi$ |\n\n</div>\n\n\r\n\nsm_nogap Triangle B\n\n<div class=\"aligned\">\n\n| | | |\n|-- :| -- | -- |\n| $4^2$ + $10^2$ | = $12^2$ | |\n| 16 + 100 | = 144 | $\\Chi$ |\n\n</div>\n\n<br>\n\n$\\therefore$ Neither are right-angled."}]},{"vars":[{"varval":"Which of the triangles listed in the table below are right-angled?\n\n<br>\n\n<div class=\"outline\">\n\n> > | | Side 1 | Side 2 | Side 3 |\n> > | ---------- | ------ | ------ | ------ |\n> > | Triangle A | 2 cm | 7 cm | 5 cm |\n> > | Triangle B | 6 cm | 10 cm | 8 cm |\n\n</div>\r\n"},{"varval":"Use Pythagoras to verify if each triangle is right-angled:\n\nsm_nogap Triangle A\n\n<div class=\"aligned\">\n\n| | | |\n|-- :| -- | -- |\n| $2^2$ + $5^2$ | = $7^2$ | |\n| 4 + 25 | = 49 | $\\Chi$ |\n\n</div>\n\n\r\n\nsm_nogap Triangle B\n\n<div class=\"aligned\">\n\n| | | |\n|-- :| -- | -- |\n| $6^2$ + $8^2$ | = $10^2$ | |\n| 36 + 64 | = 100 | $\\checkmark$ |\n\n</div>\n\n<br>\n\n$\\therefore$ Triangle B only is right-angled."}]},{"vars":[{"varval":"Which of the triangles listed in the table below are right-angled?\n\n<br>\n\n<div class=\"outline\">\n\n> > | | Side 1 | Side 2 | Side 3 |\n> > | ---------- | ------ | ------ | ------ |\n> > | Triangle A | 5 cm | 3 cm | 4 cm |\n> > | Triangle B | 8 cm | 6 cm | 10 cm |\n\n</div>\r\n"},{"varval":"Use Pythagoras to verify if each triangle is right-angled:\n\nsm_nogap Triangle A\n\n<div class=\"aligned\">\n\n| | | |\n|-- :| -- | -- |\n| $3^2$ + $4^2$ | = $5^2$ | |\n| 9 + 16 | = 25 | $\\checkmark$ |\n\n</div>\n\n\n\r\n\nsm_nogap Triangle B\n\n<div class=\"aligned\">\n\n| | | |\n|-- :| -- | -- |\n| $6^2$ + $8^2$ | = $10^2$ | |\n| 36 + 64 | = 100 | $\\checkmark$ |\n\n</div>\n\n\n<br>\n\n$\\therefore$ Both triangles are right-angled.\r\n"}]}]

  784. RAPH - JOB #5 - Q9 Numbers 41-45

    <div class="sm_mode"> {{part1}} </div>

    [{"vars":[{"varval":"How many faces does a rectangular prism have?"},{"varval":"sm_img https://teacher.smartermaths.com.au/wp-content/uploads/2021/03/RAPH-Q9_var0.svg 250 indent3 vpad\n\nA rectangular prism has 6 faces.\r\n\r"}]},{"vars":[{"varval":"How many faces does a hexagonal prism have?"},{"varval":"sm_img https://teacher.smartermaths.com.au/wp-content/uploads/2021/03/RAPH-Q9_var1.svg 250 indent3 vpad\n\nA hexagonal prism has 8 faces.\r"}]},{"vars":[{"varval":"How many faces does a pentagonal pyramid have?"},{"varval":"sm_img https://teacher.smartermaths.com.au/wp-content/uploads/2021/03/RAPH-Q9_var2.svg 250 indent3 vpad\n\nA pentagonal pyramid has 1 pentagonal base and 5 triangular faces.\r\n\n$\\therefore$ It has a total of 6 faces.\r\n"}]},{"vars":[{"varval":"How many faces does a hexagonal pyramid have?"},{"varval":"sm_img https://teacher.smartermaths.com.au/wp-content/uploads/2021/03/RAPH-Q9_var3.svg 250 indent3 vpad\n\nA hexagonal pyramid has 1 hexagonal base and 6 triangular faces.\r\n\r\n\n$\\therefore$ It has a total of 7 faces.\r\n"}]},{"vars":[{"varval":"How many faces does a square pyramid have?\r\n"},{"varval":"sm_img https://teacher.smartermaths.com.au/wp-content/uploads/2021/03/RAPH-Q9_var4.svg 250 indent3 vpad\n\nA square pyramid has 1 square base and 4 triangular faces.\r\n\r\n\n$\\therefore$ It has a total of 5 faces.\r\n"}]}]

  785. RAPH - JOB #5 - Q12 Numbers 56-60

    <div class="sm_mode"> {{question}} </div>

    [{"vars":[{"varval":"Adam has 15 boxes.\n\nEach box can hold 16 apples.\r\n\nWhat is the greatest number of apples that can be packed into these boxes?\r\n"},{"varval":"sm_nogap One strategy:\n\n<div class=\"aligned\">\n\n| | |\n|:|: ------------------------------------------- |\n| 15 $\\times$ 16 | \\= 15 $\\times$ 10 + 15 $\\times$ 6|\n| | = 150 + 90|\n| | \\= {{correctAnswer}} |\n\n</div>\r\n"}]},{"vars":[{"varval":"12 buses are booked for a school excursion.\r\n\nEach bus can take 16 students.\r\n\nWhat is the greatest number of students that can catch these buses for the school excursion?\r\n"},{"varval":"sm_nogap One strategy:\n\n<div class=\"aligned\">\n\n| | |\n|:|: ------------------------------------------- |\n| 12 $\\times$ 16 | \\= 12 $\\times$ 10 + 12 $\\times$ 6|\n| | = 120 + 72|\n| | \\= {{correctAnswer}} |\n\n</div>\r\n"}]},{"vars":[{"varval":"A resort has 12 conference rooms.\r\n\nEach room can accommodate 18 people.\r\n\nWhat is the greatest number of people that can be accommodated in these rooms?\r\n"},{"varval":"sm_nogap One strategy:\n\n<div class=\"aligned\">\n\n| | |\n|:|: ------------------------------------------- |\n| 12 $\\times$ 18 | \\= 12 $\\times$ 10 + 12 $\\times$ 8|\n| | = 120 + 96|\n| | \\= {{correctAnswer}} |\n\n</div>\n\n"}]},{"vars":[{"varval":"There are 14 water coolers placed around an office building.\r\n\nEach water cooler can store 15 litres of water.\r\n\nWhat is the maximum volume of water, in litres, that can be kept in these coolers?\r\n"},{"varval":"sm_nogap One strategy:\n\n<div class=\"aligned\">\n\n| | |\n|:|: ------------------------------------------- |\n| 14 $\\times$ 15 | \\= 14 $\\times$ 10 + 14 $\\times$ 5|\n| | = 140 + 70|\n| | \\= {{correctAnswer}} |\n\n</div>\r\n"}]},{"vars":[{"varval":"There are 14 vegetable rows in a garden bed.\n\n\r\nEach row can fit 22 carrot plants.\n\n\r\nWhat is the greatest number of carrots that can be planted in the garden bed?\r\n"},{"varval":"sm_nogap One strategy:\n\n<div class=\"aligned\">\n\n| | |\n|:|: ------------------------------------------- |\n| 14 $\\times$ 22 | \\= 14 $\\times$ 20 + 14 $\\times$ 2|\n| | = 280 + 28|\n| | \\= {{correctAnswer}} |\n\n</div>\r\n"}]}]

  786. RAPH - JOB #5 - Q8 Numbers 36-40

    <div class="sm_mode"> {{image1}} {{part2}} </div>

    [{"vars":[{"varval":"sm_img https://teacher.smartermaths.com.au/wp-content/uploads/2021/03/RAPH-Q8_var0.svg 350 indent3 vpad"},{"varval":"Expressed as a decimal, what is the total value of these coins as a proportion of $1.00?"},{"varval":"sm_nogap Total value of coins\n\n<div class=\"aligned\">\r\n\r\n\n> > | |\r\n> > | ------- |\r\n> > | = 0.50 + 0.20 + 0.10 |\n> > | = $0.80 |\n\r\n\r\n\r\n\n</div>\n\n<br>\n\n$\\therefore$ Total as a proportion of $1.00 = 0.8"}]},{"vars":[{"varval":"sm_img https://teacher.smartermaths.com.au/wp-content/uploads/2021/03/RAPH-Q8_var1.svg 400 indent3 vpad"},{"varval":"Expressed as a decimal, what is the total value of these coins as a proportion of $1.00?\r\n"},{"varval":"sm_nogap Total value of coins\n\n<div class=\"aligned\">\r\n\r\n\n> > | |\r\n> > | ------- |\r\n> > | = 0.20 + 0.20 + 0.10 + 0.05 |\n> > | = $0.55 |\n\r\n\r\n\r\n\n</div>\n\n<br>\n\n$\\therefore$ Total as a proportion of $1.00 = 0.55"}]},{"vars":[{"varval":"sm_img https://teacher.smartermaths.com.au/wp-content/uploads/2021/03/RAPH-Q8_var2.svg 350 indent3 vpad"},{"varval":"Expressed as a decimal, what is the total value of these coins as a proportion of $1.00?\r\n"},{"varval":"sm_nogap Total value of coins\n\n<div class=\"aligned\">\r\n\r\n\n> > | |\r\n> > | ------- |\r\n> > | = 0.20 + 0.20 + 0.50 |\n> > | = $0.90 |\n\r\n\r\n\r\n\n</div>\n\n<br>\n\n$\\therefore$ Total value as a proportion of $1.00 = 0.9"}]},{"vars":[{"varval":"sm_img https://teacher.smartermaths.com.au/wp-content/uploads/2021/03/RAPH-Q8_var3.svg 450 indent3 vpad"},{"varval":"Expressed as a decimal, what is the total value of these coins as a proportion of $1.00?\r\n"},{"varval":"sm_nogap Total value of coins\n\n<div class=\"aligned\">\r\n\r\n\n> > | |\r\n> > | ------- |\r\n> > | = 0.20 + 0.05 + 0.05 + 0.10 |\n> > | = $0.40|\n\r\n</div>\n\n<br>$\\therefore$ Total value as a proportion of $1.00 = {{{correctAnswer}}}"}]},{"vars":[{"varval":"sm_img https://teacher.smartermaths.com.au/wp-content/uploads/2021/03/RAPH-Q8_var4.svg 350 indent3 vpad"},{"varval":"Expressed as a decimal, what is the total value of these coins as a proportion of $1.00?"},{"varval":"sm_nogap Total value of coins\n\n<div class=\"aligned\">\r\n\r\n\n> > | |\r\n> > | ------- |\r\n> > | = 5 + 5 + 5 + 20 + 50 |\n> > | = 85 cents |\n> > | = $0.85 |\n\r\n\r\n\r\n\n</div>\n\n<br>\n\n$\\therefore$ Total as a proportion of $1.00 = 0.85"}]}]

  787. RAPH - JOB #5 - Q7 Numbers 31-35

    <div class="sm_mode"> {{part1}} </div>

    [{"vars":[{"varval":"What is the sum of $\\dfrac{1}{2}$ and $\\dfrac{2}{5}$ ?"},{"varval":"sm_nogap Establish a common denominator:\n\n<div class=\"aligned\">\r\n\r\n\n| |\r\n| ------- |\r\n| $\\dfrac{1}{2} \\times \\dfrac{5}{5}$ = $\\dfrac{5}{10}$|\n| |\n| $\\dfrac{2}{5} \\times \\dfrac{2}{2}$ = $\\dfrac{4}{10}$|\r\n\r\n\r\n\n</div>\n\n<br>\n\nsm_nogap Adding the fractions:\n\n<div class=\"aligned\">\r\n\r\n\n| |\r\n| ------- |\r\n| $\\dfrac{5}{10} + \\dfrac{4}{10}$ = $\\dfrac{9}{10}$|\n\r\n\r\n\r\n\n</div>"}]},{"vars":[{"varval":"What is the sum of $\\dfrac{1}{4}$ and $\\dfrac{2}{3}$?"},{"varval":"sm_nogap Establish a common denominator:\n\n<div class=\"aligned\">\r\n\r\n\n| |\r\n| ------- |\r\n| $\\dfrac{1}{4} \\times \\dfrac{3}{3}$ = $\\dfrac{3}{12}$|\n| |\n| $\\dfrac{2}{3} \\times \\dfrac{4}{4}$ = $\\dfrac{8}{12}$|\r\n\r\n\r\n\n</div>\n\n<br>\n\nsm_nogap Adding the fractions:\n\n<div class=\"aligned\">\r\n\r\n\n| |\r\n| ------- |\r\n| $\\dfrac{3}{12} + \\dfrac{8}{12}$ = $\\dfrac{11}{12}$|\n\r\n\r\n\r\n\n</div>"}]},{"vars":[{"varval":"What is the sum of $\\dfrac{3}{4}$ and $\\dfrac{1}{5}$ ?"},{"varval":"sm_nogap Establish a common denominator:\n\n<div class=\"aligned\">\r\n\r\n\n| |\r\n| ------- |\r\n| $\\dfrac{3}{4} \\times \\dfrac{5}{5}$ = $\\dfrac{15}{20}$|\n| |\n| $\\dfrac{1}{5} \\times \\dfrac{4}{4}$ = $\\dfrac{4}{20}$|\r\n\r\n</div>\n\n<br>\n\nsm_nogap Adding the fractions:\n\n<div class=\"aligned\">\r\n\r\n\n| |\r\n| ------- |\r\n| $\\dfrac{15}{20} + \\dfrac{4}{20}$ = $\\dfrac{19}{20}$|\n\r\n\r\n\r\n\n</div>"}]},{"vars":[{"varval":"What is the sum of $\\dfrac{1}{6}$ and $\\dfrac{1}{3}$?"},{"varval":"sm_nogap Establish a common denominator:\n\n<div class=\"aligned\">\r\n\r\n\n| |\r\n| ------- |\r\n| $\\dfrac{1}{3} \\times \\dfrac{2}{2}$ = $\\dfrac{2}{6}$|\n\r\n</div>\n\n<br>\n\nsm_nogap Then add the fractions:\n\n<div class=\"aligned\">\r\n\r\n\n| |\r\n| ------- |\r\n| $\\dfrac{1}{6} + \\dfrac{2}{6}$ = $\\dfrac{3}{6}$ = $\\dfrac{1}{2}$|\n\r\n\r\n\r\n\n</div>"}]},{"vars":[{"varval":"What is the sum of $\\dfrac{3}{8}$ and $\\dfrac{1}{4}$?"},{"varval":"sm_nogap Establish a common denominator:\n\n<div class=\"aligned\">\r\n\r\n\n| |\r\n| ------- |\r\n| $\\dfrac{1}{4} \\times \\dfrac{2}{2}$ = $\\dfrac{2}{8}$|\n\r\n</div>\n\n<br>\n\nsm_nogap Adding the fractions:\n\n<div class=\"aligned\">\r\n\r\n\n| |\r\n| ------- |\r\n| $\\dfrac{3}{8} + \\dfrac{2}{8}$ = $\\dfrac{5}{8}$|\n\r\n\r\n\r\n\n</div>"}]}]

  788. RAPH - JOB #5 - Q6 Numbers 26-30

    <div class="sm_mode"> {{part1}} {{image1}} {{part2}} </div>

    [{"vars":[{"varval":"The bar chart below shows the average cost of 1 gigabite (GB) of mobile data in 25 countries in 2020.\r\n"},{"varval":"sm_img https://teacher.smartermaths.com.au/wp-content/uploads/2021/03/RAPH-Q6_var0-1.svg 770 indent vpad"},{"varval":"In how many countries is the cost of 1 GB of mobile data less than 10% of the average cost of all the countries?"},{"varval":"<div class=\"aligned\">\r\n\r\n| | |\r\n| -------: | -- |\r\n| 10% of the average cost | = 6.64 $\\times$ 0.10 |\r\n| | = $0.664 per GB|\n\n<br>\n\n\r\n$\\therefore$ There are 6 countries (Indonesia, China, Vietnam, Russia, Italy and India) where 1 GB of mobile data costs less than $0.66 per GB.\n\n\r\n\r\n</div>"}]},{"vars":[{"varval":"An international human rights group researched the average minimum wage per hour across numerous countries.\r\n\nThe results are recorded in the bar chart below."},{"varval":"sm_img https://teacher.smartermaths.com.au/wp-content/uploads/2021/03/RAPH-Q6_var1.svg 600 indent vpad"},{"varval":"In how many countries is the minimum wage per hour more than 50% greater than the average minimum wage?"},{"varval":"<div class=\"aligned\">\r\n\r\n\n| | |\r\n| -------: | -- |\r\n| 150% of average wage | = 5.23 + 5.23 $\\times$ 0.5 |\n| | = 5.23 + 2.615 |\r\n| | = 7.845 |\n\n</div>\n\n<br>\n\n\r\n$\\therefore$ There are 5 countries (Netherlands, France, Ireland, Luxembourg and Australia) with an average wage higher than $7.85.\n\n\r\n\r\n"}]},{"vars":[{"varval":"A think tank's research gathered data on the average minimum wage per hour across a number of countries.\n\nThe results are recorded in the bar chart below."},{"varval":"sm_img https://teacher.smartermaths.com.au/wp-content/uploads/2021/03/RAPH-Q6_var2.svg 600 indent3 vpad"},{"varval":"In how many countries is the minimum wage per hour less than 50% of the average minimum wage?"},{"varval":"<div class=\"aligned\">\r\n\r\n| | |\r\n| -------: | -- |\r\n| 50% of average cost | = 5.23 $\\times$ 0.50 |\r\n| | = $2.615 |\n\n<br>\n\n\r\n$\\therefore$ There are 5 countries (Hungary, Estonia, Chile, Latvia and Mexico) that have a minimum wage less than $2.62.\n\n\r\n\r\n</div>"}]},{"vars":[{"varval":"An insurance company researched the average life expectancy in a number of countries.\n\nThe results are recorded in the bar chart below."},{"varval":"sm_img https://teacher.smartermaths.com.au/wp-content/uploads/2021/03/RAPH-Q6_var3.svg 770 indent vpad"},{"varval":"In how many countries is the life expectancy more than 1% greater than the average of the listed countries?"},{"varval":"<div class=\"aligned\">\r\n\r\n\n| | |\r\n| -------: | -- |\r\n| 101% of average life | = 82.6 + 82.6 $\\times$ 0.01 |\n| | = 82.6 + 0.826|\r\n| | = 83.426 |\n\n</div>\n\n<br>\n\n\r\n$\\therefore$ There is only 1 country (Japan) that has life expectancy greater than 83.426.\n\n\r\n\r"}]},{"vars":[{"varval":"The Bill Gates Foundation is researching the 2020 population of a number of selected countries.\n\nThe results are recorded in the bar chart below."},{"varval":"sm_img https://teacher.smartermaths.com.au/wp-content/uploads/2021/03/RAPH-Q6_var4.svg 350 indent3 vpad"},{"varval":"In how many countries is the population more than 50% greater than the average of the selected countries?"},{"varval":"sm_nogap Calculating in millions for simplicity:\n\n\n<div class=\"aligned\">\r\n\r\n\n| | |\r\n| -------: | -- |\r\n| 150% of average population | $\\approx$ 147 + 147 $\\times$ 0.5 |\n| | $\\approx$ 147 + 73.5 |\r\n| | $\\approx$ 220.5 million |\n\n</div>\n\n<br>\n\n\r\n$\\therefore$ There are 3 countries (United States, Indonesia and Pakistan) that have a population more than 220.5 million.\n\n\r\n\r\n"}]}]

  789. RAPH - JOB #5 - Q5 Numbers 21-25

    <div class="sm_mode"> {{part1}} {{{table1}}} <br> {{part2}} </div>

    [{"vars":[{"varval":"Penelope conducts a survey and records the height of 25 Year 8 students in the table below.\n"},{"varval":"<div class=\"outline\">\n\n> > | Height (in cm) | Frequency | Cumulative Frequency |\n> > | :------------: | :---------: | :------------------: |\n> > | 100-109 | 4 | 4 |\n> > | 110-119 | 5 | |\n> > | 120-129 | 3 | |\n> > | 130-139 | 6 | |\n> > | 140-149 | 2 | |\n> > | 150-159 | 5 | |\n\n</div>"},{"varval":"What are the missing numbers from the Cumulative Frequency column?"},{"varval":"sm_nogap Cumulative frequency on any row equals the frequency on the same row added to the previous cumulative frequency on the row above.\n\n<div class=\"aligned\">\n\n> > | |\n> > | ------- |\n> > | 4 + 5 = 9 |\n> > | 9 + 3 = 12 |\n> > | 12 + 6 = 18 |\n> > | 18 + 2 = 20 |\n> > | 20 + 5 = 25 |\n\n</div>"}]},{"vars":[{"varval":"Roger did a survey and recorded the weights of 30 kindergarten students in the table below."},{"varval":"<div class=\"outline\">\n\n> > | Weight (in kg) | Frequency | Cumulative Frequency |\n> > | :------------: | :---------: | :--------------------: |\n> > | 31-35 | 8 | 8 |\n> > | 36-40 | 14 | |\n> > | 41-45 | 5 | |\n> > | 46-50 | 3 | |\n\n</div>"},{"varval":"What are the missing numbers from the Cumulative Frequency column?"},{"varval":"sm_nogap Cumulative frequency on any row equals the frequency on the same row added to the previous cumulative frequency on the row above.\r\n\r\n<div class=\"aligned\">\r\n\r\n> > | |\r\n> > | ------- |\r\n> > | 8 + 14 = 22 |\r\n> > | 22 + 5 = 27 |\r\n> > | 27 + 3 = 30 |\r\n\n\r\n\r\n</div>"}]},{"vars":[{"varval":"Jennifer was an official in the long jump event at a State championships athletic carnival. \n\nShe recorded the length of the first jump of 20 athletes in the table below."},{"varval":"<div class=\"outline\">\r\n\r\n> > | Distance (in ft) | Frequency | Cumulative Frequency |\r\n> > | :------------: | :---------: | :--------------------: |\r\n> > | 19-20 | 2 | 2 |\r\n> > | 21-22 | 5 | |\r\n> > | 23-24 | 6 | |\r\n> > | 25-26 | 4 | |\n> > | 27-28 | 3 | |\r\n\r\n</div>\r\n"},{"varval":"What are the missing numbers from the Cumulative Frequency column?"},{"varval":"sm_nogap Cumulative frequency on any row equals the frequency on the same row added to the previous cumulative frequency on the row above.\r\n\r\n<div class=\"aligned\">\r\n\r\n> > | |\r\n> > | ------- |\r\n> > | 2 + 5 = 7 |\r\n> > | 7 + 6 = 13 |\r\n> > | 13 + 4 = 17 |\r\n> > | 17 + 3 = 20 |\r\n\r\n\r\n</div>"}]},{"vars":[{"varval":"A dentist recorded the ages of 35 people who visited her practice and recorded the results in the table below."},{"varval":"<div class=\"outline\">\r\n\r\n> > | Age | Frequency | Cumulative Frequency |\r\n> > | :------------: | :---------: | :--------------------: |\r\n> > | 0-9 | 10 | 10 |\r\n> > | 10-19 | 7 | |\r\n> > | 20-29 | 6 | |\r\n> > | 30-39 | 9 | |\n> > | 40-49 | 3 | |\r\n\r\n</div>\r\n"},{"varval":"What are the missing numbers from the Cumulative Frequency column?\r\n"},{"varval":"sm_nogap Cumulative frequency on any row equals the frequency on the same row added to the previous cumulative frequency on the row above.\r\n\r\n<div class=\"aligned\">\r\n\r\n> > | |\r\n> > | ------- |\r\n> > | 10 + 7 = 17 |\r\n> > | 17 + 6 = 23 |\r\n> > | 23 + 9 = 32 |\r\n> > | 32 + 3 = 35 |\r\n\n\r\n\r\n</div>"}]},{"vars":[{"varval":"Burnie conducts a survey and asks 24 people what their daily wage is, in Australian dollars.\n\nHe records the results in the table below."},{"varval":"<div class=\"outline\">\r\n\r\n> > | Daily Wage (in $) | Frequency | Cumulative Frequency |\r\n> > | :-----------: | :---------: | :--------------------: |\r\n> > | 201-250 | 5 | 5 |\r\n> > | 251-300 | 6 | |\r\n> > | 301-350 | 8 | |\r\n> > | 351-400 | 3 | |\n> > | 401-450 | 2 | |\r\n\r\n</div>\r\n"},{"varval":"What are the missing numbers from the Cumulative Frequency column?\r\n"},{"varval":"sm_nogap Cumulative frequency on any row equals the frequency on the same row added to the previous cumulative frequency on the row above.\r\n\r\n<div class=\"aligned\">\r\n\r\n> > | |\r\n> > | ------- |\r\n> > | 5 + 6 = 11 |\r\n> > | 11 + 8 = 19 |\r\n> > | 19 + 3 = 22 |\r\n> > | 22 + 2 = 24 |\r\n\n\r\n\r\n</div>"}]}]

  790. RAPH - JOB #5 - Q4

    <div class="sm_mode"> {{part1}} {{image1}} {{part2}} </div>

    [{"vars":[{"varval":"sm_img https://teacher.smartermaths.com.au/wp-content/uploads/2021/03/RAPH-Q4_var0.svg 350 indent3 vpad"},{"varval":"The image shows a wrapped chocolate candy being measured with a ruler. "},{"varval":"What is the distance from end to end, in centimetres, of the wrapped chocolate candy?"},{"varval":"sm_nogap Subtract the two measurement readings\n\n<div class=\"aligned\">\n\n> > | |\n> > | ------------------------------------------- |\n> > | 5.7 – 1.2 = 4.5 cm|\n\n\n</div>"}]},{"vars":[{"varval":"sm_img https://teacher.smartermaths.com.au/wp-content/uploads/2021/03/RAPH-Q4_var1.svg 350 indent3 vpad"},{"varval":"The image shows the width of a tomato being measured with a ruler. "},{"varval":"What is the width, in centimetres, of the tomato?"},{"varval":"sm_nogap Subtract the two measurement readings\n\n<div class=\"aligned\">\n\n> > | |\n> > | ------------------------------------------- |\n> > | 6.6 – 2.4 = 4.2 cm|\n\n\n</div>"}]},{"vars":[{"varval":"sm_img https://teacher.smartermaths.com.au/wp-content/uploads/2021/03/RAPH-Q4_var2.svg 350 indent3 vpad"},{"varval":"The width of a sand shoe is measured with a ruler. "},{"varval":"What is the length of the sand shoe, in centimetres?"},{"varval":"sm_nogap Subtract the two measurement readings\n\n<div class=\"aligned\">\n\n> > | |\n> > | ------------------------------------------- |\n> > | 10.1 – 1.6 = 8.5 cm|\n\n\n</div>"}]},{"vars":[{"varval":"sm_img https://teacher.smartermaths.com.au/wp-content/uploads/2021/03/RAPH-Q4_var3.svg 350 indent3 vpad"},{"varval":"A ruler is used to measure the width of a beach ball."},{"varval":"What is the width, in centimetres, of the beach ball?"},{"varval":"sm_nogap Subtract the two measurement readings\n\n<div class=\"aligned\">\n\n> > | |\n> > | ------------------------------------------- |\n> > | 9.5 – 2.7 = 6.8 cm|\n\n\n</div>"}]},{"vars":[{"varval":"sm_img https://teacher.smartermaths.com.au/wp-content/uploads/2021/03/RAPH-Q4_var4.svg 350 indent3 vpad"},{"varval":"A ruler is used to measure the length of a crayon. "},{"varval":"What is the length, in centimetres, of the crayon?"},{"varval":"sm_nogap Subtract the two measurement readings\n\n<div class=\"aligned\">\n\n> > | |\n> > | ------------------------------------------- |\n> > | 8.5 – 3.0 = 5.5 cm|\n\n\n</div>"}]}]

  791. RAPH - JOB #5 - Q2

    <div class="sm_mode"> {{part1}} <br> {{image1}} {{part2}} </div>

    [{"vars":[{"varval":"Jessica’s hobby is making flower pots and she records the number of flower pots she makes every month. \n\nThe graph below shows the number of flower pots she made over two years."},{"varval":"sm_img https://teacher.smartermaths.com.au/wp-content/uploads/2021/03/RAPH-Q2_var0.svg 600 indent vpad"},{"varval":"What was the difference in the total numbers of flower pots she made in 2014 and 2015?"},{"varval":"sm_nogap Total number of pots made in 2014\n\n<div class=\"aligned\">\n\n> > | |\n> > | ------------------------------------------- |\n> > | = 4 + 6 + 5 + 7 + 3 + 7 + 8 + 2 + 9 + 4 + 6 + 5 |\n> > | = 66|\n\n</div>\n\n<br>\n\nsm_nogap Total number of pots made in 2015\n\n<div class=\"aligned\">\n\n> > | |\n> > | ------------------------------------------- |\n> > | = 8 + 2 + 1 + 9 + 4 + 6 + 8 + 5 + 2 + 1 + 7 + 9|\n> > | = 62 |\n\n</div>\n\n<br>\n\n<div class=\"aligned\">\n\n| | |\n| --------------------- | ------------------------------------------- |\n| $\\therefore$ Difference| \\= 66 $–$ 62|\n| | \\= 4 |\n\n</div>"}]},{"vars":[{"varval":"Gary is a chef and grows his own fruit.\n\nIn September, he picked fruit from two trees every third day and recorded the information in the graph below."},{"varval":"sm_img https://teacher.smartermaths.com.au/wp-content/uploads/2021/03/RAPH-Q2_var1.svg 660 indent vpad"},{"varval":"What was the difference in the total number of pieces of fruit picked from each tree?"},{"varval":"sm_nogap Total fruit picked from Tree A\r\n\r\n<div class=\"aligned\">\r\n\r\n> > | |\r\n> > | ------------------------------------------- |\r\n> > | = 4 + 1 + 3 + 4 + 2 + 5 + 5 + 7 + 6 + 8 |\n> > | = 45 |\n\n\r\n\r\n</div>\n\n<br>\n\n\r\n\r\nsm_nogap Total fruit picked from Tree B\n\n\r\n\r\n<div class=\"aligned\">\r\n\r\n> > | |\r\n> > | ------------------------------------------- |\r\n> > | = 5 + 6 + 3 + 5 + 7 + 2 + 8 + 4 + 1 + 9|\n> > | = 50 |\n\n\r\n\r\n<br>\n\n</div>\n\n\r\n\r\n<div class=\"aligned\">\r\n\r\n| | |\r\n| --------------------- | ------------------------------------------- |\r\n| $\\therefore$ Difference | = 50 $-$ 45|\r\n| | = {{{correctAnswer}}} |\r\n\r\n</div>"}]},{"vars":[{"varval":"Amy saves money every week. The graph shows her savings over two months."},{"varval":"sm_img https://teacher.smartermaths.com.au/wp-content/uploads/2021/03/RAPH-Q2_var2.svg 585 indent vpad"},{"varval":"What was the difference in her total savings in May and June?"},{"varval":"sm_nogap Total savings in May\n\n\r\n\r\n<div class=\"aligned\">\n\n\r\n\r\n> > | |\r\n> > | ------------------------------------------- |\r\n> > | = 150 + 350 + 200 + 100|\n> > | = $800 |\n\r\n\r\n\n</div>\n\n<br>\n\n\r\n\r\nsm_nogap Total savings in June\n\n\r\n\r\n<div class=\"aligned\">\r\n\r\n> > | |\r\n> > | ------------------------------------------- |\r\n> > | = 250 + 300 + 150 + 200|\n> > | = $900 |\n\r\n\r\n\n</div>\n\n<br>\n\n\r\n\r\n<div class=\"aligned\">\n\n\r\n\r\n| | |\r\n| --------------------- | ------------------------------------------- |\r\n| $\\therefore$ Difference | \\= 900 $-$ 800|\r\n| | \\= $100 |\r\n\r\n\n</div>"}]},{"vars":[{"varval":"In a basketball game, Zach and Edward got the highest scores for their respective teams. \n\nThe graph shows their individual scores in each quarter."},{"varval":"sm_img https://teacher.smartermaths.com.au/wp-content/uploads/2021/03/RAPH-Q2_var3.svg 600 indent vpad"},{"varval":"What was the difference in the total score of the two players?"},{"varval":"sm_nogap Zach's total score\n\n\r\n\r\n<div class=\"aligned\">\n\n\r\n\r\n> > | |\r\n> > | ------------------------------------------- |\r\n> > | = 10 + 8 + 4 + 12 |\n> > | = 34 |\n\n\r\n\r\n</div>\n\n<br>\n\n\r\n\r\nsm_nogap Edward's total score\n\n\r\n\r\n<div class=\"aligned\">\r\n\r\n> > | |\r\n> > | ------------------------------------------- |\r\n> > | = 2 + 16 + 6 + 12|\n> > | = 36 |\n\n\r\n\r\n</div>\n\n<br>\n\n\r\n\r\n<div class=\"aligned\">\r\n\r\n| | |\r\n| --------------------- | ------------------------------------------- |\r\n| $\\therefore$ Difference | \\= 36 $–$ 34|\r\n| | \\= 2 |\r\n\r\n\n</div>"}]},{"vars":[{"varval":"The graph below shows the number of people who joined a Sports Club and a Music Club over a 12 month period."},{"varval":"sm_img https://teacher.smartermaths.com.au/wp-content/uploads/2021/03/RAPH-Q2_var4.svg 600 indent vpad"},{"varval":"What was the difference in the total number of people that joined each club?"},{"varval":"sm_nogap Total that joined Sports Club\n\n\r\n\r\n<div class=\"aligned\">\n\n\r\n\r\n> > | |\r\n> > | ------------------------------------------- |\r\n> > | = 3 + 7 + 8 + 9 + 2 + 2 + 3 + 1 + 4 + 6 + 5 + 2|\n> > | = 52 |\n\n\r\n\r\n</div>\n\n<br>\n\n\r\n\r\nsm_nogap Total that joined Music Club\n\n\r\n\r\n<div class=\"aligned\">\n\n\r\n\r\n> > | |\r\n> > | ------------------------------------------- |\r\n> > | = 6 + 5 + 3 + 9 + 4 + 2 + 3 + 3 + 5 + 7 + 2 + 1|\n> > | = 50 |\n\n\n\r\n\r\n</div>\n\n<br>\n\n\r\n\r\n<div class=\"aligned\">\r\n\r\n| | |\r\n| --------------------- | ------------------------------------------- |\r\n| $\\therefore$ Difference | \\= 52 $–$ 50 |\r\n| | \\= 2 |\r\n\r\n\n</div>"}]}]

  792. RAPH - JOB #5 - Q3

    <div class="sm_mode"> {{part1}} {{image1}} {{part2}} </div>

    [{"vars":[{"varval":"sm_img https://teacher.smartermaths.com.au/wp-content/uploads/2021/03/RAPH-Q3_var0.svg 350 indent3 vpad"},{"varval":"In the following figures, all angles are 90$\\degree$."},{"varval":"For which figures is the length of the perimeter the same?"},{"varval":"Calculate the perimeter of each figure\n\n<div class=\"aligned\">\r\n\r\n| | |\r\n| --------------------- | ------------------------------------------- |\r\n| $$ \\text{Perimeter I} $$ | \\= 4+2+2+4+4+2+2+4 |\r\n| | \\= 24 |\r\n\r\n</div>\n\n<br>\n\n<div class=\"aligned\">\r\n\r\n| | |\r\n| --------------------- | ------------------------------------------- |\r\n| $$ \\text{Perimeter II} $$ | \\= 6+6+6+2+2+2+2+2 |\r\n| | \\= 28 |\r\n\r\n</div>\n\n<br>\n\n<div class=\"aligned\">\r\n\r\n| | |\r\n| --------------------- | ------------------------------------------- |\r\n| $$ \\text{Perimeter III} $$ | \\= 6+6+6+6 |\r\n| | \\= 24 |\r\n\r\n</div>\n\n<br>\n\n$\\therefore$ Figures I and III have the same perimeter\n\n"}]},{"vars":[{"varval":"sm_img https://teacher.smartermaths.com.au/wp-content/uploads/2021/03/RAPH-Q3_var1.svg 350 indent3 vpad"},{"varval":"In the following figures, all angles are 90$\\degree$."},{"varval":"For which figures is the length of the perimeter the same?"},{"varval":"Calculate the perimeter of each figure\n\n<div class=\"aligned\">\r\n\r\n| | |\r\n| --------------------- | ------------------------------------------- |\r\n| $$ \\text{Perimeter I} $$ | \\= 2+2+2+2+2+2+2+2+2+2+2+2 |\r\n| | \\= 24 |\r\n\r\n</div>\n\n<br>\n\n<div class=\"aligned\">\r\n\r\n| | |\r\n| --------------------- | ------------------------------------------- |\r\n| $$ \\text{Perimeter II} $$ | \\= 6+6+6+6 |\r\n| | \\= 24 |\r\n\r\n</div>\n\n<br>\n\n<div class=\"aligned\">\r\n\r\n| | |\r\n| --------------------- | ------------------------------------------- |\r\n| $$ \\text{Perimeter III} $$ | \\= 6+2+2+2+2+2+6+2+2+2+2+2 |\r\n| | \\= 32 |\r\n\r\n</div>\n\n<br>\n\n$\\therefore$ Figures I and II have the same perimeter\r\n\n\n"}]},{"vars":[{"varval":"sm_img https://teacher.smartermaths.com.au/wp-content/uploads/2021/03/RAPH-Q3_var2.svg 350 indent3 vpad"},{"varval":"In the following figures, all angles are 90$\\degree$."},{"varval":"For which figures is the length of the perimeter the same?"},{"varval":"Calculate the perimeter of each figure\n\n<div class=\"aligned\">\r\n\r\n| | |\r\n| --------------------- | ------------------------------------------- |\r\n| $$ \\text{Perimeter I} $$ | \\= 6+6+6+6 |\r\n| | \\= 24 |\r\n\r\n</div>\n\n<br>\n\n<div class=\"aligned\">\r\n\r\n| | |\r\n| --------------------- | ------------------------------------------- |\r\n| $$ \\text{Perimeter II} $$ | \\= 6+6+2+2+2+2+2+2 |\r\n| | \\= 24 |\r\n\r\n</div>\n\n<br>\n\n<div class=\"aligned\">\r\n\r\n| | |\r\n| --------------------- | ------------------------------------------- |\r\n| $$ \\text{Perimeter III} $$ | \\= 4+4+2+2+4+4+2+2 |\r\n| | \\= 24 |\r\n\r\n</div>\n\n<br>\n\n$\\therefore$ All the figures have the same perimeter\r\n\n\n"}]},{"vars":[{"varval":"sm_img https://teacher.smartermaths.com.au/wp-content/uploads/2021/03/RAPH-Q3_var3.svg 350 indent3 vpad"},{"varval":"In the following figures, all angles are 90$\\degree$."},{"varval":"For which figures is the length of the perimeter the same?"},{"varval":"Calculate the perimeter of each figure\n\n<div class=\"aligned\">\r\n\r\n| | |\r\n| --------------------- | ------------------------------------------- |\r\n| $$ \\text{Perimeter I} $$ | \\= 4+4+2+2+4+2+2+4 |\r\n| | \\= 24 |\r\n\r\n</div>\n\n<br>\n\n<div class=\"aligned\">\r\n\r\n| | |\r\n| --------------------- | ------------------------------------------- |\r\n| $$ \\text{Perimeter II} $$ | \\= 2+4+4+2+6+6 |\r\n| | \\= 24 |\r\n\r\n</div>\n\n<br>\n\n<div class=\"aligned\">\r\n\r\n| | |\r\n| --------------------- | ------------------------------------------- |\r\n| $$ \\text{Perimeter III} $$ | \\= 6+6+6+6 |\r\n| | \\= 24 |\r\n\r\n</div>\n\n<br>\n\n$\\therefore$ All the figures have the same perimeter\r\n\n\n"}]},{"vars":[{"varval":"sm_img https://teacher.smartermaths.com.au/wp-content/uploads/2021/03/RAPH-Q3_var4.svg 350 indent3 vpad"},{"varval":"In the following figures, all angles are 90$\\degree$."},{"varval":"For which figures is the length of the perimeter the same?"},{"varval":"Calculate the perimeter of each figure\n\n<div class=\"aligned\">\r\n\r\n| | |\r\n| --------------------- | ------------------------------------------- |\r\n| $$ \\text{Perimeter I} $$ | \\= 6+6+6+6 |\r\n| | \\= 24 |\r\n\r\n</div>\n\n<br>\n\n<div class=\"aligned\">\r\n\r\n| | |\r\n| --------------------- | ------------------------------------------- |\r\n| $$ \\text{Perimeter II} $$ | \\= 4+2+2+4+2+2+2+2+2+2 |\r\n| | \\= 24 |\r\n\r\n</div>\n\n<br>\n\n<div class=\"aligned\">\r\n\r\n| | |\r\n| --------------------- | ------------------------------------------- |\r\n| $$ \\text{Perimeter III} $$ | \\= 6+6+6+2+4+2+4+2 |\r\n| | \\= 32 |\r\n\r\n</div>\n\n<br>\n\n$\\therefore$ Figures I and II have the same perimeter\r\n\n\n"}]}]

  793. RAPH - JOB #5 - Q1

    <div class="sm_mode"> {{part1}} <br> {{image1}} {{part2}} </div>

    [{"vars":[{"varval":"A vegetable garden measures 2 metres by 1.5 metres.\n\nJim plants rows of lettuces in the garden, making sure there is a 20 cm gap between the garden edge and a plant, and 40 cm between each plant."},{"varval":"sm_img https://teacher.smartermaths.com.au/wp-content/uploads/2021/03/RAPH-Q1_var0-1.svg 350 indent3 vpad"},{"varval":"sm_nogap What is the maximum number of lettuces Jim can plant?"},{"varval":"sm_nogap Remove the 20 cm gaps from the garden edges:\n\n<div class=\"aligned\">\n\n| | |\n| ------- :| ----------------------------------- |\n| Effective length| = $200 - 20 - 20$ = 160 cm |\n| Effective width | = $150 - 20 - 20$ = 110 cm |\n\n</div>\n\n\n\nsm_nogap Rows\n\n<div class=\"aligned\">\n\n| |\n| ------------------------------------------------------------------ |\n| $\\dfrac{160}{40} = 4 \\Rightarrow 5 \\ \\text{rows}$ |\n\n\n\nsm_nogap Columns\n\n| |\n| ---------------------------------------------------------------------- |\n| $\\dfrac{110}{40} = 2+ \\Rightarrow$ 3 columns |\n\n</div>\n\n<br>\n\n<div class=\"aligned\">\n\n| | |\n| ------------------------------- | --------------- |\n| Number of lettuces | = 5 $\\times$ 3 |\n| | = 15 |\n\n</div>"}]},{"vars":[{"varval":"A vegetable garden measures 3 metres by 1.5 metres.\n\nSinead plants rows of sweet potatoes in the garden, making sure there is a 30 cm gap between the garden edge and a plant, and 20 cm between each plant."},{"varval":"\nsm_img https://teacher.smartermaths.com.au/wp-content/uploads/2021/03/RAPH-Q1_var1.svg 350 indent3 vpad"},{"varval":"What is the maximum number of sweet potatoes that Sinead can plant?"},{"varval":"sm_nogap Remove the 30 cm gaps from the garden edges:\n\n<div class=\"aligned\">\n\n| | |\n| ------- :| ----------------------------------- |\n| Effective length| = $300 - 30 - 30$ = 240 cm |\n| Effective width | = $150 - 30 - 30$ = 90 cm |\n\n</div>\n\n<br>\n\nsm_nogap Rows\n\n<div class=\"aligned\">\n\n| |\n| ------------------------------------------------------------------ |\n| $\\dfrac{240}{20} = 12 \\Rightarrow 13 \\ \\text{rows}$ |\n\n\n\nsm_nogap Columns\n\n| |\n| ---------------------------------------------------------------------- |\n| $\\dfrac{90}{20} = 4.5 \\Rightarrow$ 5 columns |\n\n</div>\n\n<br>\n\n<div class=\"aligned\">\n\n| | |\n| ------------------------------- | --------------- |\n| Number of sweet potatoes | = 13 $\\times$ 5 |\n| | = 65 |\n\n</div>"}]},{"vars":[{"varval":"A vegetable garden measures 3 metres by 1 metres.\n\nMarley plants rows of celeriac in the garden, making sure there is a 15 cm gap between the garden edge and a plant, and 30 cm between each plant."},{"varval":"sm_img https://teacher.smartermaths.com.au/wp-content/uploads/2021/03/RAPH-Q1_var2.svg 350 indent3 vpad"},{"varval":"What is the maximum number of celeriac plants that Marley can fit in the vegetable garden?"},{"varval":"sm_nogap Remove the 15 cm gaps from the garden edges:\n\n<div class=\"aligned\">\n\n| | |\n| ------- :| ----------------------------------- |\n| Effective length| = $300 - 15 - 15$ = 270 cm |\n| Effective width | = $100 - 15 - 15$ = 70 cm |\n\n</div>\n\n<br>\n\nsm_nogap Rows\n\n<div class=\"aligned\">\n\n| |\n| ------------------------------------------------------------------ |\n| $\\dfrac{270}{30} = 9 \\Rightarrow 10 \\ \\text{rows}$ |\n\n\n\nsm_nogap Columns\n\n| |\n| ---------------------------------------------------------------------- |\n| $\\dfrac{70}{30} = 2+ \\Rightarrow$ 3 columns |\n\n</div>\n\n<br>\n\n<div class=\"aligned\">\n\n| | |\n| ------------------------------- | --------------- |\n| Number of celeriac plants | = 10 $\\times$ 3 |\n| | = 30 |\n\n</div>"}]},{"vars":[{"varval":"A vegetable garden measures 2 metres by 1.5 metres.\n\nBjork plants rows of beetroot in the garden, making sure there is a 10 cm gap between the garden edge and a plant, and 30 cm between each plant."},{"varval":"sm_img https://teacher.smartermaths.com.au/wp-content/uploads/2021/03/RAPH-Q1_var3.svg 350 indent3 vpad"},{"varval":"What is the maximum number of beetroot plants that Bjork can plant?"},{"varval":"sm_nogap Remove the 10 cm gaps from the garden edges:\n\n<div class=\"aligned\">\n\n| | |\n| ------- :| ----------------------------------- |\n| Effective length| = $200 - 10 - 10$ = 180 cm |\n| Effective width | = $150 - 10 - 10$ = 130 cm |\n\n</div>\n\n<br>\n\n\nsm_nogap Rows\n\n<div class=\"aligned\">\n\n| |\n| ------------------------------------------------------------------ |\n| $\\dfrac{180}{30} = 6 \\Rightarrow 7 \\ \\text{rows}$ |\n\n\n\nsm_nogap Columns\n\n| |\n| ---------------------------------------------------------------------- |\n| $\\dfrac{130}{30} = 4+ \\Rightarrow$ 5 columns |\n\n</div>\n\n<br>\n\n<div class=\"aligned\">\n\n| | |\n| ------------------------------- | --------------- |\n| Number of beetroot plants | = 7 $\\times$ 5 |\n| | = 35 |\n\n</div>\n\r\n"}]},{"vars":[{"varval":"A vegetable garden measures 3 metres by 1 metre.\n\nBeau plants rows of garlic in the garden, making sure there is a 15 cm gap between the garden edge and a plant, and 20 cm between each plant."},{"varval":"sm_img https://teacher.smartermaths.com.au/wp-content/uploads/2021/03/RAPH-Q1_var4.svg 350 indent3 vpad"},{"varval":"What is the maximum number of garlic plants that Beau can fit in the vegetable garden?"},{"varval":"sm_nogap Remove the 15 cm gaps from the garden edges:\n\n<div class=\"aligned\">\n\n| | |\n| ------- :| ----------------------------------- |\n| Effective length| = $300 - 15 - 15$ = 270 cm |\n| Effective width | = $100 - 15 - 15$ = 70 cm |\n\n</div>\n\n<br>\n\n\nsm_nogap Rows\n\n<div class=\"aligned\">\n\n| |\n| ------------------------------------------------------------------ |\n| $\\dfrac{270}{20} = 13.5 \\Rightarrow 14 \\ \\text{rows}$ |\n\n\n\nsm_nogap Columns\n\n| |\n| ---------------------------------------------------------------------- |\n| $\\dfrac{70}{20} = 3.5 \\Rightarrow$ 4 columns |\n\n</div>\n\n<br>\n\n<div class=\"aligned\">\n\n| | |\n| ------------------------------- | --------------- |\n| Number of garlic plants | = 14 $\\times$ 4 |\n| | = 56 |\n\n</div>\n\r\n\n\r\n"}]}]

  794. NAP9-A-02

    <div class="sm_mode"> At {{time1}} the temperature in {{place}} was {{temp1}}$\degree C$ At {{time2}} it was {{temp2}} $\degree C$ warmer. At {{time3}} it was {{temp3}} cooler than at {{time2}}. \ What was the temperature at {{time3}}? </div>

    [{"vars":[{"varval":"6 am"},{"varval":"San Diego"},{"varval":"12.7"},{"varval":"2 pm"},{"varval":"7.5"},{"varval":"8 pm"},{"varval":"9.6"}]},{"vars":[{"varval":"4:30 am"},{"varval":"Kurri Kurri"},{"varval":"8.6"},{"varval":"11 am"},{"varval":"13.7"},{"varval":"7 pm"},{"varval":"14.8"}]},{"vars":[{"varval":"5 am"},{"varval":"Fitzroy"},{"varval":"5.8"},{"varval":"1 pm"},{"varval":"14.3"},{"varval":"6 pm"},{"varval":"12.8"}]}]

  795. NAP9-A-01 Added categories.

    <div class="sm_mode"> The digital clock below tells the time on the 24-hour system. {{image}} What is the equivalent time in the 12-hour system? </div>

    [{"vars":[{"varval":"\n\nsm_img https://teacher.smartermaths.com.au/wp-content/uploads/2021/02/NAP9-A-01_var0.svg 200 indent3 vpad"},{"varval":"1225"}]},{"vars":[{"varval":"sm_img https://teacher.smartermaths.com.au/wp-content/uploads/2021/02/NAP9-A-01_var1.svg 200 indent3 vpad"},{"varval":"1175"}]},{"vars":[{"varval":"\n\nsm_img https://teacher.smartermaths.com.au/wp-content/uploads/2021/02/NAP9-A-01_var2.svg 200 indent3 vpad"},{"varval":"851"}]}]

  796. S003

    <div class="sm_mode"> A bag of marbles contain only {{color1}} and {{color2}} marbles. <br> {{image}} <br>What fraction of the total marbles are {{color3}}? </div>

    [{"vars":[{"varval":"white"},{"varval":"blue"},{"varval":"sm_img https://teacher.smartermaths.com.au/wp-content/uploads/2021/02/S003_VAR0.svg 200 indent3 vpad"},{"varval":"blue"},{"varval":"\\text{blue marbles}"}]},{"vars":[{"varval":"red"},{"varval":"grey"},{"varval":"sm_img https://teacher.smartermaths.com.au/wp-content/uploads/2021/02/S003_VAR1.svg 200 indent3 vpad"},{"varval":"red"},{"varval":"\\text{red marbles}"}]},{"vars":[{"varval":"green"},{"varval":"grey"},{"varval":"sm_img https://teacher.smartermaths.com.au/wp-content/uploads/2021/02/S003_VAR2.svg 200 indent3 vpad"},{"varval":"green"},{"varval":"\\text{green marbles}"}]},{"vars":[{"varval":"grey"},{"varval":"orange"},{"varval":"sm_img https://teacher.smartermaths.com.au/wp-content/uploads/2021/02/S003_VAR3.svg 200 indent3 vpad"},{"varval":"orange"},{"varval":"\\text{orange marbles}"}]},{"vars":[{"varval":"red"},{"varval":"green"},{"varval":"sm_img https://teacher.smartermaths.com.au/wp-content/uploads/2021/02/S003_VAR4.svg 200 indent3 vpad"},{"varval":"green"},{"varval":"\\text{green marbles}"}]}]

  797. S002

    <div class="sm_mode"> Which number is exactly halfway between {{frac1}} and {{frac2}} ? </div>

    [{"vars":[{"varval":"$2 \\dfrac{1}{4}$"},{"varval":"$4 \\dfrac{3}{4}$"},{"varval":"7"}]},{"vars":[{"varval":"$3\\dfrac{1}{2}$"},{"varval":"6"},{"varval":"$9\\dfrac{1}{2}$"}]},{"vars":[{"varval":"$2\\dfrac{2}{3}$"},{"varval":"$5\\dfrac{1}{3}$"},{"varval":"8"}]},{"vars":[{"varval":"$1\\ \\dfrac{1}{3}$"},{"varval":"$4\\ \\dfrac{2}{3}$"},{"varval":"6"}]}]

  798. S001

    <div class="sm_mode"> {{name1}} has {{objects1}}. He gives {{frac}} of them to his {{name2}}. How many {{objects2}} does {{who}} </div>

    [{"vars":[{"varval":"Peter"},{"varval":"16 marbles"},{"varval":"$\\dfrac{1}{4}$"},{"varval":"friend Roger"},{"varval":"marbles"},{"varval":"Peter have left?"},{"varval":"$\\dfrac{1}{4} \\times 16 = 4$\n\n<div class=\"aligned\">\n\n|||\n|-:|-|\n|$\\therefore \\text{Marbles left}$|= 16 $-$ 4|\n||= 12|\n\n</div>\n\n\n"}]},{"vars":[{"varval":"Andrew"},{"varval":"caught 12 fish"},{"varval":"$\\dfrac{1}{3}$"},{"varval":"friend Kelly"},{"varval":"fish"},{"varval":"Kelly receive?"},{"varval":"$\\dfrac{1}{3} \\times12 = 4$\n\n$\\therefore \\text{Kelly has 4 fish}$\n"}]},{"vars":[{"varval":"Eli"},{"varval":"24 goldfish"},{"varval":"$\\dfrac{1}{4}$"},{"varval":"brother Zac"},{"varval":"goldfish"},{"varval":"Zac get from Eli?"},{"varval":"$\\dfrac{1}{4} \\times 24 =6$\n\n$\\therefore \\text{Zac gets 6 goldfish}$"}]},{"vars":[{"varval":"Callen"},{"varval":"21 one-dollar coins"},{"varval":"$\\dfrac{1}{3}$"},{"varval":"friend Mia"},{"varval":"one-dollar coins"},{"varval":"Calllen have left?"},{"varval":"$\\dfrac{1}{3} \\times 21 = 7$\n\n<div class=\"aligned\">\n\n|||\n|-:|-|\n|$\\therefore \\text{Coins left}$|= 21 $-$ 7|\n||= 14|\n\n</div>\n"}]},{"vars":[{"varval":"Lindon "},{"varval":"18 pencils"},{"varval":"$\\dfrac{1}{3}$"},{"varval":"sister Molly"},{"varval":"pencils"},{"varval":"Lindon have left?"},{"varval":"$\\dfrac{1}{3} \\times 18 = 6$\n\n<div class=\"aligned\">\n\n|||\n|-:|-|\n|$\\therefore \\text{Pencils left}$|= 18 $-$ 6|\n||= 12|\n\n</div>\n"}]},{"vars":[{"varval":"Ali"},{"varval":"15 lollipops"},{"varval":"$\\dfrac{1}{3}$"},{"varval":"sister Domi"},{"varval":"lollipops"},{"varval":"his sister Domi get?"},{"varval":"$\\dfrac{1}{3} \\times 15 = 5$\n\n$\\therefore$ Domi gets 5 lollipops."}]}]

  799. Q7-8

    <div class="sm_mode"> sm_nogap An item costs {{value}} including {{percent}} shipping fee. What is the cost before the shipping fee is added? </div>

    [{"vars":null}]

  800. Solution incorrect. Added worked solution instead of just answer. Added categories.

    <div class="sm_mode"> A class of 33 students is to be divided into smaller groups. If each group is to contain 4, 5, or 6 people, what is the largest number of groups possible? </div>

    [{"vars":null}]

  801. Added worked solution and corrected 2nd sentence. Added categories.

    <div class="sm_mode"> A class of 28 students is to be divided into smaller groups. If each group is to contain 2, 3, or 4 people, what is the largest number of groups possible? </div>

    [{"vars":null}]

  802. Corrected answer from 50 to 350. Added worked solution instead of just correct answer. Added categories. Needs difficulty.

    <div class="sm_mode"> The heights of two buildings are in the ratio of 1 : 8. If together, their height is 450 metres, how much taller is the larger building, in metres? </div>

    [{"vars":null}]

  803. Corrected answer. Added worked solution. Added categories.

    <div class="sm_mode"> The weights of two trucks are in the ratio of 4 : 7. If together, they weigh 3300 tonnes, how many more tonnes does the larger truck weigh? </div>

    [{"vars":null}]

  804. Corrected wording in question.

    <div class="sm_mode"> {{{question}}} </div>

    [{"vars":[{"varval":"Of the following units, which would be the best for measuring the mass of an elephant?\n"},{"varval":"{{{correctAnswer}}}"}]},{"vars":[{"varval":"Of the following units, which would be the best for measuring the mass of a car?"},{"varval":"{{{correctAnswer}}}"}]},{"vars":[{"varval":"Of the following units, which would be the best for measuring the mass of a Year 3 student?"},{"varval":"{{{correctAnswer}}}"}]},{"vars":[{"varval":"Of the following units, which would be the best for measuring the mass of a full backpack?"},{"varval":"{{{correctAnswer}}}"}]},{"vars":[{"varval":"Of the following units, which would be the best for measuring the mass of an ant?"},{"varval":"{{{correctAnswer}}}"}]},{"vars":[{"varval":"Of the following units, which would be the best for measuring the mass of a pencil?"},{"varval":"{{{correctAnswer}}}"}]}]

  805. Corrected wording. Added categories.

    <div class="sm_mode"> {{{question}}} </div>

    [{"vars":[{"varval":"Which of the following units would be the best for measuring the distance between two cities?\n"},{"varval":"{{{correctAnswer}}}"}]},{"vars":[{"varval":"Which of the following units would be the most appropriate for measuring the length of an ant?\n"},{"varval":"{{{correctAnswer}}}"}]},{"vars":[{"varval":"Which of the following units would be the best for measuring the distance between the goal posts on a soccer field?\n"},{"varval":"{{{correctAnswer}}}"}]},{"vars":[{"varval":"Which of the following units would be the best for measuring the distance between your hand and your elbow?\n"},{"varval":"{{{correctAnswer}}}"}]},{"vars":[{"varval":"Which of the following units would be the best for measuring the distance between London and New York?\n"},{"varval":"{{{correctAnswer}}}"}]},{"vars":[{"varval":"Which of the following units would be the best for measuring the distance between Sydney and Brisbane?\n"},{"varval":"{{{correctAnswer}}}"}]}]

  806. Added categories. Added markdown table. Corrected wording in question.

    <div class="sm_mode"> What is the numeral representing 22 hundreds, 9 ones, 7 hundredths and 2 thousandths? </div>

    [{"vars":null}]

  807. Corrected Answer and wording. Added worked solution. Added categories.

    <div class="sm_mode"> Three boys start with different amounts of money. If Ryan gives Jeet $1 and Jeet gives Kevin $5, then the three boys will then have the same amount of money. If Kevin had $2 originally, how much **more** money did Jeet have than Ryan at the start? </div>

    [{"vars":null}]

  808. Corrected question and changed variables and setting out in worked solution. Added categories.

    <div class="sm_mode"> Three boys start with different amounts of money. If Joe gives Mike $5 and Mike gives Steven $7, the three boys will then have the same amount of money. If Steven had $10 originally, how much **more** money did Joe have than Mike at the start? </div>

    [{"vars":null}]

  809. Added detail to worked solution. Added categories.

    <div class="sm_mode"> Steve finds that by wearing different combinations of the jackets, shirts and pairs of trousers that he owns, he can make up 240 different outfits. If he owns 6 pairs of trousers and 8 shirts, how many jackets does he own? </div>

    [{"vars":null}]

  810. Corrected question wording and added detail to worked solution. Added categories.

    <div class="sm_mode"> Maurice finds that by wearing different combinations of the jackets, shirts and pairs of trousers that he owns, he can make up 180 different outfits. If Maurice owns 9 jackets and 2 shirts, how many pairs of trousers does he own? </div>

    [{"vars":null}]

  811. Corrected wording in question and added detail and corrected solution. Added categories. Needs difficulty.

    <div class="sm_mode"> A small patio required 16 slabs to cover it. The cost of using 12 colored slabs and 4 white ones is $68. If 8 colored slabs and 8 white ones are used instead, the cost becomes $56. sm_img https://teacher.smartermaths.com.au/wp-content/uploads/2021/01/Hamda_Q22a.svg 80 indent vpad Find the cost of the arrangement below which uses 4 colored slabs and 12 white ones. sm_img https://teacher.smartermaths.com.au/wp-content/uploads/2021/01/Hamda_Q22b.svg 80 indent vpad </div>

    [{"vars":null}]

  812. #21

    <div class="sm_mode"> A small patio required 16 slabs to cover it. The cost of using 4 colored slabs and 12 blank ones is $20. If 8 colored slabs and 8 blank ones are used instead, the cost becomes $24. Find the cost of this arrangement which is made up by using 2 colored slabs and 10 blank ones. </div>

    [{"vars":null}]

  813. #20 Added 4 variants. Corrected Var1 answer. Added categories and staging_suejones to var0, 2, 3, 4 (accidentally moved original var0 to var1) Needs difficulty.

    <div class="sm_mode"> {{{question}}} </div>

    [{"vars":[{"varval":"Molly's weekly salary is $45 less than Jim's, whose weekly salary is $70 more than Artie's. If Artie earns $375 per week, how much in total do they earn per week?"},{"varval":"<div class=\"aligned\">\n\n|||\n|-|-|\n|Total wages|= Artie + Jim + Molly|\n||= 375 + (375 + 70) + (375 + 70 $-$ 45)|\n||= 375 + 445 + 400|\n||= {{{correctAnswer}}}|\n\n</div>"}]},{"vars":[{"varval":"Nancy's weekly salary is $70 less than David's, whose weekly salary is $30 more than Sam's. If Sam earns $490 per week, how much in total do they earn per week?"},{"varval":"<div class=\"aligned\">\n\n|||\n|-|-|\n|Total wages|= Sam + David + Nancy|\n||= 490 + (490 + 30) + (490 + 30 $-$ 70)|\n||= 490 + 520 + 450|\n||= {{{correctAnswer}}}|\n\n</div>"}]},{"vars":[{"varval":"Mavis's weekly salary is $105 less than Pete's, whose weekly salary is $172 more than Leonardo's. If Leonardo earns $1003 per week, how much in total do they earn per week?"},{"varval":"<div class=\"aligned\">\n\n|||\n|-|-|\n|Total wages|= Leonardo + Pete + Mavis|\n||= 1003 + (1003 + 172) + (1003 + 172 $-$ 105)|\n||= 1003 + 1175 + 1070|\n||= {{{correctAnswer}}}|\n\n</div>"}]},{"vars":[{"varval":"Bligh's weekly salary is $99 less than Burke's, whose weekly salary is $43 more than Brenda's. If Brenda earns $567 per week, how much in total do they earn per week?"},{"varval":"<div class=\"aligned\">\n\n|||\n|-|-|\n|Total wages|= Brenda + Burke + Bligh|\n||= 567 + (567 + 43) + (567 + 43 $-$ 99)|\n||= 567 + 610 + 511|\n||= {{{correctAnswer}}}|\n\n</div>"}]},{"vars":[{"varval":"Fern's weekly salary is $240 less than Sage's, whose weekly salary is $116 more than Summer's. If Summer earns $876 per week, how much in total do they earn per week?"},{"varval":"<div class=\"aligned\">\n\n|||\n|-|-|\n|Total wages|= Summer + Sage + Fern|\n||= 876 + (876 + 116) + (876 + 116 $-$ 240)|\n||= 876 + 992 + 752|\n||= {{{correctAnswer}}}|\n\n</div>"}]}]

  814. #19 Added categories

    <div class="sm_mode"> Jane's weekly salary is $50 less than Taylor's, whose weekly salary is $80 more than Katy's. If Katy earns $310 per week, how much in total do they earn per week? </div>

    [{"vars":null}]

  815. #18

    <div class="sm_mode"> In the figure below, square ABCD has area 25 cm$^2$ square CEFG has area 49 cm$^2$ and square GHDI has area 16 cm$^2$. What is the area triangle DCI? sm_img https://teacher.smartermaths.com.au/wp-content/uploads/2021/01/Hamda_Qn.svg 200 indent vpad </div>

    [{"vars":null}]

  816. Corrected diagram - no "F" in original and size of yellow square was 25 sq metres which could not have been the case with the dimensions of the other squares. Added categories.

    <div class="sm_mode"> In the figure below, square ABCD has area 64 cm$^2$ square DEFG has area 25 cm$^2$ and square AHIJ has area 100 cm$^2$. What is the area triangle AGF? sm_img https://teacher.smartermaths.com.au/wp-content/uploads/2022/11/Measurement_70036_v0_a.svg 300 indent vpad </div>

    [{"vars":null}]

  817. Added diagram and wrote question to match. Added categories.

    <div class="sm_mode"> In the figure below, square ABCD has area 100 cm$^2$ square EFGB has area 36 cm$^2$ and square FHIC has area 196 cm$^2$. What is the area triangle BEC? sm_img https://teacher.smartermaths.com.au/wp-content/uploads/2022/11/Measurement_70035_v0.svg 480 indent vpad </div>

    [{"vars":null}]

  818. <div class="sm_mode"> On a scaled map, a distance of 4 centimetres represents 5 kilometres. If the distance between two sets of traffic lights is 250 metres, what is the distance on the map, in centimetres? </div>

    [{"vars":null}]

  819. <div class="sm_mode"> On a scaled map, a distance of 1 centimetre represents 2 kilometres. If the distance between two restaurants is 600 metres, what is the distance on the map, in centimetres? </div>

    [{"vars":null}]

  820. Duplicate of 700032. Changed numbers - now a different question. Added categories.

    <div class="sm_mode"> {{{question}}} </div>

    [{"vars":[{"varval":"If the average of 4, 10, 20 and $\\large n$ is 19, then 4 + 10 + 20 + $\\large n$ = ?"},{"varval":"If the average of 4, 10, 20 and $\\large n$ is 19 then\n\n<div class=\"aligned\">\n\n>>|||\n|-|-|\n|$\\dfrac{4 + 10 + 20 + \\large n }{4}$|= 19|\n|4 + 10 + 20 + $\\large n$|= 19 $\\times$ 4|\n\n</div>\n\n>><br>$\\therefore$ 4 + 10 + 20 + $\\large n$ = {{{correctAnswer}}}"}]},{"vars":[{"varval":"If the average of 7, 11, 18 and $\\large w$ is 17, then 7 + 11 + 18 + $\\large w$ = ?"},{"varval":"If the average of 7, 11, 18 and $\\large w$ is 17 then\n\n<div class=\"aligned\">\n\n>>|||\n|-|-|\n|$\\dfrac{7 + 11 + 18 + \\large w}{4}$|= 17|\n|7 + 11 + 18 + $\\large w$|= 17 $\\times$ 4|\n\n</div>\n\n>><br>$\\therefore$ 7 + 11 + 18 + $\\large w$ = {{{correctAnswer}}}"}]},{"vars":[{"varval":"If the average of 7, 8, 15 and $\\large c$ is 13, then 7 + 8 + 15 + $\\large c$ = ?"},{"varval":"If the average of 7, 8, 15 and $\\large c$ is 13 then\n\n<div class=\"aligned\">\n\n>>|||\n|-|-|\n|$\\dfrac{7 + 8 + 15 + \\large c }{4}$|= 13|\n|7 + 8 + 15 + $\\large c$|= 13 $\\times$ 4|\n\n</div>\n\n>><br>$\\therefore$ 7 + 8 + 15 + $\\large c$ = {{{correctAnswer}}}"}]}]

  821. Duplicate of 700031. Changed numbers = different question. Added categories.

    <div class="sm_mode"> {{{question}}} </div>

    [{"vars":[{"varval":"If the average of 15, 22, 39 and $\\large y$ is 27, then 15 + 22 + 39 + $\\large y$ = ?"},{"varval":"If the average of 15, 22, 39 and $\\large y$ is 27 then\n\n<div class=\"aligned\">\n\n>|||\n|-|-|\n|$\\dfrac{15 + 22 + 39 + \\large y }{4}$|= 27|\n|15 + 22 + 39 + $\\large y$| = 27 $\\times$ 4|\n\n</div>\n\n<br>\n\n>>$\\therefore$ 15 + 22 + 39 + $\\large y$ = {{{correctAnswer}}}"}]},{"vars":[{"varval":"If the average of 11, 14, 17 and $\\large t$ is 15, then 11 + 14 + 17 + $\\large t$ = ?"},{"varval":"If the average of 11, 14, 17 and $\\large t$ is 15 then\n\n<div class=\"aligned\">\n\n>|||\n|-|-|\n|$\\dfrac{11 + 14 + 17 + \\large t }{4}$|= 15|\n|11 + 14 + 17 + $\\large t$| = 15 $\\times$ 4|\n\n</div>\n\n<br>\n\n>>$\\therefore$ 11 + 14 + 17 + $\\large t$ = {{{correctAnswer}}}"}]},{"vars":[{"varval":"If the average of 21, 24, 31 and $\\large m$ is 25, then 21 + 24 + 31 + $\\large m$ = ?"},{"varval":"If the average of 21, 24, 31 and $\\large m$ is 25 then\n\n<div class=\"aligned\">\n\n>|||\n|-|-|\n|$\\dfrac{21 + 24 + 31 + \\large m }{4}$|= 25|\n|21 + 24 + 31 + $\\large m$| = 25 $\\times$ 4|\n\n</div>\n\n<br>\n\n>>$\\therefore$ 21 + 24 + 31 + $\\large m$ = {{{correctAnswer}}}"}]}]

  822. Changed variable p to correct size in question/solution. Changed option 3 in answers from 16 to 48 as 16 was also option 1. Added categories.

    <div class="sm_mode"> {{{question}}} </div>

    [{"vars":[{"varval":"If the average of 2, 9, 15 and $\\large p$ is 16, then 2 + 9 + 15 + $\\large p$ = ?"},{"varval":"The average of 2, 9, 15 and $\\large p$ is 16 then\n\n<div class=\"aligned\">\n\n>>|||\n|-|-|\n|$\\dfrac{2 + 9 + 15 + \\large p}{4}$|= 16 |\n|2 + 9 + 15 + $\\large p$|= 16 $\\times$ 4|\n\n</div>\n\n<br>\n\n>>$\\therefore$ 2 + 9 + 15 + $\\large p$ = {{{correctAnswer}}}\n"}]},{"vars":[{"varval":"If the average of 1, 7, 9 and $\\large q$ is 8, then 1 + 7 + 9 + $\\large q$ = ?"},{"varval":"The average of 1, 7, 9 and $\\large q$ is 8 then\n\n<div class=\"aligned\">\n\n>>|||\n|-|-|\n|$\\dfrac{1 + 7 + 9 + \\large q}{4}$|= 8 |\n|1 + 7 + 9 + $\\large q$|= 8 $\\times$ 4|\n\n</div>\n\n<br>\n\n>>$\\therefore$ 1 + 7 + 9 + $\\large q$ = {{{correctAnswer}}}\n"}]},{"vars":[{"varval":"If the average of 5, 12, 19 and $\\large x$ is 16, then 5 + 12 + 19 + $\\large x$ = ?"},{"varval":"The average of 2, 9, 15 and $\\large x$ is 16 then\n\n<div class=\"aligned\">\n\n>>|||\n|-|-|\n|$\\dfrac{5 + 12 + 19 + \\large x}{4}$|= 16 |\n|5 + 12 + 19 + $\\large x$|= 16 $\\times$ 4|\n\n</div>\n\n<br>\n\n>>$\\therefore$ 5 + 12 + 19 + $\\large x$ = {{{correctAnswer}}}\n"}]}]

  823. Added categories. Changed variable "p" to correct size.

    <div class="sm_mode"> {{{question}}} </div>

    [{"vars":[{"varval":"If the average of 5, 12, 19 and $\\large p$ is 12, then 5 + 12 + 19 + $\\large p$ = ?"},{"varval":"If the average of 5, 12, 19 and $\\large p$ is 12, then \n\n<div class=\"aligned\">\n\n>>|||\n|-|-|\n|$\\dfrac{5 + 12 + 19 +\\large p }{4}$|= 12|\n|5 + 12 + 19 + $\\large p$|= 12 $\\times$ 4|\n\n</div>\n\n<br>\n\n>>$\\therefore$ 5 + 12 + 19 + $\\large p$ = {{{correctAnswer}}}\n\n"}]},{"vars":[{"varval":"If the average of 9, 17, 21 and $\\large a$ is 18, then 9 + 17 + 21 + $\\large a$ = ?"},{"varval":"If the average of 9, 17, 21 and $\\large a$ is 18, then \n\n<div class=\"aligned\">\n\n>>|||\n|-|-|\n|$\\dfrac{9 + 17 + 21 + \\large a}{4}$|= 18|\n|9 + 17 + 21 + $\\large a$|= 18 $\\times$ 4|\n\n</div>\n\n<br>\n\n>>$\\therefore$ 9 + 17 + 21 + $\\large a$ = {{{correctAnswer}}}"}]},{"vars":[{"varval":"If the average of 7, 15, 17 and $\\large q$ is 14, then 7 + 15 + 17 + $\\large q$ = ?"},{"varval":"If the average of 7, 15, 17 and $\\large q$ is 14, then \n\n<div class=\"aligned\">\n\n>>|||\n|-|-|\n|$\\dfrac{7 + 15 + 17 +\\large q }{4}$|= 14|\n|7 + 15 + 17 + $\\large q$|= 14 $\\times$ 4|\n\n</div>\n\n<br>\n\n>>$\\therefore$ 7 + 15 + 17 + $\\large q$ = {{{correctAnswer}}}"}]}]

  824. Corrected answer from 2 to 3. Added categories.

    <div class="sm_mode"> What is the greatest integer that will always evenly divide the sum of three consecutive odd numbers? </div>

    [{"vars":null}]

  825. Corrected answer from 2 to 4. Added categories. Needs difficulty.

    <div class="sm_mode"> What is the greatest integer that will always evenly divide the sum of two consecutive odd numbers? </div>

    [{"vars":null}]

  826. Q1 and Q2 Simplified the working in the worked solution in var0 and var1. Added categories. Needs difficulty.

    <div class="sm_mode"> What number could replace A in the following equation? >>>$\dfrac{ {{val1}} }{ {{val2}} \times A } = {{frac1}}$ </div>

    [{"vars":[{"varval":"36"},{"varval":"2"},{"varval":"1 \\dfrac{1}{5}"},{"varval":"18"},{"varval":"\\dfrac{6}{5}"},{"varval":"\\dfrac{3}{3}"}]},{"vars":[{"varval":"48"},{"varval":"8"},{"varval":"1 \\dfrac{1}{2}"},{"varval":"6"},{"varval":"\\dfrac{3}{2}"},{"varval":"\\dfrac{2}{2}"}]},{"vars":[{"varval":"24"},{"varval":"3"},{"varval":"1 \\dfrac{1}{3}"},{"varval":"8"},{"varval":"\\dfrac{4}{3}"},{"varval":"\\dfrac{2}{2}"}]},{"vars":[{"varval":"50"},{"varval":"5"},{"varval":"1 \\dfrac{1}{4}"},{"varval":"10"},{"varval":"\\dfrac{5}{4}"},{"varval":"\\dfrac{2}{2}"}]},{"vars":[{"varval":"64"},{"varval":"2"},{"varval":"1\\dfrac{3}{5}"},{"varval":"32"},{"varval":"\\dfrac{8}{5}"},{"varval":"\\dfrac{4}{4}"}]},{"vars":[{"varval":"108"},{"varval":"9"},{"varval":"1\\dfrac{1}{5}"},{"varval":"12"},{"varval":"\\dfrac{6}{5}"},{"varval":"\\dfrac{2}{2}"}]}]

  827. Q3 and Q4 Added categories. Don't like the solution, but it works. The many variables don't really seem to make this question any easier to set up.

    <div class="sm_mode"> sm_nogap Express ${{val1}} as a percentage of ${{val2}}. </div>

    [{"vars":[{"varval":"20"},{"varval":"500"},{"varval":"4"},{"varval":"50"},{"varval":"0.4"},{"varval":"0.04"},{"varval":"5"}]},{"vars":[{"varval":"18"},{"varval":"600"},{"varval":"3"},{"varval":"60"},{"varval":"0.3"},{"varval":"0.03"},{"varval":"6"}]}]

  828. Nearly a repeat of question: cea77c1c-befd-49d3-ab3f-ccd8dfb5af36 Added detail to worked solution. Added categories.

    <div class="sm_mode"> {{{question}}} </div>

    [{"vars":[{"varval":"Only one of the following number sentences is equal to 6. \n\nWhich one is it?\n"},{"varval":"<div class=\"aligned\">\n\n|||\n|-|-|\n|{{{correctAnswer}}}|= $8 - 16 \\div 8$|\n||= 8 - 2 (order of operations)|\n||= 6|\n\n</div>\n\n<br>\n\n$\\therefore$ Answer is {{{correctAnswer}}}"}]},{"vars":[{"varval":"Only one of the following number sentences is equal to 10. \n\nWhich one is it?\n"},{"varval":"<div class=\"aligned\">\n\n|||\n|-|-|\n|{{{correctAnswer}}}|= $8 + 16 \\div 8$|\n||= 8 + 2 (order of operations)|\n||= 10|\n\n</div>\n\n<br>\n\n$\\therefore$ Answer is {{{correctAnswer}}}"}]},{"vars":[{"varval":"Only one of the following number sentences is equal to 8. \n\nWhich one is it?\n"},{"varval":"<div class=\"aligned\">\n\n|||\n|-|-|\n|{{{correctAnswer}}}|= $6 + 12 \\div 6$|\n||= 6 + 2 (order of operations)|\n||= 8|\n\n</div>\n\n<br>\n\n$\\therefore$ Answer is {{{correctAnswer}}}"}]},{"vars":[{"varval":"Only one of the following number sentences is equal to 4. \n\nWhich one is it?\n"},{"varval":"<div class=\"aligned\">\n\n|||\n|-|-|\n|{{{correctAnswer}}}|= $6 - 12 \\div 6$|\n||= 6 - 2 (order of operations)|\n||= 4|\n\n</div>\n\n<br>\n\n$\\therefore$ Answer is {{{correctAnswer}}}"}]},{"vars":[{"varval":"Only one of the following number sentences is equal to 10. \n\nWhich one is it?\n"},{"varval":"<div class=\"aligned\">\n\n|||\n|-|-|\n|{{{correctAnswer}}}|= $(121 - 11) \\div 11$|\n||= $110 \\div 11$ (order of operations)|\n||= 10|\n\n</div>\n\n<br>\n\n$\\therefore$ Answer is {{{correctAnswer}}}"}]},{"vars":[{"varval":"Only one of the following number sentences is equal to 11. \n\nWhich one is it?\n"},{"varval":"<div class=\"aligned\">\n\n|||\n|-|-|\n|{{{correctAnswer}}}|= ($100 + 10) \\div 10$|\n||= $110 \\div 10$ (order of operations)|\n||= 11|\n\n</div>\n\n<br>\n\n$\\therefore$ Answer is {{{correctAnswer}}}"}]}]

  829. Added detail to Q and WS. Added categories.

    <div class="sm_mode"> {{{question}}} </div>

    [{"vars":[{"varval":"\nOnly one of the following number sentences is equal to 4. \n\nWhich number sentence is it?"},{"varval":"<div class=\"aligned\">\n\n|||\n|-|-|\n|$(3 \\times 3 + 3) \\div 3$|= $(9 + 3) \\div 3$|\n||= $12 \\div 3$ (order of operations)|\n||= 4|\n\n</div>\n\n<br>\n\n$\\therefore$ The correct number sentence is {{{correctAnswer}}}.\n"}]},{"vars":[{"varval":"\nOnly one of the following number sentences is equal to 5. \n\nWhich number sentence is it?"},{"varval":"<div class=\"aligned\">\n\n|||\n|-|-|\n|{{{correctAnswer}}}|= $(16 + 4) \\div 4$|\n||= $20 \\div 4$ (order of operations)|\n||= 5|\n\n</div>\n\n<br>\n\n$\\therefore$ The correct number sentence is {{{correctAnswer}}}.\n"}]},{"vars":[{"varval":"\nOnly one of the following number sentences is equal to 3. \n\nWhich number sentence is it?"},{"varval":"<div class=\"aligned\">\n\n|||\n|-|-|\n|{{{correctAnswer}}}|= $(16 - 4) \\div 4$|\n||= $12 \\div 4$ (order of operations)|\n||= 3|\n\n</div>\n\n<br>\n\n$\\therefore$ The correct number sentence is {{{correctAnswer}}}.\n"}]},{"vars":[{"varval":"\nOnly one of the following number sentences is equal to 2. \n\nWhich number sentence is it?"},{"varval":"<div class=\"aligned\">\n\n|||\n|-|-|\n|{{{correctAnswer}}}|= $(9 - 3) \\div 3$|\n||= $6 \\div 3$ (order of operations)|\n||= 2|\n\n</div>\n\n<br>\n\n$\\therefore$ The correct number sentence is {{{correctAnswer}}}.\n"}]},{"vars":[{"varval":"\nOnly one of the following number sentences is equal to 4. \n\nWhich number sentence is it?"},{"varval":"<div class=\"aligned\">\n\n|||\n|-|-|\n|{{{correctAnswer}}}|= $(25 - 5) \\div 5$|\n||= $20 \\div 5$ (order of operations)|\n||= 4|\n\n</div>\n\n<br>\n\n$\\therefore$ The correct number sentence is {{{correctAnswer}}}.\n"}]},{"vars":[{"varval":"\nOnly one of the following number sentences is equal to 6. \n\nWhich number sentence is it?"},{"varval":"<div class=\"aligned\">\n\n|||\n|-|-|\n|{{{correctAnswer}}}|= $(25 + 5) \\div 5$|\n||= $30 \\div 5$ (order of operations)|\n||= 6|\n\n</div>\n\n<br>\n\n$\\therefore$ The correct number sentence is {{{correctAnswer}}}.\n"}]}]

  830. Swapped solution 1 and 2 so easier solution was first. Added categories.

    <div class="sm_mode"> {{{question}}} </div>

    [{"vars":[{"varval":"(6 - 3)$^2$ = ?"},{"varval":"Solution 1:\n\n<div class=\"aligned\">\n\n|||\n|-|-|\n|(6 - 3)$^2$|= 3$^2$\n||= {{{correctAnswer}}}|\n\n\n</div>\n\nsm_nogap Solution 2: Advanced\n\n<div class=\"aligned\">\n\n|||\n|-|-|\n|(6 - 3)$^2$|= 6$^2 - 2 \\times 6 \\times 3 + 3^2$ |\n||= 36 - 36 + 9|\n||= {{{correctAnswer}}}|\n\n\n</div>\n"}]},{"vars":[{"varval":"(9 - 3)$^2$ = ?"},{"varval":"Solution 1:\n\n<div class=\"aligned\">\n\n|||\n|-|-|\n|(9 - 3)$^2$|= 6$^2$\n||= {{{correctAnswer}}}|\n\n\n</div>\n\nsm_nogap Solution 2: Advanced\n\n<div class=\"aligned\">\n\n|||\n|-|-|\n|(9 - 3)$^2$|= 9$^2 - 2 \\times 9 \\times 3 + 3^2$ |\n||= 81 - 54 + 9|\n||= {{{correctAnswer}}}|\n\n\n</div>\n"}]},{"vars":[{"varval":"(5 - 2)$^2$ = ?"},{"varval":"Solution 1:\n\n<div class=\"aligned\">\n\n|||\n|-|-|\n|(5 - 2)$^2$|= 3$^2$\n||= {{{correctAnswer}}}|\n\n\n</div>\n\nsm_nogap Solution 2: Advanced\n\n<div class=\"aligned\">\n\n|||\n|-|-|\n|(5 - 2)$^2$|= 5$^2 - 2 \\times 5 \\times 2 + 2^2$ |\n||= 25 - 20 + 4|\n||= {{{correctAnswer}}}|\n\n\n</div>\n"}]},{"vars":[{"varval":"(8 - 4)$^2$ = ?"},{"varval":"Solution 1:\n\n<div class=\"aligned\">\n\n|||\n|-|-|\n|(8 - 4)$^2$|= 4$^2$\n||= {{{correctAnswer}}}|\n\n\n</div>\n\nsm_nogap Solution 2: Advanced\n\n<div class=\"aligned\">\n\n|||\n|-|-|\n|(8 - 4)$^2$|= 8$^2 - 2 \\times 8 \\times 4 + 4^2$ |\n||= 64 - 64 + 16|\n||= {{{correctAnswer}}}|\n\n\n</div>\n"}]},{"vars":[{"varval":"(10 - 5)$^2$ = ?"},{"varval":"Solution 1:\n\n<div class=\"aligned\">\n\n|||\n|-|-|\n|(10 - 5)$^2$|= 5$^2$\n||= {{{correctAnswer}}}|\n\n\n</div>\n\nsm_nogap Solution 2: Advanced\n\n<div class=\"aligned\">\n\n|||\n|-|-|\n|(10 - 5)$^2$|= 10$^2 - 2 \\times 10 \\times 5 + 5^2$ |\n||= 100 - 100 + 25|\n||= {{{correctAnswer}}}|\n\n\n</div>\n"}]},{"vars":[{"varval":"(7 - 5)$^2$ = ?"},{"varval":"Solution 1:\n\n<div class=\"aligned\">\n\n|||\n|-|-|\n|(7 - 5)$^2$|= 2$^2$\n||= {{{correctAnswer}}}|\n\n\n</div>\n\nsm_nogap Solution 2: Advanced\n\n<div class=\"aligned\">\n\n|||\n|-|-|\n|(7 - 5)$^2$|= 7$^2 - 2 \\times 7 \\times 5 + 5^2$ |\n||= 49 - 70 + 25|\n||= {{{correctAnswer}}}|\n\n\n</div>\n"}]}]

  831. No question. Copied from 70021. Changed worked solution so each option is checked. Added categories.

    <div class="sm_mode"> Which of the following would result in the largest number? </div>

    [{"vars":null}]

  832. Changed question to "smallest" from "largest" to match solution. Added check all options to worked solution. Added categories.

    <div class="sm_mode"> Which of the following would result in the smallest number? </div>

    [{"vars":null}]

  833. Corrected answer and added detail to worked solution. Added categories.

    <div class="sm_mode"> The price of oil in 2009 was $50 per barrel. In 2013, the price was 200% more than in 2009. What was the price per barrel in 2013? </div>

    [{"vars":null}]

  834. Corrected worked solution and added detail. Added categories.

    <div class="sm_mode"> The price of computer A is $500 and the price of computer B is 300% more than the price of computer A. What is the price of computer B? </div>

    [{"vars":null}]

  835. Added categories. Needs difficulty.

    <div class="sm_mode"> Tina works as a designer and is normally paid $800 for a 20-hour week. For how many hours would she have to work in a week if her total pay is $1280 and all overtime is paid at the time and a half rate? </div>

    [{"vars":null}]

  836. Added categories. Needs difficulty.

    <div class="sm_mode"> Ava works as a chef and is normally paid $1200 for a 40-hour week. For how many hours would she have to work in a week if her total pay is $1425 and all overtime is paid at the time and a half rate? </div>

    [{"vars":null}]

  837. Added categories.

    <div class="sm_mode"> {{{question}}} </div>

    [{"vars":[{"varval":"Which of the following expressions is equal to the value of (10 $\\times$ 10) $\\times$ (600 $\\times$ 40)?\n"},{"varval":"In (10 $\\times$ 10) $\\times$ (600 $\\times$ 40), there are 5 zeros.\n\nTherefore, the correct choice should also have 5 zeros.\n\nCorrect answer has 5 zeros.\n\n{{{correctAnswer}}}"}]},{"vars":[{"varval":"Which of the following expressions is equal to the value of (20 $\\times$ 100) $\\times$ (7000 $\\times$ 10)?\n"},{"varval":"In (20 $\\times$ 100) $\\times$ (7000 $\\times$ 10), there are 6 zeros.\n\nTherefore, the correct choice should also have 6 zeros.\n\nCorrect answer has 6 zeros.\n\n{{{correctAnswer}}}"}]},{"vars":[{"varval":"Which of the following expressions is equal to the value of (100 $\\times$ 10) $\\times$ (800 $\\times$ 5000)?\n"},{"varval":"In (100 $\\times$ 10) $\\times$ (800 $\\times$ 5000), there are 8 zeros.\n\nTherefore, the correct choice should also have 8 zeros.\n\nCorrect answer has 8 zeros.\n\n{{{correctAnswer}}}"}]},{"vars":[{"varval":"Which of the following expressions is equal to the value of (900 $\\times$ 20) $\\times$ (100 $\\times$ 100)?\n"},{"varval":"In (900 $\\times$ 20) $\\times$ (100 $\\times$ 100), there are 7 zeros.\n\nTherefore, the correct choice should also have 7 zeros.\n\nCorrect answer has 7 zeros.\n\n{{{correctAnswer}}}"}]},{"vars":[{"varval":"Which of the following expressions is equal to the value of (10 $\\times$ 100) $\\times$ (500 $\\times$ 30)?\n"},{"varval":"In (10 $\\times$ 100) $\\times$ (500 $\\times$ 30), there are 6 zeros.\n\nTherefore, the correct choice should also have 6 zeros.\n\nCorrect answer has 6 zeros.\n\n{{{correctAnswer}}}"}]},{"vars":[{"varval":"Which of the following expressions is equal to the value of (4000 $\\times$ 500) $\\times$ (10 $\\times$ 10)?\n"},{"varval":"In (4000 $\\times$ 500) $\\times$ (10 $\\times$ 10), there are 7 zeros.\n\nTherefore, the correct choice should also have 7 zeros.\n\nCorrect answer has 7 zeros.\n\n{{{correctAnswer}}}"}]}]

  838. Same as question 9a0b0ad6-d5ba-4cca-81ec-bc02b70f46b8 Changed numbers, added categories and added difficulty.

    <div class="sm_mode"> Which of the following expressions is equal to the value of (100 $\times$ 100) $\times$ (5 $\times$ 300)? </div>

    [{"vars":null}]

  839. Corrected grammar in question. Added categories.

    <div class="sm_mode"> Peter went on a 150 kilometre drive to the lake. The graph below shows the distance driven, in kilometres, and the time, in hours, taken for the trip. sm_img https://teacher.smartermaths.com.au/wp-content/uploads/2021/01/var70014.svg 580 indent vpad What was the average speed of Peter's car during the first 6 hours? </div>

    [{"vars":null}]

  840. Added categories.

    <div class="sm_mode"> Zach is saving money every year. The graph shows how much money is in his bank account at the end of each year. sm_img https://teacher.smartermaths.com.au/wp-content/uploads/2021/01/var70013.svg 580 indent vpad What was Zach's average amount of money saved per year during the first 5 years? </div>

    [{"vars":null}]

  841. Corrected grammar in question. Added "0" to all answers eg $4.80. Added categories.

    <div class="sm_mode"> A pump delivers 5 L of water every 30 mins. What is the cost to deliver 60 L of water if the cost of the electricity to run the pump is $0.80 per hour? </div>

    [{"vars":null}]

  842. Corrected question and worked solution. Added zeros to the $ amounts in answers. Added categories. .

    <div class="sm_mode"> A truck can travel 8 kilometres on 400 mL of petrol. What is the cost of petrol to travel 440 kilometres if petrol costs $1.15 per litre? </div>

    [{"vars":null}]

  843. Corrected working in worked solution. Added categories.

    <div class="sm_mode"> The area of a rectangle is 36 cm$^2$ and the perimeter is 30 cm. What is the ratio of the width to the length? </div>

    [{"vars":null}]

  844. Corrected question. Added categories.

    <div class="sm_mode"> Carl had $\dfrac{5}{6}$ as much money as Harry. He gave $50 to Harry, who now has $140. How much money did Carl have at the beginning? </div>

    [{"vars":null}]

  845. Added categories.

    <div class="sm_mode"> Mark had $\dfrac{3}{4}$ as much money as John. He gave $70 to John, who now has $190. How much money did Mark have at the beginning? </div>

    [{"vars":null}]

  846. Replaced a, b and c with correct variables in worked solution. Added categories.

    <div class="sm_mode"> P, Q and R stand for three different numbers. The average of P and Q is 30. The average of Q and R is 45. P + Q + R = 120 Calculate the values of P, Q and R. </div>

    [{"vars":null}]

  847. <div class="sm_mode"> P, Q and R stand for three different numbers. The average of P and Q is 64. The average of Q and R is 50. P + Q + R = 150 Calculate the values of P, Q and R. </div>

    [{"vars":null}]

  848. Corrected question to match worked solution. Added categories.

    <div class="sm_mode"> 180 people attended the opening of the new theme park. The chart below shows the number of people still in the park at different times. sm_img https://teacher.smartermaths.com.au/wp-content/uploads/2021/01/var70002.svg 580 indent vpad What fraction of the people had gone home by 2 pm? </div>

    [{"vars":null}]

  849. Added categories. Needs difficulty.

    <div class="sm_mode"> 120 pies are being sold at the market fair. The chart below shows the number of pies that are left at the end of each day. sm_img https://teacher.smartermaths.com.au/wp-content/uploads/2021/01/var70001.svg 580 indent vpad What fraction of the total number of pies sold from Monday to Thursday? </div>

    [{"vars":null}]

  850. <div class="sm_mode"> {{name}} is building a {{object1}} using {{object2}} that are {{width}} centimetres wide. If the length of the fence is {{length}} metres with no gaps, which equation should {{name}} use to calculate the number of {{object3}} he will need? </div>

    [{"vars":[{"varval":"Bob"},{"varval":"fence"},{"varval":"vertical wooden planks"},{"varval":"14"},{"varval":"16.8"},{"varval":"wooden planks"},{"varval":"14 cm = 0.14 metres"}]},{"vars":[{"varval":"Keith"},{"varval":"fence"},{"varval":"vertical planks of wood"},{"varval":"18"},{"varval":"28.8"},{"varval":"planks"},{"varval":"18 cm = 0.18 metres"}]},{"vars":[{"varval":"Dan"},{"varval":"fence"},{"varval":"vertical wooden planks"},{"varval":"15"},{"varval":"19.2"},{"varval":"planks"},{"varval":"15 cm = 0.15 metres"}]},{"vars":[{"varval":"Oscar"},{"varval":"fence"},{"varval":"vertical planks of wood"},{"varval":"11"},{"varval":"24.2"},{"varval":"planks"},{"varval":"11 cm = 0.11 metres"}]},{"vars":[{"varval":"Karl"},{"varval":"fence"},{"varval":"vertical wooden planks"},{"varval":"9"},{"varval":"14.4"},{"varval":"planks"},{"varval":"9 cm = 0.09 metres"}]}]

  851. <div class="sm_mode"> {{name}} needs to reduce her discretionary spending which is shown in the table below. <br> <div class="outline"> > > | Spend | Amount |How Often| > > | ------ | ----------------: |-| > > | {{spend1}}| ${{amount1}}|{{often1}}| > > | {{spend2}}| ${{amount2}} |{{often2}}| > > | {{spend3}}| ${{amount3}} |{{often3}}| > > | {{spend4}}| ${{amount4}} |{{often4}}| </div> <br> <br>Which action will save her the most? </div>

    [{"vars":[{"varval":"Rhonda"},{"varval":"Car Wash"},{"varval":"Hairdresser"},{"varval":"Thai take-away"},{"varval":"Streaming services"},{"varval":"80"},{"varval":"150"},{"varval":"15"},{"varval":"12"},{"varval":"Monthly"},{"varval":"Quarterly"},{"varval":"2 days per week"},{"varval":"Weekly"},{"varval":"Car Wash = $80\r\n\r\nHairdresser = 150 $\\div$ 3 = \\$50\r\n\r\nThai = 15 $\\times$ 4 = $60\r\n\r\nStreaming = 12 $\\times$ 4 = $48"}]},{"vars":[{"varval":"Sharnie"},{"varval":"Foxtel"},{"varval":"Theatre"},{"varval":"Uber Eats"},{"varval":"Swimming Lessons"},{"varval":"70"},{"varval":"120"},{"varval":"20"},{"varval":"15"},{"varval":"Monthly"},{"varval":"Quarterly"},{"varval":"twice per week"},{"varval":"Weekly"},{"varval":"Foxtel = $70\r\n\r\nTheatre = 120 $\\div$ 3 = \\$40\r\n\r\nUber Eats = 20 $\\times$ 4 = $80\r\n\r\nSwimming Lessons = 15 $\\times$ 4 = $60"}]},{"vars":[{"varval":"Jenny"},{"varval":"Gym Membership"},{"varval":"Air Travel"},{"varval":"KFC Lunch"},{"varval":"Penrith Panthers Ticket"},{"varval":"50"},{"varval":"120"},{"varval":"14"},{"varval":"20"},{"varval":"Monthly"},{"varval":"Quarterly"},{"varval":"2 days per week"},{"varval":"Weekly"},{"varval":"Gym Membership = $50\r\n\r\nAir Travel = 120 $\\div$ 3 = \\$40\r\n\r\nKFC Lunch = 14 $\\times$ 4 = $56\r\n\r\nPenrith Panthers Tickets = 20 $\\times$ 4 = $80"}]},{"vars":[{"varval":"Veronica"},{"varval":"Coffee Beans"},{"varval":"Streaming Services"},{"varval":"Beauty Salon"},{"varval":"Pizza Delivery"},{"varval":"10"},{"varval":"50"},{"varval":"90"},{"varval":"15"},{"varval":"Weekly"},{"varval":"Monthly"},{"varval":"Quarterly"},{"varval":"2 days per week"},{"varval":"Coffee Beans = 4 $\\times$ 10 = $40\r\n\r\nStreaming Services = 50 $-$ 15 = \\$35\r\n\r\nBeauty Salon = 90 $\\div$ 3 = $30\n\r\nPizza Delivery = 15 $\\times$ 4 = $60"}]},{"vars":[{"varval":"Zilda"},{"varval":"Theatre"},{"varval":"Coffee"},{"varval":"Car Detailing"},{"varval":"Chocolate"},{"varval":"90"},{"varval":"5"},{"varval":"150"},{"varval":"12"},{"varval":"Monthly"},{"varval":"5 days per week"},{"varval":"Quarterly"},{"varval":"Weekly"},{"varval":"Theatre = 90 $-$ 30 = $60\r\n\r\nCoffee = 5 $\\times$ 4 $\\times$ 4 = \\$80\r\n\r\nCar Detailing = 150 $\\div$ 3 = $50\r\n\r\nChocolate = 4 $\\times$ 12 = $48"}]}]

  852. <div class="sm_mode"> Concrete is poured to make a {{object}}. The dimensions of the slab are shown in the diagram below. <br> {{image}} <br>If the concrete costs \${{cost}} per cubic metre to pour, what is the cost of the slab? </div>

    [{"vars":[{"varval":"pathway"},{"varval":"sm_img https://teacher.smartermaths.com.au/wp-content/uploads/2021/01/5var326_i.svg 380 indent3 vpad"},{"varval":"180"},{"varval":"30 cm = 0.3 metres"},{"varval":"8 $\\times$ 2 $\\times$ 0.3"},{"varval":"4.8"}]},{"vars":[{"varval":"driveway"},{"varval":"sm_img https://teacher.smartermaths.com.au/wp-content/uploads/2021/01/5var326_ii.svg 380 indent3 vpad"},{"varval":"220"},{"varval":"40 cm = 0.4 metres"},{"varval":"4 $\\times$ 5 $\\times$ 0.4"},{"varval":"8"}]},{"vars":[{"varval":"golf-cart pathway"},{"varval":"sm_img https://teacher.smartermaths.com.au/wp-content/uploads/2021/01/5var326_iii.svg 370 indent3 vpad"},{"varval":"300"},{"varval":"20 cm = 0.2 metres"},{"varval":"10 $\\times$ 2 $\\times$ 0.2"},{"varval":"4"}]},{"vars":[{"varval":"swimming pool base"},{"varval":"sm_img https://teacher.smartermaths.com.au/wp-content/uploads/2021/01/5var326_iv.svg 360 indent3 vpad"},{"varval":"210"},{"varval":"60 cm = 0.6 metres"},{"varval":"8 $\\times$ 5 $\\times$ 0.6"},{"varval":"24"}]},{"vars":[{"varval":"back deck"},{"varval":"sm_img https://teacher.smartermaths.com.au/wp-content/uploads/2021/01/5var326_v.svg 370 indent3 vpad"},{"varval":"240"},{"varval":"15 cm = 0.15 metres"},{"varval":"4 $\\times$ 7 $\\times$ 0.15"},{"varval":"4.2"}]}]

  853. <div class="sm_mode"> {{person}} {{verb}} that is available in four different sizes at the {{place}}. Which of the following is the best buy? </div>

    [{"vars":[{"varval":"Fabio"},{"varval":"has released a new cologne"},{"varval":"chemist"},{"varval":"100 mL for \\$5.20 $\\rArr$ \\$52/L\n\n200 mL for \\$11.00 $\\rArr$ \\$55/L\n\n500 mL for \\$29.00 $\\rArr$ \\$58/L\n\n1 L for \\$54.50"}]},{"vars":[{"varval":"Dove"},{"varval":"has released a new hand sanitiser"},{"varval":"chemist"},{"varval":"100 mL for \\$1.65 $\\rArr$ \\$16.50/L\n\n200 mL for \\$3.00 $\\rArr$ \\$15.00/L\n\n250 mL for \\$4.00 $\\rArr$ \\$16.00/L\n\n1 L for \\$16.20"}]},{"vars":[{"varval":"Palmolive"},{"varval":"has released a new liquid soap"},{"varval":"supermarket"},{"varval":"200 mL for \\$1.80 $\\rArr$ \\$9.00/L\n\n250 mL for \\$2.00 $\\rArr$ \\$8.00/L\n\n500 mL for \\$3.90 $\\rArr$ \\$7.80/L\n\n1 L for \\$7.50"}]},{"vars":[{"varval":"Ronaldo"},{"varval":"has released a new shampoo"},{"varval":"supermarket"},{"varval":"\r100 mL for \\$1.05 $\\rArr$ \\$10.50/L\r\n\r\n250 mL for \\$2.40 $\\rArr$ \\$9.60/L\r\n\r\n500 mL for \\$5.50 $\\rArr$ \\$11.00/L\r\n\r\n1 L for \\$10.40"}]},{"vars":[{"varval":"Woof Dog"},{"varval":"has released a new dog shampoo"},{"varval":"pet store"},{"varval":"\r\n100 mL for \\$0.90 $\\rArr$ \\$9.00/L\r\n\r\n250 mL for \\$2.50 $\\rArr$ \\$10.00/L\r\n\r\n500 mL for \\$5.10 $\\rArr$ \\$10.20/L\r\n\r\n1 L for \\$9.50"}]}]

  854. <div class="sm_mode"> The opposite faces of a standard die always add up to seven. <br> sm_img https://teacher.smartermaths.com.au/wp-content/uploads/2021/01/5var324_Q.svg 100 indent3 vpad <br>Which of the following nets can be folded into a standard die? </div>

    [{"vars":null},{"vars":null},{"vars":null},{"vars":null},{"vars":null}]

  855. <div class="sm_mode"> The measurements of the prisms below are all in centimetres. Which prism has the capacity to hold exactly {{litre}} of water? </div>

    [{"vars":[{"varval":"1 litre"},{"varval":"1 L $\\rArr\\ 1000 \\ \\text{cm}^3$"},{"varval":"10 $\\times$ 5 $\\times$ 20"},{"varval":"10 $\\times$ 100"},{"varval":"1000"}]},{"vars":[{"varval":"1 litre"},{"varval":"1 L $\\rArr\\ 1000\\ \\text{cm}^3$"},{"varval":"50 $\\times$ 4 $\\times$ 5"},{"varval":"200 $\\times$ 5"},{"varval":"1000"}]},{"vars":[{"varval":"2 litres"},{"varval":"2 L $\\rArr\\ 2000\\ \\text{cm}^3$"},{"varval":"10 $\\times$ 20 $\\times$ 10"},{"varval":"200 $\\times$ 10"},{"varval":"2000"}]},{"vars":[{"varval":"1 litre"},{"varval":"1 L $\\rArr\\ 1000\\ \\text{cm}^3$"},{"varval":"20 $\\times$ 5 $\\times$ 10"},{"varval":"100 $\\times$ 10"},{"varval":"1000"}]},{"vars":[{"varval":"2 litres"},{"varval":"2 L $\\rArr\\ 2000\\ \\text{cm}^3$"},{"varval":"40 $\\times$ 10 $\\times$ 5"},{"varval":"400 $\\times$ 5"},{"varval":"2000"}]}]

  856. <div class="sm_mode"> A wallet contains only ${{note1}} and ${{note2}} notes in the ratio {{ratio}}. If there is a total of {{number1}} notes, how much money is in the wallet? </div>

    [{"vars":[{"varval":"10"},{"varval":"20"},{"varval":"3 : 1"},{"varval":"12"},{"varval":"$\\dfrac{3}{4}$"},{"varval":"9"},{"varval":"$\\dfrac{1}{4}$"},{"varval":"3"},{"varval":"90"},{"varval":"60"}]},{"vars":[{"varval":"5"},{"varval":"20"},{"varval":"2 : 3"},{"varval":"15"},{"varval":"$\\dfrac{2}{5}$"},{"varval":"6"},{"varval":"$\\dfrac{3}{5}$"},{"varval":"9"},{"varval":"30"},{"varval":"180"}]},{"vars":[{"varval":"10"},{"varval":"5"},{"varval":"5 : 1"},{"varval":"18"},{"varval":"$\\dfrac{5}{6}$"},{"varval":"15"},{"varval":"$\\dfrac{1}{6}$"},{"varval":"3"},{"varval":"150"},{"varval":"15"}]},{"vars":[{"varval":"20"},{"varval":"10"},{"varval":"2 : 1"},{"varval":"12"},{"varval":"$\\dfrac{2}{3}$"},{"varval":"8"},{"varval":"$\\dfrac{1}{3}$"},{"varval":"4"},{"varval":"160"},{"varval":"4"},{"varval":"40"}]},{"vars":[{"varval":"50"},{"varval":"20"},{"varval":"1 : 3"},{"varval":"16"},{"varval":"$\\dfrac{1}{4}$"},{"varval":"4"},{"varval":"$\\dfrac{3}{4}$"},{"varval":"12"},{"varval":"200"},{"varval":"240"}]}]

  857. <div class="sm_mode"> {{girl}} and {{number}} of her friends took turns {{verb1}}. They {{verb2}} between {{time1}} and {{time2}}. If they {{verb3}} an equal amount of time, how many minutes did each person {{verb4}}? </div>

    [{"vars":[{"varval":"Majella"},{"varval":"four"},{"varval":"playing a video game"},{"varval":"played"},{"varval":"1:20 pm"},{"varval":"3:30 pm"},{"varval":"played"},{"varval":"play"},{"varval":"130"},{"varval":"$\\dfrac{130}{5}$"}]},{"vars":[{"varval":"Sonja"},{"varval":"five"},{"varval":"batting in a cricket net"},{"varval":"practiced"},{"varval":"10:15 am"},{"varval":"11:45 am"},{"varval":"batted"},{"varval":"bat for"},{"varval":"90"},{"varval":"$\\dfrac{90}{6}$"}]},{"vars":[{"varval":"Neli"},{"varval":"four"},{"varval":"singing at karaoke"},{"varval":"sang"},{"varval":"8:30 pm"},{"varval":"9:50 pm"},{"varval":"had the microphone"},{"varval":"sing for"},{"varval":"80"},{"varval":"$\\dfrac{80}{5}$"}]},{"vars":[{"varval":"Naomi"},{"varval":"four"},{"varval":"carrying a backpack on a walk"},{"varval":"walked"},{"varval":"1:20 pm"},{"varval":"4:00 pm"},{"varval":"carried the backpack"},{"varval":"have it for"},{"varval":"160"},{"varval":"$\\dfrac{160}{5}$"}]},{"vars":[{"varval":"Jackie"},{"varval":"four"},{"varval":"swimming in an ocean relay event"},{"varval":"swam"},{"varval":"6:30 am"},{"varval":"8:50 am"},{"varval":"swam"},{"varval":"swim for"},{"varval":"140"},{"varval":"$\\dfrac{140}{5}$"}]}]

  858. Myca, can you make the distance labels on the wheels go from the hub to the edge of the rubber ridges on all images.

    <div class="sm_mode"> A {{type}} has a hub with {{measure1}} of {{length1}} millimetres and an outer rubber layer with a width of {{length2}} millimetres. {{image}} What distance, in millimetres, would the truck cover in one full rotation of the wheel? </div>

    [{"vars":[{"varval":"truck wheel"},{"varval":"radius"},{"varval":"250"},{"varval":"150"},{"varval":"sm_img https://teacher.smartermaths.com.au/wp-content/uploads/2021/01/5var_320_150mm.svg 350 indent vpad"},{"varval":"400"}]},{"vars":[{"varval":"coal truck wheel"},{"varval":"radius"},{"varval":"2400"},{"varval":"600"},{"varval":"sm_img https://teacher.smartermaths.com.au/wp-content/uploads/2021/01/5var_320_600mm.svg 350 indent vpad"},{"varval":"3000"}]},{"vars":[{"varval":"monster truck wheel"},{"varval":"radius"},{"varval":"3200"},{"varval":"500"},{"varval":"sm_img https://teacher.smartermaths.com.au/wp-content/uploads/2021/01/5var_320_500mm.svg 350 indent vpad"},{"varval":"3700"}]},{"vars":[{"varval":"truck tyre"},{"varval":"radius"},{"varval":"380"},{"varval":"140"},{"varval":"sm_img https://teacher.smartermaths.com.au/wp-content/uploads/2021/01/5var_320_140mm.svg 350 indent vpad"},{"varval":"520"}]},{"vars":[{"varval":"mining truck tyre"},{"varval":"radius"},{"varval":"1900"},{"varval":"600"},{"varval":"sm_img https://teacher.smartermaths.com.au/wp-content/uploads/2021/01/5var_320_600mm.svg 350 indent vpad"},{"varval":"2500"}]}]

  859. <div class="sm_mode"> A {{job}} makes a {{object1}} with a height of {{length1}} cm and a width of {{length2}} cm. She makes another {{object2}} in the same proportion that has a height of {{length3}} cm. What is the width of this {{object2}}? </div>

    [{"vars":[{"varval":"framer"},{"varval":"picture frame"},{"varval":"30"},{"varval":"20"},{"varval":"frame"},{"varval":"45"},{"varval":"2 : 3"},{"varval":"$\\dfrac{3}{2}$"}]},{"vars":[{"varval":"graphic designer"},{"varval":"poster"},{"varval":"50"},{"varval":"40"},{"varval":"poster"},{"varval":"75"},{"varval":"2 : 3"},{"varval":"$\\dfrac{3}{2}$"}]},{"vars":[{"varval":"graphic designer"},{"varval":"poster"},{"varval":"60"},{"varval":"80"},{"varval":"poster"},{"varval":"120"},{"varval":"1 : 2"},{"varval":"2"}]},{"vars":[{"varval":"framer"},{"varval":"picture frame"},{"varval":"12"},{"varval":"10"},{"varval":"frame"},{"varval":"18"},{"varval":"2 : 3"},{"varval":"$\\dfrac{3}{2}$"}]},{"vars":[{"varval":"sketch artist"},{"varval":"sketch"},{"varval":"8"},{"varval":"10"},{"varval":"sketch"},{"varval":"20"},{"varval":"2 : 5"},{"varval":"$\\dfrac{5}{2}$"}]}]

  860. <div class="sm_mode"> {{people}} are driving from {{place1}} to {{place2}} which is a distance of {{distance}} kilometres. After every two hours of driving, they rest for {{rest}} minutes and swap drivers. How long will their trip take if they average {{speed}} km/h when driving? </div>

    [{"vars":[{"varval":"Oscar and Lucinda"},{"varval":"Dungog"},{"varval":"Bourke"},{"varval":"735"},{"varval":"15"},{"varval":"70"},{"varval":"10.5 h"},{"varval":"5"},{"varval":"10 h 30 m + (5 $\\times$ 15 m)"},{"varval":"10 h 30 m + 1 h 15 m"}]},{"vars":[{"varval":"Roy and Siegfried"},{"varval":"Broken Hill"},{"varval":"Albury"},{"varval":"840"},{"varval":"20"},{"varval":"80"},{"varval":"10.5"},{"varval":"5"},{"varval":"10 h 30 m + (5 $\\times$ 20 m)"},{"varval":"10 h 30 m + 1 h 40 m"}]},{"vars":[{"varval":"Banjo and Henry"},{"varval":"Bowral"},{"varval":"Glen Innes"},{"varval":"760"},{"varval":"30"},{"varval":"80"},{"varval":"9.5"},{"varval":"4"},{"varval":"9 h 30 min + (4 $\\times$ 30 m)"},{"varval":"9 h 30 m + 2 h"}]},{"vars":[{"varval":"Tics and Rooster"},{"varval":"Coffs Harbour"},{"varval":"Nowra"},{"varval":"690"},{"varval":"15"},{"varval":"60"},{"varval":"11.5"},{"varval":"5"},{"varval":"11 h 30 m + (5 $\\times$ 15 m)"},{"varval":"11 h 30 m + 1 h 15 m"}]},{"vars":[{"varval":"Johnno and Zoey"},{"varval":"Wahroonga"},{"varval":"Ballarat"},{"varval":"825"},{"varval":"20"},{"varval":"100"},{"varval":"8.25"},{"varval":"4"},{"varval":"8 h 15 m + (4 $\\times$ 20)"},{"varval":"8 hr 15 m + 1 h 20 m"}]}]

  861. <div class="sm_mode"> On a map, {{places}} are {{dist1}} centimetres apart. If the scale of the map is {{scale}}, how many kilometres are the {{places}} apart? </div>

    [{"vars":[{"varval":"two towns"},{"varval":"6.5"},{"varval":"1 \\: 30 000"},{"varval":"30 000"},{"varval":"195 000 cm"},{"varval":"1950 m"}]},{"vars":[{"varval":"two museums"},{"varval":"7.5"},{"varval":"1 \\: 20 000"},{"varval":"20 000"},{"varval":"150 000 cm"},{"varval":"1500 m"}]},{"vars":[{"varval":"two beaches"},{"varval":"12.5"},{"varval":"1 \\: 40 000"},{"varval":"40 000"},{"varval":"500 000 cm"},{"varval":"5000 m"}]},{"vars":[{"varval":"two mountains"},{"varval":"14.5"},{"varval":"1 \\: 80 000"},{"varval":"80 000"},{"varval":"1 160 000 cm"},{"varval":"11 600 m"}]},{"vars":[{"varval":"two hospitals"},{"varval":"3.5"},{"varval":"1 \\: 50 000"},{"varval":"50 000"},{"varval":"175 000 cm"},{"varval":"1750 m"}]}]

  862. <div class="sm_mode"> {{name1}}, {{name2}} and {{name3}} are {{activity}}. {{name1}} {{frac1}} and {{frac2}}. What fraction of the {{frac3}}? </div>

    [{"vars":[{"varval":"Kelly"},{"varval":"Megan"},{"varval":"Narelle"},{"varval":"running in a 3-person relay"},{"varval":"completes half of the course"},{"varval":"Megan completes one-fifth"},{"varval":"original distance does Narelle have left"},{"varval":"$\\dfrac{1}{2} + \\dfrac{1}{5}$"},{"varval":"$\\dfrac{5}{10} + \\dfrac{2}{10}$"},{"varval":"$\\dfrac{7}{10}$"}]},{"vars":[{"varval":"Ian"},{"varval":"Greg"},{"varval":"Trevor"},{"varval":"swimming in an ocean relay event"},{"varval":"completes half of the course"},{"varval":"Greg completes one-third"},{"varval":"original distance does Trevor have left"},{"varval":"$\\dfrac{1}{2} + \\dfrac{1}{3}$"},{"varval":"$\\dfrac{3}{6} + \\dfrac{2}{6}$"},{"varval":"$\\dfrac{5}{6}$"}]},{"vars":[{"varval":"Chris"},{"varval":"Liam"},{"varval":"Luke"},{"varval":"walking in a 3 person charity relay"},{"varval":"completes one-fifth of the total distance"},{"varval":"Liam completes one-quarter"},{"varval":"total distance is left for Luke to walk"},{"varval":"$\\dfrac{1}{5} + \\dfrac{1}{4}$"},{"varval":"$\\dfrac{4}{20} + \\dfrac{5}{20}$"},{"varval":"$\\dfrac{9}{20}$"}]},{"vars":[{"varval":"Andrea"},{"varval":"Caroline"},{"varval":"Sharon"},{"varval":"laying planks for an outside deck"},{"varval":"lays two-fifths of the planks"},{"varval":"Caroline lays one-quarter"},{"varval":"total planks are left for Sharon to lay"},{"varval":"$\\dfrac{2}{5} + \\dfrac{1}{4}$"},{"varval":"$\\dfrac{8}{20} + \\dfrac{5}{20}$"},{"varval":"$\\dfrac{13}{20}$"}]},{"vars":[{"varval":"Larry"},{"varval":"Mo"},{"varval":"Curley"},{"varval":"laying bricks for a fence"},{"varval":"lays one-third of the bricks"},{"varval":"Mo lays one-quarter"},{"varval":"total bricks are left for Curley to lay"},{"varval":"$\\dfrac{1}{3} + \\dfrac{1}{4}$"},{"varval":"$\\dfrac{4}{12} + \\dfrac{3}{12}$"},{"varval":"$\\dfrac{7}{12}$"}]}]

  863. VARIANT 3 ... adjust image so that the connection A to B (3.1) is removed.

    <div class="sm_mode"> The letters below represent {{landmark1}}. The {{connect}} between {{landmark2}} and their distances, in kilometres, are shown on the diagram. <br> {{network}} <br>If travel can only be made on the {{connect}} shown on the diagram, which of the following trips is the shortest? </div>

    [{"vars":[{"varval":"country properties"},{"varval":"roads"},{"varval":"properties"},{"varval":"sm_img https://teacher.smartermaths.com.au/wp-content/uploads/2020/12/variant312_1.svg 300 indent3 vpad"},{"varval":"A to C: 5.3 + 2.8 = 8.1\n\nB to E: 2.8 + 4.9 = 7.7\n\nD to C: 3.3 + 4.9 = 8.2\n\nA to E: 4.2 + 3.3 = 7.5"}]},{"vars":[{"varval":"a city's museums"},{"varval":"pathways"},{"varval":"museums"},{"varval":"sm_img https://teacher.smartermaths.com.au/wp-content/uploads/2020/12/variant312_2.svg 250 indent3 vpad"},{"varval":"A to C: 2.7 + 3.6 = 6.3\n\nB to D: 3.6 + 2.9 = 6.5\n\nD to E: 2.9 + 3.5 = 6.4\n\nB to E: 3.6 + 3.5 = 7.1"}]},{"vars":[{"varval":"a city's parks"},{"varval":"pathways"},{"varval":"parks"},{"varval":"sm_img https://teacher.smartermaths.com.au/wp-content/uploads/2020/12/variant312_3.svg 240 indent3 vpad"},{"varval":"A to E: 2.3 + 4.1 = 6.4\n\nB to D: 2.3 + 3.1 = 5.4\n\nA to C: 2.3 + 3.8 = 6.1\n\nF to A: 2.9 + 3.1 = 6.0"}]},{"vars":[{"varval":"a city's train stations"},{"varval":"train lines"},{"varval":"stations"},{"varval":"sm_img https://teacher.smartermaths.com.au/wp-content/uploads/2021/02/variant312_4rev.svg 200 indent3 vpad"},{"varval":"A to D: 4.5 + 3.6 = 8.1\n\nE to B: 3.4 + 3.9 = 7.3\n\nB to C: 3.9 + 3.6 = 7.5\n\nC to E: 3.6 + 3.4 = 7.0"}]},{"vars":[{"varval":"mining tenements"},{"varval":"roads"},{"varval":"tenements"},{"varval":"sm_img https://teacher.smartermaths.com.au/wp-content/uploads/2020/12/variant312_5.svg 270 indent3 vpad"},{"varval":"A to D: 4.5 + 5.1 = 9.6\n\nB to C: 5.1 + 3.9 = 9.0\n\nF to C: 4.3 + 3.9 = 8.2\n\nE to F: 4.9 + 4.3 = 9.2"}]}]

  864. <div class="sm_mode"> {{name}} {{activity1}}. After {{time}}, he has {{activity2}}. What fraction still needs to be {{verb}}? </div>

    [{"vars":[{"varval":"Angus"},{"varval":"was downloading a movie on Netflix"},{"varval":"30 seconds"},{"varval":"downloaded 85% of the file"},{"varval":"downloaded"},{"varval":"$\\dfrac{85}{100}$ = $\\dfrac{17}{20}$"},{"varval":"$1 - \\dfrac{17}{20}$"}]},{"vars":[{"varval":"Sandeep"},{"varval":"is filling up his new pool with water"},{"varval":"4 hours"},{"varval":"filled up 45% of the pool"},{"varval":"filled"},{"varval":"$\\dfrac{45}{100}$ = $\\dfrac{9}{20}$"},{"varval":"$1 - \\dfrac{9}{20}$"}]},{"vars":[{"varval":"Con"},{"varval":"is packing mangoes into boxes"},{"varval":"2 hours"},{"varval":"packed 35% of the total mangoes"},{"varval":"packed"},{"varval":"$\\dfrac{35}{100}$ = $\\dfrac{7}{20}$"},{"varval":"$1 - \\dfrac{7}{20}$"}]},{"vars":[{"varval":"Carl"},{"varval":"is doing a walk for charity"},{"varval":"3 hours"},{"varval":"completed 75% of the walk"},{"varval":"completed"},{"varval":"$\\dfrac{75}{100}$ = $\\dfrac{15}{20}$"},{"varval":"$1 - \\dfrac{15}{20}$"}]},{"vars":[{"varval":"Krusty"},{"varval":"is raising money for a charity"},{"varval":"3 weeks"},{"varval":"raised 65% of his target"},{"varval":"raised"},{"varval":"$\\dfrac{65}{100}$ = $\\dfrac{13}{20}$"},{"varval":"$1 - \\dfrac{13}{20}$"}]}]

  865. FIX Table ... "Movee" should be "Moree"

    <div class="sm_mode"> The distances between major towns in New South Wales are shown in the table below. sm_img https://teacher.smartermaths.com.au/wp-content/uploads/2020/12/variant_310.svg 620 indent vpad {{name1}} is travelling from {{location1}} to {{location2}}. {{name2}} is travelling from {{location3}} to {{location4}}. How much farther is {{name2}} travelling than {{name1}}? </div>

    [{"vars":[{"varval":"Mark"},{"varval":"Bourke"},{"varval":"Dungog"},{"varval":"Alan"},{"varval":"Bowral"},{"varval":"Broken Hill"},{"varval":"750"},{"varval":"1189"}]},{"vars":[{"varval":"Albert"},{"varval":"Bowral"},{"varval":"Bourke"},{"varval":"Koko"},{"varval":"Albury"},{"varval":"Moree"},{"varval":"783"},{"varval":"908"}]},{"vars":[{"varval":"Veronica"},{"varval":"Dungog"},{"varval":"Bowral"},{"varval":"Israel"},{"varval":"Bourke"},{"varval":"Cooma"},{"varval":"321"},{"varval":"839"}]},{"vars":[{"varval":"Darryl"},{"varval":"Dungog"},{"varval":"Albury"},{"varval":"Murray"},{"varval":"Broken Hill"},{"varval":"Grafton"},{"varval":"757"},{"varval":"1330"}]},{"vars":[{"varval":"Eliza"},{"varval":"Moree"},{"varval":"Bourke"},{"varval":"Claudia"},{"varval":"Broken Hill"},{"varval":"Albury"},{"varval":"439"},{"varval":"851"}]}]

  866. <div class="sm_mode"> Students were given a survey where they were asked {{question1}}. The results are shown in the table below. <div class="outline"> > > | | {{object1}} | {{object2}} | Total | > > | ------ | :---------: | :---------: | :---: | > > | Male | {{number1}} | {{number4}} | 50 | > > | Female | {{number2}} | {{number5}} | 50 | > > | Total | {{number3}} | {{number6}} | 100 | </div> <br> What percentage of {{gender}} students {{question2}}? </div>

    [{"vars":[{"varval":"if their house had solar panels"},{"varval":"Solar Panels"},{"varval":"31"},{"varval":"33"},{"varval":"64"},{"varval":"No Solar Panels"},{"varval":"19"},{"varval":"17"},{"varval":"36"},{"varval":"did not have solar panels"},{"varval":"$\\dfrac{\\text{Males with no solar panels}}{\\text{Total Males}}\\ \\times$ 100"},{"varval":"$\\dfrac{19}{50}\\ \\times$ 100"},{"varval":"male"}]},{"vars":[{"varval":"if they had a part-time job"},{"varval":"Part-time Job"},{"varval":"34"},{"varval":"37"},{"varval":"71"},{"varval":"No Part-time Job"},{"varval":"16"},{"varval":"13"},{"varval":"29"},{"varval":"did not have a part-time job"},{"varval":"$\\dfrac{\\text{Females with no part-time job}}{\\text{Total Females}}$"},{"varval":"$\\dfrac{13}{50}\\ \\times$ 100"},{"varval":"female"}]},{"vars":[{"varval":"if they caught a train to get to school"},{"varval":"Train"},{"varval":"24"},{"varval":"27"},{"varval":"51"},{"varval":"No Train"},{"varval":"26"},{"varval":"23"},{"varval":"49"},{"varval":"did not catch a train to school"},{"varval":"$\\dfrac{\\text{Females(no train)}}{\\text{Total Females}}$"},{"varval":"$\\dfrac{23}{50}\\ \\times$ 100"},{"varval":"female"}]},{"vars":[{"varval":"if they owned a pet"},{"varval":"Own Pet"},{"varval":"22"},{"varval":"19"},{"varval":"41"},{"varval":"Don't Own Pet"},{"varval":"28"},{"varval":"31"},{"varval":"59"},{"varval":"did not own a pet"},{"varval":"$\\dfrac{\\text{Males that do not own pet}}{\\text{Total Males}}$"},{"varval":"$\\dfrac{28}{50}\\ \\times$ 100"},{"varval":"male"}]},{"vars":[{"varval":"if they study Biology"},{"varval":"Study Biology"},{"varval":"14"},{"varval":"23"},{"varval":"37"},{"varval":"Don't Study Biology"},{"varval":"36"},{"varval":"27"},{"varval":"63"},{"varval":"don't study Biology"},{"varval":"$\\dfrac{\\text{Males not studying Biology}}{\\text{Total Males}}$"},{"varval":"$\\dfrac{36}{50}\\ \\times$ 100"},{"varval":"male"}]}]

  867. <div class="sm_mode"> The cost of {{activity1}} is shown in the table below. <div class="outline"> > > | Rental period | Cost | > > | ------ | -----------------: | > > | Up to 1 hour| {{cost1}} | > > | Following half hour or part thereof| {{cost2}} | </div> <br> {{name}} {{activity2}}. If she was charged {{total}} when she returned it, at what time could she possibly have done this? </div>

    [{"vars":[{"varval":"renting a surfboard"},{"varval":"$10"},{"varval":"$3"},{"varval":"Kim"},{"varval":"rented a surfboard at 9:00 am"},{"varval":"$13"},{"varval":"Kim has had the surfboard between 1 $-$ 1.5 hours"}]},{"vars":[{"varval":"renting a kayak"},{"varval":"$8"},{"varval":"$3"},{"varval":"Denise"},{"varval":"rented a kayak at 3:30 pm"},{"varval":"$14"},{"varval":"Denise has had the kayak between 1.5 $-$ 2 hours"}]},{"vars":[{"varval":"renting a drone"},{"varval":"$12"},{"varval":"$6"},{"varval":"Dolly"},{"varval":"rented a drone at 7:00 am"},{"varval":"$18"},{"varval":"Dolly has had the drone between 1 $-$ 1.5 hours"}]},{"vars":[{"varval":"renting a jetski"},{"varval":"$15"},{"varval":"$3"},{"varval":"Jill"},{"varval":"rented a jetski at 2:00 pm"},{"varval":"$24"},{"varval":"Jill has had the jetski between 2 $-$ 2.5 hours"}]},{"vars":[{"varval":"renting a bowling alley"},{"varval":"$14"},{"varval":"$4"},{"varval":"Billie"},{"varval":"rented a bowling alley at 7:30 pm"},{"varval":"$22"},{"varval":"Billie has had the bowling alley between 1.5 $-$ 2 hours"}]}]

  868. <div class="sm_mode"> {{number1}} {{object1}} weigh {{mass1}} kilograms. If each {{object2}} has the same mass, approximately what would {{number2}} {{object1}} weigh? </div>

    [{"vars":[{"varval":"Fifteen"},{"varval":"concrete bricks"},{"varval":"40.8"},{"varval":"brick"},{"varval":"8"},{"varval":"15"},{"varval":"2.72"},{"varval":"21.76"}]},{"vars":[{"varval":"Eighteen"},{"varval":"containers"},{"varval":"51.3"},{"varval":"container"},{"varval":"5"},{"varval":"18"},{"varval":"2.85"},{"varval":"14.25"}]},{"vars":[{"varval":"A dozen"},{"varval":"watermelons"},{"varval":"67.8"},{"varval":"watermelon"},{"varval":"5"},{"varval":"12"},{"varval":"5.65"},{"varval":"28.25"}]},{"vars":[{"varval":"Twenty-five"},{"varval":"chairs"},{"varval":"105.5"},{"varval":"chair"},{"varval":"7"},{"varval":"25"},{"varval":"4.22"},{"varval":"29.54"}]},{"vars":[{"varval":"Sixteen"},{"varval":"chaff bags"},{"varval":"151.2"},{"varval":"chaff bag"},{"varval":"6"},{"varval":"16"},{"varval":"9.45"},{"varval":"56.7"}]}]

  869. <div class="sm_mode"> {{scenario}} Which combination {{question}}? </div>

    [{"vars":[{"varval":"A television producer is designing the number and length of episodes in a new series."},{"varval":"gives the new series the longest screen time"},{"varval":"10 $\\times$ 32 = 320 minutes\n\n8 $\\times$ 45 = 360 minutes\n\n6 $\\times$ 56 = 336 minutes\n\n5 $\\times$ 70 = 350 minutes\n\n$\\therefore$ 8 episodes of 45 minutes"}]},{"vars":[{"varval":"A dance school is designing the number and length of dance lessons for their Learn to Dance course."},{"varval":"provides the longest total lesson time"},{"varval":"8 $\\times$ 30 = 240 mins\n\n6 $\\times$ 45 = 270 mins\n\n5 $\\times$ 50 = 250 mins\n\n4 $\\times$ 70 = 280 mins\n\n$\\therefore$ 4 lessons of 1 hour 10 minutes"}]},{"vars":[{"varval":"A swim school is designing the number and length of swimming lessons for its Summer squad program."},{"varval":"provides the longest total lesson time for the program"},{"varval":"10 $\\times$ 45 = 450 mins\n\n8 $\\times$ 54 = 432 mins\n\n7 $\\times$ 65 = 455 mins\n\n6 $\\times$ 70 = 420 mins\n\n$\\therefore$ 7 lessons of 1 hour and 5 minutes"}]},{"vars":[{"varval":"A guitar teacher is designing the number and length of guitar lessons she can do in one day."},{"varval":"gives the longest total teaching time"},{"varval":"7 $\\times$ 40 = 280 mins\n\n5 $\\times$ 58 = 290 mins\n\n4 $\\times$ 65 = 260 mins\n\n3 $\\times$ 90 = 270 mins\n\n$\\therefore$ 5 lessons of 58 minutes"}]},{"vars":[{"varval":"A scuba diving instructor is designing the number and length of dives for her advanced diving course."},{"varval":"gives the longest total dive time"},{"varval":"8 $\\times$ 40 = 320 mins\n\n6 $\\times$ 52 = 312 mins\n\n5 $\\times$ 66 = 330 mins\n\n4 $\\times$ 71 = 284 mins\n\n$\\therefore$ 5 lessons of 1 hour and 6 minutes"}]}]

  870. <div class="sm_mode"> {{name}} was measuring the {{activity}}. He recorded {{type1}} of {{data}}. What was the median {{type2}} recorded? </div>

    [{"vars":[{"varval":"Ernest"},{"varval":"crabs he had caught"},{"varval":"widths"},{"varval":"15, 15.4, 16.5, 16 and 15.2 centimetres"},{"varval":"width"},{"varval":"15, 15.2, 15.4, 16, 16.5"}]},{"vars":[{"varval":"Sam"},{"varval":"lengths of cockroaches he had caught for a science experiment"},{"varval":"lengths"},{"varval":"34.3, 36, 34, 34.8, 36 millimetres"},{"varval":"length"},{"varval":"34, 34.3, 34.8, 36, 36"}]},{"vars":[{"varval":"Fernando"},{"varval":"head diameter of Tasmanian devils at the zoo"},{"varval":"diameters"},{"varval":"92, 94, 92.7, 92.3 and 94 millimetres"},{"varval":"diameter"},{"varval":"92, 92.3, 92.7, 94, 94"}]},{"vars":[{"varval":"Murray"},{"varval":"length of the tuna fish he had caught"},{"varval":"lengths"},{"varval":"61, 61.4, 63, 63, and 61.9 centimetres"},{"varval":"length"},{"varval":"61, 61.4, 61.9, 63, 63"}]},{"vars":[{"varval":"Sehwag"},{"varval":"lengths of fairy penguins in the wild"},{"varval":"lengths"},{"varval":"45.2, 45, 47, 45.6, 47 centimetres"},{"varval":"length"},{"varval":"45, 45.2, 45.6, 47, 47"}]}]

  871. <div class="sm_mode"> Which set of numbers is arranged from the smallest to the largest? </div>

    [{"vars":[{"varval":"$-3$"},{"varval":"$-2$"},{"varval":"25$\\%$"},{"varval":"$\\dfrac{5}{4}$"}]},{"vars":[{"varval":"$-2$"},{"varval":"$-1$"},{"varval":"85$\\%$"},{"varval":"$\\dfrac{5}{3}$"}]},{"vars":[{"varval":"$-5$"},{"varval":"$-3$"},{"varval":"10$\\%$"},{"varval":"$\\dfrac{3}{2}$"}]},{"vars":[{"varval":"$-7$"},{"varval":"$-6$"},{"varval":"80$\\%$"},{"varval":"$\\dfrac{5}{4}$"}]},{"vars":[{"varval":"$-5$"},{"varval":"$-4$"},{"varval":"75$\\%$"},{"varval":"$\\dfrac{5}{4}$"}]}]

  872. <div class="sm_mode"> {{name}} creates a pattern with {{object1}}. {{patternimage}} How many {{object2}} are needed for {{figure}} in this pattern? </div>

    [{"vars":[{"varval":"Alan"},{"varval":"square tiles"},{"varval":"sm_img https://teacher.smartermaths.com.au/wp-content/uploads/2020/12/20341_var0.svg 400 indent vpad"},{"varval":"tiles"},{"varval":"Figure 6"},{"varval":"Figure 1: 1\n\nFigure 2: 1 + 4 = 5\n\nFigure 3: 5 + 4 = 9\n\nsm_nogap $\\Rarr$ Add 4 each figure\n\n<div class=\"aligned\">\n\n| | |\n| --------------------- | ------------------- |\n| $\\therefore$ Figure 6 | = 1 + 5 $\\times$ 4 |\n| | = 21 |\n\n</div>"}]},{"vars":[{"varval":"Sydney"},{"varval":"matchsticks"},{"varval":"sm_img https://teacher.smartermaths.com.au/wp-content/uploads/2020/12/20341_var1.svg 480 indent vpad"},{"varval":"matchsticks"},{"varval":"Figure 7"},{"varval":"Figure 1: 4\n\nFigure 2: 4 + 3 = 7\n\nFigure 3: 7 + 3 = 10\n\nsm_nogap $\\Rarr$ Add 3 each figure\n\n<div class=\"aligned\">\n\n| | |\n| --------------------- | ------------------- |\n| $\\therefore$ Figure 7 | = 4 + 6 $\\times$ 3 |\n| | = 22 |\n\n</div>"}]},{"vars":[{"varval":"Xanthia"},{"varval":"matchsticks"},{"varval":"sm_img https://teacher.smartermaths.com.au/wp-content/uploads/2020/12/20341_var2.svg 440 indent vpad"},{"varval":"matchsticks"},{"varval":"Figure 6"},{"varval":"Figure 1: 4\n\nFigure 2: 4 + 9 = 13\n\nFigure 3: 13 + 9 = 22\n\nsm_nogap $\\Rarr$ Add 9 each figure\n\n<div class=\"aligned\">\n\n| | |\n| --------------------- | ------------------- |\n| $\\therefore$ Figure 6 | = 4 + 5 $\\times$ 9 |\n| | = 49 |\n\n</div>"}]},{"vars":[{"varval":"Jack"},{"varval":"triangular tiles"},{"varval":"sm_img https://teacher.smartermaths.com.au/wp-content/uploads/2020/12/20341_var3.svg 440 indent vpad"},{"varval":"tiles"},{"varval":"Figure 5"},{"varval":"Figure 1: 1\n\nFigure 2: 1 + 3 = 4\n\nFigure 3: 4 + 5 = 9\n\nFigure 4: 9 + 7 = 16\n\nFigure 5: 16 + 9 = 25"}]},{"vars":[{"varval":"Alvin"},{"varval":"grey square tiles"},{"varval":"sm_img https://teacher.smartermaths.com.au/wp-content/uploads/2020/12/20341_var4.svg 480 indent vpad"},{"varval":"tiles"},{"varval":"Figure 6"},{"varval":"Figure 1: 8\n\nFigure 2: 8 + 5 = 13\n\nFigure 3: 13 + 5 = 18\n\nsm_nogap $\\Rarr$ Add 5 each figure\n\n<div class=\"aligned\">\n\n| | |\n| --------------------- | ------------------- |\n| $\\therefore$ Figure 6 | = 8 + 5 $\\times$ 5 |\n| | = 33 |\n\n</div>"}]}]

  873. <div class="sm_mode"> {{name1}} starts his morning {{activity1}} at {{time1}} and {{activity2}} for {{duration1}}. {{name2}} starts her morning {{activity1}} at {{time2}} and {{activity2}} for {{duration2}}. Who finished their {{activity1}} first and by how long? </div>

    [{"vars":[{"varval":"Rob"},{"varval":"jog"},{"varval":"6:20 am"},{"varval":"jogs"},{"varval":"1 hour and 49 minutes"},{"varval":"Angie"},{"varval":"6:50 am"},{"varval":"55 minutes"},{"varval":"1 hr 49 m"},{"varval":"8:09 am"},{"varval":"55 m"},{"varval":"7:45 am"}]},{"vars":[{"varval":"Ben"},{"varval":"walk"},{"varval":"5:30 am"},{"varval":"walks"},{"varval":"2 hours and 17 minutes"},{"varval":"Courtney"},{"varval":"6:10 am"},{"varval":"1 hour and 55 minutes"},{"varval":"2 hr 17 m"},{"varval":"7:47 am"},{"varval":"1 hr 55 m"},{"varval":"8:05 am"}]},{"vars":[{"varval":"Chandler"},{"varval":"row"},{"varval":"7:30 am"},{"varval":"rows"},{"varval":"2 hours and 27 minutes"},{"varval":"Debbie"},{"varval":"8:45 am"},{"varval":"57 minutes"},{"varval":"2 hr 23 m"},{"varval":"9:57 am"},{"varval":"57 m"},{"varval":"9:42 am"}]},{"vars":[{"varval":"Steve"},{"varval":"walk"},{"varval":"6:15 am"},{"varval":"walks"},{"varval":"2 hours 46 minutes"},{"varval":"Clare"},{"varval":"7:00 am"},{"varval":"1 hour and 58 minutes"},{"varval":"2 hr 46 min"},{"varval":"9:01 am"},{"varval":"1 hr 58 min"},{"varval":"8:58 am"}]},{"vars":[{"varval":"Dan"},{"varval":"paddle"},{"varval":"5:40 am"},{"varval":"paddles"},{"varval":"1 hour and 39 minutes"},{"varval":"Shelly"},{"varval":"6:10 am"},{"varval":"56 minutes"},{"varval":"1 hr 39 min"},{"varval":"7:19 am"},{"varval":"56 min"},{"varval":"7:06 am"}]}]

  874. <div class="sm_mode"> {{{unknown}}} = {{number1}} $-$ {{number2}} The value of &nbsp;{{{unknown}}}, rounded to the nearest 10 is which of the following? </div>

    [{"vars":[{"varval":"<span class=\"sm-text color4\">?</span>"},{"varval":"368"},{"varval":"126"},{"varval":"242"}]},{"vars":[{"varval":"<span class=\"sm-text color1\">?</span>"},{"varval":"478"},{"varval":"137"},{"varval":"341"}]},{"vars":[{"varval":"<span class=\"sm-text color3\">?</span>"},{"varval":"369"},{"varval":"131"},{"varval":"238"}]},{"vars":[{"varval":"<span class=\"sm-text color4\">?</span>"},{"varval":"457"},{"varval":"135"},{"varval":"322"}]},{"vars":[{"varval":"<span class=\"sm-text color5\">?</span>"},{"varval":"349"},{"varval":"113"},{"varval":"236"}]}]

  875. Algebra, NAPX-G4-NC14

    <div class="sm_mode"> {{{question}}} </div>

    [{"vars":[{"varval":"Peter has 6 more golf balls than Roger.\n\nUsing $\\large p$ for the number of Peter's golf balls and $\\large r$ for Roger's golf balls, which equation correctly describes this fact?"},{"varval":"{{{correctAnswer}}}\n"}]},{"vars":[{"varval":"Bart has 12 less chocolates than Lisa.\n\nUsing $\\large b$ for the number of Bart's chocolates and $\\large l$ for Lisa's chocolates, which equation correctly describes this fact?"},{"varval":"{{{correctAnswer}}}"}]},{"vars":[{"varval":"Ben has 8 more ice creams than Jerry.\n\nUsing $\\large b$ for the number of Ben's ice creams and $\\large j$ for Jerry's ice creams, which equation correctly describes this fact?"},{"varval":"{{{correctAnswer}}}"}]},{"vars":[{"varval":"Anthony made 6 more bowls of fruit salad than twice what Simon made.\n\nUsing $\\large a$ for the number of Anthony's bowls of fruit salad and $\\large s$ for Simon's bowls of fruit salad, which equation correctly describes this fact?"},{"varval":"{{{correctAnswer}}}"}]},{"vars":[{"varval":"Eilidh read 2 less books than three times what Cormac read.\n\nUsing $\\large e$ for the number of books Eilidh has read and $\\large c$ for number of books Cormac has read, which equation correctly describes this fact?"},{"varval":"{{{correctAnswer}}}"}]},{"vars":[{"varval":"Stephen streamed 10 less movies than twice what Bill streamed last month.\n\nUsing $\\large s$ for the number of movies Stephen streamed and $\\large b$ for number of movies Bill streamed, which equation correctly describes this fact?"},{"varval":"{{{correctAnswer}}}"}]}]

  876. Algebra, NAP-C3-CA19

    <div class="sm_mode"> {{{question}}} </div>

    [{"vars":[{"varval":"Kate is 4 years older than Jess.\n\nSarah is 6 years older than Jess.\n\nWhich equation shows how Kate's age relates to Sarah's age?\n"},{"varval":"sm_nogap Sarah is older than Kate.\n\nsm_nogap Difference between Sarah's and Kate's ages\n\n<div class=\"aligned\">\r\n\r\n>>| |\r\n| ---------- |\r\n| \\= $6 \\ − \\ 4$ |\n| \\= 2 |\r\n\r\n\r\n</div>\r\n\n<br>\n\n$\\therefore$ {{{correctAnswer}}}"}]},{"vars":[{"varval":"Jemma is 10 years older than Boris.\n\nMagnus is 15 years older than Boris.\n\nWhich equation shows how Jemma's age relates to Magnus' age?\n"},{"varval":"sm_nogap Magnus is older than Jemma.\n\nsm_nogap Difference between Magnus' and Jemma's ages\n\n<div class=\"aligned\">\r\n\r\n>>| |\r\n| ---------- |\r\n| \\= $15 \\ − \\ 10$ |\n| \\= 5 |\r\n\r\n\r\n</div>\r\n\n<br>\n\n$\\therefore$ {{{correctAnswer}}}"}]},{"vars":[{"varval":"Corey is 14 years older than Louis.\n\nSteph is 20 years older than Louis.\n\nWhich equation shows how Corey's age relates to Steph's age?\n"},{"varval":"sm_nogap Steph is older than Corey.\n\nsm_nogap Difference between Steph's and Corey's ages\n\n<div class=\"aligned\">\r\n\r\n>>| |\r\n| ---------- |\r\n| \\= $20 \\ − \\ 14$ |\n| \\= 6 |\r\n\r\n\r\n</div>\r\n\n<br>\n\n$\\therefore$ {{{correctAnswer}}}"}]},{"vars":[{"varval":"Deiter is 2 years older than Brigette.\n\nBligh is 6 years older than Brigette.\n\nWhich equation shows how Deiter's age relates to Bligh's age?\n"},{"varval":"sm_nogap Bligh is older than Deiter.\n\nsm_nogap Difference between Bligh's and Deiter's ages\n\n<div class=\"aligned\">\r\n\r\n>>| |\r\n| ---------- |\r\n| \\= $6 \\ − \\ 2$ |\n| \\= 4 |\r\n\r\n\r\n</div>\r\n\n<br>\n\n$\\therefore$ {{{correctAnswer}}}"}]},{"vars":[{"varval":"Brooke is 7 years older than Kirsten.\n\nSandy is 10 years older than Kirsten.\n\nWhich equation shows how Brooke's age relates to Sandy's age?\n"},{"varval":"sm_nogap Sandy is older than Brooke.\n\nsm_nogap Difference between Sandy's and Brooke's ages\n\n<div class=\"aligned\">\r\n\r\n>>| |\r\n| ---------- |\r\n| \\= $10 \\ − \\ 7$ |\n| \\= 3 |\r\n\r\n\r\n</div>\r\n\n<br>\n\n$\\therefore$ {{{correctAnswer}}}"}]},{"vars":[{"varval":"Jason is 9 years older than Buck.\n\nLou Lou is 20 years older than Buck.\n\nWhich equation shows how Jason's age relates to Lou Lou's age?\n"},{"varval":"sm_nogap Lou Lou is older than Jason.\n\nsm_nogap Difference between Lou Lou's and Jason's ages\n\n<div class=\"aligned\">\r\n\r\n>>| |\r\n| ---------- |\r\n| \\= $20 \\ − \\ 9$ |\n| \\= 11 |\r\n\r\n\r\n</div>\r\n\n<br>\n\n$\\therefore$ {{{correctAnswer}}}"}]}]

  877. Algebra, NAP-G3-CA21

    <div class="sm_mode"> {{{question}}} </div>

    [{"vars":[{"varval":"Pasta is sold in bags of 3 sizes: small, medium, and large.\n\nA medium bag of pasta costs \\$3.25 more than a small bag.\n\nA large bag of pasta costs \\$4.75 more than a small bag.\n\nWhich equation correctly states the cost of a medium bag of pasta?"},{"varval":"sm_nogap The extra cost of a large pasta over a medium pasta\n\n<div class=\"aligned\">\n\n>>| | \n| ----------------------- |\n|= 4.75 $-$ 3.25|\n|= $1.50 |\n\n</div>\n\n<br>\n\n$\\therefore$ {{{correctAnswer}}}"}]},{"vars":[{"varval":"Olive oil is sold in bottles of 3 sizes: small, medium, and large.\n\nA medium bottle of olive oil costs \\$2.50 more than a small bottle.\n\nA large bottle of olive oil costs \\$5.10 more than a small bottle.\n\nWhich equation correctly states the cost of a medium bottle of olive oil?"},{"varval":"sm_nogap The extra cost of a large bottle over a medium bottle\n\n<div class=\"aligned\">\n\n>>| | \n| ----------------------- |\n|= 5.10 $-$ 2.50|\n|= $2.60 |\n\n</div>\n\n<br>\n\n$\\therefore$ {{{correctAnswer}}}"}]},{"vars":[{"varval":"Engine oil is sold in containers of 3 sizes: small, medium, and large.\n\nA medium container of engine oil costs \\$8.40 more than a small container.\n\nA large container of engine oil costs \\$15.60 more than a small container.\n\nWhich equation correctly states the cost of a medium container of engine oil?"},{"varval":"sm_nogap The extra cost of a large container over a medium container\n\n<div class=\"aligned\">\n\n>>| | \n| ----------------------- |\n|= 15.60 $-$ 8.40|\n|= $7.20 |\n\n</div>\n\n<br>\n\n$\\therefore$ {{{correctAnswer}}}"}]},{"vars":[{"varval":"Breakfast cereal is sold in boxes of 3 sizes: small, medium, and large.\n\nA medium box of breakfast cereal costs \\$2.05 more than a small box.\n\nA large box of breakfast cereal costs \\$4.30 more than a small box.\n\nWhich equation correctly states the cost of a medium box of breakfast cereal?"},{"varval":"sm_nogap The extra cost of a large box over a medium box\n\n<div class=\"aligned\">\n\n>>| | \n| ----------------------- |\n|= 4.30 $-$ 2.05|\n|= $2.25 |\n\n</div>\n\n<br>\n\n$\\therefore$ {{{correctAnswer}}}"}]},{"vars":[{"varval":"Concert tickets are sold in 3 price categories: standard, premium, and VIP.\n\nA premium concert ticket costs $45 more than a standard ticket.\n\nA VIP concert ticket costs $105 more than a standard ticket.\n\nWhich equation correctly states the cost of a premium concert ticket?"},{"varval":"sm_nogap The extra cost of a VIP ticket over a Premium ticket\n\n<div class=\"aligned\">\n\n>>| | \n| ----------------------- |\n|= 105 $-$ 45|\n|= $60 |\n\n</div>\n\n<br>\n\n$\\therefore$ {{{correctAnswer}}}"}]},{"vars":[{"varval":"Pizzas are sold in 3 sizes: Extra Large, Family, and Regular.\n\nA family pizza costs \\$3.50 more than a regular pizza.\n\nAn extra large pizza costs \\$4.90 more than a regular pizza.\n\nWhich equation correctly states the cost of a family pizza?"},{"varval":"sm_nogap The extra cost of an Extra Large pizza over a Family Pizza\n\n<div class=\"aligned\">\n\n>>| | \n| ----------------------- |\n|= 4.90 $-$ 3.50|\n|= $1.40 |\n\n</div>\n\n<br>\n\n$\\therefore$ {{{correctAnswer}}}"}]}]

  878. Algebra, NAP-F3-CA17

    <div class="sm_mode"> {{{question}}} </div>

    [{"vars":[{"varval":"There are 74 year 7 students enrolled at a school.\n\nThere are 12 more girls enrolled than boys.\n\nHow many boys are enrolled in year 7?\n"},{"varval":"Strategy 1\r\n\nBy trial and error using given options:\r\n\n$30 + (30 + 12) = 72$ &nbsp;x\n\n$31 + (31 + 12) = 74$ &nbsp;$\\checkmark$\n\n$\\therefore$ There are {{correctAnswer}} boys enrolled.\n\n<br>\n\nStrategy 2\n\n<div class=\"no-margin-bottom\">\n\nLet &nbsp;$\\large n$ = number of boys enrolled\n\n</div>\n\n<div class=\"no-margin-bottom\">\n\n$\\Rightarrow \\large n$ + 12 = number of girls enrolled\n\n</div>\n\n<div class=\"aligned\">\n\n| | |\n| --------------------: | -------------- |\n| $\\large n + n$ + 12 | \\= 74 |\n| $2\\large n$ | \\= 62 |\n| $\\large n$ | \\= {{correctAnswer}} |\n\n$\\therefore$ There are {{correctAnswer}} boys enrolled.\n\n</div>"}]},{"vars":[{"varval":"There are 190 year 9 students enrolled at a school.\n\n\r\n\r\nThere are 16 more boys enrolled than girls.\n\n\r\n\r\nHow many girls are enrolled in year 9?"},{"varval":"Strategy 1\r\n\nBy trial and error using given options:\r\n\n$87 + (87 + 16) = 190$ &nbsp;$\\checkmark$\n\n$\\therefore$ There are {{correctAnswer}} girls enrolled.\n\n<br>\n\nStrategy 2\n\n<div class=\"no-margin-bottom\">\n\nLet &nbsp;$\\large n$ = number of girls enrolled\n\n</div>\n\n<div class=\"no-margin-bottom\">\n\n$\\Rightarrow \\large n$ + 16 = number of boys enrolled\n\n</div>\n\n<div class=\"aligned\">\n\n| | |\n| --------------------: | -------------- |\n| $\\large n + n$ + 16 | \\= 190 |\n| $2\\large n$ | \\= 174 |\n| $\\large n$ | \\= {{correctAnswer}} |\n\n$\\therefore$ There are {{correctAnswer}} girls enrolled.\n\n</div>"}]},{"vars":[{"varval":"A junior soccer club has 315 registered players.\n\nThere are 51 less boys registered than girls.\n\nHow many of the registered players are girls?\n"},{"varval":"Strategy 1\r\n\nBy trial and error using given options:\r\n\n$132 + (132$ $-$ $51) = 213$ &nbsp;x\n\n$180 + (180$ $-$ $51) = 309$ &nbsp;x\n\n$183 + (183$ $-$ $51) = 315$ &nbsp;$\\checkmark$\n\n$\\therefore$ There are {{correctAnswer}} girls registered.\n\n<br>\n\nStrategy 2\n\n<div class=\"no-margin-bottom\">\n\nLet &nbsp;$\\large n$ = number of girls registered\n\n</div>\n\n<div class=\"no-margin-bottom\">\n\n$\\Rightarrow \\large n$ $-$ 51 = number of boys registered\n\n</div>\n\n<div class=\"aligned\">\n\n| | |\n| --------------------: | -------------- |\n| $\\large n + n$ $-$ 51 | \\= 315 |\n| $2\\large n$ | \\= 315 + 51 |\n| $2\\large n$ | \\= 366 |\n| $\\large n$ | \\= {{correctAnswer}} |\n\n$\\therefore$ There are {{correctAnswer}} girls registered.\n\n</div>"}]},{"vars":[{"varval":"An over 18's concert was attended by 7000 people over the three day event.\n\nThere were 5200 more people in the age group 30 or under than the over 30 age group.\n\nHow many of the concert goers were over the age of 30?\n"},{"varval":"Strategy 1\r\n\nBy trial and error using given options:\r\n\n$900 + (900 + 5200) = 7000$ &nbsp;$\\checkmark$\n\n$\\therefore$ There are {{correctAnswer}} at concert older than 30.\n\n<br>\n\nStrategy 2\n\n<div class=\"no-margin-bottom\">\n\nLet &nbsp;$\\large n$ = number at concert older than 30\n\n</div>\n\n<div class=\"no-margin-bottom\">\n\n$\\Rightarrow \\large n$ + 5200 = number at concert 30 or under\n\n</div>\n\n<div class=\"aligned\">\n\n| | |\n| --------------------: | -------------- |\n| $\\large n + n$ + 5200 | \\= 7000 |\n| $2\\large n$ | \\= 7000 $-$ 5200 |\n| $2\\large n$ | \\= 1800 |\n| $\\large n$ | \\= {{correctAnswer}} |\n\n$\\therefore$ There are {{correctAnswer}} at concert older than 30.\n\n</div>"}]},{"vars":[{"varval":"A rose farm produced a total of 1500 red and white long stemmed roses for Valentine's Day.\n\nThere are 600 less white roses picked than red.\n\nHow many of the roses picked are red?\n"},{"varval":"Strategy 1\r\n\nBy trial and error using given options:\r\n\n$150 + (150$ $-$ $600) = − 300$ &nbsp;x\n\n$750 + (750$ $-$ $600) = 900$ &nbsp;x\n\n$1050 + (1050$ $-$ $600) = 1500$ &nbsp;$\\checkmark$\n\n$\\therefore$ There are {{correctAnswer}} red roses.\n\n<br>\n\nStrategy 2\n\n<div class=\"no-margin-bottom\">\n\nLet &nbsp;$\\large n$ = number of red roses\n\n</div>\n\n<div class=\"no-margin-bottom\">\n\n$\\Rightarrow \\large n$ $-$ 600 = number of white roses\n\n</div>\n\n<div class=\"aligned\">\n\n| | |\n| --------------------: | -------------- |\n| $\\large n + n$ $-$ 600 | \\= 1500 |\n| $2\\large n$ | \\= 1500 + 600 |\n| $2\\large n$ | \\= 2100 |\n| $\\large n$ | \\= {{correctAnswer}} |\n\n$\\therefore$ There are {{correctAnswer}} red roses.\n\n</div>"}]},{"vars":[{"varval":"There are 156 birds in an aviary.\n\nAll the birds are either canaries or finches and there are 82 more finches than canaries.\n\nHow many canaries are in the aviary?\n"},{"varval":"Strategy 1\r\n\nBy trial and error using given options:\r\n\n$34 + (34 + 82) = 150$ &nbsp;x\n\n$37 + (37 + 82) = 156$ &nbsp;$\\checkmark$\n\n$\\therefore$ There are {{correctAnswer}} canaries in the aviary.\n\n<br>\n\nStrategy 2\n\n<div class=\"no-margin-bottom\">\n\nLet &nbsp;$\\large n$ = number of canaries in the aviary\n\n</div>\n\n<div class=\"no-margin-bottom\">\n\n$\\Rightarrow \\large n$ + 82 = number of finches in the aviary\n\n</div>\n\n<div class=\"aligned\">\n\n| | |\n| --------------------: | -------------- |\n| $\\large n + n$ + 82 | \\= 156 |\n| $2\\large n$ | \\= 156 $-$ 82 |\n| $2\\large n$ | \\= 74 |\n| $\\large n$ | \\= {{correctAnswer}} |\n\n$\\therefore$ There are {{correctAnswer}} canaries in the aviary.\n\n</div>"}]}]

  879. Algebra, NAP-K3-CA23

    <div class="sm_mode"> {{{question}}} </div>

    [{"vars":[{"varval":"Leia does a writing course that charges a fee per session and a one off $80 administration fee.\n\nThe overall cost ($C$) is represented by the formula  $C = 25\\large s$ + 80  where $\\large s$ is the number of sessions Leia attends.\n\nLeia attends 6 sessions in total.\n\nHow much does she pay?"},{"varval":"<div class=\"aligned\">\n\n| | |\n| ----------------- | ------------------------ |\n| Cost | = $25\\large s$ + 80 |\n| | = $25 \\times 6 + 80$ |\n| | = {{correctAnswer}} |\n\n</div>"}]},{"vars":[{"varval":"Briony does an online course that charges a fee per session and a one off $100 administration fee.\n\nThe overall cost ($C$) is represented by the formula  $C = 45\\large s$ + 100  where $\\large s$ is the number of sessions Briony completes.\n\nBriony completes 8 sessions in total.\n\nHow much does she pay?"},{"varval":"<div class=\"aligned\">\n\n| | |\n| ----------------- | ------------------------ |\n| Cost | = $45\\large s$ + 100 |\n| | = $45 \\times 8 + 100$ |\n| | = {{correctAnswer}} |\n\n</div>"}]},{"vars":[{"varval":"Angus does a scuba diving course that charges a fee per day and a one off $199 wetsuit purchase fee.\n\nThe overall cost ($C$) is represented by the formula  $C = 135\\large s$ + 199  where $\\large s$ is the number of days Angus attends.\n\nAngus attends 3 days in total.\n\nHow much does he pay?"},{"varval":"<div class=\"aligned\">\n\n| | |\n| ----------------- | ------------------------ |\n| Cost | = $135\\large s$ + 199 |\n| | = $135 \\times 3 + 199$ |\n| | = {{correctAnswer}} |\n\n</div>"}]},{"vars":[{"varval":"Henry does a sailing course that charges a fee per session and a one off reduction of $50 if he books before the end of the month.\n\nThe overall cost ($C$) is represented by the formula  $C = 55\\large s$ $-$ 50  where $\\large s$ is the number of sessions Henry attends.\n\nHenry attends 6 sessions in total.\n\nHow much does he pay?"},{"varval":"<div class=\"aligned\">\n\n| | |\n| ----------------- | ------------------------ |\n| Cost | = $55\\large s$ $-$ 50 |\n| | = $55 \\ \\times$ 6 $-$ 50 |\n| | = {{correctAnswer}} |\n\n</div>"}]},{"vars":[{"varval":"Jo Jo enrols in a graphic design course that charges a fee per session and a one off reduction of $100 if she books before the end of the month.\n\nThe overall cost ($C$) is represented by the formula  $C = 60\\large s$ $-$ 100  where $\\large s$ is the number of sessions Jo Jo completes.\n\nJo Jo completes 5 sessions in total.\n\nHow much does she pay?"},{"varval":"<div class=\"aligned\">\n\n| | |\n| ----------------- | ------------------------ |\n| Cost | = $60\\large s$ $-$ 100 |\n| | = $60 \\ \\times$ 5 $-$ 100 |\n| | = {{correctAnswer}} |\n\n</div>"}]},{"vars":[{"varval":"Benson joins a rock climbing gym that charges a fee per session and a one off joining fee of $60.\n\nThe overall cost ($C$) is represented by the formula  $C = 30\\large s$ + 60  where $\\large s$ is the number of sessions Benson completes.\n\nBenson completes 7 sessions in total.\n\nHow much does he pay?"},{"varval":"<div class=\"aligned\">\n\n| | |\n| ----------------- | ------------------------ |\n| Cost | = $30\\large s$ + 60 |\n| | = $30 \\ \\times$ 7 + 60 |\n| | = {{correctAnswer}} |\n\n</div>"}]}]

  880. Algebra, NAP-I3-NC21

    <div class="sm_mode"> {{{question}}} </div>

    [{"vars":[{"varval":"Stu and Tarly collect sea-shells on the beach.\n\nTarly collects 3 times as many sea-shells as Stu plus an additional 5.\n\nLet $\\large s$ be the number of shells Stu collects.\n\nWhich expression correctly shows the number of shells Tarly collects?\n"},{"varval":"{{{correctAnswer}}}\n"}]},{"vars":[{"varval":"Bick and Dixie collect aluminium cans for recycling.\n\nDixie collects 2 times as many aluminium cans as Bick plus an additional 3.\n\nLet $\\large b$ be the number of aluminium cans Bick collects.\n\nWhich expression correctly shows the number of aluminium cans Dixie collects?\n"},{"varval":"{{{correctAnswer}}}\n"}]},{"vars":[{"varval":"Barclay and Isla picked punnets of strawberries.\n\nIsla picks 4 times as many punnets as Barclay less 3.\n\nLet $\\large b$ be the number of punnets Barclay picks.\n\nWhich expression correctly shows the number of punnets of strawberries Isla picks?\n"},{"varval":"{{{correctAnswer}}}"}]},{"vars":[{"varval":"Brutus and Cindy are watching the same series on a streaming channel.\n\nCindy has watched 2 times as many episodes as Brutus plus an additional 4.\n\nLet $\\large b$ be the number of episodes Brutus has watched.\n\nWhich expression correctly shows the number of episodes Cindy watches?"},{"varval":"{{{correctAnswer}}}"}]},{"vars":[{"varval":"Jacqui and Rina are competing in a swim-a-thon at their local swimming pool.\n\nRina swims 4 times as many laps as Jacqui less 1.\n\nLet $\\large j$ be the number of laps Jacqui swims.\n\nWhich expression correctly shows the number of laps Rina swims?\n"},{"varval":"{{{correctAnswer}}}\n"}]},{"vars":[{"varval":"Blaxland and Lawson hiked over the mountains taking different routes.\n\nLawson hiked 2 times as many kilometres as Blaxland less 6.\n\nLet $\\large b$ be the number of kilometres Blaxland hikes.\n\nWhich expression correctly shows the number of kilometres Lawson hiked?\n"},{"varval":"{{{correctAnswer}}}"}]}]

  881. Algebra, NAP-K3-CA08

    <div class="sm_mode"> {{{question}}} </div>

    [{"vars":[{"varval":"Raphael is 2 years younger than 3 times his sister's age.\n\nIf $\\large s$ represents his sister's age, which expression represents Raphael's age?"},{"varval":"Let $\\ \\large s$ = sister's age\n\n$\\therefore \\text{Raphael's age}$ = {{correctAnswer}}"}]},{"vars":[{"varval":"Peter is 3 years older than twice his sister's age.\n\nIf $\\large x$ represents his sister's age, which expression represents Peter's age?"},{"varval":"Let $\\ \\large x$ = sister's age\n\n$\\therefore \\text{Peter's age}$ = {{correctAnswer}}"}]},{"vars":[{"varval":"Maria is 4 years younger than 5 times her brother's age.\n\nIf $\\large x$ represents her brother's age, which expression represents Maria's age?"},{"varval":"Let $\\ \\large x$ = brother's age\n\n$\\therefore \\text{Maria's age}$ = {{correctAnswer}}"}]},{"vars":[{"varval":"Julio is 10 years older than 2 times his brother's age.\n\nIf $\\large x$ represents his brother's age, which expression represents Julio's age?"},{"varval":"Let $\\ \\large x$ = brother's age\n\n$\\therefore \\text{Julio's age}$ = {{correctAnswer}}"}]},{"vars":[{"varval":"Bernice is 8 years younger than 3 times her sister's age.\n\nIf $\\large x$ represents her sister's age, which expression represents Bernice's age?"},{"varval":"Let $\\ \\large x$ = sister's age\n\n$\\therefore \\text{Bernice's age}$ = {{correctAnswer}}"}]},{"vars":[{"varval":"Oliver is 25 years younger than 3 times his mother's age.\n\nIf $\\large x$ represents his mother's age, which expression represents Oliver's age?"},{"varval":"Let $\\ \\large x$ = mother's age\n\n$\\therefore \\text{Oliver's age}$ = {{correctAnswer}}"}]}]

  882. Algebra, NAPX-G3-CA21

    Soup is sold in cans of 3 sizes: small, medium, and large. A small can of soup costs \$1.40 less than a medium can. A small can of soup costs \$4.20 less than a large can. Which equation correctly states the cost of a medium can of soup?

    [{"vars":null}]

  883. Algebra, NAPX-I4-CA11

    A corner store sells hamburgers for \$7 and milkshakes for \$5 each. Gia wrote the equation $5 \large h$ + 7$\large m$ = $70$ to calculate the number of hamburgers and milkshakes she could buy for exactly \$70. How many hamburgers and milkshakes did Gia buy for \$70?

    [{"vars":null}]

  884. Algebra, NAPX-I4-NC13, APX-I3-NC21

    Ryan and Oliver breed hamsters. Each counts the number they have and Ryan has 2 times as many as Oliver plus another 7. Let $\large h$ be the number of hamsters Oliver has. Which expression represents the number of hamsters Ryan has?

    [{"vars":null}]

  885. Algebra, NAPX-H3-CA12 ISSUE: Incorrect image - table does not fit the question? Table still does not make sense? Can you check the link vs the original.

    sm_img https://teacher.smartermaths.com.au/wp-content/uploads/2018/07/NAPX-H3-CA12.svg 435 indent3 vpad <br> What is the rule connecting dress sizes in Country A and Country B?

    [{"vars":null}]

  886. Algebra, NAPX-G4-NC02

    Twinky-Winky spent three times as much money as La-La. If they spent a total of $120, how much did La-La spend?

    [{"vars":null}]

  887. Algebra, NAP-G4-NC14

    Aaron has \$3 less than Brendon. Using $\large a$ for Aaron's money and $\large b$ for Brendon's money, which equation correctly describes this fact?

    [{"vars":null}]

  888. Algebra, NAP-A4-CA12

    <div class="sm_mode"> {{{question}}} </div>

    [{"vars":[{"varval":"Here is a table of values for $\\large x$ and $\\large y$.\n\n<div class=\"sm-table col1-color7\">\n\n>>| $\\large x$|0|0.5|1|1.5|2|\n|:-:|:-:|:-:|:-:|:-:|:-:|\n| $\\large y$ | 0|1|4|9|16|\n\n</div>\n\n<br>\n\nWhich of these is a correct rule for $\\large y$ in terms of $\\large x$?"},{"varval":"By trial and error for each given equation:\n\n<div class=\"no-margin-bottom\">\n\nConsider &nbsp;$\\large y$ = 4$\\large x$$^2$\n\n</div>\n\n<div class=\"aligned\">\n\n>>| | \n| ----------------------- |\n|$0 = 4 × 0^2$ &nbsp;$\\checkmark$|\n|$1 = 4 × 0.5^2$ &nbsp;$\\checkmark$|\n|$4 = 4 × 1^2$ &nbsp;$\\checkmark$|\n\n<br>\n\n$\\therefore\\ \\large y$ = 4$\\large x^2$ &nbsp;is the correct rule.\n\n</div>"}]},{"vars":[{"varval":"Here is a table of values for $\\large x$ and $\\large y$.\n\n<div class=\"sm-table col1-color3\">\n\n>>| $\\large x$|0|0.5|1|1.5|2|\n|:-:|:-:|:-:|:-:|:-:|:-:|\n| $\\large y$ | 0|2|8|18|32|\n\n</div>\n\n<br>\n\nWhich of these is a correct rule for $\\large y$ in terms of $\\large x$?"},{"varval":"By trial and error for each given equation:\n\n<div class=\"no-margin-bottom\">\n\nConsider &nbsp;$\\large y$ = 8$\\large x$$^2$\n\n</div>\n\n<div class=\"aligned\">\n\n>>| | \n| ----------------------- |\n|$0 = 8 × 0^2$ &nbsp;$\\checkmark$|\n|$2 = 8 × 0.5^2$ &nbsp;$\\checkmark$|\n|$8 = 8 × 1^2$ &nbsp;$\\checkmark$|\n\n<br>\n\n$\\therefore\\ \\large y$ = 8$\\large x^2$ &nbsp;is the correct rule.\n\n</div>"}]},{"vars":[{"varval":"Here is a table of values for $\\large x$ and $\\large y$.\n\n<div class=\"sm-table col1-color8\">\n\n>>| $\\large x$|0|1|2|3|4|\n|:-:|:-:|:-:|:-:|:-:|:-:|\n| $\\large y$ | $-1$|1|3|5|7|\n\n</div>\n\n<br>\n\nWhich of these is a correct rule for $\\large y$ in terms of $\\large x$?"},{"varval":"By trial and error for each given equation:\n\n<div class=\"no-margin-bottom\">\n\nConsider &nbsp;$\\large y$ = 2$\\large x$ $-$ 1\n\n</div>\n\n<div class=\"aligned\">\n\n>>| | \n| ----------------------- |\n|$-1 = 2 × 0 - 1$ &nbsp;$\\checkmark$|\n|$1 = 2 × 1 - 1$ &nbsp;$\\checkmark$|\n|$3 = 2 × 2 - 1$ &nbsp;$\\checkmark$|\n|$5 = 2 × 3 - 1$ &nbsp;$\\checkmark$|\n\n<br>\n\n$\\therefore$ &nbsp;$\\large y$ = 2$\\large x$ $-$ 1 &nbsp;is the correct rule.\n\n</div>"}]},{"vars":[{"varval":"Here is a table of values for $\\large x$ and $\\large y$.\n\n<div class=\"sm-table col1-color7\">\n\n>>| $\\large x$|0|0.5|1|1.5|2|\n|:-:|:-:|:-:|:-:|:-:|:-:|\n| $\\large y$ | 0| 0.5|2|4.5|8|\n\n</div>\n\n<br>\n\nWhich of these is a correct rule for $\\large y$ in terms of $\\large x$?"},{"varval":"By trial and error for each given equation:\n\n<div class=\"no-margin-bottom\">\n\nConsider &nbsp;$\\large y$ = 2$\\large x$$^2$\n\n</div>\n\n<div class=\"aligned\">\n\n>>| | \n| ----------------------- |\n|$0 = 2 × 0^2$ &nbsp;$\\checkmark$|\n|$0.5 = 2 × 0.5^2$ &nbsp;$\\checkmark$|\n|$2 = 2 × 1^2$ &nbsp;$\\checkmark$|\n|$4.5 = 2 × 1.5^2$ &nbsp;$\\checkmark$|\n|$8 = 2 × 2^2$ &nbsp;$\\checkmark$|\n\n\n<br>\n\n$\\therefore\\ \\large y$ = 2$\\large x$$^2$ &nbsp;is the correct rule.\n\n</div>"}]},{"vars":[{"varval":"Here is a table of values for $\\large x$ and $\\large y$.\n\n<div class=\"sm-table col1-color2\">\n\n>>| $\\large x$|0|0.5|1|1.5|2|\n|:-:|:-:|:-:|:-:|:-:|:-:|\n| $\\large y$ | 0|$-$ 0.5|$-$ 2|$-$ 4.5|$-$ 8|\n\n</div>\n\n<br>\n\nWhich of these is a correct rule for $\\large y$ in terms of $\\large x$?"},{"varval":"By trial and error for each given equation:\n\n<div class=\"no-margin-bottom\">\n\nConsider &nbsp;$\\large y$ = $-$ 2$\\large x$$^2$\n\n</div>\n\n<div class=\"aligned\">\n\n>>| | \n| ----------------------- |\n|0 = $-$ $2 × 0^2$ &nbsp;$\\checkmark$|\n|$-$ 0.5 = $-$ $2 × 0.5^2$ &nbsp;$\\checkmark$|\n|$-$ 2 = $-$ $2 × 1^2$ &nbsp;$\\checkmark$|\n|$-$ 4.5 = $-$ $2 × 1.5^2$ &nbsp;$\\checkmark$|\n|$-$ 8 = $-$ $2 × 2^2$ &nbsp;$\\checkmark$|\n\n\n<br>\n\n$\\therefore\\ \\large y$ = $-$ 2$\\large x$$^2$ &nbsp;is the correct rule.\n\n</div>"}]},{"vars":[{"varval":"Here is a table of values for $\\large x$ and $\\large y$.\n\n<div class=\"sm-table col1-color7\">\n\n>>| $\\large x$|0|1|2|3|4|\n|:-:|:-:|:-:|:-:|:-:|:-:|\n| $\\large y$ | 0| 1|4|9|16|\n\n</div>\n\n<br>\n\nWhich of these is a correct rule for $\\large y$ in terms of $\\large x$?"},{"varval":"By trial and error for each given equation:\n\n<div class=\"no-margin-bottom\">\n\nConsider &nbsp;$\\large y$ = $\\large x$$^2$\n\n</div>\n\n<div class=\"aligned\">\n\n>>| | \n| ----------------------- |\n|$0 = 0^2$ &nbsp;$\\checkmark$|\n|$1 = 1^2$ &nbsp;$\\checkmark$|\n|$4 = 2^2$ &nbsp;$\\checkmark$|\n|$9 = 3^2$ &nbsp;$\\checkmark$|\n|$16 = 4^2$ &nbsp;$\\checkmark$|\n\n\n<br>\n\n$\\therefore\\ \\large y$ = $\\large x$$^2$ &nbsp;is the correct rule.\n\n</div>"}]}]

  889. Algebra, NAP-E4-NC11

    <div class="sm_mode"> {{{question}}} </div>

    [{"vars":[{"varval":"Zoe collected 8 baskets of eggs from her chicken coup.\n\nEach basket had the same number of eggs and a total of 96 eggs were collected.\n\nWhich equation shows the average number of eggs,  $\\large x$,  in each basket?\n"},{"varval":"sm_nogap Let &nbsp;$\\large x$ = Eggs in 1 basket\n\n<div class=\"aligned\">\n\n| | |\n| --------------------: | -------------- |\n| $\\large x$ | \\= $\\dfrac {\\text{total eggs}}{\\text{number of baskets}}$ |\n| | |\n| $\\large x$| \\= $\\dfrac{96}{8}$ |\n| $\\therefore \\large x$ $×\\ 8$ | \\= $96$ |\n\n</div>\n"}]},{"vars":[{"varval":"Beau picked 18 baskets of peaches from the orchard.\n\nEach basket had the same number of peaches and a total of 450 peaches were picked.\n\nWhich equation shows the average number of peaches,  $\\large x$,  in each basket?\n"},{"varval":"sm_nogap Let &nbsp;$\\large x$ = Peaches in 1 basket\n\n<div class=\"aligned\">\n\n| | |\n| --------------------: | -------------- |\n| $\\large x$ | \\= $\\dfrac {\\text{total peaches}}{\\text{number of baskets}}$ |\n| | |\n| $\\large x$| \\= $\\dfrac{450}{18}$ |\n| $\\therefore \\large x$ $×\\ 18$ | \\= $450$ |\n\n</div>\n"}]},{"vars":[{"varval":"Belinda raised money for charity on 12 consecutive days.\n\nEach day she collected same amount of money and a total of $1320 was collected.\n\nWhich equation shows the average amount collected,  $\\large x$, each day?\n"},{"varval":"sm_nogap Let &nbsp;$\\large x$ = Amount collected in 1 day\n\n<div class=\"aligned\">\n\n| | |\n| --------------------: | -------------- |\n| $\\large x$ | \\= $\\dfrac {\\text{total collected}}{\\text{number of days}}$ |\n| | |\n| $\\large x$| \\= $\\dfrac{1320}{12}$ |\n| $\\therefore \\large x$ $×\\ 12$ | \\= $1320$ |\n\n</div>"}]},{"vars":[{"varval":"Sam worked for 35 hours last week.\n\nEach hour he worked he earned the same amount of money and he earned a total of $630 last week.\n\nWhich equation shows the average amount Sam earned,  $\\large x$, per hour?\n"},{"varval":"sm_nogap Let &nbsp;$\\large x$ = Amount earned in 1 hour\n\n<div class=\"aligned\">\n\n| | |\n| --------------------: | -------------- |\n| $\\large x$ | \\= $\\dfrac {\\text{total earned}}{\\text{number of hours}}$ |\n| | |\n| $\\large x$| \\= $\\dfrac{630}{35}$ |\n| $\\therefore \\large x$ $×\\ 35$ | \\= $630$ |\n\n</div>"}]},{"vars":[{"varval":"Levi collected 42 kilograms of apples from the orchard.\n\nEach kilogram had the same number of apples and a total of 336 apples were picked.\n\nWhich equation shows the average number of apples,  $\\large x$,  per kilogram?"},{"varval":"sm_nogap Let &nbsp;$\\large x$ = Apples in 1 kilogram\n\n<div class=\"aligned\">\n\n| | |\n| --------------------: | -------------- |\n| $\\large x$ | \\= $\\dfrac {\\text{total apples}}{\\text{number of kilograms}}$ |\n| | |\n| $\\large x$| \\= $\\dfrac{336}{42}$ |\n| $\\therefore \\large x$ $×\\ 42$ | \\= $336$ |\n\n</div>\n"}]},{"vars":[{"varval":"Ronan planted 25 boxes of seedlings.\n\nEach box had the same number of seedlings and a total of 1000 seedlings were planted.\n\nWhich equation shows the average number of seedlings,  $\\large x$,  per box?"},{"varval":"sm_nogap Let &nbsp;$\\large x$ = Seedlings in 1 box\n\n<div class=\"aligned\">\n\n| | |\n| --------------------: | -------------- |\n| $\\large x$ | \\= $\\dfrac {\\text{total seedlings}}{\\text{number of boxes}}$ |\n| | |\n| $\\large x$| \\= $\\dfrac{1000}{25}$ |\n| $\\therefore \\large x$ $×\\ 25$ | \\= $1000$ |\n\n</div>"}]}]

  890. Algebra, NAP-L4-CA12

    <div class="sm_mode"> {{{question}}} </div>

    [{"vars":[{"varval":"A kayak can be hired for different numbers of hours, as shown in the table below.\n\n<br>\n\n<div class=\"sm-table col1-color7\">\n\n>>| Number of hours | 1 |2|3|4|5|6|\n|:-:|:-:|:-:|:-:|:-:|:-:|:-:|\n| Cost ($) | 30|40|50|60|70|80|\n\n</div>\n\n<br>\n\nSelect the answer that shows how to calculate the cost of hiring a kayak."},{"varval":"By trial and error,\n\nConsider the 4th option:\n\n\\$10 for every hour plus \\$20\n\nTest:\n\n 2 hours = 2 × 10 + 20 =\\$40 ✔\n\n 5 hours = 5 × 10 + 20 =\\$70 ✔"}]},{"vars":[{"varval":"A sailing boat can be hired for different numbers of hours, as shown in the table below.\n\n<br>\n\n<div class=\"sm-table col1-color8\">\n\n>>| Number of hours | 1 |2|3|4|5|6|\n|:-:|:-:|:-:|:-:|:-:|:-:|:-:|\n| Cost ($) | 30|45|60|75|90|105|\n\n</div>\n\n<br>\n\nSelect the answer that shows how to calculate the cost of hiring one of the sailing boats."},{"varval":"By trial and error,\n\nConsider the 3rd option:\n\n$15 for every hour plus $15\n\nTest:\n\n 2 hours = 2 × 15 + 15 = $45 ✔\n\n 5 hours = 5 × 15 + 15 = $90 ✔"}]}]

  891. Algebra, NAP-I4-NC13

    {{{question}}}

    [{"vars":[{"varval":"Ryan and Oliver are buying goldfish for their fish tank.\n\nOliver buys 3 times the number of goldfish as Ryan does, plus another 5.\n\nLet $\\large m$ be the number of goldfish that Ryan buys.\n\nWhich expression represents the number of goldfish Oliver buys?"},{"varval":"sm_nogap Goldfish bought by Oliver\n\n<div class=\"aligned\">\n\n>>| | \n| ----------------------- |\n|= {{{correctAnswer}}}|\n\n</div>"}]},{"vars":[{"varval":"Gilbo and Burger breed hamsters.\n\nEach counts the number they have and Gilbo has 2 times as many as Burger plus another 5.\n\nLet $\\large b$ be the number of hamsters Burger has.\n\nWhich expression represents the number of hamsters Gilbo has?"},{"varval":"sm_nogap Gilbo's hamsters\n\n<div class=\"aligned\">\n\n>>| | \n| ----------------------- |\n|= {{{correctAnswer}}} |\n\n</div>"}]}]

  892. Algebra, NAPX-J4-CA08, NAPX-J3-CA16.

    <div class="sm_mode"> Ringo places a disc at (−5, −1). He then moves the disc up 2 units. What are the coordinates of the new position of the disc? <br> sm_img https://teacher.smartermaths.com.au/wp-content/uploads/2018/04/NAPX-J4-CA08.svg 430 indent vpad </div>

    [{"vars":null}]

  893. Algebra, NAPX-I4-NC05, NAPX-I3-NC07

    <div class="sm_mode"> Point $P$ is translated to the left 3 units. sm_img https://teacher.smartermaths.com.au/wp-content/uploads/2018/04/NAPX-I4-NC05.svg 300 indent3 vpad What are the coordinates of the new position of point $P$? </div>

    [{"vars":null}]

  894. Algebra, NAP-B3-NC13

    <div class="sm_mode"> Jamie is drawing a square on this grid. He has drawn two corner points as shown. <br> sm_img https://teacher.smartermaths.com.au/wp-content/uploads/2017/02/NAP-B3-NC13.png 300 indent vpad <br>Jamie draws the third corner at (5, 6). Where will the other corner of the square be? </div>

    [{"vars":null}]

  895. Algebra, NAP-J3-CA16

    <div class="sm_mode"> Bobby places a disc at  (4, −2). He then moves the disc up 4 units. What are the coordinates of the new position of the disc? <br> sm_img https://teacher.smartermaths.com.au/wp-content/uploads/2017/09/NAP-J1-081.png 380 indent vpad </div>

    [{"vars":null}]

  896. Algebra, NAP-I3-NC07

    <div class="sm_mode"> {{{question}}} </div>

    [{"vars":[{"varval":"Point $Q$ is translated down 3 units.\n\n<br>\n\nsm_img https://teacher.smartermaths.com.au/wp-content/uploads/2017/01/NAP-2016-Y7-NC07.png 290 indent3 vpad\n\n<br>What are the new coordinates of $Q$?\n"},{"varval":"$Q$ has coordinates ( 4 , 2 )\n\nWhen translated 3 down, the $\\large x$-value stays the same and the \n$\\large y$-value decreases by 3.\n\n$\\therefore$ New coordinates are (4 , 2 $-$ 3 ) = {{{correctAnswer}}}."}]},{"vars":[{"varval":"Point $M$ is translated to the right 5 units.\n\n<br>\n\nsm_img https://teacher.smartermaths.com.au/wp-content/uploads/2023/08/Algebra-NAP_20333v1.svg 300 indent2 vpad\n\n<br>What are the new coordinates of $M$?\n"},{"varval":"$M$ has coordinates ( $-$ 1 , 2 )\n\nWhen translated 5 right, the $\\large x$-value increases by 5 and the \n$\\large y$-value stays the same.\n\n$\\therefore$ New coordinates are ( $-$ 1 + 5 , 2 ) = {{{correctAnswer}}}."}]},{"vars":[{"varval":"Point $B$ is translated left 4 units.\n\n<br>\n\nsm_img https://teacher.smartermaths.com.au/wp-content/uploads/2023/08/Algebra-NAP_20333v2.svg 300 indent2 vpad\n\n<br>What are the new coordinates of $B$?\n"},{"varval":"$Q$ has coordinates ( 3 , $-$ 2 )\n\nWhen translated 4 left, the $\\large x$-value decreases by 4 and the \n$\\large y$-value stays the same.\n\n$\\therefore$ New coordinates are ( 3 $-$ 4 , $-$ 2) = {{{correctAnswer}}}."}]},{"vars":[{"varval":"Point $A$ is translated up 5 units.\n\n<br>\n\nsm_img https://teacher.smartermaths.com.au/wp-content/uploads/2023/08/Algebra-NAP_20333v3.svg 300 indent2 vpad\n\n<br>What are the new coordinates of $A$?\n"},{"varval":"$A$ has coordinates ($-$ 2, $-$ 3 )\n\nWhen translated 5 up, the $\\large x$-value stays the same and the \n$\\large y$-value increases by 5.\n\n$\\therefore$ New coordinates are ($-$ 2, $-$ 3 + 5 ) = {{{correctAnswer}}}."}]},{"vars":[{"varval":"Point $C$ is translated down 3 units and left 2 units.\n\n<br>\n\nsm_img https://teacher.smartermaths.com.au/wp-content/uploads/2023/08/Algebra-NAP_20333v4.svg 320 indent2 vpad\n\n\n<br>What are the new coordinates of $C$?\n"},{"varval":"$C$ has coordinates ( 4 , 2 )\n\nWhen translated 3 down and 2 left the $\\large x$-value decreases by 2 and the $\\large y$-value decreases by 3.\n\nsm_img https://teacher.smartermaths.com.au/wp-content/uploads/2023/08/Algebra-NAP_20333v4ws-min.svg 320 indent2 vpad\n\n$\\therefore$ New coordinates are ( 4 $-$ 2 , 2 $-$ 3 ) = {{{correctAnswer}}}."}]},{"vars":[{"varval":"Point $D$ is translated right 6 units and down 5 units.\n\n<br>\n\nsm_img https://teacher.smartermaths.com.au/wp-content/uploads/2023/08/Algebra-NAP_20333v5.svg 300 indent2 vpad\n\n\n<br>What are the new coordinates of $D$?\n"},{"varval":"$D$ has coordinates ( $-$ 2 , 3 )\n\nWhen translated right 6 and 3 down, the $\\large x$-value increases by 6 and the $\\large y$-value decreases by 5.\n\nsm_img https://teacher.smartermaths.com.au/wp-content/uploads/2023/08/Algebra-NAP_20333v5ws-min.svg 300 indent2 vpad\n\n\n$\\therefore$ New coordinates are ( $-$ 2 + 6 , 3 $-$ 5 ) = {{{correctAnswer}}}."}]},{"vars":[{"varval":"Point $W$ is translated left 7 units and down 4 units.\n\n<br>\n\nsm_img https://teacher.smartermaths.com.au/wp-content/uploads/2023/08/Algebra-NAP_20333v6.svg 300 indent2 vpad\n\n<br>What are the new coordinates of $W$?"},{"varval":"$W$ has coordinates ( 4 , 5 )\n\nWhen translated left 7 and 4 down, the $\\large x$-value decreases by 7 and the $\\large y$-value decreases by 4.\n\nsm_img https://teacher.smartermaths.com.au/wp-content/uploads/2023/08/Algebra-NAP_20333v6ws-min.svg 300 indent2 vpad\n\n\n$\\therefore$ New coordinates are ( 4 $-$ 7 , 5 $-$ 4 ) = {{{correctAnswer}}}."}]}]

  897. Algebra, NAPX-H4-CA20

    <div class="sm_mode"> {{{question}}} </div>

    [{"vars":[{"varval":"The cost of hiring a stand-up paddle board for $\\large d$ days is\n\n> Hire cost = 25 + 12.5$\\large d$\n\n<br>\n\nIf Mick has \\$250 to spend, what is the maximum number of days he can hire the board for?"},{"varval":"<div class=\"aligned\">\n\n| | |\n| ----------------------: | ------------------------ |\n| $25 + 12.5\\large d$ | \\= 250 |\n| $12.5\\large d$ | \\= 250 $-$ 25 |\n| $\\therefore \\large d$ | \\= $\\dfrac{225}{12.5}$ |\n| | \\= {{{correctAnswer}}} |\n\n</div>"}]},{"vars":[{"varval":"The cost of hiring an electric bicycle for $\\large h$ hours is\n\n> Hire cost = 40 + 30.25$\\large h$\n\n<br>\n\nIf Boris has \\$282 to spend, what is the maximum number of hours he can hire the electric bicycle for?"},{"varval":"<div class=\"aligned\">\n\n| | |\n| ----------------------: | ------------------------ |\n| $40 + 30.25\\large h$ | \\= 282 |\n| $30.25\\large h$ | \\= 282 $-$ 40 |\n| $\\therefore \\large h$ | \\= $\\dfrac{242}{30.25}$ |\n| | \\= {{{correctAnswer}}} |\n\n</div>"}]},{"vars":[{"varval":"The cost of hiring a box trailer for $\\large d$ days is\n\n> Hire cost = 100 + 48$\\large d$\n\n<br>\n\nIf Jace has \\$340 to spend, what is the maximum number of days he can hire the box trailer for?"},{"varval":"<div class=\"aligned\">\n\n| | |\n| ----------------------: | ------------------------ |\n| $100 + 48\\large d$ | \\= 340 |\n| $48\\large d$ | \\= 340 $-$ 100 |\n| $\\therefore \\large d$ | \\= $\\dfrac{240}{48}$ |\n| | \\= {{{correctAnswer}}} |\n\n</div>"}]},{"vars":[{"varval":"The cost of hiring a steam cleaner for $\\large h$ hours is\n\n> Hire cost = 60 + 15.8$\\large h$\n\n<br>\n\nIf Bronnie has \\$139 to spend, what is the maximum number of hours she can hire the steam cleaner for?"},{"varval":"<div class=\"aligned\">\n\n| | |\n| ----------------------: | ------------------------ |\n| $60 + 15.8\\large h$ | \\= 139 |\n| $15.8\\large h$ | \\= 139 $-$ 60 |\n| $\\therefore \\large h$ | \\= $\\dfrac{79}{15.8}$ |\n| | \\= {{{correctAnswer}}} |\n\n</div>"}]},{"vars":[{"varval":"The cost of hiring a truck for $\\large d$ days is\n\n> Hire cost = 100 + 320$\\large d$\n\n<br>\n\nIf Ferris has \\$3940 to spend, what is the maximum number of days he can hire the truck for?"},{"varval":"<div class=\"aligned\">\n\n| | |\n| ----------------------: | ------------------------ |\n| $100 + 320\\large d$ | \\= 3940 |\n| $320\\large d$ | \\= 3940 $-$ 100 |\n| $\\therefore \\large d$ | \\= $\\dfrac{3840}{320}$ |\n| | \\= {{{correctAnswer}}} |\n\n</div>"}]},{"vars":[{"varval":"The cost of hiring a wedding venue for $\\large h$ hours is\n\n> Hire cost = 700 + 120$\\large d$\n\n<br>\n\nIf Romeo has \\$2140 to spend, what is the maximum number of hours he can hire the wedding venue for?"},{"varval":"<div class=\"aligned\">\n\n| | |\n| ----------------------: | ------------------------ |\n| $700 + 120\\large h$ | \\= 2140 |\n| $120\\large h$ | \\= 2140 $-$ 700 |\n| $\\therefore \\large h$ | \\= $\\dfrac{1440}{120}$ |\n| | \\= {{{correctAnswer}}} hours |\n\n</div>"}]}]

  898. Algebra, NAPX-H4-NC11

    <div class="sm_mode"> Elvis walks from his home to the beach through a park. <br> sm_img https://teacher.smartermaths.com.au/wp-content/uploads/2018/05/NAPX-H4-NC11-SA1.svg 325 indent vpad <br>Which of the following situations best fits the distance/time graph above, where distance is Elvis' distance from home? </div>

    [{"vars":null}]

  899. ebra, NAPX-J4-CA14

    <div class="sm_mode"> Dom and Marty each bought saffron from the local farmers' market. Dom bought 4 grams for \$5. Marty bought 6 grams for \$7.50. Which graph best represents the cost of saffron at the farmers' market? </div>

    [{"vars":null}]

  900. Algebra, NAPX-I4-CA09

    <div class="sm_mode"> {{{question}}} </div>

    [{"vars":[{"varval":"A nursery sells potting mix in two sizes.\n\nA 30 kilogram bag costs \\$5.50 and a 60 kilogram bag costs \\$11.00.\n\nWhich graph could show the relationship between the amount of potting mix and its cost?​"},{"varval":"{{{correctAnswer}}}\n"}]},{"vars":[{"varval":"A nursery sells potting mix in two sizes.\n\nA 25 kilogram bag costs \\$4.50 and a 50 kilogram bag costs \\$9.00.\n\nWhich graph could show the relationship between the amount of potting mix and its cost?​"},{"varval":"{{{correctAnswer}}}"}]},{"vars":[{"varval":"A nursery sells potting mix in two sizes and adds a flat rate of $2 for delivery in the local area.\n\nA 60 kilogram bag costs \\$7.00 and a 30 kilogram bag costs \\$3.50.\n\nWhich graph could show the relationship between the amount of potting mix and its cost?​"},{"varval":"{{{correctAnswer}}}"}]},{"vars":[{"varval":"A nursery sells potting mix in 60 kilogram bags for a cost of \\$5.50 per bag and adds a flat rate of $5.50 for delivery in the local area.\n\nWhich graph could show the relationship between the amount of potting mix and its cost?​"},{"varval":"{{{correctAnswer}}}"}]}]

  901. lgebra, NAP-J4-CA14

    <div class="sm_mode"> {{{question}}} </div>

    [{"vars":[{"varval":"Paddy and Miffy each bought potatoes from the local farmers' market.\n\nPaddy bought 2 kilograms for \\$2.50.\n\nMiffy bought 6 kilograms for \\$7.50.\n\nWhich graph best represents the cost of potatoes at the farmers' market?\n"},{"varval":"{{{correctAnswer}}}"}]},{"vars":[{"varval":"Jake and Amie each bought apples from the local orchard.\n\nJake bought 2 kilograms for \\$3.00.\n\nAmie bought 6 kilograms for \\$9.00\n\nWhich graph best represents the cost of apples at the local orchard?\n"},{"varval":"{{{correctAnswer}}}"}]},{"vars":[{"varval":"Billy and Bob each bought grapes from the local vineyard.\n\nBilly bought 2 kilograms for \\$2.00.\n\nBob bought 9 kilograms for \\$9.00.\n\nWhich graph best represents the cost of grapes at the local vineyard?\n"},{"varval":"{{{correctAnswer}}}"}]},{"vars":[{"varval":"\nBalou and Akela each bought bananas from the local market.\n\nBalou bought 4 kilograms for \\$3.20.\n\nAkela bought 5 kilograms for \\$4.00\n\nWhich graph best represents the cost of bananas at the local market?\n\n"},{"varval":"{{{correctAnswer}}}"}]},{"vars":[{"varval":"Buzz and Darby each bought rice at the supermarket.\n\nBuzz bought 2 kilograms for \\$2.50.\n\nDarby bought 5 kilograms for \\$6.25\n\nWhich graph best represents the cost of rice at the supermarket?\n\n"},{"varval":"{{{correctAnswer}}}"}]},{"vars":[{"varval":"Nemo and Dory each bought fish food at the pet shop.\n\nNemo bought 1 kilograms for \\$1.50.\n\nDory bought 5 kilograms for \\$7.50\n\nWhich graph best represents the cost of fish food at the pet shop?\n"},{"varval":"{{{correctAnswer}}}"}]}]

  902. Algebra, NAP-K4-CA15

    <div class="sm_mode"> {{{question}}} </div>

    [{"vars":[{"varval":"Surf Legends charge an hourly rate for surfing lessons and add a one-off charge of \\$50 for insurance.\n\nThe overall cost ($C$) is represented by the formula, $C = 40\\large h +$ 50, where $\\large h$ is the number of hours of lessons.\n\nKelly has 15 hours of lessons with Surf Legends.\n\nHow much does he pay?\n"},{"varval":"<div class=\"aligned\">\n\n| | |\n| --------------------- | -------------------------------------------- |\n| $C$ | = 40 $\\times$ 15 + 50 |\n| | = 600 + 50 |\n| | = {{{correctAnswer}}} |\n\n</div>"}]},{"vars":[{"varval":"Surf Legends charge an hourly rate for surfing lessons and add a one-off charge of \\$50 for insurance.\n\nThe overall cost ($C$) is represented by the formula, $C = 40\\large h +$ 50, where $\\large h$ is the number of hours of lessons.\n\nAmber's surf lessons with Surf Legends cost $330.\n\nHow many hours of lessons did she have?\n"},{"varval":"<div class=\"aligned\">\n\n| | |\n| ---------------------: | -------------------------------------------- |\n| 40$\\large h$ + 50 | = 330 |\n| 40$\\large h$ |\\= 330 $-$ 50 |\n| 40$\\large h$ |\\= 280|\n| $\\large h$ | \\= {{{correctAnswer}}} |\n\n</div>\n"}]},{"vars":[{"varval":"DriveToday charge an hourly rate for driving lessons and add a one-off charge of \\$38 for insurance.\n\nThe overall cost ($C$) is represented by the formula, $C = 25\\large h +$ 38, where $\\large h$ is the number of hours of lessons.\n\nBenjamin has 5 hours of lessons with DriveToday.\n\nHow much does he pay?\n"},{"varval":"<div class=\"aligned\">\n\n| | |\n| --------------------- | -------------------------------------------- |\n| $C$ | = 25 $\\times$ 5 + 38 |\n| | = 125 + 38 |\n| | = {{{correctAnswer}}} |\n\n</div>"}]},{"vars":[{"varval":"DriveToday charge an hourly rate for driving lessons and add a one-off charge of \\$38 for insurance.\n\nThe overall cost ($C$) is represented by the formula, $C = 25\\large h +$ 38, where $\\large h$ is the number of hours of lessons.\n\nRosa's driving lessons with DriveToday cost $363.\n\nHow many hours of lessons did she have?\n"},{"varval":"<div class=\"aligned\">\n\n| | |\n| ---------------------: | -------------------------------------------- |\n| 25$\\large h$ + 38 | = 363 |\n| 25$\\large h$ |\\= 363 $-$ 38 |\n| 25$\\large h$ |\\= 325|\n| $\\large h$ | \\= {{{correctAnswer}}} |\n\n</div>\n"}]},{"vars":[{"varval":"High Tree Adventures charge an hourly rate for the high ropes course and add a one-off charge of \\$55 for insurance.\n\nThe overall cost ($C$) is represented by the formula, $C = 21\\large h +$ 55, where $\\large h$ is the number of hours of on the course.\n\nDora spends 1.5 hours on the high ropes course.\n\nHow much does she pay?\n"},{"varval":"<div class=\"aligned\">\n\n| | |\n| --------------------- | -------------------------------------------- |\n| $C$ | = 21 $\\times$ 1.5 + 55 |\n| | = 31.50 + 55 |\n| | = {{{correctAnswer}}} |\n\n</div>"}]},{"vars":[{"varval":"High Tree Adventures charge an hourly rate for the high ropes course and add a one-off charge of $55 for insurance.\n\nThe overall cost ($C$) is represented by the formula, $C = 21\\large h +$ 55, where $\\large h$ is the number of hours on the course.\n\nBanjo's charge for the high ropes course was $102.25.\n\nFor how many hours was he on the high ropes course?\n"},{"varval":"<div class=\"aligned\">\n\n| | |\n| ---------------------: | -------------------------------------------- |\n|21$\\large h$ + 55 | = 102.25 |\n| 21$\\large h$ |\\= 102.25 $-$ 55 |\n| 21$\\large h$ |\\= 47.25|\n| $\\large h$ | \\= {{{correctAnswer}}} |\n\n</div>\n"}]}]

  903. gebra, NAP-D4-NC14

    <div class="sm_mode"> Farmer Jim uses 5-strand wire fencing around his paddocks. His fences have 3 barbed wire strands and 2 plain wire strands, as shown in the diagram below. <br> sm_img https://teacher.smartermaths.com.au/wp-content/uploads/2017/02/NAP-D4-NC14.png 300 indent3 vpad <br>Barbed wire costs $\text{\textdollar}\large b$ per metre. Plain wire costs $\text{\textdollar}\large w$ per metre. Which of these expressions gives the total cost of the wire needed for a fence of length $f$ metres? </div>

    [{"vars":null}]

  904. Algebra, NAP-I4-CA09

    <div class="sm_mode"> {{{question}}} </div>

    [{"vars":[{"varval":"At a pool supply store, 20 kilograms of salt costs \\$6.50 and 40 kilograms of the same salt costs \\$13.00.\n\nWhich graph could show the relationship between the amount of salt and its cost?\n"},{"varval":"{{{correctAnswer}}}\n"}]},{"vars":[{"varval":"At a pool supply store, 20 kilograms of salt costs $8.50 and 40 kilograms of the same salt costs $17.00.\n\n\r\nWhich graph could show the relationship between the amount of salt and its cost?\r\n"},{"varval":"{{{correctAnswer}}}\n"}]},{"vars":[{"varval":"At a pool supply store, 15 kilograms of salt costs $6.00 and 30 kilograms of the same salt costs $12.00.\r\nWhich graph could show the relationship between the amount of salt and its cost?\r\n\n"},{"varval":"{{{correctAnswer}}}\n"}]},{"vars":[{"varval":"At a pool supply store, 20 kilograms of salt costs $7.00 and 40 kilograms of the same salt costs $14.00.\n\n\r\nWhich graph could show the relationship between the amount of salt and its cost?\r\n"},{"varval":"{{{correctAnswer}}}"}]},{"vars":[{"varval":"At a pool supply store, 10 kilograms of salt costs $4.00 and 30 kilograms of the same salt costs $12.00.\r\n\nWhich graph could show the relationship between the amount of salt and its cost?\r\n"},{"varval":"{{{correctAnswer}}}"}]},{"vars":[{"varval":"At a pool supply store, 20 kilograms of salt costs $7.50 and 40 kilograms of the same salt costs $15.00.\r\n\nWhich graph could show the relationship between the amount of salt and its cost?\r\n"},{"varval":"{{{correctAnswer}}}"}]}]

  905. Algebra, NAP-I4-CA11

    <div class="sm_mode"> {{{question}}} </div>

    [{"vars":[{"varval":"A store sells cups for \\$4 each and bowls for \\$5 each.\n\nAlex wrote the equation  $4 \\large c$ + 5$\\large b$ = 65  to calculate the number of cups and bowls he could buy for exactly \\$65.\n\nHow many cups and bowls did Alex buy for \\$65?"},{"varval":"By trial and error\n\nConsider the 3rd option:\n\n<div class=\"sm_mode\">\n\n(4 $\\times$ 5) + (5 $\\times$ 9) = 20 + 45 = \\$65\n\n$\\therefore$ {{{correctAnswer}}}\n\n</div>"}]},{"vars":[{"varval":"A store sells mugs for \\$2 each and plates for \\$4 each.\n\nBianca wrote the equation  $2 \\large m$ + 4$\\large p$ = 42  to calculate the number of mugs and plates she could buy for exactly \\$42.\n\nHow many mugs and plates did Bianca buy for \\$42?"},{"varval":"By trial and error\n\nConsider the 2nd option:\n\n<div class=\"sm_mode\">\n\n(2 $\\times$ 5) + (4 $\\times$ 8) = 10 + 32 = \\$65\n\n$\\therefore$ {{{correctAnswer}}}\n\n</div>"}]},{"vars":[{"varval":"A store sells hand towels for \\$8 each and bath towels for \\$15 each.\n\nBrent wrote the equation  $8 \\large h$ + 15$\\large b$ = 84  to calculate the number of hand towels and bath towels he could buy for exactly \\$84.\n\nHow many hand towels and bath towels did Brent buy for \\$84?"},{"varval":"By trial and error\n\nConsider the 3rd option:\n\n<div class=\"sm_mode\">\n\n(8 $\\times$ 3) + (15 $\\times$ 4) = 24 + 60 = \\$84\n\n$\\therefore$ {{{correctAnswer}}}\n\n</div>"}]},{"vars":[{"varval":"A store sells teaspoons for \\$2 each and forks for \\$3 each.\n\nPriscilla wrote the equation  $2 \\large t$ + 3$\\large f$ = 61  to calculate the number of teaspoons and forks she could buy for exactly \\$61.\n\nHow many teaspoons and forks did Priscilla buy for \\$61?"},{"varval":"By trial and error\n\nConsider the 1st option:\n\n<div class=\"sm_mode\">\n\n(2 $\\times$ 8) + (3 $\\times$ 15) = 16 + 45 = \\$61\n\n$\\therefore$ {{{correctAnswer}}}\n\n</div>"}]},{"vars":[{"varval":"A store sells small candles for \\$6 each and large candles for \\$10.50 each.\n\nBeau wrote the equation  $6 \\large s$ + 10.50$\\large l$ = 90  to calculate the number of small candles and large candles he could buy for exactly \\$90.\n\nHow many small candles and large candles did Beau buy for \\$90?"},{"varval":"By trial and error\n\nConsider the 4th option:\n\n<div class=\"sm_mode\">\n\n(6 $\\times$ 8) + (10.50 $\\times$ 4) = 48 + 42 = \\$90\n\n$\\therefore$ {{{correctAnswer}}}\n\n</div>"}]},{"vars":[{"varval":"A store sells pens for \\$4.50 each and rulers for \\$1.50 each.\n\nCandy wrote the equation  $4.50 \\large p$ + 1.50$\\large r$ = 40.50  to calculate the number of pens and rulers she could buy for exactly \\$40.50.\n\nHow many pens and rulers did Candy buy for \\$40.50?"},{"varval":"By trial and error\n\nConsider the 2nd option:\n\n<div class=\"sm_mode\">\n\n(4.50 $\\times$ 6) + (10.50 $\\times$ 9) = 27 + 13.50 = \\$40.50\n\n$\\therefore$ {{{correctAnswer}}}\n\n</div>"}]}]

  906. Algebra, NAP-D4-NC08

    <div class="sm_mode"> Jeremy sold ice creams out of his ice cream truck. He drew the graph below to show how the number of ice creams he sells in a week is related to their price. <br> sm_img https://teacher.smartermaths.com.au/wp-content/uploads/2017/02/NAP-D3-NC10.png 490 indent vpad <br>Which statement best describes the graph? </div>

    [{"vars":null}]

  907. Q51

    <div class="sm_mode"> {{name}} wants to make as many different {{type1}} as possible using the {{item1}}: <br> >>>{{item2}} <br>{{phrase}}, how many different {{type2}} can she make? </div>

    [{"vars":[{"varval":"Dani"},{"varval":"four-digit numbers"},{"varval":"numbers"},{"varval":"sm_img https://teacher.smartermaths.com.au/wp-content/uploads/2020/08/Q52var1.svg 200 indent vpad"},{"varval":"In total"},{"varval":"numbers"},{"varval":"digit"},{"varval":"4"},{"varval":"3"},{"varval":"2"}]},{"vars":[{"varval":"Cynthia"},{"varval":"letter arrangements"},{"varval":"four letters"},{"varval":"sm_img https://teacher.smartermaths.com.au/wp-content/uploads/2020/08/Q52var2.svg 200 indent vpad"},{"varval":"In total"},{"varval":"arrangements"},{"varval":"letter"},{"varval":"4"},{"varval":"3"},{"varval":"2"}]},{"vars":[{"varval":"Jane"},{"varval":"four digit numbers"},{"varval":"numbers"},{"varval":"sm_img https://teacher.smartermaths.com.au/wp-content/uploads/2020/08/Q52var3.svg 200 indent vpad"},{"varval":"If the four digit number must start with 3"},{"varval":"numbers"},{"varval":"number"},{"varval":"1"},{"varval":"3"},{"varval":"2"}]},{"vars":[{"varval":"Gaby"},{"varval":"letter arrangements"},{"varval":"four letters"},{"varval":"sm_img https://teacher.smartermaths.com.au/wp-content/uploads/2020/08/Q52var4.svg 200 indent vpad"},{"varval":"If the letter arrangement must start with L"},{"varval":"arrangement"},{"varval":"letter"},{"varval":"1"},{"varval":"3"},{"varval":"2"}]},{"vars":[{"varval":"Miranda"},{"varval":"side-by-side arrangements"},{"varval":"four shapes"},{"varval":"sm_img https://teacher.smartermaths.com.au/wp-content/uploads/2020/08/Q51var5.svg 200 indent vpad"},{"varval":"In total"},{"varval":"arrangements"},{"varval":"shape"},{"varval":"4"},{"varval":"3"},{"varval":"2"}]}]

  908. Number, NAPX-J4-CA12, NAPX-J3-CA21

    <div class="sm_mode"> Which number line correctly shows the positions of 0.05, 0.405 and 0.45. </div>

    [{"vars":null}]

  909. Algebra, NAPX-K2-20

    <div class="sm_mode"> {{{question}}} </div>

    [{"vars":[{"varval":"On Tuesday, Dinesh went to the doctor and was given 28 tablets.\r\n\r\nDinesh was to take 6 tablets each day starting from Tuesday.\r\n\r\nOn which day did Dinesh take the last tablet?\n"},{"varval":"<div class=\"aligned\">\n\n| | |\n| --------------------: | -------------- |\n| Number of days | = 28 $\\div$ 6 |\n| | = 4 remainder 4 |\n\n</div>\n\n<br>$\\Rightarrow$ Last tablet taken on the 5th day.\n\n\n$\\therefore$ {{{correctAnswer}}}\n"}]},{"vars":[{"varval":"On Monday, Kermit went to the doctor and was given 30 tablets.\r\n\r\nKermit was to take 4 tablets each day starting from Tuesday.\r\n\r\nOn which day did Kermit take the last tablet?\n"},{"varval":"<div class=\"aligned\">\n\n| | |\n| --------------------: | -------------- |\n| Number of days | = 30 $\\div$ 4 |\n| | = 7 remainder 2 |\n\n</div>\n\n<br>$\\Rightarrow$ Last tablet taken on the 8th day.\n\n\n$\\therefore$ {{{correctAnswer}}}\n"}]},{"vars":[{"varval":"On Wednesday, Margarita went to the doctor and was given 20 antibiotic capsules.\r\n\r\nMargarita was to take 3 antibiotic capsules each day starting from Wednesday.\r\n\r\nOn which day did Margarita take the last tablet?\n"},{"varval":"<div class=\"aligned\">\n\n| | |\n| --------------------: | -------------- |\n| Number of days | = 20 $\\div$ 3 |\n| | = 6 remainder 2 |\n\n</div>\n\n<br>$\\Rightarrow$ Last tablet taken on the 7th day.\n\n\n$\\therefore$ {{{correctAnswer}}}\n"}]},{"vars":[{"varval":"On Saturday, Hendrik went to the doctor and was given 25 tablets.\r\n\r\nHendrik was to take 3 tablets each day starting from Sunday.\r\n\r\nOn which day did Hendrik take the last tablet?\n"},{"varval":"<div class=\"aligned\">\n\n| | |\n| --------------------: | -------------- |\n| Number of days | = 25 $\\div$ 3 |\n| | = 8 remainder 1 |\n\n</div>\n\n<br>$\\Rightarrow$ Last tablet taken on the 9th day.\n\n\n$\\therefore$ {{{correctAnswer}}}\n"}]},{"vars":[{"varval":"Gabriela started taking flu tablets on Thursday from a box containing 16 tablets.\r\n\r\nGabriela takes 1 tablet every 8 hours.\r\n\r\nOn which day will Gabriela take the last tablet?\n"},{"varval":" Tablets per day = 24 $\\div$ 8 = 3 \n\n<div class=\"aligned\">\n\n| | |\n| --------------------: | -------------- |\n| Number of days | = 16 $\\div$ 3 |\n| | = 5 remainder 1 |\n\n</div>\n\n<br>$\\Rightarrow$ Last tablet taken on the 6th day.\n\n\n$\\therefore$ {{{correctAnswer}}}\n"}]},{"vars":[{"varval":"Antionette started taking iron tablets on Wednesday from a bottle containing 25 tablets.\r\n\r\nAntionette takes 1 tablet every 12 hours.\r\n\r\nOn which day will Antionette take the last tablet?\n"},{"varval":" Tablets per day = 24 $\\div$ 12 = 2 \n\n<div class=\"aligned\">\n\n| | |\n| --------------------: | -------------- |\n| Number of days | = 25 $\\div$ 2 |\n| | = 12 remainder 1 |\n\n</div>\n\n<br>$\\Rightarrow$ Last tablet taken on the 13th day.\n\n\n$\\therefore$ {{{correctAnswer}}}"}]}]

  910. Algebra, NAPX-p122508v01

    <div class="sm_mode"> On Wednesday, Albert went to the doctor and was given 26 tablets. Albert was to take 4 tablets each day starting from Wednesday. On which day did Albert take the last tablet? </div>

    [{"vars":null}]

  911. Algebra, NAPX-p122351v02

    <div class="sm_mode"> Tiger plays golf. During one round, he found 12 golf balls and lost 3. At the end of the round, Tiger had a total of 15 golf balls. <br> sm_img https://teacher.smartermaths.com.au/wp-content/uploads/2019/01/NAPX-K2-18v3.svg 350 indent3 vpad <br>How many golf balls did Tiger start with? </div>

    [{"vars":null}]

  912. Number, NAP-08275

    <div class="sm_mode"> The number 27 982 can be rounded in different ways. Which two ways of rounding give the same answer? </div>

    [{"vars":null}]

  913. Measurement, NAPX-H2-36

    <div class="sm_mode"> {{{question}}} </div>

    [{"vars":[{"varval":"Bob lays pavers on his back deck, as shown below.\n\n<br>\n\n<div class=\"indent2\">\n\nsm_img https://teacher.smartermaths.com.au/wp-content/uploads/2019/01/NAPX-H2-36.svg 250 indent vpad\n\n</div>\n\nWhat is the area of his back deck in square metres?\n"},{"varval":"<div class=\"indent2\">\n\nsm_img https://teacher.smartermaths.com.au/wp-content/uploads/2019/01/NAPX-H2-36-Answer.svg 250 indent vpad\n\n</div>\n\n<div class=\"aligned\">\n\n| | |\n| ---- | --------------------------------------- |\n| Area | \\= (20 $\\times$ 7) + (5 $\\times$ 8) |\n| | \\= 140 + 40 |\n| | \\= {{{correctAnswer}}} m$^2$ |\n\n</div>"}]},{"vars":[{"varval":"Celeste lays tiles on her living room floor, as shown below.\n\n<br>\n\n<div class=\"indent2\">\n\nsm_img https://teacher.smartermaths.com.au/wp-content/uploads/2022/08/Measurement_20321_v1q.svg 300 indent vpad\n\n</div>\n\nWhat is the area of her living room in square metres?\n"},{"varval":"<div class=\"indent2\">\n\nsm_img https://teacher.smartermaths.com.au/wp-content/uploads/2022/08/Measurement_20321_v1ws.svg 330 indent vpad\n\n</div>\n\n<div class=\"aligned\">\n\n| | |\n| ---- | --------------------------------------- |\n| Area | \\= (3 $\\times$ 12) + (2 $\\times$ 3) |\n| | \\= 36 + 6 |\n| | \\= {{{correctAnswer}}} m$^2$ |\n\n</div>"}]},{"vars":[{"varval":"Bronte lays artificial turf in her pool area, as shown below.\n\n<br>\n\n<div class=\"indent2\">\n\nsm_img https://teacher.smartermaths.com.au/wp-content/uploads/2022/08/Measurement_20321_v2ws_1.svg 220 indent vpad\n\n</div>\n\nWhat is the area of her pool area in square metres?\n"},{"varval":"<div class=\"indent2\">\n\nsm_img https://teacher.smartermaths.com.au/wp-content/uploads/2022/08/Measurement_20321_v2q_1.svg 220 indent vpad\n\n</div>\n\n<div class=\"aligned\">\n\n| | |\n| ---- | --------------------------------------- |\n| Area | \\= (10 $\\times$ 2) + (2 $\\times$ 4) |\n| | \\= 20 + 8 |\n| | \\= {{{correctAnswer}}} m$^2$ |\n\n</div>"}]},{"vars":[{"varval":"Jeremy lays timber on his front balcony, as shown below.\n\n<br>\n\n<div class=\"indent2\">\n\nsm_img https://teacher.smartermaths.com.au/wp-content/uploads/2022/08/Measurement_20321_v3q.svg 370 indent vpad\n\n</div>\n\nWhat is the area of his front balcony in square metres?\n"},{"varval":"<div class=\"indent2\">\n\nsm_img https://teacher.smartermaths.com.au/wp-content/uploads/2022/08/Measurement_20321_v3ws.svg 370 indent vpad\n\n</div>\n\n<div class=\"aligned\">\n\n| | |\n| ---- | --------------------------------------- |\n| Area | \\= (4.2 $\\times$ 4.6) + (1.8 $\\times$ 1.6) |\n| | \\= 19.32 + 2.88 |\n| | \\= {{{correctAnswer}}} m$^2$ |\n\n</div>"}]},{"vars":[{"varval":"Mitchell lays rubber matting in his gym, as shown below.\n\n<br>\n\n<div class=\"indent2\">\n\nsm_img https://teacher.smartermaths.com.au/wp-content/uploads/2022/08/Measurement_20321_v4q.svg 330 indent vpad\n\n</div>\n\nWhat is the area of his gym in square metres?\n"},{"varval":"<div class=\"indent2\">\n\nsm_img https://teacher.smartermaths.com.au/wp-content/uploads/2022/08/Measurement_20321_v4ws_1.svg 330 indent vpad\n\n</div>\n\n<div class=\"aligned\">\n\n| | |\n| ---- | --------------------------------------- |\n| Area | \\= (3.7 $\\times$ 11) + (1.5 $\\times$ 0.8) |\n| | \\= 40.7 + 1.2 |\n| | \\= {{{correctAnswer}}} m$^2$ |\n\n</div>"}]},{"vars":[{"varval":"Lilo lays turf on his terrace, as shown below.\n\n<br>\n\n<div class=\"indent2\">\n\nsm_img https://teacher.smartermaths.com.au/wp-content/uploads/2022/08/Measurement_20321_v5q.svg 380 indent vpad\n\n</div>\n\nWhat is the area of his terrace in square metres?\n"},{"varval":"<div class=\"indent2\">\n\nsm_img https://teacher.smartermaths.com.au/wp-content/uploads/2022/08/Measurement_20321_v5ws.svg 380 indent vpad\n\n</div>\n\n<div class=\"aligned\">\n\n| | |\n| ---- | --------------------------------------- |\n| Area | \\= (15 $\\times$ 5) + (35.8 $\\times$ 8.2) |\n| | \\= 75 + 293.56 |\n| | \\= {{{correctAnswer}}} m$^2$ |\n\n</div>"}]}]

  914. Measurement, NAPX-p122415v01

    <div class="sm_mode"> {{{question}}} </div>

    [{"vars":[{"varval":"Which one of these shapes has the longest perimeter?"},{"varval":"Perimeter of 20 units (largest).\n\n{{{correctAnswer}}}"}]},{"vars":[{"varval":"Which one of these shapes has the longest perimeter?"},{"varval":"Perimeter of 24 units (largest).\n\n{{{correctAnswer}}}"}]},{"vars":[{"varval":"Which one of these shapes has the longest perimeter?"},{"varval":"Perimeter of 20 units (largest).\n\n{{{correctAnswer}}}"}]},{"vars":[{"varval":"Which one of these shapes has the longest perimeter?"},{"varval":"Perimeter of 22 units (largest).\n\n{{{correctAnswer}}}"}]},{"vars":[{"varval":"Which one of these shapes has the longest perimeter?"},{"varval":"Perimeter of 22 units (largest).\n\n{{{correctAnswer}}}"}]},{"vars":[{"varval":"Which one of these shapes has the longest perimeter?"},{"varval":"Perimeter of 24 units (largest).\n\n{{{correctAnswer}}}"}]}]

  915. Measurement, NAPX-J2-32 Minstd was the only category so added 7-10PAV.

    <div class="sm_mode"> {{{question}}} </div>

    [{"vars":[{"varval":"Ken made these solid prisms out of identical cubes.\n\nWhich prism has the largest volume?"},{"varval":"Volume of each prism:\n\nOption 1:  5 $\\times$ 2 $\\times$ 3 = 30 cubes\n\nOption 2:  3 $\\times$ 3 $\\times$ 3 = 27 cubes\n\nOption 3:  7 $\\times$ 2 $\\times$ 2 = 28 cubes\n\nOption 4:  4 $\\times$ 4 $\\times$ 2 = 32 cubes\n\n$\\therefore$ The prism with the largest volume is\n\n{{{correctAnswer}}}\n"}]}]

  916. Measurement, NAP-H3-CA23

    <div class="sm_mode"> {{{question}}} </div>

    [{"vars":[{"varval":"<div class=\"indent\">\n\nsm_img https://teacher.smartermaths.com.au/wp-content/uploads/2017/01/NAP-H3-CA23.png 240 indent vpad\n\n</div>\n\n<br>Which of these could be used to calculate the area of the shape in square centimetres?\n"},{"varval":"<div class=\"aligned\">\n\n| | |\n| --------------------- | -------------------------------------------- |\n| Area | = Area of large rectangle $-$ Area of cut-out rectangle |\n| | = {{{correctAnswer}}} |\n\n</div>"}]},{"vars":[{"varval":"<div class=\"indent\">\n\nsm_img https://teacher.smartermaths.com.au/wp-content/uploads/2022/08/Measurement_20317_v1.svg 350 indent vpad\n\n</div>\n\nWhich of these could be used to calculate the area of the shape in square centimetres?\n"},{"varval":"<div class=\"aligned\">\n\n| | |\n| --------------------- | -------------------------------------------- |\n| Area | = Area of large rectangle $-$ Area of cut-out rectangle |\n| | = {{{correctAnswer}}} |\n\n</div>"}]},{"vars":[{"varval":"<div class=\"indent\">\n\nsm_img https://teacher.smartermaths.com.au/wp-content/uploads/2022/08/Measurement_20317_v2.svg 300 indent vpad\n\n</div>\n\nWhich of these could be used to calculate the area of the shape in square millimetres?\n"},{"varval":"<div class=\"aligned\">\n\n| | |\n| --------------------- | -------------------------------------------- |\n| Area | = Area of large rectangle $-$ Area of cut-out rectangle |\n| | = {{{correctAnswer}}} |\n\n</div>"}]},{"vars":[{"varval":"<div class=\"indent\">\n\nsm_img https://teacher.smartermaths.com.au/wp-content/uploads/2022/08/Measurement_20317_v3.svg 390 indent vpad\n\n</div>\n\nWhich of these could be used to calculate the area of the shape in square centimetres?\n"},{"varval":"<div class=\"aligned\">\n\n| | |\n| --------------------- | -------------------------------------------- |\n| Area | = Area of large rectangle $-$ Area of cut-out square |\n| | = {{{correctAnswer}}} |\n\n</div>"}]},{"vars":[{"varval":"<div class=\"indent\">\n\nsm_img https://teacher.smartermaths.com.au/wp-content/uploads/2022/08/Measurement_20317_v5.svg 400 indent vpad\n\n</div>\n\nWhich of these could be used to calculate the area of the shape in square centimetres?\n"},{"varval":"<div class=\"aligned\">\n\n| | |\n| --------------------- | -------------------------------------------- |\n| Area | = Area of large rectangle $-$ Area of cut-out smaller rectangle |\n| | = {{{correctAnswer}}} |\n\n</div>"}]},{"vars":[{"varval":"<div class=\"indent\">\n\nsm_img https://teacher.smartermaths.com.au/wp-content/uploads/2022/08/Measurement_20317_v4.svg 350 indent vpad\n\n</div>\n\nWhich of these could be used to calculate the area of the shape in square centimetres?\n"},{"varval":"<div class=\"aligned\">\n\n| | |\n| --------------------- | -------------------------------------------- |\n| Area | = Area of large horizontal rectangle + Area of small vertical rectangle |\n| | = {{{correctAnswer}}} |\n\n</div>"}]}]

  917. Number, NAP-83471

    <div class="sm_mode"> {{{question}}} </div>

    [{"vars":[{"varval":"28.3 $\\times$ 10 ="},{"varval":"\n{{{correctAnswer}}}\n\n\nWhen multiplying by 10, move the decimal\npoint 1 position to the right."}]},{"vars":[{"varval":"90.2 $\\times$ 10 ="},{"varval":"{{{correctAnswer}}}\n\n\nWhen multiplying by 10, move the decimal\npoint 1 position to the right."}]},{"vars":[{"varval":"53.05 $\\times$ 10 ="},{"varval":"{{{correctAnswer}}}\n\n\nWhen multiplying by 10, move the decimal\npoint 1 position to the right."}]}]

  918. Measurement, NAP-A3-CA22

    <div class="sm_mode"> {{{question}}} </div>

    [{"vars":[{"varval":"This shape is made from eight equilateral triangles and one large parallelogram.\n\n<br>\n<div class=\"indent2\">\n\nsm_img https://teacher.smartermaths.com.au/wp-content/uploads/2017/12/NAP-A3-CA221.svg 200 indent vpad\n\n</div>\n\n<br>Each side of all the small triangles is 4 cm long.\n\nWhat is the perimeter of the shape?"},{"varval":"sm_nogap Perimeter from the top left corner (clockwise)\n\n<div class=\"aligned\">\n\n>>| |\n| --------------------- |\n| = 12 + 4 + 4 + 4 + 4 + 8 + 4 + 8 |\n| = {{{correctAnswer}}} |\n\n</div>"}]},{"vars":[{"varval":"This shape is made from ten equilateral triangles and one large parallelogram.\n\n<br>\n<div class=\"indent2\">\n\nsm_img https://teacher.smartermaths.com.au/wp-content/uploads/2022/08/Measurement_20316_v1.svg 250 indent vpad\n\n</div>\n\n<br>Each side of all the small triangles is 5 cm long.\n\nWhat is the perimeter of the shape?"},{"varval":"sm_nogap Perimeter from the top left corner (clockwise)\n\n<div class=\"aligned\">\n\n>>| |\n| --------------------- |\n| = 10 + 8 $\\times$ 5 + 20 |\n| = 10 + 40 + 20 |\n| = {{{correctAnswer}}} |\n\n</div>"}]},{"vars":[{"varval":"This shape is made from eight equilateral triangles and one large parallelogram.\n\n<br>\n<div class=\"indent2\">\n\nsm_img https://teacher.smartermaths.com.au/wp-content/uploads/2022/08/Measurement_20316_v2.svg 270 indent vpad\n\n</div>\n\n<br>Each side of all the small triangles is 2 cm long.\n\nWhat is the perimeter of the shape?"},{"varval":"sm_nogap Perimeter from the top left corner (clockwise)\n\n<div class=\"aligned\">\n\n>>| |\n| --------------------- |\n| = 4 + 7 $\\times$ 2 + 6 |\n| = 4 + 14 + 6 |\n| = {{{correctAnswer}}} |\n\n</div>"}]},{"vars":[{"varval":"This shape is made from sixteen equilateral triangles and one large parallelogram.\n\n<br>\n<div class=\"indent2\">\n\nsm_img https://teacher.smartermaths.com.au/wp-content/uploads/2022/08/Measurement_20316_v3.svg 330 indent vpad\n\n</div>\n\n<br>Each side of all the small triangles is 6 cm long.\n\nWhat is the perimeter of the shape?"},{"varval":"sm_nogap Perimeter from furthest left corner (clockwise)\n\n<div class=\"aligned\">\n\n>>| |\n| --------------------- |\n| = 7 $\\times$ 6 + 18 + 7 $\\times$ 6 + 18 |\n| = 42 + 18 + 42 + 18 |\n| = {{{correctAnswer}}} |\n\n</div>"}]},{"vars":[{"varval":"This shape is made from twelve equilateral triangles and one large parallelogram.\n\n<br>\n<div class=\"indent2\">\n\nsm_img https://teacher.smartermaths.com.au/wp-content/uploads/2022/08/Measurement_20316_v4.svg 270 indent vpad\n\n</div>\n\n<br>Each side of all the small triangles is 11 cm long.\n\nWhat is the perimeter of the shape?"},{"varval":"sm_nogap Perimeter from the top corner (clockwise)\n\n<div class=\"aligned\">\n\n>>| |\n| --------------------- |\n| = 9 $\\times$ 11 + 22 + 55 |\n| = 99 + 22 + 55 |\n| = {{{correctAnswer}}} |\n\n</div>"}]},{"vars":[{"varval":"This shape is made from twelve equilateral triangles and one large parallelogram.\n\n<br>\n<div class=\"indent2\">\n\nsm_img https://teacher.smartermaths.com.au/wp-content/uploads/2022/08/Measurement_20316_v5a.svg 270 indent vpad\n\n</div>\n\n<br>Each side of all the small triangles is 12 cm long.\n\nWhat is the perimeter of the shape?"},{"varval":"sm_nogap Perimeter from the top corner (clockwise)\n\n<div class=\"aligned\">\n\n>>| |\n| --------------------- |\n| = 9 $\\times$ 12 + 36 + 48 |\n| = 108 + 36 + 48 |\n| = {{{correctAnswer}}} |\n\n</div>"}]}]

  919. Measurement, NAP-H3-CA18

    <div class="sm_mode"> {{{question}}} </div>

    [{"vars":[{"varval":"A rectangle has a length of 20 cm and a width of 12 cm.\n\nA square has the same perimeter as this rectangle.\n\nWhat is the side length of this square in centimetres?"},{"varval":"<div class=\"aligned\">\n\n| | |\n| --------------------- | -------------------------------------------- |\n| Perimeter of rectangle | = (2 $\\times$ 12) + (2 $\\times$ 20) |\n| | = 64 cm |\n\n</div>\n\n<br>\n\n<div class=\"aligned\">\n\n| | |\n| --------------------- | -------------------------------------------- |\n| Side length of square | = $\\dfrac{64}{4}$ |\n| | = {{{correctAnswer}}} |\n\n</div>"}]},{"vars":[{"varval":"A rectangle has a length of 41.5 cm and a width of 19 cm.\n\nA square has the same perimeter as this rectangle.\n\nWhat is the side length of this square in centimetres?"},{"varval":"<div class=\"aligned\">\n\n| | |\n| --------------------- | -------------------------------------------- |\n| Perimeter of rectangle | = (2 $\\times$ 41.5) + (2 $\\times$ 19) |\n| | = 121 cm |\n\n</div>\n\n<br>\n\n<div class=\"aligned\">\n\n| | |\n| --------------------- | -------------------------------------------- |\n| Side length of square | = $\\dfrac{121}{4}$ |\n| | = {{{correctAnswer}}} |\n\n</div>"}]},{"vars":[{"varval":"A rectangle has a length of 31 cm and a width of 53.5 cm.\n\nA square has the same perimeter as this rectangle.\n\nWhat is the side length of this square in centimetres?"},{"varval":"<div class=\"aligned\">\n\n| | |\n| --------------------- | -------------------------------------------- |\n| Perimeter of rectangle | = (2 $\\times$ 31) + (2 $\\times$ 53.5) |\n| | = 169 cm |\n\n</div>\n\n<br>\n\n<div class=\"aligned\">\n\n| | |\n| --------------------- | -------------------------------------------- |\n| Side length of square | = $\\dfrac{169}{4}$ |\n| | = {{{correctAnswer}}} |\n\n</div>"}]},{"vars":[{"varval":"A rectangle has a length of 9 cm and a width of 31.5 cm.\n\nA square has the same perimeter as this rectangle.\n\nWhat is the side length of this square in centimetres?"},{"varval":"<div class=\"aligned\">\n\n| | |\n| --------------------- | -------------------------------------------- |\n| Perimeter of rectangle | = (2 $\\times$ 9) + (2 $\\times$ 31.5) |\n| | = 81 cm |\n\n</div>\n\n<br>\n\n<div class=\"aligned\">\n\n| | |\n| --------------------- | -------------------------------------------- |\n| Side length of square | = $\\dfrac{81}{4}$ |\n| | = {{{correctAnswer}}} |\n\n</div>"}]},{"vars":[{"varval":"A rectangular paddock has a length of 140 m and a width of 102 m.\n\nAn adjoining paddock is a square and has the same perimeter as the rectangular paddock.\n\nWhat is the side length of the square paddock in metres?"},{"varval":"<div class=\"aligned\">\n\n| | |\n| --------------------- | -------------------------------------------- |\n| Perimeter of rectangular paddock | = (2 $\\times$ 140) + (2 $\\times$ 102) |\n| | = 484 m |\n\n</div>\n\n<br>\n\n<div class=\"aligned\">\n\n| | |\n| --------------------- | -------------------------------------------- |\n| Side length of square paddock | = $\\dfrac{484}{4}$ |\n| | = {{{correctAnswer}}} |\n\n</div>"}]},{"vars":[{"varval":"A rectangular table has a length of 2.8 m and a width of 1.2 m.\n\nA square table has the same perimeter as the rectangular table.\n\nWhat is the side length of the square table in metres?"},{"varval":"<div class=\"aligned\">\n\n| | |\n| --------------------- | -------------------------------------------- |\n| Perimeter of rectangular paddock | = (2 $\\times$ 2.8) + (2 $\\times$ 1.2) |\n| | = 8 m |\n\n</div>\n\n<br>\n\n<div class=\"aligned\">\n\n| | |\n| --------------------- | -------------------------------------------- |\n| Side length of square paddock | = $\\dfrac{8}{4}$ |\n| | = {{{correctAnswer}}} |\n\n</div>"}]}]

  920. Measurement, NAP-J3-CA11

    <div class="sm_mode"> A triangle is drawn on grid paper. sm_img https://teacher.smartermaths.com.au/wp-content/uploads/2017/09/NAP-J1-CA11_1.png 300 indent vpad What is the area of the triangle? </div>

    [{"vars":null}]

  921. Number, NAP-06955

    <div class="sm_mode"> Locky got a new car and recorded how many litres of petrol he put into the car over 4 weeks. Week 1: 62.35 litres Week 2: 58.65 litres Week 3: 44.95 litres Week 4: 53.75 litres To estimate how many litres of petrol he put into his car over the 4 weeks, Locky rounded the weekly amounts to the nearest 10 litres and added them. How much was Locky's estimate? </div>

    [{"vars":null}]

  922. Number, NAP-49345

    <div class="sm_mode"> {{{question}}} </div>

    [{"vars":[{"varval":"Roger had tests to check his health that are summarised in the table below.\n\n<br>\n\n<div class=\"sm-table row1-color8\">\n\n>| **Test** | **Normal Range** | **Roger's Results** |\n|:-:|:-:|:-:|\n| Resting Heart Rate | 60 to 95| 92|\n| Blood Sugar | 4.2 to 6.1| 4.1|\n| Vitamin C | 0.4 to 1.7| 0.5|\n\n</div>\n\n<br>For which tests were Roger's results within the normal range?"},{"varval":"Checking each test: \n\n92 is within 60 and 95 &nbsp;$\\checkmark$\n\n4.1 is not within 4.2 and 6.1 &nbsp;X\n\n0.5 is within 0.4 and 1.7 &nbsp;$\\checkmark$\n\n$\\therefore$ {{{correctAnswer}}} are within the normal range."}]},{"vars":[{"varval":"Albert had tests to check his health that are summarised in the table below.\n\n<br>\n\n<div class=\"sm-table row1-color8\">\n\n>| **Test** | **Normal Range** | **Albert's Results** |\n|:-:|:-:|:-:|\n| Triglycerides | 40 to 75 | 69 |\n| Blood Sugar | 4.5 to 7.2| 3.8|\n| Malevoids | 0.6 to 1.4| 0.9|\n\n</div>\n\n<br>For which tests were Albert's results within the normal range?"},{"varval":"Checking each test: \n\n69 is within 40 and 75 &nbsp;$\\checkmark$\n\n3.8 is not within 4.5 and 7.2 &nbsp;X\n\n0.9 is within 0.6 and 1.4 &nbsp;$\\checkmark$\n\n$\\therefore$ {{{correctAnswer}}} are within the normal range."}]},{"vars":[{"varval":"Mattel had tests to check her health that are summarised in the table below.\n\n<br>\n\n<div class=\"sm-table row1-color8\">\n\n>| **Test** | **Normal Range** | **Mattel's Results** |\n|:-:|:-:|:-:|\n| Vitamin D | 3.1 to 4.7| 3.05|\n| Cholesterol | 10.3 to 11.2| 10.9|\n| Lipids | 28 to 65| 49|\n\n</div>\n\n<br>For which tests were Mattel's results within the normal range?"},{"varval":"Checking each test: \n\n3.05 is not within 3.1 and 4.7 &nbsp;X\n\n10.9 is within 10.3 and 11.2 &nbsp;$\\checkmark$\n\n49 is within 28 and 65 &nbsp;$\\checkmark$\n\n$\\therefore$ {{{correctAnswer}}} are within the normal range."}]},{"vars":[{"varval":"George had tests to check his health that are summarised in the table below.\n\n<br>\n\n<div class=\"sm-table row1-color8\">\n\n>| **Test** | **Normal Range** | **George's Results** |\n|:-:|:-:|:-:|\n|pH | 7.35 to 7.45 | 7.55|\n| Fasting Blood Glucose | 4.0 to 6.0| 8.9|\n| Triglycerides | 0.55 to 1.90| 1.89|\n\n</div>\n\n<br>For which tests were George's results within the normal range?"},{"varval":"Checking each test: \n\n7.55 is not within 7.35 and 7.45 &nbsp;X\n\n8.9 is not within 4.0 and 6.0 &nbsp;X\n\n1.89 is within 0.55 and 1.90 &nbsp;$\\checkmark$\n\n$\\therefore$ {{{correctAnswer}}} are within the normal range."}]},{"vars":[{"varval":"Lexi had tests to check her health that are summarised in the table below.\n\n<br>\n\n<div class=\"sm-table row1-color8\">\n\n>| **Test** | **Normal Range** | **Lexi's Results** |\n|:-:|:-:|:-:|\n| Vitamin D | 3.1 to 4.7| 2.5|\n| Folate| greater than 4.9 | 6.0|\n| Vitamin B12 | 180 to 1000| 120|\n\n</div>\n\n<br>For which tests were Lexi's results within the normal range?"},{"varval":"Checking each test: \n\n2.5 is not within 3.1 and 4.7 &nbsp;X\n\n6.0 is greater than 4.9 &nbsp;$\\checkmark$\n\n120 is not within 180 and 1000 &nbsp;X\n\n$\\therefore$ {{{correctAnswer}}} is within the normal range."}]},{"vars":[{"varval":"Braydon had tests to check his health that are summarised in the table below.\n\n<br>\n\n<div class=\"sm-table row1-color8\">\n\n>| **Test** | **Normal Range** | **Braydon's Results** |\n|:-:|:-:|:-:|\n| Ferratin | 25 to 350| 150|\n| Haemoglobin| 130 to 180| 120|\n| Lipids | 28 to 65| 70|\n\n</div>\n\n<br>For which tests were Braydon's results within the normal range?"},{"varval":"Checking each test: \n\n150 is within 25 and 350 &nbsp;$\\checkmark$\n\n120 is not within 130 and 180 &nbsp;X\n\n70 is not within 28 and 65 &nbsp;X\n\n$\\therefore$ {{{correctAnswer}}} is within the normal range."}]}]

  923. Number, NAP-78782

    <div class="sm_mode"> {{{question}}} </div>

    [{"vars":[{"varval":"The area of New South Wales is 1 896 790 square kilometres.\r\n\r\nWhat is the area rounded to the nearest thousand square kilometres?"},{"varval":"{{{correctAnswer}}}"}]},{"vars":[{"varval":"The area of Western Australia is 2 527 013 square kilometres.\r\n\r\nWhat is the area rounded to the nearest thousand square kilometres?"},{"varval":"\t\r\n{{{correctAnswer}}}"}]},{"vars":[{"varval":"The area of Australia is approximately 7 617 930 square kilometres.\r\n\r\nWhat is the area rounded to the nearest thousand square kilometres?"},{"varval":"{{{correctAnswer}}}"}]},{"vars":[{"varval":"The storage capacity of Warragamba dam is 2 064 680 megalitres.\n\nThat is this capacity rounded to the nearest thousand megalitres?"},{"varval":"{{{correctAnswer}}}"}]},{"vars":[{"varval":"The population of NSW as at December 2020 was 8 172 500 residents.\r\n\r\nWhat is the population rounded to the nearest thousand residents?"},{"varval":"{{{correctAnswer}}}"}]},{"vars":[{"varval":"There were 294 369 registered births in 2020.\r\n\r\nWhat is this rounded to the nearest thousand births?"},{"varval":"{{{correctAnswer}}}"}]}]

  924. Number, NAP-43430

    <div class="sm_mode"> {{{question}}} </div>

    [{"vars":[{"varval":"Which one of these numbers is a factor of 34?\n"},{"varval":"{{{correctAnswer}}}"}]},{"vars":[{"varval":"Which one of these numbers is a factor of 26?"},{"varval":"{{{correctAnswer}}}"}]},{"vars":[{"varval":"Which one of these numbers is a factor of 51?"},{"varval":"{{{correctAnswer}}}"}]},{"vars":[{"varval":"Which one of these numbers is a factor of 63?"},{"varval":"\t\r\n{{{correctAnswer}}}"}]},{"vars":[{"varval":"Which one of these numbers is a factor of 94?"},{"varval":"{{{correctAnswer}}}"}]},{"vars":[{"varval":"Which one of these numbers is a factor of 41?"},{"varval":"{{{correctAnswer}}}"}]}]

  925. Number, NAP-43426

    <div class="sm_mode"> Con packs oranges into boxes for the market. Each box holds 20 oranges. Con fills 3 boxes and has 6 oranges left over. <br> sm_img https://teacher.smartermaths.com.au/wp-content/uploads/2019/12/NAPLAN-2019-Y7-3-MC.svg 550 indent vpad <br>How many oranges does Con have altogether? </div>

    [{"vars":null}]

  926. Number, NAPX-J3-NC07, NAPX-J2-39

    <div class="sm_mode"> {{{question}}} </div>

    [{"vars":[{"varval":"Which of these numbers has a 7 in the thousandths place?"},{"varval":"Digits to the right of the decimal place are (in order): \n\n$\\rightarrow$ tenths, hundredths, thousandths, ...\n\n$\\therefore$ {{{correctAnswer}}} is correct.\n"}]},{"vars":[{"varval":"Which of these numbers has a 2 in the thousandths place?"},{"varval":"Digits to the right of the decimal place are (in order): \n\n$\\rightarrow$ tenths, hundredths, thousandths, ...\n\n$\\therefore$ {{{correctAnswer}}} is correct.\n"}]},{"vars":[{"varval":"Which of these numbers has a 7 in the tenths place?"},{"varval":"Digits to the right of the decimal place are (in order): \n\n$\\rightarrow$ tenths, hundredths, thousandths, ...\n\n$\\therefore$ {{{correctAnswer}}} is correct.\n"}]},{"vars":[{"varval":"Which of these numbers has a 7 in the hundredths place?"},{"varval":"Digits to the right of the decimal place are (in order): \n\n$\\rightarrow$ tenths, hundredths, thousandths, ...\n\n$\\therefore$ {{{correctAnswer}}} is correct."}]},{"vars":[{"varval":"Which of these numbers has a 4 in the thousandths place?"},{"varval":"Digits to the right of the decimal place are (in order): \n\n$\\rightarrow$ tenths, hundredths, thousandths, ...\n\n$\\therefore$ {{{correctAnswer}}} is correct.\n"}]},{"vars":[{"varval":"Which of these numbers has a 4 in the hundredths place?"},{"varval":"Digits to the right of the decimal place are (in order): \n\n$\\rightarrow$ tenths, hundredths, thousandths, ...\n\n$\\therefore$ {{{correctAnswer}}} is correct."}]}]

  927. Number, NAPX-E3-CA20

    <div class="sm_mode"> {{{question}}} </div>

    [{"vars":[{"varval":"The number 97 697 can be rounded in different ways.\r\n\r\nWhich two ways of rounding give the same answer?"},{"varval":"Rounding to the nearest ten:\n\n97 697 $\\to$ 97 700\n\nRounding to the nearest hundred:\n\n97 697 $\\to$ 97 700\n\n$\\therefore$ {{{correctAnswer}}}"}]},{"vars":[{"varval":"The number 27 982 can be rounded in different ways.\r\n\r\nWhich two ways of rounding give the same answer?"},{"varval":"27 982 = 28 000 (rounding to nearest 100)\n\n27 982 = 28 000 (rounding to nearest 1000)\n\n$\\therefore$ {{{correctAnswer}}} give the same answer."}]},{"vars":[{"varval":"The number 45 965 can be rounded in different ways.\r\n\r\nWhich two ways of rounding give the same answer?"},{"varval":"45 965 = 46 000 (rounding to nearest 100)\n\n45 965 = 46 000 (rounding to nearest 1000)\n\n$\\therefore$ {{{correctAnswer}}} give the same answer."}]},{"vars":[{"varval":"The number 34 599 can be rounded in different ways.\r\n\r\nWhich two ways of rounding give the same answer?"},{"varval":"34 599 = 34 600 (rounding to nearest 10)\n\n34 599 = 34 600 (rounding to nearest 100)\n\n$\\therefore$ {{{correctAnswer}}} give the same answer."}]},{"vars":[{"varval":"The number 59 869 can be rounded in different ways.\r\n\r\nWhich two ways of rounding give the same answer?"},{"varval":"59 869 = 60 000 (rounding to nearest 1000)\n\n59 869 = 60 000 (rounding to nearest 10 000)\n\n$\\therefore$ {{{correctAnswer}}} give the same answer."}]},{"vars":[{"varval":"The number 12 396 can be rounded in different ways.\r\n\r\nWhich two ways of rounding give the same answer?"},{"varval":"12 396 = 13 400 (rounding to nearest 10)\n\n12 396 = 13 400 (rounding to nearest 100)\n\n$\\therefore$ {{{correctAnswer}}} give the same answer."}]}]

  928. mjdtest

    <div class="sm_mode"> b3l2i1 </div>

    [{"vars":null},{"vars":null},{"vars":null},{"vars":null},{"vars":null},{"vars":null},{"vars":null},{"vars":null},{"vars":null},{"vars":null}]

  929. mjdtest

    <div class="sm_mode"> b3l1i1 </div>

    [{"vars":null},{"vars":null},{"vars":null},{"vars":null},{"vars":null},{"vars":null},{"vars":null},{"vars":null},{"vars":null},{"vars":null}]

  930. Measurement, NAP-A3-NC14

    <div class="sm_mode"> sm_img https://teacher.smartermaths.com.au/wp-content/uploads/2018/01/nap-A3-nc14.svg 310 indent3 vpad <br>What is the perimeter of this shape? </div>

    [{"vars":null}]

  931. Measurement, NAP-F3-CA12

    <div class="sm_mode"> Michael cut a diagonal into a rectangular piece of wood. <br> <div class="indent2"> sm_img https://teacher.smartermaths.com.au/wp-content/uploads/2017/02/naplan-Y7-2013-12mca-700x400.png 330 indent vpad </div> <br>What is the missing length? </div>

    [{"vars":null}]

  932. Number, NAPX-I3-NC19

    <div class="sm_mode"> Monty wants to buy some cricket equipment. The sports shop's prices of the equipment he wants is listed in the table below. <br> sm_img https://teacher.smartermaths.com.au/wp-content/uploads/2018/06/NAPX-I3-NC19.svg 220 indent3 vpad <br>Monty buys all 4 items. Which of these gives the correct range for the total cost of this equipment? </div>

    [{"vars":null}]

  933. Number, NAPX-H3-CA03

    <div class="sm_mode"> {{{question}}} </div>

    [{"vars":[{"varval":"Mick is selling egg and bacon rolls at a school fete.\r\n\r\nHe makes $54 from selling 9 egg and bacon rolls.\r\n\r\nAll egg and bacon rolls cost the same.\r\n\r\nHow much will Mick make if he sells 11 egg and bacon rolls?"},{"varval":"Price of 1 egg and bacon roll = $\\dfrac{54}{9}$ = $6\n\n\n<div class=\"aligned\">\n\n| | |\n| --------------------: | -------------- |\n| $\\therefore$ Price of 11 rolls | = 11 $\\times$ $6 |\n| | = {{{correctAnswer}}} |\n\n\n</div>"}]},{"vars":[{"varval":"Josie is selling olive oil at a farmer's market.\r\n\r\nShe makes $42 from selling 6 bottles of olive oil.\r\n\r\nAll bottles of olive oil cost the same.\r\n\r\nHow much will Josie make if she sells 8 bottles of olive oil?"},{"varval":"Price of 1 bottle of olive oil = $\\dfrac{42}{6}$ = $7\n\n\n<div class=\"aligned\">\n\n| | |\n| --------------------: | -------------- |\n| $\\therefore$ Price of 8 bottles | = 8 $\\times$ $7 |\n| | = {{{correctAnswer}}} |\n\n\n</div>"}]},{"vars":[{"varval":"Brian is selling toy lawn mowers on Marketplace.\r\n\r\nHe makes $220 from selling 11 toy lawn mowers.\r\n\r\nAll toy lawn mowers cost the same.\r\n\r\nHow much will Brian make if he sells 7 toy lawn mowers?"},{"varval":"Price of 1 toy lawn mower = $\\dfrac{220}{11}$ = $20\n\n\n<div class=\"aligned\">\n\n| | |\n| --------------------: | -------------- |\n| $\\therefore$ Price of 7 toy lawn mowers | = 7 $\\times$ $20 |\n| | = {{{correctAnswer}}} |\n\n\n</div>"}]},{"vars":[{"varval":"Jack is selling drones on eBay.\r\n\r\nHe makes $6000 from selling 4 drones.\r\n\r\nAll drones cost the same.\r\n\r\nHow much will Jack make if he sells 9 drones?"},{"varval":"Price of 1 drone = $\\dfrac{6000}{4}$ = $1500\n\n\n<div class=\"aligned\">\n\n| | |\n| --------------------: | -------------- |\n| $\\therefore$ Price of 9 drones | = 9 $\\times$ $1500 |\n| | = {{{correctAnswer}}} |\n\n\n</div>"}]},{"vars":[{"varval":"The hardware store is having a sale on ladders.\r\n\r\nThey make $1800 from selling 3 ladders.\r\n\r\nAll ladders on sale cost the same.\r\n\r\nHow much will the hardware store make if they sell 8 ladders?"},{"varval":"Price of 1 ladder = $\\dfrac{1800}{3}$ = $600\n\n\n<div class=\"aligned\">\n\n| | |\n| --------------------: | -------------- |\n| $\\therefore$ Price of 8 ladders | = 8 $\\times$ $600 |\n| | = {{{correctAnswer}}} |\n\n\n</div>"}]},{"vars":[{"varval":"Mega Office Supplies is selling Casio calculators.\r\n\r\nThey make $138 from selling 6 toy Casio calculators.\r\n\r\nAll Casio calculators cost the same.\r\n\r\nHow much will Mega Office Supplies make if they sell 10 Casio calculators?"},{"varval":"Price of 1 Casio calculator = $\\dfrac{138}{6}$ = $23\n\n\n<div class=\"aligned\">\n\n| | |\n| --------------------: | -------------- |\n| $\\therefore$ Price of 10 Casio calculators | = 10 $\\times$ $23|\n| | = {{{correctAnswer}}} |\n\n\n</div>"}]}]

  934. Number, NAPX-L4-CA05 v2

    <div class="sm_mode"> At 8 pm in the Arctic circle, the temperature is 4°C. At 2 am the temperature is $-14$°C. Which calculation below can be used to work out how many degrees warmer it is at 8 pm than at 2 am? </div>

    [{"vars":null}]

  935. Measurement, NAPX-F4-CA16

    <div class="sm_mode"> {{{question}}} </div>

    [{"vars":[{"varval":"A child's bike tyre has a circumference of 64 cm.\n\nWhich of these is closest to the radius of the tyre?\n"},{"varval":"<div class=\"aligned\">\n\n| | |\n| ----------------: | ----------------------- |\n| $C$ | \\= 2$\\large \\pi \\large r$ |\n| 64 | \\= 2$\\large \\pi \\large r$ |\n| $\\therefore\\ \\large r$ | \\= $\\dfrac{64}{2\\large \\pi}$ |\n| | $\\approx$ {{{correctAnswer}}} |\n\n</div>"}]},{"vars":[{"varval":"A child's bike tyre has a circumference of 81 cm.\n\nWhich of these is closest to the radius of the tyre?\n"},{"varval":"<div class=\"aligned\">\n\n| | |\n| ----------------: | ----------------------- |\n| $C$ | \\= 2$\\large \\pi \\large r$ |\n| 81 | \\= 2$\\large \\pi \\large r$ |\n| $\\therefore\\ \\large r$ | \\= $\\dfrac{81}{2\\large \\pi}$ |\n| | $\\approx$ {{{correctAnswer}}} |\n\n</div>"}]},{"vars":[{"varval":"A mountain bike tyre has a circumference of 205 cm.\n\nWhich of these is closest to the radius of the tyre?\n"},{"varval":"<div class=\"aligned\">\n\n| | |\n| ----------------: | ----------------------- |\n| $C$ | \\= 2$\\large \\pi \\large r$ |\n| 205 | \\= 2$\\large \\pi \\large r$ |\n| $\\therefore\\ \\large r$ | \\= $\\dfrac{205}{2\\large \\pi}$ |\n| | $\\approx$ {{{correctAnswer}}} |\n\n</div>"}]},{"vars":[{"varval":"An electric scooter tyre has a circumference of 35 cm.\n\nWhich of these is closest to the radius of the tyre?\n"},{"varval":"<div class=\"aligned\">\n\n| | |\n| ----------------: | ----------------------- |\n| $C$ | \\= 2$\\large \\pi \\large r$ |\n| 35 | \\= 2$\\large \\pi \\large r$ |\n| $\\therefore\\ \\large r$ | \\= $\\dfrac{35}{2\\large \\pi}$ |\n| | $\\approx$ {{{correctAnswer}}} |\n\n</div>"}]},{"vars":[{"varval":"A car's tyre has a circumference of 223 cm.\n\nWhich of these is closest to the radius of the tyre?\n"},{"varval":"<div class=\"aligned\">\n\n| | |\n| ----------------: | ----------------------- |\n| $C$ | \\= 2$\\large \\pi \\large r$ |\n| 223 | \\= 2$\\large \\pi \\large r$ |\n| $\\therefore\\ \\large r$ | \\= $\\dfrac{223}{2\\large \\pi}$ |\n| | $\\approx$ {{{correctAnswer}}} |\n\n</div>"}]},{"vars":[{"varval":"An old wagon wheel has a circumference of 600 cm.\n\nWhich of these is closest to the radius of the wheel?\n"},{"varval":"<div class=\"aligned\">\n\n| | |\n| ----------------: | ----------------------- |\n| $C$ | \\= 2$\\large \\pi \\large r$ |\n| 600 | \\= 2$\\large \\pi \\large r$ |\n| $\\therefore\\ \\large r$ | \\= $\\dfrac{600}{2\\large \\pi}$ |\n| | $\\approx$ {{{correctAnswer}}} |\n\n</div>"}]}]

  936. Measurement, NAPX-H3-CA23

    <div class="sm_mode"> sm_img https://teacher.smartermaths.com.au/wp-content/uploads/2018/07/NAPX-H3-CA23.svg 420 indent vpad <br>Which of these could be used to calculate the area of the shape in square centimetres? </div>

    [{"vars":null}]

  937. Measurement, NAPX-H3-CA30

    <div class="sm_mode"> {{{question}}} </div>

    [{"vars":[{"varval":"Squares with sides 5 cm are cut out from the corners of a rectangular piece of cardboard.\n\nThe sides are then folded to make a rectangular box with no lid.\n\n<br>\n\nsm_img https://teacher.smartermaths.com.au/wp-content/uploads/2018/07/NAPX-H3-CA30.svg 710 indent vpad\n\nWhat is the volume of the box?\n"},{"varval":"<div class=\"aligned\">\n\n| | |\n| --------------------- | ------------------------------------------- |\n| Volume | \\= base area $\\times$ height |\n| | \\= (6 $\\times$ 2) $\\times$ 5 |\n| | \\= {{{correctAnswer}}} |\n\n</div>"}]},{"vars":[{"varval":"Squares with sides 4 cm are cut out from the corners of a rectangular piece of cardboard.\n\nThe sides are then folded to make a rectangular box with no lid.\n\n<br>\n\nsm_img https://teacher.smartermaths.com.au/wp-content/uploads/2022/08/Measurement_20309_v1.svg 770 indent vpad\n\nWhat is the volume of the box?\n"},{"varval":"<div class=\"aligned\">\n\n| | |\n| --------------------- | ------------------------------------------- |\n| Volume | \\= base area $\\times$ height |\n| | \\= (12 $\\times$ 4) $\\times$ 4 |\n| | \\= {{{correctAnswer}}} |\n\n</div>"}]},{"vars":[{"varval":"Squares with sides 3 cm are cut out from the corners of a rectangular piece of cardboard.\n\nThe sides are then folded to make a rectangular box with no lid.\n\n<br>\n\nsm_img https://teacher.smartermaths.com.au/wp-content/uploads/2022/08/Measurement_20309_v2_d.svg 770 indent vpad\n\nWhat is the volume of the box?\n"},{"varval":"<div class=\"aligned\">\n\n| | |\n| --------------------- | ------------------------------------------- |\n| Volume | \\= base area $\\times$ height |\n| | \\= (10 $\\times$ 5) $\\times$ 3 |\n| | \\= {{{correctAnswer}}} |\n\n</div>"}]},{"vars":[{"varval":"Squares with sides 6 cm are cut out from the corners of a rectangular piece of cardboard.\n\nThe sides are then folded to make a rectangular box with no lid.\n\n<br>\n\nsm_img https://teacher.smartermaths.com.au/wp-content/uploads/2022/08/Measurement_20309_v3.svg 600 indent vpad\n\nWhat is the volume of the box?"},{"varval":"<div class=\"aligned\">\n\n| | |\n| --------------------- | ------------------------------------------- |\n| Volume | \\= base area $\\times$ height |\n| | \\= (9 $\\times$ 21) $\\times$ 6 |\n| | \\= {{{correctAnswer}}} |\n\n</div>"}]},{"vars":[{"varval":"Squares with sides 4 cm are cut out from the corners of a rectangular piece of cardboard.\n\nThe sides are then folded to make a rectangular box with no lid.\n\n<br>\n\nsm_img https://teacher.smartermaths.com.au/wp-content/uploads/2022/08/Measurement_20309_v4.svg 600 indent vpad\n\nWhat is the volume of the box?"},{"varval":"<div class=\"aligned\">\n\n| | |\n| --------------------- | ------------------------------------------- |\n| Volume | \\= base area $\\times$ height |\n| | \\= (5 $\\times$ 7) $\\times$ 4 |\n| | \\= {{{correctAnswer}}} |\n\n</div>"}]},{"vars":[{"varval":"Squares with sides 10 cm are cut out from the corners of a rectangular piece of cardboard.\n\nThe sides are then folded to make a rectangular box with no lid.\n\n<br>\n\nsm_img https://teacher.smartermaths.com.au/wp-content/uploads/2022/08/Measurement_20309_v5.svg 600 indent vpad\n\nWhat is the volume of the box?"},{"varval":"<div class=\"aligned\">\n\n| | |\n| --------------------- | ------------------------------------------- |\n| Volume | \\= base area $\\times$ height |\n| | \\= (15 $\\times$ 9) $\\times$ 10 |\n| | \\= {{{correctAnswer}}} |\n\n</div>"}]}]

  938. Measurement, NAPX-I4-NC10

    <div class="sm_mode"> Rod is mowing a lawn in the shape of a parallelogram. <br> <div class="indent2"> sm_img https://teacher.smartermaths.com.au/wp-content/uploads/2018/04/NAPX-I4-NC10.svg 320 indent vpad </div> <br>What is the perimeter of the mown lawn? </div>

    [{"vars":null}]

  939. Measurement, NAPX-H3-CA18

    <div class="sm_mode"> {{{question}}} </div>

    [{"vars":[{"varval":"A rectangle has a length of 12 cm and a width of 7 cm.\n\nA square has the same perimeter as this rectangle.\n\nWhat is the side length of this square in centimetres?"},{"varval":"<div class=\"aligned\">\n\n| | |\n| --------------------- | ------------------------------------------- |\n| Perimeter | \\= (2 $\\times$ 12) + (2 $\\times$ 7) |\n| | \\= 38 cm |\n\n</div>\n\n<br>\n\n<div class=\"aligned\">\n\n| | |\n| --------------------- | ------------------------------------------- |\n| $\\therefore$ Square side | \\= 38 $\\div$ 4 |\n| | \\= {{{correctAnswer}}} cm |\n\n</div>\n"}]},{"vars":[{"varval":"A rectangle has a length of 8 cm and a width of 5 cm.\n\nA square has the same perimeter as this rectangle.\n\nWhat is the side length of this square in centimetres?"},{"varval":"<div class=\"aligned\">\n\n| | |\n| --------------------- | ------------------------------------------- |\n| Perimeter | \\= (2 $\\times$ 8) + (2 $\\times$ 5) |\n| | \\= 26 cm |\n\n</div>\n\n<br>\n\n<div class=\"aligned\">\n\n| | |\n| --------------------- | ------------------------------------------- |\n| $\\therefore$ Square side | \\= 26 $\\div$ 4 |\n| | \\= {{{correctAnswer}}} cm |\n\n</div>\n"}]},{"vars":[{"varval":"A rectangle has a length of 16 cm and a width of 19 cm.\n\nA square has the same perimeter as this rectangle.\n\nWhat is the side length of this square in centimetres?"},{"varval":"<div class=\"aligned\">\n\n| | |\n| --------------------- | ------------------------------------------- |\n| Perimeter | \\= (2 $\\times$ 16) + (2 $\\times$ 19) |\n| | \\= 70 cm |\n\n</div>\n\n<br>\n\n<div class=\"aligned\">\n\n| | |\n| --------------------- | ------------------------------------------- |\n| $\\therefore$ Square side | \\= 70 $\\div$ 4 |\n| | \\= {{{correctAnswer}}} cm |\n\n</div>\n"}]},{"vars":[{"varval":"A rectangle has a length of 41 cm and a width of 65 cm.\n\nA square has the same perimeter as this rectangle.\n\nWhat is the side length of this square in centimetres?"},{"varval":"<div class=\"aligned\">\n\n| | |\n| --------------------- | ------------------------------------------- |\n| Perimeter | \\= (2 $\\times$ 41) + (2 $\\times$ 65) |\n| | \\= 212 cm |\n\n</div>\n\n<br>\n\n<div class=\"aligned\">\n\n| | |\n| --------------------- | ------------------------------------------- |\n| $\\therefore$ Square side | \\= 212 $\\div$ 4 |\n| | \\= {{{correctAnswer}}} cm |\n\n</div>\n"}]},{"vars":[{"varval":"A rectangle has a length of 1.1 metres and a width of 65 cm.\n\nA square has the same perimeter as this rectangle.\n\nWhat is the side length of this square in centimetres?"},{"varval":"Using &nbsp;1 m = 100 cm\n\n<div class=\"aligned\">\n\n| | |\n| --------------------- | ------------------------------------------- |\n| Perimeter | \\= (2 $\\times$ 110) + (2 $\\times$ 65) |\n| | \\= 350 cm |\n\n</div>\n\n<br>\n\n<div class=\"aligned\">\n\n| | |\n| --------------------- | ------------------------------------------- |\n| $\\therefore$ Square side | \\= 350 $\\div$ 4 |\n| | \\= {{{correctAnswer}}} cm |\n\n</div>\n"}]},{"vars":[{"varval":"A rectangle has a length of 40 cm and a width of 58 cm.\n\nA square has the same perimeter as this rectangle.\n\nWhat is the side length of this square in centimetres?"},{"varval":"<div class=\"aligned\">\n\n| | |\n| --------------------- | ------------------------------------------- |\n| Perimeter | \\= (2 $\\times$ 40) + (2 $\\times$ 58) |\n| | \\= 196 cm |\n\n</div>\n\n<br>\n\n<div class=\"aligned\">\n\n| | |\n| --------------------- | ------------------------------------------- |\n| $\\therefore$ Square side | \\= 196 $\\div$ 4 |\n| | \\= {{{correctAnswer}}} cm |\n\n</div>\n"}]}]

  940. Measurement, NAPX-H3-CA06

    <div class="sm_mode"> What shape has an area greater than 4 square units? <br> <div class="indent1"> sm_img https://teacher.smartermaths.com.au/wp-content/uploads/2018/07/NAPX-H3-CA06.svg 390 indent vpad </div> </div>

    [{"vars":null}]

  941. Number, NAP-20261

    <div class="sm_mode"> {{{question}}} </div>

    [{"vars":[{"varval":"At sunrise the temperature is $-4$°C.\r\n\r\nAt midday the temperature is 13°C.\r\n\r\nWhich one of these calculations can be used to work out how many degrees warmer it is at midday than at sunrise? "},{"varval":"sm_nogap Degrees warmer\n\n>>= 13 $- (-4)$\n\n>>= {{{correctAnswer}}}\n"}]},{"vars":[{"varval":"At sunrise the temperature is $-6$°C.\r\n\r\nAt midday the temperature is 2°C.\r\n\r\nWhich one of these calculations can be used to work out how many degrees warmer it is at midday than at sunrise? "},{"varval":"sm_nogap Degrees warmer\n\n>>= 2 $- (-6)$\n\n>>= {{{correctAnswer}}}\n"}]},{"vars":[{"varval":"At midday the temperature is $12$°C.\r\n\r\nAt 9pm the temperature is −3°C.\r\n\r\nWhich one of these calculations can be used to work out how many degrees warmer it is at midday than at 9 pm? "},{"varval":"sm_nogap Degrees warmer\n\n>>= 12 $- (-3)$\n\n>>= {{{correctAnswer}}}\n"}]},{"vars":[{"varval":"At sunrise the temperature is $-4$°C.\r\n\r\nAt 11pm the temperature is $-2$°C.\r\n\r\nWhich one of these calculations can be used to work out how many degrees warmer it is at 11 pm than at sunrise? "},{"varval":"sm_nogap Degrees warmer\n\n>>= $-2$ $- (-4)$\n\n>>= {{{correctAnswer}}}\n"}]},{"vars":[{"varval":"At sunrise the temperature is $-8$°C.\r\n\r\nAt sunset the temperature is $-4$°C.\r\n\r\nWhich one of these calculations can be used to work out how many degrees warmer it is at sunset than at sunrise? "},{"varval":"sm_nogap Degrees warmer\n\n>>= $-4$ $- (-8)$\n\n>>= {{{correctAnswer}}}\n"}]},{"vars":[{"varval":"At midday the temperature is $10$°C.\r\n\r\nAt sunset the temperature is $-6$°C.\r\n\r\nWhich one of these calculations can be used to work out how many degrees warmer it is at midday than at sunset? "},{"varval":"sm_nogap Degrees warmer\n\n>>= $10$ $- (-6)$\n\n>>= {{{correctAnswer}}}\n"}]}]

  942. Measurement, NAP-F4-CA16

    <div class="sm_mode"> A child's bike tyre has a circumference of 90 cm. Which of these is closest to the radius of the circle? </div>

    [{"vars":null}]

  943. Measurement, NAP-A4-CA17

    <div class="sm_mode"> {{{question}}} </div>

    [{"vars":[{"varval":"Here is a plan of Dave's backyard.\n\n<br>\n\nsm_img https://teacher.smartermaths.com.au/wp-content/uploads/2017/12/NAP-A4-CA17-1.svg 450 indent vpad\n\n<br>The total area of the paving stones is 18 m$^2$.\n\nWhat is the total area of the grass in Dave's backyard?\n"},{"varval":"9 paving stones = 18 m$^2$\n\n$\\therefore$ 1 grid square = 2 m$^2$\n\n<div class=\"aligned\">\n\n| | |\n| --------------------- | -------------------------------------------- |\n| Total grass area | = 27 grid squares |\n| | = 27 $\\times$ 2 |\n| | = {{{correctAnswer}}} |\n\n</div>\n"}]},{"vars":[{"varval":"Here is a plan of Garth's tiled outdoor area.\n\n<br>\n\nsm_img https://teacher.smartermaths.com.au/wp-content/uploads/2022/08/Measurement_20303_v1a.svg 400 indent vpad\n\n<br>The total area of the dark tiles is 48 m$^2$.\n\nWhat is the total area of the light tiles in Garth's outdoor area?\n"},{"varval":"24 dark tiles = 48 m$^2$\n\n$\\therefore$ 1 grid square = 2 m$^2$\n\n<div class=\"aligned\">\n\n| | |\n| --------------------- | -------------------------------------------- |\n| Total light tile area | = [(11 $\\times$ 5) $-$ 24] grid squares |\n| | = 31 $\\times$ 2 |\n| | = {{{correctAnswer}}} |\n\n</div>\n"}]},{"vars":[{"varval":"Here is a plan of Rita's tiled hallway area.\n\n<br>\n\nsm_img https://teacher.smartermaths.com.au/wp-content/uploads/2022/08/Measurement_20303_v2.svg 350 indent vpad\n\n<br>The total area of the dark tiles is 15 m$^2$.\n\nWhat is the total area of the light tiles in Rita's hallway area?\n"},{"varval":"30 dark tiles = 15 m$^2$\n\n$\\therefore$ 1 grid square = 0.5 m$^2$\n\n<div class=\"aligned\">\n\n| | |\n| --------------------- | -------------------------------------------- |\n| Total light tile area | = [(10 $\\times$ 4) $-$ 30] grid squares |\n| | = 10 $\\times$ 0.5 |\n| | = {{{correctAnswer}}} |\n\n</div>\n"}]},{"vars":[{"varval":"Here is a plan of Reece's tiled courtyard area.\n\n<br>\n\nsm_img https://teacher.smartermaths.com.au/wp-content/uploads/2022/08/Measurement_20303_v3.svg 400 indent vpad\n\n<br>The total area of the dark tiles is 66 m$^2$.\n\nWhat is the total area of the light tiles in Reece's courtyard area?\n"},{"varval":"22 dark tiles = 66 m$^2$\n\n$\\therefore$ 1 grid rectangle = 3 m$^2$\n\n<div class=\"aligned\">\n\n| | |\n| --------------------- | -------------------------------------------- |\n| Total light tile area | = [(13 $\\times$ 5) $-$ 22] grid rectangles |\n| | = 43 $\\times$ 3 |\n| | = {{{correctAnswer}}} |\n\n</div>\n"}]},{"vars":[{"varval":"Here is a plan of a stained glass window in a cathedral.\n\n<br>\n\nsm_img https://teacher.smartermaths.com.au/wp-content/uploads/2022/08/Measurement_20303_v4a.svg 410 indent vpad\n\n<br>The total area of the clear glass is 36 m$^2$.\n\nWhat is the total area of the coloured glass in the cathedral window?"},{"varval":"18 clear glass squares = 36 m$^2$\n\n$\\therefore$ 1 grid square = 2 m$^2$\n\n\n<div class=\"aligned\">\n\n| | |\n| --------------------- | -------------------------------------------- |\n| Total coloured glass area | = [(8 $\\times$ 4) $-$ 18] grid squares |\n| | = 14 $\\times$ 2 |\n| | = {{{correctAnswer}}} |\n\n</div>\n"}]},{"vars":[{"varval":"Here is a plan of Jonathon's vegetable garden area.\n\n<br>\n\nsm_img https://teacher.smartermaths.com.au/wp-content/uploads/2022/08/Measurement_20303_v5.svg 500 indent vpad\n\n<br>The total area of the seedlings is 12 m$^2$.\n\nWhat is the total area of the woodchips in Jonathon's vegetable garden area?"},{"varval":"24 seedlings = 12 m$^2$\n\n$\\therefore$ 1 grid square = 0.5 m$^2$\n\n<div class=\"aligned\">\n\n| | |\n| --------------------- | -------------------------------------------- |\n| Total light tile area | = [(12 $\\times$ 5) $-$ 24] grid rectangles |\n| | = 36 $\\times$ 0.5 |\n| | = {{{correctAnswer}}} |\n\n</div>\n"}]}]

  944. Measurement, NAP-A4-CA15

    <div class="sm_mode"> {{{question}}} </div>

    [{"vars":[{"varval":"Pillar draws a circle with a diameter of 3.2 cm.\n\nWhat is the circumference of the circle to the nearest centimetre?"},{"varval":"<div class=\"aligned\">\n\n| | |\n| --------------------- | -------------------------------------------- |\n| $C$ | = 2$\\large \\pi r$ |\n| | = $\\large \\pi d$ |\n||= $3.142 \\ \\times$ 3.2|\n| | $\\approx$ {{{correctAnswer}}} |\n\n</div>"}]},{"vars":[{"varval":"Michail draws a circle with a diameter of 4.5 cm.\n\nWhat is the circumference of the circle to the nearest centimetre?"},{"varval":"<div class=\"aligned\">\n\n| | |\n| --------------------- | -------------------------------------------- |\n| $C$ | = 2$\\large \\pi r$ |\n| | = $\\large \\pi d$ |\n||= $3.142 \\ \\times$ 4.5|\n| | $\\approx$ {{{correctAnswer}}} |\n\n</div>"}]},{"vars":[{"varval":"Vanessa draws a circle with a diameter of 22.6 cm.\n\nWhat is the circumference of the circle to the nearest centimetre?"},{"varval":"<div class=\"aligned\">\n\n| | |\n| --------------------- | -------------------------------------------- |\n| $C$ | = 2$\\large \\pi r$ |\n| | = $\\large \\pi d$ |\n||= $3.142 \\ \\times$ 22.6|\n| | $\\approx$ {{{correctAnswer}}} |\n\n</div>"}]},{"vars":[{"varval":"Prince draws a circle with a diameter of 28 mm.\n\nWhat is the circumference of the circle to the nearest millimetre?"},{"varval":"<div class=\"aligned\">\n\n| | |\n| --------------------- | -------------------------------------------- |\n| $C$ | = 2$\\large \\pi r$ |\n| | = $\\large \\pi d$ |\n||= $3.142 \\ \\times$ 28|\n| | $\\approx$ {{{correctAnswer}}} |\n\n</div>"}]},{"vars":[{"varval":"Scott marks a circle on the ground with a diameter of 19.5 m.\n\nWhat is the circumference of the circle to the nearest metre?"},{"varval":"<div class=\"aligned\">\n\n| | |\n| --------------------- | -------------------------------------------- |\n| $C$ | = 2$\\large \\pi r$ |\n| | = $\\large \\pi d$ |\n||= $3.142 \\ \\times$ 19.5|\n| | $\\approx$ {{{correctAnswer}}} |\n\n</div>"}]},{"vars":[{"varval":"Tommy draws a circle on the whiteboard with a diameter of 16.3 cm.\n\nWhat is the circumference of the circle to the nearest centimetre?"},{"varval":"<div class=\"aligned\">\n\n| | |\n| --------------------- | -------------------------------------------- |\n| $C$ | = 2$\\large \\pi r$ |\n| | = $\\large \\pi d$ |\n||= $3.142 \\ \\times$ 16.3|\n| | $\\approx$ {{{correctAnswer}}} |\n\n</div>"}]}]

  945. Measurement, NAP-D4-NC13

    <div class="sm_mode"> {{{question}}} </div>

    [{"vars":[{"varval":"Pablo cuts a square out of a rectangular piece of paper, as shown below.\n\n<br>\n\nsm_img https://teacher.smartermaths.com.au/wp-content/uploads/2022/08/Measurement_NAP-D4-NC13_v0.svg 330 indent3 vpad\n\n<br>The square Pablo cut out has a side length of 6 cm.\n\nWhich of these expressions gives the area of Pablo's piece of paper after cutting out the square?\n"},{"varval":"<div class=\"aligned\">\n\n| | |\n| --------------------- | -------------------------------------------- |\n| Area | = Area of larger rectangle $-$ Area of square |\n| | = {{{correctAnswer}}} |\n\n</div>"}]},{"vars":[{"varval":"Jacko cuts a square out of a rectangular piece of paper, as shown below.\n\n<br>\n\nsm_img https://teacher.smartermaths.com.au/wp-content/uploads/2022/08/Measurement_NAP-D4-NC13_v1.svg 310 indent3 vpad\n\n<br>The square Jacko cut out has a side length of 5 cm.\n\nWhich of these expressions gives the area of Jacko's piece of paper after cutting out the square?\n"},{"varval":"<div class=\"aligned\">\n\n| | |\n| --------------------- | -------------------------------------------- |\n| Area | = Area of larger rectangle $-$ Area of square |\n| |= {{{correctAnswer}}}|\n\n</div>"}]},{"vars":[{"varval":"Clint cuts a square out of a rectangular piece of paper, as shown below.\n\n<br>\n\nsm_img https://teacher.smartermaths.com.au/wp-content/uploads/2022/08/Measurement_NAP-D4-NC13_v2.svg 330 indent3 vpad\n\n<br>The square Clint cut out has a side length of 12 cm.\n\nWhich of these expressions gives the area of Clint's piece of paper after cutting out the square?\n"},{"varval":"<div class=\"aligned\">\n\n| | |\n| --------------------- | -------------------------------------------- |\n| Area | = Area of larger rectangle $-$ Area of square |\n| | = {{{correctAnswer}}} |\n\n</div>"}]},{"vars":[{"varval":"Mateo cuts a square out of a rectangular piece of paper, as shown below.\n\n<br>\n\nsm_img https://teacher.smartermaths.com.au/wp-content/uploads/2022/08/Measurement_NAP-D4-NC13_v3.svg 330 indent3 vpad\n\n<br>The square Mateo cut out has a side length of 20 cm.\n\nWhich of these expressions gives the area of Mateo's piece of paper after cutting out the square?\n"},{"varval":"<div class=\"aligned\">\n\n| | |\n| --------------------- | -------------------------------------------- |\n| Area | = Area of larger rectangle $-$ Area of square |\n| | = {{{correctAnswer}}} |\n\n</div>"}]},{"vars":[{"varval":"Lola cuts a square out of a rectangular piece of paper, as shown below.\n\n<br>\n\nsm_img https://teacher.smartermaths.com.au/wp-content/uploads/2022/08/Measurement_NAP-D4-NC13_v4.svg 370 indent3 vpad\n\n<br>The square Lola cut out has a side length of 25 cm.\n\nWhich of these expressions gives the area of Lola's piece of paper after cutting out the square?\n"},{"varval":"<div class=\"aligned\">\n\n| | |\n| --------------------- | -------------------------------------------- |\n| Area | = Area of larger rectangle $-$ Area of square |\n| | = {{{correctAnswer}}} |\n\n</div>"}]},{"vars":[{"varval":"Leticia cuts a square out of a rectangular piece of paper, as shown below.\n\n<br>\n\nsm_img https://teacher.smartermaths.com.au/wp-content/uploads/2022/08/Measurement_NAP-D4-NC13_v5.svg 400 indent3 vpad\n\n<br>The square Leticia cut out has a side length of 4 cm.\n\nWhich of these expressions gives the area of Leticia's piece of paper after cutting out the square?\n"},{"varval":"<div class=\"aligned\">\n\n| | |\n| --------------------- | -------------------------------------------- |\n| Area | = Area of larger rectangle $-$ Area of square |\n| | = {{{correctAnswer}}} |\n\n</div>"}]}]

  946. Michelle is tiling a walkway that is 5.6 metres long.

    <div class="sm_mode"> {{{question}}} </div>

    [{"vars":[{"varval":"Michelle is tiling a walkway that is 5.6 metres long.\n\nShe is using cement tiles that are 0.08 metres wide.\n\nThere are no gaps between the tiles.\n\n<br>\n\nsm_img https://teacher.smartermaths.com.au/wp-content/uploads/2017/02/naplan-Y7-2009-15mc.png 220 indent vpad\n\n<br>How should Michelle calculate how many cement tiles she will need?\n"},{"varval":"{{{correctAnswer}}}\n"}]},{"vars":[{"varval":"Mitchell is covering a walkway that is 8.4 metres long.\n\nHe is using timber slats that are 0.04 metres wide.\n\nThere are no gaps between the slats.\n\n<br>\n\nsm_img https://teacher.smartermaths.com.au/wp-content/uploads/2022/08/Measurement_20300_v2.svg 360 indent3 vpad\n\n<br>How should Mitchell calculate how many timber slats he will need?\n"},{"varval":"{{{correctAnswer}}}\n"}]},{"vars":[{"varval":"Gladys is tiling a wall that is 3.6 metres high.\n\nShe is using ceramic tiles that are 0.06 metres wide.\n\nThere are no gaps between the tiles.\n\n<br>\n\nsm_img https://teacher.smartermaths.com.au/wp-content/uploads/2022/08/Measurement_20300_v1.svg 300 indent3 vpad\n\n<br>How should Gladys calculate how many ceramic tiles she will need?\n"},{"varval":"{{{correctAnswer}}}\n"}]},{"vars":[{"varval":"Jerome is laying flooring on a balcony that is 6.5 metres long.\n\nHe is using laminate boards that are 0.15 metres wide.\n\nThere are no gaps between the boards.\n\n<br>\n\nsm_img https://teacher.smartermaths.com.au/wp-content/uploads/2022/08/Measurement_20300_v3.svg 270 indent3 vpad\n\n<br>How should Jerome calculate how many laminate boards he will need?\n"},{"varval":"{{{correctAnswer}}}\n"}]},{"vars":[{"varval":"Maxine is laying tiles on a balcony that is 4.0 metres long.\n\nShe is using paving tiles that are 0.2 metres wide.\n\nThere are no gaps between the tiles.\n\n<br>\n\nsm_img https://teacher.smartermaths.com.au/wp-content/uploads/2022/08/Measurement_20300_v4.svg 250 indent3 vpad\n\n<br>How should Maxine calculate how many paving tiles she will need?\n"},{"varval":"{{{correctAnswer}}}"}]},{"vars":[{"varval":"Louis is laying pavers on a footpath that is 9.6 metres long.\n\nHe is using concrete pavers that are 0.30 metres wide.\n\nThere are no gaps between the pavers.\n\n<br>\n\nsm_img https://teacher.smartermaths.com.au/wp-content/uploads/2022/08/Measurement_20300_v5.svg 230 indent3 vpad\n\n<br>How should Louis calculate how many concrete pavers he will need?"},{"varval":"{{{correctAnswer}}}\n"}]}]

  947. Algebra, NAP-D4-CA07

    <div class="sm_mode"> The surface area ($\large S$) of the prism below can be calculated by multiplying the perimeter of its face ($\large p$) by its height ($\large h$), and adding twice the area of its face ($F$). <br> sm_img https://teacher.smartermaths.com.au/wp-content/uploads/2017/02/naplan-2011-7mc.png 200 indent3 vpad <br>Which one of these formulas could be used for this calculation? </div>

    [{"vars":null}]

  948. Algebra, NAP-F4-NC12

    <div class="sm_mode"> The rhombus below has a side length $\large d$. <br> sm_img https://teacher.smartermaths.com.au/wp-content/uploads/2016/12/NAP-F4-NC12.png 150 indent3 vpad <br>Which expression **cannot** be used for the perimeter? </div>

    [{"vars":null}]

  949. Measurement, NAP-I4-NC10

    <div class="sm_mode"> {{{question}}} </div>

    [{"vars":[{"varval":"Oliver is mowing a lawn in the shape of a parallelogram.\n\n<br>\n\nsm_img https://teacher.smartermaths.com.au/wp-content/uploads/2016/12/NAP-169-NC010.png 280 indent3 vpad\n\n<br>What is the perimeter of the mown lawn?\n"},{"varval":"<div class=\"aligned\">\n\n| | |\n| --------------------- | -------------------------------------------- |\n| Perimeter | = 2 $\\times$ (50 + 65) |\n| | = 2 $\\times$ 115 |\n| | = {{{correctAnswer}}} |\n\n</div>\n"}]},{"vars":[{"varval":"Joyce is walking her dog around a paddock in the shape of a parallelogram.\n\n<br>\n\nsm_img https://teacher.smartermaths.com.au/wp-content/uploads/2022/08/Measurement_20297_v1.svg 280 indent3 vpad\n\n<br>What is the perimeter of the paddock?\n"},{"varval":"<div class=\"aligned\">\n\n| | |\n| --------------------- | -------------------------------------------- |\n| Perimeter | = 2 $\\times$ (1.4 + 2.3) |\n| | = 2 $\\times$ 3.7 |\n| | = {{{correctAnswer}}} |\n\n</div>\n"}]},{"vars":[{"varval":"Barnaby is running around a track in the shape of a parallelogram.\n\n<br>\n\nsm_img https://teacher.smartermaths.com.au/wp-content/uploads/2022/08/Measurement_20297_v2.svg 330 indent3 vpad\n\n<br>What is the perimeter of the running track?\n"},{"varval":"<div class=\"aligned\">\n\n| | |\n| --------------------- | -------------------------------------------- |\n| Perimeter | = 2 $\\times$ (315 + 247) |\n| | = 2 $\\times$ 562 |\n| | = {{{correctAnswer}}} |\n\n</div>\n"}]},{"vars":[{"varval":"Marina is cutting a piece of material in the shape of a parallelogram.\n\n<br>\n\nsm_img https://teacher.smartermaths.com.au/wp-content/uploads/2022/08/Measurement_20297_v3.svg 300 indent3 vpad\n\n<br>What is the perimeter of the piece of material?\n"},{"varval":"<div class=\"aligned\">\n\n| | |\n| --------------------- | -------------------------------------------- |\n| Perimeter | = 2 $\\times$ (15.5 + 3.5) |\n| | = 2 $\\times$ 19 |\n| | = {{{correctAnswer}}} |\n\n</div>\n"}]},{"vars":[{"varval":"Suki is using tiles in the shape of a parallelogram to tile her bathroom.\n\n<br>\n\nsm_img https://teacher.smartermaths.com.au/wp-content/uploads/2022/08/Measurement_20297_v4.svg 310 indent3 vpad\n\n<br>What is the perimeter of the tiles?\n"},{"varval":"<div class=\"aligned\">\n\n| | |\n| --------------------- | -------------------------------------------- |\n| Perimeter | = 2 $\\times$ (15 + 10) |\n| | = 2 $\\times$ 25 |\n| | = {{{correctAnswer}}} |\n\n</div>\n"}]},{"vars":[{"varval":"Bowen painting a mural in the shape of a parallelogram.\n\n<br>\n\nsm_img https://teacher.smartermaths.com.au/wp-content/uploads/2022/08/Measurement_20297_v5.svg 280 indent3 vpad\n\n<br>What is the perimeter of the mural?\n"},{"varval":"<div class=\"aligned\">\n\n| | |\n| --------------------- | -------------------------------------------- |\n| Perimeter | = 2 $\\times$ (5.2 + 3.8) |\n| | = 2 $\\times$ 9 |\n| | = {{{correctAnswer}}} |\n\n</div>"}]}]

  950. Algebra, NAP-B4-NC08

    <div class="sm_mode"> {{{question}}} </div>

    [{"vars":[{"varval":"Olive drew this plan of her lawn.\n\n<br>\n\nsm_img https://teacher.smartermaths.com.au/wp-content/uploads/2017/12/nap-b4-nc08a.svg 220 indent vpad\n\n<br>Which expression gives the area of Olive's lawn?\n"},{"varval":"sm_img https://teacher.smartermaths.com.au/wp-content/uploads/2017/12/nap-b4-nc08b.svg 240 indent vpad\n\n<div class=\"aligned\">\n\n| | |\n| --------------------- | -------------------------------------------- |\n| Total Area | = Area 1 + Area 2 |\n| | = {{{correctAnswer}}} |\n\n</div>"}]},{"vars":[{"varval":"Atticus drew this plan of his living area.\n\n<br>\n\nsm_img https://teacher.smartermaths.com.au/wp-content/uploads/2022/08/Measurement_20296_v1q.svg 320 indent3 vpad\n\n<br>Which expression gives the area of Atticus's living area?\n"},{"varval":"sm_img https://teacher.smartermaths.com.au/wp-content/uploads/2022/08/Measurement_20296_v1ws.svg 320 indent3 vpad\n\n<div class=\"aligned\">\n\n| | |\n| --------------------- | -------------------------------------------- |\n| Total Area | = Area 1 + Area 2 |\n| | = {{{correctAnswer}}} |\n\n</div>"}]},{"vars":[{"varval":"Vera drew this plan of her entertaining area.\n\n<br>\n\nsm_img https://teacher.smartermaths.com.au/wp-content/uploads/2022/08/Measurement_20296_v2q.svg 340 indent3 vpad\n\n<br>Which expression gives the area of Vera's entertaining area?\n"},{"varval":"sm_img https://teacher.smartermaths.com.au/wp-content/uploads/2022/08/Measurement_20296_v2ws_2.svg 340 indent3 vpad\n\n<div class=\"aligned\">\n\n| | |\n| --------------------- | -------------------------------------------- |\n| Total Area | = Area 1 + Area 2 + Area 3 |\n| | = {{{correctAnswer}}} |\n\n</div>"}]},{"vars":[{"varval":"Bernie drew this plan of his timber deck.\n\n<br>\n\nsm_img https://teacher.smartermaths.com.au/wp-content/uploads/2022/08/Measurement_20296_v3q.svg 400 indent3 vpad\n\n<br>Which expression gives the area of Bernie's timber deck?\n"},{"varval":"sm_img https://teacher.smartermaths.com.au/wp-content/uploads/2022/08/Measurement_20296_v3ws.svg 400 indent3 vpad\n\n<div class=\"aligned\">\n\n| | |\n| --------------------- | -------------------------------------------- |\n| Total Area | = Area 1 $-$ Area 2 |\n| | = {{{correctAnswer}}} |\n\n</div>"}]},{"vars":[{"varval":"Tran drew this plan of the gym area in his garage.\n\n<br>\n\nsm_img https://teacher.smartermaths.com.au/wp-content/uploads/2022/08/Measurement_20296_v4q.svg 350 indent3 vpad\n\n<br>Which expression gives the area of Tran's gym area?\n"},{"varval":"sm_img https://teacher.smartermaths.com.au/wp-content/uploads/2022/08/Measurement_20296_v4ws.svg 350 indent3 vpad\n\n<div class=\"aligned\">\n\n| | |\n| --------------------- | -------------------------------------------- |\n| Total Area | = Area 1 + Area 2 |\n| | = {{{correctAnswer}}} |\n\n</div>"}]},{"vars":[{"varval":"Aubrey drew this plan of his garden area.\n\n<br>\n\nsm_img https://teacher.smartermaths.com.au/wp-content/uploads/2022/08/Measurement_20296_v5q.svg 350 indent3 vpad\n\n<br>Which expression gives the area of Aubrey's garden area?\n"},{"varval":"sm_img https://teacher.smartermaths.com.au/wp-content/uploads/2022/08/Measurement_20296_v5ws.svg 350 indent3 vpad\n\n<div class=\"aligned\">\n\n| | |\n| --------------------- | -------------------------------------------- |\n| Total Area | = Area 1 $-$ Area 2 |\n| | = {{{correctAnswer}}} |\n\n</div>"}]}]

  951. Measurement, NAP-K4-CA10 ISSUE: Question image .. 1- move "not to scale" further right, and 2-colour the image

    <div class="sm_mode"> {{{question}}} </div>

    [{"vars":[{"varval":"Adam is laying tiles that are shaped like a parallelogram.\n\n<br>\n\nsm_img https://teacher.smartermaths.com.au/wp-content/uploads/2018/06/NAPX-K4-CA10.svg 218 indent3 vpad\n\n<br>The longer sides are 2.5 times the length of the shorter sides.\n\nWhat is the perimeter of one of the tiles?\n"},{"varval":"<div class=\"aligned\">\n\n| | |\n| --------------------- | -------------------------------------------- |\n| Perimeter | = 12 $\\times$ 2 + (12 $\\times$ 2.5) $\\times$ 2 |\n| | = 24 + 30 $\\times$ 2 |\n| | = 24 + 60 |\n| | = {{{correctAnswer}}} |\n\n</div>"}]},{"vars":[{"varval":"Fumiko is laying pavers that are shaped like a parallelogram.\n\n<br>\n\nsm_img https://teacher.smartermaths.com.au/wp-content/uploads/2022/08/Measurement_20295_v1.svg 240 indent3 vpad\n\n<br>The longer sides are 3.5 times the length of the shorter sides.\n\nWhat is the perimeter of one of the pavers?\n"},{"varval":"<div class=\"aligned\">\n\n| | |\n| --------------------- | -------------------------------------------- |\n| Perimeter | = 8 $\\times$ 2 + (8 $\\times$ 3.5) $\\times$ 2 |\n| | = 16 + 28 $\\times$ 2 |\n| | = 16 + 56 |\n| | = {{{correctAnswer}}} |\n\n</div>"}]},{"vars":[{"varval":"Arima is laying tiles that are shaped like a parallelogram.\n\n<br>\n\nsm_img https://teacher.smartermaths.com.au/wp-content/uploads/2022/08/Measurement_20295_v2.svg 350 indent3 vpad\n\n<br>The longer sides are 4.25 times the length of the shorter sides.\n\nWhat is the perimeter of one of the tiles?\n"},{"varval":"<div class=\"aligned\">\n\n| | |\n| --------------------- | -------------------------------------------- |\n| Perimeter | = 4 $\\times$ 2 + (4 $\\times$ 4.25) $\\times$ 2 |\n| | = 8 + 17 $\\times$ 2 |\n| | = 8 + 34 |\n| | = {{{correctAnswer}}} |\n\n</div>"}]},{"vars":[{"varval":"Bjork is laying ice bricks that are shaped like a parallelogram.\n\n<br>\n\nsm_img https://teacher.smartermaths.com.au/wp-content/uploads/2022/08/Measurement_20295_v3.svg 260 indent3 vpad\n\n<br>The shorter sides are half the length of the longer sides.\n\nWhat is the perimeter of one of the bricks?\n"},{"varval":"<div class=\"aligned\">\n\n| | |\n| --------------------- | -------------------------------------------- |\n| Perimeter | = $\\bigg( \\dfrac{1}{2} \\times 26 \\bigg)\\times$ 2 + 26 $\\times$ 2 |\n| | = 13 $\\times$ 2 + 52|\n| | = 26 + 52|\n| | = {{{correctAnswer}}} |\n\n</div>"}]},{"vars":[{"varval":"Vladimir is cutting a piece of coloured glass into the shape of a parallelogram to use in a leadlight window.\n\n<br>\n\nsm_img https://teacher.smartermaths.com.au/wp-content/uploads/2022/08/Measurement_20295_v4.svg 300 indent3 vpad\n\n<br>The shorter sides are one-third the length of the longer sides.\n\nWhat is the perimeter of the piece of coloured glass?\n"},{"varval":"<div class=\"aligned\">\n\n| | |\n| --------------------- | -------------------------------------------- |\n| Perimeter | = $\\bigg( \\dfrac{1}{3} \\times 36 \\bigg)\\times$ 2 + 36 $\\times$ 2 |\n| | = 12 $\\times$ 2 + 72|\n| | = 24 + 72|\n| | = {{{correctAnswer}}} |\n\n</div>"}]},{"vars":[{"varval":"Tatiana is cutting a piece of material in the shape of a parallelogram for a sewing project.\n\n<br>\n\nsm_img https://teacher.smartermaths.com.au/wp-content/uploads/2022/08/Measurement_20295_v5.svg 300 indent3 vpad\n\n<br>The longer sides are one and two-thirds the length of the shorter sides.\n\nWhat is the perimeter of the piece of material?\n"},{"varval":"<div class=\"aligned\">\n\n| | |\n| --------------------- | -------------------------------------------- |\n| Perimeter | = $\\bigg($ 18 + $\\bigg( \\dfrac{2}{3} \\times 18 \\bigg)\\bigg)\\times$ 2 + 18 $\\times$ 2 |\n| | = (18 + 12) $\\times$ 2 + 36|\n| | = 60 + 36|\n| | = {{{correctAnswer}}} |\n\n</div>"}]}]

  952. Measurement, NAP-C4-CA05

    <div class="sm_mode"> A shape, pictured below, is made with 5 rhombuses. <br> sm_img https://teacher.smartermaths.com.au/wp-content/uploads/2017/01/naplan-Y7-2010-5mca.png 220 indent3 vpad <br>What is the perimeter of the shape? </div>

    [{"vars":null}]

  953. Measurement, NAP-A4-NC02

    <div class="sm_mode"> The table shows the lengths and widths of rectangles with an area of 15 cm$^2$. <br> <div class="sm-table col1-color3"> >>| Length (cm) | 1|1.5|3|5|10|15| |:-:|:-:|:-:|:-:|:-:|:-:|:-:| | Width (cm) | 15|10|5|3|1.5|1| </div> <br>Which graph shows the information in the table for length against width? </div>

    [{"vars":null}]

  954. Number, NAP-72098

    <div class="sm_mode"> {{{question}}} </div>

    [{"vars":[{"varval":"Which of the following changes is the smallest?"},{"varval":"sm_nogap Consider each option:\n\n\n<div class=\"aligned\">\n\n| | |\n| --------------------: | -------------- |\n| −8$\\degree$C to −4$\\degree$C | = 4$\\degree$ change |\n|−5$\\degree$C to −2$\\degree$C| = 3$\\degree$ change |\n| −2$\\degree$C to 3$\\degree$C| = 5$\\degree$ change |\n| 3$\\degree$C to 7$\\degree$C| = 4$\\degree$ change |\n\n</div>\n\n<br>\n\n\n$\\therefore$ {{{correctAnswer}}} is the smallest change\n"}]},{"vars":[{"varval":"Which of the following changes is the largest?"},{"varval":"sm_nogap Consider each option:\n\n\n<div class=\"aligned\">\n\n| | |\n| --------------------: | -------------- |\n| −8$\\degree$C to −4$\\degree$C | = 4$\\degree$ change |\n|−5$\\degree$C to −2$\\degree$C| = 3$\\degree$ change |\n| −2$\\degree$C to 3$\\degree$C| = 5$\\degree$ change |\n| 3$\\degree$C to 7$\\degree$C| = 4$\\degree$ change |\n\n</div>\n\n<br>\n\n\n$\\therefore$ {{{correctAnswer}}} is the largest change"}]},{"vars":[{"varval":"Which of the following changes is the smallest?"},{"varval":"sm_nogap Consider each option:\n\n\n<div class=\"aligned\">\n\n| | |\n| --------------------: | -------------- |\n| −1$\\degree$C to −4$\\degree$C | = 3$\\degree$ change |\n|−3$\\degree$C to 1$\\degree$C| = 4$\\degree$ change |\n| −10$\\degree$C to −8$\\degree$C| = 2$\\degree$ change |\n| 1$\\degree$C to 4$\\degree$C| = 3$\\degree$ change |\n\n</div>\n\n<br>\n\n\n$\\therefore$ {{{correctAnswer}}} is the smallest change\n"}]},{"vars":[{"varval":"Which of the following changes is the largest?"},{"varval":"sm_nogap Consider each option:\n\n\n<div class=\"aligned\">\n\n| | |\n| --------------------: | -------------- |\n| −1$\\degree$C to −4$\\degree$C | = 3$\\degree$ change |\n|−3$\\degree$C to 1$\\degree$C| = 4$\\degree$ change |\n| −10$\\degree$C to −8$\\degree$C| = 2$\\degree$ change |\n| 1$\\degree$C to 4$\\degree$C| = 3$\\degree$ change |\n\n</div>\n\n<br>\n\n\n$\\therefore$ {{{correctAnswer}}} is the largest change\n"}]},{"vars":[{"varval":"Which of the following changes is the smallest?"},{"varval":"sm_nogap Consider each option:\n\n\n<div class=\"aligned\">\n\n| | |\n| --------------------: | -------------- |\n| −7$\\degree$C to −3$\\degree$C | = 4$\\degree$ change |\n|−1$\\degree$C to 4$\\degree$C| = 5$\\degree$ change |\n| −3$\\degree$C to 1$\\degree$C| = 4$\\degree$ change |\n| −5$\\degree$C to −2$\\degree$C| = 3$\\degree$ change |\n\n</div>\n\n<br>\n\n\n$\\therefore$ {{{correctAnswer}}} is the smallest change\n"}]},{"vars":[{"varval":"Which of the following changes is the largest?"},{"varval":"sm_nogap Consider each option:\n\n\n<div class=\"aligned\">\n\n| | |\n| --------------------: | -------------- |\n|−1$\\degree$C to 4$\\degree$C| = 5$\\degree$ change |\n| −7$\\degree$C to −3$\\degree$C | = 4$\\degree$ change |\n| −3$\\degree$C to 1$\\degree$C| = 4$\\degree$ change |\n| −5$\\degree$C to −2$\\degree$C| = 3$\\degree$ change |\n\n</div>\n\n<br>\n\n\n$\\therefore$ {{{correctAnswer}}} is the largest change\n"}]}]

  955. Number, NAP-49700

    <div class="sm_mode"> Two arrows point to numbers on a number line. <br> {{image}} <br>Which one of these numbers lies between the two arrows? </div>

    [{"vars":[{"varval":"sm_img https://teacher.smartermaths.com.au/wp-content/uploads/2016/12/naplan-2014-7mci.png 350 indent3 vpad"}]},{"vars":[{"varval":"sm_img https://teacher.smartermaths.com.au/wp-content/uploads/2018/05/NAPX-G4-CA07.svg 350 indent3 vpad"}]}]

  956. Number, NAPX-H2-19

    <div class="sm_mode"> Magnus bought 3 chocolate bars that cost \$2.30 each. He gave the shopkeeper a \$10 note. How much change should Magnus expect? </div>

    [{"vars":null}]

  957. Number, NAPX-p121319v01

    <div class="sm_mode"> {{{question}}} </div>

    [{"vars":[{"varval":"Jeeves buys 3 water bottles for \\$1.10 each.\n\nHe pays for these water bottles with a \\$5 note.\n\nHow much change should Jeeves receive?"},{"varval":"<div class=\"aligned\">\n\n| | |\n| --------------------- | -------------------------------------------- |\n| Change | = \\$5 $-$ (3 $\\times$ \\$1.10) |\n| | = \\$5 $-$ \\$3.30 |\n| | = {{{correctAnswer}}} |\n\n</div>"}]},{"vars":[{"varval":"\nSam buys 5 water bottles for \\$1.50 each.\n\nHe pays for these water bottles with a \\$10 note.\n\nHow much change should Sam receive?"},{"varval":"<div class=\"aligned\">\n\n| | |\n| --------------------- | -------------------------------------------- |\n| Change | = \\$10 $-$ (5 $\\times$ \\$1.50) |\n| | = \\$10 $-$ \\$7.50 |\n| | = {{{correctAnswer}}} |\n\n</div>"}]}]

  958. Number, NAP-48085

    <div class="sm_mode"> A solar panel grid on a school roof produces an average of 8.6 kWh of energy per day. How much energy will the grid produce for the school on average over 7 days? </div>

    [{"vars":null}]

  959. Number, NAP-89411

    <div class="sm_mode"> A television broadcasting tower is 800 metres high. A model of the tower is built with a scale of 1 : 4000. What is the height of the model? </div>

    [{"vars":null}]

  960. Number, NAP-66982

    <div class="sm_mode"> A soft drink factory can make 950 cans of lemonade and 600 cans of ginger ale **per hour**. The factory runs non-stop and each can weighs 300 grams. How many kilograms of soft drink does the factory produce in **1 full day**? </div>

    [{"vars":null}]

  961. Number, NAP-42341

    <div class="sm_mode"> Milly is using this cupcake recipe. <br> sm_img https://teacher.smartermaths.com.au/wp-content/uploads/2018/01/nap-A3-nc15.svg 200 indent3 vpad <br>How many cups of sugar are needed for 20 cupcakes? </div>

    [{"vars":null}]

  962. Number, NAP-55287

    <div class="sm_mode"> Johnno was standing 300 metres away from the stage at a rock concert. If the sound traveled at 330 metres per second from the stage, how many seconds did the sound take to get to Johnno? </div>

    [{"vars":null}]

  963. Number, NAP-78810

    <div class="sm_mode"> Curly measures the position of glaciers in the antarctic. His measurements showed that in 1 full year, a glacier moved 88 cm. On average, how many centimetres did the glacier move **per day**? </div>

    [{"vars":null}]

  964. Number, NAP-84726

    <div class="sm_mode"> Vladimir uses a 6 cm high print to make a sign. <br> sm_img https://teacher.smartermaths.com.au/wp-content/uploads/2017/12/NAP-A3-CA0132.svg 300 indent3 vpad <br>If the actual sign is 120 cm wide, what is its height? </div>

    [{"vars":null}]

  965. Number, NAP-36394

    <div class="sm_mode"> Muriel made a batch of cookies. <br> sm_img https://teacher.smartermaths.com.au/wp-content/uploads/2017/01/NAP-E3-NC05.png 200 indent3 vpad <br>Each cookie had 4 chocolate chips and 3 jelly snakes on it. Muriel used 39 jelly snakes in the batch of cookies. How many chocolate chips did she use? </div>

    [{"vars":[]}]

  966. Number, NAP-71775

    <div class="sm_mode"> The actual body length of a beetle Brad has caught is 24 mm. A scale drawing of the beetle is shown below. <br> sm_img https://teacher.smartermaths.com.au/wp-content/uploads/2017/12/nap-A4-nc08rev.svg 200 indent3 vpad <br>What scale is used in the drawing? </div>

    [{"vars":null}]

  967. Number, NAP-37682

    <div class="sm_mode"> A ute uses an average of 12 litres of diesel fuel for every 100 km travelled. At this rate, how many litres would the ute use to travel 450 km? </div>

    [{"vars":null}]

  968. Number, NAP-E3-CA16 Var 2: Changed question to $65 000 to match solution (was $120 000)

    <div class="sm_mode"> {{{question}}} </div>

    [{"vars":[{"varval":"Samuel earns \\$45 000 per year.\n\nHe is paid in equal monthly payments.\n\nHow much does he earn in 3 months?"},{"varval":"<div class=\"aligned\">\n\n| | |\n| --------------------- | -------------------------------------------- |\n| Earnings in 3 months | = $\\dfrac{3}{12}\\ \\times$ 45 000 |\n| | = $\\dfrac{1}{4}\\ \\times$ 45 000 |\n| | = {{{correctAnswer}}} |\n\n</div>"}]},{"vars":[{"varval":"Robyn earns \\$105 000 per year.\n\nShe is paid in equal monthly payments.\n\nHow much does she earn in 4 months?"},{"varval":"<div class=\"aligned\">\n\n| | |\n| --------------------- | -------------------------------------------- |\n| Earnings in 4 months | = $\\dfrac{4}{12}\\ \\times$ 105 000 |\n| | = $\\dfrac{1}{3}\\ \\times$ 105 000 |\n| | = {{{correctAnswer}}} |\n\n</div>"}]},{"vars":[{"varval":"Jarmello earns $65 000 per year.\n\nShe is paid in equal monthly payments.\n\nHow much does she earn in 3 months?"},{"varval":"<div class=\"aligned\">\n\n| | |\n| --------------------- | -------------------------------------------- |\n| Earnings in 3 months | = $\\dfrac{3}{12}\\ \\times$ 65 000 |\n| | = $\\dfrac{1}{4}\\ \\times$ 65 000 |\n| | = {{{correctAnswer}}} |\n\n</div>"}]},{"vars":[{"varval":"Chilla earns $33 000 per year.\n\nHe is paid in equal monthly payments.\n\nHow much does he earn in 3 months? "},{"varval":"<div class=\"aligned\">\n\n| | |\n| --------------------- | -------------------------------------------- |\n| Earnings in 3 months | = $\\dfrac{3}{12}\\ \\times$ 33 000 |\n| | = $\\dfrac{1}{4}\\ \\times$ 33 000 |\n| | = {{{correctAnswer}}} |\n\n</div>"}]},{"vars":[{"varval":"Snixon earns $51 000 per year.\n\nHe is paid in equal monthly payments.\n\nHow much does he earn in 3 months? "},{"varval":"<div class=\"aligned\">\n\n| | |\n| --------------------- | -------------------------------------------- |\n| Earnings in 3 months | = $\\dfrac{3}{12}\\ \\times$ 51 000 |\n| | = $\\dfrac{1}{4}\\ \\times$ 51 000 |\n| | = {{{correctAnswer}}} |\n\n</div>"}]},{"vars":[{"varval":"Alyce earns $87 000 per year.\n\nShe is paid in equal monthly payments.\n\nHow much does she earn in 3 months? "},{"varval":"<div class=\"aligned\">\n\n| | |\n| --------------------- | -------------------------------------------- |\n| Earnings in 3 months | = $\\dfrac{3}{12}\\ \\times$ 87 000 |\n| | = $\\dfrac{1}{4}\\ \\times$ 87 000 |\n| | = {{{correctAnswer}}} |\n\n</div>"}]}]

  969. Number, NAP-G3-CA14

    <div class="sm_mode"> Camilla has only 10-cent, 20-cent and 50-cent coins in her purse. She buys a coffee that costs her $3.80 and pays the exact price. What is the smallest number of coins she could use to pay for her coffee? </div>

    [{"vars":null}]

  970. Number, NAP-H3-NC18

    <div class="sm_mode"> This table shows the prices of different kinds of nuts sold at a farmers' market. <br> <div class="sm-table col1-color1"> >>| Nuts | pecans | cashews | almonds | walnuts | |:-:|:-:|:-:|:-:|:-:| | Price| $1.80| $6.00 | $7.60 | $3.90| | Quantity | 250 grams|750 grams | 1 kilogram | 500 grams | </div> <br>Which one of these are the most expensive nuts per kilogram? </div>

    [{"vars":null}]

  971. Number, NAP-F3-NC21

    <div class="sm_mode"> Brandon has a money jar that contains 50 cent, 20 cent, 10 cent and 5 cent pieces only. If he buys a pie worth \$2.65, what is the minimum amount of coins he can pay with? </div>

    [{"vars":null}]

  972. Number, NAP-B3-CA17

    <div class="sm_mode"> Sarah bought 4 pears and a \$4 bunch of grapes. The total cost was \$10.60. What would 2 pears and a \$2 bunch of grapes cost? </div>

    [{"vars":null}]

  973. Algebra, NAP-G3-CA15

    <div class="sm_mode"> {{{question}}} </div>

    [{"vars":[{"varval":"Kate takes part in a swim-a-thon to raise money for charity.\n\nHer grandmother sponsors her $20 in total.\n\nHer uncle sponsors her $1.25 for each lap of the pool she swims.\n\nIn order to raise $100, how many laps does Kate need to swim?"},{"varval":"<div class=\"no-margin-bottom\">\n\nLet &nbsp;$\\large x$ = number of laps that Kate needs to swim\n\n</div>\n\n<div class=\"aligned\">\n\n| | |\n| ----------------------: | --------------------------- |\n| 100 | \\= 1.25$\\large x$ + 20 |\n| 1.25$\\large x$ | \\= 100 $-$ 20 |\n| $\\therefore \\large x$ | \\= $\\dfrac{80}{1.25}$ |\n| | \\= {{{correctAnswer}}} laps |\n\n</div>"}]},{"vars":[{"varval":"Jennifer takes part in a \"Jump Rope for Heart\" marathon to raise money for charity.\n\nHer aunty sponsors her $30 in total.\n\nHer sister sponsors her $0.05 for each jump she does.\n\nIn order to raise $50, how many jumps does Jennifer need to complete?"},{"varval":"<div class=\"no-margin-bottom\">\n\nLet &nbsp;$\\large x$ = number of jumps that Jennifer needs to complete\n\n</div>\n\n<div class=\"aligned\">\n\n| | |\n| ----------------------: | --------------------------- |\n| 50 | \\= 0.05$\\large x$ + 30 |\n| 0.05$\\large x$ | \\= 50 $-$ 30 |\n| $\\therefore \\large x$ | \\= $\\dfrac{20}{0.05}$ |\n| | \\= {{{correctAnswer}}} jumps |\n\n</div>"}]},{"vars":[{"varval":"Finn takes part in a read-a-thon to raise money for charity.\n\nHis mother sponsors him $60 in total.\n\nHis sister sponsors him $4.00 for each book he reads.\n\nIn order to raise $100, how many books does Finn need to read?"},{"varval":"<div class=\"no-margin-bottom\">\n\nLet &nbsp;$\\large x$ = number of books that Finn needs to read\n\n</div>\n\n<div class=\"aligned\">\n\n| | |\n| ----------------------: | --------------------------- |\n| 100 | \\= 4$\\large x$ + 60 |\n| 4$\\large x$ | \\= 100 $-$ 60 |\n| $\\therefore \\large x$ | \\= $\\dfrac{40}{4}$ |\n| | \\= {{{correctAnswer}}} books |\n\n</div>"}]},{"vars":[{"varval":"Baby takes part in a dance-a-thon to raise money for charity.\n\nHer father sponsors her $50 in total.\n\nHer aunty sponsors her $5 for each hour she dances.\n\nIn order to raise $90, how many hours does Baby need to dance?"},{"varval":"<div class=\"no-margin-bottom\">\n\nLet &nbsp;$\\large x$ = number of hours that Baby needs to dance\n\n</div>\n\n<div class=\"aligned\">\n\n| | |\n| ----------------------: | --------------------------- |\n| 90 | \\= 5$\\large x$ + 50 |\n| 5$\\large x$ | \\= 90 $-$ 50 |\n| $\\therefore \\large x$ | \\= $\\dfrac{40}{5}$ |\n| | \\= {{{correctAnswer}}} hours |\n\n</div>"}]},{"vars":[{"varval":"Mitch takes part in a row-a-thon to raise money for charity.\n\nHis football team sponsors him $150 in total.\n\nHis wife sponsors him $3 for each kilometre he rows.\n\nIn order to raise $300, for how many kilometres does Mitch need to row?"},{"varval":"<div class=\"no-margin-bottom\">\n\nLet &nbsp;$\\large x$ = number of kilometres that Mitch needs to row\n\n</div>\n\n<div class=\"aligned\">\n\n| | |\n| ----------------------: | --------------------------- |\n| 300 | \\= 3$\\large x$ + 150 |\n| 3$\\large x$ | \\= 300 $-$ 150 |\n| $\\therefore \\large x$ | \\= $\\dfrac{150}{3}$ |\n| | \\= {{{correctAnswer}}} kilometres |\n\n</div>"}]},{"vars":[{"varval":"Dunlop takes part in a walk-a-thon to raise money for charity.\n\nHis tennis team sponsors him $40 in total.\n\nHis sister sponsors him $5 for each kilometre he walks.\n\nIn order to raise $150, for how many kilometres does Dunlop need to walk?"},{"varval":"<div class=\"no-margin-bottom\">\n\nLet &nbsp;$\\large x$ = number of kilometres that Dunlop needs to walk\n\n</div>\n\n<div class=\"aligned\">\n\n| | |\n| ----------------------: | --------------------------- |\n| 150 | \\=5$\\large x$ + 40 |\n| 5$\\large x$ | \\= 150 $-$ 40 |\n| $\\therefore \\large x$ | \\= $\\dfrac{110}{5}$ |\n| | \\= {{{correctAnswer}}} kilometres |\n\n</div>"}]}]

  974. Number, NAP-E3-CA18

    <div class="sm_mode"> {{{question}}} </div>

    [{"vars":[{"varval":"Sandy works part time in a shoe shop.\n\nOn weekends, she earns 1.5 times as much per hour as she earns on weekdays.\n\nOne week, she works 2 hours on a weekday and 2 hours on the weekend.\n\nHer pay for the week was \\$80.\n\nHow much does she earn in 1 hour on a weekday?"},{"varval":"sm_nogap Let &nbsp;$\\large x$ = pay per hour on a weekday\n\n<div class=\"aligned\">\n\n| | |\n| -----------------------------------------: | ---------------------- |\n| 2$\\large x$ + (2 $\\times\\ 1.5\\large x$) | \\= 80 |\n| 2$\\large x$ + 3$\\large x$ | \\= 80 |\n| 5$\\large x$ | \\= 80 |\n| $\\therefore\\large x$ | \\= $\\dfrac{80}{5}$ |\n| | \\= {{{correctAnswer}}} |\n\n</div>\n"}]},{"vars":[{"varval":"Blenham works part time in a bakery.\n\nOn weekends, he earns 2.5 times as much per hour as he earns on weekdays.\n\nOne week, he works 4 hours on a weekday and 2 hours on the weekend.\n\nHis pay for the week was \\$151.20\n\nHow much does he earn in 1 hour on a weekday?\n\n"},{"varval":"sm_nogap Let &nbsp;$\\large x$ = pay per hour on a weekday\n\n<div class=\"aligned\">\n\n| | |\n| -----------------------------------------: | ---------------------- |\n| 4$\\large x$ + (2 $\\times\\ 2.5\\large x$) | \\= 151.20 |\n| 4$\\large x$ + 5$\\large x$ | \\= 151.20 |\n| 9$\\large x$ | \\= 151.20 |\n| $\\therefore\\large x$ | \\= $\\dfrac{151.20}{9}$ |\n| | \\= {{{correctAnswer}}} |\n\n</div>"}]},{"vars":[{"varval":"Nicole works part time in a movie cinema.\n\nOn public holidays, she earns 3 times as much per hour as she earns on normal weekdays.\n\nOne week, she works 6 hours on a normal weekday and 4 hours on a public holiday.\n\nHer pay for the week was $450.\n\nHow much does she earn in 1 hour on a normal weekday?"},{"varval":"sm_nogap Let &nbsp;$\\large x$ = pay per hour on a weekday\n\n<div class=\"aligned\">\n\n| | |\n| -----------------------------------------: | ---------------------- |\n|6$\\large x$ + (4 $\\times\\ 3\\large x$) | \\= 450 |\n| 6$\\large x$ + 12$\\large x$ | \\= 450 |\n| 18$\\large x$ | \\= 450 |\n| $\\therefore\\large x$ | \\= $\\dfrac{450}{18}$ |\n| | \\= {{{correctAnswer}}} |\n\n</div>\n"}]},{"vars":[{"varval":"Ronald works part time in a fast food outlet.\n\nOn weekends, he earns 2.5 times as much per hour as he earns on weekdays.\n\nOne week, he works 12 hours on weekdays and 4 hours on the weekend.\n\nHis pay for the week was \\$258.50\n\nHow much does he earn in 1 hour on a weekday?\n"},{"varval":"sm_nogap Let &nbsp;$\\large x$ = pay per hour on a weekday\n\n<div class=\"aligned\">\n\n| | |\n| -----------------------------------------: | ---------------------- |\n| 12$\\large x$ + (4 $\\times\\ 2.5\\large x$) | \\= 258.50 |\n| 12$\\large x$ + 10$\\large x$ | \\= 258.50 |\n| 22$\\large x$ | \\= 258.50 |\n| $\\therefore\\large x$ | \\= $\\dfrac{258.50}{22}$ |\n| | \\= {{{correctAnswer}}} |\n\n</div>"}]},{"vars":[{"varval":"Tabitha works part time in a pet shop.\n\nOn weekends, she earns 1.5 times as much per hour as she earns on weekdays.\n\nOne week, she works 15 hours on weekdays and 6 hours on the weekend.\n\nHer pay for the week was \\$288.\n\nHow much does she earn in 1 hour on a weekday?"},{"varval":"sm_nogap Let &nbsp;$\\large x$ = pay per hour on a weekday\n\n<div class=\"aligned\">\n\n| | |\n| -----------------------------------------: | ---------------------- |\n|15$\\large x$ + (6 $\\times\\ 1.5\\large x$) | \\= 288 |\n| 15$\\large x$ + 9$\\large x$ | \\= 288 |\n| 24$\\large x$ | \\= 288 |\n| $\\therefore\\large x$ | \\= $\\dfrac{288}{24}$ |\n| | \\= {{{correctAnswer}}} |\n\n</div>\n"}]},{"vars":[{"varval":"Cole works part time in a supermarket.\n\nOn public holidays, he earns 2.5 times as much per hour as he earns on weekdays.\n\nOne week, he works 16 hours on weekdays and 8 hours on a public holiday.\n\nHis pay for the week was $756.00\n\nHow much does he earn in 1 hour on a weekday?\n"},{"varval":"sm_nogap Let &nbsp;$\\large x$ = pay per hour on a weekday\n\n<div class=\"aligned\">\n\n| | |\n| -----------------------------------------: | ---------------------- |\n| 16$\\large x$ + (8 $\\times\\ 2.5\\large x$) | \\= 756.00 |\n| 16$\\large x$ + 20$\\large x$ | \\= 756.00 |\n| 36$\\large x$ | \\= 756.00 |\n| $\\therefore\\large x$ | \\= $\\dfrac{756.00}{36}$ |\n| | \\= {{{correctAnswer}}} |\n\n</div>"}]}]

  975. Number, NAP-K3-CA14

    <div class="sm_mode"> Kim-chee makes 5 bowls of hot noodle soup at a school fete that cost her $24 to make in total. She sells three of the bowls for $6 each and the 4th bowl for $4.50. For what price must she sell the 5th bowl of noodle soup to make a profit? </div>

    [{"vars":null}]

  976. Algebra, NAP-I3-NC15

    <div class="sm_mode"> {{{question}}} </div>

    [{"vars":[{"varval":"Lee pays a monthly membership of \\$36 to have unlimited entry to the local swimming pool.\n\nIf she does squad training, she needs to pay an extra \\$8 per session to the swim coach.\n\nIf Lee does $\\large n$ squad sessions in a month, which expression represents her monthly bill?"},{"varval":"Monthly membership = \\$36\n\nCost of $\\large n$ squad sessions = \\$8$\\large n$\n\n$\\therefore$ Total monthly bill = {{{correctAnswer}}}\n"}]},{"vars":[{"varval":"Ian pays a monthly membership of \\$54 to have unlimited entry to the local swimming pool.\n\nIf he does squad training, he needs to pay an extra \\$12 per session to the swim coach.\n\nIf Ian does $\\large n$ squad sessions in a month, which expression represents his monthly bill?"},{"varval":"Monthly membership = \\$54\n\nCost of $\\large n$ squad sessions = \\$12$\\large n$\n\n$\\therefore$ Total monthly bill = {{{correctAnswer}}}\n"}]},{"vars":[{"varval":"Usain pays a monthly membership of \\$50 to have unlimited entry to the local running track.\n\nIf he does track training, he needs to pay an extra \\$10 per session to the track coach.\n\nIf Usain does $\\large n$ track sessions in a month, which expression represents his monthly bill?"},{"varval":"Monthly membership = \\$50\n\nCost of $\\large n$ track sessions = \\$10$\\large n$\n\n$\\therefore$ Total monthly bill = {{{correctAnswer}}}\n"}]},{"vars":[{"varval":"Olivia pays a monthly membership of \\$42 to have unlimited entry to the local CrossFit gym.\n\nIf she does Ninja Warrior training, she needs to pay an extra \\$19 per session to the head trainer.\n\nIf Olivia does $\\large n$ Ninja Warrior training sessions in a month, which expression represents her monthly bill?"},{"varval":"Monthly membership = \\$42\n\nCost of $\\large n$ Ninja Warrior sessions = \\$19$\\large n$\n\n$\\therefore$ Total monthly bill = {{{correctAnswer}}}\n"}]},{"vars":[{"varval":"Dominic pays a monthly membership of \\$27 to have unlimited entry to the local trampoline sports gym.\n\nIf he does personal lessons, he needs to pay an extra $15 per lesson to the gym.\n\nIf Dominic does $\\large n$ personal lessons in a month, which expression represents his monthly bill?"},{"varval":"Monthly membership = \\$27\n\nCost of $\\large n$ personal lessons = \\$15$\\large n$\n\n$\\therefore$ Total monthly bill = {{{correctAnswer}}}\n"}]},{"vars":[{"varval":"Serena pays a monthly membership of \\$35 to have unlimited entry to the local tennis centre.\n\nIf she does individual lessons, she needs to pay an extra \\$12 per lesson to the head coach.\n\nIf Serena does $\\large n$ individual lessons in a month, which expression represents her monthly bill?"},{"varval":"Monthly membership = \\$35\n\nCost of $\\large n$ individual lessons = \\$12$\\large n$\n\n$\\therefore$ Total monthly bill = {{{correctAnswer}}}\n"}]}]

  977. Number, NAP-I3-NC14

    <div class="sm_mode"> Giselle does odd jobs around the house to earn pocket money. She gets \$3 per job. The table below shows all the jobs she did in one week. <br> sm_img https://teacher.smartermaths.com.au/wp-content/uploads/2017/01/NAP-I3-14.png 280 indent vpad <br>If Giselle does the same jobs for two weeks, how much money will she earn altogether? </div>

    [{"vars":null}]

  978. Number, NAP-F3-NC15

    <div class="sm_mode"> Andy buys a chocolate bar for 95 cents from the shop. He has 4 coins in his hand and pays the exact amount. Which coin is not **not** in Andy's hand? </div>

    [{"vars":null}]

  979. Number, NAP-F3-CA16 var5 question image ... "Sauce 5%" on graph has a comma and a grey line that shouldn't be there ***var 0 of this question has accommodation spelled incorrectly also. I have redone the pie chart. Var1 ... think the question should give a second solution strategy where petrol = 1/4 x 1080 = $270 (recognise 1/4 of pie chart as 25%). can you adjust accomm and meals amounts on chart to adjust this **Done**

    <div class="sm_mode"> {{{question}}} </div>

    [{"vars":[{"varval":"Kat went on holiday and spent her money on accommodation, meals and shopping.\n\nShe spent \\$1200 in total and the pie chart below shows how she spent it.\n\n\n\nsm_img https://teacher.smartermaths.com.au/wp-content/uploads/2022/09/Stats_Prob_20273_v0a.svg 450 indent vpad\n\nHow much money did Kat spend on meals on her holiday?\n"},{"varval":"sm_nogap Money spent on shopping\n\n<div class=\"aligned\">\n\n>>| |\n| --------------------- |\n| = 30% $\\times$ 1200 |\n| = \\$360 |\n\n</div>\n\n<br>\n\n<div class=\"no-margin-bottom\">\n\n$\\therefore$ Money spent on meals\n\n</div>\n\n<div class=\"aligned\">\n\n>>| |\n| --------------------- |\n| = 1200 $-$ (540 + 360) |\n| = {{{correctAnswer}}} |\n\n</div>\n"}]},{"vars":[{"varval":"Evan went on fishing trip and spent his money on accommodation, meals, bait and petrol.\n\nHe spent \\$1080 in total and the pie chart below shows how he spent it.\n\n\n\nsm_img https://teacher.smartermaths.com.au/wp-content/uploads/2022/09/Stat_Prob_20273_v1.svg 550 vpad\n\nHow much money did Evan spend on petrol on his fishing trip?\n"},{"varval":"sm_nogap Money spent on bait\n\n<div class=\"aligned\">\n\n>>| |\n| --------------------- |\n| = 15% $\\times$ 1080 |\n| = \\$162 |\n\n</div>\n\n<br>\n\n<div class=\"no-margin-bottom\">\n\n$\\therefore$ Money spent on petrol\n\n</div>\n\n<div class=\"aligned\">\n\n>>| |\n| --------------------- |\n| = 1080 $-$ (378 + 216 +162) |\n| = {{{correctAnswer}}} |\n\n</div>\n"}]},{"vars":[{"varval":"Jade was planning her birthday party and was allocating money to catering, decorations and marquee hire.\n\nShe budgeted \\$5000 in total and the pie chart below shows how she allocated the money.\n\nsm_img https://teacher.smartermaths.com.au/wp-content/uploads/2022/08/Stats_Prob_20273_v2.svg 450 indent vpad\n\nHow much money did Jade allocate to catering for her party?\n"},{"varval":"sm_nogap Money allocated to decorations\n\n<div class=\"aligned\">\n\n>>| |\n| --------------------- |\n| = 11% $\\times$ 5000 |\n| = \\$550 |\n\n</div>\n\n<br>\n\n<div class=\"no-margin-bottom\">\n\n$\\therefore$ Money allocated to catering\n\n</div>\n\n<div class=\"aligned\">\n\n>>| |\n| --------------------- |\n| = 5000 $-$ (550 + 1200) |\n| = {{{correctAnswer}}} |\n\n</div>\n"}]},{"vars":[{"varval":"Oliver was building a deck beside his pool.\n\nHe allocated \\$7500 in total and the pie chart below shows how he allocated the money.\n\n<br>\n\nsm_img https://teacher.smartermaths.com.au/wp-content/uploads/2022/09/Stats_Prob_20273_v3_1.svg 550 indent vpad\n\n<br>How much money did Oliver allocate to the timber framework for his deck?\n"},{"varval":"sm_nogap Money allocated to Ekodecking\n\n<div class=\"aligned\">\n\n>>| |\n| --------------------- |\n| = 80% $\\times$ 7500 |\n| = \\$6000 |\n\n</div>\n\n<br>\n\n<div class=\"no-margin-bottom\">\n\n$\\therefore$ Money allocated to timber framework\n\n</div>\n\n<div class=\"aligned\">\n\n>>| |\n| --------------------- |\n| = 7500 $-$ (6000 + 90 + 210) |\n| = {{{correctAnswer}}} |\n\n</div>\n"}]},{"vars":[{"varval":"Phoebe is calculating her mobile phone and broadband costs from her latest bill.\n\nThe total of her bill was \\$252 and the pie chart below shows the breakdown of her bill.\n\n<br>\n\nsm_img https://teacher.smartermaths.com.au/wp-content/uploads/2022/08/Stats_Prob_20273_v4a.svg 450 indent vpad\n\n<br>How much does Phoebe pay for her Premium Plan?\n"},{"varval":"<div class=\"aligned\">\n\n| | |\n| --------------------- |-------------- |\n| iPhone cost | = 38% $\\times$ 252 |\n| |= \\$95.76 |\n\n</div>\n\n<br>\n\n<div class=\"no-margin-bottom\">\n\n$\\therefore$ Premium Plan\n\n</div>\n\n<div class=\"aligned\">\n\n>>| |\n| --------------------- |\n| = 252 $-$ (95.76 + 75.60) |\n| = {{{correctAnswer}}} |\n\n</div>\n"}]},{"vars":[{"varval":"Grant was organising a sausage sizzle for charity.\n\nHe spent \\$600 in total and the pie chart below shows how he spent it.\n\nsm_img https://teacher.smartermaths.com.au/wp-content/uploads/2022/09/Stats_Prob_20273_v5a.svg 480 indent vpad\n\nHow much money did Grant spend on onions for the sausage sizzle?\n"},{"varval":"sm_nogap Money spent on bread and sauce\n\n<div class=\"aligned\">\n\n>>| |\n| --------------------- |\n| = 30% $\\times$ 600 |\n| = \\$180 |\n\n</div>\n\n<br>\n\n<div class=\"no-margin-bottom\">\n\n$\\therefore$ Money spent on onions\n\n</div>\n\n<div class=\"aligned\">\n\n>>| |\n| --------------------- |\n| = 600 $-$ (180 + 410) |\n| = {{{correctAnswer}}} |\n\n</div>\n"}]}]

  980. Number, NAP-F3-NC14

    <div class="sm_mode"> Ten people share in a lottery prize of \$5650. They keep \$550 each and give the rest to charity. How much money in total do they have to give to charity? </div>

    [{"vars":null}]

  981. Number, NAP-D3-NC12

    <div class="sm_mode"> A service station sells bottles of water for \$2 each. It also has a special of 3 bottles for \$5. What is the lowest cost of 8 bottles of water? </div>

    [{"vars":null}]

  982. Number, NAP-D3-CA05

    <div class="sm_mode"> Bronwyn receives \$1.85 in change at the shop. She receives only coins. What is the **least** number of coins that she could receive? </div>

    [{"vars":null}]

  983. Number, NAP-59918

    <div class="sm_mode"> This picture shows a stone vase. <br> sm_img https://teacher.smartermaths.com.au/wp-content/uploads/2017/01/NAP-C3-NC081.png 200 indent3 vpad <br>The picture is 2 cm high. The actual vase is 40 cm high. What scale is used in the picture? </div>

    [{"vars":null}]

  984. Number, NAP-02305

    <div class="sm_mode"> Patrick rode his scooter at a speed of 5 metres per second. If he rode for 30 seconds, how far did he go? </div>

    [{"vars":null}]

  985. Number, NAP-H3-NC05

    <div class="sm_mode"> Nick had a \$10 note only. At the service station, he bought 5 iceblocks that cost \$1.20 each. How much change should he get? </div>

    [{"vars":null}]

  986. Number, NAP-C3-CA06 SA

    <div class="sm_mode"> sm_img https://teacher.smartermaths.com.au/wp-content/uploads/2017/01/naplan-Y7-2010-6mc.png 420 indent vpad Geoffrey buys 4 peaches and 1 watermelon from a fresh fruit market. How much does Geoffrey pay altogether? </div>

    [{"vars":null}]

  987. Number, NAP-49370

    <div class="sm_mode"> Sisko is making red cordial for his daughter's birthday party. Red cordial is made by adding red concentrate with water. Sisko adds 60 millilitres (mL) of red concentrate to 1 litre (L) of water. How much red cordial has Sisko made? </div>

    [{"vars":null}]

  988. Number, NAP-31738

    <div class="sm_mode"> {{{question}}} </div>

    [{"vars":[{"varval":"Barry travelled 120 kilometres in 2 hours.\n\nWhat was his average speed in kilometres per hour?"},{"varval":"<div class=\"aligned\">\n\n| | |\n| --------------------: | -------------- |\n| Average speed | = $\\dfrac{\\text{distance}}{\\text{time}}$ |\n|| = $\\dfrac{120}{2}$ |\n| | = {{{correctAnswer}}} km/h |\n\n</div>"}]},{"vars":[{"varval":"Andrea rode his bike 72 kilometres in 3 hours.\n\nWhat was his average speed in kilometres per hour?"},{"varval":"<div class=\"aligned\">\n\n| | |\n| --------------------: | -------------- |\n| Average speed | = $\\dfrac{\\text{distance}}{\\text{time}}$ |\n|| = $\\dfrac{72}{3}$ |\n| | = {{{correctAnswer}}} km/h |\n\n</div>"}]},{"vars":[{"varval":"Celeste completed a 360 kilometre off-road rally in 5 hours.\n\nWhat was her average speed in kilometres per hour?"},{"varval":"<div class=\"aligned\">\n\n| | |\n| --------------------: | -------------- |\n| Average speed | = $\\dfrac{\\text{distance}}{\\text{time}}$ |\n|| = $\\dfrac{360}{5}$ |\n| | = {{{correctAnswer}}} km/h |\n\n</div>"}]},{"vars":[{"varval":"Jerry ran 1500 metres in 6 minutes.\n\nWhat was his average speed in metres per minute?"},{"varval":"<div class=\"aligned\">\n\n| | |\n| --------------------: | -------------- |\n| Average speed | = $\\dfrac{\\text{distance}}{\\text{time}}$ |\n|| = $\\dfrac{1500}{6}$ |\n| | = {{{correctAnswer}}} metres/minute |\n\n</div>"}]},{"vars":[{"varval":"Gayle decorated 162 cookies in 9 hours.\n\nWhat was her average decorating speed in cookies per hour?"},{"varval":"<div class=\"aligned\">\n\n| | |\n| --------------------: | -------------- |\n| Average speed | = $\\dfrac{\\text{c}}{\\text{time}}$ |cookies baked\n|| = $\\dfrac{162}{9}$ |\n| | = {{{correctAnswer}}} cookies/hour |\n\n</div>"}]},{"vars":[{"varval":"Hans completed a 378 kilometre cycling race in 9 hours.\n\nWhat was his average speed in kilometres per hour?"},{"varval":"<div class=\"aligned\">\n\n| | |\n| --------------------: | -------------- |\n| Average speed | = $\\dfrac{\\text{distance}}{\\text{time}}$ |\n|| = $\\dfrac{378}{9}$ |\n| | = {{{correctAnswer}}} km/h |\n\n</div>"}]}]

  989. Number, NAP-J3-NC02

    <div class="sm_mode"> Patrick gets \$7.35 in pocket money each week. He does extra jobs one week and earns \$4.75 more. How much money did Patrick receive in total in the week? </div>

    [{"vars":null}]

  990. Number, NAP-H3-CA03

    <div class="sm_mode"> Tim is selling milkshakes at a school fete. He makes \$48 from selling 8 milkshakes. All milkshakes cost the same. How much will Tim make if he sells 11 milkshakes. </div>

    [{"vars":null}]

  991. Number, NAP-K3-NC01

    <div class="sm_mode"> Jazz buys 4 avocados for \$1.20 each. He pays for the avocados with a \$5 note. How much change should Jazz receive? </div>

    [{"vars":null}]

  992. Number, NAP-K3-CA05

    <div class="sm_mode"> The entry fees to visit the Toorak Aquarium are \$15 for an adult and \$7 for a child. A group of 8 people paid \$88 in total to visit the aquarium. How many adults are in the group? </div>

    [{"vars":null}]

  993. Number, NAPX-E4-CA06

    <div class="sm_mode"> Barry lives 30 kilometres from the library. On Tuesday, he drove to the library and averaged 90 kilometres per hour. On Thursday, he took the train which averaged 30 kilometres per hour. What was the extra time of the train journey, in minutes, compared to when he drove on Tuesday? </div>

    [{"vars":null}]

  994. Number, NAPX-I4-CA13, NAPX-I3-CA21

    <div class="sm_mode"> Om is using a vegetarian recipe that uses 3 cups of brown rice for every 2 cups of dates. Select the correct combination of brown rice and dates. </div>

    [{"vars":null}]

  995. Number, NAPX-E4-NC14, NAPX-E3-NC18

    <div class="sm_mode"> Keenak is making a fruit cake. The recipe says he needs 1 cup of sultanas for every 4 cups of flour. If 7 cups of flour are used, how many cups of sultanas are needed? </div>

    [{"vars":null}]

  996. Number, NAPX-F4-CA14

    <div class="sm_mode"> Kelly drives her moped to a beach 100 km away at an average speed of 60 km. How long does the trip take? </div>

    [{"vars":null}]

  997. Number, NAPX-H3-CA24

    <div class="sm_mode"> Tim lives 6 km from the park where he trains for soccer. He jogs there at a constant speed of 10 kilometres per hour. How many minutes does it take for Tim to get to the park? </div>

    [{"vars":null}]

  998. Algebra, NAPX-p111726v01

    <div class="sm_mode"> Chakra has a toy horse that is 4 cm high. His father Dinesh has a real horse that is 2 metres high. How many times as high as Chakra's toy horse is Dinesh's real horse? </div>

    [{"vars":null}]

  999. Number, NAPX-H3-NC19, NAPX-H2-31

    <div class="sm_mode"> Basil has a van that is 3 meters long. His son Cady has a toy van that is 5 cm long. How many times as long as Cady's toy van is Basil's truck? </div>

    [{"vars":null}]

  1000. Number, NAPX-L4-CA18, NAPX-L3-CA29

    <div class="sm_mode"> A school's chess club has 6 girls and 5 boys. 2 girls and some more boys then joined the club. The number of boys in the club was now double the number of girls. How many boys joined the club? </div>

    [{"vars":null}]

  1001. Number, NAP-F3-NC04

    <div class="sm_mode"> Missy has \$5.40 in loose change and bought a cup of coffee that cost her \$4.85. How much money does she have left? </div>

    [{"vars":null}]

  1002. Number, NAP-D3-CA03

    <div class="sm_mode"> Srinath spends \$12 a month on coffee. After how many months will his total spending on coffee amount to \$180? </div>

    [{"vars":null}]

  1003. Number, NAPX-I4-CA24

    <div class="sm_mode"> A supermarket sells four sizes of chocolate bar. sm_img https://teacher.smartermaths.com.au/wp-content/uploads/2018/06/NAPX-I4-CA24_2.svg 550 indent vpad Which chocolate bar costs the least per gram? </div>

    [{"vars":null}]

  1004. Number NAPX-F4-NC15, NAPX-F3-NC21

    <div class="sm_mode"> Barry has a money jar that contains 1 dollar, 50 cent, 20 cent, 10 cent and 5 cent coins only. If he buys a pavlova worth \$3.65, what is the minimum amount of coins he can pay with? </div>

    [{"vars":null}]

  1005. Number, NAPX-I4-CA15, NAPX-I3-CA23

    <div class="sm_mode"> {{{question}}} </div>

    [{"vars":[{"varval":"Jarren is buying a spare tyre for his car.\n\nThe table below lists the original price and the amount of discount on a single tyre at four different tyre retailers.\n\n<br>\n\n<div class=\"sm-table row1-color1 heading-color2\">\n\n>>Tyre Prices\n\n>>| Shop | Original price | Discount |\n|:-:|:-:|:-:|\n| A | $40 | 40%|\n| B | $36 | 25%|\n| C | $29| $\\dfrac{1}{5}$\n| D | $28| $3 off|\n\n</div>\n\n<br>Which shop has the lowest sale price for the tyre?\n"},{"varval":"Consider the sale price at each shop:\n\n\nA = 40 − (40% $\\times$ 40) = 40 $-$ 16 = $24\n\n\nB = 36 − (25% $\\times$ 36) = 36 $-$ 9 = $27\n\n\nC = 29 − (20% $\\times$ 29) = 29 $-$ 5.80 = $23.20\n\n\nD = 28 − 3 = $25\n\n \n\n$\\therefore$ Shop {{{correctAnswer}}} has the lowest price.\n"}]},{"vars":[{"varval":"Bec is buying 40 kilograms of dry dog food for her bullmastiff.\n\nThe table below lists the original price and the amount of discount on a 40 kilogram bag of dry dog food at four different pet stores.\n\n<br>\n\n\n<div class=\"sm-table row1-color1 heading-color2\">\n\n>>40 kg Dry Dog Food Prices\n\n>>| Shop | Original price | Discount |\n|:-:|:-:|:-:|\n| A | $220 | 20%|\n| B | $245 | 25%|\n| C | $250| $\\dfrac{1}{5}$\n| D | $230| $35 off|\n\n</div>\n\n<br>Which pet store has the lowest sale price for the 40kg bag of dog food?\n"},{"varval":"Consider the sale price at each shop:\n\n\nA = 220 − (20% $\\times$ 220) = 220 $-$ 44 = $176\n\n\nB = 245 − (25% $\\times$ 245) = 245 $-$ 61.25 = $183.75\n\n\nC = 250 − (20% $\\times$ 250) = 250 $-$ 50= $200\n\n\nD = 230 − 35 = $195\n\n \n\n$\\therefore$ Shop {{{correctAnswer}}} has the lowest price.\n"}]},{"vars":[{"varval":"Arnold is buying protein powder.\n\nThe table below lists the original price and the amount of discount on a container of protein powder at four different supplement stores.\n\n<br>\n\n<div class=\"sm-table row1-color1 heading-color2\">\n\n>>Protein Powder Prices\n\n>>| Shop | Original price | Discount |\n|:-:|:-:|:-:|\n| A | $75 | 10%|\n| B | $85 | 20%|\n| C | $92| $\\dfrac{1}{4}$\n| D | $80| $11.50 off|\n\n</div>\n\n<br>Which shop has the lowest sale price for the protein powder?\n"},{"varval":"Consider the sale price at each shop:\n\n\nA = 75 − (10% $\\times$ 75) = 75 $-$ 7.50 = $67.50\n\n\nB = 85 − (20% $\\times$ 85) = 85 $-$ 17 = $68\n\n\nC = 92 − (25% $\\times$ 92) = 92 $-$ 23 = $69\n\n\nD = 80 − 11.50 = $68.50\n\n \n\n$\\therefore$ Shop {{{correctAnswer}}} has the lowest price.\n"}]},{"vars":[{"varval":"Jake is buying a 2 person pop up tent.\n\nThe table below lists the original price and the amount of discount on a 2 person tent at four different camping supply retailers.\n\n<br>\n\n<div class=\"sm-table row1-color1 heading-color2\">\n\n>>Tent Prices\n\n>>| Shop | Original price | Discount |\n|:-:|:-:|:-:|\n| A | $199 | 12%|\n| B | $250 | 30%|\n| C | $220| $\\dfrac{1}{5}$\n| D | $210| $33 off|\n\n</div>\n\n<br>Which shop has the lowest sale price for the tent?\n"},{"varval":"Consider the sale price at each shop:\n\n\nA = 199 − (12% $\\times$ 199) = 199 $-$ 23.88 = $175.12\n\n\nB = 250 − (30% $\\times$ 250) = 250 $-$ 75 = $175\n\n\nC = 220 − (20% $\\times$ 220) = 220 $-$ 44 = $176\n\n\nD = 210 − 33 = $177\n\n \n\n$\\therefore$ Shop {{{correctAnswer}}} has the lowest price.\n"}]},{"vars":[{"varval":"Fleetwood is buying an acoustic drum kit.\n\nThe table below lists the original price and the amount of discount on a drum kit at four different music retailer stores.\n\n<br>\n\n<div class=\"sm-table row1-color1 heading-color2\">\n\n>>Drum Kit Prices\n\n>>| Shop | Original price | Discount |\n|:-:|:-:|:-:|\n| A | $1195 | 15%|\n| B | $1250 | 20%|\n| C | $1350| $\\dfrac{1}{4}$\n| D | $1299| $285 off|\n\n</div>\n\n<br>Which shop has the lowest sale price for the drum kit?\n"},{"varval":"Consider the sale price at each shop:\n\n\nA = 1195 − (15% $\\times$ 1195) = 1195 $-$ 179.25 = $1015.75\n\n\nB = 1250 − (20% $\\times$ 1250) = 1250 $-$ 250 = $1000\n\n\nC = 1350 − (25% $\\times$ 1350) = 1350 $-$ 337.50 = $1012.50\n\n\nD = 1299 − 285 = $1014\n\n \n\n$\\therefore$ Shop {{{correctAnswer}}} has the lowest price.\n"}]},{"vars":[{"varval":"Harvey is buying a trampoline for his grandson's birthday.\n\nThe table below lists the original price and the amount of discount on the trampoline at four different online stores.\n\n<br>\n\n<div class=\"sm-table row1-color1 heading-color2\">\n\n>>Trampoline Prices\n\n>>| Shop | Original price | Discount |\n|:-:|:-:|:-:|\n| A | $1320 | 10%|\n| B | $1480 | 25%|\n| C | $1250| $\\dfrac{1}{10}$\n| D | $1699| $600 off|\n\n</div>\n\n<br>Which online store has the lowest sale price for the trampoline?\n"},{"varval":"Consider the sale price at each shop:\n\n\nA = 1320 − (10% $\\times$ 1320) = 1320 $-$ 132 = $1188\n\n\nB = 1480 − (25% $\\times$ 1480) = 1480 $-$ 370 = $1110\n\n\nC = 1250 − (10% $\\times$ 1250) = 1250 $-$ 125 = $1125\n\n\nD = 1699 − 600 = $1099\n\n \n\n$\\therefore$ Shop {{{correctAnswer}}} has the lowest price.\n"}]}]

  1006. Number, NAPX-E3-CA16

    <div class="sm_mode"> Rafty earns \$75 000 per year. He is paid in equal monthly payments. How much does he earn in 4 months? </div>

    [{"vars":null}]

  1007. Number, NAPX-G4-CA13

    <div class="sm_mode"> Kylie has 5-cent, 20-cent and 50-cent coins in her pocket. In total she has \$2.35. What is the smallest number of coins she could have? </div>

    [{"vars":null}]

  1008. Number, NAPX-G4-NC06, NAPX-G3-NC13

    <div class="sm_mode"> 20 school children went to Leyland Brothers World for an excursion. Entry into Leyland Brothers World was \$15 per student and the total cost of the bus was \$140. Which expression shows the total cost of the excursion for everybody? </div>

    [{"vars":null}]

  1009. Number, NAPX-L4-CA18 v1, NAPX-L3-CA29 v1

    <div class="sm_mode"> A school's drama club has 7 girls and 9 boys. 4 girls and some boys then joined the club. The number of boys in the club is now double the number of girls. How many boys joined the club? </div>

    [{"vars":null}]

  1010. Number, NAPX-L4-CA18 v2, NAPX-L3-CA29 v2

    <div class="sm_mode"> A school's garden club has 4 girls and 6 boys. Two boys and some girls then joined the club. The number of girls in the club was now double the number of boys. How many girls joined the club? </div>

    [{"vars":null}]

  1011. Number, NAPX-G4-CA09

    <div class="sm_mode"> The colour pink can be made by mixing 5 litres of white paint with 3 litres of red paint. If 80 litres of white paint are used in this mixture, how much red paint is needed? </div>

    [{"vars":null}]

  1012. Number, NAPX-J4-NC04

    <div class="sm_mode"> Crystal is making red cordial for her daughter's birthday party. She needs to add 280 millilitres of concentrate to every 2 liters of water. If Crystal only has 0.5 litres of water left, how many millilitres of concentrate should she add? </div>

    [{"vars":null}]

  1013. Number, NAPX-I4-CA05 SA

    <div class="sm_mode"> Chakra earns $23.88 per hour at a homewares shop. In one week, she worked $15 \dfrac{1}{4}$ hours. She used the money to buy scented candles for her friends. Each candle costs $31.25. What is the maximum number of candles Chakra can buy? </div>

    [{"vars":null}]

  1014. Number, NAPX-J4-CA06

    <div class="sm_mode"> Ticka is a baker and sells 50 loaves of bread at the markets. Each loaf costs him \$3.75 to make. He sells 40 loaves for \$7.95 each. In the last hour of the market, he sells the last 10 loaves for \$2.50 each. What is the total profit or loss of the 50 loaves? </div>

    [{"vars":null}]

  1015. <div class="sm_mode"> Chloe has \$20 to buy fruit from the local farmer's market. She buys one avocado and a pineapple. sm_img https://teacher.smartermaths.com.au/wp-content/uploads/2018/07/NAPX-G4-NC08_6.svg 300 indent3 vpad <br>How much change should Chloe be given? </div>

    [{"vars":null}]

  1016. Number, NAPX-I3-CA19

    <div class="sm_mode"> Santiago needs to buy some olive oil. The farmers' market sells Santiago's favourite olive oil in a number of different sizes. Which size costs him the least per mL? </div>

    [{"vars":null}]

  1017. Number, NAPX-I3-CA17

    <div class="sm_mode"> {{{question}}} </div>

    [{"vars":[{"varval":"A soft drink factory can make 750 cans of lemonade and 1100 cans of ginger ale per hour.\n\nEach can weighs 300 grams.\n\nHow many kilograms of soft drink does the factory produce in 1 hour?"},{"varval":"\nsm_nogap Total cans made per hour\n\n\n<div class=\"aligned\">\n\n>>| |\n| -------------- |\n| = 750 + 1100 |\n| = 1850 |\n\n</div>\n\n\n<br>\n\n\n<div class=\"no-margin-bottom\">\n\n$\\therefore$ Kilograms made per hour\n\n</div>\n\n\n<div class=\"aligned\">\n\n>>| |\n| -------------- |\n| = 1850 $\\times$ 300 |\n| = 555 000 grams |\n| = 555 kilograms |\n\n</div>"}]},{"vars":[{"varval":"A pet food factory can make 950 cans of dog food and 1450 cans of cat food per hour.\n\nEach can weighs 550 grams.\n\nHow many kilograms of pet food does the factory produce in 1 hour?"},{"varval":"\nsm_nogap Total cans made per hour\n\n\n<div class=\"aligned\">\n\n>>| |\n| -------------- |\n| = 950 + 1450 |\n| = 2400 |\n\n</div>\n\n\n<br>\n\n\n<div class=\"no-margin-bottom\">\n\n$\\therefore$ Kilograms made per hour\n\n</div>\n\n\n<div class=\"aligned\">\n\n>>| |\n| -------------- |\n| = 2400 $\\times$ 550 |\n| = 1 320 000 grams |\n| = 1320 kilograms |\n\n</div>"}]}]

  1018. Number, NAPX-G3-CA14

    <div class="sm_mode"> Butch has only 5-cent, 10-cent, 20-cent and 50-cent coins in his pocket. He buys a doughnut that costs him \$2.65 and pays the exact price. What is the smallest number of coins he could use to pay for his doughnut? </div>

    [{"vars":null}]

  1019. Number, NAPX-E3-CA13

    <div class="sm_mode"> Alan planted a tree in his yard. His measurements showed that in 1 full year, the height of the tree increased 65 centimetres. On average, how many centimetres did the tree grow per day? </div>

    [{"vars":null}]

  1020. Number, NAPX-H3-NC18 (1.25 min)

    <div class="sm_mode"> This table shows the prices of different kinds of nuts sold at a farmers' market. sm_img https://teacher.smartermaths.com.au/wp-content/uploads/2018/07/NAPX-H3-NC18.svg 530 indent vpad Which one of these are the most expensive nuts per kilogram? </div>

    [{"vars":null}]

  1021. Number, NAPX-I4-CA03

    <div class="sm_mode"> {{{question}}} </div>

    [{"vars":[{"varval":"On a dairy farm, a farmer allocates 3 hectares of land to one cow.\n\nHow many cows would the farmer allocate to a 30 hectare paddock?"},{"varval":"<div class=\"aligned\">\n\n| | |\n| --------------------: | -------------- |\n| Number of Cows | = $\\dfrac{30}{3}$ |\n| | = {{{correctAnswer}}} |\n\n\n</div>"}]}]

  1022. Number, NAPX-E3-CA18

    <div class="sm_mode"> {{{question}}} </div>

    [{"vars":[{"varval":"Brin works part time in a coffee shop.\n\nOn weekends, he earns 1.5 times as much per hour as he earns on weekdays.\n\nOne week, he works 9 hours on a weekday and 4 hours on the weekend.\n\nHis pay for the week was \\$270.\n\nHow much does he earn in 1 hour on a weekday?\n"},{"varval":"<div class=\"no-margin-bottom\">\n\nLet $\\ \\large x$ = pay per hour on a weekday\n\n</div>\n\n<div class=\"aligned\">\n\n| | |\n| --------------------: | ---------------------- |\n| 9$\\large x$ + (4 $\\times$ 1.5$\\large x$)\t | \\= \\$270 |\n| 15$\\large x$ | \\= 270 |\n| $\\large x$ | \\= 270 $\\div$ 15 |\n| | \\= {{{correctAnswer}}} |\n\n</div>"}]},{"vars":[{"varval":"Betty works part time in a clothing shop.\n\nOn weekends, she earns 2 times as much per hour as she earns on weekdays.\n\nOne week, she works 15 hours on weekdays and 3 hours on the weekend.\n\nHer pay for the week was \\$367.50.\n\nHow much does she earn in 1 hour on a weekday?\n"},{"varval":"<div class=\"no-margin-bottom\">\n\nLet $\\ \\large x$ = pay per hour on a weekday\n\n</div>\n\n<div class=\"aligned\">\n\n| | |\n| --------------------: | ---------------------- |\n| 15$\\large x$ + (3 $\\times$ 2$\\large x$)\t | \\= \\$367.50 |\n| 21$\\large x$ | \\= 367.50 |\n| $\\large x$ | \\= 367.50 $\\div$ 21 |\n| | \\= {{{correctAnswer}}} |\n\n</div>"}]},{"vars":[{"varval":"Clay has a casual job at the local movie cinema.\n\nOn weekends, he earns 1.5 times as much per hour as he earns on weekdays.\n\nOne week, he works 20 hours on weekdays and 6 hours on the weekend.\n\nHis pay for the week was \\$498.80.\n\nHow much does he earn in 1 hour on a weekday?\n"},{"varval":"<div class=\"no-margin-bottom\">\n\nLet $\\ \\large x$ = pay per hour on a weekday\n\n</div>\n\n<div class=\"aligned\">\n\n| | |\n| --------------------: | ---------------------- |\n| 20$\\large x$ + (6 $\\times$ 1.5$\\large x$)\t | \\= \\$498.80 |\n| 29$\\large x$ | \\= 498.80 |\n| $\\large x$ | \\= 498.80 $\\div$ 29 |\n| | \\= {{{correctAnswer}}} |\n\n</div>"}]},{"vars":[{"varval":"Fleur works part time in a flower shop.\n\nOn weekends, she earns 2 times as much per hour as she earns on weekdays.\n\nOne week, she works 23 hours on weekdays and 1.5 hours on the weekend.\n\nHer pay for the week was \\$351.\n\nHow much does she earn in 1 hour on a weekday?"},{"varval":"<div class=\"no-margin-bottom\">\n\nLet $\\ \\large x$ = pay per hour on a weekday\n\n</div>\n\n<div class=\"aligned\">\n\n| | |\n| --------------------: | ---------------------- |\n| 23$\\large x$ + (1.5 $\\times$ 2$\\large x$)\t | \\= \\$351 |\n| 26$\\large x$ | \\= 351 |\n| $\\large x$ | \\= 351 $\\div$ 26 |\n| | \\= {{{correctAnswer}}} |\n\n</div>"}]},{"vars":[{"varval":"Bodhi works part time in a surf shop.\n\nOn weekends, he earns 1.5 times as much per hour as he earns on weekdays.\n\nOne week, he works 7.5 hours on weekdays and 3 hours on the weekend.\n\nHis pay for the week was \\$150.\n\nHow much does he earn in 1 hour on a weekday?"},{"varval":"<div class=\"no-margin-bottom\">\n\nLet $\\ \\large x$ = pay per hour on a weekday\n\n</div>\n\n<div class=\"aligned\">\n\n| | |\n| --------------------: | ---------------------- |\n| 7.5$\\large x$ + (1.5 $\\times$ 3$\\large x$)\t | \\= \\$150 |\n| 12$\\large x$ | \\= 150 |\n| $\\large x$ | \\= 150 $\\div$ 12 |\n| | \\= {{{correctAnswer}}} |\n\n</div>"}]},{"vars":[{"varval":"Aurora works part time in a donut shop.\n\nOn weekends, she earns 2.5 times as much per hour as she earns on weekdays.\n\nOne week, she works 14.5 hours on weekdays and 3 hours on the weekend.\n\nHer pay for the week was \\$319.\n\nHow much does she earn in 1 hour on a weekday?"},{"varval":"<div class=\"no-margin-bottom\">\n\nLet $\\ \\large x$ = pay per hour on a weekday\n\n</div>\n\n<div class=\"aligned\">\n\n| | |\n| --------------------: | ---------------------- |\n| 14.5$\\large x$ + (2.5 $\\times$ 3$\\large x$)\t | \\= \\$319 |\n| 22$\\large x$ | \\= 319 |\n| $\\large x$ | \\= 319 $\\div$ 22 |\n| | \\= {{{correctAnswer}}} |\n\n</div>"}]}]

  1023. Number, NAPX-I3-NC14, NAPX-I2-30

    <div class="sm_mode"> Mia does odd jobs around the house to earn pocket money. She gets \$4 per job. The table below shows all the jobs she did in one week. sm_img https://teacher.smartermaths.com.au/wp-content/uploads/2018/06/NAPX-I3-NC14.svg 190 indent vpad If Mia does the same jobs for two weeks, how much money will she earn altogether? </div>

    [{"vars":null}]

  1024. Number, NAPX-F3-CA11

    <div class="sm_mode"> Gia was standing 250 metres away from a gun that is fired. If the sound of the shot travelled at 330 metres per second from the gun, how many seconds did the sound take to get to Gia? </div>

    [{"vars":null}]

  1025. Number, NAPX-G3-NC11

    <div class="sm_mode"> {{{question}}} </div>

    [{"vars":[{"varval":"The Park Run is a 5 kilometre fun run.\n\nOlive completes the Park Run in 35 minutes.\n\nPopeye runs at the same speed as Olive.\n\nHow long will it take him to run 3 kilometres?\n"},{"varval":"sm_nogap Olive's speed\n\n\n<div class=\"aligned\">\n\n> > | |\n| -------------- |\n| = 35 $\\div$ 5 |\n| = 7 mins/km |\n\n</div>\n\n\n<br>\n\n\n<div class=\"no-margin-bottom\">\n\n$\\therefore$ Time for Popeye to run 3 kilometres\n\n</div>\n\n\n<div class=\"aligned\">\n\n> > | |\n| -------------- |\n| = 3 $\\times$ 7 |\n| = {{{correctAnswer}}} minutes |\n\n</div>"}]},{"vars":[{"varval":"The Harbour Classic is a 12 kilometre fun run.\n\nClint completes the fun run in 60 minutes.\n\nEzzy runs at the same speed as Clint.\n\nHow long will it take her to run 5 kilometres?\n"},{"varval":"sm_nogap Clint's speed\n\n\n<div class=\"aligned\">\n\n> > | |\n| -------------- |\n| = 60 $\\div$ 12 |\n| = 5 mins/km |\n\n</div>\n\n\n<br>\n\n\n<div class=\"no-margin-bottom\">\n\n$\\therefore$ Time for Ezzy to run 5 kilometres\n\n</div>\n\n\n<div class=\"aligned\">\n\n> > | |\n| -------------- |\n| = 5 $\\times$ 5 |\n| = {{{correctAnswer}}} minutes |\n\n</div>"}]},{"vars":[{"varval":"Jordy enters a 20 kilometre mountain bike race.\n\nJordy completes the race in 80 minutes.\n\nKane rides at the same speed as Jordy.\n\nHow long will it take him to ride 8 kilometres?\n"},{"varval":"sm_nogap Jordy's speed\n\n\n<div class=\"aligned\">\n\n> > | |\n| -------------- |\n| = 80 $\\div$ 20 |\n| = 4 mins/km |\n\n</div>\n\n\n<br>\n\n\n<div class=\"no-margin-bottom\">\n\n$\\therefore$ Time for Kane to ride 8 kilometres\n\n</div>\n\n\n<div class=\"aligned\">\n\n> > | |\n| -------------- |\n| = 8 $\\times$ 4 |\n| = {{{correctAnswer}}} minutes |\n\n</div>"}]},{"vars":[{"varval":"Ben and Beccy enter a pie eating contest.\n\nBen can eat 9 pies in 18 minutes.\n\nBeccy eats pies at the same speed as Ben.\n\nHow long will it take her to eat 15 pies?"},{"varval":"sm_nogap Ben's speed\n\n\n<div class=\"aligned\">\n\n> > | |\n| -------------- |\n| = 18 $\\div$ 9 |\n| = 2 mins/pie |\n\n</div>\n\n\n<br>\n\n\n<div class=\"no-margin-bottom\">\n\n$\\therefore$ Time for Beccy to eat 15 pies\n\n</div>\n\n\n<div class=\"aligned\">\n\n> > | |\n| -------------- |\n| = 15 $\\times$ 2 |\n| = {{{correctAnswer}}} minutes |\n\n</div>"}]},{"vars":[{"varval":"Suzi enters a sail boarding race.\n\nSuzi finishes a 6 kilometre race 42 minutes.\n\nStacey sails at the same speed as Suzi.\n\nHow long will it take Stacey to complete a 5 kilometre race?\n"},{"varval":"sm_nogap Suzi's speed\n\n\n<div class=\"aligned\">\n\n> > | |\n| -------------- |\n| = 42 $\\div$ 6 |\n| = 7 mins/kilometre |\n\n</div>\n\n\n<br>\n\n\n<div class=\"no-margin-bottom\">\n\n$\\therefore$ Time for Stacey to sail 5 kilometres\n\n</div>\n\n\n<div class=\"aligned\">\n\n> > | |\n| -------------- |\n| = 5 $\\times$ 7 |\n| = {{{correctAnswer}}} minutes |\n\n</div>"}]},{"vars":[{"varval":"The River Run is a 10 kilometre fun run.\n\nJoey completes the fun run in 80 minutes.\n\nMaude runs at the same speed as Joey.\n\nHow long will it take her to run 7 kilometres?\n"},{"varval":"sm_nogap Joey's speed\n\n\n<div class=\"aligned\">\n\n> > | |\n| -------------- |\n| = 80 $\\div$ 10 |\n| = 8 mins/kilometre |\n\n</div>\n\n\n<br>\n\n\n<div class=\"no-margin-bottom\">\n\n$\\therefore$ Time for Maude to run 7 kilometres\n\n</div>\n\n\n<div class=\"aligned\">\n\n> > | |\n| -------------- |\n| = 7 $\\times$ 8 |\n| = {{{correctAnswer}}} minutes |\n\n</div>"}]}]

  1026. Number, NAPX-I3-CA02

    <div class="sm_mode"> {{{question}}} </div>

    [{"vars":[{"varval":"Simon owns a dairy farm.\n\nHe keeps 2 cows on every acre of the property.\n\nHow many acres would he need for 12 cows?\n"},{"varval":"sm_nogap 2 cows per acre\n\n\n<div class=\"aligned\">\n\n| | |\n| --------------------: | -------------- |\n| $\\therefore$ Acres needed for 12 cows | = $\\dfrac{12}{2}$ |\n| | = {{{correctAnswer}}} acres |\n\n</div>"}]},{"vars":[{"varval":"Benson owns a pineapple farm.\n\nHe plants 8 pineapples per square metre of land. \n\nHow many square metres of land would he need to plant 96 pineapples?"},{"varval":"sm_nogap 8 pineapples per square metre\n\n\n<div class=\"aligned\">\n\n| | |\n| --------------------: | -------------- |\n| $\\therefore$ Square metres needed for 96 pineapples | = $\\dfrac{96}{8}$ |\n| | = {{{correctAnswer}}} square metres|\n\n</div>"}]},{"vars":[{"varval":"Emma manages a wildlife park.\n\nThe kangaroo enclosure houses 25 kangaroos per hectare.\n\nHow many hectares would she need for 200 kangaroos?\n"},{"varval":"sm_nogap 25 kangaroos per hectare\n\n\n<div class=\"aligned\">\n\n| | |\n| --------------------: | -------------- |\n| $\\therefore$ Hectares needed for 200 kangaroos | = $\\dfrac{200}{25}$ |\n| | = {{{correctAnswer}}} hectares|\n\n</div>"}]},{"vars":[{"varval":"Boyd owns a chicken farm.\n\nHe grazes 80 free range chickens per hectare.\n\nHow many hectares would he need for 400 chickens?\n"},{"varval":"sm_nogap 80 chickens per hectare\n\n\n<div class=\"aligned\">\n\n| | |\n| --------------------: | -------------- |\n| $\\therefore$ Hectares needed for 400 chickens| = $\\dfrac{400}{80}$ |\n| | = {{{correctAnswer}}} hectares|\n\n</div>"}]},{"vars":[{"varval":"Sue-Anne owns an elephant sanctuary.\n\nShe keeps 12 elephants per hectare.\n\nHow many hectares would she need for 72 elephants?\n"},{"varval":"sm_nogap 12 elephants per hectare\n\n\n<div class=\"aligned\">\n\n| | |\n| --------------------: | -------------- |\n| $\\therefore$ Hectares needed for 72 elephants| = $\\dfrac{72}{12}$ |\n| | = {{{correctAnswer}}} hectares|\n\n</div>"}]},{"vars":[{"varval":"Jock owns an apple orchard.\n\nHe grows 20 apple trees per hectare.\n\nHow many hectares would he need for 90 apple trees?"},{"varval":"sm_nogap 20 apple trees per hectare\n\n\n<div class=\"aligned\">\n\n| | |\n| --------------------: | -------------- |\n| $\\therefore$ Hectares needed for 90 apple trees| = $\\dfrac{90}{20}$ |\n| | = {{{correctAnswer}}} hectares|\n\n</div>"}]}]

  1027. Statistics and Probability, NAPX-H3-CA25

    <div class="sm_mode"> Skylar records the number of students who can play different instruments in his class. He records the results in the table below. <br> sm_img https://teacher.smartermaths.com.au/wp-content/uploads/2018/07/NAPX-H3-CA25.svg 470 indent3 vpad <br> Some students play more than one instrument. What is the least number of boys that could be in Skylar's class? </div>

    [{"vars":null}]

  1028. Statistics, NAPX-p122499v02

    <div class="sm_mode"> {{{question}}} </div>

    [{"vars":[{"varval":"<div class=\"sm_mode\">\n\nAt the start of school, every child chooses one sport.\n\nThe table shows how many children chose each sport.\n\n<br>\n\n<div class=\"sm-table col1-color1 row1-color1 top-left-cell-hidden\">\n\n>>| | Rugby | Swimming |AFL| Tennis|\n|:-:|:-:|:-:|:-:|:-:|\n| Girls | 40| 35|15|30|\n| Boys| 20| 55|25|10|\n\n</div>\n\n<br>Select the statement that is true.\n\n</div>"},{"varval":"By trial and error, consider option 3:\n\nTotal girls = 40 + 35 + 15 + 30 = 120\n\n<div class=\"aligned\">\n\n| | |\n| --------------------: | -------------- |\n| $\\dfrac{1}{4} \\times 120$ | = 30 |\n\n</div>\n\n<br>\n\n$\\therefore$ {{{correctAnswer}}}"}]},{"vars":[{"varval":"<div class=\"sm_mode\">\n\nAt the start of the cricket season, every player chooses one specialty to practice.\n\nThe table shows how many players chose each specialty.\n\n<br>\n\n<div class=\"sm-table col1-color3 row1-color3 top-left-cell-hidden\">\n\n>>| | Bowling | Out Field |Slips| Batting|\n|:-:|:-:|:-:|:-:|:-:|\n| Females | 10| 12|17|21|\n| Males| 22| 8|15|25|\n\n</div>\n\n<br>Select the statement that is true.\n\n</div>"},{"varval":"By trial and error, consider option 2:\n\nTotal females = 10 + 12 + 17 + 21 = 60\n\nHalf females = 30\n\nFemales bowling or batting = 10 + 21 = 31\n\n$\\therefore$ {{{correctAnswer}}}"}]},{"vars":[{"varval":"<div class=\"sm_mode\">\n\nAt the school canteen, every child chooses one lunch option.\n\nThe table shows how many children chose each lunch.\n\n<br>\n\n<div class=\"sm-table col1-color5 row1-color5 top-left-cell-hidden\">\n\n>>| | Salad Bowl | Sandwich |Wrap| Fruit Salad Bowl|\n|:-:|:-:|:-:|:-:|:-:|\n| Girls | 15| 21|10|14|\n| Boys| 12| 25|15|20|\n\n</div>\n\n<br>Select the statement that is true.\n\n</div>"},{"varval":"By trial and error, consider option 4:\n\nTotal boys = 12 + 25 + 15 + 20 = 72\n\n<div class=\"aligned\">\n\n| | |\n| --------------------: | -------------- |\n| $\\dfrac{1}{6} \\times 72$ | = 12 |\n\n</div>\n\n<br>\n\n$\\therefore$ {{{correctAnswer}}}"}]},{"vars":[{"varval":"<div class=\"sm_mode\">\n\nAt the start of Year 8, every child chooses one language.\n\nThe table shows how many children chose each language.\n\n<br>\n\n<div class=\"sm-table col1-color8 row1-color8 top-left-cell-hidden\">\n\n>>| | Spanish| German |French| Japanese|\n|:-:|:-:|:-:|:-:|:-:|\n| Girls | 21| 16|23|24|\n| Boys| 22| 21|16|21|\n\n</div>\n\n<br>Select the statement that is true.\n\n</div>"},{"varval":"By trial and error, consider option 1:\n\nTotal girls = 21 + 16 + 23 + 24 = 84\n\n$\\dfrac{1}{4}\\ \\times$ girls = 21 &nbsp;$\\Rightarrow$ &nbsp;$\\dfrac{3}{4} = 3 \\times 21 = 63$\n\nGerman + French + Japanese = 16 + 23 + 24 = 63\n\n\n$\\therefore$ {{{correctAnswer}}}"}]},{"vars":[{"varval":"<div class=\"sm_mode\">\n\nAt a children's party, each child can choose one pizza flavour.\n\nThe table shows how many children chose each type of pizza.\n\n<br>\n\n<div class=\"sm-table col1-color1 row1-color1 top-left-cell-hidden\">\n\n>>| | Cheese| Margarita |Pepperoni| Hawaiian|\n|:-:|:-:|:-:|:-:|:-:|\n| Girls | 9| 6|5|4|\n| Boys| 8| 4|10|2|\n\n</div>\n\n<br>Select the statement that is true.\n\n</div>"},{"varval":"By trial and error, consider option 2:\n\n\n\n<div class=\"aligned\">\n\n| | | |\n| --------------------: | -------------- | -------------- |\n| Total girls| = 9 + 6 + 5 + 4 | = 24 |\n| Total boys| = 8 + 4 + 10 + 2| = 24 |\n\n</div>\n\n<br>\n\n$\\therefore$ {{{correctAnswer}}}"}]},{"vars":[{"varval":"<div class=\"sm_mode\">\n\nAt the start of athletics training, every child chooses one field event.\n\nThe table shows how many children chose each field event.\n\n<br>\n\n<div class=\"sm-table col1-color5 row1-color5 top-left-cell-hidden\">\n\n>>| | Shotput| Discus |Long Jump| High Jump|\n|:-:|:-:|:-:|:-:|:-:|\n| 8 Year Olds | 12| 16|10|18|\n| 9 Year Olds| 15| 14|24|17|\n\n</div>\n\n<br>Select the statement that is true.\n\n</div>"},{"varval":"By trial and error, consider option 3:\n\nTotal 8 Year olds = 12 + 16 + 10 + 18 = 56\n\n8 Year olds choosing discus = $\\dfrac{16}{56}$ = $\\dfrac{2}{7}$\n\nTotal 9 Year olds = 15 + 14 + 24 + 17 = 70\n\n9 Year olds choosing discus = $\\dfrac{14}{70}$ = $\\dfrac{1}{5}$\n\nSince $\\dfrac{1}{5}$ < $\\dfrac{2}{7}$,\n\n$\\therefore$ {{{correctAnswer}}}"}]}]

  1029. Statistics, NAPX-K2-19

    <div class="sm_mode"> Before attending a school camp, each child chose their favourite activity. The table shows how many children chose each activity. <div class="sm-table col1-color8 row1-color8 top-left-cell-hidden"> >>| | Boating | Spear Fishing | Bushwalking|Bird Watching| |:-:|:-:|:-:|:-:|:-:| | Girls | 20| 55|15|20| | Boys| 20| 35|25|20| </div> <br> Select the statement that is true. </div>

    [{"vars":null}]

  1030. Statistics, NAPX-G2-30

    <div class="sm_mode"> {{{question}}} </div>

    [{"vars":[{"varval":"Anna is captain of the chess club and keeps a record of the number of students in the club at the end of each week.\n\n<br>\n\nsm_img https://teacher.smartermaths.com.au/wp-content/uploads/2019/01/NAPX-G2-30.svg 335 indent vpad\n\n<br>\n\nIf no students have left the club, how many joined in Week 3?"},{"varval":"\nStudents in club:\n\n\n>At the end of week 2 = 11\n\n>At the end of week 3 = 19\n\n<br>\n<div class=\"no-margin-bottom\">\n\n$\\therefore$ Students who joined in week 3\n\n</div>\n\n\n<div class=\"aligned\">\n\n\n>>| |\n| ----------------------- |\n|= 19 $-$ 11|\n|= {{{correctAnswer}}} |\n\n\n</div>"}]},{"vars":[{"varval":"Mathew is captain of the drama club and keeps a record of the number of students in the club at the end of each week.\n\n<br>\n\nsm_img https://teacher.smartermaths.com.au/wp-content/uploads/2023/08/Statistics-NAP_50148_v1.svg 250 indent vpad\n\n<br>\n\nIf no students have left the club, how many joined in Week 3?"},{"varval":"Students in club:\n\n\n>At the end of week 2 = 22\n\n>At the end of week 3 = 33\n\n<br>\n<div class=\"no-margin-bottom\">\n\n$\\therefore$ Students who joined in week 3\n\n</div>\n\n\n<div class=\"aligned\">\n\n\n>>| |\n| ----------------------- |\n|= 33 $-$ 22|\n|= {{{correctAnswer}}} |\n\n\n</div>"}]},{"vars":[{"varval":"Natalie is the supervisor of the Maths club and keeps a record of the number of students in the club at the end of each week.\n\n<br>\n\nsm_img https://teacher.smartermaths.com.au/wp-content/uploads/2023/08/Statistics-NAP_50148_v2-min.svg 280 indent vpad\n\n<br>\n\nIf no students have left the club, how many joined in Week 3?"},{"varval":"Students in club:\n\n\n>At the end of week 2 = 16\n\n>At the end of week 3 = 21\n\n<br>\n<div class=\"no-margin-bottom\">\n\n$\\therefore$ Students who joined in week 3\n\n</div>\n\n\n<div class=\"aligned\">\n\n\n>>| |\n| ----------------------- |\n|= 21 $-$ 16|\n|= {{{correctAnswer}}} |\n\n\n</div>"}]},{"vars":[{"varval":"Zhang is a trainer at the Little Athletics club and keeps a record of the number of children in the club at the end of each week.\n\n<br>\n\nsm_img https://teacher.smartermaths.com.au/wp-content/uploads/2023/08/Statistics-NAP_50148_v3-min.svg 290 indent vpad\n\n<br>\n\nIf no children have left the club, how many joined in Week 3?"},{"varval":"Children in club:\n\n\n>At the end of week 2 = 25\n\n>At the end of week 3 = 37\n\n<br>\n<div class=\"no-margin-bottom\">\n\n$\\therefore$ Students who joined in week 3\n\n</div>\n\n\n<div class=\"aligned\">\n\n\n>>| |\n| ----------------------- |\n|= 37 $-$ 25|\n|= {{{correctAnswer}}} |\n\n\n</div>"}]},{"vars":[{"varval":"Adeben is a trainer at the gymnastics club and keeps a record of the number of children in the club at the end of each week.\n\n<br>\n\nsm_img https://teacher.smartermaths.com.au/wp-content/uploads/2023/08/Statistics-NAP_50148_v4-min.svg 290 indent vpad\n\n<br>\n\nIf no children have left the club, how many joined in Week 3?"},{"varval":"Children in club:\n\n\n>At the end of week 2 = 31\n\n>At the end of week 3 = 39\n\n<br>\n<div class=\"no-margin-bottom\">\n\n$\\therefore$ Students who joined in week 3\n\n</div>\n\n\n<div class=\"aligned\">\n\n\n>>| |\n| ----------------------- |\n|= 39 $-$ 31|\n|= {{{correctAnswer}}} |\n\n\n</div>"}]},{"vars":[{"varval":"Fern is a supervisor at the walking club and keeps a record of the number of people in the club at the end of each week.\n\n<br>\n\nsm_img https://teacher.smartermaths.com.au/wp-content/uploads/2023/08/Statistics-NAP_50148_v5-min.svg 290 indent vpad\n\n<br>\n\nIf no people have left the club, how many joined in Week 3?"},{"varval":"People in club:\n\n\n>At the end of week 2 = 19\n\n>At the end of week 3 = 28\n\n<br>\n<div class=\"no-margin-bottom\">\n\n$\\therefore$ Students who joined in week 3\n\n</div>\n\n\n<div class=\"aligned\">\n\n\n>>| |\n| ----------------------- |\n|= 28 $-$ 19|\n|= {{{correctAnswer}}} |\n\n\n</div>"}]}]

  1031. Statistics, NAPX-G2-15

    <div class="sm_mode"> {{{question}}} </div>

    [{"vars":[{"varval":"Christie measured the temperature every 3 hours from 6:00 am to 3:00 pm..\n\n<br>\n\n<div class=\"sm-table col1-color1\">\n\n>>| Time of the day | 6:00 am |9:00 am |12:00 pm |3:00 pm |\n|:-:|:-:|:-:|:-:|:-:|\n| Temperature ($\\degree$C) | 22|27|32|26|\n\n</div>\n\n<br>\n\nWhich graph shows Christie's results?"},{"varval":"1st increase = 27 − 22 = 5°\n\n2nd increase = 32 − 27 = 5°\n\n$\\Rightarrow$ Temperature then drops 6° to 26°\n\n{{{correctAnswer}}}"}]},{"vars":[{"varval":"Darren measured the temperature every 3 hours from 6:00 am to 3:00 pm.\n\n<br>\n\n<div class=\"sm-table col1-color4\">\n\n>>| Time of the day | 6:00 am |9:00 am |12:00 pm |3:00 pm |\n|:-:|:-:|:-:|:-:|:-:|\n| Temperature ($\\degree$C) | 18|25|28|27|\n\n</div>\n\n<br>\n\nWhich graph shows Darren's results?"},{"varval":"1st increase = 25 − 18 = 7°\n\n2nd increase = 28 − 25 = 3°\n\n$\\Rightarrow$ Temperature then drops 1° to 27°\n\n{{{correctAnswer}}}"}]}]

  1032. Statistics, NAPX-p124001v01

    <div class="sm_mode"> {{{question}}} </div>

    [{"vars":[{"varval":"Charlie counts the number of moths he has collected.\n\n* 4 Bogon moths\n\n* 12 Atlas moths\n\n* 8 Gypsy moths\n\n<table>\n<tbody>\n<tr>\n<td style=\"padding-left: 1px; vertical-align: middle;\">In each picture graph below,</td>\n<td style=\"padding-left: 1px;\"><img class=\"alignnone size-full wp-image-124006\"\nsrc=\"https://teacher.smartermaths.com.au/wp-content/uploads/2019/01/NAPX-J2-29-v1.svg\" alt=\"\" /></td>\n<td style=\"padding-left: 1px;\">= 4 moths.</td>\n</tr>\n</tbody>\n</table>\n\nSelect the picture graph that shows the number of moths Charlie counts."},{"varval":"<table>\n<tbody>\n<tr>\n<td style=\"padding-left: 1px;\"><img class=\"alignnone size-full wp-image-124006\"\nsrc=\"https://teacher.smartermaths.com.au/wp-content/uploads/2019/01/NAPX-J2-29-v1.svg\" alt=\"\" /></td>\n<td style=\"padding-left: 1px;\">= 4 moths</td>\n</tr>\n</tbody>\n</table>\n\n<table>\n<tbody>\n<tr>\n<td style=\"padding-left: 1px;\"><img class=\"alignnone size-full wp-image-124006\"\nsrc=\"https://teacher.smartermaths.com.au/wp-content/uploads/2019/01/NAPX-J2-29-v1.svg\" alt=\"\" /> <img class=\"alignnone size-full wp-image-124006\"\nsrc=\"https://teacher.smartermaths.com.au/wp-content/uploads/2019/01/NAPX-J2-29-v1.svg\" alt=\"\" /></td>\n\n<td style=\"padding-left: 1px;\">= 8 moths</td>\n</tr>\n</tbody>\n</table>\n\n<table>\n<tbody>\n<tr>\n<td style=\"padding-left: 1px;\"><img class=\"alignnone size-full wp-image-124006\"\nsrc=\"https://teacher.smartermaths.com.au/wp-content/uploads/2019/01/NAPX-J2-29-v1.svg\" alt=\"\" /> <img class=\"alignnone size-full wp-image-124006\"\nsrc=\"https://teacher.smartermaths.com.au/wp-content/uploads/2019/01/NAPX-J2-29-v1.svg\" alt=\"\" /> <img class=\"alignnone size-full wp-image-124006\"\nsrc=\"https://teacher.smartermaths.com.au/wp-content/uploads/2019/01/NAPX-J2-29-v1.svg\" alt=\"\" /></td>\n\n<td style=\"padding-left: 1px;\">= 12 moths</td>\n</tr>\n</tbody>\n</table>\n\n{{{correctAnswer}}}\n"}]},{"vars":[{"varval":"Silk counts the number of moths she has collected.\n\n* 5 Bogon moths\n\n* 30 Atlas moths\n\n* 10 Gypsy moths\n\n<table>\n<tbody>\n<tr>\n<td style=\"padding-left: 1px; vertical-align: middle;\">In each picture graph below,</td>\n<td style=\"padding-left: 1px;\"><img class=\"alignnone size-full wp-image-124006\"\nsrc=\"https://teacher.smartermaths.com.au/wp-content/uploads/2019/01/NAPX-J2-29-v1.svg\" alt=\"\" /></td>\n<td style=\"padding-left: 1px;\">= 5 moths.</td>\n</tr>\n</tbody>\n</table>\n\nSelect the picture graph that shows the number of moths Silk counts."},{"varval":"<table>\n<tbody>\n<tr>\n<td style=\"padding-left: 1px;\"><img class=\"alignnone size-full wp-image-124006\"\nsrc=\"https://teacher.smartermaths.com.au/wp-content/uploads/2019/01/NAPX-J2-29-v1.svg\" alt=\"\" /></td>\n<td style=\"padding-left: 1px;\">= 5 moths</td>\n</tr>\n</tbody>\n</table>\n\n<table>\n<tbody>\n<tr>\n<td style=\"padding-left: 1px;\"><img class=\"alignnone size-full wp-image-124006\"\nsrc=\"https://teacher.smartermaths.com.au/wp-content/uploads/2019/01/NAPX-J2-29-v1.svg\" alt=\"\" /> <img class=\"alignnone size-full wp-image-124006\"\nsrc=\"https://teacher.smartermaths.com.au/wp-content/uploads/2019/01/NAPX-J2-29-v1.svg\" alt=\"\" /> <img class=\"alignnone size-full wp-image-124006\"\nsrc=\"https://teacher.smartermaths.com.au/wp-content/uploads/2019/01/NAPX-J2-29-v1.svg\" alt=\"\" /> <img class=\"alignnone size-full wp-image-124006\"\nsrc=\"https://teacher.smartermaths.com.au/wp-content/uploads/2019/01/NAPX-J2-29-v1.svg\" alt=\"\" /> <img class=\"alignnone size-full wp-image-124006\"\nsrc=\"https://teacher.smartermaths.com.au/wp-content/uploads/2019/01/NAPX-J2-29-v1.svg\" alt=\"\" /> <img class=\"alignnone size-full wp-image-124006\"\nsrc=\"https://teacher.smartermaths.com.au/wp-content/uploads/2019/01/NAPX-J2-29-v1.svg\" alt=\"\" /></td>\n\n<td style=\"padding-left: 1px;\">= 30 moths</td>\n</tr>\n</tbody>\n</table>\n\n<table>\n<tbody>\n<tr>\n<td style=\"padding-left: 1px;\"><img class=\"alignnone size-full wp-image-124006\"\nsrc=\"https://teacher.smartermaths.com.au/wp-content/uploads/2019/01/NAPX-J2-29-v1.svg\" alt=\"\" /> <img class=\"alignnone size-full wp-image-124006\"\nsrc=\"https://teacher.smartermaths.com.au/wp-content/uploads/2019/01/NAPX-J2-29-v1.svg\" alt=\"\" /></td>\n\n<td style=\"padding-left: 1px;\">= 10 moths</td>\n</tr>\n</tbody>\n</table>\n\n{{{correctAnswer}}}"}]},{"vars":[{"varval":"Butch counts the number of moths he has collected.\n\n* 6 Bogon moths\n\n* 24 Atlas moths\n\n* 12 Gypsy moths\n\n<table>\n<tbody>\n<tr>\n<td style=\"padding-left: 1px; vertical-align: middle;\">In each picture graph below,</td>\n<td style=\"padding-left: 1px;\"><img class=\"alignnone size-full wp-image-124006\"\nsrc=\"https://teacher.smartermaths.com.au/wp-content/uploads/2019/01/NAPX-J2-29-v1.svg\" alt=\"\" /></td>\n<td style=\"padding-left: 1px;\">= 6 moths.</td>\n</tr>\n</tbody>\n</table>\n\nSelect the picture graph that shows the number of moths Butch counts."},{"varval":"<table>\n<tbody>\n<tr>\n<td style=\"padding-left: 1px;\"><img class=\"alignnone size-full wp-image-124006\"\nsrc=\"https://teacher.smartermaths.com.au/wp-content/uploads/2019/01/NAPX-J2-29-v1.svg\" alt=\"\" /></td>\n<td style=\"padding-left: 1px;\">= 6 moths</td>\n</tr>\n</tbody>\n</table>\n\n<table>\n<tbody>\n<tr>\n<td style=\"padding-left: 1px;\"><img class=\"alignnone size-full wp-image-124006\"\nsrc=\"https://teacher.smartermaths.com.au/wp-content/uploads/2019/01/NAPX-J2-29-v1.svg\" alt=\"\" /> <img class=\"alignnone size-full wp-image-124006\"\nsrc=\"https://teacher.smartermaths.com.au/wp-content/uploads/2019/01/NAPX-J2-29-v1.svg\" alt=\"\" /> <img class=\"alignnone size-full wp-image-124006\"\nsrc=\"https://teacher.smartermaths.com.au/wp-content/uploads/2019/01/NAPX-J2-29-v1.svg\" alt=\"\" /> <img class=\"alignnone size-full wp-image-124006\"\nsrc=\"https://teacher.smartermaths.com.au/wp-content/uploads/2019/01/NAPX-J2-29-v1.svg\" alt=\"\" /></td>\n\n<td style=\"padding-left: 1px;\">= 24 moths</td>\n</tr>\n</tbody>\n</table>\n\n<table>\n<tbody>\n<tr>\n<td style=\"padding-left: 1px;\"><img class=\"alignnone size-full wp-image-124006\"\nsrc=\"https://teacher.smartermaths.com.au/wp-content/uploads/2019/01/NAPX-J2-29-v1.svg\" alt=\"\" /> <img class=\"alignnone size-full wp-image-124006\"\nsrc=\"https://teacher.smartermaths.com.au/wp-content/uploads/2019/01/NAPX-J2-29-v1.svg\" alt=\"\" /></td>\n\n<td style=\"padding-left: 1px;\">= 12 moths</td>\n</tr>\n</tbody>\n</table>\n\n{{{correctAnswer}}}"}]},{"vars":[{"varval":"Charlie counts the number of moths he has collected.\n\n* 7 Bogon moths\n\n* 42 Atlas moths\n\n* 14 Gypsy moths\n\n<table>\n<tbody>\n<tr>\n<td style=\"padding-left: 1px; vertical-align: middle;\">In each picture graph below,</td>\n<td style=\"padding-left: 1px;\"><img class=\"alignnone size-full wp-image-124006\"\nsrc=\"https://teacher.smartermaths.com.au/wp-content/uploads/2019/01/NAPX-J2-29-v1.svg\" alt=\"\" /></td>\n<td style=\"padding-left: 1px;\">= 7 moths.</td>\n</tr>\n</tbody>\n</table>\n\nSelect the picture graph that shows the number of moths Charlie counts."},{"varval":"<table>\n<tbody>\n<tr>\n<td style=\"padding-left: 1px;\"><img class=\"alignnone size-full wp-image-124006\"\nsrc=\"https://teacher.smartermaths.com.au/wp-content/uploads/2019/01/NAPX-J2-29-v1.svg\" alt=\"\" /></td>\n<td style=\"padding-left: 1px;\">= 7 moths</td>\n</tr>\n</tbody>\n</table>\n\n<table>\n<tbody>\n<tr>\n<td style=\"padding-left: 1px;\"><img class=\"alignnone size-full wp-image-124006\"\nsrc=\"https://teacher.smartermaths.com.au/wp-content/uploads/2019/01/NAPX-J2-29-v1.svg\" alt=\"\" /> <img class=\"alignnone size-full wp-image-124006\"\nsrc=\"https://teacher.smartermaths.com.au/wp-content/uploads/2019/01/NAPX-J2-29-v1.svg\" alt=\"\" /> <img class=\"alignnone size-full wp-image-124006\"\nsrc=\"https://teacher.smartermaths.com.au/wp-content/uploads/2019/01/NAPX-J2-29-v1.svg\" alt=\"\" /> <img class=\"alignnone size-full wp-image-124006\"\nsrc=\"https://teacher.smartermaths.com.au/wp-content/uploads/2019/01/NAPX-J2-29-v1.svg\" alt=\"\" /> <img class=\"alignnone size-full wp-image-124006\"\nsrc=\"https://teacher.smartermaths.com.au/wp-content/uploads/2019/01/NAPX-J2-29-v1.svg\" alt=\"\" /> <img class=\"alignnone size-full wp-image-124006\"\nsrc=\"https://teacher.smartermaths.com.au/wp-content/uploads/2019/01/NAPX-J2-29-v1.svg\" alt=\"\" /></td>\n\n<td style=\"padding-left: 1px;\">= 42 moths</td>\n</tr>\n</tbody>\n</table>\n\n<table>\n<tbody>\n<tr>\n<td style=\"padding-left: 1px;\"><img class=\"alignnone size-full wp-image-124006\"\nsrc=\"https://teacher.smartermaths.com.au/wp-content/uploads/2019/01/NAPX-J2-29-v1.svg\" alt=\"\" /> <img class=\"alignnone size-full wp-image-124006\"\nsrc=\"https://teacher.smartermaths.com.au/wp-content/uploads/2019/01/NAPX-J2-29-v1.svg\" alt=\"\" /></td>\n\n<td style=\"padding-left: 1px;\">= 14 moths</td>\n</tr>\n</tbody>\n</table>\n\n{{{correctAnswer}}}"}]}]

  1033. Number, NAP-36810

    <div class="sm_mode"> In a tap dancing class, the ratio of boys to girls is 5 : 7. There are 6 more girls than boys in this group. How many students are in this group altogether? </div>

    [{"vars":null}]

  1034. Number, NAP-79168

    <div class="sm_mode"> A health food packet had the following information on its label. <br> sm_img https://teacher.smartermaths.com.au/wp-content/uploads/2017/02/naplan-2011-17mc.png 300 indent3 vpad <br>What was the mass of the health food packet to the nearest gram? </div>

    [{"vars":null}]

  1035. Number, NAP-79136

    <div class="sm_mode"> Fleur lives 15 kilometres from her work. On Wednesday, she drove to work and averaged 60 kilometres per hour. On Thursday, she took the bus which averaged 15 kilometres per hour. What was the extra time of the bus journey, in minutes, compared to when she drove on Wednesday? </div>

    [{"vars":null}]

  1036. Statistics and Probability, NAPX-J4-CA19, NAPX-J3-CA26

    <div class="sm_mode"> {{{question}}} </div>

    [{"vars":[{"varval":"This graph shows a company's profit over a four year period.\n\n<br>\n\nsm_img https://teacher.smartermaths.com.au/wp-content/uploads/2018/06/NAPX-J4-CA192.svg 435 indent vpad\n\n<br>\n\nWhich conclusion can be reached from the graph?\n"},{"varval":"Important to note the profit (y-axis) in this graph starts at $20 000, not zero.\n\nsm_nogap The lowest profit in any year in this graph is $25 000.\n\n<div class=\"aligned\">\n\n| | |\n| --------------------: | -------------- |\n| $\\therefore$ | Profits were greater than \\$20 000 in the period |\n| | between Year 1 and Year 4. |\n\n</div> "}]},{"vars":[{"varval":"This graph shows a company's profit over a five year period.\n\n<br>\n\nsm_img https://teacher.smartermaths.com.au/wp-content/uploads/2022/08/Stat_Prob_50144_v1.svg 435 indent3 vpad\n\n<br>\n\nWhich conclusion can be reached from the graph?\n"},{"varval":"sm_nogap Important to note that profits (on the y-axis) in this table begin at \\$20 000, not zero.\n\n<div class=\"aligned\">\n\n| | |\n| --------------------: | -------------- |\n| $\\therefore$ | {{{correctAnswer}}} |\n\n</div> "}]},{"vars":[{"varval":"This graph shows a company's profit over a five year period.\n\n<br>\n\nsm_img https://teacher.smartermaths.com.au/wp-content/uploads/2022/08/Stat_Prob_50144_v2.svg 435 indent vpad\n\n<br>\n\nWhich conclusion can be reached from the graph?\n"},{"varval":"Important to note that profits (y-axis) on this graph begin at \\$10 000, not zero.\n\nBy testing each option:\n\nConsider option 2\n\nProfit in year 4 = $50 000\n\nProfit in year 1 = \\$25 000 = $\\dfrac{1}{2}\\ \\times$ $50 000\n\n<div class=\"aligned\">\n\n| | |\n| --------------------: | -------------- |\n| $\\therefore$ | {{{correctAnswer}}} \n\n\n</div> "}]},{"vars":[{"varval":"This graph shows a company's profit over a five year period.\n\n<br>\n\nsm_img https://teacher.smartermaths.com.au/wp-content/uploads/2022/08/Stat_Prob_50144_v3_1.svg 435 indent vpad\n\n<br>\n\nWhich conclusion can be reached from the graph?\n"},{"varval":"Important to note that profits (y-axis) on this graph begin at \\$6000, not zero.\n\nBy testing each option:\n\nConsider option 3\n\nProfit in year 5 = $15 000\n\nProfit in year 1 = \\$30 000 = 2 $\\times$ \\$15 000\n\n<div class=\"aligned\">\n\n| | |\n| --------------------: | -------------- |\n| $\\therefore$ | {{{correctAnswer}}} |\n\n\n</div> "}]},{"vars":[{"varval":"This graph shows a company's profit over a five year period.\n\n<br>\n\nsm_img https://teacher.smartermaths.com.au/wp-content/uploads/2022/08/Stat_Prob_50144_v4.svg 435 indent vpad\n\n<br>\n\nWhich conclusion can be reached from the graph?\n"},{"varval":"Important to note that profits (y-axis) on this graph begin at \\$15 000, not zero.\n\nBy testing each option:\n\nConsider option 4\n\nProfit increase Y1 to Y2 = 60 000 $-$ 25 000 = $35 000\n\nProfit decrease Y4 to Y5 = 70 000 $−$ 35 000 = $35 000\n\n<div class=\"aligned\">\n\n| | |\n| --------------------: | -------------- |\n| $\\therefore$ | The increase in profit from Year 1 to Year 2 is the same as|\n| | the decrease in profit from Year 4 to Year 5. |\n\n</div> "}]},{"vars":[{"varval":"This graph shows a company's profit over a five year period.\n\n<br>\n\nsm_img https://teacher.smartermaths.com.au/wp-content/uploads/2022/08/Stat_Prob_50144_v5.svg 450 indent vpad\n\n<br>\n\nWhich conclusion can be reached from the graph?\n"},{"varval":"Important to note that profits (y-axis) on this graph begin at \\$50 000, not zero.\n\nBy testing each option:\n\nsm_nogap Consider option 4\n\n<br>\n<div class=\"aligned\">\n\n| | |\n| --------------------: | -------------- |\n| Mean Profit |= $\\dfrac{80\\ 000+95\\ 000+60\\ 000+110\\ 000+105\\ 000}{5}$ ||\n| |= $90 000|\n| ||\n\n\n$\\therefore$ {{{correctAnswer}}}\n\n</div> "}]}]

  1037. Statistics and Probability, NAPX-L4-CA21

    <div class="sm_mode"> {{{question}}} </div>

    [{"vars":[{"varval":"Benzin bought 16 pencils.\n\nHe bought:\n* one packet of 10 pencils for $19.00, and\n* three packets of 2 pencils for $5.20.\n\nApproximately, what is the mean price of the 16 pencils?"},{"varval":"<div class=\"aligned\">\n\n| | |\n| --------------------: | -------------- |\n| Total spent | \\= 19.00 + 3 $\\times$ 5.20 |\n| | \\= 19.00 + 15.60 |\n| | \\= \\$34.60 |\n\n| | |\n| ----------------------------: | -------------- |\n| $\\therefore$ Mean price | \\= $\\dfrac{34.60}{16}$ |\n| | \\= {{{correctAnswer}}} |\n\n</div>"}]},{"vars":[{"varval":"Jessie bought 17 sewing kits for her textiles class.\n\nShe bought:\n\n* one box of 9 sewing kits for $32.90, and\n* two boxes of 4 sewing kits for $16.20\n\nApproximately, what is the mean price of the 17 sewing kits?"},{"varval":"<div class=\"aligned\">\n\n| | |\n| --------------------: | -------------- |\n| Total spent | \\= 32.90 + 2 $\\times$ 16.20 |\n| | \\= 32.90 + 32.40 |\n| | \\= $65.30 |\n\n| | |\n| ----------------------------: | -------------- |\n| $\\therefore$ Mean price | \\= $\\dfrac{65.30}{17}$ |\n| | \\= {{{correctAnswer}}} |\n\n</div>"}]},{"vars":[{"varval":"Clem bought 26 eggs from the farmers' market.\n\nHe bought:\n* two cartons of 10 eggs for \\$5.20, and\n* six extra eggs at 63 cents each.\n\nWhat is the mean price of the 26 eggs?"},{"varval":"<div class=\"aligned\">\n\n| | |\n| ------------------------ | --------------- |\n| $\\text{Total spent}$ | = 2 $\\times$ 5.20 + 6 $\\times$ 0.65 |\n| | = 10.40 + 3.90 |\n| | = $14.30 |\n\n| | |\n| ---------------------------------- | --------------- |\n| $\\therefore \\text{Mean price}$ | = $\\dfrac{14.30}{26}$ |\n| | = {{{correctAnswer}}} |\n\n</div>"}]}]

  1038. Statistics and Probability, NAPX-LA-CA08 o1 Safari browser is making the colours look poor near the lines. Var3 is the worst but noticeable on most. We can sort out early next week.

    <div class="sm_mode"> {{{question}}} </div>

    [{"vars":[{"varval":"Tilba needs to find out the number of students studying both Biology and Physics.\n\nWhich shaded region in the Venn Diagrams represents the students that Tilba is looking for?"},{"varval":"{{{correctAnswer}}}\n"}]},{"vars":[{"varval":"Georgie needs to find out the number of entrants running either the 10 kilometre Fun Run or the 5 kilometre Walk, but not both.\n\nWhich shaded region in the Venn Diagrams represents the entrants that Georgie is looking for?"},{"varval":"{{{correctAnswer}}}\n"}]},{"vars":[{"varval":"Preston needs to find out the number of diners choosing a dessert but not an entree in his restaurant tonight.\n\nWhich shaded region in the Venn Diagrams represents the diners that Preston is looking for?"},{"varval":"{{{correctAnswer}}}\n"}]},{"vars":[{"varval":"Brady needs to find out the number of customers who have either milk or sugar in their coffee, including those that have both.\n\nWhich shaded region in the Venn Diagrams represents the customers that Brady is looking for?"},{"varval":"{{{correctAnswer}}}\n"}]},{"vars":[{"varval":"Jocelyn needs to find out the number of day-care students who have fruit only for morning tea.\n\nWhich shaded region in the Venn Diagrams represents the day-care students that Jocelyn is looking for?"},{"varval":"{{{correctAnswer}}}\n"}]},{"vars":[{"varval":"Jorga needs to find out the number of students who travel to school by bus or travel to school using both bus and train.\n\nWhich shaded region in the Venn Diagrams represents the students that Jorga is looking for?"},{"varval":"{{{correctAnswer}}}\n"}]}]

  1039. Statistics and Probability, NAPX-LA-CA08 o2 Var3 - question and answer images ... colour looks patchy around the lines (let me know if this is just my browser) Colour palette: https://cms.smarterschool.com.au/item/3e6b9cdd-fd78-4ea0-a45d-ec5a6e90fb05/ Fixed var1 - var5 using Geogebra

    <div class="sm_mode"> {{{question}}} </div>

    [{"vars":[{"varval":"Murray wants to find the number of boys in his class that play AFL or soccer but don't play both.\n\nWhich shaded region in the Venn diagrams represents the boys that Murray is seeking?"},{"varval":"{{{correctAnswer}}}"}]},{"vars":[{"varval":"Anna wants to find the number of people in her office that only drink coffee.\n\nWhich shaded region in the Venn diagrams represents the people that Anna is seeking?"},{"varval":"{{{correctAnswer}}}"}]},{"vars":[{"varval":"Domino wants to find the number of players in his football team that like both cheese and supreme pizza toppings.\n\nWhich shaded region in the Venn diagrams represents the players that Domino is seeking?"},{"varval":"{{{correctAnswer}}}"}]},{"vars":[{"varval":"A trail walking event has a 20 kilometre hike on Saturday and a 10 kilometre hike on Sunday.\n\nWhich shaded region shows the number of hikers who are participating in the 20 kilometre hike or the 10 kilometre hike, including those who are participating in both hikes?"},{"varval":"{{{correctAnswer}}}"}]},{"vars":[{"varval":"Darcy wants to find the number of people who are taking part in the whale watch cruise only.\n\nWhich shaded region in the Venn diagrams represents the people that Darcy is seeking?"},{"varval":"{{{correctAnswer}}}"}]},{"vars":[{"varval":"Walt wants to find the number of people who subscribe to Netflix including those that subscribe to Netflix and Disney+.\n\nWhich shaded region in the Venn diagrams represents the people that Walt is seeking?"},{"varval":"{{{correctAnswer}}}"}]}]

  1040. Statistics and Probability, NAPX-E4-NC08

    This graph shows the price per tonne of resources in a country. <br> sm_img https://teacher.smartermaths.com.au/wp-content/uploads/2018/06/NAPX-E4-NC08_2.svg 700 indent vpad <br> The price of coal was less than the price of iron ore for a number of quarters, as shown by the graph. In one of those quarters, the price of aluminium scrap metal fell below \$50 per tonne. Which quarter was this?

    [{"vars":null}]

  1041. Statistics and Probability, NAPX-F4-CA10

    <div class="sm_mode"> {{{question}}} </div>

    [{"vars":[{"varval":"In 2017, an estimated 8.7 million vehicle owners in Australia registered their cars.\n\nThe average cost of one registration was estimated at \\$473.\n\nWhat was the total amount paid for registrations in 2017?"},{"varval":"<div class=\"aligned\">\n\n| | |\n| --------------------: | -------------- |\n| Total | \\= 8 700 000 × 473 |\n| | \\= {{{correctAnswer}}} |\n\n</div>"}]},{"vars":[{"varval":"In 2021, an estimated 5.89 million motor vehicles were registered in NSW.\n\nThe average annual cost of compulsory Third Party insurance was estimated at \\$478.50.\n\nOn average, what was the total amount paid for compulsory Third Party Greenslips in NSW in 2021?"},{"varval":"<div class=\"aligned\">\n\n| | |\n| --------------------: | -------------- |\n| Total | \\= 5 890 000 × 478.50 |\n| | \\= {{{correctAnswer}}} |\n\n</div>"}]},{"vars":[{"varval":"The 2021, the estimated sales of SUV's in a country was 1.4 million.\n\nThe average cost of one SUV in 2021 was estimated at \\$48 350.\n\nWhat was the total amount paid for SUV's in 2021?"},{"varval":"<div class=\"aligned\">\n\n| | |\n| --------------------: | -------------- |\n| Total | \\= 1 400 000 × 48 350 |\n| | \\= {{{correctAnswer}}} |\n\n</div>"}]},{"vars":[{"varval":"Over a 10 year period, the estimated attendance at music festivals across Australia was 5.34 million people in total.\n\nThe average cost of one ticket for the festival was $184.\n\nWhat was the total amount paid for festival tickets?"},{"varval":"<div class=\"aligned\">\n\n| | |\n| --------------------: | -------------- |\n| Total | \\= 5 340 000 × 184 |\n| | \\= {{{correctAnswer}}} |\n\n</div>"}]},{"vars":[{"varval":"In the second quarter of 2022, an estimated 733 homes sold in Manhattan.\n\nDuring this period, the average house price in Manhattan was $1.882 million.\n\nWhat was the total value of homes sold in Manhattan in the second quarter of 2022?"},{"varval":"<div class=\"aligned\">\n\n| | |\n| --------------------: | -------------- |\n| Total | \\= 1 882 000 × 733 |\n| | \\= {{{correctAnswer}}} |\n\n</div>"}]},{"vars":[{"varval":"In the first half of 2022, an estimated 3.9 million mobile phones were sold in Australia.\n\nThe average cost of one a mobile phone was estimated at \\$208.\n\nOn average, what was the total amount paid for mobile phones in the first half of 2022?"},{"varval":"<div class=\"aligned\">\n\n| | |\n| --------------------: | -------------- |\n| Total | \\= 3 900 000 × 208 |\n| | \\= {{{correctAnswer}}} |\n\n</div>"}]}]

  1042. Statistics and Probability, NAPX-H4-NC05

    The world high jump record between 1960 and 1995 is graphed below. <br> sm_img https://teacher.smartermaths.com.au/wp-content/uploads/2018/04/NAPX-H4-NC05.svg 563 indent vpad <br> By about how many centimeters did the world record increase between 1965 and 1995?

    [{"vars":null}]

  1043. Statistics and Probability, NAPX-H3-NC13

    <div class="sm_mode"> sm_nogap Tranquilla's class were asked to choose their favourite vegetable among four choices. * 5 voted for zuccini * 5 voted for carrots * 6 voted for snow peas * The rest voted for spinach <br> sm_img https://teacher.smartermaths.com.au/wp-content/uploads/2018/08/NAPX-H3-NC13_2.svg 370 indent3 vpad <br>How many students in Tranquilla's class chose spinach? </div>

    [{"vars":null}]

  1044. Statistics and Probability, NAPX-F3-CA16 I have taken the white space out around all the images and think these look pretty good.

    <div class="sm_mode"> {{{question}}} </div>

    [{"vars":[{"varval":"Aaron went on holiday and spent his money on accommodation, golf and meals.\n\nHe spent \\$1500 in total and the pie chart below shows how he spent it.\n\nsm_img https://teacher.smartermaths.com.au/wp-content/uploads/2022/09/Stat_Prob_50133_v0b.svg 320 indent vpad\n\nHow much money did Aaron spend on meals on his holiday?"},{"varval":"sm_nogap Percentage spent on accommodation\n\n<div class=\"aligned\">\n\n>>| | \n| ----------------------- |\n|= $\\dfrac{675}{1500} \\times 100$|\n|= 45% |\n\n</div>\n\n<br>\n\n$\\Rightarrow$ Percentage on meals = 100 − (40 + 45) = 15%\n\n<div class=\"no-margin-bottom\">\n\n$\\therefore$ Amount spent on meals\n\n</div>\n\n<div class=\"aligned\">\n\n>>| | \n| ----------------------- |\n|= 15% × 1500|\n|= {{{correctAnswer}}} |\n\n</div>"}]},{"vars":[{"varval":"Ian is budgeting for his next holiday and is allocating money to accommodation, airfares and day trips.\n\nHe is planning to spend \\$4500 in total and the pie chart below shows how he is allocating his money.\n\n\nsm_img https://teacher.smartermaths.com.au/wp-content/uploads/2022/09/Stat_Prob_50133_v1b.svg 320 indent vpad\n\n\nHow much money did Ian allocate for airfares in his holiday budget?"},{"varval":"sm_nogap Percentage spent on accommodation\n\n<div class=\"aligned\">\n\n>>| | \n| ----------------------- |\n|= $\\dfrac{1305}{4500} \\times 100$|\n|= 29% |\n\n</div>\n\n<br>\n\n$\\Rightarrow$ Percentage on Airfares = 100 − (15 + 29) = 56%\n\n<div class=\"no-margin-bottom\">\n\n$\\therefore$ Amount spent on airfares\n\n</div>\n\n<div class=\"aligned\">\n\n>>| | \n| ----------------------- |\n|= 56% × 4500|\n|= {{{correctAnswer}}} |\n\n</div>"}]},{"vars":[{"varval":"Burke is measuring the amount, in kilograms, of each vegetable he has harvested from his garden this winter. His garden harvest is made up of carrots, zucchinis and pumpkins.\n\nHe harvested 400 kilograms in total and the pie chart below shows the amount of each vegetable he harvested.\n\n\n\nsm_img https://teacher.smartermaths.com.au/wp-content/uploads/2022/09/Stat_Prob_50133_v2b1.svg 280 indent vpad\n\n\n\nHow many kilograms of carrots did Burke harvest this winter?"},{"varval":"sm_nogap Percentage of zucchinis harvested\n\n<div class=\"aligned\">\n\n>>| | \n| ----------------------- |\n|= $\\dfrac{112}{400} \\times 100$|\n|= 28% |\n\n</div>\n\n<br>\n\n$\\Rightarrow$ Percentage of Carrots = 100 − (40 + 28) = 32%\n\n<div class=\"no-margin-bottom\">\n\n$\\therefore$ Kilograms of carrots harvested\n\n</div>\n\n<div class=\"aligned\">\n\n>>| | \n| ----------------------- |\n|= 32% × 400|\n|= {{{correctAnswer}}} |\n\n</div>"}]},{"vars":[{"varval":"Karen has allocated year 9 students to either tennis, soccer or ultimate frisbee for their term 3 sport.\n\nThere are 225 year 9 students in total and the pie chart below shows how the sports were allocated.\n\n<br>\n\nsm_img https://teacher.smartermaths.com.au/wp-content/uploads/2022/09/Stat_Prob_50133_v3b.svg 310 indent vpad\n\n\n\nHow many students were allocated ultimate frisbee as their term 3 sport?"},{"varval":"sm_nogap Percentage allocated to tennis\n\n<div class=\"aligned\">\n\n>>| | \n| ----------------------- |\n|= $\\dfrac{54}{225} \\times 100$|\n|= 24% |\n\n</div>\n\n<br>\n\n$\\Rightarrow$ Percentage allocated to ultimate frisbee = 100 − (24 + 8) = 68%\n\n<div class=\"no-margin-bottom\">\n\n$\\therefore$ Students allocated to ultimate frisbee\n\n</div>\n\n<div class=\"aligned\">\n\n>>| | \n| ----------------------- |\n|= 68% × 225|\n|= {{{correctAnswer}}} |\n\n</div>"}]},{"vars":[{"varval":"Joy set up a cardio gym in her garage and purchased a treadmill, a rowing machine and a spin bike.\n\nShe spent \\$6000 in total and the pie chart below shows how she spent it.\n\n\n\nsm_img https://teacher.smartermaths.com.au/wp-content/uploads/2022/09/Stat_Prob_50133_v4b.svg 310 indent vpad\n\n<br>\n\nHow much money did Joy spend on the spin bike?"},{"varval":"sm_nogap Percentage spent on the treadmill\n\n<div class=\"aligned\">\n\n>>| | \n| ----------------------- |\n|= $\\dfrac{2460}{6000} \\times 100$|\n|= 41% |\n\n</div>\n\n<br>\n\n$\\Rightarrow$ Percentage on spin bike = 100 − (41 + 28) = 31%\n\n<div class=\"no-margin-bottom\">\n\n$\\therefore$ Amount spent on spin bike\n\n</div>\n\n<div class=\"aligned\">\n\n>>| | \n| ----------------------- |\n|= 31% × 6000|\n|= {{{correctAnswer}}} |\n\n</div>"}]},{"vars":[{"varval":"Jye was checking his phone battery usage and found he spent his time listening to music, using social media and viewing YouTube videos.\n\nLast week he spent 25 hours in total using his phone and the pie chart below shows the breakdown of his usage.\n\n\n<br>\n\nsm_img https://teacher.smartermaths.com.au/wp-content/uploads/2022/09/Stat_Prob_50133_v5b.svg 300 indent vpad\n\nHow much time did Jye spend on social media?"},{"varval":"sm_nogap Percentage of usage watching YouTube videos\n\n<div class=\"aligned\">\n\n>>| | \n| ----------------------- |\n|= $\\dfrac{8}{25} \\times 100$|\n|= 32% |\n\n</div>\n\n<br>\n\n$\\Rightarrow$ Percentage of usage on social media = 100 − (48 + 32) = 20%\n\n<div class=\"no-margin-bottom\">\n\n$\\therefore$ Amount of usage on social media\n\n</div>\n\n<div class=\"aligned\">\n\n>>| | \n| ----------------------- |\n|= 20% × 25|\n|= {{{correctAnswer}}} |\n\n</div>"}]}]

  1045. Number, NAPX-E4-NC04

    <div class="sm_mode"> Ash bought 7 candy canes and 2 chocolate bars from the shop. The candy canes cost 25 cents each and the chocolate bars cost \$1.20 each. How much did Ash pay? </div>

    [{"vars":null}]

  1046. Number, NAPX-F4-CA07

    <div class="sm_mode"> {{{question}}} </div>

    [{"vars":[{"varval":"Darryl buys some jousting sticks online that costs him $213.\n\nThe postage costs involved are listed in the table below.\n\n<div class=\"sm-table col1-color1\">\n\n> > | <div style=\"line-height: 3rem\">Weight</div> | Up to <br> 500 g | 500 g to less <br> than 1 kg | 1 kg to less <br> than 1.25 kg | 1.25 kg to less <br> than 1.5 kg |\n> > | :-------------------------------------------: | :----------------: | :----------------------------: | :------------------------------: | :--------------------------------: |\n> > | Postage | $12 | $17.50 | $21.75 | $25 |\n\n</div>\n\n<br>\n\nThe jousting sticks weigh 1.3 kg.\n\nWhat does Darryl have to pay, in total, to purchase the jousting sticks and have them delivered?"},{"varval":"<div class=\"aligned\">\n\n| | |\n| ---------- | ---------------------- |\n| Total cost | \\= 213 + delivery |\n| | \\= 213 + 25 |\n| | \\= {{{correctAnswer}}} |\n\n</div>"}]},{"vars":[{"varval":"Dale buys a trombone online that costs him $78.\n\nThe postage costs involved are listed in the table below.\n\n<div class=\"sm-table col1-color8\">\n\n> > | <div style=\"line-height: 3rem\">Weight</div> | Up to <br> 500 g | 500 g to less <br> than 1 kg | 1 kg to less <br> than 1.25 kg | 1.25 kg to less <br> than 1.5 kg |\n> > | :-------------------------------------------: | :----------------: | :----------------------------: | :------------------------------: | :--------------------------------: |\n> > | Postage | $12 | $17.50 | $21.75 | $25 |\n\n</div>\n\n<br>\n\nThe trombone weighs 1.2 kg.\n\nWhat does Dale have to pay, in total, to purchase the trombone and have it delivered?"},{"varval":"<div class=\"aligned\">\n\n| | |\n| ---------- | ---------------------- |\n| Total cost | \\= 78 + delivery |\n| | \\= 78 + 21.75 |\n| | \\= {{{correctAnswer}}} |\n\n</div>"}]},{"vars":[{"varval":"Steve buys an ergonomic foot massager online that costs him $62.\n\nThe postage costs involved are listed in the table below.\n\n<div class=\"sm-table col1-color8\">\n\n> > | <div style=\"line-height: 3rem\">Weight</div> | Up to <br> 500 g | 500 g to less <br> than 1 kg | 1 kg to less <br> than 1.25 kg | 1.25 kg to less <br> than 1.5 kg |\n> > | :-------------------------------------------: | :----------------: | :----------------------------: | :------------------------------: | :--------------------------------: |\n> > | Postage | $12 | $17.50 | $21.75 | $25 |\n\n</div>\n\n<br>\n\nThe foot massager weighs 1.4 kg.\n\nWhat does Steve have to pay, in total, to purchase the foot massager and have it delivered?"},{"varval":"<div class=\"aligned\">\n\n| | |\n| ---------- | ---------------------- |\n| Total cost | \\= 62 + delivery |\n| | \\= 62 + 25 |\n| | \\= {{{correctAnswer}}} |\n\n</div>"}]},{"vars":[{"varval":"Geraldine buys some perfume online that costs her $48.\n\nThe postage costs involved are listed in the table below.\n\n<div class=\"sm-table col1-color7\">\n\n> > | <div style=\"line-height: 3rem\">Weight</div> | Up to <br> 500 g | 500 g to less <br> than 1 kg | 1 kg to less <br> than 1.25 kg | 1.25 kg to less <br> than 1.5 kg |\n> > | :-------------------------------------------: | :----------------: | :----------------------------: | :------------------------------: | :--------------------------------: |\n> > | Postage | $8.50 | $11.75 | $14 | $18.75 |\n\n</div>\n\n<br>\n\nThe perfume weighs 660 grams.\n\nWhat does Geraldine have to pay, in total, to purchase the perfume and have it delivered?"},{"varval":"<div class=\"aligned\">\n\n| | |\n| ---------- | ---------------------- |\n| Total cost | \\= 48 + delivery |\n| | \\= 48 + 11.75 |\n| | \\= {{{correctAnswer}}} |\n\n</div>"}]},{"vars":[{"varval":"Colin buys some protein powder online that costs him $73.\n\nThe postage costs involved are listed in the table below.\n\n<div class=\"sm-table col1-color7\">\n\n> > | <div style=\"line-height: 3rem\">Weight</div> | Up to <br> 500 g | 500 g to less <br> than 1 kg | 1 kg to less <br> than 1.35 kg | 1.35 kg to less <br> than 1.8 kg |\n> > | :-------------------------------------------: | :----------------: | :----------------------------: | :------------------------------: | :--------------------------------: |\n> > | Postage | $8.50 | $11.75 | $14 | $18.75 |\n\n</div>\n\n<br>\n\nThe protein powder weighs 1.5 kg.\n\nWhat does Colin have to pay, in total, to purchase the protein powder and have it delivered?"},{"varval":"<div class=\"aligned\">\n\n| | |\n| ---------- | ---------------------- |\n| Total cost | \\= 73 + delivery |\n| | \\= 73 + 18.75 |\n| | \\= {{{correctAnswer}}} |\n\n</div>"}]},{"vars":[{"varval":"Joline buys some multi-vitamins online that costs her $37.\n\nThe postage costs involved are listed in the table below.\n\n<div class=\"sm-table col1-color7\">\n\n> > | <div style=\"line-height: 3rem\">Weight</div> | Up to <br> 500 g | 500 g to less <br> than 1 kg | 1 kg to less <br> than 1.35 kg | 1.35 kg to less <br> than 1.8 kg |\n> > | :-------------------------------------------: | :----------------: | :----------------------------: | :------------------------------: | :--------------------------------: |\n> > | Postage | $8.50 | $11.75 | $14 | $18.75 |\n\n</div>\n\n<br>\n\nThe multi-vitamins weigh 1.3 kg.\n\nWhat does Joline have to pay, in total, to purchase the multi-vitamins and have them delivered?"},{"varval":"<div class=\"aligned\">\n\n| | |\n| ---------- | ---------------------- |\n| Total cost | \\= 37 + delivery |\n| | \\= 37 + 14 |\n| | \\= {{{correctAnswer}}} |\n\n</div>"}]}]

  1047. Number, NAPX-L4-CA03 v1

    <div class="sm_mode"> Kerry-Anne buys an apple for 55 cents and a banana for 80 cents. She pays with a \$2 coin. How much change should Kerry-Anne get? </div>

    [{"vars":null}]

  1048. Number, NAPX-L4-CA03 v2

    <div class="sm_mode"> Windy buys an apple for 65 cents and a banana for 90 cents. She pays with a \$2 coin. How much change should Windy get? </div>

    [{"vars":null}]

  1049. Number, NAPX-H4-NC02

    <div class="sm_mode"> Lily has \$12 to buy batteries for her torch. Each battery costs \$2.40 and Lily buys 4 batteries. Which expression shows how much money she has left? </div>

    [{"vars":null}]

  1050. Number, NAPX-H3-NC05

    <div class="sm_mode"> Nug had a \$10 note only. At the service station, he bought 6 sherbert cones that cost \$0.80 each. How much change should he get? </div>

    [{"vars":null}]

  1051. Number, NAPX-J3-NC02, NAPX-J2-10

    <div class="sm_mode"> Petunia gets \$6.45 in pocket money each week. She does extra jobs one week and earns \$3.75 more. How much money did Petunia receive in total in the week? </div>

    [{"vars":null}]

  1052. Number, NAPX-J3-NC01, NAPX-J2-08

    <div class="sm_mode"> Miley has 65 cents in 5-cent pieces. How many 5-cent pieces does she have? </div>

    [{"vars":null}]

  1053. Statistics and Probability, NAPX-E4-CA03, NAPX-E3-CA04

    Joseph asked children in his class what their favourite sport is. Their answers were used to draw the pie chart below. <br> sm_img https://teacher.smartermaths.com.au/wp-content/uploads/2018/05/NAPX-E4-CA03.svg 270 indent3 vpad <br> Around a quarter of the students preferred which sport?

    [{"vars":null}]

  1054. Statistics and Probability, NAPX-L4-CA03 o1

    Year 9 students were surveyed to find their favourite type of movie. The results were recorded in this graph <br> sm_img https://teacher.smartermaths.com.au/wp-content/uploads/2020/03/NAPX-LA-CA03-o1.svg 250 indent3 vpad <br> What percentage of Year 9 student's favourite movie type was comedy?

    [{"vars":null}]

  1055. Statistics and Probability, NAPX-H3-CA08

    A table tennis club gathers data on its members and displays it in the graph below. <br> sm_img https://teacher.smartermaths.com.au/wp-content/uploads/2018/07/NAPX-H3-CA08.svg 480 indent3 vpad <br> How many members in the table tennis club are 50 or younger?

    [{"vars":null}]

  1056. Statistics and Probability, NAPX-I3-CA09

    The graph shows the origin and type of all vehicles in a city. <br> sm_img https://teacher.smartermaths.com.au/wp-content/uploads/2018/06/NAPX-I3-NC09.svg 590 indent vpad <br> Which statement is most accurate based on the graph?

    [{"vars":null}]

  1057. Number, NAPX-F3-NC04

    <div class="sm_mode"> Ivanka has \$6.60 and bought a muffin that cost her \$3.95. How much money does she have left? </div>

    [{"vars":null}]

  1058. #50 QUICK FIXES: Graph 1 (used for Var1,2,3) ... English Boys column needs to be shortened to 3 (from 4) Graph 2 (used for Var4,5) ... Chemistry boys needs to be increased to 6 (from 5)

    <div class="sm_mode"> A group of students are surveyed and asked what is their favourite {{what}} at school. The results are graphed below. <br> {{image}} <br>Which {{what}} was {{explain1}}? </div>

    [{"vars":[{"varval":"subject"},{"varval":"sm_img https://teacher.smartermaths.com.au/wp-content/uploads/2020/08/dQ50_graph-A_r.svg 600 indent vpad"},{"varval":"the favourite of twice as many girls as boys"},{"varval":"3"},{"varval":"6"},{"varval":"the favourite subject to twice as many girls as boys"}]},{"vars":[{"varval":"subject"},{"varval":"sm_img https://teacher.smartermaths.com.au/wp-content/uploads/2020/08/dQ50_graph-A_r.svg 600 indent vpad"},{"varval":"the favourite of half as many girls as boys"},{"varval":"8"},{"varval":"4"},{"varval":"the favourite subject to half as many girls as boys"}]},{"vars":[{"varval":"subject"},{"varval":"sm_img https://teacher.smartermaths.com.au/wp-content/uploads/2020/08/dQ50_graph-A_r.svg 600 indent vpad"},{"varval":"equally favoured between boys and girls"},{"varval":"5"},{"varval":"5"},{"varval":"equally favoured between boys and girls"}]},{"vars":[{"varval":"science subject"},{"varval":"sm_img https://teacher.smartermaths.com.au/wp-content/uploads/2020/08/dQ50_graph-B_r.svg 600 indent vpad"},{"varval":"the favourite of twice as many boys as girls"},{"varval":"6"},{"varval":"3"},{"varval":"the favourite subject to twice as many boys as girls"}]},{"vars":[{"varval":"science subject"},{"varval":"sm_img https://teacher.smartermaths.com.au/wp-content/uploads/2020/08/dQ50_graph-B_r.svg 600 indent vpad"},{"varval":"the favourite of half as many boys as girls"},{"varval":"4"},{"varval":"8"},{"varval":"the favourite subject to half as many boys as girls"}]}]

  1059. #49

    <div class="sm_mode"> {{name}} {{work}} that is sold at the market. In a {{object}} that weighs {{mass1}} {{units}}, {{name}} can fit {{number}} {{item1}} that weigh {{mass2}} {{units}} each. How much, in {{units}}, does a full {{object}} of {{item1}} weigh? </div>

    [{"vars":[{"varval":"Joel"},{"varval":"makes confectionery"},{"varval":"jar"},{"varval":"$\\large j$"},{"varval":"grams"},{"varval":"$\\large n$"},{"varval":"chocolate macadamia nuts"},{"varval":"$\\large m$"},{"varval":"nut"},{"varval":"nuts"}]},{"vars":[{"varval":"Isaac"},{"varval":"grows fruit"},{"varval":"box"},{"varval":"$\\large b$"},{"varval":"grams"},{"varval":"$\\large n$"},{"varval":"cherries"},{"varval":"$\\large c$"},{"varval":"cherry"},{"varval":"cherries"}]},{"vars":[{"varval":"Veronica"},{"varval":"makes confectionery"},{"varval":"container"},{"varval":"$\\large c$"},{"varval":"grams"},{"varval":"$\\large n$"},{"varval":"lollipops"},{"varval":"$\\large l$"},{"varval":"lollipop"},{"varval":"lollipops"}]},{"vars":[{"varval":"Willie"},{"varval":"makes confectionery"},{"varval":"box"},{"varval":"$\\large b$"},{"varval":"grams"},{"varval":"$\\large n$"},{"varval":"pralines"},{"varval":"$\\large p$"},{"varval":"praline"},{"varval":"pralines"}]},{"vars":[{"varval":"Reg"},{"varval":"grows fruit"},{"varval":"box"},{"varval":"$\\large b$"},{"varval":"grams"},{"varval":"$\\large n$"},{"varval":"gooseberries"},{"varval":"$\\large g$"},{"varval":"gooseberry"},{"varval":"gooseberries"}]}]

  1060. Statistics and Probability, NAPX-I3-NC09, NAPX-I2-22

    The number of students that could kick a soccer ball more than 50 metres were counted at 7 different primary schools. The results were recorded in the graph below. <br> sm_img https://teacher.smartermaths.com.au/wp-content/uploads/2018/06/NAPX-I3-NC09-1.svg 435 indent3 vpad <br> How many students, in total, could kick the soccer ball more than 50 metres?

    [{"vars":null}]

  1061. Statistics and Probability, NAPX-G3-NC06

    Rupert measured the temperature every 3 hours from 6:00 am to 3:00 pm. <br> sm_img https://teacher.smartermaths.com.au/wp-content/uploads/2018/08/NAPX-G3-NC06.svg 470 indent3 vpad <br> Which graph shows Rupert's results?

    [{"vars":null}]

  1062. Statistics and Probability, NAPX-G4-CA03

    <div class="sm_mode"> {{{question}}} </div>

    [{"vars":[{"varval":"10 of the tallest mountains in the United States are listed in the table below:\n\n<br>\n\n<div class=\"sm-table row1-color8\">\n\n>>| Name of Mountain | Height (m) | Location |\n|:-:|:-:|:-:|\n| Denali | 6190| Alaska|\n| Mount Bona | 5044| Alaska|\n| Mount Massive | 4398| Colorado|\n| Castle Peak | 4352| Colorado|\n| Mount Evans | 4350| Colorado|\n| Mount Gabb | 4190| California|\n| Mount Loa | 4169| Hawaii|\n| Kings Peak| 4125| Utah|\n| Wheeler Peak | 4013| New Mexico|\n| Mount Bear | 3540| Alaska |\n\n\n</div>\n\n<br>How much taller than Colorado's tallest mountain is Alaska's tallest mountain?"},{"varval":"<div class=\"aligned\">\n\n| | |\n| --------------------- | --------------------- |\n| $\\text{Extra Height}$ | = 6190 − 4398 |\n| | = {{correctAnswer}} |\n\n</div>"}]},{"vars":[{"varval":"7 of the world's longest rivers are listed in the table below.\n\n<br>\n\n<div class=\"sm-table row1-color8\">\n\n>>| Name of River | Length (km) | Location |\n|:-:|:-:|:-:|\n| Yangtze River | 6300| China|\n| Yellow River| 5464| China|\n| Mekong River| 4350| China|\n| Hazma River| 5920| Brazil|\n| Amazon | 6760| Brazil|\n| Lena River| 4294| Russia|\n| Irtysh River| 4248| Russia|\n\n\n</div>\n\n<br>How much shorter is China's longest river compared to Brazil's longest river?"},{"varval":"sm_nogap Amazon River – Yangtze\n\n<div class=\"aligned\">\r\n\r\n>>| |\r\n| ---------- |\r\n| \\= $6760\\ − \\ 6300$ |\r\n| \\= {{{correctAnswer}}} |\r\n\r\n</div>\r"}]},{"vars":[{"varval":"Eight of the tallest mountains in the United States are listed in the table below:\n\n<br>\n\n<div class=\"sm-table row1-color2\">\n\n>>| Name of Mountain | Height (m) | Location |\n|:-:|:-:|:-:|\n| Denali | 6190| Alaska|\n| Mount Bona | 5044| Alaska|\n| Mount Massive | 4398| Colorado|\n| Castle Peak | 4352| Colorado|\n| Mount Evans | 4350| Colorado|\n| Mount Gabb | 4190| California|\n| Kings Peak| 4125| Utah|\n| Wheeler Peak | 4013| New Mexico|\n\n\n</div>\n\n<br>\nHow much taller than Utah's tallest mountain is Colorado's tallest mountain?"},{"varval":"<div class=\"sm_mode\">\n\nsm_nogap Mount Massive $–$ Kings Peak\n\n<div class=\"aligned\">\n\n>>| | \n| ----------------------- |\n|= 4398 $-$ 4125|\n|= {{{correctAnswer}}} |\n\n</div>\n\n</div>"}]}]

  1063. Statistics, NAPX-L4-CA01 v1

    This graph shows the amount of electricity generated by solar power and wind power in the Australian states. <br> sm_img https://teacher.smartermaths.com.au/wp-content/uploads/2019/11/nap-L4-ca01-ver1.svg 680 indent vpad <br> Which state generates the least electricity using solar power?

    [{"vars":null}]

  1064. Q50

    <div class="sm_mode"> sm_img https://teacher.smartermaths.com.au/wp-content/uploads/2020/07/Q50-diagram.png 650 indent vpad <br>{{1}} got off the bus at the {{2}} on {{3}}. What direction does {{1}} need to walk to get to the {{4}}? </div>

    [{"vars":[{"varval":"Lily"},{"varval":"park"},{"varval":"Ivy Avenue"},{"varval":"YMCA"}]},{"vars":[{"varval":"Connor"},{"varval":"medical centre"},{"varval":"Railway Street"},{"varval":"bakery"}]},{"vars":[{"varval":"Justine"},{"varval":"court house"},{"varval":"Railway Street"},{"varval":"park"}]},{"vars":[{"varval":"Rambo"},{"varval":"supermarket"},{"varval":"Eve Avenue"},{"varval":"medical centre"}]},{"vars":[{"varval":"Ganus"},{"varval":"bakery"},{"varval":"Railway Street"},{"varval":"supermarket"}]}]

  1065. Q49

    <div class="sm_mode"> The entrance heights of four {{1}} are measured and recorded in the table below. {{{2}}} <br>{{3}} is driving a {{4}} that is {{5}} metres high and {{6}} is driving a {{4}} that is {{7}} metres high. Which of the following two {{8}} can {{3}} and {{6}} both safely use? </div>

    [{"vars":[{"varval":"carparks"},{"varval":"<div class=\"sm-table col1-color3 row1-color3 top-left-cell-hidden\">\n\n>>| | Entrance Height (metres)|\n|:-:|:-:|\n| Carpark A| 2.6|\n| Carpark B| 3.0|\n| Carpark C| 2.6|\n| Carpark D| 2.8|\n\n</div>"},{"varval":"Mike"},{"varval":"caravan"},{"varval":"2.7"},{"varval":"Mal"},{"varval":"2.5"},{"varval":"carparks"},{"varval":"Mike"}]},{"vars":[{"varval":"petrol stations"},{"varval":"<div class=\"sm-table col1-color1 row1-color1 top-left-cell-hidden\">\n\n>>| | Entrance Height (metres)|\n|:-:|:-:|\n| Caltex| 3.8|\n| BP| 3.3|\n| 7-11| 3.5|\n| Shell| 3.3|\n\n</div>"},{"varval":"Michael"},{"varval":"caravan"},{"varval":"3.4"},{"varval":"Tito"},{"varval":"3.2"},{"varval":"petrol stations"},{"varval":"Michael"}]},{"vars":[{"varval":"bridges"},{"varval":"<div class=\"sm-table col1-color1 row1-color1 top-left-cell-hidden\">\n\n>>| | Entrance Height (metres)|\n|:-:|:-:|\n| Castle Bridge| 4.3|\n| Hanging Bridge| 4.8|\n| Pont Bridge| 4.2|\n| Fenton Bridge| 4.7|\n\n</div>"},{"varval":"Dawn"},{"varval":"truck"},{"varval":"4.2"},{"varval":"Frank"},{"varval":"4.5"},{"varval":"bridges"},{"varval":"Frank"}]},{"vars":[{"varval":"petrol stations"},{"varval":"<div class=\"sm-table col1-color1 row1-color1 top-left-cell-hidden\">\n\n>>| | Entrance Height (metres)|\n|:-:|:-:|\n| Texaco| 4.7|\n| Mobil| 5.0 |\n| BP| 4.7|\n| Shell| 4.9|\n\n</div>"},{"varval":"Clyde"},{"varval":"truck"},{"varval":"4.6"},{"varval":"Burt"},{"varval":"4.8"},{"varval":"stations"},{"varval":"Burt"}]},{"vars":[{"varval":"bridges"},{"varval":"<div class=\"sm-table col1-color8 row1-color8 top-left-cell-hidden\">\n\n>>| | Entrance Height (metres)|\n|:-:|:-:|\n| Hasler Bridge| 3.7|\n| Cherry Bridge| 4.1 |\n| Lyons Bridge| 3.7|\n| Menzies Bridge| 3.9|\n\n</div>"},{"varval":"Sally"},{"varval":"caravan"},{"varval":"3.6"},{"varval":"Eliza"},{"varval":"3.8"},{"varval":"bridges"},{"varval":"Eliza"}]}]

  1066. #48

    <div class="sm_mode"> {{name}} is using the roast calculator, pictured below, to cook a {{mass}} {{meat1}}. <br> {{image}} <br>She intends to {{instructions}} the {{meat2}} before serving it at {{t1}}. What is the latest time {{name}} can start {{verb}} the {{meat1}}? </div>

    [{"vars":[{"varval":"Shelly"},{"varval":"4.0 kilogram"},{"varval":"leg of lamb"},{"varval":"sm_img https://teacher.smartermaths.com.au/wp-content/uploads/2020/07/roast1.png 480 indent3 vpad"},{"varval":"brown, cook and then rest"},{"varval":"lamb"},{"varval":"6:30 pm"},{"varval":"20 min + 2 hr 40 min + 30 min"},{"varval":"3 hr 30 min"},{"varval":"browning"}]},{"vars":[{"varval":"Sam"},{"varval":"2.5 kilogram"},{"varval":"leg of lamb"},{"varval":"sm_img https://teacher.smartermaths.com.au/wp-content/uploads/2020/07/roast2.png 450 indent3 vpad"},{"varval":"brown, cook and then rest"},{"varval":"lamb"},{"varval":"8:15 pm"},{"varval":"20 min + 1 hr 40 min + 30 min"},{"varval":"2 hr 30 min"},{"varval":"browning"}]},{"vars":[{"varval":"Deli"},{"varval":"5.0 kilogram"},{"varval":"turkey"},{"varval":"sm_img https://teacher.smartermaths.com.au/wp-content/uploads/2020/07/roast3.png 450 indent3 vpad"},{"varval":"cook and then rest"},{"varval":"turkey"},{"varval":"7:30 pm"},{"varval":"3 hr 10 min + 20 min"},{"varval":"3 hr 30 min"},{"varval":"cooking"}]},{"vars":[{"varval":"Katerina"},{"varval":"2.0 kilogram"},{"varval":"chicken"},{"varval":"sm_img https://teacher.smartermaths.com.au/wp-content/uploads/2020/07/roast4.png 500 indent3 vpad"},{"varval":"brown, cook and then rest"},{"varval":"chicken"},{"varval":"12:30 pm"},{"varval":"20 min + 1 hr 20 min + 15 min"},{"varval":"1 hr 55 min"},{"varval":"browning"}]},{"vars":[{"varval":"Dianne"},{"varval":"5.0 kilogram"},{"varval":"duck"},{"varval":"sm_img https://teacher.smartermaths.com.au/wp-content/uploads/2020/07/roast5.png 500 indent3 vpad"},{"varval":"brown, cook and then rest"},{"varval":"duck"},{"varval":"1:30 pm"},{"varval":"20 min + 3 hr 45 min + 15 min"},{"varval":"4 hr 20 min"},{"varval":"browning"}]}]

  1067. Q48

    <div class="sm_mode"> {{1}} $\times$ {{2}} = <span class="sm-text">?</span> After rounding both numbers to the nearest 10, which of the following is the best estimate of this equation? </div>

    [{"vars":[{"varval":"162"},{"varval":"205"},{"varval":"160"},{"varval":"210"},{"varval":"$\\begin{array}{ccccccccccc}\n\\ \\ \\ \\ \\ \\ \\ {160} \\times \\\\ \n\\ \\ \\ \\ \\underset{\\text{------------}}{210} \\\\\n\\ \\ {1600} \\\\\n \\underset{\\text{--------------}}{32000} \\\\\n{33600} \\\\\n\\end{array}$\n"}]},{"vars":[{"varval":"195"},{"varval":"144"},{"varval":"200"},{"varval":"140"},{"varval":"$\\begin{array}{ccccccccccc}\n\\ \\ \\ \\ \\ \\ \\ {200} \\times \\\\ \n\\ \\ \\ \\ \\underset{\\text{------------}}{140} \\\\\n\\ \\ {8000} \\\\\n \\underset{\\text{--------------}}{20000} \\\\\n\\ {28000} \\\\\n\\end{array}$\n"}]},{"vars":[{"varval":"232"},{"varval":"155"},{"varval":"230"},{"varval":"160"},{"varval":"$\\begin{array}{ccccccccccc}\n\\ \\ \\ \\ \\ \\ \\ {230} \\times \\\\ \n\\ \\ \\ \\ \\underset{\\text{------------}}{160} \\\\\n\\ {13800} \\\\\n \\underset{\\text{--------------}}{23000} \\\\\n{36800} \\\\\n\\end{array}$"}]},{"vars":[{"varval":"424"},{"varval":"115"},{"varval":"420"},{"varval":"120"},{"varval":"$\\begin{array}{ccccccccccc}\n\\ \\ \\ \\ \\ \\ \\ {420} \\times \\\\ \n\\ \\ \\ \\ \\underset{\\text{------------}}{120} \\\\\n\\ \\ {8400} \\\\\n \\underset{\\text{--------------}}{42000} \\\\\n\\ {50400} \\\\\n\\end{array}$"}]},{"vars":[{"varval":"565"},{"varval":"123"},{"varval":"570"},{"varval":"120"},{"varval":"$\\begin{array}{ccccccccccc}\n\\ \\ \\ \\ \\ \\ \\ {570} \\times \\\\ \n\\ \\ \\ \\ \\underset{\\text{------------}}{120} \\\\\n \\ {11400} \\\\\n \\underset{\\text{--------------}}{57000} \\\\\n{68400} \\\\\n\\end{array}$"}]}]

  1068. Statistics and Probability, NAPX-E3-CA07

    <div class="sm_mode"> {{{question}}} </div>

    [{"vars":[{"varval":"Jaya did a survey of the number of female toilets in three shopping centres.\n\nThe results were recorded in the table below but the key has been left off the graph?\n\n<br>\n\nsm_img https://teacher.smartermaths.com.au/wp-content/uploads/2018/08/NAPX-E3-CA07.svg 385 indent3 vpad\n\n<br>\n\nThe total number of female toilets was 50.\n\n<div class=\"sm_mode\">\n\n<table>\n<tbody>\n<tr>\n<td style=\"padding-left: 1px; text-align: right; vertical-align: middle;\">How many toilets does</td>\n<td style=\"vertical-align: middle;\"><a target=\"_blank\" href=\"https://teacher.smartermaths.com.au/wp-content/uploads/2017/02/naplan-Y7-2012-7mci.png\" rel=\"noopener\"><img class=\"alignnone wp-image-69222\" src=\"https://teacher.smartermaths.com.au/wp-content/uploads/2017/02/naplan-Y7-2012-7mci.png\" width=\"61\" height=\"61\" /></a></td>\n<td style=\"padding-left: 1px; text-align: right; vertical-align: middle;\">represent in the graph?</td>\n</tr>\n</tbody>\n</table>\n\n</div>\n"},{"varval":"sm_nogap Total number of female symbols\n\n<div class=\"aligned\">\n\n>>| | \n| ----------------------- |\n|= 5 + 4.5 + 3|\n|= 12.5 |\n\n</div>\n\n<br>\n\n<div class=\"no-margin-bottom\">\n\n$\\therefore$ The number of toilets one symbol represents\n\n</div>\n\n<div class=\"aligned\">\n\n>>| | \n| ----------------------- |\n|= $\\dfrac{50}{12.5}$|\n|= {{{correctAnswer}}} |\n\n</div>"}]},{"vars":[{"varval":"Scott did a survey of the number of male toilets in four shopping centres.\n\nThe results were recorded in the table below but the key has been left off the graph?\n\n<br>\n\nsm_img https://teacher.smartermaths.com.au/wp-content/uploads/2023/08/Statistics-NAP_50122_v1.svg 340 indent3 vpad\n\n<br>\n\nThe total number of male toilets was 60.\n\n<div class=\"sm_mode\">\n\n<table>\n<tbody>\n<tr>\n<td style=\"padding-left: 1px; text-align: right; vertical-align: middle;\">How many toilets does</td>\n<td style=\"vertical-align: middle;\"><a target=\"_blank\" href=\"https://teacher.smartermaths.com.au/wp-content/uploads/2023/08/Statistics-NAP_50122-min.svg\" rel=\"noopener\"><img class=\"alignnone wp-image-69222\" src=\"https://teacher.smartermaths.com.au/wp-content/uploads/2023/08/Statistics-NAP_50122_1-min.svg\" width=\"22\" height=\"22\" /></a></td>\n<td style=\"padding-left: 1px; text-align: right; vertical-align: middle;\">represent in the graph?</td>\n</tr>\n</tbody>\n</table>\n\n</div>\n"},{"varval":"sm_nogap Total number of male symbols\n\n<div class=\"aligned\">\n\n>>| | \n| ----------------------- |\n|= 5.5 + 6.5 + 4.5 + 3.5|\n|= 20 |\n\n</div>\n\n<br>\n\n<div class=\"no-margin-bottom\">\n\n$\\therefore$ The number of toilets one symbol represents\n\n</div>\n\n<div class=\"aligned\">\n\n>>| | \n| ----------------------- |\n|= $\\dfrac{60}{20}$|\n|= {{{correctAnswer}}} |\n\n</div>"}]}]

  1069. Statistics and Probability, NAPX-J3-CA01

    <div class="sm_mode"> {{{question}}} </div>

    [{"vars":[{"varval":"Four students recorded the number of times they played different sports in one week in the table below.\n\n<br>\n\n<div class=\"sm-table col1-color2 row1-color2 top-left-cell-hidden\">\n\n>>| | AFL | Soccer | Tennis | Netball|\n|:-:|:-:|:-:|:-:|:-:|\n| Ben | 0| 3|2|2|\n|Jack | 2| 0|6|0|\n| Daisy | 4| 5|0|0|\n| Molly | 3| 2|4|4|\n\n</div>\n\n<br>In total, which sport was played the most times?"},{"varval":"Times each sport was played:\n<div class=\"aligned\">\n\n| | |\n| ------------- | ---------- |\n| AFL | \\= 0 + 2 + 4 + 3 = 9|\n| Soccer| \\= 3 + 0 + 5 + 2 = 10 |\n| Tennis| \\= 2 + 6 + 0 + 4 = 12 |\n| Netball| \\= 2 + 0 + 0 + 4 = 6 |\n\n</div>\n\n<br>\n\n$\\therefore$ {{{correctAnswer}}} was played the most."}]},{"vars":[{"varval":"Four students recorded the number of times they played different sports in one week in the table below.\n\n<br>\n\n<div class=\"sm-table col1-color3 row1-color3 top-left-cell-hidden\">\n\n>>| | Badminton | Cricket | Ice Hockey | Water Polo|\n|:-:|:-:|:-:|:-:|:-:|\n| Bob | 0| 1|2|3|\n|Ally | 2| 0|1|2|\n| Drew | 3| 2|0|0|\n| Candice | 1| 2|0|3|\n\n</div>\n\n<br>In total, which sport was played the most times?"},{"varval":"Times each sport was played:\n<div class=\"aligned\">\n\n| | |\n| ------------- | ---------- |\n| Badminton | \\= 0 + 2 + 3 + 1 = 6|\n| Cricket| \\= 1 + 0 + 2 + 2 = 5 |\n| Ice Hockey| \\= 2 + 1 + 0 + 0 = 3 |\n| Water Polo| \\= 3 + 2 + 0 + 3 = 8 |\n\n</div>\n\n<br>\n\n\n$\\therefore$ {{{correctAnswer}}} was played the most."}]},{"vars":[{"varval":"In the table below, four friends recorded the number of times they participated in certain leisure activities during the holidays.\n\n<br>\n\n<div class=\"sm-table col1-color4 row1-color4 top-left-cell-hidden\">\n\n>>| | Movies | Beach | Bowling | Laser Tag|\n|:-:|:-:|:-:|:-:|:-:|\n| Joy | 2| 5|0|1|\n|Alice| 0| 3|2|2|\n| Denny | 4| 5|0|0|\n| Mort | 3| 0|4|1|\n\n</div>\n\n<br>Which of the friends participated in the most leisure activities?"},{"varval":"Number of times each friend participated:\n<div class=\"aligned\">\n\n| | |\n| ------------- | ---------- |\n| Joy | \\= 2 + 5 + 0 + 1 = 8|\n| Alice| \\= 0 + 3 + 2 + 2 = 7 |\n| Denny| \\= 4 + 5 + 0 + 0 = 9 |\n| Mort| \\= 3 + 0 + 4 + 1 = 8 |\n\n</div>\n\n<br>\n\n$\\therefore$ {{{correctAnswer}}} participated in the most leisure activities."}]},{"vars":[{"varval":"In the table below, four friends recorded the number of times they participated in certain leisure activities during the holidays.\n\n<br>\n\n<div class=\"sm-table col1-color5 row1-color5 top-left-cell-hidden\">\n\n>>| | Movies | Beach | Bowling | Laser Tag|\n|:-:|:-:|:-:|:-:|:-:|\n| Joy | 2| 5|0|1|\n|Alice| 0| 3|2|2|\n| Denny | 4| 5|0|0|\n| Mort | 3| 0|4|1|\n\n</div>\n\n<br>In total, which leisure activity was attended the most times?"},{"varval":"Times each sport was played:\n<div class=\"aligned\">\n\n| | |\n| ------------- | ---------- |\n| Movies | \\= 2 + 0 + 4 + 3 = 9|\n| Beach | \\= 5 + 3 + 5 + 0 = 13|\n| Bowling | \\= 0 + 2 + 0 + 4 = 6|\n| Laser Tag | \\= 1 + 2 + 0 + 1 = 4|\n\n</div>\n\n<br>\n\n$\\therefore$ {{{correctAnswer}}} was played the most."}]},{"vars":[{"varval":"In the table below, four students recorded the number of times they practised different sports at Little Athletics over a four week period.\n\n<br>\n\n<div class=\"sm-table col1-color6 row1-color6 top-left-cell-hidden\">\n\n>>| | 100 metres | Discus | Long Jump | Shotput|\n|:-:|:-:|:-:|:-:|:-:|\n| Mike | 6| 4|5|3|\n|Jesse | 8| 5|4|5|\n| Susie | 5| 4|8|3|\n| Seb | 4| 2|4|9|\n\n</div>\n\n<br>In total, which sport was played the most times?"},{"varval":"Times each sport was practised:\n\n<div class=\"aligned\">\n\n| | |\n| ------------- | ---------- |\n| 100 metres | \\= 6 + 8 + 5 + 4 = 23|\n| Discus| \\= 4 + 5 + 4 + 2 = 15|\n| Long Jump | \\= 5 + 4 + 8 + 4 = 21|\n| Shotput | \\= 3 + 5 + 3 + 9 = 20|\n\n</div>\n\n<br>\n\n$\\therefore$ {{{correctAnswer}}} was practised the most."}]},{"vars":[{"varval":"Four students recorded the number of hours they spent studying for different subjects before their exams in the table below.\n\n<br>\n\n<div class=\"sm-table col1-color8 row1-color8 top-left-cell-hidden\">\n\n>>| | English | Mathematics | Science | Geography | History |\n|:-:|:-:|:-:|:-:|:-:|:-:|\n| Mandy | 2 | 3 | 2 | 1 | 1 |\n| Kin | 1 | 1 | 2 | 0 | 3 |\n| Rupert | 2 | 2 | 2 | 2 | 2 |\n| Celeste | 3 | 2 | 4 | 0 | 0 |\n\n</div>\n\n<br>In total, which student did the most hours of study?"},{"varval":"Hours each student studied:\n\n<div class=\"aligned\">\n\n| | |\n| ------------- | ---------- |\n| Mandy | \\= 2 + 3 + 2 + 1 + 1 = 9|\n| Kin | \\= 1 + 1 + 2 + 0 + 3 = 7|\n| Rupert | \\= 2 + 2 + 2 + 2 + 2 = 10|\n| Celeste | \\= 3 + 2 + 4 + 0 + 0 = 9|\n\n</div>\n\n<br>\n\n$\\therefore$ {{{correctAnswer}}} studied the most hours."}]}]

  1070. Number, NAP-I4-CA24

    <div class="sm_mode"> A shop sells four sizes of chocolate bar. sm_img https://teacher.smartermaths.com.au/wp-content/uploads/2016/12/naplan-2016-24.png 518 indent vpad <br>Which packet costs the least per gram? </div>

    [{"vars":null}]

  1071. Number, NAP-F4-NC23 SA

    <div class="sm_mode"> John, Olivia and Louis are picking grapes to earn money. Their pay is based on the number of tonnes of grapes they pick. <br> sm_img https://teacher.smartermaths.com.au/wp-content/uploads/2017/01/NAP-F4-NC231.png 210 indent3 vpad <br>Their total pay is \$640. How much does Olivia earn? </div>

    [{"vars":null}]

  1072. Number, NAP-C4-CA19

    <div class="sm_mode"> Pamela looks at the price of four sunscreens. <br> sm_img https://teacher.smartermaths.com.au/wp-content/uploads/2017/05/naplan-2010-19mc.png 630 indent vpad <br>Which sunscreen is the cheapest per litre? </div>

    [{"vars":null}]

  1073. <div class="sm_mode"> David has a job selling mobile phone plans. His weekly salary, $W$ dollars, is calculated using the rule below: >$W = 300 + 0.05P$ <br>where $P$ is the total value in dollars of the mobile phone plans he sells that week. David sold \$32 000 worth of mobile phone plans in a given week. What was David's salary in the week? </div>

    [{"vars":null}]

  1074. Number, NAP-F4-NC15

    <div class="sm_mode"> Brandon has a money jar that contains 50 cent, 20 cent, 10 cent and 5 cent pieces only. If he buys a pie worth \$2.65, what is the minimum amount of coins he can pay with? </div>

    [{"vars":null}]

  1075. Number, NAP-I4-CA15 Vars 0,2,3,4,5 have tables with inconsistent border weights. - looks browser specific with no problem on safari. Will check test

    <div class="sm_mode"> {{{question}}} </div>

    [{"vars":[{"varval":"The table below lists the original price and the amount of discount of a pair of jeans at four different shops.\n\n<br>\n\n<div class=\"sm-table row1-color3 heading-color6\">\n\n>>JEANS SALE\n\n>>| Shop | Original Price | Discount|\n|:-:|:-:|:-:|\n| A | $20 |25%|\n| B | $21 |$\\dfrac{1}{3}$|\n| C | $18|20%|\n| D | $17|$2 off|\n\n</div>\n\n<br>Which shop has the lowest sale price for the jeans?\n"},{"varval":"Consider the sale price at each shop:\n\n\nA = 20 − (25\\% $\\times$ 20) = \\$15\n\nB = 21 − $\\bigg( \\dfrac{1}{3}\\ \\times 21 \\bigg)$ = \\$14\n\nC = 18 − (20\\% $\\times$ 18) = \\$14.40\n\nD = 17 − 2 = \\$15\n\n$\\therefore$ Shop {{{correctAnswer}}} has the lowest sale price."}]},{"vars":[{"varval":"The table below lists the original price and the amount of discount on beach towels at four different shops.\n\n<br>\n\n<div class=\"sm-table row1-color1 heading-color2\">\n\n>>BEACH TOWEL SALE\n\n>>| Shop | Original Price | Discount|\n|:-:|:-:|:-:|\n| A | $25 |$7 off|\n| B | $24 |$\\dfrac{1}{3}$|\n| C | $18|10%|\n| D | $20|25%|\n\n</div>\n\n<br>Which shop has the lowest sale price for a beach towel?"},{"varval":"Consider the sale price at each shop:\n\n\nA = 25 - 7 = $18\n\nB = 24 − $\\bigg( \\dfrac{1}{3}\\ \\times 24 \\bigg)$ = 24 $-$ 8 = \\$16\n\nC = 18 − (10\\% $\\times$ 18) = 18 $-$ 1.80 = \\$16.20\n\nD = 20 − (25\\% $\\times$ 20) = 20 $-$ 5 = \\$15\n\n$\\therefore$ Shop {{{correctAnswer}}} has the lowest sale price."}]},{"vars":[{"varval":"The table below lists the original price and the amount of discount on sandshoes at four different shops.\n\n<br>\n\n<div class=\"sm-table row1-color7 heading-color8\">\n\n>>SANDSHOE SALE\n\n>>| Shop | Original Price | Discount|\n|:-:|:-:|:-:|\n| A | $27 |$6 off|\n| B | $33 |$\\dfrac{1}{3}$|\n| C | $30|20%|\n| D | $36|25%|\n\n</div>\n\n<br>Which shop has the lowest sale price for the sandshoes?"},{"varval":"Consider the sale price at each shop:\n\n\nA = 27 $-$ 6 = $21\n\nB = 33 − $\\bigg( \\dfrac{1}{3}\\ \\times 33 \\bigg)$ = 33 $-$ 11 = \\$22\n\nC = 30 − (20\\% $\\times$ 30) = 30 $-$ 6 = \\$24\n\nD = 36 − (25\\% $\\times$ 36) = 36 $-$ 9 = \\$27\n\n$\\therefore$ Shop {{{correctAnswer}}} has the lowest sale price."}]},{"vars":[{"varval":"The table below lists the original price and the amount of discount on sunglasses at four different shops.\n\n<br>\n\n<div class=\"sm-table row1-color7 heading-color8\">\n\n>>SUNGLASSES SALE\n\n>>| Shop | Original Price | Discount|\n|:-:|:-:|:-:|\n| A | $36 |$5 off|\n| B | $48 |$\\dfrac{1}{3}$|\n| C | $40|20%|\n| D | $44|25%|\n\n</div>\n\n<br>Which shop has the lowest sale price for the sunglasses?"},{"varval":"Consider the sale price at each shop:\n\n\nA = 36 $-$ 5 = $31\n\nB = 48 − $\\bigg( \\dfrac{1}{3}\\ \\times 48 \\bigg)$ = 48 $-$ 16 = \\$32\n\nC = 40 − (20\\% $\\times$ 40) = 40 $-$ 8 = \\$32\n\nD = 44 − (25\\% $\\times$ 44) = 44 $-$ 11 = \\$33\n\n$\\therefore$ Shop {{{correctAnswer}}} has the lowest sale price."}]},{"vars":[{"varval":"The table below lists the original price and the amount of discount on teddy bears at four different shops.\n\n<br>\n\n<div class=\"sm-table row1-color8 heading-color6\">\n\n>>TEDDY BEAR SALE\n\n>>| Shop | Original Price | Discount|\n|:-:|:-:|:-:|\n| A | $39 |$8 off|\n| B | $48 |$\\dfrac{1}{3}$|\n| C | $35|10%|\n| D | $40|25%|\n\n</div>\n\n<br>Which shop has the lowest sale price for a teddy bear?"},{"varval":"Consider the sale price at each shop:\n\n\nA = 39 $-$ 8 = $31\n\nB = 48 − $\\bigg( \\dfrac{1}{3}\\ \\times 48 \\bigg)$ = 48 $-$ 16 = \\$32\n\nC = 35 − (10\\% $\\times$ 35) = 35 $-$ 3.50 = \\$31.50\n\nD = 40 − (25\\% $\\times$ 40) = 40 $-$ 10 = \\$30\n\n$\\therefore$ Shop {{{correctAnswer}}} has the lowest sale price."}]},{"vars":[{"varval":"The table below lists the original price and the amount of discount on fidget spinners at four different shops.\n\n<br>\n\n<div class=\"sm-table row1-color1 heading-color2\">\n\n>>FIDGET SPINNER SALE\n\n>>| Shop | Original Price | Discount|\n|:-:|:-:|:-:|\n| A | $15 |$\\dfrac{1}{3}$|\n| B | $12 |25%|\n| C | $11|20%|\n| D | $14|$4.50 off|\n\n</div>\n\n<br>Which shop has the lowest sale price for a fidget spinner?"},{"varval":"Consider the sale price at each shop:\n\n\nA = 15 − $\\bigg( \\dfrac{1}{3}\\ \\times 15 \\bigg)$ = 15 $-$ 5 = \\$10\n\nB = 12 − (25\\% $\\times$ 12) = 12 $-$ 3 = \\$9\n\nC = 11 − (20\\% $\\times$ 11 ) = 11 $-$ 2.20 = \\$8.80\n\nD = 14 $-$ 4.50 = $9.50\n\n$\\therefore$ Shop {{{correctAnswer}}} has the lowest sale price."}]}]

  1076. Statistics and Probability, NAP-D4-CA16

    <div class="sm_mode"> {{{question}}} </div>

    [{"vars":[{"varval":"Lea collects comic books.\n\nShe sells some of them.\n\nThe selling prices are listed below.\n\n> > \\$5, \\$7, \\$7, \\$7, \\$9, \\$12, \\$12, \\$14, \\$35\n\n<br>\n\nWhat is their mean (average) selling price?\n"},{"varval":"sm_nogap Total sales \n\n>>= 5 + 7 + 7 + 7 + 9 + 12 + 12 + 14 + 35\n\n>>= \\$108\n\n<br>\n\n<div class=\"aligned\">\n\n| | |\n| --------------------- | -------------------------------------------- |\n| $\\therefore \\text{Average selling price}$ | = $\\dfrac{108}{9}$ |\n| | = {{correctAnswer}} |\n\n</div> "}]},{"vars":[{"varval":"Janet sells grazing boxes.\n\nShe has several orders for the weekend.\n\nThe selling prices of the grazing boxes ordered are listed below.\n\n> > \\$50, \\$70, \\$35, \\$50, \\$50, \\$35, \\$40, \\$70, \\$50\n\n<br>\n\nWhat is the mean (average) selling price of the grazing boxes?\n"},{"varval":"sm_nogap Total sales \n\n>>= 50 + 70 + 35 + 50 + 50 + 35 + 40 + 70 + 50\n\n>>= \\$450\n\n<br>\n\n<div class=\"aligned\">\n\n| | |\n| --------------------- | -------------------------------------------- |\n| $\\therefore \\text{Average selling price}$ | = $\\dfrac{450}{9}$ |\n| | = {{correctAnswer}} |\n\n</div> "}]},{"vars":[{"varval":"Jeremiah sells tropical fish.\n\nHe sells some every day last week.\n\nThe selling prices are listed below.\n\n> > \\$15, \\$20, \\$16, \\$20, \\$20, \\$34, \\$50\n\n<br>\n\nWhat is their mean (average) selling price?\n"},{"varval":"sm_nogap Total sales \n\n>>= 15 + 20 + 16 + 20 + 20 + 34 + 50\n\n>>= \\$175\n\n<br>\n\n<div class=\"aligned\">\n\n| | |\n| --------------------- | -------------------------------------------- |\n| $\\therefore \\text{Average selling price}$ | = $\\dfrac{175}{7}$ |\n| | = {{correctAnswer}} |\n\n</div> "}]},{"vars":[{"varval":"Bliss collects antique dolls.\n\nShe sells some of them.\n\nThe selling prices are listed below.\n\n> > \\$150, \\$120, \\$135, \\$190, \\$120\n\n<br>\n\nWhat is their mean (average) selling price?\n"},{"varval":"sm_nogap Total sales \n\n>>= 150 + 120 + 135 + 190 + 120 \n\n>>= \\$715\n\n<br>\n\n<div class=\"aligned\">\n\n| | |\n| --------------------- | -------------------------------------------- |\n| $\\therefore \\text{Average selling price}$ | = $\\dfrac{715}{5}$ |\n| | = {{correctAnswer}} |\n\n</div> "}]},{"vars":[{"varval":"Dave collects Marvel characters.\n\nHe sells some of them.\n\nThe selling prices are listed below.\n\n> > \\$320, \\$180, \\$75, \\$55, \\$280, \\$104\n\n<br>\n\nWhat is their mean (average) selling price?\n"},{"varval":"sm_nogap Total sales \n\n>>= 320 + 180 + 75 + 55 + 280 + 104\n\n>>= \\$108\n\n<br>\n\n<div class=\"aligned\">\n\n| | |\n| --------------------- | -------------------------------------------- |\n| $\\therefore \\text{Average selling price}$ | = $\\dfrac{1014}{6}$ |\n| | = {{correctAnswer}} |\n\n</div> "}]},{"vars":[{"varval":"Brodie is a personal trainer.\n\nLast week she had several personal training sessions.\n\nThe session prices are listed below.\n\n> > \\$55, \\$115, \\$140, \\$85, \\$85, \\$85, \\$140, \\$140, \\$55\n\n<br>\n\nWhat is the mean (average) price of the training sessions?\n"},{"varval":"sm_nogap Total sales \n\n>>= 55 + 115 + 140 + 85 + 85 + 85 + 140 + 140 + 55\n\n>>= \\$900\n\n<br>\n\n<div class=\"aligned\">\n\n| | |\n| --------------------- | -------------------------------------------- |\n| $\\therefore \\text{Average price per session}$ | = $\\dfrac{900}{9}$ |\n| | = {{correctAnswer}} |\n\n</div> "}]}]

  1077. Number, NAP-D4-CA15

    <div class="sm_mode"> {{{question}}} </div>

    [{"vars":[{"varval":"<div class=\"sm-table col1-color1 heading-color1\">\n\n>>Type II Diabetes in USA\n\n>>| Year | 2007 | 2008 | 2009 | 2010 |\n|:-:|:-:|:-:|:-:|:-:|\n| Number of people (millions) | 31.16| 32.42 |32.93|33.21|\n\n</div>\n\n<br>Between 2007 and 2010, the number of people with type II diabetes in the USA increased by about"},{"varval":"33.21 − 31.16 = 2.05\n\n$\\therefore$ The number increased by about {{{correctAnswer}}}"}]},{"vars":[{"varval":"<div class=\"sm-table col1-color1 heading-color1\">\n\n>>Population of Texas\n\n>>| Year | 2012 | 2014 | 2016 | 2018 |\n|:-:|:-:|:-:|:-:|:-:|\n| Number of people (millions) | 25.71| 26.46|27.19|28.63|\n\n</div>\n\n<br>Between 2012 and 2018, the population of Texas increased by about"},{"varval":"28.63 − 25.71 = 2.92\n\n$\\therefore$ The population increased by about {{{correctAnswer}}}"}]}]

  1078. #47

    <div class="sm_mode"> {{name}} budgets for the following expenses each week: * \${{amt1}} for rent * \${{amt2}} for {{expense1}} * \${{amt3}} for food * \${{amt4}} for {{expense2}} If {{gender}} takes home \${{wage}} from work each week, how much can {{gender}} save in a fortnight? </div>

    [{"vars":[{"varval":"Karen"},{"varval":"395"},{"varval":"45"},{"varval":"petrol"},{"varval":"210"},{"varval":"35"},{"varval":"electricity"},{"varval":"950"},{"varval":"she"},{"varval":"265"}]},{"vars":[{"varval":"Tom"},{"varval":"180"},{"varval":"85"},{"varval":"utilities"},{"varval":"225"},{"varval":"60"},{"varval":"transport"},{"varval":"730"},{"varval":"he"},{"varval":"180"}]},{"vars":[{"varval":"Ella"},{"varval":"225"},{"varval":"65"},{"varval":"taxis"},{"varval":"130"},{"varval":"90"},{"varval":"utilities"},{"varval":"615"},{"varval":"she"},{"varval":"105"}]},{"vars":[{"varval":"Mia"},{"varval":"205"},{"varval":"35"},{"varval":"petrol"},{"varval":"120"},{"varval":"115"},{"varval":"utilities"},{"varval":"760"},{"varval":"she"},{"varval":"285"}]},{"vars":[{"varval":"Dirk"},{"varval":"165"},{"varval":"55"},{"varval":"uber rides"},{"varval":"190"},{"varval":"55"},{"varval":"electricity"},{"varval":"580"},{"varval":"he"},{"varval":"115"}]}]

  1079. Q47

    <div class="sm_mode"> A {{food1}} restaurant chain announces that it will give {{amt1}} to {{charity}} for every {{number1}} {{food2}} it sells in {{month}}. If it sells {{number2}} {{food2}} in {{month}}, how much will the {{place}} donate to {{charity}}. </div>

    [{"vars":[{"varval":"hamburger"},{"varval":"\\$1"},{"varval":"the Melanoma Foundation"},{"varval":"30"},{"varval":"hamburgers"},{"varval":"October"},{"varval":"12 000"},{"varval":"hamburger chain"},{"varval":"400"}]},{"vars":[{"varval":"Mexican"},{"varval":"\\$0.50"},{"varval":"cancer research"},{"varval":"10"},{"varval":"tacos"},{"varval":"July"},{"varval":"9000"},{"varval":"restaurant chain"},{"varval":"900"}]},{"vars":[{"varval":"hamburger"},{"varval":"\\$5"},{"varval":"vaccine research"},{"varval":"20"},{"varval":"hamburgers"},{"varval":"February"},{"varval":"6000"},{"varval":"hamburger chain"},{"varval":"300"}]},{"vars":[{"varval":"pizza"},{"varval":"\\$2"},{"varval":"the Kidney Foundation"},{"varval":"10"},{"varval":"pizzas"},{"varval":"January"},{"varval":"5000"},{"varval":"pizza chain"},{"varval":"500"}]},{"vars":[{"varval":"pizza"},{"varval":"\\$10"},{"varval":"the Mark Hughes Foundation"},{"varval":"25"},{"varval":"pizzas"},{"varval":"April"},{"varval":"5000"},{"varval":"pizza chain"},{"varval":"200"}]}]

  1080. Number, NAP-G4-CA13

    <div class="sm_mode"> Patrick has 5-cent, 20-cent and 50-cent coins in his pocket. In total he has \$1.95. What is the smallest number of coins he could have? </div>

    [{"vars":null}]

  1081. Number, NAP-B4-CA15

    <div class="sm_mode"> {{{question}}} </div>

    [{"vars":[{"varval":"Jake bought a mobile phone at 25\\% off the original price.\n\nThe original price was \\$380.\n\nHow much did Jake pay for the mobile phone?\n"},{"varval":"sm_nogap Strategy 1:\n\n<div class=\"aligned\">\n\n| | |\n| --------------- | ---------------------------- |\n| Price of mobile | = $380 - (25\\% \\times 380)$ |\n| | = $380 - 95$ |\n| | = {{{correctAnswer}}} |\n\n\nsm_nogap Strategy 2:\n\n| | |\n| --------------- | ---------------------------- |\n| Price of mobile | = ($100\\% - 25\\% )\\times 380$|\n| | = $75\\% \\times 380$ |\n| | = {{{correctAnswer}}} |\n\n</div>"}]},{"vars":[{"varval":"George bought a smart watch at 20% off the original price.\n\nThe original price was $450.\n\nHow much did George pay for the smart watch?\n"},{"varval":"sm_nogap Strategy 1:\n\n<div class=\"aligned\">\n\n| | |\n| --------------- | ---------------------------- |\n| Price of smart watch| = $450 - (20\\% \\times 450)$ |\n| | = $450 - 90$ |\n| | = {{{correctAnswer}}} |\n\n\nsm_nogap Strategy 2:\n\n| | |\n| --------------- | ---------------------------- |\n| Price of smart watch | = ($100\\% - 20\\% )\\times 450$|\n| | = $80\\% \\times 450$ |\n| | = {{{correctAnswer}}} |\n\n\n</div>"}]},{"vars":[{"varval":"Yves bought a sewing machine at 35% off the original price.\n\nThe original price was $2500.\n\nHow much did Yves pay for the sewing machine?\n"},{"varval":"sm_nogap Strategy 1:\n\n<div class=\"aligned\">\n\n| | |\n| --------------- | ---------------------------- |\n| Price of sewing machine| = $2500 - (35\\% \\times 2500)$ |\n| | = $2500 - 875$ |\n| | = {{{correctAnswer}}} |\n\n\nsm_nogap Strategy 2:\n\n| | |\n| --------------- | ---------------------------- |\n| Price of sewing machine | = ($100\\% - 35\\% )\\times 2500$ |\n| | = $65\\% \\times 2500$ |\n| | = {{{correctAnswer}}} |\n\n</div>"}]},{"vars":[{"varval":"Indira bought a new car at 15% off the original price.\n\nThe original price was $52 000.\n\nHow much did Indira pay for the new car?\n"},{"varval":"sm_nogap Strategy 1:\n\n<div class=\"aligned\">\n\n| | |\n| --------------- | ---------------------------- |\n| Price of new car | = $52\\ 000 - (15\\% \\times 52\\ 000)$ |\n| | = $52\\ 000 - 7\\ 800$ |\n| | = {{{correctAnswer}}} |\n\n\nsm_nogap Strategy 2:\n\n| | |\n| --------------- | ---------------------------- |\n| Price of new car | = ($100\\% - 15\\% )\\times 52\\ 000$ |\n| | = $85\\% \\times 52\\ 000$ |\n| | = {{{correctAnswer}}} |\n\n</div>"}]},{"vars":[{"varval":"Billy bought a new saddle for his horse at 60% off the original price.\n\nThe original price was $995.\n\nHow much did Billy pay for the saddle?\n"},{"varval":"sm_nogap Strategy 1:\n\n<div class=\"aligned\">\n\n| | |\n| --------------- | ---------------------------- |\n| Price of saddle | = $995 - (60\\% \\times 995)$ |\n| | = $995 - 597$ |\n| | = {{{correctAnswer}}} |\n\n\nsm_nogap Strategy 2:\n\n| | |\n| --------------- | ---------------------------- |\n| Price of saddle | = ($100\\% - 60\\% )\\times 995$ |\n| | = $40\\% \\times 995$ |\n| | = {{{correctAnswer}}} |\n\n</div>"}]},{"vars":[{"varval":"Prue bought a painting at 70% off the original price.\n\nThe original price was $2680.\n\nHow much did Prue pay for the painting?\n"},{"varval":"sm_nogap Strategy 1:\n\n<div class=\"aligned\">\n\n| | |\n| --------------- | ---------------------------- |\n| Price of painting | = $2680 - (70\\% \\times 2680)$ |\n| | = $2680 - 1876$ |\n| | = {{{correctAnswer}}} |\n\n\nsm_nogap Strategy 2:\n\n| | |\n| --------------- | ---------------------------- |\n| Price of painting | = ($100\\% - 70\\% )\\times 2680$ |\n| | = $30\\% \\times 2680$ |\n| | = {{{correctAnswer}}} |\n\n</div>"}]}]

  1082. Algebra, NAP-K4-CA15

    <div class="sm_mode"> {{{question}}} </div>

    [{"vars":[{"varval":"Surf Grommets charge an hourly rate for surfing lessons and add a one-off charge of \\$45 for insurance.\n\nThe overall cost $(C)$ is represented by the formula, $C = 30\\large h$ + 45, where $\\large h$ is the number of hours of lessons.\n\nMark has 11 hours of lessons with Surf Grommets.\n\nHow much does he pay?\n"},{"varval":"<div class=\"aligned\">\n\n| | |\n| ------------------------------: | ----------------------------------- |\n| Cost |= $30\\large h$ + 45 |\n| |= 30 $\\times$ 11 + 45|\n| |= {{{correctAnswer}}} |\n\n</div>"}]},{"vars":[{"varval":"An online tutoring school charges an hourly rate for tuition and adds a one-off administration charge of \\$50.\n\nThe overall cost $(C)$ is represented by the formula, $C = 25\\large h$ + 50, where $\\large h$ is the number of hours of tuition.\n\nMitch has 15 hours of online tuition.\n\nHow much does he pay?\n"},{"varval":"<div class=\"aligned\">\n\n| | |\n| ------------------------------: | ----------------------------------- |\n| Cost |= 25$\\large h$ + 50 |\n| |= 25 $\\times$ 15 + 50|\n| |= {{{correctAnswer}}} |\n\n</div>"}]},{"vars":[{"varval":"A physiotherapist charges an hourly rate for services and adds a one-off charge for remedial massage of \\$80.\n\nThe overall cost $(C)$ is represented by the formula, $C = 70\\large h$ + 80, where $\\large h$ is the number of hourly sessions taken.\n\nBrendan has 4 hours of physiotherapy sessions booked.\n\nHow much does he pay?\n"},{"varval":"<div class=\"aligned\">\n\n| | |\n| ------------------------------: | ----------------------------------- |\n| Cost |= 70$\\large h$ + 80 |\n| |= 70 $\\times$ 4 + 80|\n| |= {{{correctAnswer}}} |\n\n</div>"}]},{"vars":[{"varval":"A cat boarding house charges a daily rate for boarding and adds a one-off administration charge of \\$40.\n\nThe overall cost $(C)$ is represented by the formula, $C = 52\\large d$ + 40, where $\\large d$ is the number of days board booked.\n\nDella is booking her cat in for 7 days of boarding.\n\nHow much does she pay?\n"},{"varval":"<div class=\"aligned\">\n\n| | |\n| ------------------------------: | ----------------------------------- |\n| Cost |= 52$\\large d$ + 40 |\n| |= 52 $\\times$ 7 + 40|\n| |= {{{correctAnswer}}} |\n\n</div>"}]},{"vars":[{"varval":"A golf driving range charges a fee per bucket of balls and adds a one-off charge of \\$5 for golf clubs if you don't have your own.\n\nThe overall cost $(C)$ is represented by the formula, $C = 10\\large b$ + 5, where $\\large b$ is the number of buckets used.\n\nStuart purchases 3 buckets of balls and hires golf clubs.\n\nHow much does he pay?\n"},{"varval":"<div class=\"aligned\">\n\n| | |\n| ------------------------------: | ----------------------------------- |\n| Cost |= 10$\\large b$ + 5 |\n| |= 10 $\\times$ 3 + 5|\n| |= {{{correctAnswer}}} |\n\n</div>"}]},{"vars":[{"varval":"A wedding venue charges an hourly rate for weddings and add a one-off charge of \\$250 for cleaning.\n\nThe overall cost $(C)$ is represented by the formula, $C = 189\\large h$ + 250, where $\\large h$ is the number of hours of venue use.\n\nRomeo and Juliet have booked the venue for 5 hours.\n\nHow much will they pay?"},{"varval":"<div class=\"aligned\">\n\n| | |\n| ------------------------------: | ----------------------------------- |\n| Cost |= 189$\\large h$ + 250 |\n| |= 189 $\\times$ 5 + 250|\n| |= {{{correctAnswer}}} |\n\n</div>"}]}]

  1083. Number, NAP-G4-CA14

    <div class="sm_mode"> Riley participates in a walkathon at the local athletics track. His uncle donates \$40. His father donates \$8 per lap of the track that Riley walks. How many laps does Riley need to walk to raise \$160? </div>

    [{"vars":null}]

  1084. Number, NAP-L4-NC03

    <div class="sm_mode"> Santana is buying a guitar string. Some of the strings are on sale. Select the string that will be cheapest. </div>

    [{"vars":null}]

  1085. Number, NAP-G4-NC06

    <div class="sm_mode"> 12 people went to Sea World together in a bus. Entry into Sea World was \$44 per person and the total cost of the bus was \$120. Which expression shows the total cost of the trip for everybody? </div>

    [{"vars":null}]

  1086. Q46

    <div class="sm_mode"> {{name}} is using the {{recipe}} recipe, pictured below, to cook dinner for his {{group}}. <br> {{image}} <br>{{name}} is cooking {{meat1}} and wants it to be ready by {{time1}}. What is the latest time he can put the {{meat2}} in the oven for it to be ready on time? </div>

    [{"vars":[{"varval":"Jamie"},{"varval":"roasted pork shoulder"},{"varval":"family"},{"varval":"2.5 kilograms of pork shoulder"},{"varval":"7:45 pm"},{"varval":"3 hours"},{"varval":"pork shoulder"},{"varval":"sm_img https://teacher.smartermaths.com.au/wp-content/uploads/2020/07/Q46var1.png 380 indent3 vpad"}]},{"vars":[{"varval":"Lionel"},{"varval":"lamb roast"},{"varval":"friends"},{"varval":"4.0 kilograms of lamb roast"},{"varval":"4$\\dfrac{1}{2}$ hours"},{"varval":"8:15 pm"},{"varval":"lamb"},{"varval":"sm_img https://teacher.smartermaths.com.au/wp-content/uploads/2020/07/Q46var2.png 380 indent3 vpad"}]},{"vars":[{"varval":"Jocky"},{"varval":"roast duck"},{"varval":"relatives"},{"varval":"5.0 kilograms of roast duck"},{"varval":"12:15 pm"},{"varval":"duck"},{"varval":"$3 \\dfrac{1}{2}$ hours"},{"varval":"sm_img https://teacher.smartermaths.com.au/wp-content/uploads/2020/07/Q46var3.png 380 indent3 vpad"}]},{"vars":[{"varval":"Gary"},{"varval":"roast chicken"},{"varval":"family"},{"varval":"3.0 kilograms of roast chicken"},{"varval":"1:15 pm"},{"varval":"chicken"},{"varval":"2 hours"},{"varval":"sm_img https://teacher.smartermaths.com.au/wp-content/uploads/2020/07/Q46var4.png 380 indent3 vpad"}]},{"vars":[{"varval":"Serge"},{"varval":"roast venison"},{"varval":"friends"},{"varval":"3.5 kilograms of roast venison"},{"varval":"7:15 pm"},{"varval":"venison"},{"varval":"3 hours"},{"varval":"sm_img https://teacher.smartermaths.com.au/wp-content/uploads/2020/07/Q46var5.png 380 indent3 vpad"}]}]

  1087. #46

    <div class="sm_mode"> {{name}} bought a {{item}} that was on sale at 50% discount. {{gender}} paid \${{price1}} for the {{item}}. Which amount below is the best estimate of the original price of the {{item}}? </div>

    [{"vars":[{"varval":"Corinne"},{"varval":"dress"},{"varval":"She"},{"varval":"39\\.95"},{"varval":"40"}]},{"vars":[{"varval":"Zane"},{"varval":"soccer ball"},{"varval":"He"},{"varval":"19\\.95"},{"varval":"20"}]},{"vars":[{"varval":"Mack"},{"varval":"pair of swimming goggles"},{"varval":"He"},{"varval":"59\\.95"},{"varval":"60"}]},{"vars":[{"varval":"Susie"},{"varval":"swimsuit"},{"varval":"She"},{"varval":"69\\.95"},{"varval":"70"}]},{"vars":[{"varval":"Charlie"},{"varval":"toy dinosaur"},{"varval":"He"},{"varval":"17\\.95"},{"varval":"18"}]}]

  1088. ISSUE VAR 4 ... Solution Image: Label the point on the compass "South-east (it is currently South-west) #45 QUICK FIXES 1- VAR3 solution image ... label the compass point "South-wes (it is currently "North-west") 2- VAR4 ... I have altered this question. Please adjust SOLUTION image so that the compass points to "South-east" - this will involve redrawing the solution image so that 180 degrees semi circle is on the top - simlar to the VAR3 solution image except the compass points to the bottom right. Thx

    <div class="sm_mode"> A {{vehicle1}} is facing {{direction}}. <br> {{image}} <br>If it rotates {{rotate}}, what direction will it be facing? </div>

    [{"vars":[{"varval":"tank"},{"varval":"south"},{"varval":"sm_img https://teacher.smartermaths.com.au/wp-content/uploads/2020/08/dQ49_var1_q.svg 270 indent3 vpad"},{"varval":"225$\\degree$ clockwise"},{"varval":"sm_img https://teacher.smartermaths.com.au/wp-content/uploads/2020/08/dQ49_var1_solution.svg 220 indent3 vpad"},{"varval":"tank"}]},{"vars":[{"varval":"tank"},{"varval":"west"},{"varval":"\n\nsm_img https://teacher.smartermaths.com.au/wp-content/uploads/2020/08/dQ49_var2_q.svg 350 indent3 vpad"},{"varval":"225$\\degree$ anti-clockwise"},{"varval":"sm_img https://teacher.smartermaths.com.au/wp-content/uploads/2020/08/dQ49_var2_solution.svg 260 indent3 vpad"},{"varval":"tank"}]},{"vars":[{"varval":"robot vacuum cleaner"},{"varval":"east"},{"varval":"sm_img https://teacher.smartermaths.com.au/wp-content/uploads/2020/08/dQ49_var3_q.svg 250 indent3 vpad"},{"varval":"225$\\degree$ anti-clockwise"},{"varval":"sm_img https://teacher.smartermaths.com.au/wp-content/uploads/2020/08/dQ49_var3_solution_r.svg 270 indent3 vpad"},{"varval":"robot vacuum cleaner"}]},{"vars":[{"varval":"robot vacuum cleaner"},{"varval":"west"},{"varval":"sm_img https://teacher.smartermaths.com.au/wp-content/uploads/2020/08/dQ49_var4_q.svg 260 indent3 vpad"},{"varval":"225$\\degree$ clockwise"},{"varval":"sm_img https://teacher.smartermaths.com.au/wp-content/uploads/2020/08/dQ49_var4_solution_r.svg 270 indent3 vpad"},{"varval":"robot vacuum cleaner"}]},{"vars":[{"varval":"tank"},{"varval":"north"},{"varval":"sm_img https://teacher.smartermaths.com.au/wp-content/uploads/2020/08/dQ49_var5_q.svg 260 indent3 vpad"},{"varval":"225$\\degree$ anti-clockwise"},{"varval":"sm_img https://teacher.smartermaths.com.au/wp-content/uploads/2020/08/dQ49_var5_solution.svg 220 indent3 vpad"},{"varval":"tank"}]}]

  1089. Statistics and Probability, NAP-L4-CA21

    <div class="sm_mode"> Mr Squiggle bought 13 pencils. He bought: * one packet of 10 pencils for \$14.50, and * another packet of 3 pencils for \$5.65 What is the mean price of the 13 pencils? </div>

    [{"vars":null}]

  1090. Statistics and Probability, NAP-C4-CA09

    <div class="sm_mode"> {{{question}}} </div>

    [{"vars":[{"varval":"Luke picked bananas for 30 days. He counted the total number of bananas he picked each day and created the graph shown below.\n\n<br>\n\nsm_img //teacher.smartermaths.com.au/wp-content/uploads/2017/03/naplan-2010-9mc.png 320 indent3 vpad\n\n<br>\n\nOn how many days did Luke pick less than 145 bananas?"},{"varval":"sm_nogap Days where less than 145 bananas picked\n\n<div class=\"aligned\">\n\n>>| | \n| ----------------------- |\n|= 4 + 5 + 8|\n|= {{{correctAnswer}}} |\n\n</div>"}]},{"vars":[{"varval":"Ethel made soup for charity for 40 days. She counted the total number of litres of soup she made each day and created the graph shown below.\n\n<br>\n\nsm_img https://teacher.smartermaths.com.au/wp-content/uploads/2022/09/Stat_Prob_50112_v1a.svg 400 indent vpad\n\n<br>\n\nOn how many days did Ethel make less than 60 litres of soup?"},{"varval":"sm_nogap Days where less than 60 litres of soup made\n\n<div class=\"aligned\">\n\n>>| | \n| ----------------------- |\n|= 6 + 9 + 10|\n|= {{{correctAnswer}}} |\n\n</div>"}]},{"vars":[{"varval":"Avery delivered pamphlets for 30 days. He counted the total number of pamphlets he delivered each day and created the graph shown below.\n\n<br>\n\nsm_img https://teacher.smartermaths.com.au/wp-content/uploads/2022/09/Stat_Prob_50112_v2.svg 340 indent vpad\n\n<br>\n\nOn how many days did Avery deliver more than 150 pamphlets?"},{"varval":"sm_nogap Days where more than 150 pamphlets were delivered\n\n<div class=\"aligned\">\n\n>>| | \n| ----------------------- |\n|= 6 + 8|\n|= {{{correctAnswer}}} |\n\n</div>"}]},{"vars":[{"varval":"Abebe was training for a marathon and tracked his running over a period of 30 weeks. \n\nHe counted the total number of kilometres he ran each week and created the graph shown below.\n\n<br>\n\nsm_img https://teacher.smartermaths.com.au/wp-content/uploads/2022/09/Stat_Prob_50112_v3_1.svg 330 indent vpad\n\n<br>\n\nOn how many weeks did Abebe run less than 80 kilometres?"},{"varval":"sm_nogap Weeks where less than 80 kilometres were run\n\n<div class=\"aligned\">\n\n>>| | \n| ----------------------- |\n|= 4 + 6 + 10|\n|= {{{correctAnswer}}} |\n\n</div>"}]},{"vars":[{"varval":"Ben has logged driving hours on 35 days since getting his L-plates. \n\nHe has counted the total number of hours he has driven each of those days and created the graph shown below.\n\n<br>\n\nsm_img https://teacher.smartermaths.com.au/wp-content/uploads/2022/09/Stat_Prob_50112_v4.svg 380 indent vpad\n\n<br>\n\nOn how many days did Ben drive more than 3.5 hours?"},{"varval":"sm_nogap Days where more than 3.5 hours driven\n\n<div class=\"aligned\">\n\n>>| | \n| ----------------------- |\n|= 2 + 3 + 4 + 5|\n|= {{{correctAnswer}}} |\n\n</div>"}]},{"vars":[{"varval":"Beth knitted beanies for charity for 30 weeks. \n\nShe counted the total number of beanies she had knitted each week and created the graph shown below.\n\n<br>\n\nsm_img https://teacher.smartermaths.com.au/wp-content/uploads/2022/09/Stat_Prob_50112_v5.svg 360 indent vpad\n\n<br>\n\nOn how many weeks did Beth knit less than 5 beanies?"},{"varval":"sm_nogap Days where less than 5 beanies knitted\n\n<div class=\"aligned\">\n\n>>| | \n| ----------------------- |\n|= 6 + 7 + 8|\n|= {{{correctAnswer}}} |\n\n</div>"}]}]

  1091. Q45

    <div class="sm_mode"> {{name}} is a {{sport}} in {{where}} and is using the tidal charts pictured below to plan his next {{event}}. <br> {{image}} <br>When will the {{tide1}} over the 2 day period covered by these charts occur? </div>

    [{"vars":[{"varval":"Aaram"},{"varval":"scuba diver"},{"varval":"Port Macquarie"},{"varval":"dive"},{"varval":"sm_img https://teacher.smartermaths.com.au/wp-content/uploads/2020/07/Q45tide1.png 720 indent vpad"},{"varval":"lowest high tide"},{"varval":"High tides: 1.00 m, 1.38 m, 0.98 m, 1.46 m"},{"varval":"Lowest high tide"},{"varval":"0.98 m"}]},{"vars":[{"varval":"Zane"},{"varval":"fisherman"},{"varval":"Wollongong"},{"varval":"fishing trip"},{"varval":"sm_img https://teacher.smartermaths.com.au/wp-content/uploads/2020/07/Q45tide1.png 720 indent vpad"},{"varval":"highest low tide"},{"varval":"Low tides: 0.30 m, 0.33 m, 0.29 m"},{"varval":"Highest low tide"},{"varval":"0.33 m"}]},{"vars":[{"varval":"Caspian"},{"varval":"abalone farmer"},{"varval":"Hobart"},{"varval":"dive"},{"varval":"sm_img https://teacher.smartermaths.com.au/wp-content/uploads/2020/07/Q45tide2.png 720 indent vpad"},{"varval":"lowest high tide"},{"varval":"High tides: 1.05 m, 1.29 m, 1.04 m, 1.19 m"},{"varval":"Lowest high tide"},{"varval":"1.04 m"}]},{"vars":[{"varval":"Drake"},{"varval":"scuba diver"},{"varval":"Crescent Head"},{"varval":"dive"},{"varval":"sm_img https://teacher.smartermaths.com.au/wp-content/uploads/2020/07/Q45tide2.png 720 indent vpad"},{"varval":"highest low tide"},{"varval":"Low tides: 0.20 m, 0.30 m, 0.22 m, 0.36 m"},{"varval":"Highest low tide"},{"varval":"0.36 m"}]},{"vars":[{"varval":"Erasmus"},{"varval":"tuna fisherman"},{"varval":"Kingscliff"},{"varval":"fishing trip"},{"varval":"sm_img https://teacher.smartermaths.com.au/wp-content/uploads/2020/07/Q45tide3.png 720 indent vpad"},{"varval":"lowest high tide"},{"varval":"High tides: 1.20 m, 1.49 m, 1.24 m, 1.41 m"},{"varval":"Lowest high tide"},{"varval":"1.20 m"}]}]

  1092. Q44

    <div class="sm_mode"> A circular thermometer is pictured below. {{image}} What temperature in $\degree$C does the thermometer record? </div>

    [{"vars":[{"varval":"sm_img https://teacher.smartermaths.com.au/wp-content/uploads/2020/07/Q44var1.png 250 indent3 vpad"},{"varval":"Left-hand"},{"varval":"negative"}]},{"vars":[{"varval":"sm_img https://teacher.smartermaths.com.au/wp-content/uploads/2020/07/Q44var2.png 250 indent3 vpad"},{"varval":"Left-hand"},{"varval":"negative"}]},{"vars":[{"varval":"sm_img https://teacher.smartermaths.com.au/wp-content/uploads/2020/07/Q44var3.png 250 indent3 vpad"},{"varval":"Right-hand"},{"varval":"positive"}]},{"vars":[{"varval":"sm_img https://teacher.smartermaths.com.au/wp-content/uploads/2020/07/Q44var4.png 250 indent3 vpad"},{"varval":"Right-hand"},{"varval":"positive"}]},{"vars":[{"varval":"sm_img https://teacher.smartermaths.com.au/wp-content/uploads/2020/07/Q44var5.png 250 indent3 vpad"},{"varval":"Left-hand"},{"varval":"positive"}]}]

  1093. Q42

    <div class="sm_mode"> {{name}} receives a bottle of medicine from {{gender}} {{person}}. The dosage instructions on the label are shown below. <br> {{image}} <br>If one spoonful contains {{ml1}} mL, how much is left in the bottle after {{name}} has been taking the medicine for {{time}}? </div>

    [{"vars":[{"varval":"Renee"},{"varval":"her"},{"varval":"doctor"},{"varval":"sm_img https://teacher.smartermaths.com.au/wp-content/uploads/2020/07/Q42var1.png 180 indent3 vpad"},{"varval":"7"},{"varval":"a week"},{"varval":"2"},{"varval":"2"},{"varval":"28"},{"varval":"7"},{"varval":"196"},{"varval":"250"}]},{"vars":[{"varval":"Justine"},{"varval":"her"},{"varval":"doctor"},{"varval":"sm_img https://teacher.smartermaths.com.au/wp-content/uploads/2020/07/Q42var2.png 180 indent3 vpad"},{"varval":"4"},{"varval":"two weeks"},{"varval":"2"},{"varval":"2"},{"varval":"16"},{"varval":"14"},{"varval":"224"},{"varval":"250"}]},{"vars":[{"varval":"Jill"},{"varval":"her"},{"varval":"doctor"},{"varval":"sm_img https://teacher.smartermaths.com.au/wp-content/uploads/2020/07/Q42var3.png 180 indent3 vpad"},{"varval":"8"},{"varval":"five days"},{"varval":"3"},{"varval":"2"},{"varval":"48"},{"varval":"5"},{"varval":"240"},{"varval":"300"}]},{"vars":[{"varval":"Winona"},{"varval":"her"},{"varval":"doctor"},{"varval":"sm_img https://teacher.smartermaths.com.au/wp-content/uploads/2020/07/Q42var4.png 150 indent3 vpad"},{"varval":"4"},{"varval":"a fortnight"},{"varval":"1"},{"varval":"2"},{"varval":"8"},{"varval":"14"},{"varval":"112"},{"varval":"200"}]},{"vars":[{"varval":"Kate"},{"varval":"her"},{"varval":"doctor"},{"varval":"sm_img https://teacher.smartermaths.com.au/wp-content/uploads/2020/07/Q42var5.png 180 indent3 vpad"},{"varval":"5"},{"varval":"eight days"},{"varval":"2"},{"varval":"4"},{"varval":"40"},{"varval":"8"},{"varval":"320"},{"varval":"350"}]}]

  1094. Number, NAP-I4-CA05 SA

    <div class="sm_mode"> Olivia earned \$17.24 per hour working at a pizza store. This week she worked for $8\dfrac{1}{4}$ hours. She used the money she earned this week to buy concert tickets for herself and her friends. Each concert ticket cost \$14.15. What is the maximum number of concert tickets Olivia can buy? </div>

    [{"vars":null}]

  1095. Algebra, NAP-I4-NC08

    <div class="sm_mode"> Ali joins a gym that has a fortnightly fee of \$33. Members pay an extra \$12 per session if they want to do a special yoga class. Which expression represents Ali's fortnightly gym bill if she does $\large x$ yoga classes during the fortnight? </div>

    [{"vars":null}]

  1096. Number, NAP-A4-NC07 SA

    <div class="sm_mode"> sm_img https://teacher.smartermaths.com.au/wp-content/uploads/2017/12/nap-A4-nc07.svg 300 indent vpad How many US dollars could be bought with 50 Australian dollars using this exchange rate? </div>

    [{"vars":null}]

  1097. Number, NAP-J4-CA06

    <div class="sm_mode"> Tran is a baker and sells 100 loaves of bread at the markets. Each loaf costs him \$2.25 to make. He sells 80 loaves for \$4.95 each. In the last hour of the market, he sells the last 20 loaves for \$2 each. What is the total profit or loss of the 100 loaves? </div>

    [{"vars":null}]

  1098. Q43

    <div class="sm_mode"> A group of students were given a list of 5 {{item1}} and asked to choose their favourite. The results were recorded and graphed below. <br> {{image}} <br>What is the angle at the centre of the piechart for the sector representing students whose favourite {{item2}} is {{item3}}? </div>

    [{"vars":[{"varval":"citrus fruits"},{"varval":"sm_img https://teacher.smartermaths.com.au/wp-content/uploads/2020/07/Q43var1.svg 300 indent3 vpad"},{"varval":"citrus fruit"},{"varval":"grapefruit"},{"varval":"20%"},{"varval":"$\\dfrac{1}{5}$"}]},{"vars":[{"varval":"animals"},{"varval":"sm_img https://teacher.smartermaths.com.au/wp-content/uploads/2020/07/Q43var2.svg 300 indent3 vpad"},{"varval":"animal"},{"varval":"the kangaroo"},{"varval":"30%"},{"varval":"$\\dfrac{3}{10}$"}]},{"vars":[{"varval":"animals"},{"varval":"sm_img https://teacher.smartermaths.com.au/wp-content/uploads/2020/07/Q43var3.svg 300 indent3 vpad"},{"varval":"animal"},{"varval":"the fox"},{"varval":"10%"},{"varval":"$\\dfrac{1}{10}$"}]},{"vars":[{"varval":"birds"},{"varval":"sm_img https://teacher.smartermaths.com.au/wp-content/uploads/2020/07/Q43var4.svg 300 indent3 vpad"},{"varval":"bird"},{"varval":"the Myna"},{"varval":"5%"},{"varval":"$\\dfrac{1}{20}$"}]},{"vars":[{"varval":"birds"},{"varval":"sm_img https://teacher.smartermaths.com.au/wp-content/uploads/2020/07/Q43var5.svg 300 indent3 vpad"},{"varval":"bird"},{"varval":"the seagull"},{"varval":"20%"},{"varval":"$\\dfrac{1}{5}$"}]}]

  1099. Number, NAP-G4-NC08

    <div class="sm_mode"> Charlie has \$20 to buy a can of soda and a chocolate bar from the shop. <br> sm_img https://teacher.smartermaths.com.au/wp-content/uploads/2016/12/NAP-149-NC08.png 250 indent3 vpad <br>How much change should Charlie be given? </div>

    [{"vars":null}]

  1100. <div class="sm_mode"> {{{question}}} </div>

    [{"vars":[{"varval":"Harmony makes four tie-dye shirts for a total cost of $120.\n\nShe sells two of the shirts for $30 each and another for $45.\r\n\r\nFor what price must she sell the fourth shirt to achieve a profit?"},{"varval":"\nTotal cost = $120 \n\n\nSales = 2 $\\times$ 30 + 45 = $105\n\n\n\nProfit occurs when sales price > cost \n\n\n\n∴ Sales price of 4th shirt must be {{{correctAnswer}}}"}]},{"vars":[{"varval":"Elroy is selling 30 kilograms of organic bananas at the market that cost him $6 per kilogram to grow.\n\nHe sells the first 10 kilograms for $90 and the second 10 kilograms for $70.\n\nWhat price must he sell the last 10 kilograms to achieve a profit? "},{"varval":"Total cost = 30 $\\times$ 6 = $180 \n\nSales = 10 $\\times$ 9 + 10 $\\times$ 7 = $160\n\nProfit occurs when sales price > cost \n\n∴ Sales price of the last 10 kilograms must be {{{correctAnswer}}}"}]},{"vars":[{"varval":"Kolin is selling 4 wooden chopping boards at the market that cost him $100 to make, in total.\n\nHe sells the first two chopping boards for $45 each and the third for $25.\n\nWhat price must he sell the 4th chopping board to achieve a profit? "},{"varval":"Total cost = $100 \n\nSales = 2 $\\times$ 45 + 25 = $115\n\nProfit occurs when sales price > cost \n\n∴ {{{correctAnswer}}}"}]}]

  1101. Number, NAP-E4-NC04

    <div class="sm_mode"> Max bought 5 lollipops and 3 chocolate bars from the shop. The lollipops cost 30 cents each and the chocolate bars cost \$1.20 each. How much did Max pay? </div>

    [{"vars":null}]

  1102. Number, NAP-F4-CA07

    <div class="sm_mode"> David buys a surfboard cover online that costs him $128. The postage costs involved are listed in the table below. <br> sm_img https://teacher.smartermaths.com.au/wp-content/uploads/2016/12/naplan-2013-7mc.png 450 indent vpad <br>The surfboard cover weighs 1.1 kg What does David have to pay, in total, to purchase the surfboard and have it delivered? </div>

    [{"vars":null}]

  1103. #44

    <div class="sm_mode"> A {{object}} is pictured below. <br> {{image}} The {{measure1}} of the {{object}} could be approximately equal to which of the following? </div>

    [{"vars":[{"varval":"pencil"},{"varval":"sm_img https://teacher.smartermaths.com.au/wp-content/uploads/2020/07/dQ44var1.png 150 indent3 vpad"},{"varval":"mass"},{"varval":"Mass"},{"varval":"grams"}]},{"vars":[{"varval":"drop of water"},{"varval":"sm_img https://teacher.smartermaths.com.au/wp-content/uploads/2020/07/dQ44var2.png 180 indent3 vpad"},{"varval":"volume"},{"varval":"Volume"},{"varval":"millilitres"}]},{"vars":[{"varval":"paper clip"},{"varval":"sm_img https://teacher.smartermaths.com.au/wp-content/uploads/2020/07/dQ44var3.png 120 indent3 vpad"},{"varval":"mass"},{"varval":"Mass"},{"varval":"grams"}]},{"vars":[{"varval":"house key"},{"varval":"sm_img https://teacher.smartermaths.com.au/wp-content/uploads/2020/07/dQ44var4.png 140 indent3 vpad"},{"varval":"mass"},{"varval":"Mass"},{"varval":"grams"}]},{"vars":[{"varval":"plastic ruler"},{"varval":"sm_img https://teacher.smartermaths.com.au/wp-content/uploads/2020/07/dQ44var5.png 300 indent3 vpad"},{"varval":"mass"},{"varval":"Mass"},{"varval":"grams"}]}]

  1104. #43

    <div class="sm_mode"> A {{collective}} of {{who}} were {{work}} {{what}}. In a full day's work, the mean number of {{verb}} by the {{who}} was {{number}}. Which of the following statements MUST be true? </div>

    [{"vars":[{"varval":"group"},{"varval":"seasonal workers"},{"varval":"harvesting"},{"varval":"lettuces"},{"varval":"lettuces picked"},{"varval":"246"},{"varval":"$\\dfrac{\\text{total lettuces picked}}{\\text{number of workers}}$"}]},{"vars":[{"varval":"team"},{"varval":"labourers"},{"varval":"harvesting"},{"varval":"pineapples"},{"varval":"pineapples picked"},{"varval":"196"},{"varval":"$\\dfrac{\\text{total pineapples picked}}{\\text{number of labourers}}$"}]},{"vars":[{"varval":"group"},{"varval":"workers"},{"varval":"picking"},{"varval":"avocados"},{"varval":"avocados picked"},{"varval":"512"},{"varval":"$\\dfrac{\\text{total avocados picked}}{\\text{number of workers}}$"}]},{"vars":[{"varval":"team"},{"varval":"labourers"},{"varval":"harvesting"},{"varval":"durians"},{"varval":"durians picked"},{"varval":"162"},{"varval":"$\\dfrac{\\text{total durians picked}}{\\text{number of pickers}}$"}]},{"vars":[{"varval":"team"},{"varval":"pickers"},{"varval":"harvesting"},{"varval":"watermelons"},{"varval":"watermelons picked"},{"varval":"81"},{"varval":"$\\dfrac{\\text{total watermelons picked}}{\\text{number of pickers}}$"}]}]

  1105. #42 QUICK FIX 1- Var2 solution image: dotted line should point to "NORTH-EAST" 2- Var4 solution image: arrow pointing up should be pointing to "WEST" 3- Var5 soltion image: dotted line should be pointing to "NORTH-EAST"

    <div class="sm_mode"> A map of {{name}}'s {{area}} is shown below. {{image1}} In what direction is {{{place1}}} from {{{place2}}}? </div>

    [{"vars":[{"varval":"Cam"},{"varval":"neighbourhood"},{"varval":"sm_img https://teacher.smartermaths.com.au/wp-content/uploads/2020/07/dQ42var1.png 480 indent vpad"},{"varval":"the park"},{"varval":"Cam's house"},{"varval":"sm_img https://teacher.smartermaths.com.au/wp-content/uploads/2020/07/dQ42var1-solution.png 400 indent vpad"}]},{"vars":[{"varval":"Kerryn"},{"varval":"street"},{"varval":"sm_img https://teacher.smartermaths.com.au/wp-content/uploads/2020/07/dQ42var2.png 480 indent vpad"},{"varval":"Kerryn's house"},{"varval":"the library"},{"varval":"sm_img https://teacher.smartermaths.com.au/wp-content/uploads/2020/07/dQ42var2-solution_R.png 400 indent vpad"}]},{"vars":[{"varval":"Andrea"},{"varval":"neighbourhood"},{"varval":"sm_img https://teacher.smartermaths.com.au/wp-content/uploads/2020/07/dQ42var3.png 450 indent vpad"},{"varval":"the Olympic pool"},{"varval":"Andrea's house"},{"varval":"\n\nsm_img https://teacher.smartermaths.com.au/wp-content/uploads/2020/07/dQ42var3-solution.png 350 indent vpad"}]},{"vars":[{"varval":"Fran"},{"varval":"street"},{"varval":"sm_img https://teacher.smartermaths.com.au/wp-content/uploads/2020/07/dQ42var4.png 450 indent vpad"},{"varval":"the cafe"},{"varval":"Fran's house"},{"varval":"sm_img https://teacher.smartermaths.com.au/wp-content/uploads/2020/07/dQ42var4-solution_R.png 350 indent vpad"}]},{"vars":[{"varval":"Olivia"},{"varval":"street"},{"varval":"sm_img https://teacher.smartermaths.com.au/wp-content/uploads/2020/07/dQ42var5.png 450 indent vpad"},{"varval":"the gym"},{"varval":"Olivia's house"},{"varval":"sm_img https://teacher.smartermaths.com.au/wp-content/uploads/2020/07/dQ42var5-solution_R.png 350 indent vpad"}]}]

  1106. #41 Should Variant 3's answer be 66.7% not 66.6%?

    <div class="sm_mode"> {{name}} is filling a {{number1}}-litre {{container}} with {{content}}. If {{name}} pours {{number2}} of {{content}} into the {{container}}, what percentage of the {{container}}'s full capacity remains available for more {{content}}? </div>

    [{"vars":[{"varval":"Richard"},{"varval":"1.5"},{"varval":"jug"},{"varval":"cordial"},{"varval":"1 litre"},{"varval":"0.5"},{"varval":"$\\dfrac{1}{3}$"}]},{"vars":[{"varval":"Sally-Anne"},{"varval":"15"},{"varval":"jerry can"},{"varval":"water"},{"varval":"6 litres"},{"varval":"9"},{"varval":"$\\dfrac{3}{5}$"}]},{"vars":[{"varval":"Byron"},{"varval":"6"},{"varval":"petrol can"},{"varval":"petrol"},{"varval":"4 litres"},{"varval":"2"},{"varval":"$\\dfrac{1}{3}$"}]},{"vars":[{"varval":"Kevin"},{"varval":"12"},{"varval":"jerry can"},{"varval":"water"},{"varval":"4 litres"},{"varval":"8"},{"varval":"$\\dfrac{2}{3}$"}]},{"vars":[{"varval":"Marty"},{"varval":"15"},{"varval":"container"},{"varval":"liquid chlorine"},{"varval":"12 litres"},{"varval":"3"},{"varval":"$\\dfrac{1}{5}$"}]}]

  1107. Q41 QUICK FIX Variant 2 ... question image. 2 adjustments: 1- shorten BC so the point C is at the same horizontal level as A and D 2- move D to the left so that angle ABD is slightly less than a right-angle Thx

    <div class="sm_mode"> In the picture below: * <span class="sm_katex-font">{{angle1}}</span> = {{size1}}$\degree$ <br> * <span class="sm_katex-font">{{angle2}}</span> = {{size2}}$\degree$ <br> {{image}} <br>What is the size of <span class="sm_katex-font">{{angle3}}?</span> </div>

    [{"vars":[{"varval":"$\\angle$PQR"},{"varval":"56"},{"varval":"$\\angle$RQS"},{"varval":"67"},{"varval":"sm_img https://teacher.smartermaths.com.au/wp-content/uploads/2020/07/Q41var1.svg 150 indent3 vpad"},{"varval":"$\\angle$PQS"}]},{"vars":[{"varval":"$\\angle$ABC"},{"varval":"18"},{"varval":"$\\angle$CBD"},{"varval":"66"},{"varval":"sm_img https://teacher.smartermaths.com.au/wp-content/uploads/2020/07/Q41var2r.svg 200 indent3 vpad"},{"varval":"$\\angle$ABD"}]},{"vars":[{"varval":"$\\angle$PRQ"},{"varval":"43"},{"varval":"$\\angle$PRS"},{"varval":"78"},{"varval":"sm_img https://teacher.smartermaths.com.au/wp-content/uploads/2020/07/Q41var3.svg 160 indent3 vpad"},{"varval":"$\\angle$QRS"}]},{"vars":[{"varval":"$\\angle$PQR"},{"varval":"107"},{"varval":"$\\angle$RQS"},{"varval":"27"},{"varval":"$\\angle$PQS"},{"varval":"sm_img https://teacher.smartermaths.com.au/wp-content/uploads/2020/07/Q41var4.svg 300 indent3 vpad"}]},{"vars":[{"varval":"$\\angle$PQR"},{"varval":"19"},{"varval":"$\\angle$RQS"},{"varval":"55"},{"varval":"sm_img https://teacher.smartermaths.com.au/wp-content/uploads/2020/07/Q41var5.svg 160 indent3 vpad"},{"varval":"$\\angle$PQS"}]}]

  1108. Q40

    <div class="sm_mode"> The table below records the 2019 car sales in Asia of 4 car markers. <br> <div class="outline"> > > | Car Maker | Sales in Asia | > > | --------- | ------------- | > > | {{make1}} | {{number1}} | > > | {{make2}} | {{number2}} | > > | {{make3}} | {{number3}} | > > | {{make4}} | {{number4}} | </div> <br> What was the average (mean) car sales for the four car makers during 2019? </div>

    [{"vars":[{"varval":"BMW"},{"varval":"1 152 322"},{"varval":"Ford"},{"varval":"2 901 907"},{"varval":"Toyota"},{"varval":"5 838 818"},{"varval":"Nissan"},{"varval":"2 143 805"},{"varval":"$\\dfrac{12\\ 036\\ 852}{4}$\n"},{"varval":"$\\approx \\dfrac{1.2 + 2.9 + 5.8 + 2.1}{4}$"},{"varval":"$\\approx \\dfrac{12.0}{4}$"}]},{"vars":[{"varval":"Mercedes"},{"varval":"1 234 726"},{"varval":"Toyota"},{"varval":"5 838 818"},{"varval":"Honda"},{"varval":"2 309 222"},{"varval":"Tesla"},{"varval":"641 730"},{"varval":"$\\dfrac{10\\ 024\\ 496}{4}$"},{"varval":"$\\approx \\dfrac{1.2 + 5.8 + 2.3 + 0.6}{4}$"},{"varval":"$\\approx \\dfrac{9.9}{4}$"}]},{"vars":[{"varval":"Toyota"},{"varval":"5 838 818"},{"varval":"Volkswagon"},{"varval":"5 127 382"},{"varval":"Tesla"},{"varval":"641 730"},{"varval":"Nissan"},{"varval":"4 272 578"},{"varval":"$\\dfrac{15\\ 880\\ 508}{4}$"},{"varval":"$\\approx \\dfrac{5.8 + 5.1 + 0.6 + 4.3}{4}$"},{"varval":"$\\approx \\dfrac{15.8}{4}$"}]},{"vars":[{"varval":"Nissan"},{"varval":"4 272 578"},{"varval":"Chevrolet"},{"varval":"2 392 112"},{"varval":"Mercedes"},{"varval":"1 234 726"},{"varval":"Proton"},{"varval":"207 807"},{"varval":"$\\dfrac{8\\ 107\\ 223}{4}$"},{"varval":"$\\approx \\dfrac{4.3 + 2.4 + 1.2 + 0.2}{4}$"},{"varval":"$\\approx \\dfrac{8.1}{4}$"}]},{"vars":[{"varval":"Honda"},{"varval":"2 309 222"},{"varval":"Volkswagen"},{"varval":"5 127 382"},{"varval":"Chevrolet"},{"varval":"2 392 112"},{"varval":"Kia"},{"varval":"2 068 573"},{"varval":"$\\dfrac{11\\ 897\\ 289}{4}$"},{"varval":"$\\approx \\dfrac{2.3 + 5.1 + 2.4 + 2.1}{4}$"},{"varval":"$\\approx \\dfrac{11.9}{4}$"}]}]

  1109. Q39 QUICK FIX - please make all the graph lines in red as per the original question. Thx

    <div class="sm_mode"> {{name}} runs a {{business}}. The number of {{item1}} {{gender}} has {{verb}} over the last six years is recorded in the graph shown below. <br> {{image}} <br>What was the average number of {{item2}} each year? </div>

    [{"vars":[{"varval":"Carol"},{"varval":"tiger sanctuary"},{"varval":"tiger cubs"},{"varval":"she"},{"varval":"bred"},{"varval":"cubs born"},{"varval":"sm_img https://teacher.smartermaths.com.au/wp-content/uploads/2020/07/q39var1_r.svg 440 indent3 vpad"},{"varval":"$\\dfrac{8 + 16 + 12 + 20 + 12 + 4}{6}$"},{"varval":"$\\dfrac{72}{6}$"}]},{"vars":[{"varval":"Byrne"},{"varval":"real estate agency"},{"varval":"houses"},{"varval":"he"},{"varval":"sold"},{"varval":"houses sold by Byrne"},{"varval":"sm_img https://teacher.smartermaths.com.au/wp-content/uploads/2020/07/q39var2_r.svg 440 indent3 vpad"},{"varval":"$\\dfrac{10 + 15 + 10 + 20 + 20 + 15}{6}$"},{"varval":"$\\dfrac{90}{6}$"}]},{"vars":[{"varval":"Denis"},{"varval":"ship building company"},{"varval":"yachts"},{"varval":"he"},{"varval":"built"},{"varval":"yachts built"},{"varval":"sm_img https://teacher.smartermaths.com.au/wp-content/uploads/2020/07/q39var3_r.svg 440 indent3 vpad"},{"varval":"$\\dfrac{16 + 8 + 10 + 10 + 20 + 4}{6}$"},{"varval":"$\\dfrac{68}{6}$"}]},{"vars":[{"varval":"Ben"},{"varval":"gymnasium"},{"varval":"treadmills"},{"varval":"he"},{"varval":"purchased"},{"varval":"treadmills purchased"},{"varval":"sm_img https://teacher.smartermaths.com.au/wp-content/uploads/2020/07/q39var4_r.svg 440 indent3 vpad"},{"varval":"$\\dfrac{5 + 5 + 10 + 20 + 15 + 25}{6}$"},{"varval":"$\\dfrac{80}{6}$"}]},{"vars":[{"varval":"Joe"},{"varval":"zoo"},{"varval":"tigers"},{"varval":"he"},{"varval":"bred"},{"varval":"tigers bred"},{"varval":"sm_img https://teacher.smartermaths.com.au/wp-content/uploads/2020/07/q39var5_r.svg 440 indent3 vpad"},{"varval":"$\\dfrac{25 + 20 + 25 + 15 + 10 + 20}{6}$"},{"varval":"$\\dfrac{115}{6}$"}]}]

  1110. Statistics and Probability, NAP-I4-NC04

    <div class="sm_mode"> {{{question}}} </div>

    [{"vars":[{"varval":"Patrick is choosing his school sport and wants to find the number of people who are enrolled in either a karate class or a surfing class but not both.\n\nWhich shaded region in the Venn diagrams represents the information Patrick is looking for?"},{"varval":"{{{correctAnswer}}}\n\nKarate or surfing but not both."}]},{"vars":[{"varval":"\nLeon is buying some new sneakers and wants to find the number of his friends who own both Vans and Converse sneakers.\n\nWhich shaded region in the Venn diagrams represents the information Leon is looking for?"},{"varval":"{{{correctAnswer}}}\n\nBoth Vans and Converse sneakers."}]},{"vars":[{"varval":"Joey owns an ice cream store and wants to find the number of people who like either vanilla or chocolate ice cream, including those that like both.\n\nWhich shaded region in the Venn diagrams represents the information Joey is looking for?"},{"varval":"{{{correctAnswer}}}\n\nEither vanilla or chocolate ice cream, including those that like both."}]},{"vars":[{"varval":"Dominique asks her friends if they play netball and basketball. She want to know who doesn't play either sport.\n\nWhich shaded region in the Venn diagrams represents the information Dominique is looking for?"},{"varval":"{{{correctAnswer}}}\n\nPlay neither Netball or Basketball."}]},{"vars":[{"varval":"Sharon asked her kindergarten students to raise their hands if they liked the TV show Bluey and then asked those who did not like the Wiggles to take their hand down.\n\nWhich shaded region in the Venn diagrams represents the kindergarten students who still had their hand up?"},{"varval":"{{{correctAnswer}}}\n\nLike Bluey but not the Wiggles."}]},{"vars":[{"varval":"Hannah runs a skiing competition and wants to know the total number of people competing in the moguls event only as well as those competing in both moguls and slalom.\n\nWhich shaded region in the Venn diagrams represents the information Hannah is looking for?"},{"varval":"{{{correctAnswer}}}\n\nCompeting in moguls and competing in both."}]}]

  1111. Statistics and Probability, NAP-F4-CA10

    <div class="sm_mode"> {{{question}}} </div>

    [{"vars":[{"varval":"In 2009, an estimated 1.7 million registrations were made by Australian children to participate in organised sport outside school.\n\nThe average cost of one registration was estimated at \\$392.\n\nWhat was the total amount paid for registrations in 2009?"},{"varval":"sm_nogap 1 700 000 × 392\n\n<div class=\"aligned\">\n\n>>| | \n| ----------------------- |\n|= {{{correctAnswer}}} |\n\n</div>"}]},{"vars":[{"varval":"A study shows that Australians are expected to renew 23.4 million streaming subscriptions as at the end of June 2022.\n\nThe average cost of one subscription was estimated at \\$55.\n\nWhat was the total estimated amount paid to renew subscriptions at the end of June 2022?"},{"varval":"sm_nogap 23 400 000 × 55\n\n<div class=\"aligned\">\n\n>>| | \n| ----------------------- |\n|= {{{correctAnswer}}} |\n\n</div>"}]},{"vars":[{"varval":"In 2021, an estimated 5.9 million cars were registered in NSW. \n\nThe average cost of one registration was estimated at \\$468.\n\nWhat was the total amount paid for car registrations in 2021?"},{"varval":"sm_nogap 5 900 000 × 468\n\n<div class=\"aligned\">\n\n>>| | \n| ----------------------- |\n|= {{{correctAnswer}}} |\n\n</div>"}]},{"vars":[{"varval":"In NSW in 2021, an estimated $17.7 million revenue was raised from the sale of recreational fishing licences.\n\nIf 474 517 recreational fishing licences were sold in 2021, what was the average cost of one licence?\n"},{"varval":"sm_nogap Cost of one fishing licence\n\n<div class=\"aligned\">\n\n>>| | \n| ----------------------- |\n|= $\\dfrac{17\\ 700\\ 000}{474\\ 517}$|\n|= 37.301... |\n| $\\approx$ {{{correctAnswer}}} |\n\n</div>\n"}]},{"vars":[{"varval":"A news website reported that in 2019, an estimated $325.8 million was spent by residents of the United States of America on interstate air travel.\n\nIf 1.2 million residents booked interstate flights, what was the average cost of interstate air travel per person?"},{"varval":"sm_nogap Cost of interstate air travel per person\n\n<div class=\"aligned\">\n\n>>| | \n| ----------------------- |\n|= $\\dfrac{325\\ 800\\ 000}{1\\ 200\\ 000}$|\n| = {{{correctAnswer}}} |\n\n</div>\n"}]},{"vars":[{"varval":"In 2018, an estimated $157.3 million was received by a union as fees from its members.\n\nIf there were 274 956 registered fee paying members of the union, what was the average annual contribution per member?"},{"varval":"sm_nogap Annual contribution per union member\n\n<div class=\"aligned\">\n\n>>| | \n| ----------------------- |\n|= $\\dfrac{157\\ 300\\ 000}{274\\ 956}$|\n|= 572.0915....|\n| $\\approx$ {{{correctAnswer}}} |\n\n</div>"}]}]

  1112. Statistics and Probability, NAP-E4-NC08

    <div class="sm_mode"> {{{question}}} </div>

    [{"vars":[{"varval":"This graph shows the price per tonne of resources in a country.\n\n<br>\n\nsm_img //teacher.smartermaths.com.au/wp-content/uploads/2017/01/NAP-E4-NC081.png 640 indent vpad\n\n<br>\n\nThe price of coal was greater than the price of iron ore for a number of quarters, as shown by the graph.\n\nIn one of those quarters, the price of aluminium scrap metal fell below \\$60 per tonne.\n\nWhich quarter was this?"},{"varval":"{{{correctAnswer}}}"}]},{"vars":[{"varval":"This graph shows the price per tonne of resources in a country.\n\n<br>\n\nsm_img //teacher.smartermaths.com.au/wp-content/uploads/2017/01/NAP-E4-NC081.png 640 indent vpad\n\n<br>\n\nOn three occasions the price of coal was equal to the price of iron ore, as shown by the graph.\n\nOn which one of these occasions, was the price of aluminium scrap metal equal to approximately \\$62 per tonne?\n\n"},{"varval":"{{{correctAnswer}}}"}]}]

  1113. Statistics and Probability, NAP-H4-NC05

    <div class="sm_mode"> {{{question}}} </div>

    [{"vars":[{"varval":"The progression of the world pole vault record is graphed below.\n\n<br>\n\nsm_img //teacher.smartermaths.com.au/wp-content/uploads/2016/12/NAP-169-NC05_1.png 465 indent vpad\n\n<br>\n\nBy about how many centimetres did the world record increase between 1985 and 2005?"},{"varval":"<div class=\"aligned\">\n\n| | |\n| --------------------- | -------------------------------------------- |\n| $\\text{Increase}$ | = 6.05 m $−$ 5.84 m |\n| | = 605 cm $-$ 584 cm |\n| | = {{correctAnswer}} cm |\n\n</div>"}]},{"vars":[{"varval":"The progression of the world men's 100 metres sprinting record is graphed below.\n\n<br>\n\nsm_img https://teacher.smartermaths.com.au/wp-content/uploads/2022/09/Stat_Prob_50108_v1.svg 750 indent vpad\n\n<br>\n\nBy about how many seconds did the world record decrease between 1960 and the end of 1987?"},{"varval":"<div class=\"aligned\">\n\n| | |\n| --------------------- | -------------------------------------------- |\n| $\\text{Decrease}$ | = 9.93 seconds $−$ 9.58 seconds |\n| | = {{correctAnswer}} seconds |\n\n</div>"}]},{"vars":[{"varval":"The graph below shows the progression of Johan's heart rate, in beats per minute (bpm), at the end of each phase of a cardio workout.\n\n<br>\n\nsm_img https://teacher.smartermaths.com.au/wp-content/uploads/2022/09/Stat_Prob_50108_v2.svg 600 indent vpad\n\n<br>\n\nBy approximately how many beats per minute did Johan's heart rate increase between the start and the end of Circuit 7?"},{"varval":"<div class=\"aligned\">\n\n| | |\n| --------------------- | -------------------------------------------- |\n| $\\text{Increase}$ | = 174 bpm $−$ 54 bpm |\n| | = {{correctAnswer}} bpm |\n\n</div>"}]},{"vars":[{"varval":"The progression of the women's world triple jump record is graphed below\n\n<br>\n\nsm_img https://teacher.smartermaths.com.au/wp-content/uploads/2022/09/Stat_Prob_50108_v3.svg 700 indent vpad\n\n<br>\n\nBy approximately how many centimetres did the world record increase between the start of 1984 and the end 1995?"},{"varval":"<div class=\"aligned\">\n\n| | |\n| --------------------- | -------------------------------------------- |\n| $\\text{Increase}$ | = 1550 cm $−$ 1300 cm |\n| | = {{correctAnswer}} |\n\n</div>"}]},{"vars":[{"varval":"The progression of the women's short course 100 metres freestyle record is graphed below.\n\n<br>\n\nsm_img https://teacher.smartermaths.com.au/wp-content/uploads/2022/09/Stat_Prob_50108_v4.svg 560 indent vpad\n\n<br>\n\nBy about how many seconds did the world record decrease between 1994 and 2018?"},{"varval":"<div class=\"aligned\">\n\n| | |\n| --------------------- | -------------------------------------------- |\n| $\\text{Increase}$ | $\\approx$ 53.00 seconds $−$ 50.25 seconds |\n| | $\\approx$ {{correctAnswer}} |\n\n</div>"}]},{"vars":[{"varval":"The graph below shows the progression of Rumour's 10 kilometre running time, in minutes, over a period of 12 months.\n\n<br>\n\nsm_img https://teacher.smartermaths.com.au/wp-content/uploads/2022/09/Stat_Prob_50108_v5_1.svg 500 indent vpad\n\n<br>\n\nBy approximately how many minutes and seconds did Rumour's running time decrease from January to December?"},{"varval":"<div class=\"aligned\">\n\n| | |\n| --------------------- | -------------------------------------------- |\n| $\\text{Decrease}$ | = 7.75 minutes $−$ 4.5 minutes |\n| | = 3.25 minutes |\n| | = {{correctAnswer}} |\n\n</div>"}]}]

  1114. Statistics and Probability, NAP-K4-CA03

    <div class="sm_mode"> {{{question}}} </div>

    [{"vars":[{"varval":"Five students do a standing long jump at their athletics carnival and the length of their jumps, in centimetres, are recorded in the table below.\n\n<br>\n\n<div class=\"sm-table col1-color7\">\n\n>>| Student | Rob| Mary | Stew | Lenny | Kath |\n|:-:|:-:|:-:|:-:|:-:|:-:|\n| Distance (cm) | 70|75|66|95|50|\n\n</div>\n\n\n\n<br>\n\nIf Kath's distance is removed from the data, what happens to the mean distance that is jumped from this group?"},{"varval":"The mean increases because the shortest distance is removed from the data set.\n"}]},{"vars":[{"varval":"Five competitors throw the discus at their athletics carnival and the length of their throws, in metres, are recorded in the table below.\n\n<br>\n\n<div class=\"sm-table col1-color1\">\n\n>>| Competitor | Bill| Tracey | Allen | Kelvin | Jen |\n|:-:|:-:|:-:|:-:|:-:|:-:|\n| Distance (m) | 9.7|10.2|10.9|12.1|9.6|\n\n</div>\n\n\n\n<br>\n\nIf Kelvin's distance is removed from the data, what happens to the mean distance that is thrown from this group?"},{"varval":"The mean decreases because the longest distance is removed from the data set.\n"}]},{"vars":[{"varval":"Five friends complete a fun run on the weekend and their times for the 5 kilometre event, in minutes and seconds, are recorded in the table below.\n\n<br>\n\n<div class=\"sm-table col1-color2\">\n\n>>| Name | John| Carrie | Beau | Zim | Kelly |\n|:-:|:-:|:-:|:-:|:-:|:-:|\n| Times | 25 m 35 s|24 m 15 s|21 m 08 s|19 m 46 s|31 m 18 s|\n\n</div>\n\n\n\n<br>\n\nIf Kelly's time is removed from the data, what happens to the median time that is run by this group?"},{"varval":"Scores in order: 19 m 46 s, 21 m 8 s, 24 m 15 s, 25 m 35s , 31 m 18 s\n\nThe median was originally 24 m 15 s and the new median, with the highest score removed, is the average of 21 m 08 s and 24 m 15 s which will be less than the original.\n\nTherefore, the median decreases because the 5th score is removed from the data set. \n\n"}]},{"vars":[{"varval":"Five pythons at the Reptile Park are measured and their lengths, in centimetres, are recorded in the table below.\n\n<br>\n\n<div class=\"sm-table col1-color3\">\n\n>>| Python| Burmese| Indian | Sumatran | Borneo | Angolan |\n|:-:|:-:|:-:|:-:|:-:|:-:|\n| Length (cm) | 270|325|166|128|183|\n\n</div>\n\n\n\n<br>\n\nIf the Sumatran Python is removed from the data, what happens to the median length of the pythons in this group?"},{"varval":"Scores in order: 128, 166, 183, 270, 325\n\nThe median was originally 183 cm and the new median is the average of 183 cm and 270 cm which will be more than the original.\n\nTherefore, the median increases because the 2nd shortest length is removed from the data set. \n"}]},{"vars":[{"varval":"Dan caught 5 fish on the weekend and their weights, in kilograms, are recorded in the table below.\n\n<br>\n\n<div class=\"sm-table col1-color8\">\n\n>>| Fish | Bream 1| Bream 2 | Kingfish | Flathead | Snapper |\n|:-:|:-:|:-:|:-:|:-:|:-:|\n| Weight (kg) | 1.1 | 1.4 | 5.6 | 2.2 | 1.8 |\n\n</div>\n\n\n\n<br>\n\nIf Bream 2 is removed from the data, what happens to the range of weights for this group?"},{"varval":"Original range &nbsp;5.6 $-$ 1.1 = 4.5 \n\nRange after Bream 2 weight removed &nbsp;5.6 $-$ 1.1 = 4.5.\n\nTherefore, the range stays the same if Bream 2 is removed from the data set.\n\n"}]},{"vars":[{"varval":"Five people measure their resting heart rate and their results, in beats per minute (BPM), are recorded in the table below.\n\n<br>\n\n<div class=\"sm-table col1-color1\">\n\n>>| Name | Cliff| Carina | Cathy | Clive | Corinne |\n|:-:|:-:|:-:|:-:|:-:|:-:|\n| Resting Heart Rate (BPM) | 72|55|79|45|64|\n\n</div>\n\n\n\n<br>\n\nIf Cathy's resting heart rate is removed from the data, what happens to the range of heart rates for this group?"},{"varval":"Original range &nbsp;79 $-$ 45 = 34 \n\nRange after Cathy's data point is removed &nbsp;72 $-$ 45 = 27.\n\nTherefore, the range decreases if Cathy's resting heart rate measure is removed from the data set."}]}]

  1115. Number, NAP-F4-CA04

    <div class="sm_mode"> {{{question}}} </div>

    [{"vars":[{"varval":"The first three days of the Brisbane cricket test had the following attendances:\n\n<div class=\"sm-table row1-color1 heading-color1\">\n\n>>| Day | Crowd Size |\n|:-:|:-:|\n| 1 | 20 156 |\n| 2 | 18 397 |\n| 3 | 29 981 |\n\n</div>\n\n<br>\n\nWhat was the total crowd over the first 3 days, to the nearest 1000?"},{"varval":"sm_nogap 20 156 + 18 397 + 29 981\n\n<div class=\"aligned\">\n\n>>| | \n| ----------------------- |\n|= 68 534|\n|= {{{correctAnswer}}} (nearest 1000) |\n\n</div>"}]},{"vars":[{"varval":"The first three days of the Ashes cricket test in Tasmania had the following attendances:\n\n<div class=\"sm-table row1-color4 heading-color4\">\n\n>>| Day | Crowd Size |\n|:-:|:-:|\n| 1 | 9 002 |\n| 2 | 8 711 |\n| 3 | 8 088 |\n\n</div>\n\n<br>\n\nWhat was the total crowd over the first 3 days, to the nearest 1000?"},{"varval":"sm_nogap 9 002 + 8 711 + 8 088\n\n<div class=\"aligned\">\n\n>>| | \n| ----------------------- |\n|= 25 801|\n|= {{{correctAnswer}}} (nearest 1000) |\n\n</div>"}]},{"vars":[{"varval":"The last three Sydney Swans AFL games had the following attendances:\n\n<div class=\"sm-table row1-color2 heading-color2\">\n\n>>| Game | Crowd Size |\n|:-:|:-:|\n| 8/7/22 | 26 226 |\n| 2/7/22 | 43 163 |\n| 25/6/22 | 31 513 |\n\n</div>\n\n<br>\n\nWhat was the total crowd over the last 3 Swans games, to the nearest 1000?"},{"varval":"sm_nogap 26 226 + 43 163 + 31 513\n\n<div class=\"aligned\">\n\n>>| | \n| ----------------------- |\n|= 100 902|\n|= {{{correctAnswer}}} (nearest 1000) |\n\n</div>"}]},{"vars":[{"varval":"The 2018 State of Origin Rugby League matches had the following attendances:\n\n<div class=\"sm-table row1-color3 heading-color3\">\n\n>>| Game | Crowd Size |\n|:-:|:-:|\n| I | 87 122 |\n| II | 82 223 |\n| III | 59 358 |\n\n</div>\n\n<br>\n\nWhat was the total crowd over the 3 game series, to the nearest 1000?"},{"varval":"sm_nogap 87 122 + 82 223 + 59 358\n\n<div class=\"aligned\">\n\n>>| | \n| ----------------------- |\n|= 228 703|\n|= {{{correctAnswer}}} (nearest 1000) |\n\n</div>"}]},{"vars":[{"varval":"The table shows the total rainfall, in millimetres, that fell on Sydney during the three years from 2019 to 2021.\n\n<div class=\"sm-table row1-color6 heading-color6\">\n\n>>| Year | Annual Rainfall (mm) |\n|:-:|:-:|\n| 2019 | 852 |\n| 2020 | 1223 |\n| 2021 | 1190 |\n\n</div>\n\n<br>\n\nWhat was the total rainfall over the 3 year period, to the nearest hundred?"},{"varval":"sm_nogap 852 + 1223 + 1190\n\n<div class=\"aligned\">\n\n>>| | \n| ----------------------- |\n|= 3 265|\n|= {{{correctAnswer}}} (nearest 100) |\n\n</div>"}]},{"vars":[{"varval":"The table shows the number of new jobs created in Australia each year from 2017 to 2019.\n\n<div class=\"sm-table row1-color2 heading-color2\">\n\n>>| Year | Number of new jobs|\n|:-:|:-:|\n| 2017 | 2 931 553 |\n| 2018 | 2 995 979 |\n| 2019 | 3 098 896 |\n\n</div>\n\n<br>\n\nWhat was the total number of new jobs created over the 3 year period, to the nearest ten thousand?"},{"varval":"sm_nogap 2 931 553 + 2 995 979 + 3 098 896\n\n<div class=\"aligned\">\n\n>>| | \n| ----------------------- |\n|= 9 026 428|\n|= {{{correctAnswer}}} (nearest 10 000) |\n\n</div>"}]}]

  1116. Q38

    <div class="sm_mode"> {{name}} produces {{product}} that he sells online. The cost of producing different quantities of {{product}} is graphed below. <br> {{image}} <br>Which equation below can be used to calculate {{name}}'s cost of production. </div>

    [{"vars":[{"varval":"Michael"},{"varval":"dog bowls"},{"varval":"sm_img https://teacher.smartermaths.com.au/wp-content/uploads/2020/07/Q38_var1.svg 470 indent3 vpad"},{"varval":"\\$5"},{"varval":"\\$5"},{"varval":"bowl"}]},{"vars":[{"varval":"Chez"},{"varval":"bandanas"},{"varval":"sm_img https://teacher.smartermaths.com.au/wp-content/uploads/2020/07/Q38_var2.svg 540 indent3 vpad"},{"varval":"\\$3"},{"varval":"\\$3"},{"varval":"bandana"}]},{"vars":[{"varval":"Mick"},{"varval":"dog collars"},{"varval":"sm_img https://teacher.smartermaths.com.au/wp-content/uploads/2020/07/Q38_var3.svg 510 indent3 vpad"},{"varval":"\\$40"},{"varval":"\\$20"},{"varval":"collar"}]},{"vars":[{"varval":"Kilgour"},{"varval":"model dinosaurs"},{"varval":"sm_img https://teacher.smartermaths.com.au/wp-content/uploads/2020/07/Q38_var4.svg 450 indent3 vpad"},{"varval":"\\$20"},{"varval":"\\$10"},{"varval":"dinosaur"}]},{"vars":[{"varval":"Lance"},{"varval":"costume swords"},{"varval":"sm_img https://teacher.smartermaths.com.au/wp-content/uploads/2020/07/Q38_var5.svg 450 indent3 vpad"},{"varval":"\\$10"},{"varval":"\\$5"},{"varval":"sword"}]}]

  1117. Number, NAP-L4-CA03

    <div class="sm_mode"> Candice buys an apple for 75 cents and a banana for 70 cents. She pays with a \$2 coin. How much change should Candice get? </div>

    [{"vars":null}]

  1118. Number, NAP-H4-NC02

    <div class="sm_mode"> Andreas has \$8 to buy batteries for his toy racing car. Each battery costs \$1.60 and he buys 4 batteries. Which expression shows how much money he has left? </div>

    [{"vars":null}]

  1119. Number, NAP-E4-NC01

    <div class="sm_mode"> Penny has four 20-cent pieces. Fi has three 20-cent pieces. Sophie has four 20-cent pieces. How much money do they have in total? </div>

    [{"vars":null}]

  1120. Number, NAPX-L4-CA09 v2

    <div class="sm_mode"> Elvis is reading the temperature outside his ski lodge. What is the value of the point marked **P** on this temperature scale? <br> sm_img https://teacher.smartermaths.com.au/wp-content/uploads/2019/12/nap-L4-09-ver3.svg 270 indent2 vpad <br> </div>

    [{"vars":null}]

  1121. Number, NAPX-I2-27

    <div class="sm_mode"> Curly, Larry and Moe share a bag of strawberries. Curly and Larry get $\dfrac{1}{8}$ of the strawberries each. What fraction of the bag of strawberries does Moe get? </div>

    [{"vars":null}]

  1122. Statistics and Probability, NAP-A4-CA04

    <div class="sm_mode"> This table summarises the time Tutty spent training her parrot over five days. <br> <div class="sm-table row1-color8 heading-color8"> >>Time Spent Training Parrot >>| Day | Time | |:-:|:-:| | Monday | 25 minutes | | Tuesday | 55 minutes | | Wednesday | 1 hour | | Thursday | 94 minutes | | Friday | 46 minutes | </div> <br> What was the average (mean) time for training the parrot each day? </div>

    [{"vars":null}]

  1123. Statistics and Probability, NAP-A4-CA03

    A dealership sells new and used cars. The graph shows the price of 2 similar cars and their age in years <br> sm_img https://teacher.smartermaths.com.au/wp-content/uploads/2017/12/NAP-A4-CA3_1.svg 220 indent3 vpad <br> Which one of these statements is true?

    [{"vars":null}]

  1124. Statistics and Probability, NAP-E4-CA03

    Anne taught a kindergarten class and asked her students what their favorite fruit is. Their answers were used to draw the pie chart below. <br> sm_img //teacher.smartermaths.com.au/wp-content/uploads/2017/01/naplan-2012-3mc.png 295 indent3 vpad <br> Around a quarter of her students preferred which fruit?

    [{"vars":null}]

  1125. Statistics and Probability, NAP-J4-CA03

    A survey was conducted to determine the number of children who have a pool or a trampoline at home. The results are shown below. <br> sm_img https://teacher.smartermaths.com.au/wp-content/uploads/2017/09/NAP-J1-03.png 320 indent3 vpad <br> How many children have a pool at home, but not a trampoline?

    [{"vars":null}]

  1126. Statistics and Probability, NAP-G4-CA03

    Ten of the tallest mountains in the United States are listed in the table below: <br> sm_img https://teacher.smartermaths.com.au/wp-content/uploads/2020/09/naplan-2014-3mc-2.svg 375 indent3 vpad <br> How much taller than Utah's tallest mountain is Colorado's tallest mountain?

    [{"vars":null}]

  1127. Statistics and Probability, NAP-F4-NC02

    This graph shows the number of bats in a fruit tree at 15 minute intervals over 4 hours. <br> sm_img //teacher.smartermaths.com.au/wp-content/uploads/2017/04/NAP-F4-NC02.png 482 indent3 vpad <br> At which time were the highest number of bats in the fruit tree?

    [{"vars":null}]

  1128. Statistics, NAP-L4-CA01

    This graph shows the amount of electricity generated by solar power and wind power in the Australian states. <br> sm_img https://teacher.smartermaths.com.au/wp-content/uploads/2019/11/nap-L4-ca01-ver1.svg 680 indent vpad <br> Which state generates the most electricity using wind power?

    [{"vars":null}]

  1129. Statistics and Probability, NAP-C4-NC02

    This diagram shows the proportion of milkshake flavours sold by a corner store over a 1 month period. <br> sm_img //teacher.smartermaths.com.au/wp-content/uploads/2017/01/NAP-C4-NC02.png 330 indent3 vpad <br> Which flavour makes up 55% of the milkshakes sold?

    [{"vars":null}]

  1130. A lizard enclosure contains 9 chameleons and 17 blue-tongue lizards. Tran uses blocks to make rectangular prisms. <br> sm_img https://teacher.smartermaths.com.au/wp-content/uploads/2018/07/NAP-K3-CA07.svg 225 indent3 vpad <br> How many blocks does Tran use altogether? Approximately what percentage of the lizards are blue-tongue?

    [{"vars":null}]

  1131. Number, NAPX-p117016v01

    <div class="sm_mode"> Kirk, Spock and McCoy share a box of plums. Kirk and Spock get $\dfrac{1}{7}$ of the plums each. What fraction of the box of the plums does McCoy get? </div>

    [{"vars":null}]

  1132. Number, NAPX-p122724v01

    <div class="sm_mode"> {{name}} cuts a pizza into {{number}} smaller pieces along the lines shown. <br> {{image}} <br>How many pieces are in {{frac1}} of the whole pizza? </div>

    [{"vars":[{"varval":"Gary"},{"varval":"6"},{"varval":"one-third"},{"varval":"$\\dfrac{1}{3}$"},{"varval":"sm_img https://teacher.smartermaths.com.au/wp-content/uploads/2019/01/NAPX-K2-21v3.svg 300 indent3 vpad"}]},{"vars":[{"varval":"Chezzie"},{"varval":"6"},{"varval":"two-thirds"},{"varval":"$\\dfrac{2}{3}$"},{"varval":"sm_img https://teacher.smartermaths.com.au/wp-content/uploads/2019/01/NAPX-K2-21v3.svg 300 indent3 vpad"}]},{"vars":[{"varval":"Mike"},{"varval":"8"},{"varval":"three-quarters"},{"varval":"$\\dfrac{3}{4}$"},{"varval":"sm_img https://teacher.smartermaths.com.au/wp-content/uploads/2018/12/nap-K2-21rev.svg 300 indent3 vpad"}]}]

  1133. #40

    <div class="sm_mode"> A {{factory}} produced {{pieces}} {{product1}} in {{month1}}. If its production increased by {{percent}} in {{month2}}, how many {{product1}} did it produce? </div>

    [{"vars":[{"varval":"sawmill"},{"varval":"200 000"},{"varval":"timber planks"},{"varval":"September"},{"varval":"30%"},{"varval":"October"},{"varval":"$\\dfrac{3}{10}$"},{"varval":"60 000"},{"varval":"Planks"}]},{"vars":[{"varval":"factory"},{"varval":"400 000"},{"varval":"silicon chips"},{"varval":"January"},{"varval":"20%"},{"varval":"February"},{"varval":"$\\dfrac{2}{10}$"},{"varval":"80 000"},{"varval":"Silicon chips"}]},{"vars":[{"varval":"factory"},{"varval":"200 000"},{"varval":"face masks"},{"varval":"April"},{"varval":"40%"},{"varval":"May"},{"varval":"$\\dfrac{4}{10}$"},{"varval":"80 000"},{"varval":"Face masks"}]},{"vars":[{"varval":"sawmill"},{"varval":"60 000"},{"varval":"chair legs"},{"varval":"November"},{"varval":"10%"},{"varval":"December"},{"varval":"$\\dfrac{1}{10}$"},{"varval":"6000"},{"varval":"Chair legs"}]},{"vars":[{"varval":"pharmaceutical factory"},{"varval":"300 000"},{"varval":"testing swabs"},{"varval":"May"},{"varval":"30%"},{"varval":"June"},{"varval":"$\\dfrac{3}{10}$"},{"varval":"90 000"},{"varval":"Swabs"}]}]

  1134. #39

    <div class="sm_mode"> A given rectangle has one side that is {{frac}} the length of another side. If the {{which1}} side is {{dist1}} centimetres, what is the {{pv1}} of the rectangle? </div>

    [{"vars":[{"varval":"half"},{"varval":"long"},{"varval":"8"},{"varval":"area"},{"varval":"Long"},{"varval":"Short"},{"varval":"4"},{"varval":"Area"},{"varval":"8 $\\times$ 4"}]},{"vars":[{"varval":"three times"},{"varval":"long"},{"varval":"12"},{"varval":"perimeter"},{"varval":"Long"},{"varval":"Short"},{"varval":"4"},{"varval":"Perimeter"},{"varval":"2 $\\times$ (12 + 4)"}]},{"vars":[{"varval":"half"},{"varval":"short"},{"varval":"8"},{"varval":"perimeter"},{"varval":"Short"},{"varval":"Long"},{"varval":"16"},{"varval":"Perimeter"},{"varval":"2 $\\times$ (8 + 16)"}]},{"vars":[{"varval":"twice"},{"varval":"long"},{"varval":"6"},{"varval":"area"},{"varval":"Long"},{"varval":"Short"},{"varval":"3"},{"varval":"Area"},{"varval":"6 $\\times$ 3"}]},{"vars":[{"varval":"one-third"},{"varval":"short"},{"varval":"9"},{"varval":"perimeter"},{"varval":"Short"},{"varval":"Long"},{"varval":"27"},{"varval":"Perimeter"},{"varval":"2 $\\times$ (9 + 27)"}]}]

  1135. Q33

    Temperatures read from a weather thermometer are recorded each day. What temperature, in degrees {{type1}}, would be read from the thermometer below? <br> {{image}}

    [{"vars":[{"varval":"Celsius"},{"varval":"sm_img https://teacher.smartermaths.com.au/wp-content/uploads/2020/07/q33var1_r.png 120 indent3 vpad"},{"varval":"below"}]},{"vars":[{"varval":"Celsius"},{"varval":"sm_img https://teacher.smartermaths.com.au/wp-content/uploads/2020/07/q33var2_r.png 120 indent3 vpad"},{"varval":"below"}]},{"vars":[{"varval":"Celsius"},{"varval":"sm_img https://teacher.smartermaths.com.au/wp-content/uploads/2020/07/q33var3_r.png 120 indent3 vpad"},{"varval":"below"}]},{"vars":[{"varval":"Celsius"},{"varval":"sm_img https://teacher.smartermaths.com.au/wp-content/uploads/2020/07/q33var4_r.png 120 indent3 vpad"},{"varval":"above"}]},{"vars":[{"varval":"Celsius"},{"varval":"sm_img https://teacher.smartermaths.com.au/wp-content/uploads/2020/07/q33var5_r.png 120 indent3 vpad"},{"varval":"below"}]}]

  1136. Q36

    <div class="sm_mode"> {{name}} designs {{object}} ramps. A plan of {{gender}} latest design is shown below. <br> {{image}} <br>The size of the ramp angle maked $\large \alpha \degree$ is closest to </div>

    [{"vars":[{"varval":"Sabre"},{"varval":"skateboard"},{"varval":"her"},{"varval":"sm_img https://teacher.smartermaths.com.au/wp-content/uploads/2020/07/q36variant1.svg 370 indent3 vpad"}]},{"vars":[{"varval":"Poppy"},{"varval":"skateboard"},{"varval":"her"},{"varval":"\n\nsm_img https://teacher.smartermaths.com.au/wp-content/uploads/2020/07/q36variant2.svg 220 indent3 vpad"}]},{"vars":[{"varval":"Aiden"},{"varval":"mountain bike"},{"varval":"his"},{"varval":"sm_img https://teacher.smartermaths.com.au/wp-content/uploads/2020/07/q36variant3.svg 220 indent3 vpad"}]},{"vars":[{"varval":"Elijah"},{"varval":"ski"},{"varval":"his"},{"varval":"sm_img https://teacher.smartermaths.com.au/wp-content/uploads/2020/07/q36variant4.svg 360 indent3 vpad"}]},{"vars":[{"varval":"Starr"},{"varval":"skateboard"},{"varval":"her"},{"varval":"sm_img https://teacher.smartermaths.com.au/wp-content/uploads/2020/07/q36variant5.svg 250 indent3 vpad"}]}]

  1137. Q35 QUICK FIX Myca, can you include the full number of articles in each attachment. The questions also test the students in making sure they pick out the correct items. This means you should only need one image per question. Thx

    <div class="sm_mode"> {{name}} purchased one {{item1}} and two {{item2}} at the sale price. {{image}} Approximately how much did {{name}} save compared to the normal price for his three items? </div>

    [{"vars":[{"varval":"Raphael"},{"varval":"Fila PLR Polo"},{"varval":"Babolat UK Country Tees"},{"varval":"polo"},{"varval":"58.00"},{"varval":"43.50"},{"varval":"14.50"},{"varval":"tee"},{"varval":"34.00"},{"varval":"27.20"},{"varval":"6.80"},{"varval":"28.10"},{"varval":"sm_img https://teacher.smartermaths.com.au/wp-content/uploads/2020/07/Q35_var1-r.png 900 indent vpad"}]},{"vars":[{"varval":"Lola"},{"varval":"Fur Hood Parka"},{"varval":"Hooded Gillets"},{"varval":"parka"},{"varval":"105.00"},{"varval":"55.00"},{"varval":"50.00"},{"varval":"gillet"},{"varval":"75.00"},{"varval":"39.20"},{"varval":"35.80"},{"varval":"121.60"},{"varval":"sm_img https://teacher.smartermaths.com.au/wp-content/uploads/2020/07/Q35_var2-r.png 900 indent vpad"}]},{"vars":[{"varval":"Aly"},{"varval":"pair of Otik Workboots"},{"varval":"pairs of Ariana Buckle Boots"},{"varval":"workboots"},{"varval":"79.99"},{"varval":"39.98"},{"varval":"40.01"},{"varval":"buckle boots"},{"varval":"49.99"},{"varval":"24.98"},{"varval":"25.01"},{"varval":"90.03"},{"varval":"sm_img https://teacher.smartermaths.com.au/wp-content/uploads/2020/07/Q35_var3-r.png 900 indent vpad"}]},{"vars":[{"varval":"Rudolf"},{"varval":"Asics Travel Polo"},{"varval":"pair of Asics Shorts"},{"varval":"polo"},{"varval":"85.00"},{"varval":"59.50"},{"varval":"25.50"},{"varval":"shorts"},{"varval":"35.00"},{"varval":"22.00"},{"varval":"13.00"},{"varval":"51.50"},{"varval":"sm_img https://teacher.smartermaths.com.au/wp-content/uploads/2020/07/Q35_var4-r.png 900 indent vpad"}]},{"vars":[{"varval":"Anton"},{"varval":"pair of Adidas gloves"},{"varval":"pairs of Adidas shin guards"},{"varval":"gloves"},{"varval":"25.00"},{"varval":"17.50"},{"varval":"7.50"},{"varval":"shin guards"},{"varval":"20.00"},{"varval":"14.00"},{"varval":"6.00"},{"varval":"19.50"},{"varval":"sm_img https://teacher.smartermaths.com.au/wp-content/uploads/2020/07/Q35_var5-r.png 900 indent vpad"}]}]

  1138. Q34

    <div class="sm_mode"> {{name}}'s credit card gives {{gender}} a breakdown of {{gender}} credit card purchases, by category, every month. The pie chart below shows {{gender}} expenditure in June. <br> {{image}} <br>What percentage of {{gender}} total credit card bill did {{name}} spend on {{category1}} in June? </div>

    [{"vars":[{"varval":"Penelope"},{"varval":"her"},{"varval":"sm_img https://teacher.smartermaths.com.au/wp-content/uploads/2020/07/q34var1.svg 365 indent3 vpad"},{"varval":"Food"},{"varval":"Clothes"},{"varval":"25%"},{"varval":"$\\dfrac{1}{3}$"}]},{"vars":[{"varval":"Clarissa"},{"varval":"her"},{"varval":"sm_img https://teacher.smartermaths.com.au/wp-content/uploads/2020/07/q34var2.svg 365 indent3 vpad"},{"varval":"Clothes"},{"varval":"Transport"},{"varval":"25%"},{"varval":"$\\dfrac{2}{3}$"}]},{"vars":[{"varval":"Kate"},{"varval":"her"},{"varval":"sm_img https://teacher.smartermaths.com.au/wp-content/uploads/2020/07/q34var3.svg 365 indent vpad"},{"varval":"Pets"},{"varval":"Food"},{"varval":"33%"},{"varval":"$\\dfrac{1}{2}$"}]},{"vars":[{"varval":"Gloria"},{"varval":"her"},{"varval":"sm_img https://teacher.smartermaths.com.au/wp-content/uploads/2020/07/q34var4.svg 365 indent3 vpad"},{"varval":"Fuel"},{"varval":"Pets and Rent"},{"varval":"50%"},{"varval":"$\\dfrac{1}{3}$"}]},{"vars":[{"varval":"Jane"},{"varval":"her"},{"varval":"sm_img https://teacher.smartermaths.com.au/wp-content/uploads/2020/07/q34var5.svg 365 indent3 vpad"},{"varval":"Gifts"},{"varval":"Vet"},{"varval":"25%"},{"varval":"$\\dfrac{1}{2}$"}]}]

  1139. #38

    <div class="sm_mode"> {{1}} $\div$ {{2}} = <span class="sm-text">?</span> </div>

    [{"vars":[{"varval":"0.324"},{"varval":"8"},{"varval":"$\\begin{array}{ccccccccccc} \\ \\ \\ \\ \\underset{\\text{\\ \\ \\ \\ \\ ----------------}}{\\ \\ \\ \\ \\ \\ \\ 0.0405\\ \\ \\ \\ } \\\\ 8\\ \\text{\\textbar} 0.324 \\\\ \\ \\ \\ \\ \\underset{\\text{------------}}{0.320} \\\\ \\ \\ \\ \\ \\ \\ \\ {0.0040} \\\\ \\ \\ \\ \\ \\ \\ \\ \\underset{\\text{--------------}}{0.0040} \\\\ \\ \\ \\ \\ \\ \\ \\ \\ \\ \\ \\ \\ \\ \\ \\ \\ {0} \\\\ \\end{array}$\n"}]},{"vars":[{"varval":"0.102"},{"varval":"5"},{"varval":"$\\begin{array}{ccccccccccc}\n\\ \\ \\ \\ \\underset{\\text{\\ \\ \\ \\ \\ ----------------}}{\\ \\ \\ \\ \\ \\ \\ 0.0204\\ \\ \\ \\ } \\\\\n5\\ \\text{\\textbar} 0.102 \\\\\n\\ \\ \\ \\ \\underset{\\text{------------}}{0.100} \\\\\n\\ \\ \\ \\ \\ \\ \\ {0.0020} \\\\\n\\ \\ \\ \\ \\ \\ \\ \\underset{\\text{--------------}}{0.0020} \\\\\n\\ \\ \\ \\ \\ \\ \\ \\ \\ \\ \\ \\ \\ \\ \\ \\ {0} \\\\\n\\end{array}$\n"}]},{"vars":[{"varval":"0.543"},{"varval":"6"},{"varval":"$\\begin{array}{ccccccccccc}\n\\ \\ \\ \\underset{\\text{\\ \\ \\ \\ \\ ----------------}}{\\ \\ \\ \\ \\ \\ \\ 0.0905\\ \\ \\ } \\\\\n6\\ \\text{\\textbar} 0.543 \\\\\n\\ \\ \\ \\ \\underset{\\text{------------}}{0.540} \\\\\n\\ \\ \\ \\ \\ \\ \\ {0.0030} \\\\\n\\ \\ \\ \\ \\ \\ \\ \\underset{\\text{--------------}}{0.0030} \\\\\n\\ \\ \\ \\ \\ \\ \\ \\ \\ \\ \\ \\ \\ \\ \\ \\ {0} \\\\\n\\end{array}$\n"}]},{"vars":[{"varval":"0.246"},{"varval":"3"},{"varval":"$\\begin{array}{ccccccccccc}\n\\ \\ \\ \\underset{\\text{\\ \\ \\ \\ \\ ----------------}}{\\ \\ \\ \\ 0.082\\ \\ \\ } \\\\\n3\\ \\text{\\textbar} 0.246 \\\\\n\\ \\ \\ \\ \\underset{\\text{------------}}{0.240} \\\\\n\\ \\ \\ \\ \\ {0.006} \\\\\n\\ \\ \\ \\ \\ \\underset{\\text{--------------}}{0.006} \\\\\n\\ \\ \\ \\ \\ \\ \\ \\ \\ \\ \\ \\ \\ \\ \\ \\ {0} \\\\\n\\end{array}$\n"}]},{"vars":[{"varval":"0.724"},{"varval":"8"},{"varval":"$\\begin{array}{ccccccccccc}\n\\ \\ \\ \\underset{\\text{\\ \\ \\ \\ \\ ----------------}}{\\ \\ \\ \\ \\ \\ \\ 0.0905\\ \\ \\ } \\\\\n8\\ \\text{\\textbar} 0.724 \\\\\n\\ \\ \\ \\ \\underset{\\text{------------}}{0.720} \\\\\n\\ \\ \\ \\ \\ \\ \\ {0.0040} \\\\\n\\ \\ \\ \\ \\ \\ \\ \\underset{\\text{--------------}}{0.0040} \\\\\n\\ \\ \\ \\ \\ \\ \\ \\ \\ \\ \\ \\ \\ \\ \\ \\ {0} \\\\\n\\end{array}$\n"}]}]

  1140. #37

    <div class="sm_mode"> The nutritional information on a {{food1}} is shown below. <br> {{image}} <br>{{name}} expends {{number1}} kJ of energy by {{sport1}} for {{time}} minutes. If she consumes {{number2}} grams of the {{food2}}, approximately how long should she {{sport2}} to use up the energy it provides? </div>

    [{"vars":[{"varval":"breakfast cereal"},{"varval":"sm_img https://teacher.smartermaths.com.au/wp-content/uploads/2020/07/NutInfo_ver1.svg 400 indent3 vpad"},{"varval":"Marjorie"},{"varval":"1000"},{"varval":"jogging"},{"varval":"30"},{"varval":"100"},{"varval":"jog"},{"varval":"1320"},{"varval":"1.3"},{"varval":"cereal"}]},{"vars":[{"varval":"breakfast cereal"},{"varval":"sm_img https://teacher.smartermaths.com.au/wp-content/uploads/2020/07/NutInfo_ver1.svg 400 indent3 vpad"},{"varval":"Anna"},{"varval":"650"},{"varval":"swimming"},{"varval":"30"},{"varval":"100"},{"varval":"swim"},{"varval":"1320"},{"varval":"2"},{"varval":"cereal"}]},{"vars":[{"varval":"power bar"},{"varval":"sm_img https://teacher.smartermaths.com.au/wp-content/uploads/2020/07/NutInfo_ver2.svg 400 indent3 vpad"},{"varval":"Sandra"},{"varval":"900"},{"varval":"running"},{"varval":"20"},{"varval":"100"},{"varval":"run"},{"varval":"1810"},{"varval":"2"},{"varval":"power bar"}]},{"vars":[{"varval":"breakfast cereal"},{"varval":"sm_img https://teacher.smartermaths.com.au/wp-content/uploads/2020/07/NutInfo_ver3.svg 400 indent3 vpad"},{"varval":"Kylie"},{"varval":"1000"},{"varval":"rowing"},{"varval":"30"},{"varval":"100"},{"varval":"row"},{"varval":"2460"},{"varval":"2.5"},{"varval":"cereal"}]},{"vars":[{"varval":"packet of biscuits"},{"varval":"sm_img https://teacher.smartermaths.com.au/wp-content/uploads/2022/01/NutInfo_ver4.svg 400 indent3 vpad"},{"varval":"Yvonne"},{"varval":"1000"},{"varval":"skipping"},{"varval":"20"},{"varval":"100"},{"varval":"skip"},{"varval":"1810"},{"varval":"1.8"},{"varval":"cereal"}]}]

  1141. #36

    <div class="sm_mode"> A {{event}} has {{number1}} {{pens}} that can each hold {{number2}} {{animal1}}. If each yard is {{frac1}} full, how many {{animal1}}, in total, are for sale at the auction? </div>

    [{"vars":[{"varval":"cattle auction"},{"varval":"90"},{"varval":"holding pens"},{"varval":"24"},{"varval":"cattle"},{"varval":"one-third"},{"varval":"Cattle"},{"varval":"$\\dfrac{1}{3}$"},{"varval":"8"}]},{"vars":[{"varval":"sheep auction"},{"varval":"140"},{"varval":"holding pens"},{"varval":"12"},{"varval":"sheep"},{"varval":"one-quarter"},{"varval":"Sheep"},{"varval":"$\\dfrac{1}{4}$"},{"varval":"3"}]},{"vars":[{"varval":"cattle auction"},{"varval":"110"},{"varval":"holding pens"},{"varval":"36"},{"varval":"cattle"},{"varval":"one-quarter"},{"varval":"Cattle"},{"varval":"$\\dfrac{1}{4}$"},{"varval":"9"}]},{"vars":[{"varval":"sheep auction"},{"varval":"70"},{"varval":"holding pens"},{"varval":"12"},{"varval":"sheep"},{"varval":"two-thirds"},{"varval":"Sheep"},{"varval":"$\\dfrac{2}{3}$"},{"varval":"8"}]},{"vars":[{"varval":"cattle auction"},{"varval":"160"},{"varval":"holding pens"},{"varval":"20"},{"varval":"cattle"},{"varval":"one-quarter"},{"varval":"Cattle"},{"varval":"$\\dfrac{1}{4}$"},{"varval":"5"}]}]

  1142. #35

    <div class="sm_mode"> {{name}} supplies {{business}} with {{stuff}}. {{gender}} supplies one {{place}} with {{mass1}} kilograms of {{object}} on Monday, {{mass2}} kilograms of {{object}} on Tuesday and {{mass3}} kilograms on Friday. How many kilograms of {{object}}, in total, did {{name}} supply to the {{place}} during the week? </div>

    [{"vars":[{"varval":"Francis"},{"varval":"building sites"},{"varval":"building materials"},{"varval":"He"},{"varval":"site"},{"varval":"282"},{"varval":"bricks"},{"varval":"200"},{"varval":"138"}]},{"vars":[{"varval":"Valerie"},{"varval":"restaurant chains"},{"varval":"potatoes"},{"varval":"She"},{"varval":"restaurant"},{"varval":"150"},{"varval":"potatoes"},{"varval":"376"},{"varval":"134"}]},{"vars":[{"varval":"Simon"},{"varval":"building sites"},{"varval":"building materials"},{"varval":"He"},{"varval":"site"},{"varval":"200"},{"varval":"bricks"},{"varval":"248"},{"varval":"262"}]},{"vars":[{"varval":"Zac"},{"varval":"bakeries"},{"varval":"flour"},{"varval":"He"},{"varval":"bakery"},{"varval":"164"},{"varval":"flour"},{"varval":"146"},{"varval":"200"}]},{"vars":[{"varval":"Beth"},{"varval":"landscape designers"},{"varval":"potting mix"},{"varval":"She"},{"varval":"site"},{"varval":"172"},{"varval":"potting mix"},{"varval":"248"},{"varval":"500"}]}]

  1143. Number, NAPX-L4-CA04 v1

    In one year, $\dfrac{3}{10}$ of the puppies that a dog breeder sells are ridgebacks. What is $\dfrac{3}{10}$ as percentage?

    [{"vars":null}]

  1144. Number, NAPX-L4-CA04 v2

    In one term, $\dfrac{7}{10}$ of students in a school had at least 1 day off sick. What is $\dfrac{7}{10}$ as percentage?

    [{"vars":null}]

  1145. #34

    <div class="sm_mode"> {{name}}'s watch is fast and gains {{gain}} minutes every {{interval1}}. If the current time is {{t1}}, what time will {{name}}'s watch show in exactly {{interval2}}' time? </div>

    [{"vars":[{"varval":"Lindon"},{"varval":"15"},{"varval":"week"},{"varval":"9:28 pm"},{"varval":"six weeks"},{"varval":"6"},{"varval":"90"},{"varval":"90 minutes"},{"varval":"90 minutes"}]},{"vars":[{"varval":"Helen"},{"varval":"3"},{"varval":"hour"},{"varval":"9:12 am"},{"varval":"six hours"},{"varval":"6"},{"varval":"18"},{"varval":"18 minutes"},{"varval":"6 hours 18 minutes"}]},{"vars":[{"varval":"Shelly"},{"varval":"9"},{"varval":"fortnight"},{"varval":"9:40 am"},{"varval":"six weeks"},{"varval":"3"},{"varval":"27"},{"varval":"27 minutes"},{"varval":"27 minutes"}]},{"vars":[{"varval":"Patrick"},{"varval":"4"},{"varval":"2 hours"},{"varval":"8:25 am"},{"varval":"eight hours"},{"varval":"4"},{"varval":"16"},{"varval":"16 minutes"},{"varval":"8 hours 16 minutes"}]},{"vars":[{"varval":"Wendel"},{"varval":"7"},{"varval":"2 hours"},{"varval":"10:43 am"},{"varval":"eight hours"},{"varval":"4"},{"varval":"28"},{"varval":"28 minutes"},{"varval":"8 hours 28 minutes"}]}]

  1146. #33

    <div class="sm_mode"> The exchange rate between Australian dollars and Euro dollars (€) is **A\$1 = {{rate1}}.** {{name}} is in {{place}} and {{activity}} that costs {{cost}}. What change, in Euro dollars (€), will {{name}} receive from A\${{money}}? </div>

    [{"vars":[{"varval":"€0.5"},{"varval":"Murray"},{"varval":"Europe"},{"varval":"takes a taxi"},{"varval":"€20"},{"varval":"50"},{"varval":"0.5"},{"varval":"€25"}]},{"vars":[{"varval":"€0.5"},{"varval":"Leisa"},{"varval":"Paris"},{"varval":"buys a baguette"},{"varval":"€12"},{"varval":"50"},{"varval":"0.5"},{"varval":"€25"}]},{"vars":[{"varval":"€0.5"},{"varval":"Paul"},{"varval":"Berlin"},{"varval":"has a schnitzel for lunch"},{"varval":"€30"},{"varval":"100"},{"varval":"0.5"},{"varval":"€50"}]},{"vars":[{"varval":"€0.5"},{"varval":"Lily"},{"varval":"Rome"},{"varval":"buys a pizza for lunch"},{"varval":"€18"},{"varval":"50"},{"varval":"0.5"},{"varval":"€25"}]},{"vars":[{"varval":"€0.5"},{"varval":"Neville"},{"varval":"Venice"},{"varval":"buys a ride on a gondola"},{"varval":"€40"},{"varval":"100"},{"varval":"0.5"},{"varval":"€50"}]}]

  1147. #32

    <div class="sm_mode"> {{name}} is a {{work}}. If {{gender}} needs {{mass1}} kilograms of {{item1}} to mix with {{item2}} and {{item3}} to make {{mass2}} kilograms of {{item4}}, how many kilograms of {{item1}} does {{gender}} need to make 1 kilogram of {{item4}}? </div>

    [{"vars":[{"varval":"Guy"},{"varval":"builder"},{"varval":"he"},{"varval":"$\\large s$"},{"varval":"sand"},{"varval":"concrete"},{"varval":"water"},{"varval":"$\\large c$"},{"varval":"cement"}]},{"vars":[{"varval":"Michelle"},{"varval":"baker"},{"varval":"she"},{"varval":"$\\large s$"},{"varval":"sugar"},{"varval":"flour, yeast"},{"varval":"water"},{"varval":"$\\large d$"},{"varval":"dough"}]},{"vars":[{"varval":"Bob"},{"varval":"pie maker"},{"varval":"he"},{"varval":"$\\large m$"},{"varval":"meat"},{"varval":"gravy"},{"varval":"salt"},{"varval":"$\\large p$"},{"varval":"pie mince"}]},{"vars":[{"varval":"Leah"},{"varval":"an Italian chef"},{"varval":"she"},{"varval":"$\\large p$"},{"varval":"flour"},{"varval":"sugar"},{"varval":"water"},{"varval":"$\\large q$"},{"varval":"pasta dough"}]},{"vars":[{"varval":"Jamie"},{"varval":"chef"},{"varval":"he"},{"varval":"$\\large c$"},{"varval":"cheese"},{"varval":"potatoes"},{"varval":"cream"},{"varval":"$\\large p$"},{"varval":"potato bake"}]}]

  1148. #31

    <div class="sm_mode"> {{name}} is a {{job}} and his {{group}} produced {{number}} {{animals}} last year. If his {{group}} produced {{frac1}} as many {{animals}} this year, how many {{animals}} were produced? </div>

    [{"vars":[{"varval":"James"},{"varval":"pastoralist"},{"varval":"flock"},{"varval":"56"},{"varval":"lambs"},{"varval":"half"},{"varval":"$\\dfrac{1}{2}$"}]},{"vars":[{"varval":"Murdoch"},{"varval":"farmer"},{"varval":"herd"},{"varval":"72"},{"varval":"calves"},{"varval":"half"},{"varval":"$\\dfrac{1}{2}$"}]},{"vars":[{"varval":"Tim"},{"varval":"rancher"},{"varval":"team of horses"},{"varval":"94"},{"varval":"foals"},{"varval":"half"},{"varval":"$\\dfrac{1}{2}$"}]},{"vars":[{"varval":"Bob"},{"varval":"farmer"},{"varval":"herd"},{"varval":"42"},{"varval":"calves"},{"varval":"one-third"},{"varval":"$\\dfrac{1}{3}$"}]},{"vars":[{"varval":"Lloyd"},{"varval":"shepherd"},{"varval":"flock"},{"varval":"54"},{"varval":"lambs"},{"varval":"half"},{"varval":"$\\dfrac{1}{2}$"}]}]

  1149. Number, NAP-02685

    A christmas pudding recipe uses 3 cups of sugar for every 4 cups of sultanas. Select the correct combination of sugar and sultanas for this recipe.

    [{"vars":null}]

  1150. Q32

    A group of students were given a list of five {{element}} and each student chose their favourite. The results were recorded and graphed below. <br> {{image}} <br> What is the angle at the centre of the pie chart that represents the students that chose {{type}} as their favourite?

    [{"vars":[{"varval":"animals"},{"varval":"Dog"},{"varval":"51 + 89 + 75 + 33"},{"varval":"248"},{"varval":"sm_img https://teacher.smartermaths.com.au/wp-content/uploads/2020/07/var1.svg 300 indent3 vpad"}]},{"vars":[{"varval":"instruments"},{"varval":"Bass"},{"varval":"30 + 41 + 90 + 123"},{"varval":"284"},{"varval":"sm_img https://teacher.smartermaths.com.au/wp-content/uploads/2020/07/var2.svg 300 indent3 vpad"}]},{"vars":[{"varval":"marsupials"},{"varval":"Opossum"},{"varval":"85 + 91 + 49 + 87"},{"varval":"312"},{"varval":"sm_img https://teacher.smartermaths.com.au/wp-content/uploads/2020/07/var3.svg 300 indent3 vpad"}]},{"vars":[{"varval":"African animals"},{"varval":"Giraffe"},{"varval":"88 + 72 + 29 + 90"},{"varval":"279"},{"varval":"sm_img https://teacher.smartermaths.com.au/wp-content/uploads/2020/07/var4.svg 300 indent3 vpad"}]},{"vars":[{"varval":"animals"},{"varval":"Emu"},{"varval":"93 + 77 + 72 + 90"},{"varval":"332"},{"varval":"sm_img https://teacher.smartermaths.com.au/wp-content/uploads/2020/07/var5.svg 300 indent3 vpad"}]}]

  1151. Q31

    {{name}}'s payslip is missing some information. <br> <div class="sm_mode"> <div class="outline"> <table style="margin-left: 50px"> <tbody> <tr> <td colspan="3" rowspan="1" style="border-bottom: none; background-color: #efe9a2"><b>PAYSLIP</b> - {{fullname}} <br>Hourly rate: ${{rate}} <br> <br>Week beginning Monday, 13-Jul-20<br> <br> </td> </tr> <tr> <td style="background-color: #efe9a2; border-top: none"> </td> <td style="text-align: center">Hours</td> <td style="text-align: center">Amount</td> </tr> <tr> <td >Standard Rate</td> <td style="text-align: center"> $X$ </td> <td></td> </tr> <tr> <td >Time and a Half</td> <td style="text-align: center">{{hours}}</td> <td> </td> </tr> <tr> <td >TOTAL</td> <td> </td> <td style="text-align: center">${{total1}}</td> </tr> </tbody> </table> </div> <br> </div> <br> How many hours, $X$, did {{name}} work at his standard rate?

    [{"vars":[{"varval":"John"},{"varval":"John Mayer"},{"varval":"40"},{"varval":"4"},{"varval":"840"},{"varval":"240"},{"varval":"600"}]},{"vars":[{"varval":"Emilio"},{"varval":"Emilio Navaro"},{"varval":"20"},{"varval":"8"},{"varval":"800"},{"varval":"240"},{"varval":"560"}]},{"vars":[{"varval":"Ryan"},{"varval":"Ryan Reign"},{"varval":"30"},{"varval":"6"},{"varval":"1050"},{"varval":"270"},{"varval":"780"}]},{"vars":[{"varval":"Rick"},{"varval":"Rick Ash"},{"varval":"40"},{"varval":"4"},{"varval":"1560"},{"varval":"240"},{"varval":"1320"}]},{"vars":[{"varval":"Perry"},{"varval":"Perry Mason"},{"varval":"60"},{"varval":"6"},{"varval":"1800"},{"varval":"540"},{"varval":"1260"}]}]

  1152. Number, NAP-L3-CA37

    <div class="sm_mode"> {{{question}}} </div>

    [{"vars":[{"varval":"Pat, Mitch and Josh entered a long distance relay race as a team.\n\nThey divided the total distance into 20 equal sections.\n\nPat ran $\\dfrac{9}{20}$ of the total distance.\n\nMitch ran $\\dfrac{3}{20}$ more of the total distance than Josh.\n\nWhat fraction of the total distance did Josh run?\n"},{"varval":"Total distance Mitch and Josh run = 1 $-$ $\\dfrac{9}{20}$ = $\\dfrac{11}{20}$\n\n$\\rArr$ Mitch runs = $\\dfrac{7}{20}$\n\n$\\rArr$ Josh runs = {{{correctAnswer}}}"}]},{"vars":[{"varval":"Jenny, Karen and Leanne entered a long distance walk for charity as a team.\n\nThey divided the total distance into 30 equal sections.\n\nJenny walked $\\dfrac{13}{30}$ of the total distance.\n\nKaren walked $\\dfrac{7}{30}$ more of the total distance than Leanne.\n\nWhat fraction of the total distance did Leanne walk?\n"},{"varval":"Total distance Karen and Leanne walk = 1 $-$ $\\dfrac{13}{30}$ = $\\dfrac{17}{30}$\n\n$\\rArr$ Karen walks = $\\dfrac{12}{30}$\n\n$\\rArr$ Leanne walks = {{{correctAnswer}}}"}]},{"vars":[{"varval":"A walking trail consists of bush walking, beach walking and a steep hill climb through a park.\n\nThe walking trail is divided into 25 equal sections.\n\nThe bush walk makes up $\\dfrac{14}{25}$ of the total walking trail.\n\nThe beach walk makes up $\\dfrac{1}{5}$ more of the total walking trail than the hill climb.\n\nWhat fraction of the total trail is the hill climb?\n"},{"varval":"Combined distance of the beach walk and hill climb = 1 $-$ $\\dfrac{14}{25}$ = $\\dfrac{11}{25}$\n\n$\\rArr$ Beach walk = $\\dfrac{8}{25}$\n\n$\\rArr$ Hill climb = {{{correctAnswer}}}"}]},{"vars":[{"varval":"Jo, Nat and Alex entered a lake swim relay as a team.\n\nThey divided the total distance into 20 equal sections.\n\nJo swam $\\dfrac{3}{20}$ of the total distance.\n\nNat swam $\\dfrac{3}{20}$ more of the total distance than Alex.\n\nWhat fraction of the total distance did Alex swim?\n"},{"varval":"Total distance Nat and Alex swim = 1 $-$ $\\dfrac{3}{20}$ = $\\dfrac{17}{20}$\n\n$\\rArr$ Nat swims = $\\dfrac{10}{20}$\n\n$\\rArr$ Alex swims = {{{correctAnswer}}}"}]},{"vars":[{"varval":"Year 9 Art classes are tiling a mural in the canteen using orange, red and black tiles.\n\nThey divided the total area into 50 equal sections.\n\nOrange tiles make up $\\dfrac{17}{50}$ of the total area.\n\nBlack tiles make up $\\dfrac{5}{50}$ more of the total area than red.\n\nWhat fraction of the total area is made up of red tiles?"},{"varval":"Total area made up of black and red tiles = 1 $-$ $\\dfrac{17}{50}$ = $\\dfrac{33}{50}$\n\n$\\rArr$ Black tiles = $\\dfrac{19}{20}$\n\n$\\rArr$ Red tiles = {{{correctAnswer}}}"}]}]

  1153. Number, NAP-A3-NC25

    <div class="sm_mode"> {{{question}}} </div>

    [{"vars":[{"varval":"A garden centre sells a potting mix made up of soil, manure and sand.\n\nSoil makes up $\\dfrac{3}{4}$ of the mix and manure makes up $\\dfrac{1}{6}$ of the mix.\n\nWhat fraction of the potting mix is sand?\n"},{"varval":"<div class=\"aligned\">\n\n| | |\n| ---------------------: | -------------------------------------------- |\n| <div style=\"vertical-align: text-bottom;\">Sand</div> | \\= $1 - \\bigg( \\dfrac{3}{4} + \\dfrac{1}{6} \\bigg)$ |\n| | \\= $1 - \\bigg( \\dfrac{9}{12} + \\dfrac{2}{12} \\bigg)$ |\n| | \\= {{{correctAnswer}}} |\n\n</div>"}]},{"vars":[{"varval":"A landscaper's concrete mix is made up of sand, cement and gravel.\n\nCement makes up $\\dfrac{1}{6}$ of the mix and gravel makes up $\\dfrac{1}{2}$ of the mix.\n\nWhat fraction of the concrete mix is sand?\n"},{"varval":"<div class=\"aligned\">\n\n| | |\n| ---------------------: | -------------------------------------------- |\n| <div style=\"vertical-align: text-bottom;\">Sand</div> | \\= $1 - \\bigg( \\dfrac{1}{6} + \\dfrac{1}{2} \\bigg)$ |\n| | \\= $1 - \\bigg( \\dfrac{1}{6} + \\dfrac{3}{6} \\bigg)$ |\n| | \\= {{{correctAnswer}}} |\n\n</div>"}]},{"vars":[{"varval":"A patchwork quilt is constructed using red, white and blue material.\n\nRed material makes up $\\dfrac{2}{5}$ of the quilt and blue material makes up $\\dfrac{3}{7}$ of the quilt.\n\nWhat fraction of the quilt material is white?\n"},{"varval":"<div class=\"aligned\">\n\n| | |\n| ---------------------: | -------------------------------------------- |\n| <div style=\"vertical-align: text-bottom;\">White</div> | \\= $1 - \\bigg( \\dfrac{2}{5} + \\dfrac{3}{7} \\bigg)$ |\n| | \\= $1 - \\bigg( \\dfrac{14}{35} + \\dfrac{15}{35} \\bigg)$ |\n| | \\= {{{correctAnswer}}} |\n\n</div>\n"}]},{"vars":[{"varval":"Students either walk to school or travel by bus or car. \n\n$\\dfrac{3}{8}$ of the students catch a bus and $\\dfrac{3}{7}$ of the students walk.\n\nWhat fraction of the students travel by car?\n"},{"varval":"<div class=\"aligned\">\n\n| | |\n| ---------------------: | -------------------------------------------- |\n| <div style=\"vertical-align: text-bottom;\">Car</div> | \\= $1 - \\bigg( \\dfrac{3}{8} + \\dfrac{3}{7} \\bigg)$ |\n| | \\= $1 - \\bigg( \\dfrac{21}{56} + \\dfrac{24}{56} \\bigg)$ |\n| | \\= {{{correctAnswer}}} |\n\n</div>"}]}]

  1154. Number, NAP-I3-NC25

    <div class="sm_mode"> {{{question}}} </div>

    [{"vars":[{"varval":"Manoj is making a number of curries. \n\nHe has 8 cups of coconut milk.\n\nManoj uses $\\dfrac{2}{3}$ of a cup of coconut milk for every curry.\r\n\r\n\nWhat is the maximum number of curries Manoj can make?\n"},{"varval":"<div class=\"sm_mode\">\n\n<div class=\"aligned\">\n\n| | |\n| --------------------- | ---------------------------------------|\n| Maximum curries | = $8 \\div \\dfrac{2}{3}$ |\n| | = $8 \\times \\dfrac{3}{2}$ |\n| | = {{correctAnswer}} |\n\n</div>\n</div>"}]},{"vars":[{"varval":"Benjamin is making a number of apple pies. \n\nHe has 12 kilograms of apples.\n\nBenjamin uses $\\dfrac{2}{5}$ of a kilogram of apples for every apple pie.\n\n\r\n\r\nWhat is the maximum number of apple pies Benjamin can make?\n"},{"varval":"<div class=\"aligned\">\n\n| | |\n| ---------------------: | ---------------------------------------|\n| Maximum apple pies| = $12 \\div \\dfrac{2}{5}$ |\n| | = $12 \\times \\dfrac{5}{2}$ |\n| | = {{correctAnswer}} |\n\n</div>"}]},{"vars":[{"varval":"Bailey is making a number of bread boards. \n\nHe has 15 metres of timber.\n\nBailey uses $\\dfrac{3}{7}$ of a metre of timber for every bread board.\n\n\r\n\r\nWhat is the maximum number of bread boards Bailey can make?"},{"varval":"<div class=\"aligned\">\n\n| | |\n| --------------------- | ---------------------------------------|\n| Maximum bread boards| = $15 \\div \\dfrac{3}{7}$ |\n| | = $15 \\times \\dfrac{7}{3}$ |\n| | = {{correctAnswer}} |\n\n</div>"}]},{"vars":[{"varval":"Sandra has 2 kilograms of flour to make cupcakes. \n\nSandra uses $\\dfrac{2}{5}$ of a kilogram of flour for every batch of 24 cupcakes she makes.\n\n\r\n\r\nWhat is the maximum number of cupcakes Sandra can make?"},{"varval":"<div class=\"aligned\">\n\n| | |\n| --------------------- | ---------------------------------------|\n| Maximum batches of cupcakes| = $2 \\div \\dfrac{2}{5}$ |\n| | = $2 \\times \\dfrac{5}{2}$ |\n| | = 5 batches |\n| | |\n\n</div>\n\n<div class=\"aligned\">\n\n| | |\n| --------------------- | ---------------------------------------|\n| Maximum cupcakes| = $24 \\times 5$ |\n| | = {{correctAnswer}} |\n\n</div>"}]},{"vars":[{"varval":"Xanto is laying a number of driveways. \n\nHis cement truck holds 7.2 cubic metres of concrete.\n\nXanto uses $2\\dfrac{2}{5}$ cubic metres of concrete for every driveway he lays.\n\n\r\n\r\nWhat is the maximum number of driveways Xanto can lay?"},{"varval":"<div class=\"aligned\">\n\n| | |\n| --------------------- | ---------------------------------------|\n| Maximum driveways| = $7.2 \\div 2\\dfrac{2}{5}$ |\n| | = 7$\\dfrac{1}{5}\\div \\dfrac{12}{5}$ |\n| | = $\\dfrac{36}{5} \\times \\dfrac{5}{12}$ |\n| | = {{correctAnswer}} |\n</div>"}]},{"vars":[{"varval":"A cement company is laying driveways in a housing development. \n\nOne cement truck holds 7.2 cubic metres of concrete.\n\nEach driveway uses $3\\dfrac{3}{5}$ cubic metres of concrete.\n\nWhat is the maximum number of driveways that can be laid if 3 trucks are used?"},{"varval":"sm_nogap Maximum driveways (one truck)\n\n<div class=\"aligned\">\n\n>| | |\n| --------------------- | ---------------------------------------|\n| | = $7.2 \\div 3\\dfrac{3}{5}$ \n| | =$7\\dfrac{1}{5}\\div \\dfrac{18}{5}$ \n| | = $\\dfrac{36}{5} \\times \\dfrac{5}{18}$ |\n| | = 2 |\n\n</div>\n\n<br>\n\n$\\therefore$ Maximum driveways (three trucks) = {{{correctAnswer}}}\n"}]}]

  1155. Number, NAP-F3-NC25

    <div class="sm_mode"> {{{question}}} </div>

    [{"vars":[{"varval":"$\\dfrac{5}{8} + \\dfrac{2}{3}$ = <span class=\"sm-text color3\">?</span>"},{"varval":"<div class=\"aligned\">\n\n| | |\n|----- | ----- |\n| $\\dfrac{5}{8} + \\dfrac{2}{3}$ | = $\\dfrac{15}{24} + \\dfrac{16}{24}$ |\n| | |\n| | = $\\dfrac{31}{24}$ |\n| | |\n| | = {{correctAnswer}} |\n\n</div>"}]},{"vars":[{"varval":"$\\dfrac{4}{7} + \\dfrac{3}{5}$ = <span class=\"sm-text color3\">?</span>"},{"varval":"<div class=\"aligned\">\n\n| | |\n|----- | ----- |\n| $\\dfrac{4}{7} + \\dfrac{3}{5}$ | = $\\dfrac{20}{35} + \\dfrac{21}{35}$ |\n| | |\n| | = $\\dfrac{41}{35}$ |\n| | |\n| | = {{correctAnswer}} |\n\n</div>"}]},{"vars":[{"varval":"$\\dfrac{7}{12} + \\dfrac{3}{4}$ = <span class=\"sm-text color3\">?</span>"},{"varval":"<div class=\"aligned\">\n\n| | |\n|----- | ----- |\n| $\\dfrac{7}{12} + \\dfrac{3}{4}$ | = $\\dfrac{7}{12} + \\dfrac{9}{12}$ |\n| | |\n| | = $\\dfrac{16}{12}$ |\n| | |\n| | = $\\dfrac{4}{3}$ |\n| | |\n| | = {{correctAnswer}} |\n\n</div>"}]},{"vars":[{"varval":"$\\dfrac{7}{9} + \\dfrac{3}{5}$ = <span class=\"sm-text color3\">?</span>"},{"varval":"<div class=\"aligned\">\n\n| | |\n|----- | ----- |\n| $\\dfrac{7}{9} + \\dfrac{3}{5}$ | = $\\dfrac{35}{45} + \\dfrac{27}{45}$ |\n| | |\n| | = $\\dfrac{62}{45}$ |\n| | |\n| | = {{correctAnswer}} |\n\n</div>"}]},{"vars":[{"varval":"$\\dfrac{3}{4} + \\dfrac{2}{3}$ = <span class=\"sm-text color3\">?</span>"},{"varval":"<div class=\"aligned\">\n\n| | |\n|----- | ----- |\n| $\\dfrac{3}{4} + \\dfrac{2}{3}$ | = $\\dfrac{9}{12} + \\dfrac{8}{12}$ |\n| | |\n| | = $\\dfrac{17}{12}$ |\n| | |\n| | = {{correctAnswer}} |\n\n</div>"}]},{"vars":[{"varval":"$\\dfrac{11}{12} + \\dfrac{4}{5}$ = <span class=\"sm-text color3\">?</span>"},{"varval":"<div class=\"aligned\">\n\n| | |\n|----- | ----- |\n| $\\dfrac{11}{12} + \\dfrac{4}{5}$ | = $\\dfrac{55}{60} + \\dfrac{48}{60}$ |\n| | |\n| | = $\\dfrac{103}{60}$ |\n| | |\n| | = {{correctAnswer}} |\n\n</div>"}]}]

  1156. Number, NAP-H3-CA22

    <div class="sm_mode"> {{{question}}} </div>

    [{"vars":[{"varval":"<span class=\"sm-text color2\">?</span> + $\\dfrac{5}{6} = \\dfrac{17}{6}$\n\n<br>\n\nWhat makes this number sentence correct?\n"},{"varval":"<div class=\"aligned\">\n\n| | |\n| --------------------- :| --------------------------------|\n| <span class=\"sm-text color2\">?</span> $+\\ \\dfrac{5}{6}$ | = $\\dfrac{17}{6}$|\n|<span class=\"sm-text color2\">?</span>| = $\\dfrac{17}{6} - \\dfrac{5}{6}$|\n| <span class=\"sm-text color2\">?</span> | = $\\dfrac{12}{6}$|\n| <span class=\"sm-text color2\">?</span> | = {{correctAnswer}} |\n\n</div>"}]},{"vars":[{"varval":"<span class=\"sm-text color3\">?</span> + $\\dfrac{4}{9} = \\dfrac{14}{9}$\n\n<br>\n\nWhat makes this number sentence correct?\n"},{"varval":"<div class=\"aligned\">\n\n| | |\n| --------------------- :| --------------------------------|\n| <span class=\"sm-text color3\">?</span> + $\\dfrac{4}{9}$ | = $\\dfrac{14}{9}$|\n|<span class=\"sm-text color3\">?</span>| = $\\dfrac{14}{9} - \\dfrac{4}{9}$|\n| <span class=\"sm-text color3\">?</span> | = $\\dfrac{10}{9}$|\n| <span class=\"sm-text color3\">?</span> | = {{correctAnswer}} |\n\n</div>"}]},{"vars":[{"varval":"<span class=\"sm-text color4\">?</span> + $\\dfrac{8}{11} = \\dfrac{20}{11}$\n\n<br>\n\nWhat makes this number sentence correct?\n"},{"varval":"<div class=\"aligned\">\n\n| | |\n| --------------------- :| --------------------------------|\n| <span class=\"sm-text color4\">?</span> + $\\dfrac{8}{11}$ | = $\\dfrac{20}{11}$|\n|<span class=\"sm-text color4\">?</span>| = $\\dfrac{20}{11} - \\dfrac{8}{11}$|\n| <span class=\"sm-text color4\">?</span> | = $\\dfrac{12}{11}$|\n| <span class=\"sm-text color4\">?</span> | = {{correctAnswer}} |\n\n</div>"}]},{"vars":[{"varval":"<span class=\"sm-text color5\">?</span> + $\\dfrac{2}{5} = \\dfrac{13}{5}$\n\n<br>\n\nWhat makes this number sentence correct?"},{"varval":"<div class=\"aligned\">\n\n| | |\n| --------------------- :| --------------------------------|\n| <span class=\"sm-text color5\">?</span> + $\\dfrac{2}{5}$ | = $\\dfrac{13}{5}$|\n|<span class=\"sm-text color5\">?</span>| = $\\dfrac{13}{5} - \\dfrac{2}{5}$|\n| <span class=\"sm-text color5\">?</span> | = $\\dfrac{11}{5}$|\n| <span class=\"sm-text color5\">?</span> | = {{correctAnswer}} |\n\n</div>"}]},{"vars":[{"varval":"<span class=\"sm-text color6\">?</span> + $\\dfrac{7}{9} = \\dfrac{32}{9}$\n\n<br>\n\nWhat makes this number sentence correct?"},{"varval":"<div class=\"aligned\">\n\n| | |\n| --------------------- :| --------------------------------|\n| <span class=\"sm-text color6\">?</span> + $\\dfrac{7}{9}$ | = $\\dfrac{32}{9}$|\n|<span class=\"sm-text color6\">?</span>| = $\\dfrac{32}{9} - \\dfrac{7}{9}$|\n| <span class=\"sm-text color6\">?</span> | = $\\dfrac{25}{9}$|\n| <span class=\"sm-text color6\">?</span> | = {{correctAnswer}} |\n\n</div>"}]},{"vars":[{"varval":"<span class=\"sm-text color6\">?</span> $-$ $\\dfrac{1}{3} = \\dfrac{5}{3}$\n\n<br>\n\nWhat makes this number sentence correct?"},{"varval":"<div class=\"aligned\">\n\n| | |\n| --------------------- :| --------------------------------|\n| <span class=\"sm-text color6\">?</span> $-$ $\\dfrac{1}{3}$ | = $\\dfrac{5}{3}$|\n|<span class=\"sm-text color6\">?</span>| = $\\dfrac{5}{3} + \\dfrac{1}{3}$|\n| <span class=\"sm-text color6\">?</span> | = $\\dfrac{6}{3}$|\n| <span class=\"sm-text color6\">?</span> | = {{correctAnswer}} |\n\n</div>"}]}]

  1157. Number, NAP-F3-NC16

    A mixture requires an amount of salt to be added. No more than 0.75 grams of salt is allowed in the mixture. Which amount below is the heaviest mass of salt that can be added?

    [{"vars":null}]

  1158. Number, NAP-B3-CA19

    A bowl has 16 pieces of fruit in it. 12 of the pieces of fruit are oranges. What fraction of the pieces of fruit are oranges?

    [{"vars":null}]

  1159. Algebra, NAP-K2-25

    A hot air balloon is 25.2 metres high. The length of the balloon section is 21.55 metres. <br> sm_img https://teacher.smartermaths.com.au/wp-content/uploads/2018/07/NAP-K3-NC041.svg 320 indent3 vpad <br> What is the height of the basket section?

    [{"vars":null}]

  1160. Number, NAP-D3-CA18

    <div class="sm_mode"> {{{question}}} </div>

    [{"vars":[{"varval":"Richard is a runner.\n\nThe length of his last 4 half marathons are recorded in the table below.\n\n<div class=\"sm-table row1-color4\">\n\n>>| Half Marathon | Distance (km) |\n|:-:|:-:|\n| 1 | 21.1|\n| 2 | 20.35|\n| 3 | 20.295|\n| 4 | 22.095|\n\n</div>\n\n\n<br>Which half marathon had the shortest distance?"},{"varval":"20.295 is the shortest distance. \n\n$\\therefore$ Half marathon {{{correctAnswer}}} was the shortest."}]},{"vars":[{"varval":"Miranda collects rats.\n\nThe length of the last 4 rodents she collected, in centimetres, are recorded in the table below.\n\n<div class=\"sm-table row1-color4\">\n\n>>| Rodent | Length (cm) |\n|:-:|:-:|\n| A | 27.3|\n| B | 26.42|\n| C | 26.184|\n| D | 28.022|\n\n</div>\n\n\n<br>Which rodent was the shortest in length?"},{"varval":"26.184 centimetres is the shortest length. \n\n$\\therefore$ Rodent {{{correctAnswer}}} was the shortest."}]}]

  1161. Numbers, NAP-I3-NC20

    Matt and Libby both bought the same sized box of chocolates. Matt ate $\dfrac{5}{6}$ of his box. Libby ate more chocolates than Matt. What fraction of her chocolates could Libby have eaten?

    [{"vars":null}]

  1162. Number, NAP-C3-CA17

    <div class="sm_mode"> {{{question}}} </div>

    [{"vars":[{"varval":"The fraction $\\dfrac{8}{10}$ and $\\dfrac{1}{5}$ have been shaded on this fraction wall.\n\n<br>\n\nsm_img https://teacher.smartermaths.com.au/wp-content/uploads/2016/12/naplan-Y7-2010-17mc.png 350 indent3 vpad\n\nWhat is $\\dfrac{8}{10} - \\dfrac{1}{5}$?"},{"varval":"The fraction wall can be used to graphically \rsee the difference as $\\dfrac{6}{10}$.\n\nAlternatively,\n\n<div class=\"aligned\">\n\n| | |\n| --------------------- | -------------------------------------------- |\n| $\\dfrac{8}{10} - \\dfrac{1}{5}$ | = $\\dfrac{8}{10} - \\dfrac{2}{10}$ |\n| | = {{correctAnswer}} |\n\n</div>"}]},{"vars":[{"varval":"The fraction $\\dfrac{8}{10}$ and $\\dfrac{1}{5}$ have been shaded on this fraction wall.\n\n<br>\n\nsm_img https://teacher.smartermaths.com.au/wp-content/uploads/2016/12/naplan-Y7-2010-17mc.png 350 indent3 vpad\n\nWhat is $\\dfrac{8}{10} + \\dfrac{1}{5}$?"},{"varval":"The fraction wall can be used to graphically \rsee the sum as 1.\n\nAlternatively,\n\n<div class=\"aligned\">\n\n| | |\n| --------------------- | -------------------------------------------- |\n| $\\dfrac{8}{10} + \\dfrac{1}{5}$ | = $\\dfrac{8}{10} + \\dfrac{2}{10}$ |\n| | = $\\dfrac{10}{10}$ |\n| | = {{correctAnswer}} |\n\n</div>"}]}]

  1163. Number, NAP-L3-NC05 SA

    <div class="sm_mode"> {{{question}}} </div>

    [{"vars":[{"varval":"The weather bureau publishes the weekday maximum temperatures in the table below.\n\n<div class=\"sm-table col1-color1 row1-color1 top-left-cell-hidden\">\n\n>>| | Mon | Tue |Wed |Thu |Fri |\n|:-:|:-:|:-:|:-:|:-:|:-:|\n| Maximum Temp ($\\degree$C) | 22.7| 21.8|20.3 | 26.4 | 28.7 |\n\n</div>\n\n\n<br>\n\nHow much warmer was it on Thursday than on Monday?"},{"varval":"<div class=\"aligned\">\n\n| | |\n| --------------------- | -------------------------------------------- |\n| Degrees warmer | = $26.4 - 22.7$ |\n| | = {{{correctAnswer}}} |\n\n\n</div>"}]},{"vars":[{"varval":"The maximum temperatures over 5 days of a cricket test are recorded in the table below.\n\n<div class=\"sm-table col1-color3 row1-color3 top-left-cell-hidden\">\n\n>>| | Day 1 | Day 2 | Day 3 |Day 4 |Day 5 |\n|:-:|:-:|:-:|:-:|:-:|:-:|\n| Maximum Temp ($\\degree$C) | 28.9| 29.4 | 33.6 | 32.2 | 35.2 |\n\n</div>\n\n\n<br>\n\nHow much warmer was it on Day 4 than on Day 2?"},{"varval":"<div class=\"aligned\">\n\n| | |\n| --------------------- | -------------------------------------------- |\n| Degrees warmer | = $32.2 - 29.4$ |\n| | = {{{correctAnswer}}} |\n\n\n</div>"}]},{"vars":[{"varval":"The maximum temperatures over 5 weekdays of in a country town are recorded in the table below.\n\n<div class=\"sm-table col1-color6 row1-color6 top-left-cell-hidden\">\n\n>>| | Mon | Tue | Wed | Thu | Fri |\n|:-:|:-:|:-:|:-:|:-:|:-:|\n| Maximum Temp ($\\degree$C) | 42.5| 44.1 | 39.3 | 43.7 | 46.2 |\n\n</div>\n\n\n<br>\n\nHow much warmer was it on Friday than on Monday?"},{"varval":"<div class=\"aligned\">\n\n| | |\n| --------------------- | -------------------------------------------- |\n| Degrees warmer | = $46.2 - 42.5$ |\n| | = {{{correctAnswer}}} |\n\n\n</div>"}]}]

  1164. David paid \$9.00 for sausages advertised at the price below. <br> sm_img https://teacher.smartermaths.com.au/wp-content/uploads/2017/12/NAP-A3-CA017.svg 297 indent3 vpad <br> How many grams of sausages did he buy?

    [{"vars":null}]

  1165. Number, NAP-C3-CA16

    Last year 4236 people visited a national park. The number of people who visited the park this year was $\dfrac{2}{3}$ of last year's figure. How many people went to the park this year?

    [{"vars":null}]

  1166. #30

    {{name}} is a {{work}} and is making {{item}}. The amount of flour she needs for one batch of {{item}} is shown below. <br> {{image}} <br> {{name}} adds {{ing1}}, {{ing2}} and {{ing3}} to make one batch of {{item}} which is used to make {{pieces}} {{product}}. {{name}} needs to make {{dozen}} dozen {{product}}. How much flour does she need?

    [{"vars":[{"varval":"Chris"},{"varval":"baker"},{"varval":"dough"},{"varval":"50 grams of yeast"},{"varval":"15 grams of salt"},{"varval":"300 mL of water"},{"varval":"8"},{"varval":"loaves of bread"},{"varval":"2"},{"varval":"24"},{"varval":"loaves"},{"varval":"3"},{"varval":"160"},{"varval":"sm_img https://teacher.smartermaths.com.au/wp-content/uploads/2020/07/q30var1.png 200 indent3 vpad"}]},{"vars":[{"varval":"Boni"},{"varval":"pastry chef"},{"varval":"cream tarts"},{"varval":"120 grams of sugar"},{"varval":"70 grams of cream"},{"varval":"250 mL of condensed milk"},{"varval":"6"},{"varval":"tarts"},{"varval":"3"},{"varval":"36"},{"varval":"tarts"},{"varval":"6"},{"varval":"120"},{"varval":"sm_img https://teacher.smartermaths.com.au/wp-content/uploads/2020/07/q30var2.png 200 indent3 vpad"}]},{"vars":[{"varval":"Rochelle"},{"varval":"baker"},{"varval":"dough"},{"varval":"70 grams of yeast"},{"varval":"25 grams of salt"},{"varval":"285 mL of water"},{"varval":"4"},{"varval":"loaves of bread"},{"varval":"2"},{"varval":"24"},{"varval":"loaves"},{"varval":"6"},{"varval":"90"},{"varval":"sm_img https://teacher.smartermaths.com.au/wp-content/uploads/2020/07/q30var3.png 200 indent3 vpad"}]},{"vars":[{"varval":"Bella"},{"varval":"chef"},{"varval":"pizza dough"},{"varval":"90 grams of yeast"},{"varval":"60 grams of sugar"},{"varval":"425 mL of water"},{"varval":"8"},{"varval":"pizza bases"},{"varval":"4"},{"varval":"48"},{"varval":"pizza bases"},{"varval":"6"},{"varval":"180"},{"varval":"sm_img https://teacher.smartermaths.com.au/wp-content/uploads/2020/07/q30var4.png 200 indent3 vpad"}]},{"vars":[{"varval":"Cassie"},{"varval":"chef"},{"varval":"pizza dough"},{"varval":"85 grams of yeast"},{"varval":"15 grams of salt"},{"varval":"575 mL of water"},{"varval":"6"},{"varval":"pizza bases"},{"varval":"2"},{"varval":"24"},{"varval":"pizza bases"},{"varval":"4"},{"varval":"170"},{"varval":"sm_img https://teacher.smartermaths.com.au/wp-content/uploads/2020/07/q30var5.png 200 indent3 vpad"}]}]

  1167. Number, NAP-F2-33

    A soccer match attracted a crowd of 17 221 people. One third of the people at the match supported the away team. Which of these is the closest to the number of people at the match who supported the away team?

    [{"vars":null}]

  1168. Number, NAP-A3-CA15

    A school library has 130 books lent out. 60 of the books on loan are overdue. The fraction of books on loan that are overdue is closest to

    [{"vars":null}]

  1169. Number, NAP-L3-CA21

    Zoey scored 88% on her Geography exam. If she achieved the same mark on her French exam, which of these could have been her mark?

    [{"vars":null}]

  1170. Number, NAP-G3-NC15

    <div class="sm_mode"> 4 teams of hikers measured the distance they hiked over a two hour period. The distances are shown in the table below. <br> <div class="sm-table col1-color1 row1-color1 top-left-cell-hidden"> >>| | Team A | Team B | Team C | Team D | |:-:|:-:|:-:|:-:|:-:| | 1st hour | 3.64 | 3.49 | 3.42| 3.56 | | 2nd hour | 3.35 | 3.61 |3.77| 3.39 | </div> <br> Which team travelled the greatest distance over the two hours? </div>

    [{"vars":null}]

  1171. Number, NAP-C3-NC13

    On a certain night half of the moon is black, $\dfrac{3}{10}$ is grey and the rest of the moon is lit up brightly. What fraction of the moon is lit up brightly?

    [{"vars":null}]

  1172. Q30

    {{name1}} collected {{number1}} {{item}} for recycling by himself. {{name2}} and her {{number2}} girlfriends collected {{item}} for recycling as well. The average number collected by each girl was half the number that {{name1}} collected. What is the total number of {{item}} collected by {{name2}} and her {{number2}} girlfriends?

    [{"vars":[{"varval":"Melvin"},{"varval":"148"},{"varval":"aluminum cans"},{"varval":"Kelly"},{"varval":"three"},{"varval":"74"},{"varval":"4"}]},{"vars":[{"varval":"Michael"},{"varval":"126"},{"varval":"bottles"},{"varval":"Sarah"},{"varval":"four"},{"varval":"63"},{"varval":"5"}]},{"vars":[{"varval":"Raphael"},{"varval":"134"},{"varval":"ink cartridges"},{"varval":"Rhonda"},{"varval":"four"},{"varval":"67"},{"varval":"5"}]},{"vars":[{"varval":"Robin"},{"varval":"86"},{"varval":"newspapers"},{"varval":"Moana"},{"varval":"five"},{"varval":"43"},{"varval":"6"}]},{"vars":[{"varval":"Winnie"},{"varval":"122"},{"varval":"bottles"},{"varval":"Lucy"},{"varval":"five"},{"varval":"61"},{"varval":"6"}]}]

  1173. Number, NAP-K3-CA12

    Lance is competing in a cycling event. On Stage 1, he had to cycle 90 kilometres. On Stage 2, he had to cycle three times as far as Stage 1. On Stage 3, he had to cycle one quarter as far as Stage 2. Which expression could Lance use to work out the number of kilometres he needed to cycle for Stage 3.

    [{"vars":null}]

  1174. Q29

    It takes {{number1}} full {{transport1}} to transport {{number2}} {{type1}} from the {{place1}} to the {{place2}}. How many {{type1}} would {{number3}} full {{transport1}} be able to transport to the {{place2}}?

    [{"vars":[{"varval":"8"},{"varval":"shuttle buses"},{"varval":"72"},{"varval":"tourists"},{"varval":"ferry terminal"},{"varval":"resort"},{"varval":"5"},{"varval":"9"},{"varval":"shuttle bus"},{"varval":"Tourists"}]},{"vars":[{"varval":"7"},{"varval":"minibuses"},{"varval":"56"},{"varval":"visitors"},{"varval":"ferry terminal"},{"varval":"hotel"},{"varval":"4"},{"varval":"8"},{"varval":"Visitors"},{"varval":"minibus"}]},{"vars":[{"varval":"7"},{"varval":"maxi-vans"},{"varval":"42"},{"varval":"supporters"},{"varval":"club"},{"varval":"sports ground"},{"varval":"3"},{"varval":"6"},{"varval":"Supporters"},{"varval":"maxi-van"}]},{"vars":[{"varval":"9"},{"varval":"carriages"},{"varval":"72"},{"varval":"royals"},{"varval":"opera house"},{"varval":"palace"},{"varval":"5"},{"varval":"8"},{"varval":"Royals"},{"varval":"carriage"}]},{"vars":[{"varval":"8"},{"varval":"carriages"},{"varval":"56"},{"varval":"royals"},{"varval":"palace"},{"varval":"ballet"},{"varval":"3"},{"varval":"7"},{"varval":"Royals"},{"varval":"carriage"}]}]

  1175. Number, NAP-L3-CA14

    Data was gathered on the country of origin of cowboys competing in a rodeo in Australia. The data showed that in one particular year: <div class="sm_mode"> <div class="aligned"> * $\dfrac{2}{15}$ of all the cowboys came from Canada. </div> * $\dfrac{1}{10}$ of all cowboys came from the USA. </div> <br> Select the correct statement about the cowboys competing in the rodeo in that year.

    [{"vars":null}]

  1176. Number, NAP-I3-CA08

    Peter had 5 cups of flour to use for baking 2 loaves of bread. He used $1\ \dfrac{1}{8}$ cups for the first loaf, and $2\ \dfrac{1}{8}$ cups for the second loaf. How many cups of flour did Peter have left? <!--EndFragment-->

    [{"vars":null}]

  1177. Number, NAP-F3-CA10

    {{{question}}}

    [{"vars":[{"varval":"The number 0.67 is between"},{"varval":"The number 0.67 is between"}]},{"vars":[{"varval":"The number 0.34 is between"},{"varval":"The number 0.34 is between"}]},{"vars":[{"varval":"The number 0.68 is between"},{"varval":"The number 0.68 is between"}]},{"vars":[{"varval":"The number 0.54 is between"},{"varval":"The number 0.54 is between"}]}]

  1178. Number, NAP-I3-CA10

    Nigella had 1 kilogram of chocolate. She used $\dfrac{3}{4}$ kilogram in a cake recipe. How much chocolate did Nigella have left?

    [{"vars":null}]

  1179. Number, NAP-J3-CA06 SA

    There are 48 Year 7 students at a high school. Each student is asked if they own a bike or not and the results are recorded. $\dfrac{3}{4}$ of the students said they owned a bike. How many Year 7 students at the school own a bike?

    [{"vars":null}]

  1180. Number, NAP-G3-NC09

    Patrick poured one quarter of a litre of milk into his glass. How many millilitres did Patrick pour into his glass?

    [{"vars":null}]

  1181. Number, NAP-F3-CA05

    {{{question}}}

    [{"vars":[{"varval":"Over a period of 2 weeks, Tony had to fight fires for 8 days.\n\nIn days, what fraction of this time period was Tony **not** fighting fires?"},{"varval":"sm_nogap Days not fighting fires\n\n<div class=\"aligned\">\n\n> > | | |\n> > | --------------------: | -------------- |\n> > | |= $14 - 8$ |\n> > | |= 6 days |\n\n</div>\n\n<br>\n\n$\\therefore$ Fraction not fighting fires = {{{correctAnswer}}}"}]},{"vars":[{"varval":"Oliver trains for soccer 10 months of the year.\r\n\r\nIn months, what fraction of the year does Oliver **not** train for soccer?"},{"varval":"sm_nogap Months not soccer training\n\n<div class=\"aligned\">\n\n> > | | |\n> > | --------------------: | -------------- |\n> > | |= $12 - 10$ |\n> > | |= 2 months |\n\n</div>\n\n<br>\n\n$\\therefore$ Fraction not soccer training = {{{correctAnswer}}}"}]},{"vars":[{"varval":"In one minute of a running workout, Patricia sprinted for 40 seconds.\n\r\nWhat fraction of the minute did Patricia **not** sprint?"},{"varval":"sm_nogap Seconds not sprinting\n\n<div class=\"aligned\">\n\n> > | | |\n> > | --------------------: | -------------- |\n> > | |= $60 - 20$ |\n> > | |= 40 seconds |\n\n</div>\n\n<br>\n\n$\\therefore$ Fraction not sprinting = {{{correctAnswer}}}"}]},{"vars":[{"varval":"In one hour of cooking, Manou spent 50 minutes chopping vegetables.\n\r\nWhat fraction of the hour was Manou **not** chopping vegetables?"},{"varval":"sm_nogap Minutes not chopping vegetables\n\n<div class=\"aligned\">\n\n> > | | |\n> > | --------------------: | -------------- |\n> > | |= $60 - 50$ |\n> > | |= 10 minutes |\n\n</div>\n\n<br>\n\n$\\therefore$ Fraction not chopping vegetables = {{{correctAnswer}}}"}]}]

  1182. Number, NAP-E3-CA05

    Michael places his business cards in a stack that has a height of 151.2 mm. Each business card has a thickness of 1.8 mm. How many business cards are in the stack?

    [{"vars":null}]

  1183. Number, NAPX-G4-CA18

    <div class="sm_mode"> {{{question}}} </div>

    [{"vars":[{"varval":"Marjorie and Deek ran in a 20 km road race.\n\nMarjorie finished in a time of 1 hour and 15 minutes.\n\nDeek took 15% longer than Marjorie.\n\nApproximately how long did Deek take to finish the race?\n"},{"varval":"sm_nogap Marjorie's time = 75 minutes\n\n<div class=\"aligned\">\n\n| | |\n| ----------- | --------------- |\n| $$\\therefore$$ Deek's time | = 75 + 15% × 75 |\n| | = 75 + 0.15 × 75 |\n| | = 86.25 minutes |\n| | $\\approx$ {{{correctAnswer}}} |\n\n\n</div>"}]},{"vars":[{"varval":"Jonah and Rachel go to the movies.\n\nJonah watches \"Minions: The Rise of Gru\" which finished in a time of 1 hour and 27 minutes.\n\nRachel watches \"Jurassic World Dominion\" which was 70% longer.\n\nApproximately how long was \"Jurassic World Dominion\"?\n"},{"varval":"sm_nogap Minion's movie time = 60 + 27 = 87 minutes\n\n<div class=\"aligned\">\n\n| | |\n| ----------- | --------------- |\n| $$\\therefore$$ Jurassic move time | = 87 + 70% × 87 |\n| | = 87 + 0.70 × 87 |\n| | = 147.9 |\n| | $\\approx$ {{{correctAnswer}}} |\n\n</div>"}]},{"vars":[{"varval":"Thorpe and Perkins competed in a marathon swim.\n\nThorpe finished in a time of 2 hour and 05 minutes.\n\nPerkins took 10% longer than Thorpe.\n\nApproximately how long did Perkins take to finish the swim?\n"},{"varval":"sm_nogap Thorpe's time = 2 × 60 + 5 = 125 minutes\n\n<div class=\"aligned\">\n\n| | |\n| ----------- | --------------- |\n| $$\\therefore$$ Perkins' time | = 125 + 10% × 125 |\n| | = 125 + 0.10 × 125 |\n| | = 137.5 |\n| | $\\approx$ {{{correctAnswer}}} |\n\n\n</div>"}]},{"vars":[{"varval":"Elona is travelling from Adelaide to Honolulu via Auckland.\n\nThe first leg from Adelaide to Auckland takes 5 hour and 35 minutes.\n\nThe Auckland to Honolulu trip takes 57% longer than the first leg.\n\nApproximately how long does the second leg take?\n"},{"varval":"sm_nogap Adelaide to Auckland time = 5 × 60 + 35 = 335 minutes\n\n<div class=\"aligned\">\n\n| | |\n| ----------- | --------------- |\n| $$\\therefore$$ Auckland to Honolulu time | = 335 + 57% × 335 |\n| | = 335 + 0.57 × 335 |\n| | = 525.95 |\n| | $\\approx$ {{{correctAnswer}}}|\n\n\n</div>"}]},{"vars":[{"varval":"The men's record for swimming the English Channel is 6 hours and 56 minutes.\n\nThe women's record is 10% longer.\n\nApproximately how long is the women's record for crossing the English Channel?\n"},{"varval":"sm_nogap Men's time = 6 × 60 + 56 = 416 minutes\n\n<div class=\"aligned\">\n\n| | |\n| ----------- | --------------- |\n| $$\\therefore$$ Women's time | = 416 + 10% × 416 |\n| | = 416 + 41.6 |\n| | = 457.6|\n| | $\\approx$ {{{correctAnswer}}} |\n\n\n</div>"}]},{"vars":[{"varval":"Brutus and Beverly went on a 50 km hike.\n\nThey finished the first 25 km in a time of 1 hour and 48 minutes.\n\nThe second 25 km took 18% longer than the first.\n\nApproximately how long did they take to finish the second 25 km?\n"},{"varval":"sm_nogap 1st 25 km time = 60 + 48 = 108 minutes\n\n<div class=\"aligned\">\n\n| | |\n| ----------- | --------------- |\n| $$\\therefore$$ 2nd 25 km time | = 108 + 18% × 108 |\n| | = 108 + 0.18 × 108 |\n| | = 127.44 |\n| | $\\approx$ {{{correctAnswer}}} |\n\n\n</div>"}]}]

  1184. Number, NAPX-H4-CA09

    A lizard enclosure contains 9 chameleons and 17 blue-tongue lizards. Approximately what percentage of the lizards are blue-tongue?

    [{"vars":null}]

  1185. Number, NAPX-F4-CA06

    <div class="sm_mode"> {{{question}}} </div>

    [{"vars":[{"varval":"Lloyd walks to work. The distance is 320 metres.\n\nA road closure means that Lloyd's walk is 5% longer.\n\nWhat distance does he need to walk now?\n"},{"varval":"<div class=\"aligned\">\n\n| | |\n| --------------------- | ---------------- |\n| $\\text{Distance}$ | = 320 + 5% × 320 |\n| | = 320 + 16 |\n| | = {{{correctAnswer}}} |\n\n</div>"}]},{"vars":[{"varval":"Django travels to work by train. The trip usually takes 40 minutes.\n\nToday there is water over the tracks making Django's trip 15% longer.\n\nHow long does it take Django to get to work today?\n"},{"varval":"<div class=\"aligned\">\n\n| | |\n| --------------------- | ---------------- |\n| $\\text{Time}$ | = 40 + 15% × 40 |\n| | = 40 + 6 |\n| | = {{{correctAnswer}}} |\n\n</div>"}]},{"vars":[{"varval":"Josie walks to her friend's house. The distance is 1.2 kilometres.\n\nA road closure means that Josie's walk is 25% longer.\n\nWhat distance does she need to walk now?\n"},{"varval":"<div class=\"aligned\">\n\n| | |\n| --------------------- | ---------------- |\n| $\\text{Distance}$ | = 1.2 + 25% × 1.2 |\n| | = 1.2 + 0.3 |\n| | = {{{correctAnswer}}} |\n\n</div>"}]},{"vars":[{"varval":"Gerry is streaming his favourite show on Paraflix. \n\nEach episode is usually 50 minutes long.\n\nGerry's internet service keeps going down and it takes 30% longer to watch tonight's episode.\n\nHow long did it take Gerry to watch tonight's episode? \n"},{"varval":"<div class=\"aligned\">\n\n| | |\n| --------------------- | ---------------- |\n| $\\text{Time}$ | = 50 + 30% × 50 |\n| | = 50 + 15 |\n| | = {{{correctAnswer}}} |\n\n</div>"}]},{"vars":[{"varval":"Minn is training to compete in a fun run. Yesterday she ran 14 kilometres.\n\nToday she increased this distance by 16%.\n\nWhat distance did Minn run today?\n"},{"varval":"<div class=\"aligned\">\n\n| | |\n| --------------------- | ---------------- |\n| $\\text{Distance}$ | = 14 + 16% × 14 |\n| | = 14 + 2.24 |\n| | = {{{correctAnswer}}} |\n\n</div>"}]},{"vars":[{"varval":"Dora was exploring the ranges. Yesterday she hiked a distance of 25 kilometres down into a valley.\n\nDue to excessive rain she had to hike an extra 15% further on her trip back home.\n\nWhat distance did Dora travel on the trip home?\n"},{"varval":"<div class=\"aligned\">\n\n| | |\n| --------------------- | ---------------- |\n| $\\text{Distance}$ | = 25 + 15% × 25 |\n| | = 25 + 3.75 |\n| | = {{{correctAnswer}}} |\n\n</div>"}]}]

  1186. Number, NAPX-J4-NC01

    Terry makes 500 kilograms of concrete by combining 100 kg of cement mix with 200 kg of sand and 200 kg of aggregate. What percentage of the concrete is made up of cement mix?

    [{"vars":null}]

  1187. Number, NAPX-H3-NC14

    Bernard wants to buy a tennis racquet that has a normal price of \$220. He is given a 10% discount. What is the value of the discount?

    [{"vars":null}]

  1188. Number, NAPX-F4-NC03, NAPX-F3-NC05

    A circle is divided into 5 equal areas and labelled, as shown in the diagram below. <br> sm_img https://teacher.smartermaths.com.au/wp-content/uploads/2018/06/NAPX-F4-NC03.svg 200 indent3 vpad <br> What percentage of the circle's area has been labelled with an even number?

    [{"vars":null}]

  1189. Number, NAPX-H3-CA10 QUICK FIX Joe the spacing of lower coding is preferred in all similar examples going forward. PLEASE SEE YOUR ORIGINAL CODING AND THEN MY CHANGES (the spacing issue has resulted in format changes on our side).

    A box of chocolates contains 7 wrapped chocolates and 16 unwrapped chocolates. About what percentage of the chocolates in the box are wrapped chocolates?

    [{"vars":null}]

  1190. Number, NAPX-F3-CA08

    The shaded area on this graph shows how much of a computer file has loaded. <br> sm_img https://teacher.smartermaths.com.au/wp-content/uploads/2018/08/NAPX-F3-CA08_1.svg 548 indent vpad <br> What percentage of the file has loaded?

    [{"vars":null}]

  1191. Number, NAP-42694

    Tony is making a fruit cake. The recipe says he needs 3 cups of sultanas for every 5 cups of flour. If 2.5 cups of flour are used, how many cups of sultanas are needed?

    [{"vars":null}]

  1192. Number, NAP-96744

    The colour orange can be made by mixing 5 litres of yellow paint with 8 litres of red paint. If 140 litres of yellow paint are used in this mixture, how much red paint is needed?

    [{"vars":null}]

  1193. Number, NAP-D4-CA21

    <div class="sm_mode"> {{{question}}} </div>

    [{"vars":[{"varval":"Which of these percentages is closest in value to $\\dfrac{5}{9}$?"},{"varval":"<div class=\"aligned\">\n\n> > | | |\n> > | ------------------ | ----------|\n> > | $\\dfrac{5}{9}$ | = 55.55…% |\n> > | | $\\approx$ {{correctAnswer}} |\n\n</div>"}]},{"vars":[{"varval":"Which of these percentages is closest in value to $\\dfrac{4}{7}$?"},{"varval":"<div class=\"aligned\">\n\n> > | | |\n> > | ------------------ | ----------|\n> > | $\\dfrac{4}{7}$ | = 57.14…% |\n> > | | $\\approx$ {{correctAnswer}} |\n\n</div>"}]},{"vars":[{"varval":"Which of these percentages is closest in value to $\\dfrac{8}{9}$?"},{"varval":"<div class=\"aligned\">\n\n> > | | |\n> > | ------------------ | ----------|\n> > | $\\dfrac{8}{9}$ | = 88.88…% |\n> > | | $\\approx$ {{correctAnswer}} |\n\n</div>"}]},{"vars":[{"varval":"Which of these percentages is closest in value to $\\dfrac{6}{11}$?"},{"varval":"<div class=\"aligned\">\n\n> > | | |\n> > | ------------------ | ----------|\n> > | $\\dfrac{6}{11}$ | = 54.54…% |\n> > | | $\\approx$ {{correctAnswer}} |\n\n</div>"}]},{"vars":[{"varval":"Which of these percentages is closest in value to $\\dfrac{7}{9}$?"},{"varval":"<div class=\"aligned\">\n\n> > | | |\n> > | ------------------ | ----------|\n> > | $\\dfrac{7}{9}$ | = 77.77…% |\n> > | | $\\approx$ {{correctAnswer}} |\n\n</div>"}]},{"vars":[{"varval":"Which of these percentages is closest in value to $\\dfrac{6}{7}$?"},{"varval":"<div class=\"aligned\">\n\n> > | | |\n> > | ------------------ | ----------|\n> > | $\\dfrac{6}{7}$ | = 85.71…% |\n> > | | $\\approx$ {{correctAnswer}} |\n\n</div>"}]}]

  1194. Number, NAP-C4-CA21

    <div class="sm_mode"> {{{question}}} </div>

    [{"vars":[{"varval":"Anna goes on a trail hike that is 3150 metres long.\n\n<br>\n\nsm_img //teacher.smartermaths.com.au/wp-content/uploads/2017/05/naplan-2010-21mc.png 520 indent vpad\n\n<br>\n\nAnna makes it to Lookout 2.\n\nWhat percentage of the trail's total distance is left until she finishes?\n"},{"varval":"<div class=\"aligned\">\n\nsm_nogap Percentage left to travel\n\n>| | \n| ----------------------- | \n| = $\\dfrac{(3150 - 2650)}{3150} \\times 100$|\n|||\n| = $\\dfrac{500}{3150} \\times 100$|\n|||\n| = {{{correctAnswer}}}|\n\n</div>\n"}]},{"vars":[{"varval":"Zeus goes on a trail hike that is 3150 metres long.\n\n<br>\n\nsm_img //teacher.smartermaths.com.au/wp-content/uploads/2017/05/naplan-2010-21mc.png 520 indent vpad\n\n<br>\n\nZeus makes it to Lookout 1.\n\nWhat percentage of the trail's total distance is left until he finishes?\n"},{"varval":"<div class=\"aligned\">\n\nsm_nogap Percentage left to travel\n\n>| | \n| ----------------------- | \n| = $\\dfrac{(3150 - 1200)}{3150} \\times 100$|\n|||\n| = $\\dfrac{1950}{3150} \\times 100$|\n|||\n| = {{{correctAnswer}}}|\n\n</div>\n"}]},{"vars":[{"varval":"Bindi goes on a trail hike that is 3150 metres long.\n\n<br>\n\nsm_img //teacher.smartermaths.com.au/wp-content/uploads/2017/05/naplan-2010-21mc.png 520 indent vpad\n\n<br>\n\nBindi makes it to Lookout 1.\n\nWhat percentage of the trail's total distance has she completed so far?\n"},{"varval":"<div class=\"aligned\">\n\nsm_nogap Percentage completed\n\n>| | \n| ----------------------- | \n| = $\\dfrac{1200}{3150} \\times 100$|\n| = {{{correctAnswer}}}|\n\n</div>"}]},{"vars":[{"varval":"Boris goes on a trail hike that is 3150 metres long.\n\n<br>\n\nsm_img //teacher.smartermaths.com.au/wp-content/uploads/2017/05/naplan-2010-21mc.png 520 indent vpad\n\n<br>\n\nBoris makes it to Lookout 2.\n\nWhat percentage of the trail's total distance has he completed so far?\n"},{"varval":"<div class=\"aligned\">\n\nsm_nogap Percentage completed\n\n>| | \n| ----------------------- | \n| = $\\dfrac{2650}{3150} \\times 100$|\n| = {{{correctAnswer}}}|\n\n</div>"}]},{"vars":[{"varval":"Michaela goes on a trail hike that is 4200 metres long.\n\n<br>\n\nsm_img //teacher.smartermaths.com.au/wp-content/uploads/2017/05/naplan-2010-21mc.png 520 indent vpad\n\n<br>\n\nMichaela makes it to Lookout 2.\n\nWhat percentage of the trail's total distance is left until she finishes?\n"},{"varval":"<div class=\"aligned\">\n\nsm_nogap Percentage left to travel\n\n>| | \n| ----------------------- | \n| = $\\dfrac{(4200 - 2650)}{4200} \\times 100$|\n|||\n| = $\\dfrac{1550}{4200} \\times 100$|\n|||\n| = {{{correctAnswer}}}|\n\n</div>"}]},{"vars":[{"varval":"Buster goes on a trail hike that is 3560 metres long.\n\n<br>\n\nsm_img //teacher.smartermaths.com.au/wp-content/uploads/2017/05/naplan-2010-21mc.png 520 indent vpad\n\n<br>\n\nBuster makes it to Lookout 2.\n\nWhat percentage of the trail's total distance is left until he finishes?\n"},{"varval":"<div class=\"aligned\">\n\nsm_nogap Percentage left to travel\n\n>| | \n| ----------------------- | \n| = $\\dfrac{(3560 - 2650)}{3560} \\times 100$|\n| = $\\dfrac{910}{3560} \\times 100$|\n| = {{{correctAnswer}}}|\n\n</div>"}]}]

  1195. Number, NAP-B4-NC17

    <div class="sm_mode"> {{{question}}} </div>

    [{"vars":[{"varval":"A local soccer club has 1600 fans.\n\nAt a game, one-quarter of the fans wear a yellow jersey and the rest wear red.\n\n25% of the red jerseys have a black stripe down the back.\n\nHow many red jerseys have a black stripe?\n"},{"varval":"<div class=\"aligned\">\n\n| | |\n| --------------------- | -------------------------------------------- |\n| $\\text{Total red jerseys}$ | = $\\dfrac{3}{4} \\times 1600$ |\n| | = 1200 |\n\n</div>\n\n<br>\n\n<div class=\"aligned\">\n\n<div class=\"no-margin-bottom\">\n\n$\\therefore$ Red jerseys with black stripe\n\n</div>\n\n>>| |\n| --------------------- | \n| = 0.25 × 1200|\n| = {{{correctAnswer}}}|\n\n</div>"}]},{"vars":[{"varval":"A rock band fan club has 2000 members.\n\nOne-fifth of the members have seen the band once and the rest have seen the band more than once.\n\n40% of those who have seen the band more than once are over 35 years old.\n\nHow many of those who have seen the band more than once are over 35 years old?"},{"varval":"<div class=\"aligned\">\n\n| | |\n| --------------------- | -------------------------------------------- |\n| $\\text{Total seen band > 1}$ | = $\\dfrac{4}{5} \\times 2000$ |\n| | = 1600 |\n\n</div>\n\n<br>\n\n<div class=\"aligned\">\n\n<div class=\"no-margin-bottom\">\n\n$\\therefore$ Seen the band > 1 and over 35\n\n</div>\n\n>>| |\n| --------------------- | \n| = 0.40 × 1600|\n| = {{{correctAnswer}}}|\n\n</div>"}]},{"vars":[{"varval":"A local netball club has 600 fans.\n\nAt a game, one-third of the fans wear a blue hat and the rest wear white.\n\n30% of the white hats have a black logo on the front.\n\nHow many white hats have a black logo?\n"},{"varval":"<div class=\"aligned\">\n\n| | |\n| --------------------- | -------------------------------------------- |\n| $\\text{Total white hats}$ | = $\\dfrac{2}{3} \\times 600$ |\n| | = 400 |\n\n</div>\n\n<br>\n\n<div class=\"aligned\">\n\n<div class=\"no-margin-bottom\">\n\n$\\therefore$ White hats with black logo\n\n</div>\n\n>>| |\n| --------------------- | \n| = 0.30 × 400|\n| = {{{correctAnswer}}}|\n\n</div>"}]},{"vars":[{"varval":"A peak hour Sydney Metro train has a maximum load of 2000 passengers.\n\nOf those passengers, one-fifth are seated and the rest are standing.\n\n40% of those standing are on the train for longer than 10 minutes.\n\nHow many standing passengers are on the train for longer than 10 minutes?\n"},{"varval":"<div class=\"aligned\">\n\n| | |\n| --------------------- | -------------------------------------------- |\n| $\\text{Total standing passengers}$ | = $\\dfrac{4}{5} \\times 2000$ |\n| | = 1600 |\n\n</div>\n\n<br>\n\n<div class=\"aligned\">\n\n<div class=\"no-margin-bottom\">\n\n$\\therefore$ Standing passengers on the train for longer than 10 minutes\n\n</div>\n\n>>| |\n| --------------------- | \n| = 0.40 × 1600|\n| = {{{correctAnswer}}}|\n\n</div>"}]},{"vars":[{"varval":"A light globe factory produces 1500 globes per day.\n\nOne-third of the globes are halogen and the rest are LED.\n\n20% of the LED globes produced are soft yellow in colour.\n\nHow many of the LED globes are soft yellow in colour?\n"},{"varval":"<div class=\"aligned\">\n\n| | |\n| --------------------- | -------------------------------------------- |\n| $\\text{Total LED globes}$ | = $\\dfrac{2}{3} \\times 1500$ |\n| | = 1000 |\n\n</div>\n\n<br>\n\n<div class=\"aligned\">\n\n<div class=\"no-margin-bottom\">\n\n$\\therefore$ LED globes soft yellow in colour\n</div>\n\n>>| |\n| --------------------- | \n| = 0.20 × 1000|\n| = {{{correctAnswer}}}|\n\n</div>"}]},{"vars":[{"varval":"A hiking group has 125 members and they embark on a 100 kilometre hike.\n\nOnly one-fifth of the hikers complete the entire distance.\n\n60% of the hikers who did not finish completed less than 75 kilometres.\n\nHow many of the hikers who did not finish completed less than 75 kilometres?"},{"varval":"<div class=\"aligned\">\n\n| | |\n| --------------------- | -------------------------------------------- |\n| $\\text{Total that did not finish}$ | = $\\dfrac{4}{5} \\times 125$ |\n| | = 100 |\n\n</div>\n\n<br>\n\n<div class=\"aligned\">\n\n<div class=\"no-margin-bottom\">\n\n$\\therefore$ Hikers who completed less than 75 kms\n\n</div>\n\n>>| |\n| --------------------- | \n| = 0.60 × 100|\n| = {{{correctAnswer}}}|\n\n</div>"}]}]

  1196. Number, NAP-F4-CA18

    Ryan's tennis racquet is pictured below. <br> sm_img //teacher.smartermaths.com.au/wp-content/uploads/2017/01/naplan-2013-18mc.png 455 indent3 vpad <br> His handle is 19 cm in length. Which of the following show the length of the handle as a percentage of the total length of the racquet?

    [{"vars":null}]

  1197. Number, NAP-K4-NC02

    <div class="sm_mode"> {{{question}}} </div>

    [{"vars":[{"varval":"Blart has a device that counts his steps each day.\n\nHe aims to complete 10 000 steps each day.\n\nThe fraction of steps he has completed toward his target one day is  $\\dfrac{6700}{10\\ 000}$.\n\nWhat percentage of steps does Blart need to complete to reach his target?\n"},{"varval":"Steps completed = 6700 ÷ 10 000 = 0.67\n\n<div class=\"no-margin-bottom\">\n\n$\\therefore$ Percentage of steps to complete\n\n</div>\n\n> > \\= 1 − 0.67\\\n> > = 0.33\\\n> > = {{{correctAnswer}}}"}]},{"vars":[{"varval":"Anna's smart watch records the kilometres she runs in her morning training session.\n\nHer aim is to complete 10 kilometres in today's session.\n\nThe fraction of kilometres she has completed toward today's target is  $\\dfrac{8.5}{10}$.\n\nWhat percentage of kilometres does Anna need to complete to reach her target?\n"},{"varval":"Kilometres completed = 8.5 ÷ 10 = 0.85\n\n<div class=\"no-margin-bottom\">\n\n$\\therefore$ Percentage of kilometres to complete\n\n</div>\n\n> > \\= 1 − 0.85\\\n> > = 0.15\\\n> > = {{{correctAnswer}}}"}]},{"vars":[{"varval":"Julian has a device that counts his steps each day.\n\nHe aims to complete 5 000 steps each day.\n\nThe fraction of steps he has completed toward his target one day is  $\\dfrac{2400}{5000}$.\n\nWhat percentage of steps does Julian need to complete to reach his target?\n"},{"varval":"Steps completed = 2400 ÷ 5000 = 0.48\n\n<div class=\"no-margin-bottom\">\n\n$\\therefore$ Percentage of steps to complete\n\n</div>\n\n> > \\= 1 − 0.48\\\n> > = 0.52\\\n> > = {{{correctAnswer}}}"}]},{"vars":[{"varval":"Shakespeare is writing a new play.\n\nHe aims to write 25 000 words per day.\n\nThe fraction of words he has completed toward his target one day is  $\\dfrac{17\\ 580}{25\\ 000}$.\n\nWhat percentage of words does Shakespeare need to complete to reach his target?\n"},{"varval":"Words completed = 17 580 ÷ 25 000 = 0.7032\n\n<div class=\"no-margin-bottom\">\n\n$\\therefore$ Percentage of words to complete\n\n</div>\n\n> > \\= 1 − 0.7032\\\n> > = 0.2968\\\n> > = {{{correctAnswer}}}"}]},{"vars":[{"varval":"Ford is a car salesman.\n\nHe aims to sell $270 000 worth of cars each month.\n\nThe fraction of sales he has completed toward his target this month is  $\\dfrac{201\\ 987}{270\\ 000}$.\n\nWhat percentage of sales does Ford need to complete to reach his target this month?\n"},{"varval":"Sales achieved = 201 987 ÷ 270 000 = 0.7481\n\n<div class=\"no-margin-bottom\">\n\n$\\therefore$ Percentage of sales to complete\n\n</div>\n\n> > \\= 1 − 0.7481\\\n> > = 0.2519\\\n> > = {{{correctAnswer}}}"}]},{"vars":[{"varval":"A factory produces electronic components.\n\nThe expected number of components produced each shift is 1500.\n\nThe fraction of components produced in a shift is $\\dfrac{1050}{1500}$.\n\nWhat percentage of components does the factory need to complete to reach the target?\n"},{"varval":"Components completed = 1050 ÷ 1500 = 0.7\n\n<div class=\"no-margin-bottom\">\n\n$\\therefore$ Percentage of components to complete\n\n</div>\n\n> > \\= 1 − 0.7\\\n> > = 0.3\\\n> > = {{{correctAnswer}}}"}]}]

  1198. Number, NAP-G4-NC16

    A birthday cake is cut into 12 equal pieces. Only the shaded pieces below are left. <br> sm_img //teacher.smartermaths.com.au/wp-content/uploads/2016/12/NAP-149-NC161.png 205 indent3 vpad <br> What percentage represents the amount of cake left?

    [{"vars":null}]

  1199. #29

    The number of {{subject1}} in {{where1}} and {{where2}} during an 8 week period is recorded and the results are graphed below. <br> {{image}} <br> In which 2 weeks did {{where3}} record less {{subject2}} than {{where4}}?

    [{"vars":[{"varval":"babies born"},{"varval":"Hospital A"},{"varval":"Hospital B"},{"varval":"sm_img https://teacher.smartermaths.com.au/wp-content/uploads/2020/07/q29var1_r.svg 630 indent vpad"},{"varval":"Hospital B"},{"varval":"babies being born"},{"varval":"Hospital A"}]},{"vars":[{"varval":"babies born"},{"varval":"Hospital A"},{"varval":"Hospital B"},{"varval":"sm_img https://teacher.smartermaths.com.au/wp-content/uploads/2020/07/q29var2_r.svg 630 indent vpad"},{"varval":"Hospital B"},{"varval":"babies being born"},{"varval":"Hospital A"}]},{"vars":[{"varval":"bush fires"},{"varval":"Town A"},{"varval":"Town B"},{"varval":"sm_img https://teacher.smartermaths.com.au/wp-content/uploads/2020/07/q29var3_r.svg 630 indent vpad"},{"varval":"Town B"},{"varval":"bush fires"},{"varval":"Town A"}]},{"vars":[{"varval":"bush fires"},{"varval":"Town A"},{"varval":"Town B"},{"varval":"sm_img https://teacher.smartermaths.com.au/wp-content/uploads/2020/07/q29var4_r.svg 630 indent vpad"},{"varval":"Town B"},{"varval":"bush fires"},{"varval":"Town A"}]},{"vars":[{"varval":"murders"},{"varval":"Mexico City"},{"varval":"San Paolo"},{"varval":"sm_img https://teacher.smartermaths.com.au/wp-content/uploads/2020/07/q29var5_r.svg 680 indent vpad"},{"varval":"San Paolo"},{"varval":"murders"},{"varval":"Mexico City"}]}]

  1200. #28

    {{name}} is transporting {{pieces}} {{item1}} to his {{place}}. The {{item2}} are transported in containers that fit {{number1}} {{item2}}. How many containers does {{name}} need to transport all of the {{item2}}?

    [{"vars":[{"varval":"Oscar"},{"varval":"140"},{"varval":"macadamia nut trees"},{"varval":"farm"},{"varval":"trees"},{"varval":"eight"},{"varval":"17.5"},{"varval":"8"}]},{"vars":[{"varval":"Nathan"},{"varval":"150"},{"varval":"lemon trees"},{"varval":"orchard"},{"varval":"trees"},{"varval":"seven"},{"varval":"21.4"},{"varval":"7"}]},{"vars":[{"varval":"Raymond"},{"varval":"180"},{"varval":"electrical transformers"},{"varval":"factory"},{"varval":"transformers"},{"varval":"eight"},{"varval":"22.5"},{"varval":"8"}]},{"vars":[{"varval":"Panus"},{"varval":"170"},{"varval":"tennis nets"},{"varval":"distribution warehouse"},{"varval":"nets"},{"varval":"six"},{"varval":"28.3"},{"varval":"6"}]},{"vars":[{"varval":"Rivaldi"},{"varval":"130"},{"varval":"orange trees"},{"varval":"orchard"},{"varval":"trees"},{"varval":"four"},{"varval":"32.5"},{"varval":"4"}]}]

  1201. Number, NAP-A4-CA14

    The population of Australian states in 2015 and 2016 is recorded in the table below. Some data for Western Australia is not shown. <br> sm_img https://teacher.smartermaths.com.au/wp-content/uploads/2017/12/NAP-A4-CA14_1.svg 550 indent3 vpad <br> What was the population of Western Australia (WA) close to in 2016?

    [{"vars":null}]

  1202. Number, NAP-A4-CA09

    <div class="sm_mode"> {{{question}}} </div>

    [{"vars":[{"varval":"There were only 17 students in Grace's class on Wednesday. The other 8 were absent.\n\nWhat percentage of Grace's class was absent?\n"},{"varval":"<div class=\"aligned\">\n\n| | |\n| --------------------------- | ------------- |\n| $\\text{Total in class}$ | = 17 + 8 |\n| | = 25 |\n\n</div>\n\n<br>\n\n<div class=\"aligned\">\n\n<div class=\"no-margin-bottom\">\n\n$\\therefore$ Percentage absent\n\n</div>\n\n>>| |\n|--------------------------- |\n|= $\\dfrac{8}{25}$ x 100 |\n|= {{{correctAnswer}}} |\n\n</div>"}]},{"vars":[{"varval":"There were only 90 Year 9 students at school on Monday. The other 60 were absent.\n\nWhat percentage of Year 9 was absent?\n"},{"varval":"<div class=\"aligned\">\n\n| | |\n| --------------------------- | ------------- |\n| $\\text{Total in year}$ | = 90 + 60 |\n| | = 150 |\n\n</div>\n\n<br>\n\n<div class=\"aligned\">\n\n<div class=\"no-margin-bottom\">\n\n$\\therefore$ Percentage absent\n\n</div>\n\n>>| |\n|--------------------------- |\n|= $\\dfrac{60}{150}$ x 100 |\n|= {{{correctAnswer}}} |\n\n</div>"}]},{"vars":[{"varval":"There were 80 people seated on the train. The other 120 seats were empty.\n\nWhat percentage of the seats were empty?\n"},{"varval":"<div class=\"aligned\">\n\n| | |\n| --------------------------- | ------------- |\n| $\\text{Total seats}$ | = 80 + 120 |\n| | = 200 |\n\n</div>\n\n<br>\n\n<div class=\"aligned\">\n\n<div class=\"no-margin-bottom\">\n\n$\\therefore$ Percentage of empty seats\n\n</div>\n\n>>| |\n|--------------------------- |\n|= $\\dfrac{120}{200}$ x 100 |\n|= {{{correctAnswer}}} |\n\n</div>"}]},{"vars":[{"varval":"A charity barbeque provided sausage sandwiches with or without sauce. \n\n45 of the sandwiches were sold with sauce and the other 75 had no sauce.\n\nWhat percentage of the sausage sandwiches were sold with no sauce?\n"},{"varval":"<div class=\"aligned\">\n\n| | |\n| --------------------------- | ------------- |\n| $\\text{Total sandwiches}$ | = 45 + 75 |\n| | = 120 |\n\n</div>\n\n<br>\n\n<div class=\"aligned\">\n\n<div class=\"no-margin-bottom\">\n\n$\\therefore$ Percentage with no sauce\n\n</div>\n\n>>| |\n|--------------------------- |\n|= $\\dfrac{75}{120}$ x 100 |\n|= {{{correctAnswer}}} |\n\n</div>"}]},{"vars":[{"varval":"Blinky was taking two days to drive to his holiday destination. \n\nOn the first day he travelled 900 kilometres and on the second day he travelled the remaining 300 kilometres.\n\nWhat percentage of the trip did he travel on the second day?\n"},{"varval":"<div class=\"aligned\">\n\n| | |\n| --------------------------- | ------------- |\n| $\\text{Total kilometres}$ | = 900 + 300 |\n| | = 1200 |\n\n</div>\n\n<br>\n\n<div class=\"aligned\">\n\n<div class=\"no-margin-bottom\">\n\n$\\therefore$ Percentage travelled on second day\n\n</div>\n\n>>| |\n|--------------------------- |\n|= $\\dfrac{300}{1200}$ x 100 |\n|= {{{correctAnswer}}} |\n\n</div>"}]},{"vars":[{"varval":"Bruce was collecting cans and bottles for recycling. \n\nHe collected 144 cans and 56 bottles.\n\nWhat percentage of the recycling was bottles?\n"},{"varval":"<div class=\"aligned\">\n\n| | |\n| --------------------------- | ------------- |\n| $\\text{Total recycling}$ | = 144 + 56 |\n| | = 200 |\n\n</div>\n\n<br>\n\n<div class=\"aligned\">\n\n<div class=\"no-margin-bottom\">\n\n$\\therefore$ Percentage of bottles\n\n</div>\n\n>>| |\n|--------------------------- |\n|= $\\dfrac{56}{200}$ x 100 |\n|= {{{correctAnswer}}} |\n\n</div>"}]}]

  1203. Number, NAP-F4-CA06

    <div class="sm_mode"> {{{question}}} </div>

    [{"vars":[{"varval":"James weighed 80 kg when he was 18 years old. \n\nOn his 21st birthday, James weighs 5% more than when he was 18.\n\nHow much does he weigh now?\n"},{"varval":"<div class=\"aligned\">\n\n| | |\n| -------------- | -------- |\n| 80 + (80 × 5%) | = 80 + 4 |\n| | = {{correctAnswer}} |\n\n</div>"}]},{"vars":[{"varval":"Kerry's puppy weighed 15 kg when he was 6 months old. \n\nOn his 1st birthday, the puppy weighs 45% more than when he was 6 months old.\n\nHow much does the puppy weigh now?\n"},{"varval":"<div class=\"aligned\">\n\n| | |\n| -------------- | -------- |\n|15 + (15 × 45%) | = 15 + 6.75 |\n| | = {{correctAnswer}} |\n\n</div>"}]},{"vars":[{"varval":"A male humpback whale is 4 metres long when it is born. \n\nIt increases in length by 205% by the time it is an adult.\n\nHow long is an adult humpback whale?\n"},{"varval":"<div class=\"aligned\">\n\n| | |\n| -------------- | -------- |\n| 4 + (4 × 205%) | = 4 + 8.2 |\n| | = {{correctAnswer}} |\n\n</div>"}]},{"vars":[{"varval":"A pot weighed 12 kg when empty. \n\nWhen filled with garden soil and plants, the pot weighs 35% more than when empty.\n\nHow much does the filled pot weigh?\n"},{"varval":"<div class=\"aligned\">\n\n| | |\n| -------------- | -------- |\n| 12 + (12 × 35%) | = 12 + 4.2 |\n| | = {{correctAnswer}} |\n\n</div>"}]},{"vars":[{"varval":"A seedling is 4 centimetres tall after 2 weeks. \n\nAfter 5 weeks it has increased in height by 175%.\n\nHow tall is the seedling now?\n"},{"varval":"<div class=\"aligned\">\n\n| | |\n| -------------- | -------- |\n| 4 + (4 × 175%) | = 4 + 7 |\n| | = {{correctAnswer}} |\n\n</div>"}]},{"vars":[{"varval":"A tree was 15 metres tall two years ago. \n\nIt was measured this year and its height had increased by 64%.\n\nHow tall is the tree now?\n"},{"varval":"<div class=\"aligned\">\n\n| | |\n| -------------- | -------- |\n| 15 + (15 × 64%) | = 15 + 9.6 |\n| | = {{correctAnswer}} |\n\n</div>"}]}]

  1204. Number, NAP-J4-NC01

    500 kilograms of a concrete material is made by mixing 200 kilograms of cement with 300 kilograms of sand. What percentage of the concrete is made up of sand?

    [{"vars":null}]

  1205. Number, NAP-L4-CA04

    In 2015, some wilderness parks in Tasmania lost up to $\dfrac{8}{10}$ of their Tasmanian devil populations. What is $\dfrac{8}{10}$ as percentage?

    [{"vars":null}]

  1206. Number, NAP-F4-NC03

    A circle is divided into 5 equal areas and labelled, as shown in the diagram below. <br> sm_img //teacher.smartermaths.com.au/wp-content/uploads/2016/12/NAP-F4-NC03.png 205 indent3 vpad <br> What percentage of the circle's area has been labelled with an odd number?

    [{"vars":null}]

  1207. #27

    {{name}} catches the {{time1}} {{transport1}} from {{place1}} to {{place2}} for work. After he wakes up, it takes him {{min1}} minutes to shower and have breakfast, {{min2}} minutes to ride his bike to the {{transport2}} and {{min3}} minutes to lock up his bike up. If {{name}} wants to get to the {{transport1}} {{early}} minutes early, what is the latest time he can wake up?

    [{"vars":[{"varval":"Brad"},{"varval":"7:22 am"},{"varval":"ferry"},{"varval":"Manly"},{"varval":"Sydney"},{"varval":"42"},{"varval":"6"},{"varval":"ferry terminal"},{"varval":"3"},{"varval":"5"},{"varval":"7:17 am"},{"varval":"51"}]},{"vars":[{"varval":"Ryan"},{"varval":"8:12 am"},{"varval":"ferry"},{"varval":"Drummoyne"},{"varval":"Sydney"},{"varval":"38"},{"varval":"14"},{"varval":"ferry terminal"},{"varval":"4"},{"varval":"3"},{"varval":"8:09 am"},{"varval":"56"}]},{"vars":[{"varval":"Vladimir"},{"varval":"6:32 am"},{"varval":"train"},{"varval":"Penrith"},{"varval":"North Sydney"},{"varval":"28"},{"varval":"21"},{"varval":"train station"},{"varval":"4"},{"varval":"2"},{"varval":"6:30 am"},{"varval":"53"}]},{"vars":[{"varval":"Igor"},{"varval":"7:23 am"},{"varval":"train"},{"varval":"Gosford"},{"varval":"Central Station"},{"varval":"34"},{"varval":"13"},{"varval":"train station"},{"varval":"2"},{"varval":"4"},{"varval":"7:19 am"},{"varval":"49"}]},{"vars":[{"varval":"Matias"},{"varval":"9:18 am"},{"varval":"ferry"},{"varval":"Newcastle"},{"varval":"Stockton"},{"varval":"41"},{"varval":"12"},{"varval":"ferry terminal"},{"varval":"4"},{"varval":"3"},{"varval":"9:15 am"},{"varval":"57"}]}]

  1208. Number, NAP-I4-NC25

    Kate has read $\dfrac{3}{8}$ of the first Harry Potter novel. Which decimal shows how much of the book Kate has read?

    [{"vars":null}]

  1209. Number, NAP-E4-NC18

    <div class="sm_mode"> {{{question}}} </div>

    [{"vars":[{"varval":"36 $\\times$ <span class=\"sm-text color3\">?</span> = 20"},{"varval":"<div class=\"sm_mode\">\n\n<div class=\"aligned\">\n\n| | |\n| --------------------- :| -------------------------------------------- |\n| 36 $\\times$ <span class=\"sm-text color3\">?</span> | = 20 |\n| <span class=\"sm-text color3\">?</span> | = $\\dfrac{20}{36}$ |\n| <span class=\"sm-text color3\">?</span> | = $\\dfrac{5}{9}$ |\n\n</div>\n\n</div>"}]},{"vars":[{"varval":"55 $\\times$ <span class=\"sm-text color3\">?</span> = 15"},{"varval":"<div class=\"sm_mode\">\n\n<div class=\"aligned\">\n\n| | |\n| --------------------- :| -------------------------------------------- |\n| 55 $\\times$ <span class=\"sm-text color3\">?</span> | = 15 |\n| <span class=\"sm-text color3\">?</span> | = $\\dfrac{15}{55}$ |\n| <span class=\"sm-text color3\">?</span> | = $\\dfrac{3}{11}$ |\n\n</div>\n\n</div>"}]},{"vars":[{"varval":"72 $\\times$ <span class=\"sm-text color8\">?</span> = 60"},{"varval":"<div class=\"sm_mode\">\n\n<div class=\"aligned\">\n\n| | |\n| --------------------- :| -------------------------------------------- |\n| 72 $\\times$ <span class=\"sm-text color8\">?</span> | = 60 |\n| <span class=\"sm-text color8\">?</span> | = $\\dfrac{60}{72}$ |\n| <span class=\"sm-text color8\">?</span> | = $\\dfrac{5}{6}$ |\n\n\n</div>\n\n</div>"}]},{"vars":[{"varval":"56 $\\times$ <span class=\"sm-text color8\">?</span> = 35"},{"varval":"<div class=\"sm_mode\">\n\n<div class=\"aligned\">\n\n| | |\n| --------------------- :| -------------------------------------------- |\n| 56 $\\times$ <span class=\"sm-text color8\">?</span> | = 35 |\n| <span class=\"sm-text color8\">?</span> | = $\\dfrac{35}{56}$ |\n| <span class=\"sm-text color8\">?</span> | = $\\dfrac{5}{8}$ |\n\n</div>\n\n</div>"}]},{"vars":[{"varval":"250 $\\times$ <span class=\"sm-text color4\">?</span> = 75"},{"varval":"<div class=\"sm_mode\">\n\n<div class=\"aligned\">\n\n| | |\n| --------------------- :| -------------------------------------------- |\n| 250 $\\times$ <span class=\"sm-text color4\">?</span> | = 75 |\n| <span class=\"sm-text color4\">?</span> | = $\\dfrac{75}{250}$ |\n| <span class=\"sm-text color4\">?</span> | = {{{correctAnswer}}} |\n\n</div>\n\n</div>"}]},{"vars":[{"varval":"28 $\\times$ <span class=\"sm-text color4\">?</span> = 8"},{"varval":"<div class=\"sm_mode\">\n\n<div class=\"aligned\">\n\n| | |\n| --------------------- :| -------------------------------------------- |\n| 28 $\\times$ <span class=\"sm-text color4\">?</span> | = 8 |\n| <span class=\"sm-text color4\">?</span> | = $\\dfrac{8}{28}$ |\n| <span class=\"sm-text color4\">?</span> | = {{{correctAnswer}}}|\n\n</div>\n\n</div>"}]}]

  1210. Number, NAP-H4-NC17

    <div class="sm_mode"> {{{question}}} </div>

    [{"vars":[{"varval":"Which of these is the closest to 1?"},{"varval":"Consider the difference of each option:\n\n$1 - 0.9 = 0.1$\n\n$1 - 0.99 = 0.01$\n\n$1.01 - 1 = 0.01$\n\n$1.001 - 1 = 0.001$ &nbsp;$\\checkmark$\n\n\n$\\therefore$ {{{correctAnswer}}} is the closest"}]},{"vars":[{"varval":"Which of these is the closest to 1?"},{"varval":"Consider the difference of each option:\n\n$1 - 0.98 = 0.02$ &nbsp;$\\checkmark$\n\n$1 - 0.9 = 0.1$\n\n$1.12 - 1 = 0.12$\n\n$1.021 - 1 = 0.021$\n\n\n$\\therefore$ {{{correctAnswer}}} is the closest"}]},{"vars":[{"varval":"Which of these is the closest to 10?"},{"varval":"Consider the difference of each option:\n\n$10 - 9.9 = 0.1$\n\n$10 - 9.97 = 0.03$\n\n$10.008 - 10 = 0.008$ &nbsp;$\\checkmark$\n\n$10.05 - 10 = 0.05$\n\n$\\therefore$ {{{correctAnswer}}} is the closest"}]},{"vars":[{"varval":"Which of these is the closest to 10?"},{"varval":"Consider the difference of each option:\n\n$10.00 - 9.98 = 0.02$\n\n$10.000 - 9.998 = 0.002$ &nbsp;$\\checkmark$\n\n$10.2 - 10.0 = 0.2$\n\n$10.02 - 10.00 = 0.02$\n\n\n$\\therefore$ {{{correctAnswer}}} is the closest"}]},{"vars":[{"varval":"Which of these is the closest to 100?"},{"varval":"Consider the difference of each option:\n\n$100.00 - 98.3 = 1.7$\n\n$100.00 - 98.33 = 1.67$\n\n$101.67 - 100.00 = 1.67$\n\n$101.067 - 100.000 = 1.067$ &nbsp;$\\checkmark$\n\n\n$\\therefore$ {{{correctAnswer}}} is the closest"}]},{"vars":[{"varval":"Which of these is the closest to 99?"},{"varval":"Consider the difference of each option:\n\n$99.0 - 98.9 = 0.1$ &nbsp;$\\checkmark$\n\n$99.00 - 98.09 = 0.91$\n\n$99.91 - 99.00 = 0.91$\n\n$100.001 - 99.00 = 1.001$\n\n\n$\\therefore$ {{{correctAnswer}}} is the closest"}]}]

  1211. Number, NAP-E4-CA17 This question is the same as c50eb14c-1028-447a-a9d7-23e60ca82bd9v0 and 4f38ef18-f081-48f8-b36c-c883052da6d7

    <div class="sm_mode"> {{{question}}} </div>

    [{"vars":[{"varval":"Ali has a bag of marbles. The marbles are either blue, black or orange.\n\n$\\dfrac{1}{6}$ of her marbles are blue and $\\dfrac{1}{4}$ are black.\n\nWhat fraction of her marbles are orange?\n"},{"varval":"sm_nogap Fraction of orange marbles\n\n<div class=\"aligned\">\n\n> > | | |\n> > | --- | ----------------------------------------------------- |\n> > | | \\= $1 - \\bigg( \\dfrac{1}{6} + \\dfrac{1}{4} \\bigg)$ |\n> > | | \\= $1 - \\bigg( \\dfrac{2}{12} + \\dfrac{3}{12} \\bigg)$ |\n> > | | \\= $1 - \\dfrac{5}{12}$ |\n> > | | \\= {{{correctAnswer}}} |\n\n</div>"}]},{"vars":[{"varval":"Julius has a bag of sweets. The sweets are either pink, white or green.\n\n$\\dfrac{2}{7}$ of his sweets are pink and $\\dfrac{1}{3}$ are white.\n\nWhat fraction of his sweets are green?\n"},{"varval":"sm_nogap Fraction of green sweets\n\n<div class=\"aligned\">\n\n> > | | |\n> > | --- | ----------------------------------------------------- |\n> > | | \\= $1 - \\bigg( \\dfrac{2}{7} + \\dfrac{1}{3} \\bigg)$ |\n> > | | \\= $1 - \\bigg( \\dfrac{6}{21} + \\dfrac{7}{21} \\bigg)$ |\n> > | | \\= $1 - \\dfrac{13}{21}$ |\n> > | | \\= {{{correctAnswer}}} |\n\n</div>"}]},{"vars":[{"varval":"Kurt has an esky containing cans of drink. The drinks are either sparkling water, cola or lemonade.\n\n$\\dfrac{3}{4}$ of the cans are sparkling water and $\\dfrac{1}{6}$ are cola.\n\nWhat fraction of the cans are lemonade?\n"},{"varval":"sm_nogap Fraction of cans of lemonade\n\n<div class=\"aligned\">\n\n> > | | |\n> > | --- | ----------------------------------------------------- |\n> > | | \\= $1 - \\bigg( \\dfrac{3}{4} + \\dfrac{1}{6} \\bigg)$ |\n> > | | \\= $1 - \\bigg( \\dfrac{9}{12} + \\dfrac{2}{12} \\bigg)$ |\n> > | | \\= $1 - \\dfrac{11}{12}$ |\n> > | | \\= {{{correctAnswer}}} |\n\n</div>"}]},{"vars":[{"varval":"Andrew and Rory went fishing and returned home with a bag of fish. \n\nThe fish in the bag are either bream, flathead or snapper.\n\n$\\dfrac{3}{8}$ of their fish are flathead and $\\dfrac{1}{5}$ are snapper.\n\nWhat fraction of their fish are bream?\n"},{"varval":"sm_nogap Fraction of bream\n\n<div class=\"aligned\">\n\n> > | | |\n> > | --- | ----------------------------------------------------- |\n> > | | \\= $1 - \\bigg( \\dfrac{3}{8} + \\dfrac{1}{5} \\bigg)$ |\n> > | | \\= $1 - \\bigg( \\dfrac{15}{40} + \\dfrac{8}{40} \\bigg)$ |\n> > | | \\= $1 - \\dfrac{23}{40}$ |\n> > | | \\= {{{correctAnswer}}} |\n\n</div>"}]},{"vars":[{"varval":"Homer has a bag of doughnuts for Bart's birthday party. \n\nThe doughnuts are either cinnamon, choc hazelnut or strawberry jam.\n\n$\\dfrac{3}{10}$ of the doughnuts are strawberry jam and $\\dfrac{1}{5}$ are cinnamon.\n\nWhat fraction of the doughnuts are choc hazelnut?\n"},{"varval":"sm_nogap Fraction of choc hazelnut doughnuts\n\n<div class=\"aligned\">\n\n> > | | |\n> > | --- | ----------------------------------------------------- |\n> > | | \\= $1 - \\bigg( \\dfrac{3}{10} + \\dfrac{1}{5} \\bigg)$ |\n> > | | \\= $1 - \\bigg( \\dfrac{3}{10} + \\dfrac{2}{10} \\bigg)$ |\n> > | | \\= $1 - \\dfrac{5}{10}$ |\n> > | | \\= {{{correctAnswer}}} |\n\n</div>"}]},{"vars":[{"varval":"Indigo has a selection of dyes. \n\nThe dyes are either blue, red or green.\n\n$\\dfrac{1}{10}$ of her dyes are red and $\\dfrac{3}{4}$ are blue.\n\nWhat fraction of her dyes are green?"},{"varval":"sm_nogap Fraction of green dyes\n\n<div class=\"aligned\">\n\n> > | | |\n> > | --- | ----------------------------------------------------- |\n> > | | \\= $1 - \\bigg( \\dfrac{1}{10} + \\dfrac{3}{4} \\bigg)$ |\n> > | | \\= $1 - \\bigg( \\dfrac{2}{20} + \\dfrac{15}{20} \\bigg)$ |\n> > | | \\= $1 - \\dfrac{17}{20}$ |\n> > | | \\= {{{correctAnswer}}} |\n\n</div>"}]}]

  1212. Number, NAP-J4-CA18 ISSUE: Image colour

    <div class="sm_mode"> {{{question}}} </div>

    [{"vars":[{"varval":"The weight of a banana is 0.1246 kg.\n\n<br>\n\nsm_img https://teacher.smartermaths.com.au/wp-content/uploads/2017/09/NAP-J1-18-300x248.png 260 indent3 vpad\n\n<br>\n\nWhat is the weight of the banana rounded to the nearest hundredth of a kilogram?\n"},{"varval":"Nearest hundredth - look at the first 3 decimal places and round to 2.\n\n0.124 $\\Rightarrow$ {{{correctAnswer}}}"}]},{"vars":[{"varval":"The weight of a banana is 0.1246 kg.\n\n<br>\n\nsm_img https://teacher.smartermaths.com.au/wp-content/uploads/2017/09/NAP-J1-18-300x248.png 260 indent3 vpad\n\n<br>\n\nWhat is the weight of the banana rounded to the nearest tenth of a kilogram?\n"},{"varval":"Nearest tenth - look at the first 2 decimal places and round to 1.\n\n0.12 $\\Rightarrow$ {{{correctAnswer}}}"}]},{"vars":[{"varval":"The weight of a banana is 0.1246 kg.\n\n<br>\n\nsm_img https://teacher.smartermaths.com.au/wp-content/uploads/2017/09/NAP-J1-18-300x248.png 260 indent3 vpad\n\n<br>\n\nWhat is the weight of the banana rounded to the nearest thousandth of a kilogram?"},{"varval":"Nearest thousandth - look at the first 4 decimal places and round to 3.\n\n0.1246 $\\Rightarrow$ {{{correctAnswer}}}"}]},{"vars":[{"varval":"The weight of one banana is 0.1246 kg.\n\nAnthony needs 10 of these bananas to make some fruit salad. \n<br>\n\nsm_img https://teacher.smartermaths.com.au/wp-content/uploads/2017/09/NAP-J1-18-300x248.png 260 indent3 vpad\n\n<br>\n\nWhat is the weight of the ten bananas rounded to the nearest tenth of a kilogram?"},{"varval":"Weight of 10 bananas = $10 \\times 0.1246$ = 1.246\n\nNearest hundredth - look at the first 3 decimal places and round to 2.\n\n1.246 $\\Rightarrow$ {{{correctAnswer}}}"}]},{"vars":[{"varval":"The weight of one banana is 0.1246 kg.\n\nSione needs three of these bananas to make some banana bread. \n<br>\n\nsm_img https://teacher.smartermaths.com.au/wp-content/uploads/2017/09/NAP-J1-18-300x248.png 260 indent3 vpad\n\n<br>\n\nWhat is the weight of the three bananas rounded to the nearest tenth of a kilogram?"},{"varval":"Weight of 3 bananas = $3 \\times 0.1246$ = 0.3738\n\nNearest tenth - look at the first 2 decimal places and round to 1.\n\n0.37 $\\Rightarrow$ {{{correctAnswer}}}"}]}]

  1213. Number, NAP-60268

    In a science experiment, Albert needs to add 60 millilitres of acid to every 2 litres of water. If Albert only has 0.5 litres of water left, how many millilitres of acid should he add?

    [{"vars":null}]

  1214. Number, NAP-61529

    Shelly and Carly collect dolls. The ratio of the number of dolls Shelly owns compared to Carly is 3 : 2. Shelley owns 12 dolls. How many dolls does Carly own?

    [{"vars":null}]

  1215. Number, NAP-02654

    <div class="sm_mode"> {{{question}}} </div>

    [{"vars":[{"varval":"On a country property, 1 acre of land is recommended for every 4 sheep.\n\nHow many acres of land would be needed for 16 sheep?"},{"varval":"<div class=\"aligned\"> \n\n| | |\n| -----: | -------------------- |\n| Acres | \\= $\\dfrac{16}{4}$ |\n| | \\= {{{correctAnswer}}} |\n\n</div>"}]},{"vars":[{"varval":"On a country property, 1 acre of land is recommended for every 5 goats.\n\nHow many acres of land would be needed for 30 goats?"},{"varval":"<div class=\"aligned\"> \n\n| | |\n| -----: | -------------------- |\n| Acres | \\= $\\dfrac{30}{5}$ |\n| | \\= {{{correctAnswer}}} |\n\n</div>"}]},{"vars":[{"varval":"In a park, 1 square metre of land is recommended for every 2 native shrubs.\n\nHow many square metres of land would be needed for 26 native shrubs?"},{"varval":"<div class=\"aligned\"> \n\n| | |\n| -----: | -------------------- |\n| Square metres | \\= $\\dfrac{26}{2}$ |\n| | \\= {{{correctAnswer}}} |\n\n</div>"}]},{"vars":[{"varval":"In a wildlife park, 1 hectare of land is recommended for every 5 camels.\n\nHow many hectares of land would be needed for 55 camels?"},{"varval":"<div class=\"aligned\"> \n\n| | |\n| -----: | -------------------- |\n| Hectares | \\= $\\dfrac{55}{5}$ |\n| | \\= {{{correctAnswer}}} |\n\n</div>"}]},{"vars":[{"varval":"In an art gallery, 1 square metre of wall space is recommended for every 2 small paintings.\n\nHow many square metres of wall space would be needed for 80 small paintings?"},{"varval":"<div class=\"aligned\"> \n\n| | |\n| -----: | -------------------- |\n| Square metres | \\= $\\dfrac{80}{2}$ |\n| | \\= {{{correctAnswer}}} |\n\n</div>"}]},{"vars":[{"varval":"At a daycare centre, 1 room is recommended for every 15 children.\n\nHow many rooms would be needed for 60 children?"},{"varval":"<div class=\"aligned\"> \n\n| | |\n| -----: | -------------------- |\n| Rooms | \\= $\\dfrac{60}{15}$ |\n| | \\= {{{correctAnswer}}} |\n\n</div>"}]}]

  1216. Number, NAP-42661 SJ v1-v5

    <div class="sm_mode"> {{{question}}} </div>

    [{"vars":[{"varval":"Pistol spent twice as much money as Boo.\n\nIf they spent a total of \\$210, how much did Pistol spend?"},{"varval":"Strategy 1\n\nBy trial and error of each option:\n\nIf Pistol spent $140, Boo spent $70\n\nTotal spent together = 140 + 70 = $210\n\n<br>\n\n\n<div class=\"aligned\">\n\nStrategy 2 \n\n| | |\n| -----: | -------------------- |\n| Let $\\ \\large x$ | \\= Amount Boo spent |\n| $2 \\large x$ + $\\large x$ | \\= 210 |\n| $\\large x$ | \\= \\$70 |\n\n</div>\n\n<br>$\\therefore$ Pistol spent \\$140."}]},{"vars":[{"varval":"Mike takes twice as many kilograms of apples to market as Rod.\n\nIf they take a total of 135 kilograms of apples to market, how many kilograms does Rod take?"},{"varval":"Strategy 1\n\nBy trial and error of each option:\n\nIf Mike has 90 kilograms, Rod has {{{correctAnswer}}} kilograms\n\nTotal kilograms together = 90 + {{{correctAnswer}}} = 135 \n\n<br>\n\n\n<div class=\"aligned\">\n\nStrategy 2 \n\n| | |\n| -----: | -------------------- |\n| Let $\\ \\large x$ | \\= kilograms Rod has |\n| $2 \\large x$ + $\\large x$ | \\= 135 |\n| $3 \\large x$ | \\= 135 |\n| $\\large x$ | \\= {{{correctAnswer}}} |\n\n</div>\n\n<br>$\\therefore$ Rod has {{{correctAnswer}}} kilograms of apples"}]},{"vars":[{"varval":"Hayley takes twice as many kilograms of pumpkins to market as Coral.\n\nIf they take a total of 360 kilograms of pumpkins to market, how many kilograms does Coral take?"},{"varval":"Strategy 1\n\nBy trial and error of each option:\n\nIf Hayley has 240 kilograms, Coral has {{{correctAnswer}}} kilograms\n\nTotal kilograms together = 240 + {{{correctAnswer}}} = 360 \n\n<br>\n\n\n<div class=\"aligned\">\n\nStrategy 2 \n\n| | |\n| -----: | -------------------- |\n| Let $\\ \\large x$ | \\= kilograms Coral has |\n| $2 \\large x$ + $\\large x$ | \\= 360 |\n| $3 \\large x$ | \\= 360 |\n| $\\large x$ | \\= {{{correctAnswer}}} |\n\n</div>\n\n<br>$\\therefore$ Coral has {{{correctAnswer}}} kilograms of pumpkins"}]},{"vars":[{"varval":"Hugh buys twice as many bales of hay as Grant.\n\nIf they buy a total of 210 bales of hay, how many bales does Grant buy?"},{"varval":"Strategy 1\n\nBy trial and error of each option:\n\nIf Hugh buys 140 bales, Grant buys {{{correctAnswer}}} bales.\n\nTotal kilograms together = 140 + {{{correctAnswer}}} = 210 \n\n<br>\n\n\n<div class=\"aligned\">\n\nStrategy 2 \n\n| | |\n| -----: | -------------------- |\n| Let $\\ \\large x$ | \\= bales Grant buys |\n| $2 \\large x$ + $\\large x$ | \\= 210 |\n| $3 \\large x$ | \\= 210 |\n| $\\large x$ | \\= {{{correctAnswer}}} |\n\n</div>\n\n<br>$\\therefore$ Grant bought {{{correctAnswer}}} bales of hay"}]},{"vars":[{"varval":"Jake has twice as many marbles as Iris.\n\nIf they have a total of 63 marbles, how many marbles does Iris have?"},{"varval":"Strategy 1\n\nBy trial and error of each option:\n\nIf Jake has 42 marbles, Iris has {{{correctAnswer}}} marbles.\n\nTotal marbles = 42 + {{{correctAnswer}}} = 63 \n\n<br>\n\n\n<div class=\"aligned\">\n\nStrategy 2 \n\n| | |\n| -----: | -------------------- |\n| Let $\\ \\large x$ | \\= marbles Iris has |\n| $2 \\large x$ + $\\large x$ | \\= 63 |\n| $3 \\large x$ | \\= 63 |\n| $\\large x$ | \\= {{{correctAnswer}}} |\n\n</div>\n\n<br>$\\therefore$ Iris has {{{correctAnswer}}} marbles"}]},{"vars":[{"varval":"Sabit has saved twice as much money as Jade.\n\nIf they have a total of $186 in savings, how much has Jade saved?"},{"varval":"Strategy 1\n\nBy trial and error of each option:\n\nIf Sabit saved $124, Jade has saved {{{correctAnswer}}}.\n\nTotal savings = $124 + {{{correctAnswer}}} = $186 \n\n<br>\n\n\n<div class=\"aligned\">\n\nStrategy 2 \n\n| | |\n| -----: | -------------------- |\n| Let $\\ \\large x$ | \\= Jade's savings |\n| $2 \\large x$ + $\\large x$ | \\= 186 |\n| $3 \\large x$ | \\= 186 |\n| $\\large x$ | \\= {{{correctAnswer}}} |\n\n</div>\n\n<br>$\\therefore$ Jade has saved {{{correctAnswer}}}"}]}]

  1217. Number, NAP-D4-CA18 SA ISSUE: Image colour

    A coffee jar weighs 2400 grams when it is purchased. <br> sm_img https://teacher.smartermaths.com.au/wp-content/uploads/2017/02/naplan-2011-18mc-153x300.png 128 indent3 vpad <br> When exactly half of the coffee has been used, the half full jar has a mass of 1750 grams. What is the mass of the coffee jar when all the coffee has been used?

    [{"vars":null}]

  1218. Number, NAP-K4-CA20

    Polly, Roger and Milly are splitting up two identical chocolate bars. Polly takes $\dfrac{1}{4}$ of the first chocolate bar and $\dfrac{3}{8}$ of the second bar. Roger takes $\dfrac{1}{2}$ of the first chocolate bar and $\dfrac{1}{4}$ of the second bar. What fraction of each bar did Milly get?

    [{"vars":null}]

  1219. Number, NAP-H4-CA13

    <div class="sm_mode"> {{{question}}} </div>

    [{"vars":[{"varval":"Alison's petrol tank was empty.\n\nShe then spent \\$55 filling her tank up to half way at a cost of \\$1.55 per litre.\n\nApproximately how much petrol can Alison's petrol tank hold when it is full?"},{"varval":"<div class=\"aligned\">\n\n| | |\n| ------------ | -------------------------------------------- |\n| Volume of half tank | = $\\dfrac{55}{1.55}$ |\n| | = 35.48 $\\dots$ |\n\n</div>\n\n<br>\n\n<div class=\"no-margin-bottom\">\n\n$\\therefore$ Volume of full tank\n\n</div>\n\n<div class=\"aligned\">\n\n> > | | |\n| ------------ | -------------------------------------------- |\n| | $\\approx$ 2 $\\times \\ 35.48$ |\n| | $\\approx$ 70.96 |\n| | $\\approx$ {{{correctAnswer}}} |\n\n</div>"}]},{"vars":[{"varval":"Ryan's petrol tank was empty.\n\nHe then spent \\$51 filling one third of his tank at a cost of \\$2.29 per litre.\n\nApproximately how much petrol can Ryan's petrol tank hold when it is full?"},{"varval":"<div class=\"aligned\">\n\n| | |\n| ------------ | -------------------------------------------- |\n| Volume of one-third tank | = $\\dfrac{51}{2.29}$ |\n| | = 22.27 $\\dots$ |\n\n</div>\n\n<br>\n\n<div class=\"no-margin-bottom\">\n\n$\\therefore$ Volume of full tank\n\n</div>\n\n<div class=\"aligned\">\n\n> > | | |\n| ------------ | -------------------------------------------- |\n| | $\\approx$ 3 $\\times \\ 22.3$ |\n| | $\\approx$ 66.9 |\n| | $\\approx$ {{{correctAnswer}}} |\n\n</div>"}]},{"vars":[{"varval":"Gerry owns a monster truck whose petrol tank was empty.\n\nHe then spent \\$87 filling one quarter of its tank at a cost of \\$2.09 per litre.\n\nApproximately how much petrol can Gerry's monster truck petrol tank hold when it is full?"},{"varval":"<div class=\"aligned\">\n\n| | |\n| ------------ | -------------------------------------------- |\n| Volume of quarter tank | = $\\dfrac{87}{2.09}$ |\n| | = 41.626 $\\dots$ |\n\n</div>\n\n<br>\n\n<div class=\"no-margin-bottom\">\n\n$\\therefore$ Volume of full tank\n\n</div>\n\n<div class=\"aligned\">\n\n> > | | |\n| ------------ | -------------------------------------------- |\n| | $\\approx$ 4 $\\times \\ 41.63$ |\n| | $\\approx$ 166.52 |\n| | $\\approx$ {{{correctAnswer}}} |\n\n</div>"}]},{"vars":[{"varval":"Batbayar's petrol tank was empty.\n\nHe then spent \\$39 filling one fifth of his tank at a cost of \\$2.11 per litre.\n\nApproximately how much petrol can Batbayar's petrol tank hold when it is full?"},{"varval":"<div class=\"aligned\">\n\n| | |\n| ------------ | -------------------------------------------- |\n| Volume of one-fifth tank | = $\\dfrac{39}{2.11}$ |\n| | = 18.483 $\\dots$ |\n\n</div>\n\n<br>\n\n<div class=\"no-margin-bottom\">\n\n$\\therefore$ Volume of full tank\n\n</div>\n\n<div class=\"aligned\">\n\n> > | | |\n| ------------ | -------------------------------------------- |\n| | $\\approx$ 5 $\\times \\ 18.483$ |\n| | $\\approx$ 92.415 |\n| | $\\approx$ {{{correctAnswer}}} |\n\n</div>"}]},{"vars":[{"varval":"Bataar's petrol tank was empty.\n\nHe then spent \\$27 filling one quarter of his tank at a cost of \\$1.88 per litre.\n\nApproximately how much petrol can Bataar's petrol tank hold when it is full?"},{"varval":"<div class=\"aligned\">\n\n| | |\n| ------------ | -------------------------------------------- |\n| Volume of one-quarter tank | = $\\dfrac{27}{1.88}$ |\n| | = 14.361 $\\dots$ |\n\n</div>\n\n<br>\n\n<div class=\"no-margin-bottom\">\n\n$\\therefore$ Volume of full tank\n\n</div>\n\n<div class=\"aligned\">\n\n> > | | |\n| ------------ | -------------------------------------------- |\n| | $\\approx$ 4 $\\times \\ 14.36$ |\n| | $\\approx$ 57.44 |\n| | $\\approx$ {{{correctAnswer}}} |\n\n</div>"}]},{"vars":[{"varval":"Amgalan's motor cycle petrol tank was empty.\n\nShe then spent \\$19 filling one half of her tank at a cost of \\$2.29 per litre.\n\nApproximately how much petrol can Amgalan's petrol tank hold when it is full?"},{"varval":"<div class=\"aligned\">\n\n| | |\n| ------------ | -------------------------------------------- |\n| Volume of a half tank | = $\\dfrac{19}{2.29}$ |\n| | = 8.296 $\\dots$ |\n\n</div>\n\n<br>\n\n<div class=\"no-margin-bottom\">\n\n$\\therefore$ Volume of full tank\n\n</div>\n\n<div class=\"aligned\">\n\n> > | | |\n| ------------ | -------------------------------------------- |\n| | $\\approx$ 2 $\\times \\ 8.3$ |\n| | $\\approx$ 16.6 |\n| | $\\approx$ {{{correctAnswer}}} |\n\n</div>"}]}]

  1220. Number, NAP-I4-NC12

    Dave and Helene had identical ice-blocks. Dave ate $\dfrac{5}{6}$ of his ice-block. Helene ate more of her ice-block than Dave. What fraction could Helene have eaten?

    [{"vars":null}]

  1221. Number, NAP-L4-CA17

    Stephan makes 14 loaves of bread to sell at his bakery. Each loaf takes $\dfrac{1}{4}$ cup of sugar. After he makes each loaf, he counts how many cups of sugar he uses in total. <div class="sm_mode"> > > $\dfrac{1}{4},\ \dfrac{1}{2},\ \dfrac{3}{4},\ 1,\ 1\dfrac{1}{4},\ 1\dfrac{1}{2} \dots$ <br>Which is the last number he will count? </div>

    [{"vars":null}]

  1222. Number, NAP-J4-NC03 SA

    Sharnie is buying eggs for her restaurant's kitchen. Each carton contains 12 eggs. If she needs 90 eggs, how many cartons will she need to buy?

    [{"vars":null}]

  1223. Number, NAP-J4-CA20 SA

    A high school has 1000 students enrolled. 37 of the 1000 students have the first name John. Write this as a decimal.

    [{"vars":null}]

  1224. Number, NAP-C4-CA13 ISSUE: Image colour

    A pile of paper is 35 mm high. <br> sm_img https://teacher.smartermaths.com.au/wp-content/uploads/2017/03/naplan-2010-13mc.png 300 indent vpad <br> If each sheet is 0.08 mm thick, how many sheets of paper are in this pile?

    [{"vars":null}]

  1225. Q28

    {{name}} has exactly {{change}} in change which is made up of {{cent1}} cent, {{cent2}} cent and {{cent3}} cent coins only. If she has exactly {{coins}} coins, how many {{cent4}} cent coins could she have?

    [{"vars":[{"varval":"Kate"},{"varval":"\\$2.60"},{"varval":"50"},{"varval":"20"},{"varval":"10"},{"varval":"8"},{"varval":"10"},{"varval":"* 4 $\\times$ 50$c$\n* 2 $\\times$ 20$c$\n* 2 $\\times$ 10$c$"}]},{"vars":[{"varval":"Mary"},{"varval":"\\$1.65"},{"varval":"50"},{"varval":"20"},{"varval":"5"},{"varval":"9"},{"varval":"5"},{"varval":"* 1 $\\times$ 50$c$\n* 5 $\\times$ 20$c$\n* 3 $\\times$ 5$c$"}]},{"vars":[{"varval":"Kirsty"},{"varval":"\\$1.15"},{"varval":"50"},{"varval":"20"},{"varval":"5"},{"varval":"8"},{"varval":"5"},{"varval":"* 1 $\\times$ 50$c$\n* 2 $\\times$ 20$c$\n* 5 $\\times$ 5$c$"}]},{"vars":[{"varval":"Sibyl"},{"varval":"\\$2.30"},{"varval":"50"},{"varval":"20"},{"varval":"10"},{"varval":"8"},{"varval":"20"},{"varval":"* 3 $\\times$ 50$c$\n* 3 $\\times$ 20$c$\n* 2 $\\times$ 10$c$"}]},{"vars":[{"varval":"Melanie"},{"varval":"\\$1.50"},{"varval":"50"},{"varval":"20"},{"varval":"10"},{"varval":"9"},{"varval":"10"},{"varval":"* 1 $\\times$ 50 $c$\n* 2 $\\times$ 20 $c$\n* 6 $\\times$ 10 $c$"}]}]

  1226. Number, NAP-B4-CA12

    <div class="sm_mode"> {{{question}}} </div>

    [{"vars":[{"varval":"36 $\\times$ <span class=\"sm-text color2\">?</span> = 27"},{"varval":"<div class=\"aligned\">\n\n| | |\n| --------------------- :| -------------------------------------------- |\n| 36$\\ \\times$ <span class=\"sm-text color2\">?</span> | = 27 |\n| <span class=\"sm-text color2\">?</span> | = $\\dfrac{27}{36}$ |\n| <span class=\"sm-text color2\">?</span> | = {{correctAnswer}} |\n\n</div>"}]},{"vars":[{"varval":"24 $\\times$ <span class=\"sm-text color2\">?</span> = 18"},{"varval":"<div class=\"aligned\">\n\n| | |\n| --------------------- :| -------------------------------------------- |\n| 24$\\ \\times$ <span class=\"sm-text color2\">?</span> | = 18 |\n| <span class=\"sm-text color2\">?</span> | = $\\dfrac{18}{24}$ |\n| <span class=\"sm-text color2\">?</span> | = {{correctAnswer}} |\n\n</div>"}]},{"vars":[{"varval":"36 $\\times$ <span class=\"sm-text color2\">?</span> = 24"},{"varval":"<div class=\"aligned\">\n\n| | |\n| --------------------- :| -------------------------------------------- |\n| 36$\\ \\times$ <span class=\"sm-text color2\">?</span> | = 24 |\n| <span class=\"sm-text color2\">?</span> | = $\\dfrac{24}{36}$ |\n| <span class=\"sm-text color2\">?</span> | = {{correctAnswer}} |\n\n</div>"}]},{"vars":[{"varval":"48 $\\times$ <span class=\"sm-text color2\">?</span> = 36"},{"varval":"<div class=\"aligned\">\n\n| | |\n| --------------------- :| -------------------------------------------- |\n| 48$\\ \\times$ <span class=\"sm-text color2\">?</span> | = 36 |\n| <span class=\"sm-text color2\">?</span> | = $\\dfrac{36}{48}$ |\n| <span class=\"sm-text color2\">?</span> | = {{correctAnswer}} |\n\n</div>"}]},{"vars":[{"varval":"27 $\\times$ <span class=\"sm-text color2\">?</span> = 18"},{"varval":"<div class=\"aligned\">\n\n| | |\n| --------------------- :| -------------------------------------------- |\n| 27$\\ \\times$ <span class=\"sm-text color2\">?</span> | = 18 |\n| <span class=\"sm-text color2\">?</span> | = $\\dfrac{18}{27}$ |\n| <span class=\"sm-text color2\">?</span> | = {{correctAnswer}} |\n\n</div>"}]}]

  1227. Number, NAP-G4-NC11

    <div class="sm_mode"> 4 groups of backpackers picked blueberries over a two hour period. The weight of the blueberries collected (in kg) by the groups in each hour is shown in the table below. <br> <div class="sm-table col1-color4 row1-color4 top-left-cell-hidden"> >>| | Group 1| Group 2|Group 3|Group 4| |:-:|:-:|:-:|:-:|:-:| | 1st hour | 2.57| 2.89|2.5|2.87| | 2nd hour | 2.35| 2.06| 2.92 | 2.53| </div> <br> Which group picked the greatest mass over the two hours? </div>

    [{"vars":null}]

  1228. Q27

    When a number is divided by {{number1}}, the answer is {{number2}}. If {{name}} multiplies the same number by {{number3}}, what will be the answer?

    [{"vars":[{"varval":"9"},{"varval":"2"},{"varval":"Erik"},{"varval":"4"},{"varval":"18"}]},{"vars":[{"varval":"12"},{"varval":"2"},{"varval":"Stanley"},{"varval":"4"},{"varval":"24"}]},{"vars":[{"varval":"6"},{"varval":"3"},{"varval":"Charlize"},{"varval":"6"},{"varval":"18"}]},{"vars":[{"varval":"4"},{"varval":"8"},{"varval":"Cameron"},{"varval":"6"},{"varval":"32"}]},{"vars":[{"varval":"12"},{"varval":"3"},{"varval":"Ralph"},{"varval":"5"},{"varval":"36"}]}]

  1229. Geometry, NAP-54032

    Christopher is walking in south-east direction and makes a quarter turn to his right. Which direction is he walking after he turns?

    [{"vars":null}]

  1230. Geometry, NAP-65861

    A car is travelling south-west on Chappell Street. The car is about to turn onto Second Street. <br> sm_img https://teacher.smartermaths.com.au/wp-content/uploads/2017/01/NAP-C3-NC14.png 250 indent3 vpad <br> In which direction will the car be travelling after it turns right?

    [{"vars":null}]

  1231. Geometry, NAP-31769

    sm_img https://teacher.smartermaths.com.au/wp-content/uploads/2017/02/naplan-Y7-2009-14mc.png 350 indent3 vpad <br> Sanjeev is travelling along Dogwood Drive towards Alex Street. What direction is Sanjeev travelling?

    [{"vars":null}]

  1232. Geometry, NAP-65860

    Two plans of a sports complex can be seen below. <br> sm_img https://teacher.smartermaths.com.au/wp-content/uploads/2017/02/NAP-D3-NC13.png 600 indent2 vpad <br> Which designated sports area is closest to the corner of Bone Street and Depp Street?

    [{"vars":null}]

  1233. Geometry, NAP-55289

    This is a map of where John and Fiona live. <br> sm_img https://teacher.smartermaths.com.au/wp-content/uploads/2017/02/naplan-Y7-2011-13mc.png 500 indent3 vpad <br> John lives on the corner of First Street and Farrar Street (map reference A2). He walks 750 metres along Farrar Street to Fiona's house. What is the map reference of Fiona's house?

    [{"vars":null}]

  1234. Geometry, NAP-55286

    sm_img https://teacher.smartermaths.com.au/wp-content/uploads/2017/01/NAP-G3-CA101.svg 400 indent3 vpad <br> Daniel is at the Racoon Ridge Inn. Which of these directions should Daniel follow to get to Darlington?

    [{"vars":null}]

  1235. Geometry, NAP-96415

    A map of the huts in Ghengis Khan's camp is drawn below. <br> sm_img https://teacher.smartermaths.com.au/wp-content/uploads/2020/10/NAP-J1-CA131.png 500 indent3 vpad <br> In what direction is Ghengis' hut from Batu's hut?

    [{"vars":null}]

  1236. Geometry, NAP-02298

    This is a street map. <br> sm_img https://teacher.smartermaths.com.au/wp-content/uploads/2017/01/NAP-H3-011.png 450 indent3 vpad <br> About how far is it from Matt's house to Ali's house?

    [{"vars":null}]

  1237. Geometry, NAP-48222

    This map shows the location of different animal enclosures at a zoo. <br> sm_img https://teacher.smartermaths.com.au/wp-content/uploads/2017/12/nap-A3-nc01.svg 350 indent3 vpad <br> Which enclosure is west of the giraffes and north of the monkeys?

    [{"vars":null}]

  1238. Geometry, NAP-73281

    <div class="sm_mode"> Spiro is making a scale drawing of his house. * The height of the garage in the scale model is 6 centimetres. * The height of Spiro's actual garage is 3 metres. What does 1 centimetre in Spiro's scale drawing represent in his real house? </div>

    [{"vars":null}]

  1239. Geometry, NAP-85080

    Libby plays on a hockey field that is 120 metres long. She makes a scale diagram of the field using a ratio of 1 : 400. <br> sm_img https://teacher.smartermaths.com.au/wp-content/uploads/2017/02/naplan-2011-13mc.png 350 indent3 vpad <br> How long should Libby make the scale diagram of the field?

    [{"vars":null}]

  1240. Geometry, NAP-73248

    Kransky has a photo which is 25 cm wide and 10 cm high. <br> sm_img https://teacher.smartermaths.com.au/wp-content/uploads/2020/01/nap-L4-13-ver1.svg 380 indent2 vpad <br> He wants to enlarge it to make a poster with a width of 50 cm. What will be the height of the poster?

    [{"vars":null}]

  1241. Geometry, NAP-32121

    A ramp rises 1 metre for every 8 metres of its base length. <br> sm_img https://teacher.smartermaths.com.au/wp-content/uploads/2016/12/naplan-2014-12mc.png 300 indent3 vpad <br> If it rises 0.7 metres, what is its base length?

    [{"vars":null}]

  1242. Geometry, NAP-07313

    Vinny made a ramp to access his tree house that is 1.5 metres high. The ratio of the horizontal length of the ramp to its height is 8:1. <br> sm_img https://teacher.smartermaths.com.au/wp-content/uploads/2016/12/NAP-169-NC07.png 430 indent vpad <br> What is the horizontal length of the ramp?

    [{"vars":null}]

  1243. Geometry, NAP-42667

    The actual body length of a beetle Brad has caught is 24 mm. A scale drawing of the beetle is shown below. sm_img https://teacher.smartermaths.com.au/wp-content/uploads/2017/12/nap-A4-nc08rev.svg 200 indent3 vpad <br>What scale is used in the drawing?

    [{"vars":null}]

  1244. Geometry, NAP-66187

    The picture below shows a flower. <br> sm_img https://teacher.smartermaths.com.au/wp-content/uploads/2017/01/NAP-C4-NC07.png 300 indent3 vpad <br>The picture is 2 cm wide. The actual flower is 40 cm wide. What scale is used in the picture?

    [{"vars":null}]

  1245. <div class="sm_mode"> > ## Valid Colors sm_img https://teacher.smartermaths.com.au/wp-content/uploads/2020/07/valid-color-palette.png 780 indent3 vpad * For one light colored background/shape only, pick one color from Palette A. * For 3D shapes/objects combine same color from Palette A & B, or B & C. Example: #efe9a2 & #eddd33 or #e69f00 & #d37508 * For three or more colored images/objects, refer to Color Wheel. <br> *** ## Textures If two light regions, then must use two complimenting colors, one color having texture: sm_img https://teacher.smartermaths.com.au/wp-content/uploads/2020/07/colcombi-1.jpg 300 indent3 vpad <br> Copy SM link for textures, open and save as SVG file. sm_img https://teacher.smartermaths.com.au/wp-content/uploads/2020/07/texture2.svg 200 indent3 vpad sm_img https://teacher.smartermaths.com.au/wp-content/uploads/2020/07/texture1.svg 200 indent3 vpad <br> Or you can create your own texture with Inkscape. </div>

    [{"vars":null}]

  1246. Number, NAP-C4-NC09

    A walking path near the beach is paved for half the length, cobbled stone for $\dfrac{3}{10}$ of the length and then grass for the rest of the path. For what fraction of the path is it grass?

    [{"vars":null}]

  1247. Number, NAP-A4-NC14

    <div class="sm_mode"> {{{question}}} </div>

    [{"vars":[{"varval":"Which fraction has the same value as &nbsp;$2\\dfrac{3}{5}$?"},{"varval":"<div class=\"aligned\">\n\n| | |\n| --------------------: | -------------- |\n| $2 \\dfrac{3}{5}$ | \\= $\\dfrac{10}{5} + \\dfrac{3}{5}$|\n| | \\= {{{correctAnswer}}} |\n\n</div>"}]},{"vars":[{"varval":"Which fraction has the same value as &nbsp;$1\\dfrac{3}{4}$?"},{"varval":"<div class=\"aligned\">\n\n| | |\n| --------------------: | -------------- |\n| $1 \\dfrac{3}{4}$ | \\= $\\dfrac{4}{4} + \\dfrac{3}{4}$|\n| | \\= {{{correctAnswer}}} |\n\n</div>"}]},{"vars":[{"varval":"Which fraction has the same value as &nbsp;$3\\dfrac{2}{5}$?"},{"varval":"<div class=\"aligned\">\n\n| | |\n| --------------------: | -------------- |\n| $3 \\dfrac{2}{5}$ | \\= $\\dfrac{15}{5} + \\dfrac{2}{5}$|\n| | \\= {{{correctAnswer}}} |\n\n</div>"}]},{"vars":[{"varval":"Which fraction has the same value as &nbsp;$2\\dfrac{4}{7}$?"},{"varval":"<div class=\"aligned\">\n\n| | |\n| --------------------: | -------------- |\n| $2 \\dfrac{4}{7}$ | \\= $\\dfrac{14}{7} + \\dfrac{4}{7}$|\n| | \\= {{{correctAnswer}}} |\n\n</div>"}]}]

  1248. Number, NAP-L4-CA09

    Ralph is reading the outside temperature on his skiing holiday. What is the value of the point marked A on this temperature scale? <br> sm_img https://teacher.smartermaths.com.au/wp-content/uploads/2019/12/nap-L4-09-ver1.svg 290 indent2 vpad

    [{"vars":null}]

  1249. Number, NAP-I4-CA07

    Luke had 6 cups of sugar. He used $\dfrac{1}{2}$ cup of sugar for one recipe and $2 \dfrac{1}{4}$ cups of sugar for another recipe. How many cups of sugar did Luke have left?

    [{"vars":null}]

  1250. Number, NAP-J4-CA05

    Megan has $\dfrac{1}{8}$ cup of sugar. She needs $1 \dfrac{1}{2}$ cups of sugar for a cake recipe she is using. How much more sugar does Megan need for the recipe?

    [{"vars":null}]

  1251. <div class="sm_mode"> {{{question}}} </div>

    [{"vars":[{"varval":"A cake is cut into 10 equal pieces.\n\nThe angle between the cuts is labelled on the diagram as $\\large a$$\\degree$.\n\n<br>\n\nsm_img https://teacher.smartermaths.com.au/wp-content/uploads/2020/03/NAPX-LA-CA12-o2-cake1.svg 220 indent3 vpad\n\n<br>What is the value of $\\large a$$\\degree$."},{"varval":"There is 360° about a point.\n\n\n\n<div class=\"aligned\">\n\n| | |\n| --------------------- | -------------------------------------------- |\n| $\\therefore\\ \\large a$° | = $\\dfrac{360}{10}$ |\n| | = 36$\\degree$|\n\n</div>"}]},{"vars":[{"varval":"A wagon wheel has 8 spokes.\n\r\nThe angle between two spokes is marked by $\\large a$$\\degree$ on the diagram.\n\n<br>\n\nsm_img https://teacher.smartermaths.com.au/wp-content/uploads/2020/04/NAPX-LA-CA12-o1wheel-revised.svg 180 indent vpad\n\n<br>What is the value of $\\large a$$\\degree$ ?"},{"varval":"There is 360° about a point.\n\n<div class=\"aligned\">\n\n|||\n|-|-|\n|$\\therefore \\large a$$\\degree$ |= $\\dfrac{360}{8}$|\n||= 45$\\degree$|\n\n</div>"}]}]

  1252. Geometry, NAPX-H4-CA16

    Paisley needs to choose a shape that has at least one pair of parallel sides and one internal reflex angle. Which of these can Paisley choose?

    [{"vars":null}]

  1253. Geometry, NAPX-H4-CA12 ****This question is the same as https://cms.smarterschool.com.au/admin/#/collections/item/entries/036f8257-e2bc-4826-8702-71af3586ee2a

    Triangle $PQR$ is an isosceles triangle. <br> sm_img https://teacher.smartermaths.com.au/wp-content/uploads/2018/04/NAPX-H4-CA121.svg 215 indent3 vpad <br> What is the size of the angle $PQR$?

    [{"vars":null}]

  1254. Geometry, NAPX-H3-CA16

    <div class="sm_mode"> An isosceles triangle is drawn below. <br> sm_img https://teacher.smartermaths.com.au/wp-content/uploads/2018/07/NAPX-H3-CA16.svg 170 indent3 vpad <br> What is the size of the angle <span class="sm_katex-font">$\angle$</span>$ABC$? </div>

    [{"vars":null}]

  1255. Geometry, NAPX-G3-CA17

    <div class="sm_mode"> {{{question}}} </div>

    [{"vars":[{"varval":"sm_img https://teacher.smartermaths.com.au/wp-content/uploads/2018/08/NAPX-G3-CA17.svg 257 indent3 vpad\n\n<br>\n\nWhich of the angles in this shape is closest to 150° ?\n"},{"varval":"By inspection:\n\nAngles $\\large b$ and $\\large d$ are both acute (less than 90°)\n\nAngle $\\large c$ is close to 100°.\n\n$\\therefore$ Angle {{{correctAnswer}}} ≈ 150°"}]},{"vars":[{"varval":"sm_img https://teacher.smartermaths.com.au/wp-content/uploads/2022/12/Geom_50069_v4.svg 210 indent2 vpad\n\n<br>\n\nWhich of the angles in this shape is closest to 100° ?\n"},{"varval":"By inspection:\n\nAngles $\\large b$ and $\\large c$ are both acute (less than 90°)\n\nAngle $\\large d$ is close to 135°.\n\n$\\therefore$ Angle {{{correctAnswer}}} ≈ 100°"}]},{"vars":[{"varval":"sm_img https://teacher.smartermaths.com.au/wp-content/uploads/2022/12/Geom_50069_v4.svg 210 indent2 vpad\n\n<br>\n\nWhich of the angles in this shape is closest to 140° ?\n"},{"varval":"By inspection:\n\nAngles $\\large b$ and $\\large c$ are both acute (less than 90°)\n\nAngle $\\large a$ is close to 90°.\n\n$\\therefore$ Angle {{{correctAnswer}}} ≈ 140°"}]}]

  1256. Geometry, NAPX-F4-CA11

    A pentagon is drawn below. <br> sm_img https://teacher.smartermaths.com.au/wp-content/uploads/2018/06/NAPX-F4-CA11.svg 315 indent3 vpad <br> What is the sum of the five interior angles? 

    [{"vars":null}]

  1257. Geometry, NAPX-F3-CA15

    sm_img https://teacher.smartermaths.com.au/wp-content/uploads/2018/08/NAPX-F3-CA15.svg 300 indent3 vpad <br> What is the size of the missing angle?

    [{"vars":null}]

  1258. Geometry, NAPX-G3-CA08

    Guthrie plotted the points $A - F$  on a grid paper, as shown below. <br> sm_img //teacher.smartermaths.com.au/wp-content/uploads/2017/01/naplan-Y7-2014-8mc.png 450 indent3 vpad <br> She then joined some of the points together with lines. Which of these pairs of lines are parallel?

    [{"vars":null}]

  1259. Geometry, NAPX-L4-CA02 o1, o2

    <div class="sm_mode"> {{{question}}} </div>

    [{"vars":[{"varval":"Calligula drew the shape below.\n\n<br>\n\nsm_img https://teacher.smartermaths.com.au/wp-content/uploads/2020/03/NAPX-L4-CA02-o2revisedshape.svg 218 indent3 vpad\n\n<br>\n\nAngle $\\large x$° is 115°.\n\nWhat is the size of angle $\\large y$°?"},{"varval":"<div class=\"sm_mode\">\n\nsm_nogap 360° about a point.\n\n<div class=\"aligned\">\n\n| | |\n| --------------------------: | ----------- |\n| $\\therefore\\ \\large y$° | = 360 $−$ 115 |\n| | = 245$\\degree$|\n\n</div>\n\n</div>"}]},{"vars":[{"varval":"Sheila drew a four sided shape.\n\n<br>\n\nsm_img https://teacher.smartermaths.com.au/wp-content/uploads/2020/03/NAPX-L4-CA02-o1revised.svg 239 indent3 vpad\n\n<br>\n\nAngle $\\large p$° is 135°.\n \nWhat is the size of angle $\\large q$°?"},{"varval":"<div class=\"sm_mode\">\n\nsm_nogap 360° about a point.\n\n<div class=\"aligned\">\n\n| | |\n| ------------------- | ----------- |\n| $\\therefore\\ \\large q$° | = 360 − 135 |\n| | = 225$\\degree$|\n\n</div>\n\n</div>"}]}]

  1260. Number, NAP-K4-NC01

    Mark is booking a hotel room online that has a rating of 3.7 out of 5. The ratings of the other 4 hotel rooms he considered are listed below. <div class="sm_mode"> <div class="aligned"> > > | | | > > | --------------------------------- | ---- | > > | $\bullet$ Hotel 57's rating | 1.8 | > > | $\bullet$ Snuggy Hotel's rating | 1.85 | > > | $\bullet$ Castle Hotel's rating | 1.91 | > > | $\bullet$ Cross Hotel's rating | 2.5 | </div> <br> Which hotel has a rating that is exactly half that of the hotel Mark booked? </div>

    [{"vars":null}]

  1261. Number, NAP-B4-CA09

    A bag of flour weighs $\dfrac{3}{4}$ of a kilogram. Peter buys two bags. How many kilograms of flour does Peter buy?

    [{"vars":[]}]

  1262. Mike is downloading a complete hard drive onto his new computer. It should take 24 minutes to download the full hard drive. Mike loses his internet connection when $\dfrac{5}{6}$ of the hard drive is downloaded. How many more minutes are needed for Mike to complete the download?

    [{"vars":null}]

  1263. #26

    {{image}} <br> ABC is a straight line. What is the size of $\large \alpha \degree$?

    [{"vars":[{"varval":"\n\nsm_img https://teacher.smartermaths.com.au/wp-content/uploads/2020/07/Q26var1.svg 140 indent3 vpad"},{"varval":"52"},{"varval":"80"},{"varval":"132"}]},{"vars":[{"varval":"sm_img https://teacher.smartermaths.com.au/wp-content/uploads/2020/07/Q26var2.svg 226 indent3 vpad"},{"varval":"46"},{"varval":"23"},{"varval":"69"}]},{"vars":[{"varval":"sm_img https://teacher.smartermaths.com.au/wp-content/uploads/2020/07/Q26var3.svg 130 indent3 vpad"},{"varval":"18"},{"varval":"49"},{"varval":"67"}]},{"vars":[{"varval":"sm_img https://teacher.smartermaths.com.au/wp-content/uploads/2020/07/Q26var4_QF.svg 100 indent3 vpad"},{"varval":"38"},{"varval":"95"},{"varval":"133"}]},{"vars":[{"varval":"sm_img https://teacher.smartermaths.com.au/wp-content/uploads/2020/07/Q26var5.svg 240 indent3 vpad"},{"varval":"93"},{"varval":"45"},{"varval":"138"}]}]

  1264. Number, NAP-L4-CA07

    <div class="sm_mode"> {{{question}}} </div>

    [{"vars":[{"varval":"The table shows the fraction of the Australian workforce in a number of industries.\n\n<br>\n\n<div class=\"sm-table row1-color8\">\n\n>>| Industry | Fraction of workforce |\n|:-:|:-:|\n| Automotive | $\\dfrac{1}{12}$|\n| Finance | $\\dfrac{1}{30}$|\n| Healthcare | $\\dfrac{1}{7}$|\n| Telecommunications | $\\dfrac{1}{10}$|\n\n</div>\n\n<br>\n\nWhich of these industries has the least number of employees in the workforce?"},{"varval":"The smallest fraction is $\\dfrac{1}{30}$.\n\n$\\therefore$ The {{{correctAnswer}}} industry has least number."}]},{"vars":[{"varval":"The table shows the fraction of the Australian workforce in a number of industries.\n\n<br>\n\n<div class=\"sm-table row1-color2\">\n\n>>| Industry | Fraction of workforce |\n|:-:|:-:|\n| Education | $\\dfrac{1}{5}$|\n| Agriculture | $\\dfrac{1}{33}$|\n| Finance | $\\dfrac{1}{15}$|\n| Telecommunications | $\\dfrac{1}{10}$|\n\n</div>\n\n<br>\n\nWhich of these industries has the second largest number of employees in the workforce?"},{"varval":"Fractions (highest to lowest value) = $\\dfrac{1}{5}$, $\\dfrac{1}{10}$, $\\dfrac{1}{15}$, $\\dfrac{1}{33}$.\n\n$\\therefore$ The {{{correctAnswer}}} industry has the second highest number of workers."}]},{"vars":[{"varval":"The table shows the fraction of the Australian workforce in a number of industries.\n\n<br>\n\n<div class=\"sm-table row1-color5\">\n\n>>| Industry | Fraction of workforce |\n|:-:|:-:|\n| Accounting | $\\dfrac{1}{40}$|\n| Engineering | $\\dfrac{1}{35}$|\n| Building | $\\dfrac{1}{9}$|\n| Transport | $\\dfrac{1}{7}$|\n\n</div>\n\n<br>\n\nWhich of these industries has the largest number of employees in the workforce?"},{"varval":"The largest fraction is $\\dfrac{1}{7}$.\n\n$\\therefore$ The {{{correctAnswer}}} industry has the largest number of workers."}]}]

  1265. #25

    {{name}} is planning a trail hike. He wants to choose a hike that is {{moreless}} than {{distance}} kilometres, and has both {{amenity11}} and {{amenity12}}. The table below shows the features of four different hikes. <br> <div class="sm_mode"> <div class="outline"> > > | Hike | Distance (km) | {{amenity21}} | {{amenity22}} | > > | ---------- | :-----------: | :-----------: | :-----------: | > > | {{trail1}} | {{dist1}} | {{{yesno11}}} | {{{yesno21}}} | > > | {{trail2}} | {{dist2}} | {{{yesno12}}} | {{{yesno22}}} | > > | {{trail3}} | {{dist3}} | {{{yesno13}}} | {{{yesno23}}} | > > | {{trail4}} | {{dist4}} | {{{yesno14}}} | {{{yesno24}}} | </div> <br> Which trail should {{name}} take? </div>

    [{"vars":[{"varval":"Dean"},{"varval":"more"},{"varval":"24"},{"varval":"sleeping cabins"},{"varval":"shower facilities"},{"varval":"Cabins"},{"varval":"Showers"},{"varval":"Queenstown trail"},{"varval":"26.5"},{"varval":"<span>Yes</span>"},{"varval":"<span>No</span>"},{"varval":"Otago Trail"},{"varval":"24.8"},{"varval":"<span>Yes</span>"},{"varval":"<span>Yes</span>"},{"varval":"Wanaka Trail"},{"varval":"22.3"},{"varval":"<span>Yes</span>"},{"varval":"<span>Yes</span>"},{"varval":"Cardrona Trail"},{"varval":"20.3"},{"varval":"<span>No</span>"},{"varval":"<span>Yes</span>"}]},{"vars":[{"varval":"Albert"},{"varval":"more"},{"varval":"18.5"},{"varval":"a river crossing"},{"varval":"toilets"},{"varval":"Crossing"},{"varval":"Toilets"},{"varval":"Scenic Trail"},{"varval":"14.9"},{"varval":"<span>Yes</span>"},{"varval":"<span>Yes</span>"},{"varval":"Rocky Trail"},{"varval":"18.9"},{"varval":"<span>Yes</span>"},{"varval":"<span>No</span>"},{"varval":"Hourglass Trail"},{"varval":"20.3"},{"varval":"<span>Yes</span>"},{"varval":"<span>Yes</span>"},{"varval":"Mexted Trail"},{"varval":"21.5"},{"varval":"<span>No</span>"},{"varval":"<span>Yes</span>"}]},{"vars":[{"varval":"Murray"},{"varval":"less"},{"varval":"17.5"},{"varval":"cabins"},{"varval":"camping grounds"},{"varval":"Cabins"},{"varval":"Camping"},{"varval":"Carter Trail"},{"varval":"14.4"},{"varval":"<span>No</span>"},{"varval":"<span>Yes</span>"},{"varval":"Nonu Trail"},{"varval":"15.8"},{"varval":"<span>Yes</span>"},{"varval":"<span>Yes</span>"},{"varval":"Ardern Trail"},{"varval":"17.2"},{"varval":"<span>Yes</span>"},{"varval":"<span>No</span>"},{"varval":"Hillary Trail"},{"varval":"23.5"},{"varval":"<span>Yes</span>"},{"varval":"<span>Yes</span>"}]},{"vars":[{"varval":"Brett"},{"varval":"more"},{"varval":"12"},{"varval":"toilet facilities"},{"varval":"camping grounds"},{"varval":"Toilets"},{"varval":"Camping"},{"varval":"Price Trail"},{"varval":"11.3"},{"varval":"<span>Yes</span>"},{"varval":"<span>Yes</span>"},{"varval":"Sterling Trail"},{"varval":"12.4"},{"varval":"<span>Yes</span>"},{"varval":"<span>No</span>"},{"varval":"Grothe Trail"},{"varval":"13.5"},{"varval":"<span>No</span>"},{"varval":"<span>Yes</span>"},{"varval":"Hilditch Trail"},{"varval":"14.6"},{"varval":"<span>Yes</span>"},{"varval":"<span>Yes</span>"}]},{"vars":[{"varval":"Scott"},{"varval":"more"},{"varval":"11"},{"varval":"cabins"},{"varval":"internet service"},{"varval":"Cabins"},{"varval":"Internet"},{"varval":"Corman Trail"},{"varval":"10.8"},{"varval":"<span>Yes</span>"},{"varval":"<span>Yes</span>"},{"varval":"Gillard Trail"},{"varval":"12.2"},{"varval":"<span>Yes</span>"},{"varval":"<span>No</span>"},{"varval":"Keating Trail"},{"varval":"13.5"},{"varval":"<span>Yes</span>"},{"varval":"<span>Yes</span>"},{"varval":"Hawke Trail"},{"varval":"14.5"},{"varval":"<span>Yes</span>"},{"varval":"<span>No</span>"}]}]

  1266. Q26

    {{name}} scored {{score1}} and {{score2}} {{type}} in the first two games of the {{sport}} season. If {{gender1}} average after the third game was {{avg}}, how many {{type}} did {{gender2}} score in the third game?

    [{"vars":[{"varval":"Sylvester"},{"varval":"24"},{"varval":"31"},{"varval":"points"},{"varval":"basketball"},{"varval":"his"},{"varval":"25"},{"varval":"75"},{"varval":"he"},{"varval":"55"},{"varval":"Points"}]},{"vars":[{"varval":"Belinda"},{"varval":"36"},{"varval":"61"},{"varval":"runs"},{"varval":"cricket"},{"varval":"her"},{"varval":"41"},{"varval":"she"},{"varval":"123"},{"varval":"97"},{"varval":"Runs"}]},{"vars":[{"varval":"Andrew"},{"varval":"31"},{"varval":"19"},{"varval":"points"},{"varval":"basketball"},{"varval":"his"},{"varval":"29"},{"varval":"he"},{"varval":"87"},{"varval":"50"},{"varval":"Points"}]},{"vars":[{"varval":"Moana"},{"varval":"27"},{"varval":"16"},{"varval":"points"},{"varval":"Aussie rules"},{"varval":"her"},{"varval":"19"},{"varval":"she"},{"varval":"57"},{"varval":"43"},{"varval":"Points"}]},{"vars":[{"varval":"Rasheed"},{"varval":"26"},{"varval":"71"},{"varval":"runs"},{"varval":"cricket"},{"varval":"his"},{"varval":"52"},{"varval":"he"},{"varval":"156"},{"varval":"97"},{"varval":"Runs"}]}]

  1267. Q25

    {{name}} purchases {{number}} {{item1}} for the office supplies cabinet. If the {{item1}} cost ${{cost}}, how much did each {{item2}} cost?

    [{"vars":[{"varval":"Bronson"},{"varval":"40"},{"varval":"pens"},{"varval":"52.80"},{"varval":"pen"}]},{"vars":[{"varval":"Angela"},{"varval":"25"},{"varval":"writing pads"},{"varval":"46.50"},{"varval":"writing pad"}]},{"vars":[{"varval":"Robyn"},{"varval":"30"},{"varval":"erasers"},{"varval":"11.40"},{"varval":"eraser"}]},{"vars":[{"varval":"Curtley"},{"varval":"40"},{"varval":"sticky note pads"},{"varval":"67.60"},{"varval":"sticky note pad"}]},{"vars":[{"varval":"Vivian"},{"varval":"40"},{"varval":"pencils"},{"varval":"17.00"},{"varval":"pencil"}]}]

  1268. Geometry, NAPX-F4-CA15, NAPX-F3-CA21

    Elvis enters a park at one of the entries shown below. The map is drawn to scale but no scale is given. <br> sm_img https://teacher.smartermaths.com.au/wp-content/uploads/2017/01/naplan-2013-15mc.png 450 indent3 vpad <br> He walks 80 metres in a south-west direction and then turns and walks 20 metres in a north-west direction. Which point on the map shows where Elvis ended up?

    [{"vars":null}]

  1269. Geometry, NAPX-H3-NC07, NAPX-H2-20

    Angelica draws a triangle in a semi-circle and labels 4 angles as shown below. <br> sm_img //teacher.smartermaths.com.au/wp-content/uploads/2017/01/NAP-H3-NC071.png 365 indent3 vpad <br> Which angle is closest in size to 60°?

    [{"vars":null}]

  1270. Geometry, NAPX-F3-CA06

    Two sides of a triangle measure 7 cm and 15 cm. Which one of these statements about the length of the third side is true?

    [{"vars":null}]

  1271. Geometry, NAPX-G4-NC01

    The two 2-dimensional shapes that make up the figure below can be best described as <br> sm_img https://teacher.smartermaths.com.au/wp-content/uploads/2018/05/NAPX-G4-NC01.svg 220 indent3 vpad

    [{"vars":null}]

  1272. Geometry, NAPX-G4-NC20 ISSUE: Question and solution image - the bottom left angle needs to labelled 20 degrees

    <div class="sm_mode"> {{{question}}} </div>

    [{"vars":[{"varval":"The triangle below is isosceles.\n\nWhat is the size of the shaded reflex angle in the diagram?\n<br>\n\nsm_img https://teacher.smartermaths.com.au/wp-content/uploads/2020/09/NAPX-G4-NC20.svg 335 indent3 vpad\n"},{"varval":"sm_img https://teacher.smartermaths.com.au/wp-content/uploads/2020/09/NAPX-G4-NC20-Answer.svg 310 indent2 vpad\n\n<div class=\"aligned\">\n\n\n\n| | |\n| -----: | -------------- |\n| $\\large x$° | \\= 180 − (15 + 15) |\n| | \\= 150° |\n\n<br>\n</div>\n\n<div class=\"aligned\">\n\n| | |\n| -----: | -------------- |\n| $\\therefore$ Shaded angle| \\= 360 $-$ 150|\n| | \\= {{{correctAnswer}}} |\n\n</div>\n"}]},{"vars":[{"varval":"The triangle below is isosceles.\n\nWhat is the size of the angle marked with an $\\large x$?\n<br>\n\nsm_img https://teacher.smartermaths.com.au/wp-content/uploads/2022/12/Geom_50056_v1.svg 300 indent3 vpad\n"},{"varval":"sm_img https://teacher.smartermaths.com.au/wp-content/uploads/2022/12/Geom_50056_v1ws.svg 300 indent2 vpad\n\n<div class=\"aligned\">\n\n\n\n| | |\n| -----: | -------------- |\n| $\\large y$° | \\= 180 − (25 + 25) |\n| | \\= 130° |\n\n<br>\n</div>\n\n<div class=\"aligned\">\n\n| | |\n| -----: | -------------- |\n| $\\therefore$ $\\large x$°| \\= 360 $-$ 130|\n| | \\= {{{correctAnswer}}} |\n\n</div>\n"}]},{"vars":[{"varval":"The triangle below is isosceles.\n\nWhat is the size of the angle marked with an $\\large x$?\n<br>\n\nsm_img https://teacher.smartermaths.com.au/wp-content/uploads/2022/12/Geom_50056_v2.svg 250 indent3 vpad\n"},{"varval":"sm_img https://teacher.smartermaths.com.au/wp-content/uploads/2022/12/Geom_50056_v2ws.svg 250 indent2 vpad\n\n<div class=\"aligned\">\n\n\n\n| | |\n| -----: | -------------- |\n| $\\large y$° | \\= 180 − (24 + 24) |\n| | \\= 132° |\n\n<br>\n</div>\n\n<div class=\"aligned\">\n\n| | |\n| -----: | -------------- |\n| $\\therefore$ $\\large x$°| \\= 360 $-$ 132|\n| | \\= {{{correctAnswer}}} |\n\n</div>\n"}]}]

  1273. Number, NAP-E4-NC03

    Bogdan grows large turnips in his garden. One turnip he picks weighs 1.2 kg. Bogdan cuts 300 grams off the turnip to cook with. What fraction has he used for cooking?

    [{"vars":null}]

  1274. Number, NAP-D4-NC04

    <div class="sm_mode"> The results of a 400 metres running race final was recorded in the table below. <br> <div class="sm-table col1-color1"> >>| 1st Place| 52.26 seconds | |:-:|:-:| | 2ndPlace| 52.62 seconds| | 3rd Place| ? | | 4th Place| 53.19 seconds| </div> <br>The time of the runner who came in 3rd place could be </div>

    [{"vars":null}]

  1275. Number, NAP-D4-CA02

    The image below shows a game of noughts and crosses in progress. <br> sm_img https://teacher.smartermaths.com.au/wp-content/uploads/2017/02/naplan-2011-2mc-300x300.png 179 indent3 vpad <br> What fraction of the available boxes have been filled in?

    [{"vars":null}]

  1276. Number, NAPX-I4-NC12, NAPX-I3-NC20

    Dave and Helene were running a half marathon. Dave had completed $\dfrac{4}{5}$ of the distance. Helena was closer to the finish line than Dave. What fraction of the race could Helene have completed?

    [{"vars":null}]

  1277. Number, NAPX-L4-CA17 v1

    Po makes 10 pavlovas to sell at her school's fete. Each pavlova uses $\dfrac{1}{3}$ kilogram of fruit salad. After she makes each pavlova, she counts the total weight of fruit salad she has used. <br> sm_img https://teacher.smartermaths.com.au/wp-content/uploads/2020/03/NAPX-L4-17-ver1.svg 210 indent vpad <br> What is the last number she will count?

    [{"vars":null}]

  1278. Number, NAPX-L4-CA17 v2

    Gary makes 12 pumpkin tarts for his bakery. Each tart uses $\dfrac{1}{8}$ kilogram of pumpkin. After he makes each tart, he counts how much pumpkin he has used in total. sm_img https://teacher.smartermaths.com.au/wp-content/uploads/2020/03/NAPX-L4-17-ver2.svg 200 indent vpad What is the last number he will count?

    [{"vars":null}]

  1279. Geometry, NAPX-H4-CA17

    Heyzeus is facing west. He turns 120° clockwise. Heyzeus then turns anticlockwise until he faces south. By how many degrees did Heyzeus turn anticlockwise?

    [{"vars":null}]

  1280. Geometry, NAPX-E4-CA22 Waiting for new images from Myca.

    <div class="sm_mode"> {{{question}}} </div>

    [{"vars":[{"varval":"Kramer walks around his local park.\n\nHe starts at point $J$ and walks east.\n\nIf he walks $\\dfrac{1}{3}$ of the distance around the park, in what direction is he heading?\n\n<br>\n\nsm_img https://teacher.smartermaths.com.au/wp-content/uploads/2018/06/NAPX-E4-CA22.svg 300 indent3 vpad"},{"varval":"sm_img https://teacher.smartermaths.com.au/wp-content/uploads/2020/09/NAPX-E4-CA22ans.svg 300 indent vpad\n\nEach edge = $\\dfrac{1}{8}$ of a full circuit.\n\nSince &nbsp;$\\dfrac{1}{4}$ < $\\dfrac{1}{3}$ < $\\dfrac{3}{8}$\n\n$\\therefore$ Kramer is heading north."}]},{"vars":[{"varval":"Etienne walks around a section of the gardens of Versailles.\n\nShe starts at point $J$ and walks north.\n\nIf she walks $\\dfrac{1}{3}$ of the distance around the garden, in what direction is she heading?\n\n<br>\n\nsm_img https://teacher.smartermaths.com.au/wp-content/uploads/2022/07/NAPX-E4-CA22-q-var1.png 350 indent3 vpad"},{"varval":"sm_img https://teacher.smartermaths.com.au/wp-content/uploads/2022/07/NAPX-E4-CA22-sol-var1.png 350 indent vpad\n\n\n\nEach edge = $\\dfrac{1}{8}$ of a full circuit.\n\nSince &nbsp;$\\dfrac{1}{4}$ < $\\dfrac{1}{3}$ < $\\dfrac{3}{8}$\n\n$\\therefore$ Etienne is heading east."}]},{"vars":[{"varval":"Shivansh walks around a section of the gardens at The Taj Mahal.\n\nHe starts at point $J$ and walks west.\n\nIf he walks $\\dfrac{3}{5}$ of the distance around the garden, in what direction is he heading?\n\n<br>\n\nsm_img https://teacher.smartermaths.com.au/wp-content/uploads/2022/07/NAPX-E4-CA22-q-var2.png 320 indent3 vpad"},{"varval":"sm_img https://teacher.smartermaths.com.au/wp-content/uploads/2022/07/NAPX-E4-CA22-sol-var2.png 360 indent vpad\n\n\n\nEach edge = $\\dfrac{1}{8}$ of a full circuit.\n\nSince &nbsp;$\\dfrac{1}{2}$ < $\\dfrac{3}{5}$ < $\\dfrac{5}{8}$\n\n$\\therefore$ Shivansh is heading east."}]},{"vars":[{"varval":"Clifford walks his dog around his local park.\n\nHe starts at point $J$ and walks south.\n\nIf he walks $\\dfrac{2}{5}$ of the distance around the park, in what direction is he heading?\n\n<br>\n\nsm_img https://teacher.smartermaths.com.au/wp-content/uploads/2022/07/NAPX-E4-CA22-q-var3.png 320 indent3 vpad"},{"varval":"sm_img https://teacher.smartermaths.com.au/wp-content/uploads/2022/07/NAPX-E4-CA22-sol-var3.png 360 indent vpad\n\n\nEach edge = $\\dfrac{1}{8}$ of a full circuit.\n\nSince &nbsp;$\\dfrac{3}{8}$ < $\\dfrac{2}{5}$ < $\\dfrac{1}{2}$\n\n$\\therefore$ Clifford is heading north-west."}]},{"vars":[{"varval":"Jackson walks around his local park.\n\nHe starts at point $J$ and walks south-east.\n\nIf he walks $\\dfrac{1}{5}$ of the distance around the park, in what direction is he heading?\n\n<br>\n\nsm_img https://teacher.smartermaths.com.au/wp-content/uploads/2022/07/NAPX-E4-CA22-q-var4.png 330 indent3 vpad"},{"varval":"sm_img https://teacher.smartermaths.com.au/wp-content/uploads/2022/07/NAPX-E4-CA22-sol-var4.png 360 indent vpad\n\n\n\nEach edge = $\\dfrac{1}{8}$ of a full circuit.\n\nSince &nbsp;$\\dfrac{1}{8}$ < $\\dfrac{1}{5}$ < $\\dfrac{1}{4}$\n\n$\\therefore$ Jackson is heading west."}]},{"vars":[{"varval":"Yui walks around the centre garden at the amusement park.\n\nShe starts at point $J$ and walks west.\n\nIf she walks $\\dfrac{4}{5}$ of the distance around the garden, in what direction is she heading?\n\n<br>\n\nsm_img https://teacher.smartermaths.com.au/wp-content/uploads/2022/07/NAPX-E4-CA22-q-var5.png 330 indent3 vpad"},{"varval":"sm_img https://teacher.smartermaths.com.au/wp-content/uploads/2022/07/NAPX-E4-CA22-sol-var5.png 365 indent vpad\n\n\nEach edge = $\\dfrac{1}{8}$ of a full circuit.\n\nSince &nbsp;$\\dfrac{3}{4}$ < $\\dfrac{4}{5}$ < $\\dfrac{7}{8}$\n\n$\\therefore$ Yui is heading south."}]}]

  1281. Geometry, NAPX-H3-CA26

    Rufus is facing east. He then turns 225° anticlockwise. Rufus then turns clockwise until he faces north. By how many degrees did Rufus turn clockwise?

    [{"vars":null}]

  1282. Geometry, NAPX-G4-CA17, NAPX-G3-CA20

    TJ walks his horse from the barn to the horse track. The distance is 8 cm on the map. <br> sm_img https://teacher.smartermaths.com.au/wp-content/uploads/2018/07/NAPX-G4-CA17_1.svg 350 indent3 vpad <br> What is the actual distance from the barn to the racetrack?

    [{"vars":null}]

  1283. Geometry, NAPX-G3-NC22, NAPX-G2-33

    sm_img https://teacher.smartermaths.com.au/wp-content/uploads/2018/08/NAPX-G3-NC22.svg 350 indent3 vpad <br> Faith walked north along Bond Street and then turned right on to Bridge Street. What was the first street she passed on her left?

    [{"vars":null}]

  1284. Geometry, NAPX-L4-CA15 v2

    Tina is a town planner and needs to know the angles between streets in the diagram below. <br> sm_img https://teacher.smartermaths.com.au/wp-content/uploads/2020/03/NAPX-L4-15-ver2.svg 350 indent3 vpad <br> Tina knows that Mork Street is parallel to Mindy Street. What is the size of the shaded angle on the map?

    [{"vars":null}]

  1285. Geometry, NAPX-p111653v01 Geometry, NAPX-H3-NC17, NAPX-H2-29

    <div class="sm_mode"> {{{question}}} </div>

    [{"vars":[{"varval":"Kelly is walking in south west direction and makes a quarter turn to her left.\n\nWhich direction is she walking after she turns?"},{"varval":"sm_img https://teacher.smartermaths.com.au/wp-content/uploads/2019/01/NAPX-H2-29-v1_1.svg 230 indent3 vpad\n\nsouth-east"}]},{"vars":[{"varval":"Hansie is travelling south-west along a highway and makes a 90° turn to his right.\n\n\r\n\r\nWhich direction is he travelling after he turns?"},{"varval":"sm_img https://teacher.smartermaths.com.au/wp-content/uploads/2018/07/NAPX-H3-NC17.svg 250 indent3 vpad\n\nnorth-west"}]}]

  1286. Number, NAPX-H3-CA22

    <div class="sm_mode"> {{{question}}} </div>

    [{"vars":[{"varval":"<div class=\"aligned\">\n\n| | |\n| ------------------------------------------------------------ | --------------- |\n| <span class=\"sm-text color3\">?</span> | $+\\ \\dfrac{7}{8}\\ = \\dfrac{23}{8}$ |\n\n</div>"},{"varval":"<div class=\"aligned\">\n\n| | |\n| ------------------------------------------------------------: | - |\n| <span class=\"sm-text color3\">?</span> + $\\dfrac{7}{8}$ | = $\\dfrac{23}{8}$ | \n| <span class=\"sm-text color3\">?</span> | $= \\dfrac{23}{8} - \\dfrac{7}{8}$ |\n| <span class=\"sm-text color3\">?</span> |$= \\dfrac{16}{8}$|\n| <span class=\"sm-text color3\">?</span> |= {{{correctAnswer}}}|\n\n</div>"}]},{"vars":[{"varval":"<div class=\"aligned\">\n\n| | |\n| ------------------------------------------------------------ | --------------- |\n| <span class=\"sm-text color3\">?</span> | $+\\ \\dfrac{3}{5}\\ = \\dfrac{18}{5}$ |\n\n</div>"},{"varval":"<div class=\"aligned\">\n\n| | |\n| ------------------------------------------------------------: | - |\n| <span class=\"sm-text color3\">?</span> + $\\dfrac{3}{5}$ | = $\\dfrac{18}{5}$ | \n| <span class=\"sm-text color3\">?</span> | $= \\dfrac{18}{5} - \\dfrac{3}{5}$ |\n| <span class=\"sm-text color3\">?</span> |$= \\dfrac{15}{5}$|\n| <span class=\"sm-text color3\">?</span> |= {{{correctAnswer}}}|\n\n</div>"}]},{"vars":[{"varval":"<div class=\"aligned\">\n\n| | |\n| ------------------------------------------------------------ | --------------- |\n| <span class=\"sm-text color3\">?</span> | $+\\ \\dfrac{9}{11}\\ = \\dfrac{20}{11}$ |\n\n</div>"},{"varval":"<div class=\"aligned\">\n\n| | |\n| ------------------------------------------------------------: | - |\n| <span class=\"sm-text color3\">?</span> + $\\dfrac{9}{11}$ | = $\\dfrac{20}{11}$ | \n| <span class=\"sm-text color3\">?</span> | $= \\dfrac{20}{11} - \\dfrac{9}{11}$ |\n| <span class=\"sm-text color3\">?</span> |$= \\dfrac{11}{11}$|\n| <span class=\"sm-text color3\">?</span> |= {{{correctAnswer}}}|\n\n</div>"}]},{"vars":[{"varval":"<div class=\"aligned\">\n\n| | |\n| ------------------------------------------------------------ | --------------- |\n| <span class=\"sm-text color3\">?</span> | $+\\ \\dfrac{4}{7}\\ = \\dfrac{46}{7}$ |\n\n</div>"},{"varval":"<div class=\"aligned\">\n\n| | |\n| ------------------------------------------------------------: | - |\n| <span class=\"sm-text color3\">?</span> + $\\dfrac{4}{7}$ | = $\\dfrac{46}{7}$ | \n| <span class=\"sm-text color3\">?</span> | $= \\dfrac{46}{7} - \\dfrac{4}{7}$ |\n| <span class=\"sm-text color3\">?</span> |$= \\dfrac{42}{7}$|\n| <span class=\"sm-text color3\">?</span> |= {{{correctAnswer}}}|\n\n</div>"}]},{"vars":[{"varval":"<div class=\"aligned\">\n\n| | |\n| ------------------------------------------------------------ | --------------- |\n| <span class=\"sm-text color3\">?</span> | $+\\ \\dfrac{5}{6}\\ = \\dfrac{35}{6}$ |\n\n</div>"},{"varval":"<div class=\"aligned\">\n\n| | |\n| ------------------------------------------------------------: | - |\n| <span class=\"sm-text color3\">?</span> + $\\dfrac{5}{6}$ | = $\\dfrac{35}{6}$ | \n| <span class=\"sm-text color3\">?</span> | $= \\dfrac{35}{6} - \\dfrac{5}{6}$ |\n| <span class=\"sm-text color3\">?</span> |$= \\dfrac{30}{6}$|\n| <span class=\"sm-text color3\">?</span> |= {{{correctAnswer}}}|\n\n</div>"}]},{"vars":[{"varval":"<div class=\"aligned\">\n\n| | |\n| ------------------------------------------------------------ | --------------- |\n| <span class=\"sm-text color3\">?</span> | $+\\ \\dfrac{3}{4}\\ = \\dfrac{19}{4}$ |\n\n\n</div>"},{"varval":"<div class=\"aligned\">\n\n| | |\n| ------------------------------------------------------------: | - |\n| <span class=\"sm-text color3\">?</span> + $\\dfrac{3}{4}$ | = $\\dfrac{19}{4}$ | \n| <span class=\"sm-text color3\">?</span> | $= \\dfrac{19}{4} - \\dfrac{3}{4}$ |\n| <span class=\"sm-text color3\">?</span> |$= \\dfrac{16}{4}$|\n| <span class=\"sm-text color3\">?</span> |= {{{correctAnswer}}}|\n\n</div>"}]}]

  1287. Geometry, NAPX-p111653v02

    Raven is walking in a north-east direction and makes a quarter turn to his right. Which direction is he walking after he turns?

    [{"vars":null}]

  1288. Geometry, NAPX-E3-CA09

    A diver completes a dive where she does one and a half somersaults before entering the water. By how many degrees has the diver rotated her body in this dive?

    [{"vars":null}]

  1289. Geometry, NAPX-J3-CA05

    Mac builds a skateboard ramp with an incline of approximately 65°. Which of these shows an angle closest to the side view of the incline of the ramp?

    [{"vars":null}]

  1290. Geometry, NAP-C3-CA15

    <div class="sm_mode"> {{{question}}} </div>

    [{"vars":[{"varval":"\nsm_img //teacher.smartermaths.com.au/wp-content/uploads/2017/02/naplan-Y7-2010-15mca.png 220 indent3 vpad\n\n<br>\nWhat is the size of the shaded angle?"},{"varval":"sm_nogap Let &nbsp;$\\large x$$\\degree$ = unknown angle\n\n<div class=\"aligned\">\n\n> > | | |\n> > | -----------------------------: | ------------------ |\n> > | 15 + 55 + $\\large x$ | \\= 180$\\degree$ |\n> > | $\\therefore \\large x$$\\degree$ | \\= 180 $-$ 70 |\n> > | | \\= {{{correctAnswer}}} |\n\n</div>"}]},{"vars":[{"varval":"sm_img https://teacher.smartermaths.com.au/wp-content/uploads/2023/01/Geom_50052_v2.svg 270 indent3 vpad\n\n<br>\n\nWhat is the size of the angle labelled $\\large m$$\\degree$?\n"},{"varval":"sm_nogap The angle sum of a triangle = 180$\\degree$\n\n<div class=\"aligned\">\n\n> > | | |\n> > | -----------------------------: | ------------------ |\n> > | 32 + 110 + $\\large m$ | \\= 180$\\degree$ |\n> > | $\\therefore \\large m$$\\degree$ | \\= 180 $-$ 142 |\n> > | | \\= {{{correctAnswer}}} |\n\n</div>"}]},{"vars":[{"varval":"sm_img https://teacher.smartermaths.com.au/wp-content/uploads/2023/01/Geom_50052_v1.svg 250 indent3 vpad\n\n<br>\n\nWhat is the size of the angle labelled $\\large x$$\\degree$?"},{"varval":"sm_nogap The angle sum of a triangle = 180$\\degree$\n\n<div class=\"aligned\">\n\n> > | | |\n> > | -----------------------------: | ------------------ |\n> > | 92 + 53 + $\\large x$ | \\= 180$\\degree$ |\n> > | $\\therefore \\large x$$\\degree$ | \\= 180 $-$ 145 |\n> > | | \\= {{{correctAnswer}}} |\n\n</div>"}]},{"vars":[{"varval":"sm_img https://teacher.smartermaths.com.au/wp-content/uploads/2023/01/Geom_50052_v3.svg 320 indent3 vpad\n\n<br>\n\nWhat is the size of the angle labelled $\\large x$$\\degree$?"},{"varval":"sm_nogap The angle sum of a triangle = 180$\\degree$\n\n<div class=\"aligned\">\n\n> > | | |\n> > | -----------------------------: | ------------------ |\n> > | 120 + 19 + $\\large x$ | \\= 180$\\degree$ |\n> > | $\\therefore \\large x$$\\degree$ | \\= 180 $-$ 139 |\n> > | | \\= {{{correctAnswer}}} |\n\n</div>"}]},{"vars":[{"varval":"sm_img https://teacher.smartermaths.com.au/wp-content/uploads/2023/01/Geom_50052_v4.svg 200 indent3 vpad\n\n<br>\n\nWhat is the size of the angle labelled $\\large p \\degree$?"},{"varval":"sm_nogap The angle sum of a triangle = 180$\\degree$\n\n<div class=\"aligned\">\n\n> > | | |\n> > | -----------------------------: | ------------------ |\n> > | 59 + 63 + $\\large p$ | \\= 180$\\degree$ |\n> > | $\\therefore \\large p \\degree$ | \\= 180 $-$ 122 |\n> > | | \\= {{{correctAnswer}}} |\n\n</div>"}]},{"vars":[{"varval":"sm_img https://teacher.smartermaths.com.au/wp-content/uploads/2023/01/Geom_50052_v5.svg 380 indent3 vpad\n\n<br>\n\nWhat is the size of the angle labelled $\\large t$$\\degree$?"},{"varval":"sm_nogap The angle sum of a triangle = 180$\\degree$\n\n<div class=\"aligned\">\n\n> > | | |\n> > | -----------------------------: | ------------------ |\n> > | 18 + 22 + $\\large t$ | \\= 180$\\degree$ |\n> > | $\\therefore \\large t$$\\degree$ | \\= 180 $-$ 40 |\n> > | | \\= {{{correctAnswer}}} |\n\n</div>"}]}]

  1291. Geometry, NAP-H3-CA04

    A regular hexagon is folded in half along the dotted line. <br> sm_img //teacher.smartermaths.com.au/wp-content/uploads/2017/01/NAP-H3-04.png 245 indent3 vpad <br> The folded shape can also be called a

    [{"vars":null}]

  1292. Geometry, NAPX-G4-CA04

    sm_img https://teacher.smartermaths.com.au/wp-content/uploads/2018/05/NAPX-G4-CA04.svg 350 indent3 vpad Maya walks along the path from $A$ to $B$. How far does she walk in **kilometres**?

    [{"vars":null}]

  1293. Geometry, NAPX-E4-CA02

    A sailing ship can travel in four different directions. <br> sm_img https://teacher.smartermaths.com.au/wp-content/uploads/2018/05/NAPX-E4-CA02.svg 450 indent3 vpad <br> If Janus wants to travel south-west which arrow should Janus follow?

    [{"vars":[]}]

  1294. Geometry, NAPX-G3-CA10

    sm_img https://teacher.smartermaths.com.au/wp-content/uploads/2018/07/NAPX-G3-CA10.svg 350 indent3 vpad Pogo is staying at the Dead Dingo Inn. Which of these directions should Pogo follow to get to the hardware store?

    [{"vars":null}]

  1295. Geometry, NAPX-J3-CA13, NAPX-J2-18

    A map of the huts in Ghengis Khan's camp is drawn below. In what direction is Kubla's hut from Altani's hut? sm_img https://teacher.smartermaths.com.au/wp-content/uploads/2017/11/NAP-J1-CA131.png 480 indent3 vpad

    [{"vars":null}]

  1296. Q24

    {{name}} is counting the {{vehicle1}} heading south on the highway through her town. A {{vehicle2}} passes once every {{seconds}} seconds. If {{name}} is counting for {{hours}} hours at this rate, how many {{vehicle1}} does she count?

    [{"vars":[{"varval":"Sandra"},{"varval":"cars"},{"varval":"car"},{"varval":"6"},{"varval":"2"},{"varval":"10"},{"varval":"Cars"},{"varval":"60"}]},{"vars":[{"varval":"Camilla"},{"varval":"cars"},{"varval":"car"},{"varval":"4"},{"varval":"4"},{"varval":"15"},{"varval":"Cars"},{"varval":"60"}]},{"vars":[{"varval":"Bronwyn"},{"varval":"trucks"},{"varval":"truck"},{"varval":"12"},{"varval":"4"},{"varval":"5"},{"varval":"Trucks"},{"varval":"60"}]},{"vars":[{"varval":"Zilda"},{"varval":"cars"},{"varval":"car"},{"varval":"3"},{"varval":"3"},{"varval":"20"},{"varval":"Cars"},{"varval":"60"}]},{"vars":[{"varval":"Domi"},{"varval":"trucks"},{"varval":"truck"},{"varval":"2"},{"varval":"3"},{"varval":"30"},{"varval":"Trucks"},{"varval":"60"}]}]

  1297. Q23

    A {{object}} is pictured below. <br> {{image}} <br/> Which of the following is the best estimate of the area of one side of the {{object}} shown, in cm$^2$.

    [{"vars":[{"varval":"circular disk"},{"varval":"sm_img https://teacher.smartermaths.com.au/wp-content/uploads/2020/07/Q23var1.svg 180 indent2 vpad"},{"varval":"10"},{"varval":"100"}]},{"vars":[{"varval":"metal disc"},{"varval":"sm_img https://teacher.smartermaths.com.au/wp-content/uploads/2020/07/Q23-var2.svg 180 indent2 vpad"},{"varval":"4"},{"varval":"16"}]},{"vars":[{"varval":"circular plate"},{"varval":"sm_img https://teacher.smartermaths.com.au/wp-content/uploads/2020/07/Q23-var3.svg 180 indent2 vpad"},{"varval":"6"},{"varval":"36"}]},{"vars":[{"varval":"circular plate"},{"varval":"sm_img https://teacher.smartermaths.com.au/wp-content/uploads/2020/07/Q23-var4.svg 180 indent2 vpad"},{"varval":"7"},{"varval":"49"}]},{"vars":[{"varval":"circular token"},{"varval":"sm_img https://teacher.smartermaths.com.au/wp-content/uploads/2020/07/Q23-var5.svg 180 indent2 vpad"},{"varval":"3"},{"varval":"9"}]}]

  1298. #24

    <div class="sm_mode"> {{shop}} has reduced the price of four items that are listed in the table below. <br> <div class="outline"> >>| Item | Regular Price | Sale Price | | --------- | :-----------: | :--------: | | {{item1}} | ${{reg1}} | ${{sale1}} | | {{item2}} | ${{reg2}} | ${{sale2}} | | {{item3}} | ${{reg3}} | ${{sale3}} | | {{item4}} | ${{reg4}} | ${{sale4}} | </div> <br> Which item has been reduced by {{frac2}}? </div>

    [{"vars":[{"varval":"The School Store"},{"varval":"Backpack"},{"varval":"140"},{"varval":"105"},{"varval":"Calculator"},{"varval":"80"},{"varval":"60"},{"varval":"Notebook"},{"varval":"200"},{"varval":"150"},{"varval":"Uniform"},{"varval":"90"},{"varval":"60"},{"varval":"$\\dfrac{1}{3}$"},{"varval":"90"},{"varval":"30"},{"varval":"60"},{"varval":"one-third"}]},{"vars":[{"varval":"The Cricket Shed"},{"varval":"Pads"},{"varval":"80"},{"varval":"60"},{"varval":"Helmet"},{"varval":"180"},{"varval":"120"},{"varval":"Bag"},{"varval":"75"},{"varval":"55"},{"varval":"Bat"},{"varval":"200"},{"varval":"150"},{"varval":"$\\dfrac{1}{3}$"},{"varval":"180"},{"varval":"60"},{"varval":"120"},{"varval":"one-third"}]},{"vars":[{"varval":"The School Store"},{"varval":"Textbooks"},{"varval":"150"},{"varval":"120"},{"varval":"Computer"},{"varval":"200"},{"varval":"150"},{"varval":"Backpack"},{"varval":"80"},{"varval":"55"},{"varval":"Uniform"},{"varval":"100"},{"varval":"80"},{"varval":"$\\dfrac{1}{4}$"},{"varval":"200"},{"varval":"50"},{"varval":"150"},{"varval":"one-quarter"}]},{"vars":[{"varval":"The Appliances Shed"},{"varval":"Microwave"},{"varval":"100"},{"varval":"80"},{"varval":"Blender"},{"varval":"150"},{"varval":"120"},{"varval":"Toaster"},{"varval":"160"},{"varval":"110"},{"varval":"Griller"},{"varval":"120"},{"varval":"90"},{"varval":"$\\dfrac{1}{4}$"},{"varval":"120"},{"varval":"30"},{"varval":"90"},{"varval":"one-quarter"}]},{"vars":[{"varval":"The Cricket Shed"},{"varval":"Thigh Guard"},{"varval":"80"},{"varval":"60"},{"varval":"Helmet"},{"varval":"120"},{"varval":"90"},{"varval":"Shoes"},{"varval":"130"},{"varval":"85"},{"varval":"Pants"},{"varval":"75"},{"varval":"50"},{"varval":"$\\dfrac{1}{3}$"},{"varval":"75"},{"varval":"25"},{"varval":"50"},{"varval":"one-third"}]}]

  1299. #23

    The distance between the {{place1}} and the {{place2}} on a map is {{dist}} centimetres. The scale of the map is {{scale}}. What is the actual distance between the {{place1}} and the {{place2}}?

    [{"vars":[{"varval":"airport"},{"varval":"town hall"},{"varval":"3.1"},{"varval":"1 : 100 000"},{"varval":"100 000"},{"varval":"310 000"},{"varval":"3100"}]},{"vars":[{"varval":"airport"},{"varval":"business head office"},{"varval":"2.6"},{"varval":"1 : 200 000"},{"varval":"200 000"},{"varval":"520 000"},{"varval":"5200"}]},{"vars":[{"varval":"beach"},{"varval":"hotel"},{"varval":"2.3"},{"varval":"1 : 1 000 000"},{"varval":"1 000 000"},{"varval":"2 300 000"},{"varval":"23 000"}]},{"vars":[{"varval":"airport"},{"varval":"town hall"},{"varval":"12.6"},{"varval":"1 : 50 000"},{"varval":"50 000"},{"varval":"630 000"},{"varval":"6300"}]},{"vars":[{"varval":"hotel"},{"varval":"adventure park"},{"varval":"11.4"},{"varval":"1 : 100 000"},{"varval":"100 000"},{"varval":"1 140 000"},{"varval":"11 400"}]}]

  1300. Number, NAPX-F3-CA22

    <div class="sm_mode"> sm_nogap Which number will complete this number sentence correctly? <div class="aligned"> >>| | | | | ----- | ------------------------------------------------------------ | --------------- | | 2.5 = | <span class="sm-text color6">?</span> | $\times\ 3.5$ | </div> </div>

    [{"vars":null}]

  1301. Number, NAPX-G4-NC11, NAPX-G3-NC15

    <div class="sm_mode"> 4 groups of backpackers picked strawberries over two hours. The weight of the strawberries collected (in kg) by the groups in each hour is shown in the table below. <br> <div class="sm-table col1-color1 row1-color1 top-left-cell-hidden"> >>| | Group 1| Group 2|Group 3|Group 4| |:-:|:-:|:-:|:-:|:-:| | 1st hour | 9.4| 9.49|10.15|10.90| | 2nd hour | 10.97| 9.86| 9.09 | 9.29| </div> <br> Which group picked the greatest mass over the two hours? </div>

    [{"vars":null}]

  1302. Geometry, NAP-J3-CA05

    Hamish builds a skateboard ramp with an incline of approximately 45°. Which of these shows an angle closest to the side view of the incline of the ramp?

    [{"vars":null}]

  1303. Geometry, NAP-I4-CA20

    <div class="sm_mode"> {{{question}}} </div>

    [{"vars":[{"varval":"Lines $AB$ and $CD$ are parallel.\n\nLine $EF$ intersects lines $AB$ and $CD$ as shown.\n\n<br>\n\nsm_img https://teacher.smartermaths.com.au/wp-content/uploads/2016/12/naplan-2016-20mc.png 320 indent vpad\n\n<br>\n\nWhich pair of angles are equal?\n"},{"varval":"{{{correctAnswer}}}\n\n(Alternate angles)"}]},{"vars":[{"varval":"Lines $AB$ and $CD$ are parallel.\n\nLine $EF$ intersects lines $AB$ and $CD$ as shown.\n\n<br>\n\nsm_img https://teacher.smartermaths.com.au/wp-content/uploads/2016/12/naplan-2016-20mc.png 320 indent vpad\n\n<br>\n\nWhich pair of angles are equal?\n"},{"varval":"{{{correctAnswer}}}\n\n(Corresponding angles) "}]},{"vars":[{"varval":"Lines $AB$ and $CD$ are parallel.\n\nLine $EF$ intersects lines $AB$ and $CD$ as shown.\n\n<br>\n\nsm_img https://teacher.smartermaths.com.au/wp-content/uploads/2016/12/naplan-2016-20mc.png 320 indent vpad\n\n<br>\n\nWhich pair of angles are equal?\n"},{"varval":"{{{correctAnswer}}}\n\n(Alternate angles)"}]},{"vars":[{"varval":"Lines $AB$ and $CD$ are parallel.\n\nLine $EF$ intersects lines $AB$ and $CD$ as shown.\n\n<br>\n\nsm_img https://teacher.smartermaths.com.au/wp-content/uploads/2016/12/naplan-2016-20mc.png 320 indent vpad\n\n<br>\n\nWhich pair of angles are equal?\n"},{"varval":"{{{correctAnswer}}}\n\n(Corresponding angles)"}]},{"vars":[{"varval":"Lines $AB$ and $CD$ are parallel.\n\nLine $EF$ intersects lines $AB$ and $CD$ as shown.\n\n<br>\n\nsm_img https://teacher.smartermaths.com.au/wp-content/uploads/2016/12/naplan-2016-20mc.png 320 indent vpad\n\n<br>\n\nWhich pair of angles are equal?\n"},{"varval":"{{{correctAnswer}}}\n\n(Corresponding angles)"}]},{"vars":[{"varval":"Lines $AB$ and $CD$ are parallel.\n\nLine $EF$ intersects lines $AB$ and $CD$ as shown.\n\n<br>\n\nsm_img https://teacher.smartermaths.com.au/wp-content/uploads/2016/12/naplan-2016-20mc.png 320 indent vpad\n\n<br>\n\nWhich pair of angles are supplementary?\n"},{"varval":"Supplementary angles sum to 180$\\degree$\n\n{{{correctAnswer}}}\n\n(Cointerior angles are supplementary)"}]}]

  1304. Geometry, NAP-D4-CA19

    <div class="sm_mode"> {{{question}}} </div>

    [{"vars":[{"varval":"Bojangles draws an irregular quadrilateral that is shown below.\n<br>\n\nsm_img //teacher.smartermaths.com.au/wp-content/uploads/2017/02/naplan-Y7-2011-22mc.png 220 indent3 vpad\n\n<br>\n\nWhich list shows the three angles $\\large a, b, c$ in **decreasing** order of size?"},{"varval":"{{{correctAnswer}}}"}]},{"vars":[{"varval":"Luther draws the irregular quadrilateral shown below.\n<br>\n\nsm_img https://teacher.smartermaths.com.au/wp-content/uploads/2022/12/Geom_50042_v1.svg 200 indent3 vpad\n\n<br>\n\nWhich list shows the three angles $\\large a, b, c$ in **decreasing** order of size?"},{"varval":"{{{correctAnswer}}}"}]},{"vars":[{"varval":"Cora draws the irregular quadrilateral shown below.\n<br>\n\nsm_img https://teacher.smartermaths.com.au/wp-content/uploads/2022/12/Geom_50042_v2.svg 180 indent3 vpad\n\n<br>\n\nWhich list shows the three angles $\\large a, b, c$ in **decreasing** order of size?"},{"varval":"{{{correctAnswer}}}"}]},{"vars":[{"varval":"Elsie draws the irregular quadrilateral shown below.\n<br>\n\nsm_img https://teacher.smartermaths.com.au/wp-content/uploads/2022/12/Geom_50042_v3.svg 180 indent3 vpad\n\n<br>\n\nWhich list shows the three angles $\\large a, b, c$ in **decreasing** order of size?"},{"varval":"{{{correctAnswer}}}"}]},{"vars":[{"varval":"Junior draws the irregular quadrilateral shown below.\n<br>\n\nsm_img https://teacher.smartermaths.com.au/wp-content/uploads/2022/12/Geom_50042_v5.svg 150 indent3 vpad\n\n<br>\n\nWhich list shows the three angles $\\large a, b, c$ in **increasing** order of size?"},{"varval":"{{{correctAnswer}}}"}]}]

  1305. Geometry, NAP-H4-NC12

    sm_img //teacher.smartermaths.com.au/wp-content/uploads/2016/12/NAP-169-NC12.png 200 indent3 vpad <br> $PQRS$ is a parallelogram. Which of these must be a property of $PQRS$?

    [{"vars":null}]

  1306. Number, NAPX-F4-NC01

    A swim squad normally swim 80 laps per training session. On a light training day, they only do three quarters of the laps of a normal session. How many laps do they swim in a light session?

    [{"vars":null}]

  1307. Number, NAPX-L4-CA09 v1

    <div class="sm_mode"> Boston is reading the outside temperature during winter in his hometown. What is the value of the point marked **P** on this temperature scale? <br> sm_img https://teacher.smartermaths.com.au/wp-content/uploads/2019/12/nap-L4-09-ver2.svg 290 indent3 vpad </div>

    [{"vars":null}]

  1308. Number, NAPX-I4-CA07

    Misty has 5 cups of pumpkin seeds. She uses $2 \dfrac{1}{2}$ cups of pumpkin seeds for one recipe and gives $\dfrac{3}{4}$ cup to a friend. How many cups of pumpkin seeds does Misty have left?

    [{"vars":null}]

  1309. Number, NAPX-G4-NC07

    Dars has 560 grams of ice cream. He serves it in 8 equal servings. How much does the ice cream in each serving weigh?

    [{"vars":null}]

  1310. Number, NAPX-J4-CA05, NAPX-J3-CA12

    Mallory has $\dfrac{3}{8}$ cup of raisins. She needs $\dfrac{3}{4}$ cup of raisins for a fruit cake she is making. How many more cups of raisins does Mallory need for the recipe?

    [{"vars":[]}]

  1311. Q22 QUICKFIX - VAR5 should have "18" as the top length ... currently it has "29". Pls fix question and solution images. - please put "NOT TO SCALE" on all question images (not required on solution images).

    The shape below has a perimeter of {{perimeter}} cm. <br> {{image1}} <br/> What is the value of $\large d$?

    [{"vars":[{"varval":"96"},{"varval":"sm_img https://teacher.smartermaths.com.au/wp-content/uploads/2020/07/car1q22.svg 410 indent3 vpad"},{"varval":"30"},{"varval":"22"},{"varval":"8"},{"varval":"18"}]},{"vars":[{"varval":"114"},{"varval":"sm_img https://teacher.smartermaths.com.au/wp-content/uploads/2020/07/varr2q22.svg 480 indent3 vpad"},{"varval":"35"},{"varval":"28"},{"varval":"7"},{"varval":"22"}]},{"vars":[{"varval":"100"},{"varval":"sm_img https://teacher.smartermaths.com.au/wp-content/uploads/2020/07/var3q22.svg 450 indent3 vpad"},{"varval":"26"},{"varval":"8"},{"varval":"18"},{"varval":"24"}]},{"vars":[{"varval":"132"},{"varval":"sm_img https://teacher.smartermaths.com.au/wp-content/uploads/2020/07/var4q22.svg 570 indent3 vpad"},{"varval":"42"},{"varval":"29"},{"varval":"13"},{"varval":"24"}]},{"vars":[{"varval":"82"},{"varval":"sm_img https://teacher.smartermaths.com.au/wp-content/uploads/2020/07/var5q22.svg 420 indent3 vpad"},{"varval":"18"},{"varval":"7"},{"varval":"11"},{"varval":"23"}]}]

  1312. Number, NAPX-H3-CA13

    <div class="sm_mode"> {{{question}}} </div>

    [{"vars":[{"varval":"Which of the following lists the numbers in increasing order?"},{"varval":"Converting non-decimals to decimals:\n\n85% = 0.85\n\n$\\dfrac{7}{8}$ = 0.875\n\n$\\therefore$ Increasing order is\n\n> {{{correctAnswer}}}"}]},{"vars":[{"varval":"Which of the following lists the numbers in increasing order?"},{"varval":"Converting non-decimals to decimals:\n\n58% = 0.58\n\n$\\dfrac{5}{8}$ = 0.625\n\n$\\therefore$ Increasing order is\n\n> {{{correctAnswer}}}"}]},{"vars":[{"varval":"Which of the following lists the numbers in increasing order?"},{"varval":"Converting non-decimals to decimals:\n\n59% = 0.59\n\n$\\dfrac{5}{9}$ = 0.55555...\n\n$\\therefore$ Increasing order is\n\n> {{{correctAnswer}}}"}]},{"vars":[{"varval":"Which of the following lists the numbers in increasing order?"},{"varval":"Converting all to decimals:\n\n36% = 0.36\n\n$\\dfrac{4}{11}$ = 0.363636.....\n\n$\\dfrac{19}{50}$ = 0.38\n\n$\\therefore$ Increasing order is\n\n> {{{correctAnswer}}}"}]},{"vars":[{"varval":"Which of the following lists the numbers in increasing order?"},{"varval":"Converting non-decimals to decimals:\n\n44% = 0.44\n\n$\\dfrac{12}{25}$ = 0.48\n\n$\\dfrac{4}{9}$ = 0.444...\n\n$\\therefore$ Increasing order is\n\n> {{{correctAnswer}}}"}]},{"vars":[{"varval":"Which of the following lists the numbers in increasing order?"},{"varval":"Converting non-decimals to decimals:\n\n65% = 0.65\n\n$\\dfrac{33}{50}$ = 0.66\n\n$\\dfrac{2}{3}$ = 0.666...\n\n$\\therefore$ Increasing order is\n\n> {{{correctAnswer}}}"}]}]

  1313. Geometry, NAP-A4-CA13

    <div class="sm_mode"> {{{question}}} </div>

    [{"vars":[{"varval":"In the diagram, $ACD$ is a straight line.\n\n<br>\n\nsm_img https://teacher.smartermaths.com.au/wp-content/uploads/2017/12/NAP-A4-CA13.svg 260 indent3 vpad\n\n<br>\n\nWhat is the size of angle $BCE$?\n"},{"varval":"sm_img https://teacher.smartermaths.com.au/wp-content/uploads/2017/12/NAP-A4-CA13-Answer.svg 260 indent vpad\n\n<div class=\"sm_mode\">\n<div class=\"aligned\">\n\n| | |\n| ----------------: | -------------- |\n| <span class=\"sm_katex-font\">$\\angle$</span>$BCA$ | \\= 180 $-$ (81 + 51) ` ` (180° in triangle) |\n| | \\= 48° |\n\n</div>\n\n<br>\n\n<div class=\"aligned\">\n\n| | |\n| ---------------------------: | -------------- |\n| $\\therefore$ <span class=\"sm_katex-font\">$\\angle$</span>$BCE$ | \\= 180 $-$ (48 + 45) ` ` (<span class=\"sm_katex-font\">$\\angle$</span>$ACD$ is a straight angle) |\n| | \\= {{{correctAnswer}}} |\n\n</div> </div>"}]},{"vars":[{"varval":"In the diagram, $ACD$ is a straight line.\n\n<br>\n\nsm_img https://teacher.smartermaths.com.au/wp-content/uploads/2022/12/Geom_50073_v1q.svg 300 indent2 vpad\n\n<br>\n\nWhat is the size of angle $BCE$?\n"},{"varval":"sm_img https://teacher.smartermaths.com.au/wp-content/uploads/2022/12/Geom_50073_v1ws.svg 300 indent vpad\n\n<div class=\"sm_mode\">\n<div class=\"aligned\">\n\n| | |\n| ----------------: | -------------- |\n| <span class=\"sm_katex-font\">$\\angle$</span>$BCA$ | \\= 180 $-$ (49 + 50) ` ` (180° in triangle) |\n| | \\= 81° |\n\n</div>\n\n<br>\n\n<div class=\"aligned\">\n\n| | |\n| ---------------------------: | -------------- |\n| $\\therefore$ <span class=\"sm_katex-font\">$\\angle$</span>$BCE$ | \\= 180 $-$ (81 + 23) ` ` (<span class=\"sm_katex-font\">$\\angle$</span>$ACD$ is a straight angle) |\n| | \\= {{{correctAnswer}}} |\n\n</div> </div>"}]},{"vars":[{"varval":"In the diagram, $ACD$ is a straight line.\n<br>\n\nsm_img https://teacher.smartermaths.com.au/wp-content/uploads/2022/12/Geom_50073_v2q.svg 380 indent vpad\n\n<br>\n\nWhat is the size of angle $BCE$?\n"},{"varval":"sm_img https://teacher.smartermaths.com.au/wp-content/uploads/2022/12/Geom_50073_v2ws.svg 380 indent vpad\n\n<div class=\"sm_mode\">\n<div class=\"aligned\">\n\n| | |\n| ----------------: | -------------- |\n| <span class=\"sm_katex-font\">$\\angle$</span>$BCA$ | \\= 180 $-$ (30 + 123) ` ` (180° in triangle) |\n| | \\= 27° |\n\n</div>\n\n<br>\n\n<div class=\"aligned\">\n\n| | |\n| ---------------------------: | -------------- |\n| $\\therefore$ <span class=\"sm_katex-font\">$\\angle$</span>$BCE$ | \\= 180 $-$ (63 + 27) ` ` (<span class=\"sm_katex-font\">$\\angle$</span>$ACD$ is a straight angle) |\n| | \\= {{{correctAnswer}}} |\n\n</div> </div>"}]},{"vars":[{"varval":"In the diagram, $ACD$ is a straight line.\n\n<br>\n\nsm_img https://teacher.smartermaths.com.au/wp-content/uploads/2022/12/Geom_50073_v3q.svg 350 indent vpad\n\n<br>\n\nWhat is the size of angle $BCE$?\n"},{"varval":"sm_img https://teacher.smartermaths.com.au/wp-content/uploads/2022/12/Geom_50073_v3ws.svg 350 indent vpad\n\n<div class=\"sm_mode\">\n<div class=\"aligned\">\n\n| | |\n| ----------------: | -------------- |\n| <span class=\"sm_katex-font\">$\\angle$</span>$BCA$ | \\= 180 $-$ (29 + 39) ` ` (180° in triangle) |\n| | \\= 112° |\n\n</div>\n\n<br>\n\n<div class=\"aligned\">\n\n| | |\n| ---------------------------: | -------------- |\n| $\\therefore$ <span class=\"sm_katex-font\">$\\angle$</span>$BCE$ | \\= 180 $-$ (112 + 20) ` ` (<span class=\"sm_katex-font\">$\\angle$</span>$ACD$ is a straight angle) |\n| | \\= {{{correctAnswer}}} |\n\n</div> </div>"}]},{"vars":[{"varval":"In the diagram, $ACD$ is a straight line.\n<br>\n\nsm_img https://teacher.smartermaths.com.au/wp-content/uploads/2022/12/Geom_50073_v4q.svg 380 indent vpad\n\n<br>\n\nWhat is the size of angle $BCE$?\n"},{"varval":"sm_img https://teacher.smartermaths.com.au/wp-content/uploads/2022/12/Geom_50073_v4ws.svg 380 indent vpad\n\n<div class=\"sm_mode\">\n<div class=\"aligned\">\n\n| | |\n| ----------------: | -------------- |\n| <span class=\"sm_katex-font\">$\\angle$</span>$BCA$ | \\= 180 $-$ (63 + 63) ` ` (180° in triangle) |\n| | \\= 54° |\n\n</div>\n\n<br>\n\n<div class=\"aligned\">\n\n| | |\n| ---------------------------: | -------------- |\n| $\\therefore$ <span class=\"sm_katex-font\">$\\angle$</span>$BCE$ | \\= 180 $-$ (54 + 44) ` ` (<span class=\"sm_katex-font\">$\\angle$</span>$ACD$ is a straight angle) |\n| | \\= {{{correctAnswer}}} |\n\n</div> </div>"}]},{"vars":[{"varval":"In the diagram, $ACD$ is a straight line.\n<br>\n\nsm_img https://teacher.smartermaths.com.au/wp-content/uploads/2022/12/Geom_50073_v5q.svg 280 indent vpad\n\n<br>\n\nWhat is the size of angle $BCE$?\n"},{"varval":"sm_img https://teacher.smartermaths.com.au/wp-content/uploads/2022/12/Geom_50073_v5ws.svg 280 indent vpad\n\n<div class=\"sm_mode\">\n<div class=\"aligned\">\n\n| | |\n| ----------------: | -------------- |\n| <span class=\"sm_katex-font\">$\\angle$</span>$BCA$ | \\= 180 $-$ (46 + 74) ` ` (180° in triangle) |\n| | \\= 60° |\n\n</div>\n\n<br>\n\n<div class=\"aligned\">\n\n| | |\n| ---------------------------: | -------------- |\n| $\\therefore$ <span class=\"sm_katex-font\">$\\angle$</span>$BCE$ | \\= 180 $-$ (39 + 60) ` ` (<span class=\"sm_katex-font\">$\\angle$</span>$ACD$ is a straight angle) |\n| | \\= {{{correctAnswer}}} |\n\n</div> </div>"}]}]

  1314. Geometry, NAP-H4-CA16

    Choon drew a shape. It had one pair of parallel sides and two internal reflex angles. Which of these could be Choon's shape?

    [{"vars":null}]

  1315. Geometry, NAP-H4-CA12

    <div class="sm_mode"> {{{question}}} </div>

    [{"vars":[{"varval":"Triangle $ABC$ is an isosceles triangle.\n\n<br>\n\nsm_img //teacher.smartermaths.com.au/wp-content/uploads/2016/12/naplan-2015-12mc.png 220 indent3 vpad\n\n<br>\n\nWhat is the size of the angle <span class=\"sm_katex-font\">$\\angle$</span>$ABC$ ?"},{"varval":"sm_nogap Let &nbsp;$\\large x$ = <span class=\"sm_katex-font\">$\\angle$</span>$ABC$ = <span class=\"sm_katex-font\">$\\angle$</span>$BCA$ ` ` (isosceles)\n\n<div class=\"aligned\">\n\n\n| | |\n| -----------------------: | -------------------------------------------- |\n| $\\large x$ + $\\large x$ + 65 | = 180 |\n| 2$\\large x$ | = 115 |\n| $\\therefore \\large x$ | = {{{correctAnswer}}} |\n\n</div>"}]},{"vars":[{"varval":"Triangle $ABC$ is an isosceles triangle.\n\n<br>\n\nsm_img https://teacher.smartermaths.com.au/wp-content/uploads/2023/01/Geom_50035_v1.svg 200 indent3 vpad\n\n<br>\n\nWhat is the size of the angle <span class=\"sm_katex-font\">$\\angle$</span>$ABC$ ?"},{"varval":"sm_nogap Let &nbsp;$\\large x$ = <span class=\"sm_katex-font\">$\\angle$</span>$ABC$ = <span class=\"sm_katex-font\">$\\angle$</span>$BAC$ ` ` (Base angles of an isosceles $\\Delta$ equal)\n\n<div class=\"aligned\">\n\n\n| | |\n| -----------------------: | -------------------------------------------- |\n| 2$\\large x$ + 40 | = 180 |\n| 2$\\large x$ | = 140 |\n| $\\therefore \\large x$ | = {{{correctAnswer}}} |\n\n</div>"}]},{"vars":[{"varval":"Triangle $ABC$ is an isosceles triangle.\n\n<br>\n\nsm_img https://teacher.smartermaths.com.au/wp-content/uploads/2023/01/Geom_50035_v2.svg 220 indent3 vpad\n\n<br>\n\nWhat is the size of the angle <span class=\"sm_katex-font\">$\\angle$</span>$ABC$ ?"},{"varval":"sm_nogap Let &nbsp;$\\large x$ = <span class=\"sm_katex-font\">$\\angle$</span>$ABC$ = <span class=\"sm_katex-font\">$\\angle$</span>$BCA$ ` ` (Base angles of an isosceles $\\Delta$ equal)\n\n<div class=\"aligned\">\n\n\n| | |\n| -----------------------: | -------------------------------------------- |\n| 2$\\large x$ + 64 | = 180 |\n| 2$\\large x$ | = 116 |\n| $\\therefore \\large x$ | = {{{correctAnswer}}} |\n\n</div>"}]},{"vars":[{"varval":"Triangle $ABC$ is an isosceles triangle.\n\n<br>\n\nsm_img https://teacher.smartermaths.com.au/wp-content/uploads/2023/01/Geom_50035_v3.svg 200 indent3 vpad\n\n<br>\n\nWhat is the size of the angle <span class=\"sm_katex-font\">$\\angle$</span>$ABC$ ?"},{"varval":"sm_nogap Let &nbsp;$\\large x$ = <span class=\"sm_katex-font\">$\\angle$</span>$ABC$ = <span class=\"sm_katex-font\">$\\angle$</span>$BAC$ ` ` (Base angles of an isosceles $\\Delta$ equal)\n\n<div class=\"aligned\">\n\n\n| | |\n| -----------------------: | -------------------------------------------- |\n| 2$\\large x$ + 44 | = 180 |\n| 2$\\large x$ | = 136 |\n| $\\therefore \\large x$ | = {{{correctAnswer}}} |\n\n</div>\n"}]},{"vars":[{"varval":"Triangle $ABC$ is an isosceles triangle.\n\n<br>\n\nsm_img https://teacher.smartermaths.com.au/wp-content/uploads/2023/01/Geom_50035_v4.svg 280 indent3 vpad\n\n<br>\n\nWhat is the size of the angle <span class=\"sm_katex-font\">$\\angle$</span>$ABC$ ?"},{"varval":"sm_nogap Let &nbsp;$\\large x$ = <span class=\"sm_katex-font\">$\\angle$</span>$ABC$ = <span class=\"sm_katex-font\">$\\angle$</span>$BCA$ ` ` (Base angles of an isosceles $\\Delta$ equal)\n\n<div class=\"aligned\">\n\n\n| | |\n| -----------------------: | -------------------------------------------- |\n| 2$\\large x$ + 152 | = 180 |\n| 2$\\large x$ | = 28 |\n| $\\therefore \\large x$ | = {{{correctAnswer}}} |\n\n</div>"}]},{"vars":[{"varval":"Triangle $ABC$ is an isosceles triangle.\n\n<br>\n\nsm_img https://teacher.smartermaths.com.au/wp-content/uploads/2023/01/Geom_50035_v5.svg 200 indent3 vpad\n\n<br>\n\nWhat is the size of the angle <span class=\"sm_katex-font\">$\\angle$</span>$ABC$ ?"},{"varval":"sm_nogap Let &nbsp;$\\large x$ = <span class=\"sm_katex-font\">$\\angle$</span>$ABC$ = <span class=\"sm_katex-font\">$\\angle$</span>$BAC$ ` ` (Base angles of an isosceles $\\Delta$ equal)\n\n<div class=\"aligned\">\n\n\n| | |\n| -----------------------: | -------------------------------------------- |\n| 2$\\large x$ + 106 | = 180 |\n| 2$\\large x$ | = 74 |\n| $\\therefore \\large x$ | = {{{correctAnswer}}} |\n\n</div>"}]}]

  1316. Geometry, NAP-L4-CA15

    Craig is a town planner and needs to know the angles that streets make with each other. <br> sm_img https://teacher.smartermaths.com.au/wp-content/uploads/2020/01/nap-L4-15-ver1.svg 250 indent3 vpad <br> He knows that Tombs Street and Horan Street are parallel. What is the size of the shaded angle on the map?

    [{"vars":null}]

  1317. Geometry, NAP-G4-CA08

    Eloise makes a sketch of the playground at her school. <br> sm_img //teacher.smartermaths.com.au/wp-content/uploads/2016/12/naplan-2014-8mc.png 220 indent3 vpad <br> What is the size of angle $\large x$°?

    [{"vars":null}]

  1318. Geometry, NAP-A4-NC03

    <div class="sm_mode"> sm_img https://teacher.smartermaths.com.au/wp-content/uploads/2017/12/nap-A4-nc03.svg 205 indent3 vpad <br> What is the value of $\large x$? </div>

    [{"vars":null}]

  1319. #21

    {{name}} is uploading a file onto her computer. If {{percent}}% of the file has been successfully uploaded, what fraction of the file remains to be uploaded?

    [{"vars":[{"varval":"Karen"},{"varval":"45"},{"varval":"$\\dfrac{9}{20}$"}]},{"vars":[{"varval":"Matilda"},{"varval":"55"},{"varval":"$\\dfrac{11}{20}$"}]},{"vars":[{"varval":"Jackie"},{"varval":"85"},{"varval":"$\\dfrac{17}{20}$"}]},{"vars":[{"varval":"Margot"},{"varval":"65"},{"varval":"$\\dfrac{13}{20}$"}]},{"vars":[{"varval":"Billie"},{"varval":"35"},{"varval":"$\\dfrac{7}{20}$"}]}]

  1320. #21

    What is the highest common factor of {{algebra1}} and {{algebra2}}?

    [{"vars":[{"varval":"$6 \\large ab$"},{"varval":"$2 \\large b$"},{"varval":"3 is not a factor of $2 \\large b$\n\n$2 \\large b$ is a factor of both $\\checkmark$\n\n$3 \\large a$ is not a factor of $2 \\large b$\n\n$6 \\large ab$ is not a factor of $2 \\large b$."}]},{"vars":[{"varval":"$8 \\large a$"},{"varval":"$4 \\large ab$"},{"varval":"$2 \\large b$ is not a factor of $8 \\large a$\n\n8 is not a factor of $4 \\large ab$\n\n$4 \\large a$ is a factor of both $\\checkmark$\n\n$\\large a$ is a (smaller) factor of both $\\checkmark$"}]},{"vars":[{"varval":"$3 \\large b$"},{"varval":"$9 \\large ab$"},{"varval":"$\\large b$ is a factor of both $\\checkmark$\n\n$3 \\large a$ is not a factor of $3 \\large b$\n\n$9 \\large ab$ is not a factor of $3 \\large b$\n\n$3 \\large b$ is a (higher) factor of both $\\checkmark$"}]},{"vars":[{"varval":"$10 \\large bc$"},{"varval":"$2 \\large c$"},{"varval":"$10 \\large bc$ is not a factor of $2 \\large c$\n\n$5 \\large b$ is not a factor of $2 \\large c$\n\n$2 \\large c$ is a factor of both $\\checkmark$\n\n2 is a (smaller) factor of both $\\checkmark$"}]},{"vars":[{"varval":"$4 \\large a$"},{"varval":"$20 \\large ab$"},{"varval":"4 is a factor of both $\\checkmark$\n\n$4 \\large a$ is a (higher) factor of both $\\checkmark$\n\n$5 \\large b$ is not a factor of $4 \\large a$\n\n$20 \\large ab$ is not a factor of $4 \\large a$"}]}]

  1321. #20

    {{name}} scores an average of {{avg1}} {{type}} in his first three {{game}} games. {{question}} does he need to get in his next game to increase his average to {{avg2}}?

    [{"vars":[{"varval":"Stan"},{"varval":"40"},{"varval":"runs"},{"varval":"cricket"},{"varval":"What score"},{"varval":"45"},{"varval":"180"},{"varval":"120"},{"varval":"Runs"}]},{"vars":[{"varval":"Luc"},{"varval":"34"},{"varval":"points"},{"varval":"basketball"},{"varval":"How many points"},{"varval":"36"},{"varval":"144"},{"varval":"102"},{"varval":"Points"}]},{"vars":[{"varval":"Don"},{"varval":"60"},{"varval":"runs"},{"varval":"cricket"},{"varval":"What score"},{"varval":"66"},{"varval":"264"},{"varval":"180"},{"varval":"Runs"}]},{"vars":[{"varval":"Lebron"},{"varval":"28"},{"varval":"points"},{"varval":"basketball"},{"varval":"How many points"},{"varval":"31"},{"varval":"124"},{"varval":"84"},{"varval":"Points"}]},{"vars":[{"varval":"Steve"},{"varval":"64"},{"varval":"runs"},{"varval":"cricket"},{"varval":"What score"},{"varval":"70"},{"varval":"280"},{"varval":"192"},{"varval":"Runs"}]}]

  1322. #20

    The diagram below shows the front view of a house in the shape of a triangular prism. <br> {{image}} <br> What is the size of the angle $\large a\degree$?

    [{"vars":[{"varval":"75"},{"varval":"sm_img https://teacher.smartermaths.com.au/wp-content/uploads/2020/07/Q19var1.svg 160 indent3 vpad"}]},{"vars":[{"varval":"65"},{"varval":"sm_img https://teacher.smartermaths.com.au/wp-content/uploads/2020/07/Q19var2.svg 240 indent3 vpad"}]},{"vars":[{"varval":"45"},{"varval":"sm_img https://teacher.smartermaths.com.au/wp-content/uploads/2020/07/Q19var3.svg 400 indent3 vpad"}]},{"vars":[{"varval":"35"},{"varval":"sm_img https://teacher.smartermaths.com.au/wp-content/uploads/2020/07/Q19var4.svg 400 indent3 vpad"}]},{"vars":[{"varval":"55"},{"varval":"sm_img https://teacher.smartermaths.com.au/wp-content/uploads/2020/07/Q19var5.svg 330 indent3 vpad"}]}]

  1323. Number, NAPX-L4-07 v2, NAPX-L3-CA07 v2

    <div class="sm_mode"> {{{question}}} </div>

    [{"vars":[{"varval":"This table shows the fractions of the Australian students in University courses.\n\n\n<div class=\"sm-table row1-color8\">\n\n>>| Course | Fraction of students |\n|:-:|:-:|\n| Nursing| $\\dfrac{1}{15}$|\n| Medicine| $\\dfrac{1}{25}$|\n| Accounting| $\\dfrac{1}{40}$|\n| Science| $\\dfrac{1}{12}$|\n\n</div>\n\n<br>Which of these courses has the least number of students?\n"},{"varval":"Smallest fraction = $\\dfrac{1}{40}$\n\n$\\therefore$ Least students are studying {{{correctAnswer}}}"}]},{"vars":[{"varval":"This table shows the fractions of total beach going tourists at some chosen Sydney beaches on the weekend.\n\n\n<div class=\"sm-table row1-color3\">\n\n>>| Course | Fraction of tourists |\n|:-:|:-:|\n| Bondi| $\\dfrac{1}{8}$|\n| Clovelly| $\\dfrac{1}{15}$|\n| Coogee| $\\dfrac{1}{12}$|\n| Narrabeen| $\\dfrac{1}{18}$|\n\n</div>\n\n<br>Which of these beaches has the least number of tourists?\n"},{"varval":"Smallest fraction = $\\dfrac{1}{18}$\n\n$\\therefore$ Least visitors are at {{{correctAnswer}}}"}]},{"vars":[{"varval":"This table shows the fractions of Year 8 students choosing technology subjects.\n\n\n<div class=\"sm-table row1-color1\">\n\n>>| Course | Fraction of students |\n|:-:|:-:|\n| Food Technology| $\\dfrac{1}{7}$|\n| Design and Technology| $\\dfrac{1}{8}$|\n| Information Technology| $\\dfrac{1}{10}$|\n| Timber Technology| $\\dfrac{1}{6}$|\n\n</div>\n\n<br>Which of these subjects has the least number of students?\n"},{"varval":"Smallest fraction = $\\dfrac{1}{10}$\n\n$\\therefore$ Least students are choosing {{{correctAnswer}}}"}]},{"vars":[{"varval":"This table shows the fractions of the Year 7 students in some language courses.\n\n\n<div class=\"sm-table row1-color5\">\n\n>>| Course | Fraction of students |\n|:-:|:-:|\n| Japanese| $\\dfrac{1}{25}$|\n| French| $\\dfrac{1}{20}$|\n| Spanish| $\\dfrac{1}{13}$|\n| German| $\\dfrac{1}{5}$|\n\n</div>\n\n<br>Which of these courses has the least number of students?\n"},{"varval":"Smallest fraction = $\\dfrac{1}{25}$\n\n$\\therefore$ Least students are studying {{{correctAnswer}}}"}]},{"vars":[{"varval":"This table shows the fractions of a company's total employees in some departments.\n\n\n<div class=\"sm-table row1-color1\">\n\n>>| Course | Fraction of employees|\n|:-:|:-:|\n| Accounting| $\\dfrac{1}{13}$|\n| Health and Safety| $\\dfrac{1}{14}$|\n| Sales| $\\dfrac{1}{4}$|\n| Management| $\\dfrac{1}{5}$|\n\n</div>\n\n<br>Which of these departments has the least number of employees?\n"},{"varval":"Smallest fraction = $\\dfrac{1}{14}$\n\n$\\therefore$ Least employees are in {{{correctAnswer}}}"}]},{"vars":[{"varval":"This table shows the fractions of the students in a school who do some organised sport outside of school.\n\n\n<div class=\"sm-table row1-color8\">\n\n>>| Sport| Fraction of students |\n|:-:|:-:|\n| Soccer| $\\dfrac{1}{3}$|\n| Netball| $\\dfrac{1}{4}$|\n| Rugby League| $\\dfrac{1}{12}$|\n| Tenpin Bowling| $\\dfrac{1}{6}$|\n\n</div>\n\n<br>Which of these sports has the least number of students?"},{"varval":"Smallest fraction = $\\dfrac{1}{12}$\n\n$\\therefore$ Least students are playing {{{correctAnswer}}}"}]}]

  1324. Q20

    <div class ="sm_mode"> <div class="outline"> The daily high and low are measured in four ski resorts and recorded in the table below. <br> > >| Ski Resort | Low | High | > | ----------- | :----------: | :------------: | > | {{resort1}} | {{lowtemp1}} | {{hightemp1}} | > | {{resort2}} | {{lowtemp2}} | {{hightemp2}} | > | {{resort3}} | {{lowtemp3}} | {{hightemp3}} | > | {{resort4}} | {{lowtemp4}} | {{hightemp4}} | <br> </div> <br> </div> Which resort had a temperature range of {{degree}} degrees?

    [{"vars":[{"varval":"Thredbo"},{"varval":"$-1\\degree$"},{"varval":"6$\\degree$"},{"varval":"Charlotte's Pass"},{"varval":"$-5\\degree$"},{"varval":"1$\\degree$"},{"varval":"Mount Hotham"},{"varval":"$-2\\degree$"},{"varval":"3$\\degree$"},{"varval":"Perisher"},{"varval":"$-4\\degree$"},{"varval":"4$\\degree$"},{"varval":"6"},{"varval":"1"},{"varval":"$-5$"}]},{"vars":[{"varval":"Crackenback"},{"varval":"$-2\\degree$"},{"varval":"4$\\degree$"},{"varval":"Perisher"},{"varval":"$-5\\degree$"},{"varval":"5$\\degree$"},{"varval":"Mount Hotham"},{"varval":"$-3\\degree$"},{"varval":"2$\\degree$"},{"varval":"Thredbo"},{"varval":"$-5\\degree$"},{"varval":"1$\\degree$"},{"varval":"5"},{"varval":"2"},{"varval":"$-3$"}]},{"vars":[{"varval":"Perisher"},{"varval":"$-2\\degree$"},{"varval":"5$\\degree$"},{"varval":"Crackenback"},{"varval":"$-7\\degree$"},{"varval":"1$\\degree$"},{"varval":"Mount Hotham"},{"varval":"$-3\\degree$"},{"varval":"3$\\degree$"},{"varval":"Thredbo"},{"varval":"$-7\\degree$"},{"varval":"7$\\degree$"},{"varval":"7"},{"varval":"5"},{"varval":"$-2$"}]},{"vars":[{"varval":"Mount Hotham"},{"varval":"$-3\\degree$"},{"varval":"2$\\degree$"},{"varval":"Thredbo"},{"varval":"$1\\degree$"},{"varval":"4$\\degree$"},{"varval":"Perisher"},{"varval":"$-1\\degree$"},{"varval":"4$\\degree$"},{"varval":"Crackenback"},{"varval":"$-2\\degree$"},{"varval":"2$\\degree$"},{"varval":"4"},{"varval":"2"},{"varval":"$-2$"}]},{"vars":[{"varval":"Crackenback"},{"varval":"$-3\\degree$"},{"varval":"4$\\degree$"},{"varval":"Perisher"},{"varval":"$-8\\degree$"},{"varval":"1$\\degree$"},{"varval":"Mount Hotham"},{"varval":"$-2\\degree$"},{"varval":"8$\\degree$"},{"varval":"Thredbo"},{"varval":"$-4\\degree$"},{"varval":"4$\\degree$"},{"varval":"8"},{"varval":"4"},{"varval":"$-4$"}]}]

  1325. Number, NAPX-E4-NC03

    Cinder grows large pumpkins in her garden. One pumpkin she picks weighs 2.4 kg. Cinder cuts 400 grams off the pumpkin to cook with. What fraction has she used for cooking?

    [{"vars":null}]

  1326. Q21

    At a {{store}}, {{mass}} kilograms of {{product}} costs ${{cost1}}. What mass of {{product}} could be purchased for ${{cost2}}?

    [{"vars":[{"varval":"health food store"},{"varval":"1.4"},{"varval":"dried fruit"},{"varval":"11.20"},{"varval":"48"},{"varval":"8.00"}]},{"vars":[{"varval":"health food store"},{"varval":"1.6"},{"varval":"mixed nuts"},{"varval":"14.40"},{"varval":"45"},{"varval":"9.00"}]},{"vars":[{"varval":"butcher"},{"varval":"1.8"},{"varval":"pork belly"},{"varval":"21.60"},{"varval":"72"},{"varval":"12.00"}]},{"vars":[{"varval":"pet store"},{"varval":"1.4"},{"varval":"dog food"},{"varval":"9.10"},{"varval":"52"},{"varval":"6.50"}]},{"vars":[{"varval":"health food store"},{"varval":"1.7"},{"varval":"walnuts"},{"varval":"18.70"},{"varval":"55"},{"varval":"11.00"}]}]

  1327. Number, NAPX-I3-CA10

    Dominique had 1 kilogram of chocolate. She used $\dfrac{2}{5}$ kilogram in a cake recipe. How much chocolate did Dominique have left?

    [{"vars":null}]

  1328. <div class="sm_mode"> {{{question}}} </div>

    [{"vars":[{"varval":"Nick had tests to check his health that are summarised in the table below.\n\n<br>\n\n<div class=\"sm-table row1-color8\">\n\n>>| Test | Normal Range| Nick's Results |\n|:-:|:-:|:-:|\n| Vitamin E| 0.3 to 2.1 | 0.9|\n| Blood sugar| 3.8 to 5.7| 3.6|\n| Stool cooties|46 to 83| 59|\n\n</div>\n\n<br>\n\nFor which tests were Nick's results within the normal range?\n"},{"varval":"Checking each test:\n\n0.9 is within 0.3 and 2.1 &nbsp;$\\checkmark$\n\n3.6 is not within 3.8 and 5.7 &nbsp;X\n\n59 is within 46 and 83 &nbsp;$\\checkmark$\n\n$\\therefore$ {{{correctAnswer}}}\n"}]},{"vars":[{"varval":"Sandra had tests to check her health that are summarised in the table below.\n\n<br>\n\n<div class=\"sm-table row1-color8\">\n\n>>| Test | Normal Range| Sandra's Results |\n|:-:|:-:|:-:|\n| Potassium| 3.5 to 5.3 | 2.9|\n| Cholesterol| less than 5.0| 4.1|\n| Chloride|98 to 106| 75|\n\n</div>\n\n<br>\n\nFor which tests were Sandra's results **not** within the normal range?\n"},{"varval":"Checking each test:\n\n2.9 is not within 3.5 and 5.3 &nbsp;X\n\n4.1 is less than 5.0 &nbsp;$\\checkmark$\n\n75 is not within 98 and 106 &nbsp;X\n\n$\\therefore$ {{{correctAnswer}}}\n"}]},{"vars":[{"varval":"Summer had tests to check her health that are summarised in the table below.\n\n<br>\n\n<div class=\"sm-table row1-color8\">\n\n>>| Test | Normal Range| Summer's Results |\n|:-:|:-:|:-:|\n| Magnesium| 0.7 to 1.0 | 0.75|\n| Blood sugar| 3.8 to 5.7| 4.8|\n| Sodium|133 to 146| 109|\n\n</div>\n\n<br>\n\nFor which tests were Summer's results within the normal range?\n"},{"varval":"Checking each test:\n\n0.75 is within 0.7 and 1.0 &nbsp;$\\checkmark$\n\n4.8 is within 3.8 and 5.7 &nbsp;$\\checkmark$\n\n109 is not within 46 and 83 &nbsp;X\n\n$\\therefore$ {{{correctAnswer}}}\n"}]},{"vars":[{"varval":"Victor had tests to check his health that are summarised in the table below.\n\n<br>\n\n<div class=\"sm-table row1-color8\">\n\n>>| Test | Normal Range| Victor's Results |\n|:-:|:-:|:-:|\n| Vitamin E| 0.3 to 2.1 | 0.2|\n| Magnesium| 0.7 to 1.0| 1.3|\n| Resting heart rate|60 to 95| 104|\n\n</div>\n\n<br>\n\nFor which tests were Victor's results not within the normal range?\n"},{"varval":"Checking each test:\n\n0.2 is not within 0.3 and 2.1 &nbsp;X\n\n1.3 is not within 0.7 and 1.0 &nbsp;X\n\n104 is not within 60 and 95 &nbsp;X\n\n$\\therefore$ {{{correctAnswer}}}\n"}]},{"vars":[{"varval":"Gino had tests to check his health that are summarised in the table below.\n\n<br>\n\n<div class=\"sm-table row1-color8\">\n\n>>| Test | Normal Range| Gino's Results |\n|:-:|:-:|:-:|\n| Potassium| 3.5 to 5.3 | 4.2|\n| Magnesium| 0.7 to 1.0| 0.89|\n| Sodium|133 to 146| 145|\n\n</div>\n\n<br>\n\nFor which tests were Gino's results within the normal range?\n"},{"varval":"Checking each test:\n\n4.2 is within 3.5 and 5.3 &nbsp;$\\checkmark$\n\n0.89 is within 0.7 and 1.0 &nbsp;$\\checkmark$\n\n145 is within 133 and 146 &nbsp;$\\checkmark$\n\n$\\therefore$ {{{correctAnswer}}}\n"}]},{"vars":[{"varval":"Trev had tests to check his health that are summarised in the table below.\n\n<br>\n\n<div class=\"sm-table row1-color8\">\n\n>>| Test | Normal Range| Trev's Results |\n|:-:|:-:|:-:|\n| Calcium| 98 to 106 | 102|\n| Albumin | 35 to 50| 75|\n| Blood sugar|3.8 to 6.1| 5.7|\n\n</div>\n\n<br>\n\nFor which tests were Trev's results within the normal range?\n"},{"varval":"Checking each test:\n\n102 is within 98 and 106 &nbsp;$\\checkmark$\n\n75 is not within 35 and 50 &nbsp;X\n\n5.7 is within 3.8 and 6.1 &nbsp;$\\checkmark$\n\n$\\therefore$ {{{correctAnswer}}}"}]}]

  1329. A netball game had a crowd of 2000 fans. One quarter of the fans supported the home team. How many fans did **not** support the home team?

    [{"vars":null}]

  1330. Number, NAPX-I3-CA08

    Rufus had 5 cups of pumpkin seeds to use for baking 2 loaves of pumpkin bread. He used $1 \dfrac{3}{4}$ cups for the first loaf, and $1 \dfrac{1}{2}$ cups for the second loaf. How many cups of pumpkin seeds did Rufus have left?

    [{"vars":null}]

  1331. Number, NAPX-J3-CA06 SA

    There are 28 Year 7 students at a small high school. Each student is asked if they have a trampoline at home or not and the results are recorded. $\dfrac{3}{4}$ of the students answered yes. How many Year 7 students at the school have a trampoline at home?

    [{"vars":null}]

  1332. Geometry, NAP-C4-NC18

    Harry lives on a street that runs directly east-west. <br> sm_img https://teacher.smartermaths.com.au/wp-content/uploads/2017/01/NAP-Y7-2010-NC23.png 400 indent3 vpad <br> Harry's house is east of town hall and north of the pool. What street does Harry live in?

    [{"vars":null}]

  1333. Geometry, NAP-G3-CA20

    The distance from Village $A$ to Village $C$ is 6 cm on the map. <br> sm_img https://teacher.smartermaths.com.au/wp-content/uploads/2016/12/naplan-2014-17mci.png 400 indent3 vpad <br> What is the actual distance from Village $A$ to Village $C$?

    [{"vars":null}]

  1334. Geometry, NAP-F3-CA21

    Brian enters a park at one of the entries shown below. The map is drawn to scale but no scale is given. <br> sm_img https://teacher.smartermaths.com.au/wp-content/uploads/2017/01/naplan-2013-15mc.png 410 indent3 vpad <br> He moves 60 metres in a north-east direction and then turns and walks 30 metres in a south-east direction. Which point on the map shows where Brian ended up?

    [{"vars":null}]

  1335. Geometry, NAP-L4-CA14

    Roger starts walking due West. He makes a quarter turn right and keeps walking. Then, he makes another quarter turn right and stops. What direction is Roger now facing?

    [{"vars":null}]

  1336. Number, NAPX-H3-NC02

    Kelsey threw a shot put in the school athletics carnival and landed it where the arrow marker is shown below. <br> sm_img https://teacher.smartermaths.com.au/wp-content/uploads/2018/07/NAPX-H3-NC02.svg 550 indent vpad <br> Which of these is closest to the length of his throw?

    [{"vars":null}]

  1337. Geometry, NAP-K4-CA12

    Cameron is camping with 4 friends. They have individual tents shown on the diagram below. <br> sm_img https://teacher.smartermaths.com.au/wp-content/uploads/2018/07/NAPX-K4-CA12.svg 450 indent3 vpad <br> Brandon's tent is north-west of Cameron and Dan's tent is to Cameron's north. What is Cameron's position?

    [{"vars":null}]

  1338. Geometry, NAP-C4-NC10

    A car travelling south-west along Jake Road takes a 90° turn left into Benson Road. <br> sm_img https://teacher.smartermaths.com.au/wp-content/uploads/2017/02/NAP-C4-NC101.png 350 indent3 vpad <br> In which direction will the car be travelling along Benson Road?

    [{"vars":null}]

  1339. Geometry, NAP-A3-CA32

    Here is a map of Crikey Island. <br> sm_img https://teacher.smartermaths.com.au/wp-content/uploads/2017/12/NAP-A4-CA10.svg 430 indent3 vpad <br> Which one of these points is on Crikey Island?

    [{"vars":null}]

  1340. Geometry, NAP-D4-NC01

    Chevy drives from the car yard to the petrol station. <br> sm_img https://teacher.smartermaths.com.au/wp-content/uploads/2017/02/NAP-D4-NC011.png 550 indent3 vpad <br> Which directions best describe his drive there?

    [{"vars":null}]

  1341. Geometry, NAP-E4-CA02

    A map of a park shows four different gates. <br> sm_img https://teacher.smartermaths.com.au/wp-content/uploads/2017/01/naplan-2012-2mc.png 450 indent3 vpad <br> If Alan enters the park and is travelling in south-east direction, what gate did he enter the park by?

    [{"vars":null}]

  1342. Number, NAP-B3-CA05

    Two places are 5.4 cm apart on a map. On the map 1 cm represents 4 km. What is the actual distance between the two places?

    [{"vars":null}]

  1343. Geometry, NAP-F4-CA02

    The distance between Newcastle and Canberra on the map below is 9 cm. <br> sm_img https://teacher.smartermaths.com.au/wp-content/uploads/2016/12/naplan-2013-2mci.png 400 indent3 vpad <br> The scale of the map is 1 cm = 50 km. What is the actual distance between Newcastle and Canberra?

    [{"vars":null}]

  1344. Geometry, NAP-B4-CA01

    Roger is driving on Bond St in the direction of the arrows in the map below. <br> sm_img https://teacher.smartermaths.com.au/wp-content/uploads/2017/12/NAP-B4-CA01.png 400 indent3 vpad <br>In what direction is Roger driving?

    [{"vars":null}]

  1345. Measurement, NAPX-J4-CA23

    Buzz is placing door trim on the top and both sides of a double door entrance, as pictured below. <br> sm_img https://teacher.smartermaths.com.au/wp-content/uploads/2018/04/NAPX-J4-CA23.svg 420 indent3 vpad <br> How many metres of door trim does he need?

    [{"vars":null}]

  1346. Measurement, NAPX-G4-CA16, NAPX-G3-CA19

    The level of water in a dam is measured by the indicator below. <br> sm_img https://teacher.smartermaths.com.au/wp-content/uploads/2018/05/NAPX-G4-CA16.svg 188 indent3 vpad <br> What is the height of the water in the dam?

    [{"vars":null}]

  1347. Measurement, NAPX-H4-NC10

    The volume of the liquid in the measuring cylinder below is closest to: <br> sm_img https://teacher.smartermaths.com.au/wp-content/uploads/2018/05/NAPX-H4-NC101.svg 150 indent3 vpad

    [{"vars":null}]

  1348. Measurement, NAPX-F4-NC10, NAPX-F3-NC19

    Merv caught a fish and measured it, as shown in the diagram below. <br> sm_img https://teacher.smartermaths.com.au/wp-content/uploads/2018/07/NAPX-F4-NC10_2.svg 319 indent vpad <br> How long was Merv's fish, in centimetres?

    [{"vars":null}]

  1349. Measurement, NAPX-H4-CA05

    A woomera can throw a spear at a speed of 75 metres per second. What is the speed of the spear in metres per minute?

    [{"vars":null}]

  1350. Measurement, NAPX-J4-CA13

    Jayden measures the width of his car parking space. It is 2525 millimetres wide. An equivalent car park width is:

    [{"vars":null}]

  1351. Measurement, NAPX-H3-NC16

    <div class="sm_mode"> {{{question}}} </div>

    [{"vars":[{"varval":"A pasta factory packs 350 gram packets of pasta into a box that can hold 7 kilograms in total.\r\n\r\nWhich one of these expressions shows how many packets of pasta will be needed to fill the box?\n"},{"varval":"Since 7 kg = 7000 grams,\n\nPackets of pasta = {{{correctAnswer}}}"}]},{"vars":[{"varval":"A soft drink factory packs 375 millilitre cans of cola into a box that can hold 9 litres in total.\r\n\r\nWhich one of these expressions shows how many cans of cola will be needed to fill the box?"},{"varval":"Since 9 L = 9000 millilitres,\n\nCans of cola = {{{correctAnswer}}}"}]},{"vars":[{"varval":"A factory worker pours 800 millilitre bottles of barbecue sauce into a container that can hold 9.6 litres in total.\r\n\r\nWhich one of these expressions shows how many bottles of barbecue sauce will be needed to fill the container?"},{"varval":"9.6 L = 9600 millilitres,\n\n$\\therefore$ Bottles of barbecue sauce = {{{correctAnswer}}}"}]},{"vars":[{"varval":"An olive oil producer pours 600 millilitre bottles of olive oil into a container that can hold 12 bottles.\r\n\r\nWhich one of these expressions shows the total volume of the full container, in litres?"},{"varval":"Volume of container = 600 $\\times$ 12 = 7200 millilitres\n\n$\\rightarrow$ 1000 millilitres = 1 litre\n\nVolume of container in litres = {{{correctAnswer}}}"}]},{"vars":[{"varval":"A biscuit factory packs 250 gram packets of chocolate chip cookies into a box that can hold 24 packets.\r\n\r\nWhich one of these expressions shows the total mass of the box, in kilograms?"},{"varval":"Mass of box = 250 $\\times$ 24 = 6000 grams\n\n$\\rightarrow$ 1000 grams = 1 kilogram\n\nTotal mass of box in kilograms = {{{correctAnswer}}}"}]},{"vars":[{"varval":"A taco factory packs 450 gram packets of soft tacos into a box that can hold 12 packets.\r\n\r\nWhich one of these expressions shows the total mass of the box, in kilograms?"},{"varval":"Mass of box = 450 $\\times$ 12 = 5400 grams\n\n$\\rightarrow$ 1000 grams = 1 kilogram,\n\nTotal mass of box in kilograms = {{{correctAnswer}}}"}]}]

  1352. Measurement, NAPX-I4-NC02, NAPX-I3-NC05

    Laken measures the height of his basketball hoop to be 3 metres and 16 centimetres . Which answer below shows how Laken can also write this measurement in metres?

    [{"vars":null}]

  1353. {{name}} teaches {{subject}}. In one class {{type}}, she gives {{mass1}} {{unit}} of {{element1}} to one group, {{mass2}} {{unit}} to a second group and {{mass3}} {{unit}} to a third group. If {{name}} started with {{mass4}} {{unit}} of {{element1}}, how many {{unit}} does she have left?

    [{"vars":[{"varval":"Miranda"},{"varval":"food technology"},{"varval":"activity"},{"varval":"1.74"},{"varval":"kilograms"},{"varval":"flour"},{"varval":"0.96"},{"varval":"0.78"},{"varval":"8"},{"varval":"3.48"},{"varval":"Flour"}]},{"vars":[{"varval":"Elle"},{"varval":"art"},{"varval":"activity"},{"varval":"2.47"},{"varval":"metres"},{"varval":"lace"},{"varval":"1.39"},{"varval":"0.88"},{"varval":"7"},{"varval":"4.74"},{"varval":"Lace"}]},{"vars":[{"varval":"Claudia"},{"varval":"industrial technology"},{"varval":"activity"},{"varval":"1.57"},{"varval":"metres"},{"varval":"wood"},{"varval":"0.94"},{"varval":"1.64"},{"varval":"10"},{"varval":"4.15"},{"varval":"Wood"}]},{"vars":[{"varval":"Cindy"},{"varval":"food technology"},{"varval":"activity"},{"varval":"0.76"},{"varval":"kilograms"},{"varval":"sugar"},{"varval":"1.87"},{"varval":"0.69"},{"varval":"5"},{"varval":"3.32"},{"varval":"Sugar"}]},{"vars":[{"varval":"Giselle"},{"varval":"Chemistry"},{"varval":"experiment"},{"varval":"1.46"},{"varval":"litres"},{"varval":"solvent"},{"varval":"0.75"},{"varval":"0.97"},{"varval":"6"},{"varval":"3.18"},{"varval":"Solvent"}]}]

  1354. Geometry, NAPX-E4-CA04, NAPX-E3-CA08

    A sculpture is pictured from 3 different angles below: <br> sm_img https://teacher.smartermaths.com.au/wp-content/uploads/2020/09/NAPX-E4-CA04rev.svg 565 indent3 vpad <br> The sculpture is in the shape of

    [{"vars":null}]

  1355. Geometry, NAPX-L4-CA05 o2

    Janice draws a rectangle on a number plane as shown below. <br> sm_img https://teacher.smartermaths.com.au/wp-content/uploads/2020/03/NAP-L4-CA05.svg 365 indent3 vpad <br> What are the coordinates of point $S$?

    [{"vars":null}]

  1356. Geometry, NAPX-L4-CA05 o1

    Karl draws a rectangle on a number plane as shown below. <br> sm_img https://teacher.smartermaths.com.au/wp-content/uploads/2020/03/NAP-L4-CA05.svg 365 indent3 vpad <br> What are the coordinates of point $R$?

    [{"vars":null}]

  1357. Geometry, NAPX-F3-CA03 Var 5 ... can you simplify this one a bit. Getting too time consuming for a single question time allocation (thought the back right two cubes allocated 3 should be 4 also). Have adjusted the figure so it is easier.

    <div class="sm_mode"> {{{question}}} </div>

    [{"vars":[{"varval":"There are six cubes in this 3D puzzle.\n\n<br>\n\nsm_img https://teacher.smartermaths.com.au/wp-content/uploads/2018/08/NAPX-F3-CA03_1.svg 170 indent3 vpad\n\n<br>\n\nThe puzzle is completely dipped into blue paint. \n\nWhen the cubes are separated, how many faces will be blue?"},{"varval":"The number of blue faces on each cube is: \n<br>\n\nsm_img https://teacher.smartermaths.com.au/wp-content/uploads/2018/08/NAPX-F3-CA03-Answer1.svg 170 indent3\n\n<br>\n\n<div class=\"sm_mode\">\n\nsm_nogap Total faces blue\n\n> > \\= 5 + 5 + 3 + 4 + 4 + 5\\\n> > = 26\n\n</div>"}]},{"vars":[{"varval":"There are eight cubes in this 3D puzzle.\n\n<br>\n\nsm_img https://teacher.smartermaths.com.au/wp-content/uploads/2022/10/Geom_50026_v1q2.svg 230 indent3 vpad\n\n<br>\n\nThe puzzle is completely dipped into red paint. \n\nWhen the cubes are separated, how many faces will be red?"},{"varval":"The number of red faces on each cube is: \n\n<br>\n\nsm_img https://teacher.smartermaths.com.au/wp-content/uploads/2022/10/Geom_50026_v1ws.svg 230 indent3\n\n<br>\n\n<div class=\"sm_mode\">\n\nsm_nogap Total faces red\n\n> > \\= 5 + 3 + 5 + 4 + 4 + 4 + 4 + 3 \\\n> > = 32\n\n</div>\n"}]},{"vars":[{"varval":"There are seven cubes in this 3D puzzle.\n\n<br>\n\nsm_img https://teacher.smartermaths.com.au/wp-content/uploads/2022/10/Geom_50026_v2q1.svg 240 indent3 vpad\n\n<br>\n\nThe puzzle is completely dipped into red paint. \n\nWhen the cubes are separated, how many faces will be red?"},{"varval":"The number of red faces on each cube is: \n<br>\n\nsm_img https://teacher.smartermaths.com.au/wp-content/uploads/2022/10/Geom_50026_v2ws.svg 240 indent3\n\n<br>\n\n<div class=\"sm_mode\">\n\nsm_nogap Total faces red\n\n> > \\= 5 + 5 + 3 + 4 + 4 + 5 + 4\\\n> > = 30\n\n</div>"}]},{"vars":[{"varval":"There are six cubes in this 3D puzzle.\n\n<br>\n\nsm_img https://teacher.smartermaths.com.au/wp-content/uploads/2022/10/Geom_50026_v3q.svg 170 indent3 vpad\n\n<br>\n\nThe puzzle is completely dipped into red paint. \n\nWhen the cubes are separated, how many faces will be red?"},{"varval":"The number of red faces on each cube is: \n<br>\n\nsm_img https://teacher.smartermaths.com.au/wp-content/uploads/2022/10/Geom_50026_v3ws.svg 170 indent3\n\n<br>\n\n<div class=\"sm_mode\">\n\nsm_nogap Total faces red\n\n> > \\= 5 + 3 + 4 + 5 + 4 + 5\\\n> > = 26\n\n</div>"}]},{"vars":[{"varval":"There are nine cubes in this 3D puzzle.\n\n<br>\n\nsm_img https://teacher.smartermaths.com.au/wp-content/uploads/2022/10/Geom_50026_v4q.svg 230 indent3 vpad\n\n<br>\n\nThe puzzle is completely dipped into red paint. \n\nWhen the cubes are separated, how many faces will be red?"},{"varval":"The number of red faces on each cube is: \n<br>\n\nsm_img https://teacher.smartermaths.com.au/wp-content/uploads/2022/10/Geom_50026_v4ws.svg 240 indent3\n\n<br>\n\n<div class=\"sm_mode\">\n\nsm_nogap Total faces red\n\n> > \\= 5 + 5 + 3 + 3 + 3 + 5 + 4 + 5 + 3\\\n> > = 36\n\n</div>"}]},{"vars":[{"varval":"There are ten cubes in this 3D puzzle. \n\n<br>\n\nsm_img https://teacher.smartermaths.com.au/wp-content/uploads/2022/10/Geom_50026_v5qa_ws.svg 260 indent3 vpad\n\n<br>\n\nThe puzzle is completely dipped into red paint. \n\nWhen the cubes are separated, how many faces will be red?"},{"varval":"The number of red faces on each cube is: \n<br>\n\nsm_img https://teacher.smartermaths.com.au/wp-content/uploads/2022/10/Geom_50026_v5qa.svg 260 indent3\n\n<br>\n\n<div class=\"sm_mode\">\n\nsm_nogap Total faces red\n\n> > \\= 5 + 4 + 5 + 3 + 3 + 3 + 3 + 4 + 3 + 5 \\\n> > = 38\n\n</div>"}]}]

  1358. Geometry, NAPX-G3-NC05

    Rufus made a sculpture. This is a drawing of the front view and the side view. <br> sm_img https://teacher.smartermaths.com.au/wp-content/uploads/2018/08/NAPX-G3-NC05.svg 250 indent3 vpad <br> Which of these is a correct top view?

    [{"vars":null}]

  1359. Geometry, NAPX-H4-CA03, NAPX-H3-CA09

    Lebron's right shoe is pictured below. <br> sm_img https://teacher.smartermaths.com.au/wp-content/uploads/2020/09/NAPX-H3-CA09.svg 225 indent3 vpad <br> His left shoe is a mirror image of his right shoe. Which of these is Lebron's left shoe?

    [{"vars":null}]

  1360. Geometry, NAPX-H4-CA01, NAPX-H3-CA02

    Boort made this 3D object using 32 small cubes. <br> sm_img https://teacher.smartermaths.com.au/wp-content/uploads/2020/09/NAPX-H4-CA012.svg 285 indent3 vpad <br> Which of these has a volume half that of Boort's object?

    [{"vars":null}]

  1361. Geometry, NAPX-G3-CA04

    Peter is making an animation of an elf digging. The picture shows six frames of his animation. <br> sm_img https://teacher.smartermaths.com.au/wp-content/uploads/2020/09/NAPX-G3-CA041.svg 360 indent3 vpad <br> He accidentally swapped 2 frames. Which two frames were swapped?

    [{"vars":null}]

  1362. Geometry, NAPX-H3-NC04

    This is a view of a solid made by stacking 21 identical cubes. <br> sm_img https://teacher.smartermaths.com.au/wp-content/uploads/2018/07/NAPX-H3-NC04.svg 235 indent3 vpad <br> How many cubes are completely hidden in this view?

    [{"vars":null}]

  1363. <div class="sm_mode"> The number of boys and girls in four {{club}} is recorded in the table below. <br> <div class="outline"> > > | Club | Boys | Girls | > > | --------- | :----: | :----: | > > | {{club1}} | {{b1}} | {{g1}} | > > | {{club2}} | {{b2}} | {{g2}} | > > | {{club3}} | {{b3}} | {{g3}} | > > | {{club4}} | {{b4}} | {{g4}} | <br> </div> <br> Which change will result in the {{clubname1}} and {{clubname2}} having the same number of players? </div>

    [{"vars":[{"varval":"soccer clubs"},{"varval":"Leopards"},{"varval":"Swans"},{"varval":"Tigers"},{"varval":"Wombats"},{"varval":"13"},{"varval":"10"},{"varval":"9"},{"varval":"9"},{"varval":"15"},{"varval":"12"},{"varval":"13"},{"varval":"9"},{"varval":"28"},{"varval":"22"},{"varval":"22"},{"varval":"18"},{"varval":"Leopards"},{"varval":"Wombats"}]},{"vars":[{"varval":"cricket clubs"},{"varval":"Lizards"},{"varval":"Bell Birds"},{"varval":"Eels"},{"varval":"Sharks"},{"varval":"16"},{"varval":"9"},{"varval":"13"},{"varval":"10"},{"varval":"12"},{"varval":"11"},{"varval":"15"},{"varval":"14"},{"varval":"28"},{"varval":"20"},{"varval":"28"},{"varval":"24"},{"varval":"Bell Birds"},{"varval":"Eels"}]},{"vars":[{"varval":"hockey clubs"},{"varval":"Lizards"},{"varval":"Bell Birds"},{"varval":"Eels"},{"varval":"Sharks"},{"varval":"16"},{"varval":"9"},{"varval":"13"},{"varval":"10"},{"varval":"12"},{"varval":"11"},{"varval":"15"},{"varval":"14"},{"varval":"28"},{"varval":"20"},{"varval":"28"},{"varval":"24"},{"varval":"Lizards"},{"varval":"Sharks"}]},{"vars":[{"varval":"Aussie Rules clubs"},{"varval":"Leopards"},{"varval":"Swans"},{"varval":"Tigers"},{"varval":"Wombats"},{"varval":"13"},{"varval":"10"},{"varval":"9"},{"varval":"9"},{"varval":"15"},{"varval":"12"},{"varval":"13"},{"varval":"9"},{"varval":"28"},{"varval":"22"},{"varval":"22"},{"varval":"18"},{"varval":"Leopards"},{"varval":"Tigers"}]},{"vars":[{"varval":"water polo clubs"},{"varval":"Rams"},{"varval":"Eagles"},{"varval":"Knights"},{"varval":"Poteroos"},{"varval":"6"},{"varval":"12"},{"varval":"11"},{"varval":"15"},{"varval":"16"},{"varval":"16"},{"varval":"14"},{"varval":"13"},{"varval":"22"},{"varval":"28"},{"varval":"25"},{"varval":"28"},{"varval":"Rams"},{"varval":"Poteroos"}]}]

  1364. <div class="sm_mode"> The quarterly cost $(C)$ of {{sport}} is \${{cost1}} for each session $(S)$ plus a fixed fee of \${{cost2}}. This can be expressed by the formula > $C = {{cost1}}S + {{cost2}}$ <br>If {{name}} pays ${{total}} for {{sport}} in a quarter, how many training sessions did she attend? </div>

    [{"vars":[{"varval":"swimming squad training"},{"varval":"5"},{"varval":"10"},{"varval":"Hannah"},{"varval":"190"},{"varval":"180"}]},{"vars":[{"varval":"tennis training"},{"varval":"4"},{"varval":"20"},{"varval":"Anna"},{"varval":"140"},{"varval":"120"}]},{"vars":[{"varval":"sprint training"},{"varval":"4"},{"varval":"30"},{"varval":"Michelle"},{"varval":"190"},{"varval":"160"}]},{"vars":[{"varval":"hockey training"},{"varval":"5"},{"varval":"10"},{"varval":"Miley"},{"varval":"160"},{"varval":"150"}]},{"vars":[{"varval":"squash training"},{"varval":"4"},{"varval":"40"},{"varval":"Rachel"},{"varval":"180"},{"varval":"140"}]}]

  1365. The shaded {{shape1}} has an area of {{area}} cm$^2$. <br> {{image}} <br> What is the volume of the {{shape2}} prism?

    [{"vars":[{"varval":"rectangle"},{"varval":"25"},{"varval":"sm_img https://teacher.smartermaths.com.au/wp-content/uploads/2020/06/VAR5Q17.svg 222 indent3 vpad"},{"varval":"rectangular"},{"varval":"8"}]},{"vars":[{"varval":"rectangle"},{"varval":"60"},{"varval":"rectangular"},{"varval":"4"},{"varval":"sm_img https://teacher.smartermaths.com.au/wp-content/uploads/2020/06/VAR2Q17.svg 120 indent3 vpad"}]},{"vars":[{"varval":"triangle"},{"varval":"80"},{"varval":"sm_img https://teacher.smartermaths.com.au/wp-content/uploads/2020/06/VAR3Q17.svg 221 indent3 vpad"},{"varval":"triangular"},{"varval":"5"}]},{"vars":[{"varval":"triangle"},{"varval":"40"},{"varval":"sm_img https://teacher.smartermaths.com.au/wp-content/uploads/2020/06/VAR4Q17.svg 120 indent3 vpad"},{"varval":"triangular"},{"varval":"3"}]},{"vars":[{"varval":"rectangle"},{"varval":"40"},{"varval":"sm_img https://teacher.smartermaths.com.au/wp-content/uploads/2020/06/VAR1Q17.svg 220 indent3 vpad"},{"varval":"rectangular"},{"varval":"8"}]}]

  1366. Q18

    How many numbers between {{limit1}} and {{limit2}} are divisible by {{divisor}}?

    [{"vars":[{"varval":"3"},{"varval":"59"},{"varval":"4"},{"varval":"10 multiples of 4 between 3 and 40."},{"varval":"4 multiples of 4 between 41 and 59"}]},{"vars":[{"varval":"5"},{"varval":"50"},{"varval":"4"},{"varval":"9 multiples of 4 between 5 and 40."},{"varval":"2 multiples of 4 between 41 and 50"}]},{"vars":[{"varval":"3"},{"varval":"80"},{"varval":"6"},{"varval":"10 multiples of 6 between 3 and 60"},{"varval":"3 multiples of 6 between 61 and 80"}]},{"vars":[{"varval":"5"},{"varval":"44"},{"varval":"3"},{"varval":"9 multiples of 3 between 5 and 30"},{"varval":"4 multiples of 3 between 31 and 44"}]},{"vars":[{"varval":"2"},{"varval":"70"},{"varval":"4"},{"varval":"10 multiples of 4 between 2 and 40"},{"varval":"7 multiples of 4 between 41 and 70"}]}]

  1367. Q17

    A farmer owns a {{size1}} acre property. {{gender1}} purchases one extra acre for every {{size2}} {{gender2}} currently owns. What is the new size of the land {{gender2}} owns?

    [{"vars":[{"varval":"20"},{"varval":"He"},{"varval":"4"},{"varval":"he"},{"varval":"5"}]},{"vars":[{"varval":"28"},{"varval":"He"},{"varval":"4"},{"varval":"he"},{"varval":"7"}]},{"vars":[{"varval":"30"},{"varval":"She"},{"varval":"5"},{"varval":"she"},{"varval":"6"}]},{"vars":[{"varval":"24"},{"varval":"He"},{"varval":"6"},{"varval":"he"},{"varval":"4"}]},{"vars":[{"varval":"32"},{"varval":"She"},{"varval":"4"},{"varval":"she"},{"varval":"8"}]}]

  1368. Geometry, NAP-B4-NC12

    A wooden block, the shape of a cylinder, is sawed into two pieces, vertically along the dashed line shown below. <br> sm_img https://teacher.smartermaths.com.au/wp-content/uploads/2017/12/nap-b4-nc12.svg 190 indent3 vpad <br> Which shows the shape of the cross-section made by the cut?

    [{"vars":null}]

  1369. Geometry, NAP-D4-CA09 Var1-5 ... the text in the plans are not centred vertically and horizontally on the version I am seeing like var0 (can you investigate if this is browser specific). I've redone var 1-5. Can you let me know how they look now?

    <div class="sm_mode"> {{{question}}} </div>

    [{"vars":[{"varval":"Jessie is arranging rectangular garden pots, as shown on the plan below.\n\n<br>\n\nsm_img //teacher.smartermaths.com.au/wp-content/uploads/2017/02/naplan-2011-9mc.png 220 indent3 vpad\n\n<br>\nWhich view of the pots would Jessie see?"},{"varval":"{{{correctAnswer}}}"}]},{"vars":[{"varval":"Joshua is arranging rectangular garden pots, as shown on the plan below.\n\n<br>\n\nsm_img https://teacher.smartermaths.com.au/wp-content/uploads/2022/09/Geom_50018_v1qa.svg 220 indent3 vpad\n\n<br>\nWhich view of the pots would Joshua see?"},{"varval":"{{{correctAnswer}}}"}]},{"vars":[{"varval":"Jo is arranging rectangular garden pots, as shown on the plan below.\n\n<br>\n\nsm_img https://teacher.smartermaths.com.au/wp-content/uploads/2022/09/Geom_50018_v2q.svg 230 indent3 vpad\n\n<br>\nWhich view of the pots would Jo see?"},{"varval":"{{{correctAnswer}}}"}]},{"vars":[{"varval":"Johnno is arranging rectangular garden pots, as shown on the plan below.\n\n<br>\n\nsm_img https://teacher.smartermaths.com.au/wp-content/uploads/2022/09/Geom_50018_v3q.svg 220 indent3 vpad\n\n<br>\nWhich view of the pots would Johnno see?"},{"varval":"{{{correctAnswer}}}"}]},{"vars":[{"varval":"Jane is arranging rectangular garden pots, as shown on the plan below.\n\n<br>\n\nsm_img https://teacher.smartermaths.com.au/wp-content/uploads/2022/09/Geom_50018_v4q.svg 220 indent3 vpad\n\n<br>\nWhich view of the pots would Jane see?"},{"varval":"{{{correctAnswer}}}"}]},{"vars":[{"varval":"Jacqui is arranging rectangular garden pots, as shown on the plan below.\n\n<br>\n\nsm_img https://teacher.smartermaths.com.au/wp-content/uploads/2022/09/Geom_50018_v5q.svg 220 indent3 vpad\n\n<br>\nWhich view of the pots would Jacqui see?"},{"varval":"{{{correctAnswer}}}"}]}]

  1370. Geometry, NAP-B4-NC09

    <div class="sm_mode"> {{{question}}} </div>

    [{"vars":[{"varval":"Grant joined the two objects below.\n\n<br>\n\nsm_img https://teacher.smartermaths.com.au/wp-content/uploads/2017/12/nap-b4-nc09.svg 160 indent3 vpad\n\n<br>\n\nWhich object below could **not** be made using Grant's 2 objects?"},{"varval":"{{{correctAnswer}}}"}]},{"vars":[{"varval":"Georgie joined the two objects below.\n\n<br>\n\nsm_img https://teacher.smartermaths.com.au/wp-content/uploads/2022/09/Geom_50017_v1.svg 160 indent3 vpad\n\n<br>\n\nWhich object below could **not** be made using Georgie's 2 objects?"},{"varval":"{{{correctAnswer}}}"}]},{"vars":[{"varval":"Graham joined the two objects below.\n\n<br>\n\nsm_img https://teacher.smartermaths.com.au/wp-content/uploads/2022/09/Geom_50017_v2.svg 160 indent3 vpad\n\n<br>\n\nWhich object below could **not** be made using Graham's 2 objects?"},{"varval":"{{{correctAnswer}}}"}]},{"vars":[{"varval":"Guilia joined the two objects below.\n\n<br>\n\nsm_img https://teacher.smartermaths.com.au/wp-content/uploads/2022/09/Geom_50017_v3.svg 200 indent3 vpad\n\n<br>\n\nWhich object below could **not** be made using Guilia's 2 objects?"},{"varval":"{{{correctAnswer}}}"}]},{"vars":[{"varval":"Griffen joined the two objects below.\n\n<br>\n\nsm_img https://teacher.smartermaths.com.au/wp-content/uploads/2022/09/Geom_50017_v4.svg 170 indent3 vpad\n\n<br>\n\nWhich object below could **not** be made using Griffen's 2 objects?"},{"varval":"{{{correctAnswer}}}"}]},{"vars":[{"varval":"Goldie joined the two objects below.\n\n<br>\n\nsm_img https://teacher.smartermaths.com.au/wp-content/uploads/2022/09/Geom_50017_v5.svg 210 indent3 vpad\n\n<br>\n\nWhich object below could **not** be made using Goldie's 2 objects?"},{"varval":"{{{correctAnswer}}}"}]}]

  1371. Geometry, NAP-G4-CA05

    <div class="sm_mode"> {{{question}}} </div>

    [{"vars":[{"varval":"Which shape below has 2 shaded triangles the same size and shape?"},{"varval":"{{{correctAnswer}}}"}]},{"vars":[{"varval":"Which shape below has 2 shaded triangles the same size and shape?"},{"varval":"{{{correctAnswer}}}"}]},{"vars":[{"varval":"Which shape below has 2 shaded triangles the same size and shape?"},{"varval":"{{{correctAnswer}}}"}]},{"vars":[{"varval":"Which shape below has 2 shaded triangles the same size and shape?"},{"varval":"{{{correctAnswer}}}"}]},{"vars":[{"varval":"Which shape below has 2 shaded triangles the same size and shape?"},{"varval":"{{{correctAnswer}}}"}]},{"vars":[{"varval":"Which shape below has 2 shaded triangles the same size and shape?"},{"varval":"{{{correctAnswer}}}"}]}]

  1372. Geometry, NAP-D4-NC11

    The top view and front view of a building are shown. <br> sm_img //teacher.smartermaths.com.au/wp-content/uploads/2017/02/NAP-D3-NC16.png 470 indent3 vpad <br> Which could be the side view of this building?

    [{"vars":null}]

  1373. Geometry, NAP-J4-CA09

    sm_img https://teacher.smartermaths.com.au/wp-content/uploads/2017/09/NAP-J1-091.png 230 indent3 vpad <br> Which of these is the front view of this object made from cubes?

    [{"vars":null}]

  1374. A {{item}} has a normal price tag of ${{price1}}. At sale time, it is reduced by 25%. What it the sale price of the {{item}}?

    [{"vars":[{"varval":"watch"},{"varval":"240"},{"varval":"60"}]},{"vars":[{"varval":"ring"},{"varval":"320"},{"varval":"80"}]},{"vars":[{"varval":"microwave"},{"varval":"160"},{"varval":"40"}]},{"vars":[{"varval":"fan"},{"varval":"80"},{"varval":"20"}]},{"vars":[{"varval":"skateboard"},{"varval":"280"},{"varval":"70"}]}]

  1375. Geometry, NAP-K4-CA01

    Which of these shows the top view of this stack of boxes? <br> sm_img https://teacher.smartermaths.com.au/wp-content/uploads/2018/06/NAPX-K4-CA01.svg 205 indent3 vpad

    [{"vars":null}]

  1376. Geometry, NAP-C4-NC01

    Seven cubes are joined to form the following object. <br> sm_img //teacher.smartermaths.com.au/wp-content/uploads/2017/01/NAP-C4-NC01.png 225 indent3 vpad <br> What will the shape look like from above?

    [{"vars":null}]

  1377. Geometry, NAP-B4-NC01

    The object below is made from 9 cubes <br> sm_img https://teacher.smartermaths.com.au/wp-content/uploads/2017/12/nap-b4-nc01rev.svg 235 indent3 vpad <br> Which one of these shows the top view?

    [{"vars":null}]

  1378. Geometry, NAP-G4-NC01

    This 2-dimensional shape pictured below is made from <br> sm_img https://teacher.smartermaths.com.au/wp-content/uploads/2016/12/NAP-149-NC01-259x300.png 190 indent3 vpad

    [{"vars":null}]

  1379. Geometry, NAP-D4-CA01

    A view of a soccer field is pictured below. <br> sm_img //teacher.smartermaths.com.au/wp-content/uploads/2017/02/naplan-2011-1mc.png 300 indent3 vpad <br> Which shape is **not** shown?

    [{"vars":null}]

  1380. Q16

    {{name}} receives ${{wage1}} an hour when he works weekdays and time and a half when he works on weekends. If he works for {{number1}} hours each day from Monday to Friday and {{number2}} hours on Sunday, how much will {{name}} be paid?

    [{"vars":[{"varval":"Rob"},{"varval":"20"},{"varval":"6"},{"varval":"3"},{"varval":"30"},{"varval":"600"},{"varval":"90"},{"varval":"\\text{\\textdollar 690}"}]},{"vars":[{"varval":"Donald"},{"varval":"30"},{"varval":"6"},{"varval":"2"},{"varval":"30"},{"varval":"900"},{"varval":"90"},{"varval":"\\text{\\textdollar 990}"}]},{"vars":[{"varval":"Roger"},{"varval":"40"},{"varval":"4"},{"varval":"5"},{"varval":"20"},{"varval":"800"},{"varval":"300"},{"varval":"\\text{\\textdollar 1100}"}]},{"vars":[{"varval":"Steve"},{"varval":"20"},{"varval":"6"},{"varval":"8"},{"varval":"30"},{"varval":"600"},{"varval":"240"},{"varval":"\\text{\\textdollar 840}"}]},{"vars":[{"varval":"Clem"},{"varval":"60"},{"varval":"4"},{"varval":"3"},{"varval":"20"},{"varval":"1200"},{"varval":"270"},{"varval":"\\text{\\textdollar 1470}"}]}]

  1381. Q15

    <div class="sm_mode"> {{name}} works at a {{market}} market. A customer purchases the following: * {{mass1}} grams of {{veg1}} at ${{cost1}} a kilogram. * {{mass2}} grams of {{veg2}} at ${{cost2}} a kilogram. * {{mass3}} grams of {{veg3}} at ${{cost3}} a kilogram. The customer pays with a ${{note}} note. What is the correct change {{name}} must give the customer? </div>

    [{"vars":[{"varval":"Con"},{"varval":"Fruit and Vegetable"},{"varval":"250"},{"varval":"grapes"},{"varval":"7"},{"varval":"300"},{"varval":"beans"},{"varval":"8"},{"varval":"750"},{"varval":"onions"},{"varval":"3"},{"varval":"20"},{"varval":"0.25"},{"varval":"0.3"},{"varval":"0.75"},{"varval":"1.75"},{"varval":"2.40"},{"varval":"2.25"}]},{"vars":[{"varval":"Alf"},{"varval":"Farmers"},{"varval":"400"},{"varval":"lemons"},{"varval":"6"},{"varval":"500"},{"varval":"kiwi fruit"},{"varval":"9"},{"varval":"250"},{"varval":"tomatoes"},{"varval":"8"},{"varval":"50"},{"varval":"0.4"},{"varval":"0.5"},{"varval":"0.25"},{"varval":"2.40"},{"varval":"4.50"},{"varval":"2.00"}]},{"vars":[{"varval":"Mimi"},{"varval":"Fresh Produce"},{"varval":"600"},{"varval":"potatoes"},{"varval":"5"},{"varval":"250"},{"varval":"apples"},{"varval":"7"},{"varval":"300"},{"varval":"grapes"},{"varval":"6"},{"varval":"20"},{"varval":"0.6"},{"varval":"0.25"},{"varval":"0.3"},{"varval":"3.00"},{"varval":"1.75"},{"varval":"1.80"}]},{"vars":[{"varval":"Chloe"},{"varval":"Fruit"},{"varval":"300"},{"varval":"passion fruit"},{"varval":"8"},{"varval":"750"},{"varval":"watermelon"},{"varval":"7"},{"varval":"500"},{"varval":"tomatoes"},{"varval":"5"},{"varval":"50"},{"varval":"0.3"},{"varval":"0.75"},{"varval":"0.5"},{"varval":"2.40"},{"varval":"5.25"},{"varval":"2.50"}]},{"vars":[{"varval":"Willy"},{"varval":"Fruit"},{"varval":"250"},{"varval":"limes"},{"varval":"9"},{"varval":"400"},{"varval":"watermelon"},{"varval":"5"},{"varval":"600"},{"varval":"grapes"},{"varval":"7"},{"varval":"50"},{"varval":"0.25"},{"varval":"0.4"},{"varval":"0.6"},{"varval":"2.25"},{"varval":"2.00"},{"varval":"4.20"}]}]

  1382. Q14

    {{name}} earns ${{wage1}} per week as a {{job1}}, and ${{wage2}} per week {{job2}}. If she saves {{frac1}} of her income each week, how many weeks will it take her to save ${{savings1}}?

    [{"vars":[{"varval":"Kate"},{"varval":"115"},{"varval":"learn to swim coach"},{"varval":"35"},{"varval":"delivering pamphlets"},{"varval":"$\\dfrac{1}{3}$"},{"varval":"2400"},{"varval":"150"},{"varval":"50"},{"varval":"50"},{"varval":"$\\dfrac{1}{3}$"}]},{"vars":[{"varval":"Fi"},{"varval":"105"},{"varval":"babysitter"},{"varval":"55"},{"varval":"delivering pizza"},{"varval":"$\\dfrac{3}{4}$"},{"varval":"4200"},{"varval":"160"},{"varval":"120"},{"varval":"$\\dfrac{3}{4}$"}]},{"vars":[{"varval":"Cianna"},{"varval":"60"},{"varval":"shop assistant"},{"varval":"45"},{"varval":"walking dogs"},{"varval":"$\\dfrac{1}{3}$"},{"varval":"1750"},{"varval":"105"},{"varval":"35"},{"varval":"$\\dfrac{1}{3}$"}]},{"vars":[{"varval":"Rhonda"},{"varval":"25"},{"varval":"guitar teacher"},{"varval":"35"},{"varval":"mowing lawns"},{"varval":"$\\dfrac{2}{3}$"},{"varval":"1800"},{"varval":"60"},{"varval":"40"},{"varval":"$\\dfrac{2}{3}$"}]},{"vars":[{"varval":"Diana"},{"varval":"90"},{"varval":"sales assistant"},{"varval":"30"},{"varval":"teaching singing"},{"varval":"$\\dfrac{1}{3}$"},{"varval":"2600"},{"varval":"120"},{"varval":"40"},{"varval":"$\\dfrac{1}{3}$"}]}]

  1383. Q13

    {{name}} works in a factory that makes {{product}}. The factory has {{mass1}} of {{ingredient}} in stock. If {{name}} uses {{mass2}} kilograms of {{ingredient}} in one batch of {{product}}, what percentage of the {{ingredient}} has he used?

    [{"vars":[{"varval":"Clive"},{"varval":"chocolate"},{"varval":"one tonne"},{"varval":"sugar"},{"varval":"40"},{"varval":"1000"}]},{"vars":[{"varval":"Burt"},{"varval":"cup cakes"},{"varval":"one tonne"},{"varval":"sugar"},{"varval":"60"},{"varval":"1000"}]},{"vars":[{"varval":"Hector"},{"varval":"tortillas"},{"varval":"one tonne"},{"varval":"flour"},{"varval":"80"},{"varval":"1000"}]},{"vars":[{"varval":"Claude"},{"varval":"croissants"},{"varval":"two tonnes"},{"varval":"sugar"},{"varval":"200"},{"varval":"2000"}]},{"vars":[{"varval":"Gerry"},{"varval":"ice cream"},{"varval":"one tonne"},{"varval":"sugar"},{"varval":"20"},{"varval":"1000"}]}]

  1384. Q12

    Four {{person}}s throw a {{object}}. The length of their throws are recorded, in millimetres, in the table below. <br> <div class="sm_mode"> <div class="outline"> > > | Name | Distance | > > | --------- | -------- | > > | {{name1}} | {{d1}} | > > | {{name2}} | {{d2}} | > > | {{name3}} | {{d3}} | > > | {{name4}} | {{d4}} | </div> </div> <br> <br> Which {{person}} threw the longest distance?

    [{"vars":[{"varval":"athlete"},{"varval":"javelin"},{"varval":"56 656"},{"varval":"65 066"},{"varval":"56 556"},{"varval":"65 666"},{"varval":"Billy"},{"varval":"Sandra"},{"varval":"Evelyn"},{"varval":"Rod"},{"varval":"65 666"}]},{"vars":[{"varval":"athlete"},{"varval":"discus"},{"varval":"76 067"},{"varval":"67 767"},{"varval":"76 667"},{"varval":"67 776"},{"varval":"Kelly"},{"varval":"Rhonda"},{"varval":"David"},{"varval":"Byron"},{"varval":"76 667"}]},{"vars":[{"varval":"cricketer"},{"varval":"cricket ball"},{"varval":"45 544"},{"varval":"54 054"},{"varval":"54 445"},{"varval":"45 545"},{"varval":"Ricky"},{"varval":"Vivat"},{"varval":"Sam"},{"varval":"Rahul"},{"varval":"54 445"}]},{"vars":[{"varval":"baseball player"},{"varval":"baseball"},{"varval":"98 098"},{"varval":"89 898"},{"varval":"98 889"},{"varval":"89 988"},{"varval":"Stuart"},{"varval":"Babe"},{"varval":"Ned"},{"varval":"Honus"},{"varval":"98 889"}]},{"vars":[{"varval":"baseball player"},{"varval":"baseball"},{"varval":"78 788"},{"varval":"87 088"},{"varval":"78 878"},{"varval":"87 778"},{"varval":"Willie"},{"varval":"Barry"},{"varval":"Ty"},{"varval":"Walter"},{"varval":"87 778"}]}]

  1385. Q10 QUICK FIX: - add colour to the stapler and credit card

    A {{item}} is measured with a ruler. {{image}} How long is the {{item}}?

    [{"vars":[{"varval":"paperclip"},{"varval":"6.5"},{"varval":"3.0"},{"varval":"sm_img https://teacher.smartermaths.com.au/wp-content/uploads/2020/06/Q10VAR1.png 400 indent vpad"}]},{"vars":[{"varval":"stapler"},{"varval":"7.5"},{"varval":"2.0"},{"varval":"sm_img https://teacher.smartermaths.com.au/wp-content/uploads/2020/06/Q10-var2.png 400 indent vpad"}]},{"vars":[{"varval":"stapler"},{"varval":"9.0"},{"varval":"3.5"},{"varval":"sm_img https://teacher.smartermaths.com.au/wp-content/uploads/2020/06/Q10-var3.png 400 indent vpad"}]},{"vars":[{"varval":"credit card"},{"varval":"8.5"},{"varval":"1.0"},{"varval":"sm_img https://teacher.smartermaths.com.au/wp-content/uploads/2020/06/Q10-var4.png 400 indent vpad"}]},{"vars":[{"varval":"paperclip"},{"varval":"7.0"},{"varval":"2.5"},{"varval":"sm_img https://teacher.smartermaths.com.au/wp-content/uploads/2020/06/Q10VAR5.png 400 indent vpad"}]}]

  1386. {{name}} has {{mass1}} tonnes of soil to add to his garden. He buys an extra {{bags}} 20 kilograms bags of soil. What is the total mass of soil, in kilograms, {{name}} has altogether?

    [{"vars":[{"varval":"Guy"},{"varval":"0.5"},{"varval":"three"},{"varval":"500"},{"varval":"3"}]},{"vars":[{"varval":"Dom"},{"varval":"0.8"},{"varval":"two"},{"varval":"800"},{"varval":"2"}]},{"vars":[{"varval":"Stan"},{"varval":"0.7"},{"varval":"two"},{"varval":"700"},{"varval":"2"}]},{"vars":[{"varval":"Ali"},{"varval":"0.4"},{"varval":"three"},{"varval":"400"},{"varval":"3"}]},{"vars":[{"varval":"Mel"},{"varval":"0.9"},{"varval":"two"},{"varval":"900"},{"varval":"2"}]}]

  1387. A summer camp has {{children}} children enrolled. {{fraction1}} chose {{activity}} as an activity. How many children chose to do {{activity}}?

    [{"vars":[{"varval":"Two-thirds"},{"varval":"kayaking"},{"varval":"$\\dfrac{2}{3}$"},{"varval":"210"}]},{"vars":[{"varval":"Three-quarters"},{"varval":"swimming"},{"varval":"$\\dfrac{3}{4}$"},{"varval":"240"}]},{"vars":[{"varval":"One-third"},{"varval":"basket weaving"},{"varval":"$\\dfrac{1}{3}$"},{"varval":"270"}]},{"vars":[{"varval":"Two-fifths"},{"varval":"volleyball"},{"varval":"$\\dfrac{2}{5}$"},{"varval":"350"}]},{"vars":[{"varval":"Three-quarters"},{"varval":"archery"},{"varval":"$\\dfrac{3}{4}$"},{"varval":"360"}]}]

  1388. {{image}} <br> Identify the point which would be located in the shaded quadrant of the number plane above.

    [{"vars":[{"varval":"sm_img https://teacher.smartermaths.com.au/wp-content/uploads/2020/06/variant1q.svg 400 indent3 vpad"},{"varval":"sm_img https://teacher.smartermaths.com.au/wp-content/uploads/2020/06/variant1sol.svg 400 indent3 vpad"}]},{"vars":[{"varval":"sm_img https://teacher.smartermaths.com.au/wp-content/uploads/2020/06/variant2q.svg 400 indent3 vpad"},{"varval":"sm_img https://teacher.smartermaths.com.au/wp-content/uploads/2020/06/variant2sol.svg 400 indent3 vpad"}]},{"vars":[{"varval":"sm_img https://teacher.smartermaths.com.au/wp-content/uploads/2020/06/variant3q.svg 400 indent3 vpad"},{"varval":"sm_img https://teacher.smartermaths.com.au/wp-content/uploads/2020/06/variant3sol.svg 400 indent3 vpad"}]},{"vars":[{"varval":"sm_img https://teacher.smartermaths.com.au/wp-content/uploads/2020/06/variant1q.svg 400 indent3 vpad"},{"varval":"sm_img https://teacher.smartermaths.com.au/wp-content/uploads/2020/06/variant1sol.svg 400 indent3 vpad"}]},{"vars":[{"varval":"sm_img https://teacher.smartermaths.com.au/wp-content/uploads/2020/06/variant2q.svg 400 indent3 vpad"},{"varval":"sm_img https://teacher.smartermaths.com.au/wp-content/uploads/2020/06/variant2sol.svg 400 indent3 vpad"}]}]

  1389. Q11 QUICK FIX Var1 images ... centre numbers, delete 15 in image 2, smaller font Var2 images ... smaller font Var3 images ... centre numbers, smaller font Var4 images ... centre numbers, smaller font Var5 images ... centre numbers, solution image 5-4 should be 6-8, smaller font

    What is the area of the shape pictured below? <br/> {{image1}}

    [{"vars":[{"varval":"4 $\\times$ 5"},{"varval":"6 $\\times$ 20"},{"varval":"20"},{"varval":"120"},{"varval":"sm_img https://teacher.smartermaths.com.au/wp-content/uploads/2020/06/rQ1.svg 360 indent3 vpad"},{"varval":"sm_img https://teacher.smartermaths.com.au/wp-content/uploads/2020/06/rA1.svg 380 indent3 vpad"}]},{"vars":[{"varval":"5 $\\times$ 5"},{"varval":"4 $\\times$ 14"},{"varval":"25"},{"varval":"56"},{"varval":"sm_img https://teacher.smartermaths.com.au/wp-content/uploads/2020/06/rQ2.svg 300 indent3 vpad"},{"varval":"sm_img https://teacher.smartermaths.com.au/wp-content/uploads/2020/06/rA2.svg 300 indent3 vpad"}]},{"vars":[{"varval":"3 $\\times$ 6"},{"varval":"6 $\\times$ 20"},{"varval":"18"},{"varval":"120"},{"varval":"sm_img https://teacher.smartermaths.com.au/wp-content/uploads/2020/06/rQ3.svg 420 indent3 vpad"},{"varval":"sm_img https://teacher.smartermaths.com.au/wp-content/uploads/2020/06/rA3.svg 420 indent3 vpad"}]},{"vars":[{"varval":"10 $\\times$ 14"},{"varval":"6 $\\times$ 30"},{"varval":"140"},{"varval":"180"},{"varval":"sm_img https://teacher.smartermaths.com.au/wp-content/uploads/2020/06/rQ4.svg 440 indent3 vpad"},{"varval":"sm_img https://teacher.smartermaths.com.au/wp-content/uploads/2020/06/rA4.svg 440 indent3 vpad"}]},{"vars":[{"varval":"5 $\\times$ 6"},{"varval":"8 $\\times$ 20"},{"varval":"30"},{"varval":"160"},{"varval":"sm_img https://teacher.smartermaths.com.au/wp-content/uploads/2020/06/rQ5.svg 300 indent3 vpad"},{"varval":"sm_img https://teacher.smartermaths.com.au/wp-content/uploads/2020/06/r_A5.svg 270 indent3 vpad"}]}]

  1390. Geometry, NAP F3 NC10

    <div class="sm_mode"> {{{question}}} </div>

    [{"vars":[{"varval":"Simon looked at a stack of cubes from the direction of the arrow, shown in the diagram below.\n\n<br>\n\nsm_img //teacher.smartermaths.com.au/wp-content/uploads/2017/01/NAP-F3-NC10.png 215 indent3 vpad\n\n<br>\nWhich is Simon's view of the cubes?"},{"varval":"{{{correctAnswer}}}"}]},{"vars":[{"varval":"May looked at a stack of cubes from the direction of the arrow, shown in the diagram below.\n\n<br>\n\nsm_img https://teacher.smartermaths.com.au/wp-content/uploads/2022/10/Geom_50008_v1q.svg 200 indent3 vpad\n\n<br>\nWhich is May's view of the cubes?"},{"varval":"{{{correctAnswer}}}"}]},{"vars":[{"varval":"Celeste looked at a stack of cubes from the direction of the arrow, shown in the diagram below.\n\n<br>\n\nsm_img https://teacher.smartermaths.com.au/wp-content/uploads/2022/10/Geom_50008_v2q1.svg 215 indent3 vpad\n\n<br>\nWhich is Celeste's view of the cubes?"},{"varval":"{{{correctAnswer}}}"}]},{"vars":[{"varval":"Sigourney looked at a stack of cubes from the direction of the arrow, shown in the diagram below.\n\n<br>\n\nsm_img https://teacher.smartermaths.com.au/wp-content/uploads/2022/10/Geom_50008_v3q.svg 225 indent3 vpad\n\n<br>\nWhich is Sigourney's view of the cubes?"},{"varval":"{{{correctAnswer}}}"}]},{"vars":[{"varval":"Miriam looked at a stack of cubes from the direction of the arrow, shown in the diagram below. \n\n<br>\n\nsm_img https://teacher.smartermaths.com.au/wp-content/uploads/2022/10/Geom_50008_v4q.svg 225 indent3 vpad\n\n<br>\nWhich is Miriam's view of the cubes?"},{"varval":"{{{correctAnswer}}}"}]},{"vars":[{"varval":"Max looked at a stack of cubes from the direction of the arrow, shown in the diagram below. \n\n<br>\n\nsm_img https://teacher.smartermaths.com.au/wp-content/uploads/2022/10/Geom_50008_v5q.svg 225 indent3 vpad\n\n<br>\nWhich is Max's view of the cubes?"},{"varval":"{{{correctAnswer}}}"}]}]

  1391. Geometry, NAP-E3-CA08

    A sculpture is pictured from 3 different angles below: <br> sm_img //teacher.smartermaths.com.au/wp-content/uploads/2017/01/naplan-2012-4mc.png 470 indent vpad <br> The sculpture is in the shape of

    [{"vars":null}]

  1392. Geometry, NAP G3 NC05

    Andre placed one block on top of another block. This is a drawing of the front view and the side view. <br> sm_img //teacher.smartermaths.com.au/wp-content/uploads/2017/01/NAP-G3-NC05.png 230 indent3 vpad <br> Which of these is a correct top view?

    [{"vars":null}]

  1393. <div class="sm_mode"> sm_nogap {{person}}'s wallet contains the following money: > * {{amount1}} five dollar notes > * {{amount2}} ten dollar notes > * {{amount3}} fifty dollar notes <br>How much cash does {{person}} have altogether? </div>

    [{"vars":[{"varval":"Gordon"},{"varval":"4"},{"varval":"2"},{"varval":"3"},{"varval":"20"},{"varval":"20"},{"varval":"150"}]},{"vars":[{"varval":"Milly"},{"varval":"6"},{"varval":"7"},{"varval":"2"},{"varval":"30"},{"varval":"70"},{"varval":"100"}]},{"vars":[{"varval":"Bill"},{"varval":"7"},{"varval":"3"},{"varval":"2"},{"varval":"35"},{"varval":"30"},{"varval":"100"}]},{"vars":[{"varval":"Elon"},{"varval":"5"},{"varval":"2"},{"varval":"3"},{"varval":"25"},{"varval":"20"},{"varval":"150"}]},{"vars":[{"varval":"Cindy"},{"varval":"8"},{"varval":"3"},{"varval":"2"},{"varval":"40"},{"varval":"30"},{"varval":"100"}]}]

  1394. {{name}} pays ${{cost1}} for {{gender1}} prepaid broadband. The next time {{gender2}} renews this, {{gender2}} decides to purchase extra broadband and pays {{percentage}}% more. What did {{name}} pay for {{gender1}} broadband renewal?

    [{"vars":[{"varval":"Penelope"},{"varval":"120"},{"varval":"her"},{"varval":"she"},{"varval":"15"},{"varval":"12"},{"varval":"$1 \\dfrac{1}{2}$"},{"varval":"18"}]},{"vars":[{"varval":"Bernice"},{"varval":"80"},{"varval":"her"},{"varval":"she"},{"varval":"15"},{"varval":"8"},{"varval":"$1 \\dfrac{1}{2}$"},{"varval":"12"}]},{"vars":[{"varval":"Sanjay"},{"varval":"160"},{"varval":"his"},{"varval":"he"},{"varval":"15"},{"varval":"16"},{"varval":"$1 \\dfrac{1}{2}$"},{"varval":"24"}]},{"vars":[{"varval":"Cary"},{"varval":"60"},{"varval":"his"},{"varval":"he"},{"varval":"15"},{"varval":"6"},{"varval":"$1 \\dfrac{1}{2}$"},{"varval":"9"}]},{"vars":[{"varval":"Olive"},{"varval":"140"},{"varval":"her"},{"varval":"she"},{"varval":"15"},{"varval":"14"},{"varval":"$1 \\dfrac{1}{2}$"},{"varval":"21"}]}]

  1395. Geometry, NAP-K3-CA07

    Tran uses blocks to make rectangular prisms. <br> sm_img https://teacher.smartermaths.com.au/wp-content/uploads/2018/07/NAP-K3-CA07.svg 225 indent3 vpad <br> How many blocks does Tran use altogether?

    [{"vars":null}]

  1396. Geometry, NAP-B3-NC04

    <div class="sm_mode"> {{{question}}} </div>

    [{"vars":[{"varval":"Johnny cuts a paper shape in half.\n\n<br>\n\nsm_img //teacher.smartermaths.com.au/wp-content/uploads/2017/01/NAP-B3-NC04.png 265 indent vpad\n\n<br>\n The shape of each half is "},{"varval":"The shape of each half has 5 sides and is {{{correctAnswer}}}."}]},{"vars":[{"varval":"Jordie cuts a paper shape in half.\n\n<br>\n\nsm_img https://teacher.smartermaths.com.au/wp-content/uploads/2022/10/Geom_50004_v1.svg 265 indent vpad\n\n<br>\n The shape of each half is "},{"varval":"The shape of each half has 6 sides and is a {{{correctAnswer}}}."}]},{"vars":[{"varval":"Jana cuts a paper shape in half.\n\n<br>\n\nsm_img https://teacher.smartermaths.com.au/wp-content/uploads/2022/10/Geom_50004_v2.svg 265 indent vpad\n\n<br>\n The shape of each half is "},{"varval":"The shape of each half has 8 sides and is {{{correctAnswer}}}."}]},{"vars":[{"varval":"Julio cuts a paper shape in half.\n\n<br>\n\nsm_img https://teacher.smartermaths.com.au/wp-content/uploads/2022/10/Geom_50004_v3.svg 265 indent2 vpad\n\n<br>\n The shape of each half is "},{"varval":"The shape of each half has 4 sides and is {{{correctAnswer}}}."}]},{"vars":[{"varval":"Jonas cuts a paper shape in half.\n\n<br>\n\nsm_img https://teacher.smartermaths.com.au/wp-content/uploads/2022/10/Geom_50004_v4.svg 165 indent2 vpad\n\n<br>\n The shape of each half is "},{"varval":"The shape of each half has 4 sides and is {{{correctAnswer}}}."}]},{"vars":[{"varval":"June cuts a paper shape in half.\n\n<br>\n\nsm_img https://teacher.smartermaths.com.au/wp-content/uploads/2022/10/Geom_50004_v5.svg 285 indent2 vpad\n\n<br>\n The shape of each half is "},{"varval":"The shape of each half has 10 sides and is {{{correctAnswer}}}."}]}]

  1397. Geometry, NAP-G3-CA04

    Herb is making an animation of a man running. The picture shows seven frames of his animation. sm_img //teacher.smartermaths.com.au/wp-content/uploads/2017/01/naplan-Y7-2014-4mci.png 598 indent vpad He has accidentally swapped two frames. Which two frames were swapped?

    [{"vars":null}]

  1398. Number, NAP-I4-NC32

    Mark and Jim have property sizes as shown in the table below: <br> sm_img https://teacher.smartermaths.com.au/wp-content/uploads/2016/12/NAP-169-NC032.png 350 indent vpad <br> What is the ratio of the land area of Mark's property to Jim's property.

    [{"vars":null}]

  1399. Measurement, NAP-I4-NC29

    Leigh has a picnic blanket with an area of 5 square metres. What is the area of her picnic blanket in square centimetres?

    [{"vars":null}]

  1400. Measurement, NAP-L4-CA23

    <div class="sm_mode"> {{{question}}} </div>

    [{"vars":[{"varval":"A rectangular garden bed is 7 metres long by 4 metres wide.\n\nThe garden bed is filled with soil to a height of 50 centimetres.\n\n<br>\n\nsm_img https://teacher.smartermaths.com.au/wp-content/uploads/2020/01/nap-L4-23-ver1.svg 400 indent3 vpad\n\n<br>\n\nHow many cubic metres of soil are needed to fill the garden bed?"},{"varval":"50 cm = 0.5 metre\n\n<div class=\"aligned\">\n\n|||\n|-|-|\n|$\\therefore$ Soil needed|= 7 $\\times\\ 4 \\times 0.5$|\n||= {{{correctAnswer}}}|\n\n</div>"}]},{"vars":[{"varval":"A square garden bed has a side length of 6 metres.\n\nThe garden bed is filled with soil to a height of 50 centimetres.\n\nHow many cubic metres of soil are needed to fill the garden bed?"},{"varval":"50 cm = 0.5 metre\n\n<div class=\"aligned\">\n\n|||\n|-|-|\n|$\\therefore$ Soil needed|= 6 $\\times\\ 6 \\times 0.5$|\n||= {{{correctAnswer}}}|\n\n</div>"}]},{"vars":[{"varval":"A rectangular cement deck is 3 metres wide by 4 metres long.\n\nThe cement deck has a depth of 25 centimetres.\n\nHow many cubic metres of cement were needed to make the cement deck?"},{"varval":"25 cm = 0.25 metre\n\n<div class=\"aligned\">\n\n|||\n|-|-|\n|$\\therefore$ Cement needed|= 3 $\\times\\ 4 \\times 0.25$|\n||= {{{correctAnswer}}}|\n\n</div>"}]},{"vars":[{"varval":"A rectangular path has been dug into the ground that is 10 metres long and 2 metres wide.\n\nThe path is 25 centimetres deep.\n\nHow many cubic metres of cement need to be poured to create the path?"},{"varval":"25 cm = 0.25 metre\n\n<div class=\"aligned\">\n\n|||\n|-|-|\n|$\\therefore$ Cement needed|= 10 $\\times\\ 2 \\times 0.25$|\n||= {{{correctAnswer}}}|\n\n</div>"}]},{"vars":[{"varval":"The rectangular base of a large sculpture is 10 metres long and 3 metres wide.\n\nThe base is made of steel and has a height of 30 centimetres.\n\nHow many cubic metres of steel are needed to make the base of the sculpture?"},{"varval":"30 cm = 0.3 metre\n\n<div class=\"aligned\">\n\n|||\n|-|-|\n|$\\therefore$ Cement needed|= 10 $\\times\\ 3 \\times 0.3$|\n||= {{{correctAnswer}}}|\n\n</div>"}]},{"vars":[{"varval":"The rectangular base of a crane is 4 metres long and 5 metres wide.\n\nThe base is made of iron and has a height of 60 centimetres.\n\nHow many cubic metres of iron are needed to make the base of the crane?"},{"varval":"60 cm = 0.6 metre\n\n<div class=\"aligned\">\n\n|||\n|-|-|\n|$\\therefore$ Cement needed|= 5 $\\times\\ 4 \\times 0.6$|\n||= {{{correctAnswer}}}|\n\n</div>"}]}]

  1401. Number, NAP-D4-NC16

    A flood level indicator measures the height of a river. <br> sm_img https://teacher.smartermaths.com.au/wp-content/uploads/2016/12/naplan-2014-16mc.png 260 indent3 vpad What is the height of the river?

    [{"vars":null}]

  1402. Number, NAP-D4-NC16

    <div class="sm_mode"> {{{question}}} </div>

    [{"vars":[{"varval":"A glass of water containing 255 mL of water is poured into a jug that already contains 1.65 L of water.\n\nHow much water is now in the jug?"},{"varval":"1 litre = 1000 millilitres\n\n<div class=\"aligned\">\n\n| | |\n| --------------------- | -------------- |\n| 1.65 L + 255 mL | = 1650 mL + 255 mL |\n| | = 1905 mL |\n| | \\= {{{correctAnswer}}} |\n\n</div>"}]},{"vars":[{"varval":"A glass of cordial containing 375 mL of cordial is poured into a jug that already contains 2.25 L of water.\n\nHow much mixture is now in the jug?"},{"varval":"1 litre = 1000 millilitres\n\n<div class=\"aligned\">\n\n| | |\n| --------------------- | -------------- |\n| 2.25 L + 375 mL | = 2250 mL + 375 mL |\n| | = 2625 mL |\n| | \\= {{{correctAnswer}}} |\n\n</div>"}]},{"vars":[{"varval":"A mower's fuel consists of petrol mixed with 2-stroke oil.\n\nJustine pours 500 mL of oil into a container that already contains 4.25 litres of petrol.\n\nHow much mower fuel is now in the container?"},{"varval":"1 litre = 1000 millilitres\n\n<div class=\"aligned\">\n\n| | |\n| --------------------- | -------------- |\n| 4.25 L + 500 mL | = 4250 mL + 500 mL |\n| | = 4750 mL |\n| | \\= {{{correctAnswer}}} |\n\n</div>"}]},{"vars":[{"varval":"A breakfast energy drink consists of watermelon juice mixed with orange juice.\n\nGerry makes the energy drink by pouring 600 mL of watermelon juice into a container that already contains 1.25 litres of orange juice.\n\nHow much energy drink has Gerry made?"},{"varval":"1 litre = 1000 millilitres\n\n<div class=\"aligned\">\n\n| | |\n| --------------------- | -------------- |\n| 1.25 L + 600 mL | = 1250 mL + 600 mL |\n| | = 1850 mL |\n| | \\= {{{correctAnswer}}} |\n\n</div>"}]},{"vars":[{"varval":"A rocket fuel is made by mixing liquid hydrogen with liquid nitrate.\n\nElton pours 400 mL of liquid nitrogen into a fuel cylinder that already contains 2.45 litres of liquid hydrogen.\n\nHow much rocket fuel has Elton made in the fuel cylinder?"},{"varval":"1 litre = 1000 millilitres\n\n<div class=\"aligned\">\n\n| | |\n| --------------------- | -------------- |\n| 2.45 L + 400 mL | = 2450 mL + 400 mL |\n| | = 2850 mL |\n| | \\= {{{correctAnswer}}} |\n\n</div>"}]},{"vars":[{"varval":"Sonnet is making a recovery shake for baby kangaroos orphaned in the bushfires.\n\nShe mixes 300 mL of maple syrup with 3.75 litres of marsupial milk.\n\nHow much recovery shake has Sonnet made?"},{"varval":"1 litre = 1000 millilitres\n\n<div class=\"aligned\">\n\n| | |\n| --------------------- | -------------- |\n| 3.75 L + 300 mL | = 3750 mL + 300 mL |\n| | = 4050 mL |\n| | \\= {{{correctAnswer}}} |\n\n</div>"}]}]

  1403. Measurement, NAP-H4-NC10

    The capacity of a jug of milk, pictured below, is closest to: sm_img https://teacher.smartermaths.com.au/wp-content/uploads/2024/06/nap-159-nc103_1-min.svg 400 indent vpad

    [{"vars":null}]

  1404. Measurement, NAP-C4-CA12

    <div class="sm_mode"> {{{question}}} </div>

    [{"vars":[{"varval":"\nThe cruising speed of a kangaroo is 720 metres per minute.\n\nWhat is this speed in metres per second?"},{"varval":"<div class=\"aligned\">\n\n|||\n|-|-|\n|Speed|= $\\dfrac{720}{60}$|\n|||\n||= {{{correctAnswer}}}|\n\n</div>"}]},{"vars":[{"varval":"The top speed of a wildebeest is 1440 metres per minute.\n\nWhat is this speed in metres per second?"},{"varval":"<div class=\"aligned\">\n\n|||\n|-|-|\n|Speed|= $\\dfrac{1440}{60}$|\n|||\n||= $\\dfrac{144}{6}$|\n||= {{{correctAnswer}}}|\n\n</div>"}]},{"vars":[{"varval":"The top speed of a leatherback sea turtle is 1800 metres per minute.\n\nWhat is this speed in metres per second?"},{"varval":"<div class=\"aligned\">\n\n|||\n|-|-|\n|Speed|= $\\dfrac{1800}{60}$|\n|||\n||= $\\dfrac{180}{6}$|\n||= {{{correctAnswer}}}|\n\n</div>"}]},{"vars":[{"varval":"The top speed of a peregrine falcon is 360 km/hr.\n\nWhat is this top speed in metres per second?"},{"varval":"1 hour = 60 minutes = 60 $\\times$ 60 seconds\n\n360 km = 360 000 metres\n\n<div class=\"aligned\">\n\n|||\n|-|-|\n|Speed|= $\\dfrac{360\\ 000}{60 \\times 60}$|\n|||\n||= $\\dfrac{3600}{36}$|\n||= {{{correctAnswer}}}|\n\n</div>"}]},{"vars":[{"varval":"A polar bear has a top speed of 420 metres per minute.\n\nWhat is this top speed in metres per second?"},{"varval":"60 seconds = 1 minute\n\n<div class=\"aligned\">\n\n|||\n|-|-|\n|Speed|= $\\dfrac{420}{60}$|\n|||\n||= $\\dfrac{42}{6}$|\n||= {{{correctAnswer}}}|\n\n</div>"}]},{"vars":[{"varval":"The top speed of an ostrich is 72 kilometres per hour.\n\nWhat is this speed in metres per second?"},{"varval":"1 hour = 60 minutes = 60 $\\times$ 60 seconds\n\n72 kilometres = 72 000 metres\n\n<div class=\"aligned\">\n\n|||\n|-|-|\n|Speed|= $\\dfrac{72\\ 000}{60 \\times 60}$|\n|||\n||= $\\dfrac{720}{6 \\times 6}$|\n||= $\\dfrac{720}{36}$|\n||= {{{correctAnswer}}}|\n\n</div>"}]}]

  1405. Measurement, NAP-F3-NC19

    Andrew caught a fish and measured it, as shown in the diagram below. <br> sm_img https://teacher.smartermaths.com.au/wp-content/uploads/2016/12/NAP-F4-NC10.png 330 indent3 vpad <br> How long was Andrew's fish, in centimetres?

    [{"vars":null}]

  1406. Measurement, NAP-J4-CA13

    <div class="sm_mode"> {{{question}}} </div>

    [{"vars":[{"varval":"Karen needs 3475 centimetres of ribbon for a science project.\n\nChoose an equivalent length of ribbon for Karen to use."},{"varval":"Convert cm to metres $\\Rightarrow$ Divide by 100\n\n$\\begin{aligned} 3475 \\ \\text{cm} &= \\frac{3475}{100} \\cr\n&=34.75 \\cr &=34\\ \\text{metres and } \\ 75\\ \\text{centimetres}\n\\end{aligned}$"}]},{"vars":[{"varval":"Morris bought a compact car that is 1515 millimetres long.\n\n\rAn equivalent car length is:\n\n"},{"varval":"1000 millimetres = 1 metre.\n\n$\\therefore$ 1515 mm = {{{correctAnswer}}}"}]}]

  1407. {{name}} is filling {{gender}} pool with water from {{gender}} rain tank. Water flows from the rain tank into the pool at a rate of {{flow}} litres per minute. After {{time}} minutes, the water tank has {{vol1}}L in it. How many litres of water were in the tank before it was used to fill the pool?

    [{"vars":[{"varval":"Stuart"},{"varval":"his"},{"varval":"100"},{"varval":"10"},{"varval":"3500"},{"varval":"1000"}]},{"vars":[{"varval":"Bryan"},{"varval":"his"},{"varval":"50"},{"varval":"10"},{"varval":"3000"},{"varval":"500"}]},{"vars":[{"varval":"Phoebe"},{"varval":"her"},{"varval":"200"},{"varval":"10"},{"varval":"7000"},{"varval":"2000"}]},{"vars":[{"varval":"Sola"},{"varval":"her"},{"varval":"100"},{"varval":"20"},{"varval":"4000"},{"varval":"2000"}]},{"vars":[{"varval":"Ed"},{"varval":"his"},{"varval":"200"},{"varval":"5"},{"varval":"5000"},{"varval":"1000"}]}]

  1408. Geometry, NAP A3 NC04

    A rectangular prism and a triangular prism are joined together to create the shape below. <br> sm_img https://teacher.smartermaths.com.au/wp-content/uploads/2017/12/nap-A3-nc04.svg 213 indent3 vpad <br> How many **faces** does the new object have?

    [{"vars":null}]

  1409. {{name}} buys a {{item}} that normally costs \${{cost}}. If it is reduced by {{percentage}} when on sale, how much does {{name}} save?

    [{"vars":[{"varval":"Belinda"},{"varval":"dress"},{"varval":"30"},{"varval":"20%"},{"varval":"$\\dfrac{1}{5}$"}]},{"vars":[{"varval":"Beth"},{"varval":"frying pan"},{"varval":"24"},{"varval":"25%"},{"varval":"$\\dfrac{1}{4}$"}]},{"vars":[{"varval":"Megan"},{"varval":"jigsaw puzzle"},{"varval":"35"},{"varval":"20%"},{"varval":"$\\dfrac{1}{5}$"}]},{"vars":[{"varval":"Richard"},{"varval":"bike pump"},{"varval":"40"},{"varval":"25%"},{"varval":"$\\dfrac{1}{4}$"}]},{"vars":[{"varval":"Vinny"},{"varval":"pingpong racket"},{"varval":"28"},{"varval":"25%"},{"varval":"$\\dfrac{1}{4}$"}]}]

  1410. Q8

    {{name}} has a taxable income of ${{income}}. {{gender}} pays ${{tax1}} tax on the first ${{amount}} of her income. A tax rate of 19% is applicable to any income earned over ${{amount}}. What is the total amount of income tax {{name}} has to pay?

    [{"vars":[{"varval":"Veronica"},{"varval":"27 000"},{"varval":"She"},{"varval":"1102"},{"varval":"24 000"},{"varval":"570"}]},{"vars":[{"varval":"Kelly"},{"varval":"30 000"},{"varval":"She"},{"varval":"912"},{"varval":"23 000"},{"varval":"1330"}]},{"vars":[{"varval":"Conrad"},{"varval":"25 000"},{"varval":"He"},{"varval":"532"},{"varval":"21 000"},{"varval":"760"}]},{"vars":[{"varval":"Tyrone"},{"varval":"32 000"},{"varval":"He"},{"varval":"1292"},{"varval":"25 000"},{"varval":"1330"}]},{"vars":[{"varval":"Darlene"},{"varval":"25 000"},{"varval":"She"},{"varval":"152"},{"varval":"19 000"},{"varval":"1140"}]}]

  1411. Identify the quadrilateral pictured below. <br> {{image}}

    [{"vars":[{"varval":"sm_img https://teacher.smartermaths.com.au/wp-content/uploads/2020/06/q7_4.svg 250 indent3 vpad"},{"varval":"One pair of opposite sides are parallel.\n\n$\\Rightarrow$ Trapezium"}]},{"vars":[{"varval":"sm_img https://teacher.smartermaths.com.au/wp-content/uploads/2020/06/q7_2.svg 150 indent3 vpad"},{"varval":"2 sets of adjacent sides are equal\n\n$\\Rightarrow$ Kite"}]},{"vars":[{"varval":"sm_img https://teacher.smartermaths.com.au/wp-content/uploads/2020/06/q7_3.svg 200 indent3 vpad"},{"varval":"All sides are equal and opposite sides are parallel.\n\n$\\Rightarrow$ Rhombus"}]},{"vars":[{"varval":"sm_img https://teacher.smartermaths.com.au/wp-content/uploads/2020/06/q7_1.svg 275 indent3 vpad"},{"varval":"Opposite sides are equal and parallel.\n\nAdjacent sides are perpendicular.\n\n$\\Rightarrow$ Rectangle"}]},{"vars":[{"varval":"sm_img https://teacher.smartermaths.com.au/wp-content/uploads/2020/06/q7_5.svg 275 indent3 vpad"},{"varval":"2 sets of opposite sides are equal and parallel.\n\n$\\Rightarrow$ Parallelogram"}]}]

  1412. MIN STD 5-Variants Q6

    {{name}} is a courier and is paid \${{pay1}} for every {{item1}} items he delivers. He receives an extra \${{pay2}} per day if the temperature exceeds 36$\degree$C. On a day with a maximum temperature of {{temp}}$\degree$C, {{name}} delivers {{item2}} items. How much should he be paid?

    [{"vars":[{"varval":"Murray"},{"varval":"48"},{"varval":"10"},{"varval":"15"},{"varval":"39"},{"varval":"15"},{"varval":"$1 \\dfrac{1}{2}$"},{"varval":"72"}]},{"vars":[{"varval":"Marty"},{"varval":"36"},{"varval":"8"},{"varval":"13"},{"varval":"41"},{"varval":"12"},{"varval":"$1 \\dfrac{1}{2}$"},{"varval":"54"}]},{"vars":[{"varval":"Jose"},{"varval":"32"},{"varval":"12"},{"varval":"11"},{"varval":"43"},{"varval":"18"},{"varval":"$1 \\dfrac{1}{2}$"},{"varval":"48"}]},{"vars":[{"varval":"Tex"},{"varval":"28"},{"varval":"12"},{"varval":"9"},{"varval":"37"},{"varval":"30"},{"varval":"$2 \\dfrac{1}{2}$"},{"varval":"70"}]},{"vars":[{"varval":"Kevin"},{"varval":"42"},{"varval":"8"},{"varval":"17"},{"varval":"44"},{"varval":"12"},{"varval":"$1 \\dfrac{1}{2}$"},{"varval":"63"}]}]

  1413. Measurement, NAP-B4-CA10 SA

    {{{question}}}

    [{"vars":[{"varval":"Which of these is the longest distance?"},{"varval":"Convert each option to metres:\n\nOption 1: &nbsp;0.2405 km = 0.2405 × 1000 = 240.5 m\n\nOption 2: &nbsp;245 m\n\nOption 3: &nbsp;2450 cm = $\\dfrac{2450}{100}$ = 24.5 m\n\nOption 4: &nbsp;2 050 mm = $\\dfrac{24\\ 050}{1000}$ = 24.05 m\n\n$\\therefore$ 245 m is the longest distance."}]},{"vars":[{"varval":"Which of these is the longest distance?"},{"varval":"Convert each option to metres:\n\nOption 1: &nbsp;35 070 mm = $\\dfrac{35\\ 070}{1000}$ = 35.07 m\n\nOption 2: &nbsp;3570 cm = $\\dfrac{3570}{100}$ = 35.7 m\n\nOption 3: &nbsp;357 m\n\nOption 4: &nbsp;0.3705 km = 0.3705 $\\times$ 1000 = 370.5 m\n\n$\\therefore$ {{{correctAnswer}}} is the longest distance."}]},{"vars":[{"varval":"Which of these is the longest distance?"},{"varval":"Convert each option to metres:\n\nOption 1: &nbsp;0.4502 km = 0.4502 $\\times$ 1000 = 450.2 m\n\nOption 2: &nbsp;451 m\n\nOption 3: &nbsp;45 200 cm = $\\dfrac{45\\ 200}{100}$ = 452 m\n\nOption 4: &nbsp;45 800 mm = $\\dfrac{45\\ 800}{1000}$ = 45.8 m\n\n$\\therefore$ {{{correctAnswer}}} is the longest distance."}]}]

  1414. {{name}} is travelling at {{speed}} km/h in his car. If {{gender}} maintains this speed, how many kilometres will {{gender}} travel in {{time}} minutes?

    [{"vars":[{"varval":"Michael"},{"varval":"100"},{"varval":"he"},{"varval":"15"},{"varval":"$\\dfrac{1}{4}$"}]},{"vars":[{"varval":"Billy Bob"},{"varval":"120"},{"varval":"he"},{"varval":"40"},{"varval":"$\\dfrac{2}{3}$"}]},{"vars":[{"varval":"Vangio"},{"varval":"80"},{"varval":"he"},{"varval":"15"},{"varval":"$\\dfrac{1}{4}$"}]},{"vars":[{"varval":"Choon"},{"varval":"90"},{"varval":"he"},{"varval":"20"},{"varval":"$\\dfrac{1}{3}$"}]},{"vars":[{"varval":"Ranjit"},{"varval":"100"},{"varval":"he"},{"varval":"45"},{"varval":"$\\dfrac{3}{4}$"}]}]

  1415. Complete this number sentence <div class="indent"> {{number1}} + {{number2}} = </div>

    [{"vars":[{"varval":"6.19"},{"varval":"4.2"}]},{"vars":[{"varval":"3.68"},{"varval":"5.2"}]},{"vars":[{"varval":"2.37"},{"varval":"4.3"}]},{"vars":[{"varval":"5.39"},{"varval":"4.1"}]},{"vars":[{"varval":"8.78"},{"varval":"3.2"}]}]

  1416. <div class="sm_mode"> {{name}} is taking up mountain bike riding with her partner. {{gender}} purchases the following three items for both of them. <br> {{product1}} {{product2}} {{product3}} </div> <br> What is the total purchase price altogether?

    [{"vars":[{"varval":"Whoopy"},{"varval":"She"},{"varval":"sm_img https://teacher.smartermaths.com.au/wp-content/uploads/2020/06/mountain-bike.png 200 indent vpad"},{"varval":"sm_img https://teacher.smartermaths.com.au/wp-content/uploads/2020/06/bikecomputer.png 202 indent vpad"},{"varval":"sm_img https://teacher.smartermaths.com.au/wp-content/uploads/2020/06/jersey.png 200 indent vpad"},{"varval":"795.00"},{"varval":"85.00"},{"varval":"75.00"},{"varval":"955"}]},{"vars":[{"varval":"Carolyn"},{"varval":"She"},{"varval":"sm_img https://teacher.smartermaths.com.au/wp-content/uploads/2020/06/mountain-bike.png 198 indent vpad"},{"varval":"sm_img https://teacher.smartermaths.com.au/wp-content/uploads/2020/06/bikehelmet.png 200 indent vpad"},{"varval":"sm_img https://teacher.smartermaths.com.au/wp-content/uploads/2020/06/bike-shoes.png 200 indent vpad"},{"varval":"795.00"},{"varval":"165.00"},{"varval":"185.00"},{"varval":"1145"}]},{"vars":[{"varval":"Therese"},{"varval":"She"},{"varval":"sm_img https://teacher.smartermaths.com.au/wp-content/uploads/2020/06/mountain-bike.png 200 indent vpad"},{"varval":"sm_img https://teacher.smartermaths.com.au/wp-content/uploads/2020/06/bikehelmet.png 200 indent vpad"},{"varval":"sm_img https://teacher.smartermaths.com.au/wp-content/uploads/2020/06/jersey.png 200 indent vpad"},{"varval":"795.00"},{"varval":"165.00"},{"varval":"75.00"},{"varval":"1035"}]},{"vars":[{"varval":"Bailey"},{"varval":"She"},{"varval":"sm_img https://teacher.smartermaths.com.au/wp-content/uploads/2020/06/mountain-bike.png 200 indent vpad"},{"varval":"sm_img https://teacher.smartermaths.com.au/wp-content/uploads/2020/06/bikecomputer.png 197 indent vpad"},{"varval":"sm_img https://teacher.smartermaths.com.au/wp-content/uploads/2020/06/jersey.png 200 indent vpad"},{"varval":"795.00"},{"varval":"85.00"},{"varval":"75.00"},{"varval":"955"}]},{"vars":[{"varval":"Koko"},{"varval":"She"},{"varval":"sm_img https://teacher.smartermaths.com.au/wp-content/uploads/2020/06/mountain-bike.png 200 indent vpad"},{"varval":"sm_img https://teacher.smartermaths.com.au/wp-content/uploads/2020/06/bikecomputer.png 200 indent vpad"},{"varval":"sm_img https://teacher.smartermaths.com.au/wp-content/uploads/2020/06/bike-shoes.png 200 indent vpad"},{"varval":"795.00"},{"varval":"85.00"},{"varval":"185.00"},{"varval":"1065"}]}]

  1417. A painter mixes {{colour1}} and {{colour2}} paint in the ratio {{ratio}} to produce a {{colour3}} colour. How many litres of {{colour1}} paint and {{colour2}} paint are required to get {{quantity1}} litres of the {{colour3}} paint?

    [{"vars":[{"varval":"red"},{"varval":"blue"},{"varval":"$1:3$"},{"varval":"purple"},{"varval":"60"},{"varval":"$\\dfrac{1}{4}$"},{"varval":"15"}]},{"vars":[{"varval":"yellow"},{"varval":"red"},{"varval":"$1:4$"},{"varval":"tangerine"},{"varval":"60"},{"varval":"$\\dfrac{1}{5}$"},{"varval":"12"}]},{"vars":[{"varval":"blue"},{"varval":"yellow"},{"varval":"$1:3$"},{"varval":"green"},{"varval":"36"},{"varval":"$\\dfrac{1}{4}$"},{"varval":"9"}]},{"vars":[{"varval":"blue"},{"varval":"yellow"},{"varval":"$1:5$"},{"varval":"teal"},{"varval":"30"},{"varval":"$\\dfrac{1}{6}$"},{"varval":"5"}]},{"vars":[{"varval":"lemon"},{"varval":"orange"},{"varval":"$1:5$"},{"varval":"marigold"},{"varval":"60"},{"varval":"$\\dfrac{1}{6}$"},{"varval":"10"}]}]

  1418. Which number is {{number}} rounded to the nearest {{rounding}}?

    [{"vars":[{"varval":"17 569"},{"varval":"thousand"}]},{"vars":[{"varval":"65 681"},{"varval":"hundred"}]},{"vars":[{"varval":"46 927"},{"varval":"hundred"}]},{"vars":[{"varval":"37 812"},{"varval":"thousand"}]},{"vars":[{"varval":"75 684"},{"varval":"hundred"}]}]

  1419. MIN STD 5-variants Q5

    When at full speed, a {{animal}} can travel at {{speed}} metres per second. How far would the {{animal}} travel if it maintained this speed over {{time}} seconds?

    [{"vars":[{"varval":"kangaroo"},{"varval":"16"},{"varval":"6"}]},{"vars":[{"varval":"possum"},{"varval":"6"},{"varval":"12"}]},{"vars":[{"varval":"cassowary"},{"varval":"9"},{"varval":"12"}]},{"vars":[{"varval":"giraffe"},{"varval":"12"},{"varval":"7"}]},{"vars":[{"varval":"hyena"},{"varval":"9"},{"varval":"8"}]}]

  1420. MINSTD 5 Variant - Question 4

    A type of mortar can be produced by mixing cement and sand in the ratio {{ratio}}. How much {{choose1}} is required to make {{mass}} kilograms of this mortar?

    [{"vars":[{"varval":"$2:3$"},{"varval":"sand"},{"varval":"30"},{"varval":"$\\dfrac{3}{5}$"}]},{"vars":[{"varval":"$3:4$"},{"varval":"sand"},{"varval":"28"},{"varval":"$\\dfrac{4}{7}$"}]},{"vars":[{"varval":"$2:5$"},{"varval":"cement"},{"varval":"35"},{"varval":"$\\dfrac{2}{7}$"}]},{"vars":[{"varval":"$3:5$"},{"varval":"sand"},{"varval":"80"},{"varval":"$\\dfrac{5}{8}$"}]},{"vars":[{"varval":"$2:3$"},{"varval":"cement"},{"varval":"30"},{"varval":"$\\dfrac{2}{5}$"}]}]

  1421. MINSTD 5 Variant - Question 3

    {{company}} import {{product}}. They sell one brand of {{product}} at {{markup}} times the wholesale price. The wholesale price is ${{wholesale}} per {{unit}}. What is the selling price per {{unit}}?

    [{"vars":[{"varval":"Earp Brothers"},{"varval":"tiles"},{"varval":"$1 \\dfrac{1}{2}$"},{"varval":"26"},{"varval":"metre"}]},{"vars":[{"varval":"Cleaning World"},{"varval":"disinfectant"},{"varval":"$2 \\dfrac{1}{2}$"},{"varval":"12"},{"varval":"litre"}]},{"vars":[{"varval":"Meat Inc."},{"varval":"steak"},{"varval":"$1 \\dfrac{3}{4}$"},{"varval":"36"},{"varval":"kilogram"}]},{"vars":[{"varval":"Smile Pty Ltd"},{"varval":"toothpaste"},{"varval":"$2 \\dfrac{1}{2}$"},{"varval":"16"},{"varval":"kilogram"}]},{"vars":[{"varval":"Transmission Oils"},{"varval":"engine oil"},{"varval":"$2 \\dfrac{1}{4}$"},{"varval":"12"},{"varval":"litre"}]}]

  1422. Number, NAP-H4-CA05

    A bullet fired from a hand gun travels at 390 metres per second. What is the speed of the bullet in metres per minute?

    [{"vars":null}]

  1423. <div class="sm_mode"> {{name}} has a stall at the market. She has {{number}} {{product}} that she wishes to arrange in rows in her display cabinet. Which combination of {{product}} per row would **not** display all her {{product}}. </div>

    [{"vars":[{"varval":"Harmony"},{"varval":"24"},{"varval":"anklets"},{"varval":"4"},{"varval":"5"},{"varval":"20"},{"varval":"anklets"}]},{"vars":[{"varval":"Alice"},{"varval":"36"},{"varval":"gemstones"},{"varval":"6"},{"varval":"5"},{"varval":"30"},{"varval":"gemstones"}]},{"vars":[{"varval":"Mandy"},{"varval":"42"},{"varval":"chocolates"},{"varval":"8"},{"varval":"5"},{"varval":"40"},{"varval":"chocolates"}]},{"vars":[{"varval":"Lulu"},{"varval":"60"},{"varval":"rings"},{"varval":"6"},{"varval":"9"},{"varval":"54"},{"varval":"rings"}]},{"vars":[{"varval":"Crystal"},{"varval":"42"},{"varval":"marbles"},{"varval":"8"},{"varval":"5"},{"varval":"40"},{"varval":"marbles"}]}]

  1424. Measurement, NAPX-E4-NC15

    Snowphish is hunting seals. He leaves his igloo at 5:25 am. If he returns at 8:15 am, how long has he been away?

    [{"vars":null}]

  1425. Measurement, NAP-E4-NC26

    <div class="sm_mode"> {{{question}}} </div>

    [{"vars":[{"varval":"Blair completed a double marathon on two consecutive days in an ultra distance running event.\n\nHis times are in the table below.\n\n<br>\n\n<div class=\"sm-table col1-color8 row1-color8 top-left-cell-hidden\">\n\n>>| | Time <br> (hours:minutes:seconds) |\n|:-:|:-:|\n| Day 1 | $11:18:48$|\n| Day 2| $10:52:52$|\n\n</div>\n\n<br>What was Blair's total time for the first two days of this event?"},{"varval":"$11:18:48$ + $10:52:52$ \n>= {{{correctAnswer}}}\n\n<br>(Remember to convert more than 60 seconds into minutes and seconds, and likewise convert more than 60 minutes into hours and minutes)"}]},{"vars":[{"varval":"Phoebe completed a double marathon on two consecutive days of a running carnival.\n\nHer times are in the table below.\n\n<div class=\"sm-table col1-color2 row1-color2 top-left-cell-hidden\">\n\n>>| | Time <br> (hours:minutes:seconds) |\n|:-:|:-:|\n| Day 1 | $12:58:46$|\n| Day 2| $13:55:35$|\n\n</div>\n\n<br>What was Phoebe's total time for the first two days of this event?"},{"varval":"$12:58:46$ + $13:55:35$\n\n>= {{{correctAnswer}}}\n\n<br>(Remember to convert more than 60 seconds into minutes and seconds, and likewise convert more than 60 minutes into hours and minutes)"}]}]

  1426. Measurement, NAP-C4-CA06

    sm_img https://teacher.smartermaths.com.au/wp-content/uploads/2017/01/naplan-Y7-2010-7mc.png 250 indent3 vpad What is the best estimate for the mass of this basketball?

    [{"vars":null}]

  1427. Measurement, NAP-C4-NC12

    Which metric unit would a builder use to measure the area of a courtyard? <br> sm_img https://teacher.smartermaths.com.au/wp-content/uploads/2017/01/NAP-C3-NC15.png 300 indent vpad

    [{"vars":null}]

  1428. Measurement, NAP-C4-NC06

    Kim bought these 5 items. <br> sm_img https://teacher.smartermaths.com.au/wp-content/uploads/2017/01/NAP-C4-NC061.png 520 indent vpad <br>The total mass of Kim's items is closest to

    [{"vars":null}]

  1429. Measurement, NAP-G4-NC04

    {{{question}}}

    [{"vars":[{"varval":"A giant earthworm measures 2.1 metres.\n\nHow long is it in centimetres?"},{"varval":"$\\begin{aligned}\r\n \\text{Length} &= 2.1 \\times 100 \\\\\n &= 210 \\ \\text{cm}\r\n\\end{aligned}$"}]},{"vars":[{"varval":"A great white shark measures 5.7 metres.\n\nHow long is it in centimetres?"},{"varval":"$\\begin{aligned}\r\n \\text{Length} &= 5.7 \\times 100 \\\\\n &= {{{correctAnswer}}} \\ \\text{cm}\r\n\\end{aligned}$"}]},{"vars":[{"varval":"The wingspan of an albatross measures 3.3 metres.\n\nHow long is it in centimetres?"},{"varval":"$\\begin{aligned}\r\n \\text{Length} &= 3.3 \\times 100 \\\\\n &= {{{correctAnswer}}}\\\r \\text{cm}\n\\end{aligned}$"}]}]

  1430. Measurement, NAP-I4-NC02

    {{{question}}}

    [{"vars":[{"varval":"Janus measures the width of his driveway to be 4 metres and 18 centimetres.\n\nWhich answer shows how Janus can write this measurement in metres?"},{"varval":"100 cm = 1 metre\n\nMeasurement = {{{correctAnswer}}}"}]},{"vars":[{"varval":"Chablis measured the length of her stand-up paddle board to be 3 metres and 21 centimetres.\n\nWhich answer shows how Chablis can write this measurement in metres?"},{"varval":"100 cm = 1 metre\n\nMeasurement = {{{correctAnswer}}}"}]},{"vars":[{"varval":"Hooley measured the length of his deck to be 10 metres and 9 centimetres.\n\nWhich answer shows how Hooley can write this measurement in metres?"},{"varval":"100 cm = 1 metre\n\nMeasurement = {{{correctAnswer}}}"}]}]

  1431. Measurement, NAP-K4-CA02

    <div class="sm_mode"> {{{question}}} </div>

    [{"vars":[{"varval":"Kim, Bob, Liz and Sarah each measure the height of the hedge in their front yards. \n\n* Kim's hedge is 0.72 metres tall\n* Bob's hedge is 815 millimetres tall\n* Liz's hedge is 68 centimetres tall\n* Sarah's hedge is 1.1 metres tall\n\nWho has the shortest hedge in their front yard?"},{"varval":"Convert each to metres:\n\nKim = 0.72 metres \n\nBob = 815 $\\div$ 1000 = 0.815 metres \n\nLiz = 68 $\\div$ 100 = 0.68 metres\n\nSarah = 1.1 metres\n\n$\\therefore$ {{{correctAnswer}}}'s hedge is the shortest."}]},{"vars":[{"varval":"Lili, Sid, Albert and Kimmy each measured the length of the biggest fish they caught. \n\n* Lili's biggest fish was 0.91 metres long\n* Sid's biggest fish was 597 millimetres long\n* Albert's biggest fish was 62 centimetres long\n* Kimmy's biggest fish was 1.2 metres long\n\nWhose biggest fish was the shortest in length?"},{"varval":"Convert each to metres:\n\nLili = 0.91 metres \n\nSid = 597 $\\div$ 1000 = 0.597 metres \n\nAlbert = 62 $\\div$ 100 = 0.62 metres\n\nKimmy = 1.2 metres\n\n$\\therefore$ {{{correctAnswer}}}'s biggest fish was the shortest in length."}]},{"vars":[{"varval":"Johnno, Zoey, Charlie and Izzy each measured the height of their dogs. \n\n* Johnno's dog is 0.84 metres high\n* Charlie's dog is 295 millimetres high\n* Izzy's dog is 1.3 metres high\n* Zoey's dog is 79 centimetres high\n\nWhose dog is the shortest?"},{"varval":"Convert each to metres:\n\nJohnno = 0.84 metres \n\nCharlie = 295 $\\div$ 1000 = 0.295 metres\n\nIzzy = 1.3 metres\n\nZoey = 79 $\\div$ 100 = 0.79 metres \n\n$\\therefore$ {{{correctAnswer}}}'s dog is the shortest."}]}]

  1432. Measurement, NAP-B4-NC03

    This dipstick measures the depth of oil in a car's oil tank. <br> sm_img https://teacher.smartermaths.com.au/wp-content/uploads/2017/01/NAP-B3-NC02.png 350 indent3 vpad <br>How deep is the oil?

    [{"vars":null}]

  1433. MIN STD - Variants 2

    {{name}} plays {{instrument}} as a session musician and is paid ${{pay1}} per hour. A band is offering {{name}} a new job that pays him an hourly rate {{fraction}} more than he currently receives. What is the hourly rate of pay the new job is offering?

    [{"vars":[{"varval":"Prince"},{"varval":"guitar"},{"varval":"36"},{"varval":"two thirds"},{"varval":"\\frac{2}{3}\n"},{"varval":"24"},{"varval":"60"}]},{"vars":[{"varval":"Peter"},{"varval":"drums"},{"varval":"18"},{"varval":"one and one third times"},{"varval":"\\frac{4}{3}"},{"varval":"24"},{"varval":"42"}]},{"vars":[{"varval":"Adrian"},{"varval":"bass guitar"},{"varval":"28"},{"varval":"one and one quarter times"},{"varval":"\\dfrac{5}{4}"},{"varval":"35"},{"varval":"63"}]},{"vars":[{"varval":"Bill"},{"varval":"keyboard"},{"varval":"27"},{"varval":"one and two thirds"},{"varval":"\\frac{5}{3}"},{"varval":"45"},{"varval":"72"}]},{"vars":[{"varval":"Jimmy"},{"varval":"lead guitar"},{"varval":"32"},{"varval":"two and three quarters"},{"varval":"\\frac{11}{4}"},{"varval":"88"},{"varval":"120"}]}]

  1434. Measurement, NAPX-H4-CA11 Stacked with 99d977ad-13e5-49ac-af22-66f3eb3cc824 Jan22

    A rodeo starts at 10:10 am and goes for 123 minutes. What time does it finish?

    [{"vars":null}]

  1435. Min Std 2 - Variants 2

    A {{club}} club decided to hold a BBQ fundraiser. The cost of the venue hire was ${{m1}}. ${{m2}} was spent on food and ${{m3}} was spent on drinks. If {{people}} people paid ${{cost}} to attend, how much profit did the {{club}} club make?

    [{"vars":[{"varval":"surf"},{"varval":"300"},{"varval":"240"},{"varval":"120"},{"varval":"300"},{"varval":"4.50"},{"varval":"660"},{"varval":"1350"},{"varval":"690"}]},{"vars":[{"varval":"soccer"},{"varval":"330"},{"varval":"240"},{"varval":"180"},{"varval":"400"},{"varval":"5.50"},{"varval":"750"},{"varval":"2200"},{"varval":"1450"}]},{"vars":[{"varval":"hockey"},{"varval":"150"},{"varval":"190"},{"varval":"170"},{"varval":"200"},{"varval":"6.50"},{"varval":"510"},{"varval":"1300"},{"varval":"790"}]},{"vars":[{"varval":"tennis"},{"varval":"160"},{"varval":"250"},{"varval":"140"},{"varval":"300"},{"varval":"3.50"},{"varval":"550"},{"varval":"1050"},{"varval":"500"}]},{"vars":[{"varval":"cricket"},{"varval":"170"},{"varval":"270"},{"varval":"120"},{"varval":"200"},{"varval":"6.50"},{"varval":"560"},{"varval":"1300"},{"varval":"740"}]}]

  1436. The running time of a documentary is 148 minutes. Paul starts watching the documentary at 2:13 pm. What time will the documentary end in 24-hour time?

    [{"vars":null}]

  1437. Measurement, NAPX-H2-33 SA

    <div class="sm_mode"> {{{question}}} </div>

    [{"vars":[{"varval":"Usain and Merv are the first two runners in a cross country relay team.\n\nUsain runs 1 lap of the course and passes a baton to Merv for the second lap.\n\nTheir results are in the table below.\n\n<br>\n\n<div class=\"sm-table row1-color1\">\n\n>>| Runner | Minutes | Seconds |\n|:-:|:-:|:-:|\n| Usain | 2|17|\n| Merv | 3| 33|\n\n</div>\n\n<br>How long after Usain **started**, in seconds, did Merv finish his lap?"},{"varval":"<div class=\"aligned\">\n\n|| |\n|-:|-|\n|Total Time | = 2:17 + 3:33||\n|| = 5 minutes 50 seconds|\n|| = (5 $\\times$ 60) + 50|\n|| = 300 + 50|\n||= {{{correctAnswer}}}|\n\n</div>"}]},{"vars":[{"varval":"<div class=\"sm_mode\">\n\nBonny and Clyde are the first two runners in a relay team.\n\nBonny runs 1 lap of the cross country course and passes a baton to Clyde for the second lap.\n\nTheir results are in the table below.\n\n<br>\n\n<div class=\"sm-table row1-color1\">\n\n>>| **Runner** | **Minutes** | **Seconds** |\n|:-:|:-:|:-:|\n| Bonny | 4|6|\n| Clyde | 5| 14|\n\n</div>\n\n<br>How long after Bonny **started**, in seconds, did Clyde finish his lap?\n\n</div>"},{"varval":"<div class=\"aligned\">\n\n|| |\n|-:|-|\n|Total Time | = 4:06 + 5:14||\n|| = 9 minutes 20 seconds|\n|| = (9 $\\times$ 60) + 20|\n|| = 540 + 20|\n||= {{{correctAnswer}}}|\n\n</div>"}]}]

  1438. Measurement, NAPX-I3-CA06

    A jeweller is making diamond rings for her collection. Which unit would be the most appropriate to record the mass of a diamond?

    [{"vars":null}]

  1439. Measurement, NAPX-J3-CA10, NAPX-J2-17

    Sheldon is estimating the length of the body of a flea in his insect collection. Which of these units of measurement would be the most helpful?

    [{"vars":null}]

  1440. Measurement, NAPX-E3-CA02

    The pictures below show a thermometer at two different times. <br> sm_img https://teacher.smartermaths.com.au/wp-content/uploads/2018/08/NAPX-E3-CA02.png 270 indent3 vpad By how much has the temperature risen?

    [{"vars":null}]

  1441. Measurement, NAP-G3-CA22

    <div class="sm_mode"> {{{question}}} </div>

    [{"vars":[{"varval":"Paul is driving from Wollongong to Taree via Hexham.\n\nHis estimated travel times are in the table below.\n\n<br>\n\n<div class=\"sm-table col1-color1\">\n\n>>| Wollongong to Hexham | 3 hours 20 minutes |\n|:-|:-:|\n| Hexham to Taree | 1 hour 20 minutes|\n\n\n</div>\n\n<br>\n\nPaul plans to leave Wollongong at 8:30 am and take a 15 minute rest stop in Hexham.\n\nUsing these driving times and rest time, when should Paul arrive in Taree?"},{"varval":"8:30 am plus 3 hours 20 minutes = 11:50 am \n\n11:50 am plus 15 minutes rest = 12:05 pm\r\n\n12:05 pm plus 1 hour 20 minutes $\\Rightarrow$ 1:25 pm \r\n\r\n$\\therefore$ Paul arrives about 1:30 pm."}]},{"vars":[{"varval":"Ginger is driving from Lennox Head to Gateshead via Bellingen.\n\nHer estimated travel times are in the table below.\n\n<br>\n\n<div class=\"sm-table col1-color1\">\n\n>>| Lennox Head to Bellingen | 3 hours 25 minutes |\n|:-|:-:|\n| Bellingen to Gateshead| 4 hours 40 minutes|\n\n\n</div>\n\n<br>\n\nGinger plans to leave Lennox Head at 6:20 am and take a 30 minute rest stop in Bellingen.\n\nUsing these driving times and rest time, when should Ginger arrive in Gateshead?"},{"varval":"6:20 am plus 3 hours 25 minutes = 9:45 am \n\n9:45 am plus 30 minutes rest = 10:15 pm\r\n\n10:15 pm plus 4 hours 40 minutes $\\Rightarrow$ 2:55 pm \r\n\r\n$\\therefore$ Ginger arrives {{{correctAnswer}}}."}]},{"vars":[{"varval":"Miley is driving from Byron Bay to Noosa via Southport.\n\nHer estimated travel times are in the table below.\n\n<div class=\"sm-table col1-color3\">\n\n>>| Byron Bay to Southport | 1 hour 15 minutes |\n|:-|:-:|\n| Southport to Noosa | 2 hours 20 minutes|\n\n</div>\n\n<br>\n\nMiley plans to leave Byron Bay at 6:30 am and take a 20 minute rest stop in Southport.\n\nUsing these driving times and rest time, when should Miley arrive in Noosa?"},{"varval":"6:30 am plus 1 hour 15 minutes = 7:45 am \n\n7:45 am plus 20 minutes rest = 8:05 am\r\n\n8:05 am plus 2 hours 20 minutes $\\Rightarrow$ 10:25 am \r\n\r\n$\\therefore$ Ginger arrives {{{correctAnswer}}}."}]}]

  1442. Measurement, NAP-C3-NC20

    When it is 10 am in Sydney, it is 6 am in Perth on the same day. A plane leaves Sydney at 11:30 am Sydney time and flies to Perth. The flight takes 4 hours and 40 minutes. What is the time in Perth when the plane arrives?

    [{"vars":null}]

  1443. Measurement, NAPX-G4-NC03 Stacked with 99d977ad-13e5-49ac-af22-66f3eb3cc824 Jan22

    A football game takes 125 minutes from start to finish. If the game started at 6:15 pm, what time will it finish?

    [{"vars":null}]

  1444. Measurement, NAPX-H3-CA14 Stacked with 99d977ad-13e5-49ac-af22-66f3eb3cc824 Jan22

    A tennis match starts at 10:21 am and goes for 135 minutes. What time does the match finish?

    [{"vars":null}]

  1445. Measurement, NAPX-K2-29

    Daisy is travelling from Ballina to Hobart. She must take two flights, one from Ballina to Melbourne and one from Melbourne to Hobart. Daisy's flight schedule is shown. <br> sm_img https://teacher.smartermaths.com.au/wp-content/uploads/2019/01/NAPX-K2-29v1.svg 580 indent vpad <br>What is Daisy's total flying time?

    [{"vars":null}]

  1446. Measurement, NAPX-I4-CA06

    Marco is travelling from Dubbo to Adelaide. He must take two flights, one from Dubbo to Sydney and one from Sydney to Adelaide. Marco's flight schedule is shown. <br> sm_img https://teacher.smartermaths.com.au/wp-content/uploads/2019/01/NAPX-K2-29v2.svg 320 indent vpad <br>What is Marco's total flying time?

    [{"vars":null}]

  1447. Measurement, NAPX-I3-NC12, NAPX-I2-25 Measurement, NAPX-p72896v02 Measurement, NAPX-p72896v01

    <div class="sm_mode"> {{{question}}} </div>

    [{"vars":[{"varval":"An end of term school assembly started at 10:30 am and went for $2 \\dfrac{3}{4}$ hours.\n\n\r\nSchool then finished $\\dfrac{1}{4}$ hour after the assembly ended.\n\n\r\n\r\nWhat time did school finish?\n"},{"varval":"<div class=\"sm_mode\">\n\nsm_nogap Finish time of assembly\n\n<div class=\"aligned\">\n\n>>||\n|-|\n|= 10:30 am + 2 hr 45 mins|\n|= 1:15 pm|\n\n</div>\n\n<br/>\n\n$\\therefore$ School finishes at {{{correctAnswer}}}.\n\n</div>"}]},{"vars":[{"varval":"Nica started cleaning her room at 8:10 am and it took her $4\\dfrac{1}{2}$ hours to finish.\n\n\r\n\r\nShe then had lunch which took $\\dfrac{1}{4}$ hour.\n\n\r\n\r\nWhat time did Nica finish eating lunch?"},{"varval":"Room cleaning finishes at 8:10 am plus 4 hours and 30 minutes, which is 12:40 pm.\n\n\r\n12:40 pm plus 15 minutes means Nica finishes eating lunch at 12:55 pm."}]},{"vars":[{"varval":"Henry attended a meeting at 1:30 pm and it lasted for $3\\dfrac{1}{2}$ hours.\n\n\r\n\r\nHe then drove home, which took him $\\dfrac{3}{4}$ hours.\n\n\r\n\r\nWhat time did Henry get home?"},{"varval":"Meeting finishes at 1:30 plus 3 hours and 30 minutes, which is 5:00 pm.\n\n\r\n5:00 pm plus 45 minutes means Henry arrives home at 5:45 pm."}]}]

  1448. Measurement, NAPX-I3-CA05

    Craig started work at 8:35 in the morning and finished at 6:50 in the evening. How many hours did Craig spend working?

    [{"vars":null}]

  1449. Measurement, NAP-B3-CA20

    Janine noticed the time difference between her watch and the wall clock. <br> sm_img https://teacher.smartermaths.com.au/wp-content/uploads/2017/02/naplan-Y7-2009-20mcb-rev.png 305 indent vpad <br><br>When the wall clock shows <img src="https://teacher.smartermaths.com.au/wp-content/uploads/2017/02/naplan-Y7-2009-20mca-rev.png" width="150" style="vertical-align: middle;"><br><br> What time does her watch show?

    [{"vars":null}]

  1450. Measurement, NAP-J3-CA23

    The running time of a documentary is 157 minutes. Michael starts watching the documentary at 12:07 pm. What time will the documentary end in 24-hour time?

    [{"vars":null}]

  1451. Measurement, NAP-A3-NC18 ISSUE: Colour in table

    <div class="sm_mode"> {{{question}}} </div>

    [{"vars":[{"varval":"Ryan and Oli are going to the cinema to watch a movie.\n\n<br>\n\n<div class=\"sm-table row1-color2\">\n\n>>| **Movie** | **Start Time** | **Length** |\n|:-|:-|:-|\n| Ted 2 | 11:30 am, 1:30 pm, 4:00 pm| 1 hour 42 minutes|\n| Deadpool | 3:30 pm, 8:00 pm| 2 hours|\n| Dirty Dancing | 11:00 am, 3:15 pm, 7:00 pm| 1 hour 50 minutes|\n| Weekend at Bernie's | 3:10 pm, 6:00 pm| 2 hours 15 minutes|\n\n</div>\n\n<br>\n\nThey arrive at the movie theatre at 3:00 pm. Their mother will pick them up at 5:15 pm.\n\nWhich movie could Ryan and Oli watch from start to finish?"},{"varval":"By trial and error:\n\nConsider Dirty Dancing time:\n\nsm_nogap 3:15 pm + 1 hour and 50 minutes\n\n>= 5:05 pm\n\n<br>\n\n$\\therefore$ Dirty Dancing can be watched from start to finish."}]},{"vars":[{"varval":"Hamish and Andy are going to the cinema to watch a movie.\n\n<br>\n\n<div class=\"sm-table row1-color1\">\n\n>>| **Movie** | **Start Time** | **Length** |\n|:-|:-|:-|\n| The Castle | 9:30 am, 2:30 pm, 4:30 pm| 2 hour 12 minutes|\n| Kenny| 2:30 pm, 6:00 pm| 2 hours|\n| Strictly Ballroom | 10:30 am, 3:45 pm, 5:00 pm| 2 hour 45 minutes|\n| Happy Feet | 9:10 pm, 4:00 pm| 1 hours 40 minutes|\n\n</div>\n\n<br>\n\nThey arrive at the movie theatre at 3:30 pm. Their mother will pick them up at 5:45 pm.\n\nWhich movie could Hamish and Andy watch from start to finish?"},{"varval":"By trial and error:\n\nConsider Happy Feet time:\n\nsm_nogap 4:00 pm + 1 hour and 40 minutes\n\n>= 5:40 pm\n\n<br>\n\n$\\therefore$ {{{correctAnswer}}} can be watched from start to finish."}]}]

  1452. Measurement, NAPX-G3-CA03

    What time is shown on the clock? <br> sm_img https://teacher.smartermaths.com.au/wp-content/uploads/2018/07/NAPX-G3-CA01.svg 250 indent2 vpad

    [{"vars":null}]

  1453. Measurement, NAP-H3-CA14

    <div class="sm_mode"> {{{question}}} </div>

    [{"vars":[{"varval":"A junior cricket game starts at 10:48 am and goes for 130 minutes.\r\n\r\nWhat time does the game finish?"},{"varval":"130 minutes = 2 hours 10 minutes\n\n10:48 am plus 2 hours and 10 minutes = {{{correctAnswer}}}."}]},{"vars":[{"varval":"A tennis match starts at 10:21 am and goes for 135 minutes.\r\n\r\nWhat time does the match finish?"},{"varval":"135 minutes = 2 hours and 15 minutes\n\n<div class=\"no-margin-bottom\">\n\n$\\therefore$ Finishing time of the tennis match\n\n</div>\n\n<div class=\"aligned\">\n\n> > | |\n> > | ------------------------------------------- |\n> > | \\= 10:21 am + 2 hours and 15 minutes |\n> > | \\= {{{correctAnswer}}} |\n\n</div>"}]},{"vars":[{"varval":"A rodeo starts at 10:10 am and goes for 123 minutes.\r\n\r\nWhat time does it finish?"},{"varval":"123 minutes = 2 hours and 3 min\n\n$\\begin{aligned}\r\n \\therefore \\text{Finish time} &=\\text{10:10 am plus 2 hours and 3 min} \\cr\n &=\\text{12:13 pm}\n\\end{aligned}$"}]},{"vars":[{"varval":"A football game takes 125 minutes from start to finish.\r\n\r\nIf the game started at 6:15 pm, what time will it finish?"},{"varval":"125 minutes = 2 hours and 5 minutes\n\nsm_nogap $\\therefore$ Finishing time of the football game\n\n<div class=\"aligned\">\n\n>>||\n|-|\n|= 6:15 pm + 2 hours and 5 minutes|\n|= {{{correctAnswer}}}|\n\n</div>"}]}]

  1454. Measurement, NAP-L4-CA30

    <div class="sm_mode"> {{{question}}} </div>

    [{"vars":[{"varval":"A truck driver plans to drive from Cairns to the Gold Coast.\n\nThe driving time for the journey is 19 hours.\r\n\nThe driver must stop for at least 8 hours rest after every 10 hours of driving.\r\n\r\nWhat is the latest time that the driver can depart Cairns on Monday to arrive in the Gold Coast at 2 pm on Tuesday?"},{"varval":"\n<div class=\"aligned\">\n\n|||\n|-|-|\n|Total travel time|= 19 + 8|\n||= 27 hours|\n\n</div>\n\n<br>\n\nsm_nogap $\\therefore$ Time driver must leave Cairns\n\n<div class=\"aligned\">\n\n>>||\n|-|\n|= 2 pm Tuesday less 27 hours|\n|= 2 pm Monday less 3 hours|\n|= 11 am Monday|\n\n</div>"}]},{"vars":[{"varval":"A long haul truck driver plans to drive from Brisbane to Kathryn.\n\nThe driving time for the journey is 15 hours.\r\n\nThe driver must stop for at least 3 hours rest after every 6 hours of driving.\r\n\r\nWhat is the latest time that the driver can depart Brisbane to arrive in Kathryn at 5 pm on Thursday?"},{"varval":"<div class=\"aligned\">\n\n|||\n|-|-|\n|Total travel time|= 6 + 3 + 6 + 3 + 3|\n||= 21 hours|\n\n</div>\n\n<br>\n\nsm_nogap $\\therefore$ Latest time to leave Brisbane\n\n<div class=\"aligned\">\n\n>>||\n|-|\n|= 5 pm Thursday less 21 hours|\n|= 5 pm Wednesday plus 3 hours|\n|= {{{correctAnswer}}}|\n\n</div>"}]},{"vars":[{"varval":"A family plans to drive from Wollongong to Coffs Harbour.\n\nThe driving time for the journey is 7 hours.\r\n\nThe driver must stop for at least 15 minutes rest after every 2 hours of driving.\r\n\r\nWhat is the latest time that the family can depart Wollongong to arrive in Coffs Harbour at 2 pm?"},{"varval":"\n<div class=\"aligned\">\n\n|||\n|-|-|\n|Total travel time|= 7 hours + 3 $\\times$ 15 minutes|\n||= 7 hours 45 minutes|\n\n</div>\n\n<br>\n\nsm_nogap $\\therefore$ Family must depart Wollongong\n\n<div class=\"aligned\">\n\n>>||\n|-|\n|= 2 pm less 7 hours 45 minutes|\n|= {{{correctAnswer}}}|\n\n</div>"}]},{"vars":[{"varval":"A removalist needs to drive from Adelaide to Melbourne for a job.\n\nThe driving time for the journey is 10 hours.\r\n\nThe driver must stop for at least 30 minutes rest after every 3 hours of driving.\r\n\r\nWhat is the latest time that the driver can depart Adelaide to arrive in Melbourne at 5 pm?"},{"varval":"<div class=\"aligned\">\n\n|||\n|-|-|\n|Total travel time|= 10 hours + 3 $\\times$ 30 minutes|\n||= 11 hours 30 minutes|\n\n</div>\n\n<br>\n\nsm_nogap $\\therefore$ Driver must leave Adelaide\n\n<div class=\"aligned\">\n\n>>||\n|-|\n|= 5 pm less 11 hours 30 minutes|\n|= {{{correctAnswer}}}|\n\n</div>"}]},{"vars":[{"varval":"Graham plans to start his university studies by driving from Broken Hill to Sydney.\n\nThe driving time for the journey is 13 hours.\r\n\nGraham plans to stop for at least 15 minutes rest after every 2 hours of driving.\r\n\r\nWhat is the latest time that Graham can depart Broken Hill to arrive in Sydney at 9 pm?"},{"varval":"<div class=\"aligned\">\n\n|||\n|-|-|\n|Total travel time|= 13 hours + 6 $\\times$ 15 minutes|\n||= 14 hours 30 minutes|\n\n</div>\n\n<br>\n\nsm_nogap $\\therefore$ Graham must leave Broken Hill\n\n<div class=\"aligned\">\n\n>>||\n|-|\n|= 9 pm less 14 hours 30 minutes|\n|= 9 am less 2 hours 30 minutes|\n|= {{{correctAnswer}}}|\n\n</div>"}]},{"vars":[{"varval":"Kim is driving back to Newcastle from Coonamble.\n\nThe driving time for the journey is 5 hours and 45 minutes.\r\n\nKim plans to stop for at least 15 minutes rest after every 2 hours of driving.\r\n\r\nWhat is the latest time that Kim can depart Coonamble to arrive in Newcastle at 3 pm?"},{"varval":"<div class=\"aligned\">\n\n|||\n|-|-|\n|Total travel time|= 5 hours 45 minutes + 2 $\\times$ 15 minutes|\n||= 6 hours 15 minutes|\n\n</div>\n\n<br>\n\nsm_nogap $\\therefore$ Kim's departure time\n\n<div class=\"aligned\">\n\n>>||\n|-|\n|= 3 pm less 6 hours 15 minutes|\n|= 9 am less 15 minutes|\n|= {{{correctAnswer}}}|\n\n</div>"}]}]

  1455. Measurement, NAP-A3-CA18

    Each morning Joel walks for 25 minutes from his home to Gateshead Station. He then travels by train to Newie Station. From Newie Station, Joel then walks for 24 minutes to the beach for a surf. <br> sm_img https://teacher.smartermaths.com.au/wp-content/uploads/2017/09/NAP-J1-31.png 550 indent vpad <br>What is the latest time Joel can leave home to arrive at the beach by 7:00?

    [{"vars":null}]

  1456. Measurement, NAP-D4-NC20

    Digby played cricket on a Saturday. His game began at 8:15 am and finished at 1:05 pm. How long did the cricket game go for?

    [{"vars":null}]

  1457. Measurement, NAP-F4-NC14

    Mike takes 2 minutes 35 seconds to walk 1 lap of the running track. Jordan runs 1 lap of the same track in 45 seconds. If they start at the same time, how long after Jordan finishes will Mike finish his lap?

    [{"vars":null}]

  1458. Number, NAP-G4-CA18

    Marty and Paul swam 3 km in an ocean swimming race. Marty finished in a time of 1 hour and 10 minutes. Paul took 25% longer than Marty. Approximately how long did Paul take to finish the race?

    [{"vars":null}]

  1459. Measurement,NAPX-K4-CA07

    <div class="sm_mode"> {{{question}}} </div>

    [{"vars":[{"varval":"Ricky lives in Bellerive. He is travelling by train to the ferry terminal to catch a 13:30 ferry.\n\nRicky needs to arrive 30 minutes before his ferry departs.\n\n<br>\n\n<div class=\"sm-table col1-color7 heading-color7\">\n\n>>**Train Timetable**\n\n>>| Glenorchy | 10:30 am | 11:30 am | 12:30 pm | 1:30 pm|\n|:-:|:-:|:-:|:-:|:-:|\n| Moonah| 10:43 am| 11:43 am | 12:43 pm | 1:43 pm|\n| Bellerive| 10:58 am|11:58 am | 12:58 pm | 1:58 pm|\n| Cambridge| 11:05 am| 12:05 pm |1:05 pm|2:05 pm|\n| Ferry Terminal| 11:17 am| 12:17 pm| 1:17 pm|2:17 pm|\n\n</div>\n\n<br>What is the latest time Ricky can catch a train from Bellerive?"},{"varval":"Working backwards:\n\nRicky must arrive at the ferry terminal before 1:00 pm\n\n$\\Rightarrow$ 12:17 pm is closest time\n\n\n$\\therefore$ 11:58 am train is the latest"}]},{"vars":[{"varval":"Jacqui lives in Moonah. She is travelling by train to the ferry terminal to catch a 12:30 pm ferry.\n\nJacqui needs to arrive 15 minutes before her ferry departs.\n\n<br>\n\n<div class=\"sm-table col1-color3 heading-color3\">\n\n>>**Train Timetable**\n\n>>| Glenorchy | 10:30 am | 11:30 am | 12:30 pm | 1:30 pm|\n|:-:|:-:|:-:|:-:|:-:|\n| Moonah| 10:43 am| 11:43 am | 12:43 pm | 1:43 pm|\n| Bellerive| 10:58 am|11:58 am | 12:58 pm | 1:58 pm|\n| Cambridge| 11:05 am| 12:05 pm |1:05 pm|2:05 pm|\n| Ferry Terminal| 11:17 am| 12:17 pm| 1:17 pm|2:17 pm|\n\n</div>\n\n<br>What is the latest time Jacqui can catch a train from Moonah?"},{"varval":"Working backwards:\n\nJacqui must arrive at the ferry terminal before 12:15 pm\n\n$\\Rightarrow$ The train arriving at the Ferry Terminal at 11:17 pm is the latest\n\n\n$\\therefore$ 10:43 am train is the latest"}]}]

  1460. Measurement, NAP-C4-NC16

    {{{question}}}

    [{"vars":[{"varval":"The sign shows opening hours of a car parking station.\n\nKenny wants to park his car for 2 hours to do some shopping, before collecting his car to drive home.\n\n<br>\n\nsm_img https://teacher.smartermaths.com.au/wp-content/uploads/2017/01/NAP-C4-NC16.png 200 indent3 vpad\n\n<br>At which time below could he safely park his car and pick it up to drive home?"},{"varval":"11:30 am on Wednesday plus 2 hours means that Kenny picks up his car at 1:30 pm which is within the opening hours of the parking station.\n\n$\\therefore$ He should park at {{{correctAnswer}}}."}]},{"vars":[{"varval":"The sign shows opening hours of a car parking station.\n\nFatima wants to park her combivan for 3.5 hours to do some shopping, before collecting her van to drive home.\n\n<br>\n\nsm_img https://teacher.smartermaths.com.au/wp-content/uploads/2017/01/NAP-C4-NC16.png 200 indent3 vpad\n\n<br>At which time below could Fatima safely park her van and pick it up to drive home?"},{"varval":"6:30 am on Saturday plus 3.5 hours means that Fatima picks up her van at 10:00 am which is within the opening hours of the parking station.\n\n$\\therefore$ She should park at {{{correctAnswer}}}."}]}]

  1461. Measurement, NAP-K3-CA16 ISSUE Image: no colour in the table

    Matt is travelling from Newcastle to Hamilton Island. He must take two flights, one from Newcastle to Brisbane and one from Brisbane to Hamilton Island. Matt's flight schedule is shown. <br> sm_img https://teacher.smartermaths.com.au/wp-content/uploads/2018/07/Measurement-NAP-K3-CA16.svg 429 indent vpad <br> What is Matt's total flying time?

    [{"vars":null}]

  1462. Measurement, NAP-D3-NC11 ISSUE Image - no colour in table

    <div class="sm_mode"> {{{question}}} </div>

    [{"vars":[{"varval":"Manou is cooking dinner for some friends.\n\nThe cooking times for the three ingredients of his main meal are in the table below.\n\n<br>\n\n<div class=\"sm-table col1-color1 row1-color1 top-left-cell-hidden\">\n\n>>| | **Cooking Time** |\n|:-:|:-:|\n| **Quail** | 1 hour 25 minutes|\n| **Brown Rice** | 20 minutes|\n| **Broccolini** | 15 minutes|\n\n</div>\n\n<br>\n\nManou starts cooking the quail at 7:15 pm.\n\nHe wants everything to finish cooking at the same time.\n\nAt what time should Manou start cooking the brown rice?"},{"varval":"The quail will finish cooking at 7:15 pm plus 1 hour 25 minutes = 8:40 pm.\n\n∴ Manou should start cooking the brown rice at {{{correctAnswer}}}."}]},{"vars":[{"varval":"Rafa is cooking dinner for his friends.\n\nThe cooking times for the three ingredients of his main meal are in the table below.\n\n<br>\n\n<div class=\"sm-table col1-color1 row1-color1 top-left-cell-hidden\">\n\n>>| | **Cooking Time** |\n|:-:|:-:|\n| **Chorizo** | 35 minutes|\n| **Arborio Rice** | 15 minutes|\n| **Prawns** | 17 minutes|\n\n</div>\n\n<br>\n\nRafa starts cooking the chorizo at 6:20 pm.\n\nHe wants everything to finish cooking at the same time.\n\nAt what time should Rafa start cooking the prawns?"},{"varval":"The chorizo will finish cooking at 6:20 pm plus 35 minutes = 6:55 pm.\n\n∴ Rafa should start cooking the prawns at {{{correctAnswer}}}."}]},{"vars":[{"varval":"Preston is cooking dinner.\n\nThe cooking times for the three ingredients of his main meal are in the table below.\n\n<br>\n\n<div class=\"sm-table col1-color1 row1-color1 top-left-cell-hidden\">\n\n>>| | **Cooking Time** |\n|:-:|:-:|\n| **Lamb Roast** | 3 hour 20 minutes|\n| **Potatoes** | 45 minutes|\n| **Beans** | 8 minutes|\n\n</div>\n\n<br>\n\nPreston starts cooking the lamb roast at 4:30 pm.\n\nHe wants everything to finish cooking at the same time.\n\nAt what time should Preston start cooking the potatoes?"},{"varval":"The lamb roast will finish cooking at 4:30 pm plus 3 hours and 20 minutes = 7:50 pm.\n\n∴ Preson should start cooking the potatoes at {{{correctAnswer}}}."}]}]

  1463. Measurement, NAP-I3-NC12

    Oliver's karate grading started at 9:15 am and went for $3 \dfrac{1}{4}$ hours. He then went straight home, which took him $\dfrac{3}{4}$ hours. What time did Oliver get home?

    [{"vars":null}]

  1464. Measurement, NAP-E3-NC13

    <div class="sm_mode"> {{{question}}} </div>

    [{"vars":[{"varval":"Tenzing started his trek at 5:30 am.\r\n\r\nHe arrived at his destination $14 \\dfrac{1}{2}$ hours later.\n\nAt what time did he arrive at his destination?"},{"varval":"One strategy:\n\n<div class=\"aligned\">\n\n|||\n|-|-|\n|5:30 am plus 14.5 hours|= 5:30 pm plus 2.5 hours|\n||= {{{correctAnswer}}}|\n\n</div>"}]},{"vars":[{"varval":"Yates started his bush walk at 6:15 am.\r\n\r\nHe arrived at his destination $13 \\dfrac{1}{3}$ hours later.\n\nAt what time did Yates arrive at his destination?"},{"varval":"One strategy:\n\n<div class=\"aligned\">\n\n|||\n|-|-|\n|6:15 am plus 13 h 20 min|= 6:15 pm plus 1 h 20 min|\n||= {{{correctAnswer}}}|\n\n</div>"}]},{"vars":[{"varval":"Carmen started her SES shift at 7:30 am.\r\n\r\nDue to being involved in flooding rescues, Carmen's shift only finished $15 \\dfrac{1}{4}$ hours later.\n\nAt what time did Carmen's shift finish?"},{"varval":"One strategy:\n\n<div class=\"aligned\">\n\n|||\n|-|-|\n|7:30 am plus 15 h 15 min|= 7:30 pm plus 3 h 15 mins|\n||= {{{correctAnswer}}}|\n\n</div>"}]},{"vars":[{"varval":"Tony started his fire fighting shift at 6:30 am.\r\n\r\nDue to the number of bush fires, Tony's shift didn't finish until $15 \\dfrac{1}{2}$ hours later.\n\nAt what time did Tony's shift finish?"},{"varval":"One strategy:\n\n<div class=\"aligned\">\n\n|||\n|-|-|\n|6:30 am plus 15.5 hours|= 6:30 pm + 3.5 hours|\n||= {{{correctAnswer}}}|\n\n</div>"}]},{"vars":[{"varval":"Al is kayaking from Sydney Harbour to Bar Beach for charity.\r\n\r\nAl left at 4:15 am but didn't arrive at Bar Beach until $13 \\dfrac{1}{3}$ hours later.\n\nAt what time did Al arrive at Bar Beach?"},{"varval":"One strategy:\n\n<div class=\"aligned\">\n\n|||\n|-|-|\n|4:15 am plus 13 h 20 mins|= 4:15 pm + 1 hour 20 mins|\n||= {{{correctAnswer}}}|\n\n</div>"}]},{"vars":[{"varval":"Shane visited a casino at 8:45 am in the morning.\n\r\nHe didn't leave the casino until $14 \\dfrac{1}{3}$ hours later.\n\nAt what time did Shane leave?"},{"varval":"One strategy:\n\n<div class=\"aligned\">\n\n|||\n|-|-|\n|8:45 am plus 14 h 20 mins|= 8:45 pm + 2 h 20 mins|\n||= {{{correctAnswer}}}|\n\n</div>"}]}]

  1465. Measurement, NAP-I3-CA05

    Roger started work at 7:15 in the morning and finished at 6:35 in the evening. How many hours did Roger spend working?

    [{"vars":null}]

  1466. Measurement, NAP-A3-NC02 ISSUE: Clock needs colour in the outline

    sm_img https://teacher.smartermaths.com.au/wp-content/uploads/2017/12/nap-A3-nc02.svg 160 indent2 vpad Which time is the same as the time shown on this digital clock?

    [{"vars":null}]

  1467. Measurement, NAP-G3-CA03

    What time is shown on the clock?<br> <br> sm_img https://teacher.smartermaths.com.au/wp-content/uploads/2017/01/naplan-Y7-2014-3mc-297x300.png 202 indent3 vpad

    [{"vars":null}]

  1468. Measurement, NAPX-J4-CA31

    Rebel walks for 5 minutes from her home to Century City Station. She then travels by train to Malibu Station. From Malibu Station, Rebel then walks for 16 minutes to the beach for a swim. <br> sm_img https://teacher.smartermaths.com.au/wp-content/uploads/2018/04/NAPX-J4-CA31.svg 580 indent vpad <br> What is the latest time Rebel can leave home to arrive at the beach by 07:00?

    [{"vars":[]}]

  1469. Measurement, NAPX-E4-NC26

    Phoebe completed a double marathon on two consecutive days of a running carnival. Her times are in the table below. sm_img https://teacher.smartermaths.com.au/wp-content/uploads/2018/06/NAPX-E4-NC26.svg 320 indent vpad What was Phoebe's total time for the first two days of this event?

    [{"vars":null}]

  1470. Measurement, NAPX-F4-NC14

    <div class="sm_mode"> {{{question}}} </div>

    [{"vars":[{"varval":"Dawn takes 4 minutes 15 seconds to swim 200 metres in the pool.\n\nMurray swims the same distance in 2 minutes 25 seconds.\n\nIf they start at the same time, how long after Murray finishes will Dawn finish?"},{"varval":"<div class=\"aligned\">\n\n|||\n|-:|-|\n| Extra time| = 4m 15s $-$ 2m 25s|\n||= 2m 15s $-$ 25s|\n||= {{{correctAnswer}}}|\n\n</div>"}]},{"vars":[{"varval":"Mike takes 2 minutes 35 seconds to walk 1 lap of the running track.\r\n\r\nJordan runs 1 lap of the same track in 55 seconds.\r\n\r\nIf they start at the same time, how long after Jordan finishes will Mike finish his lap?"},{"varval":"sm_nogap 2 minutes 35 seconds $-$ 55 seconds\n\n<div class=\"aligned\">\n\n>>||\n|-|\n|= 2 minutes $-$ 20 seconds|\n|= {{{correctAnswer}}}|\n\n</div>"}]}]

  1471. Measurement, NAPX-G3-CA22

    Miley is driving from Byron Bay to Noosa via Southport. Her estimated travel times are in the table below. sm_img https://teacher.smartermaths.com.au/wp-content/uploads/2018/08/NAPX-G3-CA22.svg 400 indent vpad Miley plans to leave Byron Bay at 6:30 am and take a 20 minute rest stop in Southport. Using these driving times and rest time, when should Miley arrive in Noosa?

    [{"vars":null}]

  1472. <div class ="sm_mode"> What is {{a}} times 6? sm_img https://teacher.smartermaths.com.au/wp-content/uploads/2019/12/nap-L4-11-ver2.svg 400 indent vpad <div class="indent2"> This is some content $$ \therefore $$ this is the answer And this is some more </div> $$ \therefore $$ this is the answer $$ \boxed{\pi=\frac c d} $$ $$ \underbrace{a+b+c}_{\text{note}} $$ sm_nogap Some writing... $$ \xcancel{ABC} $$ abcdef 1234 $$ \textsf{ abcdef 2342 } \xcancel{ABC} \forall \notni \sum \cfrac{a}{1 + \cfrac{1}{b}} $$ <div class="aligned"> | | | | ----------- | ------------------------------------- | | Cost per kg | \= $$\sf{\dfrac{23462}{645}}$$ | | | \= 23462 | </div> <br><br>And this is some more </div>

    [{"vars":[{"varval":"7"}]}]

  1473. {{name}} is catching a {{transport}}. She misses a {{transport}} by {{m1}} minutes and then waits for another {{m2}} minutes for the next {{transport}} which arrives on time at {{t1}}. At what time did the earlier {{transport}} leave?

    [{"vars":[{"varval":"Shelly"},{"varval":"tram"},{"varval":"6"},{"varval":"16"},{"varval":"11\\:17"},{"varval":"22"},{"varval":"11\\:00"},{"varval":"5"}]},{"vars":[{"varval":"Katrina"},{"varval":"bus"},{"varval":"8"},{"varval":"17"},{"varval":"5\\:12"},{"varval":"25"},{"varval":"5\\:00"},{"varval":"13"}]},{"vars":[{"varval":"Kelly"},{"varval":"ferry"},{"varval":"4"},{"varval":"15"},{"varval":"7\\:13"},{"varval":"19"},{"varval":"7\\:00"},{"varval":"6"}]},{"vars":[{"varval":"Angie"},{"varval":"train"},{"varval":"5"},{"varval":"12"},{"varval":"10\\:13"},{"varval":"17"},{"varval":"10\\:00"},{"varval":"4"}]},{"vars":[{"varval":"Constance"},{"varval":"hydrofoil"},{"varval":"13"},{"varval":"12"},{"varval":"2\\:14"},{"varval":"25"},{"varval":"2\\:00"},{"varval":"11"}]}]

  1474. {{{a}}} is catching a {{{b}}}. She misses a {{{c}}} by {{{d}}} minutes and then waits for another {{{e}}} minutes for the next {{{f}}} which arrives on time at {{{g}}} <br>At what time did the earlier tram leave?

    [{"vars":[{"varval":"Shelly"},{"varval":"tram"},{"varval":"tram"},{"varval":"6"},{"varval":"16"},{"varval":"tram"},{"varval":"11\\:17"}]}]

  1475. {{name}} starts playing a game of {{game}} at {{t1}} and finishes at {{t2}}. How long does {{name}} play {{game}} for?

    [{"vars":[{"varval":"Roger"},{"varval":"tennis"},{"varval":"7:20 pm"},{"varval":"8:05 pm"},{"varval":"8:00 pm"},{"varval":"40"},{"varval":"5"}]},{"vars":[{"varval":"Tammy"},{"varval":"netball"},{"varval":"10:50 am"},{"varval":"11:45 am"},{"varval":"11:00 am"},{"varval":"10"},{"varval":"45"}]},{"vars":[{"varval":"Michael"},{"varval":"basketball"},{"varval":"8:50 am"},{"varval":"9:25 am"},{"varval":"9:00 am"},{"varval":"10"},{"varval":"25"}]},{"vars":[{"varval":"Eddie"},{"varval":"billiards"},{"varval":"9:30 pm"},{"varval":"10:15 pm"},{"varval":"10:00 pm"},{"varval":"30"},{"varval":"15"}]},{"vars":[{"varval":"Moana"},{"varval":"Aussie Rules"},{"varval":"2:30 pm"},{"varval":"3:55 pm"},{"varval":"3:00 pm"},{"varval":"30"},{"varval":"55"}]}]

  1476. Measurement, NAPX-E4-CA11, NAPX-E3-CA15

    Big Matt's Pie shop is open 10 am to 4 pm from Monday to Friday and 10 am to 1 pm on Sunday. What is the total number of hours his shop is open in 1 week?

    [{"vars":null}]

  1477. Measurement, NAP-E4-NC15

    Azul is running a half-marathon. The event starts at 7:25 am. Azul completes the run at 9:05 am. How long did the run take Azul?

    [{"vars":null}]

  1478. Measurement, NAP-E3-CA15

    Renny's dress shop is open 9 am to 6 pm from Monday to Friday and 9 am to 1 pm on Saturday. What is the total number of hours her shop is open in 1 week?

    [{"vars":null}]

  1479. Measurement, NAPX-G4-NC03

    A football game takes 135 minutes from start to finish. If the game started at 6:15 pm, what time will it finish?

    [{"vars":null}]

  1480. Measurement, NAPX-H3-CA14

    {{{question}}}

    [{"vars":[{"varval":"A tennis match starts at 10:21 am and goes for 135 minutes.\n\nWhat time does the match finish?"},{"varval":"60 minutes in 1 hour.\n\n135 minutes = 2 hours and 15 minutes.\n\n10:21 am plus 2 hours and 15 minutes = {{{correctAnswer}}}"}]},{"vars":[{"varval":"Rafa started playing the Australian Open tennis final at 7:30 pm, in a match that took 324 minutes.\n\nAt what time did the match finish?\n"},{"varval":"60 minutes in 1 hour.\n\n324 minutes = 5 hours and 24 minutes.\n\n7:30 pm plus 5 hours and 24 minutes = {{{correctAnswer}}}"}]}]

  1481. Measurement, NAP-G4-NC03

    A television documentary goes for 115 minutes. If Damon started watching at 7:30 pm, what time will it finish?

    [{"vars":null}]

  1482. Measurement, NAP-D4-NC07

    Tran lives in Sydney, but his mother lives in Bangkok. When it is 1 pm in Sydney, it is 9 am in Bangkok on the same day. At 7 pm in Sydney, Tran calls his mother in Bangkok. What was the time in Bangkok when Tran called his mother?

    [{"vars":null}]

  1483. Measurement, NAPX-L4-CA11 v2, NAPX-L3-CA17 v2 ISSUE: incorrect timetable?

    <div class="sm_mode"> This is part of a timetable for the ferry from Mosman to Taronga Zoo. <br> <div class="sm-table col1-color1 heading-color1"> >>**Departure Times** >>| **Mosman**| 12:27 | 13:27 | 14:27 |15:27| |:-|:-:|:-:|:-:|:-:| | **Cremorne**| 12:58| 13:58 | 14:58|15:58| | **Luna Park**| 13:07| 14:07 |15:07|16:07| | **Taronga Zoo**| 13:19| 14:19 | 15:19|16:19| </div> <br>{{{a}}} is catching a ferry from {{{b}}} to Taronga Zoo. She catches the first ferry that leaves {{{b}}} after {{{c}}}. At what time will {{{a}}} arrive at Taronga Zoo? </div>

    [{"vars":[{"varval":"Anastasia"},{"varval":"Cremorne"},{"varval":"1:30 pm"},{"varval":"13:58 (1:58 pm)"}]},{"vars":[{"varval":"Precise"},{"varval":"Mosman"},{"varval":"2 pm"},{"varval":"14:27 (2:27 pm)"}]}]

  1484. Measurement, NAPX-I4-CA06

    <div class="sm_mode"> {{{question}}} </div>

    [{"vars":[{"varval":"Rex arrived at school at 8:05 in the morning and did not leave until 3:25 in the afternoon.\r\n\r\nHow long was Rex at school?"},{"varval":"Time until midday = 3 hours 55 min\n\nsm_nogap Total time at school\n\n<div class=\"aligned\">\n\n>>||\n|-|\n|\\= 3h 55 min + 3h 25 min|\n|\\= 7 hours 20 minutes|\n\n</div>"}]},{"vars":[{"varval":"Craig arrived at the Easter Show at 9:15 in the morning and did not leave until 4:30 in the afternoon.\r\n\r\nHow long was Craig at the Easter Show?"},{"varval":"Time until midday = 2 hours 45 min\n\nsm_nogap Total time at Easter Show\n\n<div class=\"aligned\">\n\n>>||\n|-|\n|\\= 2h 45 min + 4h 30 min|\n|\\= {{{correctAnswer}}}|\n\n</div>"}]},{"vars":[{"varval":"Fonzi arrived at the mall at 9:40 in the morning and did not leave until 4:20 in the afternoon.\r\n\r\nHow long was Fonzi at the mall?"},{"varval":"Time until midday = 2 hours 20 min\n\nsm_nogap Total time at mall\n\n<div class=\"aligned\">\n\n>>||\n|-|\n|\\= 2h 20 min + 4h 20 min|\n|\\= {{{correctAnswer}}}|\n\n</div>"}]},{"vars":[{"varval":"Hank arrived at the rodeo at 6:10 in the morning and did not leave until 4:30 in the afternoon.\r\n\r\nHow long was Hank at the rodeo?"},{"varval":"Time until midday = 5 hours 50 min\n\nsm_nogap Total time at rodeo\n\n<div class=\"aligned\">\n\n>>||\n|-|\n|\\= 5h 50 min + 4h 30 min|\n|\\= {{{correctAnswer}}}|\n\n</div>"}]},{"vars":[{"varval":"Kim arrived at the movie cinemas at 11:10 in the morning and did not leave until 2:30 in the afternoon.\r\n\r\nHow long was Kim at the movie cinemas?"},{"varval":"Time until midday = 50 min\n\nsm_nogap Total time at movie cinemas\n\n<div class=\"aligned\">\n\n>>||\n|-|\n|\\= 50 min + 2h 30 min|\n|\\= {{{correctAnswer}}}|\n\n</div>"}]},{"vars":[{"varval":"Rhona arrived at her hairdresser at 9:50 in the morning and did not leave until 1:15 in the afternoon.\r\n\r\nHow long was Rhonda at her hairdresser?"},{"varval":"Time until midday = 2h 10 min\n\nsm_nogap Total time at hairdresser\n\n<div class=\"aligned\">\n\n>>||\n|-|\n|\\= 2h 10 min + 1h 15 min|\n|\\= {{{correctAnswer}}}|\n\n</div>"}]}]

  1485. Measurement, NAPX-J4-CA01, NAPX-J3-CA02

    The digital clock below tells the time in the 24-hour system. sm_img https://teacher.smartermaths.com.au/wp-content/uploads/2018/06/NAPX-J4-CA01_rev2.svg 170 indent vpad What is the equivalent time in the 12-hour system?

    [{"vars":null}]

  1486. Measurement, NAP-I4-CA06

    Peter left home at 9:15 in the morning and did not return until 5:25 in the afternoon. How long was Peter away from his house?

    [{"vars":null}]

  1487. NAP-J4-CA01

    The digital clock below tells the time in the 24-hour system. <br> sm_img https://teacher.smartermaths.com.au/wp-content/uploads/2017/09/NAP-J1-01.png 160 indent vpad <br>What is the equivalent time in the 12-hour system?

    [{"vars":null}]

  1488. Measurement, NAPX-I3-NC12, NAPX-I2-25

    An end of term school assembly started at 10:30 am and went for $2\dfrac{3}{4}$ hours. School then finished $\dfrac{1}{4}$ hour after the assembly ended. What time did school finish?

    [{"vars":null}]

  1489. Measurement, NAP-L4-CA11

    This is part of a timetable for the ferry from Apollo Road to West End. <br> sm_img https://teacher.smartermaths.com.au/wp-content/uploads/2019/12/NAP-L3-CA17.svg 520 indent vpad <br> Skye is catching a ferry from Apollo Road to West End. She catches the first ferry that leaves Apollo Road after 1 pm. At what time will Skye arrive at West End?

    [{"vars":null}]

  1490. Measurement, NAP-I3-CA05

    Roger started work at 7:15 in the morning and finished at 6:35 in the evening. How many hours did Roger spend working?

    [{"vars":null}]